You are on page 1of 2947

MyPastest

Prefer to use the old MyPastest? Access it here »

Back to Filters

Question 1 of 12

An 8-year-old child attends clinic for the first time with his mother. He has achieved daytime dryness but wets the
bed several nights a week. Urinalysis is clear and there are no abnormal findings on
abdominal or neurological
examination. No previous intervention has been given to the family.

Which ONE of the following would be the most appropriate first line treatment?

A Intranasal desmopressin

B Star chart and encouragement of child and family

C Oxybutynin

D Enuresis alarm

E Imipramine

Explanation
Many children and families respond well to simple behavioural interventions and reassurance, with
encouragement of the child to participate in changing bedding. Not drinking 2 hours prior to bedtime and
micturating just before going to sleep are techniques used as a first reponse to nocturnal enuresis. After the age of
7 years, if these behavioural techniques have failed, an enuresis alarm can be given
a trial and provides more
Next Question
effective treatment than drug therapy with a low relapse rate. Desmopressin can be useful in short-term situations
in
older children but has a poor cure rate, as does imipramine, which also
has many side effects. Oxybutynin is
most useful in those with bladder instability affecting both diurnal and nocturnal continence. 11612

Tag Question

https://mypastest.pastest.com/Secure/TestMe/Browser/436619[‫ ص‬05:19:49 08/12/1437]


MyPastest

Feedback

Difficulty: Easy
Previous Question
Peer Responses

Session Progress

Responses Correct: 0

Responses Incorrect: 1

Responses Total: 1

Responses - % Correct: 0%

Blog
About Pastest
Contact Us
Help

© Pastest 2016

End Session

https://mypastest.pastest.com/Secure/TestMe/Browser/436619[‫ ص‬05:19:49 08/12/1437]


MyPastest

Prefer to use the old MyPastest? Access it here »

Back to Filters

Question 2 of 12

What BMI is diagnostic of anorexia?

A < 13.0

B < 15.5

C < 17.5

D < 19.5

E < 22.0

Explanation
This is one of the diagnostic criteria. According
to ICD-10 all of the following are required for a definite diagnosis
of
anorexia nervosa:

Quetelet’s body mass index is 17.5 or less


The weight loss is self-induced
There is body-image distortion and a dread of fatness
There
is disturbance of the hypothalamic–pituitary–gonadal axis (manifest in women as amenorrhoea and
Next Question
in men as a loss of sexual interest and potency)
If the onset is prepubertal, the sequence of pubertal events is delayed or even arrested.

12637

https://mypastest.pastest.com/Secure/TestMe/Browser/436619[‫ ص‬05:21:16 08/12/1437]


MyPastest

Tag Question

Feedback

Difficulty: Average
Previous Question
Peer Responses

Session Progress

Responses Correct: 0

Responses Incorrect: 2

Responses Total: 2

Responses - % Correct: 0%

Blog
About Pastest
Contact Us
Help

© Pastest 2016

End Session

https://mypastest.pastest.com/Secure/TestMe/Browser/436619[‫ ص‬05:21:16 08/12/1437]


MyPastest

Prefer to use the old MyPastest? Access it here »

Previous Question Back to Filters

Question 3 of 12

A teenage boy presents with bizarre delusions, a blunted affect and tangential thought processes.

Which one of the following characteristics would indicate an unfavourable diagnosis and prognosis?

A A prolonged premorbid history of social withdrawal

B A schizophrenic second-degree relative

C Rapid onset of illness

D A normal MRI brain scan

E Catatonia

Explanation
The symptoms are suggestive of schizophrenia. In schizophrenia, a sudden onset of illness has a more favourable
prognosis
than when the onset is insidious. A premorbid history of social withdrawal is predictive of more severe
and long-lasting psychopathology. A family history of schizophrenia is commonly found in second-degree
relatives and has little prognostic significance. MRI abnormalities are associated with more severe symptoms and
clinical course in people with schizophrenia. The presence of catatonic symptomatology is not associated with any
particular clinical course. 12638
Next Question

Tag Question
End Session
Feedback

https://mypastest.pastest.com/Secure/TestMe/Browser/436619[‫ ص‬05:22:13 08/12/1437]


MyPastest

Difficulty: Average

Peer Responses

Session Progress

Responses Correct: 0

Responses Incorrect: 3

Responses Total: 3

Responses - % Correct: 0%

Blog
About Pastest
Contact Us
Help

© Pastest 2016

https://mypastest.pastest.com/Secure/TestMe/Browser/436619[‫ ص‬05:22:13 08/12/1437]


MyPastest

Prefer to use the old MyPastest? Access it here »

Previous Question Back to Filters

Question 4 of 12

A 4-year-old girl fails to develop intelligible speech and simply repeats bits of sentences that she has heard. She
spends hours sitting alone, facing a wall and rocking back and forth. She shows little affection for others and
sometimes bangs her head against the floor.

What is the most likely aetiology for her symptoms?

A Childhood sexual abuse

B Emotionally distant parents

C Childhood exposure to lead

D Intrauterine rubella

E Maternal cocaine abuse during pregnancy

Explanation
The symptoms are most suggestive of autistic disorder. There is a strong association between autistic disorder and
intrauterine infections, most commonly rubella. Psychosocial factors such as emotionally distant parents and
childhood sexual abuse are not causally associated, although the symptoms may mimic an autistic disorder.
Maternal cocaine abuse during pregnancy and lead exposure during childhood are associated with behavioural and
Next Question
attention disturbances in children, but not with autistic disorder.
12639

Tag Question

https://mypastest.pastest.com/Secure/TestMe/Browser/436619[‫ ص‬05:22:33 08/12/1437]


MyPastest

Feedback

Difficulty: Difficult

Peer Responses

Session Progress

Responses Correct: 0

Responses Incorrect: 4

Responses Total: 4

Responses - % Correct: 0%

Blog
About Pastest
Contact Us
Help

© Pastest 2016

End Session

https://mypastest.pastest.com/Secure/TestMe/Browser/436619[‫ ص‬05:22:33 08/12/1437]


MyPastest

Prefer to use the old MyPastest? Access it here »

Previous Question Back to Filters

Question 5 of 12

A teenage girl complains that her mother interferes in every aspect of her life and tries to control her. The girl no
longer speaks to her parents or eats meals with them. The mother has increased her efforts to maintain control.

What condition could arise from this situation?

A Anorex ia nervosa

B Dissociative identity disorder

C Narcissistic personality disorder

D Schizophrenia

E Separation anxiety disorder

Explanation
Family dynamics are strongly implicated in the development of anorexia nervosa. Food restriction, with avoidance
of family mealtimes and weight loss are postulated as attempts to regain some control and avoid sexual issues.
Dissociative identity disorder is associated with childhood sexual abuse. Narcissistic personality and
schizophrenia are not associated with family dynamics. There is no situation here that could cause separation
anxiety disorder.
Next Question 12640

End
Tag Session
Question

Feedback

https://mypastest.pastest.com/Secure/TestMe/Browser/436619[‫ ص‬05:22:56 08/12/1437]


MyPastest

Difficulty: Average

Peer Responses

Session Progress

Responses Correct: 0

Responses Incorrect: 5

Responses Total: 5

Responses - % Correct: 0%

Blog
About Pastest
Contact Us
Help

© Pastest 2016

https://mypastest.pastest.com/Secure/TestMe/Browser/436619[‫ ص‬05:22:56 08/12/1437]


MyPastest

Prefer to use the old MyPastest? Access it here »

Previous Question Back to Filters

Question 6 of 12

A young boy resists going to school because he wants to stay with his mother. He becomes terrified whenever his
parents leave the house. During the day he worries that his family may never come back home.

What disorder in adulthood is most closely associated with these symptoms?

A Dysthymic disorder

B Obsessive–compulsive disorder

C Panic disorder

D Schizophrenia

E Pain disorder

Explanation
This scenario is indicative of separation anxiety
disorder. School avoidance is often present in children with this
disorder. A history of this disorder in childhood is common in adult patients with panic disorder. Schizophrenia is
sometimes associated with
a history of emotional withdrawal during childhood. The other conditions are not
usually associated with separation disorder.
12641

Next Question

Tag Question
End Session
Feedback

https://mypastest.pastest.com/Secure/TestMe/Browser/436619[‫ ص‬05:23:20 08/12/1437]


MyPastest

Difficulty: Easy

Peer Responses

Session Progress

Responses Correct: 0

Responses Incorrect: 6

Responses Total: 6

Responses - % Correct: 0%

Blog
About Pastest
Contact Us
Help

© Pastest 2016

https://mypastest.pastest.com/Secure/TestMe/Browser/436619[‫ ص‬05:23:20 08/12/1437]


MyPastest

Prefer to use the old MyPastest? Access it here »

Back to Filters

Question 7 of 12

A 17-year-old girl is reviewed in the psychiatric clinic. Her


expected weight is 55 kg and since follow-up began
her lowest weight has been 49.5 kg. She goes through episodes of binge-eating and vomiting
and also reports
prolonged fasts. She is scared of putting on weight and has missed her last two periods.

What is the likely diagnosis?

A Anorexia nervosa – purging type

B Anorexia nervosa – with binge-eating

C Bulimia nervosa

D Personality disorder

E Depression

Explanation
The main differential diagnoses of eating disorders are anorexia and bulimia. However, for a diagnosis of anorexia
the weight must be > 15% below the expected weight, with amenorrhoea
defined as the continuous absence of
three menstrual cycles. Binge episodes and fasting episodes can be seen in both anorexia and bulimia, but purging
behaviours, e.g. vomiting and laxative and diuretic abuse are more characteristic of bulimia. Depression and
Next Question
obsessive–compulsive disorder are frequently associated with anorexia, which has a high long-term mortality of
18–20%. Predisposing factors for bulimia include a history of anorexia, being overweight, mood disorder and
borderline personality disorder as well as
illicit drug use. The prognosis in bulimia is variable. Both conditions
are
treated with psychotherapy, cognitive therapy and self-help groups.
Antidepressants can be useful in bulimia
where fluoxetine is used (10 mg/day).
12642

https://mypastest.pastest.com/Secure/TestMe/Browser/436619[‫ ص‬05:23:43 08/12/1437]


MyPastest

Tag Question

Feedback

Previous Question Difficulty: Average

Peer Responses

Session Progress

Responses Correct: 0

Responses Incorrect: 7

Responses Total: 7

Responses - % Correct: 0%

Blog
About Pastest
Contact Us
Help

© Pastest 2016

End Session

https://mypastest.pastest.com/Secure/TestMe/Browser/436619[‫ ص‬05:23:43 08/12/1437]


MyPastest

Prefer to use the old MyPastest? Access it here »

Back to Filters

Question 8 of 12

A 9-year-old boy constantly disobeys his parents and teachers. He gets along well with his peers and completes
projects that he likes. His developmental milestones are normal and he has no history of fighting, theft or
destruction of property. Mental status examination
reveals an assertive child who tells the examiner that he does
not wish
to discuss his problems.

What is the most likely diagnosis?

A Oppositional defiant disorder

B Mental retardation

C Conduct disorder

D Childhood disintegrative disorder

E Attention-deficit hyperactivity disorder (ADHD)

Explanation
Oppositional defiant disorder involves problems in relating to authority figures. Such children get along well with
their peers and have no other problems of conduct or development. Mental retardation is associated with delayed
developmental milestones and other evidence of impaired intellectual abilities. Conduct disorder is characterised
Next Question
by violation of age-appropriate social norms, fighting, runaway behaviour, theft and destruction of property and is
less likely given the clinical scenario. Childhood disintegrative disorder involves the development of severe
disturbances in social, communicative and cognitive functions following a period of normal development.
Attention-deficit hyperactivity disorder is characterised by inattention, impulsivity and hyperactivity. Individuals
with ADHD would be unlikely to complete projects even if they were interested in them. Conduct disorder is a
sequel to oppositional defiant disorder. 12643

https://mypastest.pastest.com/Secure/TestMe/Browser/436619[‫ ص‬05:24:05 08/12/1437]


MyPastest

Tag Question

Feedback

Previous Question
Difficulty: Easy

Peer Responses

Session Progress

Responses Correct: 0

Responses Incorrect: 8

Responses Total: 8

Responses - % Correct: 0%

Blog
About Pastest
Contact Us
Help

© Pastest 2016

End Session

https://mypastest.pastest.com/Secure/TestMe/Browser/436619[‫ ص‬05:24:05 08/12/1437]


MyPastest

Prefer to use the old MyPastest? Access it here »

Previous Question Back to Filters

Question 9 of 12

Of which of the following would a 4-year-old girl be scared?

A Death

B Monsters

C Heights

D Loud noises

E Strangers

Explanation
Children are not usually scared of real dangers or afraid to interact with strangers. However, they do react to
fictitious or imaginative creatures such as ghosts or monsters, which are made to appear fearsome and scary in
comics, cartoons and movies.
12644

Next Question
Tag Question

Feedback

Difficulty: Easy End Session


Peer Responses

https://mypastest.pastest.com/Secure/TestMe/Browser/436619[‫ ص‬05:24:29 08/12/1437]


MyPastest

Session Progress

Responses Correct: 0

Responses Incorrect: 9

Responses Total: 9

Responses - % Correct: 0%

Blog
About Pastest
Contact Us
Help

© Pastest 2016

https://mypastest.pastest.com/Secure/TestMe/Browser/436619[‫ ص‬05:24:29 08/12/1437]


MyPastest

Prefer to use the old MyPastest? Access it here »

Back to Filters

Question 10 of 12

A 6-year-old girl presents with soiling and she has never been toilet trained. She regularly passes normal formed
stools into her underclothes without seemingly being concerned. Her parents separated when she was 3 and the
mother has a history of an eating disorder. The father has a history of duodenal ulcer. Her bowel habit was erratic
and has much improved with lactulose 5 mL twice daily. Her examination is normal except for a slightly distended
abdomen. The management plan most likely to succeed is:

A Increase her laxatives

B Refer to Social Services

C Liaise with her schoolteacher

D Switch off her favourite TV programme whenever she soils

E Refer to the Child and Adolescent Mental Health Services (CAMHS)

Explanation
Of the management options listed for this girl with encopresis, a referral to the CAMHS team is the most
appropriate. She is already taking mild laxatives with good effect, with little to suggest significant constipation
because she passes normal stools regularly, although not in the toilet. A social services referral may be
indicated at
some time, but is mainly indicated if there are specific child protection concerns. Behavioural programmes which
Next Question
may well be helpful, but not using negative rewards which may well be counterproductive and be explained as
punishment.
22374

Tag Question

https://mypastest.pastest.com/Secure/TestMe/Browser/436619[‫ ص‬05:24:52 08/12/1437]


MyPastest

Feedback

Difficulty: Average
Previous Question
Peer Responses

Session Progress

Responses Correct: 0

Responses Incorrect: 10

Responses Total: 10

Responses - % Correct: 0%

Blog
About Pastest
Contact Us
Help

© Pastest 2016

End Session

https://mypastest.pastest.com/Secure/TestMe/Browser/436619[‫ ص‬05:24:52 08/12/1437]


MyPastest

Prefer to use the old MyPastest? Access it here »

Back to Filters

Question 11 of 12

Theme: Psychiatric disorders

A Anorexia nervosa
B Asperger syndrome
C Attention deficit-hyperactivity disorder
D Autism
E Depression
F Factitious Illness
G Psycho-social deprivation
H Post-traumatic stress disorder
I Rett syndrome
J Temper tantrums

For
each of the following scenarios, please choose the most likely diagnosis from the list above. Each item may be
used once, more than once or not at all.

Scenario 1

A 3-year-old boy is brought to you, as his


mother is concerned about his attention span. You note that he has very
poor eye contact and poor verbal skills (having only a four-word vocabulary). His weight and height are
appropriate for his age.

Your answer was incorrect

Select one...

D - Autism

Autism is an increasing problem. It was first described by Kanner in 1943. It is a triad of communication
difficulties, socialisation problems and attention problems. This 3-year-old shows poor language skills and poor
eye contact (socialisation problem). The features of autism are mostly present before 2 years of age but the
diagnosis may not necessarily be made at that time. The language difficulties can be profound and up to 30% of
autistic children may never develop language. Treatment modalities include speech and language therapy,
behavioural therapy to improve socialisation and attention.

https://mypastest.pastest.com/Secure/TestMe/Browser/436619[‫ ص‬05:25:18 08/12/1437]


MyPastest

Scenario 2

A 4-year-old girl is referred for speech therapy as she speaks very little. She has not had her MMR or pre-school
boost. She appears withdrawn and shy but will play games if encouraged. Her weight is on the 2nd centile for her
age (2 years ago was on 50th). Her dentition is poor.

Your answer was incorrect


Previous Question
Select one...

G - Psycho-social deprivation

In a child who was previously thriving


but is now growth faltering, and who has some developmental delay, the
diagnosis of psychosocial deprivation must always be considered. This is
a diagnosis of exclusion, once organic
causes have been ruled out. In this case there are other warning sign of this diagnosis – she has
missed some
immunisations, she will play games (so probably has no socialisation difficulties) and her dentition is poor, a
possible sign of neglect.

Scenario 3

An 11-year-old girl is confrontational at home and argues with her parents. She enjoys going out with her friends.
Her school performance is falling and her concentration is poor. She has had
thoughts of deliberate self-harm in
the past and presents with some superficial lacerations to her left wrist. Taking a detailed history you
elicit that she
was sexually assaulted by a family friend 1 year ago has repeated distressing memories of the event.

Your answer was incorrect

Select one...

H - Post-traumatic stress disorder

Post-traumatic stress disorder, although still uncommon, is an increasing diagnosis in paediatric psychiatry. The
trauma of the sexual assault has led to the behavioural problems in this girl. Posttraumatic stress disorder can be
due to sexual assault, physical assault or indeed any experience of very traumatic situations (eg refugees from a
conflict zone). It can present in a myriad of ways: concentration difficulties, mood instability, aggression,
flashbacks, sleep disturbance, anxiety, depression and parasuicide. Cognitive–behavioural therapy and counselling
may help the disclosure of the trigger, and provide effective results.
22414

Next Question

Tag Question

Feedback End Session

Difficulty: Average

https://mypastest.pastest.com/Secure/TestMe/Browser/436619[‫ ص‬05:25:18 08/12/1437]


MyPastest

Session Progress

Responses Correct: 0

Responses Incorrect: 13

Responses Total: 13

Responses - % Correct: 0%

Blog
About Pastest
Contact Us
Help

© Pastest 2016

https://mypastest.pastest.com/Secure/TestMe/Browser/436619[‫ ص‬05:25:18 08/12/1437]


MyPastest

Prefer to use the old MyPastest? Access it here »

Back to Filters

Question 12 of 12

Select the MOST appropriate statement from the following with regard to somatoform disorders in childhood

A Medical investigations are rarely helpful

B They are often appropriately managed solely by the mental health services

C They occur more frequently in children whose parents have a physical illness

D They overlap with factitious disorders and malingering

E They rarely have any long-term consequences for the child or family

Explanation
Somatoform disorders are a group of psychological disorders in which a patient experiences a physical symptom
in the absence of an underlying medical condition. It is common for children to express emotional distress as
physical pain and transient episodes do not affect overall functioning. However, the persistent experiences
associated with a disorder will often interfere with school, home life and friendships. In contrast to factitious
disorders or malingering, where symptoms are intentionally produced or feigned, in somatoform disorders
intentional deception does not occur.

Headaches,
stomachaches, dizziness, chest pain and nausea are the most commonly reported symptoms. Risk
factors include presence of depression or anxiety disorder, female sex, family dysfunction and increasing age. It
also occurs more frequently in children whose parents have non-life-threatening disease or medically unexplained
symptoms. It is more likely to occur in children with a history of being abused. Exclusion of medical and
neurological conditions is the major diagnostic
concern, eg brain tumours and temporal lobe epilepsy, but
extensive unwarranted medical investigation often only delay the diagnosis and implementation of appropriate
treatment. Families may be resistant to considering a psychological cause of the child’s symptoms, so careful
explanation and reassurance are essential at all stages. Treatment involves ruling out concurrent physical problems
and maintaining or improving the overall functioning of the patient. Mental health services are helpful in
managing patients with more severe symptoms, often jointly with medical involvement.

https://mypastest.pastest.com/Secure/TestMe/Browser/436619[‫ ص‬05:25:42 08/12/1437]


MyPastest

43940

End Session

Previous Question Tag Question

Feedback

Difficulty: Average

Peer Responses

Session Progress

Responses Correct: 0

Responses Incorrect: 14

Responses Total: 14

Responses - % Correct: 0%

Blog
About Pastest
Contact Us
Help

© Pastest 2016

https://mypastest.pastest.com/Secure/TestMe/Browser/436619[‫ ص‬05:25:42 08/12/1437]


MyPastest

Prefer to use the old MyPastest? Access it here »

Back to Filters

Question 1 of 39

Which of the following statements about Wolff–Parkinson–White syndrome is most accurate?

A Digoxin is useful in the long-term treatment of patients with Wolff–Parkinson–White syndrome

B Radiofrequency ablation of the accessory pathway is potentially curable

C The
ECG of a patient with Wolff–Parkinson–White syndrome, when the patient is in normal rhythm, is
indistinguishable from normal

D Wolff–Parkinson–White
syndrome is most commonly found in patients with congenital heart disease,
most commonly with corrected transposition of the great arteries

E Wolff–Parkinson–White syndrome has an equal sex distribution

Explanation
Patients with this syndrome are prone to SVT from pre-excitation due to an anomalous atrioventricular conduction
pathway bypassing the junctional tissue. The accessory pathway allows a circuit to be formed, which facilitates a
re-entry tachycardia.

The ECG characteristics include a shortened PR interval due to rapid anterograde conduction, prolonged QRS
caused by premature action of the ventricle through the accessory pathway, followed by normal depolarisation
through the AV node and the bundle of His and delta wave
– slurring of the upstroke to the QRS complex.

Wolff–Parkinson–White syndrome (WPW) syndrome is usually associated with a structurally normal heart (70–
80%). It is more common in males (60–70%). There is a recognised association with Ebstein anomaly, corrected
transposition and cardiomyopathy.

Most patients present with tachycardia at a rate >250 beats/min and hypotension. Clinically crepitations in lung
bases may coexist secondary to pulmonary vascular congestion due to rate associated cardiac compromise.
Occasionally patients can present incidentally when characteristic ECG appearances are detected.

https://mypastest.pastest.com/Secure/TestMe/Browser/436619[‫ ص‬08:19:30 08/12/1437]


MyPastest

When a patient is known to have WPW syndrome, adenosine should be used only with extreme caution, having
the means to perform cardioversion/defibrillation immediately available due to the risk of deterioration of the
rhythm to VF as a result of blockage of the AV node
and the possibility of a partial or no effect on the accessory
pathway.
Digoxin is contraindicated for the same reason. Appropriate anti-arrhythmics include verapamil,
flecainide and procainamide.

Radiofrequency ablation of the accessory pathway is a potentially curative procedure for patients with WPW
syndrome.
11448

Tag Question

Feedback

Difficulty: Difficult

Peer Responses

Session Progress

Responses Correct: 0

Responses Incorrect: 1

Responses Total: 1

Responses - % Correct: 0%

Blog
About Pastest
Contact Us
Help

© Pastest 2016

https://mypastest.pastest.com/Secure/TestMe/Browser/436619[‫ ص‬08:19:30 08/12/1437]


MyPastest

Prefer to use the old MyPastest? Access it here »

Back to Filters

Question 2 of 39

What does a prominent left praecordium in a 16-year-old boy with an ejection murmur in the second left
intercostal space indicate?

A ASD with aortic regurgitation

B ASD with aortic stenosis

C ASD with mitral regurgitation

D ASD with pulmonary hypertension

E Uncomplicated ASD

Explanation
A prominent left praecordium suggests that the right ventricle was dilated during childhood, and also that it was
working against a high pressure. Ostium secondum atrial septal defect (ASD) in combination with rheumatic
mitral stenosis (Lutembacher’s
syndrome) can cause the same picture in advanced cases when there is pulmonary
hypertension.
12770

Tag Question

Feedback

Difficulty: Average

https://mypastest.pastest.com/Secure/TestMe/Browser/436619[‫ ص‬08:19:59 08/12/1437]


MyPastest

Peer Responses

Session Progress

Responses Correct: 0

Responses Incorrect: 2

Responses Total: 2

Responses - % Correct: 0%

Blog
About Pastest
Contact Us
Help

© Pastest 2016

https://mypastest.pastest.com/Secure/TestMe/Browser/436619[‫ ص‬08:19:59 08/12/1437]


MyPastest

Prefer to use the old MyPastest? Access it here »

Back to Filters

Question 3 of 39

A 17-year-old student, who has never been vaccinated against measles, presents to his GP with symptoms
suggestive of the disease. He is sent home and advised to rest, but later presents to A&E with anterior chest pain
that is worse on inspiration and relieved by sitting
forward. On examination there appears to be a rub on
auscultation.

What diagnosis fits best with this clinical picture?

A Viral pleurisy

B Pericarditis

C Myocardial ischaemia

D Pneumothorax

E Secondary bacterial pneumonia

Explanation
Pericarditis presents with anterior pleuritic chest pain, worse on inspiration and relieved by sitting forward. It is
associated with a pericardial friction rub, which is best heard when the
patient is upright and leaning forward.
There may be associated cardiac
tamponade, evidenced by tachycardia, low blood and pulse pressure and
distended neck veins. Pericarditis may be infectious in origin (viral, bacterial or fungal), inflammatory (e.g.
rheumatic fever, related to systemic lupus erythematosus (SLE), scleroderma or vasculitis), drug-induced,
(myocardial infarction-related), postradiotherapy, uraemic, neoplastic, related to sarcoid, or to a host of other
causes.

For
viral pericarditis, as in this case, limitation of activity is advised with additional pain relief using non-
steroidals. Prognosis varies according to the underlying cause, but recurrence of pericarditis occurs
in 10-15% of
patients within the first year.
12772

https://mypastest.pastest.com/Secure/TestMe/Browser/436619[‫ ص‬08:26:12 08/12/1437]


MyPastest

Tag Question

Feedback

Difficulty: Easy

Peer Responses

Session Progress

Responses Correct: 0

Responses Incorrect: 3

Responses Total: 3

Responses - % Correct: 0%

Blog
About Pastest
Contact Us
Help

© Pastest 2016

https://mypastest.pastest.com/Secure/TestMe/Browser/436619[‫ ص‬08:26:12 08/12/1437]


MyPastest

Prefer to use the old MyPastest? Access it here »

Back to Filters

Question 4 of 39

An Asian boy with a known history of rheumatic heart disease presents with low-grade fever for the past month.
He received a course of antibiotics from his GP a week ago.

Which of the following investigations would be most useful in the diagnosis?

A Blood culture

B Serological testing

C Echocardiogram

D C-reactive protein

E Full blood count

Explanation
Echocardiography is extremely useful in allowing vegetations in infective endocarditis to be seen. Although blood
cultures are a key diagnostic test in this condition, they may be negative if patients have recently received
antibiotic therapy. The same
reasoning applies to serological tests for Coxiella, Bartonella, Legionella, Chlamydia
and Brucella
species that may also cause infective endocarditis. Both CRP and polymorphonuclear leucocytosis
are non-specific tests that would be helpful and give further clues but not necessarily the diagnosis. 12773

Tag Question

Feedback

https://mypastest.pastest.com/Secure/TestMe/Browser/436619[‫ ص‬08:26:32 08/12/1437]


MyPastest

Difficulty: Average

Peer Responses

Session Progress

Responses Correct: 0

Responses Incorrect: 4

Responses Total: 4

Responses - % Correct: 0%

Blog
About Pastest
Contact Us
Help

© Pastest 2016

https://mypastest.pastest.com/Secure/TestMe/Browser/436619[‫ ص‬08:26:32 08/12/1437]


MyPastest

Prefer to use the old MyPastest? Access it here »

Back to Filters

Question 5 of 39

A 17-year-old boy presents with palpitations. His physical examination is normal except for a systolic murmur in
the second left intercostal space and prominent praecordial motion with a late second systolic impulse.

Which of the following conditions is he likely to have?

A Aortic stenosis

B Atrial septal defect

C Hypertrophic cardiomyopathy

D Mitral valve prolapse

E Mixed aortic valve disease

Explanation
A mid to late systolic impulse in the praecordial motion (triple ripple) is seen in patients with HOCM. The second
impulse
is due to contraction against a narrowed left ventricular outflow tract
and the third due to a late systolic
bulge in the ventricle near the end of systole. A hyperdynamic impulse is seen in mild to moderate aortic
regurgitation. Conditions A, B, D and E can produce the murmur but not this characteristic finding. 12774

Tag Question

Feedback

https://mypastest.pastest.com/Secure/TestMe/Browser/436619[‫ ص‬08:26:53 08/12/1437]


MyPastest

Difficulty: Average

Peer Responses

Session Progress

Responses Correct: 0

Responses Incorrect: 5

Responses Total: 5

Responses - % Correct: 0%

Blog
About Pastest
Contact Us
Help

© Pastest 2016

https://mypastest.pastest.com/Secure/TestMe/Browser/436619[‫ ص‬08:26:53 08/12/1437]


MyPastest

Prefer to use the old MyPastest? Access it here »

Back to Filters

Question 6 of 39

Which of the following conditions give rise to a systolic murmur with radiological signs of increased pulmonary
blood flow:

A Pulmonary Atresia

B Pulmonary stenosis

C Fallot's Tetralogy

D Ventricular septal defect

E Congenital aortic stenosis

Explanation
ASD and VSD cause a left to right shunt and therefore increased pulmonary blood flow. In pulmonary stenosis,
pulmonary atresia and Fallot's tetralogy the lung fields will appear oligaemic due to a right to left shunt. Aortic
stenosis causes an enlarged left ventricle.
14220

Tag Question

Feedback

Difficulty: Average

https://mypastest.pastest.com/Secure/TestMe/Browser/436619[‫ ص‬08:27:15 08/12/1437]


MyPastest

Peer Responses

Session Progress

Responses Correct: 0

Responses Incorrect: 6

Responses Total: 6

Responses - % Correct: 0%

Blog
About Pastest
Contact Us
Help

© Pastest 2016

https://mypastest.pastest.com/Secure/TestMe/Browser/436619[‫ ص‬08:27:15 08/12/1437]


MyPastest

Prefer to use the old MyPastest? Access it here »


Next Question

Back to Filters

Question 7 of 39

A 4-year-old girl has had recurrent blackouts over the previous six months. These were brief lasting less than a
minute during which some twitching of her hands and feet was noted. The recovery was rapid. These occurred
almost exclusively during the daytime, without warning when she was active. In the family, mother had similar
episodes until adolescence and a maternal uncle died suddenly in his teens. Clinically, and developmentally she
was normal. EEG, resting ECG, and serum biochemistry were normal.

The most likely diagnosis is?

A Generalised tonic/clonic epilepsy

B Reflex anoxic seizures

C Breath-holding attacks

D Long Q-T syndrome

E Sick sinus syndrome

Explanation
The episodes consist of sudden, brief loss of posture and awareness with rapid recovery that appear to be
precipitated
when active. They do not typify generalised epilepsy. There is no associated precipitant such as
painful knock or temper tantrum to suggest reflex anoxic seizures. The twitching noted may occur with syncopal
attacks whatever the cause. The family history is significant and points to a possible inherited abnormality. The
normal resting ECG does eliminate any evidence of heart block but continuous 24 hour Holter
monitoring is
needed to eliminate an arrhythmia. An exercise ECG will be helpful in the long Q-T syndrome, the proven
diagnosis. 14532

https://mypastest.pastest.com/Secure/TestMe/Browser/436619[‫ ص‬08:29:39 08/12/1437]


MyPastest

Tag Question

Feedback

Difficulty: Average

Peer Responses End Session

Previous Question

Session Progress

Responses Correct: 0

Responses Incorrect: 7

Responses Total: 7

Responses - % Correct: 0%

Blog
About Pastest
Contact Us
Help

© Pastest 2016

https://mypastest.pastest.com/Secure/TestMe/Browser/436619[‫ ص‬08:29:39 08/12/1437]


MyPastest

Prefer to use the old MyPastest? Access it here »


Next Question

Back to Filters

Question 8 of 39

Previous Question

A 4-week-old baby presents to the GP with poor feeding for 2 days. He is pale with poor peripheral perfusion on
arrival.

Which of the following is the single most useful thing to do in order to diagnose coarctation of the aorta?

A Auscultate the heart

B Measure four limb blood pressures

C Measure upper and lower limb saturations

D Palpate the femoral pulses

E Perform a chest X-ray

Explanation
Palpation of pulses is the diagnostic method of choice for coarctation of the aorta. Four limb blood pressure
measurements are often difficult, and not very accurate in babies but would be reasonable to attempt to measure if
coarctation was suspected. At this age, the child is likely to be symptomatic as the ductus is closing, so measuring
upper and lower limb saturation is reasonable to attempt , but they may not yield useful results. Auscultation is
rather non-specific when diagnosing coarctation. A CXR is not diagnostic at this age as rib notching is a chronic
finding due to the development of collateral vessels. 14533

Tag Question

Feedback

https://mypastest.pastest.com/Secure/TestMe/Browser/436619[‫ ص‬08:29:59 08/12/1437]


MyPastest

Difficulty: Average

Peer Responses

End Session

Session Progress

Responses Correct: 0

Responses Incorrect: 8

Responses Total: 8

Responses - % Correct: 0%

Blog
About Pastest
Contact Us
Help

© Pastest 2016

https://mypastest.pastest.com/Secure/TestMe/Browser/436619[‫ ص‬08:29:59 08/12/1437]


MyPastest

Prefer to use the old MyPastest? Access it here »

Back to Filters

Question 9 of 39

A Aortic valve stenosis


B Atrial septal defect
C Atrioventricular septal defect
D Coarctation of the aorta
E Normal heart
F Patent ductus arteriosus
G Pulmonary valve stenosis
H Ventricular septal defect
I Kartagener's syndrome
J Venous hum

For
each of the clinical scenarios below, choose the most likely diagnosis from the options above. Each option
may be used once, more than once, or
not at all.

Scenario 1

A 4-year-old girl presents to her GP with an upper respiratory tract infection. Her respiratory examination is
normal. An incidental finding in the cardiovascular examination is wide,
fixed splitting of the second heart sound
and a grade II out of VI ejection systolic murmur, loudest over the pulmonary area (upper left sternal edge).

Your answer was incorrect

Select one...

B - Atrial septal defect

In the first case a previously well child has an ASD. This is not an innocent murmur as the child has an
abnormality of the 2nd heart sound.
The
second heart sound should normally be split and the degree of splitting
varies with respiration, increasing with
insipration and decreasing with expiration. During inspiration when there is negative pressure in the thoracic
cavity, there is increased systemic venous return to the right, increasing the volume of blood in RV and delaying
closure of the pulmonary valve. When the RV is persistently volume loaded (e.g. ASD, PAPVD) there is delayed
closure which is no longer variable with inspiration (i.e. wide, fixed splitting).

https://mypastest.pastest.com/Secure/TestMe/Browser/436619[‫ ص‬08:30:20 08/12/1437]


MyPastest

Scenario 2

A 3-year-old boy presents to his GP with otitis media. He has otherwise been a well child. Cardiovascular
Next Question
examination reveals normal heart sounds and a short, soft systolic murmur at the lower left sternal edge, the
loudness of which varies on sitting and lying down.

Your answer was incorrect

Select one...
End Session
E - Normal heart

Previous Question
The second case is child who has a structurally normal heart but who has an innocent murmur. It will be easily
heard if the child is febrile or after exercise. The diagnosis is
helped by the information that the murmur changes
on position.

Innocent
(or functional murmurs) are present in the absence of anatomic abnormality. Children do not have
cardiac symptoms and ECG and chest X-ray are normal. They are soft (no thrill associated), and systolic
(exception is venous hum which has a diastolic component that is obliterated by gently pressing on the neck).
Heart sounds are normal with no ejection clicks. The murmurs do not radiate, they vary in intensity with posture
and are accentuated in high output states (e.g. fever, anaemia).

Scenario 3

A 6-week-old boy is brought to his GP for a routine check-up. He is asymptomatic and growing normally.
Cardiovascular examination reveals a thrill at the lower left sternal edge, normal heart sounds and a grade IV out
of VI pansystolic murmur at
the lower left sternal edge.

Your answer was incorrect

Select one...

H - Ventricular septal defect

This is a very classic presentation of a VSD.


A previously well child, who may or may not be thriving is found at
their 6-week check to have a murmur. The thrill at the lower left sternal edge and a pansystolic murmur are
characteristic of a VSD.
14534

Tag Question

Feedback

Difficulty: Average

https://mypastest.pastest.com/Secure/TestMe/Browser/436619[‫ ص‬08:30:20 08/12/1437]


MyPastest

Session Progress

Responses Correct: 0

Responses Incorrect: 11

Responses Total: 11

Responses - % Correct: 0%

Blog
About Pastest
Contact Us
Help

© Pastest 2016

https://mypastest.pastest.com/Secure/TestMe/Browser/436619[‫ ص‬08:30:20 08/12/1437]


MyPastest

Prefer to use the old MyPastest? Access it here »

Back to Filters

Question 10 of 39

Theme: Heart defects

A Aortic stenosis
B Atrial septal defect (ASD)
C Coarctation of the aorta
D Ebstein’s anomaly
E Innocent (flow) murmur
F Patent ductus arteriosus (PDA)
G Pulmonary stenosis
H Tetralogy of Fallot
I Transposition of the great arteries (TGA)
J Ventricular septal defect (VSD)

For
each of the following sets of clinical findings, choose the most likely
diagnosis from the above list. Each item
may be used once, more than once or not at all.

Exam question theme October 2012 and June 2015

Scenario 1

A 3-year-old child presents to his GP with a febrile illness and is noted to have a soft ejection systolic murmur at
the left sternal edge (LSE) only.

Your answer was incorrect

Select one...

E - Innocent (flow) murmur

This is an incidental finding in a presumably acyanotic, otherwise healthy 3-year-old. The murmur is soft, ejection
systolic at the LSE only, and so a flow murmur is the most likely diagnosis (a result of the increased flow because
the child is unwell). The GP should re-examine once this acute illness has resolved.

Scenario 2

A 12-hour-old newborn baby, at her discharge check, is noted to have an continuous murmur heard all over the

https://mypastest.pastest.com/Secure/TestMe/Browser/436619[‫ ص‬08:30:42 08/12/1437]


MyPastest

precordium. The femoral pulses are very easily palpable.

Your answer was incorrect


Next Question
Select one...

F - Patent ductus arteriosus (PDA)


End Session
This is not a cyanotic lesion. As she is only 12 hours old then the most likely diagnosis is a PDA that has yet to
close. Full femoral pulses are usually felt in PDA.

Scenario 3 Previous Question


A baby girl is seen for the 8-week check and is noted to have an ejection systolic murmur best heard in the
pulmonary area. The second heart sound does not vary with respiration. There is 1 cm hepatomegaly.

Your answer was incorrect

Select one...

B - Atrial septal defect (ASD)

Again this is an acyanotic lesion. The


murmur heard is a pulmonary flow murmur resulting from the increased
flow. The flow through the defect itself is not great enough (as atrial)
to produce a murmur. A fixed split of the
second heart sound is classic
of an ASD. Hepatomegaly is a characteristic sign of heart failure in infants and small
children.
22203

Tag Question

Feedback

Difficulty: Average

Session Progress

Responses Correct: 0

Responses Incorrect: 14

Responses Total: 14

Responses - % Correct: 0%

https://mypastest.pastest.com/Secure/TestMe/Browser/436619[‫ ص‬08:30:42 08/12/1437]


MyPastest

Blog
About Pastest
Contact Us
Help

© Pastest 2016

https://mypastest.pastest.com/Secure/TestMe/Browser/436619[‫ ص‬08:30:42 08/12/1437]


MyPastest

Prefer to use the old MyPastest? Access it here »


Next Question

Back to Filters

Question 11 of 39

Previous Question

An 18-month-old boy presents with poor feeding and tachypnoea


a week after coryzal illness. His cardiac
examination is unremarkable, other than a third heart sound being present. His chest radiograph shows
cardiomegaly and bilateral interstitial shadowing. FBC, renal function and anti-streptolysin O test (ASCOT) are
normal.

What is the most likely diagnosis?

A Coxsackie myocarditis

B Lyme disease

C Parvovirus B19 infection

D Rheumatic fever

E Kawasaki disease

Explanation
This clinical picture fits with cardiac failure. Myocarditis is an important cause of acquired heart failure. The
infective causes are commonly Coxsackie virus, influenza and adenoviruses, and bacterial causes as seen with
Borrelia burgdorferi
(Lyme disease). Rheumatic fever is unlikely if the ASOT is normal. A pancarditis may occur
as part of Kawasaki disease; however, this is unlikely to present in failure. 22244

Tag Question

Feedback

https://mypastest.pastest.com/Secure/TestMe/Browser/436619[‫ ص‬08:57:17 08/12/1437]


MyPastest

Difficulty: Average

Peer Responses

End Session

Session Progress

Responses Correct: 0

Responses Incorrect: 15

Responses Total: 15

Responses - % Correct: 0%

Blog
About Pastest
Contact Us
Help

© Pastest 2016

https://mypastest.pastest.com/Secure/TestMe/Browser/436619[‫ ص‬08:57:17 08/12/1437]


MyPastest

Prefer to use the old MyPastest? Access it here »


Next Question

Back to Filters

Question 12 of 39

A 12-year-old girl with a VSD presents with a week of fever, malaise and night sweats. When 2 years old she had
a rash with Amoxil prescribed for otitis media. She has recently had a holiday in Brittany but there are no known
contacts with infection although her mother works
in the local Emergency Department. Examination reveals a
temperature of
39°C, a grade 3 pansystolic murmur and some non-blanching erythematous maculopapular lesions
on her arms and legs.

The most relevant investigation is:

A Resting ECG

B Serum ASOT

C Urgent echocardiogram

D Chest radiograph

E Repeated blood cultures

Explanation
An ECG may show abnormalities such as right ventricular hypertrophy, but this will not explain the week-long
history
of symptoms. A raised ASOT will demonstrate past streptococcal infection, but the rash is not that of
rheumatic fever, i.e. erythema marginatum. The most likely diagnosis, which needs to be confirmed, is of infective
endocarditis; this needs to be proved by at least three blood cultures taken before antibiotic treatment is started. An
echocardiogram may show vegetations if present, but if absent does not exclude the diagnosis. 22370

Tag Question

https://mypastest.pastest.com/Secure/TestMe/Browser/436619[‫ ص‬08:57:48 08/12/1437]


MyPastest

Feedback

Difficulty: Average

Peer Responses
End Session

Previous Question

Session Progress

Responses Correct: 0

Responses Incorrect: 16

Responses Total: 16

Responses - % Correct: 0%

Blog
About Pastest
Contact Us
Help

© Pastest 2016

https://mypastest.pastest.com/Secure/TestMe/Browser/436619[‫ ص‬08:57:48 08/12/1437]


MyPastest

Prefer to use the old MyPastest? Access it here »

Back to Filters

Question 13 of 39

With regard to advanced life support and resuscitation in children with cardiac arrest, which of the following
statements is most factually correct?

A Basic
life support with high concentration oxygen should be continued throughout any resuscitation
with the minimum of interruptions

B Ventricular fibrillation is the most common cardiac arrest rhythm in children

C It
is important that a child's airway is secured during an arrest situation, so it is acceptable for basic life
support to be interrupted for as long as it takes to intubate the child

D The
resuscitation dose of adrenaline is 0.1 μg/kg and should be administered at 4-minute intervals in
asystolic arrests with the third dose being given as a higher dose of 1 mg/kg if no response is seen

E In
a paediatric arrest associated with PEA a 2 j/kg synchronised DC shock should be given every 2
minutes while reversible causes are being sought
and treated

Explanation
In all states of cardiac arrest the establishment and continuity of basic life support (BLS) should take precedence
over any advanced management.

Asystole is the most common arrest rhythm in children.

Pulseless
electrical activity (including electro-mechanical dissociation) should be treated as asystole. It can be
secondary to a treatable cause such as for example severe hypovolaemia or tension pneumothorax.

Ventricular
fibrillation is rare in childhood. Suspect in cases of sudden collapse
or with underlying cardiac disease,
hypothermia and/or overdose with a tricyclic antidepressant.
43784

https://mypastest.pastest.com/Secure/TestMe/Browser/436619[‫ ص‬09:00:14 08/12/1437]


MyPastest

Next Question
Tag Question

Feedback

End Session
Difficulty: Average

Peer Responses
Previous Question

Session Progress

Responses Correct: 0

Responses Incorrect: 17

Responses Total: 17

Responses - % Correct: 0%

Blog
About Pastest
Contact Us
Help

© Pastest 2016

https://mypastest.pastest.com/Secure/TestMe/Browser/436619[‫ ص‬09:00:14 08/12/1437]


MyPastest

Prefer to use the old MyPastest? Access it here »

Back to Filters

Question 14 of 39

A 14-year-old girl is referred to the paediatric outpatient clinic with a 4-month history of palpitations.

She
is normally fit and well and has not experienced any recent illness or weight loss. She is in her final year at an
all girls' private school but is generally achieving good grades in all her classes. She describes her symptoms as a
racing heart rate that comes on gradually and will slowly decrease back to normal after several hours. She has not
had associated chest pain or collapses and her symptoms usually present during school hours. She has had no
episodes associated with exercise or during her time at home. She has a sensation of difficulty breathing during the
attacks and has described 'pins and needles' in her
hands during her episodes. She has no significant past medical
history
of cardiac disease but does have mild asthma. She has no family history of heart disease, sudden death or
deafness. She had an entirely
normal examination including a normal pulse rate (70 beats/min regular); blood
pressure and 12-lead ECG in clinic were within normal limits.

Which of the following outcomes from this consultation is the most appropriate?

A Referral to paediatric cardiologist for further investigation of cardiac arrhythmia

B FBC,
U&Es, TFTs and 24-hour urinary catecholamine collection with follow-up in general paediatric
clinic to exclude medical causes for her
palpitations

C Reassurance
that her palpitations do not have a significant cause, advice about relaxation and stress
control during exam season, and discharge to follow-up from clinic

D Referral to child and adolescent psychiatry for treatment of anxiety disorder

E Initiation of propranolol treatment for panic attacks

Explanation
This young girl has got physiological tachycardia associated with situation-specific stress and requires strong
reassurance not further investigation or referral.

Palpitations are common in paediatrics. A minority will have true arrhythmias (13%).

https://mypastest.pastest.com/Secure/TestMe/Browser/436619[‫ ص‬09:00:50 08/12/1437]


MyPastest

Cardiac
arrhythmias are often described as having abrupt onset and offset. Children with premature atrial or
ventricular beats may describe a flip-flop feeling when the heart stops and then compensates with a larger stroke
volume on the following beat. Next Question
Palpitations associated with excessive periodic sweating, headaches and hypertension may suggest the rare
underlying diagnosis of phaeochromocytoma.

Palpitations with heat intolerance, sweating and weight loss may suggest hyperthyroidism.
End
Syncope
(with injury), exercise intolerance, and a past history or family history Session
of cardiac disease, sudden death or
deafness (PQT) may indicate significant underlying cardiac pathology.

Tachycardia out of proportion to fever may indicate myocarditis.


Previous Question
43785

Tag Question

Feedback

Difficulty: Average

Peer Responses

Session Progress

Responses Correct: 0

Responses Incorrect: 18

Responses Total: 18

Responses - % Correct: 0%

https://mypastest.pastest.com/Secure/TestMe/Browser/436619[‫ ص‬09:00:50 08/12/1437]


MyPastest

Blog
About Pastest
Contact Us
Help

© Pastest 2016

https://mypastest.pastest.com/Secure/TestMe/Browser/436619[‫ ص‬09:00:50 08/12/1437]


MyPastest

Prefer to use the old MyPastest? Access it here »

Back to Filters

Question 15 of 39

An 18-month-old girl is admitted to the paediatric assessment


unit with a 3-week history of worsening shortness of
breath and difficulty feeding accompanied by weight loss.

She had had symptoms of a cold and mild gastroenteritis a week before the onset of symptoms from which she had
initially appeared to recover. She had become increasingly lethargic, seemed unsettled and struggled to breathe
when laid flat, relieved by being in an up-right position or sleeping in a propped-up position in her buggy.

She had previously been a well, thriving child with no significant past medical history. On examination she had a
low-grade fever of 37.8oC and a heart rate of 190 beats/min while settled. She had increased work of breathing
with bibasilar crepitations. She had a respiratory rate of 50
beats/min with mild intercostal recession and normal
oxygen saturations
of 98% in ari. On auscultation of her heart no murmurs were heard but she was thought to have
a gallop rhythm. On examination of her abdomen a
4 cm palpable liver was found.

From the following list choose the most likely explanation for her current symptoms.

A Cystic fibrosis

B Large VSD with cardiac failure

C Pneumonia with hyperinflation of her thorax

D Supraventricular tachycardia

E Viral myocarditis

Explanation
This baby presents with a short history of new onset cardiac failure associated with faltering growth at 18 months
of age following shortly after a viral illness. This is consistent with an acquired cardiac failure not a congenital
cause:

Myocarditis - inflammation of the heart muscle

https://mypastest.pastest.com/Secure/TestMe/Browser/436619[‫ ص‬09:02:05 08/12/1437]


MyPastest

Cause of sudden death in previously healthy children


Can lead to acute dilated cardiomyopathy
Next Question
Viral
myocarditis is the leading cause in the western world - adenovirus (up to 39% of cases);
Coxsackievirus (Coxcackie B enterovirus)
Other causes include toxins, autoimmune processes, other infection (viral, bacterial, protozoal, parasitic)

Clinical presentation: prodomal viraemia (70 - 90%) - fever, myalgia, coryzalEnd


and/orSession
gastroenteritis. 43787

Previous Question

Tag Question

Feedback

Difficulty: Average

Peer Responses

Session Progress

Responses Correct: 0

Responses Incorrect: 19

Responses Total: 19

Responses - % Correct: 0%

Blog
About Pastest
Contact Us
Help

© Pastest 2016

https://mypastest.pastest.com/Secure/TestMe/Browser/436619[‫ ص‬09:02:05 08/12/1437]


MyPastest

https://mypastest.pastest.com/Secure/TestMe/Browser/436619[‫ ص‬09:02:05 08/12/1437]


MyPastest

Prefer to use the old MyPastest? Access it here »

Back to Filters

Question 15 of 39

An 18-month-old girl is admitted to the paediatric assessment


unit with a 3-week history of worsening shortness of
breath and difficulty feeding accompanied by weight loss.

She had had symptoms of a cold and mild gastroenteritis a week before the onset of symptoms from which she had
initially appeared to recover. She had become increasingly lethargic, seemed unsettled and struggled to breathe
when laid flat, relieved by being in an up-right position or sleeping in a propped-up position in her buggy.

She had previously been a well, thriving child with no significant past medical history. On examination she had a
low-grade fever of 37.8oC and a heart rate of 190 beats/min while settled. She had increased work of breathing
with bibasilar crepitations. She had a respiratory rate of 50
beats/min with mild intercostal recession and normal
oxygen saturations
of 98% in ari. On auscultation of her heart no murmurs were heard but she was thought to have
a gallop rhythm. On examination of her abdomen a
4 cm palpable liver was found.

From the following list choose the most likely explanation for her current symptoms.

A Cystic fibrosis

B Large VSD with cardiac failure

C Pneumonia with hyperinflation of her thorax

D Supraventricular tachycardia

E Viral myocarditis

Explanation
This baby presents with a short history of new onset cardiac failure associated with faltering growth at 18 months
of age following shortly after a viral illness. This is consistent with an acquired cardiac failure not a congenital
cause:

Myocarditis - inflammation of the heart muscle

https://mypastest.pastest.com/Secure/TestMe/Browser/436619[‫ ص‬09:09:40 08/12/1437]


MyPastest

Cause of sudden death in previously healthy children


Can lead to acute dilated cardiomyopathy
Next Question
Viral
myocarditis is the leading cause in the western world - adenovirus (up to 39% of cases);
Coxsackievirus (Coxcackie B enterovirus)
Other causes include toxins, autoimmune processes, other infection (viral, bacterial, protozoal, parasitic)

Clinical presentation: prodomal viraemia (70 - 90%) - fever, myalgia, coryzalEnd


and/orSession
gastroenteritis. 43787

Previous Question

Tag Question

Feedback

Difficulty: Average

Peer Responses

Session Progress

Responses Correct: 0

Responses Incorrect: 19

Responses Total: 19

Responses - % Correct: 0%

Blog
About Pastest
Contact Us
Help

© Pastest 2016

https://mypastest.pastest.com/Secure/TestMe/Browser/436619[‫ ص‬09:09:40 08/12/1437]


MyPastest

https://mypastest.pastest.com/Secure/TestMe/Browser/436619[‫ ص‬09:09:40 08/12/1437]


MyPastest

Prefer to use the old MyPastest? Access it here »

Back to Filters

Question 16 of 39

With regard to aortic stenosis, which of the following statements is most factually correct?

A An ejection click on auscultation suggests that the stenosis is supravalvular

B Aortic stenosis is the most common cardiac lesion associated with Down syndrome

C Right ventricular hypertrophy is a common finding in a child with aortic stenosis

D The most common treatment for aortic stenosis in childhood is aortic valve replacement

E There is an association with sudden death in patients with aortic stenosis

Explanation
Aortic valve stenosis accounts for 5% of congenital heart defects. It is more common in males and is associated
with Turner syndrome, Williams syndrome, Coarctation of the aorta and other cardiac abnormalities such as
hypoplastic left ventricle and mitral valve abnormalities.

The stenosis can be either supravalvular or subvalvular. There is often an associated bicuspid valve. It is usually
asymptomatic but in its most severe form can cause congestive cardiac failure, arrhythmias and sudden death in
infancy (though this is rare).

The murmur is best heard in the aortic area (upper right sternal edge) and radiates to the neck.

An
ejection click suggests valvular stenosis. A palpable thrill is usually
present in the suprasternal notch. A2 (the
aortic component of the second heart sound) is quiet.

Although left ventricular hypertrophy is common the ECG can be normal.

Treatment
is conservative in most cases avoiding valve replacement in the young patient. If the gradient across the
valve is >60 mmHg treatment is indicated. This is usually in the form of a balloon valvoplasty at cadiac catheter,
or surgical valvuloplasty.
43788

https://mypastest.pastest.com/Secure/TestMe/Browser/436619[‫ ص‬09:10:05 08/12/1437]


MyPastest

Next Question
Tag Question

Feedback

End Session
Difficulty: Average

Previous Question Peer Responses

Session Progress

Responses Correct: 0

Responses Incorrect: 20

Responses Total: 20

Responses - % Correct: 0%

Blog
About Pastest
Contact Us
Help

© Pastest 2016

https://mypastest.pastest.com/Secure/TestMe/Browser/436619[‫ ص‬09:10:05 08/12/1437]


MyPastest

Prefer to use the old MyPastest? Access it here »

Back to Filters

Question 17 of 39

Which of the following statements concerning transposition of the great arteries is most factually correct?

A The vast majority of cases of transposition of the great arteries are diagnosed on routine antenatal
ultrasonography

B Transposition of the great arteries is associated with a metabolic alkalosis at presentation

C Transposition
of the great arteries is associated with decreased pulmonary blood flow
and pulmonary
oligaemia on chest radiograph

D Transposition of the great arteries is the most common cause of cyanotic congenital heart disease in
the neonatal period

E Transposition of the great arteries occurs together with a recognised paediatric syndrome in 80% of
cases

Explanation
Transposition of the great arteries is the most common cause of cyanotic congenital heart disease in the neonatal
period and represents about 6% of all CHD. It is the most common cause of cyanotic CHD in the neonatal period
and is more common in boys than girls.

It is not usually associated with syndromes and there is usually a metabolic acidosis at presentation.

Antenatal
diagnosis on anomaly ultrasound scanning remains difficult and so the vast majority of cases are
diagnosed postnatally. The child is usually cyanotic from or shortly after birth. As the lesion is duct dependent, the
infant's condition deteriorates when the duct closes.

Later
presentation may occur where a lesion that mixes the oxygenated and deoxygenated blood is present (eg
PDA, ASD, VSD) in which case poor feeding and congestive cardiac failure will be the seen in later infancy.

A chest radiograph shows cardiomegaly and increased pulmonary vascularity.


43789

https://mypastest.pastest.com/Secure/TestMe/Browser/436619[‫ ص‬09:10:30 08/12/1437]


MyPastest

Next Question

Tag Question

Feedback
End Session
Difficulty: Average

Previous Question Peer Responses

Session Progress

Responses Correct: 0

Responses Incorrect: 21

Responses Total: 21

Responses - % Correct: 0%

Blog
About Pastest
Contact Us
Help

© Pastest 2016

https://mypastest.pastest.com/Secure/TestMe/Browser/436619[‫ ص‬09:10:30 08/12/1437]


MyPastest

Prefer to use the old MyPastest? Access it here »

Back to Filters

Question 18 of 39

Which of the following statements regarding tetralogy of Fallot is most factually correct?

A An atrial septal defect is one of the four major components of tetralogy of Fallot

B Cyanotic spells associated with tetralogy of Fallot usually begin around 4-6 months of age

C Finger clubbing usually develops within the first few months of life

D Most patients with tetralogy of Fallot have a left-to-right shunt across the VSD

E The murmur heart in a patient with tetralogy of Fallot becomes louder during cyanotic spells

Explanation
Cyanotic spells associated with tetralogy of Fallot (TOF) usually begin around 4-6 months of age.

The components of TOF are:

Ventricular septal defect


Right ventricular outflow obstruction
Right ventricular hypertrophy
Overriding aorta

The
severity of the right ventricular outflow obstruction determines the clinical picture. Most cases have faltering
growth and breathlessness on
feeding.
A mild obstruction - pink TOF denotes a left-to-right shunt across the VDS. Moderate obstruction presents with
cyanosis - right-to-left shunt across VSD. The murmur is ejection systolic due to pulmonary stenosis and the VSD
silent. Severe obstruction is duct dependent, presenting with cyanosis in the nonatal period.

Finger clubbing usually appears after the age of 1 year.

https://mypastest.pastest.com/Secure/TestMe/Browser/436619[‫ ص‬09:10:53 08/12/1437]


MyPastest

A feature of TOF is a reduction in the intensity of the murmur as cyanosis worsens.


43790

Next Question

Tag Question
End Session
Feedback

Previous Question Difficulty: Average

Peer Responses

Session Progress

Responses Correct: 0

Responses Incorrect: 22

Responses Total: 22

Responses - % Correct: 0%

Blog
About Pastest
Contact Us
Help

© Pastest 2016

https://mypastest.pastest.com/Secure/TestMe/Browser/436619[‫ ص‬09:10:53 08/12/1437]


MyPastest

Prefer to use the old MyPastest? Access it here »

Back to Filters

Question 19 of 39

Which of the following statements concerning atrial septal defects is most factually correct?

A A
pulmonary to systemic flow ratio of more than 2:1 is an indication for surgical or more commonly
percutaneous transcatheter device closure

B Atrial fibrillation is a common associated complication in the first decade of life

C In a secundum defect the ECG shows right bundle-branch block and left axis deviation

D In
children with ASDs the pulmonary vascular resistance increases in early
childhood, leading to
pulmonary hypertension typically in the first decade of life

E Ostium primum defects are more common than ostium secundum defects

Explanation
Isolated ASDs account for 8% of congenital heart disease. The defect is more common in girls and there are three
types:

1. Ostium secundum (most common)


2. Ostium primum
3. Sinus venosus

The
ECG appearance of atrial septal defects for ostium primum is right bundle-branch block, left axis deviation.
Ostium secundum shows right bundle-branch block with right axis deviation.

Atrial arrhythmias occur in adulthood but are rare in childhood.

Surgical
repair is indicated if the pulmonary-to-systemic flow ratio is >2:1.
The risk of pulmonary hypertension
increases with shunt size. Ostium secundum defects are usually asymptomatic, with pulmonary hypertension and

https://mypastest.pastest.com/Secure/TestMe/Browser/436619[‫ ص‬09:11:16 08/12/1437]


MyPastest

right ventricular failure occurring in the third and fourth decades.


43791

Next Question

Tag Question
End Session
Feedback

Previous Question Difficulty: Average

Peer Responses

Session Progress

Responses Correct: 0

Responses Incorrect: 23

Responses Total: 23

Responses - % Correct: 0%

Blog
About Pastest
Contact Us
Help

© Pastest 2016

https://mypastest.pastest.com/Secure/TestMe/Browser/436619[‫ ص‬09:11:16 08/12/1437]


MyPastest

Prefer to use the old MyPastest? Access it here »

Back to Filters

Question 20 of 39

Which of the following statements about ventricular septal defects (VSDs) is most factually correct?

A VSDs are most commonly located in the muscular part of the ventricular septum

B VSDs will cause an audible flow murmur across the defect, usually audible from birth

C Infective endocarditis is a complication seen in approximately 10% of all children with VSDs

D VSDs are associated with higher oxygen content in the blood of the right ventricle than the right
atrium

E VSDs are associated with right ventricular volume overload

Explanation
VSDs are the most common congenital heart
defect. The defect can occur in the membranous or muscular part of
the septum, although defects in the membranous septum are more common and usually single. Defects in the
muscular part of the septum are usually multiple.

The signs and symptoms depend on the haemodynamics of the defect, which depends on the size of the defect and
the pulmonary vascular resistance. With a small defect, the murmur is rarely present at birth but appears as the
pulmonary vascular resistance
falls.

Infective endocarditis occurs in less than 2% of cases and it is the left and not the right ventricle that is volume
overloaded. The shunt occurs mainly during systole when the right
ventricle is contracting, and so the shunted
blood enters the pulmonary
circulation.

The right atrium contains deoxygenated blood; the right ventricle contains deoxygenated blood from
the right
atrium and oxygenated blood from the left ventricle. At cardiac catheterisation, the oxygen content of blood in the
right ventricle is greater than that in the right atrium.
43822

https://mypastest.pastest.com/Secure/TestMe/Browser/436619[‫ ص‬09:11:37 08/12/1437]


MyPastest

Next Question
Tag Question

Feedback

End Session
Difficulty: Average

Peer Responses
Previous Question

Session Progress

Responses Correct: 0

Responses Incorrect: 24

Responses Total: 24

Responses - % Correct: 0%

Blog
About Pastest
Contact Us
Help

© Pastest 2016

https://mypastest.pastest.com/Secure/TestMe/Browser/436619[‫ ص‬09:11:37 08/12/1437]


MyPastest

Prefer to use the old MyPastest? Access it here »


Next Question

Back to Filters

Question 21 of 39

Which of the following statements about patent ductus arteriosus is most factually correct?

A The incidence of persistent ductus arteriosus is increased in males compared with females

B Cardiac catheter coil placement is the treatment of choice for symptomatic infants with persistent
ductus arteriosus post term

C PDA can be treated with prostaglandins

D PDA always closes spontaneously in term infants

E The incidence of PDA is increased in preterm infants and those with perinatal hypoxia and/or distress

Explanation
PDA is present in 40–50% of preterm
infants born weighing <1750 g, 30% of whom have a significant ductus
with congestive cardiac failure. The reason for the higher incidence in
preterm infants is that the responsiveness of
ductal smooth muscle is gestation dependent. Perinatal distress and/or hypoxia can delay the closure of the duct.

If not associated with predisposing factors PDA is more common in girls (ratio 2:1). In term infants functional
closure occurs within 10–15 hours of birth with
complete anatomical closure by 2–3 weeks of age. A persistence
of
the duct for 3 months after the baby has reached term is the definition
of a persistent ductus arteriosus.

Management of
PDA in preterm infants depends on symptoms. Spontaneous closure is likely if the duct is
asymptomatic. In symptomatic infants the management is fluid restriction, diuretics, maintenance of normal
haemoglobin and attempted medical closure with indometacin, ibuprofen or
surgical ligation. In the term infant
indometacin is not helpful and the principal therapeutic option is surgical ligation. Catheter closure of the duct is
indicated in children aged >1 year (occasionally younger). Spontaneous closure is not likely in term infants.
43823

https://mypastest.pastest.com/Secure/TestMe/Browser/436619[‫ ص‬09:12:01 08/12/1437]


MyPastest

Tag Question

Feedback

Difficulty: Average
End Session
Peer Responses

Previous Question

Session Progress

Responses Correct: 0

Responses Incorrect: 25

Responses Total: 25

Responses - % Correct: 0%

Blog
About Pastest
Contact Us
Help

© Pastest 2016

https://mypastest.pastest.com/Secure/TestMe/Browser/436619[‫ ص‬09:12:01 08/12/1437]


MyPastest

Prefer to use the old MyPastest? Access it here »


Next Question

Back to Filters

Question 22 of 39

Which of the following statements concerning coarctation of the aorta is most factually correct?

A Adult-type coarctation of the aorta is associated with blood pressure discrepancy between the upper
and lower limbs

B Coarctation is commonly associated with a tricuspid aortic valve

C Coarctation of the aorta is associated with rib notching on chest radiograph in infancy

D Coarctation of the aorta is more common in females

E It is rare to diagnose duct-dependent coarctation on antenatal ultrasonography

Explanation
Coarctation of the aorta accounts for 5% of congenital heart disease. It is more common in boys (2:1). It is
associated with cardiac anomalies such as bicuspid aortic valve (70%), mitral valve disease, subaortic stenosis and
VSD.

Adult-type
coarctation is usually just after the origin of the left subclavian artery (98%) at the level of the ductus
arteriosus. It can occur proximal to the origin of the left subclavian artery, in which case the blood pressure in the
right arm will be higher than in the left arm, the
classic discrepancy being between the upper and lower limb blood
pressure.

Rib notching does not appear until late childhood. There can be a classic '3' sign with a visible notch on
the chest
radiograph in the descending aorta.

In duct-dependent coarctation the lesion is often diagnosed antenatally on screening ultrasound examinations.
43825

https://mypastest.pastest.com/Secure/TestMe/Browser/436619[‫ ص‬09:12:23 08/12/1437]


MyPastest

Tag Question

Feedback

Difficulty: Average

Peer Responses
End Session

Previous Question

Session Progress

Responses Correct: 0

Responses Incorrect: 26

Responses Total: 26

Responses - % Correct: 0%

Blog
About Pastest
Contact Us
Help

© Pastest 2016

https://mypastest.pastest.com/Secure/TestMe/Browser/436619[‫ ص‬09:12:23 08/12/1437]


MyPastest

Prefer to use the old MyPastest? Access it here »


Next Question

Back to Filters

Question 23 of 39

Previous Question

Which of the following statements about childhood hypertension is most factually correct?

A A systolic pressure greater than the 75th centile for age and sex is defined as hypertension

B Children with primary hypertension are usually symptomatic

C In infants the most common cause of hypertension is renal parenchymal disease

D More than 25% of children with secondary hypertension will have symptoms of headache at
presentation

E Primary hypertension is more common than secondary hypertension in children

Explanation
Childhood hypertension is defined as systolic or diastolic blood pressure greater than the 95th centile for age,
recorded on three separate occasions.

There
are two types: primary (aetiology unknown) and secondary (aetiology known). Secondary hypertension is
more common in infants and younger children. Primary hypertension is more common than secondary
hypertension in adolescents and young adults when there is often a family history.

Children with primary hypertension are rarely symptomatic. Obesity is associated with primary hypertension, and
in 70–80% of children with secondary hypertension there is a renal cause.
43828

Tag Question

https://mypastest.pastest.com/Secure/TestMe/Browser/436619[‫ ص‬09:12:48 08/12/1437]


MyPastest

Feedback

Difficulty: Average

Peer Responses

End Session

Session Progress

Responses Correct: 0

Responses Incorrect: 27

Responses Total: 27

Responses - % Correct: 0%

Blog
About Pastest
Contact Us
Help

© Pastest 2016

https://mypastest.pastest.com/Secure/TestMe/Browser/436619[‫ ص‬09:12:48 08/12/1437]


MyPastest

Prefer to use the old MyPastest? Access it here »

Back to Filters

Question 24 of 39

Which of the following statements concerning infective endocarditis is most factually correct?

A Antibiotic prophylaxis is indicated to cover dental procedures in children with congenital heart disease

B Infective endocarditis is associated with a 13–20% mortality rate

C Splinter haemorrhages in the nailbeds are an almost universal finding

D The diagnosis of infective endocarditis is excluded if echocardiography is normal

E The right side of the heart is most commonly affected

Explanation
Infective endocarditis (IE) is defined as an infection of the lining of the heart, particularly affecting the heart
valves.

It
is associated with a 20% mortality rate and considerable morbidity. Lesions are usually left sided (except in
intravenous drug abusers).

The
clinical manifestations are often difficult and non-specific. 50% have
skin manifestations (caused by
circulating antibody/antigen complexes),
specifically Janeway lesions which are painless haemorrhages on the
soles and palms; Osler nodes which are painful nodules in the pads of the fingers and toes; and splinter
haemorrhages in nails.

Diagnosis
is on clinical suspicion, positive blood culture (which may need to be done repeatedly – three blood
cultures have a 95% pick-up rate) and by the demonstration of valvular vegetations on echocardiography.

Good
dental hygiene is essential. NICE (National Institute for Health and Clinical Excellence) no longer
recommends prophylactic antibiotics for routine procedures.

https://www.nice.org.uk/guidance/cg64 43829

https://mypastest.pastest.com/Secure/TestMe/Browser/436619[‫ ص‬09:13:10 08/12/1437]


MyPastest

Next Question
Tag Question

Feedback

End Session
Difficulty: Average

Peer Responses
Previous Question

Session Progress

Responses Correct: 0

Responses Incorrect: 28

Responses Total: 28

Responses - % Correct: 0%

Blog
About Pastest
Contact Us
Help

© Pastest 2016

https://mypastest.pastest.com/Secure/TestMe/Browser/436619[‫ ص‬09:13:10 08/12/1437]


MyPastest

Prefer to use the old MyPastest? Access it here »


Next Question

Back to Filters

Question 25 of 39

Which of the following statements about cardiomyopathy is the most factually correct?

A Dilated cardiomyopathy is associated with doxorubicin toxicity in children

B Endocardial fibroelastosis is a form of hypertrophic cardiomyopathy

C Hypertrophic obstructive cardiomyopathy is most commonly inherited as an X-linked disorder

D Restrictive cardiomyopathy is the commonest cardiomyopathy in childhood

E There is an increased incidence of dilated cardiomyopathy in infants of mothers with diabetes

Explanation
Dilated cardiomyopathy is associated with doxorubicin toxicity in children.

Endocardial fibroelastosis is a form of dilated cardiomyopathy and is rarely seen in childhood.

Hypertrophic
obstructive cadiomyopathy is inherited in an autosomal dominant manner in approximately 60% of
cases. A transient form is seen in infants of mothers with diabetes and infants on sterioids. Syndromic associations
include Beckwith-Weidemann and Noonan.

Restrictive
cardiomyopathy is one of the least commonest forms in childhood accounting for only around 3% of
cases. Dilated cardiomyopathy (at around 50% of cases) is the most common.

43831

Tag Question

https://mypastest.pastest.com/Secure/TestMe/Browser/436619[‫ ص‬09:13:35 08/12/1437]


MyPastest

Feedback

Difficulty: Average

Peer Responses
End Session

Previous Question

Session Progress

Responses Correct: 0

Responses Incorrect: 29

Responses Total: 29

Responses - % Correct: 0%

Blog
About Pastest
Contact Us
Help

© Pastest 2016

https://mypastest.pastest.com/Secure/TestMe/Browser/436619[‫ ص‬09:13:35 08/12/1437]


MyPastest

Prefer to use the old MyPastest? Access it here »

Back to Filters

Question 26 of 39

Which of the following statements about rheumatic fever is most factually correct?

A PR prolongation on the ECG is one of the major diagnostic criteria

B Prophylactic antibiotics should be stopped after 3 months

C Rheumatic fever is caused by an infection with group A α-haemolytic Streptococcus species

D The most common cardiac manifestation of rheumatic fever is an isolated endocarditis, occurring in
over 50% of cases

E The most commonly occurring major criterion for diagnosing rheumatic fever is polyarthritis

Explanation
Rheumatic fever develops secondary to infection with group A β-haemolytic Streptococcus sp.

Diagnosis
is by the Duckett–Jones criteria: two major OR one major and two minor criteria PLUS evidence of
recent streptococcal infection (raised ASO (antistreptolysin O) titres, anti-deoxyribonuclease B or positive throat
swab).

Major criteria:

Carditis: 50% (pancarditis affecting the pericardium, myocardium and endocardium)


Chorea:
15% (a late manifestation – purposeless and involuntary movements
and emotional lability, lasts
>6 months but has no long-term neurological sequelae)
Polyarthritis: 70% (migratory, involves large joints, transient with no long-term sequelae)
Erythema marginatum: 10% (pink rings on trunk and extensor surfaces of limbs)
Subcutaneous nodules: 1%

https://mypastest.pastest.com/Secure/TestMe/Browser/436619[‫ ص‬09:13:57 08/12/1437]


MyPastest

Minor criteria:

Arthralgia Next Question


Fever
Prolonged PR interval on ECG
Raised erythrocyte sedimentation rate (ESR) and C-reactive protein (CRP)

End Session
Pancarditis
affecting the pericardium, myocardium and endocardium is the most commonly occurring cardiac
manifestation and is seen in approximately 50% of cases. As a consequence of this 12.4% develop dysrhythmias,
and 6% develop heart block, tachycardia, cardiomegaly, congestive cardiac failure and valve disease. Its sequelae
Previous mitral
include mitral regurgitation, Question
stenosis, aortic regurgitation and tricuspid regurgitation.
The
most commonly occurring major criterion for diagnosing rheumatic fever is polyarthritis. This is seen in
approximately 70% of cases, is migratory, involves large joints but is transient with no long-term sequelae.

A 10 day course of penicillin is given for eradication of streptococci in the first instance and antibiotic prophylaxis
should be continued long term as a 3 - 4 weekly intramuscular dose of benzylpenicillin. It should continue for 10
years
in those with carditis or until aged 21 years (whichever is longer).
43833

Tag Question

Feedback

Difficulty: Average

Peer Responses

Session Progress

Responses Correct: 0

Responses Incorrect: 30

Responses Total: 30

https://mypastest.pastest.com/Secure/TestMe/Browser/436619[‫ ص‬09:13:57 08/12/1437]


MyPastest

Responses - % Correct: 0%

Blog
About Pastest
Contact Us
Help

© Pastest 2016

https://mypastest.pastest.com/Secure/TestMe/Browser/436619[‫ ص‬09:13:57 08/12/1437]


MyPastest

Prefer to use the old MyPastest? Access it here »

Back to Filters

Question 27 of 39

A 5-year-old girl is referred to a paediatric outpatient clinic for assessment of a murmur heard incidentally when
she was reviewed by her GP with a 48-hour history of fever and cough.

In clinic she is heard to have a 2/6 systolic murmur, heard loudest in the left sternal edge. The murmur is loudest
on lying flat but is audible in
the sitting position. She has no symptoms of breathlessness or exercise
intolerance.
She is active – dances and swims after school with her older siblings. There is no family history of cardiac disease
and she has never been admitted to hospital.

What is the most appropriate follow up for this young girl with regard to her murmur?

A ECG and chest radiograph

B Exercise stress test

C Immediate referral for paediatric cardiology review

D Outpatient referral to paediatric cardiology

E Reassurance that the murmur is innocent and discharge from follow-up

Explanation
It is essential that a paediatrician be familiar with the features of an innocent cardiac murmur. It is not appropriate
for all innocent murmurs to be seen in the cardiology clinic
because up to 70% of children may experience
innocent murmurs at some time in childhood. Cardiac referral and echocardiogram may aggravate parental anxiety
even if it is reported as normal. However, if the clinician is not confident of the diagnosis of an innocent murmur a
referral to a cardiologist is essential.

Features of an innocent murmur:

Localised
Poorly conducting

https://mypastest.pastest.com/Secure/TestMe/Browser/436619[‫ ص‬09:14:22 08/12/1437]


MyPastest

Musical/vibratory
Soft grade 1–2/6*
Next Question
Systolic*
Varies with posture
Present in high-output states, e.g. febrile illness, during exercise
Cardiac examination otherwise normal
Chest radiograph, ECG normal End Session

*Except venous hum.


Previous Question
Types of innocent murmurs are Still murmurs, Venous hum or Pulmonary flow murmurs.
43840

Tag Question

Feedback

Difficulty: Average

Peer Responses

Session Progress

Responses Correct: 0

Responses Incorrect: 31

Responses Total: 31

Responses - % Correct: 0%

https://mypastest.pastest.com/Secure/TestMe/Browser/436619[‫ ص‬09:14:22 08/12/1437]


MyPastest

Blog
About Pastest
Contact Us
Help

© Pastest 2016

https://mypastest.pastest.com/Secure/TestMe/Browser/436619[‫ ص‬09:14:22 08/12/1437]


MyPastest

Prefer to use the old MyPastest? Access it here »

Back to Filters

Question 28 of 39

A 3-month old child presents to the emergency department with


a 20-minute history of severe shock - pale but
responsive with a peripheral capillary refill time of 4 seconds and a central capillary refill time of 3 seconds.

She is found to have a tachycardia of 290 beats/min with a narrow complex on the ECG. Vascular access is
difficult to obtain. High-flow oxygen has been administered by the ambulance.

What is the most appropriate next management step given the most likely cardiac arrhythmia in this child?

A Amiodarone given via an intraosseous needle

B Asynchronous DC shock of 2 J/kg

C Intramuscular ceftriaxone

D Place an intraosseous needle and administer adenosine 100 μg/kg

E Synchronous shock of 1 J/kg

Explanation
Supraventricular tachycardia is the most common cardiac arrhythmia in the paediatric age group. In infants the
rate is usually >220 beats/min but can be lower in older children. The QRS complex is narrow. Classically the
onset is abrupt. It can last for a few minutes or several days, and is tolerated well by most children, although the
majority will develop cardiac failure if the arrhythmia persists.

If haemodynamically stable:

1. Vagal stimulation – facial immersion/unilateral carotid massage


2. Adenosine
into proximal vein with flush: – first dose, 100 μg/kg – second dose after 2 min 200 μg/kg – third
dose 300 μg/kg after 2 min – consider 400–500 μg/kg (>1 month of age)
3. Synchronous DC shock or amiodarone or procainamide.

If shock present:

https://mypastest.pastest.com/Secure/TestMe/Browser/436619[‫ ص‬09:14:44 08/12/1437]


MyPastest

1. Vagal manoeuvres if no delays


2. If vascular access immediately available give adenosine
3. If not give synchronous DC shock 1 J/kg Next Question
4. Further synchronised DC shock 2 J/kg
5. Consider amiodarone
43853

End Session

Tag Question
Previous Question
Feedback

Difficulty: Difficult

Peer Responses

Session Progress

Responses Correct: 0

Responses Incorrect: 32

Responses Total: 32

Responses - % Correct: 0%

Blog
About Pastest
Contact Us
Help

© Pastest 2016

https://mypastest.pastest.com/Secure/TestMe/Browser/436619[‫ ص‬09:14:44 08/12/1437]


MyPastest

Prefer to use the old MyPastest? Access it here »

Back to Filters

Question 29 of 39

Which symptom from the following list is the MOST important when considering a diagnosis of Kawasaki
disease?

A Bilateral exudative conjunctivitis

B Gallbladder hydrops

C High-grade fever for >5 days

D Petechial rash

E Thrombocytosis

Explanation
Without fever there can be no diagnosis of Kawasaki disease. It is an acute self-limiting systemic vasculitis with a
predilection for the coronary arteries.

The diagnosis is clinical.

The
definition is: fever of 5 days’ duration with no obvious underlying cause plus at least four of the following
five (or fever, coronary aneurysms and three of the five):

Bilateral conjunctivitis without exudate


Oral changes (strawberry tongue, fissuring of the lips)
Peripheral
extremity changes (erythema of palms or soles, swelling, desquamation of fingers or toes [1–2
weeks after the onset of fever])
Cervical adenopathy >1.5 cm – uncommon and may be unilateral
Pleomorphic rash (not vesicular) generalised

https://mypastest.pastest.com/Secure/TestMe/Browser/436619[‫ ص‬09:15:06 08/12/1437]


MyPastest

43864

Next Question

Tag Question

Feedback End Session

Difficulty: Average
Previous Question
Peer Responses

Session Progress

Responses Correct: 0

Responses Incorrect: 33

Responses Total: 33

Responses - % Correct: 0%

Blog
About Pastest
Contact Us
Help

© Pastest 2016

https://mypastest.pastest.com/Secure/TestMe/Browser/436619[‫ ص‬09:15:06 08/12/1437]


MyPastest

Prefer to use the old MyPastest? Access it here »


Next Question

Back to Filters

Question 30 of 39

Previous Question

Which is a sign of subacute bacterial endocarditis (SBE)?

A Cervical nodes

B Eosinophilia

C Extensor surface nodules

D Renal artery bruit

E Splinter haemorrhages

Explanation
SBE is a serious systemic infection with a cardiac focus. Underlying cardiac abnormalities (of valves or septa)
predispose to, but are not an imperative part of, the aetiology of this condition. Organisms include Streptococcus
viridans and Staphylococcus aureus. Clinical signs include splinters, retinal haemorrhages and changing cardiac
murmurs. Diagnosis rests on a positive blood culture (in practice at least three should be taken) and showing
vegetations on an echo (not 100% sensitive). Treatment consists of prolonged antibiotic
course (6 weeks
standard), valve replacement in cases of irreversible damage and (arguably) lifelong antibiotic prophylaxis,
although the NICE
guidelines published in 2008 refute the value of giving routine prophylaxis to any patient with
a condition that predisposes to SBE.

https://www.nice.org.uk/guidance/cg64 44008

Tag Question

https://mypastest.pastest.com/Secure/TestMe/Browser/436619[‫ ص‬09:15:28 08/12/1437]


MyPastest

Feedback

Difficulty: Average

Peer Responses

End Session

Session Progress

Responses Correct: 0

Responses Incorrect: 34

Responses Total: 34

Responses - % Correct: 0%

Blog
About Pastest
Contact Us
Help

© Pastest 2016

https://mypastest.pastest.com/Secure/TestMe/Browser/436619[‫ ص‬09:15:28 08/12/1437]


MyPastest

Prefer to use the old MyPastest? Access it here »


Next Question

Back to Filters

Question 31 of 39

Previous Question

A 3-month-old baby boy is admitted to hospital breathless with poor feeding. On arrival the main findings are:
pale with pulse of 240/min, liver enlarged at 3 cm below costal margin and saturation in air of 88%.

Which is the MOST appropriate first action?

A Application of ice pack over face

B Intranasal adenosine

C Intravenous adenosine

D Intravenous β blocker

E Oxygen

Explanation
This baby has SVT and is compromised cardiovascularly. This is more likely to be due to a re-entry phenomenon
than a structural defect. As in any cardiorespiratory emergency, an ABC
approach is the priority followed by
management to terminate the dysrhythmia. Vagal manoeuvres such as ice packs, carotid sinus massage (hard in a
baby) and ocular pressure may help to terminate the dysrhythmia. Parenteral adenosine given rapidly in escalating
doses is the drug treatment of choice if vagal methods fail, followed by synchronous DC shock. Recurrent SVT
may require longer-term anti-arrhythmic treatment (digoxin, flecainide, propranolol or amiodarone). In more
severe cases, pathway ablation is a final, usually successful, option.
44009

Tag Question

https://mypastest.pastest.com/Secure/TestMe/Browser/436619[‫ ص‬09:15:51 08/12/1437]


MyPastest

Feedback

Difficulty: Average

Peer Responses

End Session

Session Progress

Responses Correct: 0

Responses Incorrect: 35

Responses Total: 35

Responses - % Correct: 0%

Blog
About Pastest
Contact Us
Help

© Pastest 2016

https://mypastest.pastest.com/Secure/TestMe/Browser/436619[‫ ص‬09:15:51 08/12/1437]


MyPastest

Prefer to use the old MyPastest? Access it here »


Next Question

Back to Filters

Question 32 of 39

A 4-day-old baby girl is admitted from home with increasing cyanosis. On examination, she is deeply cyanosed
(saturation in air 78%), and has mild subcostal recession. She is alert and not distressed.

What is the MOST likely diagnosis?

A Diaphragmatic hernia

B Large ventricular septal defect

C Persistent fetal circulation

D Transposition of the great arteries

E Tricuspid atresia

Explanation
This baby clinically has cyanotic congenital heart disease. The key features are deep cyanosis without major
respiratory distress, indicative of a physiological right-to-left shunt. The most likely diagnosis is TGA. Until the
patent ductus closes there may be no symptoms or signs, but, as the pulmonary–systemic connection reaches
critical point, cyanosis becomes overt. Tetralogy of Fallot, pulmonary atresia, tricuspid atresia, anomalous
pulmonary venous
drainage and truncus arteriosus all give rise to a similar picture but are much less common. A
VSD is unlikely to cause symptoms at this age and would manifest as heart failure without cyanosis. Persistent
fetal circulation and diaphragmatic hernias present with a much ‘sicker’ baby who has respiratory signs and within
the first
few hours, if not sooner.
44010

https://mypastest.pastest.com/Secure/TestMe/Browser/436619[‫ ص‬09:16:14 08/12/1437]


MyPastest

Tag Question

Feedback

Difficulty: Average

Peer Responses
End Session

Previous Question

Session Progress

Responses Correct: 0

Responses Incorrect: 36

Responses Total: 36

Responses - % Correct: 0%

Blog
About Pastest
Contact Us
Help

© Pastest 2016

https://mypastest.pastest.com/Secure/TestMe/Browser/436619[‫ ص‬09:16:14 08/12/1437]


MyPastest

Prefer to use the old MyPastest? Access it here »

Back to Filters

Question 33 of 39

A 4-month-old is referred to the outpatient clinic with stridor since birth and faltering growth. The baby is thin
with a soft ejection systolic and diastolic murmur.

What is the investigation MOST likely to yield a unifying diagnosis in this clinical setting?

A Arteriography

B Barium swallow

C direct laryngoscopy

D ECG

E Sweat test

Explanation
Although ‘simple’ laryngomalacia is by far the most common cause of early stridor (and almost always self-
resolving), the picture in this case is different. This child is failing to thrive (always significant) and has a murmur
suggesting a vascular/aortic ring which is best elucidated by a barium swallow because it will compress the
oesophagus. You would, of course, when presented with this clinical scenario, require an expert cardiology
review
and echocardiogram to evaluate the underlying cardiac signs. In addition a respiratory or ENT bronchoscopic
evaluation of the upper airway may be helpful in due course.

Causes of stridor:

Intrinsic, e.g.

laryngomalacia
cord nodules/polyps
haemangiomas of the cords

https://mypastest.pastest.com/Secure/TestMe/Browser/436619[‫ ص‬09:16:34 08/12/1437]


MyPastest

cord palsies
laryngeal nerve palsy
Next Question
subglottic stenosis after prolonged intubation
laryngeal web

Extrinsic, e.g.
End Session
vascular ring
right atrial enlargement
cystic hygroma (obvious)
Previous Question
thyroid enlargement

44011

Tag Question

Feedback

Difficulty: Average

Peer Responses

Session Progress

Responses Correct: 0

Responses Incorrect: 37

Responses Total: 37

Responses - % Correct: 0%

https://mypastest.pastest.com/Secure/TestMe/Browser/436619[‫ ص‬09:16:34 08/12/1437]


MyPastest

Blog
About Pastest
Contact Us
Help

© Pastest 2016

https://mypastest.pastest.com/Secure/TestMe/Browser/436619[‫ ص‬09:16:34 08/12/1437]


MyPastest

Prefer to use the old MyPastest? Access it here »

Back to Filters

Question 34 of 39

A 2-year-old, fully immunised boy is referred with suspected measles. He has a week-long history of fever and
malaise and has developed a generalised morbilliform rash. On arrival he is miserable, febrile (temperature
40.1°C), cervical lymphadenopathy, conjunctivitis, desquamation over palms. Bloods apart from a high plasma
viscosity are normal.

What is the MOST appropriate first-line management?

A Full infection screen including lumbar puncture and urine and broad-spectrum antibiotic cover

B High-dose aspirin alone

C High-dose aspirin and parenteral immunoglobulin

D Parenteral steroids

E Vitamin A

Explanation
This child fulfils the criteria for Kawasaki disease (see comprehensive discussion elsewhere in this chapter). The
rash is often confused with measles (which was rare in the
UK but became more common again with the reduction
in MMR vaccine uptake). However, the extreme misery, desquamation and cervical nodes point in another
direction. Once suspected, treatment should be started without delay due to the risk of a coronary artery aneurysm
developing or enlarging with the increased duration of the febrile phase of the illness. Meta-analyses suggest that
high-dose aspirin (50 mg/kg per day in divided doses) until the fever subsides and a single dose of
immunoglobulin (2 g over 12 h i.v.) is the best management pending echocardiography, and this regimen is now
accepted practice.

Vitamin A at presentation is routine management of measles in developing country settings and reduces mortality
by up to 50%.
44012

https://mypastest.pastest.com/Secure/TestMe/Browser/436619[‫ ص‬09:16:56 08/12/1437]


MyPastest

Next Question
Tag Question

Feedback

End Session
Difficulty: Average

Peer Responses
Previous Question

Session Progress

Responses Correct: 0

Responses Incorrect: 38

Responses Total: 38

Responses - % Correct: 0%

Blog
About Pastest
Contact Us
Help

© Pastest 2016

https://mypastest.pastest.com/Secure/TestMe/Browser/436619[‫ ص‬09:16:56 08/12/1437]


MyPastest

Prefer to use the old MyPastest? Access it here »

Back to Filters

Question 35 of 39

You are urgently called to A&E to help resuscitate an 8 year old boy found unconscious in a local swimming pool.
He is intubated, ventilated and has defibrillator pads on his chest. Chest compressions are being continued by the
nursing staff. His ECG shows a polymorphic ventricular tachycardia with the QRS complexes appearing to ‘twist’
around the isoelectric line.

What drug should you give this boy as part of the APLS guidelines?

A Amiodarone

B Atropine

C Calcium

D Magnesium

E Sodium bicarbonate

Explanation
This boy has Torsades de Pointes (a form of polymorphic VT) and requires magnesium. Magnesium is a major
intracellular cation and needed as a co-factor for many enzyme reactions. It is an indicated treatment for children
with Torsades de Pointes / polymorphic VT from any cause and in documented hypomagnesaemia.

Amiodarone is a membrane-stabilising and anti-arrhythmic drug used in the treatment of shockable rhythms. It
should be given after the third defibrillation (IV 5mg/kg-1) with a repeat dose after the fifth shock if child is still
in VF/pVT.

Atropine is helpful in increasing the heart rate when bradycardia has been caused by increased vagal tone.
It has
not been shown to be useful in asphyxia bradycardia or systole and it is not routinely used in the APLS algorithm.

Calcium
should only be used when specifically indicated such as in hyperkalaemia, hypocalcaemia or an overdose
of calcium-channel – blocking drugs. It is otherwise dangerous in cardiac arrest as high levels can further harm the

https://mypastest.pastest.com/Secure/TestMe/Browser/436619[‫ ص‬09:17:18 08/12/1437]


MyPastest

ischaemic myocardium and impair cerebral recovery.

Sodium bicarbonate is not used routinely in CPR as it can exacerbate intracellular acidosis. It may be
considered
Next Question
in prolonged arrests, hyperkalaemia and tricyclic antidepressant overdose.

Website: https://www.resus.org.uk/resuscitation-guidelines/paediatric-advanced-life-support/#sequence
45423

End Session

Tag Question
Previous Question
Feedback

Difficulty: Average

Peer Responses

Session Progress

Responses Correct: 0

Responses Incorrect: 39

Responses Total: 39

Responses - % Correct: 0%

Blog
About Pastest
Contact Us
Help

© Pastest 2016

https://mypastest.pastest.com/Secure/TestMe/Browser/436619[‫ ص‬09:17:18 08/12/1437]


MyPastest

Prefer to use the old MyPastest? Access it here »

Back to Filters

Question 36 of 39

You are urgently called to A&E to help resuscitate an 8 year old boy found unconscious in a local swimming pool.
He is intubated, ventilated and has defibrillator pads on his chest. Chest compressions are being continued by the
nursing staff. His ECG shows a polymorphic ventricular tachycardia with the QRS complexes appearing to ‘twist’
around the isoelectric line. You stabilise him by following the APLS treatment guidelines.

What underlying abnormality are you likely to find on his baseline ECG?

A Coved ST segment elevation followed by a negative T wave

B Episodic SVT

C Premature atrial contractions

D Prolonged QT intervals

E Short PR interval and delta wave

Explanation
This boy has Torsades de Pointes, a specific form of polymorphic VT in patients with a long QT interval. This boy
is very likely to have congenital Long QT Syndrome, inherited as an autosomal dominant disorder with
incomplete penetrance. Torsades
de Pointes can be triggered by stress, strenuous exercise (particularly
swimming), loud noises and a slow heart rate during sleep. It is a leading cause of sudden cardiac death in young,
otherwise healthy people
and often thought to be an underlying cause of Sudden Infant Death Syndrome
(SIDS).Torsades de Pointes may be drug induced by tricyclic antidepressants, phenothiazides, and some antiviral
and antifungal drugs.

Brugada Syndrome (Type 1) is diagnosed on ECG with a coved ST segment elevation >2mm in >1 of V1-V3
followed by a negative T wave plus at least one of the following clinical criteria: documented VF or polymorphic
VT, Family history of sudden cardiac death <45 years old, coved-type ECG in family members,
syncope,
nocturnal agonal respiration, inducibility of VT with programmed electrical stimulation. It typically affects young

https://mypastest.pastest.com/Secure/TestMe/Browser/436619[‫ ص‬09:17:42 08/12/1437]


MyPastest

and middle-aged men and is also a leading cause of sudden cardiac death.

Episodic
SVT is the most common abnormal tachycardia in children and it usually is not life threatening.
Next Question
Treatment may be required for frequent or prolonged episodes.

A short PR interval and delta wave is diagnostic for Wolff-Parkinson-White (WPW). This is a congenital
condition with an abnormal electrical re-entry circuit including the AV node. Many children have no symptoms or
only occasional palpitations and SVT symptoms. WPW is also a cause of sudden
cardiac death.

Endand
Premature atrial contractions are
very common in normal children and adolescents Session
very rarely is there
any
underlying cause of significance or need to treat.
45424

Previous Question

Tag Question

Feedback

Difficulty: Average

Peer Responses

Session Progress

Responses Correct: 0

Responses Incorrect: 40

Responses Total: 40

Responses - % Correct: 0%

Blog
About Pastest
Contact Us
Help

© Pastest 2016

https://mypastest.pastest.com/Secure/TestMe/Browser/436619[‫ ص‬09:17:42 08/12/1437]


MyPastest

https://mypastest.pastest.com/Secure/TestMe/Browser/436619[‫ ص‬09:17:42 08/12/1437]


MyPastest

Prefer to use the old MyPastest? Access it here »

Back to Filters

Question 37 of 39

An 11 year old girl attends your clinic and describes episodes of feeling her heart racing and pounding, dizziness
and sweating. Her mum says she appears slightly pale and feels frightened when it happens. These episodes occur
at different times during the day,
a few times a week and do not have a clear trigger. She has no other known
medical problems.

What is the most likely underlying cause for these episodes?

A Atrial Flutter

B Atrial tachycardia

C Atrio-ventricular re-entrant tachycardia (AVRT)

D Atrio-ventricular nodal re-entrant tachycardia (AVNRT)

E Premature atrial contractions (PACs)

Explanation
This girl is presenting with features of supraventricular tachycardia (SVT). The most common form of SVT in
children over 8 years old is Atrio-ventricular nodal re-entrant tachycardia (AVNRT) due to an additional electrical
circuit in or near the AV node itself.

In atrial flutter is a form of re-entry tachycardia within the atria which then become unsynchronised with the
ventricles. It is most common in babies and children with congenital heart disease.

Atrial tachycardia is relatively uncommon in children where an area of atrium takes over the pacemaker activity of
the heart.

Atrio-ventricular
re-entrant tachycardia (AVRT) is due to an accessory electrical pathway
between the atria and
ventricles forming a re-entry circuit. This is the most common cause of SVT in children LESS than 8 years old.
Specific diagnoses of AVRT include Wolff-Parkinson-White but this is much rarer than AVNRT.

https://mypastest.pastest.com/Secure/TestMe/Browser/436619[‫ ص‬09:18:04 08/12/1437]


MyPastest

Premature atrial contractions (PAC) are very common in normal, healthy children and adolescents and give the
feeling of an occasional ‘skipped beat’. They are rarely associated with any underlying significant
pathology or
need to be treated. Next Question
45426

End Session
Tag Question

Feedback
Previous Question
Difficulty: Average

Peer Responses

Session Progress

Responses Correct: 0

Responses Incorrect: 41

Responses Total: 41

Responses - % Correct: 0%

Blog
About Pastest
Contact Us
Help

© Pastest 2016

https://mypastest.pastest.com/Secure/TestMe/Browser/436619[‫ ص‬09:18:04 08/12/1437]


MyPastest

Prefer to use the old MyPastest? Access it here »

Back to Filters

Question 38 of 39

You are asked to review the ECG of a 16 year old girl brought in unwell after a suspected beta-blocker overdose.

What ECG changes best support this history?

A Brisk sinus tachycardia and prolonged QTc

B QRS widening (>100 ms) and right axis deviation of the terminal QRS

C Sinus bradycardia, 1st degree AV block

D Subtle QRS widening

E Supraventricular tachycardia with slow ventricular response

Explanation
Beta-blockers (such as Atenolol, metoprolol, propranolol, sotalol) and cardioselective calcium-channel blockers
(verapamil, diltiazem) affect the ECG in toxicity by causing sinus bradycardia, 1st, 2nd, 3rd degree AV block,
junctional bradycardia
and ventricular bradycardia.

Massive carbamazepine overdose (>> 50mg/kg) is associated with cardiotoxicity due to fast sodium channel
blockade.This may be detectable on the ECG as subtle QRS widening or 1st degree AV block.The ECG changes
are not usually as dramatic as those seen in TCA overdose.

Digoxin
overdose may lead to a wide range of dysrhythmias, classically an SVT with slow ventricular response
(atrial tachycardia with block).

Quetiapine
(second generation atypical antipsychotic) overdose leads to characteristic ECG changes of brisk sinus
tachycardia and prolonged QTc.
Toxicity is serious and can lead to anticholinergic delirium and coma.

Tri-cyclic overdose (sodium channel blocker toxicity) ECG changes include interventricular conduction delay
(QRS widening) and right axis deviation of the terminal QRS. Tachycardia and
broad complex dysrhythmias may
also occur.

https://mypastest.pastest.com/Secure/TestMe/Browser/436619[‫ ص‬09:18:25 08/12/1437]


MyPastest

45584

Next Question

Tag Question

Feedback End Session

Difficulty: Average
Previous Question
Peer Responses

Session Progress

Responses Correct: 0

Responses Incorrect: 42

Responses Total: 42

Responses - % Correct: 0%

Blog
About Pastest
Contact Us
Help

© Pastest 2016

https://mypastest.pastest.com/Secure/TestMe/Browser/436619[‫ ص‬09:18:25 08/12/1437]


MyPastest

Prefer to use the old MyPastest? Access it here »

Back to Filters

Question 39 of 39

In the fetal circulation blood in the right atrium from the inferior vena cava (IVC) is separated into two parts.
Approximately one
third of the blood volume passes through the foramen ovale into the left atrium, left ventricle,
and ascending aorta. The remainder of the IVC blood flow passes into the right ventricle and pulmonary artery.

What is the major physiological benefit of this division of blood flow?

A Maintains high-flow blood to the high-resistance pulmonary bed.

B Maintains high PaO2 blood flow to coronary arteries and brain

C Maintains patency of the ductus arteriosus by flow-sensitive prostaglandin production

D Maintains patency of the ductus venosus by reducing PaCO2.

E Maintenance of low-pressure blood flow to protect the placental bed

Explanation
In the fetal circulation blood is oxygenated via the placenta and returns to the fetus through the umbilical vein.
Inside the fetal body, some blood passes through the liver to the hepatic vein and IVC. The remainder flows
through the ductus venosus, bypassing the liver, to enter the IVC. The IVC carries this oxygenated blood into the
right atrium. By then splitting the blood flow through the foramen ovale, high PaO2blood is directed via the
left
heart into the ascending aorta supplying coronary arteries and brain with richly oxygenated blood. This is the
major physiological benefit.

The pulmonary vascular bed is a relatively high-resistance, low-flow circuit in the fetus. There is no requirement
to maintain high flow and therefore answer A is incorrect.

Prostaglandins
in circulation and locally produced keep the ductus arteriosus patent during fetal life. There is no
evidence that the division of blood flow
from the right atrium has a major impact on this. The ductus arteriosus
closes after birth because of the rise in PaO2 rather than flow.

https://mypastest.pastest.com/Secure/TestMe/Browser/436619[‫ ص‬09:18:46 08/12/1437]


MyPastest

Answer D is incorrect as the ductus venosus channels oxygenated blood from the placenta through the liver, into
the IVC and right atrium. The division of right atrial blood flow does not have any known bearing on patency of
this duct or by impacting PaCO2. Functional closure occurs shortly after birth and its remnant is
known as the
ligamentum venosum.

Blood in the right atrium is not divided to reduce blood pressure to protect the placental bed.
45585

End Session

Previous Question Tag Question

Feedback

Difficulty: Average

Peer Responses

Session Progress

Responses Correct: 0

Responses Incorrect: 43

Responses Total: 43

Responses - % Correct: 0%

Blog
About Pastest
Contact Us
Help

© Pastest 2016

https://mypastest.pastest.com/Secure/TestMe/Browser/436619[‫ ص‬09:18:46 08/12/1437]


MyPastest

Prefer to use the old MyPastest? Access it here »

Back to Filters

Question 1 of 72

In a 3-year-old child, which one of the following developmental milestones is UNLIKELY to have been achieved?

A Build a tower of nine cubes

B Make good cuts with scissors

C Copy a circle

D Give full name

E Eat with fork and spoon

Explanation
A 3-year-old can go upstairs one foot per step and downstairs both feet per step. He can stand on one foot for a
few seconds and ride a tricycle. He can build a tower of nine cubes, can dress and undress himself with
supervision and thread beads. He can copy
a circle, understands adjectives of big and small, and knows some
nursery rhymes. He can give his full name and sex and eats well with a fork and spoon.

A 5-year-old should be able to make good cuts with scissors.

Exam question theme from June 2015

10417

Previous Question
Next Question
Tag Question

https://mypastest.pastest.com/Secure/TestMe/Browser/436619[‫ م‬11:15:09 08/12/1437]


MyPastest

Feedback

End Session
Difficulty: Average

Peer Responses

Session Progress

Responses Correct: 0

Responses Incorrect: 1

Responses Total: 1

Responses - % Correct: 0%

Blog
About Pastest
Contact Us
Help

© Pastest 2016

https://mypastest.pastest.com/Secure/TestMe/Browser/436619[‫ م‬11:15:09 08/12/1437]


MyPastest

Prefer to use the old MyPastest? Access it here »

Back to Filters

Question 2 of 72

A girl presents with dyspraxia. At mainstream school, they have noticed that she has difficulty with dressing,
undressing, doing up
her buttons and tying her shoelaces. She holds a pencil with a mature pincer grasp in her
right hand. She is able to copy a horizontal line, vertical line, circle, ladder, square, triangle and slanted rectangle,
but she has difficulty copying a diamond. She can draw her house, her father (with separate body and head) and a
ladder to stage II in all categories, and is able to write her first name but not her surname.

What is this girl’s developmental age with respect to her drawing fine motor skills?

A 5 years

B 5.5 years

C 6 years

D 6.5 years

E 7 years

Explanation
The following gives a rough estimation of a child’s drawing skills from the drawing category of the Ruth Griffiths
mental developmental scales, established from the modified Benet and Bender Gestalt test. Children are asked to
draw independently a
straight line, a horizontal line, a circle, a cross, a square or a man.
The ages at which these
are copied and notimitated are established as follows:

Can hold a pencil as if to mark a paper – 11 months


Uses pencil on paper a little – 13 months
Scribbles – 18–28 months
Circular scribble in imitation – 2.5–3 years
Perpendicular stroke or line in imitation – 2 years

https://mypastest.pastest.com/Secure/TestMe/Browser/436619[‫ م‬11:16:04 08/12/1437]


MyPastest

Perpendicular stroke without imitation – 3 years


Horizontal line – 3 years
Copies a circle – 3 years

Copies a ladder, stage I – 4 years


Draws a man, stage I, head and limbs (arms and legs) not all arising from the body – 4 years

Copies a square – 4-5 years


Draws a primitive house with windows, stage I – 5 years

Draws a man, head and body with arms and legs arising from appropriate areas – 6 years
Triangle – 5-6 years
Three letters – 6 years
Writes or prints first name – 6 years
Draws a house, stage II – 6 years
Draws a slanted rectangle – 6 years
Draws a precise ladder, stage II – 6 years

Draws a diamond – 7 years


Writes figures to 9 correctly – 7 years
Writes his or her full name – 7 years

Draws a blocked cross and circle/diamond touching – 8 years


Writes 10+ letters – 8 years
Draws a man with more defined features - ears, hair, hands and feet – 8 years

Draws a tube – 9 years

21746

Next Question

Tag Question

https://mypastest.pastest.com/Secure/TestMe/Browser/436619[‫ م‬11:16:04 08/12/1437]


MyPastest

Feedback End Session

Difficulty: Difficult

Peer Responses

Session Progress

Responses Correct: 0

Responses Incorrect: 2

Responses Total: 2

Responses - % Correct: 0%

Blog
About Pastest
Contact Us
Help

© Pastest 2016

Previous Question

https://mypastest.pastest.com/Secure/TestMe/Browser/436619[‫ م‬11:16:04 08/12/1437]


MyPastest

Prefer to use the old MyPastest? Access it here »

Back to Filters

Question 3 of 72

A 2-year-old Somali boy has attended a day nursery for the first time since arriving in the UK. The teacher has
found that the boy plays very much on his own and appears very shy. He does not seem to react to verbal
communication, loud sounds or songs sung around him. He appears to be a very happy child who gives good eye
contact; once engagement has been sought, he shows good interactive play. He sleeps for 6–7 hours during the day
as well as 12 hours at night. His four siblings suffer from otitis media, and his brother has been found eating dirt
and biting the paint off the side of the house.

Investigations
on the boy show normal electrolyte, C-reactive protein, full blood count and full septic screen
readings. Urine and serum toxicology prove negative. The only abnormality is a mildly raised thyroidstimulating
hormone level. An audiology review reports that he has bilateral sensorineural deafness.

What is a possible diagnosis for this boy?

A Lead encephalopathy

B Hypothyroidism

C Pendred’s syndrome

D Waardenburg’s syndrome

E Chronic suppurative otitis media

Explanation
Pendred’s syndrome is the most common form of inherited congenital deafness. It is inherited in an autosomal
recessive fashion and presents with a simple euthyroid goitre and mild hypothyroidism. The congenital deafness is
usually sensorineural in nature. Other genetic disorders associated with hearing loss include Down syndrome,
Velocardiofacial syndrome, Treacher Collins syndrome, Goldenhaar syndrome, Alport syndrome, CHARGE
syndrome and heterochromia.
Waardenburg syndrome is a rare autosomal dominant disorder, the deafness being
associated with pigmentary anomalies including a white forelock, heterochromia iridis and facial abnormalities.

https://mypastest.pastest.com/Secure/TestMe/Browser/436619[‫ م‬11:18:59 08/12/1437]


MyPastest

Acquired causes for hearing loss include chronic suppurative otitis media, commonly known as glue ear. This is
the most common cause of deafness and speech delay in children, causes including allergy, rhinitis, hay fever,
malformation of the eustachian tube and the accumulation of fluid
in the middle ear. Other infective causes
End Session
include otitis media, meningitis secondary to a Pneumococcus or Haemophilus influenzae infection and
encephalitis secondary to mumps or Haemophilus influenzae, but not toxoplasmosis, as this presents with
choroidoretinitis.
21756

Tag Question

Feedback

Difficulty: Average

Peer Responses

Session Progress

Responses Correct: 0

Responses Incorrect: 3

Responses Total: 3

Responses - % Correct: 0%

Blog
About Pastest
Contact Us
Help

© Pastest 2016 Previous Question


Next Question

https://mypastest.pastest.com/Secure/TestMe/Browser/436619[‫ م‬11:18:59 08/12/1437]


MyPastest

Prefer to use the old MyPastest? Access it here »

Back to Filters

Question 4 of 72

An infant presents to the GP for her routine check. During a developmental assessment, she is able to transfer a
cube from her left to her right hand and gains much excitement when placed in a forward or downward parachute
position. She no longer has a rooting, grasp or startle reflex, and her symmetrical Moro reflex has disappeared.
The infant continuously presents items to her mouth and sucks her fingers. She has a positive Babinski sign but a
negative asymmetrical tonic neck reflex.

What developmental age is this girl?

A 2 months

B 3 months

C 8 months

D 12 months

E 18 months

Explanation
The primitive reflexes are indicators of functional integrity and maturity. The majority of reflexes do not persist
beyond 6 months of age. There are 13 reflexes of importance, including:

Moro reflex
Startle reflex
Rooting reflex
Sucking reflex
Grasp reflex
Voluntary palmar grasp reflex

https://mypastest.pastest.com/Secure/TestMe/Browser/436619[‫ م‬11:19:24 08/12/1437]


MyPastest

Voluntary reach reflex


Stepping reflex
Asymmetrical tonic neck reflex (ATNR) End Session
Parachute reflex
Plantar reflex
Tendon reflex
Clonus

Note that gastrocolic reflex is not a primitive reflex.

Ninety-five per cent of reflexes will have disappeared by the ages shown in the table below.

Age Reflex lost

6 weeks Stepping

3-4 months Palmar grasp, Moro

6 months Sucking, rooting, asymmetrical tonic neck reflex (ATNR)

21758

Tag Question

Feedback

Difficulty: Average

Peer Responses

Previous Question
Next Question
Session Progress

Responses Correct: 0

Responses Incorrect: 4

https://mypastest.pastest.com/Secure/TestMe/Browser/436619[‫ م‬11:19:24 08/12/1437]


MyPastest

Responses Total: 4

Responses - % Correct: 0%

Blog
About Pastest
Contact Us
Help

© Pastest 2016

https://mypastest.pastest.com/Secure/TestMe/Browser/436619[‫ م‬11:19:24 08/12/1437]


MyPastest

Prefer to use the old MyPastest? Access it here »

Back to Filters

Question 5 of 72

Theme: Development regression

A Hypothyroidism
B Batten disease
C Aminoaciduria disorder
D Human immunodeficiency virus (HIV) encephalopathy
E Peroxisomal disorders
F Lead encephalopathy
G Subacute sclerosing panencephalopathy
H Spieler–Mayer–Schrögen syndrome
I Leigh’s encephalopathy
J Hydrocephalus secondary to a medulloblastoma

Three
children present to the child development centre for assessment of developmental regression. The diagnoses
listed above are possible causes
for developmental regression.

Match the three clinical scenarios to one diagnosis from the diagnostic suggestions above.

Scenario 1

A 3-year-old boy and his parents review a hospice. He presented previously with myoclonic jerks, optic atrophy
and
progressive dementia. His parents have found it increasingly difficult to control his myoclonic jerks and he is
becoming increasingly agitated,
requiring sedation. A bone marrow sample found lipofuscin in his marrow
cells.

What is his diagnosis?

Your answer was incorrect

Select one...

B - Batten disease

Scenario 2

A 14-year-old Iranian girl presents with a


6-month history of developmental regression. She has regressed in all
categories of her development and now wears nappies both day and night. She has ten words in her own language
and appears unable to communicate.
There is no previous medical history to note. She is unvaccinated. She has

https://mypastest.pastest.com/Secure/TestMe/Browser/436619[‫ م‬11:19:50 08/12/1437]


MyPastest

had all the usual childhood rashes. On clinical examination, she is well. Her Griffiths mental scale scoring portrays
her subquotient results as follows: gross motor 8.5 months, social skills 12 months, language and hearing 14
months, and hand and eye coordination 18 months;
she was unable to carry out the performance tasks.

What is the most likely cause of this girl’s developmental regression?

Your answer was incorrect

Select one...

G - Subacute sclerosing panencephalopathy

Scenario 3

An 8-year-old girl presents with a 12-month history of early morning headaches with associated nausea and
vomiting before breakfast. She is otherwise fit and well. Her friends have noticed that she does not participate in
lunchtime meals as she used to. On clinical examination, she is found to have increased reflexes and clonus on the
right side. On ophthalmic examination, there is papilloedema of the left eye.

What is her diagnosis?

Your answer was incorrect

Select one...

J - Hydrocephalus secondary to a medulloblastoma

The ten diagnoses are possible causes for a child presenting with developmental regression. Mitochondrial
disorders include Leigh’s encephalopathy and Batten disease. Batten disease often presents with myoclonic jerks,
dementia and optic atrophy, with lipofuscin found in the bone marrow cells.
Lysosomal
enzyme disorders include mucopolysaccharidoses, sphyngolipidoses (Gaucher’s disease) and glycogen
protein degradation disorders. These children present with abnormal facies, with or without hepatosplenomegaly.
Other lysosomal enzyme disorders include mucolipidosis, Niemann–Pick disease (sphyngomyelin and cholesterol
in the tissues, and foam cells present in bone marrow), and finally metachromic leukodystrophy, which includes
neuropsychoses with associated hyporeflexia and peripheral neuropathy.

Neurocutaneous disorders include neurofibromatosis and tuberous sclerosis. Metabolic disorders include
aminoacidurias, and infective causes such as herpes encephalopathy. Spieler–Mayer–Schrögen syndrome is
associated with decreased visual acuity, seizures, developmental deterioration, spasticity, dystonia and akinesia.

The above disorders may be a cause of a child presenting with developmental regression at less than 3 years of
age. Children presenting at over 3 years of age may have subacute sclerosing panencephalopathy secondary to
measles, mitochondrial disorders as above, or genetic disorders of the grey and white matter.
21761

Next Question

https://mypastest.pastest.com/Secure/TestMe/Browser/436619[‫ م‬11:19:50 08/12/1437]


MyPastest

Tag Question

Feedback End Session

Difficulty: Average

Session Progress

Responses Correct: 0

Responses Incorrect: 7

Responses Total: 7

Responses - % Correct: 0%

Blog
About Pastest
Contact Us
Help

© Pastest 2016

Previous Question

https://mypastest.pastest.com/Secure/TestMe/Browser/436619[‫ م‬11:19:50 08/12/1437]


MyPastest

Prefer to use the old MyPastest? Access it here »

Back to Filters

Question 6 of 72

Theme: Speech delay

A Tongue-tie
B Down syndrome
C Secretory otitis media
D Duchenne muscular dystrophy
E Autism
F Bilingual family
G Emotional deprivation
H Hypothyroidism
I Phenylketonuria
J Meningitis

The above are common causes of speech delay.

Match a diagnosis for each of the three clinical scenarios to the list above.

Scenario 1

A 3-year-old Somali girl is referred for poor hearing and an assessment for possible secretory otitis media. She
speaks only 10–20 words and has basic sentence construction. None of her seven siblings has a hearing problem.
On examination, she is quiet and withdrawn, slightly dishevelled but interacting well with other children.

What is the cause of her speech and language delay?

Your answer was incorrect

Select one...

G - Emotional deprivation

Scenario 2

A 3½-year-old boy is brought to the


GP with naughty behaviour. He does not carry out tasks asked of him and
does not listen to what he is told. Over the winter period, he has had recurrent febrile coryzal illnesses but has
otherwise remained well. He is found to have conductive hearing loss.

https://mypastest.pastest.com/Secure/TestMe/Browser/436619[‫ م‬11:20:22 08/12/1437]


MyPastest

What is his diagnosis?

Your answer was incorrect

Select one...

C - Secretory otitis media

Scenario 3

A blond, blue-eyed, 4½-year-old girl presents to the child development centre for a developmental assessment.
Her previous medical history is unremarkable, with a normal neonatal period and a normal Guthrie test. Her elder
brother has dyslexia and dyspraxia. She has a poor attention span with limited listening skills. She does not engage
fully with the assessor and gives poor eye contact. The girl is found to have marked expressive and receptive
language delay, with poor social skills; the score in all categories is reduced due to her severely impaired literacy
and numeracy
skills. On examination, she has poor speech, echolalia and repetitive movements, and is easily
distracted.

What is her diagnosis?

Your answer was incorrect

Select one...

E - Autism

In order to understand speech delay, speech milestones should be understood:

Age Achievement

3–6 months Tuneful vocalisations

Babble and repetitive syllables


6–12
months Mama, dada by 18 months said appropriately, by 12–14 months inappropriately

1–1.5 years Words understood in opposite context with a good understanding of language

1.5–2.5 Jargon and intelligible words. By 2 years, may join two or more words together; asks why? and
years where?

2.5–4 years Rapid speech development. Asks questions and may now direct speech to others

4+ years May narrate stories. Is able to use grammar and sentences by 4.5 years

Causes
of speech delay are many. Emotional deprivation is not an emotional disorder. It may be constitutional and
is due to understimulation and neglect. It often occurs in single-child families or in families with multiple siblings
where the youngest child is obviously neglected or spoken for by the elder siblings. It is not found in twins as they
often
speak to each other. Hearing deficit may be a cause of speech delay; causes include secretory otitis media
and deafness. Communication disorders are often associated with speech and language delay; this may include

https://mypastest.pastest.com/Secure/TestMe/Browser/436619[‫ م‬11:20:22 08/12/1437]


MyPastest

autistic spectrum disorder, elective mutism and bilingual children. Other important causes of speech and language
delay are listed
below.

Syndromes: End Session

Tuberous sclerosis
Down syndrome
Fragile X syndrome
Fetal alcohol syndrome

Neurological conditions:

Cerebral palsy
West syndrome
Duchenne muscular dystrophy

Metabolic disorders:

Phenylketonuria
Hypothyroidism

Infections:

Intrauterine infections (TORCH)


Meningitis

Differential diagnoses that affect but do not delay speech include tonguetie, cleft palate and malocclusion.
Cystinuria does not cause speech delay.
21762

Tag Question

Feedback

Previous Question Difficulty: Average


Next Question
Session Progress

Responses Correct: 0

Responses Incorrect: 10

https://mypastest.pastest.com/Secure/TestMe/Browser/436619[‫ م‬11:20:22 08/12/1437]


MyPastest

Responses Total: 10

Responses - % Correct: 0%

Blog
About Pastest
Contact Us
Help

© Pastest 2016

https://mypastest.pastest.com/Secure/TestMe/Browser/436619[‫ م‬11:20:22 08/12/1437]


MyPastest

Prefer to use the old MyPastest? Access it here »

Back to Filters

Question 7 of 72

Which one of the following gross motor milestones would you expect a normally developing 10-month-old infant
to have most recently acquired?

A Pulling to sit

B Pivots to reach objects

C Rolling over

D Walking up stairs with support

E Transferring hand to hand

Explanation
Neurodevelopment is a dynamic process. Infants achieve ‘milestones’ gradually; as an example they do not
suddenly sit unsupported one morning, having been unable to the previous
morning. There is an inherent variation
in when a child will achieve a ‘milestone’, and it is important to know the range of ages at which it is normal to
develop skills. If a child has not achieved a milestone by the age at which 90% of the population would be
expected to
have achieved it, then further assessment may be indicated. If a child is delayed in one of the
neurodevelopmental areas, that may well affect the development of another area.

Pulling to sit with no head lag: by 6 months


Sits steadily and pivots to reach objects: by 9 months
Rolling over: by 6 months
Walk up stairs with support: by 16/18 months (without support 24 months)
Transferring hand to hand (actually a fine motor milestone!): by 6 months

22230

https://mypastest.pastest.com/Secure/TestMe/Browser/436619[‫ م‬11:20:52 08/12/1437]


MyPastest

End Session
Tag Question

Feedback

Difficulty: Average

Peer Responses

Session Progress

Responses Correct: 0

Responses Incorrect: 11

Responses Total: 11

Responses - % Correct: 0%

Blog
About Pastest
Contact Us
Help

© Pastest 2016

Previous Question
Next Question

https://mypastest.pastest.com/Secure/TestMe/Browser/436619[‫ م‬11:20:52 08/12/1437]


MyPastest

Prefer to use the old MyPastest? Access it here »

Back to Filters

Question 8 of 72

Which one of the following fine motor milestones would you expect a normally developing 20-month-old infant to
have most recently acquired?

A Casting of objects

B Thumb/finger grip

C Banging two cubes together

D Building a tower of two cubes

E Feeding self with a spoon

Explanation

Casting of objects: by 15 months


Thumb and forefinger grasp: by 12 months
Banging two cubes together: by 12 months
Build tower of two cubes: by 15 months
Feed self with spoon: by 18 months

22231

Previous Question
Next Question
Tag Question

https://mypastest.pastest.com/Secure/TestMe/Browser/436619[‫ م‬11:21:14 08/12/1437]


MyPastest

Feedback

End Session
Difficulty: Average

Peer Responses

Session Progress

Responses Correct: 0

Responses Incorrect: 12

Responses Total: 12

Responses - % Correct: 0%

Blog
About Pastest
Contact Us
Help

© Pastest 2016

https://mypastest.pastest.com/Secure/TestMe/Browser/436619[‫ م‬11:21:14 08/12/1437]


MyPastest

Prefer to use the old MyPastest? Access it here »

End Session

Back to Filters

Question 9 of 72

Which one of the following language milestones would you expect 90% of normally developing children to have
most recently developed by 1 year?

A Turning to voice

B Imitates speech sounds

C Makes three word sentences

D Says dada/mama

E Recognises and points to some body parts

Explanation
Even very young infants recognise and respond to voice (by 8 weeks a baby should ‘still’ to their mother’s voice).
Over the first year an infant will learn noises and intonation and may not produce any words but may babble with
mature intonation. ‘Dada’ is usually the first ‘word’ as the ‘da’ sound is easier to produce than a ‘ma’. From 1
year, the number of words increases, with the child putting three or four words together in sentences by 3 years.
22233

Previous Question Tag Question

Feedback
Next Question

Difficulty: Average

https://mypastest.pastest.com/Secure/TestMe/Browser/436619[‫ م‬11:21:38 08/12/1437]


MyPastest

Peer Responses

Session Progress

Responses Correct: 0

Responses Incorrect: 13

Responses Total: 13

Responses - % Correct: 0%

Blog
About Pastest
Contact Us
Help

© Pastest 2016

https://mypastest.pastest.com/Secure/TestMe/Browser/436619[‫ م‬11:21:38 08/12/1437]


MyPastest

Prefer to use the old MyPastest? Access it here »

Back to Filters

Question 10 of 72

Which one of the following language milestones would you expect a normally developing 2.5-year-old child to
have most recently acquired?

A Understanding and following a two-step request

B Greater than 100-word vocabulary

C Three-to-four-word phrases

D Tuneful babbling

E Singing nursery rhymes

Explanation

Understanding and following a two-step request (e.g. pick up your toys and put them in the basket): by 2.5
years
> 100-word vocabulary: by 3 years
Three-to-four-word phrases: by 3 years
Tuneful babbling: by 9 months
Sing nursery rhymes: by 3 years.

22351

Next Question

Tag Question

https://mypastest.pastest.com/Secure/TestMe/Browser/436619[‫ م‬11:21:59 08/12/1437]


MyPastest

Feedback End Session

Difficulty: Difficult

Peer Responses

Session Progress

Responses Correct: 0

Responses Incorrect: 14

Responses Total: 14

Responses - % Correct: 0%

Blog
About Pastest
Contact Us
Help

© Pastest 2016

Previous Question

https://mypastest.pastest.com/Secure/TestMe/Browser/436619[‫ م‬11:21:59 08/12/1437]


MyPastest

Prefer to use the old MyPastest? Access it here »

Back to Filters

Question 11 of 72

Which one of the following social milestones would you expect a normally developing 15-month-old infant to
have most recently acquired?

A Plays alone

B Indicates toilet needs

C Wave bye-bye

D Dresses with supervision

E Finger feeds

Explanation

Plays alone: by 24 to 30 months


Indicates toilet needs: by 18 months
Wave bye-bye: by 12 months
Dresses with supervision: by 3 years
Finger feeds: by 9 months

22377

Previous Question
Next Question
Tag Question

https://mypastest.pastest.com/Secure/TestMe/Browser/436619[‫ م‬11:22:20 08/12/1437]


MyPastest

Feedback

End Session
Difficulty: Difficult

Peer Responses

Session Progress

Responses Correct: 0

Responses Incorrect: 15

Responses Total: 15

Responses - % Correct: 0%

Blog
About Pastest
Contact Us
Help

© Pastest 2016

https://mypastest.pastest.com/Secure/TestMe/Browser/436619[‫ م‬11:22:20 08/12/1437]


MyPastest

Prefer to use the old MyPastest? Access it here »

Back to Filters

Question 12 of 72

Which one of the following gross motor milestones would you expect 90% of normally developing children to
have most recently developed by 4 years?

A Bounces and catches a ball

B Skip

C Hop on one leg

D Pedals tricycle

E Walk down stairs one foot per step

Explanation

Bounces and catches a ball: by 5 years


Skip: by 5 years
Hop on one leg: by 4 years
Pedals tricycle: by 3 years
Walk down stairs one foot per step: 5 years

22379

Previous Question
Next Question
Tag Question

https://mypastest.pastest.com/Secure/TestMe/Browser/436619[‫ م‬11:22:43 08/12/1437]


MyPastest

Feedback

End Session
Difficulty: Average

Peer Responses

Session Progress

Responses Correct: 0

Responses Incorrect: 16

Responses Total: 16

Responses - % Correct: 0%

Blog
About Pastest
Contact Us
Help

© Pastest 2016

https://mypastest.pastest.com/Secure/TestMe/Browser/436619[‫ م‬11:22:43 08/12/1437]


MyPastest

Prefer to use the old MyPastest? Access it here »

End Session

Back to Filters

Question 13 of 72

A first time mother brings her 2-month-old baby to see you. She is concerned about his development.

Which one of the following statements is true regarding developmental milestones at 2 months of age?

A The infant is not expected to look at his parent

B The infant is not expected to calm himself by sucking on his hand

C The infant is not expected to mirror facial expressions like smiling

D The infant is not expected to get bored if not kept engaged

E The infant is not expected to turn towards sounds

Explanation
Developmental milestones are easy questions to ask in exams. In this case mirroring simple facial expressions
such as smiling or frowning is a milestone expected around 4 months of age.

http://www.cdc.gov/ncbddd/actearly/milestones/index.html 45381

Tag Question
Previous Question
Feedback Next Question

Difficulty: Average

https://mypastest.pastest.com/Secure/TestMe/Browser/436619[‫ م‬11:23:02 08/12/1437]


MyPastest

Peer Responses

Session Progress

Responses Correct: 0

Responses Incorrect: 17

Responses Total: 17

Responses - % Correct: 0%

Blog
About Pastest
Contact Us
Help

© Pastest 2016

https://mypastest.pastest.com/Secure/TestMe/Browser/436619[‫ م‬11:23:02 08/12/1437]


MyPastest

Prefer to use the old MyPastest? Access it here »

End Session

Back to Filters

Question 14 of 72

A nervous father brings his 4-month-old baby to see you. He is concerned about her development.

Which one of the following statements is true regarding developmental milestones at 4 months of age?

A Infants are not expected to let a parent know if they happy or sad by 4 months of age

B Smiling spontaneously is not expected at around 4 months of age

C The ability to reach for a toy with one hand is not expected at around 4 months of age

D The ability to respond to affection is not expected at around 4 months of age

E The ability to string vowels together whilst babbling is not expected at around 4 months of age

Explanation
Developmental milestones are commonly asked questions in exams such as these. The ability to string vowels
together whilst babbling is expected at around 6 months of age.

http://www.cdc.gov/ncbddd/actearly/milestones/index.html 45382

Tag Question
Previous Question
Feedback Next Question

Difficulty: Average

https://mypastest.pastest.com/Secure/TestMe/Browser/436619[‫ م‬11:23:24 08/12/1437]


MyPastest

Peer Responses

Session Progress

Responses Correct: 0

Responses Incorrect: 18

Responses Total: 18

Responses - % Correct: 0%

Blog
About Pastest
Contact Us
Help

© Pastest 2016

https://mypastest.pastest.com/Secure/TestMe/Browser/436619[‫ م‬11:23:24 08/12/1437]


MyPastest

Prefer to use the old MyPastest? Access it here »

End Session

Back to Filters

Question 15 of 72

A couple bring their 6-month-old baby to see you. They are concerned about her development.

Which one of the following statements is true regarding developmental milestones at 6 months of age?

A Can play peek-a-boo

B Recognises the word "no"

C They are weary of strangers

D They enjoy playing with their parents

E They have favourite toys

Explanation
Developmental milestones is essential reading for exams such as this.

At 6 months old children should enjoy playing with others, especially their parents. The other milestones are
expected of a 9-month-old.

http://www.cdc.gov/ncbddd/actearly/milestones/index.html 45383

Previous Question
Next Question Tag Question

Feedback

https://mypastest.pastest.com/Secure/TestMe/Browser/436619[‫ م‬11:23:46 08/12/1437]


MyPastest

Difficulty: Average

Peer Responses

Session Progress

Responses Correct: 0

Responses Incorrect: 19

Responses Total: 19

Responses - % Correct: 0%

Blog
About Pastest
Contact Us
Help

© Pastest 2016

https://mypastest.pastest.com/Secure/TestMe/Browser/436619[‫ م‬11:23:46 08/12/1437]


MyPastest

Prefer to use the old MyPastest? Access it here »

End Session

Back to Filters

Question 16 of 72

You see a 9-month-old child in clinic and are asked by her parents if she has reached her expected developmental
milestones.

Which one of the following statements is FALSE regarding her expected development?

A She is expected to be able to crawl

B She is expected to be able to wave goodbye

C She is expected to have a favorite toy

D She is expected to manoeuvre herself to a sitting position

E She is expected to understand "no"

Explanation
Developmental questions are commonplace in exams such that milestones should be well rehearsed. All the
statements but B are expected milestones for a 9-month-old infant. A child can be expected to wave goodbye at
around 1 year.

http://www.cdc.gov/ncbddd/actearly/milestones/index.html 45384

Previous Question
Next Question Tag Question

Feedback

https://mypastest.pastest.com/Secure/TestMe/Browser/436619[‫ م‬11:24:07 08/12/1437]


MyPastest

Difficulty: Average

Peer Responses

Session Progress

Responses Correct: 0

Responses Incorrect: 20

Responses Total: 20

Responses - % Correct: 0%

Blog
About Pastest
Contact Us
Help

© Pastest 2016

https://mypastest.pastest.com/Secure/TestMe/Browser/436619[‫ م‬11:24:07 08/12/1437]


MyPastest

Prefer to use the old MyPastest? Access it here »

End Session

Back to Filters

Question 17 of 72

You see a 1-year-old child in clinic. His mother is concerned about her development.

Which one of the following statements is true regarding developmental milestones?

A A 1-year-old child is expected to offer a limb when getting dressed

B A 1-year-old child is expected to eat using a spoon

C A 1-year-old child is expected to follow a simple command

D A 1-year-old child is expected to play simple pretend games such as feeding a teddy

E A 1-year-old child is expected to shake their head whilst saying "no"

Explanation
Answers B-E are milestones consistent with an 18 month old child.

http://www.cdc.gov/ncbddd/actearly/milestones/index.html 45385

Tag Question

Previous Question
Feedback
Next Question
Difficulty: Average

Peer Responses

https://mypastest.pastest.com/Secure/TestMe/Browser/436619[‫ م‬11:25:19 08/12/1437]


MyPastest

Session Progress

Responses Correct: 0

Responses Incorrect: 21

Responses Total: 21

Responses - % Correct: 0%

Blog
About Pastest
Contact Us
Help

© Pastest 2016

https://mypastest.pastest.com/Secure/TestMe/Browser/436619[‫ م‬11:25:19 08/12/1437]


MyPastest

Prefer to use the old MyPastest? Access it here »

End Session

Back to Filters

Question 18 of 72

You are talking to a group of parents on child health and development.

Which one of the following statements is FALSE with respect to seeking medical advice?

A Medical advice should be sought if an 18-month-old child begins to lose motor skills

B Medical advice should be sought if an 18-month-old child cannot walk

C Medical advice should be sought if an 18-month-old child cannot speak at least 6 words

D Medical advice should be sought if an 18-month-old child cannot use two-word phrases

E Medical advice should be sought if an 18-month-old child cannot learn new words

Explanation
Medical advice should be sought if a 24-month-old
child cannot use two-word phrases. The rest of the statements
are true.
A child of any age who begins to regress requires assessment by a paediatrician.

http://www.cdc.gov/ncbddd/actearly/milestones/index.html 45386

Tag Question
Previous Question
Feedback Next Question

Difficulty: Average

https://mypastest.pastest.com/Secure/TestMe/Browser/436619[‫ م‬11:25:41 08/12/1437]


MyPastest

Peer Responses

Session Progress

Responses Correct: 0

Responses Incorrect: 22

Responses Total: 22

Responses - % Correct: 0%

Blog
About Pastest
Contact Us
Help

© Pastest 2016

https://mypastest.pastest.com/Secure/TestMe/Browser/436619[‫ م‬11:25:41 08/12/1437]


MyPastest

Prefer to use the old MyPastest? Access it here »

End Session

Back to Filters

Question 19 of 72

You are talking to a group of parents and teachers on child development.

Which one of the following statements is true with respect to seeking medical advice?

A Medical advice should be sought if a 2-year-old child cannot appreciate what 'two' means

B Medical advice should be sought if a 2-year-old child cannot build a six block tower

C Medical advice should be sought if a 2-year-old child cannot communicate in sentences

D Medical advice should be sought if a 2-year-old child cannot undress himself

E Medical advice should be sought if a 2-year-old child does not recognise what to do with simple
objects like a spoon

Explanation
With the exception of not recognising simple objects such as a spoon, these statements apply to 3-year-olds. A 2-
year-child would be expected to know what to do with everyday objects like a fork or spoon.

http://www.cdc.gov/ncbddd/actearly/milestones/index.html 45387

Previous Question Tag Question


Next Question
Feedback

Difficulty: Average

https://mypastest.pastest.com/Secure/TestMe/Browser/436619[‫ م‬11:26:02 08/12/1437]


MyPastest

Peer Responses

Session Progress

Responses Correct: 0

Responses Incorrect: 23

Responses Total: 23

Responses - % Correct: 0%

Blog
About Pastest
Contact Us
Help

© Pastest 2016

https://mypastest.pastest.com/Secure/TestMe/Browser/436619[‫ م‬11:26:02 08/12/1437]


MyPastest

Prefer to use the old MyPastest? Access it here »

Back to Filters

Question 20 of 72

When discussing child development with a parent, which one of the following motor skills would NOT be
expected of a 3-year-old?

A Catching a bounced ball

B Climbing

C Riding a tricycle

D Running

E Walking upstairs

Explanation
A clear understanding of developmental milestones
can offer diagnostic clues. As such it is essential knowledge
for these
exams.

All but A would be expected developmental skills of a 3-year-old.

When
assessing development age in children it is important to determine the skills achieved in all four
developmental areas; Gross motor, Fine motor
and vision, Language and Social.

http://www.cdc.gov/ncbddd/actearly/milestones/index.html 45388

Previous Question
Next Question
Tag Question

Feedback

https://mypastest.pastest.com/Secure/TestMe/Browser/436619[‫ م‬11:26:23 08/12/1437]


MyPastest

End Session
Difficulty: Average

Peer Responses

Session Progress

Responses Correct: 0

Responses Incorrect: 24

Responses Total: 24

Responses - % Correct: 0%

Blog
About Pastest
Contact Us
Help

© Pastest 2016

https://mypastest.pastest.com/Secure/TestMe/Browser/436619[‫ م‬11:26:23 08/12/1437]


MyPastest

Prefer to use the old MyPastest? Access it here »

End Session

Back to Filters

Question 21 of 72

You are talking to a 3-year-old in the emergency department and ask her what she is called. She replies 'Emily
Smith'. The mother says that she always gives her full name since starting at nursery where
there are three girls
called Emily so she hears her surname frequently.

By what age should children be able to say their first and last name?

A 1 year old

B 2 years old

C 3 years old

D 4 years old

E 5 years old

Explanation
By 4 years of age children should be familiar with their first and last names. Additionally, 4-year-olds should also
be able to sing songs and tell stories.

http://www.cdc.gov/ncbddd/actearly/milestones/milestones-4yr.html 45389

Previous Question
Next Question Tag Question

Feedback

https://mypastest.pastest.com/Secure/TestMe/Browser/436619[‫ م‬11:26:45 08/12/1437]


MyPastest

Difficulty: Average

Peer Responses

Session Progress

Responses Correct: 0

Responses Incorrect: 25

Responses Total: 25

Responses - % Correct: 0%

Blog
About Pastest
Contact Us
Help

© Pastest 2016

https://mypastest.pastest.com/Secure/TestMe/Browser/436619[‫ م‬11:26:45 08/12/1437]


MyPastest

Prefer to use the old MyPastest? Access it here »

Back to Filters

Question 22 of 72

A 10-year-old boy is brought to clinic because of increasing unsteadiness on his feet. He is scared he needs glasses
as he cannot see
the board at school. He now sleeps with his light on as he cannot see anything in low light.

What other key symptoms do you need to look for to help you diagnose this peroxisomal disorder?

A Anosmia, hearing problems and itchy skin

B Loss of sensation in extremities, dysarthria and diabetes

C Numbness of the limbs, seizures and developmental delay

D Rapid chaotic eye movements, behaviour change and irritability

E Sweet smelling urine, lethargy and seizures

Explanation
This child is presenting with symptoms of
the peroxisomal disorder Refsum’s Disease. It is characterised by
anosmia, early onset retinitis pigmentosa (night blindness), chronic ataxia, variable neuropathy, deafness and
ichthyosis. Refsum's disease is an inherited disorder of fatty acid oxidationwith phytanic acid accumulation in the
blood and tissues, causing a motor and sensory neuropathy.

Answer B is suggestive of Friedrich’s Ataxia, the most common autosomal recessive cause of ataxia. Associated
features include dysarthria, scoliosis, diabetes and
hypertrophic cardiomyopathy.

Answer C suggests
a mitochondrial cytopathy such as NARP (Neuropathy, Ataxia and Retinitis Pigmentosa).
Learning difficulties, developmental delays and seizures are not uncommon, as with many mitochondrial
disorders.

Answer D suggests Opsoclonus-Myoclonus Syndrome which is thought to be a


parainfectious or paraneoplastic
(neuroblastoma) condition linked to an
abnormal immune response. Children present unwell with altered
behaviour, irritability, ataxia, random chaotic eye movements and later myoclonus.

https://mypastest.pastest.com/Secure/TestMe/Browser/436619[‫ م‬11:27:07 08/12/1437]


MyPastest

Answer E suggests Maple Syrup Urine Disease (MSUD), an autosomal recessive organic acidaemia. There is a
distinct sweet odour to the urine of affected individuals, particularly at times of acute illness. Without treatment
MSUD can lead to seizures, brain damage, coma and death. The most common and classic form affects babies
End Session
shortly after birth but variant forms may not be evident until later childhood.

Remembered exam theme October 2015


45427

Tag Question

Feedback

Difficulty: Average

Peer Responses

Session Progress

Responses Correct: 0

Responses Incorrect: 26

Responses Total: 26

Responses - % Correct: 0%

Blog
About Pastest
Contact Us
Help

© Pastest 2016 Previous Question


Next Question

https://mypastest.pastest.com/Secure/TestMe/Browser/436619[‫ م‬11:27:07 08/12/1437]


MyPastest

Prefer to use the old MyPastest? Access it here »

Back to Filters

Question 23 of 72

A 4-month-old baby girl is brought to hospital with increasing weight loss over the past month due to vomiting,
diarrhoea and poor feeding. On examination she is hypotonic, has poor head control, weak suck and you note
some involuntary muscle contractions. On the ward the baby has repeated episodes of abnormal breathing,
nystagmus and is found to have a high lactic acidosis.

Which imaging investigation would be the most helpful towards a diagnosis?

A Abdominal ultrasound

B CT head and spine

C Eye examination and retinal pictures

D MRI brain

E X-ray abdomen and chest

Explanation
This baby is presenting with failure to thrive, vomiting, diarrhoea and poor feeding plus developmental concerns,
abnormal movements, breathing problems and metabolic disturbance. This picture raises particular concerns about
metabolic disorders and mitochondrial disorders, particularly given the young age and history of deterioration over
a few weeks.

Infection and congenital abnormalities of the GI tract should be considered (such as with X rays and ultrasound),
but when making a diagnosis for this child all the features need to be taken into account and her brain must be
imaged. An MRI brain scan should show the characteristic lesions of the brain (including basal ganglia), brainstem
and demyelination in Leigh Syndrome (necrotising subacute encephalomyelopathy) caused by mitochondrial
DNA abnormalities. MRI scans are more sensitive than CT scans in detecting brain and soft tissue abnormalities.

Expert eye examination and


imaging can be very valuable when diagnosing syndromes, such as the characteristic
cherry red spot in TaySach’s Disease. In multi-system presentations with neurological concerns as above eye

https://mypastest.pastest.com/Secure/TestMe/Browser/436619[‫ م‬11:27:30 08/12/1437]


MyPastest

examination is less likely to be the key diagnostic imaging of choice although it may well be part of the diagnostic
workup.

Remembered question theme from October 2015 End Session


45428

Tag Question

Feedback

Difficulty: Average

Peer Responses

Session Progress

Responses Correct: 0

Responses Incorrect: 27

Responses Total: 27

Responses - % Correct: 0%

Blog
About Pastest
Contact Us
Help

© Pastest 2016

Previous Question
Next Question

https://mypastest.pastest.com/Secure/TestMe/Browser/436619[‫ م‬11:27:30 08/12/1437]


MyPastest

Prefer to use the old MyPastest? Access it here »

Back to Filters

Question 24 of 72

Which of the following developmental milestones would be expected of a normal 6-month-old child?

A Crawling

B Looks for a hidden toy

C Puts everything to his/her mouth

D Shows strong hand preference

E Uses a pincer grasp

Explanation
Hand preference does not begin to be clearly demonstrated until 18 months (if it occurs before this it can indicate
cerebral palsy).

A fine pincer grip is not normally present until about 10 months to one year.

Provided a child can crawl by 12 months of age, there is no cause for concern.

A normal 6-month-old:

Motor

Can roll over


Sits briefly with support

Vision / fine movement

Transfers and reaches out for objects

https://mypastest.pastest.com/Secure/TestMe/Browser/436619[‫ م‬11:27:50 08/12/1437]


MyPastest

Puts everything to mouth

Hearing / speech End Session

Quietens to mum's voice, excited at sounds he recognises, babbles


Will turn when name is called

Social

Smiles, coos, shows enjoyment - and screams in annoyance!


Plays with feet

45724

Tag Question

Feedback

Difficulty: Average

Peer Responses

Session Progress

Responses Correct: 0

Responses Incorrect: 28

Responses Total: Previous Question 28

Responses - % Correct: Next Question 0%

https://mypastest.pastest.com/Secure/TestMe/Browser/436619[‫ م‬11:27:50 08/12/1437]


MyPastest

Blog
About Pastest
Contact Us
Help

© Pastest 2016

https://mypastest.pastest.com/Secure/TestMe/Browser/436619[‫ م‬11:27:50 08/12/1437]


MyPastest

Prefer to use the old MyPastest? Access it here »

End Session

Back to Filters

Question 25 of 72

Which one of the following developmental milestones would be likely to have been most recently acquired by a
normal 4-year-old?

A Build a tower of 9 cubes

B Copy a square with a pencil

C Dresses and undresses self if helped with buttons

D Ride a tricycle unaided

E Stands on one foot for a few seconds

Explanation
By 4 years of age a child should be able to copy a cross and a square.

Building towers of bricks up to 8 or 9 high is accomplished by around 2 1/2 years of age.

Dressing with supervision is achieved around age 3, as is pedalling a tricycle and momentarily balancing on one
foot.

45725

Previous Question
Next Question
Tag Question

Feedback

https://mypastest.pastest.com/Secure/TestMe/Browser/436619[‫ م‬11:28:10 08/12/1437]


MyPastest

Difficulty: Average

Peer Responses

Session Progress

Responses Correct: 0

Responses Incorrect: 29

Responses Total: 29

Responses - % Correct: 0%

Blog
About Pastest
Contact Us
Help

© Pastest 2016

https://mypastest.pastest.com/Secure/TestMe/Browser/436619[‫ م‬11:28:10 08/12/1437]


MyPastest

Prefer to use the old MyPastest? Access it here »

End Session

Back to Filters

Question 26 of 72

When testing reflexes which one of the following would you not expect to find in a new born, full term baby?

A Moro

B Parachute

C Rooting

D Stepping

E Upgoing plantars

Explanation
The Moro, stepping and rooting reflexes are present from around 28 weeks of gestation. The parachute reflex
develops from 9 months of age and serves as a protective mechanism as the child begins to sit and learn to crawl
or walk.

Upgoing plantars are a normal finding in infants.


45726

Tag Question
Previous Question
Feedback Next Question

Difficulty: Average

https://mypastest.pastest.com/Secure/TestMe/Browser/436619[‫ م‬11:28:29 08/12/1437]


MyPastest

Peer Responses

Session Progress

Responses Correct: 0

Responses Incorrect: 30

Responses Total: 30

Responses - % Correct: 0%

Blog
About Pastest
Contact Us
Help

© Pastest 2016

https://mypastest.pastest.com/Secure/TestMe/Browser/436619[‫ م‬11:28:29 08/12/1437]


MyPastest

Prefer to use the old MyPastest? Access it here »

Back to Filters

Question 27 of 72

Which of the following developmental milestones would you expect to see in a normal 6-month-old baby?

A Be able to hold an object between finger and thumb

B Continue to display the stepping relex

C Have lost the Moro reflex

D Sit steadily

E Show hand dominance

Explanation
The Moro reflex is present from birth and
persists until 4 months of age. It is abnormal for the Moro reflex to
persist beyond this stage and if it does, cerebral palsy should be considered. You would therefore expect this to
have disappeared by 6-months.

A normal 6-month-old baby should be able to roll over from front to back, readily follow objects with their
eyes
and transfer objects from hand to hand, recognise parental voices and babble and laugh.

Pincer grip is usually seen from the age of 9 to 12 months and babies sit steadily by 9 months.

The stepping reflex generally disappears around the age of 2 months.

Hand dominance is unlikely to emerge before the age of 18 months.


45727

Next Question

Tag Question

https://mypastest.pastest.com/Secure/TestMe/Browser/436619[‫ م‬11:28:51 08/12/1437]


MyPastest

Feedback End Session

Difficulty: Average

Peer Responses

Session Progress

Responses Correct: 0

Responses Incorrect: 31

Responses Total: 31

Responses - % Correct: 0%

Blog
About Pastest
Contact Us
Help

© Pastest 2016

Previous Question

https://mypastest.pastest.com/Secure/TestMe/Browser/436619[‫ م‬11:28:51 08/12/1437]


MyPastest

Prefer to use the old MyPastest? Access it here »

Back to Filters

Question 28 of 72

A child should be investigated further if they are not:

A Dry at night by 3 years of age

B Saying single words with meaning by 18 months

C Sitting unsupported by 6 months of age

D Smiling by 4 weeks of age

E Walking unaided at 12 months of age

Explanation
At 6 months old, a child would be expected to pull to sit and stay sitting momentarily or with support.

Smiling usually occurs by six weeks of age.

Walking occurs anytime from 12-18 months.

The failure of children to say single words at 18 months should raise concern.

Children
are usually expected to be dry by day at the age of 3 years and dry by night at the age of 4 years, however
10% of 5 year old children and 5% of 10 year old children still wet the bed.
45728

Previous Question
Next Question
Tag Question

Feedback

https://mypastest.pastest.com/Secure/TestMe/Browser/436619[‫ م‬11:29:14 08/12/1437]


MyPastest

End Session
Difficulty: Average

Peer Responses

Session Progress

Responses Correct: 0

Responses Incorrect: 32

Responses Total: 32

Responses - % Correct: 0%

Blog
About Pastest
Contact Us
Help

© Pastest 2016

https://mypastest.pastest.com/Secure/TestMe/Browser/436619[‫ م‬11:29:14 08/12/1437]


MyPastest

Prefer to use the old MyPastest? Access it here »

End Session

Back to Filters

Question 29 of 72

A normally developed 3-year-old child can do which one of the following tasks?

A Copy a square

B Count to 10

C Fasten buttons fully without assistance

D Hop

E Make a tower out of 9 bricks

Explanation
A normally developed 3-year-old child can
go upstairs one foot per step and downstairs two feet per step. They
can ride a tricycle and build a tower of 9 bricks.

Counting to 10, hopping and copying a square is typical of a 4-year-old. Fastening buttons fully is typical of 5-
year-olds.
45729

Tag Question
Previous Question
Feedback Next Question

Difficulty: Average

https://mypastest.pastest.com/Secure/TestMe/Browser/436619[‫ م‬11:29:33 08/12/1437]


MyPastest

Peer Responses

Session Progress

Responses Correct: 0

Responses Incorrect: 33

Responses Total: 33

Responses - % Correct: 0%

Blog
About Pastest
Contact Us
Help

© Pastest 2016

https://mypastest.pastest.com/Secure/TestMe/Browser/436619[‫ م‬11:29:33 08/12/1437]


MyPastest

Prefer to use the old MyPastest? Access it here »

End Session

Back to Filters

Question 30 of 72

A normally developed 4-year-old would be expected to do which one of the following?

A Copy a cross

B Draw a triangle

C Dress and undress unaided

D Give their home address

E Write their name

Explanation
Drawing a triangle, being able to give their home address, write their name and dress or undress completely
unaided are more typical of a 5-year-old child. A four-year-old should be able to copy a cross and a square.
45730

Tag Question

Feedback
Previous Question
Next Question
Difficulty: Average

Peer Responses

https://mypastest.pastest.com/Secure/TestMe/Browser/436619[‫ م‬11:29:53 08/12/1437]


MyPastest

Session Progress

Responses Correct: 0

Responses Incorrect: 34

Responses Total: 34

Responses - % Correct: 0%

Blog
About Pastest
Contact Us
Help

© Pastest 2016

https://mypastest.pastest.com/Secure/TestMe/Browser/436619[‫ م‬11:29:53 08/12/1437]


MyPastest

Prefer to use the old MyPastest? Access it here »

End Session

Back to Filters

Question 31 of 72

You are assessing the development of a 12-month-old child.

Which one of the following milestones would raise concern if it was absent?

A Building a tower of 3 bricks

B Employing echolalia

C Feeding themselves with a spoon

D Throwing a ball without falling

E Walking with one hand held

Explanation
A normally developed 1-year-old would be expected to walk with one hand held.

At
15 months a child should be repeating words spoken to them (echolalia),
and a typical 18-month-old would be
expected to throw a ball without falling, feed themselves with a spoon and build a tower of 3 bricks.
45731

Previous Question Tag Question

Feedback
Next Question

Difficulty: Average

https://mypastest.pastest.com/Secure/TestMe/Browser/436619[‫ م‬11:31:08 08/12/1437]


MyPastest

Peer Responses

Session Progress

Responses Correct: 0

Responses Incorrect: 35

Responses Total: 35

Responses - % Correct: 0%

Blog
About Pastest
Contact Us
Help

© Pastest 2016

https://mypastest.pastest.com/Secure/TestMe/Browser/436619[‫ م‬11:31:08 08/12/1437]


MyPastest

Prefer to use the old MyPastest? Access it here »

Back to Filters

Question 32 of 72

You are discussing with a mother the possible reasons for speech delay in her child.

Which of the following conditions is NOT associated with speech delay?

A Autism

B Cerebral palsy

C Cleft palate

D Foetal alcohol syndrome

E Tongue tie

Explanation
Speech delay has many potential causes including autism, hearing defects, foetal alcohol syndrome, fragile X
syndrome, Down syndrome, hypothyroidism, congenital infections and emotional deprivation.

It is important to take a family history.

Cleft
palate is associated with speech and language delay. This may be a direct effect of dysarthria, because of the
commonly associated hearing problems or because of some global problem that may be associated with the cleft
palate.

Tongue tie may effect the quality of speech but should not lead to delay in the development of speech.
45732

Next Question

Tag Question

https://mypastest.pastest.com/Secure/TestMe/Browser/436619[‫ م‬11:31:28 08/12/1437]


MyPastest

Feedback End Session

Difficulty: Average

Peer Responses

Session Progress

Responses Correct: 0

Responses Incorrect: 36

Responses Total: 36

Responses - % Correct: 0%

Blog
About Pastest
Contact Us
Help

© Pastest 2016

Previous Question

https://mypastest.pastest.com/Secure/TestMe/Browser/436619[‫ م‬11:31:28 08/12/1437]


MyPastest

Prefer to use the old MyPastest? Access it here »

End Session

Back to Filters

Question 33 of 72

You are conducting a 6-week check on a normal term baby.

Which one of the following would be an expected finding?

A Fixes and follows in the horizontal and vertical plane

B Turns head to sound level with the ear

C Uses a social smile to attract attention

D Will be startled by a sudden prolonged sound

E Will roll over from prone to supine

Explanation
A normal 6-week-old child would be expected to smile but not use a 'social' smile to attract attention, startle to
loud noises and show evidence of fixing and following through
90o in the horizontal plane. Being able to fix and
follow in the horizontal and vertical planes is seen in a 12-16 week old baby.

Turning the head to sound level to the ear is normally seen around 3 months of age.

A 6-month-old can roll from prone to supine.


45733

Previous Question
Next Question
Tag Question

Feedback

https://mypastest.pastest.com/Secure/TestMe/Browser/436619[‫ م‬11:36:56 08/12/1437]


MyPastest

Difficulty: Average

Peer Responses

Session Progress

Responses Correct: 0

Responses Incorrect: 37

Responses Total: 37

Responses - % Correct: 0%

Blog
About Pastest
Contact Us
Help

© Pastest 2016

https://mypastest.pastest.com/Secure/TestMe/Browser/436619[‫ م‬11:36:56 08/12/1437]


MyPastest

Prefer to use the old MyPastest? Access it here »

End Session

Back to Filters

Question 34 of 72

Which one of the following would be within the expected developmental stage of a normal 1-year-old child?

A Build a tower of 3 blocks

B Drink from a cup

C Say 6 words with meaning

D Spontaneously scribble with a pencil

E Walk around a cot by holding on

Explanation
A normal 1-year-old child would be able to walk with one hand held and cruise around furniture including a cot.
They would be able to use two words with meaning and have a pincer grip.

Drinking from a cup is an achievement seen


around 15 months whilst building a tower of 3 blocks and
spontaneously scribbling with a pencil are more typical of an 18-month-old child.
45734

Tag Question
Previous Question
Feedback Next Question

Difficulty: Average

https://mypastest.pastest.com/Secure/TestMe/Browser/436619[‫ م‬11:37:15 08/12/1437]


MyPastest

Peer Responses

Session Progress

Responses Correct: 0

Responses Incorrect: 38

Responses Total: 38

Responses - % Correct: 0%

Blog
About Pastest
Contact Us
Help

© Pastest 2016

https://mypastest.pastest.com/Secure/TestMe/Browser/436619[‫ م‬11:37:15 08/12/1437]


MyPastest

Prefer to use the old MyPastest? Access it here »

End Session

Back to Filters

Question 35 of 72

You are observing a normal 15-month-old girl.

Which of the following is her mother likely to have noticed her daughter beginning to do, most recently?

A Crawling

B Cruising around furniture

C Demonstrating a simple pincer grip

D Echolalia

E Waving 'bye-bye'

Explanation
It would be expected that the child should be able to demonstrate a simple pincer grip, wave bye-bye, crawl and
cruise around the furniture from the age of 12 months. It is around
15 months of age that a child begins to repeat
words spoken to them (echolalia), use several words with meaning and to show understanding of
commonly used
words such as the names of brother and sisters, or 'cup',
'book', 'sleep' etc.
45735

Previous Question Tag Question


Next Question
Feedback

Difficulty: Average

https://mypastest.pastest.com/Secure/TestMe/Browser/436619[‫ م‬11:37:35 08/12/1437]


MyPastest

Peer Responses

Session Progress

Responses Correct: 0

Responses Incorrect: 39

Responses Total: 39

Responses - % Correct: 0%

Blog
About Pastest
Contact Us
Help

© Pastest 2016

https://mypastest.pastest.com/Secure/TestMe/Browser/436619[‫ م‬11:37:35 08/12/1437]


MyPastest

Prefer to use the old MyPastest? Access it here »

Back to Filters

Question 36 of 72

You observe a 5-year-old as she draws triangles whilst waiting for their appointment with you. When called, she
hops and skips
in to see you, gives her home address when requested and undresses and dresses herself in order to
be examined.

Which of these abilities is beyond her normal developmental age?

A Dressing and undressing unaided

B Giving her home address

C Skipping and hopping

D Understanding the concept of 'opposites'

E None of the above

Explanation
A normal 5-year-old should be able to skip and hop, draw a triangle, give their home address, and dress and
undress unaided.

They
will also be able to bounce and catch a ball, understand the concept of
opposites, write their name, choose
their own friends and enjoy acting out role play.
45736

Previous Question
Next Question
Tag Question

Feedback

https://mypastest.pastest.com/Secure/TestMe/Browser/436619[‫ م‬11:37:57 08/12/1437]


MyPastest

End Session
Difficulty: Average

Peer Responses

Session Progress

Responses Correct: 0

Responses Incorrect: 40

Responses Total: 40

Responses - % Correct: 0%

Blog
About Pastest
Contact Us
Help

© Pastest 2016

https://mypastest.pastest.com/Secure/TestMe/Browser/436619[‫ م‬11:37:57 08/12/1437]


MyPastest

Prefer to use the old MyPastest? Access it here »

End Session

Back to Filters

Question 37 of 72

A group of 3-year-olds are sent out to play in their nursery playground. One girl, Ella, fastens her coat buttons
unaided then selects a tricycle to ride around upon. She points to and correctly names at least two of the colours
used on an abstract mural painted on the boundary wall, and balances momentarily on one foot to dance with a
friend.

Which of these observations is a developmental milestone in advance of her age?

A Balancing on one foot

B Doing up buttons unaided

C Naming two colours

D Riding a tricycle

E None of the above

Explanation
A normal 3-year-old can ride a tricycle, balance momentarily on one foot and name 2 colours. Doing up buttons is
a
skill more usually seen in 5-year-olds.
45737

Previous Question
Next Question Tag Question

Feedback

https://mypastest.pastest.com/Secure/TestMe/Browser/436619[‫ م‬11:38:17 08/12/1437]


MyPastest

Difficulty: Average

Peer Responses

Session Progress

Responses Correct: 0

Responses Incorrect: 41

Responses Total: 41

Responses - % Correct: 0%

Blog
About Pastest
Contact Us
Help

© Pastest 2016

https://mypastest.pastest.com/Secure/TestMe/Browser/436619[‫ م‬11:38:17 08/12/1437]


MyPastest

Prefer to use the old MyPastest? Access it here »

End Session

Back to Filters

Question 38 of 72

Which of the following is NOT a recognised cause of toe walking?

A Duchenne Muscular Dystrophy

B Prematurity

C Spinal tumours

D Unilateral hip dislocation

E Waardenburg syndrome

Explanation
Recognised causes of toe walking include prematurity, unilateral hip dislocation, Duchenne muscular dystrophy,
spastic cerebral palsy and spinal tumours. It is a common normal finding in children between the ages of 1 and 2
years.

Waardenburg's
syndrome is autosomal dominantly inherited and characterised by a white
forelock, sensorieural
hearing loss and heterochromic irises. Toe walking is not a feature.
45739

Previous Question Tag Question


Next Question
Feedback

Difficulty: Average

https://mypastest.pastest.com/Secure/TestMe/Browser/436619[‫ م‬11:38:36 08/12/1437]


MyPastest

Peer Responses

Session Progress

Responses Correct: 0

Responses Incorrect: 42

Responses Total: 42

Responses - % Correct: 0%

Blog
About Pastest
Contact Us
Help

© Pastest 2016

https://mypastest.pastest.com/Secure/TestMe/Browser/436619[‫ م‬11:38:36 08/12/1437]


MyPastest

Prefer to use the old MyPastest? Access it here »

End Session

Back to Filters

Question 39 of 72

You see a child in clinic whose parents are concerned about his hearing.

Which of the following would be a reason to refer a child for a hearing test?

A If there are no single words by the age of 18 months

B If there is a strong family history of deafness

C If there is no intelligible speech by 3 years

D If there is parental concern regarding the child's hearing

E All of the above

Explanation
Children should be referred for a hearing
test if there is parental concern regarding the child's hearing, or if
there is
a strong family history of deafness. Other indications for referral include no intelligible speech by 3 years and no
single words by the age of 18 months. Additionally a hearing test is indicated following bacterial meningitis.
45740

Tag Question
Previous Question
Feedback Next Question

Difficulty: Average

https://mypastest.pastest.com/Secure/TestMe/Browser/436619[‫ م‬11:38:55 08/12/1437]


MyPastest

Peer Responses

Session Progress

Responses Correct: 0

Responses Incorrect: 43

Responses Total: 43

Responses - % Correct: 0%

Blog
About Pastest
Contact Us
Help

© Pastest 2016

https://mypastest.pastest.com/Secure/TestMe/Browser/436619[‫ م‬11:38:55 08/12/1437]


MyPastest

Prefer to use the old MyPastest? Access it here »

End Session

Back to Filters

Question 40 of 72

You are reviewing a 9-month-old baby in clinic.

Which of the following observations would be in advance of the the child's expected development stage?

A Can differentiate between strangers and familiar faces

B Claps hands

C Pokes objects with their index finger(s)

D Sits without support

E Will play peek-a-boo

Explanation
A normal 9-month-old should be able to differentiate between strangers and familiar faces (this can cause
separation and stranger anxiety), play peek-a-boo (as this depends on object permanence) and can poke objects
with their index fingers. They should be sitting without support by 8 or 9 months but clapping hands is
looked for
in a 12-month-old baby.
45742

Previous Question Tag Question


Next Question
Feedback

Difficulty: Average

https://mypastest.pastest.com/Secure/TestMe/Browser/436619[‫ م‬11:39:16 08/12/1437]


MyPastest

Peer Responses

Session Progress

Responses Correct: 0

Responses Incorrect: 44

Responses Total: 44

Responses - % Correct: 0%

Blog
About Pastest
Contact Us
Help

© Pastest 2016

https://mypastest.pastest.com/Secure/TestMe/Browser/436619[‫ م‬11:39:16 08/12/1437]


MyPastest

Prefer to use the old MyPastest? Access it here »

Back to Filters

Question 41 of 72

A 15-month-old girl should be further assessed if she can't do which one of the following?

A Build a tower of 3 bricks

B Copy a square

C Follow a one step command

D Stand holding onto furniture

E Take off shoes and socks

Explanation
An 18-month-old should be able to build a
tower of 3 bricks, a 15-month-old should be able to build a tower of 2
bricks. Following a one step command such as 'give me a doll' is also normal development for an 18-month-old as
is the ability to remove shoes
and socks.

A 4-year-old should be able to copy a square.

Children
start to walk between the ages of 12-18 months and most can walk well by the age of 15 months. Delay
in walking as evidenced by delay in pulling to stand could raise the question of an underlying problem such as
Duchenne muscular dystrophy if the child is a boy.
45743

Previous Question
Next Question
Tag Question

Feedback

https://mypastest.pastest.com/Secure/TestMe/Browser/436619[‫ م‬11:39:38 08/12/1437]


MyPastest

End Session
Difficulty: Average

Peer Responses

Session Progress

Responses Correct: 0

Responses Incorrect: 45

Responses Total: 45

Responses - % Correct: 0%

Blog
About Pastest
Contact Us
Help

© Pastest 2016

https://mypastest.pastest.com/Secure/TestMe/Browser/436619[‫ م‬11:39:38 08/12/1437]


MyPastest

Prefer to use the old MyPastest? Access it here »

End Session

Back to Filters

Question 42 of 72

Which of the following would imply the need for developmental assessment in a 2-year-old child?

A Bed wetting

B Failure to make a 3 block tower

C Inability to match 3-4 primary colours

D Still only able to go up stairs two feet per step

E Unable to copy a circle

Explanation
It is normal to go up and down stairs alone, holding on, two feet per step at 2 years.

Children can normally name some colours at 3 years and should also be able to copy a circle.

A 2-year-old would be expected to build a tower of six cubes.

A
child should be dry by day by 3-years-old and dry by night by 4 years. However, approximately 10% of children
are still wetting the bed at the age of 5 years.
45744

Previous Question
Next Question Tag Question

Feedback

https://mypastest.pastest.com/Secure/TestMe/Browser/436619[‫ م‬11:39:57 08/12/1437]


MyPastest

Difficulty: Average

Peer Responses

Session Progress

Responses Correct: 0

Responses Incorrect: 46

Responses Total: 46

Responses - % Correct: 0%

Blog
About Pastest
Contact Us
Help

© Pastest 2016

https://mypastest.pastest.com/Secure/TestMe/Browser/436619[‫ م‬11:39:57 08/12/1437]


MyPastest

Prefer to use the old MyPastest? Access it here »

End Session

Back to Filters

Question 43 of 72

Which of the following merit further investigation in a 12-month-old?

A Child who displays casting (throwing things away)

B Child who doesn't have any meaningful words

C Child who has persistent hand preference

D Child who is a 'bottom shuffler'

E Child who shouts to gain attention

Explanation
Hand preference starts to develop around the age of 18 months and settles down in the majority of children by the
age 3. Early apparent hand preference may indicate cerebral palsy.

A 12-month-old child should have two meaningful words but referral would not be warranted until the child
reaches 18 months.

'Bottom shuffling' is another form of mobility, like crawling, used by a child before they learn to walk.

Throwing toys, etc. at the age of 12 months is perfectly normal as is shouting for attention.
45745

Previous Question
Next Question
Tag Question

Feedback

https://mypastest.pastest.com/Secure/TestMe/Browser/436619[‫ م‬11:40:19 08/12/1437]


MyPastest

Difficulty: Average

Peer Responses

Session Progress

Responses Correct: 0

Responses Incorrect: 47

Responses Total: 47

Responses - % Correct: 0%

Blog
About Pastest
Contact Us
Help

© Pastest 2016

https://mypastest.pastest.com/Secure/TestMe/Browser/436619[‫ م‬11:40:19 08/12/1437]


MyPastest

Prefer to use the old MyPastest? Access it here »

Back to Filters

Question 44 of 72

You refer a child for hearing assessment who has delayed speech development.

Which of the following is not an indication for referral for hearing assessment?

A A deaf sibling

B Single failure at health visitor distraction testing

C Parental concern over hearing

D Recent meningitis

E Secretory otitis media in a 4-year-old with normal language development

Explanation
Diagnostic audiometry is mandatory in the
presence of delayed speech development even if a child passed his
screening test in infancy.

Although distraction audiometry is very effective if carried out with scrupulous attention to
detail, there are
potential pitfalls, for instance, due to limitations on space the test can be done in areas which are not soundproof.

Sensorineural
deafness is often genetically determined usually through autosomal recessive transmission, although
sex-linked and dominant inheritance occurs.

Parental concern should be taken seriously.


45746

Next Question

Tag Question

https://mypastest.pastest.com/Secure/TestMe/Browser/436619[‫ م‬11:43:06 08/12/1437]


MyPastest

Feedback End Session

Difficulty: Average

Peer Responses

Session Progress

Responses Correct: 0

Responses Incorrect: 48

Responses Total: 48

Responses - % Correct: 0%

Blog
About Pastest
Contact Us
Help

© Pastest 2016

Previous Question

https://mypastest.pastest.com/Secure/TestMe/Browser/436619[‫ م‬11:43:06 08/12/1437]


MyPastest

Prefer to use the old MyPastest? Access it here »

Back to Filters

Question 45 of 72

A 12-month-old child who is developing normally would be expected to do which of the following?

A Be able to make 2 word sentences

B Make a tower three blocks high

C Pick up a raisin between forefinger and thumb

D Remove one item of clothing on request (eg socks)

E Understand several words with meaning

Explanation
It is important to have a good grasp of important milestones - they are a common topic for the exam.

Most
12-month-old children will be mobile, by standing holding onto support,
lifting one foot and moving it
sideways ('cruising' around the furniture). They will demonstrate a neat pincer grasp, e.g. picking up a
raisin or
pellet of paper between the tip of the index finger and the thumb.

Words (or meaningful word-like utterances) are produced, but words are not usually chosen and put together
deliberately by a child until after the second birthday (typically around age of 30 months). Word combinations
used earlier than
this are likely to be an echo of a learnt phrase which may be understood by the child to be one
single word even though they are a combination of more than one word (eg daddyhome).

Building a tower of three cubes and following a one step command such as 'take off your socks', is expected at 18
months.
45747
Next Question

https://mypastest.pastest.com/Secure/TestMe/Browser/436619[‫ م‬11:43:29 08/12/1437]


MyPastest

Tag Question

Feedback End Session

Difficulty: Average

Peer Responses

Session Progress

Responses Correct: 0

Responses Incorrect: 49

Responses Total: 49

Responses - % Correct: 0%

Blog
About Pastest
Contact Us
Help

© Pastest 2016

Previous Question

https://mypastest.pastest.com/Secure/TestMe/Browser/436619[‫ م‬11:43:29 08/12/1437]


MyPastest

Prefer to use the old MyPastest? Access it here »

Back to Filters

Question 46 of 72

Which one of the following reflexes/responses should still be present in a normally developing 12-month-old
child?

A Moro reflex

B Neck-righting reflex

C Palmar reflex

D Parachute reflex

E Plantar grasp response

Explanation
The parachute reflex is the last of the postural reflexes to develop. It usually appears at about 9 months of age
around the age when a baby is beginning to learn how to balance and sit unsupported. It is a protective reflex
against falling. The baby will outstretch their hands if dropped forwards.

The
Moro response usually disappears by 4 months. Persistence beyond 6 months is always abnormal. The baby is
held supine and if their head is
allowed to fall back, their arms will open symmetrically before closing
again.

The neck-righting reflex is strongest at 3


months of age and disappears by 10 months of age. It enables the baby to
learn to roll over. It is observed when a child is lain supine and turns its shoulders, trunk and pelvis towards the
side to which the head
is turned.

The palmar grasp persists up to about 3 months. the baby closes their hand around an object placed in their palm.

The
plantar grasp response will have disappeared between 6 and 12 months of
age. The baby curls their foot
around an object placed on their sole.
45748

https://mypastest.pastest.com/Secure/TestMe/Browser/436619[‫ م‬11:43:52 08/12/1437]


MyPastest

End Session
Tag Question

Feedback

Difficulty: Average

Peer Responses

Session Progress

Responses Correct: 0

Responses Incorrect: 50

Responses Total: 50

Responses - % Correct: 0%

Blog
About Pastest
Contact Us
Help

© Pastest 2016

Previous Question
Next Question

https://mypastest.pastest.com/Secure/TestMe/Browser/436619[‫ م‬11:43:52 08/12/1437]


MyPastest

Prefer to use the old MyPastest? Access it here »

Back to Filters

Question 47 of 72

You are examining a 2-month-old premature baby girl. When the baby is touched near the corner of the mouth,
she turns towards the stimulus.

Which of the following is descriptive of this reflex?

A It can assist in establishing breastfeeding

B It is not considered to be a primitive reflex

C It rarely persists beyond 1 week

D It is seen more in males than females

E It is always present in premature infants

Explanation
The rooting reflex is seen in term newborn babies (usually develops around 24 - 28 weeks gestation) who
automatically turn their face toward the stimulus and make sucking (rooting) motions with the mouth when the
cheek or lip is touched. Brushing the nipple against the baby's cheek initiates the rooting reflex and helps to ensure
breastfeeding is successful.

Premature babies born prior to 28 weeks may not have developed this reflex.

It is a primitive reflex and is present from birth until 4 months of age. It is found in both males and females
equally.
45749

Next Question

https://mypastest.pastest.com/Secure/TestMe/Browser/436619[‫ م‬11:44:16 08/12/1437]


MyPastest

Tag Question

Feedback End Session

Difficulty: Average

Peer Responses

Session Progress

Responses Correct: 0

Responses Incorrect: 51

Responses Total: 51

Responses - % Correct: 0%

Blog
About Pastest
Contact Us
Help

© Pastest 2016

Previous Question

https://mypastest.pastest.com/Secure/TestMe/Browser/436619[‫ م‬11:44:16 08/12/1437]


MyPastest

Prefer to use the old MyPastest? Access it here »

Back to Filters

Question 48 of 72

A 3-year-old boy was referred, as the health visitor was concerned that he still produced little sound and used no
comprehensible
words, although he was apparently able to understand commands appropriate to his age.

What diagnosis is most likely in this child?

A Autistic Spectrum Disorder

B Deafness

C Elective mutism

D Tongue-tie

E All of the above

Explanation
If a child presents with expressive speech delay, but has normal or near normal receptive (i.e. understanding)
language skills, this can suggest maturation delay, developmental expressive language disorder, dysarthria, or
dyspraxia.

The
child with both receptive and expressive language delays but normal hearing is most likely to have mental
retardation, mixed receptive and expressive language disorder, or autistic spectrum disorder.

Psychological
and speech evaluations can help differentiate among these possibilities. The most common causes
of speech/language delay, in order
of frequency of association, are mental retardation, hearing impairment, autistic
spectrum disorder, cerebral palsy, craniofacial disorders (such as cleft palate), genetic disorders, and brain injury.
45750
Next Question

https://mypastest.pastest.com/Secure/TestMe/Browser/436619[‫ م‬11:44:42 08/12/1437]


MyPastest

Tag Question

Feedback End Session

Difficulty: Average

Peer Responses

Session Progress

Responses Correct: 0

Responses Incorrect: 52

Responses Total: 52

Responses - % Correct: 0%

Blog
About Pastest
Contact Us
Help

© Pastest 2016

Previous Question

https://mypastest.pastest.com/Secure/TestMe/Browser/436619[‫ م‬11:44:42 08/12/1437]


MyPastest

Prefer to use the old MyPastest? Access it here »

End Session

Back to Filters

Question 49 of 72

You are assessing a child aged 18 months and observe him performing the skills listed below.

Which of these goes beyond the development milestones expected of a normally developing child of his age?

A Feeding with a spoon

B Speaking in sentences of at least 3 words

C Throwing a ball without falling

D Using pincer finger grip

E Waving 'bye bye'

Explanation
Speaking in sentences of 3 or more words tends to occur at the age of 3 years. The other items are all seen in 18-
month-olds. 45751

Tag Question

Previous Question
Feedback

Next Question
Difficulty: Average

Peer Responses

https://mypastest.pastest.com/Secure/TestMe/Browser/436619[‫ م‬11:45:05 08/12/1437]


MyPastest

Session Progress

Responses Correct: 0

Responses Incorrect: 53

Responses Total: 53

Responses - % Correct: 0%

Blog
About Pastest
Contact Us
Help

© Pastest 2016

https://mypastest.pastest.com/Secure/TestMe/Browser/436619[‫ م‬11:45:05 08/12/1437]


MyPastest

Prefer to use the old MyPastest? Access it here »

Back to Filters

Question 50 of 72

A nursery teacher has expressed concern to the parents of an 18-month-old girl about her language development.

Select the feature below which would NOT be associated with language delay in an 18-month-old child?

A Failure to move to music

B Failure to respond to own name

C Lack of 2 words with meaning

D Lack of imitative gesture

E Unable to understand the word 'no'

Explanation
Imitative gestures include any gestures (sticking out tongue/waving bye-bye, etc) that a child may see someone
doing and copy. Lack of such gestures is not a marker for language delay.

A child of 18 months usually understands up to 50 words. Although expressive language development will vary
from child to child, between the ages of 18 months and 2 years the ability to
name objects and express commands
will expand rapidly from approximately 15 - 20 to 50 - 100 words, with word combinations increasingly used
appropriately.

Language delay is clearly associated with hearing problems and may be evidenced by a child's failure to respond
to music or their own name.
45752

Next Question

https://mypastest.pastest.com/Secure/TestMe/Browser/436619[‫ م‬11:45:31 08/12/1437]


MyPastest

Tag Question

Feedback End Session

Difficulty: Average

Peer Responses

Session Progress

Responses Correct: 0

Responses Incorrect: 54

Responses Total: 54

Responses - % Correct: 0%

Blog
About Pastest
Contact Us
Help

© Pastest 2016

Previous Question

https://mypastest.pastest.com/Secure/TestMe/Browser/436619[‫ م‬11:45:31 08/12/1437]


MyPastest

Prefer to use the old MyPastest? Access it here »

Back to Filters

Question 51 of 72

An average 8-week-old infant, born at full term will be able to do which one of the following?

A Demonstrate a startle response to a sound of 30 dB

B Follow a moving object with their eyes through 180o in the horizontal plane

C Push up onto hands

D Smile

E Support their own head fully when pulled to sitting

Explanation
Smiles in response to appropriate stimuli
are seen from around 6 weeks of age. A social smile, where the infant
has awareness that a smile attracts attention, is not observed until around 3 months of age.

At 6 weeks an infant is able to follow objects in the horizontal plane through 90o; fixing and following an object
through 180o
in the horizontal plane is a milestone achieved at 3 months, as is pushing up onto forearms. Pushing
up onto hands occurs by around 5 months.

Infants of 3 months should startle to a 60 dB sound.

45753

Previous Question
Next Question
Tag Question

https://mypastest.pastest.com/Secure/TestMe/Browser/436619[‫ م‬11:45:53 08/12/1437]


MyPastest

Feedback

End Session
Difficulty: Average

Peer Responses

Session Progress

Responses Correct: 0

Responses Incorrect: 55

Responses Total: 55

Responses - % Correct: 0%

Blog
About Pastest
Contact Us
Help

© Pastest 2016

https://mypastest.pastest.com/Secure/TestMe/Browser/436619[‫ م‬11:45:53 08/12/1437]


MyPastest

Prefer to use the old MyPastest? Access it here »

End Session

Back to Filters

Question 52 of 72

You are assessing a normally developed 4-year-old girl.

Which of the following would be a task you could reasonably expect her to be able to do?

A Copy a cross shape

B Copy a triangle

C Draw a 6 part man

D Name 10 colours

E Write her name

Explanation
A 4-year-old should be able to copy a cross and also a square.

Some
4-year-olds may be able to name a few colours (but not 10) and to draw a
basic man with a head and eyes
and 4 limbs not simply arising from the head (but not 6 parts such as ears, hair, hands and feet).

By 5 years of age a child should be able to write their name (not just trace dots of letters) and draw a triangle.
45754

Previous Question
Next Question Tag Question

Feedback

https://mypastest.pastest.com/Secure/TestMe/Browser/436619[‫ م‬11:46:14 08/12/1437]


MyPastest

Difficulty: Average

Peer Responses

Session Progress

Responses Correct: 0

Responses Incorrect: 56

Responses Total: 56

Responses - % Correct: 0%

Blog
About Pastest
Contact Us
Help

© Pastest 2016

https://mypastest.pastest.com/Secure/TestMe/Browser/436619[‫ م‬11:46:14 08/12/1437]


MyPastest

Prefer to use the old MyPastest? Access it here »

End Session

Back to Filters

Question 53 of 72

Normal motor development at 3 months includes which of the following?

A Attempts to crawl

B Can roll over

C Finger play with the hands open and brought together

D Pulling to stand

E Sitting with support

Explanation
Pulling to stand would not be expected until an infant is about 12 months old. A 6-month-old infant should be able
to roll over. Crawling is attempted at at about 8-9 months, and sitting with support is achieved by 5-6 months.
However, a 3-month-old baby should love to look at and play with their hands and fingers.
45755

Tag Question

Previous Question
Feedback

Next Question
Difficulty: Average

Peer Responses

https://mypastest.pastest.com/Secure/TestMe/Browser/436619[‫ م‬11:46:36 08/12/1437]


MyPastest

Session Progress

Responses Correct: 0

Responses Incorrect: 57

Responses Total: 57

Responses - % Correct: 0%

Blog
About Pastest
Contact Us
Help

© Pastest 2016

https://mypastest.pastest.com/Secure/TestMe/Browser/436619[‫ م‬11:46:36 08/12/1437]


MyPastest

Prefer to use the old MyPastest? Access it here »

End Session

Back to Filters

Question 54 of 72

By 2 years of age a child should be able to do which of the following?

A Bounce and catch a ball

B Pedal a tricycle

C Pull and push large wheeled toys and squat to play with toys on the floor

D Run on tiptoe

E Stand on one leg for 3 - 5 seconds

Explanation
Pushing and pulling large wheeled toys and squatting to play with toys on the floor is commonly observed in 2-
year-olds.

A child can walk on tip-toe by 2.5 and run on tip-toe by 3 years of age.

The
ability to pedal a tricycle and momentarily maintain balance using one leg should be attained by 3 years and
bouncing and catching a ball is learnt by the age of 5.
45756

Previous Question
Next Question Tag Question

Feedback

https://mypastest.pastest.com/Secure/TestMe/Browser/436619[‫ م‬11:46:55 08/12/1437]


MyPastest

Difficulty: Average

Peer Responses

Session Progress

Responses Correct: 0

Responses Incorrect: 58

Responses Total: 58

Responses - % Correct: 0%

Blog
About Pastest
Contact Us
Help

© Pastest 2016

https://mypastest.pastest.com/Secure/TestMe/Browser/436619[‫ م‬11:46:55 08/12/1437]


MyPastest

Prefer to use the old MyPastest? Access it here »

End Session

Back to Filters

Question 55 of 72

Normal hearing and speech at 3 months of age reveal themselves by which of the following?

A Babbling loudly and tunefully with repetitive strings of syllables, eg ba-ba

B Crying when uncomfortable and annoyed

C Imitating adults' playful vocalisations

D No longer being startled by sudden noises

E Understanding the word 'no'

Explanation
At 3 months of age, the child reacts to sudden noises, quietens or stills to the mother's or father's voice, vocalises
in a happy manner when spoken to, cries when uncomfortable or irritated, and stills to a bell or rattle for 3-5
seconds.

By
9 months, the child is eagerly attentive to sounds, vocalises deliberately as a means of communication, being
either friendly or annoyed, shouts for attention, babbles considerably, understands 'no' and 'bye bye', and imitates
adults engaging in playful sounds.
45757

Previous Question
Next Question Tag Question

Feedback

https://mypastest.pastest.com/Secure/TestMe/Browser/436619[‫ م‬11:47:35 08/12/1437]


MyPastest

Difficulty: Average

Peer Responses

Session Progress

Responses Correct: 0

Responses Incorrect: 59

Responses Total: 59

Responses - % Correct: 0%

Blog
About Pastest
Contact Us
Help

© Pastest 2016

https://mypastest.pastest.com/Secure/TestMe/Browser/436619[‫ م‬11:47:35 08/12/1437]


MyPastest

Prefer to use the old MyPastest? Access it here »

Back to Filters

Question 56 of 72

Damon is 18 months of age.

Which of the following gross motor skills should he have acquired?

A Carrying toys whilst walking

B Intentionally kicking a ball

C Jumping with two feet together from a small step

D Propelling a tricycle forwards by pushing with the feet on the floor

E Throwing a small ball overhand without falling

Explanation
At 18 months, the following gross motors skills would be expected:

walks with the feet slightly apart


runs carefully with the head held high
pushes and pulls objects around the floor
walks upstairs if the hand is held
kneels on a flat surface without support
carry toys while walking

At 2 years, the following would be seen:

squats to play with toys and can rise to the feet without using the hands
climbs on to furniture and can get down again

https://mypastest.pastest.com/Secure/TestMe/Browser/436619[‫ م‬11:47:54 08/12/1437]


MyPastest

walks upstairs holding on to a rail or the wall, with two feet on each step
throws a ball overhand forwards without falling over
sits on a small tricycle and propels forwards by pushing with the feet onEnd Session
the floor

At 2.5 years of age, these would be the normal gross milestones to have been achieved:

walks upstairs confidently


runs in straight lines
can jump with two feet together from the lowest step
can stand on tiptoe
intentionally kicks a ball
should be able to cast a hand-held ball somewhat stiffly at body level.

45758

Tag Question

Feedback

Difficulty: Average

Peer Responses

Session Progress

Responses Correct: 0

Previous Question
Responses Incorrect: 60

Responses Total:
Next Question 60

Responses - % Correct: 0%

https://mypastest.pastest.com/Secure/TestMe/Browser/436619[‫ م‬11:47:54 08/12/1437]


MyPastest

Blog
About Pastest
Contact Us
Help

© Pastest 2016

https://mypastest.pastest.com/Secure/TestMe/Browser/436619[‫ م‬11:47:54 08/12/1437]


MyPastest

Prefer to use the old MyPastest? Access it here »

Back to Filters

Question 57 of 72

Regarding primative reflexes, Jenny is a normal 12-month-old infant if:

A The asymmetrical tonic neck reflex is present

B The head-righting reflex is absent

C The Moro reflex is absent

D The parachute reflex is absent

E The stepping reflex is present

Explanation
The primitive reflexes are stereotyped responses that disappear in a predictable order as the child matures.

The
Moro reflex is present from birth to 4 months. Asymmetry of the Moro reflex occurs in brachial plexus injury
and fracture of the clavicle. Persistence of the Moro can be an early sign of cerebral palsy.

The
parachute reflex develops at around 9 months of age. It consists of extension of the hands and arms when the
infant in the prone position is
allowed to fall a short distance. This protective reflex persists for life.

The stepping reflex is present at birth and disappears at 6 weeks.

The asymmetrical tonic neck reflex disappears around 6 months of age. Its persistence occurs in children with
cerebral palsy.

The head-righting reflex develops at 6 months of age and persists.


45759
Next Question

https://mypastest.pastest.com/Secure/TestMe/Browser/436619[‫ م‬11:48:16 08/12/1437]


MyPastest

Tag Question

Feedback End Session

Difficulty: Average

Peer Responses

Session Progress

Responses Correct: 0

Responses Incorrect: 61

Responses Total: 61

Responses - % Correct: 0%

Blog
About Pastest
Contact Us
Help

© Pastest 2016

Previous Question

https://mypastest.pastest.com/Secure/TestMe/Browser/436619[‫ م‬11:48:16 08/12/1437]


MyPastest

Prefer to use the old MyPastest? Access it here »

Back to Filters

Question 58 of 72

Lily has recently celebrated her birthday. She is very definite about which colours she likes and chose her pink
dress and black patent shoes from her wardrobe and dressed herself with supervision for her party.

She was very excited about her party and jumped about when guests began to arrive. She happily shared her toys
with her friends.

In the evening she brushed her teeth with supervision and took herself to the toilet.

How old is Lily likely to be?

A 2 years of age

B 3 years of age

C 4 years of age

D 5 years of age

E 6 years of age

Explanation
Typical 4-year-old children can jump and are starting to hop, but are usually unable to skip. They can stand on one
foot, well.

They
are able to identify several colours, will share toys and are out of nappies by night. They can dress with
supervision and go to the toilet alone.
45760

Next Question

https://mypastest.pastest.com/Secure/TestMe/Browser/436619[‫ م‬11:48:36 08/12/1437]


MyPastest

Tag Question

Feedback End Session

Difficulty: Average

Peer Responses

Session Progress

Responses Correct: 0

Responses Incorrect: 62

Responses Total: 62

Responses - % Correct: 0%

Blog
About Pastest
Contact Us
Help

© Pastest 2016

Previous Question

https://mypastest.pastest.com/Secure/TestMe/Browser/436619[‫ م‬11:48:36 08/12/1437]


MyPastest

Prefer to use the old MyPastest? Access it here »

End Session

Back to Filters

Question 59 of 72

Emma is a typical 2-year-old girl.

Which of the following commands would she be able to follow?

A Copy a circle

B Jump up and down

C Name three colours

D Point to three body parts

E Tell a story

Explanation
A typical 2-year-old child would be able to identify and point to three body parts and identify objects and pass
them across eg 'pass me a clean nappy please!'

Jumping, copying a circle, naming three colours and beginning to tell stories are skills found in typical 3-year-
olds,
45761

Previous Question
Next Question Tag Question

Feedback

https://mypastest.pastest.com/Secure/TestMe/Browser/436619[‫ م‬11:48:56 08/12/1437]


MyPastest

Difficulty: Average

Peer Responses

Session Progress

Responses Correct: 0

Responses Incorrect: 63

Responses Total: 63

Responses - % Correct: 0%

Blog
About Pastest
Contact Us
Help

© Pastest 2016

https://mypastest.pastest.com/Secure/TestMe/Browser/436619[‫ م‬11:48:56 08/12/1437]


MyPastest

Prefer to use the old MyPastest? Access it here »

Back to Filters

Question 60 of 72

Which one of the following disorders is not associated with developmental delay?

A Duchenne muscular dystrophy

B Marfan syndrome

C Spinal muscular atrophy

D Tay-Sachs disease

E Prader-Willi syndrome

Explanation
This is not a difficult question but many
people have been caught out by it as they automatically take
developmental delay to mean mental retardation, a term that is no longer
used. Remember, developmental delay
can constitute motor, cognitive, vision, hearing or social delays and really when we refer to it we should be
specific as to which of these the child has or, if the child is delayed in all these areas, we call it global
developmental delay.

Muscle disorders are associated with developmental delay and include Duchenne muscular dystrophy (these
children can have problems with their
intellect also) and myotonic dystrophy. Tay-Sachs disease is a
neurodegenerative disease of the grey matter with most children dying between 3 to 4 years of age. Children with
spinal muscular atrophy are cognitively normal and have normal intelligence, but they are often unable to walk or
sit up due to significant motor delay. Children with Prader-Wiili often have learning difficulties and mild
cognitive delay.
Next Question
Developmental delay is not a feature of Marfan syndrome.
45762

https://mypastest.pastest.com/Secure/TestMe/Browser/436619[‫ م‬11:49:16 08/12/1437]


MyPastest

Tag Question
End Session
Feedback

Difficulty: Average

Peer Responses

Session Progress

Responses Correct: 0

Responses Incorrect: 64

Responses Total: 64

Responses - % Correct: 0%

Blog
About Pastest
Contact Us
Help

© Pastest 2016

Previous Question

https://mypastest.pastest.com/Secure/TestMe/Browser/436619[‫ م‬11:49:16 08/12/1437]


MyPastest

Prefer to use the old MyPastest? Access it here »

End Session

Back to Filters

Question 61 of 72

A healthy 12-month-old infant should be able to do which one of the following?

A Feed with a spoon

B Point to a body part

C Understand words such as 'cup'

D Walk

E Wave bye bye

Explanation
Waving bye-bye develops around 10-12 months of age.

Understanding words such as 'cup' or the names of brothers and sisters is noticeable from around 15 months.

Pointing to body parts and spoon-feeding occur at around 18 months.

Although some children can walk at 12 months, most are still cruising. They should be walking by 18 months.
45767

Previous Question Tag Question


Next Question
Feedback

Difficulty: Average

https://mypastest.pastest.com/Secure/TestMe/Browser/436619[‫ م‬11:49:37 08/12/1437]


MyPastest

Peer Responses

Session Progress

Responses Correct: 0

Responses Incorrect: 65

Responses Total: 65

Responses - % Correct: 0%

Blog
About Pastest
Contact Us
Help

© Pastest 2016

https://mypastest.pastest.com/Secure/TestMe/Browser/436619[‫ م‬11:49:37 08/12/1437]


MyPastest

Prefer to use the old MyPastest? Access it here »

End Session

Back to Filters

Question 62 of 72

Stephanie is 3-years-old. Her parents consider her to be advanced for her age. They list her accomplishments.

Which of these, listed below, is typical of a 3-year-old child?

A Copy a cross

B Count to 10

C Draw a person with three parts

D Match 2 colours

E Stand on one foot well

Explanation
A 3-year-old should be able to match two colours, copy a circle, and jump.

Copying
a cross, standing well on one foot, drawing a person with three parts and counting to 10 are all typical of
a 4-year-old.
45768

Previous Question Tag Question

Feedback
Next Question

Difficulty: Average

https://mypastest.pastest.com/Secure/TestMe/Browser/436619[‫ م‬11:49:58 08/12/1437]


MyPastest

Peer Responses

Session Progress

Responses Correct: 0

Responses Incorrect: 66

Responses Total: 66

Responses - % Correct: 0%

Blog
About Pastest
Contact Us
Help

© Pastest 2016

https://mypastest.pastest.com/Secure/TestMe/Browser/436619[‫ م‬11:49:58 08/12/1437]


MyPastest

Prefer to use the old MyPastest? Access it here »

Back to Filters

Question 63 of 72

Jess is a normally developed 2 1/2-year-old.

Which of the following is she most likely to have been able to do for at least a year?

A Copy a vertical line

B Eat with a spoon and fork

C Follow commands such as 'give me a doll please'

D Jump well

E Kick a ball

Explanation
A year ago, when Jess was 18-months-old, it is likely that she would have been able to follow a one step command
such as 'give me a doll please'. She would also have been capable of throwing a ball, stooping and retrieving
objects from the floor, and climbing upstairs holding on, but using two feet per step.

Eating with a spoon and fork, copying a vertical line and kicking a ball are all within her normal age range
accomplishments of 2 - 2 1/2.

Jumping well is typical of a 3-year-old.


45771

Previous Question
Next Question
Tag Question

https://mypastest.pastest.com/Secure/TestMe/Browser/436619[‫ م‬11:50:18 08/12/1437]


MyPastest

Feedback

End Session
Difficulty: Average

Peer Responses

Session Progress

Responses Correct: 0

Responses Incorrect: 67

Responses Total: 67

Responses - % Correct: 0%

Blog
About Pastest
Contact Us
Help

© Pastest 2016

https://mypastest.pastest.com/Secure/TestMe/Browser/436619[‫ م‬11:50:18 08/12/1437]


MyPastest

Prefer to use the old MyPastest? Access it here »

End Session

Back to Filters

Question 64 of 72

You are making a developmental assessment on a 3-year-old boy.

Assuming his development is normal, which one of the following social skills is matched to his current age?

A He comforts friends who show distress

B He can tie the laces on his shoes

C He has recently begun to choose his own friends

D He is able to dress himself with supervision

E He will share toys

Explanation
A normal 3-year-old should be able to feed, wash and dress themselves with supervision, including donning shoes,
but they are unable to manage complex buckles or laces.

They will interact with other children in both imaginary and non-imaginary play.

A
child will be actively choosing their own friends and also beginning to
recognise distress in others and offer
comfort, by the age of 5 years.
45773

Previous Question
Next Question
Tag Question

Feedback

https://mypastest.pastest.com/Secure/TestMe/Browser/436619[‫ م‬11:50:39 08/12/1437]


MyPastest

Difficulty: Average

Peer Responses

Session Progress

Responses Correct: 0

Responses Incorrect: 68

Responses Total: 68

Responses - % Correct: 0%

Blog
About Pastest
Contact Us
Help

© Pastest 2016

https://mypastest.pastest.com/Secure/TestMe/Browser/436619[‫ م‬11:50:39 08/12/1437]


MyPastest

Prefer to use the old MyPastest? Access it here »

Back to Filters

Question 65 of 72

Daniel is a 1-year-old who has been assessed as developing normally.

Which one of the following skills would he most likely be able to demonstrate?

A Talking in sentences

B Understanding the concept of 'object permanence'

C Walking unaided

D Drinking from a cup

E Scribbling in circles

Explanation
Development is difficult to revise. The easiest way is to get to know a range of normal children who you can use
to picture the key stages: infant of 6 weeks, 4-6 months, 9 months, 10-12 months, 18 months, 2, 3, 4 and 5 years.

At
the age of 1 year, children should have developed an accurate pincer grip and be able to see and co-ordinate
and manipulate fairly small objects such as a raisin.

Gross motor development varies but the average 1-year-old should be able to stand and cruise around furniture or
walk with their hands held but not necessarily walk unaided.

The average 1-year-old


will have developed 'object permanence' and understands that objects continue to exist
even when they can't be seen, e.g. looking for a fallen toy.

Drinking from a cup will become established at around 15 months and scribbling at 18 months.
Next Question 45774

https://mypastest.pastest.com/Secure/TestMe/Browser/436619[‫ م‬11:51:01 08/12/1437]


MyPastest

Tag Question
End Session
Feedback

Difficulty: Average

Peer Responses

Session Progress

Responses Correct: 0

Responses Incorrect: 69

Responses Total: 69

Responses - % Correct: 0%

Blog
About Pastest
Contact Us
Help

© Pastest 2016

Previous Question

https://mypastest.pastest.com/Secure/TestMe/Browser/436619[‫ م‬11:51:01 08/12/1437]


MyPastest

Prefer to use the old MyPastest? Access it here »

End Session

Back to Filters

Question 66 of 72

You are giving a talk on child development to nursery school teachers who have asked you to focus on play.

Which of the following would be a normal observation for a 2-year-old at play?

A Catching a large ball

B Hopping with excitement

C Pedalling a tricycle

D Playing alone

E Sharing their toys

Explanation
A normally developed 2-year-old child would be expected to play alone. They are quite likely to play 'alongside'
others but each child's 'play' is independent of those around them.

Pedalling a tricycle is a skill developed around the age of 3. Sharing toys and hopping is typical of 4-year-old
children, whilst catching a ball may not fully develop until nearer 5 years of age.
45775

Previous Question
Next Question Tag Question

Feedback

https://mypastest.pastest.com/Secure/TestMe/Browser/436619[‫ م‬11:51:22 08/12/1437]


MyPastest

Difficulty: Average

Peer Responses

Session Progress

Responses Correct: 0

Responses Incorrect: 70

Responses Total: 70

Responses - % Correct: 0%

Blog
About Pastest
Contact Us
Help

© Pastest 2016

https://mypastest.pastest.com/Secure/TestMe/Browser/436619[‫ م‬11:51:22 08/12/1437]


MyPastest

Prefer to use the old MyPastest? Access it here »

Back to Filters

Question 67 of 72

In an otherwise well child, which one of the following is an indicator of possible language delay?

A A family history of hereditary deafness

B Echolalia at the age of 2 years

C Construction of 2 word phrases only by 2 and a half years

D No repetitive babble by 8 months of age

E Only speaking single words by 18 months

Explanation
A child with a family history of deafness should be screened during early childhood particularly if the parents
have concerns regarding the child's ability to hear properly.

Most infants will be vocalising tunefully by age 6 months and using strings of repetitive same-sound syllables
(e.g. 'agagaga', 'dadada') from 9 months of age, but there is a range of normal and most referral guidelines suggest
10-12 months as an 'upper' limit for babbling. Monotonous or limited vocalisation after 9-10 months should arouse
suspicion of deafness.

Echolalia (repeating phrases others have used) is common at 2 years.

The
failure of children to say single words by 18 months should raise concern as should the failure to construct 2
word phrases by 2 1/2 years.
45776

Next Question

https://mypastest.pastest.com/Secure/TestMe/Browser/436619[‫ م‬11:51:44 08/12/1437]


MyPastest

Tag Question

Feedback End Session

Difficulty: Average

Peer Responses

Session Progress

Responses Correct: 0

Responses Incorrect: 71

Responses Total: 71

Responses - % Correct: 0%

Blog
About Pastest
Contact Us
Help

© Pastest 2016

Previous Question

https://mypastest.pastest.com/Secure/TestMe/Browser/436619[‫ م‬11:51:44 08/12/1437]


MyPastest

Prefer to use the old MyPastest? Access it here »

Back to Filters

Question 68 of 72

You are selecting a type of hearing test for a 3 1/2-year-old


child who has been referred following parental
concern over hearing.

Which of the following is the most likely test to be used?

A Brain stem evoked potentials

B Distraction testing

C Oto acoustic test

D Play audiometry

E Pure tone audiometry

Explanation
Brain stem evoked potentials are useful in babies
less than 6 months, particularly those with a high risk of hearing
impairment secondary to kernicterus or prematurity. The oto acoustic test is also useful in babies and has 100%
sensitivity for sensorineural
hearing loss in the categories of moderate, severe and profound, however it will not
pick up the mild and minimal cases.

Distraction
testing is used in children from 8 months (as it relies on the child being able to sit unsupported) up to 1
year of age.

Play audiometry asks the child to perform a simple task in response to sound using toys to show the tester that
they have heard the sound. It is most
commonly used between the ages of 2 to 3 years.

Pure tone audiometry is reliably used from 3 years upwards as it is at this age when the child is more able to
cooperate with the testing.
45777

https://mypastest.pastest.com/Secure/TestMe/Browser/436619[‫ م‬11:52:05 08/12/1437]


MyPastest

End Session
Tag Question

Feedback

Difficulty: Average

Peer Responses

Session Progress

Responses Correct: 0

Responses Incorrect: 72

Responses Total: 72

Responses - % Correct: 0%

Blog
About Pastest
Contact Us
Help

© Pastest 2016

Previous Question
Next Question

https://mypastest.pastest.com/Secure/TestMe/Browser/436619[‫ م‬11:52:05 08/12/1437]


MyPastest

Prefer to use the old MyPastest? Access it here »

End Session

Back to Filters

Question 69 of 72

When assessing language development, which of the following is typical of a 2 1/2 year old child's use of
language?

A Ask 'when' questions

B Ask 'where' questions

C Ask 'why' questions

D Have a good grasp of the correct use of grammar

E Narrate stories

Explanation
A 2 1/2-year-old child should be able to join two
or more words in phrases and simple requests in context, such as
'give me' and ask 'what' and 'where' questions. Children generally ask 'why'
questions over the age of 4 years.

A child should be able to narrate stories and have a good grasp of grammar in the 3rd and 4th years.

If a child has problems with speech development, a hearing impairment should always be excluded.
45781

Previous Question
Next Question Tag Question

Feedback

https://mypastest.pastest.com/Secure/TestMe/Browser/436619[‫ م‬11:52:25 08/12/1437]


MyPastest

Difficulty: Average

Peer Responses

Session Progress

Responses Correct: 0

Responses Incorrect: 73

Responses Total: 73

Responses - % Correct: 0%

Blog
About Pastest
Contact Us
Help

© Pastest 2016

https://mypastest.pastest.com/Secure/TestMe/Browser/436619[‫ م‬11:52:25 08/12/1437]


MyPastest

Prefer to use the old MyPastest? Access it here »

Back to Filters

Question 70 of 72

You are investigating a child for developmental regression which presented beyond the age of 2 years.

Which of the following is a cause of developmental regression typically presenting before the age of 2 years?

A Hypothyroidism

B Leukodystrophies

C Mitochondrial disorders

D Sub-acute sclerosing pan encephalitis

E Wilson's disease

Explanation
Developmental regression before the age of 2 years is typically seen in hypothyroidism, HIV encephalopathy,
aminoacidopathies and some of the lysosomal storage disorders including certain of the mucopolysaccharidoses.

Recognised causes of developmental regression beyond the age of 2 years are many and include sub-acute
sclerosing pan encephalitis (although this tends to present much later than 2 years, typically after the ages of 6-15),
other acquired CNS infection including prion disease, leukodystrophies, Wilson's disease (again symptoms occur
between 6-20 years of age), other
neural storage and mitochondrial disorders.
45783

Previous Question
Next Question
Tag Question

Feedback

https://mypastest.pastest.com/Secure/TestMe/Browser/436619[‫ م‬11:52:46 08/12/1437]


MyPastest

End Session
Difficulty: Average

Peer Responses

Session Progress

Responses Correct: 0

Responses Incorrect: 74

Responses Total: 74

Responses - % Correct: 0%

Blog
About Pastest
Contact Us
Help

© Pastest 2016

https://mypastest.pastest.com/Secure/TestMe/Browser/436619[‫ م‬11:52:46 08/12/1437]


MyPastest

Prefer to use the old MyPastest? Access it here »

Back to Filters

Question 71 of 72

Cameron is a normally developing 6-month-old boy.

Which of the following developmental milestones would you expect to observe during his assessment?

A Have a good pincer grip

B Lost Moro reflex

C Sits steadily

D Starting to stand with support

E Waves bye bye

Explanation
A normal 6-month old baby should be able to roll over from front to back, readily follow objects with their eyes,
transfer objects from hand to hand, recognise parental voices and babble
and laugh.

The Moro reflex is present from birth and persists until 4 months of age. It is abnormal for the Moro reflex to
persist much beyond this stage and if it does, cerebral palsy should be considered.

Waving bye-bye, having a pincer grip and starting to stand with support are more typical of a 9-12 month old.

Although at 6 months babies are often able to sit briefly or with support, sitting steadily is typically seen at 9
months.
45785

Next Question

Tag Question

https://mypastest.pastest.com/Secure/TestMe/Browser/436619[‫ م‬11:53:09 08/12/1437]


MyPastest

Feedback End Session

Difficulty: Average

Peer Responses

Session Progress

Responses Correct: 0

Responses Incorrect: 75

Responses Total: 75

Responses - % Correct: 0%

Blog
About Pastest
Contact Us
Help

© Pastest 2016

Previous Question

https://mypastest.pastest.com/Secure/TestMe/Browser/436619[‫ م‬11:53:09 08/12/1437]


MyPastest

Prefer to use the old MyPastest? Access it here »

Back to Filters

Question 72 of 72

A normal 6-month-old child is NOTexpected to do which of the following?

A Have no head lag

B Lift head from prone position

C Roll over

D Sit unsupported for 10 minutes

E Transfer objects from hand to hand

Explanation
A 6-month-old child may sit unsupported for a short period of time but is unlikely to do so for as long as 10
minutes until about 9 months.

Lifting the head from the prone position and a steady head when pulled to sit are seen around 3 months of age.

Rolling over and transferring objects from hand to hand are well established skills of a 6-month-old.
45853

End Session

Previous Question Tag Question

Feedback

Difficulty: Average

https://mypastest.pastest.com/Secure/TestMe/Browser/436619[‫ م‬11:53:31 08/12/1437]


MyPastest

Peer Responses

Session Progress

Responses Correct: 0

Responses Incorrect: 76

Responses Total: 76

Responses - % Correct: 0%

Blog
About Pastest
Contact Us
Help

© Pastest 2016

https://mypastest.pastest.com/Secure/TestMe/Browser/436619[‫ م‬11:53:31 08/12/1437]


MyPastest

Prefer to use the old MyPastest? Access it here »

Back to Filters

Question 1 of 9

Under which of the following circumstances would a child’s


name automatically be removed from the Child
Protection Register?

A When the child reaches the age of 18

B Following a routine review case conference

C As soon as the child moves out of the local authority’s area

D When there is dissent from the parents about their child’s name being placed on the register

E Following a professional concerns’ meeting

Explanation
The child may be de-registered for three reasons:

1. The child reaches the age of 18 years, in which case his/her name will be automatically removed.
2. Following
a review case conference in which the child is felt to no longer be at risk, in which case the case
conference needs to be satisfied that the circumstances have significantly changed.
3. Following transfer to another local authority once the receiving local authority has accepted care of the child
and organised their own case conference and appropriate registration.
Next Question
The parents should always be actively involved in the child protection process and encouraged to attend case
conferences. It is, however, not always possible to have their consent to register their children and their dissent is
not a reason for the child’s name to be removed from the register. They can write a complaint to the area services
manager for social work. A child’s name can only be placed on, or removed from, the register following a case
conference, not any other type of meeting.
11512

https://mypastest.pastest.com/Secure/TestMe/Browser/436619[‫ ص‬05:27:49 08/12/1437]


MyPastest

Tag Question

Feedback

Difficulty: Average

Peer Responses

Session Progress

Responses Correct: 0

Responses Incorrect: 1

Responses Total: 1

Responses - % Correct: 0%

Blog
About Pastest
Contact Us
Help

© Pastest 2016

Previous Question
End Session

https://mypastest.pastest.com/Secure/TestMe/Browser/436619[‫ ص‬05:27:49 08/12/1437]


MyPastest

Prefer to use the old MyPastest? Access it here »

Back to Filters

Question 2 of 9

Which of the following scenarios is most suggestive of abuse?

A Different age fractures in 7-month-old non-identical twins

B Perianal warts in a baby of 6 months age

C A tibial fracture in a toddler with blue sclera

D Three circular blisters, 1 cm round, with raised, erythematous, crusty borders occurring on the face and
hands of a toddler

E Multiple bruises of different ages in a toddler

Explanation
Non-mobile infants with fractures without
a consistent story are highly suggestive of abuse. Perianal warts can be
passed from the mother by vertical transmission or by normal childcare as well so are not pathognomonic for
abuse but can be a consequence of child sexual abuse. Osteogenesis imperfecta is characterised by blue sclera,
multiple fractures, a family history of deafness and dentinogenesis imperfecta. Cigarette lesions have a
characteristic appearance but similar lesions are present in impetigo. Multiple bruises occur in toddlers when
mobile. Next Question
Exam theme question from October 2015
11516

Tag Question

https://mypastest.pastest.com/Secure/TestMe/Browser/436619[‫ ص‬05:28:21 08/12/1437]


MyPastest

Feedback

Difficulty: Average

Peer Responses

Session Progress

Responses Correct: 0

Responses Incorrect: 2

Responses Total: 2

Responses - % Correct: 0%

Blog
About Pastest
Contact Us
Help

© Pastest 2016

Previous Question
End Session

https://mypastest.pastest.com/Secure/TestMe/Browser/436619[‫ ص‬05:28:21 08/12/1437]


MyPastest

Prefer to use the old MyPastest? Access it here »

Back to Filters

Question 3 of 9

In an unconscious 1-year-old infant, who had fallen from his/her


parent’s bed onto a carpeted floor, which of the
following signs would be most suggestive of abuse?

A Bulging fontanelle

B Haematuria

C Depressed occipital skull fracture

D Shin bruising

E Parietal crack fracture

Explanation
A bulging fontanelle and haematuria are non-specific
signs. Shin bruising occurs in mobile 1-year-old infants. A
depressed occipital skull fracture in itself is highly suggestive of abuse and indicates a significant force has been
experienced. Accidental fractures
tend to be single and in the parietal region. 11518

Next Question
Tag Question

Feedback
Previous Question
End Session
Difficulty: Average

Peer Responses

https://mypastest.pastest.com/Secure/TestMe/Browser/436619[‫ ص‬05:28:43 08/12/1437]


MyPastest

Session Progress

Responses Correct: 0

Responses Incorrect: 3

Responses Total: 3

Responses - % Correct: 0%

Blog
About Pastest
Contact Us
Help

© Pastest 2016

https://mypastest.pastest.com/Secure/TestMe/Browser/436619[‫ ص‬05:28:43 08/12/1437]


MyPastest

Prefer to use the old MyPastest? Access it here »

Back to Filters

Question 4 of 9

Which of the following is conclusive of, or very strong evidence of, sexual abuse in a child?

A Sexually transmitted disease

B Enlargement of the hymenal opening to 8 mm in a pre-pubertal child

C Patchy localised reddening on the inner sides of the labia

D Anal laxity or reduction of the power of the anal sphincter

E Genital or anal warts

Explanation
Clinical findings rarely provide conclusive proof of
sexual abuse. Many children who are sexually abused do not
have abnormal physical signs. The hymenal opening of normal pre-pubertal girls is approximately 4 mm, there are
individual variations but an opening >1 cm is not seen in normal pre-pubertal children. Hymenal size alone should
not be used as sole evidence for abuse. Patchy localised reddening can occur from local irritation by focal
infection, by underwear or the child’s own fingers resulting in vulvovaginitis. Anal laxity can occur following
constipation or anal stretching for stenosis. Genital or anal warts can be transmitted vertically to infants or be
passed on by poor family hygiene as well as by sexual abuse. Sexually transmitted diseases and pregnancy are two
signs that provide strong evidence of sexual abuse.
Next Question 11521

Previous Question
End
Tag Session
Question

Feedback

https://mypastest.pastest.com/Secure/TestMe/Browser/436619[‫ ص‬05:29:04 08/12/1437]


MyPastest

Difficulty: Easy

Peer Responses

Session Progress

Responses Correct: 0

Responses Incorrect: 4

Responses Total: 4

Responses - % Correct: 0%

Blog
About Pastest
Contact Us
Help

© Pastest 2016

https://mypastest.pastest.com/Secure/TestMe/Browser/436619[‫ ص‬05:29:04 08/12/1437]


MyPastest

Prefer to use the old MyPastest? Access it here »

Back to Filters

Question 5 of 9

In which of the following cases, does the biological mother lose her parental responsibility?

A Her child is taken into local authority care

B Her child is adopted

C The biological father takes over care of her child

D Her child moves to live with her maternal grandparents

E Her other children are taken into local authority care

Explanation
Parental responsibility is held by the biological mother of a child and only removed following adoption. The
parental responsibilities may in other situations be shared with the local authorities or other parties. Parental
responsibility is only conveyed to the father if married to the mother or, after 1 December 2003, if his
name is on
the child’s birth certificate. 11524

Next Question
Tag Question

Feedback
Previous Question
End Session
Difficulty: Easy

Peer Responses

https://mypastest.pastest.com/Secure/TestMe/Browser/436619[‫ ص‬05:29:28 08/12/1437]


MyPastest

Session Progress

Responses Correct: 0

Responses Incorrect: 5

Responses Total: 5

Responses - % Correct: 0%

Blog
About Pastest
Contact Us
Help

© Pastest 2016

https://mypastest.pastest.com/Secure/TestMe/Browser/436619[‫ ص‬05:29:28 08/12/1437]


MyPastest

Prefer to use the old MyPastest? Access it here »

Back to Filters

Question 6 of 9

You are called about a day-old baby on the postnatal ward; he


is yet to have a baby check. The mother has
revealed recreational drug use in her pregnancy. She is happy for urine toxicology to be sent on the baby. The
baby’s father arrives and is angry at this, and is threatening to take the mother and baby home against medical
advice. The
parents are unmarried.

What is the best course of action?

A Call security to stop them leaving so you can assess the baby

B Call the police to stop them leaving so you can assess the baby

C Allow them to leave but ask the GP to check on the baby

D Allow them to leave but inform the duty social worker

E Allow them to leave and take no further action

Explanation
There is no immediate risk to the child that would necessitate the police being called. Hospital security has no
power to stop someone discharging him or herself. The general practitioner might well be able to check on the
baby but there is a need
to inform Social Services. There are child protection risk factors for this baby – the
Next Question
maternal substance use and the father’s behaviour – so Social Services clearly need to be informed.
22243

Previous Question

Tag Question

https://mypastest.pastest.com/Secure/TestMe/Browser/436619[‫ ص‬05:29:48 08/12/1437]


MyPastest

Feedback

Difficulty: Average

Peer Responses

Session Progress

Responses Correct: 0

Responses Incorrect: 6

Responses Total: 6

Responses - % Correct: 0%

Blog
About Pastest
Contact Us
Help

© Pastest 2016

End Session

https://mypastest.pastest.com/Secure/TestMe/Browser/436619[‫ ص‬05:29:48 08/12/1437]


MyPastest

Prefer to use the old MyPastest? Access it here »

Back to Filters

Question 7 of 9

A 4-year-old girl is suffering physical abuse at home. Which of the following interventions would be most
appropriate to apply if her welfare is thought to be in immediate danger?

A Police Protection Order (PPO)

B Emergency Protection Order

C Court wardship

D Section 47 meeting

E Temporary foster placement

Explanation
If the child is thought to be in immediate danger
then a PPO is the quickest and most effective as it allows the
child to
be taken to a place of safety. An EPO Court Wardship and temporary fostering may occur later but not
immediately. A section 47 meeting is convened by Social Services for professionals and family to assess a child’s
needs and the ability of the current carer to ensure a safe and nurturing environment.
22380

Next Question

Tag Question
Previous Question
Feedback
End Session

Difficulty: Difficult

https://mypastest.pastest.com/Secure/TestMe/Browser/436619[‫ ص‬05:30:09 08/12/1437]


MyPastest

Peer Responses

Session Progress

Responses Correct: 0

Responses Incorrect: 7

Responses Total: 7

Responses - % Correct: 0%

Blog
About Pastest
Contact Us
Help

© Pastest 2016

https://mypastest.pastest.com/Secure/TestMe/Browser/436619[‫ ص‬05:30:09 08/12/1437]


MyPastest

Prefer to use the old MyPastest? Access it here »

Back to Filters

Question 8 of 9

Which of the following statements best describes the circumstances in which parents would be legally permitted to
smack their
child?

A Anywhere on the body, but not on the face

B Anywhere on the body, but only after consideration (not in the heat of the moment)

C Anywhere on the body, as long as only the hand is used and no mark is left

D Anywhere on the body, except the face, but only in the home and not in public

E Anywhere on the body, as long as any implement used does not leave a mark

Explanation
The hand must be open (not a clenched fist), no implement may be used and ‘no injury’ inflicted (so it must leave
no mark). It could well be argued that even if no mark is left, an
‘injury’ may still have been inflicted.

See the Children Act 2004 and any updates for the latest Information:

http://www.legislation.gov.uk/ukpga/2004/31/section/58 22399

Next Question

Previous Question Tag Question

Feedback End Session

Difficulty: Average

https://mypastest.pastest.com/Secure/TestMe/Browser/436619[‫ ص‬05:30:29 08/12/1437]


MyPastest

Peer Responses

Session Progress

Responses Correct: 0

Responses Incorrect: 8

Responses Total: 8

Responses - % Correct: 0%

Blog
About Pastest
Contact Us
Help

© Pastest 2016

https://mypastest.pastest.com/Secure/TestMe/Browser/436619[‫ ص‬05:30:29 08/12/1437]


MyPastest

Prefer to use the old MyPastest? Access it here »

Back to Filters

Question 9 of 9

Which of the following is the only definite sign of sexual abuse in children?

A Perianal soreness

B Cliteromegaly

C Pregnancy

D Anal fissures

E Anal skin tags

Explanation
There may be general signs of sexual abuse (e.g. superficial injuries, recurrent UTI); perineal signs (e.g. soreness,
vaginal discharge) and behavioural signs (e.g. sexualised behaviour, depression, bedwetting, drug dependence).
However, it is clear that most
of these signs can be seen in other diseases.Chlamydial infection and genital warts
are the common sexually transmitted infections (STIs) in child sex abuse and are also definite signs of sexual
abuse. Anal fissures, skin tags, reflex dilatation and perianal bruising also rouse suspicion of, but are not
pathognomonic for, sexual abuse, unlike pregnancy. 28969

End Session

Previous Question Tag Question

Feedback

Difficulty: Easy

https://mypastest.pastest.com/Secure/TestMe/Browser/436619[‫ ص‬05:30:55 08/12/1437]


MyPastest

Peer Responses

Session Progress

Responses Correct: 0

Responses Incorrect: 9

Responses Total: 9

Responses - % Correct: 0%

Blog
About Pastest
Contact Us
Help

© Pastest 2016

https://mypastest.pastest.com/Secure/TestMe/Browser/436619[‫ ص‬05:30:55 08/12/1437]


MyPastest

Prefer to use the old MyPastest? Access it here »

Back to Filters

Question 1 of 10

For which one of the following conditions is routine screening undertaken?

A Cerebral palsy

B Developmental dysplasia of the hip

C HIV

D Haemophilia A

E Hepatitis C

Explanation
Screening for developmental dysplasia of the hip occurs in the neonatal examination at birth and at 6 weeks of
age.

Types
of developmental hip dysplasia include dislocated hip, dislocatable hip, subluxable hip and dysplastic hip.
Clinical examination includes the Ortolani test (for reducing a dislocated hip) and the Barlow test (for dislocatable
or subluxatable hip).
46072

Tag Question

Feedback

Difficulty: Easy

https://mypastest.pastest.com/Secure/TestMe/Browser/436619[‫ ص‬05:34:56 08/12/1437]


MyPastest

Peer Responses

Session Progress

Responses Correct: 0

Responses Incorrect: 1

Responses Total: 1

Responses - % Correct: 0%

Blog
About Pastest
Contact Us
Help

© Pastest 2016

https://mypastest.pastest.com/Secure/TestMe/Browser/436619[‫ ص‬05:34:56 08/12/1437]


MyPastest

Prefer to use the old MyPastest? Access it here »

Back to Filters

Question 2 of 10

You are giving advice to a mum who has recently given birth to twins.

Which of the following statements about breastfeeding is correct?

A Breastfeeding for the first week reduces the risk of gastroenteritis

B It is impractical with twins

C It is proven to increase the risk of atherosclerosis in adult life

D It reduces the likelihood of bronchiolitis

E Vitamin D supplements must be given to all breast fed infants, especially twins

Explanation
Although there is increasing evidence that many breast fed infants have low levels of vitamin D, mandatory
supplementation is not currently recommended.

Whilst breastfeeding twins may prove tiring and time consuming, especially in the early weeks when feeding
times for each twin may not coincide, it still remains perfectly possible and the benefits are the same as
breastfeeding a single infant.

Breastfeeding for more than 13 weeks offers some protection from vomiting and diarrhoea and gastroenteritis, ear
infections, bronchiolitis and pneumonia.

Exclusive
breastfeeding is more likely to reduce the risk or severity of allergies. Breast milk also decreases the
risk of atherosclerosis, diabetes, asthma and eczema.
46073

https://mypastest.pastest.com/Secure/TestMe/Browser/436619[‫ ص‬05:35:25 08/12/1437]


MyPastest

Tag Question

Feedback

Difficulty: Average

Peer Responses

Session Progress

Responses Correct: 0

Responses Incorrect: 2

Responses Total: 2

Responses - % Correct: 0%

Blog
About Pastest
Contact Us
Help

© Pastest 2016

https://mypastest.pastest.com/Secure/TestMe/Browser/436619[‫ ص‬05:35:25 08/12/1437]


MyPastest

Prefer to use the old MyPastest? Access it here »

Back to Filters

Question 3 of 10

A 12-year-old girl was diagnosed with meningococcal meningitis. She was treated successfully and made a full
recovery.

Appropriate further action would include which of the following?

A Nasal swabbing of the child's entire class to detect carriers

B Provision of information leaflets to the child's school

C swabbing of the child’s entire school to detect carriers

D Treatment of immediate family with Rifampicin

E Treatment of the entire class with Ciprofloxacin

Explanation
People who have come into close contact with meningococcal meningitis and/or meningococcal septicaemia
require prophylactic antibiotics in the form of Ciprofloxacin. Close contacts are classified as people who are living
or sleeping in the same household as the patient or who have intimately kissed the patient. School friends and
colleagues of the patient with meningococcal disease are rarely at higher risk and do not require treatment or
testing.

The patient will have come into contact with other people during the incubation period. These people are deemed
as casual contacts and they are not at any increased risk. However, it is wise to provide information to the child's
school.
46075

https://mypastest.pastest.com/Secure/TestMe/Browser/436619[‫ ص‬05:37:00 08/12/1437]


MyPastest

Tag Question

Feedback

Difficulty: Average

Peer Responses

Session Progress

Responses Correct: 0

Responses Incorrect: 3

Responses Total: 3

Responses - % Correct: 0%

Blog
About Pastest
Contact Us
Help

© Pastest 2016

https://mypastest.pastest.com/Secure/TestMe/Browser/436619[‫ ص‬05:37:00 08/12/1437]


MyPastest

Prefer to use the old MyPastest? Access it here »

Back to Filters

Question 4 of 10

Which of the following maternal drug treatments are contra-indicated in breast feeding?

A Acetylsalicylic acid

B Amoxycillin

C Insulin

D Paracetamol

E Warfarin

Explanation
The WHO Working Group on Human Lactation classified Aspirin (acetylsalicyclic acid) as unsafe for use by
nursing women. It should be avoided due to the possible risk of Reye's syndrome and in high doses could impair
platelet function.

Antibiotics do not necessarily demand the cessation of breastfeeding although many produce very loose stools in
babies.

Warfain does not pass into breast milk.

Diabetic medications such as insulin, metformin or sulphonylureas are not contraindicated in breastfeeding
mothers.

Paracetamol
is considered the best form of pain relief whilst breastfeeding. Although paracetamol may pass into
breast milk, the amount is too small to cause any harm to the baby.
46076

https://mypastest.pastest.com/Secure/TestMe/Browser/436619[‫ ص‬05:37:20 08/12/1437]


MyPastest

Tag Question

Feedback

Difficulty: Average

Peer Responses

Session Progress

Responses Correct: 0

Responses Incorrect: 4

Responses Total: 4

Responses - % Correct: 0%

Blog
About Pastest
Contact Us
Help

© Pastest 2016

https://mypastest.pastest.com/Secure/TestMe/Browser/436619[‫ ص‬05:37:20 08/12/1437]


MyPastest

Prefer to use the old MyPastest? Access it here »

Back to Filters

Question 5 of 10

You are leading a reminder training session with a group of nurses on healthcare associated infections and hand
washing.

Which of the following statements is NOT correct?

A Ordinary soap and water is effective against bacteria

B Staff should wash hands after every patient contact

C Staff should wash hands before examining every patient

D Staff should wash hands before and after patient contact even when wearing gloves

E Pseudomonas aeruginosa cannot be removed with the use of alcohol rubs

Explanation
Formal handwashing with soap and water is required when there is soiling. When there is none the hand hygiene
liaison group now advocates that staff should use an alcohol-glycerol hand rub between patients.

Alcohol rubs can be effective against gram positive and negative bacteria, including Staphylococcus aureus,
MRSA, E-coli, Psudomonas and Enterococcus. They are also effective against viruses such as Influenza and
Herpes but not norovirus where handwashing is recommended.

NICE guidance can be found via the following link:

https://www.nice.org.uk/guidance/cg139/chapter/1-guidance 46077

https://mypastest.pastest.com/Secure/TestMe/Browser/436619[‫ ص‬05:38:00 08/12/1437]


MyPastest

Tag Question

Feedback

Difficulty: Easy

Peer Responses

Session Progress

Responses Correct: 0

Responses Incorrect: 5

Responses Total: 5

Responses - % Correct: 0%

Blog
About Pastest
Contact Us
Help

© Pastest 2016

https://mypastest.pastest.com/Secure/TestMe/Browser/436619[‫ ص‬05:38:00 08/12/1437]


MyPastest

Prefer to use the old MyPastest? Access it here »

Back to Filters

Question 6 of 10

Which of the following is not consistent with a finding of lead poisoning?

A A history of pica

B Constipation

C Abdominal pain

D Rickets

E Mental retardation

Explanation
The commonest sources of lead poisoning come from
paints and pipes in old buildings since lead used to be in
these substances. In addition to encephalopathy with drowsiness and convulsions, children may also get raised
intracranial pressure. Earlier
signs include vomiting, loss of appetite and weight, abdominal pain and
constipation
with lead fragments seen on plain abdominal X-ray. A blood
film will show basophilic stippling, anaemia
(hypochromic microcytic picture). Pica is common and the child eats non-nutritive items such as sticks, wood,
foam, paper etc.
46080

Tag Question

Feedback

https://mypastest.pastest.com/Secure/TestMe/Browser/436619[‫ ص‬05:38:21 08/12/1437]


MyPastest

Difficulty: Average

Peer Responses

Session Progress

Responses Correct: 0

Responses Incorrect: 6

Responses Total: 6

Responses - % Correct: 0%

Blog
About Pastest
Contact Us
Help

© Pastest 2016

https://mypastest.pastest.com/Secure/TestMe/Browser/436619[‫ ص‬05:38:21 08/12/1437]


MyPastest

Prefer to use the old MyPastest? Access it here »

Back to Filters

Question 7 of 10

The role of the health visitor includes:

A Child health surveillance in all children under 10 years of age

B Reviewing every child under 10 years who has attended the Emergency Department

C Responsibility for supervision of children in care

D Supervising the running of immunisation clinics

E Taking over postnatal care from the midwife at 3 weeks of age

Explanation
The health visitor relieves the midwife of responsibility at 10 days post delivery and is subsequently responsible
for child health surveillance in all children up to 5 years of age. The paediatric liaison health visitor will be
informed of every child under 5
who attends the emergency department and they are then obliged to contact the
community health visitor of any child who needs appropriate follow-up. Health visitors also supervise the running
of immunisation clinics, however social workers are responsible for children in care.
46081

Tag Question

Feedback

Difficulty: Average

https://mypastest.pastest.com/Secure/TestMe/Browser/436619[‫ ص‬05:38:40 08/12/1437]


MyPastest

Peer Responses

Session Progress

Responses Correct: 0

Responses Incorrect: 7

Responses Total: 7

Responses - % Correct: 0%

Blog
About Pastest
Contact Us
Help

© Pastest 2016

https://mypastest.pastest.com/Secure/TestMe/Browser/436619[‫ ص‬05:38:40 08/12/1437]


MyPastest

Prefer to use the old MyPastest? Access it here »

Back to Filters

Question 8 of 10

Which of the following tests is the most appropriate to check the hearing of a 5-year-old child?

A Auditory brain-stem-evoked response

B Distraction test

C Impedance audiometry

D Otoacoustic emissions

E Pure-tone audiometry

Explanation
Pure tone audiometry is performed by placing headphones on the child and playing tones of various frequencies
and intensities through them. The test requires a degree of co-operation, which should be present in a 5-year-old.
Otoacoustic emissions (measuring the sound 'echo' from the cochlear) are a useful screening test, but they cannot
distinguish between conductive and sensory deficits. Brainstem-evoked responses (measuring brain-wave
response to auditory stimulus) can be performed at any age but is a lengthy test. The distraction test is routinely
performed by health visitors at 8 months of age.
46082

Tag Question

Feedback

https://mypastest.pastest.com/Secure/TestMe/Browser/436619[‫ ص‬05:39:06 08/12/1437]


MyPastest

Difficulty: Average

Peer Responses

Session Progress

Responses Correct: 0

Responses Incorrect: 8

Responses Total: 8

Responses - % Correct: 0%

Blog
About Pastest
Contact Us
Help

© Pastest 2016

https://mypastest.pastest.com/Secure/TestMe/Browser/436619[‫ ص‬05:39:06 08/12/1437]


MyPastest

Prefer to use the old MyPastest? Access it here »

Back to Filters

Question 9 of 10

Public Health England has identified a number of areas to focus on over the next few years to protect and improve
the nation's health.

Which of the following is applicable to children and/or adolescents?

A Addressing speech and language difficulties in the under 2s

B Reducing harmful drinking

C Reducing smoking

D Tackling obesity

E All of the above

Explanation
Health promotion is a major part of health policy. Important areas relating specifically to children and adolescents
include the following:

Source: www.gov.uk/government

Smoking

In 2013 approximately 8% of 15-year-olds were regular smokers and 10% occasional smokers

Obesity

Associated
with bullying in childhood includes the promotion of a health diet and excercise. the
number of obese children doubles from reception to year 6

Alcoholic consumption

https://mypastest.pastest.com/Secure/TestMe/Browser/436619[‫ ص‬05:39:26 08/12/1437]


MyPastest

Leading risk factor for preventable accidents in the 15-years+ age group

Increasing readiness of children for learning and school

Socially
disadvantaged children are more likely to have speech and language development
problems which will decrease their ability to learn on attending school

46083

Tag Question

Feedback

Difficulty: Average

Peer Responses

Session Progress

Responses Correct: 0

Responses Incorrect: 9

Responses Total: 9

Responses - % Correct: 0%

Blog
About Pastest
Contact Us
Help

© Pastest 2016

https://mypastest.pastest.com/Secure/TestMe/Browser/436619[‫ ص‬05:39:26 08/12/1437]


MyPastest

Prefer to use the old MyPastest? Access it here »

Back to Filters

Question 10 of 10

Which of the following is an example of secondary prevention?

A Child-proof catches on cupboards

B Fire-extinguishers kept in the house

C Speed limits

D Stair gates

E teaching parents first-aid skills

Explanation
Secondary prevention aims to prevent injury should the 'accident' happen, whereas primary prevention is aimed at
preventing the 'accident' from happening and includes speed limits, stair gates, teaching road safety and child-
proof catches on cupboards.

Examples
of secondary prevention include cycling helmets, seat belts, smoke alarms and fire extinguishers kept in
the house. Note that blister packs
for prescription drugs merely limit the number of tablets that a child can get at in
a given time and are therefore a form of secondary prevention, whereas child-resistant lids are a form of primary
prevention because they prevent the child from reaching the drug.

Tertiary
prevention aims to limit the impact of an injury once the 'accident' has happened and includes teaching
parents first aid skills and providing good access to the emergency services.
46084

Tag Question

https://mypastest.pastest.com/Secure/TestMe/Browser/436619[‫ ص‬05:39:48 08/12/1437]


MyPastest

Feedback

Difficulty: Difficult

Peer Responses

Session Progress

Responses Correct: 0

Responses Incorrect: 10

Responses Total: 10

Responses - % Correct: 0%

Blog
About Pastest
Contact Us
Help

© Pastest 2016

https://mypastest.pastest.com/Secure/TestMe/Browser/436619[‫ ص‬05:39:48 08/12/1437]


Back to Filters

Question 1 of 26

Difficulty: Average

The following statements refer to oppositional defiant disorder.


Select the MOST appropriate

A Comorbidity with other disruptive behaviours is uncommon

B It can be least evident with adult and peers that the child knows well

C It is classically seen in teenagers

D It often includes behaviour that violates the law

E Toddler tantrums are a form of this

Explanation
This type of conduct disorder is characteristically seen in children below the age of 10 years. It is defined by the
presence of markedly defiant, disobedient, provocative behaviour, and by the absence of more severe dissocial or
aggressive acts that violate the law or the rights of others. The behaviour can be most evident with adults whom
the child knows well or peers.

The
behaviour must be repetitive and persistent and must be well outside the normal social expectations of that
age child, eg toddler tantrums are normal for 2–3 year olds and would not be classed as oppositional defiant
disorder.
Back to Filters

Question 2 of 26

Difficulty: Average
The following statements refer to enuresis.
Select the MOST appropriate

A Enuresis is defined as continued wetting beyond the age of 3 years

B It is more common in girls than in boys

C Nocturnal enuresis can by treated with antidiuretic hormone

D Primary enuresis is more likely than secondary enuresis to have an underlying organic cause

E Restricting fluid throughout the day should be encouraged in primary enuresis

Explanation
Enuresis is defined as wetting beyond the
age of 6 years. It is very common and affects at least 1 in 10 children
of
primary school age. It can be divided into primary enuresis, when bladder control has never been established, and
secondary enuresis when continence has been established then lost. The majority of primary enuresis, particularly
nocturnal enuresis >95%, is due to a delay in the maturation of the urethral sphincter. It is more common in boys
and can often run in families. Secondary enuresis can be due to psychological stressful events but is more likely
than primary to be due
to an organic cause.

Organic causes:

Urinary tract infection


Congenital abnormalities, eg spina bifida, ectopic ureter
Endocrine: diabetes mellitus and diabetes insipidus
Pelvic masses, eg constipation, hydrocolpos.

In the case of primary nocturnal enuresis, after a careful history and examination, advice to parents needs to be
clear that this is a common problem, which is likely to improve with time. Good volume of fluid intake needs to
be encouraged, only restricting fluid intake in the last hour before going to bed. Caffeine drinks need to be
avoided. Advice should be given on the importance of regular toileting during the day and before sleep. Reward
systems can be used but should be based on agreed behaviour rather than on dry nights. Alarm treatment and
antidiuretic hormone drug treatment (desmopressin) can then be used alongside these measures.

Treatment in organic causes involves treating the underlying cause.


MyPastest

Prefer to use the old MyPastest? Access it here »

Back to Filters

Question 3 of 26

Next Question
The following statements refer to parental responsibility.

Select the MOST appropriate

A A father loses parental responsibility if he divorces the mother of his child

B A stepfather has automatic parental responsibility for his wife’s children

C Parents lose parental responsibility when their children are voluntarily taken into local authority care

D Parental responsibility is automatically given to the mother

E Only the mother holds parental responsibility if parents are unmarried

Explanation
All mothers, whether married or unmarried, automatically have parental responsibility for any child born to them.
Having parental responsibility means assuming all the rights, duties, powers, responsibilities and authority that a
parent of a child has by law. In England and Wales, if the parents of a child are married to each
other at the time of
the birth, or if they have jointly adopted a child, then they both have parental responsibility. An unmarried father
will automatically gain parental responsibility by jointly registering the child’s birth with the mother (for births
registered after 1 December 2003), pre-December 2003 a father would need to re-register the
birth, to add the
father’s details, and gain parental responsibility. Parents do not lose parental responsibility if they divorce
irrespective of whom the child resides with nor do they lose parental responsibility if a child is placed in local
authority care. 43907

Tag Question

https://mypastest.pastest.com/Secure/TestMe/Browser/436619[‫ ص‬07:51:46 08/12/1437]


MyPastest

Feedback

Difficulty: Average

Peer Responses

Previous Question
Session Progress
End Session
Responses Correct: 1

Responses Incorrect: 2

Responses Total: 3

Responses - % Correct: 33%

Blog
About Pastest
Contact Us
Help

© Pastest 2016

https://mypastest.pastest.com/Secure/TestMe/Browser/436619[‫ ص‬07:51:46 08/12/1437]


MyPastest

Prefer to use the old MyPastest? Access it here »

Back to Filters

Question 4 of 26

A 4-year-old girl is referred to children’s outpatients with a 2-month history of passing irregular hard stool.

It
has been causing her such distress that her mother is presently not sending her to school. On examination she
has a soft abdomen but a small
anal fissure.

What is the BEST advice to give the parents?

A Advise mum to increase her fluid and fibre intake

B Her anal fissure is a worrying sign of possible abuse and will need further investigation

C Her
constipation is likely to need treatment for several months to get her stool soft and regular and
allow her anal fissure to heal

D She will need to be admitted to hospital to have an enema to get on top of it and get her back to school

E To
keep her off school until the constipation is completely treated; she doesn’t have to legally go to
school until she is 5 years of age

Explanation
Constipation in childhood is common. It is estimated that it affects 5–30% of the child population. Common times
during which constipation can occur are during infant weaning, toilet training and starting school. If there are no
signs of faecal impaction, admission to hospital should not be required. If a diagnosis of idiopathic constipation is
made, then treatment could be started. The
presence of a single anal fissure would be in keeping with a diagnosis
of idiopathic constipation. Poor fluid intake can contribute as well as concerns over toileting at school. School
refusal due to this is not common but can occur. Dietary and lifestyle changes should be encouraged
but should
not be used alone as first-line treatment.

NICE clinical guidelines exist on constipation in children and young people:

https://www.nice.org.uk/guidance/cg99 43908

https://mypastest.pastest.com/Secure/TestMe/Browser/436619[‫ ص‬07:52:44 08/12/1437]


MyPastest

Tag Question

Feedback

Difficulty: Average

Peer Responses

Next Question

Previous Question
End Session

Session Progress

Responses Correct: 1

Responses Incorrect: 3

Responses Total: 4

Responses - % Correct: 25%

Blog
About Pastest
Contact Us
Help

© Pastest 2016

https://mypastest.pastest.com/Secure/TestMe/Browser/436619[‫ ص‬07:52:44 08/12/1437]


MyPastest

Prefer to use the old MyPastest? Access it here »

Back to Filters

Question 5 of 26

A 15-year-old girl presents with severe weight loss over the last 6 months.

Her
mum reports that she has been exercising excessively and has not menstruated for several months. A number
of investigations including TFTs (thyroid function tests) and coeliac screen have not showed any abnormality. She
has a resting heart rate of 40/min and a marked postural drop in her blood pressure. She is cool peripherally.

What is the BEST advice to give parents?

A A
diagnosis of anorexia nervosa is not likely but she appears dehydrated and therefore her mother
should encourage her to take fluids

B A diagnosis of anorexia nervosa is possible and she will need to be admitted to hospital for
management of her acute state

C A diagnosis of anorexia nervosa is possible and you will refer her on to a psychiatric outpatients

D Advise her mother to stop her daughter exercising so much

E To keep a food diary over the next few weeks and come back to see you later that month

Explanation
A diagnosis of anorexia nervosa must be considered in this case. Although most patients with anorexia nervosa
should, where possible, be treated as an outpatient, this girl is showing worrying signs of the medical problems
associated with anorexia and she needs admission to stabilise her acute clinical state. At this stage she is likely to
need a careful medical evaluation including electrolytes to look for any imbalances; she may need nasogastric
feeding or intravenous fluids depending on her hydration status. This should all be managed in a setting that can
provide the skilled implementation of re-feeding with careful physical and electrolyte monitoring (particularly in
the first few days of re-feeding), in combination with psychosocial interventions.

The characteristic features of anorexia nervosa are:

https://mypastest.pastest.com/Secure/TestMe/Browser/436619[‫ ص‬07:53:32 08/12/1437]


MyPastest

Severe weight loss


Excessive exercising
Depression (in about 50%)
Self-induced vomiting
Laxative abuse
Associated distorted self-image
Morbid fear of being fat.

It
is more common in pubertal girls, in whom amenorrhoea is usually present, than in prepubertal girls. The
incidence is higher in pupils of
fee-paying schools and children of higher socioeconomic status. It is also higher in
certain populations such as ballet dancers and fashion students. Concordance is greater in monozygotic than
dizygotic twins. The prevalence of anorexia is 0.5–1.0%; 1 in 10 cases are boys, and 50% make a complete
Next Question
recovery, 25% make a partial recovery with some residual minor eating problems and the remaining 25% persist
with a chronic course of illness. The mortality rate is 5–10%.

Bulimia
nervosa is characterised by binge eating followed by self-induced vomiting and laxative abuse. These
Previous
individuals are not Questionbut share the same fear as sufferers of anorexia nervosa of becoming fat.
usually underweight
Dental caries is common, with a prevalence estimated at 1%. The severity is variable. It is more common in late
End Session
adolescence and early adult life than in childhood. The prevalence in childhood is much lower.

Medical problems:

Reduced metabolic rate with bradycardia, postural hypotension, peripheral cyanosis, cold intolerance and
lethargy
Impaired resistance to infection with bone marrow suppression which can be marked
Amenorrhoea, hypocortisolaemia, impaired thyroid function, lanugo hair, osteopenia
Hypoalbuminaemia, oedema, constipation
Electrolyte
abnormalities including secondary to laxative abuse, raised urea secondary to reduced fluid
intake, impaired liver function
Elevated amylase with parotid swelling or pancreatitis
Effects of vitamin deficiency including hair loss

NICE Guidance:

https://www.nice.org.uk/guidance/cg9 43911

Tag Question

Feedback

https://mypastest.pastest.com/Secure/TestMe/Browser/436619[‫ ص‬07:53:32 08/12/1437]


MyPastest

Difficulty: Average

Peer Responses

Session Progress

Responses Correct: 1

Responses Incorrect: 4

Responses Total: 5

Responses - % Correct: 20%

Blog
About Pastest
Contact Us
Help

© Pastest 2016

https://mypastest.pastest.com/Secure/TestMe/Browser/436619[‫ ص‬07:53:32 08/12/1437]


MyPastest

Prefer to use the old MyPastest? Access it here »

Back to Filters

Question 6 of 26

The following statements refer to scoliosis.

Select the MOST appropriate

A Idiopathic scoliosis is more common in girls

B Initiation of treatment is required once the curvature is greater than 35°

C Neuromuscular scoliosis accounts for >75% of cases

D Scoliosis is a lateral and rotational curvature of the thoracic and lumbar spine, measuring >20°

E Scoliosis repair is not advisable in patients with Duchene muscular dystrophy due to the reduced life
expectancy

Explanation
Scoliosis is a lateral and rotational curvature of the thoracic and lumbar spine measuring >10°. Idiopathic scoliosis
accounts for approximately 75% of all cases and is most common in adolescent girls, with the remaining 25% of
cases occurring in patients with an acquired deformity that results from a peripheral or central neurological
impairment or attributable to connective tissue and musculoskeletal disorders. Children with severe neurological
impairment are at high risk for the development of scoliosis, eg 90% of boys with Duchenne muscular dystrophy
(DMD) will develop scoliosis. In cerebral palsy (CP), the incidence is highest in those most severely affected,
usually with quadraplegic, hemiplegic and dystonic forms of CP.

The timing and type of surgery depend on the patient’s age, rate of progression, skeletal
maturity, symptoms,
underlying diagnosis, degree and location of curvature, and cardiopulmonary function. In idiopathic scoliosis,
curves
that exceed 40–50° before the onset of skeletal maturity usually require surgery to prevent progression and
to diminish spinal deformity. In neuromuscular scoliosis, surgery is highly dependent on aetiology and rate of
progression. For example, as scoliosis is relentlessly progressive in most cases of DMD, surgery is recommended
as
soon as a progression of curvature can be established, generally at 20–30°. This generally occurs within 2–5

https://mypastest.pastest.com/Secure/TestMe/Browser/436619[‫ ص‬07:54:42 08/12/1437]


MyPastest

years of wheelchair dependence, so the recognition at the early stage of curvature is important and requires careful
monitoring.
43913

Tag Question

Feedback

Difficulty: Average

Next Question
Peer Responses

Previous Question
End Session

Session Progress

Responses Correct: 1

Responses Incorrect: 5

Responses Total: 6

Responses - % Correct: 17%

Blog
About Pastest
Contact Us
Help

© Pastest 2016

https://mypastest.pastest.com/Secure/TestMe/Browser/436619[‫ ص‬07:54:42 08/12/1437]


MyPastest

Prefer to use the old MyPastest? Access it here »

Back to Filters

Question 7 of 26

A 7-year-old boy has a diagnosis of attention deficit hyperactivity disorder.

Over
the last 2 years his parents have had parental training to manage his condition and he has had psychological,
behavioural and educational intervention. Despite this he continues to have problems both at home and at school
with his hyperactivity, and of late his teachers feel that
it is having an adverse effect on his attainment at school.

What is the MOST appropriate next step?

A Discuss with parents the use of medication as part of a comprehensive treatment plan

B Reassure parents that he is likely to grow out of his condition, given time

C Stop the psychological intervention and start medication

D To advise eliminating artificial colouring and additives from the diet

E Treat his mother for depression

Explanation
Behavioural therapy can lead to improvement but it is intensive. Drug therapy is with stimulant drugs such as
dexamfetamine and methylphenidate, and their use should be alongside psychological, behavioural and
educational intervention. Side effects of drug therapy are common and include insomnia, suppression of appetite
and depression. Rare side effects include psychosis, growth retardation, increased frequency of tics and worsening
of epilepsy.

Modification
of diet has been reported to improve symptoms in a number of children but behavioural modification
or drug therapy or both are much more widely used.

Assessment of symptoms and effect of treatment can be aided by use of the Connor Teacher’s Rating Scale
questionnaire to assess activity and attention at school. The condition is said to persist into adult life in 10–20%.

Some other causes of hyperactivity in children are:

https://mypastest.pastest.com/Secure/TestMe/Browser/436619[‫ ص‬07:55:07 08/12/1437]


MyPastest

normal variant
understimulation/boredom
sleep disturbance
learning difficulties
anxiety disorder
autistic spectrum disorder
temporal lobe epilepsy
drugs, eg antiepileptic medication, sedatives.

43914

Next Question

Previous Question Tag Question

Feedback End Session

Difficulty: Average

Peer Responses

Session Progress

Responses Correct: 1

Responses Incorrect: 6

Responses Total: 7

Responses - % Correct: 14%

Blog
About Pastest
Contact Us
Help

https://mypastest.pastest.com/Secure/TestMe/Browser/436619[‫ ص‬07:55:07 08/12/1437]


MyPastest

© Pastest 2016

https://mypastest.pastest.com/Secure/TestMe/Browser/436619[‫ ص‬07:55:07 08/12/1437]


MyPastest

Prefer to use the old MyPastest? Access it here »

Back to Filters

Question 8 of 26

The following statements refer to school refusal.

Select the MOST appropriate statement

A Associated physical symptoms are uncommon

B Family stress may be a precipitating factor

C Parents are not aware of their child’s absence from school

D The absence from school is often intermittent

E These are often low achieving pupils

Explanation
School refusal is the reluctance or refusal to attend school due to excessive anxiety. It can lead to prolonged
absence from school (weeks to months). In contrast to truancy these children remain at home during school hours
and do not conceal their absence from their parents. It has two peaks of incidence at age 5–6 years and 11 years at
times of school change.

Precipitating factors include

transition from primary to secondary school


family stress, ie illness, marital breakdown
pressure for high academic achievement
bullying

Depression
is present in more than 50% of children and physical symptoms are also common. The outcome is
usually good with a combination of psychological and psychiatric interventions although pharmacological

https://mypastest.pastest.com/Secure/TestMe/Browser/436619#Top[‫ ص‬07:56:24 08/12/1437]


MyPastest

treatment of depression may be needed.


43916

Tag Question

Feedback

Difficulty: Average

Peer Responses

Session Progress

Responses Correct: 1

Responses Incorrect: 7

Responses Total: 8

Responses - % Correct: 13%

Blog
About Pastest
Contact Us
Help

© Pastest 2016

Previous Question Next Question

End Session

https://mypastest.pastest.com/Secure/TestMe/Browser/436619#Top[‫ ص‬07:56:24 08/12/1437]


MyPastest

Prefer to use the old MyPastest? Access it here »

Back to Filters

Question 9 of 26

A 15-year-old boy is referred to a general paediatric outpatients clinic.

His
mother has become increasingly concerned about his low mood. He no longer goes out with his friends and
has become increasingly isolated and anxious about school. She is particularly concerned with his sleep because
he is sleeping poorly and having problems getting to sleep and waking early in the morning. The 15 year old is
difficult to engage in clinic but denies any drug or alcohol consumption.

What is the MOST likely diagnosis?

A Bipolar disorder

B Drug missuse

C Childhood depression

D Normal teenage behaviour

E Schizophrenia

Explanation
Major depressive illness in children is uncommon (2 per 1000), but many children have depressive symptoms and
the incidence of both increases during adolescence. Depression can be primary (isolated depression) or secondary
(to other psychiatric disorder or physical disease):

Primary depression is often associated with a family history of depression.


Prepubertal boys are twice as likely to have depression as prepubertal girls. This sex incidence is reversed
after puberty.
Suicidal behaviour (parasuicide) is common in children who have depression.
Successful suicide in childhood is extremely rare.

https://mypastest.pastest.com/Secure/TestMe/Browser/436619#Top[‫ ص‬07:56:47 08/12/1437]


MyPastest

Symptoms
characteristically include depressed mood and tearfulness, lethargy and
loss of interest in usual
activities, feelings of guilt and self-blame,
diminished appetite and poor weight gain (but appetite can be
increased), impaired sleep (but can be increased), social withdrawal, outbursts of aggression and delusional
symptoms in the form of auditory hallucinations, accusing the child of worthlessness.
43917

Tag Question

Feedback

Difficulty: Average

Peer Responses

Session Progress

Responses Correct: 1

Responses Incorrect: 8

Responses Total: 9

Responses - % Correct: 11%

Blog
About Pastest
Contact Us
Help
Previous Question Next Question
© Pastest 2016

End Session

https://mypastest.pastest.com/Secure/TestMe/Browser/436619#Top[‫ ص‬07:56:47 08/12/1437]


MyPastest

Prefer to use the old MyPastest? Access it here »

Back to Filters

Question 10 of 26

The following statements refer to childhood squint.

Select the MOST appropriate

A Non-paralytic squint rarely requires corrective surgery

B Paralytic squint is more common than non-paralytic squint

C There is an absolute indication for testing of visual acuity

D They can only lead to amblyopia of the affected eye if the squint is paralytic

E They occur in <1% of pre-school children

Explanation
Childhood squint (strabismus) is the misalignment of the eyes during visual fixation. It is common in childhood,
affecting 4% of children aged <6 years. If left untreated it can cause secondary visual loss (amblyopia) in the
affected eye, regardless of the cause.

Squint is divided into two categories:

1. Non-paralytic squint: where there is no abnormality of the extraocular muscles or nerves supplying them
2. Paralytic
squint: where there is a weakness of one or more of the extraocular muscles leading to squint. This
is less common and when it presents a cause must be sought.

Diagnosis:

Corneal
light reflex test: the degree of squint can be assessed using prisms in
front of the eye while doing
the test (Krimsky method)
Cover testing: crucial to determine the presence and amount of ocular deviation
Visual acuity testing: this is mandatory to assess vision in both eyes

https://mypastest.pastest.com/Secure/TestMe/Browser/436619#Top[‫ ص‬08:01:40 08/12/1437]


MyPastest

Eye movements.

Treatment of non-paralytic squint:

Correction of any refractive error


Patching to the normal eye to allow the development of vision in the affected eye
Corrective
surgery: this is largely cosmetic and involves strengthening or weakening the appropriate
extraocular muscles to balance out the squint.
Often a second operation is required to align the eyes fully.

43922

Tag Question

Feedback

Difficulty: Average

Peer Responses

Session Progress

Responses Correct: 1

Responses Incorrect: 9

Responses Total: 10

Previous Question
Responses - % Correct: Next Question 10%

Blog
About Pastest
Contact Us
Help
End Session
© Pastest 2016

https://mypastest.pastest.com/Secure/TestMe/Browser/436619#Top[‫ ص‬08:01:40 08/12/1437]


MyPastest

https://mypastest.pastest.com/Secure/TestMe/Browser/436619#Top[‫ ص‬08:01:40 08/12/1437]


MyPastest

Prefer to use the old MyPastest? Access it here »

Back to Filters

Question 11 of 26

The following statements refer to hearing assessments and screening.

Select the MOST appropriate

A A distraction test is an appropriate hearing assessment for a 3-month-old child

B A normal neonatal hearing screen excludes hearing impairment as a cause of speech and language
delay

C After bacterial meningitis, a formal hearing assesment should be preformed approximately 1 year after
the illness

D Early detection and treatment of hearing impairment have no effect on developmental and educational
outcome

E Neonatal hearing screening in the UK involves automated otoacoustic emission (AOAE)

Explanation
The early detection of hearing impairment
is important; the earlier hearing impairments can be detected and
treated the more successful the developmental and educational outcome.

The
current hearing screeing in the UK is neonatal hearing screening followed by school entry hearing test. A
normal neonatal hearing screen does not exclude hearing impairment, particularly when you consider both
aquired
and progressive causes of hearing loss.

Assessment of deafness:

Otoacoustic
emissions: this is the acoustic response produced by the cochlea in response to sound emitted
by an aural probe. There is no response if hearing loss is >30 dB. It is 100% sensitive and 80% specific for
sensorineural hearing loss.

https://mypastest.pastest.com/Secure/TestMe/Browser/436619#Top[‫ ص‬08:02:49 08/12/1437]


MyPastest

Brainstem-evoked auditory responses: electrodes are placed over the skull and electrical activity is picked
up in response to auditory stimulation. This is a useful test in babies aged <6 months especially in those at
high risk
of deafness (jaundice, prematurity, low birthweight). The test is not affected by sedation.
Distraction testing is used from 6 months to 1 year of age.
Play audiometry can be used after 18 months to 2 years of age.
Pure-tone audiometry is used after 3.5 years of age because this is the age in which cooperation can be
achieved.

43923

Tag Question

Feedback

Difficulty: Average

Peer Responses

Session Progress

Responses Correct: 1

Responses Incorrect: 10

Responses Total: 11

Responses - % Correct: 9%
Previous Question Next Question

Blog
About Pastest
Contact Us
Help

© Pastest 2016 End Session

https://mypastest.pastest.com/Secure/TestMe/Browser/436619#Top[‫ ص‬08:02:49 08/12/1437]


MyPastest

https://mypastest.pastest.com/Secure/TestMe/Browser/436619#Top[‫ ص‬08:02:49 08/12/1437]


MyPastest

Prefer to use the old MyPastest? Access it here »

Back to Filters

Question 12 of 26

The following statements refer to hearing impairment.

Select the MOST appropriate statement

A Alport syndrome is associated with sensorineural hearing loss

B Children presenting aged >2 years with hearing impairment will not have a genetic cause for their
impairment

C Glue ear is an uncommon cause of hearing impairment in children

D Hearing aids are not appropriate for children aged <6 months

E Hearing loss of <20 dB is unlikely to affect development

Explanation
Deafness is conductive, sensorineural or mixed. There are many causes of deafness and these should be learned. A
number of causes are listed below:

Congenital:

autosomal dominant – 33%


autosomal recessive – 65%
X-linked – 2%

Syndromes:
Alport syndrome (with renal failure), Treacher Collins syndrome, Klippel–Feil anomalies of
the spine, Down syndrome, Waardenburg syndrome, Jervell–Lange–Nielsen syndrome associated with
prolonged QT interval and sudden death, Pendred syndrome
Acquired:

https://mypastest.pastest.com/Secure/TestMe/Browser/436619#Top[‫ ص‬08:03:11 08/12/1437]


MyPastest

prenatal: congenital infection – rubella, cytomegalovirus; drugs such as thalidomide


perinatal
– it is common in pre-term infants with a prevalence in very-low-birthweight infants
(<1500 g) of 1%. Pre-term infants are particularly susceptible to hypoxia, hyperbilirubinaemia,
intraventricular haemorrhage and the use of drugs such as gentamicin
older children
otitis media
glue ear
meningitis.

Glue
ear is the most common form of deafness in children. Causes include acute otitis media, allergy and
malformation of the eustachian tube (associated with cleft palate). These all lead to accumulation of fluid in the
middle ear which may become infected. The fluid prevents equalisation of pressures between the middle ear and
the atmosphere from
occurring and damps the response of the eardrum to sound. A large proportion of cases
resolve spontaneously or with a trial of decongestant therapy. If not, treatment is with grommet insertion. The
main indications are speech delay and severe hearing loss.

43925

Tag Question

Feedback

Difficulty: Average

Peer Responses

Previous Question Next Question


Session Progress

Responses Correct: 1

Responses Incorrect: End Session 11

https://mypastest.pastest.com/Secure/TestMe/Browser/436619#Top[‫ ص‬08:03:11 08/12/1437]


MyPastest

Responses Total: 12

Responses - % Correct: 8%

Blog
About Pastest
Contact Us
Help

© Pastest 2016

https://mypastest.pastest.com/Secure/TestMe/Browser/436619#Top[‫ ص‬08:03:11 08/12/1437]


MyPastest

Prefer to use the old MyPastest? Access it here »

Back to Filters

Question 13 of 26

The following statements refer to sudden infant death syndrome (SIDS)

Select the MOST appropriate

A It is associated with the use of pacifiers or dummies.

B It is more common in girls

C Prematurity is a risk factor

D The majority of SIDS occurs in babies over the age of 6 months

E The risk is increased with increasing maternal age

Explanation
Sudden infant death syndrome (SIDS) is the unexpected and unexplained death of an infant (on postmortem
findings and on examination of the scene of death). The peak incidence is between 2 and 4 months of age, with
95% of deaths occurring before 6 months of age. The incidence has been declining since 1989, when there were
1337 deaths, to 281 deaths in 2008. The incidence started declining
before the government ‘Back to Sleep’
campaign in 1991 which promoted sleep in the supine position, probably due to prior publicity of the need to put
babies to sleep supine. The major risk factors for sudden infant death are:

parental smoking – antenatal and postnatal


prone sleeping position
male sex
maternal age (younger)
low birthweight
prematurity

https://mypastest.pastest.com/Secure/TestMe/Browser/436619#Top[‫ ص‬08:03:33 08/12/1437]


MyPastest

febrile illness
thermal stress (high temperature, over-wrapping).

The
use of apnoea monitors has not reduced the risk of SIDS. There is no proved association of SIDS with the
type of mattress used although it is
advised that they should be clean and dry and in good condition. Recent
evidence has suggested that settling a baby to sleep with a dummy reduces the risk of SIDS.
43927

Tag Question

Feedback

Difficulty: Average

Peer Responses

Session Progress

Responses Correct: 1

Responses Incorrect: 12

Responses Total: 13

Responses - % Correct: 8%

Previous Question Next Question


Blog
About Pastest
Contact Us
Help

© Pastest 2016

End Session

https://mypastest.pastest.com/Secure/TestMe/Browser/436619#Top[‫ ص‬08:03:33 08/12/1437]


MyPastest

Prefer to use the old MyPastest? Access it here »

Back to Filters

Question 14 of 26

The following statements refer to speech delay in childhood.

Select the MOST appropriate statement

A Speech delay is associated with tongue-tie

B Speech delay is common in autism

C Speech delay is more common in first-born children

D Speech delay is more common in girls

E Speech delay is uncommon in pre-school children

Explanation
Recognised causes of speech delay include:

hearing defects
learning
disability due to any cause, eg congenital hypothyroidism, tuberous sclerosis, Down syndrome,
fragile X syndrome, intrauterine infections and fetal alcohol syndrome
cerebral palsy
developmental expressive aphasia
emotional deprivation
autism.

The
family history is usually of relevance. There is often a history of speech delay in the parents; girls tend to
speak earlier than boys, first-born children tend to speak earlier than subsequent children and twins speak later
than singletons. Tongue-tie, cleft palate and malocclusion may affect speech quality but do not cause speech

https://mypastest.pastest.com/Secure/TestMe/Browser/436619#Top[‫ ص‬08:04:22 08/12/1437]


MyPastest

delay.

Milestones in the development of speech:

3–6 months: loud tuneful vocalisations.


6–12
months: babbles in long repetitive syllables. Syllables ‘da’, ‘ma’ may be used from 8 months initially
inappropriately but use in correct context to parents by 12–14 months.
1–1.5 years: starts using words in appropriate context. Understands many more words.
1.5–2.5
years: many intelligible words mixed with jargon. Starts asking ‘why’, ‘where’ questions by 2
years. Joins two or more words in phrases by 2 years.
2.5–4
years: period of rapid speech development with the acquisition of many new words. Constantly
asking questions. Stops talking to him- or herself
during play in favour of directing speech towards others.
4+
years: can narrate stories; correct grammatical usage by 4.5 years. Delay in any of these areas is a
warning that there is a problem with either hearing or in speech acquisition.

43929

Tag Question

Feedback

Difficulty: Average

Peer Responses

Previous Question Next Question


Session Progress

Responses Correct: 1

Responses Incorrect: 13

Responses Total: 14
End Session
Responses - % Correct: 7%

https://mypastest.pastest.com/Secure/TestMe/Browser/436619#Top[‫ ص‬08:04:22 08/12/1437]


MyPastest

Blog
About Pastest
Contact Us
Help

© Pastest 2016

https://mypastest.pastest.com/Secure/TestMe/Browser/436619#Top[‫ ص‬08:04:22 08/12/1437]


MyPastest

Prefer to use the old MyPastest? Access it here »

Back to Filters

Question 15 of 26

A 9-month-old child is referred to a paediatric outpatient clinic by a health visitor because her mother has concern
about the child’s development.

On examination the child can be pulled


to sit with no head lag and will sit unaided; he is not yet pulling himself to
stand, although will stand if held. The child will grasp objects in both hands and bang them together, although
does not yet appear to have a pincer grip. His mother reports that he does babble and
say Dada but has concern
that does not yet use this specifically for his father. He appears a sociable child and plays peek-a-boo with you,
although his mother reports that he is not yet waving or drinking from a
cup.

Given this history, what is the most appropriate conclusion from the following statements, that you can make
about the development of this child?

A The child has delayed fine motor development

B The child has delayed gross motor development

C The child has delayed speech and language

D The child has global developmental delay

E The child has normal development

Explanation
This child has normal development. First-time parents in particular may become anxious when they see the
children of ante-natal group contemporaries achieve skills ahead of their own child. They may need reassurance
that although one child may learn to walk at 10 months, it is only of concern if a child reaches 15 months without
learning to walk.

A 9 month old child would be expected to sit steadily, pivot to reach objects and be able to stand holding onto
someone or something. They are likely to look for a toy which has fallen from view and be able to poke objects
with an index finger. They will have learnt that shouting gains attention, will understand the word ‘no’ and engage

https://mypastest.pastest.com/Secure/TestMe/Browser/436619#Top[‫ ص‬08:04:44 08/12/1437]


MyPastest

in two syllable babble. Socially they will enjoy picking up food to eat with their fingers and resist if objects are
removed from their grasp.

Pincer grip, pulling to stand, using two words with meaning and waving bye bye would indicate a development
age of 12 months, whilst drinking from a cup is a skill seen by 15 months.
43932

Tag Question

Feedback

Difficulty: Average

Peer Responses

Session Progress

Responses Correct: 1

Responses Incorrect: 14

Responses Total: 15

Responses - % Correct: 7%

Blog
About Pastest
Contact Us
Help
Previous Question Next Question
© Pastest 2016

End Session

https://mypastest.pastest.com/Secure/TestMe/Browser/436619#Top[‫ ص‬08:04:44 08/12/1437]


MyPastest

Prefer to use the old MyPastest? Access it here »

Back to Filters

Question 16 of 26

The following statements refer to what would be expected of a 3-year-old child.

Select the MOST appropriate

A Copy a square

B Go up stairs one foot per step

C Recognise numbers up to 20

D Ride a bicycle

E Write his or her own name

Explanation
A three year old child will be able to go up stairs one foot per step.

Copying a square and recognising numbers equates to a four year old whilst riding a bike and writing their name
are skills looked for in a five year old.
43934

Previous Question Next Question


Tag Question

Feedback

End Session
Difficulty: Average

https://mypastest.pastest.com/Secure/TestMe/Browser/436619#Top[‫ ص‬08:05:06 08/12/1437]


MyPastest

Peer Responses

Session Progress

Responses Correct: 1

Responses Incorrect: 15

Responses Total: 16

Responses - % Correct: 6%

Blog
About Pastest
Contact Us
Help

© Pastest 2016

https://mypastest.pastest.com/Secure/TestMe/Browser/436619#Top[‫ ص‬08:05:06 08/12/1437]


MyPastest

Prefer to use the old MyPastest? Access it here »

Back to Filters

Question 17 of 26

The following statements refer to children who toe walk.

Select the MOST appropriate

A An underlying neurological cause is most likely in these children

B Botulium toxin is not useful in these children

C It is uncommon in children under the age of 2 years

D It may be caused by congenital shortening of the Achilles tendon

E Treatment is usually required

Explanation
Toe walking is common between the ages of
1 and 2 years. In most cases it is a habit and these children can stand
on their heels without difficulty and ankle movements are normal.

Other causes include:

prematurity
spastic cerebral palsy
congenital shortening of the Achilles tendon
Duchenne muscular dystrophy
peroneal muscular atrophy
infantile autism
spinal tumour
unilateral hip dislocation

https://mypastest.pastest.com/Secure/TestMe/Browser/436619#Top[‫ ص‬08:05:28 08/12/1437]


MyPastest

Often no specific treatment is required. Supportive footwear can be helpful and in some cases botulinum toxin can
be useful.
43936

Tag Question

Feedback

Difficulty: Average

Peer Responses

Session Progress

Responses Correct: 1

Responses Incorrect: 16

Responses Total: 17

Responses - % Correct: 6%

Blog
About Pastest
Contact Us
Help

© Pastest 2016
Previous Question Next Question

End Session

https://mypastest.pastest.com/Secure/TestMe/Browser/436619#Top[‫ ص‬08:05:28 08/12/1437]


MyPastest

Prefer to use the old MyPastest? Access it here »

Back to Filters

Question 18 of 26

You receive a referral letter for your general paediatric clinic from a GP to see a 12-year-old girl who attends the
local fee-paying grammar school.

She complains of excessive tiredness for the last 4 months and her school attendance has fallen to less than 20% of
expected. She experiences frequent waking overnight and daytime drowsiness. There is no history of weight loss
or any systemic symptoms of note. Examination is unremarkable. The GP has already requested the following
blood tests, all of which are normal: full blood count (FBC) and film, erythrocyte sedimentation rate (ESR), C-
reactive protein (CRP), urea and electrolytes (U&Es), liver function tests (LFTs) and
TFTs, creatine kinase (CK),
glucose and Epstein–Barr virus serology. Urinalysis is also normal. The GP thinks that the diagnosis is
one of
chronic fatigue syndrome.

Which course of action would be MOST appropriate?

A After
review in clinic, prescribe low-dose antidepressants to try to normalise sleep pattern in an
attempt to improve school attendance

B After
thorough history and examination in clinic, reassure the family and patient that there is no
evidence of serious underlying pathology. Explain variable timescale and prognosis. Develop a
management plan with
patient and family agreement that addresses issues of concern, and arrange
regular reviews of progress

C Assess
in clinic and arrange further investigations to rule out underlying pathology. At review,
reassure patient and family that no organic cause has been found and problems will resolve with time

D Pass
the referral to the Child and Adolescent Mental Health Services (CAMHS)
so that they can
institute family therapy and explore the possibility of an underlying depressive illness

E See
in clinic before requesting ongoing follow-up by the physiotherapists to commence a graded
exercise programme, to try to restore a more normal
level of day-to-day activity for the child

43937

https://mypastest.pastest.com/Secure/TestMe/Browser/436619#Top[‫ ص‬08:05:52 08/12/1437]


MyPastest

Submit

Skip Question

Calculator

Blog
About Pastest
Contact Us
Help

© Pastest 2016

Previous Question

https://mypastest.pastest.com/Secure/TestMe/Browser/436619#Top[‫ ص‬08:05:52 08/12/1437]


MyPastest

Prefer to use the old MyPastest? Access it here »

Back to Filters

Question 18 of 26

You receive a referral letter for your general paediatric clinic from a GP to see a 12-year-old girl who attends the
local fee-paying grammar school.

She complains of excessive tiredness for the last 4 months and her school attendance has fallen to less than 20% of
expected. She experiences frequent waking overnight and daytime drowsiness. There is no history of weight loss
or any systemic symptoms of note. Examination is unremarkable. The GP has already requested the following
blood tests, all of which are normal: full blood count (FBC) and film, erythrocyte sedimentation rate (ESR), C-
reactive protein (CRP), urea and electrolytes (U&Es), liver function tests (LFTs) and
TFTs, creatine kinase (CK),
glucose and Epstein–Barr virus serology. Urinalysis is also normal. The GP thinks that the diagnosis is
one of
chronic fatigue syndrome.

Which course of action would be MOST appropriate?

A After
review in clinic, prescribe low-dose antidepressants to try to normalise sleep pattern in an
attempt to improve school attendance

B After
thorough history and examination in clinic, reassure the family and patient that there is no
evidence of serious underlying pathology. Explain variable timescale and prognosis. Develop a
management plan with
patient and family agreement that addresses issues of concern, and arrange
regular reviews of progress

C Assess
in clinic and arrange further investigations to rule out underlying pathology. At review,
reassure patient and family that no organic cause has been found and problems will resolve with time

D Pass
the referral to the Child and Adolescent Mental Health Services (CAMHS)
so that they can
institute family therapy and explore the possibility of an underlying depressive illness

E See
in clinic before requesting ongoing follow-up by the physiotherapists to commence a graded
exercise programme, to try to restore a more normal
level of day-to-day activity for the child

Explanation

https://mypastest.pastest.com/Secure/TestMe/Browser/436619#Top[‫ ص‬08:06:11 08/12/1437]


MyPastest

Prevalence is highest in adolescents, with females outnumbering males by approximately 3:1. Children are more
likely to make a full recovery than adults although a small percentage may remain incapacitated for years. The
onset may be gradual or sudden, with or without a preceding acute illness. Debilitating fatigue exacerbated by
activity is the most commonly reported symptom Other symptoms are malaise, headaches, sore throat, sleep
disturbance, myalgia, and abdominal and joint pains. Depression, anxiety and other psychological conditions may
coexist.

Diagnosis is based on the impact of the condition on a patient rather than the duration of symptoms as such. A
thorough history is required to include family history and emotional dimensions of illness. Physical examination
should
include height, weight, head circumference, neurological assessment, lymph nodes and sinuses, lying and
standing blood pressure, and heart rate. Routine investigations to be performed in all cases are as stated in the
question, with others dependent on any relevant findings on history and examination but it is important not to do
repeated tests. Diagnosis should be made as soon as possible and communicated to the patient and family. Other
appropriate professionals should be included in management and may include CAMHS, physiotherapy, dieticians,
and education and social services. Inpatient admission and medication may need to be considered in a small
minority. Prolonged bed rest should be avoided. Regular review is essential to monitor progress and revise
management plan as needed.

The Royal College of Paediatrics and Child Health have published extensive evidence-based guidelines in 2004
for the management of children with chronic fatigue syndrome. These are available on the College website:

http://www.rcpch.ac.uk/system/files/protected/page/RCPCH%20CFS.pdf 43937

Tag Question

Feedback

Difficulty: Average

Peer Responses

Previous Question Next Question

Session Progress

Responses Correct: End Session 1

https://mypastest.pastest.com/Secure/TestMe/Browser/436619#Top[‫ ص‬08:06:11 08/12/1437]


MyPastest

Responses Incorrect: 17

Responses Total: 18

Responses - % Correct: 6%

Blog
About Pastest
Contact Us
Help

© Pastest 2016

https://mypastest.pastest.com/Secure/TestMe/Browser/436619#Top[‫ ص‬08:06:11 08/12/1437]


MyPastest

Prefer to use the old MyPastest? Access it here »

Back to Filters

Question 19 of 26

Choose the MOST appropriate statement regarding separation anxiety from the list below

A It is developmentally normal until the age of approximately 8 years

B It is more common in girls than boys

C Presence of separation anxiety disorder in teenagers is most likely to present as school refusal and
somatic symptoms

D Separation anxiety disorder has a poor prognosis even with early intervention and treatment

E To
be diagnosed as a disorder (according to Diagnostic and Statistical Manual of Mental Disorders
[DSM]-V criteria), symptoms must have been present for a minimum of 6 months in <18s

Explanation
DSM-V criteria state that Separation Anxiety Disorder (SAD) consists of excessive anxiety beyond that expected
for a person's age and developmental level, related to separation or impending separation from the primary
caregiver or 'home'.
Previous criteria described this disorder as one usually first diagnosed in infancy, childhood or
adolescence, however it is now recognised that it may first manifest in adulthood.

In
children aged <18 years the behaviour giving rise to the diagnosis should be observed for a minimum of 4
weeks however adult onset separation anxiety disorder requires behaviour lasting for 6 months or more to avoid
over diagnosis of transient fears.

Separation
anxiety is normal until the age of approximately 3–4 years. It is
more common in children who have
suffered bereavement or who have over-protective parents or who come from a close family. Incidence is
approximately equal among boys and girls. The mean age of onset of separation anxiety disorder (SAD) is 7.5
years. Approximately three of four children with SAD will go on to develop school refusal, which has a
mean age
of onset of 10 years.

SAD manifests differently at different ages. In those aged <8 years, it tends to present with unrealistic worry with

https://mypastest.pastest.com/Secure/TestMe/Browser/436619#Top[‫ ص‬08:06:33 08/12/1437]


MyPastest

regard to harm to parents and school


refusal. In those aged 9–12 years, the most common complaint is of
excessive distress at times of separation such as overnight school trips. Those aged 12–16 years present with
school refusal and somatic problems.

SAD symptoms are frequently reinforced by family members. It is more commonly found in children where there
is a family history of anxiety disorder or if there is any history of early or traumatic separation from the attachment
figure (such as divorce). Cognitive–behaviour therapy and family therapy are helpful in returning the child to
normal patterns of behaviour. Complications include depression and substance abuse, but in general the
prognosis
is good if detected and treated early.
43938

Tag Question

Feedback

Difficulty: Average

Peer Responses

Session Progress

Responses Correct: 1

Responses Incorrect: 18

Responses Total: 19

Responses - % Correct: 5%

Previous Question Next Question

Blog
About Pastest
Contact Us
Help

© Pastest 2016
End Session

https://mypastest.pastest.com/Secure/TestMe/Browser/436619#Top[‫ ص‬08:06:33 08/12/1437]


MyPastest

https://mypastest.pastest.com/Secure/TestMe/Browser/436619#Top[‫ ص‬08:06:33 08/12/1437]


MyPastest

Prefer to use the old MyPastest? Access it here »

Back to Filters

Question 20 of 26

The following statements refer to sleep disorders in childhood.

Select the MOST appropriate statement

A Insomnia is the most frequent sleep disorder seen in postpubertal children with depression

B Melatonin is a drug licensed for treating sleep disturbance in children

C Night
terrors are short-lived episodes occurring during the first few hours of sleep, which terminate
spontaneously and of which the child has poor recollection the following day

D Sleepwalking is rare in prepubertal children

E Sleep laboratory studies are helpful in defining the nature of most sleep disorders in childhood

Explanation
Up to one in four children has some type of sleep problem, the most common being nightmares, sleepwalking and
insomnia. Learning difficulties, ADHD, family dysfunction, depression and anxiety are commonly seen in
association. Nightmares affect up to half of 3–6 year olds, being common after stressful or frightening
events.
They usually occur during the second half of the night and are well remembered the next day. Night terrors
typically occur during the first 3 hours of sleep and are associated with autonomic arousal (tachypnoea,
tachycardia). They end spontaneously and the child quickly returns to sleep with poor recall the next day.
Sleepwalking is seen most commonly in the 3- to 10-year age group, and can affect up to 30% of children. There
is usually no recollection the next day.

Chronic
insomnia is associated with the presence of another mental disorder in 35–50% of cases. Prepubertal
children with depression are most likely to experience insomnia whereas their postpubertal counterparts are more
likely to complain of hypersomnia. Substance-induced sleep disorder (drugs or alcohol) should also be considered.
Obstructive sleep
apnoea is the most common reason for referral for sleep studies. Other reasons include suspected
narcolepsy, sleep-related seizure-like activity and snoring associated with daytime somnolence. Most childhood

https://mypastest.pastest.com/Secure/TestMe/Browser/436619#Top[‫ ص‬08:06:53 08/12/1437]


MyPastest

sleep problems can be managed with behavioural techniques including emphasising the need for good sleep
hygiene (eg simple, consistent bedtime routine, dark, quiet room and removal of distractions such as a television)
and techniques such as controlled crying. Medications are rarely needed. Melatonin is used but is not licensed in
the UK for use in children.
43941

Tag Question

Feedback

Difficulty: Average

Peer Responses

Session Progress

Responses Correct: 1

Responses Incorrect: 19

Responses Total: 20

Responses - % Correct: 5%

Blog
About Pastest
Contact Us
Help

© Pastest 2016 Previous Question Next Question

End Session

https://mypastest.pastest.com/Secure/TestMe/Browser/436619#Top[‫ ص‬08:06:53 08/12/1437]


MyPastest

Prefer to use the old MyPastest? Access it here »

Back to Filters

Question 21 of 26

A school nurse discusses a 6-year-old boy with you, who teachers have noticed appears to be clumsier than his
peers in the classroom and on the playground. He struggles in particular with his handwriting and he is not
achieving at the same level as his classmates.

What would be the MOST appropriate course of action to recommend?

A Ask
for an occupational therapy assessment and treatment programme to be drawn up to follow in the
classroom for improving the child’s fine
motor skills

B Assess
in clinic to obtain a full history and perform examination in order to make an application for a
Statement of Special Educational Needs

C Make
direct contact with teachers to establish level of concern before arranging a multidisciplinary
assessment of child’s abilities with
the consent of the parents

D Referral to paediatric neurologist to rule out a possible underlying neurological disorder

E Request educational psychology opinion to look for the presence of underlying learning difficulties

Explanation
This is also known as dyspraxia and exists in varying degrees of severity with various associated comorbidities
such as learning difficulties, attention deficit and other
disabilities. A problem arises due to a failure to sequence
information
required to perform ‘complex’ tasks in a coordinated fashion. The problem area may be limited to
certain fine or gross motor skills, or include problems with speech and language or thought processing. The exact
incidence is unknown but it is likely that 5% of children worldwide have the disorder, with up to 10%
experiencing more minor forms of coordination difficulties. More boys are thought to be affected than girls.
Developmental progress in early childhood may suggest the presence of the disorder but it typically comes to light
during school years and the child’s teacher is often the first person to raise concerns. Crucial to the diagnosis is
that no underlying
neurological condition exists (eg cerebral palsy or muscular dystrophy)
but this can usually be
excluded without the need for specialist input.

https://mypastest.pastest.com/Secure/TestMe/Browser/436619#Top[‫ ص‬08:07:14 08/12/1437]


MyPastest

There are numerous strategies for evaluation of


a child with suspected developmental coordination disorder. Much
information can be gathered by observing a child performing routine daily tasks such as writing, using a knife and
fork and scissors, catching and throwing a ball, and dressing, and this is often done in an
environment familiar to
the child such as the classroom, rather than in
a clinic setting. In standardised tests, a child scoring below the 5th
centile for age is thought to be indicative of a motor problem. Those involved in the assessment process, which
should be multidisciplinary, may include occupational therapists and physiotherapists, speech and language
therapists, educational psychologists, teachers and parents. After identification of problem areas, individual
intervention programmes can be devised either to allow acquisition of skills that are
absent or to provide optimal
coping mechanisms for deficiencies.
43943

Tag Question

Feedback

Difficulty: Average

Peer Responses

Session Progress

Responses Correct: 1

Responses Incorrect: 20

Responses Total: 21

Responses - % Correct: 5%
Previous Question Next Question

Blog
About Pastest
Contact Us
Help

© Pastest 2016 End Session

https://mypastest.pastest.com/Secure/TestMe/Browser/436619#Top[‫ ص‬08:07:14 08/12/1437]


MyPastest

https://mypastest.pastest.com/Secure/TestMe/Browser/436619#Top[‫ ص‬08:07:14 08/12/1437]


MyPastest

Prefer to use the old MyPastest? Access it here »

Back to Filters

Question 22 of 26

How many babies does sudden infant death syndrome affect every year in the UK?

A 50

B 100

C 150

D 300

E 500

Explanation
Sudden infant death syndrome is thought to affect 300 babies a year in the UK. Multiple risk factors include
maternal smoking, overheating and pre-maturity.
45375

Tag Question
Previous Question Next Question
Feedback

Difficulty: Average

Peer Responses
End Session

https://mypastest.pastest.com/Secure/TestMe/Browser/436619#Top[‫ ص‬08:07:35 08/12/1437]


MyPastest

Session Progress

Responses Correct: 1

Responses Incorrect: 21

Responses Total: 22

Responses - % Correct: 5%

Blog
About Pastest
Contact Us
Help

© Pastest 2016

https://mypastest.pastest.com/Secure/TestMe/Browser/436619#Top[‫ ص‬08:07:35 08/12/1437]


MyPastest

Prefer to use the old MyPastest? Access it here »

Back to Filters

Question 23 of 26

A medical student asks about the risks associated with sudden infant death syndrome.

What is the increased risk of SIDS when sleeping prone?

A x2

B x3

C x4

D x5

E x6

Explanation
The risk of death from SIDS is six times greater if a child sleeps prone (on their front) than supine. The risk is
even more significant when a child who normally sleeps on their back is put to sleep on their front. Other risk
factors include maternal smoking, pre-maturity and an excessively warm environment.

http://www.lullabytrust.org.uk/document.doc?id=302 45376

Previous Question Next Question


Tag Question

Feedback

End Session
Difficulty: Average

https://mypastest.pastest.com/Secure/TestMe/Browser/436619#Top[‫ ص‬08:07:56 08/12/1437]


MyPastest

Peer Responses

Session Progress

Responses Correct: 1

Responses Incorrect: 22

Responses Total: 23

Responses - % Correct: 4%

Blog
About Pastest
Contact Us
Help

© Pastest 2016

https://mypastest.pastest.com/Secure/TestMe/Browser/436619#Top[‫ ص‬08:07:56 08/12/1437]


MyPastest

Prefer to use the old MyPastest? Access it here »

Back to Filters

Question 24 of 26

You are counselling a new mother on the dangers of smoking.

What is the approximate increased risk of sudden infant death syndrome when mothers smoke over 20 cigarettes a
day?

A 4-5x

B 5-6x

C 6-7x

D 7-8x

E 8-9x

Explanation
SIDS affects around 300 babies in the UK. Smoking
(including passive smoking) is a major risk factor. The
hazard ratio is
2.5-4x if mothers smoke up to 10 cigarettes a day, whereas the risk is 7-8.5x if mothers smoke over
20 a day.

http://www.lullabytrust.org.uk/document.doc?id=302 45377

Previous Question Next Question

Tag Question

Feedback
End Session

https://mypastest.pastest.com/Secure/TestMe/Browser/436619#Top[‫ ص‬08:08:16 08/12/1437]


MyPastest

Difficulty: Average

Peer Responses

Session Progress

Responses Correct: 1

Responses Incorrect: 23

Responses Total: 24

Responses - % Correct: 4%

Blog
About Pastest
Contact Us
Help

© Pastest 2016

https://mypastest.pastest.com/Secure/TestMe/Browser/436619#Top[‫ ص‬08:08:16 08/12/1437]


MyPastest

Prefer to use the old MyPastest? Access it here »

Back to Filters

Question 25 of 26

Bed sharing has been shown to increase the risk of sudden infant death syndrome.

Bed sharing with a parent who smokes increases the risk of SIDS by what factor?

A x2

B x4

C x6

D x8

E x10

Explanation
The risk of SIDS is excessive when babies sleep on their front, with parental smoking. This increases further when
multiple factors are present. The risk of SIDS in bed sharers is particularly high if parents are excessively tired,
smoke or are under the influence of alcohol.

http://www.lullabytrust.org.uk/document.doc?id=302 45378

Previous Question Next Question

Tag Question

Feedback End Session

https://mypastest.pastest.com/Secure/TestMe/Browser/436619#Top[‫ ص‬08:08:35 08/12/1437]


MyPastest

Difficulty: Average

Peer Responses

Session Progress

Responses Correct: 1

Responses Incorrect: 24

Responses Total: 25

Responses - % Correct: 4%

Blog
About Pastest
Contact Us
Help

© Pastest 2016

https://mypastest.pastest.com/Secure/TestMe/Browser/436619#Top[‫ ص‬08:08:35 08/12/1437]


MyPastest

Prefer to use the old MyPastest? Access it here »

Back to Filters

Question 26 of 26

The use of pillows has been shown to increase the risk of sudden infant death syndrome (SIDS) by what factor?

A x1.5

B x2.5

C x3.5

D x4.5

E x5.5

Explanation
Several factors increase the risks of SIDS including the use of soft bedding and pillows. Other factors include
sleeping prone, parental smoking and bed sharing.

http://www.lullabytrust.org.uk/document.doc?id=302 45379

End Session

Previous Question Tag Question

Feedback

Difficulty: Average

Peer Responses

https://mypastest.pastest.com/Secure/TestMe/Browser/436619#Top[‫ ص‬08:08:55 08/12/1437]


MyPastest

Session Progress

Responses Correct: 1

Responses Incorrect: 25

Responses Total: 26

Responses - % Correct: 4%

Blog
About Pastest
Contact Us
Help

© Pastest 2016

https://mypastest.pastest.com/Secure/TestMe/Browser/436619#Top[‫ ص‬08:08:55 08/12/1437]


MyPastest

Prefer to use the old MyPastest? Access it here »

Back to Filters

Question 1 of 9

Difficulty: Difficult

A 15-year-old girl presents complaining of an odd patch of skin that she noticed on her left thigh and which has
developed over the
past couple of weeks. On examination there is a very firm and slightly indurated pale area of
skin on her upper thigh, which is a few centimetres across, and the lesion has an erythematous border. The pale
area of skin appears to have a rather atrophic, glazed appearance.

What diagnosis fits best with this clinical picture?

A Lichen sclerosus et atrophicus

B Pityriasis vesicular

C Dermatomyositis

D Morphoea

E Pityriasis rosea

Explanation
Morphoea presents as a very firm, white or violaceous patch of skin on any body site, but more commonly on the
thighs, trunk and upper arms. It is a localised scleroderma as there is no systemic involvement. The disease occurs
most commonly in children or
young adults. Developing morphoea lesions have a well-demarcated red or
violet
peripheral edge. As disease activity burns out, the edge assumes
the same colour as the central lesion, and the
lesion itself becomes very firm with an atrophic glazed surface appearance. A linear variant of morphoea may be
seen on the scalp and face of young children, and is known as ‘en coup de sabre’.

Some commentators postulate that the disease may be linked to infection with Borrelia burgdorferi, although
circulating antibodies or spirochaetes are not identified in every patient. In support of this hypothesis, there are
anecdotal case reports that some patients with early morphoea lesions appear to respond to tetracycline antibiotics,
but UVA light can also be
used.

https://mypastest.pastest.com/Secure/TestMe/Browser/436619[‫ ص‬09:21:23 08/12/1437]


MyPastest

Prefer to use the old MyPastest? Access it here »

Back to Filters

Question 2 of 9

Difficulty: Easy

A 16-year-old girl known to suffer from acne vulgaris has been started on isotretinoin.

Which of the following statements best applies to treatment with isotretinoin?

A It is contraindicated in patients with renal artery stenosis

B It can cause hirsutism

C It can cause hyperkalaemia and hence electrolytes should be checked every month

D Pregnancy should be avoided during and 1 month after treatment

E It may cause haemoptysis

Explanation
Isotretinoin is indicated for the treatment of severe inflammatory acne. It can only be prescribed by, or under the
supervision of a consultant dermatologist. However, it causes marked dryness of the skin and mucous membranes,
especially the lips, and can result in minor nosebleeds.

It can also cause infection of the cuticles, itchiness, skin fragility, skin peeling, dry eyes, diffuse alopecia areata,
eye irritation, conjunctivitis, reduced tolerance to contact lenses, permanent thin skin, headaches, permanent hair
thinning, myalgia and/or arthralgia, back pain.

Due to its teratogenicity, pregnancy must be excluded prior to its initiation and during treatment as well as
for 1
month after treatment.

Abnormalities of serum lipids and liver function tests should be excluded before treatment and sought after 4, and
perhaps 8, weeks of treatment.

https://mypastest.pastest.com/Secure/TestMe/Browser/436619[‫ ص‬09:27:15 08/12/1437]


MyPastest

Prefer to use the old MyPastest? Access it here »

Back to Filters

Question 3 of 9

Difficulty: Average

What is the cause for tinea incognito?

A Bacterial superinfection

B Fungal superinfection

C Inappropriate treatment with steroid cream

D Inappropriate treatment with antifungal cream

E Food poisoning

Explanation
Tinea incognito is the name given to tinea when the clinical appearance has been altered by inappropriate
treatment, usually
a topical steroid cream. Tinea is an infection with a dermatophyte fungus. The result is that the
original infection slowly extends. Often the patient and/or their doctor believe they have a dermatitis, hence the
use of a topical steroid cream. The steroid cream dampens down inflammation so the condition feels less irritable.
But when the cream is stopped for a few days the itch gets worse, so the steroid cream is promptly used again. The
more steroid applied, the more extensive the fungal infection becomes. This is particularly worse in areas where
the skin is occluded such as the groin, buttock crease, armpit. 12747

https://mypastest.pastest.com/Secure/TestMe/Browser/436619[‫ ص‬09:34:11 08/12/1437]


MyPastest

Prefer to use the old MyPastest? Access it here »

Back to Filters

Question 4 of 9

Difficulty: Average

A six-month-old baby is referred with a recurrent itchy eruption affecting his trunk and soles. Examination shows
a diffuse eczema on the trunk and pink-red papules on both soles.

Which of the following is the most likely diagnosis?

A Atopic dermatitis

B Pustular psoriasis

C Scabies

D Tinea pedis

E Viral warts

Explanation
It is unlikely that the child has pustular psoriasis
as there are no pustules in the history. Tinea pedis and viral warts
do
not generally give rise to dermatitis on the trunk. Atopic dermatitis presents with a dermatitis affecting the
flexures or the face in babies,
without papules on the soles. Scabies can present with an itchy eczematous-looking
rash on the body, but the clues are at certain sites (palms, soles, genitalia, buttocks). 12748

https://mypastest.pastest.com/Secure/TestMe/Browser/436619[‫ ص‬09:34:31 08/12/1437]


MyPastest

Prefer to use the old MyPastest? Access it here »

Back to Filters

Question 5 of 9

Difficulty: Average

A 3-month-old baby girl is brought to your clinic because her


mother is concerned that her birthmark is still
present. On examination
she has a capillary haemangioma on her left thigh.

Which of the following statements is the most appropriate to tell the mother?

A The birthmark should resolve by 5 years of age

B The birthmark may get larger until 2 years of age then resolve by 5 years

C The birthmark may get larger until 2 years of age then resolve by 10 years

D The birthmark may get larger until 2 years of age and probably will have gone by 5 years but may
never resolve completely

E The birthmark may get larger until 2 years of age, 70% resolve by 5 years and 95% by 10 years but
may never completely resolve.

Explanation
This is indeed the natural course of a capillary haemangioma. It is important that parents are told that there is a
chance that the birthmark may never completely resolve.
22235

https://mypastest.pastest.com/Secure/TestMe/Browser/436619[‫ ص‬09:35:05 08/12/1437]


MyPastest

Prefer to use the old MyPastest? Access it here »

Back to Filters

Question 6 of 9

Difficulty: Average

An 18-month-old girl with eczema is on the following treatment regimen: Oilatum in baths; ‘baby’ shampoo and
soap; E45 cream to affected areas four times daily. Mother uses ‘non-biological’ washing powder. On examination
her skin is erythematous, excoriated and lichenified over the knees, thighs and flexor surfaces of the elbows.

Which of the following would be the next best step?

A Use E45 cream instead of soap; advise using a greasier emollient and try an antihistamine at night

B Use 1% hydrocortisone to affected areas and continue with other measures

C Use emollient wet wraps at night for 1 week then continue current treatment

D Use fusidic acid–hydrocortisone on the affected areas for 1 week, then continue the current regimen

E Advise mother to continue the current treatment and try to exclude dairy products from the diet

Explanation
Treatment of eczema can be problematic. It is worthwhile giving parents advice about simple, everyday measures
that can improve the eczema: using non-biological washing powder; wearing cotton clothes as opposed to artificial
fibres; and not using soaps or shampoos. Use of a bath oil (e.g. Oilatum) is beneficial and E45 cream can be used
as ‘soap’ to good effect. Regular emollient use is important; however, parents can find using very greasy products
hard work because it involves a lot of washing of clothes. The aim is to keep
the skin from feeling dry at any time
of day. Sedating with older types
of antihistamines at night does not help to reduce itching but used occasionally
in large doses provides a sedative effect which may improve
sleep. Once all these measures are in use, but the
eczema is still not controlled, escalation of treatment would be appropriate. There is no evidence for the benefit of
topical antibiotics.

https://mypastest.pastest.com/Secure/TestMe/Browser/436619[‫ ص‬09:35:27 08/12/1437]


MyPastest

Prefer to use the old MyPastest? Access it here »

Back to Filters

Question 7 of 9

Difficulty: Average

A 3-year-old boy presents with a several week history of multiple, discreet, circular, pearly white, papular lesions
on his lower
trunk and upper thigh. On closer inspection they all have a small central depression. They are causing
him no discomfort.

What is the MOST likely diagnosis?

A Eczema

B Keratosis pilari

C Molluscum contagiosum

D Papular urticaria

E Scabies

Explanation
Molluscum contagiosum is a common pox virus that affects mainly infants and young children. It presents as
multiple, discrete, pearly pink or white popular lesions with a characteristic central dimple. It most commonly
affects the trunk, face and anogenital region. The lesions are not in themselves puritic and do not usually cause
irritation or pain, although they can do if they become infected. Eczema and topical steroids can exacerbate them.
If not
complicated, they require no treatment and will usually resolve within a
year. If they are problematic due to
the number or position of lesions,
they can be treated with cryotherapy, curettage or benzoylperoxide as with
warts. An excess of molluscum may be a sign of an underlying immune
disorder and a careful history should be
taken.

Papular
uriticaria, scabies and eczema would all expect to cause some discomfort. Kerotosis pilari is the plugging
of follicles and is most likely to affect the upper arms.
43915

https://mypastest.pastest.com/Secure/TestMe/Browser/436619[‫ ص‬09:35:49 08/12/1437]


MyPastest

Prefer to use the old MyPastest? Access it here »

Back to Filters

Question 8 of 9

Difficulty: Average

A pregnant woman with atopic eczema and asthma, who has a 4-year-old child with moderately severe eczema,
requests advice about reducing the risk of eczema in her unborn child.

Select the MOST appropriate advice from the following:

A If breastfeeding is not possible, soya-based formulae should be used in preference to cows’ milk
preparations

B In
high-risk families, there is evidence to support prolonged exclusive breastfeeding (ie beyond 6
months) as a preventive measure

C Removing
certain known food allergens from the mother’s diet during pregnancy does not reduce the
risk or prevent the onset of atopic eczema

D Taking
measures to minimise exposure to house-dust mite have been shown to reduce the risk of the
development of eczema after birth

E There
is good clinical and epidemiological evidence to support the delayed introduction of potential
food allergens in weaning foods (eg milk protein and eggs)

Explanation
It affects 10–15% of children, and up to 50% of those who also have hay fever or asthma. Boys and girls are
equally affected, and it often starts in infancy. Most will improve by the teenage years but up to one in four will
experience symptoms into adulthood. Typical distribution in infancy affects the scalp and face, with extensor
surfaces also involved in some. Older children and adults have predominantly flexural involvement. Features
include pruritus and xerosis (dry skin). Genetic and environmental factors are involved and include contact
irritants (soaps, detergents, wool), aeroallergens (house-dust mite, pollen, dander), microbial agents
(Staphylococcus aureus), food allergens (cows’ milk, soya, egg) and psychological stress. Treatment consists of
adequate skin hydration, avoidance of allergenic precipitants, topical corticosteroids, systemic antihistamines,
antibiotic treatment of secondary infection and oral immunosuppression in extreme cases. Immunomodulating

https://mypastest.pastest.com/Secure/TestMe/Browser/436619[‫ ص‬09:36:11 08/12/1437]


MyPastest

topical treatment such as tacrolimus and pimecrolimus should be considered to treat moderate or severe atopic
eczema in those aged >2 years or if the maximum strength and potency of topical corticosteroid that is appropriate
for the patient’s age and the area being treated have been adequately tried and haven’t worked, where there is
serious risk of important side effects from further use of topical corticosteroids (particularly permanent damage to
the skin).

Evidence
for the effectiveness of preventive measures is generally applicable in
high-risk families only (ie family
history of atopy in parents and siblings). Removal of certain foods from the pregnant mother’s diet can be harmful
and has not been shown to reduce the risk of eczema in the unborn child. Breastfeeding is protective against the
development
of allergies when compared with formula feeding, but there is no evidence to support its exclusive
use beyond the age of 6 months. If breastfeeding is not possible in families at high risk of atopic disease, evidence
exists to support the use of hydrolysed cows’ milk formulae rather than cows’ or soya milk-based products.
Conclusive evidence to support delaying the introduction of common food allergens when weaning is lacking,
although it is frequently recommended. No benefit has been shown of the preventive effect of avoiding house-dust
mite during pregnancy and after birth, although, if combined with food allergen avoidance, some protection
against atopic eczema in infancy may be gained.
MyPastest

Prefer to use the old MyPastest? Access it here »

Back to Filters

Question 9 of 9

Difficulty: Average

A 6-week-old baby attending for developmental assessment is noted to have a well-circumscribed lesion on the
bridge of his nose measuring 5 mm diameter and with a bluish hue.

The GP refers the child to the ward. The mass feels firm to the touch. Her parents say it was not present at birth
and have noticed that it temporarily increases in size when the baby cries. Appearances are consistent with a
capillary
haemangioma.

What is the MOST appropriate advice to give to these parents?

A Arrange follow-up for the child in the outpatient clinic to monitor the growth of the lesion over the
coming weeks

B Commence a course of oral steroids to limit the growth of the lesion

C Inform parents that no further follow-up is required as the majority spontaneously involute by school
age

D Referral to a plastic surgeon is required for consideration of removal of the lesion

E Reassure parents that it is likely to heal without scarring with time

Explanation
Close follow-up would be required in this
particular case to ensure that any further growth of the naevus does not
impinge on the infant’s visual fields given its location over the nasal bridge.

Capillary haemangiomas, known as a strawberry naevi, are the most common tumour in infancy and seen in
approximately 10% of white infants. The incidence is much lower in black and Asian babies. It is more common
in preterm babies and when mothers have undergone chorionic villous sampling in pregnancy. Girls are three
times more commonly affected than boys. It presents at birth in 30%, and is a focal and solitary lesion in 80% –
most commonly found on the head and neck (60%), followed by the trunk (25%) and extremities (15%). May
appear initially as an area of blanching of skin,
followed by development of fine telangiectasiae and then a red

https://mypastest.pastest.com/Secure/TestMe/Browser/436619[‫ ص‬09:36:35 08/12/1437]


MyPastest

macule or papule. Colour depends on depth of lesion – red or crimson if superficial, purple/blue or flesh coloured
if deep.

Proliferation
of lesions occurs during first year of life, most rapidly during first few weeks and up to age of 6
months. Involution then occurs, typically from the centre. By age 5 years, 50% have completely resolved, 70% by
7 years and 90% by 9 years. Approximately 50% leave some form of permanent
skin change (eg telangiectasiae,
superficial dilated veins or epidermal
atrophy). Treatment is required for certain lesions, ie those that impinge on
vital structures (eg causing airway obstruction or impairment
of visual fields), for ulcerated, bleeding or
secondarily infected lesions and for those causing psychological distress often due to cosmetic disfigurement.
Other indications for intervention include complications such as congestive cardiac failure (seen when multiple or
visceral lesions present such as in diffuse neonatal haemangiomatosis) and in Kasabach–Merritt syndrome
(consumptive coagulopathy and thrombocytopenia). Treatment options include medical therapy with systemic or
intralesional steroids, subcutaneous interferon-α,
or laser or excision surgery. The use of propranolol has recently
been reported to be remarkably successful in the treatment of proliferating haemangiomas.
43919

Tag Question

Feedback

Peer Responses

Session Progress

Responses Correct: 0

Responses Incorrect: 9

Responses Total: 9

Responses - % Correct: 0%

https://mypastest.pastest.com/Secure/TestMe/Browser/436619[‫ ص‬09:36:35 08/12/1437]


MyPastest

Prefer to use the old MyPastest? Access it here »

Back to Filters

Question 1 of 33

A 16-year-old girl presents via her GP who is concerned that she may have an underlying endocrine problem. She
is a good student and has just got a place at college. She weighs only 38 kg (6 stone) and is 1.78 m (5ft 10 inches)
tall. She is emaciated, her skin is dry and she has excessive growth of lanugo hair. She has been amenorrhoeic for
9 months. Her cortisol level is elevated, her free T4 is normal. She has an anaemia and associated reduced white
cell and platelet count.

Which of the following diagnoses is most likely to fit with this clinical picture?

A Addison’s disease

B HIV

C Occult carcinoma

D Hypothyroidism

E Anorexia nervosa

Explanation
Anorexia nervosa has a female to male preponderance of 9:1. It is estimated that around 0.5–1% of American
women between 15 and 30 years of age have anorexia, and the numbers are likely to be similar in the UK. The
aetiology of anorexia is
unknown, but it is likely to be an interaction between environmental and genetic factors,
American studies report rates of sexual abuse as high as 50% in anorexic females.

Typical laboratory tests include decreased FSH, LH, oestrogens and 17-OH steroids. Free T4 and TSH levels are
usually normal. In patients with anorexia the release of stress hormones such as cortisol is triggered. Chronically
elevated levels of cortisol have been observed in patients with anorexia. Anaemia with decreased white and
platelet cell count may also occur. There may be metabolic alkalosis, hypocalcaemia, hypokalaemia and
hypomagnesaemia.

https://mypastest.pastest.com/Secure/TestMe/Browser/436619[‫ ص‬06:31:23 09/12/1437]


MyPastest

Treatment may involve complex psychotherapy for a number of years, and referral to a specialist in the field is
recommended.
Next Question
12156

Previous Question
End Session
Tag Question

Feedback

Difficulty: Easy

Peer Responses

Session Progress

Responses Correct: 0

Responses Incorrect: 1

Responses Total: 1

Responses - % Correct: 0%

Blog
About Pastest
Contact Us
Help

© Pastest 2016

https://mypastest.pastest.com/Secure/TestMe/Browser/436619[‫ ص‬06:31:23 09/12/1437]


MyPastest

Prefer to use the old MyPastest? Access it here »


Next Question

Previous Question Back to Filters

Question 2 of 33

A 17-year-old has been referred on to you by her gynaecologist. She has been complaining of amenorrhoea for 5
months, although no gynaecological abnormality has been found. She feels well and is very active but her weight
has decreased from 61 kg to 43 kg in the last 6 months. Her height is 168 cm. On examination her BP is 90/60
mmHg, heart rate 64 bpm.

What is the most likely diagnosis?

A Conn’s syndrome

B Crohn’s disease

C Anorexia nervosa

D Hyperthyroidism

E Diabetes mellitus

Explanation
Patients with anorexia nervosa actively maintain an unduly low body weight. According to ICD-10 diagnostic
criteria this may
be defined as a weight at least 15% below that expected for the person’s age, height and sex, or as
a body mass index below 17.5. Amenorrhoea (in postmenarchal women who are not taking an oral contraceptive)
is almost always present in these patients. 12157

Tag Question

Feedback

https://mypastest.pastest.com/Secure/TestMe/Browser/436619[‫ ص‬06:31:54 09/12/1437]


MyPastest

Difficulty: Easy

Peer Responses

End Session

Session Progress

Responses Correct: 0

Responses Incorrect: 2

Responses Total: 2

Responses - % Correct: 0%

Blog
About Pastest
Contact Us
Help

© Pastest 2016

https://mypastest.pastest.com/Secure/TestMe/Browser/436619[‫ ص‬06:31:54 09/12/1437]


MyPastest

Prefer to use the old MyPastest? Access it here »


Next Question

Previous Question Back to Filters

End Session
Question 3 of 33

A patient with type-1 diabetes mellitus has a deficiency of insulin.

Which cells secrete insulin?

A Alpha cells of the islets of Langerhans

B Beta cells of the pancreatic islets

C Hepatocytes

D Fat cells

E Melanocytes

Explanation
Insulin is synthesised and stored by the Beta cells of the pancreatic islets. The Beta cells make up 60–70% of the
volume of the islets. Glucagon is synthesised and stored in the Alpha cells (10–20% of the islets). Somatostatin is
synthesised and stored in the Delta cells (5–10% of the islets). 12158

Tag Question

Feedback

Difficulty: Easy

https://mypastest.pastest.com/Secure/TestMe/Browser/436619[‫ ص‬06:32:12 09/12/1437]


MyPastest

Peer Responses

Session Progress

Responses Correct: 0

Responses Incorrect: 3

Responses Total: 3

Responses - % Correct: 0%

Blog
About Pastest
Contact Us
Help

© Pastest 2016

https://mypastest.pastest.com/Secure/TestMe/Browser/436619[‫ ص‬06:32:12 09/12/1437]


MyPastest

Prefer to use the old MyPastest? Access it here »


Next Question

Previous Question Back to Filters

Question 4 of 33

A 16-year-old girl with Addison’s disease is intolerant


of her hydrocortisone treatment, which she takes at a dose
of 20 mg in the morning and 5 mg in the evening.

Which of the following doses of prednisolone would provide an approximate equivalent daily dose to her
hydrocortisone?

A 1 mg

B 7.5 mg

C 10 mg

D 12.5 mg

E 15 mg

Explanation
Equivalent dose of prednisolone vs hydrocortisone therapy is usually about 25% of the hydrocortisone dose.
Adequacy of steroid replacement is normally assessed by clinical well being and restoration of normal (not
excessive) weight. Cortisol levels during the
day are only a useful assessment if the patient is on hydrocortisone.
Patients normally require therapy with mineralocorticoids as well as glucocorticoid therapy. Standard therapy is
with fludrocortisone 50–300 microgram/day, and effectiveness is assessed by serum electrolytes, postural change
in blood pressure, and suppression of plasma renin activity to normal levels. 12159

Tag Question

https://mypastest.pastest.com/Secure/TestMe/Browser/436619[‫ ص‬06:32:31 09/12/1437]


MyPastest

Feedback

Difficulty: Average

Peer Responses

End Session

Session Progress

Responses Correct: 0

Responses Incorrect: 4

Responses Total: 4

Responses - % Correct: 0%

Blog
About Pastest
Contact Us
Help

© Pastest 2016

https://mypastest.pastest.com/Secure/TestMe/Browser/436619[‫ ص‬06:32:31 09/12/1437]


MyPastest

Prefer to use the old MyPastest? Access it here »


Next Question

Previous Question Back to Filters

End Session
Question 5 of 33

A 14-year-old boy presents with poor development of secondary


sex characteristics, colour blindness and a
decreased sense of smell. On examination, his testes are located in the scrotum and are small and soft.

What is the most probable diagnosis?

A Klinefelter’s syndrome

B Kallmann’s syndrome

C Mumps orchitis

D Hyperprolactinaemia

E Cryptorchidism

Explanation
Kallmann’s syndrome is due to isolated gonadotrophin-releasing hormone (GnRH) deficiency. It is often inherited
in an X-linked recessive manner. Klinefelter’s syndrome is also associated with hypogonadism, but the other
clinical features are not seen. Cryptorchidism is ruled out by the presence of testes in the scrotum. The features are
not suggestive of mumps orchitis or hyperprolactinaemia. 12783

Tag Question

Feedback

https://mypastest.pastest.com/Secure/TestMe/Browser/436619[‫ ص‬06:32:50 09/12/1437]


MyPastest

Difficulty: Easy

Peer Responses

Session Progress

Responses Correct: 0

Responses Incorrect: 5

Responses Total: 5

Responses - % Correct: 0%

Blog
About Pastest
Contact Us
Help

© Pastest 2016

https://mypastest.pastest.com/Secure/TestMe/Browser/436619[‫ ص‬06:32:50 09/12/1437]


MyPastest

Prefer to use the old MyPastest? Access it here »


Next Question

Previous Question Back to Filters

End Session
Question 6 of 33

You are called to the psychiatric unit to see an 17-year-old girl with anorexia nervosa.

What would you expect to see in the results of her biochemical investigations?

A Raised LH, FSH

B Elevated circulating cortisol

C Low resting growth hormone levels

D Increased GnRH

E Normal oestrogen levels

Explanation
The typical patient with anorexia nervosa is a female aged < 25 years with weight loss, amenorrhea and
behavioural changes. There is a long-term risk of severe osteoporosis. Endocrine abnormalities include GnRH
deficiency, low LH and FSH, low oestrogen in females, raised circulating cortisol, low to normal T4, reduced T3,
normal TSH and increased resting GH levels. 12784

Tag Question

Feedback

https://mypastest.pastest.com/Secure/TestMe/Browser/436619[‫ ص‬06:33:09 09/12/1437]


MyPastest

Difficulty: Average

Peer Responses

Session Progress

Responses Correct: 0

Responses Incorrect: 6

Responses Total: 6

Responses - % Correct: 0%

Blog
About Pastest
Contact Us
Help

© Pastest 2016

https://mypastest.pastest.com/Secure/TestMe/Browser/436619[‫ ص‬06:33:09 09/12/1437]


MyPastest

Prefer to use the old MyPastest? Access it here »


Next Question

Back to Filters

Question 7 of 33

You review a 17-year-old girl who has been referred with amenorrhoea. She is noted on routine screening to have
a raised prolactin level. She has read about her condition on the Internet and has some questions about prolactin
physiology.

Thinking of hormones in general, which of the following hormones is under continuous inhibition?

A Prolactin

B Growth hormone

C Adrenocorticotrophic hormone

D Thyroid-releasing hormone

E Testosterone

Explanation
The answer is (a) only. Prolactin is under predominantly inhibitory control, by levels of dopamine. Prolactin levels
are known to rise during pregnancy, lactation, severe stress, sleep and coitus. Mildly increased prolactin levels in
the range of 400–600 mU/litre may be physiological, but higher levels require a
diagnosis. Levels above 1000
imply the possibility of a microprolactinoma, and levels above 5000 imply the presence of a macroprolactinoma.
Drug causes of hyperprolactinaemia are, not surprisingly, the dopamine antagonists, which include
metoclopramide, domperidone and the phenothiazines. Features of hyperprolactinaemia in women include
amenorrhoea, galactorrhoea, loss of libido, subfertility and features of androgen deficiency. 12785

Tag Question

https://mypastest.pastest.com/Secure/TestMe/Browser/436619[‫ ص‬06:33:54 09/12/1437]


MyPastest

Feedback

Difficulty: Average
Previous Question
Peer Responses
End Session

Session Progress

Responses Correct: 0

Responses Incorrect: 7

Responses Total: 7

Responses - % Correct: 0%

Blog
About Pastest
Contact Us
Help

© Pastest 2016

https://mypastest.pastest.com/Secure/TestMe/Browser/436619[‫ ص‬06:33:54 09/12/1437]


MyPastest

Prefer to use the old MyPastest? Access it here »


Next Question

Previous Question Back to Filters

End Session
Question 8 of 33

A child with congenital hypothyroidism is currently on 50 µg/day of thyroxine (T4). She is seen in clinic and has
the following thyroid function tests (TFTs): free T4 20.2 (12–24) nmol/L, thyroid-stimulating hormone (TSH) 9.8
(1.2–4.0) mU/L.

Which of the following explanations is the most likely?

A Under-treatment

B Over-treatment

C Anti-T4 antibodies

D Poor compliance

E Wrong diagnosis

Explanation
This is the only explanation that fits. In poor compliance the TSH rises as the T4 is repeatedly missed; however,
when the child has to come for TFTs this prompts the correct dosage of T4 to be restarted and so the plasma T4
rapidly reverts to normal, although TSH is slower to recover. 22225

Tag Question

Feedback

https://mypastest.pastest.com/Secure/TestMe/Browser/436619[‫ ص‬06:34:15 09/12/1437]


MyPastest

Difficulty: Average

Peer Responses

Session Progress

Responses Correct: 0

Responses Incorrect: 8

Responses Total: 8

Responses - % Correct: 0%

Blog
About Pastest
Contact Us
Help

© Pastest 2016

https://mypastest.pastest.com/Secure/TestMe/Browser/436619[‫ ص‬06:34:15 09/12/1437]


MyPastest

Prefer to use the old MyPastest? Access it here »


Next Question

Back to Filters

Question 9 of 33

Which of the following is the most ideal insulin regimen for an 8-year-old child newly diagnosed as having type 1
diabetes?

A Basal–bolus regimen

B Twice daily (30% short/70% intermediate acting): at 2/3 of dose am and 1/3 of dose pm

C Twice daily (20% short/80% intermediate acting): at 1/2 of dose am 1/2 of dose pm

D Once-daily long-acting insulin

E Three times daily, short-acting insulin

Explanation
The basal–bolus system involves a dose of long-acting insulin with three doses of short-acting insulin per day to
coincide with meals, allowing greater flexibility with eating times and amounts (this is easier with older children).
The long-acting insulin ensures a background level that provides more stable glycaemic control. This is ideally the
best regimen for any diabetic child.

Younger
children are unlikely to conform to the ‘set’ three meals per day and will tend to ‘graze’ (i.e. take small
frequent bits of food). Given this eating behaviour a regimen where there is a more constant level of insulin in the
bloodstream is appropriate. Therefore, a twice-daily regimen is more likely to provide better control; indeed in
some children whose eating patterns are so erratic a once-a-day injection of a long-acting insulin provides the best
control.
22238

https://mypastest.pastest.com/Secure/TestMe/Browser/436619[‫ ص‬06:34:34 09/12/1437]


MyPastest

Tag Question

Feedback

Difficulty: Average
Previous Question
Peer Responses
End Session

Session Progress

Responses Correct: 0

Responses Incorrect: 9

Responses Total: 9

Responses - % Correct: 0%

Blog
About Pastest
Contact Us
Help

© Pastest 2016

https://mypastest.pastest.com/Secure/TestMe/Browser/436619[‫ ص‬06:34:34 09/12/1437]


MyPastest

Prefer to use the old MyPastest? Access it here »

Back to Filters

Question 10 of 33

Theme: Endocrine disorders

A Chronic corticosteroid therapy

B Cystic fibrosis

C Constitutional delay in growth and puberty

D Growth hormone deficiency

E Hypochondroplasia

F Hypothyroidism

G Psychosocial deprivation

H Rickets

I Small genetic height potential

J Turner syndrome

For
each of the following cases of poor growth, choose the most likely cause from the above list. Each item may
be used once, more than once or
not at all.

Scenario 1

A 7-year-old girl comes to outpatients with her parents, who are concerned that she is too short. She has
previously been fit and well. Her height and weight are both on the 2nd centile. Parental heights are on the 2nd
(mother) and 10th (father) centiles.

Your answer was incorrect

Select one...

I - Small genetic height potential

This child may not have a big genetic height potential (i.e. her overall potential height that she will get from her
genetic make-up) if both her parents are not tall. It is unlikely that she will be tall.

https://mypastest.pastest.com/Secure/TestMe/Browser/436619[‫ ص‬06:34:57 09/12/1437]


MyPastest

Scenario 2

A 13-year-old boy is the shortest in his class. He has no pubic or axillary hair and his testicular volume is 8 mL.
His bone age is delayed. Next Question
Your answer was incorrect

Select one...
Previous Question
End Session
C - Constitutional delay in growth and puberty

There are no clinical signs of puberty


yet in this boy except that his testicular volume is 8 ml. The pubertal
growth
spurt normally starts once the testicular volume reaches 10 ml. The delayed bone age means that there is still
growth potential in the bones so that if the bone age is delayed 2 years then once puberty starts he will have an
‘extra’ 2 years of growth compared to
his peers. Constitutional delay of growth and puberty can be managed with
reassurance, however peer pressure and bullying at school may lead some families to want intervention. In these
cases a short course of testosterone may ‘kick start’ the growth.

Scenario 3

A 9-year-old girl has previously had a bone marrow transplant (matched sibling donor) for ALL. She presents as
her growth is not keeping up with her peers. She is falling behind academically. She is gaining weight and has
constipation.

Your answer was incorrect

Select one...

F - Hypothyroidism

The features of constipation, weight gain and academic faltering are consistent with hypothyroidism. Acquired
hypothyroidism is due to two main causes: autoimmune (Hashimoto’s) thyroiditis and post-total-body irradiation
(TBI). Worldwide the most common cause is iodine deficiency. Since part of the preparation for bone marrow
transplant requires TBI this is the most likely cause. Treatment is with thyroxine replacement.
22272

Tag Question

Feedback

Difficulty: Average

Session Progress

https://mypastest.pastest.com/Secure/TestMe/Browser/436619[‫ ص‬06:34:57 09/12/1437]


MyPastest

Responses Correct: 0

Responses Incorrect: 12

Responses Total: 12

Responses - % Correct: 0%

Blog
About Pastest
Contact Us
Help

© Pastest 2016

https://mypastest.pastest.com/Secure/TestMe/Browser/436619[‫ ص‬06:34:57 09/12/1437]


MyPastest

Prefer to use the old MyPastest? Access it here »

Back to Filters

Question 11 of 33

The following statements refer to Addison disease.

Select the MOST appropriate.

A Fatigue is a common presenting feature

B It can present with hyperglycaemia if in crises

C It is more common in boys

D Serum renin is characteristically low

E The short Synacthen test is normal

Explanation
Addison disease:

Incidence 1 in 10 000
Most common cause is autoimmune
Familial incidence
Female preponderance, as with many autoimmune conditions
Autoantibodies usually present
Associated
with other autoimmune conditions including hypothyroidism, hypoparathyroidism, diabetes
mellitus, cirrhosis and alopecia

Presenting features:

Hypoglycaemia

https://mypastest.pastest.com/Secure/TestMe/Browser/436619[‫ ص‬06:35:20 09/12/1437]


MyPastest

Lethargy
Muscle weakness
Fatigue
Gastrointestinal symptoms
Hyperpigmentation

Investigations:

Hyponatraemia
Hyperkalaemia
Renin is elevated
Short Synacthen test – no rise in cortisol at 60 or 120 minutes

Note that baseline cortisol may be normal.

Addisonian crisis:

may be the presenting feature


may be precipitated by intercurrent illness or stress
includes dehydration, hypotension and collapse
includes electrolyte disturbance as above

Management:

Long-term treatment with oral hydrocortisone and fludrocortisone. Hydrocortisone needs to be increased
at times of stress
Monitoring of growth and bone age

Management of addisonian crisis:

Physiological (0.9%) saline with added dextrose


Intravenous hydrocortisone

Other causes of adrenal insufficiency:

Adrenoleukodystrophy
Adrenal
destruction, eg birth injury, Waterhouse–Friderichsen syndrome (associated with meningococcal
septicaemia), tuberculosis, tumour metastases
Congenital adrenal hyperplasia
Congenital adrenal hypoplasia
Steroid usage

https://mypastest.pastest.com/Secure/TestMe/Browser/436619[‫ ص‬06:35:20 09/12/1437]


MyPastest

Steroid withdrawal

Next Question 43920

Previous Question
End
Tag Session
Question

Feedback

Difficulty: Average

Peer Responses

Session Progress

Responses Correct: 0

Responses Incorrect: 13

Responses Total: 13

Responses - % Correct: 0%

Blog
About Pastest
Contact Us
Help

© Pastest 2016

https://mypastest.pastest.com/Secure/TestMe/Browser/436619[‫ ص‬06:35:20 09/12/1437]


MyPastest

Prefer to use the old MyPastest? Access it here »

Back to Filters

Question 12 of 33

The following are features of classic homocystinuria.

Select the MOST appropriate

A Abnormal from birth

B Aortic root dilatation

C Autosomal dominant inheritance

D Hyperextendable joints

E Predisposition to vascular thrombosis

Explanation
Homocystinuria

The
incidence of homocystinuria is 1 in 300 000. Inheritance is autosomal recessive and prenatal diagnosis is
possible. It is a disorder of the conversion of methionine into cystine with a consequent accumulation of
homocystine.

Clinical features of homocystinuria include:

normal
at birth • marfanoid features, tall and thin with long fingers, the lower segment of the body longer
than the upper segment and an arm span greater than height
subluxed lens – downward and inward
stiff joints
connective tissue weakness – hernia, scoliosis
propensity to vascular thrombosis

https://mypastest.pastest.com/Secure/TestMe/Browser/436619[‫ ص‬06:35:44 09/12/1437]


MyPastest

progressive learning disability (70%).

Next Question
Diagnosis
is based on plasma and urinary amino acids. Treatment aims to reduce the homocystine levels. Options
include pyridoxine, folic acid, a low protein diet and aspirin as an anti-thrombolytic.

Marfan syndrome
Previous Question
The
incidence of Marfan syndrome is between 1:16 000 and 1:60 000. Inheritance is autosomal dominant, with the
gene locus on chromosome 15. End Session
Clinical features of Marfan syndrome include:

usually abnormal at birth


long fingers, lower segment of the body longer than the upper segment, arm span greater than height
subluxed lens – upwards and outwards, myopia, retinal detachment, glaucoma and cataract
hyperextendable joints
connective tissue weakness – hernia and scoliosis
mitral and aortic valve disease including aortic root dilatation
pneumothorax.

Diagnosis
is clinical. Slit-lamp examination and echocardiography are useful. Plasma amino acids exclude
homocystinuria. Cardiac problems are a significant cause of morbidity.

Ectopia lentis

This is an isolated finding of a dislocated lens usually inherited as an autosomal dominant condition.
43921

Tag Question

Feedback

Difficulty: Average

Peer Responses

https://mypastest.pastest.com/Secure/TestMe/Browser/436619[‫ ص‬06:35:44 09/12/1437]


MyPastest

Session Progress

Responses Correct: 0

Responses Incorrect: 14

Responses Total: 14

Responses - % Correct: 0%

Blog
About Pastest
Contact Us
Help

© Pastest 2016

https://mypastest.pastest.com/Secure/TestMe/Browser/436619[‫ ص‬06:35:44 09/12/1437]


MyPastest

Prefer to use the old MyPastest? Access it here »

Back to Filters

Question 13 of 33

Select the MOST appropriate statement regarding congenital hypothyroidism

A A high thyroid-stimulating hormone (TSH) and normal T4 on treatment suggest poor compliance

B It is usually due to dyshormonogenesis

C It is usually symptomatic in the neonatal period

D Screening is at the 6-week check

E The incidence is 1 in 40000

Explanation

Incidence of 1 in 4000
Asymptomatic until 6–12 weeks of age
Male to female ratio is 1:2

Symptoms and signs include:

poor feeding
constipation
lethargy
jaundice
large tongue
umbilical hernia
hoarse cry

https://mypastest.pastest.com/Secure/TestMe/Browser/436619[‫ ص‬06:36:06 09/12/1437]


MyPastest

Without
treatment myxoedema (soft-tissue accumulation) and failure to thrive will occur and cretinoid facies will
Next Question
develop. Neonatal screening is done
with the Guthrie card test for phenylketonuria at 7–10 days. Usually T4
(thyroxine) will be low and TSH raised although in 10% the T4 will be normal. Neonatal screening will not
usually detect hypothyroidism due to TSH deficiency.

Congenital
hypothyroidism
PreviousisQuestion
usually due to thyroid dysgenesis, sometimes with ectopic thyroid tissue being
present. Less commonly it occurs secondary to an inborn error of hormone synthesis (dyshormonogenesis).
End Session
Treatment is lifelong with thyroxine. The best method of monitoring is by assessment of growth and development.
A normal T4 should be aimed for, but the TSH does not necessarily have to be normal. A high TSH and a normal
T4
may indicate poor compliance. Outcome is near normal (but not normal) development provided that treatment
is started early. Large cohort studies show a 5- to 10-point difference in IQ compared with a control population.
The TSH at presentation is of prognostic importance, with the higher levels being of greater concern.
43924

Tag Question

Feedback

Difficulty: Average

Peer Responses

Session Progress

Responses Correct: 0

Responses Incorrect: 15

Responses Total: 15

Responses - % Correct: 0%

https://mypastest.pastest.com/Secure/TestMe/Browser/436619[‫ ص‬06:36:06 09/12/1437]


MyPastest

Blog
About Pastest
Contact Us
Help

© Pastest 2016

https://mypastest.pastest.com/Secure/TestMe/Browser/436619[‫ ص‬06:36:06 09/12/1437]


MyPastest

Prefer to use the old MyPastest? Access it here »

Back to Filters

Question 14 of 33

Concerning Klinefelter syndrome select the MOST appropriate statement.

A Gynaecomastia is an uncommon finding in adolescents

B Infertility is rare

C The karyotype is 47 XYY

D The testes are large

E There is increased risk of leukaemia

Explanation

Incidence of 1 in 1000 males


Karyotype is 47 XXY

The
aetiology is meiotic non-dysjunction with the extra X-chromosome coming
from the father in 50% and the
mother in the other 50%. Increased maternal age is a risk factor.

There are a number of variants with more than two X-chromosomes and mosaicism is common. Children are
usually asymptomatic until the age of 5 years.

After
that they can present with behavioural problems or psychiatric disturbances. Intelligence is below average.
The children are usually tall and thin.

Puberty is delayed and infertility is common, due to azoospermia. The testes and phallus are small. Gynaecomastia
is common (80%). There is an increased risk of pulmonary disease, varicose veins, breast cancer, leukaemia and
mediastinal germ-cell tumours. The prepubertal hormone profile is normal. By midpuberty the FSH and LH levels
are raised and the testosterone is low, and testosterone replacement is required. Elevated levels of estradiol with a
high estradiol:testosterone ratio account for
the development of gynaecomastia during puberty.

https://mypastest.pastest.com/Secure/TestMe/Browser/436619[‫ ص‬06:36:27 09/12/1437]


MyPastest

43926

Next Question

Previous Question Tag Question

Feedback End Session

Difficulty: Average

Peer Responses

Session Progress

Responses Correct: 0

Responses Incorrect: 16

Responses Total: 16

Responses - % Correct: 0%

Blog
About Pastest
Contact Us
Help

© Pastest 2016

https://mypastest.pastest.com/Secure/TestMe/Browser/436619[‫ ص‬06:36:27 09/12/1437]


MyPastest

Prefer to use the old MyPastest? Access it here »

Back to Filters

Question 15 of 33

For Turner syndrome select the MOST appropriate statement.

A Fetal loss in the first trimester is common

B Hypogonadotrophic hypogonadism is a feature

C Infants are usually large for dates

D Spontaneous puberty is never seen

E The incidence increases with advancing maternal age

Explanation
The incidence is 1 in 1500–2500 live born girls. The karyotype is 45 XO, 45 XO/46 XX. Mosaicism is present
within most cell lines, and the paternal sex chromosome is lost.
Loss of the maternal sex chromosome is a lethal
deletion. There is no effect of increasing maternal age on incidence. Spontaneous fetal loss is common, usually in
the first trimester.

Clinical features of Turner syndrome include the following:

Infants are usually small for dates


Lymphoedema and feeding difficulties in the neonatal period
Neck webbing
Cubitus valgus
Cardiac abnormalities: coarctation of the aorta, bicuspid aortic valve, aortic stenosis
Renal abnormalities: pelvic kidney, single kidney, pelvic–ureteric junction obstruction
Growth failure
Failure of puberty (hypergonadotrophic hypogonadism)

https://mypastest.pastest.com/Secure/TestMe/Browser/436619[‫ ص‬06:36:56 09/12/1437]


MyPastest

Pigmented naevi

Next Question
There are two options for management of growth failure:

1. Steroid
treatment: oxandrolone, an anabolic steroid with minimal androgenic side effects, if used in low
dose, will increase final adult height
Previous
2. Growth
hormone: Question
recombinant growth hormone is given as injections and increases final height (6–8 cm).
Higher doses are needed than in growth hormone deficiency
End Session
Management of pubertal failure

In
20% of cases of Turner syndrome there is some ovarian function and development of some signs of puberty.
Most require oestrogen replacement
at 12–13 years. Once puberty is initiated, cyclical therapy with oestrogen and
progesterone leads to menstrual cycles. Successful pregnancies have been described with ovum induction and in
vitro fertilisation.
43928

Tag Question

Feedback

Difficulty: Average

Peer Responses

Session Progress

Responses Correct: 0

Responses Incorrect: 17

Responses Total: 17

Responses - % Correct: 0%

https://mypastest.pastest.com/Secure/TestMe/Browser/436619[‫ ص‬06:36:56 09/12/1437]


MyPastest

Blog
About Pastest
Contact Us
Help

© Pastest 2016

https://mypastest.pastest.com/Secure/TestMe/Browser/436619[‫ ص‬06:36:56 09/12/1437]


MyPastest

Prefer to use the old MyPastest? Access it here »

Back to Filters

Question 16 of 33

With regard to type 1 diabetes mellitus, select the MOST appropriate statement.

A It is associated with an increased risk of diabetes in siblings of the affected case

B It is associated with islet cell antibodies in 30% at diagnosis

C It is more common in children who possess the HLA-B3 antigen

D It has a peak incidence at 9–10 years of age

E It has a peak incidence during the summer months in the UK

Explanation
Type 1 diabetes occurs as the result of the destruction of pancreatic islet cells. Clinical symptoms occur when
there is approximately 20% of islet cell activity remaining. Pathogenesis is thought to be autoimmune. In 80–90%
of newly diagnosed patients with diabetes there are islet cell antibodies. There is an association with HLA
antigens: HLA-B8, HLA-BW15, HLA-DR3 and HLA-DR4 (each of these give a two- to threefold increased risk
of developing type 1 diabetes). Homozygosity to the absence of aspartic acid in the HLA-DQ β chain confers a
100-fold increased risk.

Siblings
of an affected individual are at a 1–7% risk of developing type 1
diabetes (annual UK incidence 7.7/100
000). There is a seasonal variation in incidence with peaks during the autumn and winter months. There is also an
increased incidence after Coxsackie virus, mumps and rubella epidemics, suggesting that an initial viral infection
triggers an autoimmune response against islet cells. There is a peak incidence at
age 5–7 years (when children start
school and exposure to viral infection increases) and at puberty (10–14 years).

Treatment
with immunosuppressive agents has been found to lengthen the honeymoon period. The risks of
treatment are greater than the benefits of starting
insulin later.
43930

https://mypastest.pastest.com/Secure/TestMe/Browser/436619[‫ ص‬06:37:17 09/12/1437]


MyPastest

Next Question
Tag Question

Previous Question
Feedback

End Session
Difficulty: Average

Peer Responses

Session Progress

Responses Correct: 0

Responses Incorrect: 18

Responses Total: 18

Responses - % Correct: 0%

Blog
About Pastest
Contact Us
Help

© Pastest 2016

https://mypastest.pastest.com/Secure/TestMe/Browser/436619[‫ ص‬06:37:17 09/12/1437]


MyPastest

Prefer to use the old MyPastest? Access it here »

Back to Filters

Question 17 of 33

The following refer to features of Graves disease.

Select the MOST appropriate.

A Association with HLA-DR3

B Constipation

C Inappropriate weight gain

D Male predominance

E Peak incidence in early childhood

Explanation
Graves disease is thyrotoxicosis associated with eye manifestations. Five per cent of cases present in childhood,
the peak incidence being during adolescence. There is a female predominance (5:1), and a family history is
common. There is an association with HLA-B8 and HLA-DR3.

Clinical features of Graves disease:

Emotional disturbance
Irritability
Poor attention span
Tremor
Tachycardia
Increased appetite
Weight loss
Diarrhoea

https://mypastest.pastest.com/Secure/TestMe/Browser/436619[‫ ص‬06:37:41 09/12/1437]


MyPastest

Goitre
Exophthalmus and ophthalmoplegia
Next Question
Lid lag
Cardiac involvement (rare in childhood)

Previous Question
T3 (triiodothyronine) and T4
are elevated, TSH is low. TSH receptor-stimulating antibodies are usually present at
Endcases
diagnosis and disappear as the condition remits. Approximately half the childhood Session
will remit spontaneously
within 2–4 years. Many will progress to become clinically hypothyroid. Medical treatment is with carbimazole
and propranolol (the latter to control acute symptoms). Block replacement means that both carbimazole and T4 are
used. Definitive treatment (favoured by many) is either by subtotal thyroidectomy or the use of radioactive iodine.
Severe eye disease may require treatment with prednisolone.

Neonatal
thyrotoxicosis is a transient condition resulting from the placental transfer of thyroid-stimulating
antibodies from a thyrotoxic mother.

Other diseases associated with HLA-B8/DR3:

Addison disease
Type 1 diabetes
Coeliac disease
Chronic active hepatitis
Systemic lupus erythematosus
Dermatomyositis
Autoimmune thyroiditis
Primary sclerosing cholangitis

43931

Tag Question

Feedback

Difficulty: Average

Peer Responses

https://mypastest.pastest.com/Secure/TestMe/Browser/436619[‫ ص‬06:37:41 09/12/1437]


MyPastest

Session Progress

Responses Correct: 0

Responses Incorrect: 19

Responses Total: 19

Responses - % Correct: 0%

Blog
About Pastest
Contact Us
Help

© Pastest 2016

https://mypastest.pastest.com/Secure/TestMe/Browser/436619[‫ ص‬06:37:41 09/12/1437]


MyPastest

Prefer to use the old MyPastest? Access it here »

Back to Filters

Question 18 of 33

A 13-year-old boy attends outpatient clinic because he is concerned that many of his friends are now taller than
him and he does not seem to have grown as they have.

He is worried. On examination
he has some sparse dark coarse hair over the junction of the pubes and reports that
he has noticed some enlargement of his penis and growth of his testis. He has a testicular volume of 8 ml.

Select the MOST likely diagnosis.

A Delayed puberty

B Normal puberty

C Precious puberty

D Premature adrenache

E Premature thelarche

Explanation
In the 3 years before puberty low levels of pulsatile LH become detectable during sleep. LH and FSH are
produced in the anterior pituitary and released due to pulsatile gonadotrophin-releasing hormone (GnRH) secreted
by the hypothalamus. There is an increase in the amplitude and frequency of LH secretion as puberty approaches,
which causes enlargement of the gonads. In boys, the
testicles produce testosterone and in girls the ovaries
produce estradiol and ovarian androgens, which, with the adrenal androgens, produce secondary sexual
characteristics.

Average
age at onset of puberty is 11 years in girls/11.5 years in boys. The first sign is breast bud development,
followed by the appearance of pubic hair 6–12 months later. Menarche usually occurs 2–2.5 years after breast bud
development. Peak height velocity in girls occurs
at breast stage 2–3 and virtually always precedes menarche.
Onset
of puberty in boys is at 11.5 years. The first sign is testicular enlargement (>3 ml) and thinning of the
scrotum. This is followed by pigmentation of the scrotum and growth of the penis, and pubic hair follows. Peak

https://mypastest.pastest.com/Secure/TestMe/Browser/436619[‫ ص‬06:38:22 09/12/1437]


MyPastest

height velocity (growth spurt) is 2 years later in boys than in girls and occurs at testicular stage 4–5 (ie testicular
volume 10–12 ml), which is around 13–14 years of age. Breast
enlargement occurs in 40–60% of boys (significant
Next Question
enough to cause
social embarrassment in 10%) and is a result of estradiol produced by the metabolism of
testosterone. It usually resolves within 3 years. During puberty, elongation of the eye often occurs, causing short-
sightedness.
43933
Previous Question
End Session

Tag Question

Feedback

Difficulty: Average

Peer Responses

Session Progress

Responses Correct: 0

Responses Incorrect: 20

Responses Total: 20

Responses - % Correct: 0%

Blog
About Pastest
Contact Us
Help

© Pastest 2016

https://mypastest.pastest.com/Secure/TestMe/Browser/436619[‫ ص‬06:38:22 09/12/1437]


MyPastest

Prefer to use the old MyPastest? Access it here »

Back to Filters

Question 19 of 33

Concerning 21-hydroxylase deficiency, select the MOST appropriate statement from the following.

A A reduced serum 17OH-progesterone is characteristic

B Hypertension is common

C It can present as premature isosexual development in boys

D Its inheritance is autosomal dominant

E Plasma potassium is low in salt losers

Explanation
The incidence of congenital adrenal hyperplasia is 1 in 5000. Inheritance is autosomal recessive. The gene defect
is known and is part of the human leukocyte antigen (HLA) complex
on chromosome 6. Antenatal diagnosis is
possible by either chorionic villous sampling or amniocentesis. Ninety-five per cent of defects are due to 21-
hydroxylase deficiency, 75% of which are salt losers. 11β-Hydroxylase deficiency is the second most common
type and associated with hypertension after the first few years.

Presentation can be in any of the following ways:

Salt-losing crises
Premature isosexual development (boys – small testes, large penis and scrotum)
Virilisation in girls
Hypertension (11β-hydroxylase deficiency)

Characteristic
features of a salt-losing crisis include a low plasma sodium and chloride and a raised potassium,
with an elevated plasma renin and low plasma aldosterone.

https://mypastest.pastest.com/Secure/TestMe/Browser/436619[‫ ص‬06:38:45 09/12/1437]


MyPastest

Diagnosis:

Next Question
Raised plasma 17-hydroxyprogesterone and raised urinary pregnanetriol (21-hydroxylase deficiency)
Raised plasma 11-deoxycortisol and 11-deoxycorticosterone (11β-hydroxylase deficiency)

Treatment
is by Previous Question
steroid replacement therapy. Hydrocortisone is used to replace corticosteroid activity, and
fludrocortisone to replace mineralocorticoid activity. Monitoring of treatment is controversial and
includes:
End Session
measurement of growth
bone age
blood pressure
plasma electrolytes
steroid biochemistry (plasma 17-hydroxyprogesterone profiles in 21-hydroxylase deficiency)

43935

Tag Question

Feedback

Difficulty: Average

Peer Responses

Session Progress

Responses Correct: 0

Responses Incorrect: 21

Responses Total: 21

Responses - % Correct: 0%

https://mypastest.pastest.com/Secure/TestMe/Browser/436619[‫ ص‬06:38:45 09/12/1437]


MyPastest

Blog
About Pastest
Contact Us
Help

© Pastest 2016

https://mypastest.pastest.com/Secure/TestMe/Browser/436619[‫ ص‬06:38:45 09/12/1437]


MyPastest

Prefer to use the old MyPastest? Access it here »

Back to Filters

Question 20 of 33

Regarding fragile X syndrome, choose the MOST appropriate statement.

A It causes more severe learning difficulties in girls than boys

B It is a cause of micro-orchidism in boys

C It is associated with small ears

D It is asymptomatic in carrier girls

E It occurs as a consequence of allelic expansion

Explanation
Fragile X syndrome is a common disorder among people with significant learning difficulties. The fragile site is
on the long arm of chromosome X at Xq27.3. Inheritance is X linked, but
expression is due to the process of
allelic expansion. The fragile X locus normally has 2800 trinucleotide base repeats; a small increase in the repeats
makes it unstable and through successive generations there is an expansion in the repeats (female transmission
increases the repeats) until the increase becomes clinically significant resulting in the fragile X syndrome.

Clinical features include:

learning difficulties (IQ 30–55 in boys, milder in girls)


prominent jaw
long face
large ears
macro-orchidism
hypotonia
joint laxity

https://mypastest.pastest.com/Secure/TestMe/Browser/436619[‫ ص‬06:39:06 09/12/1437]


MyPastest

The
testicular enlargement is more prominent post-puberty. Psychological features include cluttering of speech,
Next Question
hyperactivity, emotional instability and autistic features.

Other disorders associated with allelic expansion include:

Huntington disease (male


Previous transmission increases the repeat)
Question
Myotonic dystrophy (maternal transmission increases the repeat)
End Session
Allelic
expansion accounts for the phenomenon genetic anticipation, in which there is earlier onset and/or
increasing severity of disease as the expanding gene is transmitted from generation to generation.
43939

Tag Question

Feedback

Difficulty: Average

Peer Responses

Session Progress

Responses Correct: 0

Responses Incorrect: 22

Responses Total: 22

Responses - % Correct: 0%

Blog
About Pastest
Contact Us
Help

https://mypastest.pastest.com/Secure/TestMe/Browser/436619[‫ ص‬06:39:06 09/12/1437]


MyPastest

© Pastest 2016

https://mypastest.pastest.com/Secure/TestMe/Browser/436619[‫ ص‬06:39:06 09/12/1437]


MyPastest

Prefer to use the old MyPastest? Access it here »

Back to Filters

Question 21 of 33

The following statements refer to growth hormone deficiency.

Choose the MOST appropriate statement.

A It can lead to boys having large genitalia

B It does not require investigation of other hypothalamic–pituitary function

C It is the cause of short stature in Cushing syndrome

D It is usually evident at birth

E Treatment with growth hormone can improve final height

Explanation
The incidence of growth hormone deficiency is 1 in 4000. The male:female ratio is 2:1 and most cases are
idiopathic.

Causes of growth hormone deficiency:

Genetic: primary defect in growth hormone production


Congenital abnormality: associated with midline defects, eg septo-optic dysplasia, cleft lip and palate
Acquired: perinatal/postnatal infections, central nervous system infection, radiotherapy
Neoplasia: craniopharyngioma, glioma
Trauma: perinatal, basal skull fracture
Autoimmune

Treatment
of growth hormone deficiency is with growth hormone replacement therapy
by injection. Growth
hormone will increase the final adult height in children with proved growth hormone deficiency.

https://mypastest.pastest.com/Secure/TestMe/Browser/436619[‫ ص‬06:39:29 09/12/1437]


MyPastest

Guidelines of the National Institute for Health and Clinical Excellence (NICE) state that somatropin (recombinant
human growth hormone) is recommended as a treatment option for children with growth failure associated with
any of
the following conditions:
Next Question

Growth hormone deficiency


Previous
Turner syndrome Question
Prader–Willi syndrome
End Session
Chronic renal insufficiency
Born small for gestational age with subsequent growth failure at 4 years of age or later
Short stature homeobox-containing gene (SHOX) deficiency: SHOX
is located on the distal ends of X and
Y chromosomes and plays a role in long bone growth. Normal growth requires two functional copies of
the
gene

https://www.nice.org.uk/guidance/ta188/chapter/1-guidance
43942

Tag Question

Feedback

Difficulty: Average

Peer Responses

Session Progress

Responses Correct: 0

Responses Incorrect: 23

Responses Total: 23

Responses - % Correct: 0%

https://mypastest.pastest.com/Secure/TestMe/Browser/436619[‫ ص‬06:39:29 09/12/1437]


MyPastest

Blog
About Pastest
Contact Us
Help

© Pastest 2016

https://mypastest.pastest.com/Secure/TestMe/Browser/436619[‫ ص‬06:39:29 09/12/1437]


MyPastest

Prefer to use the old MyPastest? Access it here »

Back to Filters

Question 22 of 33

The following statements refer to galactosaemia.

Choose the MOST appropriate statement.

A Clinical manifestations can occur before a baby has been fed

B Hepatic failure is rare

C It can lead to delayed puberty

D It has an incidence of 1 in 1000 live births

E It is treated with a dairy-free diet

Explanation
Galactosaemia is a rare autosomal recessive disorder. The incidence is around 1 in 50 000 live births.

Three separate defects have been described:

1. Galactokinase deficiency which causes cataracts only.


2. Mild
galactose-1-phosphate uridyltransferase deficiency (Duarte variant) in which there are no symptoms,
but erythrocyte galactose-1-phosphate uridyltransferase activity is reduced.
3. Severe galactose-1-phosphate uridyltransferase deficiency: widespread, generalised disorder that produces
learning difficulties, failure to thrive, cataracts, jaundice, hypoglycaemia and hepatomegaly. There is a rapid
progression to irreversible severe learning disability and cirrhosis in undiagnosed patients.

Severe galactose-1-phosphate uridyltransferase is the most common presentation,


usually presenting in the
neonatal period. Reducing substances are present in the urine after the first feed. This is detectable on testing
with
Clinitest tablets. Urine dipstick for glucose is negative. Diagnosis is confirmed by measuring the enzymes.
Treatment is with a lactose-free diet, but the outcome is variable. Some degree of learning difficulties is usual. A
number have severe learning difficulties. In addition to these learning difficulties, galactosaemia can affect the

https://mypastest.pastest.com/Secure/TestMe/Browser/436619[‫ ص‬06:39:50 09/12/1437]


MyPastest

ovaries. Girls can have delayed puberty due to their ovaries not producing enough oestrogen.
43944

Next Question

Previous Question
Tag Question
End Session
Feedback

Difficulty: Average

Peer Responses

Session Progress

Responses Correct: 0

Responses Incorrect: 24

Responses Total: 24

Responses - % Correct: 0%

Blog
About Pastest
Contact Us
Help

© Pastest 2016

https://mypastest.pastest.com/Secure/TestMe/Browser/436619[‫ ص‬06:39:50 09/12/1437]


MyPastest

Prefer to use the old MyPastest? Access it here »

Back to Filters

Question 23 of 33

Select the MOST appropriate statements regarding the use of insulin pumps.

A Diabetic ketoacidosis will take longer to develop in those patients using them because they generally
have improved control

B Less education is required because with a continuous pump the regimen is stable

C They can be useful in infants where small alterations in insulin is difficult with standard insulin pens

D They reduce lifestyle flexibility due to the pump having to be continuously attached

E They reduce the need for blood sugar monitoring

Explanation
Insulin pumps provide continuous subcutaneous insulin infusion. Their use has increased as the size, price and
reliability of the pumps have improved.

These pumps are a complex technical medical device and their use requires a patient/parent to have in-depth
understanding not only of how
the pump functions but also of the diabetes and this requires extensive
education.

The use of an insulin pump along with the intensive intervention and education can lead to improved overall
control but, as the insulin is being continuously infused, any interruption in the delivery of this infusion will mean
a immediate cessation of insulin delivery which can lead to metabolic decompensation
more quickly. Therefore
more frequent blood sugar monitoring is required.

The delivery of insulin in young children can be quite problematic as such small amounts may be needed and
alterations in dose on standard pens or syringes can be difficult, so with motivated parents insulin pumps can be
useful.
43946

https://mypastest.pastest.com/Secure/TestMe/Browser/436619[‫ ص‬06:40:10 09/12/1437]


MyPastest

Next Question
Tag Question

Previous Question
Feedback

End Session
Difficulty: Average

Peer Responses

Session Progress

Responses Correct: 0

Responses Incorrect: 25

Responses Total: 25

Responses - % Correct: 0%

Blog
About Pastest
Contact Us
Help

© Pastest 2016

https://mypastest.pastest.com/Secure/TestMe/Browser/436619[‫ ص‬06:40:10 09/12/1437]


MyPastest

Prefer to use the old MyPastest? Access it here »

Back to Filters

Question 24 of 33

In nutritional rickets, which of the following is MOST likely to occur?

A Craniosynostosis is a clinical feature

B Exclusive breastfeeding may be protective

C Hyperphosphataemia is a cause

D It leads to increased mineralisation of the growing bone

E Vitamin D deficiency is the most common cause

Explanation
Rickets is a disorder of decreased endochondral calcification at the growth plates leading to deformities.

Vitamin
D deficiency is the most common cause of nutritional rickets worldwide and, although rare in the
developed world, it is thought to be increasing in prevalence due to children’s inadequate exposure to sunlight. A
combination of calcium deficiency and vitamin D deficiency leading to rickets can be a particular problem in
those infants exclusively breastfed.

Both vitamin D deficiency and calcium deficiency can lead to a reduced plasma level of calcium leading to
secondary hyperparathyroidism, which results in excessive bone resorption and decreased bone mass.

Clinical features:

Growth delay
Bone pain
Fractures
Skeletal abnormalities: wrist swelling, bowing of long bones, craniotabes

https://mypastest.pastest.com/Secure/TestMe/Browser/436619[‫ ص‬06:40:31 09/12/1437]


MyPastest

43947

Next Question

Previous Question Tag Question

Feedback End Session

Difficulty: Average

Peer Responses

Session Progress

Responses Correct: 0

Responses Incorrect: 26

Responses Total: 26

Responses - % Correct: 0%

Blog
About Pastest
Contact Us
Help

© Pastest 2016

https://mypastest.pastest.com/Secure/TestMe/Browser/436619[‫ ص‬06:40:31 09/12/1437]


MyPastest

Prefer to use the old MyPastest? Access it here »

Back to Filters

Question 25 of 33

Which of the following is MOST likely to cause hypercalcaemia?

A DiGeorge syndrome

B Hyperparathyroidism

C Hypothyroidism

D Phenytoin

E Vitamin D deficiency

Explanation
Causes of hypercalcaemia include the following:

Hyperparathyroidism
High bone turnover states:

hyperthyroidism
malignancy
vitamin A intoxication

Thiazide diuretics
Vitamin D intoxication
Idiopathic hypercalcaemia of infancy
Renal failure

https://mypastest.pastest.com/Secure/TestMe/Browser/436619[‫ ص‬06:40:54 09/12/1437]


MyPastest

Signs and symptoms of. hypercalcaemia may be related to the underlying cause. Hypercalcaemia can present with
the following:
Next Question
Polyuria
Polydipsia
Previous Question
Muscle weakness
Constipation and abdominal pain End Session
Fatigue and lethargy
Cardiac arrhythmias

43948

Tag Question

Feedback

Difficulty: Average

Peer Responses

Session Progress

Responses Correct: 0

Responses Incorrect: 27

Responses Total: 27

Responses - % Correct: 0%

Blog
About Pastest
Contact Us
Help

https://mypastest.pastest.com/Secure/TestMe/Browser/436619[‫ ص‬06:40:54 09/12/1437]


MyPastest

© Pastest 2016

https://mypastest.pastest.com/Secure/TestMe/Browser/436619[‫ ص‬06:40:54 09/12/1437]


MyPastest

Prefer to use the old MyPastest? Access it here »

Back to Filters

Question 26 of 33

Following a neonatal delivery, a midwife is unable to decide on the gender of the baby due to ambiguous genitalia.

Select the MOST appropriate course of action.

A After
examination of the baby inform the parents of the most likely gender, because it will cause them
unnecessary distress to make them wait

B Inform the parents that a definitive diagnosis will be reached

C Inform
the parents that a number of investigations will need to be performed and that they will need to
wait before a sex is assigned

D Inform
the parents that the chromosomes will need to be done and after this result they will be
informed what gender the child should be reared as

E Suggest
to the midwife that it is likely that the baby is the gender it most looks like so she should
inform the parents of this most likely gender, because it will cause them unnecessary distress to make
them wait

Explanation
Ambiguous genitalia in a newborn infant is a relative emergency; it is important that the baby and the parents are
seen by the most appropriate members of the team as quickly as possible. The priorities are the sex assignment
followed by the sex of rearing and identifying life-threatening disorders.

Although
there is great psychological impacts for parents when they can’t tell family and friends whether they
have had a boy or a girl, this should not encourage junior staff to ‘guess’ what sex the child may be. These patients
need a multidisciplinary approach with endocrinologists, geneticists, surgeons and psychologists.

Causes of ambiguous genitalia:

Masculinised female:

https://mypastest.pastest.com/Secure/TestMe/Browser/436619[‫ ص‬06:41:15 09/12/1437]


MyPastest

congenital adrenal hyperplasia


Next Question
maternal ovarian tumours, ie maternal androgens

Under-masculinised
Previousmales:
Question
androgen insensitivity syndrome
End Session
androgen biosynthesis defect
gonadal dysgenesis

43949

Tag Question

Feedback

Difficulty: Average

Peer Responses

Session Progress

Responses Correct: 0

Responses Incorrect: 28

Responses Total: 28

Responses - % Correct: 0%

Blog
About Pastest
Contact Us
Help

https://mypastest.pastest.com/Secure/TestMe/Browser/436619[‫ ص‬06:41:15 09/12/1437]


MyPastest

© Pastest 2016

https://mypastest.pastest.com/Secure/TestMe/Browser/436619[‫ ص‬06:41:15 09/12/1437]


MyPastest

Prefer to use the old MyPastest? Access it here »

Back to Filters

Question 27 of 33

The following statements refer to osteogenesis imperfecta.

Choose the MOST appropriate statement.

A Alkaline phosphatase levels will be raised

B Bisphosphonate infusion is potentially curative

C Blue sclerae are diagnostic

D Compression vertebral body fractures can be seen

E Type II is likely to be mild

Explanation
Osteogenesis imperfecta is a congenital disease of bone formation, which affects approximately 1 in 10 000
people. It leads to a disorganised bone structure, making the bones fragile with a low bone mass, hence the term
‘brittle bone disease’. There are several different types, which are distinguished by clinical findings, radiological
findings and bone histology. They vary widely in severity from those that are perinatally lethal (type II) to those
that are quite mild, and may be picked up during childhood with increased fractures or with scoliosis due to
vertebral fractures. Type III is the most severe form compatible with survival past the perinatal period. Mutations
in the gene for collagen type I α chains are seen in the most common types; these are mainly autosomal dominant,
although new mutations are common. Blood tests usually show no specific abnormalities. As well as the skeletal
changes it is also has extraskeletal features:

Blue/grey scelera, although blue sclera can be found in healthy babies


Dentinogenesis imperfecta: discoloured teeth that break easily
Ligamental laxity
Increased skin laxity

https://mypastest.pastest.com/Secure/TestMe/Browser/436619[‫ ص‬06:41:37 09/12/1437]


MyPastest

Hearing impairment

Next Question
Management
of osteogenesis imperfecta (OI), particularly the more severe forms, requires a multidisplinary
approach including physiotherapist, occupational therapist and the medical team. Cyclical bisphosphonate
infusions (inhibiting osteoclast function) have been used for the last decade for severe-to-moderate forms of OI in
Previous
children and adolescents Question
to
effectively treat symptoms although the length of treatment and the use
in milder
forms continues to be researched.
End Session 43952

Tag Question

Feedback

Difficulty: Average

Peer Responses

Session Progress

Responses Correct: 0

Responses Incorrect: 29

Responses Total: 29

Responses - % Correct: 0%

Blog
About Pastest
Contact Us
Help

© Pastest 2016

https://mypastest.pastest.com/Secure/TestMe/Browser/436619[‫ ص‬06:41:37 09/12/1437]


MyPastest

Prefer to use the old MyPastest? Access it here »

Back to Filters

Question 28 of 33

An infant has been diagnosed with phenylketonuria after neonatal screening.

Select the MOST appropriate statement from the following:

A It is caused by an inability to produce phenylalanine

B Seizures can occur

C The infant will be severely affected at birth

D The urine is odourless

E Untreated individuals have a normal IQ

Explanation
The metabolic block in phenylketonuria is
the conversion of phenylalanine to tyrosine due to a deficiency of
phenylalanine hydroxylase. Incidence is around 1 in 5000. Inheritance is
autosomal recessive and carrier detection
and prenatal diagnosis are possible. Affected infants are normal at birth.

Clinical features in the untreated patient:

Low IQ
Poor head growth
Seizures
Fair skin ‘dilute pigmentation’– due to inadequate melanisation
Eczema-like rash

Diagnosis
is by measuring the plasma phenylalanine, which will be raised. The urine has a mousy, pungent odour
due to the presence of phenylacetic acid, a metabolite of phenylalanine. Screening is carried out on all babies born

https://mypastest.pastest.com/Secure/TestMe/Browser/436619[‫ ص‬06:41:58 09/12/1437]


MyPastest

in the UK by the Guthrie card. A drop of blood is collected on the card, and the blood is then used to measure the
whole blood phenylalanine by a bacterial inhibition assay. This is done at age 4–5 days, ideally after a feed.
Next Question
Treatment is with a diet that is selectively low in phenylalanine. The diet needs to be lifelong, particularly during
pregnancy when untreated or partially treated phenylketonuria is teratogenic to the unborn child. The
teratogenicity manifests as severe learning difficulties with microcephaly.
43954
Previous Question
End Session

Tag Question

Feedback

Difficulty: Average

Peer Responses

Session Progress

Responses Correct: 0

Responses Incorrect: 30

Responses Total: 30

Responses - % Correct: 0%

Blog
About Pastest
Contact Us
Help

© Pastest 2016

https://mypastest.pastest.com/Secure/TestMe/Browser/436619[‫ ص‬06:41:58 09/12/1437]


MyPastest

Prefer to use the old MyPastest? Access it here »


Next Question

Back to Filters

Question 29 of 33

Which is the MOST appropriate statement about type 2 diabetes in childhood?

A It does not pose a significant problem in western society

B It is always treatable with dietary measures and oral hypoglycaemic agents

C It is rarely associated with the microvascular complications as seen in type 1 diabetes

D It is seen in association with acanthosis nigricans

E It is usually associated with weight loss at presentation

Explanation
Type 2 diabetes in childhood has become an increasingly recognised problem over recent years. Predisposing
factors include ethnicity (highest incidence in Asian and African–American racial groups), obesity, a positive
family history of type 2 diabetes and female sex. Intrauterine growth retardation is also known to be a risk factor.
Insulin resistance and impaired glucose tolerance are seen. Presentation is rarely acute, with children often having
no or mild symptoms only for a considerable length
of time before diagnosis, although ketoacidosis can occur.
Acanthosis nigricans is a marker of high insulin levels and thus helps to differentiate type 2 from type 1 diabetes
in affected individuals.

Treatment
is aimed at achieving good glycaemic control (to minimise risk of micro- and macrovascular
complications) and maintenance of a reasonable weight. There appears to be a greater risk of nephropathy than
retinopathy. Dietary adjustments and oral hypoglycaemic agents (eg metformin) can be used, but, in some cases,
insulin may be required. The
role of prevention is likely to assume greater significance.
43956

https://mypastest.pastest.com/Secure/TestMe/Browser/436619[‫ ص‬06:42:20 09/12/1437]


MyPastest

Tag Question

Feedback

Difficulty: Average
Previous Question
Peer Responses
End Session

Session Progress

Responses Correct: 0

Responses Incorrect: 31

Responses Total: 31

Responses - % Correct: 0%

Blog
About Pastest
Contact Us
Help

© Pastest 2016

https://mypastest.pastest.com/Secure/TestMe/Browser/436619[‫ ص‬06:42:20 09/12/1437]


MyPastest

Prefer to use the old MyPastest? Access it here »

Back to Filters

Question 30 of 33

Select the MOST appropriate statement regarding hypoglycaemia in neonates.

A A bolus of 2.5 ml/kg of 50% dextrose should be used to correct the abnormality

B Increased glycogen stores in large-for-date babies, such as infants of diabetic mothers, are protective

C Intramuscular glucagon is the treatment of choice

D Long-term consequences include lowered IQ and decreased head size

E The best time to take blood and urine samples is immediately after the correction of a hypoglycaemic
episode

Explanation
Causes of hypoglycaemia:

Excessive glucose utilisation


Hyperinsulinism (eg type 1 diabetes mellitus, insulin-producing tumour)
Defects in alternative fuel production (eg medium chain acyl-CoA dehydrogenase deficiency [MCAD])
Sepsis
Glucose underproduction
Inadequate stores (eg preterm, small for gestational age [SGA])
Abnormal hepatic glucose production (eg glycogen storage disease type I, galactosaemia, maple syrup
urine disease)
Hormonal abnormalities (eg panhypopituitarism, growth hormone and cortisol deficiency)
toxins (eg ethanol and propranolol)

https://mypastest.pastest.com/Secure/TestMe/Browser/436619[‫ ص‬06:42:46 09/12/1437]


MyPastest

Usual
definition of hypoglycaemia is a blood sugar of <2.6 mmol/l. To aid diagnosis of possible underlying
metabolic abnormality it is critical that samples (blood and urine) are taken at the time of a hypoglycaemic
Next Question
episode, immediately after correction of hypoglycaemia. it is important
that correction is not delayed whilst blood
samples are taken as the neurocognitive effects of hypoglycaemia can be devastating. As hypoglycaemia can be
asymptomatic, ‘at-risk’ babies such as SGA, premature and infants of mothers with diabetes should have frequent
screening for hypoglycaemia for at least the first 24 hours of life. Treatment consists of an intravenous bolus of
Previous Question
10% dextrose (2.5 ml/kg), followed by an intravenous infusion to match normal hepatic glucose production
(typically 5–8 mg/kg per min in an otherwise normal neonate). Intramuscular End glucagon is the treatment of choice
Session
within the home setting only. It is known that even asymptomatic hypoglycaemia in the neonatal period can have
long-term consequences in the form of neurocognitive impairment and abnormalities apparent on neuroimaging.
43958

Tag Question

Feedback

Difficulty: Average

Peer Responses

Session Progress

Responses Correct: 0

Responses Incorrect: 32

Responses Total: 32

Responses - % Correct: 0%

Blog
About Pastest
Contact Us
Help

© Pastest 2016

https://mypastest.pastest.com/Secure/TestMe/Browser/436619[‫ ص‬06:42:46 09/12/1437]


MyPastest

https://mypastest.pastest.com/Secure/TestMe/Browser/436619[‫ ص‬06:42:46 09/12/1437]


MyPastest

Prefer to use the old MyPastest? Access it here »

Back to Filters

Question 31 of 33

With regard to McCune–Albright syndrome, which is the MOST appropriate statement?

A A family history of café-au-lait pigmentation supports the diagnosis

B Café-au-lait pigmentation is typically present at birth

C Fibrous dysplasia commonly results in pathological fracture

D Precocious puberty caused by this condition is more commonly seen in girls than boys

E Precocious puberty is primarily due to central gonadotrophin-dependent causes

Explanation
Diagnosis of McCune–Albright syndrome requires at least two features of the triad of polyostotic fibrous
dysplasia, café-au-lait pigmentation and autonomous endocrine hyperfunction to be present. The most common
endocrine abnormality is gonadotrophin-independent precocious puberty (ie autonomous ovarian or testicular
function). It is far more common in girls than boys, with breast development and vaginal bleeding being seen
in
some cases before a year of age. Hyperthyroidism, hypercortisolism and acromegaly can also be seen.

Adrenocorticotrophic
hormone (ACTH)-independent Cushing syndrome generally results in growth
failure and
hypertension in infancy.

Fibrous dysplasia most commonly affects the long bones, ribs and skull, and can be asymptomatic or cause pain
and pathological fracture in more severe cases. A family history of café-au-lait spots raises the suspicion of
neurofibromatosis (an autosomal dominant condition) whereas
McCune–Albright syndrome occurs sporadically.
Non-endocrine abnormalities may include hypophosphataemia, chronic liver disease, tachycardia and cardiac
arrhythmia. Treatment is aimed at correcting the
underlying endocrine abnormality, with specialist orthopaedic
input if bony lesions are problematic. Potential complications include loss of final adult height potential (due to
precocious puberty and/or hypophosphataemia), pathological fractures and sudden death.
43961

https://mypastest.pastest.com/Secure/TestMe/Browser/436619[‫ ص‬06:43:07 09/12/1437]


MyPastest

Next Question
Tag Question

Previous Question
Feedback

End Session
Difficulty: Average

Peer Responses

Session Progress

Responses Correct: 0

Responses Incorrect: 33

Responses Total: 33

Responses - % Correct: 0%

Blog
About Pastest
Contact Us
Help

© Pastest 2016

https://mypastest.pastest.com/Secure/TestMe/Browser/436619[‫ ص‬06:43:07 09/12/1437]


MyPastest

Prefer to use the old MyPastest? Access it here »

Back to Filters

Question 32 of 33

Select the MOST appropriate statement about sexual determination and differentiation.

A Androgen insensitivity syndrome leads to an undermasculinised male

B Gender identity should be determined by the phenotypic appearance

C Gonadal differentiation in utero occurs in the second trimester

D If palpable gonads are present, the infant can be assigned the male sex

E Labial adhesions are a cause of ambiguous genitalia

Explanation
In normal embryological development, the presence or absence of a Y-chromosome leads to gonadal
differentiation in the sixth week. The process can be disrupted by exposure to inadequate or excessive amounts of
androgens (testosterone and dihydrotestosterone) and müllerian-inhibiting substance or by end-organ insensitivity
to their actions. Androgen insensitivity syndrome is caused by the end-organ insensitivity and leads to an under-
masculinised boy. Caution must be exercised in the presence of incompletely descended palpable gonads.
Although only testicular material descends fully, ovotestes (as seen in true hermaphroditism) can
sometimes be
felt in labioscrotal folds. Conversely, the absence of palpable gonads in an otherwise fully virilised infant should
raise the possibility of a severely virilised girl with congenital adrenal hyperplasia.

Labial adhesions are not a cause of


ambiguous genitalia but can be a cause of anxiety. The appearance is usually
typical, with fusion of the labial skin extending from the posterior fourchette to the urethral opening; spontaneous
resolution is common.

The development of gender identity is a complex, poorly understood process, with evidence showing that it is
determined not only by phenotypic appearance but by the brain’s pre- and postnatal development. Parents should
be given as much information as possible to enable them to make informed decisions about gender assignment and
possible subsequent genital surgery.

https://mypastest.pastest.com/Secure/TestMe/Browser/436619[‫ ص‬06:43:27 09/12/1437]


MyPastest

43967

Next Question

Previous Question Tag Question

Feedback End Session

Difficulty: Average

Peer Responses

Session Progress

Responses Correct: 0

Responses Incorrect: 34

Responses Total: 34

Responses - % Correct: 0%

Blog
About Pastest
Contact Us
Help

© Pastest 2016

https://mypastest.pastest.com/Secure/TestMe/Browser/436619[‫ ص‬06:43:27 09/12/1437]


MyPastest

Prefer to use the old MyPastest? Access it here »

Back to Filters

Question 33 of 33

Select the MOST appropriate statement listed below about obesity in childhood.

A It is a greater problem in girls than boys in the UK population

B It is a risk factor for subclinical coronary artery sclerosis and atherosclerosis in childhood

C It is defined as a body mass index (weight in kilograms/[height in metres]2) >30

D The
various strategies that are used to tackle obesity (ie dietary modification and increased physical
activity) have a strong evidence base supporting their use

E Weight loss is the most frequently recommended course of action to prevent long-term health
complications

Explanation
This is an increasing problem that is difficult to treat and has implications for both physical and psychosocial
health in childhood and beyond. Most children are obese due
to their lifestyle and not any underlying medical
condition. There is little evidence to support the long-term effectiveness of the currently used strategies of an
improved dietary intake and increased levels of physical activity. Risk factors associated with obesity are
deprivation and increasing age. There is no difference in prevalence of obesity between boys and girls. Parental
obesity is a risk factor for persistence of childhood obesity into adulthood. As childhood is a period of growth,
body mass index (BMI) is not a static measurement but varies with age and sex. Obesity in childhood is therefore
defined as a BMI >95th centile for age and sex.

Consequences of obesity include hypertension, an increased risk of developing or worsening asthma, and
abnormalities of foot structure and function. In addition, adverse lipid profiles, insulin resistance and
hyperinsulinaemia are also seen, leading to atherosclerosis and coronary
artery disease, which are seen with
greater frequency in obese children
post mortem. Girls are more likely than boys to suffer psychological distress
related to their obesity. Adults who were obese children are also more at risk of cardiovascular disease and have
an increased mortality risk.

https://mypastest.pastest.com/Secure/TestMe/Browser/436619[‫ ص‬06:43:47 09/12/1437]


MyPastest

43968

End Session

Previous Question Tag Question

Feedback

Difficulty: Average

Peer Responses

Session Progress

Responses Correct: 0

Responses Incorrect: 35

Responses Total: 35

Responses - % Correct: 0%

Blog
About Pastest
Contact Us
Help

© Pastest 2016

https://mypastest.pastest.com/Secure/TestMe/Browser/436619[‫ ص‬06:43:47 09/12/1437]


MyPastest

Prefer to use the old MyPastest? Access it here »

Back to Filters

Question 1 of 16

Theme: Statistics and research methods

A Incidence

B Lag time

C Lead time

D Length bias

E Likelihood ratio

F Number needed to treat

G Paired cohort

H Prevalence

I Sensitivity

J Specificity

For
each of the following definitions, choose the word that applies to it from the above list. Each item may be used
once, more than once or not at all.

Scenario 1

Interval between identification of a condition by screening and the development of symptoms.

Your answer was incorrect

Select one...

C - Lead time

Scenario 2

Odds of a positive test result in an affected individual compared with that of a positive result in an unaffected
individual.

Your answer was incorrect

https://mypastest.pastest.com/Secure/TestMe/Browser/436619[‫ ص‬10:31:03 10/12/1437]


MyPastest

Select one...

E - Likelihood ratio

Scenario 3

Proportion of people unaffected by a condition correctly identified by a designated test.

Your answer was incorrect

Select one...

J - Specificity

Lead time: the time between a condition being identified by screening and a condition becoming clinically
apparent.
Lag time: the time between an intervention being assessed as clinically useful and an intervention actually
entering everyday practice.
Sensitivity: percentage of those with a condition who correctly test positive.
Specificity: percentage of those without a condition who correctly test negative.
Likelihood ratio:
odds of a positive test result in an affected individual compared with a
positive result in
an unaffected individual – this is a positive likelihood ratio.
Number needed to treat (NNT): the number of patients who would need to have an intervention for a set
outcome to be shown in one of them.
Prevalence: the total number of cases in a population at any one time (expressed as a proportion of the
total population).
Incidence: rate at which new cases of a condition occur in a population (over a set period of time – usually
1 year).

22246
Next Question

Tag Question
End Session
Feedback

Difficulty: Average

Previous Question Session Progress

Responses Correct: 0

https://mypastest.pastest.com/Secure/TestMe/Browser/436619[‫ ص‬10:31:03 10/12/1437]


MyPastest

Responses Incorrect: 3

Responses Total: 3

Responses - % Correct: 0%

Blog
About Pastest
Contact Us
Help

© Pastest 2016

https://mypastest.pastest.com/Secure/TestMe/Browser/436619[‫ ص‬10:31:03 10/12/1437]


MyPastest

Prefer to use the old MyPastest? Access it here »

Back to Filters

Question 2 of 16

Non-parametric tests:

A Are applicable for normally distributed data

B Are less powerful than parametric tests

C Are more complex to administer

D Should only be used as a last resort

E Use most information from the data

Explanation
Non-parametric tests can be used for normally distributed data but parametric tests are preferable if normality can
be
assumed because non-parametric tests are less efficient, using less information from the data. Generally, non-
parametric tests are no more complex to administer than parametric tests, however. The decision to use a non-
Next Question
parametric test should be made according to the distribution of the data and size of the sample – they are not just
used as a last resort. Non-parametric tests are less powerful than their parametric counterparts. 23612

End Session

Tag Question

Feedback

Previous Question
Difficulty: Difficult

https://mypastest.pastest.com/Secure/TestMe/Browser/436619[‫ ص‬10:31:41 10/12/1437]


MyPastest

Peer Responses

Session Progress

Responses Correct: 0

Responses Incorrect: 4

Responses Total: 4

Responses - % Correct: 0%

Blog
About Pastest
Contact Us
Help

© Pastest 2016

https://mypastest.pastest.com/Secure/TestMe/Browser/436619[‫ ص‬10:31:41 10/12/1437]


MyPastest

Prefer to use the old MyPastest? Access it here »

Back to Filters

Question 3 of 16

A screening test correctly identifies 90 of 100 individuals with disease and falsely identifies a further 15 of 300
individuals without disease.

Which one of the following statements is true?

A A larger study is required

B Most individuals with the disease were identified

C The screening test is worthwhile

D The sensitivity of the test is 90%

E The specificity of the test is 90%

Explanation
Next Question
While most of the individuals with disease were identified, a more precise quantification is that the sensitivity is
90%, i.e. 90% of those with disease were correctly identified. The specificity of the test is 95% (300 – 15)/300. A
larger study would give more precise estimates but is not necessarily needed. How worthwhile the screening test is
depends on the cost, convenience and predictive values in different populations. 23613

End Session

Tag Question

Feedback
Previous Question

https://mypastest.pastest.com/Secure/TestMe/Browser/436619[‫ ص‬10:32:04 10/12/1437]


MyPastest

Difficulty: Easy

Peer Responses

Session Progress

Responses Correct: 0

Responses Incorrect: 5

Responses Total: 5

Responses - % Correct: 0%

Blog
About Pastest
Contact Us
Help

© Pastest 2016

https://mypastest.pastest.com/Secure/TestMe/Browser/436619[‫ ص‬10:32:04 10/12/1437]


MyPastest

Prefer to use the old MyPastest? Access it here »

Back to Filters

Question 4 of 16

The process of randomisation in a clinical trial:

A Aims to remove confounding

B Helps to ensure that patients are blinded to treatment group

C Is facilitated by the availability of suitable placebo

D Is more important in multicentre trials

E Must be performed away from the trial centre

23614

Next Question
Tag Question

Feedback

Difficulty: Average End Session

Peer Responses

Previous Question

https://mypastest.pastest.com/Secure/TestMe/Browser/436619[‫ ص‬10:32:25 10/12/1437]


MyPastest

Session Progress

Responses Correct: 0

Responses Incorrect: 6

Responses Total: 6

Responses - % Correct: 0%

Blog
About Pastest
Contact Us
Help

© Pastest 2016

https://mypastest.pastest.com/Secure/TestMe/Browser/436619[‫ ص‬10:32:25 10/12/1437]


MyPastest

Prefer to use the old MyPastest? Access it here »

Back to Filters

Question 5 of 16

Box plots:

A Are preferred to showing the actual values

B Are useful for numerical data

C Enable comparisons to be made between subgroups of the data

D Should be used for numerical and not categorical outcomes

E Show distributionally outlying values and provide data summaries that are not unduly influenced by
those outliers

Explanation
Box plots can be used to display numerical outcomes and give valid summaries (median and interquartile range or
IQR) for any
distributional form that the outcomes might take. Additionally, they display the outliers. They are not
Next Question
as informative as showing the actual values but can be used to make comparisons of medians and IQRs between
subgroups. 23615

End Session

Tag Question

Feedback

Previous Question
Difficulty: Difficult

https://mypastest.pastest.com/Secure/TestMe/Browser/436619[‫ ص‬10:32:51 10/12/1437]


MyPastest

Peer Responses

Session Progress

Responses Correct: 0

Responses Incorrect: 7

Responses Total: 7

Responses - % Correct: 0%

Blog
About Pastest
Contact Us
Help

© Pastest 2016

https://mypastest.pastest.com/Secure/TestMe/Browser/436619[‫ ص‬10:32:51 10/12/1437]


MyPastest

Prefer to use the old MyPastest? Access it here »

Back to Filters

Question 6 of 16

When sampling patients for a clinical trial the most important thing is that:

A The most seriously ill patients are selected

B The patients do not have prior opinions about the best treatment

C The study size is large

D The treatment has not been trialled before

E They are a random and representative sample

Explanation
Clinical trials can validly be performed for treatments that have previously been trialled, perhaps in other groups
of patients. The patient group selected for the trial will depend on the
specific research question and do not
necessarily need to be the most seriously ill. It does not matter if the patients have prior opinions about the best
Next Question
treatment provided they agree to be randomised and are blinded to their treatment group. It is important that the
samples used for the trial are random and so representative. 23616

End Session

Tag Question

Feedback

Previous Question
Difficulty: Easy

https://mypastest.pastest.com/Secure/TestMe/Browser/436619[‫ ص‬10:33:09 10/12/1437]


MyPastest

Peer Responses

Session Progress

Responses Correct: 0

Responses Incorrect: 8

Responses Total: 8

Responses - % Correct: 0%

Blog
About Pastest
Contact Us
Help

© Pastest 2016

https://mypastest.pastest.com/Secure/TestMe/Browser/436619[‫ ص‬10:33:09 10/12/1437]


MyPastest

Prefer to use the old MyPastest? Access it here »

Back to Filters

Question 7 of 16

We want to compare body mass index (BMI) values between children


from several ethnic groups, taking into
account differences in the age distributions of the groups. The appropriate analysis is:

A Chi-square

B Kruskal–Wallis ANOVA

C Linear regression analysis with age and ethnicity as predictors

D One-way analysis of variance

E Student’s t-test

Explanation
BMI is a numerical outcome. Chi-square is used to compare proportions and so is inappropriate for a numerical
outcome. Student’s t-test and one-way analysis of variance can be used to compare normally distributed outcomes
Next Question
in two and more than two groups respectively. Kruskal–Wallis ANOVA can be used to compare numeric
outcomes in more than two groups, regardless of the distribution
of the values in each group. However, none of
the listed tests can make
adjustment for any other variables. To compare numerical outcomes between groups
while making adjustment for other factors requires the use of linear regression analysis. For example, to compare
BMIs between different ethnic groups while taking account of differences in age distribution between groups
would require a linear regression analysis with age and ethnicity as predictors. 23617

Previous Question Tag Question

https://mypastest.pastest.com/Secure/TestMe/Browser/436619[‫ ص‬10:33:28 10/12/1437]


MyPastest

Feedback

Difficulty: Difficult

Peer Responses

Session Progress

Responses Correct: 0

Responses Incorrect: 9

Responses Total: 9

Responses - % Correct: 0%

Blog
About Pastest
Contact Us
Help

© Pastest 2016

End Session

https://mypastest.pastest.com/Secure/TestMe/Browser/436619[‫ ص‬10:33:28 10/12/1437]


MyPastest

Prefer to use the old MyPastest? Access it here »

Back to Filters

Question 8 of 16

The primary reason for a written protocol is to:

A Detail the analyses to be undertaken

B Document the process of the trial in advance

C Inform the ethical committee

D Name the investigators

E Provide a power calculation

Explanation
A written protocol should be produced prior to any study commencing and this should, amongst other things,
name the investigators and their roles and provide a power calculation and the analyses to be undertaken. The
protocol should in fact document the whole planned process of the trial in advance and any deviations from this
Next Question
should be clearly documented. One purpose of the written protocol is to ensure that data collection is not biased by
early findings and to
show that the study has been properly conducted. The ethics committee might sometimes, but
not always, want to see the written protocol. 23618

End Session

Tag Question

Feedback
Previous Question
Difficulty: Average

https://mypastest.pastest.com/Secure/TestMe/Browser/436619[‫ ص‬10:33:47 10/12/1437]


MyPastest

Peer Responses

Session Progress

Responses Correct: 0

Responses Incorrect: 10

Responses Total: 10

Responses - % Correct: 0%

Blog
About Pastest
Contact Us
Help

© Pastest 2016

https://mypastest.pastest.com/Secure/TestMe/Browser/436619[‫ ص‬10:33:47 10/12/1437]


MyPastest

Prefer to use the old MyPastest? Access it here »

Back to Filters

Question 9 of 16

Currently healthy individuals are assessed according to current body mass index (BMI). Two years later their
health status is assessed and relationships with earlier BMI determined. What best describes this type of study
design?

A Cohort

B Controlled trial

C Ecological

D Observational

E Retrospective

Explanation
When groups are classified according to one or more factors at a given time and followed forward to determine
Next Question
outcomes (usually some health status), this is a prospective observational study.
Although there is in one sense a
control group (those who do not develop the health problem), this is not generally called a ‘controlled’ trial. An
ecological study would look at outcomes in different groups (countries or regions usually) who follow different
practices. The common name for a study as described would be ‘cohort’. 23619

End Session

Tag Question

Previous Question
Feedback

https://mypastest.pastest.com/Secure/TestMe/Browser/436619[‫ ص‬10:34:06 10/12/1437]


MyPastest

Difficulty: Difficult

Peer Responses

Session Progress

Responses Correct: 0

Responses Incorrect: 11

Responses Total: 11

Responses - % Correct: 0%

Blog
About Pastest
Contact Us
Help

© Pastest 2016

https://mypastest.pastest.com/Secure/TestMe/Browser/436619[‫ ص‬10:34:06 10/12/1437]


MyPastest

Prefer to use the old MyPastest? Access it here »

Back to Filters

Question 10 of 16

Crossover trials:

A Are always more efficient

B Are best for assessing the efficacy of different treatments in giving short-term relief of chronic
conditions

C Are free from carry-over effects

D Require a long washout period

E Should be used in preference to parallel trials

Explanation
With a crossover trial each participant tries the different treatments (or treatment and placebo) in random order. It
is important that the order of treatments is random and not the same for all participants so that any carry-over
Next Question
effect, which there might or might not be, can be quantified. It is important that the individual is the same at the
start of each treatment period and a washout period between treatments might be needed to ensure this, i.e. a time
of no measurement (and possibly no treatment) to allow things to stabilise after the first treatment. Whether there
needs to be a washout period and how long this needs to be depends on the nature of the treatment. Crossover
trials are less prone to confounding and are more efficient than parallel trials (different patients on each treatment),
and should be preferred where the nature and outcome for the treatments allows them. Crossover trials are only
valid where short-term relief is given for chronic conditions so that there is something to treat during the second
phase for each patient. 23620

https://mypastest.pastest.com/Secure/TestMe/Browser/436619[‫ ص‬10:34:25 10/12/1437]


MyPastest

Tag Question

Feedback

Difficulty: Average

Peer Responses

Session Progress

Responses Correct: 0

Responses Incorrect: 12

Responses Total: 12

Responses - % Correct: 0%

Blog
About Pastest
Contact Us
Help

© Pastest 2016

End Session

Previous Question

https://mypastest.pastest.com/Secure/TestMe/Browser/436619[‫ ص‬10:34:25 10/12/1437]


MyPastest

Prefer to use the old MyPastest? Access it here »

Back to Filters

Question 11 of 16

Which one of the following is the best study design to use for testing the effectiveness of a new treatment?

A Case–control

B Cohort

C Cross-sectional

D Ecological

E Randomised controlled trial

Explanation
To test the effectiveness of a new treatment, ecological, case–control, cross-sectional and cohort studies are all
observational studies that might show a relationship. However, it cannot be inferred from any of these that the
treatment causes any differences between the treated and untreated groups because they are observational and
Next Question
open to confounding. A randomised control trial of treated versus untreated would allow causality to be inferred
and so this is the preferred study design. 23621

End Session

Tag Question

Feedback

Previous Question
Difficulty: Easy

https://mypastest.pastest.com/Secure/TestMe/Browser/436619[‫ ص‬10:34:45 10/12/1437]


MyPastest

Peer Responses

Session Progress

Responses Correct: 0

Responses Incorrect: 13

Responses Total: 13

Responses - % Correct: 0%

Blog
About Pastest
Contact Us
Help

© Pastest 2016

https://mypastest.pastest.com/Secure/TestMe/Browser/436619[‫ ص‬10:34:45 10/12/1437]


MyPastest

Prefer to use the old MyPastest? Access it here »

Back to Filters

Question 12 of 16

Consent to randomisation:

A Should be avoided by only consenting people in the treatment arm

B Is the main ethical issue

C Is not necessary

D Should be obtained as part of the overall consent to the study

E Will depend on the nature of randomisation

Explanation
It is important that consent is obtained from study participants and they should consent to all parts of the study,
including randomisation. The entire consent process is an ethical issue and the consent should include an
understanding of the randomisation process being used. Consent to randomisation has been avoided in some
Next Question
studies by only consenting those allocated to treatment (known as a Zelen design), but there are disadvantages
with this design. 23622

End Session

Tag Question

Feedback

Previous Question
Difficulty: Easy

https://mypastest.pastest.com/Secure/TestMe/Browser/436619[‫ ص‬10:35:04 10/12/1437]


MyPastest

Peer Responses

Session Progress

Responses Correct: 0

Responses Incorrect: 14

Responses Total: 14

Responses - % Correct: 0%

Blog
About Pastest
Contact Us
Help

© Pastest 2016

https://mypastest.pastest.com/Secure/TestMe/Browser/436619[‫ ص‬10:35:04 10/12/1437]


MyPastest

Prefer to use the old MyPastest? Access it here »

Back to Filters

Question 13 of 16

Power calculations for comparison of a numerical measurement between two groups:

A Are complex

B Do not help with the interpretation of results

C Should be done retrospectively when the variance of the measurement in the samples is known

D Should be performed before the study commences

E Should have the power set at 80%

Explanation
Power calculations should be performed before any study commences and do not help directly with the
interpretation of the results. They can also be done retrospectively if the estimates they were originally based on
are found to be erroneous. The level of power selected will depend on how important it is not to miss any
Next Question
difference that exists and this will vary from study to study. Power calculations for a comparison of a numerical
measurement between two groups are standard and not especially complex. 23623

End Session

Tag Question

Feedback

Previous Question
Difficulty: Average

https://mypastest.pastest.com/Secure/TestMe/Browser/436619[‫ ص‬10:35:23 10/12/1437]


MyPastest

Peer Responses

Session Progress

Responses Correct: 0

Responses Incorrect: 15

Responses Total: 15

Responses - % Correct: 0%

Blog
About Pastest
Contact Us
Help

© Pastest 2016

https://mypastest.pastest.com/Secure/TestMe/Browser/436619[‫ ص‬10:35:23 10/12/1437]


MyPastest

Prefer to use the old MyPastest? Access it here »

Back to Filters

Question 14 of 16

The best study design to determine whether a certain factor is causally implicated in the onset of a rare disease is:

A Case–control

B Cohort

C Cross-sectional

D Ecological

E Randomised controlled trial

Explanation
An ecological study might show an association between disease rates of a rare disease and the potentially causal
factor in different groups of individuals (from different countries or regions, for example), but would be open to
confounding. A randomised control trial, cross-sectional or cohort study would all require prohibitively large
Next Question
numbers if the disease is rare. A case–control
study can study past features of those with and without the rare
disease to find potentially causal agents and is the preferred study type to determine a relationship. 23624

End Session

Tag Question

Feedback

Previous Question
Difficulty: Average

https://mypastest.pastest.com/Secure/TestMe/Browser/436619[‫ ص‬10:35:42 10/12/1437]


MyPastest

Peer Responses

Session Progress

Responses Correct: 0

Responses Incorrect: 16

Responses Total: 16

Responses - % Correct: 0%

Blog
About Pastest
Contact Us
Help

© Pastest 2016

https://mypastest.pastest.com/Secure/TestMe/Browser/436619[‫ ص‬10:35:42 10/12/1437]


MyPastest

Prefer to use the old MyPastest? Access it here »

Back to Filters

Question 15 of 16

Theme: Graphical displays

Options

A. Bar chart
B. Box plot
C. Histogram
D. Line chart
E. Scatter plot
F. Side-by-side or stacked bar chart

Instructions:
For
each of the following sets of data, select the most appropriate graphical display from the list above. Each
option may be used once, more than once or not at all.

Scenario 1

Ages at operation, categorised as 0–1 month, 2–6 months, 6–12 months, 1–5 years, over 5 years.

Your answer was incorrect

Select one...

C - Histogram

Histogram

https://mypastest.pastest.com/Secure/TestMe/Browser/436619[‫ ص‬10:36:07 10/12/1437]


MyPastest

Pictorial example of a histogram

Scenario 2

Blood pressure and weight in a sample of 500 men.

Your answer was incorrect

Select one...

E - Scatter plot

Scatter plot

Pictorial example of a scatter plot

Scenario 3

IgG measurements in different ethnic groups.

Your answer was incorrect

Select one...

B - Box plot

Box plot

https://mypastest.pastest.com/Secure/TestMe/Browser/436619[‫ ص‬10:36:07 10/12/1437]


MyPastest

Example of box plot (box and whisker plot)

Example of bar chart

Bar
charts can be used to display the frequencies of categorical data or numerical data if divided into categories.
The heights of the bar show the frequency within each category.

Histograms are similar to bar charts but are used specifically for numerical
data that is categorised and show the
frequency per unit. They differ from bar charts when the numerical variable is not put into categories of equal
width (for example lengths categorised as 0–10, 11–50, 51–250, 251–1000).

Box plots can be used to summarise numerical data within several categories.

Line
charts can be used to show how a numerical variable changes over time or dosages within individuals. There
should be one line per individual.

https://mypastest.pastest.com/Secure/TestMe/Browser/436619[‫ ص‬10:36:07 10/12/1437]


MyPastest

Example of a line chart

A
scatter plot is a good way to show the relationship between two numerical variables. One variable is plotted on
each axis and a single point shows the two values for each individual.

Side-by-side or stacked bar charts are a good way to show the relationships between two categorical variables.

Example of a stacked bar chart


23625

Next Question

Tag Question

Feedback End Session

Difficulty: Average

Session Progress

Previous Question
Responses Correct: 0

Responses Incorrect: 19

https://mypastest.pastest.com/Secure/TestMe/Browser/436619[‫ ص‬10:36:07 10/12/1437]


MyPastest

Responses Total: 19

Responses - % Correct: 0%

Blog
About Pastest
Contact Us
Help

© Pastest 2016

https://mypastest.pastest.com/Secure/TestMe/Browser/436619[‫ ص‬10:36:07 10/12/1437]


MyPastest

Prefer to use the old MyPastest? Access it here »

Back to Filters

Question 16 of 16

What is the leading accidental cause of child death under the age of 15?

A Road traffic accident

B Falls

C Drowning

D Asphyxia

E Poisoning

Explanation
RTAs are the leading cause [~35%] of accidental death in children <15y old

Asphyxia is the 2nd most common [30%].

Drowning is the 3rd most common [13%].

Poisoning causes 5% of accidental deaths.

Falls cause 3.5% of accidental deaths.

[Ref Office National Statistics]


45295

End Session

Previous Question Tag Question

Feedback

https://mypastest.pastest.com/Secure/TestMe/Browser/436619[‫ ص‬10:36:29 10/12/1437]


MyPastest

Difficulty: Average

Peer Responses

Session Progress

Responses Correct: 0

Responses Incorrect: 20

Responses Total: 20

Responses - % Correct: 0%

Blog
About Pastest
Contact Us
Help

© Pastest 2016

https://mypastest.pastest.com/Secure/TestMe/Browser/436619[‫ ص‬10:36:29 10/12/1437]


MyPastest

Prefer to use the old MyPastest? Access it here »


Next Question

Back to Filters

Question 1 of 78

A 13-year-old girl has had recurrent abdominal pain and intermittent diarrhoea over the previous year. During
these episodes she
may pass 3-7 very loose stools with mucus. Over the past 3 months she has also passed stools
mixed with blood during the attacks. Though she has not lost weight, her weight has decreased by crossing a
centile. She
has not had her menarche. The mother suffers from vitiligo. Clinical examination was unremarkable.
Her blood tests are as follows: Hb 12.1 g/l, normal differential count, ESR 38mm. An autoantibody screen is
negative.

The most relevant next investigation is?

A Barium enema

B Colonoscopy

C Radio-isotope study

D Abdominal X-ray

E Angiography

Explanation
This girl is most likely to be suffering from inflammatory bowel disease, probably ulcerative colitis. The most
valuable investigation that will give an assessment of severity and extent of the disease, including the opportunity
to obtain biopsies is a
colonoscopy.

Barium studies and abdominal x-rays do not give sufficient information. Radio-isotope scans will help in
identifying a focus such as a Meckel's diverticulum and angiography is rarely indicated unless a vascular lesion is
suspected of leading to the intestinal bleed.
14562

https://mypastest.pastest.com/Secure/TestMe/Browser/436619[‫ ص‬06:47:04 09/12/1437]


MyPastest

Previous Question Tag Question

Feedback

Difficulty: Easy
End Session
Peer Responses

Session Progress

Responses Correct: 0

Responses Incorrect: 1

Responses Total: 1

Responses - % Correct: 0%

Blog
About Pastest
Contact Us
Help

© Pastest 2016

https://mypastest.pastest.com/Secure/TestMe/Browser/436619[‫ ص‬06:47:04 09/12/1437]


MyPastest

Prefer to use the old MyPastest? Access it here »


Previous Question Next Question

Back to Filters

Question 2 of 78

A 6-week-old baby boy presents to the emergency department with a history of vomiting. the vomiting has been
present since the third week of life but has suddenly increased in violence and frequency over the preceding 3
days. The baby is well but hungry and failing to thrive.

The most useful next step is?

A Take an accurate feeding history and decrease feed volumes

B Take a blood gas

C Perform a septic screen

D Start intensive gastro-oesophageal reflux therapy

E Refer surgically immediately

Explanation
The most likely diagnosis needing exclusion is pyloric stenosis, so doing a blood gas will help establish the
diagnosis first and is preferable to a referral without thought.

Speculative
reduction of feed volumes will miss the diagnosis, as would presuming it is just exacerbated gastro-
oesophageal reflux. Only once pyloric stenosis is excluded should overfeeding and gastro-oesophageal reflux be
considered in the differential diagnosis. The child is well so sepsis is least likely.
14563

Tag Question

https://mypastest.pastest.com/Secure/TestMe/Browser/436619[‫ ص‬06:47:31 09/12/1437]


MyPastest

Feedback

Difficulty: Easy

Peer Responses

End Session

Session Progress

Responses Correct: 0

Responses Incorrect: 2

Responses Total: 2

Responses - % Correct: 0%

Blog
About Pastest
Contact Us
Help

© Pastest 2016

https://mypastest.pastest.com/Secure/TestMe/Browser/436619[‫ ص‬06:47:31 09/12/1437]


MyPastest

Prefer to use the old MyPastest? Access it here »


Previous Question Next Question

Back to Filters

End Session
Question 3 of 78

A 3-year-old girl presents with abdominal pain, infrequent stools, screaming on defecation, blood per rectum, and
poor eating. On examination she is uncooperative but is noticed to have anal tag and fissure.

The commonest cause for her condition is?

A Crohn's disease

B Haemorrhoids

C Constipation

D Hirschprung's disease

E Perianal streptococcal disease

Explanation
The vignette is a classic story for constipation. Crohn's disease is far less common. Haemorrhoids very rarely
occur in children. It is a late presentation for Hirschprung's disease. Perianal streptococcal infection does occur
and causes a very painful bottom, as do pinworms but the presence of a fissure and tags does not fit this diagnosis.
14564

Tag Question

Feedback

Difficulty: Average

https://mypastest.pastest.com/Secure/TestMe/Browser/436619[‫ ص‬06:47:52 09/12/1437]


MyPastest

Peer Responses

Session Progress

Responses Correct: 0

Responses Incorrect: 3

Responses Total: 3

Responses - % Correct: 0%

Blog
About Pastest
Contact Us
Help

© Pastest 2016

https://mypastest.pastest.com/Secure/TestMe/Browser/436619[‫ ص‬06:47:52 09/12/1437]


MyPastest

Prefer to use the old MyPastest? Access it here »


Previous Question Next Question

Back to Filters

End Session
Question 4 of 78

A 5-week-old baby boy of Afro-Caribbean parents presents to your surgery with jaundice. He was initially breast
fed but has been bottle fed for the past week and has not gained any weight for nearly 2 weeks.

The most useful next action is?

A Test the stools for reducing substances

B Request urgent liver function tests

C Change the milk to a soy based baby formula

D Request a blood test for sickle cell disease

E Organise a supra-pubic aspirate for urgent microscopy, culture and sensitivity

Explanation
An infant, who is not thriving and who has persistent jaundice after switching to formula milk, must be further
investigated. This cannot simply be put down to simple prolonged breast milk jaundice. Initially liver function
tests must be taken and a total and split bilirubin levels should be sought. The most important diagnosis to exclude
is biliary atresia. 14565

Tag Question

Feedback

https://mypastest.pastest.com/Secure/TestMe/Browser/436619[‫ ص‬06:58:11 09/12/1437]


MyPastest

Difficulty: Average

Peer Responses

Session Progress

Responses Correct: 0

Responses Incorrect: 4

Responses Total: 4

Responses - % Correct: 0%

Blog
About Pastest
Contact Us
Help

© Pastest 2016

https://mypastest.pastest.com/Secure/TestMe/Browser/436619[‫ ص‬06:58:11 09/12/1437]


MyPastest

Prefer to use the old MyPastest? Access it here »

Back to Filters

Question 5 of 78

A 5-year-old girl has had a persistent nocturnal cough for over 2 years that has not responded to treatment
including inhaled steroids and bronchodilators. The cough tends to be paroxysmal and is associated with vomiting
and disturbing her sleep virtually every night.
She is happy and normally active during the day and did not have
any exercise intolerance. In other respects she has been well with no significant past history. There is no family
history of asthma/eczema/allergy. Clinically, her weight was on the 75th centile and her height on the 50th centile.
General and systematic examinations were normal. CXR - essentially normal. FBC - normal range. CRP 5. PEF
90% predicted. Mantoux negative.

The next most useful that will aid the diagnosis is?

A Bronchoscopy

B CT scan of thorax

C Ambulatory oesophageal pH study

D Barium meal

E Spirometry

Explanation
This 5-year-old had a persistent cough that was characteristically nocturnal, accompanied by vomiting and with no
evidence of any diurnal respiratory symptoms. The possibility was that the cough is related to the supine posture,
especially as there was accompanying vomiting which was a constant feature. Therefore, in the first instance, the
least invasive investigation of ambulatory oesophageal pH monitoring is done. A barium meal is done if the
possibility of a hiatus hernia is considered. A bronchoscopy and CT scan
may be indicated if the initial
investigations are negative.

This girl has severe gastro-oesophageal reflux and symptoms subsided dramatically following anti-reflux therapy.
14887

https://mypastest.pastest.com/Secure/TestMe/Browser/436619[‫ ص‬06:58:30 09/12/1437]


MyPastest

Previous Question Next Question


Tag Question

Feedback

End Session
Difficulty: Easy

Peer Responses

Session Progress

Responses Correct: 0

Responses Incorrect: 5

Responses Total: 5

Responses - % Correct: 0%

Blog
About Pastest
Contact Us
Help

© Pastest 2016

https://mypastest.pastest.com/Secure/TestMe/Browser/436619[‫ ص‬06:58:30 09/12/1437]


MyPastest

Prefer to use the old MyPastest? Access it here »

Back to Filters

Question 6 of 78

Theme: Gastro-intestinal disorders

A Abetalipoproteinaemia

B Coeliac Disease

C Constipation with overflow

D Crohn’s disease

E Giardiasis

F Ileal tuberculosis

G Lactose intolerance

H Toddler’s diarrhoea

I Ulcerative colitis

J Viral gastro-enteritis

For
each of the following cases, choose the most likely cause of diarrhoea from the above list. Each item may be
used once, more than once or not at all.

Scenario 1

A 9-year-old boy presents with a 2-month history of weight loss, abdominal pain and intermittent diarrhoea. There
is no blood in the stool. He has a mildly tender abdomen and some perianal skin tags. Investigations reveal: CRP
45 mg/dL, ESR 50 mm first
hour.

Your answer was incorrect

Select one...

D - Crohn’s disease

From the history, and the objective weight loss, there is likely to be a significant pathology ongoing. The time
course is a little lengthy to be infective, although not impossible. The inflammatory markers are raised, indicating
an ongoing inflammatory process. The perianal skin tags should strongly raise the suspicion of Crohn’s disease.

https://mypastest.pastest.com/Secure/TestMe/Browser/436619[‫ ص‬06:58:52 09/12/1437]


MyPastest

This disease is increasing in childhood and the current incidence in the UK is approximately 10 per 100,000. It is
an inflammatory process involving the whole bowel (mouth to anus). Clinically patients present with abdominal
Previous
pain, weight loss/reduced Question
growth Next Question
and diarrhoea. On examination there may mouth ulceration, and perianal lesions.
Extragastro-intestinal features may be
present – including arthralgia, anaemia and uveitis. Radiological
contrast
studies will show involvement of various parts of the bowel (skip lesions), There may also be signs of
inflammation –‘cobblestones’ or ‘rose-thorn’ ulcers. Treatment should be shared with a tertiary level,
multidisciplinary, paediatric gastro-enterological team.

Scenario 2

A 3-year-old girl has a 2-month history of


loose stools. Her parents are very concerned. The stools often contain
‘undigested food’. She is otherwise well and thriving.

Your answer was incorrect

Select one...

H - Toddler’s diarrhoea

This is a classic history for toddler’s diarrhoea. The child is well and thriving but the parents can understandably
be anxious. The cause is thought to be due a fast enteric transit time, and a brisk gastrocolic reflex. Reassurance is
vital, but simple measures such as reducing fruit juice intake may help.

Scenario 3

An 8-month-old infant has had watery diarrhoea for 4 days. There has been no recent travel. Her 2-year-old sibling
has recently had a similar illness, though less severe. She is drinking her normal formula milk well and is not
dehydrated.

Your answer was incorrect

Select one...

J - Viral gastro-enteritis

This is an acute history so an infective process is likely; the fact that her sibling had a similar illness again makes
an infective cause much more likely. A viral cause is more common in this age group (e.g. rotavirus). Viral gastro-
enteritis is a self-limiting condition but there is a risk of dehydration if oral intake is insufficient. Medications such
as loperamide or codeine to ‘reduce’ the diarrhoea have no place in management, and may lengthen the duration
of the illness. Secondary lactose intolerance is uncommon and before changing the milk to a lactose-free formula,
stool reducing sugars should be checked.
22267

Tag Question

https://mypastest.pastest.com/Secure/TestMe/Browser/436619[‫ ص‬06:58:52 09/12/1437]


MyPastest

Feedback

Difficulty: Average

Session Progress

Responses Correct: 0
End Session
Responses Incorrect: 8

Responses Total: 8

Responses - % Correct: 0%

Blog
About Pastest
Contact Us
Help

© Pastest 2016

https://mypastest.pastest.com/Secure/TestMe/Browser/436619[‫ ص‬06:58:52 09/12/1437]


MyPastest

Prefer to use the old MyPastest? Access it here »


Previous Question Next Question

Back to Filters

End Session
Question 7 of 78

A 7-week-old baby boy is referred with a 2-week history of vomiting. He is being formula fed 5 oz (approximately
150 mL)
every 2–3 h. On examination he is well, thriving and has a normal
examination. The most likely
diagnosis is:

A Pyloric stenosis

B Gastro-oesophageal reflux

C Over-feeding

D Gastroenteritis

E Jejunal stenosis

Explanation
Although all of the answers would cause the listed symptoms, pyloric stenosis, jejunal stenosis and gastroenteritis
would be very unlikely in a thriving child. The daily volume of feed is quite substantial (1200 mL) and in a 5 kg
baby would work out to 240 mL/kg per day! Approximately 150 mL/kg per day is a rough guide to a baby’s milk
requirement.
22344

Tag Question

Feedback

https://mypastest.pastest.com/Secure/TestMe/Browser/436619[‫ ص‬06:59:12 09/12/1437]


MyPastest

Difficulty: Easy

Peer Responses

Session Progress

Responses Correct: 0

Responses Incorrect: 9

Responses Total: 9

Responses - % Correct: 0%

Blog
About Pastest
Contact Us
Help

© Pastest 2016

https://mypastest.pastest.com/Secure/TestMe/Browser/436619[‫ ص‬06:59:12 09/12/1437]


MyPastest

Prefer to use the old MyPastest? Access it here »


Next Question

Back to Filters

Question 8 of 78

A 6-week-old baby is referred for back arching and crying. He


possets after feeds, especially when he lies on his
back. He is thriving. You suspect gastro-oesophageal reflux (GOR).

What is the most appropriate first line course of action?

A Barium swallow

B Trial of Gaviscon

C Trial of domperidone and ranitidine

D Reassure the parents

E A pH study

Explanation
This is a difficult situation and different practitioners may have different views. There is an argument that since
the child is thriving, the GOR is something the child will outgrow and reassurance of the parents should be first
line treatment.

In addition, since it can be difficult not to treat a child, a trial of Gaviscon is simple, has minimal side-effects, and
is a reasonable approach. If there is an improvement in the symptoms further treatment can be initiated as
warranted. An oesophageal pH study is a good investigation for diagnosing severity of GOR, whereas a contrast
upper GI study (or barium meal) is non-physiological and its main value here is in demonstrating a hiatus hernia;
however, if there are good clinical
features investigations may not be needed.
22385

https://mypastest.pastest.com/Secure/TestMe/Browser/436619[‫ ص‬06:59:31 09/12/1437]


MyPastest

Tag Question

Previous Question
Feedback

Difficulty: Average

Peer Responses
End Session

Session Progress

Responses Correct: 0

Responses Incorrect: 10

Responses Total: 10

Responses - % Correct: 0%

Blog
About Pastest
Contact Us
Help

© Pastest 2016

https://mypastest.pastest.com/Secure/TestMe/Browser/436619[‫ ص‬06:59:31 09/12/1437]


MyPastest

Prefer to use the old MyPastest? Access it here »


Previous Question Next Question

Back to Filters

End Session
Question 9 of 78

A 9-month-old infant presents with vomiting and crying. On examination she is afebrile and has a diffusely tender
abdomen. No masses are palpable. Which one of the following diagnoses is
the most important to exclude?

A Gastroenteritis

B Intussusception

C Mesenteric adenitis

D Hirschsprung’s disease

E Colic

Explanation
‘The most important to exclude’: intussusception is the only one that is potentially life threatening. The others are,
of course, possible diagnoses but not immediately life-threatening.
22417

Tag Question

Feedback

Difficulty: Easy

Peer Responses

https://mypastest.pastest.com/Secure/TestMe/Browser/436619[‫ ص‬06:59:50 09/12/1437]


MyPastest

Session Progress

Responses Correct: 0

Responses Incorrect: 11

Responses Total: 11

Responses - % Correct: 0%

Blog
About Pastest
Contact Us
Help

© Pastest 2016

https://mypastest.pastest.com/Secure/TestMe/Browser/436619[‫ ص‬06:59:50 09/12/1437]


MyPastest

Prefer to use the old MyPastest? Access it here »

Back to Filters

Question 10 of 78

Which of the following statements about complementary feeding (weaning) is most factually correct?

A Exclusive
breastfeeding for at least 6 months is associated with a decreased incidence of coeliac
disease in later life compared with the introduction of gluten between 4 and 6 months

B Exclusive breastfeeding of infants until 6 months is associated with a decreased incidence of infection

C The reported percentage of mothers in the UK who exclusively breastfeed their babies for 6 months is
around 15%

D There
is clear evidence that the introduction of certain foods between 4 and 6
months is associated
with an increased prevalence of food allergy

E There are newly emerging concerns that exclusive breastfeeding until 6 months is associated with iron
deficiency anaemia

Explanation
The WHO advice issued in 2001 stated that
all infants should be exclusively breastfed until 6 months; in 2003 this
became UK policy. Recent evidence has highlighted possible concerns
about this advice; studies are under way to
evaluate its potential consequences. It is, however, well documented that as few as 1% of infants in the UK are
exclusively breastfed until 6 months of age.

In the UK Millennium cohort study, the introduction of formula feeds (from exclusive breastfeeding) rather than
solid feeds predicted an increased hospital admission for gastroenteritis and respiratory infections. It was
estimated that 53% of admissions for gastroenteritis and 27% of admissions for chest infection could be prevented
each month by exclusive breastfeeding.

Evidence has always


existed, and continues to grow, that exclusive breastfeeding for 6 months may lead to
increased iron deficiency anaemia and its consequences of long-term motor, mental and social development.

There
is also concern that, although the growth of babies exclusively breastfed for 6 months appears to be

https://mypastest.pastest.com/Secure/TestMe/Browser/436619[‫ ص‬07:00:12 09/12/1437]


MyPastest

adequate (very small numbers), calculations about energy requirements compared with the energy content and
output of breast milk may mean exclusive breastfeeding may not support adequate growth in all children.

Limited
evidencePrevious
exists for aQuestion Next Question
bimodal distribution of increased allergy associated with the introduction of certain
foods. Exposure before 3–4 months as well as delayed exposure after 6 months may increase
the prevalence of
food allergy in a population. Exposure to gluten-containing foods between 4 and 6 months appears to be
protective,
especially when combined with continued breastfeeding, for the development of coeliac disease in later
life.
End Session
There may also be a ‘window’ to introduce bitter tasting foods (such as green vegetables) between 4 and 6 months.
43950

Tag Question

Feedback

Difficulty: Average

Peer Responses

Session Progress

Responses Correct: 0

Responses Incorrect: 12

Responses Total: 12

Responses - % Correct: 0%

Blog
About Pastest
Contact Us
Help

© Pastest 2016

https://mypastest.pastest.com/Secure/TestMe/Browser/436619[‫ ص‬07:00:12 09/12/1437]


MyPastest

https://mypastest.pastest.com/Secure/TestMe/Browser/436619[‫ ص‬07:00:12 09/12/1437]


MyPastest

Prefer to use the old MyPastest? Access it here »

Back to Filters

Question 11 of 78

Which of the following statements with regard to acute abdominal pain in children is most factually correct?

A Abdominal
ultrasonography is rarely helpful in diagnosis but can help to identify
alternative
pathology, including ovarian problems in girls

B Acute appendicitis is most common in the second decade of life

C Acute appendicitis is the most common surgical cause of an acute abdomen in the under-5 age group

D The
classic presentation of central colicky abdominal pain localising to the lower right quadrant occurs
in about 70% of patients

E The rate of perforation of the appendix at diagnosis falls from 80% to 30% between the first and
second decades of life

Explanation
The most common surgical diagnosis in all
children who present to hospital with acute abdominal pain is acute
appendicitis; intussusception is more common in children aged <5 years. In over half of all admissions to hospital
with acute abdominal pain no cause is found. The differential diagnosis is wide; a thorough history and
examination including testicular examination are essential to appropriately investigate and manage these patients.

Acute appendicitis

Classically
there is initial central colicky abdominal pain progressing to localise
pain in the right lower quadrant;
however, less than 60% of children present in this way. Fever (low grade), anorexia, nausea and vomiting are
common, loose stools and urinary symptoms may also occur. WCC is generally raised, ultrasonography being
increasingly utilised to investigate this condition and reporting sensitivity of 84% and specificity of 94% in
children. A pelvic appendix may be missed on ultrasonography and present with atypical symptoms and signs.
Appendicitis is most common in the second decade of life. The rate of perforation is increased by a delay in
diagnosis and approaches 80% in those aged <3 years, and decreases to 10–20% of 10–17 year olds. Management

https://mypastest.pastest.com/Secure/TestMe/Browser/436619[‫ ص‬07:00:32 09/12/1437]


MyPastest

is by appendectomy unless there is an appendix mass, in which case a course of intravenous antibiotics is followed
by an interval elective appendectomy. Laparoscopic techniques are increasingly being employed.
Previous Question
Differential diagnosis includes:
Next Question
Appendicitis
Intussusception
Urinary tract infection
End Session
Mesenteric adenitis
Constipation
Peptic ulceration
Meckel diverticulitis
Pancreatitis
Gastroenteritis
Ovarian pathology, eg torsion/cyst
Primary peritonitis
Henoch–Schönlein purpura
Hernia
Testicular torsion
Cholecystitis
Renal colic
Metabolic, eg acute porphyria
Trauma
Inflammatory bowel disease
Pelvic inflammatory disease
Ectopic pregnancy
Sickle cell crisis
Non-abdominal cause, eg pneumonia, diabetic ketoacidosis

43951

Tag Question

Feedback

Difficulty: Average

Peer Responses

https://mypastest.pastest.com/Secure/TestMe/Browser/436619[‫ ص‬07:00:32 09/12/1437]


MyPastest

Session Progress

Responses Correct: 0

Responses Incorrect: 13

Responses Total: 13

Responses - % Correct: 0%

Blog
About Pastest
Contact Us
Help

© Pastest 2016

https://mypastest.pastest.com/Secure/TestMe/Browser/436619[‫ ص‬07:00:32 09/12/1437]


MyPastest

Prefer to use the old MyPastest? Access it here »

Back to Filters

Question 12 of 78

Which of the following statements with regard to iron deficiency anaemia in children is most factually correct?

A Follow-on milks should be encouraged in all children to prevent iron deficiency anaemia

B Iron-rich foods include egg yolk, red meat, fortified breakfast cereals, and dried fruit, green vegetables,
beans and pulses

C Malabsorption is the major cause for iron deficiency in children in the UK

D The bioavailability of iron in breast milk is low

E The
most likely cause for non-response to oral iron treatment is an undiagnosed pathological aetiology,
eg coeliac disease, blood loss or chronic inflammation

Explanation
WHO definition of iron deficiency anaemia:

Hb <110 g/l in children aged 1–2 years


Hb <112 g/l in children aged 3–5 years
Hb <115 g/l in children aged 5–12 years
Hb <120 g/l in children aged 12–15 years.

The
prevalence in the UK is 8% in those aged under 5 years, increasing considerably in inner city areas and in
Asian children. Although malabsorption and bleeding are causes of iron deficiency anaemia in children, the
overwhelming cause is dietary, specifically the disproportionately high percentage of unmodified cows’ milk in
the
diet and the over-reliance on non-iron-containing convenience foods in the weaning diet and beyond. Iron
stores are laid down in the last trimester of pregnancy and so preterm babies require supplementation. Markers of
iron deficiency include:

https://mypastest.pastest.com/Secure/TestMe/Browser/436619[‫ ص‬07:00:53 09/12/1437]


MyPastest

low MCV (mean corpuscular volume)


low MCHPrevious Question
(mean corpuscular Hb) Next Question
low plasma ferritin (<10 υg/l) – note that raised as part of acute phase reactant.

Consequences:

Pallor, tiredness, irritability End Session


Anorexia
Increased risk of infection
Developmental delay and poor educational achievement (may persist post-treatment)
Dysphagia (oesophageal web)
Pica.

Management:

Breastfeeding – high bio-availability


Iron-fortified formula/follow-on milk (where need to maintain milk intake level).
Encourage iron-rich weaning foods and beyond
Give vitamin C-rich fruit/juices with meals to increase absorption
Avoid whole cows’ milk during the first year of life, and subsequently restrict intake to <750 ml/day.

Treatment/Prophylaxis:

3–6 mg/kg per day divided doses of elemental iron


Hb should rise by 10–20 g/l over 3–4 weeks
Treatment should be continued for 3 months after the anaemia has been corrected
Prophylaxis for neonates is 5 mg elemental iron daily, 6 weeks until weaned
Side effects of iron medication include nausea, epigastric pain, constipation, diarrhoea and black
discoloration of stools
Iron is very dangerous in overdose
Poor compliance is the most likely cause of resistant cases.

43953

Tag Question

Feedback

https://mypastest.pastest.com/Secure/TestMe/Browser/436619[‫ ص‬07:00:53 09/12/1437]


MyPastest

Difficulty: Average

Peer Responses

Session Progress

Responses Correct: 0

Responses Incorrect: 14

Responses Total: 14

Responses - % Correct: 0%

Blog
About Pastest
Contact Us
Help

© Pastest 2016

https://mypastest.pastest.com/Secure/TestMe/Browser/436619[‫ ص‬07:00:53 09/12/1437]


MyPastest

Prefer to use the old MyPastest? Access it here »

Back to Filters

Question 13 of 78

A 20-month-old Asian boy of non-consanguineous parents presents with a history of swelling of the wrists. He
complains of pain in his limbs and since mobilising his parents have become concerned about his ‘bandy legs’. He
was breastfed from birth and had solids introduced into his diet at 6 months of age. He had a green-stick
fracture
of his radius at the age of 12 months secondary to a fall at nursery.

Which of the following diagnoses is most likely in this case?

A Acute lymphoblastic leukaemia

B Juvenile idiopathic arthritis

C Non-accidental injury

D Vitamin D-deficient rickets

E X-linked hypophosphataemic rickets

Explanation
Rickets is a disease of growing bones, (failure to mineralise osteoid). Vitamin D deficiency (diet or ? sunlight) is
the main cause of rickets. Maternal vitamin D deficiency combined with low vitamin D in breast milk may lead to
neonatal rickets.
Other causes include the following:

Nutritional deficiencies of calcium and phosphorous


X-linked and autosomal recessive (AR) hyphophoshataemic rickets, AR vitamin D-dependent rickets (all
rare)
Fat malabsorption (coeliac disease, liver disease – also leads to defective metabolism of vitamin D)
Renal disease (defective metabolism of vitamin D)
Neonatal rickets of prematurity (secondary to inadequate phosphate)

https://mypastest.pastest.com/Secure/TestMe/Browser/436619[‫ ص‬07:01:15 09/12/1437]


MyPastest

Anticonvulsants.

Rickets typically presents at 4–18 months. Vitamin D deficiency during adolescence usually results in
osteomalacia.

Typical radiological features of rickets include poor mineralisation, delayed development of epiphyses,
metaphyseal cupping, fraying and splaying, best seen at the wrists, knees and ankles.

Symptoms and signs include:

Bowing of the legs


Impaired linear growth in infancy
Diffuse bone pain
Kyphoscoliosis
Flattened pelvis (obstructed labour in adult life)
The costochondral junctions become swollen (rickety rosary)
Pectus carinatum and Harrison sulci
Cartilaginous swellings around the wrists and ankles
Frontal bossing
Delayed closure of the anterior fontanelle
Dental enamel hypoplasia and delayed tooth eruption
Hypocalcaemic seizures
Tetany, apnoea and stridor
Left ventricular hypertrophy
Prolonged QTc interval and arrhythmias
Hypotension and heart failure

Blood biochemistry in vitamin D-deficient rickets:

Decreased 25OH-D (25-hydroxyvitamin D, calcidiol) in serum


Raised PTH
Decreased Ca2+
Decreased PO43-
Raised Alkaline phosphatase.

Treatment:

Ergocalciferol (vitamin D2) or cholecalciferol (vitamin D3) for dietary deficiency


Alfacalcidol
(1α-hydroxycholecalciferol) or calcitriol (1,25-dihydroxycholecalciferol) for those with liver
or kidney disease.

https://mypastest.pastest.com/Secure/TestMe/Browser/436619[‫ ص‬07:01:15 09/12/1437]


MyPastest

Sources of vitamin D include fish oil, vegetable oil and skin synthesis during exposure to sunlight.
43955

Previous Question Next Question

Tag Question
End Session
Feedback

Difficulty: Average

Peer Responses

Session Progress

Responses Correct: 0

Responses Incorrect: 15

Responses Total: 15

Responses - % Correct: 0%

Blog
About Pastest
Contact Us
Help

© Pastest 2016

https://mypastest.pastest.com/Secure/TestMe/Browser/436619[‫ ص‬07:01:15 09/12/1437]


MyPastest

Prefer to use the old MyPastest? Access it here »

Back to Filters

Question 14 of 78

A 2-year-old boy presents to the GP with a 3-month history of


recurrent episodes of small amounts of bright red
blood per rectum.

He is well grown and does not have any associated symptoms of abdominal pain, diarrhoea or constipation. There
is no family history of
GI bleeding and he does not appear clinically anaemic. He has no other cutaneous signs on
examination and has normal cardiovascular, respiratory and abdominal examinations in clinic.

What is the most likely diagnosis from the list below?

A Constipation with anal fissure

B Familial adenomatous polyposis coli

C Juvenile polyp

D Juvenile polyposis

E Peutz–Jegher syndrome

Explanation
In this case bright-red blood passing per
rectum is likely to be from the distal bowel and the absence of associated
anal pain makes a diagnosis of constipation with an anal fissure unlikely.

Juvenile polyps

More
than 90% of polyps that present in childhood are juvenile polyps. They are benign harmatomas.
They present at age 2–6 years with painless
blood per rectum; however, 5% prolapse and 10% are
associated with abdominal pain. Most polyps are solitary and located within 30 cm of the
anus. They are
not premalignant.

https://mypastest.pastest.com/Secure/TestMe/Browser/436619[‫ ص‬07:01:37 09/12/1437]


MyPastest

Juvenile polyposis

This
rare Previous Question
condition refers Next Question
to patients with more than five juvenile polyps. Presenting signs include
diarrhoea, rectal bleeding, intussusception, anaemia, prolapse and failure to thrive. This may be sporadic
or familial with an autosomal dominant pattern of inheritance. Polyps can be present throughout the GI
tract. There is considerable malignant potential (17%) and prophylactic colectomy may be advised.

End Session
Peutz–Jegher syndrome

This
condition is inherited in an autosomal dominant pattern. It consists of
hamartomatous polyps that can
occur throughout the GI tract, but most commonly in the small intestine. It is associated with
hyperpigmentation
of the buccal mucosa and lips. Patients without a family history will often present with
recurrent abdominal pain caused by intussusception secondary to small bowel polyps. It is a premalignant
condition with carcinomatous transformation of the polyps; in addition there is an increased risk of
developing pancreatic, ovarian, breast, cervical and testicular tumours.

Familial adenomatous polyposis coli and Gardner syndrome

These
conditions are inherited in an autosomal dominant fashion. Multiple polyps develop (>100), usually
in the second decade. Gastric and duodenal polyps develop in up to 50% and there is an increased risk of
thyroid and liver tumours. Most cases are diagnosed on screening children of affected individuals. Gardner
syndrome is familial adenomatous polyposis plus bony lesions, subcutaneous tumours and cysts.
Both
conditions carry a very high risk of colonic carcinoma (100%) and prophylactic colectomy at the end of
the second decade is advised.

43957

Tag Question

Feedback

Difficulty: Average

Peer Responses

https://mypastest.pastest.com/Secure/TestMe/Browser/436619[‫ ص‬07:01:37 09/12/1437]


MyPastest

Session Progress

Responses Correct: 0

Responses Incorrect: 16

Responses Total: 16

Responses - % Correct: 0%

Blog
About Pastest
Contact Us
Help

© Pastest 2016

https://mypastest.pastest.com/Secure/TestMe/Browser/436619[‫ ص‬07:01:37 09/12/1437]


MyPastest

Prefer to use the old MyPastest? Access it here »

Back to Filters

Question 15 of 78

Which of the following statements about breastfeeding is MOST accurate?

A A diagnosis of PKU is a contraindication to breastfeeding

B Breastfeeding increases IQ equally in both low-birthweight and normal-birthweight infants

C Breastfeeding offers clear benefits in the prevention of lower respiratory tract infections

D Breastfeeding reduces the rate of gastrointestinal infections of infants while it continues

E Maternal HIV, hepatitis C and hepatitis B are contraindications to breastfeeding in developed countries

Explanation
The clearest benefits to the children who
are breastfed are a decreased incidence of gastrointestinal infection and
reduced incidence of otitis media, although these effects do not persist beyond a 2-month period after the cessation
of breastfeeding. There is also clear evidence of a protective effect in the prevention of
NEC in preterm babies.
There is no evidence that breastfeeding prevents
the development of respiratory tract infections but the severity of
the
illness, as measured by the need for hospital admission, appears to be decreased. Additional benefits of
breastfeeding to the child include the
following:

Long-term reduction in blood pressure


Long-term lower cholesterol levels
Decreased incidence of childhood type 1 diabetes
May prevent or at least delay the development of coeliac disease
Increased IQ (more pronounced for low-birthweight children)
Decreased risk of developing childhood ALL (acute lymphoblastic leukaemia)
A decrease in incidence of childhood atopic disease in at-risk children

https://mypastest.pastest.com/Secure/TestMe/Browser/436619[‫ ص‬07:01:59 09/12/1437]


MyPastest

There are a few contraindications to breastfeeding:

Previous Question
Galactosaemia
Next Question
Phenlyketonuria (PKU – breastfeeds may be alternated with phenylalanine-free formulae)
Mothers
with HIV infection in developed countries (the WHO recommends that, when replacement
feeding is acceptable, feasible, affordable, sustainable and safe, avoidance of all breastfeeding by HIV-
infected mothers is recommended; otherwise, exclusive breastfeeding isEnd Sessionduring the first
recommended
months of life)
Note that NOT contraindicated in hepatitis B surface antigen or hepatitis C-positive mothers
Mothers with herpes simplex lesions on their breasts
Mothers who are positive for human T-cell lymphotrophic virus (HTLV) type I and II
Mothers who are receiving therapeutic or diagnostic radioactive isotopes
Maternal intake of any medications should be checked before breastfeeding; alternatives may be available

43959

Tag Question

Feedback

Difficulty: Average

Peer Responses

Session Progress

Responses Correct: 0

Responses Incorrect: 17

Responses Total: 17

Responses - % Correct: 0%

https://mypastest.pastest.com/Secure/TestMe/Browser/436619[‫ ص‬07:01:59 09/12/1437]


MyPastest

Blog
About Pastest
Contact Us
Help

© Pastest 2016

https://mypastest.pastest.com/Secure/TestMe/Browser/436619[‫ ص‬07:01:59 09/12/1437]


MyPastest

Prefer to use the old MyPastest? Access it here »

  Logged in as Ahmed fouad

Your

Back to Filters
Question 16 of 78

 My

 Sho

Which of the following statements is the most factually correct with regard to the administration of  Blo
Difficulty: Average
vitamin K to babies in the neonatal
period?
✉ Co
Peer Responses

A If an oral regimen is undertaken, a single dose of oral vitamin K is recommended in all  Mo
formula-fed babies
 He

B Significant
bleeding in the neonatal period normally occurs in children with significant
risk factors for vitamin K deficiency bleeding
Logg

C The vast majority of vitamin K deficiency-associated bleeding occurs within the first  Lo
week of life

Session Progress
D There is good evidence to support an increased risk of childhood leukaemia in babies
given vitamin K in the neonatal period Responses Correct: 0

Responses Incorrect: 18

E Vitamin
K deficiency bleeding is a potentially fatal disorder that is virtually
preventable Responses Total: 18
with one dose of vitamin K given intramuscularly at birth
Responses - % Correct: 0%

Explanation

The Department of Health issued guidance on the administration of vitamin K in newborn infants in
1998. In short:

All
newborn babies should receive vitamin K to prevent the rare but serious
and sometimes
fatal disorder of vitamin K deficiency bleeding (VKDB).
The available data do not support an increased risk of cancer including leukaemia caused by
vitamin K.
Intramuscular vitamin K effectively prevents VKDB in virtually all babies after a single
dose given at birth.
Oral
regimens require repeated (more than two) doses in breastfed babies*; clinicians must
ensure that all recommended oral doses are given at the appropriate ages.
A selective policy for high- or low-risk infants is not feasible because bleeding occurs
unpredictably.
Babies
at high risk of early (24 h) or classic (2–7 days) VKDB include: preterm, or failing to
take or absorb feeds, or had a complicated delivery or are ill or, if their mothers have been
receiving medication associated with a higher risk (eg anticonvulsant drugs).
Babies
at high risk of late (>7 days) VKDB: babies with liver disease (prolonged jaundice
with pale stools and/or dark urine), or babies who have bleeding or spontaneous bruising in
early infancy or who are ill from other causes.
Of the babies who do not receive vitamin K, 1
in 10 000 will develop VKDB; 50% of these
are affected late, and 50% of
those affected will have intracranial bleeding, 20% of those
will die and most survivors will have long-term neurological damage.
Many
who have severe late VKDB have minor warning bleeds in the preceding few days. In
about a third of instances the bleeding occurs without evident cause or risk factor.

*Vitamin K is added to formula milk.

http://webarchive.nationalarchives.gov.uk/+/www.dh.gov.uk/en/Publicationsandstatistics/Lettersandcirculars/Professionalletters/Chiefmedicalofficerletters/DH_4004993
43960

https://mypastest.pastest.com/Secure/TestMe/Browser/436619[‫ ص‬07:02:20 09/12/1437]


MyPastest

Previous Question Next Question


Tag Question Feedback

Blog
About Pastest
Contact Us
Help
© Pastest 2016
End Session

https://mypastest.pastest.com/Secure/TestMe/Browser/436619[‫ ص‬07:02:20 09/12/1437]


MyPastest

Prefer to use the old MyPastest? Access it here »

Back to Filters

Question 17 of 78

Which of the following statements with regard to testing for carbohydrate malabsorption is most factually correct?

A Alkaline stool pH is indicative of carbohydrate intolerance

B An
early peak of hydrogen in a hydrogen breath test, followed by a secondary peak, may be indicative
of small bowel bacterial overgrowth

C Non-resolution of symptoms on removal of suspected carbohydrate from the diet excludes the
diagnosis of carbohydrate intolerance

D Reducing substances of 0.05% in the stool are indicative of carbohydrate intolerance

E Resolution
of symptoms on removal of the suspected carbohydrate from the diet is conclusive
evidence of the presence of lactose intolerance

Explanation

Samples of liquid stool will show undigested sugars on chromatography


Reducing
substances in the stool of 0.25–0.5% are suggestive of carbohydrate malabsorption; >0.75% is
indicative of the condition. The liquid portion of the stool must be tested
Acidic stool pH <5.5 is indicative of carbohydrate malabsorption even in the absence of reducing
substances
Analysis
of small-bowel mucosal disaccharidases in a biopsy obtained endoscopically is the ‘gold
standard’, but is rarely needed.
Endoscopy may reveal small intestinal mucosal injury resulting in
secondary carbohydrate malabsorption
Clinical response to treatment with withdrawal of suspected intolerant substance leading to resolution of
symptoms and reintroduction leading to relapse is suggestive of diagnosis. However, non-resolution of

https://mypastest.pastest.com/Secure/TestMe/Browser/436619[‫ ص‬07:02:44 09/12/1437]


MyPastest

diarrhoea may not


necessarily exclude the diagnosis because diarrhoea itself may produce a
secondary
intolerance of a different substance, so the sensitivity of this as a diagnostic test can be poor. In addition
Previous Question Next Question
removal of a dietary substance may improve symptoms but not confirm a diagnosis, eg the removal of
cows’ milk may lead to the cessation of diarrhoea but this may be due to a diagnosis of lactose intolerance
or cows’ milk protein intolerance
The hydrogen breath test: the principle is that malabsorbed carbohydrate will pass into the colon where it
End Session
is metabolised by bacteria and hydrogen is released. The gas is then absorbed and released in the breath. If
there is a peak it suggests carbohydrate malabsorption. An early peak raises the possibility of bacterial
overgrowth in the small intestine, in which case a secondary peak may occur representing large bowel
fermentation. Lactulose, which is a non-absorbable carbohydrate, can be given to ensure that the colonic
flora can metabolise carbohydrate and to assess transit time. A false-negative test is possible

43962

Tag Question

Feedback

Difficulty: Average

Peer Responses

Session Progress

Responses Correct: 0

Responses Incorrect: 19

Responses Total: 19

Responses - % Correct: 0%

https://mypastest.pastest.com/Secure/TestMe/Browser/436619[‫ ص‬07:02:44 09/12/1437]


MyPastest

Blog
About Pastest
Contact Us
Help

© Pastest 2016

https://mypastest.pastest.com/Secure/TestMe/Browser/436619[‫ ص‬07:02:44 09/12/1437]


MyPastest

Prefer to use the old MyPastest? Access it here »

Back to Filters

Question 18 of 78

Which of the following statements about preterm and term formulae and breast milk is most factually correct?

A Babies
fed on fortified breast milk have a decreased incidence of late-onset sepsis and necrotising
enterocolitis when compared with babies fed on preterm formulae

B Preterm babies fed with fortified breast milk show similar weight gain profiles to preterm babies fed
on preterm formula milks

C Preterm
formula contains more fat, protein and electrolytes but the same amount
of carbohydrate as
term formula or unfortified breast milk

D Preterm formula has a lower sodium content than term formula

E Preterm formula has the same protein content as term formula

Explanation
The principal differences are that preterm formula contains more electrolytes, calories and minerals than term
formula. All of the following are higher in the preterm formulae: energy, protein, carbohydrate, fat, osmolality,
sodium, potassium, calcium, magnesium, phosphate and iron.

Breast milk fortifier is a powder designed to add to breast milk for preterm or
low-birthweight babies to provide
extra energy, protein, vitamins and minerals to support growth in this population while preserving the non-
nutritional advantages of breast milk, such as its immunological function, enzymes and growth factors. It is based
on hydrolysed cows’ milk protein. Fortified human milk may not produce as much weight gain as preterm
formulae but it is associated with a decreased incidence of NEC and late-onset sepsis.

In 100 ml Breast milk (approximately) Term Formula Preterm Formula

Energy (kcal) 60-75 67 80

Protein (g) 1.3 1.4 2.5

https://mypastest.pastest.com/Secure/TestMe/Browser/436619[‫ ص‬07:03:14 09/12/1437]


MyPastest

Carbohydrate (g) 7 7.5 7.8

Fat (g)
Previous Question
4.2
Next3.5Question4.4
Na+ (mmol) 0.65 0.7 1.7

K+ (mmol) 1.54 1.6 2.0

Ca2+ (mmol) 0.88 1.0 2.5 End Session


PO43- (mmol) 0.48 0.6 1.6

Iron (mg) 0.08 0.5 1.4

43963

Tag Question

Feedback

Difficulty: Average

Peer Responses

Session Progress

Responses Correct: 0

Responses Incorrect: 20

Responses Total: 20

Responses - % Correct: 0%

https://mypastest.pastest.com/Secure/TestMe/Browser/436619[‫ ص‬07:03:14 09/12/1437]


MyPastest

Blog
About Pastest
Contact Us
Help

© Pastest 2016

https://mypastest.pastest.com/Secure/TestMe/Browser/436619[‫ ص‬07:03:14 09/12/1437]


MyPastest

Prefer to use the old MyPastest? Access it here »

Back to Filters

Question 19 of 78

Which of the following statements with regard to infant formula and breast milk is most factually correct?

A The fat content of breast milk varies throughout the day and throughout a feed

B The iron content of first formula milks closely resembles that of breast milk

C The protein and carbohydrate components of colostrum are higher than those of mature breast milk

D The renal solute load of a casein-based formula is similar to that of a whey-based formula

E There is good evidence to show that casein-predominant formulas are more satisfying for hungrier
babies

Explanation
The composition of human milk is not static. The protein content is higher and the carbohydrate content lower
in
colostrum than in mature milk. Fat contents fluctuate with diurnal variations and within a feed, increasing towards
the end of each feed. Breast milk has whey:casein ratio (70:30), protein of high biological value, long-chain fatty
acids that may improve vision and cognition, and
a low renal solute load. In addition to its nutrients, it also
contains
immunoglobulins, antimicrobial and anti-inflammatory agents, hormones, growth factors and enzymes;
these factors contribute to the immune protection, digestion, and both intestinal and general growth and
development.

The composition of formula feeds in


contrast is closely regulated in terms of the acceptable range of nutritional
components. The two main types available differ in their protein composition with whey:casein ratios of 60:40 and
20:80 in the whey- and casein-dominant formulae respectively. The whey-based formulae
most closely resemble
breast milk and, although evidence is lacking, the casein-based formulae are marketed as more satisfying for the
hungry
baby because the casein forms curds that are more slowly digested. The casein-based formulae have a
higher renal solute load than the whey-based milks and, although suitable from birth, extra care should be
taken in
making them up correctly.

https://mypastest.pastest.com/Secure/TestMe/Browser/436619[‫ ص‬07:03:38 09/12/1437]


MyPastest

In 100 ml Breast milk (approximately) Term formula


Previous Question Next Question
Energy (kcal) 60-75 67

Protein (g) 1.3 1.4

Carbohydrate (g) 7 7.5


End Session
Fat (g) 4.2 3.5

Na+ (mmol) 0.65 0.7

K+ (mmol) 1.54 1.6

Ca2+ (mmol) 0.88 1.0

PO43- (mmol) 0.48 0.6

Iron (mg) 0.08 0.5

43964

Tag Question

Feedback

Difficulty: Average

Peer Responses

Session Progress

Responses Correct: 0

https://mypastest.pastest.com/Secure/TestMe/Browser/436619[‫ ص‬07:03:38 09/12/1437]


MyPastest

Responses Incorrect: 21

Responses Total: 21

Responses - % Correct: 0%

Blog
About Pastest
Contact Us
Help

© Pastest 2016

https://mypastest.pastest.com/Secure/TestMe/Browser/436619[‫ ص‬07:03:38 09/12/1437]


MyPastest

Prefer to use the old MyPastest? Access it here »

Back to Filters

Question 20 of 78

Which of the following statements concerning carbohydrate intolerance in childhood is most factually correct?

A Acquired post-enteritis lactose intolerance is usually transient

B Carbohydrate intolerance is usually inherited in an autosomal dominant fashion

C Symptoms of congenital carbohydrate intolerance always occur in the neonatal period

D Symptoms of inherited carbohydrate intolerance are most common in the first 4 years of life

E Symptoms of lactose intolerance often require total exclusion of all dairy products throughout life

Explanation
Carbohydrates make up at least half of the energy intake in the diet. The main carbohydrates are the storage
polysaccharides (starch, glycogen and cellulose), the disaccharides, lactose and sucrose, and the monosaccharides,
glucose and fructose.

Disorders of disaccharide absorption include the following:

Primary:

Congenital alactasia (exceptionally rare with neonatal presentation)


Sucrose–isomaltase
deficiency (even rarer than lactase deficiency): symptoms when fruit (sucrose) added
to the diet, will not tolerate glucose feeds

Secondary:

Post-enteritis (rotavirus): usually resolves after a short period – days to 2 weeks


Neonatal surgery, malnutrition

https://mypastest.pastest.com/Secure/TestMe/Browser/436619[‫ ص‬07:03:58 09/12/1437]


MyPastest

Untreated coeliac disease


Late-onset
lactose intolerance: in many populations lactase activity declines after the first few years of life.
Previous Question Next Question
Symptoms are rare before age 6 years.
Ten per cent of northern Europeans but >80% of Africans, Asians,
Inuit and American Indians. Often dose-dependent tolerance with fermented dairy products being less
problematic. Thought to be autosomal
recessive

Disorders of monosaccharide absorption End Session


Primary:
glucose–galactose malabsorption (very rare autosomal recessive) – intolerance of feeds with
watery diarrhoea from birth, stops with cessation of feeds. Will tolerate fructose feeds.
Secondary:– post-enteritis – neonatal surgery – malnutrition

Carbohydrate
intolerance is usually lactose intolerance and is usually acquired. The
deficiency is the brush border
enzyme lactase which hydrolyses lactose into glucose and galactose. The intolerance will present with
characteristic loose explosive stools. The diagnosis is made by looking for reducing substances in the stools after
carbohydrate ingestion; detection of >0.5% is significant. Treatment is with lactose-free formula in infancy and a
reduced lactose intake in later childhood. After gastroenteritis, carbohydrate intolerance can be to either
disaccharides or monosaccharides. Both types of intolerance are usually transient and both respond to removal of
the offending carbohydrate. Both result in positive reducing substances in the stools.
43966

Tag Question

Feedback

Difficulty: Average

Peer Responses

Session Progress

Responses Correct: 0

https://mypastest.pastest.com/Secure/TestMe/Browser/436619[‫ ص‬07:03:58 09/12/1437]


MyPastest

Responses Incorrect: 22

Responses Total: 22

Responses - % Correct: 0%

Blog
About Pastest
Contact Us
Help

© Pastest 2016

https://mypastest.pastest.com/Secure/TestMe/Browser/436619[‫ ص‬07:03:58 09/12/1437]


MyPastest

Prefer to use the old MyPastest? Access it here »

Back to Filters

Question 21 of 78

Which of the following statements with regard to cows’ milk, breast (human) milk and follow-on formulas is the
most
factually correct?

A Calogen contains 250 kcal/100 g

B Duocal contains carbohydrate, protein and fat

C Maxijul is a glucose polymer

D Polycal contains MCT fats

E Scandishakes powder is a cows’ milk, protein-free, calorie supplement

Explanation
Nutritional supplements are indicated for
use in patients with disease-related malnutrition, malabsorption states
or
any other conditions requiring fortification with a high or readily available calorie supplement. These include
glucose polymer-based supplements:

Maxijul super 380 kcal/100 g powder


Maxijul liquid 200 kcal/100 ml
Vitajoule powder 380 kcal/100 g

Also supplements based on maltodextrin:

Caloreen powder 390 kcal/100 g


Polycal powder 384 kcal/100 g
Polycal liquid 247 kcal/100 ml

https://mypastest.pastest.com/Secure/TestMe/Browser/436619[‫ ص‬07:04:18 09/12/1437]


MyPastest

Nutritional supplements based on fat have generally more calories per ml than their carbohydrate equivalents and
include:
Previous Question Next Question
Calogen liquid 450 kcal/100 ml
Liquigen liquid 450 kcal/100 ml

Nutritional supplements based on fat and carbohydrate includes: End Session


Duocal preparations (calorific values vary with preparations)

Nutritional supplements based on protein, fat and carbohydrate includes:

Scandishake 500 kcal/100 g – contains cows’ milk protein and lactose

43969

Tag Question

Feedback

Difficulty: Average

Peer Responses

Session Progress

Responses Correct: 0

Responses Incorrect: 23

Responses Total: 23

Responses - % Correct: 0%

https://mypastest.pastest.com/Secure/TestMe/Browser/436619[‫ ص‬07:04:18 09/12/1437]


MyPastest

Blog
About Pastest
Contact Us
Help

© Pastest 2016

https://mypastest.pastest.com/Secure/TestMe/Browser/436619[‫ ص‬07:04:18 09/12/1437]


MyPastest

Prefer to use the old MyPastest? Access it here »

Back to Filters

Question 22 of 78

Which of the following statements relating to Wilson disease is most factually correct?

A A diet low in copper should be low in shellfish, liver and chocolate

B A presentation with psychiatric or neurological symptoms is rare

C The majority of patients with Wilson disease present with liver and renal impairment in late
adolescence and early childhood

D Wilson disease has a poor outcome even if diagnosed early and treated appropriately

E Wilson disease is sporadically inherited

Explanation
Wilson disease is an autosomal recessive disorder of copper metabolism. The gene has been identified on
chromosome 13 and dysfunction of this gene decreases both copper excretion into bile and ceruloplasmin
production. Ceruloplasmin is the plasma protein that transports copper. Wilson disease presents in 40% with
features of liver disease usually between the ages of 3 and 12 years. Approximately 50% have a psychiatric or
neurological presentation, usually in adolescence or early adult life; 50% of these will have clinically detectable
liver disease. The remainder present with skeletal, renal or haemolytic disease. The effects of the disease include
the following:

Liver: chronic active hepatitis, portal hypertension and fulminant hepatic failure
Brain: progressive lenticular degradation due to copper deposition, often leads to tremor
Cornea: Kayser–Fleischer rings
Lens: sunflower cataract
Kidney: renal tubular disorders
Blood: haemolysis

https://mypastest.pastest.com/Secure/TestMe/Browser/436619[‫ ص‬07:04:39 09/12/1437]


MyPastest

Diagnosis
can be difficult. The important part is to include it in the differential of all undiagnosed liver disease in
Previous
childhood. Biochemical Question
findings include: Next Question
low plasma ceruloplasmin level <200 mg/L
Raised urinary copper >25 μmol/24 h after penicillamine (the penicillamine challenge)
Baseline urinary copper is unreliable with poor sensitivity and specificity (>5 μmol/24 h is suggestive)
Raised hepatic copper on liver biopsy
End Session
Note that serum copper levels are not helpful

If
untreated the disease is fatal, usually by the age of 30 years. If identified early and treated, the prognosis is
good. Treatment is with oral penicillamine as a copper-binding agent plus a low-copper diet (avoid excessive
chocolate, shellfish and liver). Patients on penicillamine require vitamin B6 (pyridoxine) supplementation
to avoid
neuropathy. Liver transplantation may be indicated if presentation is with fulminant liver failure. It is important to
screen siblings of patients with Wilson disease to start treatment early and improve prognosis.
43970

Tag Question

Feedback

Difficulty: Average

Peer Responses

Session Progress

Responses Correct: 0

Responses Incorrect: 24

Responses Total: 24

Responses - % Correct: 0%

https://mypastest.pastest.com/Secure/TestMe/Browser/436619[‫ ص‬07:04:39 09/12/1437]


MyPastest

Blog
About Pastest
Contact Us
Help

© Pastest 2016

https://mypastest.pastest.com/Secure/TestMe/Browser/436619[‫ ص‬07:04:39 09/12/1437]


MyPastest

Prefer to use the old MyPastest? Access it here »

Back to Filters

Question 23 of 78

Which of the following statements with regard to neonatal hyperbilirubinaemia is most factually correct?

A Babies
who become jaundiced whilst breastfeeding are significantly at risk of dehydration and require
supplementation with formula milk

B Babies who have rhesus incompatibility and haemolysis will require exchange transfusion

C Clinical examination of neonates is an acceptable way to quantify the extent of neonatal jaundice and
guide treatment

D Neonates
who develop jaundice within the first 24 h need urgent medical review to rule out underlying
potentially severe aetiology for their jaundice

E Unconjugated hyperbilirubinaemia can present with dark urine, due to high levels of bilirubin in the
urine

Explanation
Jaundice can be either conjugated (direct) or unconjugated (indirect). Unconjugated is fat soluble and does not
spill over into the urine. Up to 60% of term and 80% of preterm
newborns become clinically jaundiced in the first
week of life; approximately 10% of breastfed babies will still be jaundiced at 1 month
of age. Most neonatal
jaundice is harmless but some children will develop very high levels of unconjugated bilirubin, which can cross
the blood–brain barrier and cause long-term neurological damage. Clinical assessment of the level of bilirubin is
difficult and so, if jaundice is detected, a serum bilirubin should be checked and managed according to threshold
charts based on gestational age and treatment guidance issued by the National Institute for Health and Clinical
Excellence (NICE). Additional care should be taken if the child develops
jaundice within the first 24 hours of life.
Most cases can be managed with phototherapy and supportive measures; however, some children with very high
levels of bilirubin that persist despite treatment or any baby
with clinical signs of acute bilirubin encephalopathy
may require an exchange transfusion. Always identify and treat any underlying cause of hyperbilirubinaemia as
appropriate.

https://mypastest.pastest.com/Secure/TestMe/Browser/436619[‫ ص‬07:05:02 09/12/1437]


MyPastest

Causes of unconjugated hyperbilirubinaemia in the neonatal period include:

IncreasedPrevious
production ofQuestion Next Question
unconjugated bilirubin from haem
Haemolysis:
– rhesus incompatibility – ABO incompatibility – hereditary spherocytosis – glucose-6-
phosphate dehydrogenase deficiency – pyruvate kinase deficiency
Polycythaemia/bruising
Sepsis congenital or acquired
End Session
Decreased
bilirubin uptake or metabolism: – physiological – sepsis,
acidosis and hypoxia – Gilbert
syndrome (7% of population mild) – Crigler–Najjar syndrome (can be severe – rare) – breast milk jaundice
– hypothyroidism
Altered
enterohepatic circulation: – breast milk jaundice – antibiotic administration – intestinal
obstruction: – ileal atresia – Hirschsprung disease – cystic fibrosis including meconium ileus – pyloric
stenosis.

43972

Tag Question

Feedback

Difficulty: Average

Peer Responses

Session Progress

Responses Correct: 0

Responses Incorrect: 25

Responses Total: 25

Responses - % Correct: 0%

https://mypastest.pastest.com/Secure/TestMe/Browser/436619[‫ ص‬07:05:02 09/12/1437]


MyPastest

Blog
About Pastest
Contact Us
Help

© Pastest 2016

https://mypastest.pastest.com/Secure/TestMe/Browser/436619[‫ ص‬07:05:02 09/12/1437]


MyPastest

Prefer to use the old MyPastest? Access it here »

Back to Filters

Question 24 of 78

Which of the following statements with regard to coeliac disease is the most factually correct?

A According to NICE guidelines type 1 diabetes mellitus is an indication for screening for coeliac
disease

B A family history of coeliac disease is present in 40% of people diagnosed with coeliac disease

C Positive serology (IgA TTG, or IgG TTG if IgA deficient) is sufficient to make a diagnosis of coeliac
disease

D The
effects of gluten on serology and small bowel mucosa are long lived so it is acceptable to advise
patients with possible coeliac disease to exclude gluten whilst awaiting referral for definitive diagnosis
and management

E The
majority of patients with coeliac disease present with irritability, pallor and malabsorption
(diarrhoea and failure to thrive) in infancy after the introduction of gluten into the diet

Explanation
This is a reversible immune-mediated enteropathy of the small intestinal mucosa that occurs on exposure to
ingested gluten. Its prevalence in the UK is between 0.3% and 1.9%. Patients can present at any time throughout
life with a wide range of symptoms; more patients are diagnosed in adulthood than childhood. The gold standard
of
diagnosis is positive serology (IgA tissue transglutaminase [TTG], or IgG TTG if IgA deficient), characteristic
histology (subtotal villous atrophy) on small bowel biopsy and resolution of symptoms when gluten is
removed.
Of note, if the patient is taking insufficient gluten before serology and biopsy, both may be within the normal
range and so the diagnosis must be confirmed on gluten (>10–15 g/day, for 6 weeks to 3 months). Risk factors for
developing coeliac disease include other autoimmune diseases (eg type 1 diabetes mellitus), first-degree relative
with coeliac disease (10%) and a diagnosis of IgA deficiency.

GI symptoms and signs include:

https://mypastest.pastest.com/Secure/TestMe/Browser/436619[‫ ص‬07:05:22 09/12/1437]


MyPastest

Persistent or intermittent diarrhoea


Abdominal pain
Vomiting
Previous Question Next Question
Constipation
Abdominal bloating/distension
Anorexia
Flatulence End Session

Non-GI symptoms and signs include:

Recurrent aphthous stomatitis


Faltering growth
Weight loss
Short stature
Delayed menarche/puberty
Amenorrhoea
Dental enamel defects
Prolonged fatigue/lethargy
Iron deficiency anaemia
Rickets/osteomalacia
Osteoporosis/pathological fracture
Dermatitis herpetiformis
Irritability/depression

Treatment
is lifelong adherence to a gluten-free diet, exclusion of wheat, rye, barley and oats (due to
contamination with gluten during milling and manufacture). Consequences of non-adherence to diet include
recurrence of symptoms, poor growth, delayed puberty, dental enamel problems, osteoporosis, and small increased
risk of malignancy, eg small bowel lymphoma.

The NICE guidelines advocate targeted screening via serological testing of children with symptoms of, or risk
factors for, coeliac disease.

https://www.nice.org.uk/guidance/ng20 43973

Tag Question

Feedback

Difficulty: Average

https://mypastest.pastest.com/Secure/TestMe/Browser/436619[‫ ص‬07:05:22 09/12/1437]


MyPastest

Peer Responses

Session Progress

Responses Correct: 0

Responses Incorrect: 26

Responses Total: 26

Responses - % Correct: 0%

Blog
About Pastest
Contact Us
Help

© Pastest 2016

https://mypastest.pastest.com/Secure/TestMe/Browser/436619[‫ ص‬07:05:22 09/12/1437]


MyPastest

Prefer to use the old MyPastest? Access it here »

Back to Filters

Question 25 of 78

Which of the statements about hepatitis B is the most factually correct?

A Anti-HBs antibodies suggest that a chronic carrier state has developed after acute infection

B Hepatitis B is a DNA virus

C Interferon-γ is a recognised treatment of the chronic carrier state

D Transmission of hepatitis B is faeco-oral

E Treatment of acute hepatitis B infection is by passive immunisation

Explanation
Hepatitis B is a DNA virus. Diagnosis is by detection of the hepatitis B surface antigen (HBsAg). Hepatitis B ‘e’
antigen (HBeAg)-positive patients carry a larger virus load and are more infectious. Acute and ongoing chronic
infection is associated with anti-hepatitis B core (HBc) IgM. Anti-HBe and anti-HBs antibodies appear as an
effective immune response develops. All HBsAg-positive individuals are infective. The route of transmission is
either percutaneous (puncture through the skin) or mucosal (exposure to infectious blood or bodily fluids). The
perinatal transmission rate depends on the maternal serology. If the mother is HBsAg positive and HBeAg
negative, the risk is 12–25%. If the mother is both HBsAg and HBeAg positive the risk is 90%. The younger the
age at infection the
less the likelihood of symptomatic liver disease but the greater the risk of prolonged viral
carriage. Ninety per cent of infants infected in
the first year of life become chronic carriers.

Clinically
the disease is often asymptomatic but an acute hepatic picture can develop. Acute liver failure occurs in
less than 1%. The risk of fulminant hepatitis is increased by coinfection with hepatitis D. In those with a typical
hepatic picture the chronic carrier rate is low. Chronicity results in an increased risk of cirrhosis and hepatocellular
carcinoma. Boys are more likely to become chronic carriers than girls. Chronically infected children have a 25%
lifetime risk of cirrhosis or hepatocellular carcinoma. Prevention is by both active and passive immunisation.
Interferon-α is a recognised treatment of chronic infection.

https://mypastest.pastest.com/Secure/TestMe/Browser/436619[‫ ص‬07:05:49 09/12/1437]


MyPastest

Hepatitis
D is an RNA virus that requires the HBsAg for its assembly and virulence. It is transmitted similarly to
hepatitis B. The severity of the liver damage increases if there is coexistent hepatitis B infection.
Diagnosis is by
Previous
serology. Prevention Question
of vertical Next Question
transmission of hepatitis B is with immunisation of all neonates born to hepatitis
B-infected mothers with the addition of HBIg for all babies <1500 g, those whose mothers had acute hepatitis B
infection in pregnancy, those whose mothers are HBsAg positive plus either HBeAg positive or anti-HBe
negative.
43974

End Session

Tag Question

Feedback

Difficulty: Average

Peer Responses

Session Progress

Responses Correct: 0

Responses Incorrect: 27

Responses Total: 27

Responses - % Correct: 0%

Blog
About Pastest
Contact Us
Help

© Pastest 2016

https://mypastest.pastest.com/Secure/TestMe/Browser/436619[‫ ص‬07:05:49 09/12/1437]


MyPastest

Prefer to use the old MyPastest? Access it here »

Back to Filters

Question 26 of 78

Which of the following statements concerning hepatitis A is the most factually correct?

A Chronic liver disease commonly follows acute infection

B Diagnosis of acute infection is by stool culture

C Hepatitis A is an RNA virus

D Passive immunisation produces lifelong immunity to infection

E Transmission is most commonly by exposure to contaminated blood at delivery

Explanation
Hepatitis A is an RNA virus. Diagnosis is
by detection of the hepatitis A virus IgM. The route of transmission is
faeco-oral. There is no carrier state and fulminant hepatic failure is rare (<0.1%). Liver function, however, may be
abnormal for up to 1 year. Prevention is by either passive or active immunisation. Passive immunisation is with
immunoglobulin and lasts for 3–6 months. Active immunisation is with a live attenuated virus, with booster
immunisation being required after 12–18 months. Clinical symptoms are initially non-specific and include
anorexia, nausea, fatigue and fever associated with epigastric pain and tender hepatomegaly. The icteric phase
then develops with jaundice, pale stools and dark urine. Sometimes there is pruritus, depression and persistent
jaundice, with raised transaminases for a prolonged period. The prothrombin time (PTT) should be monitored. A
raised PTT raises the possibility of severe hepatic necrosis or decompensation of underlying liver disease.

Hepatitis
E is an RNA virus. Epidemics occur in developing countries. In the UK infection is usually seen in
travellers from endemic areas. The route of
transmission is faeco-oral. The clinical course of hepatitis E infection
is similar to hepatitis A. Complete recovery from acute infection occurs. Chronic infection has not been described,
although acute fulminant hepatic failure can occur and is more common during pregnancy. Diagnosis is by
serology. No vaccine or prophylactic treatment is available.

https://mypastest.pastest.com/Secure/TestMe/Browser/436619[‫ ص‬07:06:34 09/12/1437]


MyPastest

43975

Previous Question Next Question

Tag Question

Feedback End Session

Difficulty: Average

Peer Responses

Session Progress

Responses Correct: 0

Responses Incorrect: 28

Responses Total: 28

Responses - % Correct: 0%

Blog
About Pastest
Contact Us
Help

© Pastest 2016

https://mypastest.pastest.com/Secure/TestMe/Browser/436619[‫ ص‬07:06:34 09/12/1437]


MyPastest

Prefer to use the old MyPastest? Access it here »

Back to Filters

Question 27 of 78

Which of the following statements about hepatitis C is the most factually correct?

A Caesarean section is advised in all cases of hepatitis C in the developed world to reduce vertical
transmission

B Maternal infection with hepatitis C is a contraindication to breastfeeding

C Post-delivery immunisation is advised in all cases to decrease vertical transmission

D Transmission worldwide is most commonly by the vertical route

E Treatment with ribavirin as a monotherapy is currently recommended in the UK for children with
moderate-to-severe liver disease

Explanation
Hepatitis C is an RNA virus. The route of
transmission is most commonly vertical, otherwise infection occurs via
parenteral, sexual or contaminated blood product transfusion. The vertical transmission rate is 9%, higher in HIV-
positive mothers. Mode of delivery does not affect the risk of transmission unless coinfection with HIV is present,
in which case caesarean section in the developed world may offer some protection.

Diagnosis is usually by serology with the detection of the anti-HCV antibody. Blood and blood products for
transfusion have been screened for hepatitis C virus since 1990. Infection is usually asymptomatic or an acute
hepatitis can occur. Fulminant hepatitis is uncommon but can occur. HCV RNA detection establishes the presence
of viraemia, confirming infection
and infectivity. Persistence of HCV RNA indicates continuing infection.
Chronic infection is common (prevalence 0.2–0.7% in northern Europe, 1–2% in southern Europe and Japan),
with the development of cirrhosis and hepatocellular carcinoma in a number of cases after an interval of 10–15
years. Treatment with pegylated interferon plus ribavirin is the currently recommended treatment for children.
Duration of treatment varies depending on the genotype of the infecting virus. Good prognostic factors for
antiviral treatment are the absence of cirrhosis, young age at acquisition, and absence or coinfection with HIV
and/or hepatitis B. Breastfeeding is not contraindicated for HCV seropositive mothers but, if nipples are cracked

https://mypastest.pastest.com/Secure/TestMe/Browser/436619[‫ ص‬07:06:55 09/12/1437]


MyPastest

and bleeding, caution is advised.


43976

Previous Question Next Question

Tag Question
End Session
Feedback

Difficulty: Average

Peer Responses

Session Progress

Responses Correct: 0

Responses Incorrect: 29

Responses Total: 29

Responses - % Correct: 0%

Blog
About Pastest
Contact Us
Help

© Pastest 2016

https://mypastest.pastest.com/Secure/TestMe/Browser/436619[‫ ص‬07:06:55 09/12/1437]


MyPastest

Prefer to use the old MyPastest? Access it here »

Back to Filters

Question 28 of 78

From the following list of infective agents and disease processes, which is most likely to cause the clinical picture
of
colitis with abdominal pain and bloody diarrhoea?

A Coeliac disease

B Giardia lamblia

C Non-typhoid Salmonella sp

D Peutz–Jegher syndrome

E Rotavirus infection

Explanation
Abdominal pain associated with bloody diarrhoea is indicative of colitis. Colitis can be infective or non-infective.

Causes of infective colitis:

Salmonella spp.
Shigella spp.
Campylobacter pylori
Escherichia coli O157 (and other E. coli)
Clostridium difficile (pseudomembranous colitis)
Yersinia spp.
Tuberculosis
Cytomegalovirus
Entamoeba histolytica
Enterobius vermicularis.

https://mypastest.pastest.com/Secure/TestMe/Browser/436619[‫ ص‬07:07:17 09/12/1437]


MyPastest

Causes of non-infective colitis:


Previous Question Next Question
Ulcerative colitis
Crohn disease
Necrotising enterocolitis
Vasculitic causes, eg Henoch–Schönlein purpura End Session
Hirschsprung enterocolitis
Microscopic colitis
Allergic enterocolitis.

43977

Tag Question

Feedback

Difficulty: Average

Peer Responses

Session Progress

Responses Correct: 0

Responses Incorrect: 30

Responses Total: 30

Responses - % Correct: 0%

https://mypastest.pastest.com/Secure/TestMe/Browser/436619[‫ ص‬07:07:17 09/12/1437]


MyPastest

Blog
About Pastest
Contact Us
Help

© Pastest 2016

https://mypastest.pastest.com/Secure/TestMe/Browser/436619[‫ ص‬07:07:17 09/12/1437]


MyPastest

Prefer to use the old MyPastest? Access it here »

Back to Filters

Question 29 of 78

Which of the following statements about Giardia intestinalis is the most factually correct?

A Asymptomatic infection in developed countries should not be treated with antibiotics

B Giardia intestinalis infection rate is highest in adolescents

C Giardia intestinalis infection should be treated with erythromycin

D Giardia intestinalis is a protozoal infection, the only host of which is humans

E Infection with Giardia intestinalis can


cause chronic diarrhoea, malabsorption, failure to thrive and
intestinal histological changes similar to those seen in coeliac disease

Explanation
It is a protozoal parasite that is infective in the cyst form and can grow in the intestines of humans or animals.
Infection in humans is more common in children than in adults; the rates of infection in the UK are highest for the
1- to 4-year age group. It is endemic in areas with poor sanitation and commonly associated with outbreaks
secondary to food or water contamination. As relatively low numbers of cysts are required to produce infection,
outbreaks in the developed world are often associated with day-care settings and institutions. Children at particular
risk are those who have travelled to endemic areas or have contact with those who have travelled to endemic areas,
those who are malnourished, those with immunodeficiencies (recurrent infection is seen in those with IgA
deficiency) and those with cystic fibrosis. Clinical manifestations include:

asymptomatic infection
acute diarrhoeal illness
chronic diarrhoea and malabsorption (may last weeks to months)

Chronic
infection may be associated with partial villous atrophy in small bowel
biopsy. Diagnosis is by stool

https://mypastest.pastest.com/Secure/TestMe/Browser/436619[‫ ص‬07:07:37 09/12/1437]


MyPastest

examination for cysts or examination of the duodenal aspirate at small bowel biopsy. Treatment consists of
appropriate fluid and nutritional management and/oral metronidazole for all infected cases who are symptomatic
or asymptomaticPrevious Question
in non-endemic Next Question
areas. Good basic hygiene procedures are essential for preventing spread.
43978

End Session
Tag Question

Feedback

Difficulty: Average

Peer Responses

Session Progress

Responses Correct: 0

Responses Incorrect: 31

Responses Total: 31

Responses - % Correct: 0%

Blog
About Pastest
Contact Us
Help

© Pastest 2016

https://mypastest.pastest.com/Secure/TestMe/Browser/436619[‫ ص‬07:07:37 09/12/1437]


MyPastest

Prefer to use the old MyPastest? Access it here »

Back to Filters

Question 30 of 78

Which of the following statements with regard to ulcerative colitis is most factually correct?

A FBC and inflammatory markers within the normal range almost always exclude a diagnosis of
ulcerative colitis

B Growth failure is a common presentation of ulcerative colitis and often precedes GI symptoms by
months or even years

C Half
the patients presenting with ulcerative colitis present with classic colitic symptoms of abdominal
pain and bloody diarrhoea

D The pANCA blood tests are positive in 15% of cases of ulcerative colitis and 45% of cases of Crohn
disease

E Ulcerative colitis is more common than Crohn disease in younger children and in Asian children

Explanation
Ulcerative colitis is an inflammatory condition limited to the colon. Most children (90%) have pancolitis; the
rest
have left-sided colonic disease, or proctitis. Ulcerative colitis is a lifelong illness with 50% of patients relapsing in
any 1 year. Children with moderate-to-severe disease at diagnosis have a colectomy rate of 25% at 5 years. The
histological changes are mucosal and submucosal inflammation, goblet cell depletion, cryptitis and crypt
abscesses, but no granulomas. Twenty-five per cent of cases of inflammatory bowel disease (IBD) present in those
aged <16 years; approximately 30% is ulcerative colitis. Ulcerative colitis is the most common form of IBD in the
younger child. It is slightly more common in Asian children than in other ethnic groups. The sex distribution in
childhood ulcerative colitis is even. A family history of either Crohn disease or ulcerative colitis is common in any
index case; breastfeeding
appears to be protective and certain GI infections may increase the risk of developing
IBD. Most patients with ulcerative colitis present with symptoms of colitis: diarrhoea, abdominal pain and blood
per rectum. Rarely the disease can present with extraintestinal manifestations, including arthropathy (10%),
ankylosing spondylitis (rare in children) liver disease (sclerosing cholangitis, autoimmune liver disease) and

https://mypastest.pastest.com/Secure/TestMe/Browser/436619[‫ ص‬07:07:57 09/12/1437]


MyPastest

erythema nodosum. Growth failure is much less commonly associated than with Crohn disease and the presenting
blood panel may be normal. The peripheral antineutrophil cytoplasmic antibody (pANCA) is positive in 70% of
ulcerative colitisPrevious
and <10% ofQuestion Next Question
Crohn disease cases.

Complications include:

Toxic megacolon
Osteoporosis
Colonic cancer
End Session
Increased thrombotic tendency in severe disease

Treatments include:

5-Aminosalicylic acid (ASA) derivatives


Local or systemic steroids, intravenously in severe colitis
Antibiotics if infection is suspected
Azathioprine or 6-mercaptopurine in recurrent–relapse or treatment-resistant disease
Ciclosporin in resistant disease
Infliximab in severe non-responding ulcerative colitis to delay surgery
Surgery

43979

Tag Question

Feedback

Difficulty: Average

Peer Responses

Session Progress

https://mypastest.pastest.com/Secure/TestMe/Browser/436619[‫ ص‬07:07:57 09/12/1437]


MyPastest

Responses Correct: 0

Responses Incorrect: 32

Responses Total: 32

Responses - % Correct: 0%

Blog
About Pastest
Contact Us
Help

© Pastest 2016

https://mypastest.pastest.com/Secure/TestMe/Browser/436619[‫ ص‬07:07:57 09/12/1437]


MyPastest

Prefer to use the old MyPastest? Access it here »

Back to Filters

Question 31 of 78

Which of the statements about recurrent abdominal pain in childhood is the most factually correct?

A A childhood history of recurrent abdominal pain is rarely associated with irritable bowel syndrome in
adult life

B A family history of coeliac disease is common in these children

C A family history of migraine is associated with a subset of children with recurrent abdominal pain

D Introducing
the biopsychosocial model of illness is best accepted at the later stages of management of
these children when organic diagnoses have been excluded

E There is good evidence to suggest that the symptom of pain in these children is fictitious and made up
for secondary gain

Explanation
Recurrent abdominal pain is common in childhood, affecting 10% of the school-aged population. In the vast
majority of cases the aetiology is non-organic. The condition is more common in girls and a family history is
common. The pain is usually periumbilical and rarely associated with other GI symptoms. Symptoms suggestive
of an organic cause include:

Age <5 years


Constitutional symptoms (fever, weight loss, delayed growth, skin rashes, arthralgia)
Vomiting, especially if bilious
Nocturnal pain
Pain away from the umbilicus
Urinary symptoms
Family history of organic GI pathology

https://mypastest.pastest.com/Secure/TestMe/Browser/436619[‫ ص‬07:08:20 09/12/1437]


MyPastest

Perianal disease
Blood in the stools
Previous Question Next Question
The
ideal management is a thorough initial history and examination, to help
to exclude the rare cases of organic
disease, followed by limited investigations. Once a diagnosis is made classification into one of four
subtypes may
aid ongoing management:

1. Functional abdominal pain (syndrome) End Session


2. Functional dyspepsia
3. Irritable bowel syndrome
4. Abdominal migraine

Many
cases respond to the acknowledgement of the symptoms and reassurance about the lack of serious
underlying pathology. The acceptance by the child and parents of the bipsychosocial model of illness is an
important
factor for the resolution of symptoms (butterflies in stomach with exams). Diet and lifestyle changes,
including avoidance of trigger factors and improvement of associated constipation, increased exercise and a staged
programme of school reintroduction if applicable, can prove
helpful. The introduction of the concept of pain
management early in the therapeutic relationship is often helpful. There are data to suggest
that chronic abdominal
pain in childhood is associated with the development of irritable bowel syndrome and anxiety disorders in adult
life.
43980

Tag Question

Feedback

Difficulty: Average

Peer Responses

Session Progress

Responses Correct: 0

Responses Incorrect: 33

https://mypastest.pastest.com/Secure/TestMe/Browser/436619[‫ ص‬07:08:20 09/12/1437]


MyPastest

Responses Total: 33

Responses - % Correct: 0%

Blog
About Pastest
Contact Us
Help

© Pastest 2016

https://mypastest.pastest.com/Secure/TestMe/Browser/436619[‫ ص‬07:08:20 09/12/1437]


MyPastest

Prefer to use the old MyPastest? Access it here »

Back to Filters

Question 32 of 78

Of the following causes of portal hypertension, which can be best described as being prehepatic?

A Biliary atresia

B Budd–Chiari syndrome

C Constrictive pericarditis

D Portal vein thrombosis

E Schistosomiasis

Explanation
The portal vein carries about 1500 ml/min
of blood from the small and large bowel, spleen, stomach and pancreas
to the liver at a pressure of 5–10 mmHg. Any obstruction or increased resistance to flow or, rarely, pathological
increases in portal vein blood flow may lead to portal hypertension, defined as portal pressures >12 mmHg.

Alcoholic and viral cirrhosis are the most common causes in the western world.
Liver disease due to schistosomiasis is the main cause in other areas of the world.
Portal vein thrombosis is the most common cause in children

Aetiology of portal hypertension in children

Prehepatic:

Portal
vein thrombosis (accounts for 30% of children with bleeding varices); causes include: sepsis,
thrombophilia, umbilical cauterization, pancreatitis.

Intrahepatic:

https://mypastest.pastest.com/Secure/TestMe/Browser/436619[‫ ص‬07:08:42 09/12/1437]


MyPastest

Cirrhotic
(most common cause of portal hypertension) – extrahepatic biliary atresia – cystic fibrosis – α1-
antitrypsin deficiency – Wilson disease – schistosomiasis
Non-cirrhotic: veno-occlusive disease of the liver (sinusoidal obstruction syndrome)

Posthepatic:

Budd–Chiari syndrome: hepatic vein occlusion, usually secondary to myeloproliferative disease or


thrombophilia
Right ventricular failure
Constrictive pericarditis

Clinical features of portal hypertension

Splenomegaly
Ascites
Prominent abdominal vessels (caput medusa)
Oesophageal varices
Haemorrhoids
Rectal varices

Consequences of portal hypertension

Increased
portal pressure causes an increase in portosystemic collaterals, resulting in a disturbed intrahepatic
circulation. This may lead to the clinical complications:

Variceal bleeding
Hepatic encephalopathy
Ascites
Hepatorenal syndrome
Recurrent infection, spontaneous bacterial peritonitis
Abnormalities in coagulation

Hepatorenal syndrome

Acute oliguric renal failure resulting from intense intrarenal vasoconstriction in otherwise normal kidneys
Prognosis is poor, treatment often ineffective
In liver failure avoid nephrotoxic drugs
Prevented by avoiding excessive diuresis and by early recognition of electrolyte imbalance, bleeding or
infection

43981

https://mypastest.pastest.com/Secure/TestMe/Browser/436619[‫ ص‬07:08:42 09/12/1437]


MyPastest

Previous Question Next Question


Tag Question

Feedback

End Session
Difficulty: Average

Peer Responses

Session Progress

Responses Correct: 0

Responses Incorrect: 34

Responses Total: 34

Responses - % Correct: 0%

Blog
About Pastest
Contact Us
Help

© Pastest 2016

https://mypastest.pastest.com/Secure/TestMe/Browser/436619[‫ ص‬07:08:42 09/12/1437]


MyPastest

Prefer to use the old MyPastest? Access it here »

Back to Filters

Question 33 of 78

Which of the following statements about rectal prolapse is the MOST correct?

A It can always be resolved with a short course of laxatives and dietary advice

B It is associated with cystic fibrosis in most cases

C It is most common in the pre-school-age group

D It is usually painful

E It requires reduction under general anaesthetic

Explanation
Rectal prolapse is most common below 4 years of age and most cases occur in the first year of life. It can involve
protrusion of the mucosal layer alone or all the layers (procidential).

Causes of rectal prolapse:

Idiopathic
Increased abdominal pressure: – constipation – chronic cough – toilet training
Diarrhoeal disorders: – acute infection – malabsorption
Cystic
fibrosis (20% of patients with CF have rectal prolapse between 6 months
and 3 years and CF
accounts for 10% of all cases of rectal prolapse)
Neuromuscular syndromes
Rectal polyps
Malnutrition/anorexia
Child abuse (anal sex)

https://mypastest.pastest.com/Secure/TestMe/Browser/436619[‫ ص‬07:09:02 09/12/1437]


MyPastest

In
most cases spontaneous reduction has taken place by the time of presentation, so a brief examination in the
squatting position is advised to try to visualise a recurrence. It is usually painless, although failure to reduce the
prolapsed tissuePrevious
can result inQuestion Next Question
oedema and
ulceration. Parents can be taught to reduce a prolapse at home. Patients
with this presentation should be considered for a sweat test if
no other explanation is forthcoming. Management
should always be conservative in the first instance especially if aged <4 years (90%);
will respond by 6 years of
age. This involves treating any underlying cause and reducing straining through the use of diet and laxatives to
reduce constipation. Surgery is occasionally required for recurrent or resistant prolapse or that associated with
ulceration or pain; it can be
associated with complications. End Session
43982

Tag Question

Feedback

Difficulty: Average

Peer Responses

Session Progress

Responses Correct: 0

Responses Incorrect: 35

Responses Total: 35

Responses - % Correct: 0%

Blog
About Pastest
Contact Us
Help

© Pastest 2016

https://mypastest.pastest.com/Secure/TestMe/Browser/436619[‫ ص‬07:09:02 09/12/1437]


MyPastest

https://mypastest.pastest.com/Secure/TestMe/Browser/436619[‫ ص‬07:09:02 09/12/1437]


MyPastest

Prefer to use the old MyPastest? Access it here »

Back to Filters

Question 34 of 78

Which is the MOST appropriate statement about constipation?

A It can always be resolved with a short course of laxatives and dietary advice

B It can be precipitated by a child’s refusal to use school toilets

C It is usually associated with an underlying medical condition

D It occurs occasionally in the pre-school-age group

E It should be investigated with rectal biopsy to exclude Hirschsprung disease

Explanation
Constipation is defined (NICE guidelines) as two or more of the following:

Passage of fewer than 3 complete stools per week


Hard ‘rabbit droppings’ or large stools that may block the toilet
Soiling – the passage of smelly loose stools without sensation
Poor appetite or abdominal pain that varies with the passage of stools
Pain associated with defecation
Evidence of retentive posturing
Bleeding associated with hard stools

An
extremely common problem in childhood (5–30% of all children), it
is rarely associated with any underlying
medical problem (5%). A vicious cycle usually develops whereby a child associates pain with defecation and then
withholds stool in an attempt to avoid discomfort. This retention of stool causes enlargement of the rectum and
distal colon (megacolon) to accommodate the faeces, and the normal urge to defecate is lost. Eventually, faecal

https://mypastest.pastest.com/Secure/TestMe/Browser/436619[‫ ص‬07:09:25 09/12/1437]


MyPastest

incontinence (soiling) may ensue. Parents may be able to identify a trigger to the start of the problem, eg a change
in milk, toilet training, after an illness or starting a new
school.
Previous Question Next Question
All children should undergo examination including growth parameters, perianal, spinal, lumbosacral, lower limbs
and abdominal examination to exclude underlying pathology at
presentation. An abdominal radiograph should not
be used to diagnose idiopathic constipation. Treatment involves disimpaction (ideally via the oral route, although
occasionally if oral medications have failed enemas are required), followed by establishing a regular bowel habit
(both behavioural modification and maintenance laxatives). Treatment is usually required for many weeks or
months and must be continued for some
time after the establishment of normalEndbowelSession
habit. Treatment should be
weaned down and never stopped abruptly. Seventy per cent of children will become eventually symptom free off
medication, although a significant cohort will continue to have symptoms despite long-term treatment. Education
and reassurance of parents are essential.

https://www.nice.org.uk/guidance/cg99/evidence/full-guidance-245466253
43983

Tag Question

Feedback

Difficulty: Average

Peer Responses

Session Progress

Responses Correct: 0

Responses Incorrect: 36

Responses Total: 36

Responses - % Correct: 0%

https://mypastest.pastest.com/Secure/TestMe/Browser/436619[‫ ص‬07:09:25 09/12/1437]


MyPastest

Blog
About Pastest
Contact Us
Help

© Pastest 2016

https://mypastest.pastest.com/Secure/TestMe/Browser/436619[‫ ص‬07:09:25 09/12/1437]


MyPastest

Prefer to use the old MyPastest? Access it here »

Back to Filters

Question 35 of 78

Which is the MOST appropriate statement about pancreatitis in childhood?

A A mildly raised amylase level associated with abdominal pain is specifically diagnostic of pancreatitis

B A normal ultrasound examination of the pancreas excludes the diagnosis

C It is a common cause of admission to hospital with abdominal pain in childhood

D It can be caused by treatment with sodium valproate

E It is commonly caused by gallstones in children

Explanation
Pancreatitis is uncommon in childhood. In
up to 25% of cases the aetiology is unknown. Unlike adults, the most
common identifiable causes are abdominal trauma, viral infection (mumps,
rubella, Coxsackie B,
cytomegalovirus), medication (azathioprine, steroids, sodium valproate) and congenital abnormalities of the
pancreatobiliary system. Pseudocysts, a fibrous, walled cavity filled with pancreatic enzymes, complicates
between 10 and 23% of cases and is common in those caused by blunt abdominal trauma. Pancreatitis typically
presents with epigastric pain (which can radiate through to the back), nausea and vomiting, and low-grade fever.
Elevated serum amylase levels are seen in most cases of acute pancreatitis but can take up to 48 h to reach peak
levels. Other abdominal pathologies can cause a rise in serum
amylase but rarely to the degree seen in pancreatitis.

Medical
management consists of rehydration and analgesia, and may also include a
period of being ‘nil by mouth’,
sometimes sufficient to require total parenteral nutrition (TPN) (start at 3 days to prevent catabolism). Antibiotics
may also be indicated in some cases to treat systemic infection or sepsis. Ultrasonography and CT are the most
commonly used imaging modalities, but MRI and endoscopic retrograde cholangiopancreatography (ERCP) can
also be helpful. Findings are variable and include a normal-looking pancreas, initially in up to 20% of cases. The
length of the illness is variable but most acute cases resolve after 2–4 days. There is a recurrence rate of
approximately 9%. Surgical involvement may be required if necrosis, abscess or pseudocyst formation occurs,
although most pseudocysts will spontaneously resolve in 4–6 weeks.

https://mypastest.pastest.com/Secure/TestMe/Browser/436619[‫ ص‬07:10:06 09/12/1437]


MyPastest

43984

Previous Question Next Question

Tag Question

Feedback End Session

Difficulty: Average

Peer Responses

Session Progress

Responses Correct: 0

Responses Incorrect: 37

Responses Total: 37

Responses - % Correct: 0%

Blog
About Pastest
Contact Us
Help

© Pastest 2016

https://mypastest.pastest.com/Secure/TestMe/Browser/436619[‫ ص‬07:10:06 09/12/1437]


MyPastest

Prefer to use the old MyPastest? Access it here »

Back to Filters

Question 36 of 78

A 5-month-old baby with pH study-proven gastro-oesophageal reflux has not improved on treatment with
Gaviscon. The centile chart now shows that the weight gain is beginning to falter.

What would be the next MOST appropriate stage of treatment?

A Addition of an acid suppressant, eg proton pump inhibitor

B Reassurance that, with weaning, symptoms should improve

C Referral for fundoplication

D Trial of hydrolysed formula milk

E Use of prokinetic agent such as domperidone

Explanation
Gastro-oesophageal reflux (GOR) is defined as the passage of gastric contents into the oesophagus with or without
regurgitation or vomiting. Gastro–oesophageal reflux disease (GORD) is when GOR is accompanied by
troublesome symptoms. In addition to irritability and vomiting, affected individuals may present with feeding
difficulties, apnoeas, failure to thrive, asthma, aspiration, stridor or haematemesis. Oesophagitis and oesophageal
strictures may result from more severe cases of GORD.

If
indicated, investigation is ideally done in the form of 24-hour pH probe, increasingly accompanied by multiple
intraluminal impedance tests, which give information on acidic, weakly acidic and non-acidic reflux episodes, the
severity of which does not necessarily correlate with symptoms.

Treatment involves parental reassurance and advice about lifestyle measures. Medical treatments include:

Alginates (eg infant Gaviscon), which should not be used as sole agents, long term
Acid
suppression: – proton pump inhibitors (PPIs) are superior to H2-receptor blockers – increased risk of

https://mypastest.pastest.com/Secure/TestMe/Browser/436619[‫ ص‬07:11:27 09/12/1437]


MyPastest

community-acquired pneumonia and GI infections, particularly with PPIs


Prokinetic agents; the routine use of these agents is not currently recommended
Previous Question Next Question
Drug treatment is not indicated in all cases, but should be used when growth is affected or symptoms occur with
sufficient frequency to cause distress to the infant and/or caregivers. Medical management, as detailed above,
should be the mainstay of treatment. GOR may rarely be associated with cows’ milk protein intolerance, in which
case a hydrolysed formula can be substituted for a 4- to 6-week trial period. In those cases where medical
management fails, surgical intervention by fundoplication should be considered. End Session
Potential surgical complications
and failure rates mean that this operation should be reserved for only the most severe cases.

In symptomatic individuals, resolution of symptoms is usually seen by the age of 2 years, correlating with
developmental maturity including a more upright posture and intake of solids.
43985

Tag Question

Feedback

Difficulty: Average

Peer Responses

Session Progress

Responses Correct: 0

Responses Incorrect: 38

Responses Total: 38

Responses - % Correct: 0%

Blog
About Pastest
Contact Us
Help

https://mypastest.pastest.com/Secure/TestMe/Browser/436619[‫ ص‬07:11:27 09/12/1437]


MyPastest

© Pastest 2016

https://mypastest.pastest.com/Secure/TestMe/Browser/436619[‫ ص‬07:11:27 09/12/1437]


MyPastest

Prefer to use the old MyPastest? Access it here »

Back to Filters

Question 37 of 78

A 30-month-old girl is referred to clinic with a 6-month history of passing up to five watery stools per day.

The
mother is particularly concerned as there are frequently recognisable food particles in the stools. The child’s
centile charts reveal that she is steadily gaining weight along the 50th centile for both height and weight. Dietary
history reveals that she is a slightly fussy eater, tending to graze throughout the day rather than eating three full
meals. She drinks up to 2.5 l of fruit squash per day via a bottle. Examination is unremarkable.

Which is the MOST appropriate course of action?

A Advice
about dietary intake, especially with regard to the amount and type of fluids being offered with
further follow-up to assess weight gain

B Arrange blood tests for full blood count, total IgA and anti-gliadin antibodies

C Assay for faecal elastase to rule out pancreatic insufficiency

D Collection of a stool sample for detection of reducing substances and implementation of a lactose-free
diet

E Microbiological analysis of stool sample (culture and virology assessment) to exclude infective
diarrhoea

Explanation
There are numerous causes of diarrhoea in childhood including the following:

Infection/inflammation
Malabsorption: transient or long term, primary or secondary, fat, protein or carbohydrate
Chronic non-specific diarrhoea of childhood (toddler’s diarrhoea)
Not in fact diarrhoea; overflow constipation, fabricated or induced illness

https://mypastest.pastest.com/Secure/TestMe/Browser/436619[‫ ص‬07:11:55 09/12/1437]


MyPastest

Any
child presenting with chronic (>14 days) diarrhoea should undergo a thorough assessment including a careful
history, head-to-toe physical examination with assessment of growth and visual examination of the stool ± lab
Previous Question Next Question
microscopy and culture, to help guide appropriate ongoing investigation and avoid unnecessary invasive or
excessive procedures where appropriate. Rotavirus is the most common cause of infective diarrhoea worldwide,
followed by adenovirus. Salmonella, Shigella and Campylobacter spp. are the next most common infecting
organisms. Carbohydrate malabsorption is most frequently seen secondary to infective diarrhoea. Pancreatic
enzyme deficiency causes fat malabsorption, tested for by stool faecal elastase; false-positive results can occur in
End
conditions with villous atrophy or due to the dilutional effects of excess watery Session
stools. Protein-losing enteropathy
should be suspected in any child with low albumin and chronic diarrhoea;
it can occur in many different disease
processes and can be tested for by measuring stool α1-antitrypsin.

Chronic
non-specific diarrhoea of childhood presents with the frequent passage of loose stools, often containing
undigested food particles, and is common especially in the run-up to toilet training. The relatively rapid
gut transit
time in children, in addition to an excessive intake of fruit juices/squash (including fructose-containing apple
juice), excess fibre, low dietary fat content and emotional stress, can exacerbate the situation. There are never any
additional features of pain, blood in the
stools, nocturnal stools or associated faltering growth. Parental
reassurance is essential as well as dietary advice, including the type and amount of fluids consumed and follow-up
until the condition improves.
43986

Tag Question

Feedback

Difficulty: Average

Peer Responses

Session Progress

Responses Correct: 0

Responses Incorrect: 39

Responses Total: 39

https://mypastest.pastest.com/Secure/TestMe/Browser/436619[‫ ص‬07:11:55 09/12/1437]


MyPastest

Responses - % Correct: 0%

Blog
About Pastest
Contact Us
Help

© Pastest 2016

https://mypastest.pastest.com/Secure/TestMe/Browser/436619[‫ ص‬07:11:55 09/12/1437]


MyPastest

Prefer to use the old MyPastest? Access it here »

Back to Filters

Question 38 of 78

A 6 month old child is admitted to hospital with episodic abnormal movements. During an episode you observe
bizarre extension and
lateral turning of her head with dystonic posturing. Her parents say she has always been
fussy with feeds and they are struggling to introduce solids. Her physical examination is normal.

Which of the management choices below would be most helpful for this child?

A Dairy-free diet

B MRI Brain scan

C Prednisolone

D Upper GI endoscopy

E Vigabatrin

Explanation
This girl is showing the episodic movements of potential Sandifer’s Syndrome – posturing secondary to gastro-
intestinal reflux most commonly misdiagnosed as seizures.

In Sandifer Syndrome the child may have sudden deviation of the head and neck to one side and the legs to the
other side appearing stretched out. Often the back is arched posteriorly with hyperextension of the spine. Elbows
may be flexed and held posteriorly with hyperextendable hips. Periods of crying and apparent discomfort may
occur, often after posturing has finished. In most cases the rhythmic clonic component as seen in a seizure is not
present. Various stiff postures may be held for 1-3 minutes, often after food. Respiratory irritation may also occur
with significant gastro-oesophageal reflux. When Sandifer’s
Syndrome is suspected the child may need GI
specialist referral with an
upper GI endoscopy to look for evidence of GOR and hiatus hernia particularly.

Whilst dairy intolerance can be a cause of reflux symptoms and distress in a child it is not usually a cause of
abnormal posturing alone. Without further history or symptoms suggestive of this
it would be unusual to
commence a dairy free diet suddenly in this child. If Sandifer’s Syndrome is diagnosed then the reflux may be

https://mypastest.pastest.com/Secure/TestMe/Browser/436619[‫ ص‬07:12:18 09/12/1437]


MyPastest

modified with diet if necessary.

An MRI Brain scan is performed to look for abnormal structures or growths in the brain, which are the underlying
Previous
cause of developmental Question
abnormalities, Next Question
seizures or other symptoms. This history is not most suggestive of a brain
structural abnormality and it is not the most helpful option for the child.

Prednisolone and Vigabatrin are used to treat Infantile Spasms (West Syndrome) associated with a number of
underlying conditions such as Tuberous Sclerosis. Infantile spasms are seizures often linked with poor
developmental outcomes. The spasms have a typical pattern not described in this scenario. They usually consist of
End Session
a brief interruption of behaviour, lifting and extension of the arms, bending forward at the waist and often a rapid,
forceful head drop. Clusters last a few minutes and may be associated with a cry. Vigabatrin is usually used when
a child is not responsive to steroid treatment.

Website reference for GORD management: https://www.nice.org.uk/guidance/ng1/chapter/1-recommendations


46208

Tag Question

Feedback

Difficulty: Average

Peer Responses

Session Progress

Responses Correct: 0

Responses Incorrect: 40

Responses Total: 40

Responses - % Correct: 0%

Blog
About Pastest
Contact Us
Help

https://mypastest.pastest.com/Secure/TestMe/Browser/436619[‫ ص‬07:12:18 09/12/1437]


MyPastest

© Pastest 2016

https://mypastest.pastest.com/Secure/TestMe/Browser/436619[‫ ص‬07:12:18 09/12/1437]


MyPastest

Prefer to use the old MyPastest? Access it here »

Back to Filters

Question 39 of 78

A male term infant was admitted to the neonatal unit on day 3


for severe jaundice. The bilirubin at that time was
320 mmol/l, but had
decreased with phototherapy. At day 30, the baby was still requiring phototherapy to keep the
bilirubin below the treatment line. On examination, the skin had a tanned appearance and his sclerae were icteric.
Mother’s blood group was A+, baby’s was A+, direct
Coombs test (DCT) was negative.

What is the MOST likely diagnosis?

A ABO incompatibility

B Crigler-Najjar syndrome type 1

C Crigler-Najjar syndrome type 2

D Haemochromatosis

E Sepsis

Explanation
Crigler-Najjar syndrome type 2 is a rare inherited disorder affecting the metabolism of bilirubin, and would
explain this baby’s ongoing jaundice.

ABO incompatibility is wrong because both mother and baby have the same blood group, and DCT is negative.

Crigler-Najjar
syndrome type 1 is a possibility, but is less common than type 2. It is therefore not the most likely
diagnosis.

Haemochromatosis does not present with neonatal jaundice. The “tanned appearance” of the skin is caused by the
ongoing phototherapy.

Sepsis is an important consideration in any neonate with jaundice, particularly in the first 24 hours of life.
However, it would not explain jaundice at day 30.
46687

https://mypastest.pastest.com/Secure/TestMe/Browser/436619[‫ ص‬07:12:42 09/12/1437]


MyPastest

Previous Question Next Question


Tag Question

Feedback

End Session
Difficulty: Average

Peer Responses

Session Progress

Responses Correct: 0

Responses Incorrect: 41

Responses Total: 41

Responses - % Correct: 0%

Blog
About Pastest
Contact Us
Help

© Pastest 2016

https://mypastest.pastest.com/Secure/TestMe/Browser/436619[‫ ص‬07:12:42 09/12/1437]


MyPastest

Prefer to use the old MyPastest? Access it here »

Back to Filters

Question 40 of 78

A 3-day-old female infant had been on the postnatal ward with


mother, and had just completed 36 hours of IV
antibiotics for presumed sepsis. The baby looked well and was breastfeeding well. At the discharge baby check,
mild jaundice and bilateral talipes were noted. She vomited her feed following palpation of the abdomen.

What is the MOST likely cause for the vomit?

A Abdominal examination

B Biliary atresia

C Duodenal atresia

D Neonatal sepsis

E Spherocytosis

Explanation
This baby has physiological jaundice, which is common is breastfeeding babies.

Biliary atresia should be suspected if jaundice continues beyond 14 days in a term baby, or 21 days in a preterm
baby. This baby is 3 days old.

Duodenal atresia is a not a cause of neonatal jaundice.

Sepsis is an important consideration but this baby has completed IV antibiotics because blood cultures were
negative at 36 hours, and is otherwise well.

Hereditary spherocytosis would cause jaundice and anaemia however this baby was otherwise well and not
anaemic.

46688

https://mypastest.pastest.com/Secure/TestMe/Browser/436619[‫ ص‬07:13:04 09/12/1437]


MyPastest

Previous Question Next Question


Tag Question

Feedback

End Session
Difficulty: Average

Peer Responses

Session Progress

Responses Correct: 0

Responses Incorrect: 42

Responses Total: 42

Responses - % Correct: 0%

Blog
About Pastest
Contact Us
Help

© Pastest 2016

https://mypastest.pastest.com/Secure/TestMe/Browser/436619[‫ ص‬07:13:04 09/12/1437]


MyPastest

Prefer to use the old MyPastest? Access it here »

Back to Filters

Question 41 of 78

You are called to examine a baby who looks jaundiced at 22 hours. The baby was born at 36+4 weeks by
emergency C section for failure to progress. There were no maternal risk factors for sepsis. You cannulate the
baby, send bloods and start the baby on antibiotics. Mother’s blood group is O+, baby’s is B+, direct Coombs test
(DCT) is negative.

What is the MOST likely diagnosis?

A ABO incompatibility

B Hepatitis A

C Neonatal sepsis

D Physiological jaundice

E Pyloric stenosis

Explanation
The DCT results are positive in only 20-40% of infants with ABO incompatibility. ABO incompatibility occurs
mostly in mothers who have blood type O.

Hepatitis A is an acute infectious disease transmitted through contaminated food or water.

Neonatal sepsis is an important consideration in a jaundiced neonate <24 hours, and starting the baby on
antibiotics was an appropriate measure.

Physiological jaundice presents on day 2 onwards.

Pyloric stenosis usually presents at 4-6 weeks of life with projectile vomiting. It is not a reason for jaundice in the
neonate.
46689

https://mypastest.pastest.com/Secure/TestMe/Browser/436619[‫ ص‬07:13:27 09/12/1437]


MyPastest

Previous Question Next Question


Tag Question

Feedback

End Session
Difficulty: Average

Peer Responses

Session Progress

Responses Correct: 0

Responses Incorrect: 43

Responses Total: 43

Responses - % Correct: 0%

Blog
About Pastest
Contact Us
Help

© Pastest 2016

https://mypastest.pastest.com/Secure/TestMe/Browser/436619[‫ ص‬07:13:27 09/12/1437]


MyPastest

Prefer to use the old MyPastest? Access it here »


Previous Question Next Question

Back to Filters

Question 42 of 78

A term infant attends for a prolonged jaundice screen at 16 days. The history from mother is that there have been
pale stools.

What is the SINGLE most important investigation?

A Abdominal ultrasound

B Conjugated bilirubin

C Full blood count

D Urine dipstick

E Urine for MC&S

Explanation
The most likely diagnosis is biliary atresia. An abdominal ultrasound will confirm the diagnosis.

A conjugated bilirubin would lead us towards the diagnosis, but would not confirm the diagnosis.

An FBC alone would not help to diagnose biliary atresia. It therefore not the single most important test.

Urine
dipstick and MC&S (microscopy, culture & sensitivity) would rule out urinary tract infection, but neither
would be the appropriate single investigation of choice.
46690

Tag Question

https://mypastest.pastest.com/Secure/TestMe/Browser/436619[‫ ص‬07:13:47 09/12/1437]


MyPastest

Feedback

Difficulty: Average

Peer Responses

End Session

Session Progress

Responses Correct: 0

Responses Incorrect: 44

Responses Total: 44

Responses - % Correct: 0%

Blog
About Pastest
Contact Us
Help

© Pastest 2016

https://mypastest.pastest.com/Secure/TestMe/Browser/436619[‫ ص‬07:13:47 09/12/1437]


MyPastest

Prefer to use the old MyPastest? Access it here »

Back to Filters

Question 43 of 78

An 8-year-old boy goes to his GP following a diarrhoeal illness. On examination, he has mild jaundice, which he
has had before and his mother says it “doesn’t seem to bother him”.

What is the MOST likely diagnosis?

A Autoimmune hepatitis

B Gilbert syndrome

C Hepato-renal syndrome

D Reye syndrome

E Salmonella

Explanation
Gilbert syndrome is a genetic disorder seen in 3-12% of the population. Reduced activity of the enzyme
glucuronyltransferase during acute illness causes hyperbilirubinemia.

Autoimmune hepatitis would be more likely to present with fatigue and ill-health.

Hepato-renal syndrome is the development of renal failure in patients with chronic liver disease. In the absence of
cirrhosis and ascites, this is unlikely.

Reye syndrome is a rare rapidly progressive encephalopathy. Although associated with liver failure, jaundice does
not generally occur.

Salmonella hepatitis is a possibility, but


this is uncommon, with only 150 cases reported worldwide. Jaundice in
salmonella hepatitis usually occurs within the first 2 weeks of the febrile illness.
46691

https://mypastest.pastest.com/Secure/TestMe/Browser/436619[‫ ص‬07:14:06 09/12/1437]


MyPastest

Previous Question Next Question


Tag Question

Feedback

End Session
Difficulty: Average

Peer Responses

Session Progress

Responses Correct: 0

Responses Incorrect: 45

Responses Total: 45

Responses - % Correct: 0%

Blog
About Pastest
Contact Us
Help

© Pastest 2016

https://mypastest.pastest.com/Secure/TestMe/Browser/436619[‫ ص‬07:14:06 09/12/1437]


MyPastest

Prefer to use the old MyPastest? Access it here »

Back to Filters

Question 44 of 78

A 10-year-old boy presents to the emergency department with abdominal pain. He was previously under follow up
in the General Paediatric clinic, but had failed to attend his last clinic appointment.
Mother tells you that his pain
had been much better for a while, but had started again over the last month. He last opened his bowels this
afternoon and he reports loose, watery stool. He is on not on any regular medications. On examination, there is
generalised tenderness over the abdomen, particularly in the left iliac fossa.

What is the MOST appropriate next course of action?

A Give advice about hygiene measures to reduce faeco-oral transmission of bacteria

B Reassure and re-refer to paediatric clinic

C Refer to surgeons

D Send a stool sample

E Start a faecal disimpaction regime and discharge

Explanation
The correct answer is to start a disimpaction regime with Movicol. This boy has chronic abdominal pain due to
constipation, and the watery stool described is from overflow. His constipation was previously well controlled
with regular Movicol, but he
stopped taking his medication and did not attend his last clinic appointment.

A is wrong because the stem of the question should lead you to a diagnosis of constipation, not acute diarrhoeal
illness.

B is wrong. Although it is appropriate to re-refer him to clinic for ongoing management, this child needs treatment
for his constipation in the meantime.

C is wrong because there are no examination findings to suggest a surgical cause for his pain. Right iliac fossa,
and not left iliac fossa pain should lead you to suspect appendicitis.

https://mypastest.pastest.com/Secure/TestMe/Browser/436619[‫ ص‬07:14:26 09/12/1437]


MyPastest

D is wrong because the stem of the question should lead you to a diagnosis of constipation, not acute diarrhoeal
illness. The watery stool is overflow.
Previous Question Next Question 46692

End
Tag Session
Question

Feedback

Difficulty: Average

Peer Responses

Session Progress

Responses Correct: 0

Responses Incorrect: 46

Responses Total: 46

Responses - % Correct: 0%

Blog
About Pastest
Contact Us
Help

© Pastest 2016

https://mypastest.pastest.com/Secure/TestMe/Browser/436619[‫ ص‬07:14:26 09/12/1437]


MyPastest

Prefer to use the old MyPastest? Access it here »

Back to Filters

Question 45 of 78

A 13-year-old girl presents with abdominal pain. When you ask her to point to where the pain is she points to her
right iliac fossa. She says that she gets this pain regularly. Paracetamol sometimes helps, but not always. She is not
currently on her menstrual period and she denies any sexual activity.

What would be the MOST appropriate treatment for her pain?

A Mefenamic acid

B Oral contraceptive pill

C Oramorph

D Paracetamol

E Tranexamic acid

Explanation
The cause of this patient’s abdominal pain is most likely to be Mittelschmerz - a German word meaning “mid-
cycle pain” or “ovulation pain.” The pain
can occur suddenly and can last from a few hours to up to three days.
The combined oral contraceptive pill can be used to prevent ovulation, and there is evidence for its use in
Mittelschmerz.

A is wrong. Mefenamic acid is a non steroidal anti-inflammatory drug (NSAID) that is an effective treatment for
dysmenorrhoea, however it has no role in the management of Mittelschmerz.

Oramorph would be reserved for severe pain and would not be appropriate in this situation.

Paracetamol is inappropriate as she has already tried this and it only helps “sometimes”.

Tranexamic acid is an anti fibrinolytic that can be helpful in menorrhagia. It has no role in the management of
Mittelschmerz.

https://mypastest.pastest.com/Secure/TestMe/Browser/436619[‫ ص‬07:14:47 09/12/1437]


MyPastest

Ref: Jensen JT, Speroff L. Health benefits of oral contraceptives.ObstetGynecolClin North Am. 2000
Dec;27(4):705-21.
Previous Question Next Question 46693

End
Tag Session
Question

Feedback

Difficulty: Average

Peer Responses

Session Progress

Responses Correct: 0

Responses Incorrect: 47

Responses Total: 47

Responses - % Correct: 0%

Blog
About Pastest
Contact Us
Help

© Pastest 2016

https://mypastest.pastest.com/Secure/TestMe/Browser/436619[‫ ص‬07:14:47 09/12/1437]


MyPastest

Prefer to use the old MyPastest? Access it here »

Back to Filters

Question 46 of 78

An 8-year-old girl presents with weight loss, diarrhoea and abdominal pain. On examination, you note the
presence of apthous ulcers
and generalised abdominal tenderness. She is off her food but drinking
normally, and
her energy levels are low.

What is the MOST likely diagnosis?

A Anorexia nervosa

B Crohn’s disease

C Diabetic ketoacidosis

D Recurrent apthous stomatitis

E Ulcerative colitis

Explanation
The correct answer is Crohn’s disease, a type of inflammatory bowel disease which can affect any part of the
bowel from the mouth the anus. Symptoms include abdominal pain, diarrhoea, pyrexia and weight loss. Extra-
intestinal manifestations include arthritis, uveitis, fatigue, anaemia and rashes including pyoderma gangrenosum
and erythema nodosum.

Whilst anorexia nervosa is an important diagnosis to consider, there are no indicators in the description that she
has a fear of gaining weight or strong desire to be thin.

Diabetic ketoacidosis is incorrect because there is no polydipsia or polyuria. A patient in DKA is more likely to
present with vomiting, and not diarrhoea.

Recurrent apthous stomatitis is not a correct answer because it does not explain all of the symptoms described,
only the mouth ulcers.

Ulcerative colitis (UC) is also incorrect. UC is a form of inflammatory bowel disease that causes inflammation in

https://mypastest.pastest.com/Secure/TestMe/Browser/436619[‫ ص‬07:15:08 09/12/1437]


MyPastest

the colon. The main symptom is bloody stools, which is not mentioned as a feature in the history.
46694

Previous Question Next Question

Tag Question
End Session
Feedback

Difficulty: Average

Peer Responses

Session Progress

Responses Correct: 0

Responses Incorrect: 48

Responses Total: 48

Responses - % Correct: 0%

Blog
About Pastest
Contact Us
Help

© Pastest 2016

https://mypastest.pastest.com/Secure/TestMe/Browser/436619[‫ ص‬07:15:08 09/12/1437]


MyPastest

Prefer to use the old MyPastest? Access it here »

Back to Filters

Question 47 of 78

A type one diabetic presents with a one year history of bloating, diarrhoea and abdominal pain. Her last Hba1c
was 6.3%.

From the list below, what is the MOST appropriate investigation to instigate next?

A Coeliac screen

B Food diary

C Sickle cell percentage

D Thyroid function tests

E Urine disptick

Explanation
Coeliac disease is highly prevalent in children with type one diabetes (11%), and these children should therefore
undergo screening. Symptoms of bloating, diarrhoea and abdominal pain are common. A duodenal biopsy would
be the gold standard investigation,
but the next most appropriate would be a blood test for coeliac screen.

Although a food diary may be useful, investigations should be initiated to reach a diagnosis. B is therefore
incorrect.

A sickle cell percentage would be useful in a patient presenting with abdominal pain due to sickle cell crisis. There
is no indication that this child has sickle cell disease.

Hyper- and hypo-thyroidism are also more common in type 1 diabetes than in the general population, but the
patient history given points towards a diagnosis of coeliac. D is therefore incorrect.

Urinary tract infection should be ruled out in a child with abdominal pain. However, this question is about
chronic, not acute abdominal pain and therefore coeliac screen is the most appropriate answer.

https://mypastest.pastest.com/Secure/TestMe/Browser/436619[‫ ص‬07:15:28 09/12/1437]


MyPastest

46695

Previous Question Next Question

Tag Question

Feedback End Session

Difficulty: Average

Peer Responses

Session Progress

Responses Correct: 0

Responses Incorrect: 49

Responses Total: 49

Responses - % Correct: 0%

Blog
About Pastest
Contact Us
Help

© Pastest 2016

https://mypastest.pastest.com/Secure/TestMe/Browser/436619[‫ ص‬07:15:28 09/12/1437]


MyPastest

Prefer to use the old MyPastest? Access it here »

Back to Filters

Question 48 of 78

A 7-year-old boy attends clinic accompanied by his mother. One year ago he was admitted to hospital with severe
gastroenteritis. Stool sample was positive for cryptosporidium. When he attended clinic 6
months ago, he had
ongoing pain which was thought to be related to the initial infection. This persists and on average, his pain occurs
twice/week and usually towards the end of the day. He says he also sometimes gets headaches. On examination,
his abdomen is soft and non tender. There are no masses or organomegaly and bowel sounds are audible. CRT<2
seconds, heart sounds are normal. There are no signs of respiratory distress and his chest is clear. There are
multiple bruises over his shins bilaterally. FBC, U&Es, CRP and TFTs taken at the last clinic visit were all within
the normal range. Blood glucose was 5.4mmol/l. On further questioning he reveals that the family has recently
moved home and his mother’s partner has recently been arrested following a domestic violence incident. Social
services had been involved and did not consider the child to be at risk.

What is the MOST likely cause of his abdominal pain?

A Diabetic ketoacidosis

B Functional abdominal pain

C Ongoing abdominal cramps following the cryptosporidium infection

D Physical abuse

E Recurrence of the cryptosporidium infection

Explanation
Functional abdominal pain describes the situation
where there is no organic cause for a reported pain. It is a
common condition in children and teenagers and may be accompanied by headaches,
limb pain and difficulty
sleeping. It may be intensified by stress or anxiety. There may also be episodes of diarrhoea or constipation but
these are not the cause of the pain. The pain causes a great deal of anxiety on the part of both parents and children
and the symptoms are real even though not caused by a physical disease. The diagnosis is made
where at least
once per week for the at least 2 months there is episodic or continuous abdominal pain not compatible with any

https://mypastest.pastest.com/Secure/TestMe/Browser/436619[‫ ص‬07:15:51 09/12/1437]


MyPastest

other symptoms of functional gastrointestinal disorder and there is no evidence of an inflammatory, anatomical,
metabolic or neoplastic process
that explains the child's symptoms.
Previous Question Next Question
Diabetic ketoacidosis is a cause of acute, not chronic/recurrent abdominal pain, and you are told the blood glucose
is normal so stem A is incorrect.

The initial infection was one year ago so it is unlikely to be causing ongoing abdominal cramps therefore stem C is
incorrect.

End Session
Although
abuse should not be dismissed as a cause simply because social services
are aware of a situation,
multiple bruises over the shins is not an uncommon finding in a child of this age. Bruises over non bony areas
should raise suspicion of physical abuse.

Recurrence of the infection would not account for the chronicity of the abdominal pain and
there is no diarrhoea at
present so stem E is unlikely.
46696

Tag Question

Feedback

Difficulty: Average

Peer Responses

Session Progress

Responses Correct: 0

Responses Incorrect: 50

Responses Total: 50

Responses - % Correct: 0%

https://mypastest.pastest.com/Secure/TestMe/Browser/436619[‫ ص‬07:15:51 09/12/1437]


MyPastest

Blog
About Pastest
Contact Us
Help

© Pastest 2016

https://mypastest.pastest.com/Secure/TestMe/Browser/436619[‫ ص‬07:15:51 09/12/1437]


MyPastest

Prefer to use the old MyPastest? Access it here »

Back to Filters

Question 49 of 78

You are the Paediatric ST1 covering the assessment unit in a busy district general hospital. A 3-week-old breasfed
baby has been referred by the emergency department. His parents are very concerned because he has not opened
his bowels for three days. He has episodic screaming episodes which occur mostly in the evenings. He is the first
baby of non-consanguineous parents. He was born at full term by emergency C-section due to prolonged second
stage of labour. There was meconium stained liquor. His APGARS were 9, 10 and 10, and he was discharged
home the next day. He is otherwise well and feeding well.

What is the MOST likely diagnosis?

A Hirschsprung’s disease

B Infantile colic

C Intussusception

D Meckel’s diverticulum

E Pyloric stenosis

Explanation
Breastfed babies may only open their bowels once every few days, and this is not a cause for concern. It is
important to note whether the baby passed meconium within the first 48 hours of life.
In this case, there is a
history of meconium stained liquor. Infantile colic is common, although more so in formula fed babies. Parents
should be given advice about appropriately winding the baby after feeds.

A is incorrect. Hirschsprung’s disease is a disorder characterised by an a ganglionic segment of the distal large
bowel, and is therefore a cause of constipation in babies. In this baby, there are no symptoms of signs suggestive
of a surgical cause. There is no bilious
vomiting and the baby is feeding well.

C is incorrect. There are no symptoms or signs suggestive of a surgical cause. Intussusception


may present with a
baby who draws up its legs, and has “redcurrant jelly” stools.

https://mypastest.pastest.com/Secure/TestMe/Browser/436619#Top[‫ ص‬07:17:15 09/12/1437]


MyPastest

D is incorrect. Meckel’s diverticulum is a remnant of the vitello intestinal duct,


which has the potential to cause
lower GI obstruction. There are no symptoms or signs suggestive of a surgical cause in this baby.
Previous Question Next Question
E
is incorrect. There are no symptoms or signs suggestive of a surgical cause. The typical age for pyloric stenosis
to occur is between 4 and 6 weeks. Presentation is with a history of projectile vomiting.
46697

End Session

Tag Question

Feedback

Difficulty: Average

Peer Responses

Session Progress

Responses Correct: 0

Responses Incorrect: 52

Responses Total: 52

Responses - % Correct: 0%

Blog
About Pastest
Contact Us
Help

© Pastest 2016

https://mypastest.pastest.com/Secure/TestMe/Browser/436619#Top[‫ ص‬07:17:15 09/12/1437]


MyPastest

Prefer to use the old MyPastest? Access it here »

Back to Filters

Question 50 of 78

A 15-year-old boy is under regular follow up in the General Paediatric clinic. Today, he attends with his
grandmother whom he has lived with for the past 6 months. Grandmother also looks after his two younger
siblings, aged 9 and 6 years. Prior to this he was cared for by
his father, however his father can no longer cope
with his behaviour. He opens his bowels once every three to four days after straining, and also has problems with
smearing. He is unable to sit on the toilet and has never done so, instead he crouches and admits that this is very
uncomfortable. He has recently broken up with his girlfriend and is feeling rejected. When you look further
through his notes you see that he suffered severe physical and emotional abuse throughout his childhood, and his
mother is currently in prison. You notice scars from
self-harm on his wrists.

Which of these is the LEAST appropriate action?

A Liaise with the school nurse

B Prescribe Movicol

C Refer to CAMHS

D Refer to clinical psychology

E Refer to surgeons

Explanation
Referral to surgeons is not appropriate in this situation. The cause for this boy’s constipation is psychological.
There are no “red flags” in the history to suggest an underlying disorder or condition (see NICE guidance for red
flags).

It would be appropriate to liaise with the school nurse for the ongoing management of this boy’s constipation.

Prescribing Movicol would be beneficial and the outreach team can also be involved to reveal any underlying
issues with compliance.

https://mypastest.pastest.com/Secure/TestMe/Browser/436619[‫ ص‬07:16:30 09/12/1437]


MyPastest

A CAMHS referral is appropriate due to the obvious self-harm. The urgency


of this referral will depend on his
current ideas about self-harm and suicide, and whether he is at immediate risk of harming himself or others.
Previous Question Next Question
Clinical psychology will play an important role in improving this boy’s psychological well-being and likely his
physical symptoms.

https://www.nice.org.uk/guidance/cg99 46698

End Session

Tag Question

Feedback

Difficulty: Average

Peer Responses

Session Progress

Responses Correct: 0

Responses Incorrect: 52

Responses Total: 52

Responses - % Correct: 0%

Blog
About Pastest
Contact Us
Help

© Pastest 2016

https://mypastest.pastest.com/Secure/TestMe/Browser/436619[‫ ص‬07:16:30 09/12/1437]


MyPastest

Prefer to use the old MyPastest? Access it here »

Back to Filters

Question 51 of 78

A 6-year-old girl was seen on the children’s assessment


unit yesterday. She was started on a Movicol disimpaction
regime. She
has re-presented today with increasing abdominal pain. You assess her and are confident with a
clinical diagnosis of faecal impaction.

What is the most appropriate next course of action?

A Continue disimpaction regime

B Perform an abdominal X-ray to confirm the diagnosis

C Perform an abdominal ultrasound to confirm the diagnosis

D Phosphate enema

E Stop Movicol and change to lactulose

Explanation
The correct answer is to continue the disimpaction regime. Families and children should be informed that
disimpaction treatment can initially increase symptoms of soiling and abdominal pain.

A plain abdominal radiograph should not be used to make a diagnosis of idiopathic constipation. According to
NICE guidelines, a plain X-ray should only be used in the ongoing management of intractable idiopathic
constipation.

Abdominal US should not be


used to make a diagnosis of idiopathic constipation. According to NICE guidelines,
an abdominal US should only be used in the ongoing management of intractable idiopathic constipation.

Phosphate enemas should only be used if all oral medications and sodium citrate enemas have failed.

NICE
guidelines state that Movicol should be substituted with lactulose only
if Movicol is not tolerated. This is
not the case here.

https://mypastest.pastest.com/Secure/TestMe/Browser/436619#Top[‫ ص‬07:24:42 09/12/1437]


MyPastest

https://www.nice.org.uk/guidance/cg99 46699

Previous Question Next Question

Tag Question

Feedback
End Session

Difficulty: Average

Peer Responses

Session Progress

Responses Correct: 0

Responses Incorrect: 53

Responses Total: 53

Responses - % Correct: 0%

Blog
About Pastest
Contact Us
Help

© Pastest 2016

https://mypastest.pastest.com/Secure/TestMe/Browser/436619#Top[‫ ص‬07:24:42 09/12/1437]


MyPastest

Prefer to use the old MyPastest? Access it here »

Back to Filters

Question 52 of 78

A male infant is brought to the emergency department by his parents. He was born at 34 weeks by spontaneous
vaginal delivery and was discharged from SCBU 4 weeks ago. He is not on any regular medication. Parents tell
you that he brings up small volumes of milk after feeds. This happens approximately twice a day. Observations are
all within normal range and examination is unremarkable.

What is the MOST likely diagnosis?

A Cows’ milk protein intolerance

B Gastro-oesophageal reflux

C Meningitis

D Overfeeding

E Pyloric stenosis

Explanation
The correct answer is B. Immaturity of the lower
oesophageal sphincter results in frequent transient relaxation,
and retrograde flow of gastric contents into the oesophagus.

Although
a diagnosis of cows' milk protein intolerance should be considered, in the absence of lower GI
symptoms or history of atopy/allergy, gastro-oesophageal reflux is the more likely diagnosis.

Meningitis
can present with poor feeding and/or vomiting in an infant. However, this baby is feeding well and is
otherwise well. The possets are related to feeds and only occur twice a day, making B the more likely diagnosis.

A careful history should be taken to pick up overfeeding. However, there is no indication within the information
provided that the baby is being overfed so D is unlikely.

Pyloric stenosis typically (but not exclusively) occurs in first born male infants at 4-6 weeks of age. However,
vomiting would occur after most/all feeds and would be projectile in nature.

https://mypastest.pastest.com/Secure/TestMe/Browser/436619#Top[‫ ص‬07:30:49 09/12/1437]


MyPastest

46700

Previous Question Next Question

Tag Question

Feedback End Session

Difficulty: Average

Peer Responses

Session Progress

Responses Correct: 0

Responses Incorrect: 54

Responses Total: 54

Responses - % Correct: 0%

Blog
About Pastest
Contact Us
Help

© Pastest 2016

https://mypastest.pastest.com/Secure/TestMe/Browser/436619#Top[‫ ص‬07:30:49 09/12/1437]


MyPastest

Prefer to use the old MyPastest? Access it here »

Back to Filters

Question 52 of 78

A male infant is brought to the emergency department by his parents. He was born at 34 weeks by spontaneous
vaginal delivery and was discharged from SCBU 4 weeks ago. He is not on any regular medication. Parents tell
you that he brings up small volumes of milk after feeds. This happens approximately twice a day. Observations are
all within normal range and examination is unremarkable.

What is the MOST likely diagnosis?

A Cows’ milk protein intolerance

B Gastro-oesophageal reflux

C Meningitis

D Overfeeding

E Pyloric stenosis

Explanation
The correct answer is B. Immaturity of the lower
oesophageal sphincter results in frequent transient relaxation,
and retrograde flow of gastric contents into the oesophagus.

Although
a diagnosis of cows' milk protein intolerance should be considered, in the absence of lower GI
symptoms or history of atopy/allergy, gastro-oesophageal reflux is the more likely diagnosis.

Meningitis
can present with poor feeding and/or vomiting in an infant. However, this baby is feeding well and is
otherwise well. The possets are related to feeds and only occur twice a day, making B the more likely diagnosis.

A careful history should be taken to pick up overfeeding. However, there is no indication within the information
provided that the baby is being overfed so D is unlikely.

Pyloric stenosis typically (but not exclusively) occurs in first born male infants at 4-6 weeks of age. However,
vomiting would occur after most/all feeds and would be projectile in nature.

https://mypastest.pastest.com/Secure/TestMe/Browser/436619#Top[‫ ص‬07:31:31 09/12/1437]


MyPastest

46700

Previous Question Next Question

Tag Question

Feedback End Session

Difficulty: Average

Peer Responses

Session Progress

Responses Correct: 0

Responses Incorrect: 54

Responses Total: 54

Responses - % Correct: 0%

Blog
About Pastest
Contact Us
Help

© Pastest 2016

https://mypastest.pastest.com/Secure/TestMe/Browser/436619#Top[‫ ص‬07:31:31 09/12/1437]


MyPastest

Prefer to use the old MyPastest? Access it here »

Back to Filters

Question 53 of 78

A 7-year-old boy gets recurrent attacks of intense nausea, retching and vomiting. He is lethargic, his eyes are
sunken and he has not passed urine all day.

Which of the following is the MOST appropriate action?

A Cyclizine and oral fluids

B IV fluid bolus then IV maintenance fluids

C IV maintenance fluids

D Juice and carbonated drinks

E Oral fluid challenge

Explanation
The correct answer is B. This child is severely dehydrated and needs an IV fluid bolus, as well as maintenance
therapy. The recurrence of the vomiting points towards a diagnosis of cyclical vomiting.

Answer A is incorrect. It is unlikely that this boy will tolerate sufficient oral fluids and rehydration will require the
use of IV fluids.

C is incorrect. The signs are suggestive of severe dehydration and a fluid bolus in indicated.

D is incorrect. Juice and carbonated drinks are contraindicated, and are likely to result in more vomiting.

E is incorrect. It is unlikely that this boy will tolerate sufficient


oral fluids and rehydration will require the use of
IV fluids.

https://www.nice.org.uk/guidance/cg84/chapter/key-priorities-for-implementation 46701

https://mypastest.pastest.com/Secure/TestMe/Browser/436619#Top[‫ ص‬07:31:52 09/12/1437]


MyPastest

Previous Question Next Question


Tag Question

Feedback

End Session
Difficulty: Average

Peer Responses

Session Progress

Responses Correct: 0

Responses Incorrect: 55

Responses Total: 55

Responses - % Correct: 0%

Blog
About Pastest
Contact Us
Help

© Pastest 2016

https://mypastest.pastest.com/Secure/TestMe/Browser/436619#Top[‫ ص‬07:31:52 09/12/1437]


MyPastest

Prefer to use the old MyPastest? Access it here »

Back to Filters

Question 54 of 78

A 12-year-old boy presents with a spreading non-blanching petechial rash, photophobia, neck stiffness and
vomiting. His mother and sister have returned from a holiday to Disneyland in Florida, and have both been unwell
with diarrhoea and vomiting.

Which investigation will confirm the MOST likely diagnosis?

A Blood culture

B CRP

C Lumbar puncture

D Sputum sample for AFB

E Stool culture

Explanation
The correct investigation is lumbar puncture. The most likely diagnosis given the child’s symptoms is
meningococcal meningitis. Treatment with IV ceftriaxone should be commenced immediately.

A is incorrect. Blood culture should be performed and may or may not be positive. However, confirmation in the
cerebrospinal fluid (CSF) will allow you to determine the accurate diagnosis and the length of treatment with IV
antibiotics.

B is incorrect. C-Reactive Protein should be performed as part of the septic screen, but this will not confirm the
diagnosis.

D
is incorrect. The presence of acid fast bacilli would diagnose tuberculosis, which is not the most likely diagnosis
in this case.

E
is incorrect because gastroenteritis is not the most likely diagnosis, despite the other family members having
symptoms of gastroenteritis.

https://mypastest.pastest.com/Secure/TestMe/Browser/436619#Top[‫ ص‬07:33:03 09/12/1437]


MyPastest

http://www.meningitis.org/assets/x/50631 46702

Previous Question Next Question

Tag Question
End Session
Feedback

Difficulty: Average

Peer Responses

Session Progress

Responses Correct: 0

Responses Incorrect: 56

Responses Total: 56

Responses - % Correct: 0%

Blog
About Pastest
Contact Us
Help

© Pastest 2016

https://mypastest.pastest.com/Secure/TestMe/Browser/436619#Top[‫ ص‬07:33:03 09/12/1437]


MyPastest

Prefer to use the old MyPastest? Access it here »

Back to Filters

Question 55 of 78

A 4-week-old infant presents with vomiting. He was born at 39 weeks by spontaneous vaginal delivery (SVD)
weighing 3.6kg, and he was treated with 48 hours of IV antibiotics due to maternal pyrexia in labour and
prolonged rupture of membranes. You are not sure what his CRP was at the time. He had become jaundiced on
day 2, but when a bilirubin level was taken it was below the treatment line. He has been started on Gaviscon infant
by the GP which mother has been adding to all
his formula feeds. Despite this, the vomiting has continued and she
thinks he needs further investigations for reflux. On examination, his CRT is < 2 seconds, his weight is 4.5kg, his
fontanelle feels soft and he passes urine as you open the nappy.

What is the next MOST appropriate course of action?

A Advise mother to reduce formula feeds

B Arrange a pH study

C Arrange an impedance study

D Change to a hydrolysed formula

E Urgent blood culture

Explanation
This infant’s weight has gone from the 25th
centile to the 75th centile within four weeks. The most likely cause is
overfeeding. A careful history should be taken to determine how much
milk he is being given and the mother
advised to reduce formula feeds.

B
and C are unhelpful in arriving at a diagnosis. pH oesophageal impedance monitoring would confirm a
diagnosis of gastro-oesophageal reflux, which is not the most likely diagnosis here.

D is incorrect. A hydrolysed formula would be appropriate if cows’ milk protein intolerance was suspected. This
infant is feeding very well and is tolerating feeds well. If vomiting continues after overfeeding is corrected, then
other diagnoses should be considered.

https://mypastest.pastest.com/Secure/TestMe/Browser/436619#Top[‫ ص‬07:33:47 09/12/1437]


MyPastest

E is incorrect. Sepsis is unlikely in a very well baby who is feeding well. However, it is always important to think
about sepsis as a possible cause for vomiting.
Previous Question Next Question 46703

End Session
Tag Question

Feedback

Difficulty: Average

Peer Responses

Session Progress

Responses Correct: 0

Responses Incorrect: 57

Responses Total: 57

Responses - % Correct: 0%

Blog
About Pastest
Contact Us
Help

© Pastest 2016

https://mypastest.pastest.com/Secure/TestMe/Browser/436619#Top[‫ ص‬07:33:47 09/12/1437]


MyPastest

Prefer to use the old MyPastest? Access it here »


Previous Question Next Question

Back to Filters

End Session
Question 56 of 78

A term baby is seen on the postnatal ward with vomiting. He is two hours of age. The obstetrician informs you
that they have just commenced mother on IV antibiotics for likely sepsis.

What is the most appropriate course of action for the baby?

A Cannulate baby, take blood and culture and start IV antibiotics

B Cannulate baby, take blood and culture and wait for the CRP

C Cannulate baby, take bloods and wait for the CRP result

D Observe the baby closely on the neonatal unit

E Observe the baby on transitional care

Explanation
A is the correct answer. Maternal sepsis is an indication to commence IV antibiotics in the baby, because the risk
of early onset neonatal sepsis from trans-placental transmission is high.

All other answers are incorrect. Antibiotics should be started in the baby because of the risk of early onset
neonatal sepsis.
46704

Tag Question

Feedback

https://mypastest.pastest.com/Secure/TestMe/Browser/436619#Top[‫ ص‬07:34:44 09/12/1437]


MyPastest

Difficulty: Average

Peer Responses

Session Progress

Responses Correct: 0

Responses Incorrect: 58

Responses Total: 58

Responses - % Correct: 0%

Blog
About Pastest
Contact Us
Help

© Pastest 2016

https://mypastest.pastest.com/Secure/TestMe/Browser/436619#Top[‫ ص‬07:34:44 09/12/1437]


MyPastest

Prefer to use the old MyPastest? Access it here »

Back to Filters

Question 57 of 78

A 14-year-old girl is seen on the children’s assessment


unit with abdominal pain. The pain started yesterday
evening and paracetamol does not help. On examination, she is tender in the right iliac fossa, and there is a
palpable adnexal mass.

What is the MOST likely diagnosis?

A Acute appendicitis

B Constipation

C Ectopic pregnancy

D Mittelschmerz

E UTI

Explanation
The most likely diagnosis is C. The classical clinical triad of ectopic pregnancy is pain, amenorrhoea and vaginal
bleeding. 75% have abdominal tenderness, and 50% have a palpable adnexal
mass. Further history should be taken
and any concerns about safeguarding discussed with a senior.

Although RIF pain may suggest a diagnosis of appendicitis, the palpable “adnexal” mass is more suggestive of
uterine pathology so stem A is incorrect.

Constipation
can present with abdominal pain but this is more often generalised. In cases of constipation, a faecal
mass is more likely to be palpable in the left iliac fossa, or throughout the abdomen.

Mittelschmerz (middle pain or pain in the middle of a menstrual cycle) is wrong as this would not explain the
palpable mass.

Although
a urine dipstick is a reasonable investigation to perform in a child presenting with abdominal pain, a UTI
would not explain the palpable mass.

https://mypastest.pastest.com/Secure/TestMe/Browser/436619#Top[‫ ص‬07:35:05 09/12/1437]


MyPastest

46999

Previous Question Next Question

Tag Question

Feedback End Session

Difficulty: Average

Peer Responses

Session Progress

Responses Correct: 0

Responses Incorrect: 59

Responses Total: 59

Responses - % Correct: 0%

Blog
About Pastest
Contact Us
Help

© Pastest 2016

https://mypastest.pastest.com/Secure/TestMe/Browser/436619#Top[‫ ص‬07:35:05 09/12/1437]


MyPastest

Prefer to use the old MyPastest? Access it here »


Next Question

Back to Filters

Question 58 of 78

A 4-year-old girl presents with vomiting and abdominal pain. She has been prescribed Movicol for constipation
but does not take it regularly.

Which is the NEXT most appropriate investigation?

A Abdominal ultrasound

B Abdominal X-ray

C Amylase

D Bloods including CRP

E Urine dipstick

Explanation
Constipation is a risk factor for urinary tract infection, which is important to test for in this 4-year-old gir so E is
correct.

A is incorrect. Abdominal ultrasound may be required if symptoms continue and are unexplained but it is
important to rule out
a UTI first.

B is incorrect. Abdominal X-ray may be required if symptoms are unexplained but it is important to rule out a UTI
first.

C and D are also incorrect. If a urine dipstick shows evidence of a UTI and she is otherwise systemically well,
venepuncture may not be indicated.
47000

https://mypastest.pastest.com/Secure/TestMe/Browser/436619#Top[‫ ص‬07:35:29 09/12/1437]


MyPastest

Tag Question
Previous Question
Feedback

Difficulty: Average

Peer Responses End Session

Session Progress

Responses Correct: 0

Responses Incorrect: 60

Responses Total: 60

Responses - % Correct: 0%

Blog
About Pastest
Contact Us
Help

© Pastest 2016

https://mypastest.pastest.com/Secure/TestMe/Browser/436619#Top[‫ ص‬07:35:29 09/12/1437]


MyPastest

Prefer to use the old MyPastest? Access it here »

Back to Filters

Question 59 of 78

A 10-year-old girl is seen in the emergency department with abdominal pain. She had a sore throat a week ago but
no significant past
medical history of note. On examination, she has mild tenderness in the right iliac fossa, but no
guarding or rebound tenderness. Her urine
dipstick is normal. Bloods results are
Hb120, WCC 7.9, Plts312, Na 139, K 3.6, Creat 52, Urea 2.5, Protein 64, AST 21, Bili 8, Alb 40, Adj Ca 2.31,
CRP 2.

What is the MOST likely diagnosis?

A Appendicitis

B Mesenteric adenitis

C Psychosomatic pain

D Pyelonephritis

E Trauma

Explanation
The most likely diagnosis is mesenteric adenitis,
a self-limiting inflammatory condition that affects the mesenteric
lymph nodes. The differential diagnosis includes appendicitis. Depending on the clinical condition, referral to a
surgeon to consider a
diagnosis of appendicitis may still be an appropriate course of action.
However, in this case
the child is well and inflammatory markers are not raised.

Although appendicitis is an important diagnosis to consider, B is more likely in this case.

Psychosomatic pain would be a diagnosis of exclusion and is not correct.

Pyelonephritis is highly unlikely in the absence of a positive urine dipstick.

There is no history of trauma so E is incorrect.


47001

https://mypastest.pastest.com/Secure/TestMe/Browser/436619#Top[‫ ص‬07:35:51 09/12/1437]


MyPastest

Previous Question Next Question


Tag Question

Feedback

End Session
Difficulty: Average

Peer Responses

Session Progress

Responses Correct: 0

Responses Incorrect: 61

Responses Total: 61

Responses - % Correct: 0%

Blog
About Pastest
Contact Us
Help

© Pastest 2016

https://mypastest.pastest.com/Secure/TestMe/Browser/436619#Top[‫ ص‬07:35:51 09/12/1437]


MyPastest

Prefer to use the old MyPastest? Access it here »


Next Question

Back to Filters

Question 60 of 78

An 8-year-old girl presents with sudden onset abdominal pain.


One month ago she was admitted with a sudden
severe pain in her left arm. She is under follow up because of a haematological condition.

What is the MOST likely underlying condition?

A Beta thalassaemia

B Beta thalasseamia trait

C Iron deficiency anaemia

D Sickle cell anaemia

E Spherocytosis

Explanation
Sickle cell anaemia is the most likely underlying
condition. This is a chronic haemolytic anaemia where vaso-
occlusive crises can be precipitated by hypoxaemia, acute febrile illness and dehydration. This patient has
presented in abdominal crisis.

A and B are wrong. Thalassaemias are a group of hereditary conditions that


cause anaemia, but are not associated
with painful crises.

C is incorrect. Iron deficiency anaemia would not explain the painful events.

E is incorrect. Hereditary haemochromatosis is a haemolytic anaemic characterised by “sphere shaped” rather than
biconcave erythrocytes. Children with spherocytosis are at risk of gallstones, however the previous left arm pain in
this child makes sickle cell anaemia the more likely diagnosis.
47002

https://mypastest.pastest.com/Secure/TestMe/Browser/436619#Top[‫ ص‬07:42:35 09/12/1437]


MyPastest

Previous Question Tag Question

Feedback

Difficulty: Average
End Session
Peer Responses

Session Progress

Responses Correct: 0

Responses Incorrect: 62

Responses Total: 62

Responses - % Correct: 0%

Blog
About Pastest
Contact Us
Help

© Pastest 2016

https://mypastest.pastest.com/Secure/TestMe/Browser/436619#Top[‫ ص‬07:42:35 09/12/1437]


MyPastest

Prefer to use the old MyPastest? Access it here »

Back to Filters

Question 61 of 78

A 6-year-old boy presents with joint pain, rash and severe abdominal pain. From mother’s description, the rash has
appeared in crops on his legs over the past week.

What is the MOST likely diagnosis?

A Gastric bleed

B Gastric ulcer

C Henoch Schonlein purpura

D Systemic lupus erythematosus

E Varicella zoster (chicken pox)

Explanation
The most likely diagnosis is C, Henoch Schonlein Purpura. This is a vasculitis that most commonly occurs in
children under the age of ten. It causes a purpuric rash, usually over the legs and buttocks, joint pain and
abdominal pain. Patients should be monitored for renal complications by checking BP, urine dipsticks (to check
for haematuria and proteinuria) and examination for oedema.

Answers
A and B are incorrect. Abdominal pain in HSP is caused by oedema and inflammation in the bowel wall
and mesentery, not from gastric pathology.

D is incorrect. The rash is not characteristic of the rashes that occur in SLE which are characteristically described
as a
malar rash - an erythema over the cheeks and nasal bridge. Although joint pain occurs, abdominal pain is not
common.

E is incorrect. In chickenpox, vesicles occur in crops and usually begin on the torso.
47003

https://mypastest.pastest.com/Secure/TestMe/Browser/436619#Top[‫ ص‬07:43:48 09/12/1437]


MyPastest

Previous Question Next Question


Tag Question

Feedback

End Session
Difficulty: Average

Peer Responses

Session Progress

Responses Correct: 0

Responses Incorrect: 63

Responses Total: 63

Responses - % Correct: 0%

Blog
About Pastest
Contact Us
Help

© Pastest 2016

https://mypastest.pastest.com/Secure/TestMe/Browser/436619#Top[‫ ص‬07:43:48 09/12/1437]


MyPastest

Prefer to use the old MyPastest? Access it here »

Back to Filters

Question 62 of 78

A female infant is brought to the emergency department by her


parents. She was discharged from the postnatal
ward three days ago, following a normal vaginal delivery at full term. Pregnancy and delivery had been
uneventful. There were no maternal risk factors for sepsis, membranes ruptured 14 hours prior to delivery and
liquor was clear. Today the baby vomited dark brown blood which was mixed in with milk. She is otherwise well
and feeding well.

What is the MOST likely diagnosis?

A Ingested blood during feeding

B Ingestion of blood stained liquor

C Mallory Weiss tear

D Oesophageal varices

E Pyloric stenosis

Explanation
The most likely answer here is ingested blood during feeding. The two most likely causes in cases such as
described are swallowed blood from cracked nipples during breastfeeding, or swallowed blood-stained liquor at
delivery. However, the history notes clear liquor in this instance so the former is more likely.

Ingestion of blood stained liquor is unlikely because the liquor was reported as clear at delivery so B is incorrect.

A Mallory Weiss tear is a laceration in the mucosa at the junction between the stomach and oesophagus, and
usually occurs after severe vomiting. There is no history of recurrent episodes of severe vomiting in this infant so
C is incorrect.

Oesophageal varices most commonly occur as a result of portal hypertension. Biliary atresia and cirrhosis related
cystic fibrosis are possible causes of this in childhood. There is no suggestion of such pathology in the stem of the

https://mypastest.pastest.com/Secure/TestMe/Browser/436619#Top[‫ ص‬07:44:50 09/12/1437]


MyPastest

question so D is incorrect.

Pyloric stenosis, typically, (but not exclusively) occurs in first born male infants at 4-6 weeks of age. However,
Previous Question Next Question
vomiting would occur after most/all feeds, would be projectile
in nature, and would not contain blood so E is
incorrect..
47004

End Session

Tag Question

Feedback

Difficulty: Average

Peer Responses

Session Progress

Responses Correct: 0

Responses Incorrect: 64

Responses Total: 64

Responses - % Correct: 0%

Blog
About Pastest
Contact Us
Help

© Pastest 2016

https://mypastest.pastest.com/Secure/TestMe/Browser/436619#Top[‫ ص‬07:44:50 09/12/1437]


MyPastest

Prefer to use the old MyPastest? Access it here »

Back to Filters

Question 63 of 78

An 8-year-old boy presents with haematemesis. He had vomited


a profuse amount of bright red blood and was
therefore brought in by ambulance. On examination of his throat, you see a blood vessel on his left tonsil that is
bleeding, and he vomits more blood after ENT examination.

Which of these is the LEAST appropriate course of action action?

A Admit and observe

B Bloods for FBC, clotting, group and save

C ENT referral

D Throat swab

E Tranexamic acid

Explanation
This child most probably has haemorrhagic tonsillitis, and a throat swab is likely to result in more bleeding. It is
therefore not an appropriate course of action so D is the correct answer.

The child should be admitted for observation and appropriate monitoring so A is incorrect.

B is incorrect. It is important to know the child’s haemoglobin, coagulation status, and to take a group and save
sample in case a blood transfusion is required. The child may be continually swallowing blood, so haemoglobin
and fluid status should be monitored.

C is incorrect. ENT advice should be sought.

E is also incorrect. Tranexamic may or may not be indicated, depending on the severity of the bleeding. However,
it is not the least appropriate action.
47005

https://mypastest.pastest.com/Secure/TestMe/Browser/436619#Top[‫ ص‬07:45:32 09/12/1437]


MyPastest

Previous Question Next Question


Tag Question

Feedback

End Session
Difficulty: Average

Peer Responses

Session Progress

Responses Correct: 0

Responses Incorrect: 65

Responses Total: 65

Responses - % Correct: 0%

Blog
About Pastest
Contact Us
Help

© Pastest 2016

https://mypastest.pastest.com/Secure/TestMe/Browser/436619#Top[‫ ص‬07:45:32 09/12/1437]


MyPastest

Prefer to use the old MyPastest? Access it here »

Back to Filters

Question 64 of 78

A 15-year-old girl is seen on the children’s assessment


unit (CAU). She has vomited eight times today and has had
loose watery
stools. In her last vomit there were streaks of fresh red blood. Her observations are all within range,
and examination is unremarkable.

What is the most appropriate course of action?

A Admit and observe

B Discharge

C Discharge with follow up in outpatients clinic

D Observe on CAU, then discharge if well

E Refer to surgeons

Explanation
Observe on CAU then discharge if well is the most
appropriate course of action in this scenario. The most likely
diagnosis is a Mallory Weiss tear, however, considering the multiple episodes of vomiting and diarrhoea, she
would benefit from a period of observation to ensure that she is tolerating fluids.

Stem A is incorrect. She is not clinically dehydrated and does not need admission
based on the information given
in the question.

Stem B is incorrect. Although it is likely that she will be discharged, she should be tolerating adequate fluids first.

Stem C is incorrect. A Mallory Weiss tear is an inappropriate reason to refer to a Paediatric outpatient clinic.

Stem E is also incorrect. A Mallory Weiss tear is an inappropriate reason to refer to surgeons.
47006

https://mypastest.pastest.com/Secure/TestMe/Browser/436619#Top[‫ ص‬07:45:56 09/12/1437]


MyPastest

Previous Question Next Question


Tag Question

Feedback

End Session
Difficulty: Average

Peer Responses

Session Progress

Responses Correct: 0

Responses Incorrect: 66

Responses Total: 66

Responses - % Correct: 0%

Blog
About Pastest
Contact Us
Help

© Pastest 2016

https://mypastest.pastest.com/Secure/TestMe/Browser/436619#Top[‫ ص‬07:45:56 09/12/1437]


MyPastest

Prefer to use the old MyPastest? Access it here »

Back to Filters

Question 65 of 78

A 3-month-old male infant presents with episodes of drawing up his legs, crying and redcurrant-jelly like stools.

What is the MOST appropriate investigation to confirm the most likely diagosis?

A Abdominal CT

B Abdominal MRI

C Abdominal ultrasound

D Abdominal X-ray

E CRP

Explanation
Abdominal ultrasound is the correct answer. The most likely diagnosis is Intussusception. Ultrasound is the
investigation of choice due to its high accuracy and lack of radiation. A ‘doughnut shaped’ mass can be identified
on ultrasound.

A CT would confirm the diagnosis but would expose the patient to unnecessary radiation, which can be avoided
by ultrasound.

Whilst abdominal MRI would confirm the diagnosis, ultrasound is likely to be more readily available and is
therefore the most appropriate investigation to request.

Abdominal X-ray may be needed if an ultrasound cannot be done immediately and there are concerns about
intestinal obstruction of free intraperitoneal gas.

C-reactive protein (CRP) is a substance produced in the liver in response to inflammation. It is useful for example,
in assessing patient with, inflammatory bowel disease, and is unlikely to lead to confirmation of the diagnosis in
this case.
47007

https://mypastest.pastest.com/Secure/TestMe/Browser/436619#Top[‫ ص‬07:46:19 09/12/1437]


MyPastest

Previous Question Next Question


Tag Question

Feedback

End Session
Difficulty: Average

Peer Responses

Session Progress

Responses Correct: 0

Responses Incorrect: 67

Responses Total: 67

Responses - % Correct: 0%

Blog
About Pastest
Contact Us
Help

© Pastest 2016

https://mypastest.pastest.com/Secure/TestMe/Browser/436619#Top[‫ ص‬07:46:19 09/12/1437]


MyPastest

Prefer to use the old MyPastest? Access it here »


Next Question

Back to Filters

Question 66 of 78

The parents of a 4-month-old girl bring her to clinic. She was most recently seen in clinic due to blood in her stool,
which was thought to be secondary to cows' milk protein intolerance. Until three weeks ago, mother had been
exclusively breastfeeding. When formula milk
was introduced at that time, she had severe swelling around her
lips, face and eyes. After discussing with your consultant, you decide to do further investigations.

What is the SINGLE investigation of choice?

A Basophil histamine release

B Lymphocyte stimulation

C Mediator release assay

D Serum specific IgG tests

E Specific IgE antibody test

Explanation
The correct answer is specific IgE antibody test. This child has symptoms of IgE mediated
cows' milk protein
(CMP) allergy. It is therefore appropriate to investigate with a specific IgE antibody test, or with a skin prick test.
However, it is important to note that both of these tests have a wide range of sensitivities and specificities.

Stems A, B, C and D are incorrect. Guidelines issued by NICE and by ESPGHAN suggest that these tests are not
recommended for diagnosing food allergies in infants and children.

http://cks.nice.org.uk/cows-milk-protein-allergy-in-children#!diagnosissub
47008

https://mypastest.pastest.com/Secure/TestMe/Browser/436619#Top[‫ ص‬07:46:55 09/12/1437]


MyPastest

Tag Question

Previous Question
Feedback

Difficulty: Average

Peer Responses
End Session

Session Progress

Responses Correct: 0

Responses Incorrect: 68

Responses Total: 68

Responses - % Correct: 0%

Blog
About Pastest
Contact Us
Help

© Pastest 2016

https://mypastest.pastest.com/Secure/TestMe/Browser/436619#Top[‫ ص‬07:46:55 09/12/1437]


MyPastest

Prefer to use the old MyPastest? Access it here »


Next Question

Back to Filters

Question 67 of 78

A 9-year-old boy presents with fresh red blood mixed in with his stools. He says he has had this before, but is not
sure how long for. On examination, he looks pale but is clinically well hydrated. There are patches of
hyperpigmentation in the mouth, and on his hands and feet.

What is the MOST likely diagnosis?

A Addison’s disease

B McCune Albright syndrome

C Meckel’s diverticulum

D Peutz-Jeghers syndrome

E Salmonella

Explanation
This patient is most likely to have Peutz-Jeghers
syndrome, an autosomal dominant condition characterised by
mucocutaneous lesions, and hamartomatous polyps in the GI tract. These polyps may bleed.

The differential diagnoses for the hyperpigmented lesions are Addison’s disease and McCune Albright syndrome.
However, neither of these are a cause of PR bleeding. Stems A
and B are therefore incorrect.

Meckel’s diverticulum is a
cause of PR bleeding, but this diagnosis would not explain the lesions in the mouth,
hands and feet. Stem C is therefore incorrect.

Salmonella food poisoning is another cause of PR bleeding, but would not explain the oral and skin lesions.
47009

https://mypastest.pastest.com/Secure/TestMe/Browser/436619#Top[‫ ص‬07:47:15 09/12/1437]


MyPastest

Tag Question
Previous Question
Feedback

Difficulty: Average

Peer Responses End Session

Session Progress

Responses Correct: 0

Responses Incorrect: 69

Responses Total: 69

Responses - % Correct: 0%

Blog
About Pastest
Contact Us
Help

© Pastest 2016

https://mypastest.pastest.com/Secure/TestMe/Browser/436619#Top[‫ ص‬07:47:15 09/12/1437]


MyPastest

Prefer to use the old MyPastest? Access it here »

Back to Filters

Question 68 of 78

A 4-year-old girl is seen in the emergency department with profuse diarrhoea which contains blood. The family
ate some takeaway chicken two days ago and multiple family members have been unwell.

What is the MOST likely cause of this child’s bloody diarrhoea?

A Clostridium difficile

B E. coli

C Haemorrhoids

D Salmonella

E Shigella

Explanation
The correct answer is D, salmonella food poisoning. Salmonellosis is contracted by eating food contaminated with
salmonella; most often poultry and eggs. It can cause diarrhoea containing blood, fever and abdominal cramps.

C. difficile
most commonly occurs after hospitalisation, or after antibiotic treatment. Neither of these risk factors
are present in this child.

E. coli
can cause bloody diarrhoea. However, Salmonella more commonly occurs after contaminated chicken
ingestion so is more likely in this case.

Although haemorrhoids may cause fresh red blood to be passed, there is often also itchiness at the anus, and there
is no association with ingestion of takeaway chicken. Haemorrhoids are uncommon in children and where it does
occur there is usually a history of inability to produce a bowel movement/constipation resulting in large hard
stools causing the child to strain to pass the stools.

Shigellosis is possible and is transmitted via the faeco-oral route, but not typically with poultry. Salmonella is
therefore the most likely diagnosis.

https://mypastest.pastest.com/Secure/TestMe/Browser/436619#Top[‫ ص‬07:47:40 09/12/1437]


MyPastest

47010

Previous Question Next Question

Tag Question

Feedback End Session

Difficulty: Average

Peer Responses

Session Progress

Responses Correct: 0

Responses Incorrect: 70

Responses Total: 70

Responses - % Correct: 0%

Blog
About Pastest
Contact Us
Help

© Pastest 2016

https://mypastest.pastest.com/Secure/TestMe/Browser/436619#Top[‫ ص‬07:47:40 09/12/1437]


MyPastest

Prefer to use the old MyPastest? Access it here »

Back to Filters

Question 69 of 78

A 4-year-old boy is referred for recurrent episodes of diarrhoea and failure to thrive. An OGD is performed and
the results of
the duodenal biopsy are awaited.

What is the MOST likely diagnosis?

A Coeliac disease

B Crohn’s disease

C Duodenal ulcer

D Recurrent viral gastroenteritis

E Ulcerative colitis

Explanation
Coeliac disease is the correct answer. Children with coeliac disease may present with failure to thrive, diarrhoea,
muscle wasting and abdominal distension. Duodenal biopsy remains the gold standard investigation for coeliac
disease, and total villous atrophy is the most typical histological feature.

Crohn’s disease can occur at any point along the gastrointestinal tract. Microscopic features can be highly
variable, and include increased lamina propria plasma cells, transmural inflammation, skip lesions and submucosal
fibrosis. Features are not limited to the duodenum.

The duodenum is the most common site for a peptic ulcer, however, symptoms of diarrhoea and failure to thrive in
a 4-year-old are more fitting with a diagnosis of coeliac disease.

Recurrent viral gastroenteritis should be considered. There is, however, also failure to thrive and sufficient
concern to lead to an OGD (oesophago-gastro- duodenoscopy - also known simply as a gastroscopy or endoscopy)
and biopsy. Coeliac is therefore more likely.

Colorectal, not duodenal biopsy can be helpful in the diagnosis of ulcerative colitis.

https://mypastest.pastest.com/Secure/TestMe/Browser/436619#Top[‫ ص‬07:48:21 09/12/1437]


MyPastest

http://www.bsg.org.uk/pdf_word_docs/ideopathic.pdf
47047
Previous Question Next Question

Tag Question
End Session
Feedback

Difficulty: Average

Peer Responses

Session Progress

Responses Correct: 0

Responses Incorrect: 71

Responses Total: 71

Responses - % Correct: 0%

Blog
About Pastest
Contact Us
Help

© Pastest 2016

https://mypastest.pastest.com/Secure/TestMe/Browser/436619#Top[‫ ص‬07:48:21 09/12/1437]


MyPastest

Prefer to use the old MyPastest? Access it here »

Back to Filters

Question 70 of 78

A 6-year-old boy presents with polydipsia, polyuria and unexplained weight loss. On examination, he is lethargic
and thin. His
blood sugar is 34mmol/mol. His urine dipstick shows the presence of glucose and ketones, and his
blood ketones are 4.0. You are about to take bloods, including a blood gas.

Which ONE of these conditions should be screened for at presentation?

A Coeliac disease

B Diabetic retinopathy

C Diabetic neuropathy

D Diabetic nephropathy

E Multiple endocrine neoplasia type 1

Explanation
The correct answer is A. Coeliac disease, which is more prevalent in patients with type one diabetes, and NICE
guidelines state that it should be screened for in patients presenting with T1D. TFTs should also be sent to screen
for thyroid disease.

B, C and D develop over time. Annual reviews in patients with T1D should ensure that all necessary screening is
being undertaken. These conditions, however, are not routinely screened for at diagnosis.

E is incorrect. MEN type 1 (Werner syndrome) consists of tumours of the pancreas, pituitary and parathyroid
gland. Excess glucagon causes diabetes mellitus. There is no suggestion in the stem of the question that any other
endocrine glands are affected, and screening for MEN is not routinely performed.

https://www.nice.org.uk/guidance/ng20/chapter/key-priorities-for-implementation
47048

https://mypastest.pastest.com/Secure/TestMe/Browser/436619#Top[‫ ص‬07:48:42 09/12/1437]


MyPastest

Previous Question Next Question


Tag Question

Feedback

End Session
Difficulty: Average

Peer Responses

Session Progress

Responses Correct: 0

Responses Incorrect: 72

Responses Total: 72

Responses - % Correct: 0%

Blog
About Pastest
Contact Us
Help

© Pastest 2016

https://mypastest.pastest.com/Secure/TestMe/Browser/436619#Top[‫ ص‬07:48:42 09/12/1437]


MyPastest

Prefer to use the old MyPastest? Access it here »


Next Question

Back to Filters

Question 71 of 78

A patient with IgA deficiency is referred to your clinic. You suspect a diagnosis of coeliac disease and want to
send a blood sample.

What is the MOST appropriate blood test to perform?

A IgA tissue transglutaminase (tTG)

B IgA endomysial antibody (EMA)

C IgG tTG

D Refer urgently for a duodenal biopsy

E There is no appropriate test

Explanation
The correct answer is C, IgG tTG. NICE guidelines state that in children, laboratories should test for total IgA and
IgA tTG as the first choice. However, when IgA is deficient, laboratories should consider using IgG EMA, IgG
DGP or IgG tTG.

A and B are both incorrect because this patient is IgA deficient, so a false negative result may occur.

D is incorrect. The question specifically asks for the most appropriate blood test.

E is also incorrect, because there is an appropriate test for this scenario (as above).

https://www.nice.org.uk/guidance/ng20/chapter/key-priorities-for-implementation
47049

https://mypastest.pastest.com/Secure/TestMe/Browser/436619#Top[‫ ص‬07:49:57 09/12/1437]


MyPastest

Tag Question

Previous Question
Feedback

Difficulty: Average

Peer Responses
End Session

Session Progress

Responses Correct: 0

Responses Incorrect: 73

Responses Total: 73

Responses - % Correct: 0%

Blog
About Pastest
Contact Us
Help

© Pastest 2016

https://mypastest.pastest.com/Secure/TestMe/Browser/436619#Top[‫ ص‬07:49:57 09/12/1437]


MyPastest

Prefer to use the old MyPastest? Access it here »


Next Question

Back to Filters

Question 72 of 78

The mother of a 3-year-old girl comes to see you in the General Paediatric clinic. She is concerned that her
daughter is allergic to fish. Every time she eats fish she gets GI symptoms. Mother
has very helpfully kept a food
diary and a record of her symptoms, which she has brought to clinic today.

Which of these is LEAST likely to suggest food allergy?

A Abdominal pain

B Constipation

C Loose, frequent stools

D Pallor

E Polycythaemia

Explanation
Stem E, ‘polycythaemia’ is not a manifestation of food allergy/intolerance. Polycythaemia is a condition
in which
the proportion of red blood cells is increased above normal levels making blood less able to flow. This may be a
result of there being a lesser volume of plasma in the blood causing it to be thicker (apparent polycythaemia) or
because the body is producing additional red
blood cells (absolute polycythaemia).

A, B, C and D are all non IgE mediated signs and symptoms of possible food allergy, as outlined in the 2011
NICE Guidelines for Food Allergy in Under 19’s Assessment and Diagnosis.

http://pathways.nice.org.uk/pathways/food-allergy-in-children-and-young-people#content=view-node%3Anodes-
initial-recognition
47050

https://mypastest.pastest.com/Secure/TestMe/Browser/436619#Top[‫ ص‬07:50:36 09/12/1437]


MyPastest

Tag Question
Previous Question
Feedback

Difficulty: Average

Peer Responses
End Session

Session Progress

Responses Correct: 0

Responses Incorrect: 74

Responses Total: 74

Responses - % Correct: 0%

Blog
About Pastest
Contact Us
Help

© Pastest 2016

https://mypastest.pastest.com/Secure/TestMe/Browser/436619#Top[‫ ص‬07:50:36 09/12/1437]


MyPastest

Prefer to use the old MyPastest? Access it here »

Back to Filters

Question 73 of 78

A 7-year-old boy presents to the GP with recurrent episodes of diarrhoea and abdominal pain. He has severe
eczema which is very difficult to manage, despite the frequent, generous use of emollients and hydrocortisone
during flare ups. Mother has removed wheat from his diet, with no improvement. His height and weight are on the
2nd centile.

What is the MOST appropriate next course of action?

A Ask mother to keep a food diary and refer to allergy clinic

B Ask mother to keep a food diary and refer to endocrine clinic for growth assessment

C Food challenge at the GP surgery

D Refer to allergy clinic

E Skin prick test to wheat in the community

Explanation
The correct answer is A. This child fits the criteria for referral to secondary or specialist care (see NICE guidelines
below). He has not responded to a single allergen elimination
diet, he has faltering growth in combination with GI
symptoms and he also has significant atopic eczema. This child may have multiple food allergies.

B is not appropriate. The history points to a gastrointestinal cause for his faltering growth, rather than an endocrine
one.

C is incorrect. If a food challenge is to be undertaken then this should be done in hospital, with necessary
measures taken to prepare for
an anaphylactic reaction.

D is not the best answer. A is more appropriate because a food diary would be helpful to tease out the most likely
dietary causes(s) for the GI symptoms.

E is also incorrect. A skin prick test may be performed at the allergy clinic, but
the community is not an

https://mypastest.pastest.com/Secure/TestMe/Browser/436619#Top[‫ ص‬07:51:09 09/12/1437]


MyPastest

appropriate setting for this.

http://pathways.nice.org.uk/pathways/food-allergy-in-children-and-young-people/food-allergy-in-children-and-
Previous Question Next Question
young-people-overview#content=view-node:nodes-consider-referral-to-secondary-or-specialist-care
47051

End Session
Tag Question

Feedback

Difficulty: Average

Peer Responses

Session Progress

Responses Correct: 0

Responses Incorrect: 75

Responses Total: 75

Responses - % Correct: 0%

Blog
About Pastest
Contact Us
Help

© Pastest 2016

https://mypastest.pastest.com/Secure/TestMe/Browser/436619#Top[‫ ص‬07:51:09 09/12/1437]


MyPastest

Prefer to use the old MyPastest? Access it here »

Back to Filters

Question 74 of 78

A 10-year-old girl with a known history of peanut allergy is brought in by ambulance. She was eating a chocolate
bar, when she developed sudden onset of shortness of breath, wheeze, swelling of the face and lips and itching all
over. She now has stridor and saturations
are 88% in air. You apply facial oxygen.

What is the most appropriate IMMEDIATE course of action?

A Adrenaline 300 micrograms IM

B Adrenaline 150 micrograms IM

C Adrenaline IV 300 micrograms

D Chlorphenamine

E Salbutamol

Explanation
The correct answer is A. This is an anaphylactic reaction and IM adrenaline must be administered immediately.

B is incorrect. The correct dose for a child aged 6-12 years is 300 micrograms. 150 micrograms would be
appropriate a child under 6 years.

C is incorrect. IM and not IV adrenaline will be faster in this child who does not have a cannula.

D is incorrect. Although chlorphenamine will help the itching, adrenaline is necessary immediately to treat the
anaphylactic reaction.

E is also incorrect. Salbutamol can be helpful if the child is wheezy. The most important treatment to administer
quickly though is the IM adrenaline.

https://www.resus.org.uk/anaphylaxis/emergency-treatment-of-anaphylactic-reactions/
47052

https://mypastest.pastest.com/Secure/TestMe/Browser/436619#Top[‫ ص‬07:55:52 09/12/1437]


MyPastest

Previous Question Next Question


Tag Question

Feedback

End Session
Difficulty: Average

Peer Responses

Session Progress

Responses Correct: 0

Responses Incorrect: 76

Responses Total: 76

Responses - % Correct: 0%

Blog
About Pastest
Contact Us
Help

© Pastest 2016

https://mypastest.pastest.com/Secure/TestMe/Browser/436619#Top[‫ ص‬07:55:52 09/12/1437]


MyPastest

Prefer to use the old MyPastest? Access it here »


Next Question

Back to Filters

Question 75 of 78

A 2-year-old girl has had previous reactions to egg. Whenever she eats egg, she has a rash and itching all over.
Allergy has
been confirmed on skin-prick testing. She missed her MMR vaccine at thirteen months. Her mother
wants to know whether she can have her immunisations.

What is the MOST appropriate advice to give?

A She can have the intranasal influenza vaccine but not the MMR

B She cannot have the intranasal influenza vaccine

C She should have all immunisations at the GP practice

D She should have all immunisations under close supervision in hospital

E She should have the MMR under close supervision in hospital

Explanation
C is the correct answer. The advice from the Department of Health is that children with egg allergy should receive
all routine vaccinations as part of the childhood immunisation programme. Both intranasal and inactivated injected
influenza vaccines,
that are egg-free or have a very low ovalbumin content, are safe for children with egg allergy.
The measles vaccine is not grown in whole eggs, but in cells derived from hens (not containing egg protein). The
MMR vaccine is therefore safe in egg-allergic children. The exception to
this is children who have required
intensive care for egg allergy, who should receive the MMR in hospital.

A, B, D and E are therefore all incorrect for the reasons given above

https://www.allergyuk.org/advice-for-parents-with-a-new-baby/immunisations
47053

https://mypastest.pastest.com/Secure/TestMe/Browser/436619#Top[‫ ص‬07:56:30 09/12/1437]


MyPastest

Tag Question
Previous Question
Feedback

Difficulty: Average

Peer Responses
End Session

Session Progress

Responses Correct: 0

Responses Incorrect: 77

Responses Total: 77

Responses - % Correct: 0%

Blog
About Pastest
Contact Us
Help

© Pastest 2016

https://mypastest.pastest.com/Secure/TestMe/Browser/436619#Top[‫ ص‬07:56:30 09/12/1437]


MyPastest

Prefer to use the old MyPastest? Access it here »


Next Question

Back to Filters

Question 76 of 78

The parents of a 4-month-old boy come to see you in clinic. He has been exclusively breastfed so far. He used to
be a “sicky baby” until the mother found that if she cut out cows’ milk
from her own diet, his symptoms
improved. At four months, he is growing well along the 50th centile and looks very well. Parents would like to
introduce bottle feeds.

Which is the LEAST appropriate formula milk to give this child?

A AptamilPepti 1®

B NutramigenLipil®1

C Neocate LCP®

D PregestimilLipil®

E Wysoy®

Explanation
The correct answer is E. Wysoy® is a soy based formula. Soya based formulas are not suitable as a first-line
product for CMPA treatment. In children under 6 months, the phytate content of soy based formulas may inhibit
the absorption of minerals, and they also contain isoflavones which have oestrogenic actions.

A, B, C and D are all hydrolysed formulas that are suitable from birth in a child with cows’ milk protein allergy.

http://cks.nice.org.uk/cows-milk-protein-allergy-in-children#!scenario
47054

https://mypastest.pastest.com/Secure/TestMe/Browser/436619#Top[‫ ص‬07:56:56 09/12/1437]


MyPastest

Tag Question

Previous Question
Feedback

Difficulty: Average

Peer Responses
End Session

Session Progress

Responses Correct: 0

Responses Incorrect: 78

Responses Total: 78

Responses - % Correct: 0%

Blog
About Pastest
Contact Us
Help

© Pastest 2016

https://mypastest.pastest.com/Secure/TestMe/Browser/436619#Top[‫ ص‬07:56:56 09/12/1437]


MyPastest

Prefer to use the old MyPastest? Access it here »

Back to Filters

Question 77 of 78

A term infant who was born by SVD following an uneventful pregnancy is now 50 hours old and has still not
passed meconium. You review him due to bilious vomiting, and are worried enough to ask for a surgical review. A
contrast enema is performed and demonstrates microcolon.

Which long term medication is this child MOST likely to need?

A IM Testosterone

B Metformin

C Orlistat

D Pancrelipase

E SC Insulin

Explanation
The correct answer is D pancrelipase. This baby has meconium ileus, obstruction of the bowel by abnormally
tenacious meconium. Approximately a third of neonates with meconium ileus have cystic fibrosis. Pancreatic
status should be established at diagnosis by clinical assessment and by measuring faecal elastase. Patients with
pancreatic insufficiency should receive pancreatic enzyme replacement and fat soluble vitamins.

A is incorrect. Although delayed puberty onset is more common in both girls and boys with CF, onset is likely to
occur without the need for testosterone injections.

Metformin and orlistat are oral medications used to treat individuals with obesity. Individuals with CF are more
likely to need nutritional supplements and help with gaining weight.

SC insulin may be indicated in cases of CF related diabetes. Although CFRD is a common comorbidity in people
with CF, pancreatic insufficiency is more common.
Therefore, D is the best answer and E is incorrect.

https://www.cysticfibrosis.org.uk/

https://mypastest.pastest.com/Secure/TestMe/Browser/436619#Top[‫ ص‬07:57:19 09/12/1437]


MyPastest

47055

Previous Question Next Question

Tag Question

Feedback End Session

Difficulty: Average

Peer Responses

Session Progress

Responses Correct: 0

Responses Incorrect: 79

Responses Total: 79

Responses - % Correct: 0%

Blog
About Pastest
Contact Us
Help

© Pastest 2016

https://mypastest.pastest.com/Secure/TestMe/Browser/436619#Top[‫ ص‬07:57:19 09/12/1437]


MyPastest

Prefer to use the old MyPastest? Access it here »

Back to Filters

Question 78 of 78

A 14-year-old girl presents with severe abdominal pain radiating to the back and vomiting. Her heart rate is 200
and BP is 100/45. On examination there is tenderness and guarding in the upper abdomen. Her blood tests show
Hb 134; WCC 16; Plts 253; Na 140; K 3.5; Creat 90; Urea 6.4; Amylase 800; AST 12; ALT 16; Bili 2; Alb 4.2.

What is the MOST likely diagnosis?

A Acute hepatitis

B Chronic hepatitis

C Gastroenteritis

D Pancreatitis

E Pregnancy

Explanation
Pancreatitis is the correct answer. Although rare in childhood, in the presence of abdominal pain radiating to the
back with shock and tachycardia, and a raised amylase, pancreatitis is the most likely diagnosis. Possible causes
are trauma, drugs, viral illness, mumps, hyperparathyroidism, hyperlipidaemia and CF.

A and B are incorrect. Normal LFTs make hepatitis unlikely.

C is also incorrect. Gastroenteritis with severe diarrhoea and vomiting may account for the shock and tachycardia,
but guarding on abdominal examination and the raised amylase would not be explained by gastroenteritis.

E is also incorrect. Pregnancy is an important


diagnosis to consider in an adolescent with abdominal pain, but the
raised amylase and other features point to a diagnosis of pancreatitis.
47056

End Session

https://mypastest.pastest.com/Secure/TestMe/Browser/436619#Top[‫ ص‬07:57:41 09/12/1437]


MyPastest

Previous Question Tag Question

Feedback

Difficulty: Average

Peer Responses

Session Progress

Responses Correct: 0

Responses Incorrect: 80

Responses Total: 80

Responses - % Correct: 0%

Blog
About Pastest
Contact Us
Help

© Pastest 2016

https://mypastest.pastest.com/Secure/TestMe/Browser/436619#Top[‫ ص‬07:57:41 09/12/1437]


MyPastest

Prefer to use the old MyPastest? Access it here »

Back to Filters

Question 1 of 78

A 13-year-old girl has had recurrent abdominal pain and intermittent diarrhoea over the previous year. During
these episodes she
may pass 3-7 very loose stools with mucus. Over the past 3 months she has also passed stools
mixed with blood during the attacks. Though she has not lost weight, her weight has decreased by crossing a
centile. She
has not had her menarche. The mother suffers from vitiligo. Clinical examination was unremarkable.
Her blood tests are as follows: Hb 12.1 g/l, normal differential count, ESR 38mm. An autoantibody screen is
negative.

The most relevant next investigation is?

Your answer was incorrect

A Barium enema

B Colonoscopy

C Radio-isotope study

D Abdominal X-ray

E Angiography

Explanation
This girl is most likely to be suffering from inflammatory bowel disease, probably ulcerative colitis. The most
valuable investigation that will give an assessment of severity and extent of the disease, including the opportunity
to obtain biopsies is a
colonoscopy.

Barium studies and abdominal x-rays do not give sufficient information. Radio-isotope scans will help in
identifying a focus such as a Meckel's diverticulum and angiography is rarely indicated unless a vascular lesion is
suspected of leading to the intestinal bleed.
14562

https://mypastest.pastest.com/Secure/TestMe/Browser/436619[‫ ص‬08:09:40 11/12/1437]


MyPastest

Previous Question Next Question


Tag Question

Feedback

End Session
Difficulty: Easy

Peer Responses

Session Progress

Responses Correct: 0

Responses Incorrect: 1

Responses Total: 1

Responses - % Correct: 0%

Blog
About Pastest
Contact Us
Help

© Pastest 2016

https://mypastest.pastest.com/Secure/TestMe/Browser/436619[‫ ص‬08:09:40 11/12/1437]


MyPastest

Prefer to use the old MyPastest? Access it here »


Previous Question Next Question

Back to Filters

Question 2 of 78

A 6-week-old baby boy presents to the emergency department with a history of vomiting. the vomiting has been
present since the third week of life but has suddenly increased in violence and frequency over the preceding 3
days. The baby is well but hungry and failing to thrive.

The most useful next step is?

A Take an accurate feeding history and decrease feed volumes

B Take a blood gas

C Perform a septic screen

D Start intensive gastro-oesophageal reflux therapy

E Refer surgically immediately

Explanation
The most likely diagnosis needing exclusion is pyloric stenosis, so doing a blood gas will help establish the
diagnosis first and is preferable to a referral without thought.

Speculative
reduction of feed volumes will miss the diagnosis, as would presuming it is just exacerbated gastro-
oesophageal reflux. Only once pyloric stenosis is excluded should overfeeding and gastro-oesophageal reflux be
considered in the differential diagnosis. The child is well so sepsis is least likely.
14563

Tag Question

https://mypastest.pastest.com/Secure/TestMe/Browser/436619[‫ ص‬08:09:59 11/12/1437]


MyPastest

Feedback

Difficulty: Easy

Peer Responses

End Session

Session Progress

Responses Correct: 0

Responses Incorrect: 2

Responses Total: 2

Responses - % Correct: 0%

Blog
About Pastest
Contact Us
Help

© Pastest 2016

https://mypastest.pastest.com/Secure/TestMe/Browser/436619[‫ ص‬08:09:59 11/12/1437]


MyPastest

Prefer to use the old MyPastest? Access it here »


Previous Question Next Question

Back to Filters

End Session
Question 3 of 78

A 3-year-old girl presents with abdominal pain, infrequent stools, screaming on defecation, blood per rectum, and
poor eating. On examination she is uncooperative but is noticed to have anal tag and fissure.

The commonest cause for her condition is?

A Crohn's disease

B Haemorrhoids

C Constipation

D Hirschprung's disease

E Perianal streptococcal disease

Explanation
The vignette is a classic story for constipation. Crohn's disease is far less common. Haemorrhoids very rarely
occur in children. It is a late presentation for Hirschprung's disease. Perianal streptococcal infection does occur
and causes a very painful bottom, as do pinworms but the presence of a fissure and tags does not fit this diagnosis.
14564

Tag Question

Feedback

Difficulty: Average

https://mypastest.pastest.com/Secure/TestMe/Browser/436619[‫ ص‬08:10:18 11/12/1437]


MyPastest

Peer Responses

Session Progress

Responses Correct: 0

Responses Incorrect: 3

Responses Total: 3

Responses - % Correct: 0%

Blog
About Pastest
Contact Us
Help

© Pastest 2016

https://mypastest.pastest.com/Secure/TestMe/Browser/436619[‫ ص‬08:10:18 11/12/1437]


MyPastest

Prefer to use the old MyPastest? Access it here »


Previous Question Next Question

Back to Filters

End Session
Question 4 of 78

A 5-week-old baby boy of Afro-Caribbean parents presents to your surgery with jaundice. He was initially breast
fed but has been bottle fed for the past week and has not gained any weight for nearly 2 weeks.

The most useful next action is?

A Test the stools for reducing substances

B Request urgent liver function tests

C Change the milk to a soy based baby formula

D Request a blood test for sickle cell disease

E Organise a supra-pubic aspirate for urgent microscopy, culture and sensitivity

Explanation
An infant, who is not thriving and who has persistent jaundice after switching to formula milk, must be further
investigated. This cannot simply be put down to simple prolonged breast milk jaundice. Initially liver function
tests must be taken and a total and split bilirubin levels should be sought. The most important diagnosis to exclude
is biliary atresia. 14565

Tag Question

Feedback

https://mypastest.pastest.com/Secure/TestMe/Browser/436619[‫ ص‬08:10:37 11/12/1437]


MyPastest

Difficulty: Average

Peer Responses

Session Progress

Responses Correct: 0

Responses Incorrect: 4

Responses Total: 4

Responses - % Correct: 0%

Blog
About Pastest
Contact Us
Help

© Pastest 2016

https://mypastest.pastest.com/Secure/TestMe/Browser/436619[‫ ص‬08:10:37 11/12/1437]


MyPastest

Prefer to use the old MyPastest? Access it here »

Back to Filters

Question 5 of 78

A 5-year-old girl has had a persistent nocturnal cough for over 2 years that has not responded to treatment
including inhaled steroids and bronchodilators. The cough tends to be paroxysmal and is associated with vomiting
and disturbing her sleep virtually every night.
She is happy and normally active during the day and did not have
any exercise intolerance. In other respects she has been well with no significant past history. There is no family
history of asthma/eczema/allergy. Clinically, her weight was on the 75th centile and her height on the 50th centile.
General and systematic examinations were normal. CXR - essentially normal. FBC - normal range. CRP 5. PEF
90% predicted. Mantoux negative.

The next most useful that will aid the diagnosis is?

A Bronchoscopy

B CT scan of thorax

C Ambulatory oesophageal pH study

D Barium meal

E Spirometry

Explanation
This 5-year-old had a persistent cough that was characteristically nocturnal, accompanied by vomiting and with no
evidence of any diurnal respiratory symptoms. The possibility was that the cough is related to the supine posture,
especially as there was accompanying vomiting which was a constant feature. Therefore, in the first instance, the
least invasive investigation of ambulatory oesophageal pH monitoring is done. A barium meal is done if the
possibility of a hiatus hernia is considered. A bronchoscopy and CT scan
may be indicated if the initial
investigations are negative.

This girl has severe gastro-oesophageal reflux and symptoms subsided dramatically following anti-reflux therapy.
14887

https://mypastest.pastest.com/Secure/TestMe/Browser/436619[‫ ص‬08:11:05 11/12/1437]


MyPastest

Previous Question Next Question


Tag Question

Feedback

End Session
Difficulty: Easy

Peer Responses

Session Progress

Responses Correct: 0

Responses Incorrect: 5

Responses Total: 5

Responses - % Correct: 0%

Blog
About Pastest
Contact Us
Help

© Pastest 2016

https://mypastest.pastest.com/Secure/TestMe/Browser/436619[‫ ص‬08:11:05 11/12/1437]


MyPastest

Prefer to use the old MyPastest? Access it here »

Back to Filters

Question 6 of 78

Theme: Gastro-intestinal disorders

A Abetalipoproteinaemia

B Coeliac Disease

C Constipation with overflow

D Crohn’s disease

E Giardiasis

F Ileal tuberculosis

G Lactose intolerance

H Toddler’s diarrhoea

I Ulcerative colitis

J Viral gastro-enteritis

For
each of the following cases, choose the most likely cause of diarrhoea from the above list. Each item may be
used once, more than once or not at all.

Scenario 1

A 9-year-old boy presents with a 2-month history of weight loss, abdominal pain and intermittent diarrhoea. There
is no blood in the stool. He has a mildly tender abdomen and some perianal skin tags. Investigations reveal: CRP
45 mg/dL, ESR 50 mm first
hour.

Your answer was incorrect

Select one...

D - Crohn’s disease

From the history, and the objective weight loss, there is likely to be a significant pathology ongoing. The time
course is a little lengthy to be infective, although not impossible. The inflammatory markers are raised, indicating
an ongoing inflammatory process. The perianal skin tags should strongly raise the suspicion of Crohn’s disease.

https://mypastest.pastest.com/Secure/TestMe/Browser/436619[‫ ص‬08:11:28 11/12/1437]


MyPastest

This disease is increasing in childhood and the current incidence in the UK is approximately 10 per 100,000. It is
an inflammatory process involving the whole bowel (mouth to anus). Clinically patients present with abdominal
Previous
pain, weight loss/reduced Question
growth Next Question
and diarrhoea. On examination there may mouth ulceration, and perianal lesions.
Extragastro-intestinal features may be
present – including arthralgia, anaemia and uveitis. Radiological
contrast
studies will show involvement of various parts of the bowel (skip lesions), There may also be signs of
inflammation –‘cobblestones’ or ‘rose-thorn’ ulcers. Treatment should be shared with a tertiary level,
multidisciplinary, paediatric gastro-enterological team.

Scenario 2

A 3-year-old girl has a 2-month history of


loose stools. Her parents are very concerned. The stools often contain
‘undigested food’. She is otherwise well and thriving.

Your answer was incorrect

Select one...

H - Toddler’s diarrhoea

This is a classic history for toddler’s diarrhoea. The child is well and thriving but the parents can understandably
be anxious. The cause is thought to be due a fast enteric transit time, and a brisk gastrocolic reflex. Reassurance is
vital, but simple measures such as reducing fruit juice intake may help.

Scenario 3

An 8-month-old infant has had watery diarrhoea for 4 days. There has been no recent travel. Her 2-year-old sibling
has recently had a similar illness, though less severe. She is drinking her normal formula milk well and is not
dehydrated.

Your answer was incorrect

Select one...

J - Viral gastro-enteritis

This is an acute history so an infective process is likely; the fact that her sibling had a similar illness again makes
an infective cause much more likely. A viral cause is more common in this age group (e.g. rotavirus). Viral gastro-
enteritis is a self-limiting condition but there is a risk of dehydration if oral intake is insufficient. Medications such
as loperamide or codeine to ‘reduce’ the diarrhoea have no place in management, and may lengthen the duration
of the illness. Secondary lactose intolerance is uncommon and before changing the milk to a lactose-free formula,
stool reducing sugars should be checked.
22267

Tag Question

https://mypastest.pastest.com/Secure/TestMe/Browser/436619[‫ ص‬08:11:28 11/12/1437]


MyPastest

Feedback

Difficulty: Average

Session Progress

Responses Correct: 0
End Session
Responses Incorrect: 8

Responses Total: 8

Responses - % Correct: 0%

Blog
About Pastest
Contact Us
Help

© Pastest 2016

https://mypastest.pastest.com/Secure/TestMe/Browser/436619[‫ ص‬08:11:28 11/12/1437]


MyPastest

Prefer to use the old MyPastest? Access it here »


Previous Question Next Question

Back to Filters

End Session
Question 7 of 78

A 7-week-old baby boy is referred with a 2-week history of vomiting. He is being formula fed 5 oz (approximately
150 mL)
every 2–3 h. On examination he is well, thriving and has a normal
examination. The most likely
diagnosis is:

A Pyloric stenosis

B Gastro-oesophageal reflux

C Over-feeding

D Gastroenteritis

E Jejunal stenosis

Explanation
Although all of the answers would cause the listed symptoms, pyloric stenosis, jejunal stenosis and gastroenteritis
would be very unlikely in a thriving child. The daily volume of feed is quite substantial (1200 mL) and in a 5 kg
baby would work out to 240 mL/kg per day! Approximately 150 mL/kg per day is a rough guide to a baby’s milk
requirement.
22344

Tag Question

Feedback

https://mypastest.pastest.com/Secure/TestMe/Browser/436619[‫ ص‬08:11:47 11/12/1437]


MyPastest

Difficulty: Easy

Peer Responses

Session Progress

Responses Correct: 0

Responses Incorrect: 9

Responses Total: 9

Responses - % Correct: 0%

Blog
About Pastest
Contact Us
Help

© Pastest 2016

https://mypastest.pastest.com/Secure/TestMe/Browser/436619[‫ ص‬08:11:47 11/12/1437]


MyPastest

Prefer to use the old MyPastest? Access it here »


Next Question

Back to Filters

Question 8 of 78

A 6-week-old baby is referred for back arching and crying. He


possets after feeds, especially when he lies on his
back. He is thriving. You suspect gastro-oesophageal reflux (GOR).

What is the most appropriate first line course of action?

A Barium swallow

B Trial of Gaviscon

C Trial of domperidone and ranitidine

D Reassure the parents

E A pH study

Explanation
This is a difficult situation and different practitioners may have different views. There is an argument that since
the child is thriving, the GOR is something the child will outgrow and reassurance of the parents should be first
line treatment.

In addition, since it can be difficult not to treat a child, a trial of Gaviscon is simple, has minimal side-effects, and
is a reasonable approach. If there is an improvement in the symptoms further treatment can be initiated as
warranted. An oesophageal pH study is a good investigation for diagnosing severity of GOR, whereas a contrast
upper GI study (or barium meal) is non-physiological and its main value here is in demonstrating a hiatus hernia;
however, if there are good clinical
features investigations may not be needed.
22385

https://mypastest.pastest.com/Secure/TestMe/Browser/436619[‫ ص‬08:12:06 11/12/1437]


MyPastest

Tag Question

Previous Question
Feedback

Difficulty: Average

Peer Responses
End Session

Session Progress

Responses Correct: 0

Responses Incorrect: 10

Responses Total: 10

Responses - % Correct: 0%

Blog
About Pastest
Contact Us
Help

© Pastest 2016

https://mypastest.pastest.com/Secure/TestMe/Browser/436619[‫ ص‬08:12:06 11/12/1437]


MyPastest

Prefer to use the old MyPastest? Access it here »


Previous Question Next Question

Back to Filters

End Session
Question 9 of 78

A 9-month-old infant presents with vomiting and crying. On examination she is afebrile and has a diffusely tender
abdomen. No masses are palpable. Which one of the following diagnoses is
the most important to exclude?

A Gastroenteritis

B Intussusception

C Mesenteric adenitis

D Hirschsprung’s disease

E Colic

Explanation
‘The most important to exclude’: intussusception is the only one that is potentially life threatening. The others are,
of course, possible diagnoses but not immediately life-threatening.
22417

Tag Question

Feedback

Difficulty: Easy

Peer Responses

https://mypastest.pastest.com/Secure/TestMe/Browser/436619[‫ ص‬08:12:26 11/12/1437]


MyPastest

Session Progress

Responses Correct: 0

Responses Incorrect: 11

Responses Total: 11

Responses - % Correct: 0%

Blog
About Pastest
Contact Us
Help

© Pastest 2016

https://mypastest.pastest.com/Secure/TestMe/Browser/436619[‫ ص‬08:12:26 11/12/1437]


MyPastest

Prefer to use the old MyPastest? Access it here »

Back to Filters

Question 10 of 78

Which of the following statements about complementary feeding (weaning) is most factually correct?

A Exclusive
breastfeeding for at least 6 months is associated with a decreased incidence of coeliac
disease in later life compared with the introduction of gluten between 4 and 6 months

B Exclusive breastfeeding of infants until 6 months is associated with a decreased incidence of infection

C The reported percentage of mothers in the UK who exclusively breastfeed their babies for 6 months is
around 15%

D There
is clear evidence that the introduction of certain foods between 4 and 6
months is associated
with an increased prevalence of food allergy

E There are newly emerging concerns that exclusive breastfeeding until 6 months is associated with iron
deficiency anaemia

Explanation
The WHO advice issued in 2001 stated that
all infants should be exclusively breastfed until 6 months; in 2003 this
became UK policy. Recent evidence has highlighted possible concerns
about this advice; studies are under way to
evaluate its potential consequences. It is, however, well documented that as few as 1% of infants in the UK are
exclusively breastfed until 6 months of age.

In the UK Millennium cohort study, the introduction of formula feeds (from exclusive breastfeeding) rather than
solid feeds predicted an increased hospital admission for gastroenteritis and respiratory infections. It was
estimated that 53% of admissions for gastroenteritis and 27% of admissions for chest infection could be prevented
each month by exclusive breastfeeding.

Evidence has always


existed, and continues to grow, that exclusive breastfeeding for 6 months may lead to
increased iron deficiency anaemia and its consequences of long-term motor, mental and social development.

There
is also concern that, although the growth of babies exclusively breastfed for 6 months appears to be

https://mypastest.pastest.com/Secure/TestMe/Browser/436619[‫ ص‬08:12:45 11/12/1437]


MyPastest

adequate (very small numbers), calculations about energy requirements compared with the energy content and
output of breast milk may mean exclusive breastfeeding may not support adequate growth in all children.

Limited
evidencePrevious
exists for aQuestion Next Question
bimodal distribution of increased allergy associated with the introduction of certain
foods. Exposure before 3–4 months as well as delayed exposure after 6 months may increase
the prevalence of
food allergy in a population. Exposure to gluten-containing foods between 4 and 6 months appears to be
protective,
especially when combined with continued breastfeeding, for the development of coeliac disease in later
life.
End Session
There may also be a ‘window’ to introduce bitter tasting foods (such as green vegetables) between 4 and 6 months.
43950

Tag Question

Feedback

Difficulty: Average

Peer Responses

Session Progress

Responses Correct: 0

Responses Incorrect: 12

Responses Total: 12

Responses - % Correct: 0%

Blog
About Pastest
Contact Us
Help

© Pastest 2016

https://mypastest.pastest.com/Secure/TestMe/Browser/436619[‫ ص‬08:12:45 11/12/1437]


MyPastest

https://mypastest.pastest.com/Secure/TestMe/Browser/436619[‫ ص‬08:12:45 11/12/1437]


MyPastest

Prefer to use the old MyPastest? Access it here »

Back to Filters

Question 11 of 78

Which of the following statements with regard to acute abdominal pain in children is most factually correct?

A Abdominal
ultrasonography is rarely helpful in diagnosis but can help to identify
alternative
pathology, including ovarian problems in girls

B Acute appendicitis is most common in the second decade of life

C Acute appendicitis is the most common surgical cause of an acute abdomen in the under-5 age group

D The
classic presentation of central colicky abdominal pain localising to the lower right quadrant occurs
in about 70% of patients

E The rate of perforation of the appendix at diagnosis falls from 80% to 30% between the first and
second decades of life

Explanation
The most common surgical diagnosis in all
children who present to hospital with acute abdominal pain is acute
appendicitis; intussusception is more common in children aged <5 years. In over half of all admissions to hospital
with acute abdominal pain no cause is found. The differential diagnosis is wide; a thorough history and
examination including testicular examination are essential to appropriately investigate and manage these patients.

Acute appendicitis

Classically
there is initial central colicky abdominal pain progressing to localise
pain in the right lower quadrant;
however, less than 60% of children present in this way. Fever (low grade), anorexia, nausea and vomiting are
common, loose stools and urinary symptoms may also occur. WCC is generally raised, ultrasonography being
increasingly utilised to investigate this condition and reporting sensitivity of 84% and specificity of 94% in
children. A pelvic appendix may be missed on ultrasonography and present with atypical symptoms and signs.
Appendicitis is most common in the second decade of life. The rate of perforation is increased by a delay in
diagnosis and approaches 80% in those aged <3 years, and decreases to 10–20% of 10–17 year olds. Management

https://mypastest.pastest.com/Secure/TestMe/Browser/436619[‫ ص‬08:13:08 11/12/1437]


MyPastest

is by appendectomy unless there is an appendix mass, in which case a course of intravenous antibiotics is followed
by an interval elective appendectomy. Laparoscopic techniques are increasingly being employed.
Previous Question
Differential diagnosis includes:
Next Question
Appendicitis
Intussusception
Urinary tract infection
End Session
Mesenteric adenitis
Constipation
Peptic ulceration
Meckel diverticulitis
Pancreatitis
Gastroenteritis
Ovarian pathology, eg torsion/cyst
Primary peritonitis
Henoch–Schönlein purpura
Hernia
Testicular torsion
Cholecystitis
Renal colic
Metabolic, eg acute porphyria
Trauma
Inflammatory bowel disease
Pelvic inflammatory disease
Ectopic pregnancy
Sickle cell crisis
Non-abdominal cause, eg pneumonia, diabetic ketoacidosis

43951

Tag Question

Feedback

Difficulty: Average

Peer Responses

https://mypastest.pastest.com/Secure/TestMe/Browser/436619[‫ ص‬08:13:08 11/12/1437]


MyPastest

Session Progress

Responses Correct: 0

Responses Incorrect: 13

Responses Total: 13

Responses - % Correct: 0%

Blog
About Pastest
Contact Us
Help

© Pastest 2016

https://mypastest.pastest.com/Secure/TestMe/Browser/436619[‫ ص‬08:13:08 11/12/1437]


MyPastest

Prefer to use the old MyPastest? Access it here »

Back to Filters

Question 12 of 78

Which of the following statements with regard to iron deficiency anaemia in children is most factually correct?

A Follow-on milks should be encouraged in all children to prevent iron deficiency anaemia

B Iron-rich foods include egg yolk, red meat, fortified breakfast cereals, and dried fruit, green vegetables,
beans and pulses

C Malabsorption is the major cause for iron deficiency in children in the UK

D The bioavailability of iron in breast milk is low

E The
most likely cause for non-response to oral iron treatment is an undiagnosed pathological aetiology,
eg coeliac disease, blood loss or chronic inflammation

Explanation
WHO definition of iron deficiency anaemia:

Hb <110 g/l in children aged 1–2 years


Hb <112 g/l in children aged 3–5 years
Hb <115 g/l in children aged 5–12 years
Hb <120 g/l in children aged 12–15 years.

The
prevalence in the UK is 8% in those aged under 5 years, increasing considerably in inner city areas and in
Asian children. Although malabsorption and bleeding are causes of iron deficiency anaemia in children, the
overwhelming cause is dietary, specifically the disproportionately high percentage of unmodified cows’ milk in
the
diet and the over-reliance on non-iron-containing convenience foods in the weaning diet and beyond. Iron
stores are laid down in the last trimester of pregnancy and so preterm babies require supplementation. Markers of
iron deficiency include:

https://mypastest.pastest.com/Secure/TestMe/Browser/436619[‫ ص‬08:13:32 11/12/1437]


MyPastest

low MCV (mean corpuscular volume)


low MCHPrevious Question
(mean corpuscular Hb) Next Question
low plasma ferritin (<10 υg/l) – note that raised as part of acute phase reactant.

Consequences:

Pallor, tiredness, irritability End Session


Anorexia
Increased risk of infection
Developmental delay and poor educational achievement (may persist post-treatment)
Dysphagia (oesophageal web)
Pica.

Management:

Breastfeeding – high bio-availability


Iron-fortified formula/follow-on milk (where need to maintain milk intake level).
Encourage iron-rich weaning foods and beyond
Give vitamin C-rich fruit/juices with meals to increase absorption
Avoid whole cows’ milk during the first year of life, and subsequently restrict intake to <750 ml/day.

Treatment/Prophylaxis:

3–6 mg/kg per day divided doses of elemental iron


Hb should rise by 10–20 g/l over 3–4 weeks
Treatment should be continued for 3 months after the anaemia has been corrected
Prophylaxis for neonates is 5 mg elemental iron daily, 6 weeks until weaned
Side effects of iron medication include nausea, epigastric pain, constipation, diarrhoea and black
discoloration of stools
Iron is very dangerous in overdose
Poor compliance is the most likely cause of resistant cases.

43953

Tag Question

Feedback

https://mypastest.pastest.com/Secure/TestMe/Browser/436619[‫ ص‬08:13:32 11/12/1437]


MyPastest

Difficulty: Average

Peer Responses

Session Progress

Responses Correct: 0

Responses Incorrect: 14

Responses Total: 14

Responses - % Correct: 0%

Blog
About Pastest
Contact Us
Help

© Pastest 2016

https://mypastest.pastest.com/Secure/TestMe/Browser/436619[‫ ص‬08:13:32 11/12/1437]


MyPastest

Prefer to use the old MyPastest? Access it here »

Back to Filters

Question 13 of 78

A 20-month-old Asian boy of non-consanguineous parents presents with a history of swelling of the wrists. He
complains of pain in his limbs and since mobilising his parents have become concerned about his ‘bandy legs’. He
was breastfed from birth and had solids introduced into his diet at 6 months of age. He had a green-stick
fracture
of his radius at the age of 12 months secondary to a fall at nursery.

Which of the following diagnoses is most likely in this case?

A Acute lymphoblastic leukaemia

B Juvenile idiopathic arthritis

C Non-accidental injury

D Vitamin D-deficient rickets

E X-linked hypophosphataemic rickets

Explanation
Rickets is a disease of growing bones, (failure to mineralise osteoid). Vitamin D deficiency (diet or ? sunlight) is
the main cause of rickets. Maternal vitamin D deficiency combined with low vitamin D in breast milk may lead to
neonatal rickets.
Other causes include the following:

Nutritional deficiencies of calcium and phosphorous


X-linked and autosomal recessive (AR) hyphophoshataemic rickets, AR vitamin D-dependent rickets (all
rare)
Fat malabsorption (coeliac disease, liver disease – also leads to defective metabolism of vitamin D)
Renal disease (defective metabolism of vitamin D)
Neonatal rickets of prematurity (secondary to inadequate phosphate)

https://mypastest.pastest.com/Secure/TestMe/Browser/436619[‫ ص‬08:13:54 11/12/1437]


MyPastest

Anticonvulsants.

Rickets typically presents at 4–18 months. Vitamin D deficiency during adolescence usually results in
osteomalacia.

Typical radiological features of rickets include poor mineralisation, delayed development of epiphyses,
metaphyseal cupping, fraying and splaying, best seen at the wrists, knees and ankles.

Symptoms and signs include:

Bowing of the legs


Impaired linear growth in infancy
Diffuse bone pain
Kyphoscoliosis
Flattened pelvis (obstructed labour in adult life)
The costochondral junctions become swollen (rickety rosary)
Pectus carinatum and Harrison sulci
Cartilaginous swellings around the wrists and ankles
Frontal bossing
Delayed closure of the anterior fontanelle
Dental enamel hypoplasia and delayed tooth eruption
Hypocalcaemic seizures
Tetany, apnoea and stridor
Left ventricular hypertrophy
Prolonged QTc interval and arrhythmias
Hypotension and heart failure

Blood biochemistry in vitamin D-deficient rickets:

Decreased 25OH-D (25-hydroxyvitamin D, calcidiol) in serum


Raised PTH
Decreased Ca2+
Decreased PO43-
Raised Alkaline phosphatase.

Treatment:

Ergocalciferol (vitamin D2) or cholecalciferol (vitamin D3) for dietary deficiency


Alfacalcidol
(1α-hydroxycholecalciferol) or calcitriol (1,25-dihydroxycholecalciferol) for those with liver
or kidney disease.

https://mypastest.pastest.com/Secure/TestMe/Browser/436619[‫ ص‬08:13:54 11/12/1437]


MyPastest

Sources of vitamin D include fish oil, vegetable oil and skin synthesis during exposure to sunlight.
43955

Previous Question Next Question

Tag Question
End Session
Feedback

Difficulty: Average

Peer Responses

Session Progress

Responses Correct: 0

Responses Incorrect: 15

Responses Total: 15

Responses - % Correct: 0%

Blog
About Pastest
Contact Us
Help

© Pastest 2016

https://mypastest.pastest.com/Secure/TestMe/Browser/436619[‫ ص‬08:13:54 11/12/1437]


MyPastest

Prefer to use the old MyPastest? Access it here »

Back to Filters

Question 14 of 78

A 2-year-old boy presents to the GP with a 3-month history of


recurrent episodes of small amounts of bright red
blood per rectum.

He is well grown and does not have any associated symptoms of abdominal pain, diarrhoea or constipation. There
is no family history of
GI bleeding and he does not appear clinically anaemic. He has no other cutaneous signs on
examination and has normal cardiovascular, respiratory and abdominal examinations in clinic.

What is the most likely diagnosis from the list below?

A Constipation with anal fissure

B Familial adenomatous polyposis coli

C Juvenile polyp

D Juvenile polyposis

E Peutz–Jegher syndrome

Explanation
In this case bright-red blood passing per
rectum is likely to be from the distal bowel and the absence of associated
anal pain makes a diagnosis of constipation with an anal fissure unlikely.

Juvenile polyps

More
than 90% of polyps that present in childhood are juvenile polyps. They are benign harmatomas.
They present at age 2–6 years with painless
blood per rectum; however, 5% prolapse and 10% are
associated with abdominal pain. Most polyps are solitary and located within 30 cm of the
anus. They are
not premalignant.

https://mypastest.pastest.com/Secure/TestMe/Browser/436619[‫ ص‬08:14:17 11/12/1437]


MyPastest

Juvenile polyposis

This
rare Previous Question
condition refers Next Question
to patients with more than five juvenile polyps. Presenting signs include
diarrhoea, rectal bleeding, intussusception, anaemia, prolapse and failure to thrive. This may be sporadic
or familial with an autosomal dominant pattern of inheritance. Polyps can be present throughout the GI
tract. There is considerable malignant potential (17%) and prophylactic colectomy may be advised.

End Session
Peutz–Jegher syndrome

This
condition is inherited in an autosomal dominant pattern. It consists of
hamartomatous polyps that can
occur throughout the GI tract, but most commonly in the small intestine. It is associated with
hyperpigmentation
of the buccal mucosa and lips. Patients without a family history will often present with
recurrent abdominal pain caused by intussusception secondary to small bowel polyps. It is a premalignant
condition with carcinomatous transformation of the polyps; in addition there is an increased risk of
developing pancreatic, ovarian, breast, cervical and testicular tumours.

Familial adenomatous polyposis coli and Gardner syndrome

These
conditions are inherited in an autosomal dominant fashion. Multiple polyps develop (>100), usually
in the second decade. Gastric and duodenal polyps develop in up to 50% and there is an increased risk of
thyroid and liver tumours. Most cases are diagnosed on screening children of affected individuals. Gardner
syndrome is familial adenomatous polyposis plus bony lesions, subcutaneous tumours and cysts.
Both
conditions carry a very high risk of colonic carcinoma (100%) and prophylactic colectomy at the end of
the second decade is advised.

43957

Tag Question

Feedback

Difficulty: Average

Peer Responses

https://mypastest.pastest.com/Secure/TestMe/Browser/436619[‫ ص‬08:14:17 11/12/1437]


MyPastest

Session Progress

Responses Correct: 0

Responses Incorrect: 16

Responses Total: 16

Responses - % Correct: 0%

Blog
About Pastest
Contact Us
Help

© Pastest 2016

https://mypastest.pastest.com/Secure/TestMe/Browser/436619[‫ ص‬08:14:17 11/12/1437]


MyPastest

Prefer to use the old MyPastest? Access it here »

Back to Filters

Question 15 of 78

Which of the following statements about breastfeeding is MOST accurate?

A A diagnosis of PKU is a contraindication to breastfeeding

B Breastfeeding increases IQ equally in both low-birthweight and normal-birthweight infants

C Breastfeeding offers clear benefits in the prevention of lower respiratory tract infections

D Breastfeeding reduces the rate of gastrointestinal infections of infants while it continues

E Maternal HIV, hepatitis C and hepatitis B are contraindications to breastfeeding in developed countries

Explanation
The clearest benefits to the children who
are breastfed are a decreased incidence of gastrointestinal infection and
reduced incidence of otitis media, although these effects do not persist beyond a 2-month period after the cessation
of breastfeeding. There is also clear evidence of a protective effect in the prevention of
NEC in preterm babies.
There is no evidence that breastfeeding prevents
the development of respiratory tract infections but the severity of
the
illness, as measured by the need for hospital admission, appears to be decreased. Additional benefits of
breastfeeding to the child include the
following:

Long-term reduction in blood pressure


Long-term lower cholesterol levels
Decreased incidence of childhood type 1 diabetes
May prevent or at least delay the development of coeliac disease
Increased IQ (more pronounced for low-birthweight children)
Decreased risk of developing childhood ALL (acute lymphoblastic leukaemia)
A decrease in incidence of childhood atopic disease in at-risk children

https://mypastest.pastest.com/Secure/TestMe/Browser/436619[‫ ص‬08:14:40 11/12/1437]


MyPastest

There are a few contraindications to breastfeeding:

Previous Question
Galactosaemia
Next Question
Phenlyketonuria (PKU – breastfeeds may be alternated with phenylalanine-free formulae)
Mothers
with HIV infection in developed countries (the WHO recommends that, when replacement
feeding is acceptable, feasible, affordable, sustainable and safe, avoidance of all breastfeeding by HIV-
infected mothers is recommended; otherwise, exclusive breastfeeding isEnd Sessionduring the first
recommended
months of life)
Note that NOT contraindicated in hepatitis B surface antigen or hepatitis C-positive mothers
Mothers with herpes simplex lesions on their breasts
Mothers who are positive for human T-cell lymphotrophic virus (HTLV) type I and II
Mothers who are receiving therapeutic or diagnostic radioactive isotopes
Maternal intake of any medications should be checked before breastfeeding; alternatives may be available

43959

Tag Question

Feedback

Difficulty: Average

Peer Responses

Session Progress

Responses Correct: 0

Responses Incorrect: 17

Responses Total: 17

Responses - % Correct: 0%

https://mypastest.pastest.com/Secure/TestMe/Browser/436619[‫ ص‬08:14:40 11/12/1437]


MyPastest

Blog
About Pastest
Contact Us
Help

© Pastest 2016

https://mypastest.pastest.com/Secure/TestMe/Browser/436619[‫ ص‬08:14:40 11/12/1437]


MyPastest

Prefer to use the old MyPastest? Access it here »

  Logged in as Ahmed fouad

Your

Back to Filters
Question 16 of 78

 My

 Sho

Which of the following statements is the most factually correct with regard to the administration of  Blo
Difficulty: Average
vitamin K to babies in the neonatal
period?
✉ Co
Peer Responses

A If an oral regimen is undertaken, a single dose of oral vitamin K is recommended in all  Mo
formula-fed babies
 He

B Significant
bleeding in the neonatal period normally occurs in children with significant
risk factors for vitamin K deficiency bleeding
Logg

C The vast majority of vitamin K deficiency-associated bleeding occurs within the first  Lo
week of life

Session Progress
D There is good evidence to support an increased risk of childhood leukaemia in babies
given vitamin K in the neonatal period Responses Correct: 0

Responses Incorrect: 18

E Vitamin
K deficiency bleeding is a potentially fatal disorder that is virtually
preventable Responses Total: 18
with one dose of vitamin K given intramuscularly at birth
Responses - % Correct: 0%

Explanation

The Department of Health issued guidance on the administration of vitamin K in newborn infants in
1998. In short:

All
newborn babies should receive vitamin K to prevent the rare but serious
and sometimes
fatal disorder of vitamin K deficiency bleeding (VKDB).
The available data do not support an increased risk of cancer including leukaemia caused by
vitamin K.
Intramuscular vitamin K effectively prevents VKDB in virtually all babies after a single
dose given at birth.
Oral
regimens require repeated (more than two) doses in breastfed babies*; clinicians must
ensure that all recommended oral doses are given at the appropriate ages.
A selective policy for high- or low-risk infants is not feasible because bleeding occurs
unpredictably.
Babies
at high risk of early (24 h) or classic (2–7 days) VKDB include: preterm, or failing to
take or absorb feeds, or had a complicated delivery or are ill or, if their mothers have been
receiving medication associated with a higher risk (eg anticonvulsant drugs).
Babies
at high risk of late (>7 days) VKDB: babies with liver disease (prolonged jaundice
with pale stools and/or dark urine), or babies who have bleeding or spontaneous bruising in
early infancy or who are ill from other causes.
Of the babies who do not receive vitamin K, 1
in 10 000 will develop VKDB; 50% of these
are affected late, and 50% of
those affected will have intracranial bleeding, 20% of those
will die and most survivors will have long-term neurological damage.
Many
who have severe late VKDB have minor warning bleeds in the preceding few days. In
about a third of instances the bleeding occurs without evident cause or risk factor.

*Vitamin K is added to formula milk.

http://webarchive.nationalarchives.gov.uk/+/www.dh.gov.uk/en/Publicationsandstatistics/Lettersandcirculars/Professionalletters/Chiefmedicalofficerletters/DH_4004993
43960

https://mypastest.pastest.com/Secure/TestMe/Browser/436619[‫ ص‬08:15:01 11/12/1437]


MyPastest

Previous Question Next Question


Tag Question Feedback

Blog
About Pastest
Contact Us
Help
© Pastest 2016
End Session

https://mypastest.pastest.com/Secure/TestMe/Browser/436619[‫ ص‬08:15:01 11/12/1437]


MyPastest

Prefer to use the old MyPastest? Access it here »

Back to Filters

Question 17 of 78

Which of the following statements with regard to testing for carbohydrate malabsorption is most factually correct?

A Alkaline stool pH is indicative of carbohydrate intolerance

B An
early peak of hydrogen in a hydrogen breath test, followed by a secondary peak, may be indicative
of small bowel bacterial overgrowth

C Non-resolution of symptoms on removal of suspected carbohydrate from the diet excludes the
diagnosis of carbohydrate intolerance

D Reducing substances of 0.05% in the stool are indicative of carbohydrate intolerance

E Resolution
of symptoms on removal of the suspected carbohydrate from the diet is conclusive
evidence of the presence of lactose intolerance

Explanation

Samples of liquid stool will show undigested sugars on chromatography


Reducing
substances in the stool of 0.25–0.5% are suggestive of carbohydrate malabsorption; >0.75% is
indicative of the condition. The liquid portion of the stool must be tested
Acidic stool pH <5.5 is indicative of carbohydrate malabsorption even in the absence of reducing
substances
Analysis
of small-bowel mucosal disaccharidases in a biopsy obtained endoscopically is the ‘gold
standard’, but is rarely needed.
Endoscopy may reveal small intestinal mucosal injury resulting in
secondary carbohydrate malabsorption
Clinical response to treatment with withdrawal of suspected intolerant substance leading to resolution of
symptoms and reintroduction leading to relapse is suggestive of diagnosis. However, non-resolution of

https://mypastest.pastest.com/Secure/TestMe/Browser/436619[‫ ص‬08:15:23 11/12/1437]


MyPastest

diarrhoea may not


necessarily exclude the diagnosis because diarrhoea itself may produce a
secondary
intolerance of a different substance, so the sensitivity of this as a diagnostic test can be poor. In addition
Previous Question Next Question
removal of a dietary substance may improve symptoms but not confirm a diagnosis, eg the removal of
cows’ milk may lead to the cessation of diarrhoea but this may be due to a diagnosis of lactose intolerance
or cows’ milk protein intolerance
The hydrogen breath test: the principle is that malabsorbed carbohydrate will pass into the colon where it
End Session
is metabolised by bacteria and hydrogen is released. The gas is then absorbed and released in the breath. If
there is a peak it suggests carbohydrate malabsorption. An early peak raises the possibility of bacterial
overgrowth in the small intestine, in which case a secondary peak may occur representing large bowel
fermentation. Lactulose, which is a non-absorbable carbohydrate, can be given to ensure that the colonic
flora can metabolise carbohydrate and to assess transit time. A false-negative test is possible

43962

Tag Question

Feedback

Difficulty: Average

Peer Responses

Session Progress

Responses Correct: 0

Responses Incorrect: 19

Responses Total: 19

Responses - % Correct: 0%

https://mypastest.pastest.com/Secure/TestMe/Browser/436619[‫ ص‬08:15:23 11/12/1437]


MyPastest

Blog
About Pastest
Contact Us
Help

© Pastest 2016

https://mypastest.pastest.com/Secure/TestMe/Browser/436619[‫ ص‬08:15:23 11/12/1437]


MyPastest

Prefer to use the old MyPastest? Access it here »

Back to Filters

Question 18 of 78

Which of the following statements about preterm and term formulae and breast milk is most factually correct?

A Babies
fed on fortified breast milk have a decreased incidence of late-onset sepsis and necrotising
enterocolitis when compared with babies fed on preterm formulae

B Preterm babies fed with fortified breast milk show similar weight gain profiles to preterm babies fed
on preterm formula milks

C Preterm
formula contains more fat, protein and electrolytes but the same amount
of carbohydrate as
term formula or unfortified breast milk

D Preterm formula has a lower sodium content than term formula

E Preterm formula has the same protein content as term formula

Explanation
The principal differences are that preterm formula contains more electrolytes, calories and minerals than term
formula. All of the following are higher in the preterm formulae: energy, protein, carbohydrate, fat, osmolality,
sodium, potassium, calcium, magnesium, phosphate and iron.

Breast milk fortifier is a powder designed to add to breast milk for preterm or
low-birthweight babies to provide
extra energy, protein, vitamins and minerals to support growth in this population while preserving the non-
nutritional advantages of breast milk, such as its immunological function, enzymes and growth factors. It is based
on hydrolysed cows’ milk protein. Fortified human milk may not produce as much weight gain as preterm
formulae but it is associated with a decreased incidence of NEC and late-onset sepsis.

In 100 ml Breast milk (approximately) Term Formula Preterm Formula

Energy (kcal) 60-75 67 80

Protein (g) 1.3 1.4 2.5

https://mypastest.pastest.com/Secure/TestMe/Browser/436619[‫ ص‬08:15:45 11/12/1437]


MyPastest

Carbohydrate (g) 7 7.5 7.8

Fat (g)
Previous Question
4.2
Next3.5Question4.4
Na+ (mmol) 0.65 0.7 1.7

K+ (mmol) 1.54 1.6 2.0

Ca2+ (mmol) 0.88 1.0 2.5 End Session


PO43- (mmol) 0.48 0.6 1.6

Iron (mg) 0.08 0.5 1.4

43963

Tag Question

Feedback

Difficulty: Average

Peer Responses

Session Progress

Responses Correct: 0

Responses Incorrect: 20

Responses Total: 20

Responses - % Correct: 0%

https://mypastest.pastest.com/Secure/TestMe/Browser/436619[‫ ص‬08:15:45 11/12/1437]


MyPastest

Blog
About Pastest
Contact Us
Help

© Pastest 2016

https://mypastest.pastest.com/Secure/TestMe/Browser/436619[‫ ص‬08:15:45 11/12/1437]


MyPastest

Prefer to use the old MyPastest? Access it here »

Back to Filters

Question 19 of 78

Which of the following statements with regard to infant formula and breast milk is most factually correct?

A The fat content of breast milk varies throughout the day and throughout a feed

B The iron content of first formula milks closely resembles that of breast milk

C The protein and carbohydrate components of colostrum are higher than those of mature breast milk

D The renal solute load of a casein-based formula is similar to that of a whey-based formula

E There is good evidence to show that casein-predominant formulas are more satisfying for hungrier
babies

Explanation
The composition of human milk is not static. The protein content is higher and the carbohydrate content lower
in
colostrum than in mature milk. Fat contents fluctuate with diurnal variations and within a feed, increasing towards
the end of each feed. Breast milk has whey:casein ratio (70:30), protein of high biological value, long-chain fatty
acids that may improve vision and cognition, and
a low renal solute load. In addition to its nutrients, it also
contains
immunoglobulins, antimicrobial and anti-inflammatory agents, hormones, growth factors and enzymes;
these factors contribute to the immune protection, digestion, and both intestinal and general growth and
development.

The composition of formula feeds in


contrast is closely regulated in terms of the acceptable range of nutritional
components. The two main types available differ in their protein composition with whey:casein ratios of 60:40 and
20:80 in the whey- and casein-dominant formulae respectively. The whey-based formulae
most closely resemble
breast milk and, although evidence is lacking, the casein-based formulae are marketed as more satisfying for the
hungry
baby because the casein forms curds that are more slowly digested. The casein-based formulae have a
higher renal solute load than the whey-based milks and, although suitable from birth, extra care should be
taken in
making them up correctly.

https://mypastest.pastest.com/Secure/TestMe/Browser/436619[‫ ص‬08:16:06 11/12/1437]


MyPastest

In 100 ml Breast milk (approximately) Term formula


Previous Question Next Question
Energy (kcal) 60-75 67

Protein (g) 1.3 1.4

Carbohydrate (g) 7 7.5


End Session
Fat (g) 4.2 3.5

Na+ (mmol) 0.65 0.7

K+ (mmol) 1.54 1.6

Ca2+ (mmol) 0.88 1.0

PO43- (mmol) 0.48 0.6

Iron (mg) 0.08 0.5

43964

Tag Question

Feedback

Difficulty: Average

Peer Responses

Session Progress

Responses Correct: 0

https://mypastest.pastest.com/Secure/TestMe/Browser/436619[‫ ص‬08:16:06 11/12/1437]


MyPastest

Responses Incorrect: 21

Responses Total: 21

Responses - % Correct: 0%

Blog
About Pastest
Contact Us
Help

© Pastest 2016

https://mypastest.pastest.com/Secure/TestMe/Browser/436619[‫ ص‬08:16:06 11/12/1437]


MyPastest

Prefer to use the old MyPastest? Access it here »

Back to Filters

Question 20 of 78

Which of the following statements concerning carbohydrate intolerance in childhood is most factually correct?

A Acquired post-enteritis lactose intolerance is usually transient

B Carbohydrate intolerance is usually inherited in an autosomal dominant fashion

C Symptoms of congenital carbohydrate intolerance always occur in the neonatal period

D Symptoms of inherited carbohydrate intolerance are most common in the first 4 years of life

E Symptoms of lactose intolerance often require total exclusion of all dairy products throughout life

Explanation
Carbohydrates make up at least half of the energy intake in the diet. The main carbohydrates are the storage
polysaccharides (starch, glycogen and cellulose), the disaccharides, lactose and sucrose, and the monosaccharides,
glucose and fructose.

Disorders of disaccharide absorption include the following:

Primary:

Congenital alactasia (exceptionally rare with neonatal presentation)


Sucrose–isomaltase
deficiency (even rarer than lactase deficiency): symptoms when fruit (sucrose) added
to the diet, will not tolerate glucose feeds

Secondary:

Post-enteritis (rotavirus): usually resolves after a short period – days to 2 weeks


Neonatal surgery, malnutrition

https://mypastest.pastest.com/Secure/TestMe/Browser/436619[‫ ص‬08:16:30 11/12/1437]


MyPastest

Untreated coeliac disease


Late-onset
lactose intolerance: in many populations lactase activity declines after the first few years of life.
Previous Question Next Question
Symptoms are rare before age 6 years.
Ten per cent of northern Europeans but >80% of Africans, Asians,
Inuit and American Indians. Often dose-dependent tolerance with fermented dairy products being less
problematic. Thought to be autosomal
recessive

Disorders of monosaccharide absorption End Session


Primary:
glucose–galactose malabsorption (very rare autosomal recessive) – intolerance of feeds with
watery diarrhoea from birth, stops with cessation of feeds. Will tolerate fructose feeds.
Secondary:– post-enteritis – neonatal surgery – malnutrition

Carbohydrate
intolerance is usually lactose intolerance and is usually acquired. The
deficiency is the brush border
enzyme lactase which hydrolyses lactose into glucose and galactose. The intolerance will present with
characteristic loose explosive stools. The diagnosis is made by looking for reducing substances in the stools after
carbohydrate ingestion; detection of >0.5% is significant. Treatment is with lactose-free formula in infancy and a
reduced lactose intake in later childhood. After gastroenteritis, carbohydrate intolerance can be to either
disaccharides or monosaccharides. Both types of intolerance are usually transient and both respond to removal of
the offending carbohydrate. Both result in positive reducing substances in the stools.
43966

Tag Question

Feedback

Difficulty: Average

Peer Responses

Session Progress

Responses Correct: 0

https://mypastest.pastest.com/Secure/TestMe/Browser/436619[‫ ص‬08:16:30 11/12/1437]


MyPastest

Responses Incorrect: 22

Responses Total: 22

Responses - % Correct: 0%

Blog
About Pastest
Contact Us
Help

© Pastest 2016

https://mypastest.pastest.com/Secure/TestMe/Browser/436619[‫ ص‬08:16:30 11/12/1437]


MyPastest

Prefer to use the old MyPastest? Access it here »

Back to Filters

Question 21 of 78

Which of the following statements with regard to cows’ milk, breast (human) milk and follow-on formulas is the
most
factually correct?

A Calogen contains 250 kcal/100 g

B Duocal contains carbohydrate, protein and fat

C Maxijul is a glucose polymer

D Polycal contains MCT fats

E Scandishakes powder is a cows’ milk, protein-free, calorie supplement

Explanation
Nutritional supplements are indicated for
use in patients with disease-related malnutrition, malabsorption states
or
any other conditions requiring fortification with a high or readily available calorie supplement. These include
glucose polymer-based supplements:

Maxijul super 380 kcal/100 g powder


Maxijul liquid 200 kcal/100 ml
Vitajoule powder 380 kcal/100 g

Also supplements based on maltodextrin:

Caloreen powder 390 kcal/100 g


Polycal powder 384 kcal/100 g
Polycal liquid 247 kcal/100 ml

https://mypastest.pastest.com/Secure/TestMe/Browser/436619[‫ ص‬08:16:53 11/12/1437]


MyPastest

Nutritional supplements based on fat have generally more calories per ml than their carbohydrate equivalents and
include:
Previous Question Next Question
Calogen liquid 450 kcal/100 ml
Liquigen liquid 450 kcal/100 ml

Nutritional supplements based on fat and carbohydrate includes: End Session


Duocal preparations (calorific values vary with preparations)

Nutritional supplements based on protein, fat and carbohydrate includes:

Scandishake 500 kcal/100 g – contains cows’ milk protein and lactose

43969

Tag Question

Feedback

Difficulty: Average

Peer Responses

Session Progress

Responses Correct: 0

Responses Incorrect: 23

Responses Total: 23

Responses - % Correct: 0%

https://mypastest.pastest.com/Secure/TestMe/Browser/436619[‫ ص‬08:16:53 11/12/1437]


MyPastest

Blog
About Pastest
Contact Us
Help

© Pastest 2016

https://mypastest.pastest.com/Secure/TestMe/Browser/436619[‫ ص‬08:16:53 11/12/1437]


MyPastest

Prefer to use the old MyPastest? Access it here »

Back to Filters

Question 22 of 78

Which of the following statements relating to Wilson disease is most factually correct?

A A diet low in copper should be low in shellfish, liver and chocolate

B A presentation with psychiatric or neurological symptoms is rare

C The majority of patients with Wilson disease present with liver and renal impairment in late
adolescence and early childhood

D Wilson disease has a poor outcome even if diagnosed early and treated appropriately

E Wilson disease is sporadically inherited

Explanation
Wilson disease is an autosomal recessive disorder of copper metabolism. The gene has been identified on
chromosome 13 and dysfunction of this gene decreases both copper excretion into bile and ceruloplasmin
production. Ceruloplasmin is the plasma protein that transports copper. Wilson disease presents in 40% with
features of liver disease usually between the ages of 3 and 12 years. Approximately 50% have a psychiatric or
neurological presentation, usually in adolescence or early adult life; 50% of these will have clinically detectable
liver disease. The remainder present with skeletal, renal or haemolytic disease. The effects of the disease include
the following:

Liver: chronic active hepatitis, portal hypertension and fulminant hepatic failure
Brain: progressive lenticular degradation due to copper deposition, often leads to tremor
Cornea: Kayser–Fleischer rings
Lens: sunflower cataract
Kidney: renal tubular disorders
Blood: haemolysis

https://mypastest.pastest.com/Secure/TestMe/Browser/436619[‫ ص‬08:17:14 11/12/1437]


MyPastest

Diagnosis
can be difficult. The important part is to include it in the differential of all undiagnosed liver disease in
Previous
childhood. Biochemical Question
findings include: Next Question
low plasma ceruloplasmin level <200 mg/L
Raised urinary copper >25 μmol/24 h after penicillamine (the penicillamine challenge)
Baseline urinary copper is unreliable with poor sensitivity and specificity (>5 μmol/24 h is suggestive)
Raised hepatic copper on liver biopsy
End Session
Note that serum copper levels are not helpful

If
untreated the disease is fatal, usually by the age of 30 years. If identified early and treated, the prognosis is
good. Treatment is with oral penicillamine as a copper-binding agent plus a low-copper diet (avoid excessive
chocolate, shellfish and liver). Patients on penicillamine require vitamin B6 (pyridoxine) supplementation
to avoid
neuropathy. Liver transplantation may be indicated if presentation is with fulminant liver failure. It is important to
screen siblings of patients with Wilson disease to start treatment early and improve prognosis.
43970

Tag Question

Feedback

Difficulty: Average

Peer Responses

Session Progress

Responses Correct: 0

Responses Incorrect: 24

Responses Total: 24

Responses - % Correct: 0%

https://mypastest.pastest.com/Secure/TestMe/Browser/436619[‫ ص‬08:17:14 11/12/1437]


MyPastest

Blog
About Pastest
Contact Us
Help

© Pastest 2016

https://mypastest.pastest.com/Secure/TestMe/Browser/436619[‫ ص‬08:17:14 11/12/1437]


MyPastest

Prefer to use the old MyPastest? Access it here »

Back to Filters

Question 23 of 78

Which of the following statements with regard to neonatal hyperbilirubinaemia is most factually correct?

A Babies
who become jaundiced whilst breastfeeding are significantly at risk of dehydration and require
supplementation with formula milk

B Babies who have rhesus incompatibility and haemolysis will require exchange transfusion

C Clinical examination of neonates is an acceptable way to quantify the extent of neonatal jaundice and
guide treatment

D Neonates
who develop jaundice within the first 24 h need urgent medical review to rule out underlying
potentially severe aetiology for their jaundice

E Unconjugated hyperbilirubinaemia can present with dark urine, due to high levels of bilirubin in the
urine

Explanation
Jaundice can be either conjugated (direct) or unconjugated (indirect). Unconjugated is fat soluble and does not
spill over into the urine. Up to 60% of term and 80% of preterm
newborns become clinically jaundiced in the first
week of life; approximately 10% of breastfed babies will still be jaundiced at 1 month
of age. Most neonatal
jaundice is harmless but some children will develop very high levels of unconjugated bilirubin, which can cross
the blood–brain barrier and cause long-term neurological damage. Clinical assessment of the level of bilirubin is
difficult and so, if jaundice is detected, a serum bilirubin should be checked and managed according to threshold
charts based on gestational age and treatment guidance issued by the National Institute for Health and Clinical
Excellence (NICE). Additional care should be taken if the child develops
jaundice within the first 24 hours of life.
Most cases can be managed with phototherapy and supportive measures; however, some children with very high
levels of bilirubin that persist despite treatment or any baby
with clinical signs of acute bilirubin encephalopathy
may require an exchange transfusion. Always identify and treat any underlying cause of hyperbilirubinaemia as
appropriate.

https://mypastest.pastest.com/Secure/TestMe/Browser/436619[‫ ص‬08:17:37 11/12/1437]


MyPastest

Causes of unconjugated hyperbilirubinaemia in the neonatal period include:

IncreasedPrevious
production ofQuestion Next Question
unconjugated bilirubin from haem
Haemolysis:
– rhesus incompatibility – ABO incompatibility – hereditary spherocytosis – glucose-6-
phosphate dehydrogenase deficiency – pyruvate kinase deficiency
Polycythaemia/bruising
Sepsis congenital or acquired
End Session
Decreased
bilirubin uptake or metabolism: – physiological – sepsis,
acidosis and hypoxia – Gilbert
syndrome (7% of population mild) – Crigler–Najjar syndrome (can be severe – rare) – breast milk jaundice
– hypothyroidism
Altered
enterohepatic circulation: – breast milk jaundice – antibiotic administration – intestinal
obstruction: – ileal atresia – Hirschsprung disease – cystic fibrosis including meconium ileus – pyloric
stenosis.

43972

Tag Question

Feedback

Difficulty: Average

Peer Responses

Session Progress

Responses Correct: 0

Responses Incorrect: 25

Responses Total: 25

Responses - % Correct: 0%

https://mypastest.pastest.com/Secure/TestMe/Browser/436619[‫ ص‬08:17:37 11/12/1437]


MyPastest

Blog
About Pastest
Contact Us
Help

© Pastest 2016

https://mypastest.pastest.com/Secure/TestMe/Browser/436619[‫ ص‬08:17:37 11/12/1437]


MyPastest

Prefer to use the old MyPastest? Access it here »

Back to Filters

Question 24 of 78

Which of the following statements with regard to coeliac disease is the most factually correct?

A According to NICE guidelines type 1 diabetes mellitus is an indication for screening for coeliac
disease

B A family history of coeliac disease is present in 40% of people diagnosed with coeliac disease

C Positive serology (IgA TTG, or IgG TTG if IgA deficient) is sufficient to make a diagnosis of coeliac
disease

D The
effects of gluten on serology and small bowel mucosa are long lived so it is acceptable to advise
patients with possible coeliac disease to exclude gluten whilst awaiting referral for definitive diagnosis
and management

E The
majority of patients with coeliac disease present with irritability, pallor and malabsorption
(diarrhoea and failure to thrive) in infancy after the introduction of gluten into the diet

Explanation
This is a reversible immune-mediated enteropathy of the small intestinal mucosa that occurs on exposure to
ingested gluten. Its prevalence in the UK is between 0.3% and 1.9%. Patients can present at any time throughout
life with a wide range of symptoms; more patients are diagnosed in adulthood than childhood. The gold standard
of
diagnosis is positive serology (IgA tissue transglutaminase [TTG], or IgG TTG if IgA deficient), characteristic
histology (subtotal villous atrophy) on small bowel biopsy and resolution of symptoms when gluten is
removed.
Of note, if the patient is taking insufficient gluten before serology and biopsy, both may be within the normal
range and so the diagnosis must be confirmed on gluten (>10–15 g/day, for 6 weeks to 3 months). Risk factors for
developing coeliac disease include other autoimmune diseases (eg type 1 diabetes mellitus), first-degree relative
with coeliac disease (10%) and a diagnosis of IgA deficiency.

GI symptoms and signs include:

https://mypastest.pastest.com/Secure/TestMe/Browser/436619[‫ ص‬08:17:59 11/12/1437]


MyPastest

Persistent or intermittent diarrhoea


Abdominal pain
Vomiting
Previous Question Next Question
Constipation
Abdominal bloating/distension
Anorexia
Flatulence End Session

Non-GI symptoms and signs include:

Recurrent aphthous stomatitis


Faltering growth
Weight loss
Short stature
Delayed menarche/puberty
Amenorrhoea
Dental enamel defects
Prolonged fatigue/lethargy
Iron deficiency anaemia
Rickets/osteomalacia
Osteoporosis/pathological fracture
Dermatitis herpetiformis
Irritability/depression

Treatment
is lifelong adherence to a gluten-free diet, exclusion of wheat, rye, barley and oats (due to
contamination with gluten during milling and manufacture). Consequences of non-adherence to diet include
recurrence of symptoms, poor growth, delayed puberty, dental enamel problems, osteoporosis, and small increased
risk of malignancy, eg small bowel lymphoma.

The NICE guidelines advocate targeted screening via serological testing of children with symptoms of, or risk
factors for, coeliac disease.

https://www.nice.org.uk/guidance/ng20 43973

Tag Question

Feedback

Difficulty: Average

https://mypastest.pastest.com/Secure/TestMe/Browser/436619[‫ ص‬08:17:59 11/12/1437]


MyPastest

Peer Responses

Session Progress

Responses Correct: 0

Responses Incorrect: 26

Responses Total: 26

Responses - % Correct: 0%

Blog
About Pastest
Contact Us
Help

© Pastest 2016

https://mypastest.pastest.com/Secure/TestMe/Browser/436619[‫ ص‬08:17:59 11/12/1437]


MyPastest

Prefer to use the old MyPastest? Access it here »

Back to Filters

Question 25 of 78

Which of the statements about hepatitis B is the most factually correct?

A Anti-HBs antibodies suggest that a chronic carrier state has developed after acute infection

B Hepatitis B is a DNA virus

C Interferon-γ is a recognised treatment of the chronic carrier state

D Transmission of hepatitis B is faeco-oral

E Treatment of acute hepatitis B infection is by passive immunisation

Explanation
Hepatitis B is a DNA virus. Diagnosis is by detection of the hepatitis B surface antigen (HBsAg). Hepatitis B ‘e’
antigen (HBeAg)-positive patients carry a larger virus load and are more infectious. Acute and ongoing chronic
infection is associated with anti-hepatitis B core (HBc) IgM. Anti-HBe and anti-HBs antibodies appear as an
effective immune response develops. All HBsAg-positive individuals are infective. The route of transmission is
either percutaneous (puncture through the skin) or mucosal (exposure to infectious blood or bodily fluids). The
perinatal transmission rate depends on the maternal serology. If the mother is HBsAg positive and HBeAg
negative, the risk is 12–25%. If the mother is both HBsAg and HBeAg positive the risk is 90%. The younger the
age at infection the
less the likelihood of symptomatic liver disease but the greater the risk of prolonged viral
carriage. Ninety per cent of infants infected in
the first year of life become chronic carriers.

Clinically
the disease is often asymptomatic but an acute hepatic picture can develop. Acute liver failure occurs in
less than 1%. The risk of fulminant hepatitis is increased by coinfection with hepatitis D. In those with a typical
hepatic picture the chronic carrier rate is low. Chronicity results in an increased risk of cirrhosis and hepatocellular
carcinoma. Boys are more likely to become chronic carriers than girls. Chronically infected children have a 25%
lifetime risk of cirrhosis or hepatocellular carcinoma. Prevention is by both active and passive immunisation.
Interferon-α is a recognised treatment of chronic infection.

https://mypastest.pastest.com/Secure/TestMe/Browser/436619[‫ ص‬08:18:20 11/12/1437]


MyPastest

Hepatitis
D is an RNA virus that requires the HBsAg for its assembly and virulence. It is transmitted similarly to
hepatitis B. The severity of the liver damage increases if there is coexistent hepatitis B infection.
Diagnosis is by
Previous
serology. Prevention Question
of vertical Next Question
transmission of hepatitis B is with immunisation of all neonates born to hepatitis
B-infected mothers with the addition of HBIg for all babies <1500 g, those whose mothers had acute hepatitis B
infection in pregnancy, those whose mothers are HBsAg positive plus either HBeAg positive or anti-HBe
negative.
43974

End Session

Tag Question

Feedback

Difficulty: Average

Peer Responses

Session Progress

Responses Correct: 0

Responses Incorrect: 27

Responses Total: 27

Responses - % Correct: 0%

Blog
About Pastest
Contact Us
Help

© Pastest 2016

https://mypastest.pastest.com/Secure/TestMe/Browser/436619[‫ ص‬08:18:20 11/12/1437]


MyPastest

Prefer to use the old MyPastest? Access it here »

Back to Filters

Question 26 of 78

Which of the following statements concerning hepatitis A is the most factually correct?

A Chronic liver disease commonly follows acute infection

B Diagnosis of acute infection is by stool culture

C Hepatitis A is an RNA virus

D Passive immunisation produces lifelong immunity to infection

E Transmission is most commonly by exposure to contaminated blood at delivery

Explanation
Hepatitis A is an RNA virus. Diagnosis is
by detection of the hepatitis A virus IgM. The route of transmission is
faeco-oral. There is no carrier state and fulminant hepatic failure is rare (<0.1%). Liver function, however, may be
abnormal for up to 1 year. Prevention is by either passive or active immunisation. Passive immunisation is with
immunoglobulin and lasts for 3–6 months. Active immunisation is with a live attenuated virus, with booster
immunisation being required after 12–18 months. Clinical symptoms are initially non-specific and include
anorexia, nausea, fatigue and fever associated with epigastric pain and tender hepatomegaly. The icteric phase
then develops with jaundice, pale stools and dark urine. Sometimes there is pruritus, depression and persistent
jaundice, with raised transaminases for a prolonged period. The prothrombin time (PTT) should be monitored. A
raised PTT raises the possibility of severe hepatic necrosis or decompensation of underlying liver disease.

Hepatitis
E is an RNA virus. Epidemics occur in developing countries. In the UK infection is usually seen in
travellers from endemic areas. The route of
transmission is faeco-oral. The clinical course of hepatitis E infection
is similar to hepatitis A. Complete recovery from acute infection occurs. Chronic infection has not been described,
although acute fulminant hepatic failure can occur and is more common during pregnancy. Diagnosis is by
serology. No vaccine or prophylactic treatment is available.

https://mypastest.pastest.com/Secure/TestMe/Browser/436619[‫ ص‬08:18:41 11/12/1437]


MyPastest

43975

Previous Question Next Question

Tag Question

Feedback End Session

Difficulty: Average

Peer Responses

Session Progress

Responses Correct: 0

Responses Incorrect: 28

Responses Total: 28

Responses - % Correct: 0%

Blog
About Pastest
Contact Us
Help

© Pastest 2016

https://mypastest.pastest.com/Secure/TestMe/Browser/436619[‫ ص‬08:18:41 11/12/1437]


MyPastest

Prefer to use the old MyPastest? Access it here »

Back to Filters

Question 27 of 78

Which of the following statements about hepatitis C is the most factually correct?

A Caesarean section is advised in all cases of hepatitis C in the developed world to reduce vertical
transmission

B Maternal infection with hepatitis C is a contraindication to breastfeeding

C Post-delivery immunisation is advised in all cases to decrease vertical transmission

D Transmission worldwide is most commonly by the vertical route

E Treatment with ribavirin as a monotherapy is currently recommended in the UK for children with
moderate-to-severe liver disease

Explanation
Hepatitis C is an RNA virus. The route of
transmission is most commonly vertical, otherwise infection occurs via
parenteral, sexual or contaminated blood product transfusion. The vertical transmission rate is 9%, higher in HIV-
positive mothers. Mode of delivery does not affect the risk of transmission unless coinfection with HIV is present,
in which case caesarean section in the developed world may offer some protection.

Diagnosis is usually by serology with the detection of the anti-HCV antibody. Blood and blood products for
transfusion have been screened for hepatitis C virus since 1990. Infection is usually asymptomatic or an acute
hepatitis can occur. Fulminant hepatitis is uncommon but can occur. HCV RNA detection establishes the presence
of viraemia, confirming infection
and infectivity. Persistence of HCV RNA indicates continuing infection.
Chronic infection is common (prevalence 0.2–0.7% in northern Europe, 1–2% in southern Europe and Japan),
with the development of cirrhosis and hepatocellular carcinoma in a number of cases after an interval of 10–15
years. Treatment with pegylated interferon plus ribavirin is the currently recommended treatment for children.
Duration of treatment varies depending on the genotype of the infecting virus. Good prognostic factors for
antiviral treatment are the absence of cirrhosis, young age at acquisition, and absence or coinfection with HIV
and/or hepatitis B. Breastfeeding is not contraindicated for HCV seropositive mothers but, if nipples are cracked

https://mypastest.pastest.com/Secure/TestMe/Browser/436619[‫ ص‬08:19:01 11/12/1437]


MyPastest

and bleeding, caution is advised.


43976

Previous Question Next Question

Tag Question
End Session
Feedback

Difficulty: Average

Peer Responses

Session Progress

Responses Correct: 0

Responses Incorrect: 29

Responses Total: 29

Responses - % Correct: 0%

Blog
About Pastest
Contact Us
Help

© Pastest 2016

https://mypastest.pastest.com/Secure/TestMe/Browser/436619[‫ ص‬08:19:01 11/12/1437]


MyPastest

Prefer to use the old MyPastest? Access it here »

Back to Filters

Question 28 of 78

From the following list of infective agents and disease processes, which is most likely to cause the clinical picture
of
colitis with abdominal pain and bloody diarrhoea?

A Coeliac disease

B Giardia lamblia

C Non-typhoid Salmonella sp

D Peutz–Jegher syndrome

E Rotavirus infection

Explanation
Abdominal pain associated with bloody diarrhoea is indicative of colitis. Colitis can be infective or non-infective.

Causes of infective colitis:

Salmonella spp.
Shigella spp.
Campylobacter pylori
Escherichia coli O157 (and other E. coli)
Clostridium difficile (pseudomembranous colitis)
Yersinia spp.
Tuberculosis
Cytomegalovirus
Entamoeba histolytica
Enterobius vermicularis.

https://mypastest.pastest.com/Secure/TestMe/Browser/436619[‫ ص‬08:19:23 11/12/1437]


MyPastest

Causes of non-infective colitis:


Previous Question Next Question
Ulcerative colitis
Crohn disease
Necrotising enterocolitis
Vasculitic causes, eg Henoch–Schönlein purpura End Session
Hirschsprung enterocolitis
Microscopic colitis
Allergic enterocolitis.

43977

Tag Question

Feedback

Difficulty: Average

Peer Responses

Session Progress

Responses Correct: 0

Responses Incorrect: 30

Responses Total: 30

Responses - % Correct: 0%

https://mypastest.pastest.com/Secure/TestMe/Browser/436619[‫ ص‬08:19:23 11/12/1437]


MyPastest

Blog
About Pastest
Contact Us
Help

© Pastest 2016

https://mypastest.pastest.com/Secure/TestMe/Browser/436619[‫ ص‬08:19:23 11/12/1437]


MyPastest

Prefer to use the old MyPastest? Access it here »

Back to Filters

Question 29 of 78

Which of the following statements about Giardia intestinalis is the most factually correct?

A Asymptomatic infection in developed countries should not be treated with antibiotics

B Giardia intestinalis infection rate is highest in adolescents

C Giardia intestinalis infection should be treated with erythromycin

D Giardia intestinalis is a protozoal infection, the only host of which is humans

E Infection with Giardia intestinalis can


cause chronic diarrhoea, malabsorption, failure to thrive and
intestinal histological changes similar to those seen in coeliac disease

Explanation
It is a protozoal parasite that is infective in the cyst form and can grow in the intestines of humans or animals.
Infection in humans is more common in children than in adults; the rates of infection in the UK are highest for the
1- to 4-year age group. It is endemic in areas with poor sanitation and commonly associated with outbreaks
secondary to food or water contamination. As relatively low numbers of cysts are required to produce infection,
outbreaks in the developed world are often associated with day-care settings and institutions. Children at particular
risk are those who have travelled to endemic areas or have contact with those who have travelled to endemic areas,
those who are malnourished, those with immunodeficiencies (recurrent infection is seen in those with IgA
deficiency) and those with cystic fibrosis. Clinical manifestations include:

asymptomatic infection
acute diarrhoeal illness
chronic diarrhoea and malabsorption (may last weeks to months)

Chronic
infection may be associated with partial villous atrophy in small bowel
biopsy. Diagnosis is by stool

https://mypastest.pastest.com/Secure/TestMe/Browser/436619[‫ ص‬08:19:44 11/12/1437]


MyPastest

examination for cysts or examination of the duodenal aspirate at small bowel biopsy. Treatment consists of
appropriate fluid and nutritional management and/oral metronidazole for all infected cases who are symptomatic
or asymptomaticPrevious Question
in non-endemic Next Question
areas. Good basic hygiene procedures are essential for preventing spread.
43978

End Session
Tag Question

Feedback

Difficulty: Average

Peer Responses

Session Progress

Responses Correct: 0

Responses Incorrect: 31

Responses Total: 31

Responses - % Correct: 0%

Blog
About Pastest
Contact Us
Help

© Pastest 2016

https://mypastest.pastest.com/Secure/TestMe/Browser/436619[‫ ص‬08:19:44 11/12/1437]


MyPastest

Prefer to use the old MyPastest? Access it here »

Back to Filters

Question 30 of 78

Which of the following statements with regard to ulcerative colitis is most factually correct?

A FBC and inflammatory markers within the normal range almost always exclude a diagnosis of
ulcerative colitis

B Growth failure is a common presentation of ulcerative colitis and often precedes GI symptoms by
months or even years

C Half
the patients presenting with ulcerative colitis present with classic colitic symptoms of abdominal
pain and bloody diarrhoea

D The pANCA blood tests are positive in 15% of cases of ulcerative colitis and 45% of cases of Crohn
disease

E Ulcerative colitis is more common than Crohn disease in younger children and in Asian children

Explanation
Ulcerative colitis is an inflammatory condition limited to the colon. Most children (90%) have pancolitis; the
rest
have left-sided colonic disease, or proctitis. Ulcerative colitis is a lifelong illness with 50% of patients relapsing in
any 1 year. Children with moderate-to-severe disease at diagnosis have a colectomy rate of 25% at 5 years. The
histological changes are mucosal and submucosal inflammation, goblet cell depletion, cryptitis and crypt
abscesses, but no granulomas. Twenty-five per cent of cases of inflammatory bowel disease (IBD) present in those
aged <16 years; approximately 30% is ulcerative colitis. Ulcerative colitis is the most common form of IBD in the
younger child. It is slightly more common in Asian children than in other ethnic groups. The sex distribution in
childhood ulcerative colitis is even. A family history of either Crohn disease or ulcerative colitis is common in any
index case; breastfeeding
appears to be protective and certain GI infections may increase the risk of developing
IBD. Most patients with ulcerative colitis present with symptoms of colitis: diarrhoea, abdominal pain and blood
per rectum. Rarely the disease can present with extraintestinal manifestations, including arthropathy (10%),
ankylosing spondylitis (rare in children) liver disease (sclerosing cholangitis, autoimmune liver disease) and

https://mypastest.pastest.com/Secure/TestMe/Browser/436619[‫ ص‬08:20:08 11/12/1437]


MyPastest

erythema nodosum. Growth failure is much less commonly associated than with Crohn disease and the presenting
blood panel may be normal. The peripheral antineutrophil cytoplasmic antibody (pANCA) is positive in 70% of
ulcerative colitisPrevious
and <10% ofQuestion Next Question
Crohn disease cases.

Complications include:

Toxic megacolon
Osteoporosis
Colonic cancer
End Session
Increased thrombotic tendency in severe disease

Treatments include:

5-Aminosalicylic acid (ASA) derivatives


Local or systemic steroids, intravenously in severe colitis
Antibiotics if infection is suspected
Azathioprine or 6-mercaptopurine in recurrent–relapse or treatment-resistant disease
Ciclosporin in resistant disease
Infliximab in severe non-responding ulcerative colitis to delay surgery
Surgery

43979

Tag Question

Feedback

Difficulty: Average

Peer Responses

Session Progress

https://mypastest.pastest.com/Secure/TestMe/Browser/436619[‫ ص‬08:20:08 11/12/1437]


MyPastest

Responses Correct: 0

Responses Incorrect: 32

Responses Total: 32

Responses - % Correct: 0%

Blog
About Pastest
Contact Us
Help

© Pastest 2016

https://mypastest.pastest.com/Secure/TestMe/Browser/436619[‫ ص‬08:20:08 11/12/1437]


MyPastest

Prefer to use the old MyPastest? Access it here »

Back to Filters

Question 31 of 78

Which of the statements about recurrent abdominal pain in childhood is the most factually correct?

A A childhood history of recurrent abdominal pain is rarely associated with irritable bowel syndrome in
adult life

B A family history of coeliac disease is common in these children

C A family history of migraine is associated with a subset of children with recurrent abdominal pain

D Introducing
the biopsychosocial model of illness is best accepted at the later stages of management of
these children when organic diagnoses have been excluded

E There is good evidence to suggest that the symptom of pain in these children is fictitious and made up
for secondary gain

Explanation
Recurrent abdominal pain is common in childhood, affecting 10% of the school-aged population. In the vast
majority of cases the aetiology is non-organic. The condition is more common in girls and a family history is
common. The pain is usually periumbilical and rarely associated with other GI symptoms. Symptoms suggestive
of an organic cause include:

Age <5 years


Constitutional symptoms (fever, weight loss, delayed growth, skin rashes, arthralgia)
Vomiting, especially if bilious
Nocturnal pain
Pain away from the umbilicus
Urinary symptoms
Family history of organic GI pathology

https://mypastest.pastest.com/Secure/TestMe/Browser/436619[‫ ص‬08:20:30 11/12/1437]


MyPastest

Perianal disease
Blood in the stools
Previous Question Next Question
The
ideal management is a thorough initial history and examination, to help
to exclude the rare cases of organic
disease, followed by limited investigations. Once a diagnosis is made classification into one of four
subtypes may
aid ongoing management:

1. Functional abdominal pain (syndrome) End Session


2. Functional dyspepsia
3. Irritable bowel syndrome
4. Abdominal migraine

Many
cases respond to the acknowledgement of the symptoms and reassurance about the lack of serious
underlying pathology. The acceptance by the child and parents of the bipsychosocial model of illness is an
important
factor for the resolution of symptoms (butterflies in stomach with exams). Diet and lifestyle changes,
including avoidance of trigger factors and improvement of associated constipation, increased exercise and a staged
programme of school reintroduction if applicable, can prove
helpful. The introduction of the concept of pain
management early in the therapeutic relationship is often helpful. There are data to suggest
that chronic abdominal
pain in childhood is associated with the development of irritable bowel syndrome and anxiety disorders in adult
life.
43980

Tag Question

Feedback

Difficulty: Average

Peer Responses

Session Progress

Responses Correct: 0

Responses Incorrect: 33

https://mypastest.pastest.com/Secure/TestMe/Browser/436619[‫ ص‬08:20:30 11/12/1437]


MyPastest

Responses Total: 33

Responses - % Correct: 0%

Blog
About Pastest
Contact Us
Help

© Pastest 2016

https://mypastest.pastest.com/Secure/TestMe/Browser/436619[‫ ص‬08:20:30 11/12/1437]


MyPastest

Prefer to use the old MyPastest? Access it here »

Back to Filters

Question 32 of 78

Of the following causes of portal hypertension, which can be best described as being prehepatic?

A Biliary atresia

B Budd–Chiari syndrome

C Constrictive pericarditis

D Portal vein thrombosis

E Schistosomiasis

Explanation
The portal vein carries about 1500 ml/min
of blood from the small and large bowel, spleen, stomach and pancreas
to the liver at a pressure of 5–10 mmHg. Any obstruction or increased resistance to flow or, rarely, pathological
increases in portal vein blood flow may lead to portal hypertension, defined as portal pressures >12 mmHg.

Alcoholic and viral cirrhosis are the most common causes in the western world.
Liver disease due to schistosomiasis is the main cause in other areas of the world.
Portal vein thrombosis is the most common cause in children

Aetiology of portal hypertension in children

Prehepatic:

Portal
vein thrombosis (accounts for 30% of children with bleeding varices); causes include: sepsis,
thrombophilia, umbilical cauterization, pancreatitis.

Intrahepatic:

https://mypastest.pastest.com/Secure/TestMe/Browser/436619[‫ ص‬08:20:55 11/12/1437]


MyPastest

Cirrhotic
(most common cause of portal hypertension) – extrahepatic biliary atresia – cystic fibrosis – α1-
antitrypsin deficiency – Wilson disease – schistosomiasis
Non-cirrhotic: veno-occlusive disease of the liver (sinusoidal obstruction syndrome)

Posthepatic:

Budd–Chiari syndrome: hepatic vein occlusion, usually secondary to myeloproliferative disease or


thrombophilia
Right ventricular failure
Constrictive pericarditis

Clinical features of portal hypertension

Splenomegaly
Ascites
Prominent abdominal vessels (caput medusa)
Oesophageal varices
Haemorrhoids
Rectal varices

Consequences of portal hypertension

Increased
portal pressure causes an increase in portosystemic collaterals, resulting in a disturbed intrahepatic
circulation. This may lead to the clinical complications:

Variceal bleeding
Hepatic encephalopathy
Ascites
Hepatorenal syndrome
Recurrent infection, spontaneous bacterial peritonitis
Abnormalities in coagulation

Hepatorenal syndrome

Acute oliguric renal failure resulting from intense intrarenal vasoconstriction in otherwise normal kidneys
Prognosis is poor, treatment often ineffective
In liver failure avoid nephrotoxic drugs
Prevented by avoiding excessive diuresis and by early recognition of electrolyte imbalance, bleeding or
infection

43981

https://mypastest.pastest.com/Secure/TestMe/Browser/436619[‫ ص‬08:20:55 11/12/1437]


MyPastest

Previous Question Next Question


Tag Question

Feedback

End Session
Difficulty: Average

Peer Responses

Session Progress

Responses Correct: 0

Responses Incorrect: 34

Responses Total: 34

Responses - % Correct: 0%

Blog
About Pastest
Contact Us
Help

© Pastest 2016

https://mypastest.pastest.com/Secure/TestMe/Browser/436619[‫ ص‬08:20:55 11/12/1437]


MyPastest

Prefer to use the old MyPastest? Access it here »

Back to Filters

Question 33 of 78

Which of the following statements about rectal prolapse is the MOST correct?

A It can always be resolved with a short course of laxatives and dietary advice

B It is associated with cystic fibrosis in most cases

C It is most common in the pre-school-age group

D It is usually painful

E It requires reduction under general anaesthetic

Explanation
Rectal prolapse is most common below 4 years of age and most cases occur in the first year of life. It can involve
protrusion of the mucosal layer alone or all the layers (procidential).

Causes of rectal prolapse:

Idiopathic
Increased abdominal pressure: – constipation – chronic cough – toilet training
Diarrhoeal disorders: – acute infection – malabsorption
Cystic
fibrosis (20% of patients with CF have rectal prolapse between 6 months
and 3 years and CF
accounts for 10% of all cases of rectal prolapse)
Neuromuscular syndromes
Rectal polyps
Malnutrition/anorexia
Child abuse (anal sex)

https://mypastest.pastest.com/Secure/TestMe/Browser/436619[‫ ص‬08:21:17 11/12/1437]


MyPastest

In
most cases spontaneous reduction has taken place by the time of presentation, so a brief examination in the
squatting position is advised to try to visualise a recurrence. It is usually painless, although failure to reduce the
prolapsed tissuePrevious
can result inQuestion Next Question
oedema and
ulceration. Parents can be taught to reduce a prolapse at home. Patients
with this presentation should be considered for a sweat test if
no other explanation is forthcoming. Management
should always be conservative in the first instance especially if aged <4 years (90%);
will respond by 6 years of
age. This involves treating any underlying cause and reducing straining through the use of diet and laxatives to
reduce constipation. Surgery is occasionally required for recurrent or resistant prolapse or that associated with
ulceration or pain; it can be
associated with complications. End Session
43982

Tag Question

Feedback

Difficulty: Average

Peer Responses

Session Progress

Responses Correct: 0

Responses Incorrect: 35

Responses Total: 35

Responses - % Correct: 0%

Blog
About Pastest
Contact Us
Help

© Pastest 2016

https://mypastest.pastest.com/Secure/TestMe/Browser/436619[‫ ص‬08:21:17 11/12/1437]


MyPastest

https://mypastest.pastest.com/Secure/TestMe/Browser/436619[‫ ص‬08:21:17 11/12/1437]


MyPastest

Prefer to use the old MyPastest? Access it here »

Back to Filters

Question 34 of 78

Which is the MOST appropriate statement about constipation?

A It can always be resolved with a short course of laxatives and dietary advice

B It can be precipitated by a child’s refusal to use school toilets

C It is usually associated with an underlying medical condition

D It occurs occasionally in the pre-school-age group

E It should be investigated with rectal biopsy to exclude Hirschsprung disease

Explanation
Constipation is defined (NICE guidelines) as two or more of the following:

Passage of fewer than 3 complete stools per week


Hard ‘rabbit droppings’ or large stools that may block the toilet
Soiling – the passage of smelly loose stools without sensation
Poor appetite or abdominal pain that varies with the passage of stools
Pain associated with defecation
Evidence of retentive posturing
Bleeding associated with hard stools

An
extremely common problem in childhood (5–30% of all children), it
is rarely associated with any underlying
medical problem (5%). A vicious cycle usually develops whereby a child associates pain with defecation and then
withholds stool in an attempt to avoid discomfort. This retention of stool causes enlargement of the rectum and
distal colon (megacolon) to accommodate the faeces, and the normal urge to defecate is lost. Eventually, faecal

https://mypastest.pastest.com/Secure/TestMe/Browser/436619[‫ ص‬08:21:40 11/12/1437]


MyPastest

incontinence (soiling) may ensue. Parents may be able to identify a trigger to the start of the problem, eg a change
in milk, toilet training, after an illness or starting a new
school.
Previous Question Next Question
All children should undergo examination including growth parameters, perianal, spinal, lumbosacral, lower limbs
and abdominal examination to exclude underlying pathology at
presentation. An abdominal radiograph should not
be used to diagnose idiopathic constipation. Treatment involves disimpaction (ideally via the oral route, although
occasionally if oral medications have failed enemas are required), followed by establishing a regular bowel habit
(both behavioural modification and maintenance laxatives). Treatment is usually required for many weeks or
months and must be continued for some
time after the establishment of normalEndbowelSession
habit. Treatment should be
weaned down and never stopped abruptly. Seventy per cent of children will become eventually symptom free off
medication, although a significant cohort will continue to have symptoms despite long-term treatment. Education
and reassurance of parents are essential.

https://www.nice.org.uk/guidance/cg99/evidence/full-guidance-245466253
43983

Tag Question

Feedback

Difficulty: Average

Peer Responses

Session Progress

Responses Correct: 0

Responses Incorrect: 36

Responses Total: 36

Responses - % Correct: 0%

https://mypastest.pastest.com/Secure/TestMe/Browser/436619[‫ ص‬08:21:40 11/12/1437]


MyPastest

Blog
About Pastest
Contact Us
Help

© Pastest 2016

https://mypastest.pastest.com/Secure/TestMe/Browser/436619[‫ ص‬08:21:40 11/12/1437]


MyPastest

Prefer to use the old MyPastest? Access it here »

Back to Filters

Question 35 of 78

Which is the MOST appropriate statement about pancreatitis in childhood?

A A mildly raised amylase level associated with abdominal pain is specifically diagnostic of pancreatitis

B A normal ultrasound examination of the pancreas excludes the diagnosis

C It is a common cause of admission to hospital with abdominal pain in childhood

D It can be caused by treatment with sodium valproate

E It is commonly caused by gallstones in children

Explanation
Pancreatitis is uncommon in childhood. In
up to 25% of cases the aetiology is unknown. Unlike adults, the most
common identifiable causes are abdominal trauma, viral infection (mumps,
rubella, Coxsackie B,
cytomegalovirus), medication (azathioprine, steroids, sodium valproate) and congenital abnormalities of the
pancreatobiliary system. Pseudocysts, a fibrous, walled cavity filled with pancreatic enzymes, complicates
between 10 and 23% of cases and is common in those caused by blunt abdominal trauma. Pancreatitis typically
presents with epigastric pain (which can radiate through to the back), nausea and vomiting, and low-grade fever.
Elevated serum amylase levels are seen in most cases of acute pancreatitis but can take up to 48 h to reach peak
levels. Other abdominal pathologies can cause a rise in serum
amylase but rarely to the degree seen in pancreatitis.

Medical
management consists of rehydration and analgesia, and may also include a
period of being ‘nil by mouth’,
sometimes sufficient to require total parenteral nutrition (TPN) (start at 3 days to prevent catabolism). Antibiotics
may also be indicated in some cases to treat systemic infection or sepsis. Ultrasonography and CT are the most
commonly used imaging modalities, but MRI and endoscopic retrograde cholangiopancreatography (ERCP) can
also be helpful. Findings are variable and include a normal-looking pancreas, initially in up to 20% of cases. The
length of the illness is variable but most acute cases resolve after 2–4 days. There is a recurrence rate of
approximately 9%. Surgical involvement may be required if necrosis, abscess or pseudocyst formation occurs,
although most pseudocysts will spontaneously resolve in 4–6 weeks.

https://mypastest.pastest.com/Secure/TestMe/Browser/436619[‫ ص‬08:22:03 11/12/1437]


MyPastest

43984

Previous Question Next Question

Tag Question

Feedback End Session

Difficulty: Average

Peer Responses

Session Progress

Responses Correct: 0

Responses Incorrect: 37

Responses Total: 37

Responses - % Correct: 0%

Blog
About Pastest
Contact Us
Help

© Pastest 2016

https://mypastest.pastest.com/Secure/TestMe/Browser/436619[‫ ص‬08:22:03 11/12/1437]


MyPastest

Prefer to use the old MyPastest? Access it here »

Back to Filters

Question 36 of 78

A 5-month-old baby with pH study-proven gastro-oesophageal reflux has not improved on treatment with
Gaviscon. The centile chart now shows that the weight gain is beginning to falter.

What would be the next MOST appropriate stage of treatment?

A Addition of an acid suppressant, eg proton pump inhibitor

B Reassurance that, with weaning, symptoms should improve

C Referral for fundoplication

D Trial of hydrolysed formula milk

E Use of prokinetic agent such as domperidone

Explanation
Gastro-oesophageal reflux (GOR) is defined as the passage of gastric contents into the oesophagus with or without
regurgitation or vomiting. Gastro–oesophageal reflux disease (GORD) is when GOR is accompanied by
troublesome symptoms. In addition to irritability and vomiting, affected individuals may present with feeding
difficulties, apnoeas, failure to thrive, asthma, aspiration, stridor or haematemesis. Oesophagitis and oesophageal
strictures may result from more severe cases of GORD.

If
indicated, investigation is ideally done in the form of 24-hour pH probe, increasingly accompanied by multiple
intraluminal impedance tests, which give information on acidic, weakly acidic and non-acidic reflux episodes, the
severity of which does not necessarily correlate with symptoms.

Treatment involves parental reassurance and advice about lifestyle measures. Medical treatments include:

Alginates (eg infant Gaviscon), which should not be used as sole agents, long term
Acid
suppression: – proton pump inhibitors (PPIs) are superior to H2-receptor blockers – increased risk of

https://mypastest.pastest.com/Secure/TestMe/Browser/436619[‫ ص‬08:22:24 11/12/1437]


MyPastest

community-acquired pneumonia and GI infections, particularly with PPIs


Prokinetic agents; the routine use of these agents is not currently recommended
Previous Question Next Question
Drug treatment is not indicated in all cases, but should be used when growth is affected or symptoms occur with
sufficient frequency to cause distress to the infant and/or caregivers. Medical management, as detailed above,
should be the mainstay of treatment. GOR may rarely be associated with cows’ milk protein intolerance, in which
case a hydrolysed formula can be substituted for a 4- to 6-week trial period. In those cases where medical
management fails, surgical intervention by fundoplication should be considered. End Session
Potential surgical complications
and failure rates mean that this operation should be reserved for only the most severe cases.

In symptomatic individuals, resolution of symptoms is usually seen by the age of 2 years, correlating with
developmental maturity including a more upright posture and intake of solids.
43985

Tag Question

Feedback

Difficulty: Average

Peer Responses

Session Progress

Responses Correct: 0

Responses Incorrect: 38

Responses Total: 38

Responses - % Correct: 0%

Blog
About Pastest
Contact Us
Help

https://mypastest.pastest.com/Secure/TestMe/Browser/436619[‫ ص‬08:22:24 11/12/1437]


MyPastest

© Pastest 2016

https://mypastest.pastest.com/Secure/TestMe/Browser/436619[‫ ص‬08:22:24 11/12/1437]


MyPastest

Prefer to use the old MyPastest? Access it here »

Back to Filters

Question 37 of 78

A 30-month-old girl is referred to clinic with a 6-month history of passing up to five watery stools per day.

The
mother is particularly concerned as there are frequently recognisable food particles in the stools. The child’s
centile charts reveal that she is steadily gaining weight along the 50th centile for both height and weight. Dietary
history reveals that she is a slightly fussy eater, tending to graze throughout the day rather than eating three full
meals. She drinks up to 2.5 l of fruit squash per day via a bottle. Examination is unremarkable.

Which is the MOST appropriate course of action?

A Advice
about dietary intake, especially with regard to the amount and type of fluids being offered with
further follow-up to assess weight gain

B Arrange blood tests for full blood count, total IgA and anti-gliadin antibodies

C Assay for faecal elastase to rule out pancreatic insufficiency

D Collection of a stool sample for detection of reducing substances and implementation of a lactose-free
diet

E Microbiological analysis of stool sample (culture and virology assessment) to exclude infective
diarrhoea

Explanation
There are numerous causes of diarrhoea in childhood including the following:

Infection/inflammation
Malabsorption: transient or long term, primary or secondary, fat, protein or carbohydrate
Chronic non-specific diarrhoea of childhood (toddler’s diarrhoea)
Not in fact diarrhoea; overflow constipation, fabricated or induced illness

https://mypastest.pastest.com/Secure/TestMe/Browser/436619[‫ ص‬08:22:46 11/12/1437]


MyPastest

Any
child presenting with chronic (>14 days) diarrhoea should undergo a thorough assessment including a careful
history, head-to-toe physical examination with assessment of growth and visual examination of the stool ± lab
Previous Question Next Question
microscopy and culture, to help guide appropriate ongoing investigation and avoid unnecessary invasive or
excessive procedures where appropriate. Rotavirus is the most common cause of infective diarrhoea worldwide,
followed by adenovirus. Salmonella, Shigella and Campylobacter spp. are the next most common infecting
organisms. Carbohydrate malabsorption is most frequently seen secondary to infective diarrhoea. Pancreatic
enzyme deficiency causes fat malabsorption, tested for by stool faecal elastase; false-positive results can occur in
End
conditions with villous atrophy or due to the dilutional effects of excess watery Session
stools. Protein-losing enteropathy
should be suspected in any child with low albumin and chronic diarrhoea;
it can occur in many different disease
processes and can be tested for by measuring stool α1-antitrypsin.

Chronic
non-specific diarrhoea of childhood presents with the frequent passage of loose stools, often containing
undigested food particles, and is common especially in the run-up to toilet training. The relatively rapid
gut transit
time in children, in addition to an excessive intake of fruit juices/squash (including fructose-containing apple
juice), excess fibre, low dietary fat content and emotional stress, can exacerbate the situation. There are never any
additional features of pain, blood in the
stools, nocturnal stools or associated faltering growth. Parental
reassurance is essential as well as dietary advice, including the type and amount of fluids consumed and follow-up
until the condition improves.
43986

Tag Question

Feedback

Difficulty: Average

Peer Responses

Session Progress

Responses Correct: 0

Responses Incorrect: 39

Responses Total: 39

https://mypastest.pastest.com/Secure/TestMe/Browser/436619[‫ ص‬08:22:46 11/12/1437]


MyPastest

Responses - % Correct: 0%

Blog
About Pastest
Contact Us
Help

© Pastest 2016

https://mypastest.pastest.com/Secure/TestMe/Browser/436619[‫ ص‬08:22:46 11/12/1437]


MyPastest

Prefer to use the old MyPastest? Access it here »

Back to Filters

Question 38 of 78

A 6 month old child is admitted to hospital with episodic abnormal movements. During an episode you observe
bizarre extension and
lateral turning of her head with dystonic posturing. Her parents say she has always been
fussy with feeds and they are struggling to introduce solids. Her physical examination is normal.

Which of the management choices below would be most helpful for this child?

A Dairy-free diet

B MRI Brain scan

C Prednisolone

D Upper GI endoscopy

E Vigabatrin

Explanation
This girl is showing the episodic movements of potential Sandifer’s Syndrome – posturing secondary to gastro-
intestinal reflux most commonly misdiagnosed as seizures.

In Sandifer Syndrome the child may have sudden deviation of the head and neck to one side and the legs to the
other side appearing stretched out. Often the back is arched posteriorly with hyperextension of the spine. Elbows
may be flexed and held posteriorly with hyperextendable hips. Periods of crying and apparent discomfort may
occur, often after posturing has finished. In most cases the rhythmic clonic component as seen in a seizure is not
present. Various stiff postures may be held for 1-3 minutes, often after food. Respiratory irritation may also occur
with significant gastro-oesophageal reflux. When Sandifer’s
Syndrome is suspected the child may need GI
specialist referral with an
upper GI endoscopy to look for evidence of GOR and hiatus hernia particularly.

Whilst dairy intolerance can be a cause of reflux symptoms and distress in a child it is not usually a cause of
abnormal posturing alone. Without further history or symptoms suggestive of this
it would be unusual to
commence a dairy free diet suddenly in this child. If Sandifer’s Syndrome is diagnosed then the reflux may be

https://mypastest.pastest.com/Secure/TestMe/Browser/436619[‫ ص‬08:23:09 11/12/1437]


MyPastest

modified with diet if necessary.

An MRI Brain scan is performed to look for abnormal structures or growths in the brain, which are the underlying
Previous
cause of developmental Question
abnormalities, Next Question
seizures or other symptoms. This history is not most suggestive of a brain
structural abnormality and it is not the most helpful option for the child.

Prednisolone and Vigabatrin are used to treat Infantile Spasms (West Syndrome) associated with a number of
underlying conditions such as Tuberous Sclerosis. Infantile spasms are seizures often linked with poor
developmental outcomes. The spasms have a typical pattern not described in this scenario. They usually consist of
End Session
a brief interruption of behaviour, lifting and extension of the arms, bending forward at the waist and often a rapid,
forceful head drop. Clusters last a few minutes and may be associated with a cry. Vigabatrin is usually used when
a child is not responsive to steroid treatment.

Website reference for GORD management: https://www.nice.org.uk/guidance/ng1/chapter/1-recommendations


46208

Tag Question

Feedback

Difficulty: Average

Peer Responses

Session Progress

Responses Correct: 0

Responses Incorrect: 40

Responses Total: 40

Responses - % Correct: 0%

Blog
About Pastest
Contact Us
Help

https://mypastest.pastest.com/Secure/TestMe/Browser/436619[‫ ص‬08:23:09 11/12/1437]


MyPastest

© Pastest 2016

https://mypastest.pastest.com/Secure/TestMe/Browser/436619[‫ ص‬08:23:09 11/12/1437]


MyPastest

Prefer to use the old MyPastest? Access it here »

Back to Filters

Question 39 of 78

A male term infant was admitted to the neonatal unit on day 3


for severe jaundice. The bilirubin at that time was
320 mmol/l, but had
decreased with phototherapy. At day 30, the baby was still requiring phototherapy to keep the
bilirubin below the treatment line. On examination, the skin had a tanned appearance and his sclerae were icteric.
Mother’s blood group was A+, baby’s was A+, direct
Coombs test (DCT) was negative.

What is the MOST likely diagnosis?

A ABO incompatibility

B Crigler-Najjar syndrome type 1

C Crigler-Najjar syndrome type 2

D Haemochromatosis

E Sepsis

Explanation
Crigler-Najjar syndrome type 2 is a rare inherited disorder affecting the metabolism of bilirubin, and would
explain this baby’s ongoing jaundice.

ABO incompatibility is wrong because both mother and baby have the same blood group, and DCT is negative.

Crigler-Najjar
syndrome type 1 is a possibility, but is less common than type 2. It is therefore not the most likely
diagnosis.

Haemochromatosis does not present with neonatal jaundice. The “tanned appearance” of the skin is caused by the
ongoing phototherapy.

Sepsis is an important consideration in any neonate with jaundice, particularly in the first 24 hours of life.
However, it would not explain jaundice at day 30.
46687

https://mypastest.pastest.com/Secure/TestMe/Browser/436619[‫ ص‬08:23:31 11/12/1437]


MyPastest

Previous Question Next Question


Tag Question

Feedback

End Session
Difficulty: Average

Peer Responses

Session Progress

Responses Correct: 0

Responses Incorrect: 41

Responses Total: 41

Responses - % Correct: 0%

Blog
About Pastest
Contact Us
Help

© Pastest 2016

https://mypastest.pastest.com/Secure/TestMe/Browser/436619[‫ ص‬08:23:31 11/12/1437]


MyPastest

Prefer to use the old MyPastest? Access it here »

Back to Filters

Question 40 of 78

A 3-day-old female infant had been on the postnatal ward with


mother, and had just completed 36 hours of IV
antibiotics for presumed sepsis. The baby looked well and was breastfeeding well. At the discharge baby check,
mild jaundice and bilateral talipes were noted. She vomited her feed following palpation of the abdomen.

What is the MOST likely cause for the vomit?

A Abdominal examination

B Biliary atresia

C Duodenal atresia

D Neonatal sepsis

E Spherocytosis

Explanation
This baby has physiological jaundice, which is common is breastfeeding babies.

Biliary atresia should be suspected if jaundice continues beyond 14 days in a term baby, or 21 days in a preterm
baby. This baby is 3 days old.

Duodenal atresia is a not a cause of neonatal jaundice.

Sepsis is an important consideration but this baby has completed IV antibiotics because blood cultures were
negative at 36 hours, and is otherwise well.

Hereditary spherocytosis would cause jaundice and anaemia however this baby was otherwise well and not
anaemic.

46688

https://mypastest.pastest.com/Secure/TestMe/Browser/436619[‫ ص‬08:23:53 11/12/1437]


MyPastest

Previous Question Next Question


Tag Question

Feedback

End Session
Difficulty: Average

Peer Responses

Session Progress

Responses Correct: 0

Responses Incorrect: 42

Responses Total: 42

Responses - % Correct: 0%

Blog
About Pastest
Contact Us
Help

© Pastest 2016

https://mypastest.pastest.com/Secure/TestMe/Browser/436619[‫ ص‬08:23:53 11/12/1437]


MyPastest

Prefer to use the old MyPastest? Access it here »

Back to Filters

Question 41 of 78

You are called to examine a baby who looks jaundiced at 22 hours. The baby was born at 36+4 weeks by
emergency C section for failure to progress. There were no maternal risk factors for sepsis. You cannulate the
baby, send bloods and start the baby on antibiotics. Mother’s blood group is O+, baby’s is B+, direct Coombs test
(DCT) is negative.

What is the MOST likely diagnosis?

A ABO incompatibility

B Hepatitis A

C Neonatal sepsis

D Physiological jaundice

E Pyloric stenosis

Explanation
The DCT results are positive in only 20-40% of infants with ABO incompatibility. ABO incompatibility occurs
mostly in mothers who have blood type O.

Hepatitis A is an acute infectious disease transmitted through contaminated food or water.

Neonatal sepsis is an important consideration in a jaundiced neonate <24 hours, and starting the baby on
antibiotics was an appropriate measure.

Physiological jaundice presents on day 2 onwards.

Pyloric stenosis usually presents at 4-6 weeks of life with projectile vomiting. It is not a reason for jaundice in the
neonate.
46689

https://mypastest.pastest.com/Secure/TestMe/Browser/436619[‫ ص‬08:24:13 11/12/1437]


MyPastest

Previous Question Next Question


Tag Question

Feedback

End Session
Difficulty: Average

Peer Responses

Session Progress

Responses Correct: 0

Responses Incorrect: 43

Responses Total: 43

Responses - % Correct: 0%

Blog
About Pastest
Contact Us
Help

© Pastest 2016

https://mypastest.pastest.com/Secure/TestMe/Browser/436619[‫ ص‬08:24:13 11/12/1437]


MyPastest

Prefer to use the old MyPastest? Access it here »


Previous Question Next Question

Back to Filters

Question 42 of 78

A term infant attends for a prolonged jaundice screen at 16 days. The history from mother is that there have been
pale stools.

What is the SINGLE most important investigation?

A Abdominal ultrasound

B Conjugated bilirubin

C Full blood count

D Urine dipstick

E Urine for MC&S

Explanation
The most likely diagnosis is biliary atresia. An abdominal ultrasound will confirm the diagnosis.

A conjugated bilirubin would lead us towards the diagnosis, but would not confirm the diagnosis.

An FBC alone would not help to diagnose biliary atresia. It therefore not the single most important test.

Urine
dipstick and MC&S (microscopy, culture & sensitivity) would rule out urinary tract infection, but neither
would be the appropriate single investigation of choice.
46690

Tag Question

https://mypastest.pastest.com/Secure/TestMe/Browser/436619[‫ ص‬08:24:33 11/12/1437]


MyPastest

Feedback

Difficulty: Average

Peer Responses

End Session

Session Progress

Responses Correct: 0

Responses Incorrect: 44

Responses Total: 44

Responses - % Correct: 0%

Blog
About Pastest
Contact Us
Help

© Pastest 2016

https://mypastest.pastest.com/Secure/TestMe/Browser/436619[‫ ص‬08:24:33 11/12/1437]


MyPastest

Prefer to use the old MyPastest? Access it here »

Back to Filters

Question 43 of 78

An 8-year-old boy goes to his GP following a diarrhoeal illness. On examination, he has mild jaundice, which he
has had before and his mother says it “doesn’t seem to bother him”.

What is the MOST likely diagnosis?

A Autoimmune hepatitis

B Gilbert syndrome

C Hepato-renal syndrome

D Reye syndrome

E Salmonella

Explanation
Gilbert syndrome is a genetic disorder seen in 3-12% of the population. Reduced activity of the enzyme
glucuronyltransferase during acute illness causes hyperbilirubinemia.

Autoimmune hepatitis would be more likely to present with fatigue and ill-health.

Hepato-renal syndrome is the development of renal failure in patients with chronic liver disease. In the absence of
cirrhosis and ascites, this is unlikely.

Reye syndrome is a rare rapidly progressive encephalopathy. Although associated with liver failure, jaundice does
not generally occur.

Salmonella hepatitis is a possibility, but


this is uncommon, with only 150 cases reported worldwide. Jaundice in
salmonella hepatitis usually occurs within the first 2 weeks of the febrile illness.
46691

https://mypastest.pastest.com/Secure/TestMe/Browser/436619[‫ ص‬08:24:55 11/12/1437]


MyPastest

Previous Question Next Question


Tag Question

Feedback

End Session
Difficulty: Average

Peer Responses

Session Progress

Responses Correct: 0

Responses Incorrect: 45

Responses Total: 45

Responses - % Correct: 0%

Blog
About Pastest
Contact Us
Help

© Pastest 2016

https://mypastest.pastest.com/Secure/TestMe/Browser/436619[‫ ص‬08:24:55 11/12/1437]


MyPastest

Prefer to use the old MyPastest? Access it here »

Back to Filters

Question 44 of 78

A 10-year-old boy presents to the emergency department with abdominal pain. He was previously under follow up
in the General Paediatric clinic, but had failed to attend his last clinic appointment.
Mother tells you that his pain
had been much better for a while, but had started again over the last month. He last opened his bowels this
afternoon and he reports loose, watery stool. He is on not on any regular medications. On examination, there is
generalised tenderness over the abdomen, particularly in the left iliac fossa.

What is the MOST appropriate next course of action?

A Give advice about hygiene measures to reduce faeco-oral transmission of bacteria

B Reassure and re-refer to paediatric clinic

C Refer to surgeons

D Send a stool sample

E Start a faecal disimpaction regime and discharge

Explanation
The correct answer is to start a disimpaction regime with Movicol. This boy has chronic abdominal pain due to
constipation, and the watery stool described is from overflow. His constipation was previously well controlled
with regular Movicol, but he
stopped taking his medication and did not attend his last clinic appointment.

A is wrong because the stem of the question should lead you to a diagnosis of constipation, not acute diarrhoeal
illness.

B is wrong. Although it is appropriate to re-refer him to clinic for ongoing management, this child needs treatment
for his constipation in the meantime.

C is wrong because there are no examination findings to suggest a surgical cause for his pain. Right iliac fossa,
and not left iliac fossa pain should lead you to suspect appendicitis.

https://mypastest.pastest.com/Secure/TestMe/Browser/436619[‫ ص‬08:25:16 11/12/1437]


MyPastest

D is wrong because the stem of the question should lead you to a diagnosis of constipation, not acute diarrhoeal
illness. The watery stool is overflow.
Previous Question Next Question 46692

End
Tag Session
Question

Feedback

Difficulty: Average

Peer Responses

Session Progress

Responses Correct: 0

Responses Incorrect: 46

Responses Total: 46

Responses - % Correct: 0%

Blog
About Pastest
Contact Us
Help

© Pastest 2016

https://mypastest.pastest.com/Secure/TestMe/Browser/436619[‫ ص‬08:25:16 11/12/1437]


MyPastest

Prefer to use the old MyPastest? Access it here »

Back to Filters

Question 45 of 78

A 13-year-old girl presents with abdominal pain. When you ask her to point to where the pain is she points to her
right iliac fossa. She says that she gets this pain regularly. Paracetamol sometimes helps, but not always. She is not
currently on her menstrual period and she denies any sexual activity.

What would be the MOST appropriate treatment for her pain?

A Mefenamic acid

B Oral contraceptive pill

C Oramorph

D Paracetamol

E Tranexamic acid

Explanation
The cause of this patient’s abdominal pain is most likely to be Mittelschmerz - a German word meaning “mid-
cycle pain” or “ovulation pain.” The pain
can occur suddenly and can last from a few hours to up to three days.
The combined oral contraceptive pill can be used to prevent ovulation, and there is evidence for its use in
Mittelschmerz.

A is wrong. Mefenamic acid is a non steroidal anti-inflammatory drug (NSAID) that is an effective treatment for
dysmenorrhoea, however it has no role in the management of Mittelschmerz.

Oramorph would be reserved for severe pain and would not be appropriate in this situation.

Paracetamol is inappropriate as she has already tried this and it only helps “sometimes”.

Tranexamic acid is an anti fibrinolytic that can be helpful in menorrhagia. It has no role in the management of
Mittelschmerz.

https://mypastest.pastest.com/Secure/TestMe/Browser/436619[‫ ص‬08:25:39 11/12/1437]


MyPastest

Ref: Jensen JT, Speroff L. Health benefits of oral contraceptives.ObstetGynecolClin North Am. 2000
Dec;27(4):705-21.
Previous Question Next Question 46693

End
Tag Session
Question

Feedback

Difficulty: Average

Peer Responses

Session Progress

Responses Correct: 0

Responses Incorrect: 47

Responses Total: 47

Responses - % Correct: 0%

Blog
About Pastest
Contact Us
Help

© Pastest 2016

https://mypastest.pastest.com/Secure/TestMe/Browser/436619[‫ ص‬08:25:39 11/12/1437]


MyPastest

Prefer to use the old MyPastest? Access it here »

Back to Filters

Question 46 of 78

An 8-year-old girl presents with weight loss, diarrhoea and abdominal pain. On examination, you note the
presence of apthous ulcers
and generalised abdominal tenderness. She is off her food but drinking
normally, and
her energy levels are low.

What is the MOST likely diagnosis?

A Anorexia nervosa

B Crohn’s disease

C Diabetic ketoacidosis

D Recurrent apthous stomatitis

E Ulcerative colitis

Explanation
The correct answer is Crohn’s disease, a type of inflammatory bowel disease which can affect any part of the
bowel from the mouth the anus. Symptoms include abdominal pain, diarrhoea, pyrexia and weight loss. Extra-
intestinal manifestations include arthritis, uveitis, fatigue, anaemia and rashes including pyoderma gangrenosum
and erythema nodosum.

Whilst anorexia nervosa is an important diagnosis to consider, there are no indicators in the description that she
has a fear of gaining weight or strong desire to be thin.

Diabetic ketoacidosis is incorrect because there is no polydipsia or polyuria. A patient in DKA is more likely to
present with vomiting, and not diarrhoea.

Recurrent apthous stomatitis is not a correct answer because it does not explain all of the symptoms described,
only the mouth ulcers.

Ulcerative colitis (UC) is also incorrect. UC is a form of inflammatory bowel disease that causes inflammation in

https://mypastest.pastest.com/Secure/TestMe/Browser/436619[‫ ص‬08:26:01 11/12/1437]


MyPastest

the colon. The main symptom is bloody stools, which is not mentioned as a feature in the history.
46694

Previous Question Next Question

Tag Question
End Session
Feedback

Difficulty: Average

Peer Responses

Session Progress

Responses Correct: 0

Responses Incorrect: 48

Responses Total: 48

Responses - % Correct: 0%

Blog
About Pastest
Contact Us
Help

© Pastest 2016

https://mypastest.pastest.com/Secure/TestMe/Browser/436619[‫ ص‬08:26:01 11/12/1437]


MyPastest

Prefer to use the old MyPastest? Access it here »

Back to Filters

Question 47 of 78

A type one diabetic presents with a one year history of bloating, diarrhoea and abdominal pain. Her last Hba1c
was 6.3%.

From the list below, what is the MOST appropriate investigation to instigate next?

A Coeliac screen

B Food diary

C Sickle cell percentage

D Thyroid function tests

E Urine disptick

Explanation
Coeliac disease is highly prevalent in children with type one diabetes (11%), and these children should therefore
undergo screening. Symptoms of bloating, diarrhoea and abdominal pain are common. A duodenal biopsy would
be the gold standard investigation,
but the next most appropriate would be a blood test for coeliac screen.

Although a food diary may be useful, investigations should be initiated to reach a diagnosis. B is therefore
incorrect.

A sickle cell percentage would be useful in a patient presenting with abdominal pain due to sickle cell crisis. There
is no indication that this child has sickle cell disease.

Hyper- and hypo-thyroidism are also more common in type 1 diabetes than in the general population, but the
patient history given points towards a diagnosis of coeliac. D is therefore incorrect.

Urinary tract infection should be ruled out in a child with abdominal pain. However, this question is about
chronic, not acute abdominal pain and therefore coeliac screen is the most appropriate answer.

https://mypastest.pastest.com/Secure/TestMe/Browser/436619[‫ ص‬08:26:26 11/12/1437]


MyPastest

46695

Previous Question Next Question

Tag Question

Feedback End Session

Difficulty: Average

Peer Responses

Session Progress

Responses Correct: 0

Responses Incorrect: 49

Responses Total: 49

Responses - % Correct: 0%

Blog
About Pastest
Contact Us
Help

© Pastest 2016

https://mypastest.pastest.com/Secure/TestMe/Browser/436619[‫ ص‬08:26:26 11/12/1437]


MyPastest

Prefer to use the old MyPastest? Access it here »

Back to Filters

Question 48 of 78

A 7-year-old boy attends clinic accompanied by his mother. One year ago he was admitted to hospital with severe
gastroenteritis. Stool sample was positive for cryptosporidium. When he attended clinic 6
months ago, he had
ongoing pain which was thought to be related to the initial infection. This persists and on average, his pain occurs
twice/week and usually towards the end of the day. He says he also sometimes gets headaches. On examination,
his abdomen is soft and non tender. There are no masses or organomegaly and bowel sounds are audible. CRT<2
seconds, heart sounds are normal. There are no signs of respiratory distress and his chest is clear. There are
multiple bruises over his shins bilaterally. FBC, U&Es, CRP and TFTs taken at the last clinic visit were all within
the normal range. Blood glucose was 5.4mmol/l. On further questioning he reveals that the family has recently
moved home and his mother’s partner has recently been arrested following a domestic violence incident. Social
services had been involved and did not consider the child to be at risk.

What is the MOST likely cause of his abdominal pain?

A Diabetic ketoacidosis

B Functional abdominal pain

C Ongoing abdominal cramps following the cryptosporidium infection

D Physical abuse

E Recurrence of the cryptosporidium infection

Explanation
Functional abdominal pain describes the situation
where there is no organic cause for a reported pain. It is a
common condition in children and teenagers and may be accompanied by headaches,
limb pain and difficulty
sleeping. It may be intensified by stress or anxiety. There may also be episodes of diarrhoea or constipation but
these are not the cause of the pain. The pain causes a great deal of anxiety on the part of both parents and children
and the symptoms are real even though not caused by a physical disease. The diagnosis is made
where at least
once per week for the at least 2 months there is episodic or continuous abdominal pain not compatible with any

https://mypastest.pastest.com/Secure/TestMe/Browser/436619[‫ ص‬08:26:46 11/12/1437]


MyPastest

other symptoms of functional gastrointestinal disorder and there is no evidence of an inflammatory, anatomical,
metabolic or neoplastic process
that explains the child's symptoms.
Previous Question Next Question
Diabetic ketoacidosis is a cause of acute, not chronic/recurrent abdominal pain, and you are told the blood glucose
is normal so stem A is incorrect.

The initial infection was one year ago so it is unlikely to be causing ongoing abdominal cramps therefore stem C is
incorrect.

End Session
Although
abuse should not be dismissed as a cause simply because social services
are aware of a situation,
multiple bruises over the shins is not an uncommon finding in a child of this age. Bruises over non bony areas
should raise suspicion of physical abuse.

Recurrence of the infection would not account for the chronicity of the abdominal pain and
there is no diarrhoea at
present so stem E is unlikely.
46696

Tag Question

Feedback

Difficulty: Average

Peer Responses

Session Progress

Responses Correct: 0

Responses Incorrect: 50

Responses Total: 50

Responses - % Correct: 0%

https://mypastest.pastest.com/Secure/TestMe/Browser/436619[‫ ص‬08:26:46 11/12/1437]


MyPastest

Blog
About Pastest
Contact Us
Help

© Pastest 2016

https://mypastest.pastest.com/Secure/TestMe/Browser/436619[‫ ص‬08:26:46 11/12/1437]


MyPastest

Prefer to use the old MyPastest? Access it here »

Back to Filters

Question 49 of 78

You are the Paediatric ST1 covering the assessment unit in a busy district general hospital. A 3-week-old breasfed
baby has been referred by the emergency department. His parents are very concerned because he has not opened
his bowels for three days. He has episodic screaming episodes which occur mostly in the evenings. He is the first
baby of non-consanguineous parents. He was born at full term by emergency C-section due to prolonged second
stage of labour. There was meconium stained liquor. His APGARS were 9, 10 and 10, and he was discharged
home the next day. He is otherwise well and feeding well.

What is the MOST likely diagnosis?

A Hirschsprung’s disease

B Infantile colic

C Intussusception

D Meckel’s diverticulum

E Pyloric stenosis

Explanation
Breastfed babies may only open their bowels once every few days, and this is not a cause for concern. It is
important to note whether the baby passed meconium within the first 48 hours of life.
In this case, there is a
history of meconium stained liquor. Infantile colic is common, although more so in formula fed babies. Parents
should be given advice about appropriately winding the baby after feeds.

A is incorrect. Hirschsprung’s disease is a disorder characterised by an a ganglionic segment of the distal large
bowel, and is therefore a cause of constipation in babies. In this baby, there are no symptoms of signs suggestive
of a surgical cause. There is no bilious
vomiting and the baby is feeding well.

C is incorrect. There are no symptoms or signs suggestive of a surgical cause. Intussusception


may present with a
baby who draws up its legs, and has “redcurrant jelly” stools.

https://mypastest.pastest.com/Secure/TestMe/Browser/436619[‫ ص‬08:27:09 11/12/1437]


MyPastest

D is incorrect. Meckel’s diverticulum is a remnant of the vitello intestinal duct,


which has the potential to cause
lower GI obstruction. There are no symptoms or signs suggestive of a surgical cause in this baby.
Previous Question Next Question
E
is incorrect. There are no symptoms or signs suggestive of a surgical cause. The typical age for pyloric stenosis
to occur is between 4 and 6 weeks. Presentation is with a history of projectile vomiting.
46697

End Session

Tag Question

Feedback

Difficulty: Average

Peer Responses

Session Progress

Responses Correct: 0

Responses Incorrect: 51

Responses Total: 51

Responses - % Correct: 0%

Blog
About Pastest
Contact Us
Help

© Pastest 2016

https://mypastest.pastest.com/Secure/TestMe/Browser/436619[‫ ص‬08:27:09 11/12/1437]


MyPastest

Prefer to use the old MyPastest? Access it here »

Back to Filters

Question 50 of 78

A 15-year-old boy is under regular follow up in the General Paediatric clinic. Today, he attends with his
grandmother whom he has lived with for the past 6 months. Grandmother also looks after his two younger
siblings, aged 9 and 6 years. Prior to this he was cared for by
his father, however his father can no longer cope
with his behaviour. He opens his bowels once every three to four days after straining, and also has problems with
smearing. He is unable to sit on the toilet and has never done so, instead he crouches and admits that this is very
uncomfortable. He has recently broken up with his girlfriend and is feeling rejected. When you look further
through his notes you see that he suffered severe physical and emotional abuse throughout his childhood, and his
mother is currently in prison. You notice scars from
self-harm on his wrists.

Which of these is the LEAST appropriate action?

A Liaise with the school nurse

B Prescribe Movicol

C Refer to CAMHS

D Refer to clinical psychology

E Refer to surgeons

Explanation
Referral to surgeons is not appropriate in this situation. The cause for this boy’s constipation is psychological.
There are no “red flags” in the history to suggest an underlying disorder or condition (see NICE guidance for red
flags).

It would be appropriate to liaise with the school nurse for the ongoing management of this boy’s constipation.

Prescribing Movicol would be beneficial and the outreach team can also be involved to reveal any underlying
issues with compliance.

https://mypastest.pastest.com/Secure/TestMe/Browser/436619[‫ ص‬08:27:30 11/12/1437]


MyPastest

A CAMHS referral is appropriate due to the obvious self-harm. The urgency


of this referral will depend on his
current ideas about self-harm and suicide, and whether he is at immediate risk of harming himself or others.
Previous Question Next Question
Clinical psychology will play an important role in improving this boy’s psychological well-being and likely his
physical symptoms.

https://www.nice.org.uk/guidance/cg99 46698

End Session

Tag Question

Feedback

Difficulty: Average

Peer Responses

Session Progress

Responses Correct: 0

Responses Incorrect: 52

Responses Total: 52

Responses - % Correct: 0%

Blog
About Pastest
Contact Us
Help

© Pastest 2016

https://mypastest.pastest.com/Secure/TestMe/Browser/436619[‫ ص‬08:27:30 11/12/1437]


MyPastest

Prefer to use the old MyPastest? Access it here »

Back to Filters

Question 51 of 78

A 6-year-old girl was seen on the children’s assessment


unit yesterday. She was started on a Movicol disimpaction
regime. She
has re-presented today with increasing abdominal pain. You assess her and are confident with a
clinical diagnosis of faecal impaction.

What is the most appropriate next course of action?

A Continue disimpaction regime

B Perform an abdominal X-ray to confirm the diagnosis

C Perform an abdominal ultrasound to confirm the diagnosis

D Phosphate enema

E Stop Movicol and change to lactulose

Explanation
The correct answer is to continue the disimpaction regime. Families and children should be informed that
disimpaction treatment can initially increase symptoms of soiling and abdominal pain.

A plain abdominal radiograph should not be used to make a diagnosis of idiopathic constipation. According to
NICE guidelines, a plain X-ray should only be used in the ongoing management of intractable idiopathic
constipation.

Abdominal US should not be


used to make a diagnosis of idiopathic constipation. According to NICE guidelines,
an abdominal US should only be used in the ongoing management of intractable idiopathic constipation.

Phosphate enemas should only be used if all oral medications and sodium citrate enemas have failed.

NICE
guidelines state that Movicol should be substituted with lactulose only
if Movicol is not tolerated. This is
not the case here.

https://mypastest.pastest.com/Secure/TestMe/Browser/436619[‫ ص‬08:27:53 11/12/1437]


MyPastest

https://www.nice.org.uk/guidance/cg99 46699

Previous Question Next Question

Tag Question

Feedback
End Session

Difficulty: Average

Peer Responses

Session Progress

Responses Correct: 0

Responses Incorrect: 53

Responses Total: 53

Responses - % Correct: 0%

Blog
About Pastest
Contact Us
Help

© Pastest 2016

https://mypastest.pastest.com/Secure/TestMe/Browser/436619[‫ ص‬08:27:53 11/12/1437]


MyPastest

Prefer to use the old MyPastest? Access it here »

Back to Filters

Question 52 of 78

A male infant is brought to the emergency department by his parents. He was born at 34 weeks by spontaneous
vaginal delivery and was discharged from SCBU 4 weeks ago. He is not on any regular medication. Parents tell
you that he brings up small volumes of milk after feeds. This happens approximately twice a day. Observations are
all within normal range and examination is unremarkable.

What is the MOST likely diagnosis?

A Cows’ milk protein intolerance

B Gastro-oesophageal reflux

C Meningitis

D Overfeeding

E Pyloric stenosis

Explanation
The correct answer is B. Immaturity of the lower
oesophageal sphincter results in frequent transient relaxation,
and retrograde flow of gastric contents into the oesophagus.

Although
a diagnosis of cows' milk protein intolerance should be considered, in the absence of lower GI
symptoms or history of atopy/allergy, gastro-oesophageal reflux is the more likely diagnosis.

Meningitis
can present with poor feeding and/or vomiting in an infant. However, this baby is feeding well and is
otherwise well. The possets are related to feeds and only occur twice a day, making B the more likely diagnosis.

A careful history should be taken to pick up overfeeding. However, there is no indication within the information
provided that the baby is being overfed so D is unlikely.

Pyloric stenosis typically (but not exclusively) occurs in first born male infants at 4-6 weeks of age. However,
vomiting would occur after most/all feeds and would be projectile in nature.

https://mypastest.pastest.com/Secure/TestMe/Browser/436619[‫ ص‬08:28:15 11/12/1437]


MyPastest

46700

Previous Question Next Question

Tag Question

Feedback End Session

Difficulty: Average

Peer Responses

Session Progress

Responses Correct: 0

Responses Incorrect: 54

Responses Total: 54

Responses - % Correct: 0%

Blog
About Pastest
Contact Us
Help

© Pastest 2016

https://mypastest.pastest.com/Secure/TestMe/Browser/436619[‫ ص‬08:28:15 11/12/1437]


MyPastest

Prefer to use the old MyPastest? Access it here »

Back to Filters

Question 53 of 78

A 7-year-old boy gets recurrent attacks of intense nausea, retching and vomiting. He is lethargic, his eyes are
sunken and he has not passed urine all day.

Which of the following is the MOST appropriate action?

A Cyclizine and oral fluids

B IV fluid bolus then IV maintenance fluids

C IV maintenance fluids

D Juice and carbonated drinks

E Oral fluid challenge

Explanation
The correct answer is B. This child is severely dehydrated and needs an IV fluid bolus, as well as maintenance
therapy. The recurrence of the vomiting points towards a diagnosis of cyclical vomiting.

Answer A is incorrect. It is unlikely that this boy will tolerate sufficient oral fluids and rehydration will require the
use of IV fluids.

C is incorrect. The signs are suggestive of severe dehydration and a fluid bolus in indicated.

D is incorrect. Juice and carbonated drinks are contraindicated, and are likely to result in more vomiting.

E is incorrect. It is unlikely that this boy will tolerate sufficient


oral fluids and rehydration will require the use of
IV fluids.

https://www.nice.org.uk/guidance/cg84/chapter/key-priorities-for-implementation 46701

https://mypastest.pastest.com/Secure/TestMe/Browser/436619[‫ ص‬08:28:36 11/12/1437]


MyPastest

Previous Question Next Question


Tag Question

Feedback

End Session
Difficulty: Average

Peer Responses

Session Progress

Responses Correct: 0

Responses Incorrect: 55

Responses Total: 55

Responses - % Correct: 0%

Blog
About Pastest
Contact Us
Help

© Pastest 2016

https://mypastest.pastest.com/Secure/TestMe/Browser/436619[‫ ص‬08:28:36 11/12/1437]


MyPastest

Prefer to use the old MyPastest? Access it here »

Back to Filters

Question 54 of 78

A 12-year-old boy presents with a spreading non-blanching petechial rash, photophobia, neck stiffness and
vomiting. His mother and sister have returned from a holiday to Disneyland in Florida, and have both been unwell
with diarrhoea and vomiting.

Which investigation will confirm the MOST likely diagnosis?

A Blood culture

B CRP

C Lumbar puncture

D Sputum sample for AFB

E Stool culture

Explanation
The correct investigation is lumbar puncture. The most likely diagnosis given the child’s symptoms is
meningococcal meningitis. Treatment with IV ceftriaxone should be commenced immediately.

A is incorrect. Blood culture should be performed and may or may not be positive. However, confirmation in the
cerebrospinal fluid (CSF) will allow you to determine the accurate diagnosis and the length of treatment with IV
antibiotics.

B is incorrect. C-Reactive Protein should be performed as part of the septic screen, but this will not confirm the
diagnosis.

D
is incorrect. The presence of acid fast bacilli would diagnose tuberculosis, which is not the most likely diagnosis
in this case.

E
is incorrect because gastroenteritis is not the most likely diagnosis, despite the other family members having
symptoms of gastroenteritis.

https://mypastest.pastest.com/Secure/TestMe/Browser/436619[‫ ص‬08:28:58 11/12/1437]


MyPastest

http://www.meningitis.org/assets/x/50631 46702

Previous Question Next Question

Tag Question
End Session
Feedback

Difficulty: Average

Peer Responses

Session Progress

Responses Correct: 0

Responses Incorrect: 56

Responses Total: 56

Responses - % Correct: 0%

Blog
About Pastest
Contact Us
Help

© Pastest 2016

https://mypastest.pastest.com/Secure/TestMe/Browser/436619[‫ ص‬08:28:58 11/12/1437]


MyPastest

Prefer to use the old MyPastest? Access it here »

Back to Filters

Question 55 of 78

A 4-week-old infant presents with vomiting. He was born at 39 weeks by spontaneous vaginal delivery (SVD)
weighing 3.6kg, and he was treated with 48 hours of IV antibiotics due to maternal pyrexia in labour and
prolonged rupture of membranes. You are not sure what his CRP was at the time. He had become jaundiced on
day 2, but when a bilirubin level was taken it was below the treatment line. He has been started on Gaviscon infant
by the GP which mother has been adding to all
his formula feeds. Despite this, the vomiting has continued and she
thinks he needs further investigations for reflux. On examination, his CRT is < 2 seconds, his weight is 4.5kg, his
fontanelle feels soft and he passes urine as you open the nappy.

What is the next MOST appropriate course of action?

A Advise mother to reduce formula feeds

B Arrange a pH study

C Arrange an impedance study

D Change to a hydrolysed formula

E Urgent blood culture

Explanation
This infant’s weight has gone from the 25th
centile to the 75th centile within four weeks. The most likely cause is
overfeeding. A careful history should be taken to determine how much
milk he is being given and the mother
advised to reduce formula feeds.

B
and C are unhelpful in arriving at a diagnosis. pH oesophageal impedance monitoring would confirm a
diagnosis of gastro-oesophageal reflux, which is not the most likely diagnosis here.

D is incorrect. A hydrolysed formula would be appropriate if cows’ milk protein intolerance was suspected. This
infant is feeding very well and is tolerating feeds well. If vomiting continues after overfeeding is corrected, then
other diagnoses should be considered.

https://mypastest.pastest.com/Secure/TestMe/Browser/436619[‫ ص‬08:29:20 11/12/1437]


MyPastest

E is incorrect. Sepsis is unlikely in a very well baby who is feeding well. However, it is always important to think
about sepsis as a possible cause for vomiting.
Previous Question Next Question 46703

End Session
Tag Question

Feedback

Difficulty: Average

Peer Responses

Session Progress

Responses Correct: 0

Responses Incorrect: 57

Responses Total: 57

Responses - % Correct: 0%

Blog
About Pastest
Contact Us
Help

© Pastest 2016

https://mypastest.pastest.com/Secure/TestMe/Browser/436619[‫ ص‬08:29:20 11/12/1437]


MyPastest

Prefer to use the old MyPastest? Access it here »


Previous Question Next Question

Back to Filters

End Session
Question 56 of 78

A term baby is seen on the postnatal ward with vomiting. He is two hours of age. The obstetrician informs you
that they have just commenced mother on IV antibiotics for likely sepsis.

What is the most appropriate course of action for the baby?

A Cannulate baby, take blood and culture and start IV antibiotics

B Cannulate baby, take blood and culture and wait for the CRP

C Cannulate baby, take bloods and wait for the CRP result

D Observe the baby closely on the neonatal unit

E Observe the baby on transitional care

Explanation
A is the correct answer. Maternal sepsis is an indication to commence IV antibiotics in the baby, because the risk
of early onset neonatal sepsis from trans-placental transmission is high.

All other answers are incorrect. Antibiotics should be started in the baby because of the risk of early onset
neonatal sepsis.
46704

Tag Question

Feedback

https://mypastest.pastest.com/Secure/TestMe/Browser/436619[‫ ص‬08:29:41 11/12/1437]


MyPastest

Difficulty: Average

Peer Responses

Session Progress

Responses Correct: 0

Responses Incorrect: 58

Responses Total: 58

Responses - % Correct: 0%

Blog
About Pastest
Contact Us
Help

© Pastest 2016

https://mypastest.pastest.com/Secure/TestMe/Browser/436619[‫ ص‬08:29:41 11/12/1437]


MyPastest

Prefer to use the old MyPastest? Access it here »

Back to Filters

Question 57 of 78

A 14-year-old girl is seen on the children’s assessment


unit with abdominal pain. The pain started yesterday
evening and paracetamol does not help. On examination, she is tender in the right iliac fossa, and there is a
palpable adnexal mass.

What is the MOST likely diagnosis?

A Acute appendicitis

B Constipation

C Ectopic pregnancy

D Mittelschmerz

E UTI

Explanation
The most likely diagnosis is C. The classical clinical triad of ectopic pregnancy is pain, amenorrhoea and vaginal
bleeding. 75% have abdominal tenderness, and 50% have a palpable adnexal
mass. Further history should be taken
and any concerns about safeguarding discussed with a senior.

Although RIF pain may suggest a diagnosis of appendicitis, the palpable “adnexal” mass is more suggestive of
uterine pathology so stem A is incorrect.

Constipation
can present with abdominal pain but this is more often generalised. In cases of constipation, a faecal
mass is more likely to be palpable in the left iliac fossa, or throughout the abdomen.

Mittelschmerz (middle pain or pain in the middle of a menstrual cycle) is wrong as this would not explain the
palpable mass.

Although
a urine dipstick is a reasonable investigation to perform in a child presenting with abdominal pain, a UTI
would not explain the palpable mass.

https://mypastest.pastest.com/Secure/TestMe/Browser/436619[‫ ص‬08:30:01 11/12/1437]


MyPastest

46999

Previous Question Next Question

Tag Question

Feedback End Session

Difficulty: Average

Peer Responses

Session Progress

Responses Correct: 0

Responses Incorrect: 59

Responses Total: 59

Responses - % Correct: 0%

Blog
About Pastest
Contact Us
Help

© Pastest 2016

https://mypastest.pastest.com/Secure/TestMe/Browser/436619[‫ ص‬08:30:01 11/12/1437]


MyPastest

Prefer to use the old MyPastest? Access it here »


Next Question

Back to Filters

Question 58 of 78

A 4-year-old girl presents with vomiting and abdominal pain. She has been prescribed Movicol for constipation
but does not take it regularly.

Which is the NEXT most appropriate investigation?

A Abdominal ultrasound

B Abdominal X-ray

C Amylase

D Bloods including CRP

E Urine dipstick

Explanation
Constipation is a risk factor for urinary tract infection, which is important to test for in this 4-year-old gir so E is
correct.

A is incorrect. Abdominal ultrasound may be required if symptoms continue and are unexplained but it is
important to rule out
a UTI first.

B is incorrect. Abdominal X-ray may be required if symptoms are unexplained but it is important to rule out a UTI
first.

C and D are also incorrect. If a urine dipstick shows evidence of a UTI and she is otherwise systemically well,
venepuncture may not be indicated.
47000

https://mypastest.pastest.com/Secure/TestMe/Browser/436619[‫ ص‬08:30:22 11/12/1437]


MyPastest

Tag Question
Previous Question
Feedback

Difficulty: Average

Peer Responses End Session

Session Progress

Responses Correct: 0

Responses Incorrect: 60

Responses Total: 60

Responses - % Correct: 0%

Blog
About Pastest
Contact Us
Help

© Pastest 2016

https://mypastest.pastest.com/Secure/TestMe/Browser/436619[‫ ص‬08:30:22 11/12/1437]


MyPastest

Prefer to use the old MyPastest? Access it here »

Back to Filters

Question 59 of 78

A 10-year-old girl is seen in the emergency department with abdominal pain. She had a sore throat a week ago but
no significant past
medical history of note. On examination, she has mild tenderness in the right iliac fossa, but no
guarding or rebound tenderness. Her urine
dipstick is normal. Bloods results are
Hb120, WCC 7.9, Plts312, Na 139, K 3.6, Creat 52, Urea 2.5, Protein 64, AST 21, Bili 8, Alb 40, Adj Ca 2.31,
CRP 2.

What is the MOST likely diagnosis?

A Appendicitis

B Mesenteric adenitis

C Psychosomatic pain

D Pyelonephritis

E Trauma

Explanation
The most likely diagnosis is mesenteric adenitis,
a self-limiting inflammatory condition that affects the mesenteric
lymph nodes. The differential diagnosis includes appendicitis. Depending on the clinical condition, referral to a
surgeon to consider a
diagnosis of appendicitis may still be an appropriate course of action.
However, in this case
the child is well and inflammatory markers are not raised.

Although appendicitis is an important diagnosis to consider, B is more likely in this case.

Psychosomatic pain would be a diagnosis of exclusion and is not correct.

Pyelonephritis is highly unlikely in the absence of a positive urine dipstick.

There is no history of trauma so E is incorrect.


47001

https://mypastest.pastest.com/Secure/TestMe/Browser/436619[‫ ص‬08:30:43 11/12/1437]


MyPastest

Previous Question Next Question


Tag Question

Feedback

End Session
Difficulty: Average

Peer Responses

Session Progress

Responses Correct: 0

Responses Incorrect: 61

Responses Total: 61

Responses - % Correct: 0%

Blog
About Pastest
Contact Us
Help

© Pastest 2016

https://mypastest.pastest.com/Secure/TestMe/Browser/436619[‫ ص‬08:30:43 11/12/1437]


MyPastest

Prefer to use the old MyPastest? Access it here »


Next Question

Back to Filters

Question 60 of 78

An 8-year-old girl presents with sudden onset abdominal pain.


One month ago she was admitted with a sudden
severe pain in her left arm. She is under follow up because of a haematological condition.

What is the MOST likely underlying condition?

A Beta thalassaemia

B Beta thalasseamia trait

C Iron deficiency anaemia

D Sickle cell anaemia

E Spherocytosis

Explanation
Sickle cell anaemia is the most likely underlying
condition. This is a chronic haemolytic anaemia where vaso-
occlusive crises can be precipitated by hypoxaemia, acute febrile illness and dehydration. This patient has
presented in abdominal crisis.

A and B are wrong. Thalassaemias are a group of hereditary conditions that


cause anaemia, but are not associated
with painful crises.

C is incorrect. Iron deficiency anaemia would not explain the painful events.

E is incorrect. Hereditary haemochromatosis is a haemolytic anaemic characterised by “sphere shaped” rather than
biconcave erythrocytes. Children with spherocytosis are at risk of gallstones, however the previous left arm pain in
this child makes sickle cell anaemia the more likely diagnosis.
47002

https://mypastest.pastest.com/Secure/TestMe/Browser/436619[‫ ص‬08:31:03 11/12/1437]


MyPastest

Previous Question Tag Question

Feedback

Difficulty: Average
End Session
Peer Responses

Session Progress

Responses Correct: 0

Responses Incorrect: 62

Responses Total: 62

Responses - % Correct: 0%

Blog
About Pastest
Contact Us
Help

© Pastest 2016

https://mypastest.pastest.com/Secure/TestMe/Browser/436619[‫ ص‬08:31:03 11/12/1437]


MyPastest

Prefer to use the old MyPastest? Access it here »

Back to Filters

Question 61 of 78

A 6-year-old boy presents with joint pain, rash and severe abdominal pain. From mother’s description, the rash has
appeared in crops on his legs over the past week.

What is the MOST likely diagnosis?

A Gastric bleed

B Gastric ulcer

C Henoch Schonlein purpura

D Systemic lupus erythematosus

E Varicella zoster (chicken pox)

Explanation
The most likely diagnosis is C, Henoch Schonlein Purpura. This is a vasculitis that most commonly occurs in
children under the age of ten. It causes a purpuric rash, usually over the legs and buttocks, joint pain and
abdominal pain. Patients should be monitored for renal complications by checking BP, urine dipsticks (to check
for haematuria and proteinuria) and examination for oedema.

Answers
A and B are incorrect. Abdominal pain in HSP is caused by oedema and inflammation in the bowel wall
and mesentery, not from gastric pathology.

D is incorrect. The rash is not characteristic of the rashes that occur in SLE which are characteristically described
as a
malar rash - an erythema over the cheeks and nasal bridge. Although joint pain occurs, abdominal pain is not
common.

E is incorrect. In chickenpox, vesicles occur in crops and usually begin on the torso.
47003

https://mypastest.pastest.com/Secure/TestMe/Browser/436619[‫ ص‬08:31:25 11/12/1437]


MyPastest

Previous Question Next Question


Tag Question

Feedback

End Session
Difficulty: Average

Peer Responses

Session Progress

Responses Correct: 0

Responses Incorrect: 63

Responses Total: 63

Responses - % Correct: 0%

Blog
About Pastest
Contact Us
Help

© Pastest 2016

https://mypastest.pastest.com/Secure/TestMe/Browser/436619[‫ ص‬08:31:25 11/12/1437]


MyPastest

Prefer to use the old MyPastest? Access it here »

Back to Filters

Question 62 of 78

A female infant is brought to the emergency department by her


parents. She was discharged from the postnatal
ward three days ago, following a normal vaginal delivery at full term. Pregnancy and delivery had been
uneventful. There were no maternal risk factors for sepsis, membranes ruptured 14 hours prior to delivery and
liquor was clear. Today the baby vomited dark brown blood which was mixed in with milk. She is otherwise well
and feeding well.

What is the MOST likely diagnosis?

A Ingested blood during feeding

B Ingestion of blood stained liquor

C Mallory Weiss tear

D Oesophageal varices

E Pyloric stenosis

Explanation
The most likely answer here is ingested blood during feeding. The two most likely causes in cases such as
described are swallowed blood from cracked nipples during breastfeeding, or swallowed blood-stained liquor at
delivery. However, the history notes clear liquor in this instance so the former is more likely.

Ingestion of blood stained liquor is unlikely because the liquor was reported as clear at delivery so B is incorrect.

A Mallory Weiss tear is a laceration in the mucosa at the junction between the stomach and oesophagus, and
usually occurs after severe vomiting. There is no history of recurrent episodes of severe vomiting in this infant so
C is incorrect.

Oesophageal varices most commonly occur as a result of portal hypertension. Biliary atresia and cirrhosis related
cystic fibrosis are possible causes of this in childhood. There is no suggestion of such pathology in the stem of the

https://mypastest.pastest.com/Secure/TestMe/Browser/436619[‫ ص‬08:31:48 11/12/1437]


MyPastest

question so D is incorrect.

Pyloric stenosis, typically, (but not exclusively) occurs in first born male infants at 4-6 weeks of age. However,
Previous Question Next Question
vomiting would occur after most/all feeds, would be projectile
in nature, and would not contain blood so E is
incorrect..
47004

End Session

Tag Question

Feedback

Difficulty: Average

Peer Responses

Session Progress

Responses Correct: 0

Responses Incorrect: 64

Responses Total: 64

Responses - % Correct: 0%

Blog
About Pastest
Contact Us
Help

© Pastest 2016

https://mypastest.pastest.com/Secure/TestMe/Browser/436619[‫ ص‬08:31:48 11/12/1437]


MyPastest

Prefer to use the old MyPastest? Access it here »

Back to Filters

Question 63 of 78

An 8-year-old boy presents with haematemesis. He had vomited


a profuse amount of bright red blood and was
therefore brought in by ambulance. On examination of his throat, you see a blood vessel on his left tonsil that is
bleeding, and he vomits more blood after ENT examination.

Which of these is the LEAST appropriate course of action action?

A Admit and observe

B Bloods for FBC, clotting, group and save

C ENT referral

D Throat swab

E Tranexamic acid

Explanation
This child most probably has haemorrhagic tonsillitis, and a throat swab is likely to result in more bleeding. It is
therefore not an appropriate course of action so D is the correct answer.

The child should be admitted for observation and appropriate monitoring so A is incorrect.

B is incorrect. It is important to know the child’s haemoglobin, coagulation status, and to take a group and save
sample in case a blood transfusion is required. The child may be continually swallowing blood, so haemoglobin
and fluid status should be monitored.

C is incorrect. ENT advice should be sought.

E is also incorrect. Tranexamic may or may not be indicated, depending on the severity of the bleeding. However,
it is not the least appropriate action.
47005

https://mypastest.pastest.com/Secure/TestMe/Browser/436619[‫ ص‬08:32:10 11/12/1437]


MyPastest

Previous Question Next Question


Tag Question

Feedback

End Session
Difficulty: Average

Peer Responses

Session Progress

Responses Correct: 0

Responses Incorrect: 65

Responses Total: 65

Responses - % Correct: 0%

Blog
About Pastest
Contact Us
Help

© Pastest 2016

https://mypastest.pastest.com/Secure/TestMe/Browser/436619[‫ ص‬08:32:10 11/12/1437]


MyPastest

Prefer to use the old MyPastest? Access it here »

Back to Filters

Question 64 of 78

A 15-year-old girl is seen on the children’s assessment


unit (CAU). She has vomited eight times today and has had
loose watery
stools. In her last vomit there were streaks of fresh red blood. Her observations are all within range,
and examination is unremarkable.

What is the most appropriate course of action?

A Admit and observe

B Discharge

C Discharge with follow up in outpatients clinic

D Observe on CAU, then discharge if well

E Refer to surgeons

Explanation
Observe on CAU then discharge if well is the most
appropriate course of action in this scenario. The most likely
diagnosis is a Mallory Weiss tear, however, considering the multiple episodes of vomiting and diarrhoea, she
would benefit from a period of observation to ensure that she is tolerating fluids.

Stem A is incorrect. She is not clinically dehydrated and does not need admission
based on the information given
in the question.

Stem B is incorrect. Although it is likely that she will be discharged, she should be tolerating adequate fluids first.

Stem C is incorrect. A Mallory Weiss tear is an inappropriate reason to refer to a Paediatric outpatient clinic.

Stem E is also incorrect. A Mallory Weiss tear is an inappropriate reason to refer to surgeons.
47006

https://mypastest.pastest.com/Secure/TestMe/Browser/436619[‫ ص‬08:32:32 11/12/1437]


MyPastest

Previous Question Next Question


Tag Question

Feedback

End Session
Difficulty: Average

Peer Responses

Session Progress

Responses Correct: 0

Responses Incorrect: 66

Responses Total: 66

Responses - % Correct: 0%

Blog
About Pastest
Contact Us
Help

© Pastest 2016

https://mypastest.pastest.com/Secure/TestMe/Browser/436619[‫ ص‬08:32:32 11/12/1437]


MyPastest

Prefer to use the old MyPastest? Access it here »

Back to Filters

Question 65 of 78

A 3-month-old male infant presents with episodes of drawing up his legs, crying and redcurrant-jelly like stools.

What is the MOST appropriate investigation to confirm the most likely diagosis?

A Abdominal CT

B Abdominal MRI

C Abdominal ultrasound

D Abdominal X-ray

E CRP

Explanation
Abdominal ultrasound is the correct answer. The most likely diagnosis is Intussusception. Ultrasound is the
investigation of choice due to its high accuracy and lack of radiation. A ‘doughnut shaped’ mass can be identified
on ultrasound.

A CT would confirm the diagnosis but would expose the patient to unnecessary radiation, which can be avoided
by ultrasound.

Whilst abdominal MRI would confirm the diagnosis, ultrasound is likely to be more readily available and is
therefore the most appropriate investigation to request.

Abdominal X-ray may be needed if an ultrasound cannot be done immediately and there are concerns about
intestinal obstruction of free intraperitoneal gas.

C-reactive protein (CRP) is a substance produced in the liver in response to inflammation. It is useful for example,
in assessing patient with, inflammatory bowel disease, and is unlikely to lead to confirmation of the diagnosis in
this case.
47007

https://mypastest.pastest.com/Secure/TestMe/Browser/436619[‫ ص‬08:32:53 11/12/1437]


MyPastest

Previous Question Next Question


Tag Question

Feedback

End Session
Difficulty: Average

Peer Responses

Session Progress

Responses Correct: 0

Responses Incorrect: 67

Responses Total: 67

Responses - % Correct: 0%

Blog
About Pastest
Contact Us
Help

© Pastest 2016

https://mypastest.pastest.com/Secure/TestMe/Browser/436619[‫ ص‬08:32:53 11/12/1437]


MyPastest

Prefer to use the old MyPastest? Access it here »


Next Question

Back to Filters

Question 66 of 78

The parents of a 4-month-old girl bring her to clinic. She was most recently seen in clinic due to blood in her stool,
which was thought to be secondary to cows' milk protein intolerance. Until three weeks ago, mother had been
exclusively breastfeeding. When formula milk
was introduced at that time, she had severe swelling around her
lips, face and eyes. After discussing with your consultant, you decide to do further investigations.

What is the SINGLE investigation of choice?

A Basophil histamine release

B Lymphocyte stimulation

C Mediator release assay

D Serum specific IgG tests

E Specific IgE antibody test

Explanation
The correct answer is specific IgE antibody test. This child has symptoms of IgE mediated
cows' milk protein
(CMP) allergy. It is therefore appropriate to investigate with a specific IgE antibody test, or with a skin prick test.
However, it is important to note that both of these tests have a wide range of sensitivities and specificities.

Stems A, B, C and D are incorrect. Guidelines issued by NICE and by ESPGHAN suggest that these tests are not
recommended for diagnosing food allergies in infants and children.

http://cks.nice.org.uk/cows-milk-protein-allergy-in-children#!diagnosissub
47008

https://mypastest.pastest.com/Secure/TestMe/Browser/436619[‫ ص‬08:33:14 11/12/1437]


MyPastest

Tag Question

Previous Question
Feedback

Difficulty: Average

Peer Responses
End Session

Session Progress

Responses Correct: 0

Responses Incorrect: 68

Responses Total: 68

Responses - % Correct: 0%

Blog
About Pastest
Contact Us
Help

© Pastest 2016

https://mypastest.pastest.com/Secure/TestMe/Browser/436619[‫ ص‬08:33:14 11/12/1437]


MyPastest

Prefer to use the old MyPastest? Access it here »


Next Question

Back to Filters

Question 67 of 78

A 9-year-old boy presents with fresh red blood mixed in with his stools. He says he has had this before, but is not
sure how long for. On examination, he looks pale but is clinically well hydrated. There are patches of
hyperpigmentation in the mouth, and on his hands and feet.

What is the MOST likely diagnosis?

A Addison’s disease

B McCune Albright syndrome

C Meckel’s diverticulum

D Peutz-Jeghers syndrome

E Salmonella

Explanation
This patient is most likely to have Peutz-Jeghers
syndrome, an autosomal dominant condition characterised by
mucocutaneous lesions, and hamartomatous polyps in the GI tract. These polyps may bleed.

The differential diagnoses for the hyperpigmented lesions are Addison’s disease and McCune Albright syndrome.
However, neither of these are a cause of PR bleeding. Stems A
and B are therefore incorrect.

Meckel’s diverticulum is a
cause of PR bleeding, but this diagnosis would not explain the lesions in the mouth,
hands and feet. Stem C is therefore incorrect.

Salmonella food poisoning is another cause of PR bleeding, but would not explain the oral and skin lesions.
47009

https://mypastest.pastest.com/Secure/TestMe/Browser/436619[‫ ص‬08:33:35 11/12/1437]


MyPastest

Tag Question
Previous Question
Feedback

Difficulty: Average

Peer Responses End Session

Session Progress

Responses Correct: 0

Responses Incorrect: 69

Responses Total: 69

Responses - % Correct: 0%

Blog
About Pastest
Contact Us
Help

© Pastest 2016

https://mypastest.pastest.com/Secure/TestMe/Browser/436619[‫ ص‬08:33:35 11/12/1437]


MyPastest

Prefer to use the old MyPastest? Access it here »

Back to Filters

Question 68 of 78

A 4-year-old girl is seen in the emergency department with profuse diarrhoea which contains blood. The family
ate some takeaway chicken two days ago and multiple family members have been unwell.

What is the MOST likely cause of this child’s bloody diarrhoea?

A Clostridium difficile

B E. coli

C Haemorrhoids

D Salmonella

E Shigella

Explanation
The correct answer is D, salmonella food poisoning. Salmonellosis is contracted by eating food contaminated with
salmonella; most often poultry and eggs. It can cause diarrhoea containing blood, fever and abdominal cramps.

C. difficile
most commonly occurs after hospitalisation, or after antibiotic treatment. Neither of these risk factors
are present in this child.

E. coli
can cause bloody diarrhoea. However, Salmonella more commonly occurs after contaminated chicken
ingestion so is more likely in this case.

Although haemorrhoids may cause fresh red blood to be passed, there is often also itchiness at the anus, and there
is no association with ingestion of takeaway chicken. Haemorrhoids are uncommon in children and where it does
occur there is usually a history of inability to produce a bowel movement/constipation resulting in large hard
stools causing the child to strain to pass the stools.

Shigellosis is possible and is transmitted via the faeco-oral route, but not typically with poultry. Salmonella is
therefore the most likely diagnosis.

https://mypastest.pastest.com/Secure/TestMe/Browser/436619[‫ ص‬08:33:55 11/12/1437]


MyPastest

47010

Previous Question Next Question

Tag Question

Feedback End Session

Difficulty: Average

Peer Responses

Session Progress

Responses Correct: 0

Responses Incorrect: 70

Responses Total: 70

Responses - % Correct: 0%

Blog
About Pastest
Contact Us
Help

© Pastest 2016

https://mypastest.pastest.com/Secure/TestMe/Browser/436619[‫ ص‬08:33:55 11/12/1437]


MyPastest

Prefer to use the old MyPastest? Access it here »

Back to Filters

Question 69 of 78

A 4-year-old boy is referred for recurrent episodes of diarrhoea and failure to thrive. An OGD is performed and
the results of
the duodenal biopsy are awaited.

What is the MOST likely diagnosis?

A Coeliac disease

B Crohn’s disease

C Duodenal ulcer

D Recurrent viral gastroenteritis

E Ulcerative colitis

Explanation
Coeliac disease is the correct answer. Children with coeliac disease may present with failure to thrive, diarrhoea,
muscle wasting and abdominal distension. Duodenal biopsy remains the gold standard investigation for coeliac
disease, and total villous atrophy is the most typical histological feature.

Crohn’s disease can occur at any point along the gastrointestinal tract. Microscopic features can be highly
variable, and include increased lamina propria plasma cells, transmural inflammation, skip lesions and submucosal
fibrosis. Features are not limited to the duodenum.

The duodenum is the most common site for a peptic ulcer, however, symptoms of diarrhoea and failure to thrive in
a 4-year-old are more fitting with a diagnosis of coeliac disease.

Recurrent viral gastroenteritis should be considered. There is, however, also failure to thrive and sufficient
concern to lead to an OGD (oesophago-gastro- duodenoscopy - also known simply as a gastroscopy or endoscopy)
and biopsy. Coeliac is therefore more likely.

Colorectal, not duodenal biopsy can be helpful in the diagnosis of ulcerative colitis.

https://mypastest.pastest.com/Secure/TestMe/Browser/436619[‫ ص‬08:34:16 11/12/1437]


MyPastest

http://www.bsg.org.uk/pdf_word_docs/ideopathic.pdf
47047
Previous Question Next Question

Tag Question
End Session
Feedback

Difficulty: Average

Peer Responses

Session Progress

Responses Correct: 0

Responses Incorrect: 71

Responses Total: 71

Responses - % Correct: 0%

Blog
About Pastest
Contact Us
Help

© Pastest 2016

https://mypastest.pastest.com/Secure/TestMe/Browser/436619[‫ ص‬08:34:16 11/12/1437]


MyPastest

Prefer to use the old MyPastest? Access it here »

Back to Filters

Question 70 of 78

A 6-year-old boy presents with polydipsia, polyuria and unexplained weight loss. On examination, he is lethargic
and thin. His
blood sugar is 34mmol/mol. His urine dipstick shows the presence of glucose and ketones, and his
blood ketones are 4.0. You are about to take bloods, including a blood gas.

Which ONE of these conditions should be screened for at presentation?

A Coeliac disease

B Diabetic retinopathy

C Diabetic neuropathy

D Diabetic nephropathy

E Multiple endocrine neoplasia type 1

Explanation
The correct answer is A. Coeliac disease, which is more prevalent in patients with type one diabetes, and NICE
guidelines state that it should be screened for in patients presenting with T1D. TFTs should also be sent to screen
for thyroid disease.

B, C and D develop over time. Annual reviews in patients with T1D should ensure that all necessary screening is
being undertaken. These conditions, however, are not routinely screened for at diagnosis.

E is incorrect. MEN type 1 (Werner syndrome) consists of tumours of the pancreas, pituitary and parathyroid
gland. Excess glucagon causes diabetes mellitus. There is no suggestion in the stem of the question that any other
endocrine glands are affected, and screening for MEN is not routinely performed.

https://www.nice.org.uk/guidance/ng20/chapter/key-priorities-for-implementation
47048

https://mypastest.pastest.com/Secure/TestMe/Browser/436619[‫ ص‬08:34:37 11/12/1437]


MyPastest

Previous Question Next Question


Tag Question

Feedback

End Session
Difficulty: Average

Peer Responses

Session Progress

Responses Correct: 0

Responses Incorrect: 72

Responses Total: 72

Responses - % Correct: 0%

Blog
About Pastest
Contact Us
Help

© Pastest 2016

https://mypastest.pastest.com/Secure/TestMe/Browser/436619[‫ ص‬08:34:37 11/12/1437]


MyPastest

Prefer to use the old MyPastest? Access it here »


Next Question

Back to Filters

Question 71 of 78

A patient with IgA deficiency is referred to your clinic. You suspect a diagnosis of coeliac disease and want to
send a blood sample.

What is the MOST appropriate blood test to perform?

A IgA tissue transglutaminase (tTG)

B IgA endomysial antibody (EMA)

C IgG tTG

D Refer urgently for a duodenal biopsy

E There is no appropriate test

Explanation
The correct answer is C, IgG tTG. NICE guidelines state that in children, laboratories should test for total IgA and
IgA tTG as the first choice. However, when IgA is deficient, laboratories should consider using IgG EMA, IgG
DGP or IgG tTG.

A and B are both incorrect because this patient is IgA deficient, so a false negative result may occur.

D is incorrect. The question specifically asks for the most appropriate blood test.

E is also incorrect, because there is an appropriate test for this scenario (as above).

https://www.nice.org.uk/guidance/ng20/chapter/key-priorities-for-implementation
47049

https://mypastest.pastest.com/Secure/TestMe/Browser/436619[‫ ص‬08:34:57 11/12/1437]


MyPastest

Tag Question

Previous Question
Feedback

Difficulty: Average

Peer Responses
End Session

Session Progress

Responses Correct: 0

Responses Incorrect: 73

Responses Total: 73

Responses - % Correct: 0%

Blog
About Pastest
Contact Us
Help

© Pastest 2016

https://mypastest.pastest.com/Secure/TestMe/Browser/436619[‫ ص‬08:34:57 11/12/1437]


MyPastest

Prefer to use the old MyPastest? Access it here »


Next Question

Back to Filters

Question 72 of 78

The mother of a 3-year-old girl comes to see you in the General Paediatric clinic. She is concerned that her
daughter is allergic to fish. Every time she eats fish she gets GI symptoms. Mother
has very helpfully kept a food
diary and a record of her symptoms, which she has brought to clinic today.

Which of these is LEAST likely to suggest food allergy?

A Abdominal pain

B Constipation

C Loose, frequent stools

D Pallor

E Polycythaemia

Explanation
Stem E, ‘polycythaemia’ is not a manifestation of food allergy/intolerance. Polycythaemia is a condition
in which
the proportion of red blood cells is increased above normal levels making blood less able to flow. This may be a
result of there being a lesser volume of plasma in the blood causing it to be thicker (apparent polycythaemia) or
because the body is producing additional red
blood cells (absolute polycythaemia).

A, B, C and D are all non IgE mediated signs and symptoms of possible food allergy, as outlined in the 2011
NICE Guidelines for Food Allergy in Under 19’s Assessment and Diagnosis.

http://pathways.nice.org.uk/pathways/food-allergy-in-children-and-young-people#content=view-node%3Anodes-
initial-recognition
47050

https://mypastest.pastest.com/Secure/TestMe/Browser/436619[‫ ص‬08:35:19 11/12/1437]


MyPastest

Tag Question
Previous Question
Feedback

Difficulty: Average

Peer Responses
End Session

Session Progress

Responses Correct: 0

Responses Incorrect: 74

Responses Total: 74

Responses - % Correct: 0%

Blog
About Pastest
Contact Us
Help

© Pastest 2016

https://mypastest.pastest.com/Secure/TestMe/Browser/436619[‫ ص‬08:35:19 11/12/1437]


MyPastest

Prefer to use the old MyPastest? Access it here »

Back to Filters

Question 73 of 78

A 7-year-old boy presents to the GP with recurrent episodes of diarrhoea and abdominal pain. He has severe
eczema which is very difficult to manage, despite the frequent, generous use of emollients and hydrocortisone
during flare ups. Mother has removed wheat from his diet, with no improvement. His height and weight are on the
2nd centile.

What is the MOST appropriate next course of action?

A Ask mother to keep a food diary and refer to allergy clinic

B Ask mother to keep a food diary and refer to endocrine clinic for growth assessment

C Food challenge at the GP surgery

D Refer to allergy clinic

E Skin prick test to wheat in the community

Explanation
The correct answer is A. This child fits the criteria for referral to secondary or specialist care (see NICE guidelines
below). He has not responded to a single allergen elimination
diet, he has faltering growth in combination with GI
symptoms and he also has significant atopic eczema. This child may have multiple food allergies.

B is not appropriate. The history points to a gastrointestinal cause for his faltering growth, rather than an endocrine
one.

C is incorrect. If a food challenge is to be undertaken then this should be done in hospital, with necessary
measures taken to prepare for
an anaphylactic reaction.

D is not the best answer. A is more appropriate because a food diary would be helpful to tease out the most likely
dietary causes(s) for the GI symptoms.

E is also incorrect. A skin prick test may be performed at the allergy clinic, but
the community is not an

https://mypastest.pastest.com/Secure/TestMe/Browser/436619[‫ ص‬08:35:41 11/12/1437]


MyPastest

appropriate setting for this.

http://pathways.nice.org.uk/pathways/food-allergy-in-children-and-young-people/food-allergy-in-children-and-
Previous Question Next Question
young-people-overview#content=view-node:nodes-consider-referral-to-secondary-or-specialist-care
47051

End Session
Tag Question

Feedback

Difficulty: Average

Peer Responses

Session Progress

Responses Correct: 0

Responses Incorrect: 75

Responses Total: 75

Responses - % Correct: 0%

Blog
About Pastest
Contact Us
Help

© Pastest 2016

https://mypastest.pastest.com/Secure/TestMe/Browser/436619[‫ ص‬08:35:41 11/12/1437]


MyPastest

Prefer to use the old MyPastest? Access it here »

Back to Filters

Question 74 of 78

A 10-year-old girl with a known history of peanut allergy is brought in by ambulance. She was eating a chocolate
bar, when she developed sudden onset of shortness of breath, wheeze, swelling of the face and lips and itching all
over. She now has stridor and saturations
are 88% in air. You apply facial oxygen.

What is the most appropriate IMMEDIATE course of action?

A Adrenaline 300 micrograms IM

B Adrenaline 150 micrograms IM

C Adrenaline IV 300 micrograms

D Chlorphenamine

E Salbutamol

Explanation
The correct answer is A. This is an anaphylactic reaction and IM adrenaline must be administered immediately.

B is incorrect. The correct dose for a child aged 6-12 years is 300 micrograms. 150 micrograms would be
appropriate a child under 6 years.

C is incorrect. IM and not IV adrenaline will be faster in this child who does not have a cannula.

D is incorrect. Although chlorphenamine will help the itching, adrenaline is necessary immediately to treat the
anaphylactic reaction.

E is also incorrect. Salbutamol can be helpful if the child is wheezy. The most important treatment to administer
quickly though is the IM adrenaline.

https://www.resus.org.uk/anaphylaxis/emergency-treatment-of-anaphylactic-reactions/
47052

https://mypastest.pastest.com/Secure/TestMe/Browser/436619[‫ ص‬08:36:03 11/12/1437]


MyPastest

Previous Question Next Question


Tag Question

Feedback

End Session
Difficulty: Average

Peer Responses

Session Progress

Responses Correct: 0

Responses Incorrect: 76

Responses Total: 76

Responses - % Correct: 0%

Blog
About Pastest
Contact Us
Help

© Pastest 2016

https://mypastest.pastest.com/Secure/TestMe/Browser/436619[‫ ص‬08:36:03 11/12/1437]


MyPastest

Prefer to use the old MyPastest? Access it here »


Next Question

Back to Filters

Question 75 of 78

A 2-year-old girl has had previous reactions to egg. Whenever she eats egg, she has a rash and itching all over.
Allergy has
been confirmed on skin-prick testing. She missed her MMR vaccine at thirteen months. Her mother
wants to know whether she can have her immunisations.

What is the MOST appropriate advice to give?

A She can have the intranasal influenza vaccine but not the MMR

B She cannot have the intranasal influenza vaccine

C She should have all immunisations at the GP practice

D She should have all immunisations under close supervision in hospital

E She should have the MMR under close supervision in hospital

Explanation
C is the correct answer. The advice from the Department of Health is that children with egg allergy should receive
all routine vaccinations as part of the childhood immunisation programme. Both intranasal and inactivated injected
influenza vaccines,
that are egg-free or have a very low ovalbumin content, are safe for children with egg allergy.
The measles vaccine is not grown in whole eggs, but in cells derived from hens (not containing egg protein). The
MMR vaccine is therefore safe in egg-allergic children. The exception to
this is children who have required
intensive care for egg allergy, who should receive the MMR in hospital.

A, B, D and E are therefore all incorrect for the reasons given above

https://www.allergyuk.org/advice-for-parents-with-a-new-baby/immunisations
47053

https://mypastest.pastest.com/Secure/TestMe/Browser/436619[‫ ص‬08:36:25 11/12/1437]


MyPastest

Tag Question
Previous Question
Feedback

Difficulty: Average

Peer Responses
End Session

Session Progress

Responses Correct: 0

Responses Incorrect: 77

Responses Total: 77

Responses - % Correct: 0%

Blog
About Pastest
Contact Us
Help

© Pastest 2016

https://mypastest.pastest.com/Secure/TestMe/Browser/436619[‫ ص‬08:36:25 11/12/1437]


MyPastest

Prefer to use the old MyPastest? Access it here »


Next Question

Back to Filters

Question 76 of 78

The parents of a 4-month-old boy come to see you in clinic. He has been exclusively breastfed so far. He used to
be a “sicky baby” until the mother found that if she cut out cows’ milk
from her own diet, his symptoms
improved. At four months, he is growing well along the 50th centile and looks very well. Parents would like to
introduce bottle feeds.

Which is the LEAST appropriate formula milk to give this child?

A AptamilPepti 1®

B NutramigenLipil®1

C Neocate LCP®

D PregestimilLipil®

E Wysoy®

Explanation
The correct answer is E. Wysoy® is a soy based formula. Soya based formulas are not suitable as a first-line
product for CMPA treatment. In children under 6 months, the phytate content of soy based formulas may inhibit
the absorption of minerals, and they also contain isoflavones which have oestrogenic actions.

A, B, C and D are all hydrolysed formulas that are suitable from birth in a child with cows’ milk protein allergy.

http://cks.nice.org.uk/cows-milk-protein-allergy-in-children#!scenario
47054

https://mypastest.pastest.com/Secure/TestMe/Browser/436619[‫ ص‬08:36:46 11/12/1437]


MyPastest

Tag Question

Previous Question
Feedback

Difficulty: Average

Peer Responses
End Session

Session Progress

Responses Correct: 0

Responses Incorrect: 78

Responses Total: 78

Responses - % Correct: 0%

Blog
About Pastest
Contact Us
Help

© Pastest 2016

https://mypastest.pastest.com/Secure/TestMe/Browser/436619[‫ ص‬08:36:46 11/12/1437]


MyPastest

Prefer to use the old MyPastest? Access it here »

Back to Filters

Question 77 of 78

A term infant who was born by SVD following an uneventful pregnancy is now 50 hours old and has still not
passed meconium. You review him due to bilious vomiting, and are worried enough to ask for a surgical review. A
contrast enema is performed and demonstrates microcolon.

Which long term medication is this child MOST likely to need?

A IM Testosterone

B Metformin

C Orlistat

D Pancrelipase

E SC Insulin

Explanation
The correct answer is D pancrelipase. This baby has meconium ileus, obstruction of the bowel by abnormally
tenacious meconium. Approximately a third of neonates with meconium ileus have cystic fibrosis. Pancreatic
status should be established at diagnosis by clinical assessment and by measuring faecal elastase. Patients with
pancreatic insufficiency should receive pancreatic enzyme replacement and fat soluble vitamins.

A is incorrect. Although delayed puberty onset is more common in both girls and boys with CF, onset is likely to
occur without the need for testosterone injections.

Metformin and orlistat are oral medications used to treat individuals with obesity. Individuals with CF are more
likely to need nutritional supplements and help with gaining weight.

SC insulin may be indicated in cases of CF related diabetes. Although CFRD is a common comorbidity in people
with CF, pancreatic insufficiency is more common.
Therefore, D is the best answer and E is incorrect.

https://www.cysticfibrosis.org.uk/

https://mypastest.pastest.com/Secure/TestMe/Browser/436619[‫ ص‬08:37:07 11/12/1437]


MyPastest

47055

Previous Question Next Question

Tag Question

Feedback End Session

Difficulty: Average

Peer Responses

Session Progress

Responses Correct: 0

Responses Incorrect: 79

Responses Total: 79

Responses - % Correct: 0%

Blog
About Pastest
Contact Us
Help

© Pastest 2016

https://mypastest.pastest.com/Secure/TestMe/Browser/436619[‫ ص‬08:37:07 11/12/1437]


MyPastest

Prefer to use the old MyPastest? Access it here »

Back to Filters

Question 78 of 78

A 14-year-old girl presents with severe abdominal pain radiating to the back and vomiting. Her heart rate is 200
and BP is 100/45. On examination there is tenderness and guarding in the upper abdomen. Her blood tests show
Hb 134; WCC 16; Plts 253; Na 140; K 3.5; Creat 90; Urea 6.4; Amylase 800; AST 12; ALT 16; Bili 2; Alb 4.2.

What is the MOST likely diagnosis?

A Acute hepatitis

B Chronic hepatitis

C Gastroenteritis

D Pancreatitis

E Pregnancy

Explanation
Pancreatitis is the correct answer. Although rare in childhood, in the presence of abdominal pain radiating to the
back with shock and tachycardia, and a raised amylase, pancreatitis is the most likely diagnosis. Possible causes
are trauma, drugs, viral illness, mumps, hyperparathyroidism, hyperlipidaemia and CF.

A and B are incorrect. Normal LFTs make hepatitis unlikely.

C is also incorrect. Gastroenteritis with severe diarrhoea and vomiting may account for the shock and tachycardia,
but guarding on abdominal examination and the raised amylase would not be explained by gastroenteritis.

E is also incorrect. Pregnancy is an important


diagnosis to consider in an adolescent with abdominal pain, but the
raised amylase and other features point to a diagnosis of pancreatitis.
47056

End Session

https://mypastest.pastest.com/Secure/TestMe/Browser/436619[‫ ص‬08:37:33 11/12/1437]


MyPastest

Previous Question Tag Question

Feedback

Difficulty: Average

Peer Responses

Session Progress

Responses Correct: 0

Responses Incorrect: 80

Responses Total: 80

Responses - % Correct: 0%

Blog
About Pastest
Contact Us
Help

© Pastest 2016

https://mypastest.pastest.com/Secure/TestMe/Browser/436619[‫ ص‬08:37:33 11/12/1437]


MyPastest

Prefer to use the old MyPastest? Access it here »

Back to Filters

Question 1 of 120

A 3-month-old infant of a 17-year-old mother was referred with a 5-day history of cough, fever, rapid breathing
and excessive crying. Antenatally, the mother was involved in a car accident as a passenger with no major injuries.
The baby was born by vaginal delivery at 36 weeks and needed active resuscitation with IPPV and external
cardiac massage. Subsequent progress was uneventful. The birth weight was 2.48 kg (50th centile).

On admission, the current weight was 4.2 kg (9th centile), and the baby was irritable and crying when handled.
Respiratory rate 55/min. Vesicular breath sounds with bilateral
crepitations. CXR showed slight hyperinflation.
Unexpected fracture of left 2nd, 5th and 6th ribs with well formed callus was visible. U&E,
creatinine, serum
calcium, serum phosphate and alkaline phospatase were
all normal. A further skeletal survey did not reveal any
other fractures.

The radiological finding of rib fractures was most likely to have been caused by?

A The antenatal car accident

B The CPR at birth

C Physical abuse

D Osteogenesis imperfecta

E Pertussis

Explanation
This baby had a respiratory illness of viral origin (most likely bronchiolitis given the examination findings) and
the presence of healing rib fractures was an unexpected finding on the chest
X-ray. The presence of well formed
callus antedates the occurrence of the fractures to 1-2 weeks prior to the x-ray. Antenatal accident as a cause of
fracture is extremely rare, as is post resuscitation fracture. However, considering the age of the baby the callus
should have completely resolved if it had occured at that stage. The possibility of osteogenesis imperfecta is

https://mypastest.pastest.com/Secure/TestMe/Browser/436619[‫ ص‬10:47:50 10/12/1437]


MyPastest

usually mooted at case hearings but on the whole a completely normal bone x-ray and otherwise normal skeletal
survey negates this diagnosis. Rarely, severe pertussis may be the cause
but multiple fractures are very rare. 14569
Previous Question
End Session

Tag Question

Feedback

Difficulty: Average

Peer Responses

Session Progress

Responses Correct: 0

Responses Incorrect: 1

Responses Total: 1

Responses - % Correct: 0%

Blog
About Pastest
Contact Us
Help

© Pastest 2016

Next Question

https://mypastest.pastest.com/Secure/TestMe/Browser/436619[‫ ص‬10:47:50 10/12/1437]


MyPastest

Prefer to use the old MyPastest? Access it here »

Back to Filters

Question 2 of 120

A 7-year-old boy with cerebral palsy and developmental delay was referred for investigation because of
uncontrolled seizures and respiratory problems, intially diagnosed and treated as asthma. He attends normal school
where he has extra support. He is a 'difficult feeder', and had clinical evidence of chest infection on admission. He
was noted to have recurrent spasmodic movements with arching of the back, throwing his arms and head, and
lurching forward. These were associated with gurgling noises in the throat, but not with loss of awareness. These
were more frequent during respiratory illnesses and especially during, or immediately following meals. 2 EEGs
were reported as normal. Antiepileptic drugs have had no significant effect.

The clinical picture is most compatible with?

A Dystonic cerebral palsy

B Uncontrolled asthma

C Partial complex seizures

D Sandifer syndrome

E Immune deficiency

Explanation
This 7-year-old has 'dystonic' movements associated with feeding, and which also appears to be exacerbated with
chest infections. These have not been noted at other times, and specifically no intention tremor or incoordination
has been noted. The constellation of symptoms is compatible with severe gastro-oesophageal reflux and
oesophagitis that can lead dystonic movements due to the extreme discomfort especially at, and following, meals.
The reflux can be the cause of recurrent chestiness and wheeze that may present as or exacerbate pre-normal
EEGs. An ambulatory oesophageal pH study confirmed
severe gastro-oesophageal reflux disease and specific
treatment relieved all his symptoms. 14570

https://mypastest.pastest.com/Secure/TestMe/Browser/436619[‫ ص‬10:48:26 10/12/1437]


MyPastest

Previous Question
End Session
Tag Question

Feedback

Difficulty: Average

Peer Responses

Session Progress

Responses Correct: 0

Responses Incorrect: 2

Responses Total: 2

Responses - % Correct: 0%

Blog
About Pastest
Contact Us
Help

© Pastest 2016

Next Question

https://mypastest.pastest.com/Secure/TestMe/Browser/436619[‫ ص‬10:48:26 10/12/1437]


MyPastest

Prefer to use the old MyPastest? Access it here »

Back to Filters

Question 3 of 120

A Infantile spasms
B Childhood absence epilepsy (petit mal)
C Juvenile myoclonic epilepsy
D Febrile convulsions
E Reflex anoxic seizures
F Daydreaming
G Rigors
H Non-epileptic attack disorder syndrome
I Night terrors
J Tics

For
each of the clinical scenarios described choose the most likely diagnosis. Each option may be used once, more
than once, or not at all.

Scenario 1

A 6-year-old girl is seen by you in the GP surgery. Her school teacher noticed her to be daydreaming in school.
Her
mother has also noticed this when the child is bored. Birth history and
development were normal. Clinical
examination is normal. You notice her
go quiet and stare into space. She blinks on confrontation.

Your answer was incorrect

Select one...

F - Daydreaming

Remember there may not always be a sinister pathology going on! Childhood absence epilepsy (petit mal) may
have the same presentation. However, in an absence, the child will not blink on confrontation (e.g. wave of hand
in front of eyes). True absences can occur in any setting. They can be precipitated by hyperventilation.

Scenario 2

An 18-month-old boy is brought to the GP surgery.


He is coryzal, his temperature is 39 °C and he has a red
pharynx. His arms and legs begin to twitch and shake but he remains conscious. When you place your hands on
his limbs, the shaking stops. Birth history
and development were normal. His brother had febrile convulsions.

https://mypastest.pastest.com/Secure/TestMe/Browser/436619[‫ ص‬10:49:24 10/12/1437]


MyPastest

Your answer was incorrect

Select one... Previous Question


End Session
G - Rigors

In a febrile child who begins to twitch and shake in all four limbs the two main differential diagnoses are febrile
convulsions and rigors. As the child is still conscious they must be having a rigor which is not the case in a
seizure.

Scenario 3

A 5-month-old boy is brought to the GP with recent onset attacks of screaming with drawing of his legs and arms
up to his chest. He has stopped smiling and babbling, and has become very irritable. Mother asks if he has colic.

Your answer was incorrect

Select one...

A - Infantile spasms

The description of salaam attacks together with regression of acquired skills should lead you to the diagnosis of
infantile spasms. Infantile spasms are a severe epileptic encephalopathy, presenting normally between 3 months to
1 year. They are
characterised by runs of flexion and extension spasms. Often associated
with a peculiar cry and
mistaken initially for colic. They are important not to miss and to initiate treatment as soon as possible because
development ceases with the onset of spasms.
14571

Tag Question

Feedback

Difficulty: Average

Session Progress

Responses Correct: 0

Responses Incorrect: 5

Responses Total: 5

Responses - % Correct: 0%

Next Question

https://mypastest.pastest.com/Secure/TestMe/Browser/436619[‫ ص‬10:49:24 10/12/1437]


MyPastest

Blog
About Pastest
Contact Us
Help

© Pastest 2016

https://mypastest.pastest.com/Secure/TestMe/Browser/436619[‫ ص‬10:49:24 10/12/1437]


MyPastest

Prefer to use the old MyPastest? Access it here »

Back to Filters

Question 4 of 120

Theme: Capillary blood and electrolyte findings

pH pO2 (kPa) pCO2 (kPa) HCO3 (mmol/l) Na (mmol/l) K (mmol/l) Cl (mmol/l)

A 7.32 8.0 4.0 26 140 5.5 102

B 7.18 7.8 3.0 7 140 3.8 99

C 7.12 15.0 8.2 16 138 4.2 106

D 7.5 6.0 2.8 18 132 4.0 76

E 7.5 8.3 4.1 26 127 2.8 92

F 7.5 8.2 4.0 18 126 2.7 100

For
each of the patients described below select the most likely arterial blood and electrolyte findings. Each option
may be used once, more than once, or not at all.

Scenario 1

A 6-year-old has had polyuria, excessive thirst, and recent loss of weight. Over the last 24 hours he had vomited
several
times and has been found to be drowsy and disorientated since the previous night. Urine analysis showed
4+ glucose and 4+ ketones

Your answer was incorrect

Select one...

B - pH 7.18, pO2 7.8, pCO2 3.0, HCO3 7, Na 140, K 3.8, Cl 99

Diabetic ketoacidosis gives rise to a metabolic acidosis with a high sodium and low to normal potassium. As it
takes weeks to arrive at this stage the bicarbonate will be extremely low.

Scenario 2

A 4-week-old baby was admitted with a history of vomiting that has worsened over a 2 week period. He feeds

https://mypastest.pastest.com/Secure/TestMe/Browser/436619[‫ ص‬10:49:46 10/12/1437]


MyPastest

well but continues to lose weight. Clinically, he appeared wasted. Test feed was followed by an impressive
projectile vomit. A transient mass was felt in
the hypochondrium.
Previous Question
Your answer was incorrect
End Session
Select one...

E - pH 7.5, pO2 8.3, pCO2 4.1, HCO3 26, Na 127, K 2.8, Cl 92

Pyloric stenosis gives rise to a metabolic alkalosis due to the multiple vomits with hypokalaemia and
hypochloraemia.

Scenario 3

A 14-year-old girl with a previous history of self harm was brought in to the Emergency Department following
another attempt 10 hours previously. She was alleged to have swallowed a large number of aspirin tablets.

Your answer was incorrect

Select one...

F - pH 7.5, pO2 8.2, pCO2 4.0, HCO3 18, Na 126, K 2.7, Cl 100

The acid-base, fluid, and electrolyte abnormalities seen with salicylate toxicity can be grouped into phases.

Phase
1 of the toxicity is characterized by hyperventilation resulting from direct respiratory center stimulation,
leading to respiratory alkalosis and compensatory alkaluria. Potassium and sodium bicarbonate are excreted in the
urine. This phase may last as long as 12 hours.

In
phase 2, paradoxic aciduria in the presence of continued respiratory alkalosis occurs when sufficient potassium
has been lost from the kidneys. This phase may begin within hours and may last 12-24 hours.
Phase
3 includes dehydration,hypokalemia and progressivemetabolic acidosis. This phase may begin 4-6 hours
after ingestion in a young infant or 24 hours or more after ingestion in an adolescent or adult
14572

Tag Question

Feedback

Difficulty: Difficult

Session Progress

Responses Correct:
Next Question 0

https://mypastest.pastest.com/Secure/TestMe/Browser/436619[‫ ص‬10:49:46 10/12/1437]


MyPastest

Responses Incorrect: 8

Responses Total: 8

Responses - % Correct: 0%

Blog
About Pastest
Contact Us
Help

© Pastest 2016

https://mypastest.pastest.com/Secure/TestMe/Browser/436619[‫ ص‬10:49:46 10/12/1437]


MyPastest

Prefer to use the old MyPastest? Access it here »


Previous Question
End Session

Back to Filters

Question 5 of 120

A 6-month-old female baby is brought into the Emergency Department with a history of vomiting, colic,
listlessness, not wanting to feed and a temperature. She has passed a bloody nappy. On examination
she is
tachycardic and has cool peripheries.

The next best step is?

A Perform an abdominal x-ray (AXR) and abdominal ultrasound

B Perform a lumbar puncture

C Organise an air enema

D Resuscitate with intravenous fluids and commence triple antibiotics

E Culture the stool

Explanation
The vignette describes a sick, potentially collapsing baby. The most likely diagnosis is intussusception but
resuscitation is the first vital step, always remember to assess A, B, C
first. The other steps such as
AXR/ultrasound/air enema will probably be needed but not first. Stool culture and lumbar puncture would be
included if it turned out not to be an intussusception. 14585

Tag Question

Feedback
Next Question

https://mypastest.pastest.com/Secure/TestMe/Browser/436619[‫ ص‬10:50:09 10/12/1437]


MyPastest

Difficulty: Easy

Peer Responses

Session Progress

Responses Correct: 0

Responses Incorrect: 9

Responses Total: 9

Responses - % Correct: 0%

Blog
About Pastest
Contact Us
Help

© Pastest 2016

https://mypastest.pastest.com/Secure/TestMe/Browser/436619[‫ ص‬10:50:09 10/12/1437]


MyPastest

Prefer to use the old MyPastest? Access it here »

Back to Filters

Question 6 of 120

Theme: Analysis of blood gases

A Compensated metabolic acidosis


B Compensated respiratory acidosis
C Metabolic acidosis
D Metabolic alkalosis
E Mixed metabolic/respiratory acidosis
F Mixed metabolic/respiratory alkalosis
G Partially compensated metabolic acidosis
H Partially compensated respiratory acidosis
I Respiratory acidosis
J Respiratory alkalosis

For
each of the following cases, choose the most appropriate analysis of the blood gas results from the above list.
Each item may be used once, more than once or not at all.

Scenario 1

An ex-23-weeks’ gestation baby is discharged with home oxygen. His venous blood gas analysis before discharge
shows: pH 7.39, PaCO2 12.1 kPa, PaO2 6.1 kPa, HCO3– 42 mmol/L, BE (base excess) +13 mmol/L.

Your answer was incorrect

Select one...

B - Compensated respiratory acidosis

The PaCO2 is raised, but the pH is normal so this is a fully compensated gas. Given the history, this is a
compensated respiratory acidosis.

Scenario 2

A 6-year-old child with known diabetes presents with an intercurrent illness. His BM (blood glucose testing strip)
is 25
and an arterial blood gas shows: pH 7.32, PaCO2 2.9 kPa, PaO2 10.2 kPa, HCO3– 18 mmol/L, BE –7
mmol/L.

https://mypastest.pastest.com/Secure/TestMe/Browser/436619[‫ ص‬10:50:32 10/12/1437]


MyPastest

Your answer was incorrect

Select one...

G - Partially compensated metabolic acidosis

A pH of 7.32 is mildly acidotic, the likely diagnosis is diabetic ketoacidosis. The HCO3– is low so this is a
metabolic acidosis; however, the PaCO2 is low (2.9 kPa) so there is some respiratory compensation (though NOT
fully).

Scenario 3

A 7-week-old newborn girl presents with vomiting. A capillary blood gas shows pH 7.46, PaCO2 6.4 kPa, PaO2
5.5 kPa, HCO3– 38 mmol/L, BE +4 mmol/L.

Your answer was incorrect

Select one...

D - Metabolic alkalosis

An alkalotic gas: the PaCO2 is slightly raised and there is a very raised HCO3– ;
this is consistent with a metabolic
alkalosis. (The likely diagnosis in
this child is pyloric stenosis or possibly a pre-ampullary duodenal stenosis –
leading to loss of H+ and Cl– in the vomit.)
Normal ranges of blood gas variables

- Normal range

pH 7.35-7.45

PCO2 4.5-5.5 kPa

PO2 6.5-13.5 kPa

HCO3– 25-35 mmol/L

BE (Base excess) -1 to +1 mmol/L

Capillary samples: approximate normal ranges are as given in the table above.

However PaCO2 upper limits can be allowed up to 6.5 kPa. Higher PaCO2
values can be permitted on venous
samples; however, if oxygenation is the concern, an arterial sample should be taken (capillary samples are well
arterialised and can be interpreted as arterial).
22015

Next Question

https://mypastest.pastest.com/Secure/TestMe/Browser/436619[‫ ص‬10:50:32 10/12/1437]


MyPastest

Tag Question

Previous Question
Feedback

End Session
Difficulty: Average

Session Progress

Responses Correct: 0

Responses Incorrect: 12

Responses Total: 12

Responses - % Correct: 0%

Blog
About Pastest
Contact Us
Help

© Pastest 2016

https://mypastest.pastest.com/Secure/TestMe/Browser/436619[‫ ص‬10:50:32 10/12/1437]


MyPastest

Prefer to use the old MyPastest? Access it here »

Back to Filters

Question 7 of 120

Theme: Rashes

A Erysipelas

B Hand, foot and mouth disease

C Infectious mononucleosis

D Measles

E Molluscum contagiosum

F Pityriasis rosea

G Rubella

H Sixth disease

I Slapped cheek syndrome

J Varicella-zoster

For
each of the following clinical scenarios, choose the most likely diagnosis from the above list. Each item may
be used once, more than once or not at all.

Scenario 1

A 5-year-old boy presents with a macular rash on the trunk, some of the macules run parallel to his ribs. His
mother reports that 5 days ago there was only one spot that must have spread.

Your answer was incorrect

Select one...

F - Pityriasis rosea

The presence of the single spot and then the development of this rash fit very well with the ‘Herald’ patch and
‘Christmas tree’ rash of pityriasis rosea. Pityriasis is probably viral in origin but the exact causative organism is
unknown. The rash appears 1–2 weeks after the Herald patch, lasts 2 weeks then slowly resolves. Treatment is
symptomatic.

https://mypastest.pastest.com/Secure/TestMe/Browser/436619[‫ ص‬10:50:54 10/12/1437]


MyPastest

Scenario 2

Previous
A 4-year-old child Question
is systemically unwell; she has erythematous cheeks that are hot and very tender to touch.

Your answer was incorrect End Session


Select one...

A - Erysipelas

Slapped cheek syndrome is possible but


this child is systemically unwell and the cheeks are cellulitic. Erysipelas is
commonly caused by Group A streptococci. Treatment is with
intravenous penicillin.

Scenario 3

A 14-month-old baby girl presents with a fine pink macular rash that started on her face and spread to her trunk.
Cervical and occipital lymph nodes are easily palpable.

Your answer was incorrect

Select one...

G - Rubella

Rubella has an incubation period of 2–3 weeks. The rash appears on the face, spreads down to the trunk
and then
finally affects the limbs. Cervical, occipital and posterior auricular lymph nodes are often enlarged before the
appearance of the rash.
22022

Tag Question

Feedback

Difficulty: Average

Session Progress

Responses Correct: 0

Responses Incorrect: 15

Responses Total: 15

Responses - % Correct: 0%
Next Question

https://mypastest.pastest.com/Secure/TestMe/Browser/436619[‫ ص‬10:50:54 10/12/1437]


MyPastest

Blog
About Pastest
Contact Us
Help

© Pastest 2016

https://mypastest.pastest.com/Secure/TestMe/Browser/436619[‫ ص‬10:50:54 10/12/1437]


MyPastest

Prefer to use the old MyPastest? Access it here »

Back to Filters

Question 8 of 120

Theme: The unwell infant

A Cardiac failure
B Congenital toxoplasmosis
C Delayed group B streptococcal sepsis
D Delayed haemorrhagic disease of the newborn
E Duct-dependant cardiac lesion
F Galactosaemia
G Hereditary lactic acidosis
H Meningitis
I Non-accidental injury (NAI)
J Wilson’s disease

For
each of the following ‘unwell infant’ scenarios, choose the
most likely diagnosis from the above list. Each
item may be used once, more than once or not at all.

Scenario 1

A 3-week-old baby is brought to the Emergency Department; he has recently been having problems completing
his feeds and today appears short of breath. On examination he has 4 cm hepatomegaly. All blood tests are normal.

Your answer was incorrect

Select one...

A - Cardiac failure

Difficulty completing feeds, along with sweating, is a good indicator of heart failure in infancy. Hepatomegaly is
another sign of cardiac failure. It is important to distinguish between cardiac failure (not cyanotic and caused by a
left-to-right shunt), and duct-dependent cardiac lesions (duct dependent: so when duct
closes infant becomes
cyanotic).

Scenario 2

A 2-week-old baby boy presents to the Emergency Department; he looks unwell. He is jaundiced, peripherally
shut down and
has hepatosplenomegaly. Baseline investigations show: Hb 8 g/dL, WCC 2.9 × 109/L, plts 120 ×

https://mypastest.pastest.com/Secure/TestMe/Browser/436619[‫ ص‬10:51:14 10/12/1437]


MyPastest

109/L, international normalised ratio (INR) 8.6, ?-glutamyl transferase (?-GT) 300 IU/L, BM 2.3, urine reducing
sugars +ve.
Previous Question
Your answer was incorrect
End Session
Select one...

F - Galactosaemia

The features of hepatomegaly, jaundice and abnormal clotting go together with both sepsis and hepatitis due to
galactosaemia. The urine-reducing sugars would be negative in sepsis – although in these cases in practice you
would treat for sepsis as well.

Scenario 3

A 6-week-old bottle-fed baby boy attends the Emergency Department with his parents. His father reports that he is
always crying. On examination he is afebrile, irritable and has a bulging fontanelle. Urgent computed tomography
(CT) (brain) shows multiple haemorrhages and generalised oedema.

Your answer was incorrect

Select one...

I - Non-accidental injury (NAI)

NAI, HDN (haemorrhagic disease of the newborn) and group B streptococcal sepsis are the three main
possibilities in this case. HDN is less likely because the baby is bottle-fed and should therefore be getting
sufficient vitamin K. If this
were HDN, then it would probably be an acute event, and the fact that the baby is
‘always’ crying points against this and more towards NAI.
22025

Tag Question

Feedback

Difficulty: Average

Session Progress

Responses Correct: 0

Responses Incorrect: 18

Responses Total: Next Question 18

https://mypastest.pastest.com/Secure/TestMe/Browser/436619[‫ ص‬10:51:14 10/12/1437]


MyPastest

Responses - % Correct: 0%

Blog
About Pastest
Contact Us
Help

© Pastest 2016

https://mypastest.pastest.com/Secure/TestMe/Browser/436619[‫ ص‬10:51:14 10/12/1437]


MyPastest

Prefer to use the old MyPastest? Access it here »

Back to Filters

Question 9 of 120

Theme: Choice of investigations

A Blood cultures
B Bone marrow aspiration cytology
C Clotting screen
D C-reactive protein (CRP)
E Erythrocyte sedimentation rate (ESR)
F Full blood count (FBC)
G Glucose-6-phosphate dehydrogenase (G6PDH) assay
H Haemoglobin electrophoresis
I Monospot
J Peripheral blood film

For
each of the following cases, choose the investigation most likely to give a definitive diagnosis from the above
list. Each item may be used once, more than once or not at all.

Scenario 1

A 6-month-old African–Caribbean infant presents with dactylitis.

Your answer was incorrect

Select one...

H - Haemoglobin electrophoresis

The most likely cause of dactylitis in a 6-month-old child is sickle cell disease. The investigation that would
diagnose this is Hb electrophoresis.

Scenario 2

A 7-year-old girl, who had an upper respiratory tract infection (URTI) 1 week ago, bruises easily and has
developed petechiae.

Your answer was incorrect

https://mypastest.pastest.com/Secure/TestMe/Browser/436619[‫ ص‬10:51:35 10/12/1437]


MyPastest

Select one...

Previous
B - Bone marrow aspirationQuestion
cytology

End Session
The most likely cause for this is immune thrombocytopenic purpura (ITP), however aplastic anaemia is possible,
as is ALL. Initial investigations would include full blood count and blood film. If these revealed any concerns
about aplastic anaemia or ALL, bone marrow aspirate cytology would be the next step.

Scenario 3

A 14-year-old boy who presents with a sore throat and palatal petechiae.

Your answer was incorrect

Select one...

I - Monospot

A 14-year-old boy with this history is most likely to have glandular fever; a monospot would diagnose this
definitively.
22030

Tag Question

Feedback

Difficulty: Average

Session Progress

Responses Correct: 0

Responses Incorrect: 21

Responses Total: 21

Responses - % Correct: 0%

Blog
About Pastest
Contact Us
Help

© Pastest 2016
Next Question

https://mypastest.pastest.com/Secure/TestMe/Browser/436619[‫ ص‬10:51:35 10/12/1437]


MyPastest

https://mypastest.pastest.com/Secure/TestMe/Browser/436619[‫ ص‬10:51:35 10/12/1437]


MyPastest

Prefer to use the old MyPastest? Access it here »

Back to Filters

Question 10 of 120

Which of the following statements is most correct with regard to the consent for an operation on a 15-year-old
child whose parents aren’t married?

A Either of the parents or the child can consent to the operation

B Either of the parents but not the child can consent to the operation

C Only the mother or the child can consent to the operation

D Only the mother can consent to the operation

E Only the child can consent to the operation

Explanation
If the parents are unmarried then the father has no legal parental responsibility. Although the law was changed for
births registered in England & Wales from 1st December 2003 to give parental rights to non-married fathers who
appear on the birth certificate, this will not apply retrospectively – i.e. if a child
was registered before 1st
December 2003 the father has no parental responsibility if not married to the mother. (Please note that different
rules may apply in Scotland and N. Ireland see the following website for details:
http://www.direct.gov.uk/en/Parents/ParentsRights/DG_4002954

Therefore,
only the mother can consent to procedures on the child. The child can consent for him or herself once
‘competent’. It is very unlikely that even a 15-year-old child would be competent to consent to an operation
(appreciating the risks of anaesthesia). An important point
is that even if a child can consent to a procedure when
competent (if under 16), legally they cannot refuse one if the person with parental responsibility consents.
22213

Next Question

https://mypastest.pastest.com/Secure/TestMe/Browser/436619[‫ ص‬10:51:55 10/12/1437]


MyPastest

Tag Question

Previous Question
Feedback

End Session
Difficulty: Average

Peer Responses

Session Progress

Responses Correct: 0

Responses Incorrect: 22

Responses Total: 22

Responses - % Correct: 0%

Blog
About Pastest
Contact Us
Help

© Pastest 2016

https://mypastest.pastest.com/Secure/TestMe/Browser/436619[‫ ص‬10:51:55 10/12/1437]


MyPastest

Prefer to use the old MyPastest? Access it here »


Previous Question
End Session

Back to Filters

Question 11 of 120

Which one of the following statements best describes clinical governance?

A Performance of audit, audit direct process change and completion of cycles

B Comprehensive risk assessment and incident reporting

C Implementation and systematic review of clinical effectiveness

D Modification of healthcare systems to optimise patient care

E Systematic training of healthcare workers to effect quality provision

Explanation
This is the best interpretation of clinical governance. The other statements are all important ‘pillars’
(parts) of
clinical governance but none individually encapsulate the whole concept.
22223

Tag Question

Feedback

Difficulty: Average

Peer Responses
Next Question

https://mypastest.pastest.com/Secure/TestMe/Browser/436619[‫ ص‬10:52:16 10/12/1437]


MyPastest

Session Progress

Responses Correct: 0

Responses Incorrect: 23

Responses Total: 23

Responses - % Correct: 0%

Blog
About Pastest
Contact Us
Help

© Pastest 2016

https://mypastest.pastest.com/Secure/TestMe/Browser/436619[‫ ص‬10:52:16 10/12/1437]


MyPastest

Prefer to use the old MyPastest? Access it here »


Previous Question
End Session

Back to Filters

Question 12 of 120

A 2-month-old baby girl is brought to your surgery as


her mother is concerned about a 1 cm lump situated lateral
to her right
eyebrow. The lump is firm and not attached to the skin. Which of the following lumps is the most
likely diagnosis?

A Enlarged lymph node

B Lipoma

C Branchial cyst

D External angular dermoid

E Neurofibroma

Explanation
External angular dermoid is by far the most likely diagnosis. External angular dermoids (dermoid cysts) are
embryological remnants that contain dermal and epidermal tissues. The site of the lump means that the other
options are much less likely. Neurofibromas are uncommon in children this young.
22229

Tag Question

Feedback

Next Question
Difficulty: Average

https://mypastest.pastest.com/Secure/TestMe/Browser/436619[‫ ص‬10:52:38 10/12/1437]


MyPastest

Peer Responses

Session Progress

Responses Correct: 0

Responses Incorrect: 24

Responses Total: 24

Responses - % Correct: 0%

Blog
About Pastest
Contact Us
Help

© Pastest 2016

https://mypastest.pastest.com/Secure/TestMe/Browser/436619[‫ ص‬10:52:38 10/12/1437]


MyPastest

Prefer to use the old MyPastest? Access it here »

Back to Filters

Question 13 of 120

A 3-year-old girl was hit by a reversing car (5 mph). She sustained a head injury and loss of consciousness for 1
minute. Subsequent to this she has vomited twice. On examination her Glasgow Coma Scale (GCS) is 15/15 and
there is no focal neurology. Which
of the following is the most appropriate course of action?

A Discharge with head injury advice for the parents

B Arrange urgent computed tomography (CT) scan of her brain

C Arrange anteroposterior (AP) and lateral skull radiographs

D Admit for 24 hours of neurological observations

E Observe for 4 hours in A&E and then send home with head injury advice if well

Explanation
The National Institute for Clinical Excellence (NICE) have produced national guidelines for the management of
head injuries. They are a little more complicated in children for a number of
reasons. Children tend to vomit after
injury; this may not be a result of a significant brain injury but a response to the injury. CT (investigation of choice
for significant head injuries) requires a degree of co-operation and lying still, which may be difficult for younger
children (< 5 years). In this patient, where there was loss of consciousness and some post-event vomiting, a CT
scan may be indicated. However, as the child is 3 years old this may be difficult unless it is performed under
general anaesthetic. In these patients admission for neurological observation is preferable; if there is any decrease
in GCS or development of focal neurological signs then a CT is definitely indicated.
22232

Next Question
Tag Question

https://mypastest.pastest.com/Secure/TestMe/Browser/436619[‫ ص‬10:53:25 10/12/1437]


MyPastest

Previous Question
Feedback

End Session
Difficulty: Average

Peer Responses

Session Progress

Responses Correct: 0

Responses Incorrect: 25

Responses Total: 25

Responses - % Correct: 0%

Blog
About Pastest
Contact Us
Help

© Pastest 2016

https://mypastest.pastest.com/Secure/TestMe/Browser/436619[‫ ص‬10:53:25 10/12/1437]


MyPastest

Prefer to use the old MyPastest? Access it here »

Back to Filters

Question 14 of 120

A mother brings her 4-and-a-half-month-old boy to your clinic. She complains that for the last week he has been
‘grizzly all the time’, ‘putting his thumbs and fingers in his mouth and will chew anything he’s given’. He was
previously sleeping for 7 h at night but now wakes twice for feeds. She has been applying a topical anti-
inflammatory to his gums. On examination there is no specific finding of note; the infant is afebrile
and generally
robust. What is the most appropriate advice to give?

A This is teething – there is no specific treatment

B This is likely to be a viral URTI

C This is a normal occurrence at this age, and in the absence of any abnormal findings reassurance
should be given

D The infant is clearly underfed and should be weaned

E The infant needs to be referred for developmental assessment

Explanation
The first tooth (normally lower incisor) usually erupts from 6 months onwards. There may be some symptoms of
teething before this but this would be unusual at 4-an-a-half months. Many illnesses are attributed to teething;
however, around this time there is
a decline in maternal circulating antibodies. If there is any evidence that the
infant is underfed (which should be picked up on examination) increased milk intake is advised because current
advice is that weaning shouldn’t start until 6 months. At around 4 months there is a recognised ‘stormy period’
associated with the development of new skills and physical growth.
22241

Next Question

https://mypastest.pastest.com/Secure/TestMe/Browser/436619[‫ ص‬10:53:47 10/12/1437]


MyPastest

Tag Question

Previous Question
Feedback

End Session
Difficulty: Average

Peer Responses

Session Progress

Responses Correct: 0

Responses Incorrect: 26

Responses Total: 26

Responses - % Correct: 0%

Blog
About Pastest
Contact Us
Help

© Pastest 2016

https://mypastest.pastest.com/Secure/TestMe/Browser/436619[‫ ص‬10:53:47 10/12/1437]


MyPastest

Prefer to use the old MyPastest? Access it here »

Back to Filters

Question 15 of 120

A 5-year-old girl has had a persistent nocturnal cough for over 2 years, which has not responded to treatment
including inhaled steroids and bronchodilators. The cough tends to be paroxysmal and is associated with vomiting,
disturbing her sleep virtually every night. She is happy and normally active during the day and does not have any
exercise intolerance. In other respects she has been well with no significant past history. There is no family history
of asthma, eczema or allergy. Her weight is on the 75th centile and height on the 50th
centile. General and
systematic examinations are normal. Chest radiograph is essential normal. FBC within the normal range, ESR 12
mm, CRP 5 mg, peak expiratory flow (PEF) 90% predicted, Mantoux negative.

The next most useful investigation that will aid the diagnosis is:

A Bronchoscopy

B Spirometry

C Barium meal

D Ambulatory oesophageal pH study

E CT of thorax

Explanation
This 5-year-old had a persistent cough that was characteristically nocturnal, accompanied by vomiting and with no
evidence of any diurnal respiratory symptoms. The cough is related to the supine posture, especially as there was
accompanying vomiting, which
was a constant feature. Therefore in the first instance, the least invasive and most
useful investigation is an ambulatory oesophageal pH study. A barium meal involves radiation, is not
physiological and is indicated only if the possibility of a hiatus hernia is considered. A bronchoscopy and CT may
be indicated if the initial investigations are negative.
22242

https://mypastest.pastest.com/Secure/TestMe/Browser/436619[‫ ص‬10:54:06 10/12/1437]


MyPastest

Previous Question
End Session
Tag Question

Feedback

Difficulty: Easy

Peer Responses

Session Progress

Responses Correct: 0

Responses Incorrect: 27

Responses Total: 27

Responses - % Correct: 0%

Blog
About Pastest
Contact Us
Help

© Pastest 2016

Next Question

https://mypastest.pastest.com/Secure/TestMe/Browser/436619[‫ ص‬10:54:06 10/12/1437]


MyPastest

Prefer to use the old MyPastest? Access it here »

Back to Filters

Question 16 of 120

A set of twins are brought to see you aged 5 days. They are being exclusively breast-fed. They are both jaundiced,
requiring admission for phototherapy, and have lost 12% and 13% birthweights, respectively. They both have
serum sodium levels of 145 mmol/L.

What is the best advice about fluid management over the next 48 h?

A Intravenous 10% dextrose at 150 mL/kg day

B Continue breast-feeding both of them exclusively

C Continue breast-feeding but give full top-ups via bottle

D Give them full requirements by bottle of ERB (expressed breast milk) and additional formula if
necessary

E Start formula 150 mL/kg per day by bottle

Explanation
The twins are dehydrated, having lost a significant proportion of their birthweight. Exclusively breast-feeding the
twins is obviously the ideal; however, this requires a lot of support, without which it may be unrealistic. Mothers
should be encouraged to feed their babies by breast and supplement with formula if
necessary. It is important that
mothers should not be made to feel guilty for supplementing. In the situation above, there is most probably
an
insufficient intake. The best way of rehydrating most children is enterally – intravenous fluids would not be
necessary in this situation. Bottle feeding, either expressed breast milk or formula, would be reasonable.
22245

Next Question

https://mypastest.pastest.com/Secure/TestMe/Browser/436619[‫ ص‬10:54:26 10/12/1437]


MyPastest

Tag Question

Previous Question
Feedback

End Session
Difficulty: Average

Peer Responses

Session Progress

Responses Correct: 0

Responses Incorrect: 28

Responses Total: 28

Responses - % Correct: 0%

Blog
About Pastest
Contact Us
Help

© Pastest 2016

https://mypastest.pastest.com/Secure/TestMe/Browser/436619[‫ ص‬10:54:26 10/12/1437]


MyPastest

Prefer to use the old MyPastest? Access it here »

Back to Filters

Question 17 of 120

Theme: Immediate interventions

A Adenosine 50 µg/kg i.v.


B Amiodarone 5 mg/kg i.v.
C Asynchronous DC shock – 0.5 J/kg
D Asynchronous DC shock – 2 J/kg
E Asynchronous DC shock – 4 J/kg
F Atropine 20 µg/kg i.v.
G Lidocaine 1 mg/kg i.v.
H Sodium bicarbonate 4.2% 4 mL/kg i.v.
I Synchronous DC shock – 1J/kg
J Vagal manoeuvres

For
the cases below, choose the most appropriate immediate intervention (airway and breathing can be assumed
to be already managed) from the above list. Each item may be used once, more than once or not at all.

Scenario 1

A 3-year-old has the following trace: SVT (supraventricular tachycardia). Her capillary refill time (CRT) is 2 secs
and her her O2 saturations (SaO2) are 95% in 5 L O2 by mask.

Your answer was incorrect

Select one...

J - Vagal manoeuvres

Any child in SVT should have vagal manoeuvres


tried first as they are quick and easy (and can be effective)
particularly if there is no evidence of shock. The strategies that can be used are: an older child can try a Valsalva
manoeuvre by blowing up a
balloon. Infants can have their face immersed in cold water to try to elicit a diving
reflex. Unilateral carotid massage can also be tried.

Scenario 2

A 15-year-old girl with a history of deliberate self-harm presents acutely unwell. ECG monitoring shows the
following trace – VT (ventricular tachycardia). Her pulse is weak but present, CRT is 5 s, SaO2 90% in 10 L O2 by

https://mypastest.pastest.com/Secure/TestMe/Browser/436619[‫ ص‬10:55:11 10/12/1437]


MyPastest

mask.

Your answer was incorrect


Previous Question
Select one... End Session

I - Synchronous DC shock – 1 J/kg

The low saturations and prolonged capillary refill indicate that the child is in shock. A shocked child in VT but
with a pulse should undergo synchronous DC cardioversion at an initial power of 1 J/kg (performed after rapid
sequence induction anaesthesia) and following shocks at 2J/kg. The underlying diagnosis is likely to be a
tricyclic
antidepressant overdose.

Scenario 3

A 2-year-old child is brought in who was found unresponsive by her parents. She is being ventilated by bag–
valve–mask by the paramedics. She is pulseless and has this rhythm on the monitor: VF (ventricular fibrillation).

Your answer was incorrect

Select one...

E - Asynchronous DC shock – 4 J/kg

Treatment of VF/pulseless VT is with asynchronous DC shock of 4 J/kg (followed by a further 4 J/kg, then 4
J/kg). Cardiopulmonary resuscitation (CPR) must continue when the shocks
are not being given, and it is
important to give epinephrine (adrenaline) – but only DC shock will convert the rhythm to sinus.
These are the
Resuscitation council UK guidelines.

Check the website for latest details: www.resus.org.uk


22250

Tag Question

Feedback

Difficulty: Average

Session Progress

Responses Correct: 0

Responses Incorrect: 31

Responses Total: Next Question 31

https://mypastest.pastest.com/Secure/TestMe/Browser/436619[‫ ص‬10:55:11 10/12/1437]


MyPastest

Responses - % Correct: 0%

Blog
About Pastest
Contact Us
Help

© Pastest 2016

https://mypastest.pastest.com/Secure/TestMe/Browser/436619[‫ ص‬10:55:11 10/12/1437]


MyPastest

Prefer to use the old MyPastest? Access it here »

Back to Filters

Question 18 of 120

Theme: Neoplasms

A Acute myeloblastic leukaemia (AML)


B Acute lymphoid leukaemia (ALL)
C Atrial myxoma
D Ewing’s sarcoma
E Medulloblastoma
F Neuroblastoma
G Non-Hodgkin’s lymphoma
H Osteosarcoma
I Rhabdomyosarcoma
J Wilms’ tumour

For
each of the following cases, choose the single most likely diagnosis from the above list. Each item may be
used once, more than once or not at all.

Scenario 1

A 2-year-old presents with a 1-month history of lethargy, not being himself and bruising easily. He has
hepatosplenomegaly and a full blood count (FBC) reveals: Hb 9.2 g/dL, white cell count (WCC) 2.3 × 109/L,
platelets (plts) 50 × 109/L.

Your answer was incorrect

Select one...

B - Acute lymphoid leukaemia (ALL)

ALL is the most common malignancy in childhood (35–40% of all childhood malignancies). The incidence is
highest in early childhood. Diagnosis is made on bone marrow aspiration; however, blast cells can often be seen in
the peripheral blood. The treatment generally lasts for around 2 years. Survival rates are as high as 85% at 5 years.

Scenario 2

A 3-year-old boy has a large left flank mass and a trace of blood in the urine.

https://mypastest.pastest.com/Secure/TestMe/Browser/436619[‫ ص‬10:55:56 10/12/1437]


MyPastest

Your answer was incorrect

Select one... Previous Question


End Session
J - Wilms’ tumour

There are only two likely diagnoses in


this case: Wilms’ nephroblastoma or neuroblastoma. As there is blood in
the urine then there is probably renal involvement and therefore a Wilms’ nephroblastoma is the more likely of the
two.

Both Wilms’ nephroblastomas and neuroblastomas count for 8% each of all childhood malignancies.

Scenario 3

A 9-year-old boy presents with lethargy, shortness of breath and night sweats. On examination splenomegaly can
be
felt. There are no blast cells present on the blood film.

Your answer was incorrect

Select one...

G - Non-Hodgkin’s lymphoma

This is a typical history for non-Hodgkin’s lymphoma. The shortness of breath is caused by mediastinal
involvement (which can ultimately cause superior vena cava (SVC) obstruction). The staging and treatment of
non-Hodgkin’s lymphoma are essentially the same as in adults. Lymphomas count for 15% of all childhood
malignancies and are the second most common solid tumours after the central nervous system (CNS) tumours.
The other childhood malignancies are bone tumours (7%), retinoblastoma (2%), and others such as
rhabdomyosarcoma or adenocarcinoma.
22255

Tag Question

Feedback

Difficulty: Average

Session Progress

Responses Correct: 0

Responses Incorrect: 34

Responses Total: 34
Next Question

https://mypastest.pastest.com/Secure/TestMe/Browser/436619[‫ ص‬10:55:56 10/12/1437]


MyPastest

Responses - % Correct: 0%

Blog
About Pastest
Contact Us
Help

© Pastest 2016

https://mypastest.pastest.com/Secure/TestMe/Browser/436619[‫ ص‬10:55:56 10/12/1437]


MyPastest

Prefer to use the old MyPastest? Access it here »


Previous Question
End Session

Back to Filters

Question 19 of 120

A 3-year-old, fully immunised child presents with fever and difficulty in breathing. She has had tonsillitis over the
past
week. On examination she looks unwell, has mild recession, and a soft stridor is audible. What is the most
likely diagnosis?

A Retropharyngeal abscess

B Bacterial tracheitis

C Epiglottitis

D Severe croup

E Fulminant pneumonia

Explanation
Stridor indicates an upper airway problem. Croup is less likely in an unwell child, especially if there has been a
preceding tonsillitis. Epiglottitis would be surprising in a child immunised with Hib, but it is recognised. Bacterial
tracheitis is indeed
a possibility but, with a preceding tonsillitis a retropharyngeal abscess is more likely.
22347

Tag Question

Feedback

Next Question
Difficulty: Average

https://mypastest.pastest.com/Secure/TestMe/Browser/436619[‫ ص‬10:56:16 10/12/1437]


MyPastest

Peer Responses

Session Progress

Responses Correct: 0

Responses Incorrect: 35

Responses Total: 35

Responses - % Correct: 0%

Blog
About Pastest
Contact Us
Help

© Pastest 2016

https://mypastest.pastest.com/Secure/TestMe/Browser/436619[‫ ص‬10:56:16 10/12/1437]


MyPastest

Prefer to use the old MyPastest? Access it here »

Back to Filters

Question 20 of 120

A 13-year-old girl attends her GP surgery wanting to be prescribed the oral contraceptive pill (OCP). She is
sexually active but does not want her mother to know (despite reasoning). Which of the following would be the
most appropriate course of action?

A Refusal to prescribe as she is under the age of informed consent

B Prescribe her the OCP because it would be in her best interests

C Prescribe her the OCP if she meets the Fraser guidelines

D Prescribe her the OCP on the condition that she gets parental consent

E Refuse to prescribe because this would condone under-age sexual intercourse, which is illegal

Explanation
The House of Lords have provided the means for determination of capacity of an under 16-year-old to give
medical consent independent of their parent/s. This was set out in the ruling on
the Gillick case (Gillick v West
Norfolk & Wisbech AHA [1985] 3 All
ER 402, HL). Gillick competence is important to understand and emerging
from this judgement Lord Fraser held that a girl under 16 years of age could be prescribed the oral contraceptive
pill (OCP) if the following criteria could be met:

1. The girl can understand, retain and make informed decisions based on the information given to her.
2. That, despite discussion, she will definitely not involve her parents.
3. If she is going to have sexual intercourse whether or not the OCP is prescribed.
4. If the OCP is NOT prescribed, it will be detrimental to her health.
5. If it is in her best interests.

There is now a move away from calling these criteria ‘Gillick Competence’ to ‘Fraser Competent’ 22348

https://mypastest.pastest.com/Secure/TestMe/Browser/436619[‫ ص‬10:56:35 10/12/1437]


MyPastest

Previous Question
End Session
Tag Question

Feedback

Difficulty: Easy

Peer Responses

Session Progress

Responses Correct: 0

Responses Incorrect: 36

Responses Total: 36

Responses - % Correct: 0%

Blog
About Pastest
Contact Us
Help

© Pastest 2016

Next Question

https://mypastest.pastest.com/Secure/TestMe/Browser/436619[‫ ص‬10:56:35 10/12/1437]


MyPastest

Prefer to use the old MyPastest? Access it here »


Previous Question

Back to Filters

Question 21 of 120

A 18-month-old infant who has an upper respiratory tract infection (URTI) presents with a convulsion. Which of
the following features best fits with a typical febrile convulsion?

A One minute of left arm shaking followed by 6 min of tonic–clonic movement of all four limbs

B Eyes rolling back followed by a 3-min generalised tonic–clonic seizure; drowsy afterwards for 1 h

C Going vacant and unresponsive for 2 min; back to normal self afterwards

D A 25-min generalised tonic–clonic seizure terminated with intravenous lorazepam

E A 5-min generalised tonic–clonic seizure; drowsy afterwards then a further 15-min generalised tonic–
clonic seizure

Explanation
Typical features of a febrile convulsion are: age
6 months to 6 years and febrile (or rising temperature) at time of
convulsion. Generalised tonic–clonic seizures last up to 10 min with a post-ictal phase afterwards. The eyes rolling
back and incontinence are often seen. Focal features of secondarily generalised seizures are sometimes seen in
febrile convulsions but are not typical. A
rapid recovery is uncharacteristic of a generalised tonic–clonic seizure –
whether febrile or afebrile. The duration of the seizures in D and E mean that these are not typical. Prolonged
seizures (25 min) and repeated seizures are not typical febrile seizures.
22349

Tag Question

Feedback
Next Question

https://mypastest.pastest.com/Secure/TestMe/Browser/436619[‫ ص‬10:56:54 10/12/1437]


MyPastest

Difficulty: Easy
End Session
Peer Responses

Session Progress

Responses Correct: 0

Responses Incorrect: 37

Responses Total: 37

Responses - % Correct: 0%

Blog
About Pastest
Contact Us
Help

© Pastest 2016

https://mypastest.pastest.com/Secure/TestMe/Browser/436619[‫ ص‬10:56:54 10/12/1437]


MyPastest

Prefer to use the old MyPastest? Access it here »


Previous Question
End Session

Back to Filters

Question 22 of 120

Which of the following is the best method of vascular access in a


5-year-old child brought into the Emergency
Department resuscitation in
asystole?

A Peripheral venous cannulation

B Femoral venous central line insertion

C Long saphenous venous cannulation

D External jugular central line insertion

E Tibial intraosseous needle insertion

Explanation
APLS (Advanced Paediatric Life Support) recommendations indicate that tibial interosseus needle insertion is a
quick and easy method of securing venous access in an arrested child. The other methods are satisfactory but in a
collapsed child, may be difficult and time-consuming to insert. 22355

Tag Question

Feedback

Difficulty: Easy

Next Question
Peer Responses

https://mypastest.pastest.com/Secure/TestMe/Browser/436619[‫ ص‬10:57:13 10/12/1437]


MyPastest

Session Progress

Responses Correct: 0

Responses Incorrect: 38

Responses Total: 38

Responses - % Correct: 0%

Blog
About Pastest
Contact Us
Help

© Pastest 2016

https://mypastest.pastest.com/Secure/TestMe/Browser/436619[‫ ص‬10:57:13 10/12/1437]


MyPastest

Prefer to use the old MyPastest? Access it here »


Previous Question
End Session

Back to Filters

Question 23 of 120

A 6-year-old complains that his foreskin balloons when he passes urine. On examination you note non-retractile
foreskin with some preputial adhesions. What is the best course of action?

A Advise gentle retraction of the foreskin in the bath

B Advise that it can be normal at this age and should be left alone

C Advise applying 1% hydrocortisone cream twice daily for 1 week

D Refer for circumcision

E Send a preputial skin swab for microscopy, culture and sensitivity

Explanation
The only medical indication for circumcision is balanitis xeroderma obliterans (BXO). A non-retractile foreskin is
not uncommon until puberty. The adhesions may be released with some weak steroid cream. Ballooning of the
foreskin is not a problem in itself.
22357

Tag Question

Feedback

Difficulty: Difficult
Next Question

https://mypastest.pastest.com/Secure/TestMe/Browser/436619[‫ ص‬10:57:33 10/12/1437]


MyPastest

Peer Responses

Session Progress

Responses Correct: 0

Responses Incorrect: 39

Responses Total: 39

Responses - % Correct: 0%

Blog
About Pastest
Contact Us
Help

© Pastest 2016

https://mypastest.pastest.com/Secure/TestMe/Browser/436619[‫ ص‬10:57:33 10/12/1437]


MyPastest

Prefer to use the old MyPastest? Access it here »


Previous Question

Back to Filters

Question 24 of 120

A 9-year-old child has a large, swollen, fluctuant left submandibular lymph node. You notice that there are gross
caries in
most of her teeth. There is a buccal swelling adjacent to her lower left ‘E’ (primary molar). What is the
best course of action?

A Course of oral amoxicillin and metronidazole and referral to her general dental practitioner (GDP)

B Admission for intravenous amoxicillin and metronidazole

C Referral to maxillofacial surgery for incision and drainage (I&D)

D Trial of oral antibiotics and surgical referral if no improvement after 5 days

E Course for 5 days of intravenous ceftriaxone administered at home by community nurses, with review
after the course

Explanation
This patient has an abscess that has developed secondary to caries in her lower left ‘E’. She has lymphadenitis
secondary to this and, as the swelling is fluctuant, there
is probably an abscess there. If there is a large (> 1 cm)
abscess present, then treatment is incision and drainage with postoperative antibiotics. Antibiotics as a first line
may reduce the infection, but will not treat it completely if the abscess is large. In the case of a small
periodental/dental abscess, a course of oral antibiotics may treat
the abscess, but the patient should be referred to a
GDP for review and
follow-up.
22364

Next Question Tag Question

https://mypastest.pastest.com/Secure/TestMe/Browser/436619[‫ ص‬10:57:51 10/12/1437]


MyPastest

Feedback

Difficulty: Difficult
End Session
Peer Responses

Session Progress

Responses Correct: 0

Responses Incorrect: 40

Responses Total: 40

Responses - % Correct: 0%

Blog
About Pastest
Contact Us
Help

© Pastest 2016

https://mypastest.pastest.com/Secure/TestMe/Browser/436619[‫ ص‬10:57:51 10/12/1437]


MyPastest

Prefer to use the old MyPastest? Access it here »


Previous Question
End Session

Back to Filters

Question 25 of 120

A baby boy is noted to have hypospadias at the postnatal check. The meatus is situated ventrally, and a chordee is
noted. The most important piece of advice to give to the parents is:

A His adult sexual function should be normal

B You must be careful when washing his penis

C He must not be circumcised

D He may require surgery in later childhood

E Treat your baby as normal

Explanation
While all of the pieces of advice are valid, it is most important to advise that circumcision should not take place.
Many parents are keen for circumcision for ethnic or religious reasons; however, circumcision can make a surgical
hypospadias repair much more difficult.
22367

Tag Question

Feedback

Difficulty: Average
Next Question

https://mypastest.pastest.com/Secure/TestMe/Browser/436619[‫ ص‬10:59:41 10/12/1437]


MyPastest

Peer Responses

Session Progress

Responses Correct: 0

Responses Incorrect: 41

Responses Total: 41

Responses - % Correct: 0%

Blog
About Pastest
Contact Us
Help

© Pastest 2016

https://mypastest.pastest.com/Secure/TestMe/Browser/436619[‫ ص‬10:59:41 10/12/1437]


MyPastest

Prefer to use the old MyPastest? Access it here »


Previous Question
End Session

Back to Filters

Question 26 of 120

A 15-year-old girl presents with a 2-month history of


abdominal pain. She has had no fever and no other
symptoms. Examination
reveals 3 cm hepatomegaly and splenomegaly. The single most useful investigation
would be:

A CRP

B Blood film

C Abdominal ultrasonography

D Paul Bunnell test

E Liver function tests (LFTs) + amylase

Explanation
Some of these investigations are more useful than
others. Abdominal ultrasonography will detect the enlargement
of abdominal/para-aortic lymph nodes but not help in establishing a diagnosis. A Paul Bunnell test, if positive, will
establish a diagnosis,
but this is unlikely given the history. CRP/LFTs and amylase may indicate an inflammatory
process but again will not help establish a diagnosis. A blood film would be the most useful given the history
(leukaemia, haematopoietic disorders).
22368

Tag Question

Feedback
Next Question

https://mypastest.pastest.com/Secure/TestMe/Browser/436619[‫ ص‬11:00:02 10/12/1437]


MyPastest

Difficulty: Average

Peer Responses

Session Progress

Responses Correct: 0

Responses Incorrect: 42

Responses Total: 42

Responses - % Correct: 0%

Blog
About Pastest
Contact Us
Help

© Pastest 2016

https://mypastest.pastest.com/Secure/TestMe/Browser/436619[‫ ص‬11:00:02 10/12/1437]


MyPastest

Prefer to use the old MyPastest? Access it here »


Previous Question

Back to Filters

Question 27 of 120

An 8-month-old infant presents to the Emergency Department with a 3-day history of frequent watery diarrhoea
and vomiting. On examination she is 5% dehydrated. She is refusing to drink in the Emergency Department.
Which of the following is the most appropriate course of action?

A Admit for intravenous rehydration (maintenance and 5% deficit)

B Observe
for 4 h in the Emergency Department; if she tolerates oral fluids, discharge her; if not admit
for intravenous rehydration

C Admit for enteral rehydration via a nasogastric tube

D For intravenous rehydration (maintenance) + oral fluids as tolerated

E Short course of oral cefalexin

Explanation
Enteral rehydration using an oral rehydration solution is almost invariably the preferred way of rehydrating
children.
If a child is not tolerating small frequent feeds then nasogastric rehydration is an underused next best
step. The fluid can be run through
a continuous pump so that it is better tolerated. Intravenous fluids are effective
but can have profound effects on the serum electrolyte balance if not monitored closely. Most children will
tolerate fluids in an emergency department, but failure to take fluids orally is not an indication for intravenous
therapy. 22375

Tag Question
Next Question

https://mypastest.pastest.com/Secure/TestMe/Browser/436619[‫ ص‬11:00:22 10/12/1437]


MyPastest

Feedback

Difficulty: Difficult
End Session
Peer Responses

Session Progress

Responses Correct: 0

Responses Incorrect: 43

Responses Total: 43

Responses - % Correct: 0%

Blog
About Pastest
Contact Us
Help

© Pastest 2016

https://mypastest.pastest.com/Secure/TestMe/Browser/436619[‫ ص‬11:00:22 10/12/1437]


MyPastest

Prefer to use the old MyPastest? Access it here »


Previous Question
End Session

Back to Filters

Question 28 of 120

You see a mother who is hepatitis B surface antibody (HBsAb) +ve, and core antigen (HBeAg) +ve.

Which of the following combinations is the most appropriate for her newborn baby?

A Hepatitis B vaccine

B Hepatitis B immunoglobulin

C Hepatitis B vaccine and hepatitis B immunoglobulin

D Hepatitis B vaccine and intravenous pooled immunoglobulin

E Hepatitis B and hepatitis C vaccines combined

Explanation
HBsAb +ve, HBeAg +ve is classified as high risk of infectivity to the infant. The babies in such cases should
receive both the hepatitis B vaccine and hepatitis B immunoglobulin within the first 24 h of life. Low-risk infants
(HBsAb +ve, anti-HBeAb+-ve) should receive hepatitis vaccine only.
22376

Tag Question

Feedback

Difficulty: Easy
Next Question

https://mypastest.pastest.com/Secure/TestMe/Browser/436619[‫ ص‬11:00:41 10/12/1437]


MyPastest

Peer Responses

Session Progress

Responses Correct: 0

Responses Incorrect: 44

Responses Total: 44

Responses - % Correct: 0%

Blog
About Pastest
Contact Us
Help

© Pastest 2016

https://mypastest.pastest.com/Secure/TestMe/Browser/436619[‫ ص‬11:00:41 10/12/1437]


MyPastest

Prefer to use the old MyPastest? Access it here »


Previous Question
End Session

Back to Filters

Question 29 of 120

You are seeing a 5-year-old with primary nocturnal enuresis.

Which of the following is the most appropriate initial management strategy?

A Family counselling

B Star chart

C Verbal chastisement

D Short trial of imipramine

E Short trial of desmopressin

Explanation
A star chart for dry nights is a useful tool for managing nocturnal enuresis. Simple measures such as minimising
fluid intake before bed and ensuring the child goes to the toilet before going to bed are important. Medical
interventions are not a good first-line measure; desmopressin is effective but there is a high relapse rate off
treatment. Imipramine is infrequently used now because there are more effective treatments and it has serious
side-effects. Enuresis clinics provide good support but the majority do not accept referrals for children under 7
years old.
22378

Tag Question

Feedback Next Question

https://mypastest.pastest.com/Secure/TestMe/Browser/436619[‫ ص‬11:01:00 10/12/1437]


MyPastest

Difficulty: Easy

Peer Responses

Session Progress

Responses Correct: 0

Responses Incorrect: 45

Responses Total: 45

Responses - % Correct: 0%

Blog
About Pastest
Contact Us
Help

© Pastest 2016

https://mypastest.pastest.com/Secure/TestMe/Browser/436619[‫ ص‬11:01:00 10/12/1437]


MyPastest

Prefer to use the old MyPastest? Access it here »


Previous Question
End Session

Back to Filters

Question 30 of 120

A 4-year-old girl suffers a femoral fracture in a road traffic accident. She also has a contusion on her right
forehead. She is
screaming with the pain from her leg.

Which of the following would be the most effective and safest forms of analgesia?

A Per rectum (p.r.) diclofenac and oral codeine

B Splinting of the fractured limb

C Paracetamol, ibuprofen and codeine per os (p.o.)

D Morphine intravenously

E Femoral nerve block

Explanation
Femoral nerve block is a safe and very effective method of pain relief for limb injuries. The other options are, of
course, useful but probably not sufficiently strong for this severity of
injury. Splinting is vital but analgesia should
be given first. Intravenous morphine should be used with caution if there is the possibility of a significant head
injury. (Advanced Paediatric Life Support, 3rd edn). 22381

Tag Question

Feedback

Next Question

https://mypastest.pastest.com/Secure/TestMe/Browser/436619[‫ ص‬11:01:19 10/12/1437]


MyPastest

Difficulty: Difficult

Peer Responses

Session Progress

Responses Correct: 0

Responses Incorrect: 46

Responses Total: 46

Responses - % Correct: 0%

Blog
About Pastest
Contact Us
Help

© Pastest 2016

https://mypastest.pastest.com/Secure/TestMe/Browser/436619[‫ ص‬11:01:19 10/12/1437]


MyPastest

Prefer to use the old MyPastest? Access it here »

Back to Filters

Question 31 of 120

Which of the following diagnoses is the most likely in a 3-year-old boy who has epilepsy and, on examination, has
numerous depigmented macules and two café-au-lait spots?

A Tuberous sclerosis

B Neurofibromatosis 1

C Ataxia telangiectasia

D Incontinentia pigmenti

E Sturge–Weber syndrome

Explanation
Tuberous sclerosis – autosomal dominant, 1 in 6,000:

‘Ash leaf’ macules (from infancy): depigmented lesions approximately 1–2 cm long
‘Shagreen’ patches (from 2 years): areas of roughened skin, usually sacral, likened to shark skin
Adenoma sebaceum (from 5 years): 1- to 2-mm papules, usually facial (butterfly distribution)
Epilepsy (usually before 2 years)

Neurofibromatosis type 1 – autosomal dominant, 1 in 3,500:

Café-au-lait spots (> 2 in children under 5 years, > 5 in children over 5 years is significant)
Axillary freckling
Neurofibromata (from 12 years): papules anywhere on the body
Epilepsy only in 10%

https://mypastest.pastest.com/Secure/TestMe/Browser/436619[‫ ص‬11:01:40 10/12/1437]


MyPastest

Ataxia
telangiectasia – autosomal recessive, a chromosomal repair defect. Affected children present as late
walkers. Ataxia Previous Question
develops in early childhood and is progressive:

Conjunctival telangiectasia: develops from 5 years End Session


Incontinentia pigmenti: X-linked dominant.
Vesicular stage: neonatal period, linear distribution; resolves by 1 month
Verrucose stage: 1–4 months, warty lesions appearing mainly on limbs; resolves by 6 months
Whorl stage: by 2 years, linear and whorl pattern of hyperpigmentation on limbs
Epilepsy in over 30%

Sturge–Weber syndrome – sporadic, 1 in 50,000:

Naevus in trigeminal distribution with an ipsilateral leptomeningeal haemangioma


Intracranial calcification is common, especially in the occipital region
Seizures develop in early childhood

22382

Tag Question

Feedback

Difficulty: Average

Peer Responses

Session Progress

Responses Correct: 0

Responses Incorrect:
Next Question 47

https://mypastest.pastest.com/Secure/TestMe/Browser/436619[‫ ص‬11:01:40 10/12/1437]


MyPastest

Responses Total: 47

Responses - % Correct: 0%

Blog
About Pastest
Contact Us
Help

© Pastest 2016

https://mypastest.pastest.com/Secure/TestMe/Browser/436619[‫ ص‬11:01:40 10/12/1437]


MyPastest

Prefer to use the old MyPastest? Access it here »


Previous Question
End Session

Back to Filters

Question 32 of 120

A 15-year-old girl is referred after a deliberate paracetamol overdose. She reports having taken five pills 45
minutes ago. She has paracetamol levels well below the treatment line at 4 h. She says she only took the pills as a
‘cry for help’ and regrets doing it. What is the most appropriate course of action?

A Discharge her with an urgent child psychiatry outpatient appointment

B Discharge her and contact the school educational psychologist

C Admit her for formal assessment by a child psychiatrist

D Admit her for observation overnight and discharge her if well the next day with a child psychiatry
outpatient appointment

E Discharge her for GP follow-up

Explanation
Any child who has taken a deliberate overdose, even if no medical treatment is required, must be seen by a child
psychiatrist. It is considered best practice not to discharge until assessed safe to do so by a child and adolescent
psychiatrist. This means that most will be admitted until assessed, although some may be seen in the emergency
department depending on the level of psychiatric service provided.
22387

Tag Question

Feedback
Next Question

https://mypastest.pastest.com/Secure/TestMe/Browser/436619[‫ ص‬11:02:01 10/12/1437]


MyPastest

Difficulty: Average

Peer Responses

Session Progress

Responses Correct: 0

Responses Incorrect: 48

Responses Total: 48

Responses - % Correct: 0%

Blog
About Pastest
Contact Us
Help

© Pastest 2016

https://mypastest.pastest.com/Secure/TestMe/Browser/436619[‫ ص‬11:02:01 10/12/1437]


MyPastest

Prefer to use the old MyPastest? Access it here »

Back to Filters

Question 33 of 120

A 12-year-old boy complains of excessive tiredness and fatigue at any time of day. He has missed a lot of school
over the past 6
months and says he can’t concentrate. He complains of headaches, aching limbs and variable
appetite. He stays at home most of the time and his mother has had to give up work to look after him. On
examination
he seems quiet but not clinically depressed. A detailed general and neurological examination reveals
no abnormalities. His FBC, CRP, LFTs, U&Es, T4 (thyroxine) and urinanalysis are normal.

The most relevant management is:

A Arrange regular physiotherapy

B Prescribe supplemental iron and vitamins

C Encourage bed rest every morning

D Prescribe fluoxetine

E Organise home tuition

Explanation
The history and examination are compatible with chronic fatigue syndrome (CFS), also known generally as
myalgic encephalomyelitis (ME) or postviral fatigue syndrome. The only treatment
for which there is some
evidence base is graded mobilisation and physiotherapy. Increased rest is of no proven value and does not improve
mobilisation and exercise tolerance. Home tuition may be advised, but the primary aim is gradual reintegration
into school. Psychological input may be useful but is often resisted by parents who perceive this as primarily a
medical/organic problem. Antidepressants are mainly indicated for an associated clinical depression. 22392

Next Question

https://mypastest.pastest.com/Secure/TestMe/Browser/436619[‫ ص‬11:02:20 10/12/1437]


MyPastest

Tag Question

Previous Question
Feedback

End Session
Difficulty: Average

Peer Responses

Session Progress

Responses Correct: 0

Responses Incorrect: 49

Responses Total: 49

Responses - % Correct: 0%

Blog
About Pastest
Contact Us
Help

© Pastest 2016

https://mypastest.pastest.com/Secure/TestMe/Browser/436619[‫ ص‬11:02:20 10/12/1437]


MyPastest

Prefer to use the old MyPastest? Access it here »


Previous Question
End Session

Back to Filters

Question 34 of 120

Which of the following is the best initial management in a 5-year-old child who has only recently developed
constipation following a
viral illness?

A Dietary advice

B Seven-day trial of Movicol

C One glycerin suppository

D Two stat phosphate enemas

E Admission for administration of Klean-Prep

Explanation
Dietary advice is always an integral part of the management of constipation. A diet with adequate fibre, fruit and
vegetables, and fluid intake should be encouraged. Sometimes Movicol (now first line drug treatment for chronic
constipation as per NICE guidelines) will help soften the stools to allow a normal bowel habit to
be re-established.
Senna would also achieve this but can cause stomach cramps and less compliance. 22400

Tag Question

Feedback

Difficulty: Average
Next Question

https://mypastest.pastest.com/Secure/TestMe/Browser/436619[‫ ص‬11:02:39 10/12/1437]


MyPastest

Peer Responses

Session Progress

Responses Correct: 0

Responses Incorrect: 50

Responses Total: 50

Responses - % Correct: 0%

Blog
About Pastest
Contact Us
Help

© Pastest 2016

https://mypastest.pastest.com/Secure/TestMe/Browser/436619[‫ ص‬11:02:39 10/12/1437]


MyPastest

Prefer to use the old MyPastest? Access it here »

Back to Filters

Question 35 of 120

Theme: Special investigations

A Angiography

B Bone scan

C CT

D Echocardiogram

E Electrocardiogram

F Electroencephalogram

G Magnetic resonance imaging (MRI)

H Plain X-ray

I Positron emission tomography

J Ultrasound

For
each of the following clinical problems, please choose the most useful investigation from the list above. Each
item may be used once, more than
once or not at all.

Scenario 1

A 2-year-old boy is diagnosed and treated for Kawasaki disease. You are concerned about possible complications
and wish to investigate this.

Your answer was incorrect

Select one...

D - Echocardiogram

The single most important late complication of Kawasaki disease is development of coronary artery aneurysms.
An echocardiogram, in skilled hands, is an easy, simple and effective way of detecting aneurysms. An angiogram
is sometimes performed in tertiary paediatric cardiac centres, to further demonstrate
aneurysms seen on the
echocardiogram.

https://mypastest.pastest.com/Secure/TestMe/Browser/436619[‫ ص‬11:03:00 10/12/1437]


MyPastest

Scenario 2

Previous
A 1-year-old infant falls fromQuestion
a chair and
hits her head on the concrete floor. She has a brief (1 minute) seizure
4
hours later
End Session
Your answer was incorrect

Select one...

C - CT

Any seizure following a head injury is


an indication for further imaging. A skull X-ray may reveal a skull fracture,
but given the fact that there has been a seizure brain imaging
is required. CT scans of the brain are quick to do and
readily available and are the investigation of choice in the NICE guidelines for
the management of a head injury.

Scenario 3

A 4-year-old child is febrile and is limping, complaining of pain in her left leg. There is some cellulitis overlying
her distal left tibia.

Your answer was incorrect

Select one...

B - Bone scan

The concern in this child is that there may be an underlying osteomyelitis. A plain film may reveal some periosteal
reaction, but only if the infection has been present for over
1 week. A bone scan is the investigation of choice. An
osteomyelitis will show as a ‘hot spot’ at the point where the infection is. MRI is now generally used to look for
periosteal and intramedullary oedema.
22415

Tag Question

Feedback

Difficulty: Average

Session Progress

Responses Correct: 0

Responses Incorrect: 53
Next Question
Responses Total: 53

https://mypastest.pastest.com/Secure/TestMe/Browser/436619[‫ ص‬11:03:00 10/12/1437]


MyPastest

Responses - % Correct: 0%

Blog
About Pastest
Contact Us
Help

© Pastest 2016

https://mypastest.pastest.com/Secure/TestMe/Browser/436619[‫ ص‬11:03:00 10/12/1437]


MyPastest

Prefer to use the old MyPastest? Access it here »

Back to Filters

Question 36 of 120

Theme: Decisions about life-saving treatment

A The brain-dead child

B The ‘no chance’ situation

C The ‘no purpose’ situation

D The persistent vegetative state

E The ‘unbearable’ situation

For
each of the following cases, choose the most appropriate criterion under which life-saving treatment could be
withdrawn (as identified by the Royal College of Paediatrics and Child Health [RCPCH]) from the above list.
Each item may be used once, more than once or not at all.

Scenario 1

A 4-year-old girl who, despite maximal intensive care, is deteriorating as a result of meningococcal septicaemia.

Your answer was incorrect

Select one...

B - The ‘no chance’ situation

Scenario 2

A 5-year-old boy who has relapsed for the third time with acute myeloblastic leukaemia and doesn’t wish for
further chemotherapy.

Your answer was incorrect

Select one...

E - The ‘unbearable’ situation

Scenario 3

https://mypastest.pastest.com/Secure/TestMe/Browser/436619[‫ ص‬11:03:20 10/12/1437]


MyPastest

A 1-year-old, who having sustained such severe head injuries in a road traffic accident that he is expected to be
profoundly brain damaged, develops a pneumonia while in the paediatric intensive care unit (PICU).
Previous Question
Your answer was incorrect
End Session
Select one...

C - The ‘no purpose’ situation

The RCPCH issued guidelines in 1997 on


when it could be considered appropriate to ‘withdraw or withhold
lifesaving treatment’. The five situations are as follows:
1. The brain-dead child: the formal criteria for brain death are met.
2. PVS: the formal criteria for PVS are met.
3. ‘No
chance’ situation: despite all treatments, there is no chance of the child surviving, so it could be
considered that withdrawal of care is appropriate.
4. ‘No purpose’ situation: a child may
survive given the necessary interventions but will be so impaired that he
or she will probably not achieve ‘personhood’ or be able to make decisions about his or her own life.
5. ‘Unbearable’
situation: the treatment is so invasive and has so many side effects that it may be more than
the child can bear (the treatment is worse than
the disease).

It is important to note that withdrawing and


withholding treatment from children should be done as part of a multi-
disciplinary process with full parental involvement.
22457

Tag Question

Feedback

Difficulty: Difficult

Session Progress

Responses Correct: 0

Responses Incorrect: 56

Responses Total: 56

Responses - % Correct: 0%

Blog
About Pastest
Contact Us
Help Next Question

https://mypastest.pastest.com/Secure/TestMe/Browser/436619[‫ ص‬11:03:20 10/12/1437]


MyPastest

© Pastest 2016

https://mypastest.pastest.com/Secure/TestMe/Browser/436619[‫ ص‬11:03:20 10/12/1437]


MyPastest

Prefer to use the old MyPastest? Access it here »

Back to Filters

Question 37 of 120

A 4-year-old child presents with headache and falls several weeks after a vaccination. On examination he is ataxic
and unable to walk unaided due to hemiparesis. A brain MRI shows demyelination.

What is the most likely cause?

A Acute disseminated encephalomyelitis

B Guillain-barré syndrome

C Glioma

D Meningitis

E Multiple sclerosis

Explanation
Acute Disseminated Encephalomyelitis (ADEM):

Epidemiology: Children>adults.

Aetiology/risk factors: 1 to 20 days post viral infection. Can also occur post vaccination (measles, mumps,
rubella).

Clinical features: Acute onset CNS inflammation, Ataxia, Fatigue, Headache, Nausea Phyramidal signs,
Weakness.

Complications:
Bowel and bladder involvement, Brown-Séquard syndrome, Coma, Deafness, Devics syndrome
(optic neuritis and transverse myelitis), Hemiparesis, Optic neuritis, Permanent disability, Seizure, Transverse
myelitis.
45249

https://mypastest.pastest.com/Secure/TestMe/Browser/436619[‫ ص‬11:03:40 10/12/1437]


MyPastest

Previous Question
End Session
Tag Question

Feedback

Difficulty: Average

Peer Responses

Session Progress

Responses Correct: 0

Responses Incorrect: 57

Responses Total: 57

Responses - % Correct: 0%

Blog
About Pastest
Contact Us
Help

© Pastest 2016

Next Question

https://mypastest.pastest.com/Secure/TestMe/Browser/436619[‫ ص‬11:03:40 10/12/1437]


MyPastest

Prefer to use the old MyPastest? Access it here »


Previous Question

Back to Filters

Question 38 of 120

A female baby was noted to have lymphoedema and absent femoral pulses.

What is the most likely diagnosis?

A Alport syndrome

B Duchenne muscular dystrophy

C Noonan syndrome

D Turner syndrome

E Wiskott-Aldrichsyndrome

Explanation
Turner syndrome affects about one in 3,000 female births.

Genetic 45 X,0 or mosaicism (may present late e.g. with primary amenorrhoea, infertility).

Clinical
features: Amenorrhoea, primitive streak ovaries, wide-spaced nipples, short fourth metacarpals, low
hairline, high-arched palate, otis media, lymphoedema, osteoporosis, horseshoe kidneys; 1 in 5 have cardiac
anomalies (commonly of the aorta), Epicanthic folds, ptosis, normal IQ, halo naevus.

Complications: DM, Hashimoto's thyroiditis, Hypertension, Infertility, Keratoconus


45251

Tag Question
Next Question

https://mypastest.pastest.com/Secure/TestMe/Browser/436619[‫ ص‬11:03:59 10/12/1437]


MyPastest

Feedback

Difficulty: Average
End Session
Peer Responses

Session Progress

Responses Correct: 0

Responses Incorrect: 58

Responses Total: 58

Responses - % Correct: 0%

Blog
About Pastest
Contact Us
Help

© Pastest 2016

https://mypastest.pastest.com/Secure/TestMe/Browser/436619[‫ ص‬11:03:59 10/12/1437]


MyPastest

Prefer to use the old MyPastest? Access it here »

Back to Filters

Question 39 of 120

An infant presents with ataxia, failure to thrive, steatorrhoea and retinopathy. His blood film reveals acanthocytes.

What is the most likely diagnosis?

A Abetalipoproteinaemia

B Ataxia telangiectasia

C Charcot-Marie-Tooth disease

D Noonan syndrome

E Refsum's disease

Explanation
Abetalipoproteinaemia is a rare autosomal recessive lipoprotein synthesis disorder.

Clinical features:

Ataxia
Failure to thrive
Reinopathy (retinitis pigmentosa)
Steatorrhoea

Investigations:

FBC (acanthyocytes)
raised ALT
lowered LDL

https://mypastest.pastest.com/Secure/TestMe/Browser/436619[‫ ص‬11:04:19 10/12/1437]


MyPastest

lowered VLDL
lowered Cholesterol
Previous Question
lowered Triglycerides.
End Session

45253

Tag Question

Feedback

Difficulty: Average

Peer Responses

Session Progress

Responses Correct: 0

Responses Incorrect: 59

Responses Total: 59

Responses - % Correct: 0%

Blog
About Pastest
Contact Us
Help

© Pastest 2016

Next Question

https://mypastest.pastest.com/Secure/TestMe/Browser/436619[‫ ص‬11:04:19 10/12/1437]


MyPastest

Prefer to use the old MyPastest? Access it here »

Back to Filters

Question 40 of 120

A child with an atrio-septal defect is noted to have a smooth philtrum and thin upper lip.

What is the most likely underlying cause?

A Down syndrome

B Foetal alcohol syndome

C Melas syndrome

D Turner syndrome

E Williams syndrome

Explanation
Foetal alcohol syndrome

Epidemiology: Common

Aetiology/risk factors: Excess alcohol consumption during pregnancy (dose dependent).

Clinical features and complications:

Acute neonatal alcohol withdrawal


ASD
Ataxia
Epicanthic folds
IUGR
Learning disabilities
Microcephaly

https://mypastest.pastest.com/Secure/TestMe/Browser/436619[‫ ص‬11:04:40 10/12/1437]


MyPastest

Narrow palpebral fissures


Neonatal hypotonia
PDA
Previous Question
Ptosis End Session
Seizures
Skeletal deformities
Narrow philtrum
Thin superior lip
Urinary tract deformities
VSD

45257

Tag Question

Feedback

Difficulty: Average

Peer Responses

Session Progress

Responses Correct: 0

Responses Incorrect: 60

Responses Total: 60

Responses - % Correct: 0%

Next Question

https://mypastest.pastest.com/Secure/TestMe/Browser/436619[‫ ص‬11:04:40 10/12/1437]


MyPastest

Blog
About Pastest
Contact Us
Help

© Pastest 2016

https://mypastest.pastest.com/Secure/TestMe/Browser/436619[‫ ص‬11:04:40 10/12/1437]


MyPastest

Prefer to use the old MyPastest? Access it here »

Back to Filters

Question 41 of 120

An infant is noted to have epicanthic folds, a flat nasal bridge, learning difficulties, a murmur and hypercalcaemia.

What is the most likely diagnosis?

A Down's syndrome

B Fetal alcohol syndrome

C Noonan's syndrome

D Turner's syndrome

E William's syndrome

Explanation
Epidemiology: Rare

Aetiology/risk factors: Genetic/Often sporadic

Clinical features:

Clinodactyly
Dental anomalies
Epicanthic folds
Flat nasal bridge
Learning difficulties
Pectus excavatum

Complications:

https://mypastest.pastest.com/Secure/TestMe/Browser/436619[‫ ص‬11:05:00 10/12/1437]


MyPastest

Attention deficit
Hypercalcaemia
Previous Question
Hypertension
Pulmonary artery stenosis End Session
Pulmonary stenosis
Supravalvular aortic valve stenosis

45259

Tag Question

Feedback

Difficulty: Average

Peer Responses

Session Progress

Responses Correct: 0

Responses Incorrect: 61

Responses Total: 61

Responses - % Correct: 0%

Blog
About Pastest
Contact Us
Help

© Pastest 2016

Next Question

https://mypastest.pastest.com/Secure/TestMe/Browser/436619[‫ ص‬11:05:00 10/12/1437]


MyPastest

https://mypastest.pastest.com/Secure/TestMe/Browser/436619[‫ ص‬11:05:00 10/12/1437]


MyPastest

Prefer to use the old MyPastest? Access it here »

Back to Filters

Question 42 of 120

A deaf child with dysplastic thumbs presents with pallor and bruising. Investigations confirm acute myeloid
leukaemia.

What is the most likely over-arching diagnosis?

A Aplastic anaemia

B Fanconi anaemia

C Fanconi syndrome

D Malaria

E Sickle cell anaemia

Explanation
Fanconi anaemia is a rare autosomal recessive condition.

Features and complications:

AML
Aplastic anaemia
Cardiac
Deafness
GI and kidney malformations
Hypogonadism
Skeletal abnormalities (triangular shaped head, radius and thumb dysplasia/aplasia)
Solid tumours (especially head and neck, and gynaecological)

https://mypastest.pastest.com/Secure/TestMe/Browser/436619[‫ ص‬11:15:36 10/12/1437]


MyPastest

45260

Previous Question
End Session
Tag Question

Feedback

Difficulty: Average

Peer Responses

Session Progress

Responses Correct: 0

Responses Incorrect: 62

Responses Total: 62

Responses - % Correct: 0%

Blog
About Pastest
Contact Us
Help

© Pastest 2016

Next Question

https://mypastest.pastest.com/Secure/TestMe/Browser/436619[‫ ص‬11:15:36 10/12/1437]


MyPastest

Prefer to use the old MyPastest? Access it here »


Previous Question
End Session

Back to Filters

Question 43 of 120

Which one of the following factors may decrease the risk of sudden infant death syndrome (SIDS)?

A Ensuring babies are kept warm

B Ensuring babies sleep on their front

C Laying a baby with their feet away from the foot of the bed

D Maternal smoking

E Use of dummies

Explanation
The risk of sudden infant death syndrome (SIDS) may be reduced by breastfeeding, ensuring babies are not too
hot,
the use of dummies and adopting the 'back to sleep' and 'feet to foot' positions when in their cot or basket.
'Back to sleep' is fairly self explanatory - lying the baby on their back to sleep. 'Feet to foot' refers to ensuring the
baby's feet touch the end of the cot or basket when they are put down to sleep, so that they are less likely to
wriggle
under the covering whilst asleep.
45374

Tag Question

Feedback

Next Question
Difficulty: Average

https://mypastest.pastest.com/Secure/TestMe/Browser/436619[‫ ص‬11:15:58 10/12/1437]


MyPastest

Peer Responses

Session Progress

Responses Correct: 0

Responses Incorrect: 63

Responses Total: 63

Responses - % Correct: 0%

Blog
About Pastest
Contact Us
Help

© Pastest 2016

https://mypastest.pastest.com/Secure/TestMe/Browser/436619[‫ ص‬11:15:58 10/12/1437]


MyPastest

Prefer to use the old MyPastest? Access it here »


Previous Question
End Session

Back to Filters

Question 44 of 120

Which of the following principles is essential when palliating children?

A Effective communication

B Family centered care

C Multi-disciplinary approach

D Sensitive professional approach

E All of the above

Explanation
Palliative care in children requires a professional, sensitive team that communicates effectively and responsively
to the needs of the child and family concerned. A shared understanding of the mutually agreed plan is needed such
that everyone involved is clear of what to do when events change.

http://www.rainbows.co.uk/wp-content/uploads/2011/06/Rainbows-Hospice-Basic-Symptom-Control-In-
Paediatric-Palliative-Care-8th-Ed-2011-protected.pdf 45380

Tag Question

Feedback

Next Question
Difficulty: Average

https://mypastest.pastest.com/Secure/TestMe/Browser/436619[‫ ص‬11:16:19 10/12/1437]


MyPastest

Peer Responses

Session Progress

Responses Correct: 0

Responses Incorrect: 64

Responses Total: 64

Responses - % Correct: 0%

Blog
About Pastest
Contact Us
Help

© Pastest 2016

https://mypastest.pastest.com/Secure/TestMe/Browser/436619[‫ ص‬11:16:19 10/12/1437]


MyPastest

Prefer to use the old MyPastest? Access it here »


Previous Question
End Session

Back to Filters

Question 45 of 120

Which of the following may be a cause of anorexia in children with malignancy?

A Depression

B Oral thrush

C Pain

D Radiotherapy

E All of the above

Explanation
Anorexia may have a number of causes which, in addition to the conditions noted above, also include nausea,
medication,
cancer cachexia, taste disturbance and constipation. If no clear cause is found or if it isn't treatable,
parents may require support and education. Practical management e.g. Small meals, child's favourite food
and
even consideration of steroids may be offered.
45876

Tag Question

Feedback

Difficulty: Average
Next Question

https://mypastest.pastest.com/Secure/TestMe/Browser/436619[‫ ص‬11:17:08 10/12/1437]


MyPastest

Peer Responses

Session Progress

Responses Correct: 0

Responses Incorrect: 65

Responses Total: 65

Responses - % Correct: 0%

Blog
About Pastest
Contact Us
Help

© Pastest 2016

https://mypastest.pastest.com/Secure/TestMe/Browser/436619[‫ ص‬11:17:08 10/12/1437]


MyPastest

Prefer to use the old MyPastest? Access it here »

Previous Question   Logged in as Ahmed fouad

End Session

Back to Filters
Question 46 of 120

When palliating children, which one of the following would NOT be Difficulty: Average
considered to be a cause of dyspnoea?
Peer Responses

A Anaemia

B Pain

C Raised intracranial pressure

D Renal failure

E None of the above Session Progress

Responses Correct: 0

Explanation Responses Incorrect: 66

All four conditions A-D would be considered to be


causes of breathlessness. Responses Total: 66

The list if not exhaustive and there may be multiple factors occurring at once. Responses - % Correct: 0%
It is essential all systems are considered when assessing patients.
http://www.togetherforshortlives.org.uk/assets/0000/5325/TfSL_Basic_Symptom_Control_In_Paediatric_Palliative_Care_-
_Ninth_Edition_-_PDF.pdf 45877

Tag Question Feedback

Blog
About Pastest
Contact Us
Help

© Pastest 2016
Next Question

https://mypastest.pastest.com/Secure/TestMe/Browser/436619[‫ ص‬11:17:36 10/12/1437]


MyPastest

https://mypastest.pastest.com/Secure/TestMe/Browser/436619[‫ ص‬11:17:36 10/12/1437]


MyPastest

Prefer to use the old MyPastest? Access it here »


Previous Question
End Session

Back to Filters

Question 47 of 120

A child with a known mediastinal tumour presents with headache and facial swelling.

What is the most likely diagnosis?

A Acute disseminated encephalomyelitis

B Anaphylaxis

C Cerebral metastasis

D Nephrotic syndrome

E Superior vena cava obstruction

Explanation
Symptoms of SVCO include headache, facial plethora, dizziness and breathlessness. It's considered an oncological
emergency. Treatments include steroids, radiotherapy, chemotherapy and stenting.
45879

Tag Question

Feedback

Difficulty: Average

Next Question
Peer Responses

https://mypastest.pastest.com/Secure/TestMe/Browser/436619[‫ ص‬11:17:57 10/12/1437]


MyPastest

Session Progress

Responses Correct: 0

Responses Incorrect: 67

Responses Total: 67

Responses - % Correct: 0%

Blog
About Pastest
Contact Us
Help

© Pastest 2016

https://mypastest.pastest.com/Secure/TestMe/Browser/436619[‫ ص‬11:17:57 10/12/1437]


MyPastest

Prefer to use the old MyPastest? Access it here »


Previous Question
End Session

Back to Filters

Question 48 of 120

You are choosing an anti-emetic for a terminally ill child.

Which one of the following act on the medulla oblongata?

A Chlorpromazine

B Cyclizine

C Domperidone

D Haloperidol

E Metoclopramide

Explanation
Cyclizine works on the medulla oblongata. The other anti-emetics work by different mechanisms. Cyclizine has
anti-muscarinic properties which may cause urinary retention. It also has sedating anti-histaminic effects. It starts
to work in around 30 mins of administration and peaks at around 2 hours.
45880

Tag Question

Feedback

Difficulty: Average
Next Question

https://mypastest.pastest.com/Secure/TestMe/Browser/436619[‫ ص‬11:18:17 10/12/1437]


MyPastest

Peer Responses

Session Progress

Responses Correct: 0

Responses Incorrect: 68

Responses Total: 68

Responses - % Correct: 0%

Blog
About Pastest
Contact Us
Help

© Pastest 2016

https://mypastest.pastest.com/Secure/TestMe/Browser/436619[‫ ص‬11:18:17 10/12/1437]


MyPastest

Prefer to use the old MyPastest? Access it here »


Previous Question
End Session

Back to Filters

Question 49 of 120

What is the most common cause of hiccough in a terminally ill child?

A Cerebral metastasis

B Gastric distension

C Pleural effusion

D Renal failure

E Superior vena caval obstruction

Explanation
The commonest cause of hiccoughs in terminally ill patients is gastric distension. Gastric reflux, diaphragmatic
irritation and renal failure may also be contributory.
45881

Tag Question

Feedback

Difficulty: Average

Peer Responses
Next Question

https://mypastest.pastest.com/Secure/TestMe/Browser/436619[‫ ص‬11:18:37 10/12/1437]


MyPastest

Session Progress

Responses Correct: 0

Responses Incorrect: 69

Responses Total: 69

Responses - % Correct: 0%

Blog
About Pastest
Contact Us
Help

© Pastest 2016

https://mypastest.pastest.com/Secure/TestMe/Browser/436619[‫ ص‬11:18:37 10/12/1437]


MyPastest

Prefer to use the old MyPastest? Access it here »

Back to Filters

Question 50 of 120

You are screening for sepsis and take a temperature reading.

Which ONE of the following is a feature of a systemic inflammatory response syndrome?

A Core temperature > 37.9 degrees celcius

B Core temperature > 38.1 degrees celcius

C Core temperature > 38.3 degrees celcius

D Core temperature > 38.5 degrees celcius

E Core temperature > 38.7 degrees celcius

Explanation
Sepsis is a condition that is easily missed, particularly in the early stages.

Initial screening for sepsis criteria:

Core temp <36°C or > 38.5°C


Inappropriate tachycardia (or bradycardia) for age
Altered mental status (including sleeping/ irritability/ lethargy/ floppiness
Reduced peripheral perfusion/ prolonged capillary refill /reduced urine output or wet nappies

45884

Next Question

https://mypastest.pastest.com/Secure/TestMe/Browser/436619[‫ ص‬11:18:58 10/12/1437]


MyPastest

Tag Question

Previous Question
Feedback

End Session
Difficulty: Average

Peer Responses

Session Progress

Responses Correct: 0

Responses Incorrect: 70

Responses Total: 70

Responses - % Correct: 0%

Blog
About Pastest
Contact Us
Help

© Pastest 2016

https://mypastest.pastest.com/Secure/TestMe/Browser/436619[‫ ص‬11:18:58 10/12/1437]


MyPastest

Prefer to use the old MyPastest? Access it here »

Back to Filters

Question 51 of 120

A 5-year-old African child comes directly to the emergency department from the airport with bone pain and fever.
A diagnosis of osteomyelitis is made.

What infective agent is commonly associated with osteomyelitis in patients with sickle cell anaemia?

A Campylobacter

B E.coli

C Pseudomonas

D Salmonella species

E Staphylococcus aureus

Explanation
Complications of sickle cell anaemia include:

Chronic haemolysis and subsequent formation of pigment gallstones


Vaso-occlusive crises:
Bone:

triggered by dehydration
hypoxia
sepsis
cold climates
Avascular necrosis of the hip
Dactylitis (results in shortened fingers and toes)

https://mypastest.pastest.com/Secure/TestMe/Browser/436619[‫ ص‬11:19:21 10/12/1437]


MyPastest

Salmonella spp osteomyelitis


septic arthritis
Previous Question
Priapism, CVA, splenic infarction leading to hyposplenism
Sequestration crises: End Session
Spleen
lung

Aplastic crisis secondary to parvovirus B19 infection


Invasive bacterial infections (Strep pneumoniae, Haemophilus influenzae, Neisseria meningitidis)
Renal papillary necrosis, fat embolism

45885

Tag Question

Feedback

Difficulty: Average

Peer Responses

Session Progress

Responses Correct: 0

Responses Incorrect: 71

Responses Total: 71

Responses - % Correct: 0%

Next Question

https://mypastest.pastest.com/Secure/TestMe/Browser/436619[‫ ص‬11:19:21 10/12/1437]


MyPastest

Blog
About Pastest
Contact Us
Help

© Pastest 2016

https://mypastest.pastest.com/Secure/TestMe/Browser/436619[‫ ص‬11:19:21 10/12/1437]


MyPastest

Prefer to use the old MyPastest? Access it here »

Back to Filters

Question 52 of 120

According to the NICE/SIGN guidelines on acute asthma in children:

What respiratory rate would be consistent with a moderate attack in children older than 5 years of age?

A Less than or equal to 15 breaths/min

B Less than or equal to 20 breaths/min

C Less than or equal to 25 breaths/min

D Less than or equal to 30 breaths/min

E Less than or equal to 35 breaths/min

Explanation
The management of emergencies is essential for exams such as these.

Features of a moderate asthma attack in children older than 2 include:

Ability to talk in sentences


SpO2 ≥92%
PEF ≥50% best or predicted
Heart rate ≤140/min in children aged 2–5 years, ≤125/min in children >5 years
Respiratory rate ≤40/min in children aged 2–5 years, ≤30/min in children >5 years

https://www.brit-thoracic.org.uk/document-library/clinical-information/asthma/btssign-asthma-guideline-2014/
45886

Next Question

https://mypastest.pastest.com/Secure/TestMe/Browser/436619[‫ ص‬11:19:42 10/12/1437]


MyPastest

Previous Question Tag Question

Feedback End Session

Difficulty: Average

Peer Responses

Session Progress

Responses Correct: 0

Responses Incorrect: 72

Responses Total: 72

Responses - % Correct: 0%

Blog
About Pastest
Contact Us
Help

© Pastest 2016

https://mypastest.pastest.com/Secure/TestMe/Browser/436619[‫ ص‬11:19:42 10/12/1437]


MyPastest

Prefer to use the old MyPastest? Access it here »

Next Question

Back to Filters

Question 53 of 120

A child is brought into the emergency department with wheeze and breathlessness. The triage nurse states her
asthma attack is at least severe according to the British Thoracic Society.

Her oxygen saturations are likely to be less than which one of the following?

A 97%

B 95%

C 92%

D 90%

E 88%

Explanation
Features of an acute severe asthma attack include:

Can’t complete sentences in one breath or too breathless to talk or feed


SpO2 <92%
PEF 33–50% best or predicted
Heart rate >140/min in children aged 2–5 years, >125/min in children aged >5 years
Respiratory rate >40/min in children aged 2–5 years, >30/min in children aged aged >5 years

https://www.brit-thoracic.org.uk/document-library/clinical-information/asthma/btssign-asthma-guideline-2014/
45887

https://mypastest.pastest.com/Secure/TestMe/Browser/436619#Top[‫ ص‬11:21:37 10/12/1437]


MyPastest

Tag Question

Feedback

Previous Question Difficulty: Average

Peer Responses
End Session

Session Progress

Responses Correct: 0

Responses Incorrect: 73

Responses Total: 73

Responses - % Correct: 0%

Blog
About Pastest
Contact Us
Help

© Pastest 2016

https://mypastest.pastest.com/Secure/TestMe/Browser/436619#Top[‫ ص‬11:21:37 10/12/1437]


MyPastest

Prefer to use the old MyPastest? Access it here »

Next Question

Back to Filters

Question 54 of 120

You see a 4-year-old asthmatic patient in respiratory distress. Your colleague concludes he is having an acute
severe attack.

What is his peak flow likely to be?

A <20% best/predicted

B 20-33% best/predicted

C 33-50% best/predicted

D >50% best/predicted

E 50-75% best/predicted

Explanation
Features of an acute severe asthma attack include:

Can’t complete sentences in one breath or too breathless to talk or feed


SpO2 <92%
PEF
33–50% best or predicted Heart rate >140/min in children aged 2–5 years, >125/min in children aged
>5 years
Respiratory rate >40/min in children aged 2–5 years, >30/min in children aged aged >5 years

https://www.brit-thoracic.org.uk/document-library/clinical-information/asthma/btssign-asthma-guideline-2014/
45888

https://mypastest.pastest.com/Secure/TestMe/Browser/436619#Top[‫ ص‬11:22:00 10/12/1437]


MyPastest

Tag Question

Feedback

Previous Question Difficulty: Average

Peer Responses
End Session

Session Progress

Responses Correct: 0

Responses Incorrect: 74

Responses Total: 74

Responses - % Correct: 0%

Blog
About Pastest
Contact Us
Help

© Pastest 2016

https://mypastest.pastest.com/Secure/TestMe/Browser/436619#Top[‫ ص‬11:22:00 10/12/1437]


MyPastest

Prefer to use the old MyPastest? Access it here »

Next Question
Previous Question Back to Filters

Question 55 of 120 End Session

An unwell child is brought in to see you with fever and a headache.

Which of the following clinical features are consistent with a raised intracranial pressure?

A Bradycardia

B Hypotension

C Oliguria

D Prolonged capillary refill time

E Tachycardia

Explanation
Features of raised intracranial pressure include relative bradycardia and hypertension, altered consciousness, focal
neurology and seizures.

All other options are signs of shock but not raised intracranial pressure.

http://www.meningitis.org/assets/x/50150 45889

Tag Question

Feedback

https://mypastest.pastest.com/Secure/TestMe/Browser/436619#Top[‫ ص‬11:22:22 10/12/1437]


MyPastest

Difficulty: Average

Peer Responses

Session Progress

Responses Correct: 0

Responses Incorrect: 75

Responses Total: 75

Responses - % Correct: 0%

Blog
About Pastest
Contact Us
Help

© Pastest 2016

https://mypastest.pastest.com/Secure/TestMe/Browser/436619#Top[‫ ص‬11:22:22 10/12/1437]


MyPastest

Prefer to use the old MyPastest? Access it here »

Next Question
Previous Question Back to Filters

Question 56 of 120 End Session

You are asked by a colleague to review a child regarding the possibility of non-accidental injury (NAI).

Which of the following would raise suspicion of NAI?

A Bruising in a 2 month old baby

B Delay in seeking medical help

C Inconsistencies in the history of events from the various adults looking after a child

D The history of how an injury occurred is out of keeping with the severity of injury

E All of the above

Explanation
NAI is often subtle and easily missed. Nevertheless there are key elements in the history and examination that
doctors need to be aware of. Clear documentation and management plans are required and, when necessary,
decisive action.
45890

Tag Question

Feedback

Difficulty: Average

https://mypastest.pastest.com/Secure/TestMe/Browser/436619#Top[‫ ص‬11:23:17 10/12/1437]


MyPastest

Peer Responses

Session Progress

Responses Correct: 0

Responses Incorrect: 76

Responses Total: 76

Responses - % Correct: 0%

Blog
About Pastest
Contact Us
Help

© Pastest 2016

https://mypastest.pastest.com/Secure/TestMe/Browser/436619#Top[‫ ص‬11:23:17 10/12/1437]


MyPastest

Prefer to use the old MyPastest? Access it here »

Next Question
Previous Question Back to Filters

Question 57 of 120 End Session

You see a 2-year-old girl in clinic and a nurse mentions that


the parents' account of how the injury occurred had
changed from the history she overheard when they first arrived in the emergency department.

Which of the following would also raise concerns with respect to non-accidental injury (NAI)?

A Bruising especially found on the front of the body

B Bruising on the inner thigh and buttocks

C Bruising on the shins of a two year old

D Bruising particularly on bony prominences

E Mongolian blue spot

Explanation
Bruising on the inner thigh may suggest abuse.

Bruising
found on the front of the body, especially bony prominences, shins, in
a mobile child, are less likely to be
abusive. Mongolian blue spots are
not a sign of abuse.
45892

Tag Question

Feedback

https://mypastest.pastest.com/Secure/TestMe/Browser/436619#Top[‫ ص‬11:23:54 10/12/1437]


MyPastest

Difficulty: Average

Peer Responses

Session Progress

Responses Correct: 0

Responses Incorrect: 77

Responses Total: 77

Responses - % Correct: 0%

Blog
About Pastest
Contact Us
Help

© Pastest 2016

https://mypastest.pastest.com/Secure/TestMe/Browser/436619#Top[‫ ص‬11:23:54 10/12/1437]


MyPastest

Prefer to use the old MyPastest? Access it here »

Next Question
Previous Question Back to Filters

Question 58 of 120 End Session

An infant presents with prolonged neonatal jaundice, pale stool and dark urine.

Which one of the following is the most likely diagnosis?

A Biliary atresia

B Crigler-Nijjar syndrome

C Galactosaemia

D Gilbert syndrome

E Haemolysis

Explanation
Biliary atresia is a rare condition that causes an obstructive jaundice. Without surgical treatment eg Roux-en-Y,
Kasai procedure or liver transplantation, death is likely by 2 years of age. The other conditions do not cause a
conjugated hyperbilirubinaema.
45893

Tag Question

Feedback

Difficulty: Average

https://mypastest.pastest.com/Secure/TestMe/Browser/436619#Top[‫ ص‬11:24:15 10/12/1437]


MyPastest

Peer Responses

Session Progress

Responses Correct: 0

Responses Incorrect: 78

Responses Total: 78

Responses - % Correct: 0%

Blog
About Pastest
Contact Us
Help

© Pastest 2016

https://mypastest.pastest.com/Secure/TestMe/Browser/436619#Top[‫ ص‬11:24:15 10/12/1437]


MyPastest

Prefer to use the old MyPastest? Access it here »

Back to Filters

Question 59 of 120

A 5-year-old child has recently been bitten by a tick after a


recent trip to the New Forest. A diagnosis of Lyme
Disease is made.

Which one of the following rashes is consistent with a diagnosis of Lyme Disease?

A Erythema ab igne

B Erythema chronicum migrans

C Erythema marginatum

D Erythema multiforme

E Erythema nodosum

Explanation
Clinical features of Lyme disease:

Greater risk of transmission with tick feeding >24 hours

Early stages:

Erythema chronicum migrans (75% occur up to one month after tick bite)
fever
malaise
muscle and joint pains

Complications:

https://mypastest.pastest.com/Secure/TestMe/Browser/436619#Top[‫ ص‬11:24:36 10/12/1437]


MyPastest

Affective disorders
Arthritis
Cardiac arrhythmias, Carditis Next Question
Iritis
Previous Question
Meningo-encephalitis, Polyneuropathy
Sleep disturbance

End Session 45894

Tag Question

Feedback

Difficulty: Average

Peer Responses

Session Progress

Responses Correct: 0

Responses Incorrect: 79

Responses Total: 79

Responses - % Correct: 0%

Blog
About Pastest
Contact Us
Help

© Pastest 2016

https://mypastest.pastest.com/Secure/TestMe/Browser/436619#Top[‫ ص‬11:24:36 10/12/1437]


MyPastest

https://mypastest.pastest.com/Secure/TestMe/Browser/436619#Top[‫ ص‬11:24:36 10/12/1437]


MyPastest

Prefer to use the old MyPastest? Access it here »

Back to Filters

Question 60 of 120

You see an infant with decreased skin turgor, slightly sunken


eyes, a depressed fontanelle and a capillary refill
time of <2 s.

Which of the following is likely to be true regarding an infant with this level of dehydration?

A A normal serum potassium level at presentation excludes the development of hypokalaemia once
rehydration is underway

B Diarrhoeal fluid treatment tends to be isotonic

C Projectile vomiting is frequently caused by hiatus hernia

D Shallow respirations are a sign of acidosis

E Vomiting is an important cause of metabolic acidosis

Explanation
The clinical description is of an infant with moderate dehydration (approximately 5%). Children with 10%
dehydration (severe dehydration) usually require intravenous fluid resuscitation.)

Diarrhoeal fluid treatment should be isotonic so option B is true.

An
attempt to compensate for a metabolic acidosis leads to deep sighing respiration and tachypnoea as the child
attempts to blow off CO2.

Persistent vomiting (e.g. with pyloric stenosis) may lead to the development of a metabolic alkalosis through loss
of hydrogen ions in gastric secretions and renal losses (hydrogen ions are exchanged with potassium ions to
correct hypokalaemia).

Although vomiting may be


associated with other conditions causing dehydration (e.g. DKA, poisoning), it is
rarely responsible for the development of a metabolic acidosis.
45896

https://mypastest.pastest.com/Secure/TestMe/Browser/436619#Top[‫ ص‬11:24:58 10/12/1437]


MyPastest

Next Question
Tag Question

Previous Question
Feedback

Difficulty: Average End Session

Peer Responses

Session Progress

Responses Correct: 0

Responses Incorrect: 80

Responses Total: 80

Responses - % Correct: 0%

Blog
About Pastest
Contact Us
Help

© Pastest 2016

https://mypastest.pastest.com/Secure/TestMe/Browser/436619#Top[‫ ص‬11:24:58 10/12/1437]


MyPastest

Prefer to use the old MyPastest? Access it here »

Next Question

Back to Filters

Question 61 of 120

You see an unhappy 3-year-old child suffering from otalgia which began the previous evening. She has a slightly
raised temperature
together with discharge from an inflamed, bulging eardrum. Her parents
tell you that she had a
similar episode a few months ago.

Which of the following treatments is most likely to be recommended for this condition?

A Adenoidectomy should always be performed in patients with recurrent episodes

B Analgesia and antipyretics

C Nasal decongestant alone

D Penicillin is the treatment of choice

E No initial treatment - antibiotics if the symptoms persist beyond 3 days

Explanation
This is a case of acute otitis media (AOM), a middle ear infection, which is usually self-limiting and antibiotics
are
not required initially. The patient should be kept as comfortable as possible with analgesia and antipyretics and
generally it is only if symptoms persist beyond 2 - 3 days that antibiotics may be considered necessary.

Causes of AOM may be viral (RVS, rhinovirus) or bacterial for example streptococcus pneumoniae) and a
bacterial infection may often follow a viral infection.

Surgical treatments may include tympanocentesis and myringotomy for severe or recurrent infections.
45897

https://mypastest.pastest.com/Secure/TestMe/Browser/436619#Top[‫ ص‬11:25:18 10/12/1437]


MyPastest

Tag Question

Feedback

Difficulty: Average
Previous Question
Peer Responses

End Session

Session Progress

Responses Correct: 0

Responses Incorrect: 81

Responses Total: 81

Responses - % Correct: 0%

Blog
About Pastest
Contact Us
Help

© Pastest 2016

https://mypastest.pastest.com/Secure/TestMe/Browser/436619#Top[‫ ص‬11:25:18 10/12/1437]


MyPastest

Prefer to use the old MyPastest? Access it here »

Next Question
Previous Question Back to Filters

Question 62 of 120 End Session

A 5-year-old girl presented in the emergency department with a


brief history of high temperature and cough. On
examination she was drooling and toxic.

Which of the following are essential in this patient?

A Carefully examine throat with torch and spatula

B Give oral steroids

C Send to ward and ask nurses to observe

D Urgent lateral neck X-ray

E Urgent referral to ENT/Anaesthetics

Explanation
A diagnosis of epiglottis is possible in this patient, particularly as the patient looks toxic and is drooling.
Assistance should be requested urgently from ENT or anaesthetics as acute upper airway obstruction may occur.
Measures that may distress the
child such as examining the throat or attempting cannulation should be avoided as
this can lead to airway compromise. An X-ray will not aid the
diagnosis.
45900

Tag Question

Feedback

https://mypastest.pastest.com/Secure/TestMe/Browser/436619#Top[‫ ص‬11:25:40 10/12/1437]


MyPastest

Difficulty: Average

Peer Responses

Session Progress

Responses Correct: 0

Responses Incorrect: 82

Responses Total: 82

Responses - % Correct: 0%

Blog
About Pastest
Contact Us
Help

© Pastest 2016

https://mypastest.pastest.com/Secure/TestMe/Browser/436619#Top[‫ ص‬11:25:40 10/12/1437]


MyPastest

Prefer to use the old MyPastest? Access it here »

Next Question
Previous Question Back to Filters

Question 63 of 120 End Session

You see a child in the emergency department who has abdominal pain and blood in the stools.

Which of the following diagnoses is unlikely to be the cause?

A Campylobacter infection

B Constipation

C Meckel's diverticulum

D Mesenteric adenitis

E Ulcerative colitis

Explanation
Common causes of abdominal pain with blood in the
stools include constipation (due to an anal fissure) and
dysenteric infections, such as campylobacter, shigella or salmonella. Ulcerative colitis, Meckel's diverticulum,
volvulus, HSP, peptic ulcer disease and gastritis are other causes.

Mesenteric adenitis can present with abdominal pain, however blood in the stools is not a recognised feature.
45901

Tag Question

Feedback

https://mypastest.pastest.com/Secure/TestMe/Browser/436619#Top[‫ ص‬11:26:02 10/12/1437]


MyPastest

Difficulty: Average

Peer Responses

Session Progress

Responses Correct: 0

Responses Incorrect: 83

Responses Total: 83

Responses - % Correct: 0%

Blog
About Pastest
Contact Us
Help

© Pastest 2016

https://mypastest.pastest.com/Secure/TestMe/Browser/436619#Top[‫ ص‬11:26:02 10/12/1437]


MyPastest

Prefer to use the old MyPastest? Access it here »

Next Question

Back to Filters

Question 64 of 120

You see a 9-year-old boy who has recently moved to the area, in clinic. He has a short history of frequent bouts of
non-organic abdominal pain.

Which of the following is correct regarding this type of pain?

A Can often be associated with blood in the stools

B Pain can be triggered by stress

C Pain is commonly associated with eating

D Pain typically occurs every day

E Peak age of onset is 3-5 years

Explanation
Over 10% of children experience non-organic pain (abdominal, headache, limbs) and it is important to exclude
organic pathology quickly and to uncover any underlying stresses.

Non-organic
abdominal pain in children typically peaks around the age of 9-10 years. It can have a diverse
frequency of occurrence - from daily to once a month. It is often peri-umbilical in position and does not radiate. It
rarely wakes the child from sleep or is associated with eating. Episodes can be brought about by stressful times
such as moving house/changing schools.

If there is blood in the stools, an alternative diagnosis should be sought.


45903

https://mypastest.pastest.com/Secure/TestMe/Browser/436619#Top[‫ ص‬11:26:22 10/12/1437]


MyPastest

Tag Question

Feedback

Difficulty: Average
Previous Question
Peer Responses

End Session

Session Progress

Responses Correct: 0

Responses Incorrect: 84

Responses Total: 84

Responses - % Correct: 0%

Blog
About Pastest
Contact Us
Help

© Pastest 2016

https://mypastest.pastest.com/Secure/TestMe/Browser/436619#Top[‫ ص‬11:26:22 10/12/1437]


MyPastest

Prefer to use the old MyPastest? Access it here »

Next Question
Previous Question Back to Filters

Question 65 of 120

A 2-month-old child is admitted to the emergency department.

Which of the following are contraindications to discharge?

A Grunting on expiration

B Inconsolable crying

C Lethargy and decreased consciousness

D Temperature of 38.5 oC

E Any one of the above

Explanation
A 2-month-old child presenting with any of the above signs, which should be considered as 'red flags', should be
assessed and have a history taken and physical examination made.

Inconsolable
crying, irritability and fever can all be signs of meningitis. Lethargy
with decreased consciousness is
a worrying presentation and the baby should be immediately assessed. Grunting is a sign of respiratory distress
and measurement of the respiratory rate and oxygen saturations along with an examination of the chest should be
sought.
45905

Tag Question

Feedback

https://mypastest.pastest.com/Secure/TestMe/Browser/436619#Top[‫ ص‬11:27:09 10/12/1437]


MyPastest

Difficulty: Average

Peer Responses

End Session

Session Progress

Responses Correct: 0

Responses Incorrect: 85

Responses Total: 85

Responses - % Correct: 0%

Blog
About Pastest
Contact Us
Help

© Pastest 2016

https://mypastest.pastest.com/Secure/TestMe/Browser/436619#Top[‫ ص‬11:27:09 10/12/1437]


MyPastest

Prefer to use the old MyPastest? Access it here »

Next Question

Back to Filters

Question 66 of 120

You are counselling a couple whose baby has died at 3 days old.

Which of the following is NOT associated with increased perinatal mortality?

A Low social class

B Maternal HIV infection

C Maternal smoking

D Paternal age < 18 years

E Twin pregnancy

Explanation
The perinatal mortality rate is still births and deaths within 6 days of birth per 1000 live- and stillbirths. The rate
varies across the country and is the subject of current study.

Social/demographic
factors demonstrating increased likelihood of perinatal death include low socioeconomic
class, maternal age less than 20 or more than 40, and maternal ethnicity (Black or Black British and Asian or
Asian British).

Maternal
HIV infection increases the susceptibility to perinatal infection and there is a higher incidence of IUGR
and stillbirth. Maternal smoking also appears to be a factor as does twin pregnancy.

http://www.ons.gov.uk/ons/rel/vsob1/child-mortality-statistics--childhood--infant-and-perinatal/2012/stb-child-
mortality-stats-2012.html 45906

https://mypastest.pastest.com/Secure/TestMe/Browser/436619#Top[‫ ص‬11:27:30 10/12/1437]


MyPastest

Tag Question

Feedback

Difficulty: Average
Previous Question
Peer Responses

End Session

Session Progress

Responses Correct: 0

Responses Incorrect: 86

Responses Total: 86

Responses - % Correct: 0%

Blog
About Pastest
Contact Us
Help

© Pastest 2016

https://mypastest.pastest.com/Secure/TestMe/Browser/436619#Top[‫ ص‬11:27:30 10/12/1437]


MyPastest

Prefer to use the old MyPastest? Access it here »

Next Question

Back to Filters

Question 67 of 120

You see a 12-year-old child in clinic who has been assessed as obese. No underlying cause has been discovered
and simple obesity is
diagnosed.

Which of the following statements is correct regarding simple obesity?

A Fails to responds to dietary measures in more than 50% of cases

B Has an association with short stature in childhood

C Has a recognised association with obesity in parents

D Is associated with delayed puberty

E Is more than twice as common in girls as in boys

Explanation
80% of significantly obese children have one or both parents who are obese. Regarding risk factors for childhood
obesity, a family history of obesity predominates. Weight reduction by
dietary means can be achieved in a
significant number of children. Unfortunately, too few persist with the slimming regimens. "Relapse" rates are
high and the long term outlook is poor.

Some obese girls


have advanced puberty or early menarche. Obese children are commonly also tall for their age. .
There is no gender difference in simple childhood obesity.

Exam question theme from June 2015


45912

https://mypastest.pastest.com/Secure/TestMe/Browser/436619#Top[‫ ص‬11:28:59 10/12/1437]


MyPastest

Tag Question

Feedback

Difficulty: Average
Previous Question
Peer Responses

End Session

Session Progress

Responses Correct: 0

Responses Incorrect: 87

Responses Total: 87

Responses - % Correct: 0%

Blog
About Pastest
Contact Us
Help

© Pastest 2016

https://mypastest.pastest.com/Secure/TestMe/Browser/436619#Top[‫ ص‬11:28:59 10/12/1437]


MyPastest

Prefer to use the old MyPastest? Access it here »

Next Question

Back to Filters

Question 68 of 120

You are reviewing the case of a 5-year-old boy who has been referred with nocturnal enuresis.

Which of the following statements is correct regarding this complaint?

A Is more common in a child where a parent has suffered from enuresis

B Is more common in girls than in boys

C Is usually associated with sexual abuse

D Treatment usually requires a child to be of at least 7 years of age

E Treatment with enuresis alarms typically have a less than 50% success rate

Explanation
Nocturnal enuresis is twice as common in boys as in girls, and is three times as common in children with a parent
who has
a history of bedwetting as it is in children where there is no relevant
parental history.

Enuresis alarms have a success rate of between


60 and 90% in treating bedwetting but only in children over the
age of 7
as at this age they are able to co-operate with them.

Because a considerable number of pre-school children wet their beds, treatment of nocturnal enuresis usually
requires that the child is at least five years of age.

While a child who has been sexually abused may wet their bed, only a small proportion of children with nocturnal
enuresis will have suffered sexual abuse.
45916

https://mypastest.pastest.com/Secure/TestMe/Browser/436619#Top[‫ ص‬11:29:24 10/12/1437]


MyPastest

Tag Question

Feedback

Difficulty: Average
Previous Question
Peer Responses

End Session

Session Progress

Responses Correct: 0

Responses Incorrect: 88

Responses Total: 88

Responses - % Correct: 0%

Blog
About Pastest
Contact Us
Help

© Pastest 2016

https://mypastest.pastest.com/Secure/TestMe/Browser/436619#Top[‫ ص‬11:29:24 10/12/1437]


MyPastest

Prefer to use the old MyPastest? Access it here »

Back to Filters

Question 69 of 120

You are consenting a 16-year-old with her parents present, for an elective medical treatment. They were not
married when the child
was born however subsequently married, but are now divorced.

Which of the following should you bear in mind?

A A child aged less than 16 years may give consent for elective medical treatment

B An unmarried father, not named on the birth certificate, still has both financial and intrinsic rights for
his child

C If a child aged less than 16 years refuses surgery it must not be carried out

D If a divorced couple disagree about the need for an elective procedure in their child, the mother's
opinion prevails

E If
emergency surgery is needed and the parents are unavailable, consent must be obtained from a
person such as a teacher who is in loco parentis

Explanation
Below 16 years of age, consent of a parent or guardian is required, unless emergency treatment is necessary (when
consent from a person in loco parentis is also unnecessary) or the child
has given consent and the doctor considers
that the child is of sufficient understanding to make an informed decision about medical care
(including oral
contraception) and the child refuses to allow the parents to be asked. The circumstances in which this may occur
are very
limited as the bar for considering a child understands the implications
of the treatment has been set, by
the Courts, deliberately high.

If a child aged under 16 years refuses surgery it can still be carried out with appropriate consent, if the doctor
believes the child doesn't have sufficient understanding to make an informed decision or if a court
has considered
the child's objection and told the doctor to proceed.

A mother and father have equal parental responsibilities for their legitimate child. If parents disagree it is probably

https://mypastest.pastest.com/Secure/TestMe/Browser/436619#Top[‫ ص‬11:29:48 10/12/1437]


MyPastest

inappropriate to proceed, although only one parent is needed for consent. An unmarried father, not named on the
birth certificate, has no intrinsic rights.
45917

Next Question
Previous Question
Tag Question
End Session
Feedback

Difficulty: Average

Peer Responses

Session Progress

Responses Correct: 0

Responses Incorrect: 89

Responses Total: 89

Responses - % Correct: 0%

Blog
About Pastest
Contact Us
Help

© Pastest 2016

https://mypastest.pastest.com/Secure/TestMe/Browser/436619#Top[‫ ص‬11:29:48 10/12/1437]


MyPastest

Prefer to use the old MyPastest? Access it here »

Next Question
Previous Question Back to Filters

Question 70 of 120 End Session

What is the most common type of congenital heart disease?

A Atrio-septal defect

B Bicuspid aortic valve

C Coarctation of the aorta

D Tetralogy of Fallot

E Ventricular septal defect

Explanation
The most common congenital heart defect is the biscuspid aortic valve (BAV) followed by a ventricular septal
defect (VSD). Approximately 40% of children with congenital heart disease will have a VSD. The prevalence of
BAV is between 0.5 and 2%. It has a genetic aetiology and is more common in those who have affected first
degree relatives. Screening of first degree relatives is therefore recommended. Conditions commonly associated
with a BAV include coarctation and patent ductus arteriosus. BAV predisposes to aortic stenosis and a valve
replacement is required in a significant number of affected patients in adulthood.
45970

Tag Question

Feedback

https://mypastest.pastest.com/Secure/TestMe/Browser/436619#Top[‫ ص‬11:30:09 10/12/1437]


MyPastest

Difficulty: Average

Peer Responses

Session Progress

Responses Correct: 0

Responses Incorrect: 90

Responses Total: 90

Responses - % Correct: 0%

Blog
About Pastest
Contact Us
Help

© Pastest 2016

https://mypastest.pastest.com/Secure/TestMe/Browser/436619#Top[‫ ص‬11:30:09 10/12/1437]


MyPastest

Prefer to use the old MyPastest? Access it here »

Next Question
Previous Question Back to Filters

Question 71 of 120 End Session

What proportion of ventricular septal defects close spontaneously by one year of age?

A 10%

B 30%

C 50%

D 70%

E 90%

Explanation
VSDs are the most common congenital cardiac defect, the second most common being bicuspid aortic valve. 40%
of children with congenital cardiac disease will have a VSD. Between 70-80%
of cases affect the membranous
septum. Large defects can increase pulmonary vascular resistance and eventually lead to a right to left shunt and
Eisenmenger's syndrome.

The following article offers further reading around the topic:

http://content.onlinejacc.org/article.aspx?articleid=1128002
45971

Tag Question

Feedback

https://mypastest.pastest.com/Secure/TestMe/Browser/436619#Top[‫ ص‬11:30:29 10/12/1437]


MyPastest

Difficulty: Average

Peer Responses

Session Progress

Responses Correct: 0

Responses Incorrect: 91

Responses Total: 91

Responses - % Correct: 0%

Blog
About Pastest
Contact Us
Help

© Pastest 2016

https://mypastest.pastest.com/Secure/TestMe/Browser/436619#Top[‫ ص‬11:30:29 10/12/1437]


MyPastest

Prefer to use the old MyPastest? Access it here »

Next Question
Previous Question Back to Filters

Question 72 of 120 End Session

What proportion of children with secundum ASDs are female?

A 90%

B 80%

C 70%

D 60%

E 50%

Explanation
Around 7% of congenital heart defects are secundum ASDs. 70% of those who have a secundum ASD are female.
It may form part of a Holt-Oram syndrome which includes an ASD (or VSD) and triphalangeal thumbs.
45973

Tag Question

Feedback

Difficulty: Average

Peer Responses

https://mypastest.pastest.com/Secure/TestMe/Browser/436619#Top[‫ ص‬11:30:50 10/12/1437]


MyPastest

Session Progress

Responses Correct: 0

Responses Incorrect: 92

Responses Total: 92

Responses - % Correct: 0%

Blog
About Pastest
Contact Us
Help

© Pastest 2016

https://mypastest.pastest.com/Secure/TestMe/Browser/436619#Top[‫ ص‬11:30:50 10/12/1437]


MyPastest

Prefer to use the old MyPastest? Access it here »

Next Question
Previous Question Back to Filters

Question 73 of 120 End Session

Which of the following statements are true regarding the anatomy of the ductus arteriosus?

A The ductus arteriosus connects the left subclavian artery to the pulmonary artery

B The ductus arteriosus connects the common carotid artery to the pulmonary artery

C The ductus arteriosus connects the brachiocephalic artery to the pulmonary artery

D The ductus arteriosus connects the proximal ascending aorta to the pulmonary artery

E The ductus arteriosus connects the proximal descending aorta to the pulmonary artery

Explanation
The ductus arteriosus connects the proximal descending aorta to the pulmonary artery, close to the origin of the
left pulmonary artery. In utero, this allows blood to bypass the pulmonary vasculature. Failure of closure leads to a
defect called a patent ductus arteriosus (PDA). PDA is twice as common in females than males. Several causes
have been described including Down syndrome, Holt-Oram syndrome, prematurity, birth at high altitude and
congenital Rubella syndrome.
45976

Tag Question

Feedback

Difficulty: Average

https://mypastest.pastest.com/Secure/TestMe/Browser/436619#Top[‫ ص‬11:31:10 10/12/1437]


MyPastest

Peer Responses

Session Progress

Responses Correct: 0

Responses Incorrect: 93

Responses Total: 93

Responses - % Correct: 0%

Blog
About Pastest
Contact Us
Help

© Pastest 2016

https://mypastest.pastest.com/Secure/TestMe/Browser/436619#Top[‫ ص‬11:31:10 10/12/1437]


MyPastest

Prefer to use the old MyPastest? Access it here »

Next Question
Previous Question Back to Filters

Question 74 of 120 End Session

A child is examined at a 6-week check. The GP notes a continuous machinery type murmur at the left sternal edge.

What is the most likely cause?

A Aortic stenosis

B Atrio-septal defect

C Patent ductus arteriosus

D Tetralogy of Fallot

E Ventricular septal defect

Explanation
Around 10% of congenital cardiac defects are due to PDAs. They occur more commonly in females (2:1 F:M) and
have multiple
causes including Down syndrome, prematurity, birth at high altitude and
congenital Rubella.

PDAs typically cause a machinery murmur. A large left to right shunt through the ductus can lead to increased
pulmonary flow, pulmonary hypertension, eventual shunt reversal and Eisenmenger syndrome.
45977

Tag Question

Feedback

https://mypastest.pastest.com/Secure/TestMe/Browser/436619#Top[‫ ص‬11:31:31 10/12/1437]


MyPastest

Difficulty: Average

Peer Responses

Session Progress

Responses Correct: 0

Responses Incorrect: 94

Responses Total: 94

Responses - % Correct: 0%

Blog
About Pastest
Contact Us
Help

© Pastest 2016

https://mypastest.pastest.com/Secure/TestMe/Browser/436619#Top[‫ ص‬11:31:31 10/12/1437]


MyPastest

Prefer to use the old MyPastest? Access it here »

Next Question
Previous Question Back to Filters

Question 75 of 120 End Session

What percentage of children with Turner's syndrome have coarctation of the aorta?

A 15%

B 30%

C 45%

D 60%

E 75%

Explanation
Coarctation of the aorta is usually a sporadic condition and affects more males than females. There are some
genetic associations, however, including Turner syndrome. There is an association with VSD, PDA, mitral valve
disease and biscuspid aortic valves. There is an increased risk of hypertension and intracranial and intercostal
artery aneurysms. Presentation in children usually suggests severe disease and can be associated with cardiac
failure.
45979

Tag Question

Feedback

Difficulty: Average

https://mypastest.pastest.com/Secure/TestMe/Browser/436619#Top[‫ ص‬11:31:52 10/12/1437]


MyPastest

Peer Responses

Session Progress

Responses Correct: 0

Responses Incorrect: 95

Responses Total: 95

Responses - % Correct: 0%

Blog
About Pastest
Contact Us
Help

© Pastest 2016

https://mypastest.pastest.com/Secure/TestMe/Browser/436619#Top[‫ ص‬11:31:52 10/12/1437]


MyPastest

Prefer to use the old MyPastest? Access it here »

Next Question
Previous Question Back to Filters

Question 76 of 120 End Session

Which one of the following forms part of the classic Tetralogy of Fallot complex?

A Atrio-septal defect

B Coarctation of the aorta

C Left ventricular hypertrophy

D Patent ductus arteriosus

E Right ventricular hypertrophy

Explanation
Tetralogy of Fallot is the most common form of congenital cyanotic heart disease and consists of:

Over-riding aorta
VSD
Right ventricular outflow obstruction
Right ventricular hypertrophy

45980

Tag Question

Feedback

https://mypastest.pastest.com/Secure/TestMe/Browser/436619#Top[‫ ص‬11:32:13 10/12/1437]


MyPastest

Difficulty: Average

Peer Responses

Session Progress

Responses Correct: 0

Responses Incorrect: 96

Responses Total: 96

Responses - % Correct: 0%

Blog
About Pastest
Contact Us
Help

© Pastest 2016

https://mypastest.pastest.com/Secure/TestMe/Browser/436619#Top[‫ ص‬11:32:13 10/12/1437]


MyPastest

Prefer to use the old MyPastest? Access it here »

Next Question
Previous Question Back to Filters

Question 77 of 120 End Session

What proportion of patients with Tetralogy of Fallot have Di George syndrome?

A 5%

B 15%

C 25%

D 35%

E 55%

Explanation
Di George syndrome affects around 1 in 4000 live births and is caused by a 22q11.2 micro-deletion.

It has a number of potential features including T-cell dysfunction, thymus maldevelopment, neonatal seizures due
to hypocalcaemia, failure to thrive, low-set ears, orbital hypertelorism, cleft palate and cardiac
defects
45981

Tag Question

Feedback

Difficulty: Average

Peer Responses

https://mypastest.pastest.com/Secure/TestMe/Browser/436619#Top[‫ ص‬11:32:32 10/12/1437]


MyPastest

Session Progress

Responses Correct: 0

Responses Incorrect: 97

Responses Total: 97

Responses - % Correct: 0%

Blog
About Pastest
Contact Us
Help

© Pastest 2016

https://mypastest.pastest.com/Secure/TestMe/Browser/436619#Top[‫ ص‬11:32:32 10/12/1437]


MyPastest

Prefer to use the old MyPastest? Access it here »

Next Question
Previous Question Back to Filters

Question 78 of 120 End Session

Which one of the following could a Fontan procedure be used to repair?

A Coarctation of the aorta

B Eisenmenger syndrome

C Hypoplastic left heart

D Patent ductus arteriosus

E Transposition of the great arteries

Explanation
The underlying principle of the Fontan operation is to establish systemic venous return to the pulmonary artery
without requiring a sub-pulmonary ventricle. Ultimately, the Fontan circulation is where one ventricle supports
both the systemic and pulmonary circulations.
45983

Tag Question

Feedback

Difficulty: Average

Peer Responses

https://mypastest.pastest.com/Secure/TestMe/Browser/436619#Top[‫ ص‬11:32:52 10/12/1437]


MyPastest

Session Progress

Responses Correct: 0

Responses Incorrect: 98

Responses Total: 98

Responses - % Correct: 0%

Blog
About Pastest
Contact Us
Help

© Pastest 2016

https://mypastest.pastest.com/Secure/TestMe/Browser/436619#Top[‫ ص‬11:32:52 10/12/1437]


MyPastest

Prefer to use the old MyPastest? Access it here »

Next Question
Previous Question Back to Filters

Question 79 of 120 End Session

Which of the following conditions is a duct dependent circulation?

A Atrio-septal defect

B Hypoplastic left heart syndrome

C Patent ductus arteriosus

D Tetralogy of Fallot

E Ventricular septal defect

Explanation
A, C, D and E are shunts in their own right. Hypoplastic left heart is a triad of small left ventricle, mitral
stenosis/atresia and aortic stenosis/atresia. As such blood flows from the left to right atrium via an ASD. Systemic
flow is maintained through
the ductus arteriosus.
45984

Tag Question

Feedback

Difficulty: Average

Peer Responses

https://mypastest.pastest.com/Secure/TestMe/Browser/436619#Top[‫ ص‬11:33:11 10/12/1437]


MyPastest

Session Progress

Responses Correct: 0

Responses Incorrect: 99

Responses Total: 99

Responses - % Correct: 0%

Blog
About Pastest
Contact Us
Help

© Pastest 2016

https://mypastest.pastest.com/Secure/TestMe/Browser/436619#Top[‫ ص‬11:33:11 10/12/1437]


MyPastest

Prefer to use the old MyPastest? Access it here »

Back to Filters

Question 80 of 120

A 6-year-old girl is brought to the emergency department by ambulance following a witnessed fall of 2 metres
from the climbing frame
in her school playground. She sustained a blow to the head and lost consciousness for 1-2
minutes at the scene. She vomited twice in the playground and once more on the way to hospital.

You assess her 1


hour after the event and find her to be abnormally drowsy and disorientated. She is following
commands and speaking in full sentences with her eyes open. Her observations are as follows: HR 119, RR 26, T
36.2, BP 95/52, sats 96% on air. The blood sugar is 4.5mmol/L. There is no focal neurology. There is a 2x3cm
fluctuant swelling overlying the left temple. During your assessment she vomits again.

What is the most appropriate next step?

A Observe for next 6-8 hours and arrange CT head in case of further deterioration

B Obtain anaesthetic advice on airway management prior to CT head

C Obtain neurosurgical advice from the regional paediatric centre

D Perform CT head as soon as possible (within one hour)

E Prepare 20% mannitol infusion

Explanation
NICE recommend CT head scan within 1 hour for children with more than 1 of the following risk factors:

Loss of consciousness lasting more than 5 minutes (witnessed)


Abnormal drowsiness
Three or more discrete episodes of vomiting
Dangerous
mechanism of injury (high-speed road traffic accident either as pedestrian, cyclist or vehicle
occupant, fall from a height of greater than 3 metres, high-speed injury from a projectile or other object)

https://mypastest.pastest.com/Secure/TestMe/Browser/436619#Top[‫ ص‬11:33:34 10/12/1437]


MyPastest

Amnesia (antegrade or retrograde) lasting more than 5 minute

This 6-year-old child has had 3-4 discrete episodes of vomiting and is abnormally drowsy, hence requires CT head
Next Question
to exclude intracranial bleed requiring neurosurgical intervention.

A is incorrect as the child requires CT head as the first step prior to a period of observation.
Previous Question
B is incorrect as the GCS is 14/15 with no history of deteriorating conscious level, the observations are normal,
and no airway problems have been identified; anaesthetic input is not required prior to CT head.
Endneurosurgical
C is incorrect as there are not currently any clinical features mandating immediate Session input, defined by
NICE as:

New, surgically significant abnormalities on imaging.


Persisting coma (GCS 8 or less) after initial resuscitation.
Unexplained confusion which persists for more than 4 hours.
Deterioration in GCS score after admission (greater attention should be paid to motor response
deterioration).
Progressive focal neurological signs.
A seizure without full recovery.
Definite or suspected penetrating injury.
A cerebrospinal fluid leak.

E is incorrect as there are no signs of raised intracranial pressure except GCS 14/15 (which is more likely
explained by concussion than raised intracranial pressure).

Reference: https://www.nice.org.uk/guidance/cg176
46118

Tag Question

Feedback

Difficulty: Average

Peer Responses

https://mypastest.pastest.com/Secure/TestMe/Browser/436619#Top[‫ ص‬11:33:34 10/12/1437]


MyPastest

Session Progress

Responses Correct: 0

Responses Incorrect: 100

Responses Total: 100

Responses - % Correct: 0%

Blog
About Pastest
Contact Us
Help

© Pastest 2016

https://mypastest.pastest.com/Secure/TestMe/Browser/436619#Top[‫ ص‬11:33:34 10/12/1437]


MyPastest

Prefer to use the old MyPastest? Access it here »

Back to Filters

Question 81 of 120

An 18-month-old boy is brought by air ambulance to the regional trauma centre following a road traffic accident
in which the vehicle he was travelling in collided with another at a combined velocity of approximately 80mph.
His car seat was thrown forward into the front passenger seat. He has sustained multiple injuries including a
fractured pelvis and suspected skull fracture.

On initial assessment in the emergency department resus after disembarking from the
air ambulance, the child is
lying supine with c-spine immobilised. His eyes are closed and he is motionless. There is no response to voice. He
cries and produces weak vocal sounds to pain but does not open his eyes.
He tries to push away the examining
hand.

What is this child’s Paediatric Glasgow Coma Scale score?

A 6

B 7

C 8

D 9

E 10

Explanation
The Glasgow Coma Scale was modified for children as follows (Adelaide scale):

Eye opening Verbal response Best motor response

4 - Spontaneous 5 - Talks normally 5 - Obeys commands

3 - To sound 4 - Words 4 - Localises pain

https://mypastest.pastest.com/Secure/TestMe/Browser/436619#Top[‫ ص‬11:33:58 10/12/1437]


MyPastest

2 - To pain 3 - Vocal sounds 3 - Flexion to pain

1 - None 2 - Cries 2 - Extension to pain

1 - None Next Question


1 - None

Previous
The described child scores 1 Question
for Eye, 3 for Verbal, 4 for Motor. Total GCS is therefore 8.

References: http://glasgowcomascale.org/faq/#faq-2

Reilly
PL, Simpson DA. Sprod R. Thomas L. Assessing the conscious level inEnd Session
infants and young children: a
paediatric version of the Glasgow Coma Scale. Childs Nervous System. 4(1):30-3, 1988
46120

Tag Question

Feedback

Difficulty: Average

Peer Responses

Session Progress

Responses Correct: 0

Responses Incorrect: 101

Responses Total: 101

Responses - % Correct: 0%

Blog
About Pastest
Contact Us
Help

© Pastest 2016

https://mypastest.pastest.com/Secure/TestMe/Browser/436619#Top[‫ ص‬11:33:58 10/12/1437]


MyPastest

https://mypastest.pastest.com/Secure/TestMe/Browser/436619#Top[‫ ص‬11:33:58 10/12/1437]


MyPastest

Prefer to use the old MyPastest? Access it here »

Back to Filters

Question 82 of 120

A ten year old boy presents to rapid access clinic with the following headache-related symptoms.

Which symptom should prompt you to arrange an urgent CT head scan?

A Increasing headache frequency

B Increasing headache severity

C Morning vomiting

D Unusual sensitivity to light and sound.

E Visual loss around time of headache

Explanation
The correct answer is Morning Vomiting as if present, without any other clear cause, it is a Red Flag Symptom
possibly suggestive of raised intracranial pressure (ICP). Raised ICP may be secondary to a space occupying
lesion or structural problem that could be visualised on a CT brain scan.

All the other answers may be symptoms associated with tension headaches, cluster headaches or migraines (visual
phenomena or partial visual loss may be part of the migraine aura). These symptoms would be most likely
assessed further with a headache diary and review of medication, behavioural/non-pharmacological treatment and
possibly dietary interventions before considering imaging. This is of course in conjunction with normal
neurological examination findings.

A helpful symptom breakdown can be found in the NICE Guidance:


https://www.nice.org.uk/guidance/cg150/chapter/recommendations

Other Red Flag Symptoms of headaches in the paediatric population include:

1. A short history (‘First’ or ‘worst’), or recent recurrent severe headache for few weeks.
2. Rapid worsening of symptoms, change in character over weeks or days.

https://mypastest.pastest.com/Secure/TestMe/Browser/436619#Top[‫ ص‬11:37:01 10/12/1437]


MyPastest

3. Headache
suggesting raised intracranial pressure (early morning headache, vomiting in morning, pain
disturbing sleep, headache worse with cough or
valsalva).
4. Associated symptoms including personality changes, weakness, visual disturbances, confusion, focal
weakness, seizures or fever. Next Question
5. Underlying
history of neurocutaneous syndrome, history of systemic illnesses eg. known malignancy with
possible brain metastases, clotting disorders.
6. Young age Previous Question
of child (<3yrs).
46191

End Session

Tag Question

Feedback

Difficulty: Average

Peer Responses

Session Progress

Responses Correct: 0

Responses Incorrect: 102

Responses Total: 102

Responses - % Correct: 0%

Blog
About Pastest
Contact Us
Help

© Pastest 2016

https://mypastest.pastest.com/Secure/TestMe/Browser/436619#Top[‫ ص‬11:37:01 10/12/1437]


MyPastest

Prefer to use the old MyPastest? Access it here »

Next Question

Back to Filters

Question 83 of 120

Which of the following calculations is correct for assessing the appropriate size of an uncuffed oral tracheal tube?

A Internal diameter (mm) = (Age/2) +2. Length (cm) = (Age/4) +12

B Internal diameter (mm) = (Age/2) +2. Length (cm) = (Age/4) +15

C Internal diameter (mm) = (Age/4) +4. Length (cm) = (Age/2) +12

D Internal diameter (mm) = (Age/4) +4. Length (cm) = (Age/2) +15

E Internal diameter (mm) = (Age/4) +4. Length (cm) = (Age/4) +12

Explanation
The correct calculation for an uncuffedoral tracheal tube is C,

Internal diameter (mm) = (Age/4) +4. Length (cm) = (Age/2) +12

The correct calculation for an uncuffednasal tube is:

Internal diameter (mm) = (Age/4) +4. Length (cm) = (Age/2) +15

For cuffed tracheal tubes, the appropriate diameter can be estimated by:

Internal diameter (mm) = (Age/4) + 3.5.

The other calculations in the question are incorrect and should not be used.
46192

Tag Question

https://mypastest.pastest.com/Secure/TestMe/Browser/436619#Top[‫ ص‬11:37:24 10/12/1437]


MyPastest

Feedback

Difficulty: Average
Previous Question
Peer Responses

End Session

Session Progress

Responses Correct: 0

Responses Incorrect: 103

Responses Total: 103

Responses - % Correct: 0%

Blog
About Pastest
Contact Us
Help

© Pastest 2016

https://mypastest.pastest.com/Secure/TestMe/Browser/436619#Top[‫ ص‬11:37:24 10/12/1437]


MyPastest

Prefer to use the old MyPastest? Access it here »

Next Question
Previous Question Back to Filters

Question 84 of 120 End Session

Which of the following drugs is NOT used in anaesthetic induction for orotracheal intubation?

A Bupivacaine

B Etomidate

C Ketamine

D Propofol

E Thiopentol

Explanation
The correct answer is A – Bupivacaine is a local anaesthetic with a longer duration of action than lidocaine. It is
most often used for spinal anaesthesia. It is not part of anaesthetic
induction.

Ketamine, Etomidate, Propofol, Thiopental (thiopentone) and sedative drugs such as midazolam, fentanyl and
morphine are the most common anaesthetic induction agents.Appropriate oxygenation must also be ensured
throughout.
46193

Tag Question

Feedback

Difficulty: Average

https://mypastest.pastest.com/Secure/TestMe/Browser/436619#Top[‫ ص‬11:37:44 10/12/1437]


MyPastest

Peer Responses

Session Progress

Responses Correct: 0

Responses Incorrect: 104

Responses Total: 104

Responses - % Correct: 0%

Blog
About Pastest
Contact Us
Help

© Pastest 2016

https://mypastest.pastest.com/Secure/TestMe/Browser/436619#Top[‫ ص‬11:37:44 10/12/1437]


MyPastest

Prefer to use the old MyPastest? Access it here »

Back to Filters

Question 85 of 120

You are urgently called to a choking child in the clinic waiting room. The child is conscious but struggling to
breathe.

What do you do next?

A 5 back blows

B 5 chest thrusts / abdominal thrusts

C 5 rescue breaths

D Encourage coughing

E Open airway

Explanation
The correct answer is A – 5 back blows. This child is choking but conscious with an ineffective cough therefore
the following sequence of actions must be followed:

CONSCIOUS CHILD with INEFFECTIVE COUGH


5 back blows
5 chest/abdominal thrusts
Assess and repeat

CONSCIOUS CHILD with EFFECTIVE COUGH


Encourage coughing
Support and assess continuously

UNCONSCIOUS CHILD
Open airway
5 rescue breaths

https://mypastest.pastest.com/Secure/TestMe/Browser/436619#Top[‫ ص‬11:38:05 10/12/1437]


MyPastest

CPR 15:2
Check for foreign body

Additionally, if the choking episode is not rapidly resolved and the child clearly fit and well, help should be
Next Question
summoned urgently. The child
should be assessed and monitored after the resuscitation is completed as injury can
occur during chest, abdominal thrusts and CPR.
46194
Previous Question

End Session

Tag Question

Feedback

Difficulty: Average

Peer Responses

Session Progress

Responses Correct: 0

Responses Incorrect: 105

Responses Total: 105

Responses - % Correct: 0%

Blog
About Pastest
Contact Us
Help

© Pastest 2016

https://mypastest.pastest.com/Secure/TestMe/Browser/436619#Top[‫ ص‬11:38:05 10/12/1437]


MyPastest

Prefer to use the old MyPastest? Access it here »

Back to Filters

Question 86 of 120

A 10 year old boy is in Resus after being hit by a speeding car. His posturing is abnormal and you are extremely
concerned. He has
a stiff body, bent arms, clenched fists and outstretched legs.

What is the name of this form of posturing?

A Decerebrate

B Decorticate

C Extensor

D Opisthotonus

E Opsoclonus-Myoclonus

Explanation
This boy is showing abnormal decorticate posturing suggestive that he has had a major head injury.

Decorticate posture can be remembered as the ‘mummy posture’ as it resembles mummys from ancient tombs. It
is caused by brain lesions above the red nucleus impacting the rubriospinal and corticospinal tracts damaging the
motor neurone pathways and muscular control.

Decerebrate posture, also known as extensor posturing is evidenced by arms and legs straight, toes pointing
downward, head and
neck arched backwards. Teeth may be clenched. This occurs as the brain
stem is damaged,
specifically below the red nucleus. Progression from decorticate (arms bent) to decerebrate (arms straight) often
occurs during tonsillar brain herniation.

Opisthotonus posturing is evidenced by a rigid and arching back and head thrown backwards. It may
also be seen
in brain injury, tetanus and after drowning. It is an extrapyramidal effect caused by spasm of the axial muscles
along the spinal column.

Decorticate, decerebrate and opisthotonic posturing all suggest brain injury and the patient requires immediate

https://mypastest.pastest.com/Secure/TestMe/Browser/436619#Top[‫ ص‬11:38:28 10/12/1437]


MyPastest

attention. A patient may fluctuate between posturing forms or show decorticate on one side and decerebrate
posturing on the other.

Posturing occurs as the brain is injured and muscle groups act without opposition.
Next Question
All of the above have been reported in children with cerebral malaria.

Opsoclonus–
Myoclonus
Previous / Opsoclonus-Myoclonus
Question Syndrome (OMS) is a rare neurological disorder thought to be
autoimmune in origin. It affects 2-3% of children with neuroblastoma. Symptoms include opsoclonus ‘dancing
eyes’ with unpredictable rapid conjugate eye movements, myoclonus
(muscular twitching), cerebellar ataxia,
speech problems, drooling, vomiting and lethargy.
End Session 46195

Tag Question

Feedback

Difficulty: Average

Peer Responses

Session Progress

Responses Correct: 0

Responses Incorrect: 106

Responses Total: 106

Responses - % Correct: 0%

Blog
About Pastest
Contact Us
Help

© Pastest 2016

https://mypastest.pastest.com/Secure/TestMe/Browser/436619#Top[‫ ص‬11:38:28 10/12/1437]


MyPastest

https://mypastest.pastest.com/Secure/TestMe/Browser/436619#Top[‫ ص‬11:38:28 10/12/1437]


MyPastest

Prefer to use the old MyPastest? Access it here »

Back to Filters

Question 87 of 120

On the neonatal unit a baby is on the following ventilator settings:

SIPPV (Synchronous Positive Pressure Ventilation)


Tidal volume = 5ml/kg PEEP = 4
Inspiratory time = 0.35 seconds
Respiratory rate = 60
PIP (max) = 22

For which neonatal patient is this ventilator setting most appropriate?

A Premature baby spontaneously breathing and weaning from ventilator

B Premature baby with pneumothorax

C Stable premature baby on antibiotics

D Term baby with Meconium Aspiration Syndrome

E Term baby with Transient Tachypnoea of the Newborn (TTN)

Explanation
The correct answer is C, stable premature baby on
antibiotics. This is an appropriate initial ventilator setting for
premature babies which will then be adjusted according to clinical progress, oxygen saturations and blood gas
results.

A premature baby with a pneumothorax and a term baby with Meconium Aspiration Syndrome may well be
mechanically ventilated but it is likely their settings will be adjusted because of the respiratory compromise.

A term baby with TTN is unlikely to be ventilated and a premature baby who is spontaneously breathing may need
to be weaned to a lower breath rate and moved to a different mode of ventilation (such as SIMV +
pressure
support) or extubated.

https://mypastest.pastest.com/Secure/TestMe/Browser/436619#Top[‫ ص‬11:38:51 10/12/1437]


MyPastest

Ventilation modes and settings of preference may vary within and between neonatal units but it is important to
know safe boundaries. Your own neonatal unit is likely to have a handbook or ventilation guidebook to help you.
46196

Next Question
Previous Question
Tag Question
End Session
Feedback

Difficulty: Average

Peer Responses

Session Progress

Responses Correct: 0

Responses Incorrect: 107

Responses Total: 107

Responses - % Correct: 0%

Blog
About Pastest
Contact Us
Help

© Pastest 2016

https://mypastest.pastest.com/Secure/TestMe/Browser/436619#Top[‫ ص‬11:38:51 10/12/1437]


MyPastest

Prefer to use the old MyPastest? Access it here »

Back to Filters

Question 88 of 120

A 5-year-old is eating a bag of sweets whilst waiting for his


brother to be seen in the emergency department. He
suddenly develops stridor and a cough. Initially he has a good cough however as he tires it become ineffective.

What is the next step in his management?

A Attempt to remove the object with forceps

B Give 5 back blows followed by 5 abdominal thrusts

C Give 5 back blows followed by 5 chest thrusts

D Open his airway and give 5 rescue breaths

E Start CPR at a ratio of 15 compressions to 2 breaths

Explanation
As the child has an ineffective cough the next step would be to give 5 back blows with the child leaning forward,
if this does not expel the object, then 5 abdominal thrusts should be given. This cycle is continued until the object
is expelled or the child
becomes unconscious.

A – In an unconscious child, if you can see the object in the mouth then a single finger sweep can be attempted to
remove it. Repeated attempts or using objects in the mouth are not advised as they can further lodge the object into
the pharynx.

C – This method is used in a child under 1 year of age to simulate a cough and prevent damage to the abdominal
organs which are less protected by the ribcage.

D – In a conscious child the airway is open and this step is not required, it becomes relevant if
the child becomes
unconscious.

E – If the child becomes unconscious 5 rescue breaths followed by CPR is performed.

https://mypastest.pastest.com/Secure/TestMe/Browser/436619#Top[‫ ص‬11:39:13 10/12/1437]


MyPastest

Ref: www.resus.org.uk/resuscitation-guidelines/paediatric-basic-life-support/#choking 46647

Next Question
Previous Question Tag Question

Feedback
End Session
Difficulty: Average

Peer Responses

Session Progress

Responses Correct: 0

Responses Incorrect: 108

Responses Total: 108

Responses - % Correct: 0%

Blog
About Pastest
Contact Us
Help

© Pastest 2016

https://mypastest.pastest.com/Secure/TestMe/Browser/436619#Top[‫ ص‬11:39:13 10/12/1437]


MyPastest

Prefer to use the old MyPastest? Access it here »

Back to Filters

Question 89 of 120

An unimmunised 5-year-old is brought to your resus room. He has a short history of drooling, stridor and fever.
He is sitting in a tripod position trying to maintain his own airway.

What is your next step in management from the choices below?

A Call for ENT and anaesthetic teams

B Examine the throat

C Give nebulised Budesonide

D Give oral Dexamethasone

E Obtain IV access and bloods

Explanation
This child is unimmunised and has a short history
of an upper airway obstruction associated with fever, this is
suggestive of epiglottitis. This is rare following introduction of Haemophilus influenzae type b (Hib) vaccine, but
can still occur. A calm environment is essential; the child is currently maintaining his airway and distressing him
may jeopardise this. Diagnosis is made by ENT visualising the airway (usually in theatre) with anaesthetic
support. Once he has a stable airway antibiotics will be required.

B – Examining the throat can result in occluding the airway and must be avoided.

C – This may be used in Croup but may distress the child in this case.

D - This may be used in Croup but may distress the child in this case.

E – This will distress the child and risks causing complete airway occlusion.
46648

https://mypastest.pastest.com/Secure/TestMe/Browser/436619#Top[‫ ص‬11:41:04 10/12/1437]


MyPastest

Next Question
Tag Question

Previous Question
Feedback

Difficulty: Average End Session


Peer Responses

Session Progress

Responses Correct: 0

Responses Incorrect: 109

Responses Total: 109

Responses - % Correct: 0%

Blog
About Pastest
Contact Us
Help

© Pastest 2016

https://mypastest.pastest.com/Secure/TestMe/Browser/436619#Top[‫ ص‬11:41:04 10/12/1437]


MyPastest

Prefer to use the old MyPastest? Access it here »

Back to Filters

Question 90 of 120

A 7-year-old asthmatic is brought to your resus room. He is


unconscious following a prolonged asthma attack. On
assessment he has
no respiratory effort and no pulse. You attach cardiac monitoring and see he is in asystole. A
crash call is placed.

What is the next step in his management?

A Give a dose of IV adrenaline 1:10,000 0.1ml/kg

B Give a dose of IO adrenaline 1:10,000 0.1ml/kg

C Head tilt chin lift, followed by 5 rescue breaths

D Head in neutral position followed by 5 rescue breaths

E Start CPR at a rate of 15:2

Explanation
The first step in resuscitation efforts is to ensure that the airway is open. This can be achieved in children with a
head tilt and chin lift to provide a “sniffing” position; a
jaw thrust can also be used. For infants a neutral position
is required. 5 rescue breaths are then given as hypoxia is more likely to be the cause of arrests in children rather
than cardiac courses as seen in the adult population.

A - This is the correct dose, however airway and breathing management are required before progressing to
circulation.

B - Airway and breathing management is required before circulation management, the IO route can be used if IV
access is difficult to obtain.

D - This position is used for infants in view of their large occiputs.

E - This is the correct rate for children, however airway and breathing are the next step in management of
resuscitations.

https://mypastest.pastest.com/Secure/TestMe/Browser/436619#Top[‫ ص‬11:41:29 10/12/1437]


MyPastest

46649

Next Question
Previous Question Tag Question

Feedback

End Session
Difficulty: Average

Peer Responses

Session Progress

Responses Correct: 0

Responses Incorrect: 110

Responses Total: 110

Responses - % Correct: 0%

Blog
About Pastest
Contact Us
Help

© Pastest 2016

https://mypastest.pastest.com/Secure/TestMe/Browser/436619#Top[‫ ص‬11:41:29 10/12/1437]


MyPastest

Prefer to use the old MyPastest? Access it here »

Back to Filters

Question 91 of 120

A 9-month-old enters your emergency department with increased


work of breathing. She has a respiratory rate of
75 with saturations of 89%; she has nasal flaring, tracheal tug, intercostal and subcostal recessions, with an
occasional wet cough. On auscultation of her chest she is diffusely wheezy with crackles. Her temperature is 38
degrees.

What is the most likely diagnosis?

A Asthma

B Bronchiolitis

C Croup

D Pneumonia

E Viral induced wheeze

Explanation
The presence of cough, increased work of breathing and wheeze/crackles in a child under 2 years makes the
diagnosis of bronchiolitis most likely. This can be associated with a fever and reduced feeds. Management is
supportive with supplemental oxygen and nasogastric feeds or intravenous fluids. If the child tires despite these
measures then either non-invasive ventilation or tracheal intubation may be required.

A - This is not common under the age


of 1 year, however should be suspected if there is solely wheeze and a
family history of atopy.

C - This is associated with a barking seal like cough and stridor.

D - This is not common in children under 1 year of age, but should be suspected if there is a high temperature and
persistent focal crackles.

E - This is usually seen in pre-school children and is a wheezy episode associated with a viral illness. No interval

https://mypastest.pastest.com/Secure/TestMe/Browser/436619#Top[‫ ص‬11:41:51 10/12/1437]


MyPastest

symptoms occur, unlike with asthma.

Ref: https://www.nice.org.uk/guidance/ng9 46650

Next Question
Previous Question
Tag Question
End Session
Feedback

Difficulty: Average

Peer Responses

Session Progress

Responses Correct: 0

Responses Incorrect: 111

Responses Total: 111

Responses - % Correct: 0%

Blog
About Pastest
Contact Us
Help

© Pastest 2016

https://mypastest.pastest.com/Secure/TestMe/Browser/436619#Top[‫ ص‬11:41:51 10/12/1437]


MyPastest

Prefer to use the old MyPastest? Access it here »

Back to Filters

Question 92 of 120

A 4-year-old is brought to your resus having fallen into a pond. He is hypothermic at 29 degrees and when you
attach the cardiac pads you see that he has a shockable rhythm.

How should defibrillation be given?

A Avoid as temperature is < 30 degrees

B Give 3 shocks only whilst < 30 degrees

C Give shocks but double the time between them

D Warm to 32 degrees then start shocks

E Warm to 35 degrees then start shocks

Explanation
Hypothermia is common with drowning episodes. Whilst the temperature is < 30 degrees the chances of
converting an arrhythmia to a normal sinus rhythm is poor. Three shocks should be tried and if unsuccessful
further shocks should only be given once the core temperature is > 30 degrees.

A - Drugs should be avoided at < 30 degrees, however 3 shocks can be attempted.

C - Shocks remain at the same frequency as normal, if they are indicated. The interval between drug doses is
doubled for a temperature between 30-35 degrees.

D – 32 degrees is the temperature at which a patient should be warmed to before stopping resuscitation attempts.

E – Above 35 degrees shocks and drugs can be given as per normal.

Ref: APLS 5th edition


46651

https://mypastest.pastest.com/Secure/TestMe/Browser/436619#Top[‫ ص‬11:42:14 10/12/1437]


MyPastest

Next Question
Tag Question

Previous Question
Feedback

Difficulty: Average End Session


Peer Responses

Session Progress

Responses Correct: 0

Responses Incorrect: 112

Responses Total: 112

Responses - % Correct: 0%

Blog
About Pastest
Contact Us
Help

© Pastest 2016

https://mypastest.pastest.com/Secure/TestMe/Browser/436619#Top[‫ ص‬11:42:14 10/12/1437]


MyPastest

Prefer to use the old MyPastest? Access it here »

Back to Filters

Question 93 of 120

An 18-month-old is walking in the street when he suddenly runs away from his mother towards the road. She
catches up with him and grabs his right arm to bring him towards her. As she does this she hears a click and the
child is upset and not using his right arm, holding it pronated against his body. There is no deformity and the limb
is well perfused.

What is the most likely diagnosis?

A Dislocated Shoulder

B Fractured Clavicle

C Pulled Elbow

D Sprained Elbow

E Toddlers Fracture

Explanation
This is a common injury in young children that occurs following a history of being pulled. The radial head
partially slips out from under the ligament holding it in place. In a child with a
history suggestive of this, no X-ray
is required, instead, manipulating
the joint will reduce it into place. The child should start to use the
arm again
soon after.

A - Traumatic shoulder dislocation in young children is rare.

B - Clavicle fractures are not uncommon although usually occur following a fall onto an outstretched hand or
direct trauma, which has not occurred here.

D - A sprain may be associated with swelling or bruising depending on the mechanism. The child may be reluctant
to use the arm but should be able to with analgesia.

E - Toddlers Fractures occur at the tibia.

https://mypastest.pastest.com/Secure/TestMe/Browser/436619#Top[‫ ص‬11:42:37 10/12/1437]


MyPastest

46652

Next Question
Previous Question Tag Question

Feedback

End Session
Difficulty: Average

Peer Responses

Session Progress

Responses Correct: 0

Responses Incorrect: 113

Responses Total: 113

Responses - % Correct: 0%

Blog
About Pastest
Contact Us
Help

© Pastest 2016

https://mypastest.pastest.com/Secure/TestMe/Browser/436619#Top[‫ ص‬11:42:37 10/12/1437]


MyPastest

Prefer to use the old MyPastest? Access it here »

Back to Filters

Question 94 of 120

A 2-year-old is playing on a trampoline when he lands awkwardly, twisting his leg and starts to cry. He is
reluctant to use his right leg and brought straight to the emergency department. On examination he is unable to
weight bear on the leg but is no longer complaining of pain and playing happily on the examination bed.

What is the likely diagnosis?

A Non-accidental injury

B Septic arthritis

C Slipped Upper Femoral Epiphysis

D Toddlers fracture

E Transient Synovitis

Explanation
There has been a history of trauma and therefore a
fracture is a possibility. Toddlers fractures are undisplaced
fractures to the tibia. They occur due to twisting injuries, which may be relatively minor. The child will be
reluctant to weight bear and may
have pain over the site of the fracture on palpation.

A - NAI should always be considered, especially in the younger age groups. There are no pointers in the history
that suggest this as a cause here.

B – Septic arthritis can cause limping and pain however there is no history of fever and no joint is red or inflamed
making this unlikely.

C – SUFE can cause limp in older children who may be overweight and hold the leg externally rotated.

E – Transient synovitis is a common cause of limp and pain following a coryzal illness, with no history of trauma.
46653

https://mypastest.pastest.com/Secure/TestMe/Browser/436619#Top[‫ ص‬11:43:02 10/12/1437]


MyPastest

Next Question
Tag Question

Previous Question
Feedback

Difficulty: Average End Session


Peer Responses

Session Progress

Responses Correct: 0

Responses Incorrect: 114

Responses Total: 114

Responses - % Correct: 0%

Blog
About Pastest
Contact Us
Help

© Pastest 2016

https://mypastest.pastest.com/Secure/TestMe/Browser/436619#Top[‫ ص‬11:43:02 10/12/1437]


MyPastest

Prefer to use the old MyPastest? Access it here »

Next Question

Back to Filters

Question 95 of 120

A 5-year-old with a known background of atopy, has stridor, facial swelling, abdominal pain and a rash.

What dose and method of delivery of adrenaline should paramedics give?

A 0.15ml 1:1000 IM

B 0.15ml 1:1000 IV

C 0.15ml 1:10,000 IM

D 0.5ml 1:1000 IM

E 0.3ml 1:1000 IV

Explanation
Intramuscular adrenaline is given in anaphylaxis.
For this age group a dose of 0.15ml 1:1000 (150micrograms) is
advised as per the APLS algorithm.

B – Adrenaline is given intramuscularly in anaphylaxis not intravenously.

C – Adrenaline is given at a strength of 1:1000 in anaphylaxis as opposed to 1:10,000 in CPR.

D – This is the dose for children aged more than 12 years.

E – This is the dose given for children aged 6-12 years however not by the IV route.

Reference: APLS 5thedition


46654

https://mypastest.pastest.com/Secure/TestMe/Browser/436619#Top[‫ ص‬11:43:23 10/12/1437]


MyPastest

Tag Question

Feedback

Difficulty: Average
Previous Question
Peer Responses

End Session

Session Progress

Responses Correct: 0

Responses Incorrect: 115

Responses Total: 115

Responses - % Correct: 0%

Blog
About Pastest
Contact Us
Help

© Pastest 2016

https://mypastest.pastest.com/Secure/TestMe/Browser/436619#Top[‫ ص‬11:43:23 10/12/1437]


MyPastest

Prefer to use the old MyPastest? Access it here »

Back to Filters

Question 96 of 120

You are called to the emergency department as the paediatric SHO on call. A baby has just been born in the back
of the ambulance and
is not breathing with a heart rate of 40. On arrival in resus the baby
is placed on a warm
resuscitaire and dried.

What is your next step in management?

A Open airway to the neutral position and give 5 inflation breaths

B Open airway to the sniffing position and give 5 inflation breaths

C Start CPR at a ratio of 3 compressions to 1 breath

D Start CPR at a ratio of 15 compressions to 2 breaths

E Start CPR at a ratio of 30 compressions to 2 breaths

Explanation
Neonates have large occiputs which can result in their neck flexing and obscuring their airway. Placing their head
in the neutral position opens the airway. As the majority of neonatal arrests are from respiratory causes, giving 5
inflation breaths (prolonged breaths over 3 seconds as opposed to faster ventilation breaths) may be enough to
oxygenate blood entering the heart to increase
contractility, if not CPR at a ratio of 3:1 may be required.

B -
The sniffing position is used in children above 28 days of life, to open the airway. In children, rescue breaths
are given rather than inflation breaths.

C – This is the correct ratio to use in neonates, however as respiratory causes predominate in neonatal arrests,
inflation breaths are given first through an open airway. If the Heart rate remains <60 and unresponsive to breaths,
CPR may be started.

D – This ratio is used for children over 28 days of life.

E – This ratio is used in adults and not for neonates or children.

https://mypastest.pastest.com/Secure/TestMe/Browser/436619#Top[‫ ص‬11:43:45 10/12/1437]


MyPastest

Ref: APLS (6th ed) and NLS (3rd ed)


46655

Next Question
Previous Question
Tag Question

Feedback End Session

Difficulty: Average

Peer Responses

Session Progress

Responses Correct: 0

Responses Incorrect: 116

Responses Total: 116

Responses - % Correct: 0%

Blog
About Pastest
Contact Us
Help

© Pastest 2016

https://mypastest.pastest.com/Secure/TestMe/Browser/436619#Top[‫ ص‬11:43:45 10/12/1437]


MyPastest

Prefer to use the old MyPastest? Access it here »

Back to Filters

Question 97 of 120

A 10-year-old presents to your resus. He was a pedestrian hit by a car at speed. He is bleeding and you suspect
moderate blood loss.

Which of the features below does NOT support this?

A Absent peripheral pulses

B Lethargy

C Minimal urine output

D Significant hypotension

E Tachycardia

Explanation
ATLS defines moderate blood loss as 30%-45% volume loss (with mild losses being <30%). This can be
associated with all of the above features, however blood pressure will be in the low to normal range. It is only with
severe blood loss (>45%) that significant hypotension in children occurs. A narrowed pulse pressure will be seen
with moderate blood loss as peripheral vascular resistance increases and this should be looked for rather than
systolic blood pressure alone.

A - This may be associated with thready central pulses.

B - Initial under perfusion of the brain may result in anxiety and confusion, however this will give way to lethargy
and eventually coma.

C - Output will be less than 1-2ml/kg/hr depending on the child’s


age. This will improve when fluid resuscitation
is adequate.

E - Heart rate increases as blood is lost, however bradycardia ensues after severe blood loss and is a worrying sign.

https://mypastest.pastest.com/Secure/TestMe/Browser/436619#Top[‫ ص‬11:44:06 10/12/1437]


MyPastest

Ref: ATLS 9th edition


46656

Next Question
Previous Question
Tag Question

Feedback
End Session

Difficulty: Average

Peer Responses

Session Progress

Responses Correct: 0

Responses Incorrect: 117

Responses Total: 117

Responses - % Correct: 0%

Blog
About Pastest
Contact Us
Help

© Pastest 2016

https://mypastest.pastest.com/Secure/TestMe/Browser/436619#Top[‫ ص‬11:44:06 10/12/1437]


MyPastest

Prefer to use the old MyPastest? Access it here »

Back to Filters

Question 98 of 120

A 5-year-old child presents having accidently poured a cup of


boiling water over himself. He has blisters to his
entire chest and abdomen as well as the front of his right arm and hand, he also has areas of erythema to the right
thigh. You call your local burn centre and they ask what percentage of burns the child has.

Which of the following is correct?

A 10%

B 18%

C 22%

D 22.5%

E 30%

Explanation
The Lund and Browder formula has the torso at 13%
and the front of the entire upper limb as 5%. Areas of
erythema are not
included when calculating burn area. As this child has >10% burns and the hand is involved,
referral to a burns unit is indicated. (1)

A – The torso alone would account for 13%

C – 22% would be true if the areas of erythema were included, which they should not be.

D - 22.5% is given by the rule of 9’s formula, however cannot be used in children <14yrs.(2)

E –This is far too high a percentage and not supported by any formula.

Ref:
1. www.frontlinefirstaid.ca/free-downloads/lund-and-browder-burn-chart-emr-bc-emalb.pdf
2. Advanced paediatric Life Support
46657

https://mypastest.pastest.com/Secure/TestMe/Browser/436619#Top[‫ ص‬11:44:29 10/12/1437]


MyPastest

Next Question
Tag Question
Previous Question
Feedback

Difficulty: Average
End Session

Peer Responses

Session Progress

Responses Correct: 0

Responses Incorrect: 118

Responses Total: 118

Responses - % Correct: 0%

Blog
About Pastest
Contact Us
Help

© Pastest 2016

https://mypastest.pastest.com/Secure/TestMe/Browser/436619#Top[‫ ص‬11:44:29 10/12/1437]


MyPastest

Prefer to use the old MyPastest? Access it here »

Next Question

Back to Filters

Question 99 of 120

A 4-year-old presents with a burn on one leg with splash marks, which you suspect is non-accidental.

Which answer does NOT support this?

A Absence of splash marks

B Burn in the shape of an object

C Delayed presentation

D Inconsistent and changing story

E Ipsilateral burn with splash marks

Explanation
Bilateral burns are more likely to reflect NAI. In this answer the burn is on one leg and there are splash marks, this
could be consistent with pouring a hot drink onto himself, which splashes him as it falls.

A – Absence of splash marks may suggest a dipping injury, where the child is lowered into hot water. There may
also be a glove and stocking distribution or legs and buttocks
distribution. Clear water lines may also be seen.

B – Contact burns will be in the shape or an object i.e. iron or cigarette and are indicative of an abusive injury.

C – Delayed presentation may be seen in all types of NAI.

D – A story that does not fit the child's developmental age or burn pattern is suspicious for NAI, as is a story that
cannot be remembered or keeps changing.
46658

https://mypastest.pastest.com/Secure/TestMe/Browser/436619#Top[‫ ص‬11:45:20 10/12/1437]


MyPastest

Tag Question

Feedback

Difficulty: Average
Previous Question
Peer Responses

End Session

Session Progress

Responses Correct: 0

Responses Incorrect: 119

Responses Total: 119

Responses - % Correct: 0%

Blog
About Pastest
Contact Us
Help

© Pastest 2016

https://mypastest.pastest.com/Secure/TestMe/Browser/436619#Top[‫ ص‬11:45:20 10/12/1437]


MyPastest

Prefer to use the old MyPastest? Access it here »

Back to Filters

Question 100 of 120

A 5-year-old presents to your emergency department having sustained 15% burns following a house fire. You
assess his airway and breathing which are stable and move to circulation. You know that as he has >10% burns to
his body, he will require additional fluids on top of his maintenance.

Which formula below is correct in calculating the additional fluid?

A Percentage burn x 10

B Percentage burn x weight (kg) + 4

C Percentage burn x weight (kg) x 4

D Percentage burn + weight (kg) x 4

E Percentage burn + weight (kg) + 10

Explanation
This is the amount required in addition to maintenance IV fluids. Half of this will be given in the first 8 hours.
It is
a guide to a starting rate, subsequent fluid rates will be guided by urine output, which needs to be kept at
>2ml/kg/hr.

A - Fluid boluses may be given at 10ml/kg but additional fluids are not calculated by this formula.

B - Multiply by four would be the correct formula.

D - Multiply by weight would be the correct formula.

E - This is incorrect, you multiply by weight and then by four. 10ml/kg may be given as a fluid bolus.

Ref: APLS 5th Edition; ATLS 9th Edition


46659

https://mypastest.pastest.com/Secure/TestMe/Browser/436619#Top[‫ ص‬11:45:44 10/12/1437]


MyPastest

Next Question
Tag Question

Previous Question
Feedback

Difficulty: Average End Session


Peer Responses

Session Progress

Responses Correct: 0

Responses Incorrect: 120

Responses Total: 120

Responses - % Correct: 0%

Blog
About Pastest
Contact Us
Help

© Pastest 2016

https://mypastest.pastest.com/Secure/TestMe/Browser/436619#Top[‫ ص‬11:45:44 10/12/1437]


MyPastest

Prefer to use the old MyPastest? Access it here »

Back to Filters

Question 101 of 120

A 10-year-old patient presents with respiratory difficulty following being trapped in a house fire. He was removed
from an enclosed room by the fire crew and brought to your resus.

Which of the following is unlikely to be a cause of his respiratory compromise?

A Carbon monoxide poisoning

B Circumferential chest burn

C Inhalation injury

D Tension pneumothorax

E Upper airway oedema

Explanation
In an enclosed fire the other causes listed above
are more likely to cause respiratory embarrassment. Tension
pneumothorax is more associated with trauma and positive pressure ventilation. Had there been a blast injury
associated with the fire, then this may have been a possibility.

A - Values >20% can cause symptoms ranging from headache to death at higher levels. Cherry
red skin is rare.
Treatment is with high flow oxygen.

B – If ventilation is a problem then surgical escharotomy may be required.

C – This causes inflammation of the airways, cell necrosis, infection and an increase in mortality.

E – Thermal pharyngeal injury can result in oedema. This should be suspected if there are burns or carbon deposits
around the mouth. Early intubation may be required.

Ref: ATLS 9th edition


46660

https://mypastest.pastest.com/Secure/TestMe/Browser/436619#Top[‫ ص‬11:46:41 10/12/1437]


MyPastest

Next Question
Tag Question

Previous Question
Feedback

Difficulty: Average End Session


Peer Responses

Session Progress

Responses Correct: 0

Responses Incorrect: 121

Responses Total: 121

Responses - % Correct: 0%

Blog
About Pastest
Contact Us
Help

© Pastest 2016

https://mypastest.pastest.com/Secure/TestMe/Browser/436619#Top[‫ ص‬11:46:41 10/12/1437]


MyPastest

Prefer to use the old MyPastest? Access it here »

Back to Filters

Question 102 of 120

A 6-year-old presents with palpitations and light headedness.


An ECG reveals a heart rate of 200. You try vagal
manoeuvres and give
repeated boluses of adenosine up to 500mcg/kg.

What is the next most appropriate step in management?

A Increase dose of Adenosine to 600mcg/kg

B Repeat Adenosine at 500mcg/kg

C Synchronous DC shock 1J/kg

D Synchronous DC shock 2J/kg

E Synchronous DC shock 4J/kg

Explanation
At this step, APLS advises synchronous DC shock, amiodarone, or cardiology advice to be sought for alternative
antiarrhythmics. Therefore synchronous DC shock 1J/kg is the most suitable answer. If the SVT is prolonged then
the child may develop signs of shock as oxygen cannot be delivered to the myocardium. Adenosine has a short
half-life and should be given through a large vein
with a saline flush.

A - 500mcg/kg is the maximum single dose that is advised by APLS guidelines.

B - As Adenosine has already failed to provide an effect at the highest


recommended dose, it is reasonable to try
another method, such as synchronised shocks.

D - The first shock given is 1J/kg followed by 2J/kg for subsequent shocks.

E - 4J/kg is the voltage given in cardiac arrest.

Ref: APLS 5th edition


46661

https://mypastest.pastest.com/Secure/TestMe/Browser/436619#Top[‫ ص‬11:47:05 10/12/1437]


MyPastest

Next Question
Tag Question

Previous Question
Feedback

Difficulty: Average End Session


Peer Responses

Session Progress

Responses Correct: 0

Responses Incorrect: 122

Responses Total: 122

Responses - % Correct: 0%

Blog
About Pastest
Contact Us
Help

© Pastest 2016

https://mypastest.pastest.com/Secure/TestMe/Browser/436619#Top[‫ ص‬11:47:05 10/12/1437]


MyPastest

Prefer to use the old MyPastest? Access it here »

Next Question

Back to Filters

Question 103 of 120

Following major trauma in children, catastrophic major haemorrhage is addressed prior to airway, breathing and
circulation. Uncontrolled haemorrhage is suspected in a trauma patient brought to your department.

Which method below is not advised?

A Activating major haemorrhage protocol

B Pelvic Binder

C Tourniquet

D Tranexamic acid

E Vigorous fluid boluses

Explanation
Vigorous fluid boluses may result in dislodging early clots and diluting coagulation factors, exacerbating major
haemorrhage. Permissive hypotension is used to prevent this.

A – Major haemorrhage protocols ensure that blood products are given at the optimal ratios in a timely manner.

B – Pelvic binders stabilise a fractured pelvis and apply pressure to minimise bleeding.

C – Use of tourniquets in major arterial bleeds may be life saving.

D – Tranexamic acid is an anti-fibrinolytic drug that is given at


a dose of 15mk/kg. A loading dose is given
followed by an infusion.

Ref: APLS 2015; ATLS 9th edition


46662

https://mypastest.pastest.com/Secure/TestMe/Browser/436619#Top[‫ ص‬11:47:26 10/12/1437]


MyPastest

Tag Question

Feedback

Previous Question
Difficulty: Average

Peer Responses
End Session

Session Progress

Responses Correct: 0

Responses Incorrect: 123

Responses Total: 123

Responses - % Correct: 0%

Blog
About Pastest
Contact Us
Help

© Pastest 2016

https://mypastest.pastest.com/Secure/TestMe/Browser/436619#Top[‫ ص‬11:47:26 10/12/1437]


MyPastest

Prefer to use the old MyPastest? Access it here »

Back to Filters

Question 104 of 120

An 8-year-old boy is hit by a speeding car in a 40mph street.


He has sustained a head injury and is intubated at the
scene for reduced GCS. On arrival in the emergency department he is stable with normal examination of the chest
and some bruising to the lower abdomen.

Which imaging modalities should be used?

A CT Head and Abdomen/Pelvis with CXR and X-ray C-spine

B CT Head, C-spine and Abdomen/Pelvis

C CT Head, C-spine and Abdomen/Pelvis with CXR

D MRI Head

E Whole body CT

Explanation
The C-spine cannot be cleared, this alongside a multi-trauma presentation means a CT C-spine is indicated rather
than X-rays alone. The imaging modality for blunt trauma to the Chestis CXR, if this shows significant thoracic
trauma,a CT chest should be considered. These recommendations are from the Royal College of Radiologists and
based on injury types in children and trying to maintain as low as reasonably achievable (ALARA) radiation
doses. (1)

A
– This child is at risk of C-Spine injury following the mechanism
of trauma. He had a reduced GCS, has
undergone a multi-region trauma and is now intubated. NICE Head injury guidelines therefore recommend using
CT. (2)

B – Imaging of the chest is required following blunt trauma and for endotracheal tube position.

D – If there were concerns of abnormal neurology then spinal MRI could be considered. (1)

E - Whole Body CT is not recommended in children in view of the high radiation doses.

https://mypastest.pastest.com/Secure/TestMe/Browser/436619#Top[‫ ص‬11:47:51 10/12/1437]


MyPastest

Ref:
1. https://www.rcr.ac.uk
2. https://www.nice.org.uk/guidance/cg176
Next Question 46663

Previous Question

End
Tag Session
Question

Feedback

Difficulty: Average

Peer Responses

Session Progress

Responses Correct: 0

Responses Incorrect: 124

Responses Total: 124

Responses - % Correct: 0%

Blog
About Pastest
Contact Us
Help

© Pastest 2016

https://mypastest.pastest.com/Secure/TestMe/Browser/436619#Top[‫ ص‬11:47:51 10/12/1437]


MyPastest

Prefer to use the old MyPastest? Access it here »

Back to Filters

Question 105 of 120

A 13-year-old boy falls down a flight of stairs. There is no loss of consciousness, his GCS is 15 and he has not
vomited, however, he
is complaining of neck pain and as such is brought to your department on a trauma board.
He remains orientated with no distracting injuries. On palpation of his C-spine he complains of pain.

Which imaging modalities will you request?

A CT C-spine

B CT head and C-spine

C CT head with C-spine X ray (3 view)

D C-spine X ray (3 view)

E No imaging required

Explanation
He has a dangerous mechanism of injury (fall >
5 steps) and C-spine tenderness, therefore he requires 3-view
cervical spine X rays prior to assessing neck movements.

A – CT C-spine. He does not have any risk factors to warrant this and the increased risk of radiation to the thyroid
gland which it entails.

B – CT head and C-spine. His head injury is not significant enough to warrant CT head and he has no risk factors
to warrant a CT C-spine.

C – CT head with C-spine X ray. His head injury is not significant enough to warrant CT head in addition to X ray
imaging of the spine.

E – No imaging. He fulfils the criteria for X-ray C spine prior to assessing movements of the spine.

Ref: http://pathways.nice.org.uk/pathways/head-injury 46664

https://mypastest.pastest.com/Secure/TestMe/Browser/436619#Top[‫ ص‬11:48:12 10/12/1437]


MyPastest

Next Question
Tag Question

Previous Question
Feedback

Difficulty: Average End Session

Peer Responses

Session Progress

Responses Correct: 0

Responses Incorrect: 125

Responses Total: 125

Responses - % Correct: 0%

Blog
About Pastest
Contact Us
Help

© Pastest 2016

https://mypastest.pastest.com/Secure/TestMe/Browser/436619#Top[‫ ص‬11:48:12 10/12/1437]


MyPastest

Prefer to use the old MyPastest? Access it here »

Back to Filters

Question 106 of 120

A 6-year-old falls from a climbing frame, hitting his head. He cries immediately then vomits 4 times. He arrives to
your department 1
hour later with a GCS of 15 and a 6cm haematoma to the right side of his head. Neurological
examination is unremarkable and he can remember the day’s events.

What is the next step in his management?

A CT head within one hour

B Discharge home

C Discuss with neurosurgery

D Observe for 4 hours from the time of injury

E Observe for 24 hours from the time of injury on the ward

Explanation
NICE guidelines were updated so that if a child has more than 2 vomits as their only risk factor, then instead of
having
a CT head, they are observed for 4 hours from the time of head injury. If during this observation they have
further vomits, a reduction in consciousness or become drowsy, then a CT head should be obtained within
one
hour.

A – Vomiting alone is no longer an indication for CT head. The haematoma is large, however only requires
imaging if >5cm in a child <1yr.

B – This child needs a period of observation in view of having more than 2 vomits.

C – This child is alert with no focal neurology and does not fulfil the criteria for involving neurosurgery.

E – Whilst a responsible adult is advised to stay with the child for 24 hours from the time of injury once
discharged from hospital, this
does not need to occur within the hospital for minor head injuries.

https://mypastest.pastest.com/Secure/TestMe/Browser/436619#Top[‫ ص‬11:48:34 10/12/1437]


MyPastest

Ref: https://www.nice.org.uk/guidance/cg176 46665

Next Question
Previous Question Tag Question

Feedback
End Session
Difficulty: Average

Peer Responses

Session Progress

Responses Correct: 0

Responses Incorrect: 126

Responses Total: 126

Responses - % Correct: 0%

Blog
About Pastest
Contact Us
Help

© Pastest 2016

https://mypastest.pastest.com/Secure/TestMe/Browser/436619#Top[‫ ص‬11:48:34 10/12/1437]


MyPastest

Prefer to use the old MyPastest? Access it here »

Back to Filters

Question 107 of 120

A 15-year-old falls off his quad bike at speed. He complains of pain in his neck and paraesthesia in his limbs. He
has multiple bleeding contusions mixed in with mud. He is bruised across his chest from where he went over the
handle bars. He is hypotensive with a normal
heart rate and low saturations.

What is the cause of his shock?

A Cardiogenic

B Haemorrhagic

C Neurogenic

D Septic

E Spinal

Explanation
His C-spine injury has caused loss of sympathetic
tone to the vasculature and heart, resulting in vasodilation and
the absence of a tachycardia. Fluid resuscitation alongside inotropic support will be required to ensure tissue
oxygenation.

A - Cardiogenic shock can occur after chest trauma. Patients may have evidence of arrhythmia, or muffled heart
sounds.

B - Haemorrhagic shock is the most common cause of shock in trauma, however is usually associated with a
tachycardia.

D - Septic shock is unlikely as he has only dirty wounds and not enough time has elapsed for these to become
infected.

E - Spinal shock is damage to the spinal cord resulting in flaccidity and loss of reflexes.

https://mypastest.pastest.com/Secure/TestMe/Browser/436619#Top[‫ ص‬11:48:56 10/12/1437]


MyPastest

Ref: ATLS 9th edition


46666

Next Question
Previous Question
Tag Question

Feedback
End Session

Difficulty: Average

Peer Responses

Session Progress

Responses Correct: 0

Responses Incorrect: 127

Responses Total: 127

Responses - % Correct: 0%

Blog
About Pastest
Contact Us
Help

© Pastest 2016

https://mypastest.pastest.com/Secure/TestMe/Browser/436619#Top[‫ ص‬11:48:56 10/12/1437]


MyPastest

Prefer to use the old MyPastest? Access it here »

Next Question

Back to Filters

Question 108 of 120

You are asked to clear the C-spine of a 15-year-old child who


has just arrived by ambulance. He was playing
football and was tackled
to the ground.

Which of the below would prevent you clearing his C-spine?

A 5mg oramorph

B Absence of C-spine tenderness

C Femoral fracture

D GCS 14

E Intact sensation

Explanation
APLS advises that a C-spine cannot be cleared if there is a painful distracting injury, such as that presenting here.

A
- Intoxication or sedative drugs prevent you from clearing the C-spine.
This dose of oramorph would be
unlikely to present any such issue.

B
- With no tenderness and no other risk factors, the patient can be asked to move the neck 45 degrees to the left
and right to see if this elicits pain.

D - Normal alertness is required when assessing the spine.

E - Any focal neurological deficit would prevent you from being able to clear the C-spine.

Ref: APLS 5th edition


46667

https://mypastest.pastest.com/Secure/TestMe/Browser/436619#Top[‫ ص‬11:49:18 10/12/1437]


MyPastest

Tag Question

Feedback

Previous Question
Difficulty: Average

Peer Responses
End Session

Session Progress

Responses Correct: 0

Responses Incorrect: 128

Responses Total: 128

Responses - % Correct: 0%

Blog
About Pastest
Contact Us
Help

© Pastest 2016

https://mypastest.pastest.com/Secure/TestMe/Browser/436619#Top[‫ ص‬11:49:18 10/12/1437]


MyPastest

Prefer to use the old MyPastest? Access it here »

Back to Filters

Question 109 of 120

A 12-year-old boy presents to your emergency department with evidence of a C-spine injury; he is alert and
complaining of pain in his
neck. His head has been immobilised in the midline, by the paramedic crew.

Which of the below mechanisms would make you obtain a CT C-spine within an hour of arrival?

A Axial load to head

B Ejection from a car

C Focal peripheral neurological signs

D Quadbike accident

E Rollover motor vehicle accident

Explanation
NICE recommends that if a child has sustained a head injury and has focal peripheral neurological signs, then a
head CT within one hour is required. The criteria aims to be robust so as to avoid unnecessary radiation doses to
the thyroid gland.

A – This may be obtained from a diving mechanism. If there are no other risk factors for CT, then this requires 3
view C-Spine X-rays within an hour.

B – Being thrown out of a motor vehicle is


an indication for C-spine X-rays assuming there are no risk factors for
CT present.

D – Accidents involving motorised recreational vehicles such as these, require C-spine X-rays assuming there are
no risk factors for CT present.

E - If there are no other risks factors for CT, then this requires 3 view C-Spine X-rays within an hour.

Ref:

https://mypastest.pastest.com/Secure/TestMe/Browser/436619#Top[‫ ص‬11:49:41 10/12/1437]


MyPastest

https://www.nice.org.uk/guidance/cg176
46668

Next Question
Previous Question
Tag Question

Feedback
End Session

Difficulty: Average

Peer Responses

Session Progress

Responses Correct: 0

Responses Incorrect: 129

Responses Total: 129

Responses - % Correct: 0%

Blog
About Pastest
Contact Us
Help

© Pastest 2016

https://mypastest.pastest.com/Secure/TestMe/Browser/436619#Top[‫ ص‬11:49:41 10/12/1437]


MyPastest

Prefer to use the old MyPastest? Access it here »

Back to Filters

Question 110 of 120

A 15-year-old is riding as a passenger in his friend’s car that has crashed against a bridge wall. He has sustained
major crushing injuries to both legs.

What abnormality would NOT be expected?

A Acute renal failure

B Elevated Creatine Kinase

C Hypocalcaemia

D Hypokalaemia

E Metabolic acidosis

Explanation
This child is at risk of crush syndrome (a life threatening injury) where traumatic rhabdomyolysis can result in
acute renal failure and electrolyte disturbances. Hyperkalaemia is seen as it
leaks into the circulation from the
damaged muscle cells. If left untreated it can result in arrhythmias and cardiac arrest.

A – As the muscle breaks down it releases myoglobin which can result


in renal failure. This can be prevented by
maintaining a high urine output through IV fluids and diuretics. Myoglobin causes darkening of the urine.

B – CK is an enzyme released from damaged muscle and will be elevated.

C – Hypocalcaemia may be seen initially as calcium is deposited within muscle.

E – Acidosis may occur from the release of lactic acid from damaged muscle.

Ref: ATLS 9th edition


46669

https://mypastest.pastest.com/Secure/TestMe/Browser/436619#Top[‫ ص‬11:50:04 10/12/1437]


MyPastest

Next Question
Tag Question

Previous Question
Feedback

Difficulty: Average End Session


Peer Responses

Session Progress

Responses Correct: 0

Responses Incorrect: 130

Responses Total: 130

Responses - % Correct: 0%

Blog
About Pastest
Contact Us
Help

© Pastest 2016

https://mypastest.pastest.com/Secure/TestMe/Browser/436619#Top[‫ ص‬11:50:04 10/12/1437]


MyPastest

Prefer to use the old MyPastest? Access it here »

Back to Filters

Question 111 of 120

A 9-year-old presents to your resus following a road traffic collision with chest injury. She is intubated and
ventilated. She suddenly becomes hypotensive and tachycardic. Her neck veins are distended and trachea
displaced to one side.

What is the next step in management?

A Change endotracheal tube

B Chest drain insertion

C Needle Decompression

D Oxygen therapy

E Pericardiocentesis

Explanation
These signs are suggestive of a tension pneumothorax, due to air entering into the pleural space. The positive
pressure ventilation in this case is the likely contributing factor in a
damaged lung. If the chest is auscultated,
reduced air entry and a hyper-resonant lung field will be found. Needle decompression in the second intercostal
space midclavicular line may be life-saving.

A
- When mechanical ventilation becomes difficult you should consider the
possible causes. The mnemonic
‘DOPE’ stands for dislodged tube, obstructed tube, pneumothorax and equipment failure.

B - This will ultimately be required, however will take time to set up and perform.

D - In a simple pneumothorax in an un-intubated child, high flow oxygen is given to accelerate air absorption.

E - This is done in cardiac tamponade. Here the neck veins may be distended and heart sounds muffled.

Ref: ATLS 9th edition.


46670

https://mypastest.pastest.com/Secure/TestMe/Browser/436619#Top[‫ ص‬11:50:27 10/12/1437]


MyPastest

Next Question
Tag Question
Previous Question
Feedback

Difficulty: Average
End Session

Peer Responses

Session Progress

Responses Correct: 0

Responses Incorrect: 131

Responses Total: 131

Responses - % Correct: 0%

Blog
About Pastest
Contact Us
Help

© Pastest 2016

https://mypastest.pastest.com/Secure/TestMe/Browser/436619#Top[‫ ص‬11:50:27 10/12/1437]


MyPastest

Prefer to use the old MyPastest? Access it here »

Back to Filters

Question 112 of 120

A 6-year-old is admitted to the ward with a cast for a tibial


fracture. He is complaining of pain and you are
worried that he may have a compartment syndrome injury, as you are aware that this can be a limb threatening
injury.

Which of the following is a late sign in compartment syndrome?

A Absent distal pulses

B Altered sensation

C Increasing pain

D Pain on passive stretching of the muscle

E Tense compartment

Explanation
An absent distal pulse, is a late sign in compartment syndrome and is therefore not relied upon to make the
diagnosis.

B – Sensory deficits from ischaemic nerve damage are seen and regular testing should be employed if the
diagnosis is suspected.

C – Pain uncontrolled by analgesia or out of proportion to the injury is an early sign in compartment syndrome.

D – Pain exacerbated by passive movement is seen in compartment syndrome.

E – A tense compartment on palpation may be felt.

Ref:
1. ATLS 9th edition
2. www.uptodate.com/contents/acute-compartment-syndrome-of-the-extremities
46671

https://mypastest.pastest.com/Secure/TestMe/Browser/436619#Top[‫ ص‬11:50:49 10/12/1437]


MyPastest

Next Question
Tag Question
Previous Question
Feedback

Difficulty: Average
End Session

Peer Responses

Session Progress

Responses Correct: 0

Responses Incorrect: 132

Responses Total: 132

Responses - % Correct: 0%

Blog
About Pastest
Contact Us
Help

© Pastest 2016

https://mypastest.pastest.com/Secure/TestMe/Browser/436619#Top[‫ ص‬11:50:49 10/12/1437]


MyPastest

Prefer to use the old MyPastest? Access it here »

Back to Filters

Question 113 of 120

A 13-year-old presents with an open fracture to his right tibia following a football injury.

Which of the following should NOT be done in the Emergency department?

A Check tetanus status

B Immobilise

C Probe the wound

D Refer to orthopaedics

E Start IV antibiotics

Explanation
An open fracture is one that communicates with the external environment and therefore carries a risk of infection.
Probing the wound is not advised as it can dislodge debris further into the wound and introduce infection.

A - Tetanus status should be ascertained and prophylaxis given if required.

B - Once the wound has been examined and neurovascular status obtained, the limb should be immobilised. This
will help reduce blood loss that may be occurring, minimise pain and prevent further injury to the surrounding soft
tissues.

D - This requires surgical management and thus orthopaedic referral in a timely manner.

E - To reduce infection risk which can complicate healing and joint function, IV antibiotics are required. They
should be initiated early in
the patient’s management after discussion with the orthopaedic team.

Ref: ATLS 9th edition


46672

https://mypastest.pastest.com/Secure/TestMe/Browser/436619#Top[‫ ص‬11:52:15 10/12/1437]


MyPastest

Next Question
Tag Question

Previous Question
Feedback

Difficulty: Average End Session


Peer Responses

Session Progress

Responses Correct: 0

Responses Incorrect: 133

Responses Total: 133

Responses - % Correct: 0%

Blog
About Pastest
Contact Us
Help

© Pastest 2016

https://mypastest.pastest.com/Secure/TestMe/Browser/436619#Top[‫ ص‬11:52:15 10/12/1437]


MyPastest

Prefer to use the old MyPastest? Access it here »

Back to Filters

Question 114 of 120

A child is hit by a car at speed. They are intubated at the scene for a reduced GCS and brought to their major
trauma centre. On arrival there is slight asymmetry of the pupils, the blood pressure is elevated with a bradycardia
but no obvious signs of haemorrhage.

What is the most likely cause?

A Hypovolaemia from haemorrhage

B Neurogenic shock

C Ocular injury

D Raised intracranial pressure (RICP)

E Traumatic arrhythmia

Explanation
The child had a reduced GCS at the scene suggesting head injury. Cushing’s triad for RICP is: hypertension,
bradycardia and irregular breathing. This child is intubated and so breathing cannot be assessed however with the
mechanism of injury, reduced GCS and evidence of a blown pupil (from herniation compressing the
parasympathetic fibres of the third nerve); RICP is the most likely diagnosis.

A – Hypovolaemia from massive blood loss would be associated with a tachycardia and hypotension.

B – Neurogenic shock would be suggested by a flaccid paralysis with hypotension and bradycardia secondary to
loss of sympathetic innervation.

C – Local trauma to the eye can cause a fixed pupil, however does not explain the other signs seen in this case.

E
– Traumatic arrhythmia may occur following blunt chest trauma, however does not explain all the signs as seen
in the case above.

https://mypastest.pastest.com/Secure/TestMe/Browser/436619#Top[‫ ص‬11:52:37 10/12/1437]


MyPastest

Ref: ATLS 9th edition


46673

Next Question
Previous Question
Tag Question

Feedback
End Session

Difficulty: Average

Peer Responses

Session Progress

Responses Correct: 0

Responses Incorrect: 134

Responses Total: 134

Responses - % Correct: 0%

Blog
About Pastest
Contact Us
Help

© Pastest 2016

https://mypastest.pastest.com/Secure/TestMe/Browser/436619#Top[‫ ص‬11:52:37 10/12/1437]


MyPastest

Prefer to use the old MyPastest? Access it here »

Back to Filters

Question 115 of 120

A 21-day-old presents to your resus with a 24 hour history of


fever, lethargy and reduced feeds. On examination
he is hypotonic, tachypnoeic and has a CRT of 4 seconds, his femoral pulses are just palpable. You struggle to get
access but eventually manage to site an intraosseous needle and obtain a blood sugar which is 13. His observations
show his saturations in air to be 98%.

What is the most likely diagnosis?

A Duct dependent lesion

B Metabolic disorder

C Reye’s syndrome

D Sepsis

E Type 1 Diabetes Mellitus

Explanation
The fever, alongside the clinical presentation, make sepsis the likely diagnosis. A temperature >38 degrees in an
infant <3months of age warrants investigation.

A - Whilst the
femorals are weak, this more reflects the neonate being profoundly shut
down rather than a
coarctation. Duct dependent lesions tend to present in the first few days of life as the PDA closes. It may present
with tachypnoea and shock as well as cyanosis/hypoxia.

B - Metabolic disorders may mimic sepsis though are less common being predominantly autosomal recessive.
They may be associated with hypo/normo or hyperglycaemia, prolonged jaundice or cerebral irritation such as
seizures, depending on the inborn error of metabolism.

C - Reye’s syndrome tends to occur in older children than is the case here, having more neurological sequale. It
may follow a viral illness and is associated with aspirin use.

https://mypastest.pastest.com/Secure/TestMe/Browser/436619#Top[‫ ص‬11:52:59 10/12/1437]


MyPastest

E - Type 1 Diabetes Mellitus. The elevated sugar in this case is a stress response to the illness and repeated
cannulation attempts.
46674

Next Question
Previous Question
Tag Question
End Session
Feedback

Difficulty: Average

Peer Responses

Session Progress

Responses Correct: 0

Responses Incorrect: 135

Responses Total: 135

Responses - % Correct: 0%

Blog
About Pastest
Contact Us
Help

© Pastest 2016

https://mypastest.pastest.com/Secure/TestMe/Browser/436619#Top[‫ ص‬11:52:59 10/12/1437]


MyPastest

Prefer to use the old MyPastest? Access it here »

Back to Filters

Question 116 of 120

A 12-year-old presents to your resus with a non-blanching rash and prolonged capillary refill time. She becomes
difficult to ventilate and so is intubated. She has received two 20ml/kg of 0.9% saline boluses, however remains
hypotensive.

What is the next step in her management?

A Allow permissive hypotension

B Give 10ml/kg Packed red cells

C Give another 20ml/kg 0.9% saline

D Start Dopamine

E Start Dobutamine

Explanation
If a third fluid bolus is being considered then it is reasonable to start dopamine. This can initially be given through
a
peripheral line whilst central access is obtained.

A - Permissive hypotension may be used in trauma to minimise haemorrhage, however this is not a problem in
sepsis.

B – Packed cells would be more appropriate in the trauma setting and not in sepsis.

C – Sepsis has resulted in pulmonary oedema which has been exacerbated by the fluid boluses already given.
Inotrope use is indicated, although a further fluid bolus may be required whilst drawing
up the dopamine.

E – Further inotropes such as Dobutamine


or Adrenaline may be required if Dopamine alone has little benefit,
however these ideally require central access and so Dopamine is used first.

Ref: APLS 5th edition


46675

https://mypastest.pastest.com/Secure/TestMe/Browser/436619#Top[‫ ص‬11:53:22 10/12/1437]


MyPastest

Next Question
Tag Question
Previous Question
Feedback

Difficulty: Average
End Session

Peer Responses

Session Progress

Responses Correct: 0

Responses Incorrect: 136

Responses Total: 136

Responses - % Correct: 0%

Blog
About Pastest
Contact Us
Help

© Pastest 2016

https://mypastest.pastest.com/Secure/TestMe/Browser/436619#Top[‫ ص‬11:53:22 10/12/1437]


MyPastest

Prefer to use the old MyPastest? Access it here »

Next Question

Back to Filters

Question 117 of 120

An otherwise well 5-year-old is brought to your resus room with a reduced GCS. As part of your APLS teaching
you remember to check his blood sugar level (BSL).

At what BSL will you commence a glucose bolus?

A < 2.6 mmol/l

B < 3 mmol/l

C < 3.5 mmol/l

D < 4 mmol/l

E < 5 mmol/l

Explanation
In APLS teaching, if a BSL is <3 a glucose bolus of 2ml/kg 10% is indicated. This situation is different for
diabetic patients and for neonates. Local guidelines may also vary.

A - <2.6is the value that defines neonatal hypoglycaemia.

C - <3.5 this level is too high in the absence of diabetes.

D - <4is the value for hypoglycaemia in patients with a background of diabetes.

E - <5 is too high a threshold.

Ref: APLS 5th Edition


46676

https://mypastest.pastest.com/Secure/TestMe/Browser/436619#Top[‫ ص‬11:53:45 10/12/1437]


MyPastest

Tag Question

Feedback

Difficulty: Average
Previous Question
Peer Responses

End Session

Session Progress

Responses Correct: 0

Responses Incorrect: 137

Responses Total: 137

Responses - % Correct: 0%

Blog
About Pastest
Contact Us
Help

© Pastest 2016

https://mypastest.pastest.com/Secure/TestMe/Browser/436619#Top[‫ ص‬11:53:45 10/12/1437]


MyPastest

Prefer to use the old MyPastest? Access it here »

Back to Filters

Question 118 of 120

A 3-year-old presents with fever and irritability you suspect


meningitis and start to undertake a full septic screen
prior to starting Ceftriaxone.

Which of the following is a relative contraindication to lumbar puncture?

A Blanching rash

B GCS 14

C Haemoglobin <100g/l

D Posture of flexed arms and extended legs

E Tachycardia with hypotension

Explanation
This is a decorticate posture and suggests raised
intracranial pressure (RICP), performing a LP in this situation
may result in coning.

A - A purpuric rash in an unwell child is a contraindication and instead requires antibiotics to be given
immediately after collecting blood cultures

B - A GCS of less than 13 is a contraindication as drowsiness may be a sign of RICP.

C - Thrombocytopenia or having a coagulation disorder are contraindications, low haemoglobin level is not.

E - The reverse of this, hypertension with bradycardia forms part of Cushing’s triad with abnormal respirations
and suggests RICP.

Ref: www.meningitis.org/images/health_professionals/doctors_in_training/Junior_Doctor_Handbook_Jan_05.pdf
46677

https://mypastest.pastest.com/Secure/TestMe/Browser/436619#Top[‫ ص‬11:54:07 10/12/1437]


MyPastest

Next Question
Tag Question

Previous Question
Feedback

Difficulty: Average End Session


Peer Responses

Session Progress

Responses Correct: 0

Responses Incorrect: 138

Responses Total: 138

Responses - % Correct: 0%

Blog
About Pastest
Contact Us
Help

© Pastest 2016

https://mypastest.pastest.com/Secure/TestMe/Browser/436619#Top[‫ ص‬11:54:07 10/12/1437]


MyPastest

Prefer to use the old MyPastest? Access it here »

Back to Filters

Question 119 of 120

A 9-year-old boy has been brought to your emergency department by helicopter following being a pedestrian hit
by a car. He has sustained a head injury and is intubated. In order to prevent secondary brain injury you employ
neuroprotective measures.

Which of the below measures is NOT neuroprotective in traumatic brain injury (TBI)?

A Head up 20 degrees

B Hypothermia

C Maintain CO2 4.5-5kPa

D Normoglycaemia

E Prevent hypotension

Explanation
Hyperthermia is detrimental in TBI as it increases metabolic demand. Hypothermia has not been shown to
improve outcomes and is known to adversely effect coagulation. Therefore it is advised that a normal temperature
range is maintained i.e. 36-37degrees.

A – For the treatment of raised intracranial pressure (RICP), APLS advises maintaining the head up at 20 degrees
in the midline. This will aid with cerebral venous drainage.

C - Normocarbia is advised as hyperventilation results in vasoconstriction and may cause ischaemia, whereas
hypoventilation can result in RICP. A lower CO2 of 4-4.5KPa may be used temporarily in acute severe RICP.

D - Hyperglycaemia appears harmful to damaged brain, therefore glucose containing fluids are not used following
TBI and regular glucose monitoring is employed.

E - Normal blood pressure is required as


hypotension will result in a reduction to cerebral blood flow and risk
ischaemia, whilst hypertension risks oedema.

https://mypastest.pastest.com/Secure/TestMe/Browser/436619#Top[‫ ص‬11:55:37 10/12/1437]


MyPastest

Ref: APLS 5th edition; ATLS 9th edition


46678

Next Question
Previous Question
Tag Question

Feedback End Session

Difficulty: Average

Peer Responses

Session Progress

Responses Correct: 0

Responses Incorrect: 139

Responses Total: 139

Responses - % Correct: 0%

Blog
About Pastest
Contact Us
Help

© Pastest 2016

https://mypastest.pastest.com/Secure/TestMe/Browser/436619#Top[‫ ص‬11:55:37 10/12/1437]


MyPastest

Prefer to use the old MyPastest? Access it here »

Back to Filters

Question 120 of 120

A 5-year-old boy with sickle cell disease presents complaining of a severe headache. On examination he has facial
asymmetry
and slurred speech, with weakness in his left arm and leg. There is no history of fever and he has not
had headaches before.

What is the most likely diagnosis?

A Focal seizure

B Medulloblastoma

C Meningitis

D Migraine

E Stroke

Explanation
This child has classic features suggestive of a stroke. Whilst strokes are rare in children, he has sickle cell disease
which places him at increased risk. Other factors that carry an increased risk include congenital heart disease and
varicella infection within the previous year.

A – Focal seizures can present with motor symptoms however they tend to be localised to one muscle group.
Headaches can occur with, before or after seizures.

B – This is a common brain tumour of childhood. It is a posterior fossa tumour and tends to present with
headaches, early morning vomiting, unsteadiness/clumsiness and a sixth nerve palsy as a false localising sign.

C – Meningitis is more likely to present with a fever and irritability. In a child of this age the typical features of
neck stiffness and photophobia may also be seen.

D – A hemiplegic migraine can mimic the presentation of a stroke, however with the risk factor of sickle cell, the
young age and no previous headaches, a stroke is more likely.

https://mypastest.pastest.com/Secure/TestMe/Browser/436619#Top[‫ ص‬11:56:02 10/12/1437]


MyPastest

46679

End Session

Previous Question Tag Question

Feedback

Difficulty: Average

Peer Responses

Session Progress

Responses Correct: 0

Responses Incorrect: 140

Responses Total: 140

Responses - % Correct: 0%

Blog
About Pastest
Contact Us
Help

© Pastest 2016

https://mypastest.pastest.com/Secure/TestMe/Browser/436619#Top[‫ ص‬11:56:02 10/12/1437]


MyPastest

Prefer to use the old MyPastest? Access it here »

Back to Filters

Question 1 of 65

Theme: Genetic diseases

A Autosomal dominant

B Autosomal dominant with incomplete penetrance

C Autosomal recessive

D Autosomal recessive with incomplete penetrance

E Lionised X linked

F Robertsonian translocation

G Single gene disorder

H Uniparental disomy

I X-linked dominant

J X-linked recessive

For
each of the following conditions, choose the most appropriate mode of inheritance from the list above. Each
item may be used once, more than once or not at all.

Scenario 1

Achondroplasia

Your answer was incorrect

Select one...

A - Autosomal dominant

Achondroplasia is an autosomal dominant condition; however, 90% of occurrences are as a result of a new
mutation.

Scenario 2

Sickle cell disease

https://mypastest.pastest.com/Secure/TestMe/Browser/436619[‫ ص‬07:27:33 11/12/1437]


MyPastest

Your answer was incorrect

Select one...
End Session
C - Autosomal recessive

Sickle cell disease is a good example of an autosomal recessive inherited condition. The chance of inheritance
of
the disease (HbSS) is one in four if both parents are heterozygotes (HbSs). The prevalence of the gene in the
population is increased because the heterozygote form (sickle cell trait) provides resistance to
malaria in endemic
countries.

Scenario 3

Previous
Duchenne muscular Question
dystrophy

Your answer was incorrect

Select one...

J - X-linked recessive

Duchenne muscular dystrophy occurs in 1


in 3000 live male births. There is a significant new mutation rate (30%).
The underlying abnormality is a decreased production of dystrophin. Presentation is usually in early childhood,
and there may be
a history of delay in walking and a tendency to fall. Difficulty in getting up from a sitting
position can be demonstrated (Gower’s sign). Hip flexion contractures and calf hypertrophy lead to toe walking.
Of those affected 90% are wheel-chair bound by puberty and 30% have some learning disability.
22404

Tag Question

Feedback

Difficulty: Average

Session Progress

Responses Correct: 0

Responses Incorrect: 3

Responses Total: 3

Responses - % Correct: 0%
Next Question

https://mypastest.pastest.com/Secure/TestMe/Browser/436619[‫ ص‬07:27:33 11/12/1437]


MyPastest

Blog
About Pastest
Contact Us
Help

© Pastest 2016

https://mypastest.pastest.com/Secure/TestMe/Browser/436619[‫ ص‬07:27:33 11/12/1437]


MyPastest

Prefer to use the old MyPastest? Access it here »

Back to Filters

Question 2 of 65

Theme: Syndromes

A Achondroplasia

B Beckwith–Wiedemann syndrome

C Down syndrome

D Fragile X syndrome

E Noonan syndrome

F Patau syndrome

G Pierre Robin syndrome

H Prader–Willi syndrome

I Rubenstein–Taybi syndrome

J Turner syndrome

For
each of the following cases, choose the diagnosis from the list above that best fits with the clinical features
given. Each item may be used once, more than once or not at all.

Scenario 1

A term baby is noted to have a cleft palate and small chin. She appears to have problems breathing when placed
supine.

Your answer was incorrect

Select one...

G - Pierre Robin syndrome

These are typical features for Pierre Robin syndrome. The mandibular hypoplasia causes a degree of glossoptosis
(tongue obstructing airway posteriorly). These infants should be nursed prone and may often require a
nasopharyngeal airway to be used until the face/jaw has grown sufficiently. In the absence of associated
syndromes/features, intelligence is normal.

https://mypastest.pastest.com/Secure/TestMe/Browser/436619[‫ ص‬07:27:58 11/12/1437]


MyPastest

Scenario 2

A 4-year-old boy is having problems keeping up academically with his peers. On examination his neck is mildly
webbed, and he has low-set ears.

Your answer was incorrect


End Session

Select one...

E - Noonan syndrome

Noonan syndrome has a similar phenotype to Turner syndrome, but can occur in both sexes. There is often a
degree of learning difficulties. Noonan syndrome has an association with right-sided cardiac defects (especially
Previous Question
pulmonary stenosis).

Scenario 3

A 6-year-old boy has marked behavioural problems. On examination he has large ears and a high forehead.

Your answer was incorrect

Select one...

D - Fragile X syndrome

In any child with marked behavioural or learning difficulties, if no other cause is obvious, fragile X syndrome
should always be considered. Fragile X is the second most common genetic cause (trinucleotide repeat sequences
on the X chromosome) for learning difficulties in males (Down syndrome being the most common).
Phenotypically, children with fragile X syndrome have large ears, long noses and high foreheads. Testicular
volume is markedly
increased.
22412

Tag Question

Feedback

Difficulty: Average

Session Progress

Responses Correct: 0

Responses Incorrect: 6

Responses Total:
Next Question 6

https://mypastest.pastest.com/Secure/TestMe/Browser/436619[‫ ص‬07:27:58 11/12/1437]


MyPastest

Responses - % Correct: 0%

Blog
About Pastest
Contact Us
Help

© Pastest 2016

https://mypastest.pastest.com/Secure/TestMe/Browser/436619[‫ ص‬07:27:58 11/12/1437]


MyPastest

Prefer to use the old MyPastest? Access it here »

Back to Filters

Question 3 of 65

In Fig. 1, Alex is an apparently healthy elder brother of a girl affected by Cystic Fibrosis. He comes to the Genetic
Clinic. His partner is healthy with no family history of genetic disorders. A population carrier frequency of Cystic
Fibrosis is 1/20.

What is the chance of Alex having an affected Child?

A 1/20

B 1/30

C 1/80

D 1/120

E 1/240

Explanation
Answer stem D is the correct answer - 1/120.

A
population carrier frequency of Cystic Fibrosis is 1 in 20. The chances
of Alex’s partner being a carrier is

https://mypastest.pastest.com/Secure/TestMe/Browser/436619[‫ ص‬07:43:41 11/12/1437]


MyPastest

therefore 1/20. The chances of Alex being a carrier is 2/3. The chances of the child of 2 carriers being affected is
1/4. Therefore, the chance of Alex having an affected child is:

(Chances of Alex’s partner being


a carrier [1/20]) x (Chances of Alex being a carrier [2/3]) x (Chances of the child
of 2 carriers being affected[1/4]) = 2/240=1/120.
End Session
45364

Tag Question

Previous Question
Feedback

Difficulty: Average

Peer Responses

Session Progress

Responses Correct: 0

Responses Incorrect: 7

Responses Total: 7

Responses - % Correct: 0%

Blog
About Pastest
Contact Us
Help

© Pastest 2016

Next Question

https://mypastest.pastest.com/Secure/TestMe/Browser/436619[‫ ص‬07:43:41 11/12/1437]


MyPastest

Prefer to use the old MyPastest? Access it here »

Back to Filters

Question 4 of 65

Fig. 2
is a family pedigree of Jane, who has a family history of Duchenne Muscular Dystrophy (DMD). She has a
brother and 2 cousins (who are brothers), who are affected by DMD. Her maternal uncle with DMD died at the
age of 24.

What is the chance of Jane having an affected male?

A 1 in 64

B 1 in 32

C 1 in 16

D 1 in 8

E 1 in 4

Explanation
Answer stem D is the correct answer - 1 in 8.

Jane’s
mother must be a carrier. The chances of Jane being a carrier are 1/2. Therefore, the chances of Jane’s child

https://mypastest.pastest.com/Secure/TestMe/Browser/436619[‫ ص‬07:44:31 11/12/1437]


MyPastest

being male and affected is 1/2 x 1/2 x 1/2.

45365

End Session

Tag Question

Feedback

Difficulty: Average
Previous Question
Peer Responses

Session Progress

Responses Correct: 0

Responses Incorrect: 8

Responses Total: 8

Responses - % Correct: 0%

Blog
About Pastest
Contact Us
Help

© Pastest 2016

Next Question

https://mypastest.pastest.com/Secure/TestMe/Browser/436619[‫ ص‬07:44:31 11/12/1437]


MyPastest

Prefer to use the old MyPastest? Access it here »

End Session
Back to Filters

Question 5 of 65

Previous Question
You are a paediatric speciality trainee year 4. You see a 3-year-old boy for development delay and behavioural
problems. He has 2 maternal aunts with mild learning difficulties and a maternal uncle with
severe learning
difficulty.

He has macrocephaly and large ears, no expressive language and poor eye contact but he is able to hear normally
and follow simple commands. He is hyperactive with a short attention span. His gross and fine motor development
is normal.

What would be the best diagnostic test for him?

A Chromosomal Microarray Analysis (CMA)

B G-branded karyotype

C FMR1 mutation analysis

D FISH (Fluorescence In Situ Hybridisation) test

E Creatinine kinase

45366

Tag Question

Feedback

Next Question
Difficulty: Average

https://mypastest.pastest.com/Secure/TestMe/Browser/436619[‫ ص‬07:45:44 11/12/1437]


MyPastest

Peer Responses

Session Progress

Responses Correct: 0

Responses Incorrect: 9

Responses Total: 9

Responses - % Correct: 0%

Blog
About Pastest
Contact Us
Help

© Pastest 2016

https://mypastest.pastest.com/Secure/TestMe/Browser/436619[‫ ص‬07:45:44 11/12/1437]


MyPastest

Prefer to use the old MyPastest? Access it here »

End Session
Back to Filters

Question 6 of 65

Previous Question
You are a year 4 specialist trainee in paediatrics. You see 12-week-old Lucy with diagnosis X in the paediatric
outpatient department together with her mother.

Her birth weight was 4.05 kg. She had an omphalocele and large protruding tongue together with subtle left sided
hemi-hypertrophy. She also had hypoglycaemia requiring
correction with IV Dextrose.

Her omphalocele healed after 4 weeks of non-operative treatment. Her mother had found out that she has
a higher
risk of Wilm’s tumour.

What is the current tumour surveillance protocol for this condition?

A Abdominal ultrasound every 3 months until the age of 8 years

B Urinary homovanillic acid (HVA) and Vanillylmandelic acid (VMA) assays every 3 months until the
age of 8

C Magnetic resonance imaging (MRI) or computed tomography (CT) examination of the abdomen every
3 months until the age 8 years

D Measure serum AFP (Alpha fetoprotein) concentration every 3 months until the age of 4 years

E Measure urinary calcium/creatinine ratio every 6 months till the age of 4 years

45367

Tag Question

Feedback Next Question

https://mypastest.pastest.com/Secure/TestMe/Browser/436619[‫ ص‬07:46:05 11/12/1437]


MyPastest

Difficulty: Average

Peer Responses

Session Progress

Responses Correct: 0

Responses Incorrect: 10

Responses Total: 10

Responses - % Correct: 0%

Blog
About Pastest
Contact Us
Help

© Pastest 2016

https://mypastest.pastest.com/Secure/TestMe/Browser/436619[‫ ص‬07:46:05 11/12/1437]


MyPastest

Prefer to use the old MyPastest? Access it here »

Back to Filters

Question 7 of 65

Fig. 3 is a family pedigree with condition X.

A similar pattern of inheritance is most likely to be seen in which of the following?

A Cystic fibrosis

B Haemophilia A (factor VIII deficiency)

C Haemophilia B (factor IX deficiency)

D Incontinentia pigmenti

E Marfan syndrome

https://mypastest.pastest.com/Secure/TestMe/Browser/436619[‫ ص‬07:46:38 11/12/1437]


MyPastest

45368

End Session

Tag Question

Feedback

Difficulty: Average

Peer Responses
Previous Question

Session Progress

Responses Correct: 0

Responses Incorrect: 11

Responses Total: 11

Responses - % Correct: 0%

Blog
About Pastest
Contact Us
Help

© Pastest 2016

Next Question

https://mypastest.pastest.com/Secure/TestMe/Browser/436619[‫ ص‬07:46:38 11/12/1437]


MyPastest

Prefer to use the old MyPastest? Access it here »

End Session
Back to Filters

Question 8 of 65

Previous Question
You are Year 4 paediatric specialist trainee. You are called to see a baby boy, James, in the postnatal ward. The
midwife tells you that James is 6 hours’ old now and has had 1 episode of bilious vomiting. He has not passed
meconium. You notice that he has low set eyes, epicanthic fold, flat nasal bridge, single transverse palmar crease
and is hypotonic. His abdomen is distended.

Which of the following should be your next step?

A Obtain
blood samples for G-branded karyotype, blood culture, inflammatory marker (C-reactive
protein) and full blood count and start on antibiotics

B Refer to the surgical team

C Request contrast Upper Gastrointestinal Tract series

D Request plain abdominal X-ray

E Stop feeds and pass nasogastric tube

45369

Tag Question

Feedback

Difficulty: Average

Next Question
Peer Responses

https://mypastest.pastest.com/Secure/TestMe/Browser/436619[‫ ص‬07:47:40 11/12/1437]


MyPastest

Session Progress

Responses Correct: 0

Responses Incorrect: 12

Responses Total: 12

Responses - % Correct: 0%

Blog
About Pastest
Contact Us
Help

© Pastest 2016

https://mypastest.pastest.com/Secure/TestMe/Browser/436619[‫ ص‬07:47:40 11/12/1437]


MyPastest

Prefer to use the old MyPastest? Access it here »

End Session
Back to Filters

Question 9 of 65

Previous Question
Sophie is a 26-year-old who has been on drug X for treatment of her seizures. She stopped taking drug X as soon
as she knew that she is pregnant at about 12 weeks of pregnancy.

She delivered a term baby boy with spina bifida, hypospadias and cleft lip. He also had facial dysmorphism (flat
nasal bridge, shallow filtrum, upturned nose with thin upper lip). Additionally he was found to have a murmur and
this was confirmed to be an atrial septal defect by echocardiogram.

What is the drug 'X' likely to be?

A Phenytoin

B Phenobarbitone

C Sodium Valproate

D Carbamazepine

E Topiramate

45370

Tag Question

Feedback

Difficulty: Average
Next Question

https://mypastest.pastest.com/Secure/TestMe/Browser/436619[‫ ص‬07:47:59 11/12/1437]


MyPastest

Peer Responses

Session Progress

Responses Correct: 0

Responses Incorrect: 13

Responses Total: 13

Responses - % Correct: 0%

Blog
About Pastest
Contact Us
Help

© Pastest 2016

https://mypastest.pastest.com/Secure/TestMe/Browser/436619[‫ ص‬07:47:59 11/12/1437]


MyPastest

Prefer to use the old MyPastest? Access it here »

End Session
Back to Filters

Question 10 of 65

Previous Question
You see a 3½-year-old girl in paediatric outpatient clinic. She was born at term by normal delivery. Gross motor,
fine motor, social, emotional and language milestones were normal until 18 months of age. Her development then
slowed down and regressed. At 3 years, she lost her expressive speech, had vacant episodes, showed stereotypic
hand washing movements and lost purposeful hand skills. She was unable to hold, pick up or grasp things. There
is no history of traumatic brain injury, neuro-metabolic disease or severe infection.

On
examination, she is microcephalic. No facial dysmorphism is noted. She has a broad based gait and stereotypic
hand movements.

What is her most likely diagnosis?

A Cerebral palsy

B Rett syndrome

C Landau-Kleffner syndrome

D Autism syndrome disorder

E Angelman syndrome

45371

Tag Question

Feedback

Next Question

https://mypastest.pastest.com/Secure/TestMe/Browser/436619[‫ ص‬07:48:20 11/12/1437]


MyPastest

Difficulty: Average

Peer Responses

Session Progress

Responses Correct: 0

Responses Incorrect: 14

Responses Total: 14

Responses - % Correct: 0%

Blog
About Pastest
Contact Us
Help

© Pastest 2016

https://mypastest.pastest.com/Secure/TestMe/Browser/436619[‫ ص‬07:48:20 11/12/1437]


MyPastest

Prefer to use the old MyPastest? Access it here »

End Session
Back to Filters

Question 11 of 65

Previous Question
A 20-month-old child presents to the paediatric outpatient department with a 1 year history of progressive stridor.

On
clinical examination, she is noted to have 6 Café Au Lait spots measuring 6-10 mm in size. Her mother and 3
older sisters have neurofibromatosis type 1. She is afebrile. No coryzal symptoms. She has inspiratory stridor with
suprasternal recessions.

What is the most appropriate next plan of management for this child?

A Administer dexamethasone and admit to the children's ward

B Arrange for MRI to look for vascular anomalies

C Reassurance and follow up in 6 months’ time as it is likely to be laryngomalacia, which would resolve
spontaneously

D Refer to ENT team for flexible laryngoscopy

E Request for plain chest X-ray to look for foreign body

45372

Tag Question

Feedback

Difficulty: Average
Next Question

https://mypastest.pastest.com/Secure/TestMe/Browser/436619[‫ ص‬07:48:39 11/12/1437]


MyPastest

Peer Responses

Session Progress

Responses Correct: 0

Responses Incorrect: 15

Responses Total: 15

Responses - % Correct: 0%

Blog
About Pastest
Contact Us
Help

© Pastest 2016

https://mypastest.pastest.com/Secure/TestMe/Browser/436619[‫ ص‬07:48:39 11/12/1437]


MyPastest

Prefer to use the old MyPastest? Access it here »

Back to Filters

Question 12 of 65

A 2-year-old girl presents with 6 Café au Lait spots measuring 6-10mm in size to the paediatric outpatient
department. she has no facial dysmorphism and her height and weight are appropriate for age, as is her
development. History and clinical examination is otherwise normal.

What would be the best plan of management for her?

A Explain
to her parents that she has neurofibromatosis type 1 (NF1) and refer to
clinician skilled in NF1
including geneticist, paediatrician, neurologist and dermatologist

B Follow up as if she has NF1 as she likely to develop NF1

C Arrange for MRI of brain to look for hyperintense lesions on T2 weighted MRI brain

D Discharge as she not fulfil the diagnostic criteria for NF1

E Genetic testing for NF1 mutation analysis

Explanation
Children with 6 Café au Lait patches alone - even with no relevant family history - should be followed up as if
they do have NF1, as 95% of them will develop NF1, so B
is the best answer.

Whilst this child does not fulfil the diagnostic criteria for NF1 yet, this is not the correct answer for the reasons
given above so answer stem A and also D are incorrect.

Hyperintense
lesions on T2 weighted brain MRI (formerly called UBOs) are probably caused by aberrant
myelination or gliosis, and are pathognomonic of NF1.
They occur most commonly in children of 8-16 years old,
tend to disappear in adulthood and have a tenuous link with cognitive impairment. The presence of these lesions
can assist the diagnosis of NF1, but MRI under anaesthetic is not warranted for this purpose in young children so
option C is incorrect.

NF1 mutational analysis clarifies the diagnosis in some uncertain cases and in individuals contemplating prenatal

https://mypastest.pastest.com/Secure/TestMe/Browser/436619[‫ ص‬07:49:13 11/12/1437]


MyPastest

diagnosis. However, genetic testing is not advocated routinely and expert consultation is advised before it is
undertaken so option E is incorrect.
45373

End Session

Tag Question

Feedback

Difficulty: Average

Previous Question
Peer Responses

Session Progress

Responses Correct: 0

Responses Incorrect: 16

Responses Total: 16

Responses - % Correct: 0%

Blog
About Pastest
Contact Us
Help

© Pastest 2016

Next Question

https://mypastest.pastest.com/Secure/TestMe/Browser/436619[‫ ص‬07:49:13 11/12/1437]


MyPastest

Prefer to use the old MyPastest? Access it here »

End Session
Back to Filters

Question 13 of 65

Previous Question
You see a 1-day-old baby on the post-natal ward and note the following features:

brachycephaly
upslanting palpebral fissures
protruding tongue
wide sandal gap between first and second toes

Given the most likely diagnosis, which of the following conditions is not considered to have an increased
incidence in such individuals?

A Alzheimer's disease

B Atlanto-axial dislocation

C Crohn's disease

D Hirschsprung's disease

E Leukaemia

45704

Tag Question

Feedback
Next Question

https://mypastest.pastest.com/Secure/TestMe/Browser/436619[‫ ص‬07:49:32 11/12/1437]


MyPastest

Difficulty: Average

Peer Responses

Session Progress

Responses Correct: 0

Responses Incorrect: 17

Responses Total: 17

Responses - % Correct: 0%

Blog
About Pastest
Contact Us
Help

© Pastest 2016

https://mypastest.pastest.com/Secure/TestMe/Browser/436619[‫ ص‬07:49:32 11/12/1437]


MyPastest

Prefer to use the old MyPastest? Access it here »

End Session
Back to Filters

Question 14 of 65

Previous Question
Which of the following conditions does not manifest autosomal recessive inheritance?

A Canavan disease

B Cystic Fibrosis

C Edward syndrome

D Galactosaemia

E Phenylketonuria

45705

Tag Question

Feedback

Difficulty: Average

Peer Responses

Next Question

https://mypastest.pastest.com/Secure/TestMe/Browser/436619[‫ ص‬07:49:50 11/12/1437]


MyPastest

Session Progress

Responses Correct: 0

Responses Incorrect: 18

Responses Total: 18

Responses - % Correct: 0%

Blog
About Pastest
Contact Us
Help

© Pastest 2016

https://mypastest.pastest.com/Secure/TestMe/Browser/436619[‫ ص‬07:49:50 11/12/1437]


MyPastest

Prefer to use the old MyPastest? Access it here »

End Session
Back to Filters

Question 15 of 65

Previous Question
You are discussing Turner syndrome with prospective parents.

Which of the following statements is correct?

A Loss of the maternal sex chromosome is always lethal

B Only females are affected

C Spontaneous fetal loss occurs commonly in the second trimester

D The risk increases with increasing maternal age

E There is always loss of the paternal sex chromosome

45710

Tag Question

Feedback

Difficulty: Average

Peer Responses

Next Question

https://mypastest.pastest.com/Secure/TestMe/Browser/436619[‫ ص‬07:50:08 11/12/1437]


MyPastest

Session Progress

Responses Correct: 0

Responses Incorrect: 19

Responses Total: 19

Responses - % Correct: 0%

Blog
About Pastest
Contact Us
Help

© Pastest 2016

https://mypastest.pastest.com/Secure/TestMe/Browser/436619[‫ ص‬07:50:08 11/12/1437]


MyPastest

Prefer to use the old MyPastest? Access it here »

Back to Filters

Question 16 of 65

Which of the following is an almost universal feature of Down syndrome?

A Atlantoaxial instability

B Auto-immune hyperthyroidism

C Hypotonia

D Leukaemia

E Maternal translocation of chromosome 21

Explanation
Neonatal hypotonia (option C) is
an almost universal feature of Down syndrome. Cardiac lesions are present in
30-50% of children. The most common lesions are septal defects, 30% being atrioventricular septal defects and
30% being ventricular septal defects.

Down syndrome is associated with an increased incidence of hypothyroidism (not hyperthyroidism; option B),
cervical spine abnormalities, leukaemia (option D), Alzheimer's disease,
duodenal and oesophageal atresia,
Hirschprung's disease, pulmonary hypertension and cataracts.

The majority of cases arise from chromosomal non-dysjunction at meiosis. Although it is important to examine the
karyotype for translocations (option E; as inheritance from either parent increases the risks for future pregnancies),
they are found in fewer than 5% of children with trisomy 21.
45712

Next Question

https://mypastest.pastest.com/Secure/TestMe/Browser/436619[‫ ص‬07:50:27 11/12/1437]


MyPastest

Tag Question

Feedback

End Session
Difficulty: Average

Peer Responses

Previous Question

Session Progress

Responses Correct: 0

Responses Incorrect: 20

Responses Total: 20

Responses - % Correct: 0%

Blog
About Pastest
Contact Us
Help

© Pastest 2016

https://mypastest.pastest.com/Secure/TestMe/Browser/436619[‫ ص‬07:50:27 11/12/1437]


MyPastest

Prefer to use the old MyPastest? Access it here »

Back to Filters

Question 17 of 65

You see an infant in clinic with an occipitofrontal head circumference below two standard deviations for age, sex
and gestational
age.

This condition is not a feature of which one of the following conditions?

A Angelman syndrome

B Cri du Chat syndrome

C Down syndrome

D Edward syndrome

E Sotos syndrome

Explanation
Microcephaly refers to a small head, where the head circumference is less than 2 standard deviations below the
mean for
age, sex and gestational age. Microcephaly can be primary or secondary to other causes. Primary causes
include genetic defects such as trisomy 13 (Patau syndrome), 18 (Edward syndrome; option D) and 21 (Down
syndrome; option C), also Cri du Chat syndrome (5p-; option B), and Angelman syndrome (80% deletion on
maternal chromosome 15, 2-3% paternal uniparental disomy 15, remainder subtle mutations; option A). Secondary
microcephaly occurs due to congenital infections, placental insufficiency, maternal alcohol consumption during
pregnancy and hypoxic
ischaemic encephalopathy.

Sotos syndrome (option E) is a state of cerebral gigantism and thus macrocephaly.


45714

Next Question

https://mypastest.pastest.com/Secure/TestMe/Browser/436619[‫ ص‬07:50:47 11/12/1437]


MyPastest

Tag Question

Feedback

End Session
Difficulty: Average

Peer Responses

Previous Question

Session Progress

Responses Correct: 0

Responses Incorrect: 21

Responses Total: 21

Responses - % Correct: 0%

Blog
About Pastest
Contact Us
Help

© Pastest 2016

https://mypastest.pastest.com/Secure/TestMe/Browser/436619[‫ ص‬07:50:47 11/12/1437]


MyPastest

Prefer to use the old MyPastest? Access it here »

Back to Filters

Question 18 of 65

Previous Question
Which of the following conditions typically demonstrates an X-linked recessive inheritance pattern?

A Ataxia telangiectasia

B Galactosaemia

C Hunter syndrome (mucopolysaccharidosis II)

D Sickle cell disease

E Tay-Sachs

Explanation
In contrast to most of the other mucopolysaccharidoses which are inherited in an autosomal recessive fashion,
Hunter syndrome
(MPS II; option C) is X-linked recessive (XLR). XLR conditions result from a mutation in a
gene carried on the X chromosome and affect males because they have just one gene copy. Females are usually
unaffected but
may have mild manifestations as a result of lyonisation. New mutations are common so the mother
of an affected boy with no preceding family history, is not necessarily a carrier.

Ataxia telangiectasia (option A), Tay-Sachs (option E), galactosaemia (option B) and sickle cell disease (option D)
are inherited in an autosomal recessive manner (AR). AR conditions result from mutations in both copies of an
autosomal gene. Where both parents are carriers (one mutation and one normal copy) each pregnancy has a 1 in 4
(25%) risk of resulting in an affected child and a 2 in 4 (50%) risk of resulting in the offspring being a carrier.
45715

Next Question Tag Question

https://mypastest.pastest.com/Secure/TestMe/Browser/436619[‫ ص‬07:51:08 11/12/1437]


MyPastest

Feedback

Difficulty: Average
End Session
Peer Responses

Session Progress

Responses Correct: 0

Responses Incorrect: 22

Responses Total: 22

Responses - % Correct: 0%

Blog
About Pastest
Contact Us
Help

© Pastest 2016

https://mypastest.pastest.com/Secure/TestMe/Browser/436619[‫ ص‬07:51:08 11/12/1437]


MyPastest

Prefer to use the old MyPastest? Access it here »

Back to Filters

Question 19 of 65

Which one of the following conditions is inherited in


an autosomal recessive pattern and is associated with tri-
nucleotide repeat expansion?

A Fragile X syndrome

B Friedriech's ataxia

C Huntington's disease

D Myotonic dystrophy

E Spinocerebellar ataxia

Explanation
Friedreich's ataxia (option B) is autosomal recessive and is due to tri-nucleotide repeat expansions in the
mitochondrial protein frataxin.

Trimucleotide
repeat disorders are associated with genes containing stretches of repeating units of three
nucleotides. In normal individuals the number of repeats varies slightly but remains below a defined threshold.
Affected patients have an increased number of repeats, known as an expansion, above the disease causing
threshold. The expansion may be unstable and enlarging in subsequent generations leading to increased disease
severity (anticipation) and earlier onset.

Myotonic dystrophy (option D) is inherited in an autosomal dominant manner. It arises due to the presence of an
unstable tri-nucleotide repeat expansion.

Huntington's disease (option C) and SCA 7 (option E) are both autosomal dominant and display the phenomenon
of trinucleotide repeat patterns, as does Fragile X syndrome (option A) which is an X linked with stably inherited
trinucleotide repeats in the FMR1 gene.
45716

https://mypastest.pastest.com/Secure/TestMe/Browser/436619[‫ ص‬07:51:30 11/12/1437]


MyPastest

End
Tag Session
Question

Feedback

Difficulty: Average

Peer Responses

Previous Question

Session Progress

Responses Correct: 0

Responses Incorrect: 23

Responses Total: 23

Responses - % Correct: 0%

Blog
About Pastest
Contact Us
Help

© Pastest 2016

Next Question

https://mypastest.pastest.com/Secure/TestMe/Browser/436619[‫ ص‬07:51:30 11/12/1437]


MyPastest

Prefer to use the old MyPastest? Access it here »

End Session
Back to Filters

Question 20 of 65

Previous Question
A 5-year-old boy presents with a waddling gait, increased lumbar lordosis, and difficulty in assuming an erect
position. His maternal uncle has Duchenne Muscular Dystrophy (DMD).

Which of the following is most likely to be correct?

A His mother is unlikely to be a carrier

B His sister certainly carries the gene

C His is more likely than not to have a detectable deletion or duplication in his dystrophin gene

D Muscle biopsy is essential to confirm the diagnosis

E He will have been symptomatic since birth with greatly delayed gross motor skills

45717

Tag Question

Feedback

Difficulty: Average

Peer Responses

Next Question

https://mypastest.pastest.com/Secure/TestMe/Browser/436619[‫ ص‬07:52:05 11/12/1437]


MyPastest

Session Progress

Responses Correct: 0

Responses Incorrect: 24

Responses Total: 24

Responses - % Correct: 0%

Blog
About Pastest
Contact Us
Help

© Pastest 2016

https://mypastest.pastest.com/Secure/TestMe/Browser/436619[‫ ص‬07:52:05 11/12/1437]


MyPastest

Prefer to use the old MyPastest? Access it here »

Back to Filters

Question 21 of 65

You see a 5-year-old child in clinic with a short neck, widely spaced nipples, pectus carinatum and low-set rotated
ears. He has a mild learning disability and a cardiac defect.

Given the most likely diagnosis, which of the following cardiac defects is he most likely to display?

A Atrial septal defect

B Hypertrophic cardiomyopathy

C Pulmonary valvular stenosis

D Tetralogy of Fallot

E Ventricular septal defect

Explanation
This boy is most likely to have Noonan syndrome, a
common autosomal dominant condition, caused by mutations
in PTPN11 on chromosome 12q. The incidence is around 1 in 2500 individuals. Affected individuals are
characteristically short (mean height around 3rd centile), and have down slanting palpebral fissures, with heavy or
'hooded' eyelids. A low posterior neckline and short 'webbed' neck are often seen.

Around 80% of individuals with Noonan syndrome have a


cardiac defect. The most common defect is pulmonary
valvular stenosis (60%; option C) with hypertrophic cardiomyopathy, which may present throughout life, second
at 20% (option B). Atrial and ventricular septal
defects, Tetralogy of Fallot, branch pulmonary artery stenosis and
coarction of the aorta are also seen.

Feeding problems in infancy are very common, and this appears to correlate with mild developmental delay (seen
in about 30% of cases).
45720

https://mypastest.pastest.com/Secure/TestMe/Browser/436619[‫ ص‬07:52:26 11/12/1437]


MyPastest

End
Tag Session
Question

Feedback

Difficulty: Average

Peer Responses

Previous Question

Session Progress

Responses Correct: 0

Responses Incorrect: 25

Responses Total: 25

Responses - % Correct: 0%

Blog
About Pastest
Contact Us
Help

© Pastest 2016

Next Question

https://mypastest.pastest.com/Secure/TestMe/Browser/436619[‫ ص‬07:52:26 11/12/1437]


MyPastest

Prefer to use the old MyPastest? Access it here »

End Session
Back to Filters

Question 22 of 65

Previous Question
With regard to Turner syndrome, which of the following statements is correct?

Your answer was incorrect

A It affects one in 250 live born girls

B It
has the genotype XY; however, intrauterine development persists as essentially female due to lack of
receptors for circulating testosterone

C It is a frequent finding among early miscarriages

D There are no features apparent at birth

E Somatotrophin (human growth hormone) is a useful treatment for short stature once the epiphyses
have fused

45721

Tag Question

Feedback

Difficulty: Average

Peer Responses
Next Question

https://mypastest.pastest.com/Secure/TestMe/Browser/436619[‫ ص‬07:52:53 11/12/1437]


MyPastest

Session Progress

Responses Correct: 0

Responses Incorrect: 26

Responses Total: 26

Responses - % Correct: 0%

Blog
About Pastest
Contact Us
Help

© Pastest 2016

https://mypastest.pastest.com/Secure/TestMe/Browser/436619[‫ ص‬07:52:53 11/12/1437]


MyPastest

Prefer to use the old MyPastest? Access it here »

End Session
Back to Filters

Question 23 of 65

Previous Question
You are investigating a child for short stature.

Which of the following syndromes is a cause of short stature?

A Klinefelter syndrome

B Marfan syndrome

C Sotos syndrome

D Triple X syndrome

E Turner syndrome

45722

Tag Question

Feedback

Difficulty: Average

Peer Responses

Next Question

https://mypastest.pastest.com/Secure/TestMe/Browser/436619[‫ ص‬07:53:11 11/12/1437]


MyPastest

Session Progress

Responses Correct: 0

Responses Incorrect: 27

Responses Total: 27

Responses - % Correct: 0%

Blog
About Pastest
Contact Us
Help

© Pastest 2016

https://mypastest.pastest.com/Secure/TestMe/Browser/436619[‫ ص‬07:53:11 11/12/1437]


MyPastest

Prefer to use the old MyPastest? Access it here »

End Session
Back to Filters

Question 24 of 65

Previous Question
Which one of the following conditions is an example of monosomy?

A Angelman syndrome

B Di George syndrome

C Down syndrome

D Prader-Willi syndrome

E Turner syndrome

45741

Tag Question

Feedback

Difficulty: Average

Peer Responses

Next Question

https://mypastest.pastest.com/Secure/TestMe/Browser/436619[‫ ص‬07:53:30 11/12/1437]


MyPastest

Session Progress

Responses Correct: 0

Responses Incorrect: 28

Responses Total: 28

Responses - % Correct: 0%

Blog
About Pastest
Contact Us
Help

© Pastest 2016

https://mypastest.pastest.com/Secure/TestMe/Browser/436619[‫ ص‬07:53:30 11/12/1437]


MyPastest

Prefer to use the old MyPastest? Access it here »

End Session
Back to Filters

Question 25 of 65

Previous Question
Which of the following genetic disorders does NOT cause problems with gonadal determination and sexual
differentiation?

A 45 XO (Turner Syndrome)

B DAX-1 mutation

C SOX-9 mutation (Campomelic Dysplasia)

D WT-1 mutation (Denys-Drash syndrome)

E XXX (Triple X Syndrome)

46048

Tag Question

Feedback

Difficulty: Average

Peer Responses

Next Question

https://mypastest.pastest.com/Secure/TestMe/Browser/436619[‫ ص‬07:53:49 11/12/1437]


MyPastest

Session Progress

Responses Correct: 0

Responses Incorrect: 29

Responses Total: 29

Responses - % Correct: 0%

Blog
About Pastest
Contact Us
Help

© Pastest 2016

https://mypastest.pastest.com/Secure/TestMe/Browser/436619[‫ ص‬07:53:49 11/12/1437]


MyPastest

Prefer to use the old MyPastest? Access it here »

Back to Filters

Question 26 of 65

In children with complete Androgen Insensitivity Syndrome (AIS) what is the usual appearance of the external
genitalia?

A Enlarged clitoris

B Male genitalia, undescended testes

C Micropenis

D Unambiguously female

E Unambiguously male

Explanation
In children with complete AIS, defects in the androgen receptor gene usually result in a completely female
phenotype in a 46XY patient. External genitalia appear obviously female
(option D) with normal clitoris,
hypoplastic labia majora and blind-ending vaginal pouch. Testes may be found in the abdomen, inguinal
canal or
labia. Patients may also present as female teenagers with complete amenorrhoea. At puberty serum testosterone &
LH are raised.
Testosterone is converted to oestradiol leading to normal female breast development.Pubic and
axillary hair may be absent or reduced.

Ectopic testes are at increased risk of malignancy and are usually removed after diagnosis. Oestrogen replacement
therapy is then given.

AIS must be considered in any female patient presenting with an inguinal hernia.

References for further reading:


Tasker, McClure, Acerini, 2009. Oxford Handbook of Paediatrics. Oxford
University Press. ISBN-10:0-19957473-1. Chapter 11 Growth & Puberty, Androgen Insensitivity Syndrome
(AIS).

Exam question theme October 2015


46049

https://mypastest.pastest.com/Secure/TestMe/Browser/436619[‫ ص‬07:54:07 11/12/1437]


MyPastest

End Session
Tag Question

Feedback

Difficulty: Average

Peer Responses

Previous Question

Session Progress

Responses Correct: 0

Responses Incorrect: 30

Responses Total: 30

Responses - % Correct: 0%

Blog
About Pastest
Contact Us
Help

© Pastest 2016

Next Question

https://mypastest.pastest.com/Secure/TestMe/Browser/436619[‫ ص‬07:54:07 11/12/1437]


MyPastest

Prefer to use the old MyPastest? Access it here »

End Session
Back to Filters

Question 27 of 65

Previous Question
Which of the following genetic disorders is most associated with miscarriage of affected male children?

A Ehlers-Danlos Syndrome

B Incontinentia Pigmenti

C Neurofibromatosis Type 1

D Tuberose Sclerosis Complex

E X-linked hypohydrotic ectodermal dysplasia

46050

Tag Question

Feedback

Difficulty: Average

Peer Responses

Next Question

https://mypastest.pastest.com/Secure/TestMe/Browser/436619[‫ ص‬07:54:26 11/12/1437]


MyPastest

Session Progress

Responses Correct: 0

Responses Incorrect: 31

Responses Total: 31

Responses - % Correct: 0%

Blog
About Pastest
Contact Us
Help

© Pastest 2016

https://mypastest.pastest.com/Secure/TestMe/Browser/436619[‫ ص‬07:54:26 11/12/1437]


MyPastest

Prefer to use the old MyPastest? Access it here »

End Session
Back to Filters

Question 28 of 65

Previous Question
What is the most frequent genetic process causing both Prader-Willi and Angelman Syndromes?

A Genomic Imprinting

B Mendelian Inheritance

C Reciprocal translocation

D Robertsonian translocation

E Uniparental Disomy

46051

Tag Question

Feedback

Difficulty: Average

Peer Responses

Next Question

https://mypastest.pastest.com/Secure/TestMe/Browser/436619[‫ ص‬07:54:43 11/12/1437]


MyPastest

Session Progress

Responses Correct: 0

Responses Incorrect: 32

Responses Total: 32

Responses - % Correct: 0%

Blog
About Pastest
Contact Us
Help

© Pastest 2016

https://mypastest.pastest.com/Secure/TestMe/Browser/436619[‫ ص‬07:54:43 11/12/1437]


MyPastest

Prefer to use the old MyPastest? Access it here »

Back to Filters

Question 29 of 65

A 15-year-old basketball player was noted to have 1/6 systolic ejection murmur and referred to the cardiologist.
Her arm span
was greater than her height. She had long and thin fingers. She had a high arched palate and
excessive ligamentous laxity. She was able to bend her thumb back so that it touched the radius. ECG showed
sinus bradycardia consistent with aerobic conditioning. She had an echocardiogram done. She is diagnosed
clinically to have condition X.

What is the most serious cardiac complication of condition X?

A Aortic dissection

B Critical aortic stenosis

C Hypertrophic cardiomyopathy

D Long QT syndrome

E Mitral valve stenosis

Explanation
This is a girl with Marfan syndrome. Aortic root disease, leading to aneurysmal dilatation, aortic regurgitation and
dissection is the main cause of morbidity and mortality in Marfan syndrome. The rest of the cardiac problems
listed (B, C, D, E) are not associated with Marfan syndrome.

Ref:

1. Richard Lopez, Julie Berg-McGraw. Marfan syndrome in a female collegiate basketball player: A case
report. Journal of Athletic Training 2000;35(1):91-95
2. Michael J Wright, Heidi M Connolly. Genetics, clinical features and diagnosis of Marfan syndrome and
related disorders. www.uptodate.com.
46159

https://mypastest.pastest.com/Secure/TestMe/Browser/436619[‫ ص‬07:55:03 11/12/1437]


MyPastest

End
Tag Session
Question

Feedback

Difficulty: Average

Peer Responses

Previous Question

Session Progress

Responses Correct: 0

Responses Incorrect: 33

Responses Total: 33

Responses - % Correct: 0%

Blog
About Pastest
Contact Us
Help

© Pastest 2016

Next Question

https://mypastest.pastest.com/Secure/TestMe/Browser/436619[‫ ص‬07:55:03 11/12/1437]


MyPastest

Prefer to use the old MyPastest? Access it here »

Back to Filters

Question 30 of 65

A 5-year-old girl was referred to the paediatric cardiologist


because of heart murmurs. She had a prominent
forehead, widely-spaced eyes, short upturned nose and a wide mouth with full lips. She had a systolic murmur
grade 2-3/6 in the right upper sternal edge with radiation to right neck. She had further cardiac investigations that
showed a mild supravalvular aortic stenosis and mild multiple peripheral
pulmonary stenoses.

What is her likely diagnosis?

A Alagille syndrome

B Ehlers-Danlos syndrome

C Noonan's syndrome

D Silver-Russel syndrome

E Williams syndrome

Explanation
About ¼ th of children with supravalvular aortic stenosis will have Williams syndrome.

The rest of the responses listed are incorrect:

A and D are associated with pulmonary stenosis.


B and C are associated with pulmonary valve stenosis.

Ref:

1. Wayne R Waz. Williams-Beuren syndrome. www.uptodate.com


2. Hassan
Zamani, Kazem Babazadeh, Saeid Fattahi, Farzad Mokhtari-Esbuie. Williams-Beuren’s syndrome:
A case report. Case reports in medicine. Vol 2012 (2012). Article ID 585726, 4 pages.
46160

https://mypastest.pastest.com/Secure/TestMe/Browser/436619[‫ ص‬07:55:22 11/12/1437]


MyPastest

End
Tag Session
Question

Feedback

Difficulty: Average

Peer Responses

Previous Question

Session Progress

Responses Correct: 0

Responses Incorrect: 34

Responses Total: 34

Responses - % Correct: 0%

Blog
About Pastest
Contact Us
Help

© Pastest 2016

Next Question

https://mypastest.pastest.com/Secure/TestMe/Browser/436619[‫ ص‬07:55:22 11/12/1437]


MyPastest

Prefer to use the old MyPastest? Access it here »

Back to Filters

Question 31 of 65

Fig 1 is a family pedigree of cystic fibrosis.

What are the chances of Tom and Sally having an affected child?

A 1/120

B 1/9

C 2/9

D 1/3

E 2/3

Explanation
Both Tom and Sally have siblings with cystic fibrosis. Chances of both Tom and Sally being carriers are 2/3x2/3=
4/9. Chance of affected child is ¼. Therefore chances of Tom and Sally having an affected child is 4/9 x ¼ = 1/9

Ref:

https://mypastest.pastest.com/Secure/TestMe/Browser/436619[‫ ص‬07:55:43 11/12/1437]


MyPastest

1. Nussbaum
RL, McInnes RR, Willard HF. Patterns of Single-Gene Inheritance. Thompson & Thompson
Genetics in Medicine. 7th Edition . Chapter 7.2007 ;115-149.
2. Nussbaum RL, McInnes RR, Willard HF. Genetic Counselling and Risk Assessment. Thompson &
Thompson Genetics in Medicine. 7th Edition . Chapter 19.2007 ;507-522.
End Session 46161

Tag Question

Feedback

Previous Question
Difficulty: Average

Peer Responses

Session Progress

Responses Correct: 0

Responses Incorrect: 35

Responses Total: 35

Responses - % Correct: 0%

Blog
About Pastest
Contact Us
Help

© Pastest 2016

Next Question

https://mypastest.pastest.com/Secure/TestMe/Browser/436619[‫ ص‬07:55:43 11/12/1437]


MyPastest

Prefer to use the old MyPastest? Access it here »

Back to Filters

Question 32 of 65

This is a family pedigree of haemophilia A. Jane's partner is unaffected.

What is the chance that a son of Jane would be affected?

A 0%

B 25%

C 50%

D 75%

E 100%

https://mypastest.pastest.com/Secure/TestMe/Browser/436619[‫ ص‬07:56:04 11/12/1437]


MyPastest

Explanation
Haemophilia A is an X-linked recessive condition.
Jane’s mother must be a carrier as she has affected daughters.
Jane, therefore, is a carrier.
End Session
The sons of a carrier female (with an unaffected partner) will have a 50% chance of being affected. Daughters of a
carrier female (with an unaffected partner) will have a 50% chance of being a carrier.

Affected male (with unaffected partner) will have all daughters as carriers and all sons unaffected.

Ref:

1. Nussbaum
RL, McInnes RR, Willard HF. Patterns of Single-Gene Inheritance. Thompson & Thompson
Genetics in Medicine. 7th Edition . Chapter 7.2007 ;115-149.
2. Nussbaum RL, McInnes RR, Willard HF. Genetic Counselling and Risk Assessment. Thompson &
ThompsonPrevious
Genetics inQuestion
Medicine. 7th Edition . Chapter 19.2007 ;507-522.
46162

Tag Question

Feedback

Difficulty: Average

Peer Responses

Session Progress

Responses Correct: 0

Responses Incorrect: 36

Responses Total: 36

Responses - % Correct: 0%

Next Question

https://mypastest.pastest.com/Secure/TestMe/Browser/436619[‫ ص‬07:56:04 11/12/1437]


MyPastest

Blog
About Pastest
Contact Us
Help

© Pastest 2016

https://mypastest.pastest.com/Secure/TestMe/Browser/436619[‫ ص‬07:56:04 11/12/1437]


MyPastest

Prefer to use the old MyPastest? Access it here »

Back to Filters

Question 33 of 65

You are a specialist paediatric trainee. You see a 5-year-old


boy with lower limb weakness and calf muscle
hypertrophy. The weakness is progressive in nature in the last 1 year, and he now requires support
to climb up the
stairs.

What is the most appropriate next step in the plan of investigations?

A Anti-nuclear antibodies

B Genetic testing

C MRI

D Muscle biopsy

E Serum creatine kinase

Explanation
This is a boy with signs and symptoms suggestive of Duchenne Muscular Dystrophy (DMD). When there is a
clinical suspicion of DMD, screening should be done with creatine kinase before proceeding to further tests.

Muscle biopsy and genetic testing can be done to confirm diagnosis but they are not the first line test.

Ref:

1. Velu
Nair et al. Successful bone marrow transplantation in a patient with Diamond-Blackfan Anemia with
co-existing Duchenne Muscular Dystrophy: A Case Report. J of Medical Case Reports 2011 (5);216.
2. Marc L Miller. Approach to the patient with muscle weakness. www.uptodate.com
3. Basil T Darras. Clinical features and diagnosis of Duchenne and Becker muscular dystrophy.
www.uptodate.com
4. Katharine
Bushby et al. Diagnosis and management of Duchenne Muscular Dystrophy,
part 1; diagnosis,
and pharmacological and psychosocial management. The
Lancet Neurology Jan 2010, Vol 9 (1);77-93.
46163

https://mypastest.pastest.com/Secure/TestMe/Browser/436619[‫ ص‬07:56:25 11/12/1437]


MyPastest

End Session
Tag Question

Feedback

Difficulty: Average

Peer Responses

Previous Question

Session Progress

Responses Correct: 0

Responses Incorrect: 37

Responses Total: 37

Responses - % Correct: 0%

Blog
About Pastest
Contact Us
Help

© Pastest 2016

Next Question

https://mypastest.pastest.com/Secure/TestMe/Browser/436619[‫ ص‬07:56:25 11/12/1437]


MyPastest

Prefer to use the old MyPastest? Access it here »

Back to Filters

Question 34 of 65

You are a specialist paediatric trainee. You see a 14-year-old girl with symptoms of clumsiness in the morning.
She was diagnosed to have absence seizure at the age of 5 years but has been seizure free in the last 9 years
without any treatment. Her father has epilepsy. She had an EEG done. She is diagnosed with epilepsy.

What is the first line anti-epileptic of choice for her likely type of seizure?

A Carbamazepine

B Clonazepam

C Lamotrigine

D Levetiracetam

E Sodium Valproate

Explanation
This girl has typical presentation of juvenile myoclonic epilepsy. Approximately 3%–8% of childhood absence
epilepsies evolve into juvenile myoclonic epilepsy(JME). JME may be responsible for about 10% of all epilepsies.
80% of patients with JME begin having seizures between ages 12-16.

Lamotrigine, Levetiracetam and clonazepam are used in JME but they are not the first line treatment.

Ref:

1. R Renganathan, N Delanty. Juvenile myoclonic epilepsy: under-appreciated and under-diagnosed.Postgrad


Med J 2003;79:78-80.
2. Epilepsy Action. www.epilepsy.org.uk
3. https://www.nice.org.uk/guidance/qs27
46164

https://mypastest.pastest.com/Secure/TestMe/Browser/436619[‫ ص‬07:56:45 11/12/1437]


MyPastest

End
Tag Session
Question

Feedback

Difficulty: Average

Peer Responses

Previous Question

Session Progress

Responses Correct: 0

Responses Incorrect: 38

Responses Total: 38

Responses - % Correct: 0%

Blog
About Pastest
Contact Us
Help

© Pastest 2016

Next Question

https://mypastest.pastest.com/Secure/TestMe/Browser/436619[‫ ص‬07:56:45 11/12/1437]


MyPastest

Prefer to use the old MyPastest? Access it here »

End Session
Back to Filters

Question 35 of 65

Previous Question
You are advising an anxious primigravida about the various pre-natal screening tests available.

The following are the principal indications for prenatal diagnosis by invasive testing EXCEPT:

A Abnormal maternal serum screening and ultrasound examination on routine screening.

B Age 30 and over

C Family history of Duchenne Muscular Dystrophy

D Presence of structural chromosomal abnormality in one of the parents

E Risk of a neural tube defect

Explanation
Advanced maternal age of 35 and over is an indication for prenatal diagnosis by invasive testing.

Screening tests UK
Ref:

1. Nussbaum
RL, McInnes RR, Willard HF. Prenatal Diagnosis.Thompson & Thompson Genetics in
Medicine. 7thEdition . Chapter 15 .2007 ;443-459.
46165

Tag Question

Feedback
Next Question

https://mypastest.pastest.com/Secure/TestMe/Browser/436619[‫ ص‬07:57:04 11/12/1437]


MyPastest

Difficulty: Average

Peer Responses

Session Progress

Responses Correct: 0

Responses Incorrect: 39

Responses Total: 39

Responses - % Correct: 0%

Blog
About Pastest
Contact Us
Help

© Pastest 2016

https://mypastest.pastest.com/Secure/TestMe/Browser/436619[‫ ص‬07:57:04 11/12/1437]


MyPastest

Prefer to use the old MyPastest? Access it here »

Back to Filters

Question 36 of 65

Jane, a 40-year-old pregnant woman at 14 weeks’ gestation underwent routine screening - maternal serum
screening (triple screen examination). She has been taking folic acid 1 month before conception till now.

Results show elevated alpha-fetoprotein (AFP).

These are the likely causes of increased alpha-fetoprotein in Jane EXCEPT:

A Anencephaly

B Liver necrosis

C Sacrococcygealteratoma

D Spina bifida

E Trisomy 21

Explanation
Trisomy 21 causes reduced levels of AFP.

All other answers are possible causes of elevated maternal serum AFP.

Ref:

1. Nussbaum
RL, McInnes RR, Willard HF. Prenatal Diagnosis.Thompson & Thompson Genetics in
Medicine. 7thEdition . Chapter 15 .2007 ;443-459.
46166

Next Question

https://mypastest.pastest.com/Secure/TestMe/Browser/436619[‫ ص‬07:57:22 11/12/1437]


MyPastest

Tag Question

Feedback

End Session
Difficulty: Average

Peer Responses

Previous Question

Session Progress

Responses Correct: 0

Responses Incorrect: 40

Responses Total: 40

Responses - % Correct: 0%

Blog
About Pastest
Contact Us
Help

© Pastest 2016

https://mypastest.pastest.com/Secure/TestMe/Browser/436619[‫ ص‬07:57:22 11/12/1437]


MyPastest

Prefer to use the old MyPastest? Access it here »

Back to Filters

Question 37 of 65

You are a Specialist Paediatric Trainee. You see a 4-year-old


boy with behavioural problems in the community
paediatric clinic. He is noted to have thin upper lip, smooth philtrum, and short palpebral fissures.

There is a history of probable maternal alcohol exposure.

What is your next step in management?

A Chromosomal micro-array analysis

B FISH test to look for 22q deletion

C Karyotype analysis

D MRI

E Referral to the relevant professionals involved in the management of fetal alcohol syndrome

Explanation
When there is uncertainty about the alcohol use, it is important to rule out alternative diagnoses by doing a micro-
array test.

Referral to relevant professionals in the management of fetal alcohol syndrome should only be done once
alternative diagnoses are excluded and
a diagnosis of fetal alcohol syndrome is confirmed.

Ref:

1. Sofia
Douzgou et al. Diagnosing fetal alcohol syndrome: new insights from newer genetic technologies.
Arch Dis Child 2012;97:812-817.
46167

Next Question

https://mypastest.pastest.com/Secure/TestMe/Browser/436619[‫ ص‬07:57:40 11/12/1437]


MyPastest

Tag Question

Feedback
End Session
Difficulty: Average

Peer Responses

Previous Question

Session Progress

Responses Correct: 0

Responses Incorrect: 41

Responses Total: 41

Responses - % Correct: 0%

Blog
About Pastest
Contact Us
Help

© Pastest 2016

https://mypastest.pastest.com/Secure/TestMe/Browser/436619[‫ ص‬07:57:40 11/12/1437]


MyPastest

Prefer to use the old MyPastest? Access it here »

Back to Filters

Question 38 of 65

A full term baby boy, Sam, was born to an epileptic mother, Claire. Claire has been on drug X till 10 weeks of
pregnancy. Once pregnancy was confirmed, it was stopped. Sam was born with a depressed nasal bridge,
hypertelorism, short nose, cleft palate, hypoplasia of nails and hirsutism.

What is drug X likely to be?

A Clonazepam

B Ethosuximide

C Lamotrigine

D Phenytoin

E Sodium valproate

Explanation
Sam has distinctive features suggestive of fetal
hydantoin syndrome which results from maternal use of the
antiepileptic
drug, phenytoin.

Major malformations associated with Sodium valproate are spina bifida, atrial septal defect and hypospadias.

Ref:

1. National organization for rare disorders. rarediseases.org.


2. Shakya NS, Gurubacharya SM, Aryal DR. Fetal Hydantoin Syndrome: A Case Report. Journal of Nepal
Paediatric Society 2010.Vol 30 (1):57-59
46168

Next Question

https://mypastest.pastest.com/Secure/TestMe/Browser/436619[‫ ص‬07:57:58 11/12/1437]


MyPastest

Tag Question

Feedback

End Session
Difficulty: Average

Peer Responses

Previous Question

Session Progress

Responses Correct: 0

Responses Incorrect: 42

Responses Total: 42

Responses - % Correct: 0%

Blog
About Pastest
Contact Us
Help

© Pastest 2016

https://mypastest.pastest.com/Secure/TestMe/Browser/436619[‫ ص‬07:57:58 11/12/1437]


MyPastest

Prefer to use the old MyPastest? Access it here »

Back to Filters

Question 39 of 65

You are a specialist paediatric trainee. You see a 4-year-old


boy with intellectual disability in the Community
Paediatric Clinic. He has normal head circumference. and hypertelorism. The rest of his clinical examination is
unremarkable. There is no known family history of intellectual disability. You decide that he will need genetic
testing.

What is the first line test?

A Chromosomal microarray analysis

B FISH test to look for 22q 11 gene deletion

C Fragile X test

D G-banded karyotype

E Genome Sequencing

Explanation
Where there is no specific disorder clinically suspected, then the first line genetic test for unexplained intellectual
disability is chromosomal microarray analysis also known as CGH microarray (comparative genomic
hybridization). This is a method of more detailed chromosome analysis than that provided by karyotyping. Patient
genomic DNA and control genomic DNA are differentially labelled with different fluorescent probes and then
hybridized together. The ratio of fluorescent intensity between patient and control DNA is then compared which
detects areas of copy number difference. This can detect
microdeletions and microduplications as well as
anomalies that would have been visible on karyotype.

FISH testing (fluorescent in situ hybridization) is used to assess the copy number of specific DNA
sequences in
the genome. FISH can be performed much more rapidly than formal karyotyping however CGH microarray
together with other types of testing has largely superseded this process except in testing other members of a family
for a known chromosomal anomoly. There is no known history here.

https://mypastest.pastest.com/Secure/TestMe/Browser/436619[‫ ص‬07:58:17 11/12/1437]


MyPastest

Ref:

1. Penelope Pivalizza, Seema R Lalani. Intellectual disability (mental retardation) in children: Evaluation for a
cause. www.uptodate.com
46169
End Session

Tag Question

Feedback

Previous Question Difficulty: Average

Peer Responses

Session Progress

Responses Correct: 0

Responses Incorrect: 43

Responses Total: 43

Responses - % Correct: 0%

Blog
About Pastest
Contact Us
Help

© Pastest 2016

Next Question

https://mypastest.pastest.com/Secure/TestMe/Browser/436619[‫ ص‬07:58:17 11/12/1437]


MyPastest

Prefer to use the old MyPastest? Access it here »

Back to Filters

Question 40 of 65

You are a specialist paediatric trainee. You see a 4-year-old


boy with intellectual disability. On further
questioning, his mum tells
you that he also has lower limb weakness and that he requires support to walk up the
stairs. On examination, he has calf muscle pseudohypertrophy.

What would be the most appropriate first step to investigate his cause of intellectual disability?

A Chromosomal microarray analysis

B Creatine kinase

C G-banded karyotype

D Muscle biopsy

E Thyroid function test

Explanation
For boys with intellectual disability and proximal muscle weakness, the most appropriate first line test would be
serum creatine kinase to screen for Duchenne Muscular Dystrophy. Patients with characteristics suggesting
specific disorder should undergo specific testing to confirm or rule out that disorder.

Muscle biopsy may be indicated at a later stage but it is not a first line test.

Chromosomal microarray analysis would be used where there patients exhibit no characteristics suggestive of a
specific disorder.

Ref:

1. Penelope Pivalizza, Seema R Lalani. Intellectual disability (mental retardation) in children: Evaluation for a
cause. www.uptodate.com
46170

https://mypastest.pastest.com/Secure/TestMe/Browser/436619[‫ ص‬07:58:37 11/12/1437]


MyPastest

End
Tag Session
Question

Feedback

Difficulty: Average

Peer Responses

Previous Question

Session Progress

Responses Correct: 0

Responses Incorrect: 44

Responses Total: 44

Responses - % Correct: 0%

Blog
About Pastest
Contact Us
Help

© Pastest 2016

Next Question

https://mypastest.pastest.com/Secure/TestMe/Browser/436619[‫ ص‬07:58:37 11/12/1437]


MyPastest

Prefer to use the old MyPastest? Access it here »

Back to Filters

Question 41 of 65

A newborn screening lab scientist phones to inform you that the TSH for a 10-day-old baby, John, is 132 mU/L.

John’s mum and dad are first cousins.

Considering the most likely diagnosis and cause in John, what is the recurrence risk?

A 0

B 1/8

C 1/4

D 1/2

E 1

Explanation
John has congenital hypothyroidism as his TSH levels are high. As there is parental consanguinity, the most likely
cause of this is thyroid dyshormonogenesis. Thyroid dyshormonogenesis is
an autosomal recessive condition with
a 1:4 recurrence risk and is a permanent thryoid hormone deficiency that is present from birth.

Ref:

1. Congenital hypothyroidism. Initial Clinical Referral Standards and Guidelines. January 2013.
www.bsped.org.uk
46171

Next Question Tag Question

https://mypastest.pastest.com/Secure/TestMe/Browser/436619[‫ ص‬07:58:55 11/12/1437]


MyPastest

Feedback

End Session
Difficulty: Average

Peer Responses

Previous Question

Session Progress

Responses Correct: 0

Responses Incorrect: 45

Responses Total: 45

Responses - % Correct: 0%

Blog
About Pastest
Contact Us
Help

© Pastest 2016

https://mypastest.pastest.com/Secure/TestMe/Browser/436619[‫ ص‬07:58:55 11/12/1437]


MyPastest

Prefer to use the old MyPastest? Access it here »

Back to Filters

Question 42 of 65

Previous Question
Chromosomal microarray analysis can be used to detect these genetic conditions except:

A Angelman syndrome

B DiGeorge syndrome

C Fragile X

D Prader-Willi syndrome

E Williams syndrome

Explanation
Chromosomal microarray analysis does not detect tiny duplications and deletions of DNA segments within a
single gene as in Fragile X. The diagnosis of Fragile X syndrome is confirmed by molecular genetic testing of the
FMR1 gene.

Ref:

1. Wattendorf DJ et al. Diagnosis and management of Fragile X syndrome. AmFam Physician 2005 Jul
1;72(01):111-113.
2. National Coalition for Health Professional Education in Genetics. Genomic Tools: Chromosomal
microarray. www.nhcpeg.org.
46172

Tag Question
Next Question

https://mypastest.pastest.com/Secure/TestMe/Browser/436619[‫ ص‬07:59:13 11/12/1437]


MyPastest

Feedback

Difficulty: Average
End Session
Peer Responses

Session Progress

Responses Correct: 0

Responses Incorrect: 46

Responses Total: 46

Responses - % Correct: 0%

Blog
About Pastest
Contact Us
Help

© Pastest 2016

https://mypastest.pastest.com/Secure/TestMe/Browser/436619[‫ ص‬07:59:13 11/12/1437]


MyPastest

Prefer to use the old MyPastest? Access it here »

End Session
Back to Filters

Question 43 of 65

Previous Question
Recombinant human growth hormone (Somatropin) is currently the only active treatment option for growth failure
in children with the following conditions EXCEPT:

A Achondroplasia

B Prader-Willi syndrome

C Short Stature Homeobox (SHOX) gene deficiency

D Small for gestational age

E Turner syndrome

Explanation
Achondroplasia is not an indication for Somatropin. The rest of the answers are indication for Somatropin as per
NICE guidance.

Ref:

1. Human growth hormone (Somatropin) for the treatment of growth failure in children. NICE guidance.
www.nice.org.uk
46173

Tag Question

Feedback
Next Question

https://mypastest.pastest.com/Secure/TestMe/Browser/436619[‫ ص‬07:59:32 11/12/1437]


MyPastest

Difficulty: Average

Peer Responses

Session Progress

Responses Correct: 0

Responses Incorrect: 47

Responses Total: 47

Responses - % Correct: 0%

Blog
About Pastest
Contact Us
Help

© Pastest 2016

https://mypastest.pastest.com/Secure/TestMe/Browser/436619[‫ ص‬07:59:32 11/12/1437]


MyPastest

Prefer to use the old MyPastest? Access it here »

End Session
Back to Filters

Question 44 of 65

Previous Question
You are a specialist paediatric trainee. You are called by the midwife as she is concerned about the low blood
sugars in Oliver, a 20-hour-old term baby. Oliver has anterior linear ear lobe creases, large protruding tongue and
umbilical hernia. His birth weight is 4.5 kg. He is clinically suspected to have condition X.

From the following list, which other condition has a similar pattern of inheritance to condition X?

A Angelman syndrome

B Csytis fibrosis

C Haemophilia A

D Neurofibromatosis type 1

E Vitamin D resistant rickets

46174

Tag Question

Feedback

Difficulty: Average

Peer Responses

Next Question

https://mypastest.pastest.com/Secure/TestMe/Browser/436619[‫ ص‬07:59:51 11/12/1437]


MyPastest

Session Progress

Responses Correct: 0

Responses Incorrect: 48

Responses Total: 48

Responses - % Correct: 0%

Blog
About Pastest
Contact Us
Help

© Pastest 2016

https://mypastest.pastest.com/Secure/TestMe/Browser/436619[‫ ص‬07:59:51 11/12/1437]


MyPastest

Prefer to use the old MyPastest? Access it here »

Back to Filters

Question 45 of 65

John is a 6-day-old baby with Tetralogy of Fallot and a cleft


palate. He has a broad nasal bridge with a bulbous
nasal tip, narrow palpebral fissures and micrognathia. He has had genetic testing to confirm the suspected clinical
diagnosis. He presents with seizure to the emergency department.

Considering his likely diagnosis, what is the most likely cause of his seizure?

A Hypertension

B Hypocalcemia

C Hypoglycemia

D Hyponatraemia

E Intracranial bleed

Explanation
John has characteristic features of DiGeorge syndrome - a 22q11.2 deletion. The main clinical presentation of
DiGeorge syndrome are cardiac anomalies (tetralogy of Fallot, truncus arteriosus, Interrupted aortic arch or
aberrant right subclavian artery), palatal abnormalities, immunodeficiency, hypocalaemia and developmental
delay. In the newborn period, hypocalcaemia can present as
seizures.

Ref:

1. Christine M Seroogy. DiGeorge(22q11.2 deletion) syndrome: Clinical features and diagnosis.


www.uptodate.com.
2. Erawati V Bawle. DiGeorge Syndrome. Emedicine.medscape.com
3. DiGeorge syndrome (22q11 deletion). www.nhs.uk.
46175

https://mypastest.pastest.com/Secure/TestMe/Browser/436619[‫ ص‬08:00:10 11/12/1437]


MyPastest

End
Tag Session
Question

Feedback

Difficulty: Average

Peer Responses

Previous Question

Session Progress

Responses Correct: 0

Responses Incorrect: 49

Responses Total: 49

Responses - % Correct: 0%

Blog
About Pastest
Contact Us
Help

© Pastest 2016

Next Question

https://mypastest.pastest.com/Secure/TestMe/Browser/436619[‫ ص‬08:00:10 11/12/1437]


MyPastest

Prefer to use the old MyPastest? Access it here »

Back to Filters

Question 46 of 65

Previous Question
An 11-year-old boy presents to the emergency department with a
history of multiple episodes of generalised tonic
clonic seizure for the last 10 days. In the past, he has had multiple admissions for the same reason and was on anti-
epileptic drugs since the age of 1 year with
poor control.

He was born of non-consanguineous parents with an


uneventful birth history. There is no history of seizure in
other family members. He has multiple hyperpigmented papules (adenoma sebaceum) over the nasolabial region
and 5 hypopigmented macules (ash leaf) over the lower limbs along with a shagreen patch over the lateral
aspect
of the left buttock. He is clinically diagnosed to have condition X.

Condition X has similar pattern of inheritance as:

A Beckwith-Wiedemann syndrome

B Cystic Fibrosis

C Duchenne Muscular Dystrophy

D Neurofibromatosis Type 1

E Hypophosphataemic rickets

Explanation

46176

Tag Question
Next Question

https://mypastest.pastest.com/Secure/TestMe/Browser/436619[‫ ص‬08:00:29 11/12/1437]


MyPastest

Feedback

Difficulty: Average
End Session
Peer Responses

Session Progress

Responses Correct: 0

Responses Incorrect: 50

Responses Total: 50

Responses - % Correct: 0%

Blog
About Pastest
Contact Us
Help

© Pastest 2016

https://mypastest.pastest.com/Secure/TestMe/Browser/436619[‫ ص‬08:00:29 11/12/1437]


MyPastest

Prefer to use the old MyPastest? Access it here »

Back to Filters

Question 47 of 65

Here is a family pedigree of condition X.

Another condition with similar pattern of inheritance is:

Your answer was correct

A Duchenne Muscular Dystrophy

B Gaucher disease

C Incontinenta pigmenti

D Marfan syndrome

E Prader-Willi syndrome

https://mypastest.pastest.com/Secure/TestMe/Browser/436619[‫ ص‬08:00:52 11/12/1437]


MyPastest

46177

End Session
Tag Question

Feedback

Difficulty: Average

Peer Responses

Previous Question

Session Progress

Responses Correct: 1

Responses Incorrect: 50

Responses Total: 51

Responses - % Correct: 2%

Blog
About Pastest
Contact Us
Help

© Pastest 2016

Next Question

https://mypastest.pastest.com/Secure/TestMe/Browser/436619[‫ ص‬08:00:52 11/12/1437]


MyPastest

Prefer to use the old MyPastest? Access it here »

Back to Filters

Question 48 of 65

This is a family pedigree of condition X.

Another condition with similar pattern of inheritance is:

A Angelman syndrome

B Cystic fibrosis

C Haemophilia A

D Hereditary spherocytosis

https://mypastest.pastest.com/Secure/TestMe/Browser/436619[‫ ص‬08:01:13 11/12/1437]


MyPastest

E Vitamin D resistant rickets

46178
End Session

Tag Question

Feedback

Previous Question Difficulty: Average

Peer Responses

Session Progress

Responses Correct: 1

Responses Incorrect: 51

Responses Total: 52

Responses - % Correct: 2%

Blog
About Pastest
Contact Us
Help

© Pastest 2016

Next Question

https://mypastest.pastest.com/Secure/TestMe/Browser/436619[‫ ص‬08:01:13 11/12/1437]


MyPastest

Prefer to use the old MyPastest? Access it here »

End Session
Back to Filters

Question 49 of 65

Previous Question
A baby boy has been diagnosed with Down Syndrome on the postnatal ward. You speak with his parents and
discover that the baby has two first cousins with Down Syndrome.

What is likely genetic basis of this baby’s Down Syndrome?

Your answer was incorrect

A Balanced Robertsonian translocation

B Ring chromosome 21

C Trisomy of chromosome 21

D Unbalanced Robertsonian translocation

E Uniparental disomy

46209

Tag Question

Feedback

Difficulty: Average

Next
PeerQuestion
Responses

https://mypastest.pastest.com/Secure/TestMe/Browser/436619[‫ ص‬08:01:34 11/12/1437]


MyPastest

Session Progress

Responses Correct: 1

Responses Incorrect: 52

Responses Total: 53

Responses - % Correct: 2%

Blog
About Pastest
Contact Us
Help

© Pastest 2016

https://mypastest.pastest.com/Secure/TestMe/Browser/436619[‫ ص‬08:01:34 11/12/1437]


MyPastest

Prefer to use the old MyPastest? Access it here »

End Session
Back to Filters

Question 50 of 65

Previous Question
Which of the following statements are true regarding embryological development?

A Week 3 – neural crest visible, eyelid folds develop

B Week 6 – upper limb bud nerves form, hCG stimulates testosterone production

C Week 12 – lungs secrete surfactant, intestines sited within abdominal cavity

D Week 20- fetus startles to sound, skin hair develops

E Week 24- lung terminal sac development, male testes descend

46210

Tag Question

Feedback

Difficulty: Average

Peer Responses

Next Question

https://mypastest.pastest.com/Secure/TestMe/Browser/436619[‫ ص‬08:01:53 11/12/1437]


MyPastest

Session Progress

Responses Correct: 1

Responses Incorrect: 53

Responses Total: 54

Responses - % Correct: 2%

Blog
About Pastest
Contact Us
Help

© Pastest 2016

https://mypastest.pastest.com/Secure/TestMe/Browser/436619[‫ ص‬08:01:53 11/12/1437]


MyPastest

Prefer to use the old MyPastest? Access it here »

End Session
Back to Filters

Question 51 of 65

Previous Question
A seven month old boy with Tetralogy of Fallot attends pre-operative clinic. There have been concerns about his
weight gain and
his mother has noted that, on occasion, milk comes from his nasal passages when he is feeding.
His pre-operative blood tests reveal the presence of hypocalcaemia.

Which investigation would be the most appropriate next step to reveal a unifying diagnosis?

A Array CGH (Comparative Genomic Hybridisation)

B FISH (Fluorescence in-situ hybridisation)

C Karyotype analysis

D Single gene sequencing of CHD7 regarding CHARGE syndrome

E Whole genome sequencing

47179

Tag Question

Feedback

Difficulty: Average

Peer Responses

Next Question

https://mypastest.pastest.com/Secure/TestMe/Browser/436619[‫ ص‬08:02:13 11/12/1437]


MyPastest

Session Progress

Responses Correct: 1

Responses Incorrect: 54

Responses Total: 55

Responses - % Correct: 2%

Blog
About Pastest
Contact Us
Help

© Pastest 2016

https://mypastest.pastest.com/Secure/TestMe/Browser/436619[‫ ص‬08:02:13 11/12/1437]


MyPastest

Prefer to use the old MyPastest? Access it here »

Back to Filters

Question 52 of 65

A ten year old boy attends a specialist neurology clinic as he is starting to have difficulty with walking. Members
of his family, including his mother, uncle, grandmother and other relatives; as demonstrated by the pedigree drawn
by the nurse specialist in the clinic, have experienced progressive ataxia and visual loss.

Which pattern of inheritance is most likely in this family?

A Autosomal dominant

B Autosomal recessive

C Mitochondrial

D X-linked dominant

https://mypastest.pastest.com/Secure/TestMe/Browser/436619[‫ ص‬08:02:34 11/12/1437]


MyPastest

E X-linked recessive

47180
End Session

Tag Question

Feedback

Previous Question Difficulty: Average

Peer Responses

Session Progress

Responses Correct: 1

Responses Incorrect: 55

Responses Total: 56

Responses - % Correct: 2%

Blog
About Pastest
Contact Us
Help

© Pastest 2016

Next Question

https://mypastest.pastest.com/Secure/TestMe/Browser/436619[‫ ص‬08:02:34 11/12/1437]


MyPastest

Prefer to use the old MyPastest? Access it here »

End Session
Back to Filters

Question 53 of 65

Previous Question
A four week old baby is seen in paediatric respiratory clinic
after being diagnosed with cystic fibrosis. He has
undergone genetic testing following an abnormal newborn screening result and his CFTR gene
mutations have
been identified. His mother has brought her 6 year old son with her and would like him to be tested as well as he is
“always coughing”.

What is the most appropriate next step?

A Arrange to refer the older boy to Clinical Genetics

B Explain that a GP referral to a Paediatrician would be appropriate for assessment and any relevant
investigation.

C Explain that you cannot investigate the older child as he has not been referred.

D In
view of the fact that the causative mutations in the family are known, offer genetic testing for the
older child, after obtaining informed consent from the mother.

E Undertake
a clinical assessment of the older boy to look for any signs that he may have CF in order to
inform the appropriate next steps in his management

47181

Tag Question

Feedback

Next Question
Difficulty: Average

https://mypastest.pastest.com/Secure/TestMe/Browser/436619[‫ ص‬08:02:56 11/12/1437]


MyPastest

Peer Responses

Session Progress

Responses Correct: 1

Responses Incorrect: 56

Responses Total: 57

Responses - % Correct: 2%

Blog
About Pastest
Contact Us
Help

© Pastest 2016

https://mypastest.pastest.com/Secure/TestMe/Browser/436619[‫ ص‬08:02:56 11/12/1437]


MyPastest

Prefer to use the old MyPastest? Access it here »

End Session
Back to Filters

Question 54 of 65

Previous Question
You are asked to attend the delivery of a baby in view of the
fact that the pregnancy has been concealed. Although
the estimated gestation is term, there is some uncertainty and neonatal input may be required.

The infant’s weight is 4.1kg and it has the appearance of macrosomia. The infant does not require resuscitation,
however on your initial assessment there are some signs of respiratory distress and the presence of hypotonia. You
therefore arrange for admission to the neonatal unit.

Further examination and investigation on the unit reveals the presence of anal atresia, a deep sacral pit and a
cardiac echo reveals a VSD. The blood tests reveal the presence of polycythaemia but no other significant
abnormalities.

Which of the following investigations is most likely to give the cause for the infant’s problems?

A A chest X-ray and renal ultrasound scan to investigate the possibility of VACTERL association

B A maternal HbA1c measurement

C An array CGH (microarray)

D Enquiring about antenatal exposure to recreational drugs

E Undertaking a Ballard Maturational Assessment of gestational age

47182

Tag Question

Feedback Next Question

https://mypastest.pastest.com/Secure/TestMe/Browser/436619[‫ ص‬08:03:19 11/12/1437]


MyPastest

Difficulty: Average

Peer Responses

Session Progress

Responses Correct: 1

Responses Incorrect: 57

Responses Total: 58

Responses - % Correct: 2%

Blog
About Pastest
Contact Us
Help

© Pastest 2016

https://mypastest.pastest.com/Secure/TestMe/Browser/436619[‫ ص‬08:03:19 11/12/1437]


MyPastest

Prefer to use the old MyPastest? Access it here »

Back to Filters

Question 55 of 65

A 17 year old is attending a follow up appointment in the Neurology clinic to plan her transition to adult services.
She and several members of her family have been affected by lower limb weakness and spasticity and require the
use of a wheelchair. The nurse specialist
has drawn the pedigree, noting that the patient is currently 10 weeks
pregnant and is married to her first cousin. The patient would like to know the chances that her baby will also be
affected with the condition.

What is the most likely chance that the unborn baby, indicated by by the diamond shape, will be affected by the
condition?

A 0%

B 25%

C 50%

https://mypastest.pastest.com/Secure/TestMe/Browser/436619[‫ ص‬08:03:41 11/12/1437]


MyPastest

D 100%

E It depends on the gender of the child


End Session
47502

Tag Question

Feedback
Previous Question

Difficulty: Average

Peer Responses

Session Progress

Responses Correct: 1

Responses Incorrect: 58

Responses Total: 59

Responses - % Correct: 2%

Blog
About Pastest
Contact Us
Help

© Pastest 2016

Next Question

https://mypastest.pastest.com/Secure/TestMe/Browser/436619[‫ ص‬08:03:41 11/12/1437]


MyPastest

Prefer to use the old MyPastest? Access it here »

End Session
Back to Filters

Question 56 of 65

Previous Question
You are called to attend the delivery of a term infant in view of abnormalities detected antenatally. Ultrasound
scanning in utero
revealed the presence of a raised nuchal translucency measurement and a
double bubble sign.
Subsequent investigations confirmed the suspected diagnosis.

In terms of the chances of the same condition happening again within the family, which test is the most
appropriate to arrange for the neonate?

A Array CGH (microarray)

B FISH

C Karyotype

D QF-PCR

E Whole genome analysis

47503

Tag Question

Feedback

Difficulty: Average

Peer Responses
Next Question

https://mypastest.pastest.com/Secure/TestMe/Browser/436619[‫ ص‬08:04:01 11/12/1437]


MyPastest

Session Progress

Responses Correct: 1

Responses Incorrect: 59

Responses Total: 60

Responses - % Correct: 2%

Blog
About Pastest
Contact Us
Help

© Pastest 2016

https://mypastest.pastest.com/Secure/TestMe/Browser/436619[‫ ص‬08:04:01 11/12/1437]


MyPastest

Prefer to use the old MyPastest? Access it here »

End Session
Back to Filters

Question 57 of 65

Previous Question
You have just seen a 4 year old boy in the Community Paediatric clinic after referral from his health visitor. Your
assessment revealed mild-moderate global developmental delay. The boy had difficulty focussing on the tasks
requested and on the physical examination you noted microcephaly and a thin vermilion border of the upper lip.
His facial appearance is similar to that of his father, however both parents have head circumference measurements
that are within normal limits.

Which next step is most likely to lead to the diagnosis in this child?

A Array CGH (microarray)

B Asking about potential alcohol exposure in utero

C Testing for Fragile X syndrome

D Thyroid function testing

E Undertaking an Autism Diagnosis Observation Schedule (ADOS) assessment

47504

Tag Question

Feedback

Difficulty: Average

Next Question
Peer Responses

https://mypastest.pastest.com/Secure/TestMe/Browser/436619[‫ ص‬08:04:22 11/12/1437]


MyPastest

Session Progress

Responses Correct: 1

Responses Incorrect: 60

Responses Total: 61

Responses - % Correct: 2%

Blog
About Pastest
Contact Us
Help

© Pastest 2016

https://mypastest.pastest.com/Secure/TestMe/Browser/436619[‫ ص‬08:04:22 11/12/1437]


MyPastest

Prefer to use the old MyPastest? Access it here »

Back to Filters

Question 58 of 65

Previous Question
You are seeing a five year old girl for a follow up in the Community Paediatric clinic. She has a complex medical
history including
a ventricular septal defect, which required surgical closure, and a cleft palate. She has learning
difficulty with particular problems with language and communication. Her facial features are unusual; her
forehead is broad with bifrontal narrowing, she has epicanthic folds, a broad upturned nose, a long and shallow
philtrum and a long upper lip. In her family history, her mother has epilepsy and has been on medication since she
was a child, her father has severe dyslexia and a maternal cousin has been diagnosed with autism.

The mother would like to have another baby but is concerned about another child having similar medical
problems.

What is the most appropriate advice to give the mother to aim to reduce this risk?

A Advise her to avoid alcohol in a future pregnancy

B Advise that she take folic acid daily for 6 weeks prior to conception until the end of the first trimester

C Ask her GP to refer her to a Neurologist to review her anticonvulsant prescription and dosing

D Ensure that her blood glucose levels are monitored closely in pregnancy and treated appropriately

E Refer
the daughter to a Clinical Geneticist for investigation as, if a genetic cause is identified, she
could have testing in a future pregnancy

47505

Tag Question

Feedback Next Question

https://mypastest.pastest.com/Secure/TestMe/Browser/436619[‫ ص‬08:04:44 11/12/1437]


MyPastest

Difficulty: Average
End Session
Peer Responses

Session Progress

Responses Correct: 1

Responses Incorrect: 61

Responses Total: 62

Responses - % Correct: 2%

Blog
About Pastest
Contact Us
Help

© Pastest 2016

https://mypastest.pastest.com/Secure/TestMe/Browser/436619[‫ ص‬08:04:44 11/12/1437]


MyPastest

Prefer to use the old MyPastest? Access it here »

End Session
Back to Filters

Question 59 of 65

Previous Question
In the general paediatric clinic you have just seen a two year old girl after she was referred by her GP in view of
an incidental pick-up of a heart murmur. She is otherwise well, but there is a grade 2/6 systolic murmur most
audible in the 2nd intercostal space
immediately to the left of the sternum. She is pink and has normal pulses
palpable. You are arranging for her to have an echocardiogram to confirm your suspicions.

You note that her height is just above the 0.4th


centile; her mother is 5’9” and her father, who is not present, is
5’6” tall. She has downslanting palpebral fissures and strikingly blue eyes. Her ears are low set and are rotated
posteriorly. Her mother says that she resembles her father. You also note a broad prominent chest. Her
development to date has been normal.

Which genetic investigation is most likely to reveal the diagnosis for this child?

A An array CGH or karyotype for assessment of the sex chromosomes

B Genetic testing of CHD7 regarding CHARGE syndrome

C Genetic testing of KRAS and related genes for Costello syndrome

D Genetic testing of PTPN11 and related genes for Noonan syndrome

E Whole genome screening to aim to delineate a single gene cause

47506

Tag Question

Feedback
Next Question

https://mypastest.pastest.com/Secure/TestMe/Browser/436619[‫ ص‬08:05:05 11/12/1437]


MyPastest

Difficulty: Average

Peer Responses

Session Progress

Responses Correct: 1

Responses Incorrect: 62

Responses Total: 63

Responses - % Correct: 2%

Blog
About Pastest
Contact Us
Help

© Pastest 2016

https://mypastest.pastest.com/Secure/TestMe/Browser/436619[‫ ص‬08:05:05 11/12/1437]


MyPastest

Prefer to use the old MyPastest? Access it here »

End Session
Back to Filters

Question 60 of 65

Previous Question
You are seeing a 14 year old girl in clinic as her mother is concerned that she is the shortest in her class and she
doesn’t seem to be having a growth spurt like her friends.

Her height is on the 0.4th


centile and you notice some differences in her appearance. There is a degree of webbing
of her neck and her nipples appear widely spaced. Assessment of her pubertal status reveals tanner stage I
regarding breast and pubic hair development. Her mother recalls starting her periods at age 15 years.

Which of the following investigations is most likely to give the cause for the girl’s problems?

A Array CGH (comparative genomic hybridisation)

B Genetic testing of PTPN11 and related genes regarding Noonan syndrome

C Growth hormone analysis

D No
investigation is required at present; it is likely familial delayed menarche and therefore follow up
in clinic is more appropriate

E Thyroid function testing

47507

Tag Question

Feedback

Next Question
Difficulty: Average

https://mypastest.pastest.com/Secure/TestMe/Browser/436619[‫ ص‬08:05:26 11/12/1437]


MyPastest

Peer Responses

Session Progress

Responses Correct: 1

Responses Incorrect: 63

Responses Total: 64

Responses - % Correct: 2%

Blog
About Pastest
Contact Us
Help

© Pastest 2016

https://mypastest.pastest.com/Secure/TestMe/Browser/436619[‫ ص‬08:05:26 11/12/1437]


MyPastest

Prefer to use the old MyPastest? Access it here »

Back to Filters

Question 61 of 65

In outpatient clinic, you are seeing a 6 year old girl in view of developmental concerns. Her family has moved to
the UK relatively recently from Brazil. In the history you note that there were
no significant antenatal concerns
and there is no family history of learning difficulty. She was born at full term via a normal delivery weighing
3200g with a head circumference of 35cm.

Her early development was reported to be normal. She sat at 6 months, crawled at 10 months and had two words
at 16 months. Her mother first had concerns at the age of 2 years when the girl stopped making eye contact and
didn’t use any words to communicate.

On examination you notice that the girl is difficult to engage, she is able to sit independently and makes regular
wringing hand movements. Her head circumference is 1cm less than the 0.4th centile. You assess her development
as severely delayed in all areas.

Which of the following investigations is most likely to give the cause for the girl’s problems?

A Array CGH (comparative genomic hybridisation)

B Autism diagnosis observation scale (ADOS) assessment

C Chromosome 15q methylation analysis for Angelman syndrome

D Maternal serology for Zika antibodies

E MECP2 analysis for Rett syndrome

47508

Next Question Tag Question

https://mypastest.pastest.com/Secure/TestMe/Browser/436619[‫ ص‬08:05:48 11/12/1437]


MyPastest

Feedback

End Session
Difficulty: Average

Peer Responses

Previous Question

Session Progress

Responses Correct: 1

Responses Incorrect: 64

Responses Total: 65

Responses - % Correct: 2%

Blog
About Pastest
Contact Us
Help

© Pastest 2016

https://mypastest.pastest.com/Secure/TestMe/Browser/436619[‫ ص‬08:05:48 11/12/1437]


MyPastest

Prefer to use the old MyPastest? Access it here »

Back to Filters

Question 62 of 65

Previous Question
A twelve year old boy is brought to see you in Paediatric outpatients. His mother has heard, via her brother-in-law,
that her estranged husband has recently been in hospital for emergency surgery to
repair his aorta as it “was going
to burst”. Her ex-husband
is 34 years old and is 190cm tall. The mother is concerned as the boy is tall and
apparently resembles his father. The mother has very little contact with the father, but the brother-in-law said that
his doctors mentioned this sort of thing can run in families.

The twelve year old is physically well and his height is on the 98th centile. His examination is unremarkable.

In addition to arranging an echocardiogram for the boy, which would be the most appropriate next step in terms of
investigation?

A Arrange for genetic testing of the gene FBN1 regarding Marfan syndrome in the boy

B Contact the ex-husband to ask for consent to contact his doctors for more information regarding his
health

C Continue to offer regular echocardiograms to monitor the boy’s aorta

D Ensure
that the GP refers the boy to an adult Cardiologist, when he is entering adulthood, for
appropriate assessment and monitoring

E Refer the boy to a Clinical Geneticist, ideally through a joint cardiac-genetics clinic if available

47509

Tag Question

Feedback Next Question

https://mypastest.pastest.com/Secure/TestMe/Browser/436619[‫ ص‬08:06:10 11/12/1437]


MyPastest

Difficulty: Average
End Session
Peer Responses

Session Progress

Responses Correct: 1

Responses Incorrect: 65

Responses Total: 66

Responses - % Correct: 2%

Blog
About Pastest
Contact Us
Help

© Pastest 2016

https://mypastest.pastest.com/Secure/TestMe/Browser/436619[‫ ص‬08:06:10 11/12/1437]


MyPastest

Prefer to use the old MyPastest? Access it here »

Back to Filters

Question 63 of 65

In the metabolic clinic you are seeing a 3 year old girl for her follow up appointment. She is known to the service
having presented very early in life with growth failure and coarse physical features. She
has marked
developmental delay and has suffered hip dislocation in the past. Her one year old brother is also known to the
clinic with similar problems.

The nurse specialist reports that the mother’s sister is expecting her first child. The mother is very concerned that
the child could have the same condition.

From the pedigree, calculate the risk that the unborn baby, represented by the diamond shape, will also have the
same condition.

Your answer was correct

A 6.25% (1/16)

B 25% (¼)

https://mypastest.pastest.com/Secure/TestMe/Browser/436619[‫ ص‬08:06:33 11/12/1437]


MyPastest

C 50% (½)

D 100%
End Session
E It depends upon the gender of the child

47510

Previous Question Tag Question

Feedback

Difficulty: Average

Peer Responses

Session Progress

Responses Correct: 2

Responses Incorrect: 65

Responses Total: 67

Responses - % Correct: 3%

Blog
About Pastest
Contact Us
Help

© Pastest 2016

Next Question

https://mypastest.pastest.com/Secure/TestMe/Browser/436619[‫ ص‬08:06:33 11/12/1437]


MyPastest

Prefer to use the old MyPastest? Access it here »

Back to Filters

Question 64 of 65

You are seeing an 8 month old girl for a follow up appointment in view of her feeding and weight gain. She has
had problems
with nasal regurgitation of milk, poor weight gain and the blood tests you arranged had revealed
hypocalcaemia. In view of your clinical suspicion you arranged a microarray, which revealed the presence of
22q11 deletion syndrome in the infant. The parents are aware of the diagnosis and, at the last appointment, you
arranged to test the parents
as per the advice on the laboratory report (as approximately 5% of cases are known to
be inherited).

You have received the parents’ results. The mother’s test is normal, however the father’s shows that he too has
22q11 deletion syndrome. As you go to call the family in for the appointment, you notice that only the mother has
attended with the infant. You review the consent forms which confirm the consent for testing but there is no
mention of who the results can be discussed with.

What would be the most appropriate next step?

A Cancel the appointment

B Explain
that you will be able to review the infant and discuss the mother’s results but that you cannot
discuss the father’s results in his absence

C Phone
the father, using the mobile number provided on the consent form, to ask whether he gives
consent for his result to be discussed at the appointment, in his absence

D Undertake
the appointment as planned, including discussing the genetic results as
the parents were
both aware that there is a chance that they could also
have the condition

E Undertake
the review aspect of the appointment regarding the girl’s health but explain that the parental
results would need to be discussed at the next appointment

47511

Next Question

https://mypastest.pastest.com/Secure/TestMe/Browser/436619[‫ ص‬08:06:55 11/12/1437]


MyPastest

Tag Question

Feedback End Session

Difficulty: Average

Peer Responses

Previous Question

Session Progress

Responses Correct: 2

Responses Incorrect: 66

Responses Total: 68

Responses - % Correct: 3%

Blog
About Pastest
Contact Us
Help

© Pastest 2016

https://mypastest.pastest.com/Secure/TestMe/Browser/436619[‫ ص‬08:06:55 11/12/1437]


MyPastest

Prefer to use the old MyPastest? Access it here »

Back to Filters

Question 65 of 65

You are seeing a 5 year old boy in the metabolic clinic. His condition puts him at risk of decompensating with a
build-up of ammonia in his blood, leading to altered conscious levels or seizures. He requires a controlled diet to
manage this. Other members of his family have been affected as shown in the pedigree. The mother’s uncle passed
away in the neonatal period and was presumed to have the condition.

The boy’s mother is extremely concerned for her


sibling’s unborn baby (represented by the diamond shape) and
asks
for your opinion of the chance they will have the condition.

Which
is the most appropriate advice as to the level of risk for the unborn baby, represented by the diamond shape
in the pedigree above?

A 0%

B 25%

https://mypastest.pastest.com/Secure/TestMe/Browser/436619[‫ ص‬08:07:17 11/12/1437]


MyPastest

C 50%

D 100%

E Virtually 0%

47512

End Session

Previous Question Tag Question

Feedback

Difficulty: Average

Peer Responses

Session Progress

Responses Correct: 2

Responses Incorrect: 67

Responses Total: 69

Responses - % Correct: 3%

Blog
About Pastest
Contact Us
Help

© Pastest 2016

https://mypastest.pastest.com/Secure/TestMe/Browser/436619[‫ ص‬08:07:17 11/12/1437]


MyPastest

Prefer to use the old MyPastest? Access it here »

Back to Filters

Question 1 of 17

Theme: Growth

A 12 months
B 15 months
C 18 months
D 2 years
E 2½ years
F 3 years
G 4 years
H 5 years
I 6 years

For each of the following cases, select the most likely developmental age from the list above.
Each option may be used once, more than once or not at all.

Scenario 1

Hops on one foot, throws ball over arm, goes to toilet alone, draws square from copy.

Your answer was incorrect

Select one...

G - 4 years

Other milestones of a 4-year-old include: standing on one foot well, building steps with cubes, drawing a
man with
two to four parts, counting to ten, able to undress, sharing toys and beginning of social interaction.

A useful mnemonic for remembering the order of copying shapes is CCST: Circle (3 years), Cross
(3½ years),
Square (4 years) and Triangle (5 years).

Scenario 2

Rides tricycle, washes hands, knows age and sex, imitates bridge of three cubes.

Your answer was incorrect

https://mypastest.pastest.com/Secure/TestMe/Browser/436619[‫ ص‬10:02:27 10/12/1437]


MyPastest

Select one...

F - 3 years

Other milestones of a 3-year-old include: briefly balancing on one foot, walking on tiptoes, building tower of nine
cubes, copying circle, knows name, speaking three to four word sentences, helping with dressing, eating with
knife and fork.

It is easy to remember some of the three-year-old milestones; such as riding a tricycle and imitating a bridge with
3 cubes.

Scenario 3

Walks up and down stairs one step at a time, jumps with both feet, scribbles spontaneously, can build tower of
more than 4 cubes.

Your answer was incorrect

Select one...

D - 2 years

Other milestones of a 2-year-old include: running well, kicking ball, climbing on furniture, building tower of six
cubes, copying vertical line, using plurals, two to three word sentences, feeding with fork and spoon. Children
initially learn to
climb stairs one step at a time around 2–2½ years. By around 3 they can walk upstairs one foot
per step, but come down with two. Between 4–5 they learn to descend with one foot per step.

It
is important to remember that whilst the age at which milestones are attained differs between children, the order
of skill development does not. Children with severe developmental delay acquire milestones in exactly the same
order as everybody else, albeit at a different rate and
with a different end-point.
It is vital that you know your developmental milestones. They are important not only as knowledge for your
exams but also in your everyday clinical practice.
10404

Tag Question
Next Question
Feedback
Previous Question
Difficulty: Easy

End Session
Session Progress

Responses Correct: 0

https://mypastest.pastest.com/Secure/TestMe/Browser/436619[‫ ص‬10:02:27 10/12/1437]


MyPastest

Responses Incorrect: 3

Responses Total: 3

Responses - % Correct: 0%

Blog
About Pastest
Contact Us
Help

© Pastest 2016

https://mypastest.pastest.com/Secure/TestMe/Browser/436619[‫ ص‬10:02:27 10/12/1437]


MyPastest

Prefer to use the old MyPastest? Access it here »

Back to Filters

Question 2 of 17

A 3-day-old baby was noticed by the mother to develop rhythmic, focal, myoclonic jerks at a rate of 1-3 per
second, followed by generalised fits.

The diagnosis is:

A Febrile convulsion

B Infantile spasms

C Late neonatal hypocalcaemia

D Hypoglycaemia

E Early neonatal hypocalcaemia

Explanation
Newborn infants with tetany develop rhythmic, focal, myoclonic jerks at a rate of 1-3/sec, sometimes followed by
generalised fits.Early neonatal hypocalcaemia appears in the first 72 hours after birth. It occurs primarily in infants
of who have experienced complicated pregnancies, or deliveries, or both.

It occurs more commonly in infants of diabetic mothers or mothers with pre-eclamptic toxaemia.

Late neonatal hypocalcaemia has peak occurrence around 6th


day of life, and historically has been primarily
associated with high phosphate concentration in cow’s milk-based infant feeds. This is now rare with modern
formulae and is more often associated with hypoparathryoidism or maternal disorders of calcium homeostasis.

Treatment
is with intravenous infusion of 10% calcium gluconate giving at 0.2ml/kg slowly under ECG
monitoring with 2-4ml/kg/24hrs given as a maintenance dose. With persistent convulsions, hypomagnesaemia
should be
sought and if confirmed, treated with MgSO4 , given as 0.2ml/kg of a 50% solution given every 4-8hrs.

Febrile
convulsions typically occur between the ages of 6 months - 5 yrs with seizures provoked by fever. In the
scenario, the infant is not in this age range and there is no mention of fever.

https://mypastest.pastest.com/Secure/TestMe/Browser/436619[‫ ص‬10:02:56 10/12/1437]


MyPastest

Infantile spasms occur


between 3 months - 1yr. Usually the infant has a flexion-extension spasm. M > F. The EEG
is chaotic and typically shows hypsarrhythmia.
11714

Tag Question

Feedback

Difficulty: Difficult

Peer Responses

Session Progress

Responses Correct: 0

Responses Incorrect: 4

Responses Total: 4

Responses - % Correct: 0%

Blog
About Pastest
Contact Us
Help

© Pastest 2016 Next Question


Previous Question

End Session

https://mypastest.pastest.com/Secure/TestMe/Browser/436619[‫ ص‬10:02:56 10/12/1437]


MyPastest

Prefer to use the old MyPastest? Access it here »

Back to Filters

Question 3 of 17

Growth hormone therapy:

A Useful in the treatment of idiopathic short stature

B Increase in the final height of 3-4cm compared to untreated children

C Increase in final height of 1-2 mm

D The therapy has no adverse effects, hence needs no monitoring

E Final height after treatment will be 7mm greater than without the treatment

Explanation
Growth hormone therapy is given to children with growth hormone deficiency, Turner syndrome, chronic renal
insufficiency or Prader-Willi syndrome. There are specific NICE guidelines for its use.

Controversies still surround it use in short, normal children


and children categorised as having idiopathic short
stature, growth hormone is not licensed for these cases. Studies suggest a modest increase in final height to the
extent of 3-4cm.

Potential adverse effects during therapy need monitoring.


11715
Next Question
Previous Question

Tag Question
End Session
Feedback

https://mypastest.pastest.com/Secure/TestMe/Browser/436619[‫ ص‬10:03:16 10/12/1437]


MyPastest

Difficulty: Average

Peer Responses

Session Progress

Responses Correct: 0

Responses Incorrect: 5

Responses Total: 5

Responses - % Correct: 0%

Blog
About Pastest
Contact Us
Help

© Pastest 2016

https://mypastest.pastest.com/Secure/TestMe/Browser/436619[‫ ص‬10:03:16 10/12/1437]


MyPastest

Prefer to use the old MyPastest? Access it here »

Back to Filters

Question 4 of 17

Puberty in girls:

A Menstruation can occur anytime from the ages of 7-14 yrs

B Most girls have their growth spurt in the year preceding the onset of the menstruation

C Scoliosis is sideways curvature of the spine and always occurs after the peak height velocity

D Most girls stop having bone growth within 4-5 years after the onset of menses

E Once menstruation occurs girls will have a predictable menstrual cycle in the first 1-2 years

Explanation
The onset of the menstruation cycle (Menarche) begins about 2 to 2.5 years after breast development. The
menstrual cycle can occur from ages 9 to 15 years; most girls do not have a regular, predictable menstrual cycle
for another year or two. Most girls
have their growth spurt in the year preceding the onset of the menstrual cycle
and will have reached close to their final adult height,
unlike the boys who have their growth spurt later in puberty.

Most
girls stop having bone growth within 18-24 month after the onset of menses. Scoliosis is more common in
children between the ages of 9 to 14
during the growth spurt of puberty.
11717

Next Question
Previous Question

Tag Question
End Session
Feedback

https://mypastest.pastest.com/Secure/TestMe/Browser/436619[‫ ص‬10:03:35 10/12/1437]


MyPastest

Difficulty: Average

Peer Responses

Session Progress

Responses Correct: 0

Responses Incorrect: 6

Responses Total: 6

Responses - % Correct: 0%

Blog
About Pastest
Contact Us
Help

© Pastest 2016

https://mypastest.pastest.com/Secure/TestMe/Browser/436619[‫ ص‬10:03:35 10/12/1437]


MyPastest

Prefer to use the old MyPastest? Access it here »

Back to Filters

Question 5 of 17

Precocious puberty:

A For
the majority of girls there is an underlying medical problem such as a brain tumour, ovarian
diseases, thyroid gland problems

B In about 5% of boys, precocious puberty is inherited from the father

C More than 2% of girls affected by precocious puberty have inherited the condition from their mother

D The onset of puberty is normally triggered by the pituitary gland

E In precocious puberty children achieved their full adult height potential

Explanation
Precocious puberty is the onset of signs of puberty before the ages 7 or 8 in girls and 9 in boys. This can be
physically and emotionally difficult for the children.

Signs are:

Girls:
Breast development, pubic or underarm hair development, rapid height growth, onset of
menstruation, acne, and mature body odour.
Boys:
Enlargement of testicles or penis, pubic or underarm or facial hair development, rapid height
growth, acne, voice deepening, and mature body
odour.

Those children who show some but not all of the early signs of puberty have what’s known as partial precocious
puberty and require evaluation to rule out true precocious puberty.

The
onset of puberty is normally triggered by the hypothalamus. It signals the pituitary gland to release hormones
that stimulate the ovaries or testicles to make sex hormones. In the majority of girls no underlying medical

https://mypastest.pastest.com/Secure/TestMe/Browser/436619[‫ ص‬10:03:54 10/12/1437]


MyPastest

problem is found. In boys the condition is less common, and more


likely to be associated with an underlying
medical problem.

In 5%
of boys, precocious puberty is inherited from their father, less than 1% of girls affected by precocious
puberty have inherited the condition from their mother.
11718

Tag Question

Feedback

Difficulty: Difficult

Peer Responses

Session Progress

Responses Correct: 0

Responses Incorrect: 7

Responses Total: 7

Responses - % Correct: 0%

Blog
About Pastest
Contact Us
Help Next Question
© Pastest 2016
Previous Question

End Session

https://mypastest.pastest.com/Secure/TestMe/Browser/436619[‫ ص‬10:03:54 10/12/1437]


MyPastest

Prefer to use the old MyPastest? Access it here »

Back to Filters

Question 6 of 17

Given a suspected diagnosis of precocious puberty, which one of the following is true?

A Blood and urine should be obtained to detect elevated levels of sex hormones

B X-rays of the wrist and hand are needed to show fracture

C An MRI scan is the initial diagnostic investigation to confirm precocious puberty

D CT brain scans are carried out to rule out pituitary tumour

E A CT abdominal scan for an intra abdominal malignancy mass like neuroblastoma is essential

Explanation
The physical changes boys and girls go through during puberty are usually evident to a doctor during an
examination. To
confirm a diagnosis of precocious puberty, order blood and urine tests to detect elevated levels of
sex hormones. X-rays of a child’s left wrist and hand can show whether the bones are maturing too rapidly.

Imaging
and scanning tests such as CT scan, MRI and ultrasound studies can help
rule out specific causes of
precocious puberty, such as a tumour in the
brain, ovary or testicle.
11719

Next Question
Previous Question
Tag Question

Feedback End Session

Difficulty: Average

https://mypastest.pastest.com/Secure/TestMe/Browser/436619[‫ ص‬10:04:15 10/12/1437]


MyPastest

Peer Responses

Session Progress

Responses Correct: 0

Responses Incorrect: 8

Responses Total: 8

Responses - % Correct: 0%

Blog
About Pastest
Contact Us
Help

© Pastest 2016

https://mypastest.pastest.com/Secure/TestMe/Browser/436619[‫ ص‬10:04:15 10/12/1437]


MyPastest

Prefer to use the old MyPastest? Access it here »

Back to Filters

Question 7 of 17

Treatment of precocious puberty:

A Lowering the high levels of sex hormones with medication will not stop sexual development from
progressing

B LHRH analogs’ treatment results are usually seen within 2 months of starting the treatment

C The
goal of treatment is to halt or even reverse sexual development and stop the rapid growth and bone
maturation that can eventually result in adult short stature

D Treatment
of precocious puberty with LHRH analogs result in a lot of side effects, hence blood tests
need to be monitored to detect early changes of abnormality

E LHRH analogs' mechanism of action is to stimulate the production of sex hormones that are causing
the early puberty

Explanation
The goal of treating precocious puberty is to halt or even reverse sexual development and stop the rapid growth
and bone maturation that can result in adult short stature.

Depending on the cause there are two possible approaches to treatment:

1. Treat the underlying cause or disease such as tumour


2. Lowering the high levels of sex hormones with medication to stop sexual development from progressing.

The
current approved hormone treatment is LHRH analogs. These are synthetic
hormones that block the body’s
production of sex hormones that are causing the early puberty. Dramatic results are usually seen within a
year of
starting treatment with an LHRH analog, which is generally safe
and usually causes no side effects in children. In
girls breast size may decrease or there will be no further development. In boys, penis and
testicles may shrink back
to the size expected for their age. Growth in
height will also slow down to a rate expected for the children before
puberty.

https://mypastest.pastest.com/Secure/TestMe/Browser/436619[‫ ص‬10:04:35 10/12/1437]


MyPastest

11720

Tag Question

Feedback

Difficulty: Average

Peer Responses

Session Progress

Responses Correct: 0

Responses Incorrect: 9

Responses Total: 9

Responses - % Correct: 0%

Blog
About Pastest
Contact Us
Help

© Pastest 2016

Next Question
Previous Question

End Session

https://mypastest.pastest.com/Secure/TestMe/Browser/436619[‫ ص‬10:04:35 10/12/1437]


MyPastest

Prefer to use the old MyPastest? Access it here »

Back to Filters

Question 8 of 17

Delayed puberty:

A Is more common in girls and usually idiopathic

B Is rare in boys and usually pathological

C Is more common in boys and usually pathological

D Is more common in boys and is usually idiopathic

E Means no signs of puberty at age 13 years in the boys

Explanation
Delayed puberty means the appearance of the following after 13 years of age in girls:

Breast development, pubic or underarm hair, onset of menstruation, acne, mature body odour and growth
spurt.

And the following after 14 years of age in boys:

Next Question
Enlargement
of the testicles or penis, pubic, underarm or facial hair, growth spurt, voice deepening, acne
and mature body odour.

Delayed puberty is common in boys and usually idiopathic. In girls it is rare and usually pathological.
11721

https://mypastest.pastest.com/Secure/TestMe/Browser/436619[‫ ص‬10:05:20 10/12/1437]


MyPastest

Tag Question

Feedback

Difficulty: Average

Peer Responses

Session Progress

Responses Correct: 0

Responses Incorrect: 10

Responses Total: 10

Responses - % Correct: 0%

Blog
About Pastest
Contact Us
Help

© Pastest 2016

Previous Question

End Session

https://mypastest.pastest.com/Secure/TestMe/Browser/436619[‫ ص‬10:05:20 10/12/1437]


MyPastest

Prefer to use the old MyPastest? Access it here »

Back to Filters

Question 9 of 17

Theme: Diagnosis

A Turner syndrome
B Prader-willi syndrome
C Noonan syndrome
D Achondroplasia
E Down syndrome
F Patau syndrome
G William syndrome
H Small for gestational age
I Psychosocial deprivation
J Constitutional delay
K Chronic childhood illness
L Edward syndrome

Match
the most appropriate diagnosis to each clinical scenario below. Each option may be used once, more than
once, or not at all.

Scenario 1

A 6-year-old boy was brought in by his mother to the clinic because she was worried about his height. On
thorough examination he was found to be short for his age. There were no signs of
other organic disease, no
features to suggest hypopituitarism, hypogonadism or space occupying lesion.

Your answer was incorrect

Select one...

J - Constitutional delay

Constitutional delay in growth and puberty occurs more often in boys. They have slow growth in the preceding 2-
3 years before puberty, no other organic disease, and there will be no features to suggest hypopitutarism,
hypogonadism and space occupying lesion. There will be delayed bone age and short stature.

Scenario 2

https://mypastest.pastest.com/Secure/TestMe/Browser/436619[‫ ص‬10:05:44 10/12/1437]


MyPastest

A baby boy born without any complication, was noticed by the mother to have feeding problems, he gets tired
easily
during feeding. On examination by a paediatrician, he was found to have
ejection systolic murmur at the left
3rd and 4th intercostal space. The eyes are rounded with downwards slope to the eye socket on the outer edge.

The gap between the eyes are wider and flattened, the neck short with extra folds of skin.

Your answer was incorrect

Select one...

C - Noonan syndrome

Noonan syndrome patients present with ejection on left sternum at the 3rd or 4th intercostals space radiating to the
back.

Scenario 3

A 5-year-old girl with short stature, wide spaced


nipples, dysplastic nails and normal intelligence. An ejection
systolic
murmur is heard at the right sternum in the 2nd intercostal space.

Your answer was incorrect

Select one...

A - Turner syndrome

Turner syndrome is karyotype 45,XO. Patients will have coarctation of the aorta, ejection systolic murmur on the
right sternum at the 2nd intercostals space. Other features are lymph oedema at birth, dysplastic nails, short stature,
ovarian dysgenesis, normal intelligence, renal anomalies with premature osteoporosis.

Scenario 4

You see a baby with severe hypotonia, small hands


and feet. He had previous feeding difficulties but developed
overeating
in second year with obesity.

Your answer was incorrect

Select one...

B - Prader-willi syndrome

Prader willi syndrome patients have severe hypotonia at birth, with feeding difficulty and failure to thrive.
Overeating begins in the second year, with subsequent obesity. Others signs are hypogonadism and small hands
and feet. Mild to moderate retardation. 55-70% have a small paternally derived deletion of the long
arm of
chromosome 15.

Scenario 5

You see a baby whose weight at birth was 2.4kg at


term. He is feeding well, with his weight and height on the

https://mypastest.pastest.com/Secure/TestMe/Browser/436619[‫ ص‬10:05:44 10/12/1437]


MyPastest

growth chart continuing on his birth centile.

Your answer was incorrect

Select one...

H - Small for gestational age

Small for dates (gestational age) means that the height of the fetus is less than the tenth centile for its gestation.
However, most foetuses are simply constitutionally small and the catch up in weight and height at age 4-5 years.
11722

Tag Question

Feedback

Difficulty: Average

Session Progress

Responses Correct: 0

Responses Incorrect: 15

Responses Total: 15

Responses - % Correct: 0%

Blog
About Pastest
Contact Us
Help

© Pastest 2016

Next Question
Previous Question

End Session

https://mypastest.pastest.com/Secure/TestMe/Browser/436619[‫ ص‬10:05:44 10/12/1437]


MyPastest

Prefer to use the old MyPastest? Access it here »

Back to Filters

Question 10 of 17

Theme: Diagnosis

A Psychosocial deprivation and neglect


B Anorexia nervosa
C Acquired hypothyroidism
D Growth hormone deficiency
E Chronic asthma
F Panhypopituitarism
G Glucocorticoid
H Achondroplasia
I Obsessive compulsive disorder
J Dyschondro-osteosis
K Down syndrome
L Prader-willi syndrome

For
each of the scenarios below select the most appropriate diagnosis. Each
option may be used once, more than
once, or not at all.

Scenario 1

A 2 ½-year-old boy is referred to the clinic with poor growth. Their birth weight was 2280g at term and he
had
progressed poorly physically and developmentally, but was said to have a good appetite.

He was withdrawn and miserable; playing with


his hand and rocking. He was undernourished with scratched dry
skin, and developmentally was markedly retarded.

Investigations revealed
absence of growth hormone response to adequate insulin hypoglycaemia. He was placed
away from his home. The growth chart shows a dramatic changed in weight, height on each occasion that he was
away from home.

Your answer was incorrect

Select one...

A - Psychosocial deprivation and neglect

https://mypastest.pastest.com/Secure/TestMe/Browser/436619[‫ ص‬10:06:12 10/12/1437]


MyPastest

Psychosocial deprivation and neglect is emotional, physical and mental illness to the child. They are
undernourished. This is known to cause reduced secretion of growth hormone and eventually retard growth.

Scenario 2

A 7 year old girl, whose birth weight was 3860g presents to your clinic. From the age of 18 months her growth
had fallen off - her height much more than weight - and she became obese. At
the age of 3 she developed asthma
and required regular inhaled steroid treatment. Her development and health otherwise were good. A bone age at
the age of 6 years corresponded radiologically to 2 ½ years.

Investigations show normal thyroid function but an almost total failure of response of growth hormone to
stimulation.

Your answer was incorrect

Select one...

D - Growth hormone deficiency

Growth hormone deficiency produces a delay in bone age, consequently the children are short. A deviation from
the normal growth rate occurs after the age of about 6 months. It also produces hypoglycaemia early in life.
Hypothyroidism also produces delay
in bone age but the level is normal in this child.

Provocation tests are required to confirm GH deficiency. Example is insulin stress test. Insulin is injected into the
child and growth hormone measured at 15, 30, 45, 60 and 120 min,with hypoglycaemic symptom and signs
occurring at 15-30 min. There should be a reduction in basal glucose value by more than 50% or < 2.2mmol/l. If
peak GH levels are less than < 7mU/l then this suggests GH deficiency and levels in excess of
15mU/l exclude the
diagnosis.

Scenario 3

A 15-year-old girl, who was growing normally, lost 14kg of weight over a year and this called for concern. She
denied any problems and maintained she was healthy and happy at school and fully active physically.

It became known that she was not eating adequately, was very dependent on her mother, and was fearful of
growing up.

Psychiatric
assessment revealed that she was a girl with a rather obsessional personality. She responded well to
psychiatric counselling.

Your answer was incorrect

Select one...

B - Anorexia nervosa

Anorexia nervosa is a disturbed sense of body image with a morbid fear of obesity and relentless pursuit of low
body weight. Physically there is a body weight more than 25% below the standard weight and there is often
amenorrhea.

It is about 10 times more common in females, with average age of onset during adolescence, male mean age 12

https://mypastest.pastest.com/Secure/TestMe/Browser/436619[‫ ص‬10:06:12 10/12/1437]


MyPastest

years, female 16 years.

Other features are constipation, postural hypotension, bradycardia, hypothermia, hypoglycaemia, alopecia, cardiac
arrhythmia, dental erosion
and knuckle calluses (Russell sign) and tooth decay (as a result of self induced
vomiting).

Hormonal changes: decrease in FSH, LSH, testosterone, oestrodiol, T3, and T4; increase in prolactin, GH and
cortisol.

Scenario 4

You see in clinic an 11-year-old boy whose


birth weight was 3500g at term. He was born to parents of above
average
stature. It was at 18 months of age that his parents noticed he was short and overweight. His limbs were
markedly short compared to his normal sized trunk and head. Otherwise his general health, development and
intellect were normal.

Your answer was incorrect

Select one...

H - Achondroplasia

Achondroplasia displays autosomal dominant inheritance. 80% as spontaneous mutation. Endochondral bone
formation is abnormal. The head is of normal size (but
can appear to be large in comparison to limbs). There are
prominent frontal, parietal, and occipital bones. The trunk is normal but the limbs abnormally short. They have
small foramen magnum: the resulting basilar compression may result in hydrocephalus.

Lumbar
lordosis, vertebral disc herniation is common. Spinal stenosis results in root compression which may
cause neurological symptoms in the lower limbs resulting in waddling gait and pelvic tilt.

Scenario 5

A 4 year old boy who attends with his mother presents with a history of cough and recurrent chest infection. He is
on treatment with digoxin and diuretics. Endocrine function throughout has been normal and bone age appropriate
for chronological age.

On examination he is of short stature, his ears are small with an over folding helix and his nasal bridge is flat. The
tongue appears large and may protrude because the mouth is relatively small. The outer canthus is higher than the
inner canthus. The hands are short and broad; the 5th finger is short and incurved.

Your answer was incorrect

Select one...

K - Down syndrome

Down syndrome (Trisomy 21). Incidence 1:800 live births, increases with maternal age.

Clinical features: short stature, intellectual (IQ < 50) and developmental delay. Brachycephaly, central hypotoniac
in 80%.

Eye: upward slanting palpebral fissures, brushfield, epicanthic folds, and cataracts.

https://mypastest.pastest.com/Secure/TestMe/Browser/436619[‫ ص‬10:06:12 10/12/1437]


MyPastest

Ear: small ear, hearing defeat, frequent middle ear infection.

Mouth:
downward turned mouth, protruding tongue. Congenital hypothyroidism occurs in 1:300 and autoimmune
type presenting later in childhood.

Cardiac: Atrioventricular septa defeat occurs in 40%.

Bowel:
trachea-oesophageal atresia, duodenal atresia (10%) double bubble x-ray
appearance. Annular pancreas
which can result in obstruction, hirschprung disease in 3%.

Hand: short and broad hands and fingers. Single palmer crease, clinodactyly.

Feet: short and broad feet and toes, increase space between the first and second toe (sandle toe).

Skin: cutis marmorata, loosely adherent skin.

There is increased risk of leukaemia and Alzheimer disease linked to chromosome 21.
11723

Tag Question

Feedback

Difficulty: Average

Session Progress

Responses Correct: 0

Responses Incorrect: 20

Responses Total: 20

Responses - % Correct: 0%

Blog
About Pastest
Contact Us
Help

© Pastest 2016 Next Question


Previous Question

End Session

https://mypastest.pastest.com/Secure/TestMe/Browser/436619[‫ ص‬10:06:12 10/12/1437]


MyPastest

Prefer to use the old MyPastest? Access it here »

Back to Filters

Question 11 of 17

Theme: Diagnosis

A Vitamin K deficiency
B Factor V deficiency
C Hypothyroidism
D Genetic short stature
E Growth failure due to inhaled steroid
F Growth hormone deficiency
G Marfan syndrome
H Di-George syndrome
I Zinc deficiency
J Psychosocial deprivation

Match each scenario to the most appropriate diagnosis. Each option may be used once, more than once, or not at
all.

Scenario 1

A 2-month-old preterm baby feeds on breast milk only. The child develops blisters at the angle of the mouth, has
discharging eyes, a bright reddish, non-scaly macular rash and patches on the face mostly around the mouth.
Serum zinc is 5 micromol/L.

Your answer was incorrect

Select one...

I - Zinc deficiency

Zinc, atomic number 30, is essential in structure and function of bio membranes. Zinc deficiency may cause
oxidation damage to membrane. Zinc absorption takes place in the small intestine. 60-80% is absorbed.
Zinc deficiency is common in premature infants, the concentration of zinc in the foetal liver from 20-40 weeks
gestation ranges from 100-300 microgram/g. Requirement for growth on average is 250 microgram/kg/day. Also
deficiency is noticed in
preterm infants breast-fed by mothers whose mammary glands have a defective ability to

https://mypastest.pastest.com/Secure/TestMe/Browser/436619[‫ ص‬10:06:36 10/12/1437]


MyPastest

secrete normal quantities of zinc into their milk.

Clinical
features: decreased growth, stomatitis, glossitis. Bright reddish, on-scaly maculaes and patches acro-
orificial distribution. (Acrodermatitis enteropathica). Exematoid, psoriasifrom, vesiculo-bullous and pustular
lesions may be present.

Scenario 2

A 3 day old baby is born at 30 weeks of gestational age. The mother suffers from controlled epilepsy and is on
phenytoin. The baby develops nasal bleeding. The PTT and PT are prolonged.

Your answer was incorrect

Select one...

A - Vitamin K deficiency

Haemorrhagic disease of the new born is vitamin K dependent. Commoner in preterm babies who have decrease in
factors ii, vii, ix and x. These are manufactured from the liver and are
vitamin K dependent.
A normal decrease of the factors in all newborn babies is seen from 48-72 hours and a return to birth levels occurs
by 7-10 days.

Breast milk is a poor source of vitamin K and


haemorrhagic complications have appeared more in breast-fed than
formula-fed infants. Presentations are GI bleeding, epistaxsis and intracranial bleed. Administration of vitamin K
1mg i.m at birth prevents bleeding.

Scenario 3

An 8 year old boy’s weight is above the 99th centile and his height is below the 3rd centile for his age. He has
no
history of chronic illness and asthma. There is growth failure despite weight gain and obesity. T3 and T4 are low.

Your answer was incorrect

Select one...

C - Hypothyroidism

Juvenile hypothyroidism. The most common cause of acquired hypothyroidism is lymphocyte thyroiditis. Some
patients with congenital thyroid dysgenesis, or with incomplete genetic defects in thyroid hormone synthesis, may
not develop clinical manifestation until childhood and therefore appear to have acquired hypothyroidism. Most
patients with these conditions are now detected in the newborn screening program.

Other causes: Subtotal thyroidectomy for thyrotoxicosis or cancer may result in hypothyroidism.

Irradiation to the area of the thyroid occurs incidentally during the treatment of Hodgkin disease or malignancies.

Clinical
manifestations: Retarded growth, myxedematous changes of the skin, constipation, cold intolerance,
decrease energy, weight gain and delayed
bone age.

Some children present with headache, visual problems, precocious puberty or galactorrhoea.

All
these changes return to normal with adequate treatment with thyroxine but in children with longstanding

https://mypastest.pastest.com/Secure/TestMe/Browser/436619[‫ ص‬10:06:36 10/12/1437]


MyPastest

hypothyroidism, catch-up growth may be


incomplete.

Scenario 4

A 9 year old boy eating well and is healthy but his height is below the target range for his age. Both his parents are
short, and family history shows that this is normal in their family.

Your answer was incorrect

Select one...

D - Genetic short stature

Scenario 5

A 7 year old known asthmatic with frequent attacks is on regular inhaled beclomethasone and salbutamol. His
current
growth is on the chart at below 3rd centile. No other chronic illness is noted.

Your answer was incorrect

Select one...

E - Growth failure due to inhaled steroid

Steroid treatment in children can arrest growth and is seen in cases of asthmatic patients who are on high doses of
inhaled steroid.
11724

Tag Question

Feedback

Difficulty: Average

Session Progress

Responses Correct:
Next Question 0

Previous Question
Responses Incorrect: 25

Responses Total: 25

Responses - % Correct: 0%
End Session

https://mypastest.pastest.com/Secure/TestMe/Browser/436619[‫ ص‬10:06:36 10/12/1437]


MyPastest

Blog
About Pastest
Contact Us
Help

© Pastest 2016

https://mypastest.pastest.com/Secure/TestMe/Browser/436619[‫ ص‬10:06:36 10/12/1437]


MyPastest

Prefer to use the old MyPastest? Access it here »

Back to Filters

Question 12 of 17

Theme: Diagnosis

A Congenital heart disease


B Intrauterine growth retardation
C Small for gestational age
D Cystic fibrosis
E Gilbert syndrome
F Insulin dependent DM
G Marfan syndrome
H Pneumonia
I Klinefelter syndrome
J Excessive growth hormone secretion
K Global developmental delay

For each scenario select the most likely option. Each option may be used once, more than once, or not at all.

Scenario 1

An 8-month-old baby girl has feeding problems. She cannot breastfeed for long, and her mother reports that the
baby is often breathless and seems to tire easily. She gets exhausted before she
achieves her full volume of
required milk. Her weight is 6kg.

Your answer was incorrect

Select one...

A - Congenital heart disease

Children with severe uncorrected congenital heart disease are frequently small. Usually their poor growth
and
thinness is apparent from the first year of life and may continue throughout childhood.
Poor growth was particularly associated with
persistent congestive heart failure and pulmonary hypertension,
double outlet right ventricle, tetralogy of fallot and pulmonary atresia.

Patent
ductus arteriosis used to be associated with severe failure to thrive in the first year of life but now corrective
surgery is safe even in low
birth weight infants. Children with persistent left ventricular failure
and those with

https://mypastest.pastest.com/Secure/TestMe/Browser/436619[‫ ص‬10:07:00 10/12/1437]


MyPastest

cyanosis are breathless and tired easily, becoming exhausted before they achieve adequate milk requirement.

Scenario 2

A 2 ½-year-old girl is brought to the clinic by her mum. She is unable to walk, cannot speak, her cry is low
pitched and she is still bottle fed by mum. Her hearing test is abnormal. Her height is 70 cm and her weight is 8
kg. On examination she
has microcephaly but no other signs. Other siblings are normal.

Your answer was incorrect

Select one...

K - Global developmental delay

A child with global developmental delay will have delays in all area of development. This will affect speech and
language, fine and gross motor skills, social skills and weight and height.
Causes include: Prematurity, cerebral malformation, chromosome disorders, infection, progressive encephalopathy
(Metabolic disease, hypothyroidism, neurocutaneous syndromes like neurofibromatosis, tuberous sclerosis).

Signs of motor delay are a child not able to bring his hands together by 4 months, not rolling over by 6 months,
not crawling by 12 months and not walking by 15 months.

Speech delay: 5-6 months most infants can say mama, dada. 10-20 months the child should be able to say 4-6
words and have a 50 word vocabulary at 16-24 months.

Scenario 3

A 6-month-old baby boy, delivered at 35 weeks of pregnancy. He was born by a normal delivery and has been
feeding well. His weight is 6kg, and his height is below 10th centile.

Your answer was incorrect

Select one...

C - Small for gestational age

Small for gestational age are children


in whom no pathological environment or genetic conditions restricting foetal
growth can be determined. Postnatal catch up growth is frequently
observed.
Intrauterine growth retardation is observed in mother
with hypertension in pregnancy or malnutrition, or with
decrease uteroplacental blood flow. The growth is asymmetrical; the head is not growing in proportion to the
body.

Scenario 4

A 10-year-old boy, newly diagnosed with diabetes,


has a past history of frequent admissions with respiratory tract
infections. Since birth, his weight and height have been between the 3rd
and 10th centiles.

Your answer was incorrect

https://mypastest.pastest.com/Secure/TestMe/Browser/436619[‫ ص‬10:07:00 10/12/1437]


MyPastest

Select one...

D - Cystic fibrosis

Cystic fibrosis defect is on long arm of chromosome 7, with most likely manifestation the f508 mutation. Patients
mostly have severe lung infection, malabsorption caused by pancreatic enzyme deficiency and failure to thrive in
early childhood due to very mild pulmonary involvement.

The growth, puberty development and final height of individual with cystic fibrosis have assumed greater
importance. Failure to maintain body weight is a predictor of mortality risk for cystic fibrosis.

Growth delay was associated with increasingly severe respiratory disease, weight more affected than height.

Scenario 5

A 15-year-old has had long legs since childhood. Thorough examination shows gynaecomastia, small testes,
azospermia and karyotype 47XXY.

Your answer was incorrect

Select one...

I - Klinefelter syndrome

The incidence of Klinefelter syndrome (47XXY) is 1:500 to 1000 live births. There is tall stature for the family
with disproportionately long legs from childhood, small testes, and azospermia.
There is marked gynaecomastia, hypogonadism or infertility. The long legs relate to testosterone secretion being
inadequate for epiphysial closure at appropriate time. Pubertal onset is
generally not delayed. Mean IQ is lower
than normal.
11725

Tag Question

Feedback

Next Question
Difficulty: Average

Previous Question
Session Progress

Responses Correct: 0

Responses Incorrect:
End Session 30

Responses Total: 30

https://mypastest.pastest.com/Secure/TestMe/Browser/436619[‫ ص‬10:07:00 10/12/1437]


MyPastest

Responses - % Correct: 0%

Blog
About Pastest
Contact Us
Help

© Pastest 2016

https://mypastest.pastest.com/Secure/TestMe/Browser/436619[‫ ص‬10:07:00 10/12/1437]


MyPastest

Prefer to use the old MyPastest? Access it here »

Back to Filters

Question 13 of 17

Theme: Infant milk formulae

A Breast milk
B Elemental formula
C ‘Follow on’ milk
D Goats’ milk
E High-energy formula
F Hydrolysed protein formula
G Pre-term infant formula
H Term infant formula
I Soya infant formula
J Soya milk

For
each of the following cases, choose the most appropriate milk from the above list. Each item may be used
once, more than once or not at all.

Scenario 1

A 7-month-old infant who has chronic lung disease of prematurity and is on home O2 and is failing to thrive on
term infant formula.

Your answer was incorrect

Select one...

E - High-energy formula

The chronic lung disease means that the infant uses more calories to breathe and therefore has a higher than
normal daily calorie requirement. A higher calorie formula is needed and
a high-energy formula would be most
appropriate. Pre-term formulae are more calorific but unsuitable for infants of this age.

Scenario 2

A newborn baby whose mother has hepatitis C.

Your answer was incorrect

https://mypastest.pastest.com/Secure/TestMe/Browser/436619[‫ ص‬10:07:22 10/12/1437]


MyPastest

Select one...

A - Breast milk

In the UK, maternal HIV, current cytotoxic therapy or galactosaemia in the infant are the only medical reasons not
to breast-feed.

Scenario 3

A 2-month-old baby who suffers from gastro-oesophageal reflux, is failing to thrive and has a sibling who ‘can’t
have dairy’.

Your answer was incorrect

Select one...

F - Hydrolysed protein formula

The clinical scenario fits with a cow’s


milk protein intolerance. Gastro-oesophageal reflux is associated with
cow’s milk protein intolerance as a result of eosinophil infiltration in the oesophagus. Soya formulae are not
recommended for infants under 6 months as they contain phyto-oestrogens.
22018

Tag Question

Feedback

Difficulty: Average

Session Progress

Responses Correct: 0

Responses Incorrect: 33

Responses Total: Next Question 33

Responses - % Correct: 0%
Previous Question

End Session
Blog
About Pastest
Contact Us
Help

© Pastest 2016

https://mypastest.pastest.com/Secure/TestMe/Browser/436619[‫ ص‬10:07:22 10/12/1437]


MyPastest

https://mypastest.pastest.com/Secure/TestMe/Browser/436619[‫ ص‬10:07:22 10/12/1437]


MyPastest

Prefer to use the old MyPastest? Access it here »

Back to Filters

Question 14 of 17

Theme: Essential nutrients

A Calcium

B Folic acid

C Iron

D Vitamin A

E Vitamin B6

F Vitamin B12

G Vitamin C

H Vitamin D

I Vitamin E

J Zinc

For
each of the cases below please choose the nutritional element most likely to be deficient from the above list.
Each item may be used once, more than once or not at all.

Scenario 1

A 6-year-old boy with cystic fibrosis is thought to have poor compliance with his medicines. He is failing to thrive
and has mild ataxia and weakness.

Your answer was incorrect

Select one...

I - Vitamin E

Ataxia and weakness are signs of vitamin E deficiency. Vitamin E is routinely supplemented in children with
cystic fibrosis. This is because vitamin E is fat soluble (along with A, D and K) and may be malabsorbed due to
exocrine pancreatic deficiency.

https://mypastest.pastest.com/Secure/TestMe/Browser/436619[‫ ص‬10:07:42 10/12/1437]


MyPastest

Scenario 2

A 4-year-old’s parents are strict vegans. He is given iron supplements but is noted to be pale. His Hb is 9.2.

Your answer was incorrect

Select one...

F - Vitamin B12

Folic deficiency is unlikely in a vegan diet as green vegetables and legumes are good sources. Vitamin B12,
however, is mainly obtained from dairy products and meat products and deficiency results in megaloblastic
anaemia. Treatment with B12 supplements stimulates a good bone marrow response.

Scenario 3

A 2-year-old child is still exclusively breast fed and has not yet started walking. He has prominent wrists and
lumps round his sternum.

Your answer was incorrect

Select one...

H - Vitamin D

Vitamin D deficiency leads to reduced Ca2+ uptake from the gut. Ca2+ is resorbed (stimulated by PTH) from the
bone to maintain serum levels. Alkaline phosphatase levels may be increased reflecting osteoblastic activity.
Clinically this leads to rickets. There may be prominent wrists (due to the ‘cupped, splayed, frayed’ appearance of
the radial and ulna metaphyses (and
those of the other long bones)). A ‘rachitic rosary’ may be
present due to
swelling of the costochondral junctions. Treatment is with vitamin D supplementation.
22027

Tag Question

Feedback

Next Question
Difficulty: Average
Previous Question
Session Progress

Responses Correct: 0
End Session
Responses Incorrect: 36

Responses Total: 36

https://mypastest.pastest.com/Secure/TestMe/Browser/436619[‫ ص‬10:07:42 10/12/1437]


MyPastest

Responses - % Correct: 0%

Blog
About Pastest
Contact Us
Help

© Pastest 2016

https://mypastest.pastest.com/Secure/TestMe/Browser/436619[‫ ص‬10:07:42 10/12/1437]


MyPastest

Prefer to use the old MyPastest? Access it here »

Back to Filters

Question 15 of 17

Which of the following is the first sign of puberty in girls?

A Menarche

B Breast development

C Growth spurt

D Axillary hair development

E Pubic hair development

Explanation
Breast development is the first sign of puberty in girls (from 8 years). Menarche occurs approximately 2 years
later (at
Tanner stage 4). Peak height velocity will usually coincide with menarche. There is a wide range in the
times of onset of the stages.

(A
good reference for the sequence of development is: Marshall, W.A., and Tanner, J.M. 1969. Variations in
pattern of pubertal changes in girls. Archives of Diseases in Childhood 44:291–303)
22215

Next Question
Previous Question
Tag Question

Feedback End Session

Difficulty: Easy

https://mypastest.pastest.com/Secure/TestMe/Browser/436619[‫ ص‬10:08:02 10/12/1437]


MyPastest

Peer Responses

Session Progress

Responses Correct: 0

Responses Incorrect: 37

Responses Total: 37

Responses - % Correct: 0%

Blog
About Pastest
Contact Us
Help

© Pastest 2016

https://mypastest.pastest.com/Secure/TestMe/Browser/436619[‫ ص‬10:08:02 10/12/1437]


MyPastest

Prefer to use the old MyPastest? Access it here »

Back to Filters

Question 16 of 17

Which of the following is the first sign of puberty in boys?

A Growth spurt

B Pubic hair development

C Deepening of the voice

D Increase in testicular volume

E Axillary hair development

Explanation
Increase in testicular volume is the first sign of puberty in boys (from 10 years). Voice change occurs at around 14
years. Peak height velocity is when testicular volume reaches 10 mL (pubic hair stage 4). There is a wide range in
times of onset of the stages in individuals.

A good reference for sequence of development is Marshall, WA and Tanner, JM (1970) Variations in the pattern
of pubertal changes in boys. Archives of Diseases in Childhood, 45: 13–23.
22345

Next Question
Previous Question
Tag Question

Feedback End Session

Difficulty: Easy

https://mypastest.pastest.com/Secure/TestMe/Browser/436619[‫ ص‬10:08:22 10/12/1437]


MyPastest

Peer Responses

Session Progress

Responses Correct: 0

Responses Incorrect: 38

Responses Total: 38

Responses - % Correct: 0%

Blog
About Pastest
Contact Us
Help

© Pastest 2016

https://mypastest.pastest.com/Secure/TestMe/Browser/436619[‫ ص‬10:08:22 10/12/1437]


MyPastest

Prefer to use the old MyPastest? Access it here »

Back to Filters

Question 17 of 17

A 13-year-old boy develops gynaecomastia and comes to


you because he is concerned. He reveals that he is being
bullied at school as a result. What is the most appropriate course of action?

A Referral to a paediatric surgeon for consideration of surgical reduction of the breast tissue

B Arrange for assessment of the hypothalamic–pituitary axis

C Reassure him that this is physiological and a normal part of puberty

D Reassure him and arrange for clinical psychology support

E Reassure him and contact the school (with his consent) about the bullying

Explanation
Gynaecomastia is not uncommon in puberty in boys and resolves spontaneously. Bullying is not acceptable and
schools have very good anti-bullying policies. Surgical reduction is a last resort in
gynaecomastia that doesn’t
resolve. Psychological support may be useful but will not stop the bullying itself.
22353

End Session

Previous Question Tag Question

Feedback

Difficulty: Average

https://mypastest.pastest.com/Secure/TestMe/Browser/436619[‫ ص‬10:08:41 10/12/1437]


MyPastest

Peer Responses

Session Progress

Responses Correct: 0

Responses Incorrect: 39

Responses Total: 39

Responses - % Correct: 0%

Blog
About Pastest
Contact Us
Help

© Pastest 2016

https://mypastest.pastest.com/Secure/TestMe/Browser/436619[‫ ص‬10:08:41 10/12/1437]


MyPastest

Prefer to use the old MyPastest? Access it here »

Back to Filters

Question 1 of 63

A 3-year-old boy fell while playing in the garden. He developed a very painful swelling of his right knee,
aspiration of which
revealed the presence of blood. The following laboratory parameters have been obtained: INR
normal, APTT increased, fibrinogen normal, antithrombin III level normal, bleeding time 3.3 min.

What is the most likely cause?

A Fibrinogen deficiency

B Glucose-6-phosphate dehydrogenase deficiency

C Prothrombin deficiency

D Haemophilia

E Factor VII deficiency

Explanation
Deficiency of either factor VIII (haemophilia A) or factor IX (haemophilia B), which together make up the factor
VIIIa/factor IXa intrinsic tenase enzymatic complex, results in the clinical phenotype commonly known as
‘haemophilia’.

Haemophilia
principally presents with haematoma formation, easy bruising and bleeding at the site of
venepuncture during the toddler period. In the untreated patient with severe disease, haemophilic arthropathy and
joint
deformity are inevitable complications. In decreasing order of involvement, the most commonly affected
joints include the knee, elbow, ankle, shoulder, wrist and hip. Recurrent bleeding episodes create a hypertrophic
synovial lining with chronic inflammation; however, the pathophysiology responsible for recurrent joint bleeding
remains unknown. Arthropathies commonly necessitate replacement of affected joints for pain control and

https://mypastest.pastest.com/Secure/TestMe/Browser/436619[‫ م‬12:19:19 10/12/1437]


MyPastest

improvement of mobility. Soft tissue haemorrhages frequently complicate haemophilia; further complications due
End Session
to these haemorrhages include compartment syndrome, neurological damage and extensive blood loss from
retroperitoneal bleeds. Haematoma formation, a frequent complication of haemophilia, may arise spontaneously or
with trauma and require extensive factor replacement and fasciotomy, the necessity for which can be assessed by
measuring the
mean arterial pressure in a compartment.

The management of haemophilia predominantly involves administering the missing protein (factor VIII or factor
IX) to a patient. Factor-replacement therapy is most commonly administered in a so-called ‘on-demand regimen’,
when a patient’s symptomatology necessitates treatment.
12179

Tag Question

Feedback

Difficulty: Easy

Peer Responses

Session Progress

Responses Correct: 0

Responses Incorrect: 1

Responses Total: 1

Responses - % Correct: 0%

Next Question
Blog
About Pastest
Contact Us
Help

© Pastest 2016
Previous Question

https://mypastest.pastest.com/Secure/TestMe/Browser/436619[‫ م‬12:19:19 10/12/1437]


MyPastest

https://mypastest.pastest.com/Secure/TestMe/Browser/436619[‫ م‬12:19:19 10/12/1437]


MyPastest

Prefer to use the old MyPastest? Access it here »

Back to Filters

Question 2 of 63

A 16-year-old young man with sickle-cell anaemia is admitted with recent breathlessness. He is febrile and has a
clear chest with saturations of 98% on air. From his out-patient notes his usual Hb is 9 g/dl. FBC taken in A&E
shows WCC 8.6 × 103 / mm3, Hb 4.7 g/dl, platelets 573 × 103 / mm3 with a bilirubin 25 mmol/l.

Which investigation is the most useful to perform next?

A Serum haptoglobin

B Urinary haemosiderin

C Parvovirus serology

D Reticulocyte count

E Chest X-ray

Explanation
Reductions in Hb in sickle-cell disease are common, and determining the pathophysiological process is crucial for
the management of such patients. Haemolysis is less likely as the bilirubin is not high given the large drop in Hb.
The one investigation that will confirm an aplastic crisis is a reticulocyte count – if this is very low with such a
severe anaemia then the marrow is not responding appropriately and urgent supportive measures are needed with
very close monitoring of haematological indices. Parvovirus serology may help in confirming the cause of an
aplastic crisis but does not in itself demonstrate the lack of appropriate marrow response. Evidence for haemolysis
would also be sought in this situation, but from the data given, an aplastic crisis seems more likely. The drop in Hb
can occur suddenly from parvovirus infection as there is markedly reduced red cell
survival in sickle-cell disease,
Next Question
implying that the circulating red cell
count is heavily dependent on current marrow function. 12180

https://mypastest.pastest.com/Secure/TestMe/Browser/436619[‫ م‬12:19:45 10/12/1437]


MyPastest

End Session
Tag Question

Feedback

Difficulty: Difficult

Peer Responses

Session Progress

Responses Correct: 0

Responses Incorrect: 2

Responses Total: 2

Responses - % Correct: 0%

Blog
About Pastest
Contact Us
Help

© Pastest 2016

Previous Question

https://mypastest.pastest.com/Secure/TestMe/Browser/436619[‫ م‬12:19:45 10/12/1437]


MyPastest

Prefer to use the old MyPastest? Access it here »


End Session

Back to Filters

Question 3 of 63

Which one of the following is the MOST common cause of aplastic crisis in a patient with sickle cell disease?

A Dehydration

B Respiratory syncytial virus infection

C Human parvovirus B19 infection

D Repeated blood transfusion

E Haemophilus influenzae septicaemia

Explanation
Aplastic anaemic crisis occur more frequently in sickle cell children with a parvovirus infection, which is
characterised
by mild fever, dyspnoea on exertion, anorexia, and pallor. Recovery from aplastic crisis typically
requires a week, but the patient may need
to be transfused with packed erythrocytes until marrow recovery. Well-
known precipitants of vaso-occlusive pain crises (VPC) include cold
weather, relative high haemoglobin
concentration, dehydration, infection, exercise, dampness, poor diet, hypoxia, acidosis, emotional stress, and
fatigue. 12181

Next Question Tag Question

Feedback

Previous Question
Difficulty: Easy

https://mypastest.pastest.com/Secure/TestMe/Browser/436619[‫ م‬12:20:04 10/12/1437]


MyPastest

Peer Responses

Session Progress

Responses Correct: 0

Responses Incorrect: 3

Responses Total: 3

Responses - % Correct: 0%

Blog
About Pastest
Contact Us
Help

© Pastest 2016

https://mypastest.pastest.com/Secure/TestMe/Browser/436619[‫ م‬12:20:04 10/12/1437]


MyPastest

Prefer to use the old MyPastest? Access it here »

Back to Filters

Question 4 of 63

A 17-year-old student attends the blood transfusion service wishing to donate blood. She is currently well, has
never had a serious illness and weighs 60 kg. Almost 6 months ago she spent the summer doing
voluntary work in
Nigeria. She also had her ears pierced 3 years ago and had a tattoo put on her left arm 2 years ago. She is not
acceptable as a donor.

What is the reason for her rejection for blood donation in the UK?

A Body piercing

B Travel to an endemic area

C Tattoo

D Age

E Underweight

Explanation
Blood donors are deferred if they are less than 17 years of age or under 50 kg in weight. They are also deferred for
12 months after body piercing or tattoos, and now only for 6 months following these incidents if the anti-hepatitis
B core test is negative.
They are deferred for 6 months after travel to an endemic malarial area, or longer if the
donor fell ill abroad or was resident for more than 6 months in Sub-Saharan Africa. They will be deferred for a
period after pregnancy. Absolute deferral now occurs if they have had any organ
transplant or blood transfusion in
the UK because of the risk of variant Creutzfeldt–Jakob disease. Note that a recent case has been reported where
probable transmission of variant Creutzfeldt–Jakob disease occurred following a transfusion from a donor
Next Question
incubating variant Creutzfeldt–Jakob disease. Any risk of drug abuse or exposure to HIV would be reasons for
absolute deferral. 12182

https://mypastest.pastest.com/Secure/TestMe/Browser/436619[‫ م‬12:20:24 10/12/1437]


MyPastest

End Session
Tag Question

Feedback

Difficulty: Average

Peer Responses

Session Progress

Responses Correct: 0

Responses Incorrect: 4

Responses Total: 4

Responses - % Correct: 0%

Blog
About Pastest
Contact Us
Help

© Pastest 2016

Previous Question

https://mypastest.pastest.com/Secure/TestMe/Browser/436619[‫ م‬12:20:24 10/12/1437]


MyPastest

Prefer to use the old MyPastest? Access it here »

Back to Filters

Question 5 of 63

A 16-year-old boy is visiting relatives in West Africa after schooling in England for his GCSEs. As part of his
preparation to travel
he has to be vaccinated against cholera and hepatitis A, and he has to take antimalarials such
as primaquine. Two days after starting he notices that his urine is dark and he experiences back pain. He is also
jaundiced.

Which of these enzyme abnormalities is most likely to be responsible for his clinical symptoms?

A Pyruvate kinase deficiency

B Triose phosphate isomerase deficiency

C Glucose–6-phosphate dehydrogenase deficiency

D Phosphoenolpyruvate deficiency

E Hexokinase deficiency

Explanation
Glucose–6-phosphate dehydrogenase deficiency (G6PD) is the commonest inherited red cell enzyme abnormality
in African
Americans and Africans. It is found in 11% of African Americans. It is inherited as a sex-linked
disorder and affects males except in extreme cases of lyonisation (abnormal inactivation of the X chromosome), in
which it affects carrier females. The symptoms occur within 1–3 days of drug ingestion and result in an episodic
intravascular haemolysis with haemoglobinuria, a self-limiting anaemia and, in most cases, abdominal and back
pain. All the enzymes listed above are part of
the Embden–Meyerhof pathway and can cause haemolysis. 12838
Next Question

https://mypastest.pastest.com/Secure/TestMe/Browser/436619[‫ م‬12:20:44 10/12/1437]


MyPastest

End
Tag Session
Question

Feedback

Difficulty: Easy

Peer Responses

Session Progress

Responses Correct: 0

Responses Incorrect: 5

Responses Total: 5

Responses - % Correct: 0%

Blog
About Pastest
Contact Us
Help

© Pastest 2016

Previous Question

https://mypastest.pastest.com/Secure/TestMe/Browser/436619[‫ م‬12:20:44 10/12/1437]


MyPastest

Prefer to use the old MyPastest? Access it here »


End Session

Back to Filters

Question 6 of 63

Which one of the following statements BEST describes haemophilia A?

A Petechiae are more common than soft tissue bleeding

B Bleeding time is prolonged

C Factor 8 inhibitors occur in 20% of patients receiving multiple factor 8 transfusion

D Iron deficiency anaemia is a frequent and persistent problem

E Joint deformity is rare despite the fact that haemarthrosis is one of the main recurrent manifestations

Explanation
Petechiae are an extremely important clue
in the diagnosis of haemostatic disorder and are only seen in platelet
disorders, not in haemophilia which is due to factor deficiency. Haemarthroses are almost pathognomonic of
severe haemophilia and often lead to joint damage and deformities. The bleeding in haemophilia is usually
confined to soft tissue, muscle and other body compartments. The
iron is usually recycled from these sites and iron
deficiency anaemia is not a frequent feature of haemophilia. APTT is prolonged, PT and bleeding time are
typically normal. Prolonged bleeding time is caused either by thrombocytopenia or by qualitative platelet
dysfunction.
12839

Next Question
Tag Question

Previous Question
Feedback

https://mypastest.pastest.com/Secure/TestMe/Browser/436619[‫ م‬12:21:19 10/12/1437]


MyPastest

Difficulty: Average

Peer Responses

Session Progress

Responses Correct: 0

Responses Incorrect: 6

Responses Total: 6

Responses - % Correct: 0%

Blog
About Pastest
Contact Us
Help

© Pastest 2016

https://mypastest.pastest.com/Secure/TestMe/Browser/436619[‫ م‬12:21:19 10/12/1437]


MyPastest

Prefer to use the old MyPastest? Access it here »

Back to Filters

Question 7 of 63

Some but not all units of blood are screened for certain infectious agents.

Patients with impaired immunology should receive blood products screened for which organism?

A Cytomegalovirus

B Human immunodeficiency virus-2

C Treponema pallidum

D Hepatitis C virus

E Hepatitis B virus

Explanation
All units of blood collected in the UK are currently tested for the following:

Human immunodeficiency virus-1 (HIV-1) and HIV-2 by antibody testing


Hepatitis B by hepatitis B surface antigen
Hepatitis C (HCV) by anti-HCV antibody and batch polymerase chain reaction (PCR) testing (nucleic acid
test, NAT)
Syphilis by Venereal Disease Research Laboratories (VDRL)
Human T-cell leukaemia virus-1 (HTLV-1) testing was introduced in 2003

A
proportion of units are screened for cytomegalovirus (CMV), just enough
to give CMV-negative units to
immunocompromised recipients (such as children with acute leukaemias) at risk of severe CMV infection.
12840

https://mypastest.pastest.com/Secure/TestMe/Browser/436619[‫ م‬12:21:39 10/12/1437]


MyPastest

End Session

Tag Question

Feedback

Difficulty: Easy

Peer Responses

Session Progress

Responses Correct: 0

Responses Incorrect: 7

Responses Total: 7

Responses - % Correct: 0%

Blog
About Pastest
Contact Us
Help

© Pastest 2016

Next Question

Previous Question

https://mypastest.pastest.com/Secure/TestMe/Browser/436619[‫ م‬12:21:39 10/12/1437]


MyPastest

Prefer to use the old MyPastest? Access it here »

Back to Filters

Question 8 of 63

What is the average lifespan of an erythrocyte once it has entered the bloodstream?

A 1 day

B 10 days

C 120 days

D 200 days

E 360 days

Explanation
Normal red cells have a finite lifespan of 120 ± 20 days. With red cell ageing, metabolic changes decrease their
flexibility as they traverse through the microvasculature and promote their lysis or phagocytosis. Thus, the red
cell’s longevity and ability to carry out its proper function is critically dependent on cell-membrane structure and
metabolism. The red cell membrane consists of a lipid bilayer and structural and integral membrane proteins that
provide a lattice network under the bilayer and create the red cell cytoskeleton. Inherited defects in protein
structure (hereditary spherocytosis, hereditary elliptocytosis, hereditary pyropoikilocytosis etc) lead to haemolytic
anaemia. Enzyme defects in metabolic pathways (pyruvate kinase, hexokinase, glucose-6-phosphate
dehydrogenase deficiency), and haemoglobin defects (sickle-cell anaemia), can also increase the haemolytic
potential of the red cell. 12841

Next Question

Tag Question
Previous Question

https://mypastest.pastest.com/Secure/TestMe/Browser/436619[‫ م‬12:22:01 10/12/1437]


MyPastest

Feedback End Session

Difficulty: Easy

Peer Responses

Session Progress

Responses Correct: 0

Responses Incorrect: 8

Responses Total: 8

Responses - % Correct: 0%

Blog
About Pastest
Contact Us
Help

© Pastest 2016

https://mypastest.pastest.com/Secure/TestMe/Browser/436619[‫ م‬12:22:01 10/12/1437]


MyPastest

Prefer to use the old MyPastest? Access it here »

Back to Filters

Question 9 of 63

A 4-year-old who has been on treatment for epilepsy with Carbamazepine without incident for some considerable
time was admitted with a history of multiple bruising. She had a seizure the previous day and had knocked her
head during the attack, and now has a large bruise over the forehead in addition to several over her arms, legs and
back. She had recently recovered from a 'flu' like illness and has had a course of Cephalexin. She was previously
on the 'at risk' register because of neglect and a violent father. Clinically, she appeared well and not anaemic, with
small shotty lymph nodes on both sides of the neck. Multiple bruises and fine petechiae noted over the trunk and
limbs. Hb 11.3 g/dl, WBC 8.0 x109/l, normal differential count, platelets < 10x109/l, elevated platelet-associated
IgG. Bone marrow showed increase in megakaryocytes.

Based on the above information, the most likely cause of her bruising is?

A Glandular fever

B Acute lymphoblastic leukaemia (ALL)

C Idiopathic thrombocytopaenic purpura

D Carbamazepine induced bone marrow suppression

E Physical abuse

Explanation
All the given causes could cause bruising, the first
four being potential causes of thrombocytopaenia. However,
the clinical
haematological, bone marrow, and immunological findings are most compatible with ITP. Though she
has a 'viral' illness it was very unlikely to have been glandular fever. The marrow and haematology did not have
any features of ALL, e.g blast cells on microscopy. She had been on carbamazepine for a considerable time and
tolerated it well. The
haematological effect reported is a general bone marrow depression with
agranulocytosis that
occurs within the first few months of treatment. Physical abuse should be considered and on the initial differential
diagnosis list, but given the haematological, bone marrow and immunological findings, this should then be
removed.

https://mypastest.pastest.com/Secure/TestMe/Browser/436619[‫ م‬12:22:22 10/12/1437]


MyPastest

14578
End Session

Tag Question

Feedback

Difficulty: Easy

Peer Responses

Session Progress

Responses Correct: 0

Responses Incorrect: 9

Responses Total: 9

Responses - % Correct: 0%

Blog
About Pastest
Contact Us
Help

© Pastest 2016

Next Question

Previous Question

https://mypastest.pastest.com/Secure/TestMe/Browser/436619[‫ م‬12:22:22 10/12/1437]


MyPastest

Prefer to use the old MyPastest? Access it here »

Back to Filters

Question 10 of 63

Theme: Diagnostic investigations

A Full blood count


B Clotting screen
C Peripheral blood screen
D Monospot
E Haemoglobin electrophoresis
F Bone marrow aspiration cytology
G C-Reactive protein
H Erythrocyte sedimentation rate
I Total immunoglobulins
J Urine CMV culture

For
each of the clinical scenarios below select the investigation most likely to give a definitive diagnosis. Each
option may be used once, more than once, or not at all.

Scenario 1

A 6-month-old Afro-Caribbean child who presents with dactylitis.

Your answer was incorrect

Select one...

E - Haemoglobin electrophoresis

In a child with dactylitis and of Afro-Caribbean origin, sickle cell disease should be considered. This would most
easily be diagnosed by haemoglobin electrophoresis.

Scenario 2

A 7-year-old girl, who had an URTI 7 days ago, bruises easily and has developed petechiae.

Your answer was incorrect

Select one...

https://mypastest.pastest.com/Secure/TestMe/Browser/436619[‫ م‬12:22:42 10/12/1437]


MyPastest

F - Bone marrow aspiration cytology


End Session

In a child with bruises and petechiae,


although initially a full blood count would be carried out, a bone marrow
would give the definitive diagnosis, whether this be ITP, ALL, etc.

Scenario 3

A 14-year-old boy who presents with a sore throat and palatal petechiae.

Your answer was incorrect

Select one...

D - Monospot

Palatal petechiae are highly suggestive of EBV infection, therefore the Monospot would be diagnostic.
14584

Tag Question

Feedback

Difficulty: Average

Session Progress

Responses Correct: 0

Responses Incorrect: 12

Responses Total: 12

Responses - % Correct: 0%

Blog
About Pastest
Contact Us
Help

© Pastest 2016
Next Question

Previous Question

https://mypastest.pastest.com/Secure/TestMe/Browser/436619[‫ م‬12:22:42 10/12/1437]


MyPastest

Prefer to use the old MyPastest? Access it here »

Back to Filters

Question 11 of 63

A 7-month-old baby boy from the Indian subcontinent has been in the UK for 1 month. At a routine health visit he
is noted to be pale.
Subsequent investigations show FBC: Hb 9.3 g/dL, WCC 4.5 × 109/L, plts 240 × 109/L,
mean
corpuscular volume (MCV) 70 fL, ferritin 16 ng/mL (normal range 100–500 ng/mL), reticulocytes 1.6%, Hb
electrophoresis 98% HbA.

Given these results, what is the most likely cause for the anaemia?

A Physiological anaemia of infancy

B α-Thalassaemia trait

C Chronic helminthic infection

D Dietary iron deficiency

E Dietary folate deficiency

Explanation
Iron deficiency is the most likely because there is a
microcytic anaemia with a low serum ferritin. The most likely
cause of this in the UK is a dietary deficiency. Chronic helminth infection is a common cause worldwide. Folate
deficiency leads to megaloblastic anaemia. As the Hb electrophoresis shows 98% HbA (normal) then α-
thalassaemia is ruled out. 22239

Next Question

Tag Question
Previous Question
Feedback

https://mypastest.pastest.com/Secure/TestMe/Browser/436619[‫ م‬12:23:24 10/12/1437]


MyPastest

End Session

Difficulty: Easy

Peer Responses

Session Progress

Responses Correct: 0

Responses Incorrect: 13

Responses Total: 13

Responses - % Correct: 0%

Blog
About Pastest
Contact Us
Help

© Pastest 2016

https://mypastest.pastest.com/Secure/TestMe/Browser/436619[‫ م‬12:23:24 10/12/1437]


MyPastest

Prefer to use the old MyPastest? Access it here »

Back to Filters

Question 12 of 63

A 4-year-old child presents following a viral URTI with bruising and petechiae, but is afebrile and well. On
examination there are petechiae on the trunk and legs, bruises on the leg, no petechiae in
the mouth or lips and no
other abnormal findings. His platelet count is
9 × 109/L; the rest of the FBC, coagulation screen and
urine dipstick
are normal. A diagnosis of ITP (immune-mediated thrombocytopenia purpura) is made.

What is the best course of action from this point?

A Admit and observe for 48 h with daily platelet counts

B Admit and start intravenous immunoglobulin

C Admit and start intravenous immunoglobulin followed by 1 unit platelets

D Admit and arrange for a bone marrow aspiration, with a view to starting steroids

E Discharge home but arrange for daily review and FBC

Explanation
Since there is no ‘mucosal’ bleeding (i.e. lips/gums/tongue, urine or bowel) there is a lower risk of serious
intracranial bleeding. There is no need to treat the lower risk groups acutely. The parents should have the
diagnosis explained to them and the
child should have frequent reviews. The natural course is that the average
time to normal platelet numbers is 1 month; most others will be normal by 6 months but a small proportion will
have chronic ITP. If there is mucosal bleeding, there may be an increased risk of intracranial bleeding and active
treatment may be advised. 22240
Next Question

Tag Question

https://mypastest.pastest.com/Secure/TestMe/Browser/436619[‫ م‬12:23:45 10/12/1437]


MyPastest

End Session
Feedback

Difficulty: Average

Peer Responses

Session Progress

Responses Correct: 0

Responses Incorrect: 14

Responses Total: 14

Responses - % Correct: 0%

Blog
About Pastest
Contact Us
Help

© Pastest 2016

Previous Question

https://mypastest.pastest.com/Secure/TestMe/Browser/436619[‫ م‬12:23:45 10/12/1437]


MyPastest

Prefer to use the old MyPastest? Access it here »

Back to Filters

Question 13 of 63

Theme: Blood disorders

A Acute lymphoblastic leukaemia


B Aplastic anaemia
C ß-Thalassaemia intermedia
D Christmas disease
E G6PDH (glucose-6-phosphate dehydrogenase) deficiency
F Haemophilia A
G Henoch–Schönlein purpura
H Immune-mediated thrombocytopenia purpura
I Meningococcal septicaemia
J Sickle cell disease

For
each of the following cases, please choose the diagnosis from the list above that best fits the clinical and
laboratory information. Each item may be used once, more than once or not at all.

Scenario 1

A 4-year-child of eastern Mediterranean origin presents pale and tired following a UTI treated with antibiotics.
His full blood count (FBC) reveals Hb 4.2 g/dL, white cell count (WCC) 12 × 109/L, platelets (plts) 332 × 109/L,
reticulocytes 6%.

Your answer was incorrect

Select one...

E - G6PDH (glucose-6-phosphate dehydrogenase) deficiency

G6PDH deficiency is more common in people of Mediterranean origin (up to 35%). Haemolysis can be triggered
by a number of factors (favism, antimalarials, sulphonamides, etc) – in
this case probably nitrofurantoin
sometimes given for UTIs. The picture
is of a haemolytic anaemia: there is a very good reticulocyte response
showing that bone marrow production is good. Thalassaemia is unlikely because this presented as an acute
problem.

Scenario 2

https://mypastest.pastest.com/Secure/TestMe/Browser/436619[‫ م‬12:24:05 10/12/1437]


MyPastest

A 6-year-old child develops a petechial rash. Her FBC shows Hb 11 g/dL, WCC 6.5 × 109/L, plts 9 × 109/L. A
End Session
bone marrow trephine shows an increase in megakaryocytes.

Your answer was incorrect

Select one...

H - Immune-mediated thrombocytopenia purpura

This is a typical picture of immune-mediated thrombocytopenia purpura. The low platelet count is a result of
peripheral destruction of platelets, not failure of production (megakaryocytes are precursors of platelets in the
bone marrow).

Scenario 3

A 9-year-old child has recently recovered from a febrile illness and is now well but develops a purpuric rash on his
lower limbs. His FBC shows Hb 13 g/dL, WCC 12 × 109/L (neutrophils 2 × 109/L, lymphocytes 10 × 109/L), plts
540 × 109/L.
Coagulation profile: international normalized ratio (INR) 1.2, acitvated partial thromboplastin time
(APTT) 28 s, prothrombin time (PT)
11 s.

Your answer was incorrect

Select one...

G - Henoch–Schönlein purpura

The distribution of the rash is consistent with Henoch–Schönlein purpura. In a well child, with normal platelets
and clotting, meningococcal septicaemia is less likely; however, it should always be considered.
22405

Tag Question

Feedback

Difficulty: Average

Session Progress

Responses Correct: Next Question 0

Responses Incorrect: 17

Responses Total: 17
Previous Question
Responses - % Correct: 0%

https://mypastest.pastest.com/Secure/TestMe/Browser/436619[‫ م‬12:24:05 10/12/1437]


MyPastest

Blog
About Pastest
Contact Us
Help

© Pastest 2016

https://mypastest.pastest.com/Secure/TestMe/Browser/436619[‫ م‬12:24:05 10/12/1437]


MyPastest

Prefer to use the old MyPastest? Access it here »

Back to Filters

Question 14 of 63

With regard to features of TAR syndrome, select the MOST appropriate.

A Death from acute haemorrhage in late childhood

B Digit deformities

C Normal platelet count

D T-cell defect

E X-linked inheritance

Explanation
Inheritance is autosomal recessive, although autosomal dominance with incomplete penetrance has been
postulated.

Clinically:

Absent radii and other limb abnormalities


Thrombocytopenia
Variable other manifestations including cardiac, renal, gastrointestinal and skeletal

The
bone marrow shows myeloid hyperplasia with an absence of megakaryocytes. Haemorrhagic manifestations
can range from petechiae to fatal intracranial haemorrhages, but for patients who survive the first year the platelet
Next Question
count tends to stabilise and the prognosis is better.
43987

https://mypastest.pastest.com/Secure/TestMe/Browser/436619[‫ م‬12:24:25 10/12/1437]


MyPastest

End Session
Tag Question

Feedback

Difficulty: Average

Peer Responses

Session Progress

Responses Correct: 0

Responses Incorrect: 18

Responses Total: 18

Responses - % Correct: 0%

Blog
About Pastest
Contact Us
Help

© Pastest 2016

Previous Question

https://mypastest.pastest.com/Secure/TestMe/Browser/436619[‫ م‬12:24:25 10/12/1437]


MyPastest

Prefer to use the old MyPastest? Access it here »

Back to Filters

Question 15 of 63

With regard to tumour lysis syndrome, select the MOST appropriate statement

A It is more common in patients with a low tumour burden

B Metabolic complications include hypokalaemia and hypercalcaemia

C Packed red blood cell transfusions are often indicated

D Potassium should be added to all fluids

E Treatment includes hyper hydration

Explanation
Tumour lysis syndrome is an oncological emergency. It most commonly occurs in patients with a high tumour
burden
such as ALL or lymphoma. It may be precipitated by starting chemotherapy or a single dose of steroids but
can occur spontaneously. Rapid cell proliferation accompanied by lysis and cell death leads to release of
intracellular metabolites in quantities that saturate the body’s usual renal and physiological buffering mechanisms.

Laboratory finding include:

Hyperuricaemia
Hyperkalaemia
Hyperphosphataemia
Hypocalcaemia
Renal failure

Management includes:

https://mypastest.pastest.com/Secure/TestMe/Browser/436619[‫ م‬12:24:46 10/12/1437]


MyPastest

Hyperhydration: potassium should not be administrated until the tumour lysis is under control
End Session
Diuretics if necessary to maintain good urine output
Allopurinol to treat hyperuricaemia and reduce urate precipitation
Infusion of insulin/dextrose, salbutamol to treat hyperkalaemia
Dialysis may be required

43988

Tag Question

Feedback

Difficulty: Average

Peer Responses

Session Progress

Responses Correct: 0

Responses Incorrect: 19

Responses Total: 19

Responses - % Correct: 0%

Blog
About Pastest
Contact Us
Help
Next Question
© Pastest 2016

Previous Question

https://mypastest.pastest.com/Secure/TestMe/Browser/436619[‫ م‬12:24:46 10/12/1437]


MyPastest

Prefer to use the old MyPastest? Access it here »

Back to Filters

Question 16 of 63

Concerning Hodgkin lymphoma, select the MOST appropriate statement

A Bone marrow involvement occurs in >50% of patients

B Lymphadenopathy is rare

C Mediastinal masses can lead to airway obstruction

D More common in girls

E Presentation tends to be in early childhood

Explanation
Hodgkin disease is characterised by progressive painless lymph node enlargement. Lymphadenopathy occurs in
more than 80% of cases and mediastinal adenopathy in 60%. Patients may also present with a non-productive
cough or complications of a mediastinal mass such as superior vena caval and airway obstruction.

Histopathology
of the lymph nodes shows Reed–Sternberg cells which are multinucleated giant cells. Bone
marrow involvement is rare, with dissemination to other organs being a late complication.

Associations:

Epstein–Barr virus (EBV) infection


Systemic lupus erythematosus (SLE)
Rheumatoid arthritis
Ataxia telangiectasia

Prognosis is good with a >90% 5-year survival rate.


43989

https://mypastest.pastest.com/Secure/TestMe/Browser/436619[‫ م‬12:25:06 10/12/1437]


MyPastest

End Session

Tag Question

Feedback

Difficulty: Average

Peer Responses

Session Progress

Responses Correct: 0

Responses Incorrect: 20

Responses Total: 20

Responses - % Correct: 0%

Blog
About Pastest
Contact Us
Help

© Pastest 2016

Next Question

Previous Question

https://mypastest.pastest.com/Secure/TestMe/Browser/436619[‫ م‬12:25:06 10/12/1437]


MyPastest

Prefer to use the old MyPastest? Access it here »

Back to Filters

Question 17 of 63

Which one of the following is a feature of hereditary spherocytosis?

A Conjugated hyperbilirubinaemia

B Haemolytic crises following fava bean ingestion

C Short stature

D Splenomegaly

E X-linked inheritance

Explanation
The incidence of hereditary spherocytosis is
1 in 5000 people of northern European extraction. It is inherited as an
autosomal dominant with variable penetrance. It usually presents in childhood with the classic triad of anaemia,
jaundice and splenomegaly. The jaundice is unconjugated and worsens with viral infections. Pigmented gallstones
are present in 85% by the second decade. Neonatal jaundice is common. Aplastic crises can occur. The diagnosis
is made on clinical grounds, by observing the spheroidal cells on a blood film and by the increased osmotic
fragility of red cells when tested. Treatment is symptomatic or by splenectomy usually done laparoscopically. The
spleen is the site of red cell destruction. Removal of the spleen will reduce haemolysis and the incidence of
gallstones. There is, however, an
increased risk of pneumococcal and other infections with splenectomy, and
therefore it should be ensured that the patient has had pneumococcal
immunisation as part of the routine schedule
and receive lifelong prophylactic penicillin. 43990

Next Question

Previous Question
Tag Question

https://mypastest.pastest.com/Secure/TestMe/Browser/436619[‫ م‬12:25:26 10/12/1437]


MyPastest

Feedback End Session

Difficulty: Average

Peer Responses

Session Progress

Responses Correct: 0

Responses Incorrect: 21

Responses Total: 21

Responses - % Correct: 0%

Blog
About Pastest
Contact Us
Help

© Pastest 2016

https://mypastest.pastest.com/Secure/TestMe/Browser/436619[‫ م‬12:25:26 10/12/1437]


MyPastest

Prefer to use the old MyPastest? Access it here »

Back to Filters

Question 18 of 63

Glanzmann thrombasthenia.

Select the MOST appropriate statement

A It is inherited as an autosomal dominant

B Its gene locus is known

C It requires long-term steroid treatment

D Splenectomy may be beneficial

E The platelet count is low

Explanation
This is autosomally recessively inherited, with the gene locus on chromosome 7. Pathogenesis is a failure of
platelet aggregation in response to ADP, collagen and thrombin. This is due to a defect in glycoprotein IIb and
IIIa, both being part of the platelet membrane and deficiency resulting in a failure of the platelet to bind
fibrinogen. It is a rare condition and presents in early childhood with recurrent bleeding. Platelet count and
morphology are normal. The only possible long-term treatment is bone marrow transplantation. In the short term
tranexamic acid can be used to
control acute bleeding.

Bernard–Soulier syndrome:

This
is an autosomal recessive condition. The characteristic features are giant platelets with a reduced life span
and hence thrombocytopenia, and
a prolonged bleeding time. Platelet aggregation is normal.
Next Question 43991

https://mypastest.pastest.com/Secure/TestMe/Browser/436619[‫ م‬12:25:46 10/12/1437]


MyPastest

End
Tag Session
Question

Feedback

Difficulty: Average

Peer Responses

Session Progress

Responses Correct: 0

Responses Incorrect: 22

Responses Total: 22

Responses - % Correct: 0%

Blog
About Pastest
Contact Us
Help

© Pastest 2016

Previous Question

https://mypastest.pastest.com/Secure/TestMe/Browser/436619[‫ م‬12:25:46 10/12/1437]


MyPastest

Prefer to use the old MyPastest? Access it here »

Back to Filters

Question 19 of 63

Concerning rhesus haemolytic disease, select the MOST appropriate statement.

A It can present as hydrops fetalis

B It cannot occur in firstborn children

C It is an indication for elective caesarean section

D It occurs as a consequence of the transplacental passage of IgM

E The most common antibody type is anti-C

Explanation
Haemolytic disease of the newborn occurs as a consequence of the transplacental passage of anti-D antibodies
(IgG) from the rhesus-negative mother to the rhesus-positive fetus. This
can occur only if the mother has been
sensitised. Other rhesus antibodies (eg anti-C and -E) can occur and produce less severe disease.
The use of anti-D
passive immunisation in mothers within 72 hours of delivery has reduced the incidence of the disease.
Sensitisation in the mother (rhesus negative) can occur after miscarriages, abortions and previous deliveries of
rhesus-positive fetuses.

Haemolytic disease can occur in utero


and may produce hepatosplenomegaly, ascites and hydrops (which can be
detected on ultrasonography). Screening is done on rhesus-negative mothers at booking, 28 weeks, 34 weeks and
at time of delivery. If anti-D antibodies are found the tests are repeated every 2 weeks. If titres are high (>10
IU/ml) referral to a specialist centre is indicated. Titres >100 IU/ml indicate severe disease. If the anti-D antibody
titres are high then in utero monitoring of the fetus is carried out by amniocentesis to measure bilirubin levels in
the amniotic fluid. If amniotic bilirubin levels indicate that a high degree
of haemolysis is present, cord blood
sampling is performed. Fetal anaemia will be the consequence of severe haemolysis and can be treated with in
utero blood transfusions. Severe disease is an indication for premature delivery (at 34–36 weeks), the infant being
likely to require exchange transfusion after birth. The severity of disease will increase with successive
pregnancies.

https://mypastest.pastest.com/Secure/TestMe/Browser/436619[‫ م‬12:26:57 10/12/1437]


MyPastest

ABO haemolytic disease: End Session


This
usually occurs in mothers with blood group O who have IgG anti-A or anti-B antibodies, which will cross the
placenta and react if the baby is blood group A or B and cause haemolysis. The degree of haemolysis is generally
only mild. It occurs in 3% of births. Phototherapy usually controls the jaundice. Diagnosis is by measuring anti-A
or anti-B antibodies in a blood group O mother with a blood group A, B or AB neonate with a positive direct
Coombs test.
43992

Tag Question

Feedback

Difficulty: Average

Peer Responses

Session Progress

Responses Correct: 0

Responses Incorrect: 23

Responses Total: 23

Responses - % Correct: 0%

Blog
About Pastest
Contact Us
Help
Next Question
© Pastest 2016

Previous Question

https://mypastest.pastest.com/Secure/TestMe/Browser/436619[‫ م‬12:26:57 10/12/1437]


MyPastest

Prefer to use the old MyPastest? Access it here »

Back to Filters

Question 20 of 63

In the case of anaemia of chronic disease, select the MOST appropriate statement

A Iron supplements are contraindicated

B The anaemia is usually normocytic

C The haemoglobin is usually <8 g/dl

D The serum iron is usually increased

E The total iron binding capacity is raised

Explanation
Anaemia can occur in any disease with chronic inflammation. Characteristic features include:

normal MCV (mean corpuscular volume)


normochromia
normal or low reticulocyte count
low serum iron
low iron-binding capacity and transferrin saturation
normal or raised ferritin
increased reticuloendothelial stores of iron.

Several factors are thought to contribute to the anaemia, including decreased red cell survival, inflammatory
mediators suppressing erythrocyte production, a blunting of the erythropoietin response to anaemia and trapping
of iron by macrophages, reducing utilisation for new red cells. Anaemia is usually not <9 g/dl. Although most
cases will not respond to iron, supplements are given if serum ferritin is <50 μg/ml and may improve anaemia if

https://mypastest.pastest.com/Secure/TestMe/Browser/436619[‫ م‬12:27:40 10/12/1437]


MyPastest

there is coexistent iron deficiency. Anaemia secondary to chronic renal failure is due to absence
of erythropoietin
End Session
production and responds to treatment with recombinant
erythropoietin. Ferritin can also be raised and serum iron
low as part of the acute phase response
43993

Tag Question

Feedback

Difficulty: Average

Peer Responses

Session Progress

Responses Correct: 0

Responses Incorrect: 24

Responses Total: 24

Responses - % Correct: 0%

Blog
About Pastest
Contact Us
Help

© Pastest 2016

Next Question

Previous Question

https://mypastest.pastest.com/Secure/TestMe/Browser/436619[‫ م‬12:27:40 10/12/1437]


MyPastest

Prefer to use the old MyPastest? Access it here »

Back to Filters

Question 21 of 63

With regard to DiGeorge syndrome, select the MOST appropriate statement

A Hypercalcaemia is common

B Inheritance is autosomal recessive

C It is associated with cardiac defects

D The lymphocyte count is always reduced

E The thymus is hyperplastic

Explanation
This is usually sporadic although familial clustering has been reported. The gene has been localised to
chromosome 22.

Clinical features of DiGeorge syndrome are as follows:

Thymic aplasia/hypoplasia (hypoplasia is more common than aplasia)


Hypoparathyroidism
Presents with hypocalcaemic seizures (neonatal tetany)
Absent parathyroid hormone
Congenital heart disease
Right-sided aortic arch
Truncus arteriosus
Interrupted aortic arch
Atrioventricular septal defect
Ventricular septal defect

https://mypastest.pastest.com/Secure/TestMe/Browser/436619[‫ م‬12:28:01 10/12/1437]


MyPastest

Hypoplastic pulmonary artery End Session


Pulmonary atresia
Facial abnormalities
Hypertelorism
Cleft lip or palate
Low-set ears
Anti-mongoloid slant
Others:
– imperforate anus, oesophageal atresia – failure to thrive – chronic infection (otitis media,
pneumonia, diarrhoea) – deafness.

The total lymphocyte count can be low, normal or raised. Assessment needs to be made of the percentage of
circulating mature lymphocytes by assessing their response
to phytohaemagglutinin (PHA). Treatment first aims
to deal with manifestations of the syndrome (hypocalcaemia, cardiac defects). In the long term, bone marrow
transplantation is the treatment of choice. Graft-versus-host disease can occur after cardiac bypass and irradiated
blood products need to be given to try to avoid this.
43994

Tag Question

Feedback

Difficulty: Average

Peer Responses

Session Progress
Next Question
Responses Correct: 0

Responses Incorrect: 25

Responses Total: Previous Question 25

https://mypastest.pastest.com/Secure/TestMe/Browser/436619[‫ م‬12:28:01 10/12/1437]


MyPastest

Responses - % Correct: 0%

Blog
About Pastest
Contact Us
Help

© Pastest 2016

https://mypastest.pastest.com/Secure/TestMe/Browser/436619[‫ م‬12:28:01 10/12/1437]


MyPastest

Prefer to use the old MyPastest? Access it here »

Back to Filters

Question 22 of 63

Regarding osteosarcoma, select the MOST appropriate statement

A Fever at presentation is common

B Lung metastasis at presentation occur in 80%

C Outcome <10% survival overall

D Usually occurs in the metaphyseal region of growing bones

E Usually presents in the first decade

Explanation
The male:female ratio of osteosarcoma is 1.5:1.0, with a peak incidence in the second decade. There is an
increased risk associated with retinoblastoma, previous chemotherapy and
radiotherapy. Tumours occur in the
metaphyseal region of long bones, with the distal femur being the most common site followed by the proximal
tibia and proximal humerus. It usually presents with pain, and metastases (lung and bone) are present at
presentation in 20%. Diagnosis
is by radiology and biopsy, with bone appearing sclerotic on a plain radiograph.
Treatment is with chemotherapy and surgery. Prognosis is 60%
cure if metastases are not present at diagnosis and
20% if they are. The differential diagnosis includes Ewing sarcoma and osteomyelitis.

Ewing sarcoma:

This
has a male:female ratio of 1.5:1.0. It usually presents in the second decade and there are no known risk
factors. Tumours may arise in any bone but are found most often in flat bones (pelvis, chest wall, vertebrae) and
the diaphyseal region of long bones. The presentation is with local pain and swelling. Fever and a raised
erythrocyte sedimentation rate (ESR) are common. The appearance on a radiograph is of a lytic lesion affecting
the medullary cavity and cortical bone. The tumour elevates the periosteum giving an ‘onion skin’ appearance.
Metastases are present in 25% at presentation. Treatment is with radiotherapy and chemotherapy; surgery is not
always needed. Survival depends on whether metastases are present at diagnosis and is 20% cure if they are and

https://mypastest.pastest.com/Secure/TestMe/Browser/436619[‫ م‬12:28:23 10/12/1437]


MyPastest

70% if not.
End Session
Differential diagnosis of a lytic bone lesion:

Ewing sarcoma
Langerhans cell histiocytosis
Osteomyelitis
Lymphoma
Neuroblastoma
Metastatic sarcoma.

43995

Tag Question

Feedback

Difficulty: Average

Peer Responses

Session Progress

Responses Correct: 0

Responses Incorrect: 26

Responses Total: 26

Responses - % Correct: 0%
Next Question

Previous Question
Blog
About Pastest
Contact Us
Help

https://mypastest.pastest.com/Secure/TestMe/Browser/436619[‫ م‬12:28:23 10/12/1437]


MyPastest

© Pastest 2016

https://mypastest.pastest.com/Secure/TestMe/Browser/436619[‫ م‬12:28:23 10/12/1437]


MyPastest

Prefer to use the old MyPastest? Access it here »

Back to Filters

Question 23 of 63

Considering accidental iron ingestion.

What is MOST likely to occur?

A Haematemesis within 2 hours of ingestion

B Hepatic necrosis within 30 minutes of ingestion

C Hypertension within 2 hours of ingestion

D Lactic acidosis within the first 2 hours

E Pyloric stenosis 2 days after ingestion

Explanation
Ingestion of 60 mg/kg or above of iron may result in systemic iron toxicity. An abdominal radiograph may be
useful to confirm iron ingestion because the tablets are radio-opaque.

The effects of iron toxicity are dependent on the time after ingestion:

Stage
1 (30 min–2 h): local effects of gastrointestinal irritation including diarrhoea and vomiting.
Haematemesis and hypotension may occur.
Stage 2 (2–6 h): apparent recovery during which iron absorption and accumulation of iron in tissues and
mitochondria occur.
Stage 3 (12 h): cellular and mitochondrial damage occurs with hypoglycaemia and lactic acidosis.
Stage
4 (2–4 days): severe hepatic necrosis with raised aspartate aminotransferase, alanine
aminotransferase, bilirubin and abnormal prothrombin time.
Stage 5 (2–4 weeks): late effects with scarring and stenosis of the pylorus.

https://mypastest.pastest.com/Secure/TestMe/Browser/436619[‫ م‬12:28:47 10/12/1437]


MyPastest

End radiograph.
Investigation
of iron poisoning includes free iron levels in serum and an abdominal Session Although
investigations are useful, it is important to consider the child’s symptoms as a guide to toxicity. The treatment is
general and specific. Emesis and gastric lavage are not useful.

Desferrioxamine (iron chelator) is the main therapeutic agent available. Supportive treatment needs to be given
for
hypotension or shock. If free iron is >50 mg/dl or total iron is
>350 mg/dl then parenteral desferrioxamine is
indicated. Desferrioxamine can cause anaphylaxis; it causes the urine to turn red while chelated iron is being
excreted. Oral desferrioxamine may actually
promote iron absorption and should not be used.
43996

Tag Question

Feedback

Difficulty: Average

Peer Responses

Session Progress

Responses Correct: 0

Responses Incorrect: 27

Responses Total: 27

Responses - % Correct: 0%

Blog
About Pastest
Contact Us
Help
Next Question
© Pastest 2016

Previous Question

https://mypastest.pastest.com/Secure/TestMe/Browser/436619[‫ م‬12:28:47 10/12/1437]


MyPastest

https://mypastest.pastest.com/Secure/TestMe/Browser/436619[‫ م‬12:28:47 10/12/1437]


MyPastest

Prefer to use the old MyPastest? Access it here »

Back to Filters

Question 24 of 63

Carbon monoxide: select the MOST appropriate statement

A Acute CO toxicity can cause neuropsychiatric problems

B CO combines with haemoglobin to form carboxyhaemoglobin

C CO increases the oxygen-carrying capacity of the blood

D CO shifts the oxygen dissociation curve to the right.

E CO toxicity occurs if the carboxyhaemoglobin level is 3–5%

Explanation
Carbon monoxide is a tasteless, odourless, colourless and non-irritant gas. It binds to haemoglobin to form
carboxyhaemoglobin, which reduces the oxygen-carrying capacity of the blood and shifts the oxygen dissociation
curve to the left. The affinity of haemoglobin for carbon monoxide is 250 times greater than that for oxygen.
Endogenous production occurs and maintains a resting carboxyhaemoglobin level of 1–3%. Smoking increases
carboxyhaemoglobin levels. Other sources of raised levels include car exhaust fumes, poorly maintained heating
systems and smoke from fires.

Clinical features of carbon monoxide poisoning occur as a result of tissue hypoxia. PaO2
is normal but the oxygen
content of the blood is reduced. Toxicity relates loosely to the maximum carboxyhaemoglobin concentration.
Other factors include duration of exposure and age of the patient.

Maximum carboxyhaemoglobin concentration:

10%: not normally associated with symptoms


10–30%: headache and dyspnoea
60%: coma, convulsions and death.

https://mypastest.pastest.com/Secure/TestMe/Browser/436619[‫ م‬12:29:07 10/12/1437]


MyPastest

Endmonoxide
Neuropsychiatric
problems can occur with chronic exposure. Treatment of carbon Sessionpoisoning is with
100% oxygen, which will reduce the carboxyhaemoglobin concentration. Hyperbaric oxygen is said to reduce the
carboxyhaemoglobin level quicker.
43998

Tag Question

Feedback

Difficulty: Average

Peer Responses

Session Progress

Responses Correct: 0

Responses Incorrect: 28

Responses Total: 28

Responses - % Correct: 0%

Blog
About Pastest
Contact Us
Help

© Pastest 2016

Next Question

Previous Question

https://mypastest.pastest.com/Secure/TestMe/Browser/436619[‫ م‬12:29:07 10/12/1437]


MyPastest

Prefer to use the old MyPastest? Access it here »

Back to Filters

Question 25 of 63

In an overdose of tricyclic antidepressants, which of following statements is most likely?

A Bradycardia precedes coma

B Cardiac arrhythmias are atrial and unlikely to cause significant clinical features

C Dialysis is effective at removing the drug

D Multiple doses of charcoal therapy can be used

E The pupils are constricted

Explanation
Mortality of deliberate tricyclic antidepressant overdose is 7–12%. Effects of tricyclic antidepressants include the
following:

Anticholinergic: causing tachycardia, pupil dilatation, dry mucous membranes, urinary retention,
hallucinations and flushing
Adrenergic (early): causing hypertension and tachycardia
α-Adrenergic receptor blocking: causing prolonged hypotension
Central
inhibition of neuronal re-uptake of noradrenaline, 5-hydroxytryptamine,
serotonin and dopamine:
leading to convulsions and coma
Cardiac: mainly ventricular tachycardia and fibrillation.
Arrhythmias are the main cause of death.

Treatment of overdose:
Initial
treatment is aimed at preventing absorption of the drug. Emetics should
be used only if there is no CNS

https://mypastest.pastest.com/Secure/TestMe/Browser/436619[‫ م‬12:29:27 10/12/1437]


MyPastest

depression. Activated charcoal should be given every 2–4 hours. Arrhythmias may respond to correction of
End Session
hypoxia and correction of acidosis with sodium bicarbonate, aiming for a
pH of 7.45–7.55. Anti-arrhythmics are
best avoided. Convulsions should be treated with intravenous diazepam. Diazepam can also be used to treat
delirium and agitation during recovery. Hypotension may respond
to treatment with intravenous fluids and colloid.
In overdosage, tissue
concentrations quickly rise, giving tissue:plasma ratios of between 10:1 and 30:1. The drug
in plasma is extensively bound to plasma proteins and removal by dialysis is ineffective.
43999

Tag Question

Feedback

Difficulty: Average

Peer Responses

Session Progress

Responses Correct: 0

Responses Incorrect: 29

Responses Total: 29

Responses - % Correct: 0%

Blog
About Pastest
Contact Us
Help

© Pastest 2016 Next Question

Previous Question

https://mypastest.pastest.com/Secure/TestMe/Browser/436619[‫ م‬12:29:27 10/12/1437]


MyPastest

Prefer to use the old MyPastest? Access it here »

Back to Filters

Question 26 of 63

Concerning aspirin overdose, select the MOST appropriate statement from the following:

A Deafness is a recognised complication

B Gastric lavage and activated charcoal have no place in the management

C Hyperkalaemia can occur

D Hypoventilation is common

E Normal biochemistry at 4 hours excludes significant overdose

Explanation
Clinical features

In
young children there is dehydration and tachypnoea. Older children and adults have tachypnoea and vomiting
with progressive lethargy. There is tinnitus and deafness, and hypoglycaemia or hyperglycaemia can occur. There
are three phases.

Phase 1

May last up to 12 hours


Salicylates
directly stimulate the respiratory centre, resulting in a respiratory alkalosis with a
compensatory alkaline urine with bicarbonate sodium and
potassium loss.

Phase 2

May begin straight away, particularly in a young child, and last 12–24 hours
Hypokalaemia with, as a consequence, a paradoxical aciduria despite the alkalosis.

https://mypastest.pastest.com/Secure/TestMe/Browser/436619[‫ م‬12:29:48 10/12/1437]


MyPastest

End Session
Phase 3

After 6–24 hours


Dehydration, hypokalaemia and progressive lactic acidosis, the acidosis now predominating
Can progress to pulmonary oedema with respiratory failure, disorientation and coma.

Management

Gastric lavage up to 4 hours. Activated charcoal for sustained-release preparations.


Level at 6 hours plotted on a nomogram.
Alkalisation
of the urine to aid drug excretion, adequate fluids including bicarbonate sodium and
potassium, with close monitoring of acid–base and electrolytes.
Discuss with poisons centre.

44000

Tag Question

Feedback

Difficulty: Average

Peer Responses

Session Progress

Responses Correct: Next Question 0

Responses Incorrect: 30

Responses Total: 30
Previous Question
Responses - % Correct: 0%

https://mypastest.pastest.com/Secure/TestMe/Browser/436619[‫ م‬12:29:48 10/12/1437]


MyPastest

Blog
About Pastest
Contact Us
Help

© Pastest 2016

https://mypastest.pastest.com/Secure/TestMe/Browser/436619[‫ م‬12:29:48 10/12/1437]


MyPastest

Prefer to use the old MyPastest? Access it here »

Back to Filters

Question 27 of 63

Lead poisoning: what is MOST likely to occur?

A Encephalopathy

B Greying of the skin

C Haematemesis

D Obesity

E Thrombocytosis

Explanation
Lead poisoning is uncommon but potentially very serious. It often results from pica (persistent eating of non-
nutritive substances, eg soil) and is therefore more common in pre-school-age children. Other causes include
sucking/ingesting lead paint, lead pipes, discharge from lead batteries and substance abuse of leaded petrol. Lead
intoxication can be divided into acute and chronic effects, and results from its combination with and disruption of
vital physiological enzymes. In acute intoxication there is a reversible, renal, Fanconi-like syndrome.

In chronic intoxication:

Failure to thrive
Abdominal upset: pain/anorexia/vomiting/constipation
Lead encephalopathy: behavioural and cognitive disturbance drowsiness, seizures, neuropathies, coma
Glomerulonephritis and renal failure
Anaemia: microcytic/hypochromic, basophilic stippling of red cells.

Haematemesis occurs in iron poisoning; skin and hair changes are very common in arsenic intoxication.

https://mypastest.pastest.com/Secure/TestMe/Browser/436619[‫ م‬12:30:12 10/12/1437]


MyPastest

44001
End Session

Tag Question

Feedback

Difficulty: Average

Peer Responses

Session Progress

Responses Correct: 0

Responses Incorrect: 31

Responses Total: 31

Responses - % Correct: 0%

Blog
About Pastest
Contact Us
Help

© Pastest 2016

Next Question

Previous Question

https://mypastest.pastest.com/Secure/TestMe/Browser/436619[‫ م‬12:30:12 10/12/1437]


MyPastest

Prefer to use the old MyPastest? Access it here »


End Session

Back to Filters

Question 28 of 63

With regard to typical features of sickle cell anaemia, which of the following is MOST likely?

A Increased susceptibility to group B streptococcal infections

B Increased susceptibility to malaria

C Massive splenomegaly at the age of 10 years

D Presentation in the first month of life

E Short stature

Explanation
Sickle cell disease causes chronic anaemia with multisystem manifestations aside from the typical ‘crises’
(sequestration, aplastic, haemolytic). First presentation is usually a sequestration crisis in the phalangeal bones at
about 6 months, a dactylitis. The continual ischaemic insults to the spleen result in ‘autoinfarction’ (and
hyposplenism) in the first few years, and a palpable spleen beyond this age suggests another diagnosis. The
hyposplenism increases susceptibility to polysaccharide-capsulated organisms particularly pneumococci. It is
thought that the relative deformability of the cell membrane protects against malaria. 44002

Next Question Tag Question

Feedback

Previous Question
Difficulty: Average

https://mypastest.pastest.com/Secure/TestMe/Browser/436619[‫ م‬12:30:32 10/12/1437]


MyPastest

Peer Responses

Session Progress

Responses Correct: 0

Responses Incorrect: 32

Responses Total: 32

Responses - % Correct: 0%

Blog
About Pastest
Contact Us
Help

© Pastest 2016

https://mypastest.pastest.com/Secure/TestMe/Browser/436619[‫ م‬12:30:32 10/12/1437]


MyPastest

Prefer to use the old MyPastest? Access it here »

Back to Filters

Question 29 of 63

Which of the following is considered to be a good prognostic signs in acute lymphoblastic leukaemia?

A Common cell type ALL

B High white cell count at diagnosis

C Less than 2 years at presentation

D Male sex

E Mediastinal widening in chest radiograph

Explanation
Good prognostic signs are:

female sex
age >2 years at presentation
normal white cell count
common cell type
normal chest radiograph

44003

Next Question

Previous Question Tag Question

https://mypastest.pastest.com/Secure/TestMe/Browser/436619[‫ م‬12:30:51 10/12/1437]


MyPastest

Feedback End Session

Difficulty: Average

Peer Responses

Session Progress

Responses Correct: 0

Responses Incorrect: 33

Responses Total: 33

Responses - % Correct: 0%

Blog
About Pastest
Contact Us
Help

© Pastest 2016

https://mypastest.pastest.com/Secure/TestMe/Browser/436619[‫ م‬12:30:51 10/12/1437]


MyPastest

Prefer to use the old MyPastest? Access it here »

Back to Filters

Question 30 of 63

An otherwise well 2-year-old presents with extreme pallor. Growth and development are normal. The child is pale
but there is no lymphadenopathy or splenomegaly. Full blood count shows haemoglobin 4 g/dl (40 g/l), MCV 59.
Other indices are normal.

What is the MOST appropriate management option?

A Bone marrow aspirate

B Chelation therapy for lead

C Dietary review

D Small bowel technetium scan

E Start gluten-free diet

Explanation
Although this could be the picture of small bowel gastrointestinal loss (eg through a Meckel diverticulum), the
likeliest (and still surprisingly common) cause is over-consumption of cows’ milk which has a poor bioavailability
of iron and fills children up, reducing their appetite for high iron-containing foods. Transfusion is not usually
required unless the child is significantly compromised or it is contraindicated because by definition the iron
deficiency is long-standing and well compensated. Treatment is a change in diet with oral iron supplements at
least until the haemoglobin starts
to rise to reasonable levels. Untreated iron deficiency has adverse developmental
consequences. 44004
Next Question

Tag Question

https://mypastest.pastest.com/Secure/TestMe/Browser/436619[‫ م‬12:31:11 10/12/1437]


MyPastest

End Session
Feedback

Difficulty: Average

Peer Responses

Session Progress

Responses Correct: 0

Responses Incorrect: 34

Responses Total: 34

Responses - % Correct: 0%

Blog
About Pastest
Contact Us
Help

© Pastest 2016

Previous Question

https://mypastest.pastest.com/Secure/TestMe/Browser/436619[‫ م‬12:31:11 10/12/1437]


MyPastest

Prefer to use the old MyPastest? Access it here »


End Session

Back to Filters

Question 31 of 63

In a child presenting with lymphadenopathy, which of the following is the BEST indicator of need for excision
node biopsy?

A Associated chronic fatigue

B Known to have congenital heart disease

C Multiple anterior cervical nodes

D Post-auricular nodes

E Single supraclavicular node

Explanation
Isolated lymphadenopathy is generally benign
in children. It may be related to infection in the area of local
drainage. If multiple areas are involved the illness is likely to be systemic, eg a mononucleosis-like illness, but it is
rare even in these cases for diagnosis to require histology. The main exception to this rule is enlargement of a
supraclavicular node (due to its regional drainage), which cannot be ignored and is much more likely to herald
serious (especially malignant) pathology. In such cases a chest radiograph should be undertaken to look for a
possible mediastinal mass. 44005

Next Question
Tag Question

Feedback
Previous Question

https://mypastest.pastest.com/Secure/TestMe/Browser/436619[‫ م‬12:31:30 10/12/1437]


MyPastest

Difficulty: Average

Peer Responses

Session Progress

Responses Correct: 0

Responses Incorrect: 35

Responses Total: 35

Responses - % Correct: 0%

Blog
About Pastest
Contact Us
Help

© Pastest 2016

https://mypastest.pastest.com/Secure/TestMe/Browser/436619[‫ م‬12:31:30 10/12/1437]


MyPastest

Prefer to use the old MyPastest? Access it here »

Back to Filters

Question 32 of 63

In chronic lead poisoning, what is MOST likely to occur?

A Cardiomyopathy

B Delayed developmental milestones

C Macrocytic anaemia

D Osteomalacia

E Pulmonary fibrosis

Explanation
Lead poisoning is rarely seen nowadays due to house paints being lead free. Above a certain concentration it is
neurotoxic and the developmental effect is only partially reversible by
chelation.

Classic features of lead poisoning are:

microcytic anaemia
developmental delay
constipation
vomiting.

A
coexisting iron deficiency is common which first further exacerbates the anaemia and second contributes to
increased lead absorption. Basophilic stippling is due to inhibition of pyrimidine 5'- nucleotidase
and results in
accumulation of denatured RNA.

Treatment involves:

https://mypastest.pastest.com/Secure/TestMe/Browser/436619[‫ م‬12:31:50 10/12/1437]


MyPastest

removing source
End Session
chelation:
mild oral D-penicillamine
severe intravenous sodium calcium edetate (EDTA)
very severe intramuscular injections of dimercaprol to increase effect of EDTA.

44007

Tag Question

Feedback

Difficulty: Average

Peer Responses

Session Progress

Responses Correct: 0

Responses Incorrect: 36

Responses Total: 36

Responses - % Correct: 0%

Blog
About Pastest
Contact Us
Help
Next Question
© Pastest 2016

Previous Question

https://mypastest.pastest.com/Secure/TestMe/Browser/436619[‫ م‬12:31:50 10/12/1437]


MyPastest

Prefer to use the old MyPastest? Access it here »

Back to Filters

Question 33 of 63

A 7 year old boy is undergoing chemotherapy for Wilm's tumour. He presents to clinic for his planned
chemotherapy but has marked neutropenia. You have to decide which of the agents that are due
today, he can
have, given his low counts.

Which of the following treatment can he definitely NOT have today?

A Etoposide

B Vincristine

C Methotrexate

D Peg-arsparaginase

E Analgesia

Explanation
From the chemotherapeutic agents listed above, the only one whose administration depends on the neutrophil
count is methotrexate. The rest of the agents are not “count-dependent” as they as not as toxic to the neutrophils
as
methotrexate is.

Analgesia should be given to anyone undergoing chemotherapy as required.

NSAIDs
are not used in paediatric cancer patients because of their effects in inducing bleeding and potential renal
toxicities. Also, caution needs to be taken in the use of paracetamol in a neutropenic patient as it may
mask febrile
neutropenia which can be life threatening.
Next Question 44947

https://mypastest.pastest.com/Secure/TestMe/Browser/436619[‫ م‬12:32:10 10/12/1437]


MyPastest

End
Tag Session
Question

Feedback

Difficulty: Average

Peer Responses

Session Progress

Responses Correct: 0

Responses Incorrect: 37

Responses Total: 37

Responses - % Correct: 0%

Blog
About Pastest
Contact Us
Help

© Pastest 2016

Previous Question

https://mypastest.pastest.com/Secure/TestMe/Browser/436619[‫ م‬12:32:10 10/12/1437]


MyPastest

Prefer to use the old MyPastest? Access it here »

Back to Filters

Question 34 of 63

A 5-year-old boy is being treated for β-cell ALL. He is


in his third cycle of maintenance and is generally doing
well. He comes to his regular clinic follow up and his mum describes that he limps after receiving a certain
chemotherapeutic agent. The limping seems to get better after a few days and then recurs when he is exposed to
the specific agent again.

He also complains of occasional back pains which improve with analgesia.

Which is the most likely agent causing these side effects?

A Methotrexate

B Dexamethasone

C 6-Mercaptopurine

D Vincristine

E Etoposide

Explanation
The core agents used in the maintenance part of the ALL treatment are oral methotrexate and 6-mercaptopurine at
home and methotrexate and vincristine, with pulses of oral dexamethasone
and also peg-asparaginase.

Vincristine commonly causes bone pains, change in sensation, constipation, hair loss and headaches.

Neurotoxicity
is a typical side effect of vincristine and it can manifest in many ways including mobility issues,
Next Question
limb pains, back pain, and cranial nerve
neuropathies.
44948

https://mypastest.pastest.com/Secure/TestMe/Browser/436619[‫ م‬12:32:30 10/12/1437]


MyPastest

End Session
Tag Question

Feedback

Difficulty: Average

Peer Responses

Session Progress

Responses Correct: 0

Responses Incorrect: 38

Responses Total: 38

Responses - % Correct: 0%

Blog
About Pastest
Contact Us
Help

© Pastest 2016

Previous Question

https://mypastest.pastest.com/Secure/TestMe/Browser/436619[‫ م‬12:32:30 10/12/1437]


MyPastest

Prefer to use the old MyPastest? Access it here »

Back to Filters

Question 35 of 63

Anna, a 9-year-old girl, is referred from the GP with a 2-week history of a swelling on the left side of her face.
The swelling was first noted while on holiday and it has been there ever since. Mum also noted that her eye on that
side seems to be poking out. Anna has not been having any headaches or problems with her vision. There has been
no pain anywhere and no weight-loss or fatigue.

Examination reveals a firm swelling of about 4cm on the left temporal side, extending to the eye which is not
painful or tender to palpation and is not mobile or erythematous. The rest of the physical examination is normal,
apart from mild pallor. Baseline bloods including FBC, U+es, LFTs, Calcium, and Phosphate were performed and
were normal. Urine catecholamines were normal. A CT scan revealed a mass in the left temporal region with
abnormally enhancing tissue associated with the pterygoid muscle and with orbital involvement. No other lesions.
No hydrocephalus. An abdominal CT was performed which was normal.

What is the most likely diagnosis?

A Acute lymphoma

B Ependymoma

C Metastasis from other solid tumour

D Neuroblastoma

E Sarcoma

Explanation
Rhabdomyosarcoma (RMS) is the most commonly occurring soft-tissue sarcoma in children. When the disease is
localised, overall 5-year survival is more than 80% with the combination
of surgery, radiation and chemotherapy.

It usually presents with


proptosis when it affects soft tissues of the head. In this case, the bloods were normal
which excludes a lymphoma. The abdominal CT was normal and the very short history of 2 weeks in a previously
fit child, excludes metastasis.Urine catecholamines were normal which excludes neuroblastoma. There is nothing

https://mypastest.pastest.com/Secure/TestMe/Browser/436619[‫ م‬12:32:49 10/12/1437]


MyPastest

to suggest ependymoma which is a type of


brain tumour.
End Session 44949

Tag Question

Feedback

Difficulty: Average

Peer Responses

Session Progress

Responses Correct: 0

Responses Incorrect: 39

Responses Total: 39

Responses - % Correct: 0%

Blog
About Pastest
Contact Us
Help

© Pastest 2016

Next Question

Previous Question

https://mypastest.pastest.com/Secure/TestMe/Browser/436619[‫ م‬12:32:49 10/12/1437]


MyPastest

Prefer to use the old MyPastest? Access it here »

Back to Filters

Question 36 of 63

A 15-year-old boy presents with cough, sore throat, pyrexia and bleeding from his gums. He also noticed a rash on
his legs. He was previously fit and well, and three weeks ago he started with coryzal symptoms. He is not on any
medication and he denies smoking, alcohol consumption and use of illicit substances.

On examination, he is
pale, with bilateral sub-conjunctival haemorrhages, erythematous throat
and some petechiae
on his legs. There is no lymphadenopathy or hepatosplenomegaly.

Full blood count reveals: Hb 8.9 g/dl, MCV 110 fl, WBC 2 x 10^9/l, Neutrophils 0.3 x 109/l, Lymphocytes 1.5 x
109, Platelets 13 x 109/l Reticulocytes: 30 x 109/l (normal range 20-100 x 109/l).
Coagulation screen was normal A
bone marrow was reported as hypoplastic. A urine dipstic reveals trace of blood in the urine.

What is the most likely cause?

A Meningococcal septicaemia

B Aplastic anaemia

C Acute leukaemia

D Non-Hodgkin's lymphoma

E Haemophilia

Explanation
A decrease in the Hb, WBC and platelets demonstrates pancytopenia. A raised MCV means that the red cells are
bigger than normal, which suggests that the red cells are not maturing normally in the bone marrow or that there is
an increase in reticulocytes. The reticulocyte count is normal so the large red cells can only be attributed to a
defect in red cell maturation (a serum B12 or folate deficiency should also be excluded).

A bone marrow problem is also apparent by the fact that all lines are affected. There
is no abnormal cells in the
blood film, hence the most likely differential which is leukaemia, is eliminated as an answer. Aplastic anaemia is a

https://mypastest.pastest.com/Secure/TestMe/Browser/436619[‫ م‬12:33:10 10/12/1437]


MyPastest

rare condition in which the bone marrow elements are extremely reduced. The aetiology is unknown and it is
End Session
thought to be autoimmune mediated. There are some drugs and viruses that have been linked to aplastic anaemia.

The most effective treatment for aplastic anaemia is sibling stem cell transplant. The spleen is not enlarged in
aplastic anaemia, which differentiates it from other causes of pancytopenia.
44950

Tag Question

Feedback

Difficulty: Average

Peer Responses

Session Progress

Responses Correct: 0

Responses Incorrect: 40

Responses Total: 40

Responses - % Correct: 0%

Blog
About Pastest
Contact Us
Help

© Pastest 2016

Next Question

Previous Question

https://mypastest.pastest.com/Secure/TestMe/Browser/436619[‫ م‬12:33:10 10/12/1437]


MyPastest

Prefer to use the old MyPastest? Access it here »

Back to Filters

Question 37 of 63

Sofia, a 14-month-old girl from Cyprus, presents with pallor and her mum thinks she is always tired. She was born
in rural Cyprus and the family moved to the UK a few months ago. Sofia has never had any blood tests or any
other investigations done since she was born. Her mum describes her as a fussy eater. She does manage to have a
fairly balanced diet. She has not lost any weight but looking at her growth chart she has been on the 2nd centile
since she started being plotted 6 months ago. You perform a full blood count and ferritin with the following
results:

Hb 5.7 g/dl, MCV 55 fl, WBC 4.2 x 109/l, Platelets 420 x 109/l, Reticulocyte count 520 x 109/l (normal range 20-
100 x 109/l).
Ferritin 250 ng/ml (normal range: 20-300 ng/ml) Blood film: hypochromic microcytic red cells,
polychromasia, fragmented red cells, nucleated red cells.

What is the most likely diagnosis?

A Iron deficiency anaemia

B Aplastic anaemia

C Vitamin B12 deficiency

D Thalassaemia

E Sickle cell disease

Explanation
Sofia comes from Cyprus where thalassaemia is common. The main differentials here were between iron
deficiency anaemia and a haemoglobinopathy. The clue towards haemoglobinopathy was
the mention of no
previous blood tests in the past. The FBC and film point towards thalassaemia. There is anaemia and evidence of
haemolysis
which would not be seen in an iron deficiency picture. Also, the ferritin is within normal range which
completely excludes iron deficiency.
44952

https://mypastest.pastest.com/Secure/TestMe/Browser/436619[‫ م‬12:33:32 10/12/1437]


MyPastest

End Session

Tag Question

Feedback

Difficulty: Average

Peer Responses

Session Progress

Responses Correct: 0

Responses Incorrect: 41

Responses Total: 41

Responses - % Correct: 0%

Blog
About Pastest
Contact Us
Help

© Pastest 2016

Next Question

Previous Question

https://mypastest.pastest.com/Secure/TestMe/Browser/436619[‫ م‬12:33:32 10/12/1437]


MyPastest

Prefer to use the old MyPastest? Access it here »

Back to Filters

Question 38 of 63

Sally, a 16-year-old, presented with a 5 week history of generally feeling tired and out of energy. She had seen her
GP on two previous occasions with a sore throat for which she was treated with antibiotics. She now has noticed a
swelling on the left side of her neck. The swelling has been there for a couple of weeks now. It has become bigger
but it has never been painful. She has lost some weight and she has had night sweats on a few occasions in the past
month. She has a previous history of mild asthma on inhalers. She lives with her parents and drinks and smokes
socially. She said she has experienced some pain in her back and her legs after having a few drinks with her
friends for her birthday.

On examination, she is pale with a 2x2


mass on her left supra-clavicular area which was not tender and fixed.
There was no other lymphadenopathy and no hepatosplenomegaly. She was pyrexial at 38 degrees. She generally
appeared unwell.

A full blood count revealed: Hb 85 g/dl, MCV 70 fl, WBC 9 x 109/l, Neutrophils 6 x 109/l, Platelets 820 x 109/l,
Reticulocytes 85 x 109/l (normal range 50-100), ESR 60mm/hour (normal 0-15mm/hour). Blood film: increased
number of platelets but no abnormal cells.

What is the most likely diagnosis?

A Acute lymphoblastic leukaemia

B EBV

C Hodgkin's disease

D Juvenile Idiopathic arthritis

E SLE

Explanation
Non-Hodgkin's disease is a general term used to describe all lymphomas except Hodgkin's lymphoma. It usually
presents with weight loss, fatigue, coughing, night sweats, swollen lymph nodes, difficulty to swallow and

https://mypastest.pastest.com/Secure/TestMe/Browser/436619[‫ م‬12:33:53 10/12/1437]


MyPastest

sometimes difficulty to breath (in case of mediastinal mass).


End Session
Bone pains, after alcohol consumption, is a rare but characteristic manifestation of Hodgkin's disease.

With
EBV, you would expect the presence of atypical mononuclear cells on the
blood film. The increase of the
platelets is commonly found in malignancy and may also be a manifestation of iron deficiency or
myeloproliferative disorders.

The raised ESR is a non specific finding of infection or malignancy.


46035

Tag Question

Feedback

Difficulty: Average

Peer Responses

Session Progress

Responses Correct: 0

Responses Incorrect: 42

Responses Total: 42

Responses - % Correct: 0%

Blog
About Pastest
Contact Us
Help Next Question
© Pastest 2016

Previous Question

https://mypastest.pastest.com/Secure/TestMe/Browser/436619[‫ م‬12:33:53 10/12/1437]


MyPastest

Prefer to use the old MyPastest? Access it here »

Back to Filters

Question 39 of 63

A 5-year-old boy is on treatment for ALL. He is in the maintenance phase of his treatment and he has a normal
neutrophil count.
You see him in clinic every week to monitor his treatment. He presents in clinic and his mum
says that he has "tingling" in his mouth and it hurts him when he eats crisps or other salty food. You examine him
and you find multiple small red lesions in his buccal mucosa and on the roof
of his mouth. There is no white
coating of his tongue. Nothing on his lips or on his face and the rest of his body. His physical examination is
otherwise normal. His observations are all within normal range and he
is apyrexial.

What would be your management plan?

A Do nothing and monitor until next week

B Fungal swab and ring them with results if treatment is required

C Fungal swab and start on PO fluconazole

D Viral swab from the lesions and blood cultures and admit to the ward for IV aciclovir

E Viral swab from the lesions and send home with PO aciclovir

Explanation
Children undergoing treatment for malignancy are susceptible to infections. The significant, potentially life-
threatening, episodes are those associated with febrile neutropenia.

In this case, the most likely cause is oral herpetic infection in a child who is systemically well and with normal
neutrophils. Therefore, the management plan would be to take a swab to confirm, and discharge home with PO
aciclovir and advice to present to the ward if he were to become pyrexial.

Should the scenario have described the case of a neutropenic child, the answer would have been to admit for IV
aciclovir.
46036

https://mypastest.pastest.com/Secure/TestMe/Browser/436619[‫ م‬12:34:14 10/12/1437]


MyPastest

End Session

Tag Question

Feedback

Difficulty: Average

Peer Responses

Session Progress

Responses Correct: 0

Responses Incorrect: 43

Responses Total: 43

Responses - % Correct: 0%

Blog
About Pastest
Contact Us
Help

© Pastest 2016

Next Question

Previous Question

https://mypastest.pastest.com/Secure/TestMe/Browser/436619[‫ م‬12:34:14 10/12/1437]


MyPastest

Prefer to use the old MyPastest? Access it here »

Back to Filters

Question 40 of 63

A 6-month-old girl was referred to the haematology unit for investigation of pancytopenia. She was previously fit
and well. She has two older sisters who are fit and well.

Examination reveals a pale child who is active and alert. Her chest is clear and she has heptosplenomegaly.

Full blood count revealed: Hb 7 g/dl, WCC 2 x 109/l, Neutrophils 0.4 x 109/l, platelets 50 z 109/l, reticulocyte
count 20 x 109/l (normal range 20-100 x 109/l).
While her bone marrow results were pending, she developed a
temperature of 39 degrees and extensive bruising. Her coagulation screen showed: PT 20 seconds, APTT 50
seconds, Fibrinogen 0.8 g/dl LFTs:
bilirubin 20micromol/l, albumin 25 g/l, ALT 200 iu/l.

What is the most likely explanation of her bruising and clotting findings?

A Acute lymphoblastic leukaemia

B DIC

C Hepatic failure

D Juvenile myelomonocytic leukaemia

E Meningococcal septicaemia in an immunocompromised child

Explanation
Most likely this bruising and clotting abnormality is caused by DIC from consumption of clotting factors. The DIC
is most likely to have been precipitated by an acute infective illness.

Next Question
Management should be supportive with replacement of coagulation factors with FFP, cryoprecipitate and platelet
transfusion as indicated. A septic screen should be performed and cover with broad spectrum antibiotics.
46037

https://mypastest.pastest.com/Secure/TestMe/Browser/436619[‫ م‬12:34:36 10/12/1437]


MyPastest

End Session
Tag Question

Feedback

Difficulty: Average

Peer Responses

Session Progress

Responses Correct: 0

Responses Incorrect: 44

Responses Total: 44

Responses - % Correct: 0%

Blog
About Pastest
Contact Us
Help

© Pastest 2016

Previous Question

https://mypastest.pastest.com/Secure/TestMe/Browser/436619[‫ م‬12:34:36 10/12/1437]


MyPastest

Prefer to use the old MyPastest? Access it here »

Back to Filters

Question 41 of 63

A 14-year-old boy presents to his GP with a testicular swelling. He has no other symptoms.

On examination, the GP finds a unilateral left testicular swelling. The


young man also looked mildly pale. He
refers him to the hospital and sends a full blood count which reveals: Hb 8 g/dl, WCC 20 x 106(/l, neutrophils 3.5
x 109/l, Platelets 110 x 109/l.

What is the most likely diagnosis?

A Acute lymphoblastic leukaemia

B Hodgkin's lymphoma

C Non-Hodkin's lymphoma

D Seminoma

E Testicular carcinoma

Explanation
Testicular disease is a rare presentation of acute leukaemia in boys. 10-30% of boys have testicular involvement at
diagnosis. Other potential areas of disease outside the bone marrow are CNS, rarely the eye and the kidney.

Boys are currently being treated for 3 years in total for ALL as opposed to girls who are treated
for a year less. In
post-treatment male patients, testes should always be examined for potential relapse.
46038

Next Question

Previous Question Tag Question

https://mypastest.pastest.com/Secure/TestMe/Browser/436619[‫ م‬12:34:55 10/12/1437]


MyPastest

Feedback End Session

Difficulty: Average

Peer Responses

Session Progress

Responses Correct: 0

Responses Incorrect: 45

Responses Total: 45

Responses - % Correct: 0%

Blog
About Pastest
Contact Us
Help

© Pastest 2016

https://mypastest.pastest.com/Secure/TestMe/Browser/436619[‫ م‬12:34:55 10/12/1437]


MyPastest

Prefer to use the old MyPastest? Access it here »

Back to Filters

Question 42 of 63

A 5-year-old boy is being treated for Ewing's sarcoma. He is admitted for chemotherapy and the nurse asks you to
see him because he is drowsy and his mum is worried. He finished the infusion 12 hours ago and has stayed in for
hyperhydration and sickness management. The nurse thinks that he is drowsy as a side effect of one of the anti-
emetics he is on.

Which one of the following medications should be adjusted or crossed off?

A Chlorpromazine

B Dexamethasone

C Levomepromazine

D Metoclopramide

E Ondansetron

Explanation
From all the drugs listed that are anti-emetics (chlorpromazine is an anti-psychotic drug), the only one that can
induce
drowsiness is levomepromazine. Levomepromazine is a phenothiazine neuroleptic drug which can cause
lethargy and drowsiness.

Metoclopramide is used for nausea as it helps with gastric emptying.

Ondansetron is a serotonin 5-HT3 receptor antagonist.

Dexamethasone can be used as an antiemetic. Next Question


46039

https://mypastest.pastest.com/Secure/TestMe/Browser/436619[‫ م‬12:35:16 10/12/1437]


MyPastest

End Session
Tag Question

Feedback

Difficulty: Average

Peer Responses

Session Progress

Responses Correct: 0

Responses Incorrect: 46

Responses Total: 46

Responses - % Correct: 0%

Blog
About Pastest
Contact Us
Help

© Pastest 2016

Previous Question

https://mypastest.pastest.com/Secure/TestMe/Browser/436619[‫ م‬12:35:16 10/12/1437]


MyPastest

Prefer to use the old MyPastest? Access it here »

Back to Filters

Question 43 of 63

A 15-year-old boy has had problems with easy bruising, epistaxis since childhood. He was referred by the dental
team for a bleeding episode that lasted 30 hours after a dental extraction.

His results are as follows: APTT 36 seconds, PT 15 seconds, Fibrinogen


2.5 g/l, Factor VIIIC 0.4 iu/ml (normal
range 0.5-1.5), vWF antigen 0.35 iu/ml (normal range 0.45-1.4), vWFristocetin co-factor 0.05 iu/ml (normal 0.45-
1.35), Platelets 230 x 10^9, Bleeding time 12 mins (normally up to 8 minutes)

What is the diagnosis?

A Haemophilia A

B Haemophilia B

C von Willebrand disease type I

D von Willebrand disease type II

E von Willebrand disease type III

Explanation
The diagnosis is that of von Willebrand disease (vWD) given the life long history of bleeding tendency. The
ristocetin co-factor activity is very much less than the factor VIIIC and vWF antigen concentration, which is
consistent with type II vWD.

In the treatment of vWD, it is best to avoid the use of blood products. Type II can be hard to manage and may
require recombinant factor if there is not enough response to DDAVP.
Next Question 46040

https://mypastest.pastest.com/Secure/TestMe/Browser/436619[‫ م‬12:35:35 10/12/1437]


MyPastest

End
Tag Session
Question

Feedback

Difficulty: Average

Peer Responses

Session Progress

Responses Correct: 0

Responses Incorrect: 47

Responses Total: 47

Responses - % Correct: 0%

Blog
About Pastest
Contact Us
Help

© Pastest 2016

Previous Question

https://mypastest.pastest.com/Secure/TestMe/Browser/436619[‫ م‬12:35:35 10/12/1437]


MyPastest

Prefer to use the old MyPastest? Access it here »

Back to Filters

Question 44 of 63

A 12-year-old boy from West Africa presented with a single large lymph node in the left side of his neck. The
mass was noted three weeks ago and it has gone up in size a lot.

On examination, there
is no other lymphadenopathy or hepatosplenomegaly. The results of his work up are as
follows: Hb 13.4 d/dl, WCC 8.7 x 109/l (normal differential) Platelets 172 x 109/l Lymph node biopsy:
replacement with large lymphoid cells with foamy cytoplasm containing numerous vacuoles.

What is the most likely diagnosis?

A Acute lymphoblastic leukaemia

B Burkitt's lymphoma

C Cutaneous T-cell lymphoma

D Malaria

E Non-Hodgkin's lymphoma

Explanation
This is a typical presentation and morphological findings of a patient with Burkitt lymphoma. The disease tends to
be multi-focal at diagnosis. Staging investigations include CT chest and abdomen. A bone marrow aspirate, which
typically does not show infiltration. Antibody to Epstein-Barr Virus (EBV) is usually positive.
46041

Next Question

Previous Question Tag Question

https://mypastest.pastest.com/Secure/TestMe/Browser/436619[‫ م‬12:35:53 10/12/1437]


MyPastest

Feedback End Session

Difficulty: Average

Peer Responses

Session Progress

Responses Correct: 0

Responses Incorrect: 48

Responses Total: 48

Responses - % Correct: 0%

Blog
About Pastest
Contact Us
Help

© Pastest 2016

https://mypastest.pastest.com/Secure/TestMe/Browser/436619[‫ م‬12:35:53 10/12/1437]


MyPastest

Prefer to use the old MyPastest? Access it here »

Back to Filters

Question 45 of 63

Ben, a 9-year-old boy is referred from the GP with a 2-week history of a swelling on the right side of his face. The
swelling was first noted while on holiday and it has been there since. Mum also noted
that his eye on that side
seems to be poking out. He has not been having any headaches or problems with her vision. There has been no
pain
anywhere and no weight-loss or fatigue. He has had no pyrexial or coryzal illness preceeding this.
Examination reveals a firm swelling of about 4cm on the right side of his face right under his eye. The swelling
seems to be spreading onto his nose and when examined with the mouth open, the swelling is seen coming through
the top of his palate.The swelling is not painful or tender to palpation and it not mobile or erythematous. There is
no rash. The rest of the physical examination is normal, apart from mild pallor.

What is the test that would be most diagnostic?

A Biopsy of the lesion

B Bone marrow aspirate

C Full body imaging

D LDH

E MRI head

Explanation
The history is consistent with a new tumour. The most likely type would be a rhabdomyosarcoma and in reality
the first thing that would happen would be MRI of the head and face and baseline bloods. However, what would
give the diagnosis, would be a biopsy of the
lesion.

Rhabdomyosarcoma (RMS) is the most commonly occurring soft-tissue sarcoma in children. When the disease is
localised, overall 5-year survival is more than 80% with the combination of surgery, radiation and chemotherapy.
46042

https://mypastest.pastest.com/Secure/TestMe/Browser/436619#Top[‫ م‬12:36:44 10/12/1437]


MyPastest

Tag Question

Feedback

Difficulty: Average

Peer Responses
Next Question
Previous Question

End Session

Session Progress

Responses Correct: 0

Responses Incorrect: 50

Responses Total: 50

Responses - % Correct: 0%

Blog
About Pastest
Contact Us
Help

© Pastest 2016

https://mypastest.pastest.com/Secure/TestMe/Browser/436619#Top[‫ م‬12:36:44 10/12/1437]


MyPastest

Prefer to use the old MyPastest? Access it here »

Back to Filters

Question 46 of 63

A 14-year-old boy with known sickle cell disease presents to the emergency department in the early hours. He is
short of breath and pyrexial. He says he has been coughing a lot and he has severe chest pain and pain on his left
lower leg. His observations are: Respiratory rate 30 bpm, Heart rate 120 bpm, BP 143/78, Sat O2 86% in room air,
Temperature 38.4 degrees.

What is the most life saving intervention?

A Analgesia

B IV antibiotics

C High flow oxygen

D Packed red cell transfusion

E 2L/m2 IV fluids

Explanation
Sickle cell disease is a haemoglobinopathy in which there is substitution of a single amino acid, valine, for
glutamic
acid in the β chain which leads to an abnormal haemoglobin molecule which is less soluble. In the event
of hypoxia, the haemoglobin
polymerises and causes the red cells to change in shape and become sickle cells. This
leads to vaso-occlusion and may cause infarction which can be fatal. It also causes haemolysis which leads to
anaemia.

This patient is suffering from "chest syndrome" with chest pain, fever, cough and evidence of lung infiltrates.

The
mainstay of treatment is oxygen support, (ventilation if required), hydration, antibiotics and analgesia. A
blood transfusion might help as the patient is severely anaemic and has low oxygen saturation. However, in these
polytransfused patients, blood matching can be time consuming and difficult as he may have developed
alloantibodies.
46043

https://mypastest.pastest.com/Secure/TestMe/Browser/436619#Top[‫ م‬12:37:05 10/12/1437]


MyPastest

Tag Question

Feedback

Difficulty: Average

Peer Responses
Next Question
Previous Question

End Session

Session Progress

Responses Correct: 0

Responses Incorrect: 50

Responses Total: 50

Responses - % Correct: 0%

Blog
About Pastest
Contact Us
Help

© Pastest 2016

https://mypastest.pastest.com/Secure/TestMe/Browser/436619#Top[‫ م‬12:37:05 10/12/1437]


MyPastest

Prefer to use the old MyPastest? Access it here »

Back to Filters

Question 47 of 63

A 3-year-old presents with one month history of having constant infections and generally not being his normal
self. His mum says he has had cough after cough and been given 2 courses of antibiotics by the GP which didn't
seem to change anything. He remains grumpy and not himself. He has been bruising very easily. He has also been
complaining of pain in his back. On further questioning, he has not travelled abroad or had any contacts from
abroad. He has been fully immunised. He was previously fit and well. He lives at home with both parents and his
5-year-old brother who has been well.

What test would you do first?

A Blood culture and CRP

B Full blood count and blood film

C Full blood count and clotting

D Rheumatoid factor

E Skeletal survey

Explanation
The most important diagnosis to exclude at this age group would be that of an acute leukaemia. ALL is the most
common malignancy in children with high incidence in this age group. Patients tend to present with a history of
having been non-specifically unwell for a few weeks. Some of them have a history of multiple infections,
bleeding, bruising and most patients are pale. All these symptoms stem from the fact that the bone marrow is taken
over by blast cells and therefore normal haemopoiesis is affected.

The first test to be considered would be a full blood count and a blood film. You would expect to find an increased
white cell count with low haemoglobin and low or normal platelets. On the film, blasts would be seen to confirm
malignancy. However, the patient would have to be transferred to a regional paediatric oncology unit for further
testing to confirm the diagnosis and the type of leukaemia.

https://mypastest.pastest.com/Secure/TestMe/Browser/436619#Top[‫ م‬12:37:25 10/12/1437]


MyPastest

46045

Tag Question

Feedback

Difficulty: Average

Next Question
Peer Responses

Previous Question

End Session

Session Progress

Responses Correct: 0

Responses Incorrect: 51

Responses Total: 51

Responses - % Correct: 0%

Blog
About Pastest
Contact Us
Help

© Pastest 2016

https://mypastest.pastest.com/Secure/TestMe/Browser/436619#Top[‫ م‬12:37:25 10/12/1437]


MyPastest

Prefer to use the old MyPastest? Access it here »

Back to Filters

Question 48 of 63

Matthew, a 2-year-old boy, presents to the paediatric assessment unit with a 3 week history of being generally
unwell. The mother is very worried and has been to the GP four times in this time, and having been sent home
diagnosed with a viral illness.

Matthew has been coryzal and generally quiet and “clingy”, according to his mum. He seems to have pain when
he is picked up and when he has his nappy changed. He is not as mobile as he used to be. Mum
thinks he looks
pale. She was worried today as in the past few days he has been getting a lot of bruises.

He has a low grade pyrexia and mild tachycardia. Examination reveals a pale looking child with bruises of
multiple ages. He avoids walking around. He is crying a lot when examined. Chest is clear and heart sounds are
normal. No obvious joint swelling noted.

What is the most likely explanation for his symptoms?

A Acute leukaemia

B Juvenile idiopathic arthritis

C Meningococcal septicaemia

D Non accidental injury

E Neuroblastoma

Explanation
The most important diagnosis to exclude at this age group would be that of an acute leukaemia. ALL is the most
common malignancy in children, with high incidence in this age group. Patients tend to present with a history of
having been non-specifically unwell for a few weeks. Some of them have a history of multiple infections,
bleeding, bruising and most patients are pale. All these symptoms stem from the fact that the bone marrow is taken
over by blast cells and therefore normal haemopoiesis is affected.

https://mypastest.pastest.com/Secure/TestMe/Browser/436619#Top[‫ م‬12:37:46 10/12/1437]


MyPastest

The presence of bone pain in any child should alarm you and prompt you to ask for a full blood count in the first
instance as this could be the only presenting symptom of a haematological malignancy. Non-verbal children are
harder to assess but clues of bone pain can be given by crying at nappy changing, crying when picked up, and
reduced mobility.

All the listed differentials above need to be excluded and in reality a lot of tests would happen at the same time.
However, in this context, leukaemia
is the most likely explanation of all the above. The test for this would be a full
blood count and a blood film. You would expect to find an increased white cell count with low haemoglobin and
low or normal platelets. On the film, blasts would be seen to confirm malignancy. However, the patient would
have to be transferred to a regional paediatric oncology unit for further testing to confirm the diagnosis and the
type of leukaemia.
46133

Next Question
Previous Question

Tag Question
End Session
Feedback

Difficulty: Average

Peer Responses

Session Progress

Responses Correct: 0

Responses Incorrect: 52

Responses Total: 52

Responses - % Correct: 0%

Blog
About Pastest
Contact Us
Help

© Pastest 2016

https://mypastest.pastest.com/Secure/TestMe/Browser/436619#Top[‫ م‬12:37:46 10/12/1437]


MyPastest

https://mypastest.pastest.com/Secure/TestMe/Browser/436619#Top[‫ م‬12:37:46 10/12/1437]


MyPastest

Prefer to use the old MyPastest? Access it here »

Back to Filters

Question 49 of 63

A 17-year-old boy was taken to the GP by his mother as she was worried that he was losing weight and had
diarrhoea. The young man described that he had to wear his 15-year-old brother's trousers as his were too loose.
He said that he had been feeling unwell for the past few months but has never been ill before. There is no relevant
family history and everybody else at home has been fine. There has been no change in his behaviour. He eats at
home, regular meals cooked by his mother. There has been no blood or mucus in the stools, nor has there been any
vomiting or loss of appetite.

Physical examination revealed conjunctival and palmar pallor and obvious weight loss. His abdomen was soft and
non tender with no hepatosplenomegaly. No lymphadenopathy is noted.

These are the results of his initial work up:

Hb 10 g/dl
MCV 105 fl
MCH 31 pg/cell
MCHC 33 g/dl
WBC 3.6 x 109/l
Platelets 125 x 109/l
Reticulocyte count 30 x 109/l (normal range 20-100 x 109/l)
Blood film: oval macrocytes and poikilocytosis with less than normal platelets
Bilirubin 22 micromol/l, LDH 3000 Iμ/l (normal range 230-450 Iμ/l)
Red cell folate 75ng/l of packed red cells (normal range 150-1000micrograms/l)

What is the most likely cause of this young man's symptoms and blood results?

A Acute lymphoblastic leukaemia

B Coeliac disease

C Dietary deficiency

https://mypastest.pastest.com/Secure/TestMe/Browser/436619#Top[‫ م‬12:38:06 10/12/1437]


MyPastest

D Langerhan's histiocytocis

E Non-Hodgkin's lymphoma

Explanation
A presentation of weight loss can be hard to estimate with no previous records of weights. However, always look
out for cues about clothing and maternal concerns. Here, the use of his younger brother's clothes implies a rather
significant weight loss. Change in bowel habits is also a clue. There is no blood or mucus, hence
moving away
from an inflammatory bowel disease type of picture. The absence of organomegaly and lymphadenopathy should
Next Question
guide you away from a
haematological malignancy diagnosis.

His blood work Previous


up is in line with a megaloblastic anaemia and that in context with the weight loss point towards a
Question
malabsorption or nutritional deficiency type of picture. He is eating normally, hence a malabsorption process is
more likely which is why coeliac is the answer. Macrocytic anaemia with pancytopenia suggest malabsorption.
The most likely cause of this is folate or vitamin B12 deficiency. The raise in End
the bilirubin
Sessionand the LDH
can be
caused by prematurely destroyed erythrocytes.

Remember,
that if the mother has folate deficiency during pregnancy then the fetus is at increased risk of spinal
defects, as spina bifida.
46134

Tag Question

Feedback

Difficulty: Average

Peer Responses

Session Progress

Responses Correct: 0

https://mypastest.pastest.com/Secure/TestMe/Browser/436619#Top[‫ م‬12:38:06 10/12/1437]


MyPastest

Responses Incorrect: 53

Responses Total: 53

Responses - % Correct: 0%

Blog
About Pastest
Contact Us
Help

© Pastest 2016

https://mypastest.pastest.com/Secure/TestMe/Browser/436619#Top[‫ م‬12:38:06 10/12/1437]


MyPastest

Prefer to use the old MyPastest? Access it here »

Back to Filters

Question 50 of 63

Adam, a 15-year-old, is referred through the GP for jaundice and fatigue occurring over the last couple of weeks.
He is also complaining of an intermittent pain in his epigastrium. He has not had any weight loss. There has been
no diarrhoea or vomiting. His appetite is normal. A detailed past medical history reveals that his fatigue has
been
ongoing all his life, with him missing out on sports at regular intervals at school. His mum said he had 2 episodes
of "hepatitis" at ages 6 and 8 years. There is no family history of jaundice. He has only travelled within Europe
and has not been away in the last 6 months.

His physical examination reveals jaundice of the sclera and a palpable


spleen tip at 3cm with some tenderness of
the right upper quadrant. He is mildly tachycardic with normal blood pressure and apyrexial.

You perform an FBC, bilirubin, DCT and an abdominal USS. The results are as follows:

Hb 12.5 g/dl
MCV 103 fl
MCHC 37 g/dl
WCC 10.5 x 109
Reticulocytes 150 x 109 (normal range 20-100 x 109/l)
Bilirubin 35 micromol/l, LDH 650 Iμ/l (normal range 230-450 Iμ/l)
Direct Coomb’s test: negative
Abdominal USS: enlarged spleen measuring 16 cm.

What is the most likely diagnosis?

A Acquired autoimmune haemolitic anaemia (AIHA)

B Acute hepatitis

C Cholocystitis

D Gilbert's syndrome

https://mypastest.pastest.com/Secure/TestMe/Browser/436619#Top[‫ م‬12:38:28 10/12/1437]


MyPastest

E Hereditary spherocytosis

Explanation
Jaundice and abdominal pain should make you think of
hepatitis or cholecystitis in the first instance. The patient is
apyrexial which is against acute cholecystitis and also the past medical
history reveals a chronic type of fatigue
and two previous episodes of "hepatitis". It is unlikely that he has contracted infective hepatitis twice and now a
third time! The commonest cause of hepatitis in childhood is hepatitis A and he has not travelled in endemic areas
and also hepatitis A is normally a mild, self-limiting disease. The chronicity of his fatigue and the blood results
lead towards a haematological diagnosis. His Hb is low with high MCV and high reticulocytes. Chronic
Next Question
haemolytic anaemia is the most likely cause and the negative Coombs excluded AIHA. The answer is hereditary
spherocytosis which is a disorder that makes the cytoskeleton of red cells more fragile and therefore leads to red
cell death and splenomegaly. Gallstones are a result of this red cell destruction and increased haem metabolism.
Previous Question 46135

End Session

Tag Question

Feedback

Difficulty: Average

Peer Responses

Session Progress

Responses Correct: 0

Responses Incorrect: 54

Responses Total: 54

Responses - % Correct: 0%

https://mypastest.pastest.com/Secure/TestMe/Browser/436619#Top[‫ م‬12:38:28 10/12/1437]


MyPastest

Blog
About Pastest
Contact Us
Help

© Pastest 2016

https://mypastest.pastest.com/Secure/TestMe/Browser/436619#Top[‫ م‬12:38:28 10/12/1437]


MyPastest

Prefer to use the old MyPastest? Access it here »

Back to Filters

Question 51 of 63

A 7-year-old boy from Somalia presents to the emergency department with his mother who does not speak
English. He is really struggling to breathe and is distressed. His heart rate is 180 bpm, his BP is 130/90 mmHg, his
respiratory rate is 37/min, his temperature 38 oC and his Sat O2
83% in room air. He is holding his chest in distress
and crying and he has marked recessions and tracheal tug. His mum is crying and is speaking in her language but
there is no interpreter available.

You
put the boy on high flow oxygen and get a CXR and some baseline bloods.
CXR shows bilateral pulmonary
infiltrates. FBC: Hb 6.6 g/dl, MCV 85 fl, MCHC 36.0 g/dl, WBC 19.6 x 109, Neutrophils 15.3 x 109, Platelets 350
x 109, CRP 50.

His mum pulls out some tablets from her bag that she has given her son
since he was little every day and you see
that it is folic acid, hydroxyurea and Penicilin V.

What is the most likely diagnosis?

A Malaria with supra added chest infection

B HIV with chest infection

C Septic shock

D Sickle cell chest crisis

E Thalassaemia with chest infection

Explanation
Sickle cell disease is a haemoglobinopathy in which there is substitution of a single amino acid, valine for
glutamic acid in the β chain which leads to an abnormal haemoglobin molecule
which is less soluble. In the event
of hypoxia, the haemoglobin polymerises and causes the red cells to change in shape and become sickle cells. This
leads to vaso-occlusion and may cause infarction which can be fatal. It also causes haemolysis which leads to
anaemia.

https://mypastest.pastest.com/Secure/TestMe/Browser/436619#Top[‫ م‬12:38:48 10/12/1437]


MyPastest

This patient is suffering from "chest syndrome" with chest pain, fever, cough and evidence of lung infiltrates. The
mainstay of treatment
is oxygen support, (ventilation if required), hydration, antibiotics and analgesia. A blood
transfusion might help as the patient is severely anaemic and has low oxygen saturation. However, in these
polytransfused patients, blood matching can be time consuming and difficult as he may have developed
alloantibodies. The hint towards haemoglobinopathy in this patient were the medications that he is on, you would
not expect a patient with any of the above differential to be on all three at the same time and long term.
46136

Next Question Tag Question

Previous Question
Feedback

Difficulty: Average
End Session
Peer Responses

Session Progress

Responses Correct: 0

Responses Incorrect: 55

Responses Total: 55

Responses - % Correct: 0%

Blog
About Pastest
Contact Us
Help

© Pastest 2016

https://mypastest.pastest.com/Secure/TestMe/Browser/436619#Top[‫ م‬12:38:48 10/12/1437]


MyPastest

Prefer to use the old MyPastest? Access it here »

Back to Filters

Question 52 of 63

Eve, a 15-year-old girl, presented to her GP with her mother.


She has noticed multiple bruises on her arms and
legs in the past 4-5 days. The bruises seem to be increasing every day, with no apparent reason. They are not
painful. Eve has had no runny nose, cough or temperature. She is normally fit and on no regular medications. On
examination, the only positive finding were multiple bruises varying in size and age all over her limbs. No joint
swellings. No hepatosplenomegaly or lymphadenopathy. She generally appeared really well, otherwise.

The result of her initial work up revealed:

Full blood count:

Hb 13 g/dl
WCC 6 x 109/l
Neut 3.5 x 109/l
Platelets 22 x 109/l
Neutrophils 4 x 109/l
Lymphocytes 1.5 x 109/l

Coagulation screen normal.


Blood film: reduced number of platelets.

Following these results, Eve was referred to the haematology unit.

What is the first test that the haematologist will run?

A Abdominal USS

B Autoimmune studies

C Bone marrow sampling

D Repeat FBC

https://mypastest.pastest.com/Secure/TestMe/Browser/436619#Top[‫ م‬12:39:08 10/12/1437]


MyPastest

E U+Es and LFTs

Explanation
In a patient with spontaneous bruising, the potential causes are threefold. Either low platelets, or reduced function
of the platelets or vasculitis type of picture, with inflammation of the blood vessel wall causing leakage of red
cells into the skin.

The low platelet count is the only positive finding in this case. The Hb and WCC count are normal, making bone
marrow failure unlikely. The coagulation screen is normal.

Her bone marrow biopsy


revealed a raised number of megakaryocytes which suggest that the platelet production is
Next Question
normal. The platelets must, therefore, be reduced
secondary to premature destruction in her circulation. There is
no splenomegaly so the platelets are not decreased because of pooling in an
enlarged spleen.
Previous Question
The most likely diagnosis is that of ITP. ITP is caused by destruction of platelets through an auto-antibody binding
on to a protein on the membrane of the platelets. If the bone marrow production of platelets is at a slower rate than
the destruction of platelets through phagocytosis, then the patient becomes symptomatic and
the platelet count can
End Session
drop to single digits.
46137

Tag Question

Feedback

Difficulty: Average

Peer Responses

Session Progress

Responses Correct: 0

Responses Incorrect: 56

Responses Total: 56

https://mypastest.pastest.com/Secure/TestMe/Browser/436619#Top[‫ م‬12:39:08 10/12/1437]


MyPastest

Responses - % Correct: 0%

Blog
About Pastest
Contact Us
Help

© Pastest 2016

https://mypastest.pastest.com/Secure/TestMe/Browser/436619#Top[‫ م‬12:39:08 10/12/1437]


MyPastest

Prefer to use the old MyPastest? Access it here »

Back to Filters

Question 53 of 63

You are on call for neonates and you are asked to see the newborn baby of a fit and healthy mother. The baby was
born through normal vaginal delivery, assisted with forceps and the midwife has called you because she has
noticed a large swelling on the baby’s head. The physical examination reveals an extensive haematoma on the
little boy’s scalp. The rest of the examination is normal. You take a history from the mother and she says that as
far as she knows there was no family history of bleeding problems on either side.

A full blood count was performed on the baby and it showed:

Hb 12 g/dl
MCV 120 fl
Platelets 440 x 109.

Clotting studies: PT 13.5 sec, APTT 80 sec, Thrombin Time 15 sec Fibrinogen 3 g/l
Factor VIII is 0% and factor IX is 60%.

What is the most likely diagnosis?

A Mild haemophilia A

B Mild haemophilia B

C Severe haemophilia A

D Severe haemophilia B

E Vitamin K deficiency

Explanation
The APTT is prolonged and the PT is normal which points towards an abnormality in the intrinsic pathway of the

https://mypastest.pastest.com/Secure/TestMe/Browser/436619#Top[‫ م‬12:39:28 10/12/1437]


MyPastest

clotting cascade. This is most likely due to haemophilia as lack of either factor
VII or IX would lead to a
prolonged APTT and a normal PT.

This is not vitamin K deficiency because the PT is normal.

A family history if very important in cases of bleeding problems. Remember that the site of bleeding can give you
clues for the underlying
disorder, bleeding in the muscles and joints are mainly caused by coagulation factor
deficiencies. Bleeding due to low platelets causes bruises and mucosal bleeding. Excessive menstrual bleeding
may be the only sign of a bleeding disorder.

Classification of Haemophilia

Severity Factor Level


Next Question
Mild >5% to 35% of normal

Moderate Previous Question


1% to 5% of normal

Severe <1% of normal


End Session 46138

Tag Question

Feedback

Difficulty: Average

Peer Responses

Session Progress

Responses Correct: 0

Responses Incorrect: 57

Responses Total: 57

Responses - % Correct: 0%

https://mypastest.pastest.com/Secure/TestMe/Browser/436619#Top[‫ م‬12:39:28 10/12/1437]


MyPastest

Blog
About Pastest
Contact Us
Help

© Pastest 2016

https://mypastest.pastest.com/Secure/TestMe/Browser/436619#Top[‫ م‬12:39:28 10/12/1437]


MyPastest

Prefer to use the old MyPastest? Access it here »

Back to Filters

Question 54 of 63

A 13-year-old boy with sickle cell disease presents with visual problems. The problems started the day before and
he finds it hard to focus his eyes. He has no headache and he is generally pain free. No other neurological signs at
all.

Looking at his notes, he has had a few episodes of painful crises, none of which required PICU
admission or any
respiratory support other than supplemental oxygen via
face mask. He has never suffered a stroke and he has not
had a splenectomy.

What’s the most likely cause of his eye symptoms?

A Complication of HbSC disease

B Complication of HbSS disease

C Complication of sickle thalassaemia

D Painful sickle crisis

E Stroke

Explanation
The question states clearly that there are no neurological signs or pain present. Therefore, options d and e are
immediately excluded.

With regards to subtypes of sickle cell disease, individuals with HbSC disease are susceptible to more severe
retinal disease and are therefore monitored closely. HbSC tends to run a
milder course than HbSS disease. These
patients retain some splenic function until their second decade and this makes them susceptible to splenic
sequestration events even later in childhood.

HbSS disease is caused by mutation in both of the beta globin genes which results in replacement of glutamic acid
with valine at position 6 of the
globin chain. This results in an abnormal haemoglobin, haemoglobin S (HbS),

https://mypastest.pastest.com/Secure/TestMe/Browser/436619#Top[‫ م‬12:39:49 10/12/1437]


MyPastest

which polymerises in its de-oxygenated state and causes a change in the shape of the red cell. The red cell takes an
abnormal elongated shape, which is what gives the name to the disease. These abnormal red cells don’t flow
normally in the circulation and cause increased viscosity and slugging which in turn can cause obstruction of
blood flow
in smaller vessels. This vaso-occlusive phenomenon is the basis of the complications of the disease,
including painful crises and stroke. There
is also ongoing haemolysis which causes a chronic anaemia picture.
46139

Tag Question
Next Question
Feedback

Previous Question
Difficulty: Average

Peer Responses End Session

Session Progress

Responses Correct: 0

Responses Incorrect: 58

Responses Total: 58

Responses - % Correct: 0%

Blog
About Pastest
Contact Us
Help

© Pastest 2016

https://mypastest.pastest.com/Secure/TestMe/Browser/436619#Top[‫ م‬12:39:49 10/12/1437]


MyPastest

Prefer to use the old MyPastest? Access it here »

Back to Filters

Question 55 of 63

Next Question
You are asked to review a two- day old full term baby boy who
started bleeding excessively following
circumcision. You request a full
blood count and clotting. The full blood count is normal and the APTT is
prolonged at 95 seconds. PT is normal.

What is the most likely diagnosis?

A Factor V Leiden disease

B Haemophilia

C ITP

D Severe von Willebrand’s disease

E Vitamin K deficiency

Explanation
Haemophilia A (factor VIII) and haemophilia B (factor IX) are the two most common congenital factor
deficiencies that cause prolongation of the APTT. These are both x-linked recessive and hence affect males. When
they present at the neonatal period they are most likely the severe form (factor levels <1%).

The FBC is normal which excludes ITP. PT is normal which excludes vitamin K deficiency. Von Willebrand’s
disease, even severe, is unlikely to
cause isolated prolongation of the APTT.

Factor V Leiden disease is a thrombophilia, which leads to hyper-coagulopathy not bleeding.


46140

https://mypastest.pastest.com/Secure/TestMe/Browser/436619#Top[‫ م‬12:40:08 10/12/1437]


MyPastest

Tag Question

Feedback

Difficulty: Average

Peer Responses

Previous Question

End Session
Session Progress

Responses Correct: 0

Responses Incorrect: 59

Responses Total: 59

Responses - % Correct: 0%

Blog
About Pastest
Contact Us
Help

© Pastest 2016

https://mypastest.pastest.com/Secure/TestMe/Browser/436619#Top[‫ م‬12:40:08 10/12/1437]


MyPastest

Prefer to use the old MyPastest? Access it here »

Back to Filters

Question 56 of 63

A term baby girl presents with bruising and blood in the nappy. She is otherwise really well with no signs of
infection. She is the first baby of healthy parents and there is no risk of perinatal infections.

Her FBC reveals normal haemoglobin and white cell count and very low platelets of 4. Her clotting studies are
normal. Her blood film reveals low platelet numbers with no morphological abnormalities.

Maternal platelets are normal and have been normal throughout pregnancy.

There is no family history of blood disorders.

What is the most likely cause of her bleeding?

A Bernard- Soulier syndrome

B Maternal autoimmune disease

C Neonatal allo-immune thrombocytopenia (NAIT)

D Neonatal autoimmune thrombocytopenia

E Rhesus haemolytic disease

Explanation
NAIT results from immune-mediated platelet destruction caused by alloantibodies produced by the mother and
transferred through the placenta to the baby’s circulation. These alloantibodies are against a certain foetal platelet
antigen that the baby inherited from the father and that is absent in the mother. This is
the platelet equivalent of
Rhesus haemolytic disease of the new born. NAIT can occur in the first pregnancy in 50% of cases.

In neonatal autoimmune thrombocytopenia, the mother has an underlying autoimmune disease or immune
thrombasthenia, which results in IgG antibodies passively transferred to the foetus who may then develop
thrombocytopenia. The antibodies are targeting platelet antigens that are present in both maternal and foetal
circulation. Therefore, in neonatal autoimmune thrombocytopenia, the mother’s platelet count is low as well. This

https://mypastest.pastest.com/Secure/TestMe/Browser/436619#Top[‫ م‬12:40:28 10/12/1437]


MyPastest

is one of the differentiating factors from NAIT, where maternal platelet count is normal. Hence answers B and D
are eliminated. The blood film shown no morphological abnormalities which excludes answer A.

Rhesus haemolytic disease is one of the causes


of neonatal thrombocytopenia. It is more commonly seen in the
second pregnancy with the first-borns being unaffected. Also, the rest of the FBC in this case is normal, without
any evidence of haemolysis.
46141

Tag Question
Next Question
Feedback
Previous Question
Difficulty: Average

End Session
Peer Responses

Session Progress

Responses Correct: 0

Responses Incorrect: 60

Responses Total: 60

Responses - % Correct: 0%

Blog
About Pastest
Contact Us
Help

© Pastest 2016

https://mypastest.pastest.com/Secure/TestMe/Browser/436619#Top[‫ م‬12:40:28 10/12/1437]


MyPastest

Prefer to use the old MyPastest? Access it here »

Back to Filters

Question 57 of 63

You are asked to see a 5-year-old with a history of headaches


every day for the last 3 weeks or so. The headache
always happens in the morning and he has missed a lot of school. His mum is worried because he has started
complaining of seeing double in the last few days. He also has been vomiting. The past few days she has noted he
is a
bit off balance sometimes. There has been no recent illnesses. No fever, no diarrhoea and no other pain
anywhere else. He has not travelled recently. There has been no rash or other skin changes. No seizures.

Your examination reveals no weakness or sensation abnormalities. His pupils are equal and reactive. His cranial
nerves are
intact. There is mild ataxia. No rash or meningism. The rest of his physical examination is normal.

What is the most likely diagnosis?

A Acute cerebellar ataxia

B Functional headache

C Infratentorial brain tumour

D Supratentorial brain tumour

E Viral meningitis

Explanation
Morning headache should always alarm you towards brain tumours in the exam and in clinical practice in general.
In this case there are neurological findings as well.

Brain tumours can present with persisting vomiting, morning headaches, diplopia and other visual disturbances,
endocrine problems, ataxia, head tilt. In younger children the symptoms can be very non-specific with
developmental delay,
loss of appetite or irritability.

The location of the tumour determines the symptomatology. Supratentorial tumours mainly present with headache,
sensory and motor impairment +/- seizures +/= change in behaviour/school problems. Infratentorial present with

https://mypastest.pastest.com/Secure/TestMe/Browser/436619#Top[‫ م‬12:40:48 10/12/1437]


MyPastest

headache, vomiting, diplopia and ataxia.

The peak age group for childhood brain tumours is 3-7 years.

There is no pyrexia or no proceeding illness which points away viral meningitis and acute cerebellar ataxia, which
tends to be post-infectious.

Functional headache is a diagnosis of exclusion and in this case there is multitude of neurological findings.
46142

Next Question Tag Question

Previous Question
Feedback

Difficulty: Average
End Session
Peer Responses

Session Progress

Responses Correct: 0

Responses Incorrect: 61

Responses Total: 61

Responses - % Correct: 0%

Blog
About Pastest
Contact Us
Help

© Pastest 2016

https://mypastest.pastest.com/Secure/TestMe/Browser/436619#Top[‫ م‬12:40:48 10/12/1437]


MyPastest

Prefer to use the old MyPastest? Access it here »

Back to Filters

Question 58 of 63

Next Question
Previous
A 13-year-old girl Question
presents with a short history of headache and weakness of her right leg. Her parents are really
worried as they feel that she doesn't act like her normal self and she is very snappy and aggressive at times.

You perform an MRI head which reveals a fronto-parietal tumour.

What is the most likely diagnosis?

A Craniopharyngioma

B Ependymoma

C High grade astrocytoma

D Low grade glioma

E Medulloblastoma

Explanation
The symptom duration is very brief here and the tumour is in the fronto-parietal region. These are typical
characteristics of a high grade astrocytoma. These account for 10% of childhood tumours. They can recur.

The current 5 year survival is 35-40% and the treatment consists of resection and post-operative radiotherapy and
chemotherapy. The location of the tumour excludes ependymoma and medulloblastoma. The acute onset excludes
low grade glioma. Craniopharyngiomas arise from the suprasellar region near the anterior pituitary gland.
46143

Tag Question

https://mypastest.pastest.com/Secure/TestMe/Browser/436619#Top[‫ م‬12:41:08 10/12/1437]


MyPastest

Feedback

Difficulty: Average

Peer Responses

Session Progress End Session


Responses Correct: 0

Responses Incorrect: 62

Responses Total: 62

Responses - % Correct: 0%

Blog
About Pastest
Contact Us
Help

© Pastest 2016

https://mypastest.pastest.com/Secure/TestMe/Browser/436619#Top[‫ م‬12:41:08 10/12/1437]


MyPastest

Prefer to use the old MyPastest? Access it here »

Back to Filters

Question 59 of 63

Next Question
A 6-year-old girl presents with vomiting, headaches and double vision in the last 4 weeks. Her parents say that her
vomiting has
become worse. She vomits 3-4 times a day. No diarrhoea and no abdominal
pain. Also they have
noticed that she is unsteady on her feet. She gets
her headaches in the morning. They are really concerned.

You request an MRI head which reveals a contrast enhancing tumour filling the 4th ventricle with evidence of
obstructive hydrocephalus.

What is the most likely diagnosis?

A Craniopharyngioma

B Diffuse intrinsic brain stem glioma

C Ependymoma

D Low grade astrocytoma

E Medulloblastoma

Explanation
Medulloblastomas are cerebellar primitive neuro-ectodermal tumours. They are the most common childhood brain
tumours. Medullobastomas are always located in the cerebellum. They are fast growing tumours. They are
unlikely to spread outside the brain and spinal cord. Overall survival is more than 50%. Standard treatment
comprises of surgical resection, and postoperative cranio-spinal irradiation. The aim of treatment is complete
resection and children benefit from adjuvant chemotherapy. They can recur. The symptomatology and location of
the tumour excludes all other options. Astrocytomas could be located anywhere, however the sudden onset of
symptoms excludes
a low grade nature. 46144

https://mypastest.pastest.com/Secure/TestMe/Browser/436619#Top[‫ م‬12:41:26 10/12/1437]


MyPastest

Tag Question

Feedback

Difficulty: Average

Peer Responses

Previous Question

End Session

Session Progress

Responses Correct: 0

Responses Incorrect: 63

Responses Total: 63

Responses - % Correct: 0%

Blog
About Pastest
Contact Us
Help

© Pastest 2016

https://mypastest.pastest.com/Secure/TestMe/Browser/436619#Top[‫ م‬12:41:26 10/12/1437]


MyPastest

Prefer to use the old MyPastest? Access it here »

Back to Filters

Question 60 of 63

Next Question
A 5-year-old girl presents with headache, vomiting and with unilateral 4th nerve palsy. The symptoms started
abruptly 3 weeks ago with the cranial neuropathy presenting in the last few days. On high suspicion of a brain
tumour, you perform an MRI head which reveals a tumour filling the 4th ventricle and also extending into the
upper cervical spinal cord with obstructive hydrocephalus.

What is the most likely diagnosis?

A Craniopharyngioma

B Ependymoma

C Low-grade astrocytoma

D Low grade glioma

E Medulloblastoma

Explanation
This is most likely an ependymoma which is a type
of glial tumour that is derived from the ependymal cells. In 30-
40% of cases, the tumour can extend through the foramen magnum into the upper cervical cord. After full
resection there is 5 year survival in excess of 60%. If the tumour has infiltrate the brain stem and there is
involvement of cranial nerve nuclei, the chance of successful resection is limited and therefore the prognosis is
worse. If the resection is incomplete, the tumour will recur despite radiation and chemotherapy.

The location of this tumour and its spread inferiorly excludes all other options. Astrocytomas could be located
anywhere, however the sudden onset of symptoms excludes a low grade nature.
46145

https://mypastest.pastest.com/Secure/TestMe/Browser/436619#Top[‫ م‬12:41:46 10/12/1437]


MyPastest

Tag Question

Feedback

Difficulty: Average

Peer Responses

Previous Question

End Session
Session Progress

Responses Correct: 0

Responses Incorrect: 64

Responses Total: 64

Responses - % Correct: 0%

Blog
About Pastest
Contact Us
Help

© Pastest 2016

https://mypastest.pastest.com/Secure/TestMe/Browser/436619#Top[‫ م‬12:41:46 10/12/1437]


MyPastest

Prefer to use the old MyPastest? Access it here »

Back to Filters

Question 61 of 63

Next Question
A 9-year-old boy presents with impaired vision, headaches and
has been recently diagnosed with diabetes
insipidus. He has been complaining of trouble with his eyes and headaches for a few weeks but his mum thought
he was trying to avoid his homework and going to school as he has problems with bullying. The development of
insensible thirst really alarmed his mum and lead to further investigations. You are awaiting for the cranial MRI
and you are thinking of possible causes for
these symptoms.

What would you most likely expect to see on the MRI?

A Cerebellar astrocytoma

B Craniopharyngioma

C Ependymoma

D Low grade glioma

E Medulloblastoma

Explanation
Craniopharyngiomas arise from the suprasellar region from remnants of embryonic tissue located in Rathke’s
pouch, which forms the anterior pituitary gland. The triad of headache, visual disturbances along with hormonal
imbalances is typical of a craniopharyngioma. Symptoms of raised intracranial pressure may also be seen. They
are benign tumours however they are hard to remove due to their location.

The symptoms are typical of a craniopharymgioma.


The only other differentials would be optic chiasm gliomas or
suprasellargerminomas which are not listed as options.
46146

https://mypastest.pastest.com/Secure/TestMe/Browser/436619#Top[‫ م‬12:42:06 10/12/1437]


MyPastest

Tag Question

Feedback

Difficulty: Average

Peer Responses

Previous Question

End Session
Session Progress

Responses Correct: 0

Responses Incorrect: 65

Responses Total: 65

Responses - % Correct: 0%

Blog
About Pastest
Contact Us
Help

© Pastest 2016

https://mypastest.pastest.com/Secure/TestMe/Browser/436619#Top[‫ م‬12:42:06 10/12/1437]


MyPastest

Prefer to use the old MyPastest? Access it here »

Back to Filters

Question 62 of 63

A 2-year-old boy presents with vomiting, loss of appetite, weight loss and abdominal distension. His mum thinks
there is a little lump on the right side of his belly. He has been pale in the last few weeks as well. There is no
history of fevers or foreign travel. He is fully immunised and lives at home with his parents and his little sister.
Nothing of significance in the family history.

Examination reveals a miserable, pale boy with evidence of wasting on this limbs and a dilated abdomen. You can
feel a round mass measuring about 2x2 cms on the edge of his liver. It is non tender and feels hard.
There is no
ascites. No jaundice and no lymphadenopathy.

You perform some initial investigations. There is low haemoglobin, normal white cell count and raised platelets.
The a-fetoprotein (AFP) is very elevated. Clotting is normal. Urine catecholamine normal. An abdominal USS has
revealed a solid lesion arising from the hepatic parenchyma.

What is the most likely diagnosis?

A Focal benign hamartoma

B Haemangioma

C Hepatoblastoma

D Hepatocellular carcinoma

E Neuroblastoma

Explanation
Liver tumours can be benign or malignant. The liver is the third most common place of intra-abdominal
malignancy in children with adrenal neuroblastomas and Wilms tumours being the most common. Hepatoblastoma
and hepatocellular carcinoma are the most common;
they represent the 75% of all liver tumours. Benign liver
tumours can be haemangiomas, hamartomas and focal nodular hyperplasia.

https://mypastest.pastest.com/Secure/TestMe/Browser/436619#Top[‫ م‬12:42:25 10/12/1437]


MyPastest

Hepatoblastomas are the most common liver tumours in children. They most
commonly present in children under
3 years with abdominal distension, anorexia, fevers, weight loss, failure to thrive, and anaemia. There is AFP
elevation in 90% of the cases. They are commonly associated with thrombocytosis. Hepatocellular carcinomas on
the other hand, are most commonly associated with polycythaemia. Jaundice occurs in about a quarter of
hepatocellular carcinomas and they can also lead to acute abdomen picture secondary to rupture. Acute abdomen
and jaundice are rare for hepatoblastomas.

In this case the mass is solid which


excludes the latter two options and the urine catecholamine is normal which
points away from the neuroblastoma diagnosis. Also the tumour arises from the liver which is not true for
neuroblastomas which are neuroendocrine tumours.
46147

Next Question
Previous Question
Tag Question

Feedback End Session

Difficulty: Average

Peer Responses

Session Progress

Responses Correct: 0

Responses Incorrect: 66

Responses Total: 66

Responses - % Correct: 0%

Blog
About Pastest
Contact Us
Help

© Pastest 2016

https://mypastest.pastest.com/Secure/TestMe/Browser/436619#Top[‫ م‬12:42:25 10/12/1437]


MyPastest

https://mypastest.pastest.com/Secure/TestMe/Browser/436619#Top[‫ م‬12:42:25 10/12/1437]


MyPastest

Prefer to use the old MyPastest? Access it here »

Back to Filters

Question 63 of 63

Randal, a 3-year-old boy is referred to you by the GP who has


noted a mass on the left side of his abdomen. The
mass was initially noted by his mum when she was bathing him. Randal has been his normal self with no other
symptoms at all. He has not complained of any pain, is passing urine and opening his bowels as usual. He is eating
as normal.

Examination revels a large mass on the right side of his abdomen. No other findings.

You suspect Wilms tumour.

Which of the following, is NOT a congenital anomaly associated with Wilms tumour?

A Aniridia

B Cryptorchidism

C Hemihypertrophy

D Hypospadias

E Spinal abnormalities

Explanation
Wilms tumour typically presents as an asymptomatic abdominal mass.

It is associated with hemihypertrophy (Beckwith-Wiedemann syndrome), aniridia, cryptorchidism and


hypospadias (WAGR).

The syndromes associated with Wilms include:

Beckwith-Wiedemann syndrome: hemihypertrophy, macroglossia, omphalocele, visceromegaly, mental


retardation.

https://mypastest.pastest.com/Secure/TestMe/Browser/436619#Top[‫ م‬12:42:47 10/12/1437]


MyPastest

WAGR syndrome: Wilms tumour, aniridia, genitourinary malformations and mental retardation.
Denys-Drash syndrome: organomegaly, macrosomia, nephropathy
Sotos’ syndrome: large at birth with excessive growth in childhood, developmental delay, dysmorphism.
Perlman syndrome: macrosomia, macrocephaly, organomegaly, characteristic facies.
Simpson-Golabi-Behmel
syndrome: x-linked, overgrowth, coarse facies, heart defects, visceromegaly,
diaphragmatic hernia and gastrointestinal malformations.

46148

End Session

Previous Question Tag Question

Feedback

Difficulty: Average

Peer Responses

Session Progress

Responses Correct: 0

Responses Incorrect: 67

Responses Total: 67

Responses - % Correct: 0%

Blog
About Pastest
Contact Us
Help

© Pastest 2016

https://mypastest.pastest.com/Secure/TestMe/Browser/436619#Top[‫ م‬12:42:47 10/12/1437]


MyPastest

https://mypastest.pastest.com/Secure/TestMe/Browser/436619#Top[‫ م‬12:42:47 10/12/1437]


MyPastest

Prefer to use the old MyPastest? Access it here »

Back to Filters

Question 1 of 22

In order to ensure meticulous care in hygiene, which measure is NOT acceptable when preparing bottle feeds?

A Using boiled water

B Allowing the liquid to cool

C Placing in the refrigerator

D Using sterilised bottles

E Warming the milk twice

Explanation
Babies are at risk of infection due to their relatively poor immunity and by warming the milk twice, you are
allowing
bacteria a chance to culture in the milk. 11505

Next Question
Tag Question

Feedback
Previous Question
Difficulty: Easy
End Session
Peer Responses

https://mypastest.pastest.com/Secure/TestMe/Browser/436619[‫ ص‬10:10:46 10/12/1437]


MyPastest

Session Progress

Responses Correct: 0

Responses Incorrect: 1

Responses Total: 1

Responses - % Correct: 0%

Blog
About Pastest
Contact Us
Help

© Pastest 2016

https://mypastest.pastest.com/Secure/TestMe/Browser/436619[‫ ص‬10:10:46 10/12/1437]


MyPastest

Prefer to use the old MyPastest? Access it here »

Back to Filters

Question 2 of 22

Which of the following is NOT a contra-indication to MMR vaccination:

A Children receiving high-dose cortico-steroids

B Children with an anaphylactic allergy to egg

C Children with an allergy to neomycin

D Children with an acute febrile illness

E Children who have received an immunoglobulin injection within the last 3 months

Explanation
All of the above are contra-indications as written in the BNF except
egg allergy - this is a common misconception.
In children with a significant history of an anaphylactic reaction to eggs, or who have had
egg allergy and chronic
severe asthma, the vaccination should be administered in hospital. The vaccine is not however contraindicated in
such patients.

Contraindications include: acute fever, recent immunoglobulin administration, administration of another live
vaccine in
the past 3 weeks, immunosuppression, allergy to neomycin or kanamycin, pregnancy. Steroids, such as
Next Question
prednisolone, given to children either orally or rectally at a daily dose of 2mg/kg/day for at least one week or
1mg/kg/day for one month should be deemed as a contraindication and administration on live vaccines should be
postponed for at least 3 months after immunosuppressive treatment has stopped.
11677
Previous Question

Tag Question

https://mypastest.pastest.com/Secure/TestMe/Browser/436619[‫ ص‬10:11:10 10/12/1437]


MyPastest

Feedback

Difficulty: Average

Peer Responses

Session Progress

Responses Correct: 0

Responses Incorrect: 2

Responses Total: 2

Responses - % Correct: 0%

Blog
About Pastest
Contact Us
Help

© Pastest 2016

End Session

https://mypastest.pastest.com/Secure/TestMe/Browser/436619[‫ ص‬10:11:10 10/12/1437]


MyPastest

Prefer to use the old MyPastest? Access it here »

Back to Filters

Question 3 of 22

Which vaccination is given by the intradermal route usually?

A Inactivated Polio

B Meningococcal C

C MMR

D DTP

E BCG

Explanation
BCG is the only vaccine given intradermally. The rest are given by the intramuscular or deep subcutaneous route.
11678

Next Question Tag Question

Feedback

Previous Question Difficulty: Easy

End Session
Peer Responses

https://mypastest.pastest.com/Secure/TestMe/Browser/436619[‫ ص‬10:11:29 10/12/1437]


MyPastest

Session Progress

Responses Correct: 0

Responses Incorrect: 3

Responses Total: 3

Responses - % Correct: 0%

Blog
About Pastest
Contact Us
Help

© Pastest 2016

https://mypastest.pastest.com/Secure/TestMe/Browser/436619[‫ ص‬10:11:29 10/12/1437]


MyPastest

Prefer to use the old MyPastest? Access it here »

Back to Filters

Question 4 of 22

Which vaccination is NOT included in the 2, 3, 4 month ages of a child according to the UK immunisation
schedule?

A Polio

B Hib

C DTP

D Meningococcal C

E BCG

Explanation
BCG is usually given at birth in high risk neonates or at ages 10-14. Universal BCG immunisation of teenagers
has recently been removed from the immunisation schedule, with a targeted strategy now recommended to
immunise groups at increased risk in infancy, at birth or adulthood. 11680

Next Question

Tag Question
Previous Question
Feedback
End Session
Difficulty: Easy

Peer Responses

https://mypastest.pastest.com/Secure/TestMe/Browser/436619[‫ ص‬10:11:48 10/12/1437]


MyPastest

Session Progress

Responses Correct: 0

Responses Incorrect: 4

Responses Total: 4

Responses - % Correct: 0%

Blog
About Pastest
Contact Us
Help

© Pastest 2016

https://mypastest.pastest.com/Secure/TestMe/Browser/436619[‫ ص‬10:11:48 10/12/1437]


MyPastest

Prefer to use the old MyPastest? Access it here »

Back to Filters

Question 5 of 22

Regarding a squint in a child, which statement is correct?

A The child does not require further investigation if it has been present for over 6 months

B May be regarded as within acceptable limits if transient and only lasting a few seconds up to the age of
18 months

C Is more frequent in children who have not been vaccinated

D Is more frequent in children with myopia than hypermetropia

E In a younger child is more likely to be divergent rather than convergent

Explanation
It is important to refer children with squints over the age of 6 months for further investigation and prevent
permanent blindness in that eye. Transient squints of only a few seconds duration may be acceptable up to 18
months of age. Hypermetropia is a common cause of squints in children. Convergent squints are more common. 11681

Next Question

Tag Question
Previous Question
Feedback
End Session
Difficulty: Average

Peer Responses

https://mypastest.pastest.com/Secure/TestMe/Browser/436619[‫ ص‬10:12:06 10/12/1437]


MyPastest

Session Progress

Responses Correct: 0

Responses Incorrect: 5

Responses Total: 5

Responses - % Correct: 0%

Blog
About Pastest
Contact Us
Help

© Pastest 2016

https://mypastest.pastest.com/Secure/TestMe/Browser/436619[‫ ص‬10:12:06 10/12/1437]


MyPastest

Prefer to use the old MyPastest? Access it here »

Back to Filters

Question 6 of 22

Consider the options presented below.

Which screening does NOT occur in the United Kingdom?

A Heel Prick Test to screen for congenital hypothyroidism

B Red Reflex to screen for congenital cataract

C Echocardiogram in children at 6 weeks to screen for / diagnose cardiac structural disease

D Otoacoustic emission to screen for hearing impairment

E Heel prick test to screen for phenylketonuria

Explanation
Auscultation is used to pick up murmurs in children,
not routine echocardiograms at 6 weeks. The Guthrie test
(heel prick test/blood spot screening test) in the UK screens for phenylketonuria, congenital hypothyroidism,
sickle cell disorders, cystic fibrosis and Medium Chain Acyl CoA Dehydrogenase Deficiency (MCADD). 11682

Next Question

Tag Question
Previous Question
Feedback
End Session

Difficulty: Easy

https://mypastest.pastest.com/Secure/TestMe/Browser/436619[‫ ص‬10:12:25 10/12/1437]


MyPastest

Peer Responses

Session Progress

Responses Correct: 0

Responses Incorrect: 6

Responses Total: 6

Responses - % Correct: 0%

Blog
About Pastest
Contact Us
Help

© Pastest 2016

https://mypastest.pastest.com/Secure/TestMe/Browser/436619[‫ ص‬10:12:25 10/12/1437]


MyPastest

Prefer to use the old MyPastest? Access it here »

Back to Filters

Question 7 of 22

A Primary Prevention
B Secondary Prevention
C Tertiary Prevention
D Health Promotion
E Health Education
F Child Health Surveillance
G Public health

The definitions below are best described by which of the options above?

Each option may be used once, more than once, or not at all.

Scenario 1

Reduction in number of new cases of a disease, disorder or condition (accidents)

Your answer was incorrect

Select one...

A - Primary Prevention

Scenario 2

Any activity which promotes health through learning

Your answer was incorrect

Select one...

E - Health Education

Scenario 3

Reduction of impairment and disability, and minimising suffering (e.g. multidisciplinary approach of a child with
Down syndrome)

https://mypastest.pastest.com/Secure/TestMe/Browser/436619[‫ ص‬10:12:46 10/12/1437]


MyPastest

Your answer was incorrect

Select one...

C - Tertiary Prevention

It is important to consider the child in society and the importance of looking at child health in the community as a
whole. Primary prevention is reducing number of cases of disease, disorder, conditions or accident from the very
onset. Secondary prevention is reduction of prevalence of disease by shortening duration or diminishing the
impact through early detection and prompt intervention (e.g. screening). Tertiary prevention is the reduction of
impairment and disability in a child with a pre-existing illness or condition. Health promotion is any measure that
improves health or prevents disease. Public health is a science preventing disease, prolonging life and promoting
health through the organised efforts of society.

Explanation
It
is important to consider the child in society and the importance of looking at child health in the community as a
whole. Secondary prevention is reduction of prevalence of disease by shortening duration or diminishing the
impact through early detection and prompt intervention (eg screening). Health promotion is any measure that
improves health or prevents disease. Public health is a science preventing disease, prolonging life and promoting
health through the organised efforts of society.
11683

Tag Question

Feedback

Difficulty: Average

Session Progress

Responses Correct: 0

Responses Incorrect: 9

Responses Total:
Next Question 9

Responses - % Correct: 0%

Previous Question
End Session
Blog
About Pastest
Contact Us
Help

© Pastest 2016

https://mypastest.pastest.com/Secure/TestMe/Browser/436619[‫ ص‬10:12:46 10/12/1437]


MyPastest

https://mypastest.pastest.com/Secure/TestMe/Browser/436619[‫ ص‬10:12:46 10/12/1437]


MyPastest

Prefer to use the old MyPastest? Access it here »

Back to Filters

Question 8 of 22

A Polio
B Diptheria
C Tetanus
D Pertussis
E Measles
F Mumps
G Rubella
H Haemophilus influenzae type B

For
each of the clinical presentations listed below, choose the most likely
option from the list above. Each option
may be used once, more than once, or not at all.

Scenario 1

A child develops fever and parotid gland enlargement

Your answer was incorrect

Select one...

F - Mumps

Scenario 2

A child has prolonged URTI with paroxysms of coughing, followed by a whoop with vomiting. Diagnosis is
confirmed by a
nasopharyngeal culture.

Your answer was incorrect

Select one...

D - Pertussis

Scenario 3

A child has a throat infection with pharyngeal exudates. There is membrane formation and upper airway

https://mypastest.pastest.com/Secure/TestMe/Browser/436619[‫ ص‬10:13:07 10/12/1437]


MyPastest

obstruction.

Your answer was incorrect

Select one...

B - Diptheria

Scenario 4

A child develops mild fever and is then left with residual paralysis.

Your answer was incorrect

Select one...

A - Polio

Scenario 5

A child has a maculopapular rash, fever, coryza, cough and conjunctivitis

Your answer was incorrect

Select one...

E - Measles

These conditions are not common due to the widespread uptake of immunisations in the UK, however it is
important to
recognise the symptoms of these serious diseases especially if a child has not been vaccinated against
them.

Explanation
It is important to recognise the symptoms of these serious diseases especially if a child has not been vaccinated
against them.
11684

Next Question

Tag Question
Previous Question
Feedback
End Session

Difficulty: Average

https://mypastest.pastest.com/Secure/TestMe/Browser/436619[‫ ص‬10:13:07 10/12/1437]


MyPastest

Session Progress

Responses Correct: 0

Responses Incorrect: 14

Responses Total: 14

Responses - % Correct: 0%

Blog
About Pastest
Contact Us
Help

© Pastest 2016

https://mypastest.pastest.com/Secure/TestMe/Browser/436619[‫ ص‬10:13:07 10/12/1437]


MyPastest

Prefer to use the old MyPastest? Access it here »

Back to Filters

Question 9 of 22

Theme: Immunisation

A No live vaccines
B No vaccines at all
C Normal immunisation schedule
D Normal schedule with separate pertussis vaccine
E Normal schedule without BCG (Bacilel Camille–Guérin)
F Normal schedule and unconjugated pneumococcal vaccination
G Normal schedule without polio vaccine
H Normal schedule with substitution of single vaccines for MMR (measles, mumps, rubella)
I Normal schedule without MMR
J Normal schedule without MMR or BCG

For
each of the following children, choose the most appropriate immunisation policy from the above list. Each
item may be used once, more than once or not at all.

Exam question theme from June 2015

Scenario 1

A child born in the UK with vertically acquired human immunodeficiency virus (HIV) infection.

Your answer was incorrect

Select one...

E - Normal schedule without BCG (Bacilel Camille–Guérin)

Measles vaccine has been reported to cause fatal disease in one severely immunosuppressed HIV positive adult.
Some experts recommend measles vaccine is not given to HIV positive children with severe immunosuppression
(i.e. CD4 <15%).

No harmful effects have been reported following live attenuated vaccines for mumps or rubella in HIV positive
children.

Scenario 2

https://mypastest.pastest.com/Secure/TestMe/Browser/436619[‫ ص‬10:13:28 10/12/1437]


MyPastest

A child diagnosed on the autistic spectrum.

Your answer was incorrect

Select one...

C - Normal immunisation schedule D

Autism is not a contraindication to any immunisation.

Scenario 3

A child who has sickle cell trait.

Your answer was incorrect

Select one...

C - Normal immunisation schedule D

Children with sickle cell trait should


receive the universal schedule. Those with sickle cell disease should receive
the unconjugated vaccine at age 2-3 years, in addition to the conjugated vaccine as part of the universal schedule
(see http://www.immunisation.nhs.uk/Immunisation_Schedule).
Note that BCG is NOT part of the UK’s universal schedule but may be offered in areas of high TB incidence.
22017

Tag Question

Feedback

Difficulty: Average

Session Progress
Next Question
Responses Correct: 0

Responses Incorrect: 17

Responses Total: Previous Question 17

Responses - % Correct: End Session 0%

https://mypastest.pastest.com/Secure/TestMe/Browser/436619[‫ ص‬10:13:28 10/12/1437]


MyPastest

Blog
About Pastest
Contact Us
Help

© Pastest 2016

https://mypastest.pastest.com/Secure/TestMe/Browser/436619[‫ ص‬10:13:28 10/12/1437]


MyPastest

Prefer to use the old MyPastest? Access it here »

Back to Filters

Question 10 of 22

Theme: Immunisations

A Acellular pertussis vaccine

B Conjugate pneumococcal vaccine

C Hepatitis A vaccine

D Hepatitis B vaccine

E Influenza vaccine

F Palivizumab (anti-RSV [respiratory syncytial virus] immunoglobulin)

G Ribavirin

H Salk polio vaccine

I Single measles vaccine

J Unconjugated pneumococcal vaccine

For
the following situations, choose one vaccine from the list above that should be used instead of, or in addition
to, the routine scheduled vaccines. Each item may be used once, more than once or not at all.

Scenario 1

An ex-premature baby is discharged in October with home oxygen. He has had the first three sets of the universal
schedule (while an inpatient).

Your answer was incorrect

Select one...

F - Palivizumab (anti-RSV [respiratory syncytial virus] immunoglobulin)

Ex-pre-term infants with chronic lung disease are at a much higher risk from respiratory infections. RSV is a
major concern for babies with chronic lung disease, leading to high rates of hospital admission, PICU admission
and even mortality. The evidence for passive immunisation against RSV shows that it will not prevent infection or
hospital admission with RSV bronchiolitis (in babies with chronic lung disease). However, it seems to reduce the

https://mypastest.pastest.com/Secure/TestMe/Browser/436619[‫ ص‬10:13:50 10/12/1437]


MyPastest

severity of the illness, such that it reduces the need for PICU admission (NNT = 10 to prevent 1 PICU admission).
There is some debate about whether this is cost-effective. Most neonatal ICUs will arrange the immunisation of
babies with chronic lung disease.

Scenario 2

A 4-year-old whose sibling is receiving chemotherapy for AML.

Your answer was incorrect

Select one...

E - Influenza vaccine

A child receiving chemotherapy for AML


will be significantly immunosuppressed. If such a child has siblings, it
is important that they are immunised against influenza, which can be devastating to an immunocompromised
child. Previously a sibling would have received inactivated polio rather than live oral polio vaccine; however,
since the change to the universal schedule (where all polio is inactivated) this is no longer a consideration.

Scenario 3

A 3-year-old infant has a splenectomy following a road traffic accident.

Your answer was incorrect

Select one...

J - Unconjugated pneumococcal vaccine

This child may or may not have been part of the conjugated pneumococcal vaccine catch-up programme; however,
as she is over 2 years old, it is important that she receive the unconjugated vaccine. At this age she will be able to
make a good antibody response to the vaccine and the unconjugated vaccine protects against 30-40 pneumococcal
serotypes (compared with 13 serotypes for the
conjugated vaccine).
22413

Next Question Tag Question

Feedback

Previous Question
Difficulty: Average
End Session
Session Progress

Responses Correct: 0

https://mypastest.pastest.com/Secure/TestMe/Browser/436619[‫ ص‬10:13:50 10/12/1437]


MyPastest

Responses Incorrect: 20

Responses Total: 20

Responses - % Correct: 0%

Blog
About Pastest
Contact Us
Help

© Pastest 2016

https://mypastest.pastest.com/Secure/TestMe/Browser/436619[‫ ص‬10:13:50 10/12/1437]


MyPastest

Prefer to use the old MyPastest? Access it here »

Back to Filters

Question 11 of 22

A 1-year-old boy from eastern Europe emigrates to the UK. He is suspected of exposure to mumps. He is
otherwise well and thriving. His parents are unsure about his MMR status.

What is the next BEST step?

A Advise against the MMR immunisation

B Check mumps serology

C Vaccinate with the MMR immunisation

D Vaccinate with the single measles and rubella immunisations

E None of the above

Explanation
MMR should be offered to unimmunised individuals if there is contact with suspected measles, mumps or rubella.
If the individual is already incubating measles, mumps or rubella, MMR immunisation will not exacerbate the
symptoms. If there is doubt about an individual’s MMR status, MMR should be given. The diagnosis of measles,
mumps and rubella can be confirmed by non-invasive
means from saliva samples. Notification to the Health
Protection Unit (HPU) should be based on clinical suspicion and should not await lab confirmation.

Common events after MMR

Mumps component:

Malaise, fever and/or rash a week after immunisation (lasting 2–3/7)


Parotid swelling (1% of those aged <4 years) usually in third week.

Rubella component is likely to cause the following:

https://mypastest.pastest.com/Secure/TestMe/Browser/436619[‫ ص‬10:14:12 10/12/1437]


MyPastest

Idiopathic thrombocytopenia (ITP) within 6/52 – resolves spontaneously


Arthropathy D14-21 (arthralgia or arthritis) – rare.

There
is overwhelming evidence that MMR does not cause autism, and there is no evidence that it causes
inflammatory bowel disease
44017

Tag Question

Feedback

Difficulty: Average

Peer Responses

Session Progress

Responses Correct: 0

Responses Incorrect: 21

Responses Total: 21

Responses - % Correct: 0%

Next Question

Blog
About Pastest
Contact Us
Help
Previous Question
© Pastest 2016
End Session

https://mypastest.pastest.com/Secure/TestMe/Browser/436619[‫ ص‬10:14:12 10/12/1437]


MyPastest

Prefer to use the old MyPastest? Access it here »

  Logged in as Ahmed fouad

Back to Filters
Question 12 of 22

A 6-year-old boy is seen at his GP surgery with a rapidly spreading non-blanching rash, fever of Difficulty: Average
38°C, heart rate 170 beats/min, central refill time 4 seconds. He is given intramuscular
benzylpenicillin by his GP before being brought into hospital by an ambulance. Peer Responses
Which investigation is most likely to lead to a diagnosis?

A Blood culture for MC&S

B Lumbar puncture sending CSF off for microscopy, culture and sensitivity (MC&S)

C Meningococcal PCR (poymerase chain reaction)

D Pneumococcal urinary antigen Session Progress

Responses Correct: 0
E None of the above Responses Incorrect: 22

Responses Total: 22
Explanation
Responses - % Correct: 0%

The sensitivity of meningococcal PCR is not affected by prior antibiotic treatment. Meningococci
are Gram-negative diplococci. Neisseria meningitidis group B is the most common cause of
infection (85%) since introduction of the immunisation against group C in 1999. Since September
2015 the routine immunisation schedule in the UK has also included protection against group B
infection. It is not yet
clear how this will translate into numbers of prevented cases or lives saved.
Group C used to account for 40% of infection in the UK before vaccine introduction, but is now
responsible for <10% of cases.

Group A: 1–2% of cases in UK but is common in other parts of the world.


Peak ages are 0–5 and 15–19 years. Meningococcal carriage rate is 5–15%.

Transmission is by droplet spread, and the incubation period is 2–7 days. Symptoms vary from
fulminant disease to insidious with mild prodromal symptoms. Mortality rate is around 10% (higher
in septicaemia). Early on the rash may be non-specific, but later it may be
petechial or purpuric and
does not blanch with pressure. One in eight people who recover from meningococcal disease will
experience long-term effects – headaches, joint problems, epileptic fits, deafness and learning
difficulties. Antibiotics are recommended for close contacts of
the index case.

There is a NICE guideline on the management of bacterial meningitis and meningococcal


septicaemia (September 2010) – an evolving and rapidly progressing rash is a contraindication to
lumbar puncture
https://www.nice.org.uk/guidance/CG102/chapter/1-Guidance

Immunisation:

Vaccine against group B meningococcal introduced in September 2015 and given at 2


months, 4 months and between 12 and 13 months of age.
Meningococcal
plain polysaccharide A and C vaccine is effective against A and C. Young

https://mypastest.pastest.com/Secure/TestMe/Browser/436619[‫ ص‬10:14:33 10/12/1437]


MyPastest

infants respond less well than older children. Immunity lasts 3–5 years. Immunity to C group
is only transitory.
Meningococcal
conjugate group C vaccine is effective against group C. It is protective in
younger children. It was introduced as part of the routine
child immunisation programme in
autumn 1999. Schedule

See website below:

https://www.gov.uk/government/uploads/system/uploads/attachment_data/file/473570/9406_PHE_2015_Complete_Immunisation_Schedule_A4_21.pdf

nb
this information was correct at the beginning of 2016. Always check the
most up-to-date
immunisation schedule as these are regularly revised.
44018

Tag Question Feedback

Blog
About Pastest
Contact Us
Help

© Pastest 2016

Next Question

Previous Question
End Session

https://mypastest.pastest.com/Secure/TestMe/Browser/436619[‫ ص‬10:14:33 10/12/1437]


MyPastest

Prefer to use the old MyPastest? Access it here »

Back to Filters

Question 13 of 22

Which of the following statements is true regarding meningitis B vaccination in England?

A Meningococcal B vaccination was introduced routinely for children in England in 1992

B Meningococcal B vaccination was introduced routinely for children in England in 1999

C Meningococcal B vaccination was introduced routinely for children in England in 2004

D Meningococcal B vaccination was introduced routinely for children in England in 2012

E Meningococcal B vaccination was introduced routinely for children in England in 2015

Explanation
There are 12 capsular groups of Neisseria meningitidis, the most common in the UK being B, C, W and Y.

Type
B accounts for 70% of lab confirmed cases and more than 90% of cases in
children and adolescents. The UK
has the highest incidence of invasive meningococcal disease in Europe hence aggressive control is essential. The
meningitis C vaccination was introduced in 1999 in England.
45076

Next Question

Previous Question Tag Question

Feedback End Session

Difficulty: Average

https://mypastest.pastest.com/Secure/TestMe/Browser/436619[‫ ص‬10:14:54 10/12/1437]


MyPastest

Peer Responses

Session Progress

Responses Correct: 0

Responses Incorrect: 23

Responses Total: 23

Responses - % Correct: 0%

Blog
About Pastest
Contact Us
Help

© Pastest 2016

https://mypastest.pastest.com/Secure/TestMe/Browser/436619[‫ ص‬10:14:54 10/12/1437]


MyPastest

Prefer to use the old MyPastest? Access it here »


End Session

Back to Filters

Question 14 of 22

You are asked by a new mum about vaccination against meningitis B.

Which of the following statements is true regarding the schedule for the meningococcal B vaccination in England?

A The meningococcal B vaccination should offered to children aged 13 months and 13 years

B The meningococcal B vaccination should offered to children aged 13 months and 3 years

C The meningococcal B vaccination should offered to children aged 2 months, 13 months and 3 years

D The meningococcal B vaccination should offered to children aged 2 months, 4 months and 13 months

E The meningococcal B vaccination should offered to children aged 2 months, 4 months and 3 years

Explanation
Meningococcal B vaccination was introduced in on 1st
September 2015 in England. This is an important public
concern given the UK has the incidence of invasive meningococcal disease in the Europe. Public Health England
has recommended a dosing regime of 2 months, 4 months and 12-13 months. Children are not currently eligible
for the vaccination over the age of 2.

www.gov.uk
45077

Previous Question Next Question


Tag Question

Feedback

https://mypastest.pastest.com/Secure/TestMe/Browser/436619#Top[‫ ص‬10:16:05 10/12/1437]


MyPastest

Difficulty: Average

Peer Responses

Session Progress

Responses Correct: 0

Responses Incorrect: 24

Responses Total: 24

Responses - % Correct: 0%

Blog
About Pastest
Contact Us
Help

© Pastest 2016

https://mypastest.pastest.com/Secure/TestMe/Browser/436619#Top[‫ ص‬10:16:05 10/12/1437]


MyPastest

Prefer to use the old MyPastest? Access it here »

Back to Filters

Question 15 of 22

Some parents are concerned about giving their baby the rotavirus vaccination and would to know more about the
condition. They would like to know when the peak incidence of rotavirus occurs.

Which month does the incidence of rotavirus peak in the UK?

A January

B March

C May

D November

E September

Explanation
Rotavirus is the commonest cause of gastroenteritis in children accounting for 50% of cases. It is estimated
children will have had at least one episode of rotavirus infection by age 5, the peak occurring around March each
year.

A
significant number of children are admitted for supportive treatment due to dehydration. As little as 10-100 viral
particles may cause infection. Virus may be present in faeces before and after symptoms present. In addition to the
faeco-oral route, it may also be transmitted
through the respiratory route. The vaccine, which is live attenuated, is
given orally at 8 and 12 weeks of age.
45078
Next Question

Tag Question

https://mypastest.pastest.com/Secure/TestMe/Browser/436619#Top[‫ ص‬10:16:24 10/12/1437]


MyPastest

End Session
Feedback

Difficulty: Average

Peer Responses

Session Progress

Responses Correct: 0

Responses Incorrect: 25

Responses Total: 25

Responses - % Correct: 0%

Blog
About Pastest
Contact Us
Help

© Pastest 2016

Previous Question

https://mypastest.pastest.com/Secure/TestMe/Browser/436619#Top[‫ ص‬10:16:24 10/12/1437]


MyPastest

Prefer to use the old MyPastest? Access it here »

Back to Filters

Question 16 of 22

Which one of the following vaccinations are NOT offered to infants aged 2 months old in the UK?

A Haemophilus B

B Measles

C Meningococcal B

D Polio

E Rotavirus

Explanation
The following are offered to infants aged 2 months in the UK:

Diphtheria
Tetanus
Pertussis (whooping cough)
Polio
Haemophilus influenzae type B (Hib)
Meningococcal group b disease (MenB)
Pneumococcal disease
Rotavirus

Measles vaccination is given as part of the MMR at around 12 months.

www.gov.uk 45079

https://mypastest.pastest.com/Secure/TestMe/Browser/436619#Top[‫ ص‬10:16:44 10/12/1437]


MyPastest

End Session

Tag Question

Feedback

Difficulty: Average

Peer Responses

Session Progress

Responses Correct: 0

Responses Incorrect: 26

Responses Total: 26

Responses - % Correct: 0%

Blog
About Pastest
Contact Us
Help

© Pastest 2016

Previous Question Next Question

https://mypastest.pastest.com/Secure/TestMe/Browser/436619#Top[‫ ص‬10:16:44 10/12/1437]


MyPastest

Prefer to use the old MyPastest? Access it here »

Back to Filters

Question 17 of 22

You are counselling a couple regarding the UK immunisation schedule.

Which one of the following would you say is NOT offered at age 12-13 months?

A Haemophilus B

B Measles

C Meningococcal B

D Pneumococcal disease

E Polio

Explanation
Polio is offered at 2, 3 and 4 months. The following are currently offered to children 12-13 months in the UK:

Haemophilus influenzaze type b (Hib)


MenC
MenB
Measles, Mumps, Rubella
Pneumococcal disease

www.gov.uk
Next Question
45080

https://mypastest.pastest.com/Secure/TestMe/Browser/436619#Top[‫ ص‬10:17:04 10/12/1437]


MyPastest

End
Tag Session
Question

Feedback

Difficulty: Average

Peer Responses

Session Progress

Responses Correct: 0

Responses Incorrect: 27

Responses Total: 27

Responses - % Correct: 0%

Blog
About Pastest
Contact Us
Help

© Pastest 2016

Previous Question

https://mypastest.pastest.com/Secure/TestMe/Browser/436619#Top[‫ ص‬10:17:04 10/12/1437]


MyPastest

Prefer to use the old MyPastest? Access it here »


End Session

Back to Filters

Question 18 of 22

A nurse contacts you regarding a child with known egg allergy who is due for his measles, mumps and rubella
(MMR) vaccination.

Which of the following statements is true regarding the safety of giving MMR vaccination in this case?

A If the child has had a previous anaphylactic reaction to egg, the MMR should not be given

B Measles, Mumps and Rubella should be given as separate vaccines

C MMR is safe to give as the vaccine is grown on chick cells, not the white or yolk

D MMR may be given but a pre dose of antihistamine is required

E MMR should only be given if the previous reaction was mild

Explanation
The national advice for children with egg allergy is:

"The
MMR vaccine can safely be given to children who have had a severe allergy (anaphylactic reaction) to egg.
This is because MMR vaccine is grown on chick cells, not the egg white or yolk"

www.gov.uk 45081

Previous Question Next Question


Tag Question

Feedback

https://mypastest.pastest.com/Secure/TestMe/Browser/436619#Top[‫ ص‬10:17:23 10/12/1437]


MyPastest

Difficulty: Average

Peer Responses

Session Progress

Responses Correct: 0

Responses Incorrect: 28

Responses Total: 28

Responses - % Correct: 0%

Blog
About Pastest
Contact Us
Help

© Pastest 2016

https://mypastest.pastest.com/Secure/TestMe/Browser/436619#Top[‫ ص‬10:17:23 10/12/1437]


MyPastest

Prefer to use the old MyPastest? Access it here »


End Session

Back to Filters

Question 19 of 22

Which one of the following statements is true regarding vaccination against tuberculosis (BCG)?

A BCG forms part of the routine immunisation programme given at 2 months of age

B BCG is given as a routine pre-school vaccination

C BCG is only given to those likely to come into contact with TB

D BCG is reserved for at risk babies

E BCG should only be given to babies living in inner cities

Explanation
Contrary to previous policies, the current NHS policy is to offer BCG to children at risk of coming into contact
with TB. Risk factors include living in inner cities, over crowding, children
who have recently arrived from a high
risk country, those who have been
in contact with somebody who has open TB, those whose parents or
grandparents come from a high risk country.

www.gov.uk 45082

Previous Question Next Question Tag Question

Feedback

Difficulty: Average

https://mypastest.pastest.com/Secure/TestMe/Browser/436619#Top[‫ ص‬10:17:42 10/12/1437]


MyPastest

Peer Responses

Session Progress

Responses Correct: 0

Responses Incorrect: 29

Responses Total: 29

Responses - % Correct: 0%

Blog
About Pastest
Contact Us
Help

© Pastest 2016

https://mypastest.pastest.com/Secure/TestMe/Browser/436619#Top[‫ ص‬10:17:42 10/12/1437]


MyPastest

Prefer to use the old MyPastest? Access it here »

Back to Filters

Question 20 of 22

Which one of the following statements is a contraindication to routine immunisation?

A Breast feeding

B Family history of any adverse reactions following immunisation

C Imminent general anaesthesia

D Personal history of febrile convulsion

E None of the above

Explanation
According to the green book (see ref), the following are not contraindications to routine vaccination:-

https://www.gov.uk/government/uploads/system/uploads/attachment_data/file/147824/Green-Book-Chapter-6-
v2_0.pdf

Family history of any adverse reactions following immunisation


Previous history of the disease (with the exception of BCG for people who have evidence of past exposure
to tuberculosis)
Contact with an infectious disease
Premature birth
Stable neurological conditions such as cerebral palsy and Down syndrome
Asthma, eczema or hay fever
Mild self-limiting illness without fever, e.g. runny nose
Treatment with antibiotics or locally acting (e.g. topical or inhaled) steroids
Child’s mother or someone in the household being pregnant

https://mypastest.pastest.com/Secure/TestMe/Browser/436619#Top[‫ ص‬10:18:02 10/12/1437]


MyPastest

Currently breast-feeding or being breast-fed


End Session
History of jaundice after birth under a certain weight
Being over the age recommended in the routine childhood immunisation schedule
Personal history of febrile convulsions or epilepsy
Close family history (parent or sibling) of febrile convulsions or epilepsy
Being a sibling or close contact of an immunosuppressed individual
Recent or imminent elective surgery

45083

Tag Question

Feedback

Difficulty: Average

Peer Responses

Session Progress

Responses Correct: 0

Responses Incorrect: 30

Responses Total: 30

Responses - % Correct: 0%

Previous Question Next Question


Blog
About Pastest
Contact Us
Help

© Pastest 2016

https://mypastest.pastest.com/Secure/TestMe/Browser/436619#Top[‫ ص‬10:18:02 10/12/1437]


MyPastest

https://mypastest.pastest.com/Secure/TestMe/Browser/436619#Top[‫ ص‬10:18:02 10/12/1437]


MyPastest

Prefer to use the old MyPastest? Access it here »

Back to Filters

Question 21 of 22

According to Public Health England, if 30% of children were vaccinated against influenza, how many
hospitalisations could be saved annually?

A 2000

B 5000

C 9000

D 11000

E 15000

Explanation
According to public health England, vaccinating children against influenza can have significant health benefits
and potentially save 2000 lives per year. Infants less than 6 months of age are particularly at risk and are often
likely to pass the infection onto
adults caring for them. It is this age group, however, that has the poorest uptake
compared to older children. School delivery programmes tend to have better uptakes than programmes delivered
by GP surgeries and community pharmacies. Live vaccines are most effective in children.

See download:

www.gov.uk 45084

Previous Question Next Question

Tag Question

https://mypastest.pastest.com/Secure/TestMe/Browser/436619#Top[‫ ص‬10:18:22 10/12/1437]


MyPastest

Feedback End Session

Difficulty: Average

Peer Responses

Session Progress

Responses Correct: 0

Responses Incorrect: 31

Responses Total: 31

Responses - % Correct: 0%

Blog
About Pastest
Contact Us
Help

© Pastest 2016

https://mypastest.pastest.com/Secure/TestMe/Browser/436619#Top[‫ ص‬10:18:22 10/12/1437]


MyPastest

Prefer to use the old MyPastest? Access it here »

Back to Filters

Question 22 of 22

A mother enquires about the HPV vaccination for her daughter.

What age are girls offered the HPV vaccination?

A 5-6

B 7-8

C 9-10

D 11-12

E 12-13

Explanation
The HPV vaccination programme is offered to girls
aged 12-13. Gardasil protects against HPV 16 and 18 which
cause cervical cancer, and 6 and 11 which cause genital warts.

www.gov.uk 45085

End Session

Previous Question Tag Question

Feedback

Difficulty: Average

https://mypastest.pastest.com/Secure/TestMe/Browser/436619#Top[‫ ص‬10:18:41 10/12/1437]


MyPastest

Peer Responses

Session Progress

Responses Correct: 0

Responses Incorrect: 32

Responses Total: 32

Responses - % Correct: 0%

Blog
About Pastest
Contact Us
Help

© Pastest 2016

https://mypastest.pastest.com/Secure/TestMe/Browser/436619#Top[‫ ص‬10:18:41 10/12/1437]


MyPastest

Prefer to use the old MyPastest? Access it here »

Back to Filters

Question 1 of 24

Which of the following is true concerning complement activation?

A IgG and IgE are the main antibody classes involved in classical pathway activation

B C1q binds to the Fab regions of antigen-complexed IgG antibodies

C The alternative, but not the classical C3, convertase enzyme involves C3b

D Elevated serum C3dg is a good marker of complement activation

E The membrane-attack complex involves polymerisation of C7

Explanation
Previous Question
IgG and IgM are the main antibody classes that activate the classical pathway. C1q binds to the Fc rather than the
Fab fragment that binds antibody (Fc, crystallisable fragment; Fab antigen-binding fragment). It is C9 that
polymerises in the membrane-attack complex. Clotting of blood can lead to complement activation, and hence
complement conversion products must be measured on
plasma and not serum. 12138

Next Question
Tag Question

Feedback

End Session
Difficulty: Difficult

Peer Responses

https://mypastest.pastest.com/Secure/TestMe/Browser/436619[‫ ص‬10:20:29 10/12/1437]


MyPastest

Session Progress

Responses Correct: 0

Responses Incorrect: 1

Responses Total: 1

Responses - % Correct: 0%

Blog
About Pastest
Contact Us
Help

© Pastest 2016

https://mypastest.pastest.com/Secure/TestMe/Browser/436619[‫ ص‬10:20:29 10/12/1437]


MyPastest

Prefer to use the old MyPastest? Access it here »

Back to Filters

Question 2 of 24

Which of the following is true concerning immunity to viruses?

A IgA can offer protection at mucosal surfaces

B Cytotoxic T cells are activated before natural killer cells during the course of infection

C Viruses stimulate the non-immune cells that they infect to produce interferon-gamma

D Non-enveloped viruses are susceptible to damage by complement

E Influenza virus can avoid antibody recognition by mutational changes in its nucleocapsid proteins

Explanation
Previous Question
Natural killer cells are activated faster than cytotoxic T cells. Infected non-immune cells produce interferon-α and
-ß, whereas interferon-γ is produced by T cells. Influenza virus mutates its surface neuraminidase and
haemagglutinin to avoid antibody recognition. Enveloped viruses are susceptible to complement attack. 12139

Next Question
Tag Question

Feedback

Difficulty: Average End Session

Peer Responses

https://mypastest.pastest.com/Secure/TestMe/Browser/436619[‫ ص‬10:20:54 10/12/1437]


MyPastest

Session Progress

Responses Correct: 0

Responses Incorrect: 2

Responses Total: 2

Responses - % Correct: 0%

Blog
About Pastest
Contact Us
Help

© Pastest 2016

https://mypastest.pastest.com/Secure/TestMe/Browser/436619[‫ ص‬10:20:54 10/12/1437]


MyPastest

Prefer to use the old MyPastest? Access it here »

Back to Filters

Question 3 of 24

Which of the following statements is true concerning immunity to bacteria?

A Antibodies to secreted bacterial products play no protective role

B Bacteria opsonised by antibodies and complement are more effectively phagocytosed than those
opsonised by antibodies alone

C Humoral rather than cellular immunity is predominant in protection against all types of bacteria

D Phagocytes cannot engulf bacteria in the absence of antibodies

E Endotoxin induces shock mainly through the activation of T cells

Previous Question
Explanation
Antibodies such as those to cholera, diphtheria and tetanus toxin can play a major role in protective immunity in
these infections. Cellular immunity is essential in protection against intracellular bacteria, e.g. mycobacteria.
Phagocytes interact directly but weakly with bacteria, or strongly if they are complement-opsonised. Endotoxin
activates macrophages by binding to CD14. 12140

Next Question

Tag Question

Feedback
End Session

Difficulty: Average

https://mypastest.pastest.com/Secure/TestMe/Browser/436619[‫ ص‬10:21:12 10/12/1437]


MyPastest

Peer Responses

Session Progress

Responses Correct: 0

Responses Incorrect: 3

Responses Total: 3

Responses - % Correct: 0%

Blog
About Pastest
Contact Us
Help

© Pastest 2016

https://mypastest.pastest.com/Secure/TestMe/Browser/436619[‫ ص‬10:21:12 10/12/1437]


MyPastest

Prefer to use the old MyPastest? Access it here »

Back to Filters

Question 4 of 24

In a patient developing anaphylaxis, which of the following is true?

A May be exacerbated by exercise

B Involves leukotriene A4

C Initial symptoms include a sensation of coldness

D 20% of fatalities are due to respiratory complications

E Less than 10% of patients have a second episode within 24 hours

Explanation
Previous Question
Anaphylaxis can be associated with leukotrienes B4, C4, D4 and E4. The first three are the mediators that make up
the slow-reacting substance of anaphylaxis (SRSA). Flushing, warmth and tingling are typical initial symptoms.
At least 50% of fatalities are due to respiratory complications, and more than 20% of patients will have a second
episode within 8 hours. 12141

Next Question
Tag Question

Feedback

End Session
Difficulty: Difficult

Peer Responses

https://mypastest.pastest.com/Secure/TestMe/Browser/436619[‫ ص‬10:21:31 10/12/1437]


MyPastest

Session Progress

Responses Correct: 0

Responses Incorrect: 4

Responses Total: 4

Responses - % Correct: 0%

Blog
About Pastest
Contact Us
Help

© Pastest 2016

https://mypastest.pastest.com/Secure/TestMe/Browser/436619[‫ ص‬10:21:31 10/12/1437]


MyPastest

Prefer to use the old MyPastest? Access it here »

Back to Filters

Question 5 of 24

In a patient with nickel-associated contact dermatitis, which one of the following statements is true?

A Mediated by mast cells

B Mediated by IgE

C Skinprick testing is the best way to establish the sensitising antigen

D Systemic cytokine release induces skin inflammation

E Elimination of the responsible agent is the most important goal

Explanation
Previous Question
Contact dermatitis is caused by T lymphocytes reacting with haptens conjugated to skin proteins. Patch testing is
the classical in vivo mechanism for ascertaining the sensitising antigen and
is read at 48–96 hours. Skinprick
testing investigates IgE-mediated reactions and is typically read after 15 minutes of using a
positive histamine
control and a negative solution of diluent. Local cytokine release produces the local response typical of these
contact reactions. 12142

Next Question

Tag Question

Feedback
End Session

Difficulty: Average

https://mypastest.pastest.com/Secure/TestMe/Browser/436619[‫ ص‬10:21:50 10/12/1437]


MyPastest

Peer Responses

Session Progress

Responses Correct: 0

Responses Incorrect: 5

Responses Total: 5

Responses - % Correct: 0%

Blog
About Pastest
Contact Us
Help

© Pastest 2016

https://mypastest.pastest.com/Secure/TestMe/Browser/436619[‫ ص‬10:21:50 10/12/1437]


MyPastest

Prefer to use the old MyPastest? Access it here »

Back to Filters

Question 6 of 24

Which of the following excludes a general, rather than a selective, antibody deficiency?

A Normal serum immunoglobulins

B Good antibody responses to immunisations

C The presence of existing antibody responses to past infections

D Normal IgG subclasses

E Normal peripheral blood lymphocyte subpopulations

Explanation
Previous Question
Normal immunoglobulins, including subclasses, do not
exclude antibody deficiency. Hence in patients with a
good history of recurrent (proven) bacterial infections, responses to Haemophilus influenzae, Pneumococcus
spp.
and tetanus toxoid should all be assessed, as should postimmunisation responses if required. Antibodies to past
infections and haemagglutinins can be helpful in assessing a patient. However, IgG responses to previous
vaccinations would not pick up a selective IgA deficiency. 12143

Next Question

Tag Question

Feedback
End Session

Difficulty: Difficult

https://mypastest.pastest.com/Secure/TestMe/Browser/436619[‫ ص‬10:22:08 10/12/1437]


MyPastest

Peer Responses

Session Progress

Responses Correct: 0

Responses Incorrect: 6

Responses Total: 6

Responses - % Correct: 0%

Blog
About Pastest
Contact Us
Help

© Pastest 2016

https://mypastest.pastest.com/Secure/TestMe/Browser/436619[‫ ص‬10:22:08 10/12/1437]


MyPastest

Prefer to use the old MyPastest? Access it here »

Back to Filters

Question 7 of 24

Parents of a boy with suspected peanut allergy want answers to many questions, including prediction of the
severity of future reactions and whether he will grow out the allergy.

Which of the following is a valid statement?

A The weal size resulting from the skinprick test is a good indicator for the severity of the next
anaphylactic reaction

B The amount of plasma-specific IgE to peanuts/treenuts is a good indicator for the severity of the next
anaphylactic reaction

C The weal size resulting from the skinprick test is an excellent predictor of a positive food challenge to
peanuts

D A negative skinprick is sufficient evidence to definitely exclude peanut allergy

E More than 50% of peanut allergic individuals will outgrow these allergies

Explanation
The severity of the next allergic reaction cannot be
predicted by skinprick test or specific IgE, as the severity
depends on
other major factors such as the amount consumed and intercurrent viral infections. The positive
Next Question
predictive value of a negative skinprick test to foods is high: that is, for all nuts, skinprick tests miss only 0.5% of
cases, whereas specific IgE will miss 22%. Less than 20% of individuals will grow out of peanut allergy, but they
are usually those in whom the reactions are mild and started under 1 year of age. 12144

Tag Question

https://mypastest.pastest.com/Secure/TestMe/Browser/436619[‫ ص‬10:22:27 10/12/1437]


MyPastest

Feedback

Difficulty: Difficult

Peer Responses

Session Progress

Responses Correct: 0

Responses Incorrect: 7

Responses Total: 7

Responses - % Correct: 0%

Blog
About Pastest
Contact Us
Help
Previous Question
© Pastest 2016

End Session

https://mypastest.pastest.com/Secure/TestMe/Browser/436619[‫ ص‬10:22:27 10/12/1437]


MyPastest

Prefer to use the old MyPastest? Access it here »

Back to Filters

Question 8 of 24

Theme: Investigations

A Allergen specific IgE


B IgA anti-tissue transglutaminase antibodies
C Full blood count
D C1-esterase inhibitor assay
E Sub-dermal skin tests
F Food elimination and re-introduction
G Total IgE
H Skin patch testing
I Stool elastase
J Vega testing

For
each of the clinical scenarios below choose the most useful test. Each option may be used once, more than
once, or not at all.

Scenario 1

A 3-year-old eats a chocolate nut bar for the first time, and within 15 minutes develops wheeze, stridor and a rash.
She attends the Emergency Department and receives intramuscular epinephrine, hydrocortisone and
chlorpeniramine. She has had one or two similar but milder reactions in the past to a particular breakfast cereal.

Your answer was incorrect

Select one...

A - Allergen specific IgE

In the first instance, nut allergy should be suspected. From the list the most appropriate test would be an allergic
specific IgE (RAST), although an oral challenge to nut within the hospital environment would be better.

Scenario 2

An 8-month-old boy has severe eczema despite a lot of topical treatments while his weight has crossed from the
50th to the 25th centile. He is breast fed and also on a mixed diet and some SMA
gold cap. He is also on soy
yoghurt and soy cheese on the advice of a neighbour.

https://mypastest.pastest.com/Secure/TestMe/Browser/436619[‫ ص‬10:22:49 10/12/1437]


MyPastest

Your answer was incorrect

Select one...

F - Food elimination and re-introduction

An 8-month-old child with severe eczema may be due to cow’s milk protein intolerance. Although there is little
evidence for a dairy free diet, it may be beneficial to eliminate cow’s milk from the diet and re-introduce it at a
later date.

Scenario 3

You see a rather irritable 15-month-old who is the fifth child of a travelling family. He has slightly loose stools
and
occasional tummy ache. There are no past recorded growth measurements and he has had no immunisations.
The only family history is a cousin with some sort of food allergy. Examination reveals a pale boy with a
distended abdomen.

Your answer was incorrect

Select one...

B - IgA anti-tissue transglutaminase antibodies

Loose stools, with abdominal pain and a child


with a distended abdomen should lead you towards the diagnosis of
coeliac disease. The most helpful test leading towards a diagnosis would
be IgA anti-tissue transglutaminase
antibodies. It is important to also
measure the IgA level.
14583

Previous Question

Tag Question

Feedback

Difficulty: Average
Next Question
Session Progress

Responses Correct: 0

Responses Incorrect: 10

Responses Total: End Session 10

Responses - % Correct: 0%

https://mypastest.pastest.com/Secure/TestMe/Browser/436619[‫ ص‬10:22:49 10/12/1437]


MyPastest

Blog
About Pastest
Contact Us
Help

© Pastest 2016

https://mypastest.pastest.com/Secure/TestMe/Browser/436619[‫ ص‬10:22:49 10/12/1437]


MyPastest

Prefer to use the old MyPastest? Access it here »

Back to Filters

Question 9 of 24

Theme: Immunodeficiency

A IgA deficiency
B Severe combined immunodeficiency
C Selective IgG deficiency
D Chronic granulomatous disease
E HIV infection
F Bruton’s agammaglobinaemia
G DiGeorge syndrome
H Wiscott–Aldrich syndrome
I Ataxia telangiectasia
J Leukocyte adhesion defect

From
the list above, select the most likely diagnosis for each of the cases below. Each item may be used once,
more than once or not at all.

Scenario 1

A 3-year-old in seen in clinic after two previous admissions for pneumonia. He recently had grommets inserted.
His parents report that he always has a runny nose. You know that he has
eczema but no other skin lesions.

Your answer was incorrect

Select one...

A - IgA deficiency

IgA has an incidence of approximately 1


in 500. Recurrent upper and lower respiratory infections are the most
common symptoms. Recurrent otitis media is often reported. There is an association with atopic and autoimmune
conditions. Treatment is with antibiotics for acute infections and consideration of prophylactic antibiotics.

Scenario 2

A 3-and-a-half-month-old boy is admitted for atypical pneumonia. His mother says that he has recurrent diarrhea.
On examination you note that he is growth faltering and that he also has
severe oral candidiasis.

https://mypastest.pastest.com/Secure/TestMe/Browser/436619[‫ ص‬10:23:09 10/12/1437]


MyPastest

Your answer was incorrect

Select one...

B - Severe combined immunodeficiency

Children with severe combined immunodeficiency (SCID) are often well for the first couple of months. The usual
presentation is at 3-4 months with recurrent/persistent diarrhoea and growth faltering. Candidiasis is often present.
Other presentations include atypical infections and infections (especially chest) that are difficult to treat.

Scenario 3

A 4-month-old South-African child presents


with growth faltering. On examination he is noted to have eczema
and oral candidiasis. The full blood count (FBC) is normal. IgG is raised, IgM low and IgA low.

Your answer was incorrect

Select one...

E - HIV infection

The risk of vertical transmission of HIV in Europe is 10-25%. The risk is minimised by anti-retroviral agents
and
delivery by caesarean section. Presentation in infancy is often with failure to thrive. Gastrointestinal symptoms are
common. Serum immunoglobulins may show a hyper-IgG globinaemia with a low IgM/IgA. Diagnosis is on HIV
PCR (polymerase chain reaction) and p24 antigen detection (HIV antibodies may be present as a result of maternal
infection and placental passage).
22026

Previous Question

Tag Question

Feedback

Difficulty: Average

Next Question
Session Progress

Responses Correct: 0

Responses Incorrect: 13

Responses Total: 13
End Session
Responses - % Correct: 0%

https://mypastest.pastest.com/Secure/TestMe/Browser/436619[‫ ص‬10:23:09 10/12/1437]


MyPastest

Blog
About Pastest
Contact Us
Help

© Pastest 2016

https://mypastest.pastest.com/Secure/TestMe/Browser/436619[‫ ص‬10:23:09 10/12/1437]


MyPastest

Prefer to use the old MyPastest? Access it here »

Back to Filters

Question 10 of 24

A parent suspects that her 2-year-old son has a food intolerance and wants ‘tests’. He is well and growing along
the appropriate centile. What is the most appropriate course of action?

A Advise her that there are no investigations that can 100% determine food intolerances

B Advise
her to try to exclude the suspected food and see if this produces an improvement in his
symptoms; if so then reintroduce and see if the symptoms return

C Perform skin-prick testing to the suspected allergen and other common allergens

D Perform radioallergosorbent test (RAST) to the suspected allergens and other common allergens

E Perform baseline coeliac screen, stool-reducing sugars, and inflammatory markers to exclude more
seriousPrevious
pathology Question

Explanation
Food intolerance is a difficult area, with parents wanting definitive tests and answers. The only scientific and
practical way of ‘detecting’ intolerance is to remove the suspected foodstuffs and see if there is an improvement in
symptoms. A staggered reintroduction of individual foods can then be tried to see if
the symptoms recur. RAST
and skin prick tests have some limitations in both specificity and sensitivity, and need to be interpreted in
Next Question
combination with the clinical history. A well, thriving child is less likely to have serious pathology (although you
should always keep this in the back of your mind).
22228

Tag Question

https://mypastest.pastest.com/Secure/TestMe/Browser/436619[‫ ص‬10:23:28 10/12/1437]


MyPastest

Feedback

Difficulty: Average

Peer Responses

Session Progress

Responses Correct: 0

Responses Incorrect: 14

Responses Total: 14

Responses - % Correct: 0%

Blog
About Pastest
Contact Us
Help

© Pastest 2016

End Session

https://mypastest.pastest.com/Secure/TestMe/Browser/436619[‫ ص‬10:23:28 10/12/1437]


MyPastest

Prefer to use the old MyPastest? Access it here »

Back to Filters

Question 11 of 24

In the treatment of anaphylaxis, what is the correct dose of intramuscular adrenaline (1:1000) for a child between
6 and 12 years of age?

A 150 micrograms

B 300 micrograms

C 300 milligrams

D 500 micrograms

E 500 milligrams

Previous Question
Explanation
IM doses of 1:1000 adrenaline for anaphylaxis (repeat after 5 min if no better)

Adult or child more than 12 years: 500 micrograms IM (0.5 mL)


Child 6 - 12 years: 300 micrograms IM (0.3 mL)
Child 6 months - 6 years and child less than 6 months: 150 micrograms IM (0.15 mL)

Next Question
https://www.resus.org.uk/ 45051

Tag Question

https://mypastest.pastest.com/Secure/TestMe/Browser/436619[‫ ص‬10:24:12 10/12/1437]


MyPastest

Feedback

Difficulty: Average

Peer Responses

Session Progress

Responses Correct: 0

Responses Incorrect: 15

Responses Total: 15

Responses - % Correct: 0%

Blog
About Pastest
Contact Us
Help

© Pastest 2016

End Session

https://mypastest.pastest.com/Secure/TestMe/Browser/436619[‫ ص‬10:24:12 10/12/1437]


MyPastest

Prefer to use the old MyPastest? Access it here »

Back to Filters

Question 12 of 24

What is the recommended volume that should be given as a fluid bolus to a child in anaphylaxis?

A 5ml/kg body weight

B 10ml/kg body weight

C 15ml/kg body weight

D 20ml/kg body weight

E 25ml/kg body weight

Explanation
Previous Question
Anaphylaxis can present as respiratory distress with either stridor or wheeze, or as cardiovascular collapse. Shock
is defined as inadequate perfusion and oxygenation of the tissues and in the case of anaphylaxis is described as
distributive shock.

The management of anaphylaxis is essential reading for this exam. More information can be found at:

https://www.resus.org.uk/_resources/assets/attachment/full/0/825.pdf 45052

Next Question

Tag Question

Feedback
End Session

Difficulty: Average

https://mypastest.pastest.com/Secure/TestMe/Browser/436619[‫ ص‬10:24:31 10/12/1437]


MyPastest

Peer Responses

Session Progress

Responses Correct: 0

Responses Incorrect: 16

Responses Total: 16

Responses - % Correct: 0%

Blog
About Pastest
Contact Us
Help

© Pastest 2016

https://mypastest.pastest.com/Secure/TestMe/Browser/436619[‫ ص‬10:24:31 10/12/1437]


MyPastest

Prefer to use the old MyPastest? Access it here »

Back to Filters

Question 13 of 24

A child in anaphylaxis is given 50mg of intravenous hydrocortisone after having received 150 micrograms of
adrenaline IM, fluids and 2.5mg of chlorphenamine.

How old is the child likely to be?

A 4 days

B 4 months

C 4 weeks

D 4 years

E 14 years

Explanation
The management of anaphylaxis is essential reading for this exam.

Treatment:

Remove allergen
Assess ABCDE
Give high-flow oxygen
Adrenaline 1:1000 i.m.
If complete obstruction, obtain a definitive airway
If partial obstruction:

Give nebulised adrenaline 5ml of 1:1000


Give hydrocortisone

https://mypastest.pastest.com/Secure/TestMe/Browser/436619[‫ ص‬10:24:51 10/12/1437]


MyPastest

If wheeze present, give nebulised salbutamol


Consider intravenous salbutamol
If shock present give fluid bolus and consider adrenaline infusion
Chlorpheniramine

More information including doses can be found at:

http://www.resus.org.uk/_resources/assets/attachment/full/0/792.pdf
45053

Tag Question

Feedback

Difficulty: Average

Peer Responses

Previous Question

Session Progress

Responses Correct: 0

Responses Incorrect: 17

Responses Total: 17

Responses - % Correct: Next Question 0%

Blog
About Pastest
Contact Us
Help
End Session
© Pastest 2016

https://mypastest.pastest.com/Secure/TestMe/Browser/436619[‫ ص‬10:24:51 10/12/1437]


MyPastest

https://mypastest.pastest.com/Secure/TestMe/Browser/436619[‫ ص‬10:24:51 10/12/1437]


MyPastest

Prefer to use the old MyPastest? Access it here »

Back to Filters

Question 14 of 24

Regarding the treatment of anaphylaxis, what is the best site for administering instramuscular adrenaline?

A Abdomen

B Anterolateral thigh

C Anteromedial thigh

D Deltoid

E Upper outer buttock

Explanation
Previous Question
The ideal site for IM adrenaline in the context of anaphylaxis treatment is the anterolateral middle third of the
thigh. A 25mm standard blue needle is recommended for those who draw
up adrenaline rather than using auto-
injector. Orange needles should not be used as there is a risk of injecting subcutaneously rather than
intramuscularly.
45054

Next Question

Tag Question

Feedback
End Session
Difficulty: Average

https://mypastest.pastest.com/Secure/TestMe/Browser/436619[‫ ص‬10:25:23 10/12/1437]


MyPastest

Peer Responses

Session Progress

Responses Correct: 0

Responses Incorrect: 18

Responses Total: 18

Responses - % Correct: 0%

Blog
About Pastest
Contact Us
Help

© Pastest 2016

https://mypastest.pastest.com/Secure/TestMe/Browser/436619[‫ ص‬10:25:23 10/12/1437]


MyPastest

Prefer to use the old MyPastest? Access it here »

Back to Filters

Question 15 of 24

In the management of anaphylaxis, which of the following statements are true regarding mast cell tryptase levels?

A Mast cell tryptase levels peak 1-2 hours after the onset of symptoms

B Mast cell tryptase levels peak 2-4 hours after the onset of symptoms

C Mast cell tryptase levels peak 4-6 hours after the onset of symptoms

D Mast cell tryptase levels peak 6-8 hours after the onset of symptoms

E Mast cell tryptase levels peak 8-10 hours after the onset of symptoms

Explanation
Previous Question
Serial Mast cell tryptase levels are useful in managing the follow up of patients with anaphylaxis. They are not
useful in the immediate phase and should not delay treatment. Levels
rise after 30 mins from onset of symptoms
and peak around 1-2 hours. The half life of tryptase is 2 hours. It's vey important to note the time when samples
are taken.
45055

Next Question

Tag Question

Feedback
End Session
Difficulty: Average

https://mypastest.pastest.com/Secure/TestMe/Browser/436619[‫ ص‬10:25:42 10/12/1437]


MyPastest

Peer Responses

Session Progress

Responses Correct: 0

Responses Incorrect: 19

Responses Total: 19

Responses - % Correct: 0%

Blog
About Pastest
Contact Us
Help

© Pastest 2016

https://mypastest.pastest.com/Secure/TestMe/Browser/436619[‫ ص‬10:25:42 10/12/1437]


MyPastest

Prefer to use the old MyPastest? Access it here »

Back to Filters

Question 16 of 24

A parent asks you how common food allergies are.

What is the prevalence of food allergy in westernised countries?

A 0.05%

B 0.1%

C 0.5%

D 1%

E 5%

Explanation
Food allergy is thought to affect 5% of children in western countries and its prevelance has increased dramatically
in recent years. Food allergies are immune meditated reactions to food antigen (protein) and can be broadly
classified into IgE meditated and non IgE meditated. The former tends to produce more immediate reactions
(anaphylaxis, urticaria and atopic dermatitis) whereas the later produces delayed reaction and implies intolerance.
Cows milk protein allergy affects around 2% of infants.

For further information see: Next Question


http://www.nice.org.uk/guidance/CG116/chapter/introduction
Sicherer SH, Sampson HA; Food allergy. J Allergy Clin Immuno. 2010 Feb; 125 (2 Suppl 2): S116-25. Epub 2009
Dec 29.

45061

https://mypastest.pastest.com/Secure/TestMe/Browser/436619[‫ ص‬10:26:03 10/12/1437]


MyPastest

Tag Question

Feedback

Difficulty: Average

Peer Responses

Session Progress

Responses Correct: 0

Responses Incorrect: 20

Responses Total: 20

Responses - % Correct: 0%

Previous Question
Blog
About Pastest
Contact Us
Help

© Pastest 2016

End Session

https://mypastest.pastest.com/Secure/TestMe/Browser/436619[‫ ص‬10:26:03 10/12/1437]


MyPastest

Prefer to use the old MyPastest? Access it here »

Back to Filters

Question 17 of 24

Immediate hypersensitivity reactions to foods are usually meditated by which antibody?

A IgA

B IgD

C IgE

D IgG

E IgM

Explanation
Previous Question
Immediate hypersensitivity reactions are IgE mediated. These reactions may be severe and lead to anaphylaxis and
even death.

Common
allergies include to egg, cows' milk and peanut. Clinical features occur soon after ingestion of the
offending agent and can include angioedema, urticaria, nausea, vomiting, wheeze and circulatory collapse.

Some patients have a mixture of IgE and non-IgE mediated responses hence careful history taking is crucial or
else the diagnosis may be missed. Delayed reactions may cause several non-specific symptoms including failure to
thrive, reflux, fatigue or irritability. Next Question
45062

Tag Question

Feedback

https://mypastest.pastest.com/Secure/TestMe/Browser/436619[‫ ص‬10:26:23 10/12/1437]


MyPastest

Difficulty: Average

Peer Responses

Session Progress

Responses Correct: 0

Responses Incorrect: 21

Responses Total: 21

Responses - % Correct: 0%

Blog
About Pastest
Contact Us
Help

© Pastest 2016

End Session

https://mypastest.pastest.com/Secure/TestMe/Browser/436619[‫ ص‬10:26:23 10/12/1437]


MyPastest

Prefer to use the old MyPastest? Access it here »

Back to Filters

Question 18 of 24

Which one of the following statements illustrate the underlying mechanism behind lactose intolerance?

A Anti-lactase antibodies

B Galactose intolerance

C IgE meditated immunity

D Lactase deiciency

E Lactose deficiency

Explanation
Previous Question
Lactose intolerance is a caused by a deficiency of the intestinal enzyme lactase. Lactase breaks down lactose
into
glucose and galactose. Primary lactase deficiency, which is autosomal recessive (adult type), is uncommon
whereas as a temporary lactase deficiency is common and often occurs secondary to bowel disorders such as
gastroenteritis, inflammatory bowel disease and coeliac disease.

Premature babies may also be deficient in lactase.

A
rare autosomal recessive lactose intolerance has been documented in which there is either very little or no
lactase from childbirth. Next Question
45063

Tag Question

Feedback

https://mypastest.pastest.com/Secure/TestMe/Browser/436619[‫ ص‬10:26:42 10/12/1437]


MyPastest

Difficulty: Average

Peer Responses

Session Progress

Responses Correct: 0

Responses Incorrect: 22

Responses Total: 22

Responses - % Correct: 0%

Blog
About Pastest
Contact Us
Help

© Pastest 2016

End Session

https://mypastest.pastest.com/Secure/TestMe/Browser/436619[‫ ص‬10:26:42 10/12/1437]


MyPastest

Prefer to use the old MyPastest? Access it here »

Back to Filters

Question 19 of 24

Regarding reactions to food, which one of the following symptoms is more suggestive of non IgE meditated
allergy?

A Eczema

B Pruritis

C Rhinorrhoea

D Urticaria

E Wheeze

Previous Question
Explanation
Features of non IgE mediated allergies are several fold and include eczema, failure to thrive, irritability and
reflux.
Investigations should start with a detailed history and consideration of a food diary. For non IgE mediated
allergies, an exclusion diet with slow introduction is often helpful in diagnosis.
45064

Next Question

Tag Question

Feedback
End Session
Difficulty: Average

https://mypastest.pastest.com/Secure/TestMe/Browser/436619[‫ ص‬10:27:01 10/12/1437]


MyPastest

Peer Responses

Session Progress

Responses Correct: 0

Responses Incorrect: 23

Responses Total: 23

Responses - % Correct: 0%

Blog
About Pastest
Contact Us
Help

© Pastest 2016

https://mypastest.pastest.com/Secure/TestMe/Browser/436619[‫ ص‬10:27:01 10/12/1437]


MyPastest

Prefer to use the old MyPastest? Access it here »

Back to Filters

Question 20 of 24

Which one of the following food allergies are children least likely to 'grow out' of?

A Eggs

B Milk

C Peanut

D Soya

E Wheat

Explanation
Previous Question
The prognosis of food allergies vary. In
some cases an elimination diet is required. Children with reactions to
peanuts, sea food and tree nuts tend not to grow out of their allergies.
The natural history of cows' milk intolerance
is one of resolution with 80-90% back on a normal diet by age 3.
45065

Next Question
Tag Question

Feedback

Difficulty: Average
End Session

Peer Responses

https://mypastest.pastest.com/Secure/TestMe/Browser/436619[‫ ص‬10:27:19 10/12/1437]


MyPastest

Session Progress

Responses Correct: 0

Responses Incorrect: 24

Responses Total: 24

Responses - % Correct: 0%

Blog
About Pastest
Contact Us
Help

© Pastest 2016

https://mypastest.pastest.com/Secure/TestMe/Browser/436619[‫ ص‬10:27:19 10/12/1437]


MyPastest

Prefer to use the old MyPastest? Access it here »

Back to Filters

Question 21 of 24

Which one of the following symptoms may be features of Cows' milk protein allergy?

A Angioedema

B Diarrhoea

C Urticaria

D Vomiting

E All of the above

Explanation
Previous Question
Cows' milk protein allergy tends to manifest at the ages when cows' milk is introduced into the diet. It can,
however, present earlier in sensitive breastfeeding infants if the mother has cows' milk in her diet. It may be IgE or
non-IgE meditated.

Manifestations
vary including life threatening anaphylaxis and shock, gastrointestinal
symptoms (diarrhoea which
may be bloody), vomiting, respiratory (e.g. wheeze) and dermatological (e.g. eczema, urticaria).

Next Question 45763

End
Tag Session
Question

Feedback

https://mypastest.pastest.com/Secure/TestMe/Browser/436619[‫ ص‬10:27:37 10/12/1437]


MyPastest

Difficulty: Average

Peer Responses

Session Progress

Responses Correct: 0

Responses Incorrect: 25

Responses Total: 25

Responses - % Correct: 0%

Blog
About Pastest
Contact Us
Help

© Pastest 2016

https://mypastest.pastest.com/Secure/TestMe/Browser/436619[‫ ص‬10:27:37 10/12/1437]


MyPastest

Prefer to use the old MyPastest? Access it here »

Back to Filters

Question 22 of 24

What system is most commonly affected by IgE mediated cows' milk protein allergy?

A Cardiovascular

B Constitutional (Generalised)

C Gastrointestinal

D Respiratory

E Skin

Explanation
Previous Question
Cows' milk protein allergy may be IgE or non IgE mediated. The commonest symptoms of IgE mediated disease
are dermatological, followed by gastrointestinal, respiratory and rarely cardiovascular.

Constitutional symptoms such as irritability, failure to thrive and even anaphylaxis are also seen. Anaphylaxis may
present with floppiness and pallor hence careful history
taking is required or else the diagnosis may be missed.

Non IgE mediated disease (delayed hypersensitivity) usually presents with gastrointestinal symptoms.

Next Question 45764

End Session
Tag Question

Feedback

https://mypastest.pastest.com/Secure/TestMe/Browser/436619[‫ ص‬10:27:57 10/12/1437]


MyPastest

Difficulty: Average

Peer Responses

Session Progress

Responses Correct: 0

Responses Incorrect: 26

Responses Total: 26

Responses - % Correct: 0%

Blog
About Pastest
Contact Us
Help

© Pastest 2016

https://mypastest.pastest.com/Secure/TestMe/Browser/436619[‫ ص‬10:27:57 10/12/1437]


MyPastest

Prefer to use the old MyPastest? Access it here »

Back to Filters

Question 23 of 24

What is the prevalance of cows' milk protein allergy in formula or mixed-fed (breast and formula-fed) infants?

A 3%

B 5%

C 7%

D 9%

E 11%

Explanation
Previous Question
The prevalence of cows' milk protein varies according to the literature however NICE places the figure at 7%. It is
the second most common allergy; secondary to egg allergy. It may be either IgE or non IgE meditated. The
commonest presentation of IgE meditated disease is dermatological, followed by intestinal, respiratory
and
cardiovascular. Generalised symptoms include failure to thrive, irritability and anapylaxis.

http://cks.nice.org.uk/cows-milk-protein-allergy-in-children#!backgroundsub:2 45766

Next Question

Tag Question

Feedback End Session

Difficulty: Average

https://mypastest.pastest.com/Secure/TestMe/Browser/436619[‫ ص‬10:28:16 10/12/1437]


MyPastest

Peer Responses

Session Progress

Responses Correct: 0

Responses Incorrect: 27

Responses Total: 27

Responses - % Correct: 0%

Blog
About Pastest
Contact Us
Help

© Pastest 2016

https://mypastest.pastest.com/Secure/TestMe/Browser/436619[‫ ص‬10:28:16 10/12/1437]


MyPastest

Prefer to use the old MyPastest? Access it here »

Back to Filters

Question 24 of 24

A 5 year old boy presents to A&E with a fever of 39°C, red eyes, strawberry tongue and cracked lips, periungual
desquamation of his fingers and toes, polymorphous rash and cervical lymphadenopathy. He has been too unwell
to attend school for a week.

Which one of the following treatments will afford him the best long term prognosis?

A Antibiotics

B Aspirin

C Immunoglobulin

D Steroids

E TNF alpha antagonists

Explanation
This boy meets the criteria for a diagnosis of Kawasaki Disease - an acute, self-limiting vasculitis of childhood.

DiagnosticCriteria:

Fever (38.5°C) for over 5 days, without other explanation

In the presence of 4 of the 5 following criteria:

Bilateral congestion of the ocular conjunctivae (non-purulent).


Changes to the lips and oral cavity (at least one of: dryness, erythema, lip fissuring, strawberry tongue,
diffuse erythema of the oral and pharyngeal mucosa).
Changes of the extremities (at least one of: erythema of palms and soles, indurative oedema, periungual

https://mypastest.pastest.com/Secure/TestMe/Browser/436619[‫ ص‬10:28:37 10/12/1437]


MyPastest

desquamation of fingers and toes).


Polymorphous exanthema.
Non-suppurative cervical lymphadenopathy.

The most concerning effect of Kawasaki Disease is the formation of coronary artery aneurysms or ectasia which
develop in 15 – 25% of untreated children. This may lead to Ischaemic Heart Disease or sudden death. This must
be managed optimally for best long term prognosis.

Answer C, Immunoglobulin (IV gamma globulin) in the acute phase reduces


the prevalence of coronary artery
abnormalities. The mechanism is not fully understood and there are numerous possible modes of action.
Immunoglobulin has the most important impact on long term prognosis. Children should also receive cardiac
follow up with imaging of the heart
and vessels.

Aspirin has been used for many years to treat Kawasaki Disease. It has important anti-inflammatory effects (in
high dose) and anti-platelet effects (in low dose). It does not appear to lower the incidence of coronary artery
abnormalities.

Steroids were used as the initial therapy in Kawasaki Disease before immunoglobulin efficacy was reported. There
is no evidence they reduce coronary artery abnormalities and their role in overall treatment is not
clear.

TNF alpha antagonists (such as infliximab) are not yet in the standard treatment regime but may be used in severe
and refractory Kawasaki Disease where immunoglobulin has failed.TNF alpha antagonists may reduce coronary
artery abnormalities but further research outcomes are awaited.

Antibiotics are not part of routine Kawasaki Disease treatment.


46205

End Session

Previous Question Tag Question

Feedback

Difficulty: Average

Peer Responses

https://mypastest.pastest.com/Secure/TestMe/Browser/436619[‫ ص‬10:28:37 10/12/1437]


MyPastest

Session Progress

Responses Correct: 0

Responses Incorrect: 28

Responses Total: 28

Responses - % Correct: 0%

Blog
About Pastest
Contact Us
Help

© Pastest 2016

https://mypastest.pastest.com/Secure/TestMe/Browser/436619[‫ ص‬10:28:37 10/12/1437]


MyPastest

Prefer to use the old MyPastest? Access it here »

Back to Filters

Question 1 of 90

Theme: Common infections

A Adenovirus
B ß-Haemolytic streptococcus
C Coxsackie virus A16
D Epstein–Barr virus (EBV)
E Human herpesvirus 6
F Mycoplasma pneumoniae
G Parvovirus B19
H Poxvirus
I Rhinovirus
J Varicella-zoster virus

For
each of the following scenarios, choose the most likely underlying causative organism from the above list.
Each item may be used once, more
than once or not at all.

Scenario 1

A 12-year-old boy has a painful erythematous throat for which he is prescribed a broad-spectrum antibiotic. He
represents the following day with a florid maculopapular rash all over his trunk.

Your answer was incorrect

Select one...

D - Epstein–Barr virus (EBV)

Glandular fever should always be considered in older children with sore throats. If amoxicillin is given when a
child has EBV infection, it will cause a florid maculopapular rash. This
is one way of making the diagnosis!

Scenario 2

A 3-year-old is at nursery and her mother notices


that she is unwell, refusing foods, and has some vesicles on her
palms and toes.

Your answer was incorrect

https://mypastest.pastest.com/Secure/TestMe/Browser/436619[‫ ص‬06:35:23 11/12/1437]


MyPastest

Select one...

End Session
C - Coxsackie virus A16

Vesicles on the hands and feet, and probably in the mouth if refusing food, are the reason why hand, foot and
mouth disease has that name. It is caused by Coxsackie virus A16 and is very contagious. Nurseries often close
during an outbreak.

Scenario 3

A 7-year-old boy presents with feeling unwell and


has a macular rash all over his body. There are some ‘target
lesions’ visible.

Previous
Your answer was incorrect Question
Select one...

F - Mycoplasma pneumoniae

Target lesions are seen in erythema multiforme (EM). In an unwell child with arythema multiforme mycoplasma
infection should always be considered, as must herpes simplex infection.
Stevens–Johnson syndrome is an
important complication.
22237

Tag Question

Feedback

Difficulty: Average

Session Progress

Responses Correct: 0

Responses Incorrect: 3

Responses Total: 3

Responses - % Correct: 0%

Next Question
Blog
About Pastest
Contact Us
Help

https://mypastest.pastest.com/Secure/TestMe/Browser/436619[‫ ص‬06:35:23 11/12/1437]


MyPastest

© Pastest 2016

https://mypastest.pastest.com/Secure/TestMe/Browser/436619[‫ ص‬06:35:23 11/12/1437]


MyPastest

Prefer to use the old MyPastest? Access it here »

Back to Filters

Question 2 of 90

Theme: Antimicrobial therapy

A Intravenous benzylpenicillin and gentamicin


B Ceftriaxone 50 mg/kg i.v.
C Ceftriaxone 80 mg/kg i.v
D Oral tobramycin
E Oral trimethoprim
F Intravenous flucloxacillin
G Intravenous cefuroxime 50 mg/kg
H Oral co-amoxiclav
I Oral ciprofloxacin
J Oral cefalexin

From
the list above, select the most appropriate antibiotic for each of the clinical scenarios below. Each item may
be used once, more than once or not at all.

Scenario 1

A 3-year-old child presents to the Emergency Department unwell, tachycardic and with an evolving purpuric rash.

Your answer was incorrect

Select one...

C - Ceftriaxone 80 mg/kg i.v

This is presumed meningococcal sepsis until proven otherwise.

A
high-dose, intravenous, broad-spectrum antibiotic must be given as soon
as possible – intravenous ceftriaxone is
a commonly used antibiotic and should be given at high dose of 80 mg/kg.

Scenario 2

A 2-day-old baby collapses on the postnatal ward with probable sepsis. His mother had grown group B
streptococci on a high vaginal swab at 32 weeks.

Your answer was incorrect

https://mypastest.pastest.com/Secure/TestMe/Browser/436619[‫ ص‬06:35:53 11/12/1437]


MyPastest

Select one...
End Session
A - Intravenous benzylpenicillin and gentamicin

The most likely diagnosis is group B Streptococcal (GBS) sepsis, which is highly sensitive to penicillin – the
treatment of choice. However, it would not be used as a monotherapy because there is minimal gram-negative
cover so it is combined with gentamicin.

Scenario 3

A 7-year-old boy with cystic fibrosis develops a productive cough. He is not exhibiting any signs of respiratory
distress and is systemically well.
Previous
Your answer was incorrect Question

Select one...

H - Oral co-amoxiclav

Children with cystic fibrosis who have


a chest exacerbation but remain well will normally be treated with an
antibiotic that should cover the possibility of secondary infection with
e.g. Haemophilus influenza or
Streptococcus pneumonia. If the patient has chronic Pseudomonas infection then oral ciprofloxacin may be
prescribed. If there are signs of respiratory distress or the child is unwell, admission and use of intravenous
antibiotics (e.g. tobramycin) should be seriously considered.
22261

Tag Question

Feedback

Difficulty: Average

Session Progress

Responses Correct: 0

Responses Incorrect: 6

Responses Total: 6

Responses - % Correct: 0%
Next Question

https://mypastest.pastest.com/Secure/TestMe/Browser/436619[‫ ص‬06:35:53 11/12/1437]


MyPastest

Blog
About Pastest
Contact Us
Help

© Pastest 2016

https://mypastest.pastest.com/Secure/TestMe/Browser/436619[‫ ص‬06:35:53 11/12/1437]


MyPastest

Prefer to use the old MyPastest? Access it here »

End Session

Back to Filters

Question 3 of 90

Previous Question
Which of the following is the most appropriate treatment for the contacts of a child who is admitted with
meningococcal sepsis?

A Treat all family members with rifampicin

B Treat close contacts with rifampicin

C Treat those contacts as advised by the Consultant for Communicable Disease Control (CCDC)

D Treat close contacts: children with penicillin and adults with rifampicin

E Leave the contact tracing and treatment to CCDC

Explanation
Although it is within the remit of the CCDC – part of the public health department– to contact, trace and treat the
necessary family members, it is good practice to liaise with them and treat the contacts at risk. This is easier for
the acute clinicians to do because the contacts (usually family members) are often
with the unwell child.
22354

Tag Question

Feedback

Next Question
Difficulty: Average

https://mypastest.pastest.com/Secure/TestMe/Browser/436619[‫ ص‬06:36:13 11/12/1437]


MyPastest

Peer Responses

Session Progress

Responses Correct: 0

Responses Incorrect: 7

Responses Total: 7

Responses - % Correct: 0%

Blog
About Pastest
Contact Us
Help

© Pastest 2016

https://mypastest.pastest.com/Secure/TestMe/Browser/436619[‫ ص‬06:36:13 11/12/1437]


MyPastest

Prefer to use the old MyPastest? Access it here »

End Session

Back to Filters

Question 4 of 90

Previous Question
A 9-month-old infant presents with coryzal symptoms and a hoarse barking cough. Select the most likely
causative agent from the list below.

A Adenovirus

B Haemophilus influenzae tybe b

C Parainfluenza virus type 3

D Parvovirus

E Corynebacterium species

Explanation
These symptoms are classic of croup. The most common cause is parainfluenza 3.
22418

Tag Question

Feedback

Difficulty: Easy
Next Question
Peer Responses

https://mypastest.pastest.com/Secure/TestMe/Browser/436619[‫ ص‬06:36:33 11/12/1437]


MyPastest

Session Progress

Responses Correct: 0

Responses Incorrect: 8

Responses Total: 8

Responses - % Correct: 0%

Blog
About Pastest
Contact Us
Help

© Pastest 2016

https://mypastest.pastest.com/Secure/TestMe/Browser/436619[‫ ص‬06:36:33 11/12/1437]


MyPastest

Prefer to use the old MyPastest? Access it here »

End Session

Back to Filters

Question 5 of 90

Previous Question
A five year old child has had a fever for 3 days followed by a maculopapular rash and a severe cough.

Which of the following is the likely diagnosis?

A Roseola infantum

B Rubella

C Measles

D Chickenpox

E Glandular fever

Explanation
Roseola infantum usually occurs in children less than 2 years old, rubella and glandular fever are not normally
associated with a severe cough and the rash in chicken pox is vesicular. 30846

Tag Question

Feedback

Next Question
Difficulty: Average

Peer Responses

https://mypastest.pastest.com/Secure/TestMe/Browser/436619[‫ ص‬06:36:53 11/12/1437]


MyPastest

Session Progress

Responses Correct: 0

Responses Incorrect: 9

Responses Total: 9

Responses - % Correct: 0%

Blog
About Pastest
Contact Us
Help

© Pastest 2016

https://mypastest.pastest.com/Secure/TestMe/Browser/436619[‫ ص‬06:36:53 11/12/1437]


MyPastest

Prefer to use the old MyPastest? Access it here »

Back to Filters

Question 6 of 90

A 2-year-old girl presents with a mild fever and an erythematous rash on her cheeks. The rash spreads to the trunk
and persists for several days.

What is the BEST advice?

A Avoid pregnant women, immunocompromised patients and those with haematological conditions

B Blood testing is recommended to confirm diagnosis

C Exclude from nursery for 7 days after onset of rash.

D This is a very mild childhood illness in healthy children

E None of the above

Explanation
This is a description of parovirus infection. Avoid immunocompromised patients, pregnant women and those with
haematological conditions. Parvovirus can cause a transient aplastic crisis (and thrombocytopenia and
neutropenia).

Slapped
cheek is a mild illness with a rash and mild fever, and occurs in mainly in the spring. It is also known as
Fifth disease or human parvovirus infection.

The face rash (reddening of the cheeks) is followed by a lace-like rash on the trunk and limbs; 25% of patients are
asymptomatic.

The incubation
period is 4–20 days. Children are NOT contagious after the appearance of the rash. Unless the
child is obviously unwell, there is no need for school/nursery exclusion after the onset of the rash.

Parvovirus in pregnancy:

Most
women who are infected with parvovirus B19 in pregnancy have a satisfactory outcome. However, infection

https://mypastest.pastest.com/Secure/TestMe/Browser/436619[‫ ص‬06:37:13 11/12/1437]


MyPastest

before 20 weeks’ gestation may result in hydrops or death. Mortality rate from untreated hydrops is 50% (reduced
to 18% by transfusion); >20/40 no risk of hydrops.
End Session
Other groups are at risk of serious sequelae (haemolytic anaemia and immunocompromised individuals).

Replication
of parvovirus in the bone marrow can lead to red cell aplasia. Parvovirus can cause transient aplastic
crisis in patients with chronic anaemias, eg sickle cell, β thalassaemia and hereditary spherocytosis. Red cell
aplasia and chronic anaemia may occur in immunodeficient patients.
44013

Tag Question
Previous Question
Feedback

Difficulty: Average

Peer Responses

Session Progress

Responses Correct: 0

Responses Incorrect: 10

Responses Total: 10

Responses - % Correct: 0%

Blog
About Pastest
Contact Us
Help

© Pastest 2016
Next Question

https://mypastest.pastest.com/Secure/TestMe/Browser/436619[‫ ص‬06:37:13 11/12/1437]


MyPastest

Prefer to use the old MyPastest? Access it here »

Back to Filters

Question 7 of 90

A 13-month-old boy has a high fever for 4 days, conjunctivitis, a macular rash and is miserable. He has induration
at the site of his BCG scar.

What is the MOST likely diagnosis?

A Kawasaki disease

B Measles

C Rubella

D Streptococcal infection

E Toxic shock syndrome

Explanation
Kawasaki disease is an acute, self-limiting, systemic vasculitis of unknown aetiology that affects infants and
young children. The vasculitis involves medium and small vessels. Clinical and epidemiological characteristics
suggest toxin-induced superantigen stimulation. Person-to-person transmission has not been documented.

Diagnostic criteria for Kawasaki disease:

Fever persisting for 5 days or more


Four
of the following: – bilateral non-suppurative conjunctivitis – polymorphous exanthema – cervical
lymphadenopathy – inflammation of the tongue, lips and oral mucosa – oedema
and erythema of the hands
and feet – illness not explained by a known disease process.

Atypical Kawasaki disease implies that not all the diagnostic criteria are met but clinically the child is felt to have
Kawasaki disease, and later coronary artery dilatation or other pathognomonic criteria develop. Atypical Kawasaki

https://mypastest.pastest.com/Secure/TestMe/Browser/436619[‫ ص‬06:37:35 11/12/1437]


MyPastest

disease is more common in infants than in older children.

There are no diagnostic tests for Kawasaki disease and, although not part of the clinical criteria, the following
findings may aid diagnosis: End Session

Perianal erythema
Children are very miserable
Induration at site of BCG
Gallbladder hydrops.

There are three stages:

1. Acute
stage (1–11days): fever, conjunctivitis, oral changes, irritability, rash and lymphadenopathy. There is
usually a Previous Question
leukocytosis,
raised C-reactive protein (CRP), raised erythrocyte sedimentation rate (ESR) and a
transient disturbance of liver function in the acute stage.
2. Subacute
stage (11–30 days) lasts for 21 days and is when skin peeling, thrombocytosis and coronary artery
aneurysms occur. Thrombocytosis is a late feature after week 1.
3. Convalescent phase (>30 days): expansion of aneurysm and possible myocardial infarction.
Smaller
aneurysms may resolve (60% cases).

Complications:
coronary artery aneurysms occur in 10–40% of untreated patients. Most patients do well, but 1–
2% die of cardiac complications, eg myocardial ischaemia. Infants have a higher incidence of coronary artery
aneurysms.

Treatment: intravenous immunoglobulin and aspirin:

Intravenous
immunoglobulin (IVIG) is given within the first 10 days of the illness at a dose of 2 g/kg. This is
associated with a reduction in symptoms and
the incidence of coronary artery aneurysms. Aspirin: anti-
inflammatory and anti-platelet effects. A high dose of 100 mg/kg per day is used in the acute phase, with a
maintenance dose of 5 mg/kg per day in the convalescent phase. Follow-up: all children in whom the diagnosis of
Kawasaki disease is suspected should be referred for an echocardiogram.
44014

Tag Question

Feedback

Difficulty: Average

Peer Responses

Next Question

https://mypastest.pastest.com/Secure/TestMe/Browser/436619[‫ ص‬06:37:35 11/12/1437]


MyPastest

Session Progress

Responses Correct: 0

Responses Incorrect: 11

Responses Total: 11

Responses - % Correct: 0%

Blog
About Pastest
Contact Us
Help

© Pastest 2016

https://mypastest.pastest.com/Secure/TestMe/Browser/436619[‫ ص‬06:37:35 11/12/1437]


MyPastest

Prefer to use the old MyPastest? Access it here »

  Logged in as Ahmed fouad

Back to Filters
Question 8 of 90

A 6-year-old boy is involved in a road traffic accident and requires a splenectomy. Difficulty: Average
What is the BEST advice to avoid future pneumococcal infections?

A None of the below

B Recommend immunisation with 13-valent PCV

C Recommend immunisation with 7-valent pneumococcal conjugate vaccine (PCV)

D Recommend immunisation with 23-valent pneumococcal poysaccharide vaccine (PPV)

E Treat suspected bacterial sepsis early and with intravenous antibiotics

Explanation

There should be continued vigilance and early use of intravenous antibiotics for suspected bacterial
sepsis.
Pneumococci (Streptococcus pneumoniae)
are Gram-positive diplococci and there are more than 80
distinct serotypes. More than 60% of the population carry pneumococci in their nasopharynx, most
of these being strains of low virulence.
Diseases caused by S. pneumoniae:

Otitis media
Sinusitis
Scarlet fever
Impetigo
Pneumonia
Septicaemia
Peritonitis
Septic arthritis
Osteomyelitis
Meningitis
Brain abscess.

Pneumovax
or 23-valent PPV accounts for 96% of the pneumococccal isolates that cause serious
infection in the UK (information from the HPA 2003). Children aged >2 years who are at risk and
adults >65 years should
receive the 23-valent PPV. Reimmunisation is recommended every 5 years.
The Department of Health has a wealth of information available online (often referred to as the
Green immunisations book) – https://www.gov.uk/government/collections/immunisation-against-
infectious-disease-the-green-book

Clinical at-risk group:

Asplenia or splenic dysfunction


Chronic
respiratory disease: cystic fibrosis, cerebral palsy or neuromuscular disorder with

https://mypastest.pastest.com/Secure/TestMe/Browser/436619[‫ ص‬06:38:01 11/12/1437]


MyPastest

risk of aspiration, bronchopulmonary dysplasia


Chronic heart disease
Chronic renal disease (nephrotic, chronic renal failure and renal transplantation)
Chronic liver disease
End Session
Diabetes requiring medication
Immunosuppression (antibody deficiency, immunodeficiency, HIV, leukaemia)
Individuals with cochlear implants
Individuals with CSF leak (after trauma or major skull surgery).

The childhood immunisation schedule:

The original Prevenar (was 7-valent): PCV introduced in 2006


The latest Prevenar 13x is now 13-valent (six extra strains)
https://www.gov.uk/government/uploads/system/uploads/attachment_data/file/473570/9406_PHE_2015_Complete_Immunisation_Schedule_A4_21.pdf

Previous Question 44015

Tag Question Feedback

Blog
About Pastest
Contact Us
Help

© Pastest 2016

Next Question

https://mypastest.pastest.com/Secure/TestMe/Browser/436619[‫ ص‬06:38:01 11/12/1437]


MyPastest

Prefer to use the old MyPastest? Access it here »

  Logged in as Ahmed fouad

End Session Your subsc

MRCPC

Back to Filters
Question 9 of 90
 My Acco

 Shop

A 29-week gestation baby on the neonatal intensive care unit is 8/52 old.  Blog
Difficulty: Average
With regard to childhood immunisations, what is the BEST advice to give to ✉ Contact
parents? Peer Responses
 Mobile A

A Previous Question
Immunise at corrected gestational age of 2 months  Help

B Immunise at term
Logged in

C Immunise just before discharge from neonatal unit  Log out

D Immunise now
Session Progress

E Warn parents of the increased risk of side effects from Responses Correct: 0
immunisations in preterm infants Responses Incorrect: 13

Responses Total: 13
Explanation
Responses - % Correct: 0%

Preterm babies may be at increased risk of infection. They should be


immunised in line with the recommended schedule from 2 months of age
after birth, no matter how preterm they are.
The immunisation schedule current at Spring 2016 may be found here:
https://www.gov.uk/government/uploads/system/uploads/attachment_data/file/523050/PHE_Complete_Immunisation_Schedule_SPRING16.pdf

NB:
This schedule is updated regularly - please ensure you use the most up-
to-date schedule when revising immunisations. An internet search of 'UK
immunisation schedule 2016' will give you the most recent version. Please
advise us through feedback, if this link requires updating. Thank you.

44016

Tag Question Feedback

Blog
About Pastest
Contact Us
Help

© Pastest 2016 Next Question

https://mypastest.pastest.com/Secure/TestMe/Browser/436619[‫ ص‬06:38:22 11/12/1437]


MyPastest

Prefer to use the old MyPastest? Access it here »

Back to Filters

Question 10 of 90

A 10-year-old girl is brought into hospital after a camping trip in the New Forest. She has developed a ring-like
rash on her leg.

What is the next best step?

A Prescribe amoxicillin (if she is not penicillin allergic)

B Prescribe an antihistamine

C Prescribe doxycyline

D Take acute and convalescent borrelia serology and arrange outpatient follow-up

E Take blood for borrelia serology

Explanation
The standard treatment course is 2 weeks in a case such as this.

This history is typical of Lyme disease (https://www.gov.uk/government/collections/lyme-disease-guidance-data-


and-analysis).
Lyme borreliosis is an infection caused by spirochaete bacteria carried
by ticks. The ixodid tick is
often found on deer. Late spring, early summer and autumn are peak times for tick feeding (and camping!). At
least 50% of infections acquired in the UK are known to have occurred in
the southern counties. High-risk areas in
the UK include the New Forest, Exmoor, the Lake District and the Scottish highlands.

Avoidance
of tick-infested areas, the use of insect repellents and early removal of ticks are important because
there is no vaccine against Borrelia spp.

Symptoms

The
most common symptom is a rash (may appear after 3–30 days, spreading from the site of the bite). The ‘bull’s
eye’
rash or erythema migrans (formerly erythema chronicum migrans) may be seen with Lyme disease; it may be
painless or can feel itchy and hot. Flu-like symptoms, facial (Bell) palsy, nerve damage, meningitis, chronic

https://mypastest.pastest.com/Secure/TestMe/Browser/436619[‫ ص‬06:38:45 11/12/1437]


MyPastest

fatigue, headaches, myalgia and arthritis may also occur.

Empirical treatment with antibiotics is advised. Blood should also be taken to confirm the diagnosis (with acute
and convalescent samples taken for borrelia serology). IgM antibodies usuallyEndoccurSession
3 weeks after
infection, IgG
after 4–6 weeks. Usually the acute phase is indicated by high titres of IgM antibodies.

Treatment with antibiotics is useful for all stages of Lyme disease but is most successful early in the course of the
illness.

In general:

Oral amoxicillin (children >12 years: doxycycline)


2/52: erythema migrans and isolated facial palsy (Bell palsy)
4/52:
Lyme arthritis. Just please clarify that standard treatment is two weeks Antihistamines may be tried.
Previous
Doxycycline Question
is prescribed in those aged >12 years or on an individual basis after microbiology advice.

Antihistamines
may be tried. Doxycycline is prescribed in those aged >12 years or on an individual basis after
microbiology advice.

http://cks.nice.org.uk/lyme-disease
44019

Tag Question

Feedback

Difficulty: Average

Peer Responses

Session Progress

Responses Correct: Next Question 0

Responses Incorrect: 14

https://mypastest.pastest.com/Secure/TestMe/Browser/436619[‫ ص‬06:38:45 11/12/1437]


MyPastest

Responses Total: 14

Responses - % Correct: 0%

Blog
About Pastest
Contact Us
Help

© Pastest 2016

https://mypastest.pastest.com/Secure/TestMe/Browser/436619[‫ ص‬06:38:45 11/12/1437]


MyPastest

Prefer to use the old MyPastest? Access it here »

Back to Filters

Question 11 of 90

A 3-year-old boy develops chickenpox. He complains of severe and increasing pain in his leg. There is rapidly
developing erythema and
discoloration of the skin.

What is the most likely aetiology?

A Disseminated intravascular coagulation

B Group A streptococcus

C Staphylococcus aureus

D Staphylococcus epidermidis

E Staphylococcal toxin

Explanation
Necrotising fasciitis should be suspected
in any child with a history of varicella infection and an increasing
complaint of pain and swelling in an extremity or other body area, associated with increasing fever, erythema and
lethargy.

Management of necrotising fasciitis to prevent muscle necrosis, major limb dysfunction and death:

Resuscitate appropriately
Early and aggressive surgical debridement
Intensive antibiotic therapy (usually including clindamycin).

Clinicians
should be alert to the possibility of increased risk of necrotising fasciitis when ibuprofen (non-steroidal
anti-inflammatory drug or NSAID)
is administered for varicella virus infection. The mechanism may be by either
impairment of the immune response or masking the symptoms of secondary infection, leading to delayed

https://mypastest.pastest.com/Secure/TestMe/Browser/436619[‫ ص‬06:39:08 11/12/1437]


MyPastest

diagnosis and treatment. NSAIDs are cyclo-oxygenase inhibitors and may have adverse effects on neutrophils,
cell-mediated immunity and cytokines.
44020
End Session

Tag Question

Feedback

Difficulty: Average
Previous Question
Peer Responses

Session Progress

Responses Correct: 0

Responses Incorrect: 15

Responses Total: 15

Responses - % Correct: 0%

Blog
About Pastest
Contact Us
Help

© Pastest 2016

Next Question

https://mypastest.pastest.com/Secure/TestMe/Browser/436619[‫ ص‬06:39:08 11/12/1437]


MyPastest

Prefer to use the old MyPastest? Access it here »

Back to Filters

Question 12 of 90

A 12-year-old girl presents with upper respiratory tract symptoms and a high fever for 3 days followed by
erythema and swelling around the left eye. She complains of double vision.

What is the next BEST step?

A Commence intravenous antibiotics and organise urgent CT scan of head

B Commence oral antibiotics

C Prescribe oral antihistamines

D Reassess after 48 hours of intravenous antibiotics

E Refer for ophthalmology outpatient assessment

Explanation
This is a case of orbital cellulitis which may cause blindness and is potentially fatal. Management would be to
commence intravenous antibiotics and organise an urgent CT scan of
the head.

The following may be associated with fever, headache and systemic malaise:

Proptosis
Pain associated with blurred vision – painful ophthalmoplegia
Evidence of optic neuropathy (optic disc oedema)

Orbital
cellulitis may occur as a result of either extension of periorbital (preseptal cellulitis –
secondary to sinusitis) or orbital trauma.

Management of orbital cellulitis:

https://mypastest.pastest.com/Secure/TestMe/Browser/436619[‫ ص‬06:39:29 11/12/1437]


MyPastest

CT of the sinuses and orbit if intracranial abscess is suspected


Refer to ophthalmologists and ENT surgeons End Session
Broad-spectrum intravenous antibiotics ± surgery if there is evidence of an orbital infection

44021

Tag Question

Feedback
Previous Question

Difficulty: Average

Peer Responses

Session Progress

Responses Correct: 0

Responses Incorrect: 16

Responses Total: 16

Responses - % Correct: 0%

Blog
About Pastest
Contact Us
Help

© Pastest 2016

Next Question

https://mypastest.pastest.com/Secure/TestMe/Browser/436619[‫ ص‬06:39:29 11/12/1437]


MyPastest

Prefer to use the old MyPastest? Access it here »

Back to Filters

Question 13 of 90

A 7-year-old child with sickle cell disease develops a painful limp with restriction of hip movement. Two days
previously he had a sore throat and fever. He now has a normal hip ultrasound scan.

WBC 26 x 109/l

Neutrophils 24.9 x 109/l

Hb 9.7 g/dl

ESR 87

CRP 110

What is the MOST likely diagnosis?

A Avascular necrosis of femoral head

B Irritable hip

C Post-streptococcal arthritis

D Salmonella osteomyelitis

E Septic arthritis

Explanation
The correct answer is salmonella osteomyelitis

Children
with sickle cell disease are at risk of vascular occlusive crises (and the long-term complication of
avascular necrosis), septic arthritis and osteomyelitis of the joints.

This child had a fever, making infection most likely. A normal ultrasound scan excludes a
septic arthritis. Post-
streptococcal arthritis typically occurs at least 10 days (usually 2–3/52) after the infection has cleared and
is felt to
be an autoimmune response.

https://mypastest.pastest.com/Secure/TestMe/Browser/436619[‫ ص‬06:39:51 11/12/1437]


MyPastest

There is some debate as to whether Salmonella sp. or Staphylococcus aureus causes the most common
presentation of osteomyelitis in patients with sickle cell.
End Session
Salmonella
osteomyelitis is rare in children; however, in patients with sickle cell disease it is the responsible
pathogen in more than 50% of cases.

The
differentiation between the much more common bone crisis due to vascular occlusive crises and osteomyelitis
is difficult to make. Antibiotic therapy should cover for Salmonella sp. and Staph. aureus.

Adequate analgesia and fluid hydration are important aspects in the management of sickle cell disease.
44022

Previous Question
Tag Question

Feedback

Difficulty: Average

Peer Responses

Session Progress

Responses Correct: 0

Responses Incorrect: 17

Responses Total: 17

Responses - % Correct: 0%

Blog
About Pastest
Contact Us
Help
Next Question
© Pastest 2016

https://mypastest.pastest.com/Secure/TestMe/Browser/436619[‫ ص‬06:39:51 11/12/1437]


MyPastest

https://mypastest.pastest.com/Secure/TestMe/Browser/436619[‫ ص‬06:39:51 11/12/1437]


MyPastest

Prefer to use the old MyPastest? Access it here »

Back to Filters

Question 14 of 90

A 7-year-old develops a fever with sore throat, cervical lymphadenopathy and palatal petechiae. The monospot
test is negative.

Which investigation is most likely to lead to a diagnosis?

A Blood culture for MC&S

B Blood film

C CMV (cytomegalovirus) in urine

D EBV (Epstein–Barr virus) serology

E Throat swab for MC&S

Explanation
This is a description of Epstein–Barr virus

Infectious
mononucleosis (glandular fever) is caused by infection with the EBV. An infectious mononucleosis-like
illness can be caused by other agents including CMV, adenovirus and toxoplasmosis.

EBV
infection is often subclinical. Clinical infection is rare in the preschool group. Spread is by transmission of
oral secretions (‘kissing disease’).

The clinical features are of fever, pharyngitis and lymphadenopathy.

The
syndrome often persists for some weeks and a postviral fatigue syndrome
can occur. The appearance of a
maculopapular rash after the administration of ampicillin is quite characteristic. Splenomegaly is seen in 50%. A
smaller number of patients develop mild jaundice and hepatomegaly.

Laboratory features include an atypical lymphocytosis, mild thrombocytopenia and a transient elevation of the
transaminases. Confirmation is by the Paul Bunnell test or EBV serology/PCR.

https://mypastest.pastest.com/Secure/TestMe/Browser/436619[‫ ص‬06:40:15 11/12/1437]


MyPastest

It
is important to realise that, particularly in children <5 years, the
Paul Bunnell test can be negative in glandular
fever due to EBV infection. In such cases the presence of IgM antibody to viral capsid antigen or PCR is
diagnostic.
End Session
Complications
are rarely seen but include splenic rupture, airway obstruction, Guillain–Barré syndrome, nerve VII
palsy, agranulocytosis and pancarditis. Treatment is supportive.

Infections commonly associated with an atypical lymphocytosis:

CMV
Malaria
Toxoplasmosis
TB
Mumps. Previous Question

EBV is associated with:

Nasopharyngeal carcinoma
Burkitt lymphoma
B-cell-driven lymphomas.

Differential diagnosis of cervical lymphadenopathy:

Cervical abscess
TB
Cat-scratch fever
Mumps
Salivary stone.

44023

Tag Question

Feedback

Difficulty: Average

Peer Responses
Next Question

https://mypastest.pastest.com/Secure/TestMe/Browser/436619[‫ ص‬06:40:15 11/12/1437]


MyPastest

Session Progress

Responses Correct: 0

Responses Incorrect: 18

Responses Total: 18

Responses - % Correct: 0%

Blog
About Pastest
Contact Us
Help

© Pastest 2016

https://mypastest.pastest.com/Secure/TestMe/Browser/436619[‫ ص‬06:40:15 11/12/1437]


MyPastest

Prefer to use the old MyPastest? Access it here »

Back to Filters

Question 15 of 90

A 16-year-old girl develops tender, raised, purple lesions on


her shins in addition to persistent cervical
lymphadenopathy. She has a
new kitten. She is on the oral contraceptive pill.

What is the most likely cause of her lesions?

A Bartonella henselae

B Epstein–Barr virus

C Oral contraceptive pill

D Streptococcal infection

E Tuberculosis

Explanation
The oral contraceptive pill does cause erythema nodosum but, with cervical lymphadenopathy and history of a
kitten, cat-scratch disease is the most likely diagnosis in this case ( https://www.evidence.nhs.uk/Search?
q=cat+scratch ).

Cat-scratch
disease often begins with a small papule developing at the site of a cat (often kitten) scratch. Nearby
lymph nodes become swollen and can persist for months. Other symptoms include fever, malaise, headache and
poor appetite. A blood test for Bartonella henselae antibodies confirms the diagnosis.

In healthy individuals antibiotics are not recommended.

Causes of erythema nodosum:

Idiopathic
Streptococcal infection
TB

https://mypastest.pastest.com/Secure/TestMe/Browser/436619[‫ ص‬06:40:46 11/12/1437]


MyPastest

Leptospirosis
Histoplasmosis
EBV infection End Session
Herpes simplex virus
Yersinia sp.
Sulphonamides
Oral contraceptive pill
Systemic lupus erythematosus (SLE)
Crohn disease
Ulcerative colitis
Behçet syndrome
Previous Question
Sarcoidosis
Hodgkin disease

44024

Tag Question

Feedback

Difficulty: Average

Peer Responses

Session Progress

Responses Correct: 0

Responses Incorrect: 19
Next Question
Responses Total: 19

Responses - % Correct: 0%

https://mypastest.pastest.com/Secure/TestMe/Browser/436619[‫ ص‬06:40:46 11/12/1437]


MyPastest

Blog
About Pastest
Contact Us
Help

© Pastest 2016

https://mypastest.pastest.com/Secure/TestMe/Browser/436619[‫ ص‬06:40:46 11/12/1437]


MyPastest

Prefer to use the old MyPastest? Access it here »

Back to Filters

Question 16 of 90

An otherwise healthy 3-year-old developed a persistent submandibular swelling with no systemic symptoms or
fever and has a normal white count. Treatment with co-amoxiclav was not beneficial. Needle aspiration resulted in
formation of a fistula.

What is the most likely diagnosis?

A Adenovirus

B Atypical mycobacterial infection

C Cat-scratch disease

D Lymphoma

E Mycobacterium tuberculosis

Explanation
The is likely to prove to be a diagnosis of atypical mycobacterial infection (ATB). The key point is that of fistula
formation in a previously well child with no response to antibiotics. Needle aspiration and incomplete excision of
atypical mycobacteria result in chronic sinus or fistula formation.

Important differentials of persistent cervical lymphadenopathy

Tuberculous adenitis
EBV
Adenovirus
CMV
Cat-scratch disease
Brucellosis

https://mypastest.pastest.com/Secure/TestMe/Browser/436619[‫ ص‬06:41:09 11/12/1437]


MyPastest

Actinomycosis
Toxoplasmosis
Malignancies (especially lymphoma) End Session
Cystic hygroma.

Generalised
lymphadenopathy is associated with most viral causes. Kawasaki disease is an important cause of
cervical lymphadenopathy. Ultrasonography may be helpful in the work-up.

TB: often a history of TB contact, weight loss or fevers. Mantoux/other TB testing indicated.
Cat-scratch disease: history of scratch from kitten (check for Bartonella henselae antibodies).
Lymphoma/leukaemia:
usually associated with larger nodes (>3 cm diameter), may be firm, fixed and
Previous
painless. Look Question
for associated hepatosplenomegaly, pallor and bruising.
Adenovirus: often associated with flu-like illness and generalised lymphadenopathy (check serology/PCR
for adenovirus).

44025

Tag Question

Feedback

Difficulty: Average

Peer Responses

Session Progress

Responses Correct: 0

Responses Incorrect: Next Question 20

Responses Total: 20

https://mypastest.pastest.com/Secure/TestMe/Browser/436619[‫ ص‬06:41:09 11/12/1437]


MyPastest

Responses - % Correct: 0%

Blog
About Pastest
Contact Us
Help

© Pastest 2016

https://mypastest.pastest.com/Secure/TestMe/Browser/436619[‫ ص‬06:41:09 11/12/1437]


MyPastest

Prefer to use the old MyPastest? Access it here »

Back to Filters

Question 17 of 90

A baby is born with growth retardation, microcephaly, hepatosplenomegaly and petechiae. Cranial ultrasound scan
demonstrates hydrocephalus.

What is the most likely diagnosis?

A Congenital CMV

B Congenital herpes simplex virus

C Congenital rubella

D Congenital syphilis

E Congenital toxoplasmosis

Explanation
The most likely diagnosis is congenital toxoplasmosis. The TORCH screen (toxoplasmosis, rubella,
cytomegalovirus and herpes) is often used for the investigation of unexpected small-for-date babies. It is better to
test for specific conditions depending on clinical presentation.

Hydrocephalus is a characteristic of congenital toxoplasmosis and not seen in congenital rubella or CMV
infections.

Microcephaly and growth retardation may be seen in all three infections.

Congenital rubella

Sensorineural hearing loss


Microcephaly
Cataracts
Growth retardation

https://mypastest.pastest.com/Secure/TestMe/Browser/436619[‫ ص‬06:41:33 11/12/1437]


MyPastest

Hepatosplenomegaly
Petechiae (‘blue berry muffin’ rash
Cardiac anomalies
End Session
Bony involvement.

Congenital CMV

Sensorineural hearing loss


Microcephaly
Periventricular calcification (compare with congenital toxoplasmosis: widespread calcifications)
Chorioretinitis
Previous Question
Pronounced growth retardation.

Congenital syphilis

High
transmission rate; 40% mortality rate if untreated. Consider in a very ‘snuffly’ child with
hepatosplenomegaly and rash. Osteochondritis and periostitis.

Congenital herpes simplex virus - Cutaneous scars or vesicles. Microcephaly and intracranial calcifications.

Congenital toxoplasmosis

When previously uninfected women become infected with Toxoplasma sp. for
the first time in pregnancy, the
infection can be passed on to the fetus and cause congenital toxoplasmosis. There is a 40% risk of transmission if
the mother is infected during pregnancy. The risk of infection is higher the more advanced the pregnancy.
However, manifestations are more serious the earlier infection occurs. The classic triad of retinochoroiditis,
hydrocephalus and intracerebral calcification is uncommon, and the clinical manifestations are usually non-
specific. If infection is diagnosed during pregnancy then termination is an option; alternatively spiramycin has
been given (see www.hpa.org.uk for flow chart of management in pregnancy
) Neither pyrimethamine nor
sulfadiazine can be given during the first trimester of pregnancy because both are teratogenic. They are used later
in pregnancy. Of congenitally infected infants 90% are asymptomatic in the neonatal period. The infant may
develop clinical symptoms months or years after birth, usually with eye lesions.
44026

Tag Question

Feedback

Difficulty: Average
Next Question
Peer Responses

https://mypastest.pastest.com/Secure/TestMe/Browser/436619[‫ ص‬06:41:33 11/12/1437]


MyPastest

Session Progress

Responses Correct: 0

Responses Incorrect: 21

Responses Total: 21

Responses - % Correct: 0%

Blog
About Pastest
Contact Us
Help

© Pastest 2016

https://mypastest.pastest.com/Secure/TestMe/Browser/436619[‫ ص‬06:41:33 11/12/1437]


MyPastest

Prefer to use the old MyPastest? Access it here »

Back to Filters

Question 18 of 90

A toddler presents with a 3-day history of high fever. He develops a truncal rash that appears when the fever
settles. He has a simple febrile convulsion.

What is the MOST likely diagnosis?

A Adenovirus

B Enterovirus

C Human herpesvirus 6 (HHV-6)

D Meningococcal septicaemia

E Parvovirus B19

Explanation
HHV-6 (roseola infantum or exanthem subitum)

This
is a common cause of febrile seizures in children aged 6 months to 2 years. Sudden onset of high fever (up to
40°C), which abruptly stops
at day 3–4, at which point a rash may appear. The rash starts on the trunk and spreads
to the face and limbs. This rash is found in a minority of patients.

Children can contract primary HHV-6 without a rash. Encephalitis may develop in children with HHV-6 infection.
HHV-6 may increase the severity of CMV infection in immunocompromised individuals and transplant recipients.
It induces bone marrow suppression, respiratory failure and other problems in transplant or stem cell recipients.
44027

Next Question

https://mypastest.pastest.com/Secure/TestMe/Browser/436619[‫ ص‬06:42:00 11/12/1437]


MyPastest

Tag Question

Feedback End Session

Difficulty: Average

Peer Responses

Previous Question

Session Progress

Responses Correct: 0

Responses Incorrect: 22

Responses Total: 22

Responses - % Correct: 0%

Blog
About Pastest
Contact Us
Help

© Pastest 2016

https://mypastest.pastest.com/Secure/TestMe/Browser/436619[‫ ص‬06:42:00 11/12/1437]


MyPastest

Prefer to use the old MyPastest? Access it here »

Back to Filters

Question 19 of 90

A 5-year-old child presents with chronic diarrhoea, generalised lymphadenopathy, oral candida infection and
bilateral parotitis.

What is the MOST likely diagnosis?

A Acute leukaemia

B Chronic granulomatous disease

C Di George syndrome

D HIV

E Wiscott–Aldrich syndrome

Explanation
This child has signs of immunodeficiency and the key point in the history that points to HIV is the bilateral
parotitis.

Children
with HIV can present as failure to thrive or pyrexia of unknown origin,
and have generalised
lymphadenopathy and/or hepatosplenomegaly with candidiasis and parotitis. They may present with infections
secondary to immunodeficiency.

Diagnosis (WHO recommendations)

In infants exposed to HIV, the mother’s antibodies to HIV may be found in the child’s blood up to age 18 months.
Accurate HIV testing <18/12 requires specialised testing. The appropriate resource depends on availability of
resources and expertise.

Age <9 months PCR


Age >9 months, consider antibody testing.)

https://mypastest.pastest.com/Secure/TestMe/Browser/436619[‫ ص‬06:42:27 11/12/1437]


MyPastest

Note that antibody testing may not be diagnostic in children <18/12. Virology testing is recommended for children
<18/12.

Management principles

Avoid live vaccines including live polio and BCG


Co-trimoxazole prophylaxis when CD4 counts low (to protect against Pneumocystis sp.)

highly active antiretroviral therapy (HAART) – nucleoside analogues (eg zidovudine) and non-
nucleoside inhibitors (eg nevarapine) and protease inhibitors
regular follow-up for growth and development, side effects of treatment.

Of children with AIDS in the UK, 85% acquire the disease vertically. This can occur during pregnancy, during
childbirth or through breastfeeding. Current
data show that HIV is passed on to 25% of babies born to untreated
HIV-infected mothers, but only 4% of treated HIV-infected individuals in
the USA. Internationally, the risk of
transmission is up to 48% in breastfed infants, but 24% if no breastfeeding occurs.

Risk factors for vertical transmission of HIV:

Advanced maternal disease


Symptomatic AIDS
Low CD4 count
Presence of other sexually transmitted infections
Higher viral load.
In the perinatal period:
Amnionitis
Vaginal delivery
Preterm and post-term delivery
Low birthweight
Prolonged rupture of membranes
Invasive procedures
Breastfeeding
Maternal immunity
Cigarette smoking.

Factors that reduce the transmission rate:

Delivery by caesarean section


Avoidance of breastfeeding
Antiretroviral therapy
Zidovudine (nucleoside reverse transcriptase inhibitor)

https://mypastest.pastest.com/Secure/TestMe/Browser/436619[‫ ص‬06:42:27 11/12/1437]


MyPastest

Orally from second trimester onwards


Intravenously during delivery
Orally to newborn for 6 weeks. End Session
Combination
antiretroviral therapy to women with more advanced disease or higher viral load (zidovudine
plus second nucleoside reverse transcriptase inhibitor plus non-nucleoside reverse transcriptase inhibitor or
protease inhibitor).
Other antiretroviral therapy is undergoing evaluation.
Avoidance of rupturing of membranes if possible
Avoidance of invasive procedures, eg fetal scalp blood sampling
Passive immunisation of mother and child with HIV hyperimmune immunoglobulin
Vitamin A supplementation to mother.
Previous Question
Chronic granulomatous disease

This is a rare genetic disorder resulting in abnormality of neutrophil membrane and inability to produce
superoxide radicals. It is most susceptible to catalase-positive bacteria (eg Staph. aureus, Salmonella sp., E. coli,
Candida and Aspergillus spp. Most common presentation is with severe infections of the skin and lymph nodes
with abscess formation. Chronic diarrhoea, hepatosplenomegaly, pneumonia and faltering growth are common.

DiGeorge syndrome

Syndrome caused by 22q11 deletion affecting third and fourth pharyngeal pouches. Features vary widely.
Characteristic
features: congenital heart defect (especially conotruncal malformations
such as truncus arteriosus),
palate defects, learning disability and recurrent infections. Thymic hypoplasia (absent thymus on neonatal chest
radiograph) results in reduced T cells and increased infections. Hypocalcaemia results from absent parathyroid
glands and may cause hypocalcaemic tetany.

Wiscott–Aldrich syndrome

X-linked
disorder of B cells (resulting in Haemophilus sp. and Strep. pneumoniae) and T cells (resulting in herpes,
CMV and Pneumocystis sp.) causing:

immunodeficiency
chronic eczema
thrombocytopenia (very small platelets)

44028

Tag Question

Feedback Next Question

Difficulty: Average

https://mypastest.pastest.com/Secure/TestMe/Browser/436619[‫ ص‬06:42:27 11/12/1437]


MyPastest

Peer Responses

Session Progress

Responses Correct: 0

Responses Incorrect: 23

Responses Total: 23

Responses - % Correct: 0%

Blog
About Pastest
Contact Us
Help

© Pastest 2016

https://mypastest.pastest.com/Secure/TestMe/Browser/436619[‫ ص‬06:42:27 11/12/1437]


MyPastest

Prefer to use the old MyPastest? Access it here »

Back to Filters

Question 20 of 90

A mother who is hepatitis B positive asks you about the risk of vertical transmission. She is HBsAg positive,
HBeAg positive and anti-e antibody negative.

What is the best advice?

A Hepatitis B immunoglobulin should be given within 72 hours of birth

B The baby should receive both the hepatitis B immunisation (at birth, 1 and 6 months) and hepatitis B
immunoglobulin

C The baby should receive only hepatitis B immunoglobulin at birth

D The baby should receive only hepatitis B immunisation (at birth, 1 and 6 months)

E The risk of transmission is 90%.

Explanation
Vertical transmission is thought to account for 40% of hepatitis B worldwide. The hallmark of ongoing infection is
the presence of HBsAg. The presence of antibody to HBsAg alone suggests successful immunisation; its presence
along with anti-HBcAg suggests resolved infection. This is mainly thought to occur around the time of birth.

The
risk of transmission is increased to >90% if the mother is hepatitis
B ‘e’ antigen positive: advise immunisation
and hepatitis B
immunoglobulin.

Hepatitis B immunoglobulin given at birth alone reduces the risk of vertical transmission. The
effect of giving
active immunisation (hepatitis B vaccine at birth, 1, 2
and 12 months) and passive immunisation with hepatitis B
immunoglobulin
is additive.

Protection is achieved in 93% of neonates.

Active immunisation does not seem to be affected by transmitted maternal IgG.

https://mypastest.pastest.com/Secure/TestMe/Browser/436619[‫ ص‬06:42:55 11/12/1437]


MyPastest

Hepatitis
B is spread by an infected mother passing it to her baby or by sexual contact or from blood/blood
products or sharing of needles, etc. It is important to prevent transmission and be immunised.
End
Hepatitis
B immunisation is indicated for anyone at increased risk of infection, Session
eg workers who will be exposed to
blood through their occupation (healthcare workers), people who live in close contact with someone infected with
hepatitis B. Also:

Those with haemophilia requiring treatment


Prison inmates
Travellers to countries where hepatitis B is common.

44029

Previous Question

Tag Question

Feedback

Difficulty: Average

Peer Responses

Session Progress

Responses Correct: 0

Responses Incorrect: 24

Responses Total: 24

Responses - % Correct: 0%

Next Question
Blog
About Pastest
Contact Us
Help

https://mypastest.pastest.com/Secure/TestMe/Browser/436619[‫ ص‬06:42:55 11/12/1437]


MyPastest

© Pastest 2016

https://mypastest.pastest.com/Secure/TestMe/Browser/436619[‫ ص‬06:42:55 11/12/1437]


MyPastest

Prefer to use the old MyPastest? Access it here »

Back to Filters

Question 21 of 90

After travel to India, a child develops massive splenomegaly,


pancytopenia and hepatic failure. She has very dry
skin and her skin is
dark in places.

What is the MOST likely diagnosis?

A Brucellosis

B Epstein–Barr virus

C Malaria

D Schistosomiasis

E Visceral leishmaniasis

Explanation
This clinical picture is similar to that of malaria but the key point in the history is the dry, DARK skin –
blackening of the skin gave visceral leishmaniasis its common name in India: kala-azar, ‘black fever’.

Causes of massive splenomegaly:

Visceral leishmaniaisis
Tropical splenomegaly
Malaria (hyper-reactive malarial splenomegaly)
Schistosomiasis:
symptoms vary with species of the parasite; invasion of the skin may cause a rash –
‘swimmer’s itch’
Gaucher syndrome (genetic disorder resulting from glucocerebrosidase enzyme deficiency – storage
diseases. Fatty substance accumulates in spleen, liver and bone marrow)
(Myelofibrosis)

https://mypastest.pastest.com/Secure/TestMe/Browser/436619[‫ ص‬06:43:20 11/12/1437]


MyPastest

(Chronic myeloid leukaemia)

Leishmaniasis

Leishmaniasis is a zoonotic infection caused by a protozoon that belongs to Leishmania sp. It


is transmitted by
sandflies (not found on beaches but in forests/stone or mud walls). Reported by US troops stationed in Saudia
Arabia and Iraq, and also soldiers in Afghanistan.

Cutaneous (Leishmania tropica and L. brasiliensis):

<90% in Afghanistan, Iran, Peru, Saudia Arabia


presents as skin lesion that develops weeks or months after infection; round ulcers to granulation-
based ulcers
diagnosis: skin smear with Giemsa stain and microscopy

Diffuse cutaneous:

mucocutaneous: 90% in S. America


visceral
(L. donovani, L. infantum and L. chagasi): the most serious and potentially fatal; presents
with fevers, weight loss, splenomegaly (may be massive), thrombocytopenia, anaemia, leukopenia
and hypergammglobulinaemia
disease of reticuloendothelial system – mainly India, Bangladesh, S . America, Middle East and
Africa.

Visceral and cutaneous are the types most commonly diagnosed in the UK.

The
incidence of leishmaniasis is increasing because of environmental changes (urbanisation) that increase
exposure to the sandfly. Co-infection with HIV has led to the spread of leishmaniasis, typically a
rural disease, to
cities. Leishmaniasis accelerates the onset of AIDS in those with HIV.

Diagnosis

By direct visualisation of Leishman–Donovan bodies, stains with Giemsa or culture. PCR and serology may be
helpful.

Prevention

No vaccine for prevention currently. Insect avoidance measures. Impregnate mosquito net with permethrin.

Treatment

Refer
to specialist tropical centre. Several treatment options including sodium stibogluconate (pentivalent
antimonial) and amphotericin have been used.
44030
Next Question

https://mypastest.pastest.com/Secure/TestMe/Browser/436619[‫ ص‬06:43:20 11/12/1437]


MyPastest

Tag Question
End Session
Feedback

Difficulty: Average

Peer Responses

Previous Question

Session Progress

Responses Correct: 0

Responses Incorrect: 25

Responses Total: 25

Responses - % Correct: 0%

Blog
About Pastest
Contact Us
Help

© Pastest 2016

https://mypastest.pastest.com/Secure/TestMe/Browser/436619[‫ ص‬06:43:20 11/12/1437]


MyPastest

Prefer to use the old MyPastest? Access it here »

  Logged in as Ahmed fouad

Back to Filters
Question 22 of 90

A primary school trip is planned to a petting farm. Many parents have Difficulty: Average
expressed concerns about farm-related infections.
What is the BEST advice to give after handling animals at the farm? Peer Responses

A Hand gels alone are adequate protection

B Handwashing with hot running water and soap

C Handwashing with hot running water and soap, drying followed


by sanitising hand gels may provide extra benefit

D Handwashing with water is sufficient Session Progress

Responses Correct: 0

E Hand wipes alone are adequate protection Responses Incorrect: 26

Responses Total: 26
Explanation
Responses - % Correct: 0%

Washing hands in hot running water with soap followed by careful drying
after children have touched animals, removed shoes, touched fences or any
surface within the farm zone. Whilst additional use of hand gels may confer
additional benefit this does not form part of the current guidance.

The HPA (April 2010) reviewed infections between 1992 and 2009.

Diarrhoea, and even haemolytic uraemic syndrome, have been linked with
petting farms.
Hand gels and wipes are not effective in killing E. coli or Cryptosporidium
sp., which can be found in animal droppings and on contaminated surfaces
around farms so are not sufficient on their own.
https://www.gov.uk/government/uploads/system/uploads/attachment_data/file/322846/Farm_visits_avoiding_infection.pdf

44031

https://mypastest.pastest.com/Secure/TestMe/Browser/436619[‫ ص‬06:43:43 11/12/1437]


MyPastest

End Session
Tag Question Feedback

Blog
About Pastest
Contact Us
Help

© Pastest 2016

Previous Question

Next Question

https://mypastest.pastest.com/Secure/TestMe/Browser/436619[‫ ص‬06:43:43 11/12/1437]


MyPastest

Prefer to use the old MyPastest? Access it here »

Back to Filters

Question 23 of 90

A 13-year-old girl returns from a holiday to Africa. She has a fever, lethargy, headache, myalgia and diarrhoea.

What is the best NEXT investigation?

A Lumbar puncture

B Rapid influenza diagnostic test (RIDT)

C Stool for microscopy, culture, ova, cystsa and parasites

D Thick and thin films

E Throat swab for MC&S

Explanation
Flu-like symptoms of lethargy, headache and myalgia, with diarrhoea and fever are malaria until proven otherwise
in a child who has been to an infected area. Malaria should be suspected if symptoms occur within a year of return
from infected area.

Malaria is a potentially fatal disease caused by the bite of an infected female (Anopheles) mosquito with the
protozoon, Plasmodium sp. Malaria is one of the leading causes of death in the world.

Thick film = quick diagnosis


Thin film = species identification.

The
gold standard is a thick film because it screens a greater quantity of blood than a thin film but the appearance
of the organism is distorted. Thin films allow species identification. The film should be repeated if there is any
doubt. Blood cultures are important because the malaria screen may prove negative.

Malaria

https://mypastest.pastest.com/Secure/TestMe/Browser/436619[‫ ص‬06:44:08 11/12/1437]


MyPastest

Fever along with flu-like symptoms are common. Be aware of the predominance of gastrointestinal symptoms.
Incorrect or delayed diagnosis is often made assuming viral illnesses, influenza, gastroenteritis and hepatitis.
Pallor, splenomegaly and thrombocytopenia are late presentations. End Session
Poor prognostic features for malaria:

Altered conscious level


Seizures
Prostration (inability to sit/stand normally)
Respiratory distress
Metabolic acidosis
Haemoglobin <5 g/l.
Previous Question
Hypoglycaemia is common and is exacerbated by quinine treatment (which stimulates insulin secretion).

Treatment of falciparum:

Broad-spectrum intravenous antibiotic cover


Intravenous quinine
Then oral quinine for 1/52
+ Fansidar (sulfadoxine pyrimethamine).

44032

Tag Question

Feedback

Difficulty: Average

Peer Responses

Next Question

https://mypastest.pastest.com/Secure/TestMe/Browser/436619[‫ ص‬06:44:08 11/12/1437]


MyPastest

Session Progress

Responses Correct: 0

Responses Incorrect: 27

Responses Total: 27

Responses - % Correct: 0%

Blog
About Pastest
Contact Us
Help

© Pastest 2016

https://mypastest.pastest.com/Secure/TestMe/Browser/436619[‫ ص‬06:44:08 11/12/1437]


MyPastest

Prefer to use the old MyPastest? Access it here »

Back to Filters

Question 24 of 90

According to NICE guidelines regarding the measurement of body temperature, which ONE of the following
statements is considered to be true?

A The routine use of oral thermometers is recommended for children aged 0-5 years of age

B The routine use of rectal thermometers is recommended for children aged 0-5 years of age

C Infra-red tympanic thermometers should routinely be used for infants under the age of 4 weeks

D Forehead thermometers should be used first line by healthcare professionals in clinical settings.

E Healthcare professionals should consider parental reported perception as valid

Explanation
The accurate measurement of body temperature is critical to robust clinical assessment.

Neither oral nor rectal thermometers are recommended in routine clinical practice.

Whilst
electronic thermometer in the axilla, chemical dot thermometer in the axilla or infra-red tympanic
thermometer are all acceptable in children from 4 weeks to 5 years, electronic axillary readings should be used for
infants under the age of 4 weeks.

Doctors should not ignore parental perception of fever.

http://www.nice.org.uk/guidance/cg160/chapter/1-recommendations
44919

Next Question

Tag Question

https://mypastest.pastest.com/Secure/TestMe/Browser/436619[‫ ص‬06:44:31 11/12/1437]


MyPastest

Feedback End Session

Difficulty: Average

Peer Responses

Previous Question

Session Progress

Responses Correct: 0

Responses Incorrect: 28

Responses Total: 28

Responses - % Correct: 0%

Blog
About Pastest
Contact Us
Help

© Pastest 2016

https://mypastest.pastest.com/Secure/TestMe/Browser/436619[‫ ص‬06:44:31 11/12/1437]


MyPastest

Prefer to use the old MyPastest? Access it here »

Back to Filters

Question 25 of 90

Regarding the assessment of children with a fever (NICE 2013), which one of the following features suggests at
least an intermediate risk for serious illness?

A heart rate of 140 in an 8 month old

B heart rate of 150 in a 3 year old

C heart rate of 140 in a 18 month old

D heart rate of 120 in a 2 year old

E heart rate of 140 in a 1 year old

Explanation
Knowledge of the advanced paediatric life support (APLS) criteria and NICE guidelines are essential for this
examination.

Tachycardia
may reflect a serious febrile illness and measurement of the pulse rate
is a critical component of the
assessment in these children.

The following table is taken from NICE and is a guide for at least an intermediate risk for serious illness.

http://www.nice.org.uk/guidance/cg160/

Age Heart rate (bpm)

<12 months >160

12 - 24 months >150

2 - 5 years >140

https://mypastest.pastest.com/Secure/TestMe/Browser/436619[‫ ص‬06:44:51 11/12/1437]


MyPastest

44921

End Session

Tag Question

Feedback

Difficulty: Average

Peer Responses
Previous Question

Session Progress

Responses Correct: 0

Responses Incorrect: 29

Responses Total: 29

Responses - % Correct: 0%

Blog
About Pastest
Contact Us
Help

© Pastest 2016

Next Question

https://mypastest.pastest.com/Secure/TestMe/Browser/436619[‫ ص‬06:44:51 11/12/1437]


MyPastest

Prefer to use the old MyPastest? Access it here »

Back to Filters

Question 26 of 90

Previous Question
Which one of the following criteria are not in keeping with a diagnosis of Kawasaki disease?

A Fever for longer than 5 days

B Cervical lymphadenopathy

C Bilateral purulent conjunctivitis

D Skin desquamation

E Strawberry tongue

Explanation
Kawasaki disease is rare in the UK and has about x20 greater incidence in Japan.

Around 70% of cases are under five years of age.

Features
in the acute stages include fever more than five days, malaise, irritability, anorexia, lymphadenopathy
>15mm (unilateral), non-purulent conjunctivitis, strawberry tongue, fissured lips, pharyngitis, widespread rash,
skin desquamation, finger and toe oedema, aseptic meningitis (50%), diarrhoea, vomiting, arthritis, and urethritis.

Complications include cardiac arrhythmias, coronary artery aneurysms (up to 70% cases, 10% cases are persistent,
aneurysms <4mm regress, these may rupture – risk of
rupture persists into adulthood), coronary artery thrombosis,
MI, myocarditis and hepatitis.
44922

Next Question
Tag Question

https://mypastest.pastest.com/Secure/TestMe/Browser/436619[‫ ص‬06:45:13 11/12/1437]


MyPastest

Feedback

End Session
Difficulty: Average

Peer Responses

Session Progress

Responses Correct: 0

Responses Incorrect: 30

Responses Total: 30

Responses - % Correct: 0%

Blog
About Pastest
Contact Us
Help

© Pastest 2016

https://mypastest.pastest.com/Secure/TestMe/Browser/436619[‫ ص‬06:45:13 11/12/1437]


MyPastest

Prefer to use the old MyPastest? Access it here »

End Session

Back to Filters

Question 27 of 90

Previous Question
Which one of the following statements is true regarding the management of fever in children?

A Anti-pyretic agents should be used with the aim of reducing the risk of febrile convulsion

B Tepid sponging is recommended in the management of fever

C Children with fever should be stripped to reduce the risk of over-heating

D Anti-pyretic agents should be used solely with the aim of reducing body temperature

E Anti-pyretic agents should be used with the aim of reducing distress in a febrile child

Explanation
Anti-pyretic agents should be used to reduce distress but not with the sole aim of reducing body temperature.
There is no evidence they reduce the risk of febrile convulsions.

Paracetamol and ibuprofen should not be used simultaneously.

Stripping
and sponging should not be used as measures to reduce fever, they may cause distress and shivering
which can increase body temperature.
44926

Tag Question

Feedback
Next Question

https://mypastest.pastest.com/Secure/TestMe/Browser/436619[‫ ص‬06:45:36 11/12/1437]


MyPastest

Difficulty: Average

Peer Responses

Session Progress

Responses Correct: 0

Responses Incorrect: 31

Responses Total: 31

Responses - % Correct: 0%

Blog
About Pastest
Contact Us
Help

© Pastest 2016

https://mypastest.pastest.com/Secure/TestMe/Browser/436619[‫ ص‬06:45:36 11/12/1437]


MyPastest

Prefer to use the old MyPastest? Access it here »

Back to Filters

Question 28 of 90

According to NICE, which one of the following microbes is a common, healthcare associated, viral infection?

A Clostridium difficile

B Escherichia coli

C Meticillin resistant staphylococcus aureus

D Meticillin sensitive staphylococcus aureus

E Norovirus

Explanation
Healthcare associated infections (HCAi) pose a significant risk to patients and cost to the NHS.

They
may either be related to care delivered in hospitals, the community or indeed caused microbes brought in by
patients, visitors or even staff.

Common responsible bacteria include meticillin-resistant Staphylococcus aureus (MRSA), meticillin-sensitive


Staphylococcus aureus (MSSA), Clostridium difficile (C. difficile) and Escherichia coli (E. coli).

Norovirus is a virus often brought into hospitals by visitors or staff. It is commonly known as the winter vomiting
virus.

http://www.nice.org.uk/guidance/PH36
44927

Next Question

Tag Question

https://mypastest.pastest.com/Secure/TestMe/Browser/436619[‫ ص‬06:45:55 11/12/1437]


MyPastest

Feedback End Session

Difficulty: Average

Peer Responses

Previous Question

Session Progress

Responses Correct: 0

Responses Incorrect: 32

Responses Total: 32

Responses - % Correct: 0%

Blog
About Pastest
Contact Us
Help

© Pastest 2016

https://mypastest.pastest.com/Secure/TestMe/Browser/436619[‫ ص‬06:45:55 11/12/1437]


MyPastest

Prefer to use the old MyPastest? Access it here »

Back to Filters

Question 29 of 90

Previous Question
Which one of the following should be prioritised by healthcare providers to reduce healthcare associated
infections according to a report by the national audit office?

A To monitor and record hospital antibiotic prescriptions

B A whole-system approach

C To ensure staff compliance with good infection control practice

D A culture of continuous improvement

E All of the above

Explanation
In their guidance on Healthcare-associated infections: prevention and control (2011), NICE referred to a series of
recommendations to reduce preventable nosocomial infections.

The
personal and financial cost to patients and the NHS is significant. It is essential all healthcare providers
consider infection control as a priority and that they and individuals are held to account.

Clinical
Commissioning Groups are also held to account for the performance of the trusts they commission in
order to encourage quality improvement.

https://www.nice.org.uk/guidance/ph36
44928

Next Question
Tag Question

https://mypastest.pastest.com/Secure/TestMe/Browser/436619[‫ ص‬06:46:17 11/12/1437]


MyPastest

Feedback

End Session
Difficulty: Average

Peer Responses

Session Progress

Responses Correct: 0

Responses Incorrect: 33

Responses Total: 33

Responses - % Correct: 0%

Blog
About Pastest
Contact Us
Help

© Pastest 2016

https://mypastest.pastest.com/Secure/TestMe/Browser/436619[‫ ص‬06:46:17 11/12/1437]


MyPastest

Prefer to use the old MyPastest? Access it here »

Back to Filters

Question 30 of 90

According to the World Health Organisation, which one


of the following statements is true regarding the impact of
healthcare associated infections?

A 7% of hospitalised patients will acquire at least one healthcare associated infection in the developed
world

B 9% of hospitalised patients will acquire at least one healthcare associated infection in the developed
world

C 11% of hospitalised patients will acquire at least one healthcare associated infection in the developed
world

D 13% of hospitalised patients will acquire at least one healthcare associated infection in the developed
world

E 15% of hospitalised patients will acquire at least one healthcare associated infection in the developed
world

Explanation
Healthcare associated infections (HCAi) affects a significant number of patients.

7% of hospitalised patients in the developed world and 10% of those in developing countries will acquire at least
one HCAi.

With
good leadership, accountability, education and systems, the risk of acquiring HCAi can be significantly
reduced. MRSA and C. Difficile are the most common pathogens and have a significant morbidity, mortality and
socio-economic burden.

The true cost of HCAi to the NHS may exceed £1billion/year.

http://www.who.int/gpsc/country_work/gpsc_ccisc_fact_sheet_en.pdf
44930

https://mypastest.pastest.com/Secure/TestMe/Browser/436619[‫ ص‬06:46:37 11/12/1437]


MyPastest

End Session

Tag Question

Feedback

Difficulty: Average

Peer Responses

Previous Question

Session Progress

Responses Correct: 0

Responses Incorrect: 34

Responses Total: 34

Responses - % Correct: 0%

Blog
About Pastest
Contact Us
Help

© Pastest 2016

Next Question

https://mypastest.pastest.com/Secure/TestMe/Browser/436619[‫ ص‬06:46:37 11/12/1437]


MyPastest

Prefer to use the old MyPastest? Access it here »

End Session

Back to Filters

Question 31 of 90

Previous Question
Which one of the following statements is true regarding health care associated infections?

A Education does not influence the risk of healthcare associated infections

B Low
staffing levels reduces the risk of healthcare associated infections by
reducing the number of
bodies who could potentially spread disease

C Prolonged use of antibiotics protects patients against healthcare associated infections

D Robust hospital infrastructure is required to systematically reduce healthcare associated infections

E The systematic use of complex clinical procedures is required to reduce the risk of healthcare
associated infections

Explanation
The risk of acquiring healthcare associated infections is increased by over-crowding, low staffing levels, poor
education, prolonged use of antibiotics and complex procedures.

Robust structures and systems, good leadership and holding people to account reduces risks.

http://www.who.int/gpsc/country_work/gpsc_ccisc_fact_sheet_en.pdf
44931

Tag Question
Next Question
Feedback

https://mypastest.pastest.com/Secure/TestMe/Browser/436619[‫ ص‬06:46:57 11/12/1437]


MyPastest

Difficulty: Average

Peer Responses

Session Progress

Responses Correct: 0

Responses Incorrect: 35

Responses Total: 35

Responses - % Correct: 0%

Blog
About Pastest
Contact Us
Help

© Pastest 2016

https://mypastest.pastest.com/Secure/TestMe/Browser/436619[‫ ص‬06:46:57 11/12/1437]


MyPastest

Prefer to use the old MyPastest? Access it here »

End Session

Back to Filters

Question 32 of 90

Previous Question
Which one of the following statements is false regarding MRSA healthcare associated infection?

A MRSA may be transmitted through contact with contaminated towels

B MRSA infection is often associated with hospital environments due to poor hygiene practices

C MRSA may be transmitted through contact with contaminated floors

D MRSA may gain entry through urinary catheters

E MRSA infection is unique to hospital environments

Explanation
Meticillin resistant S. Aureus may be transmitted via patients, staff, visitors and contaminated dressings, towels,
sheets, floors, door handles etc.

Infection
is not unique to hospitals but often occurs in hospital environments due to the close proximity of
vulnerable patients with disease, catheters and wounds.

Poor hygiene practices significantly increase the risk of MRSA infection, bacteraemia and death.
44932

Tag Question

Feedback
Next Question

https://mypastest.pastest.com/Secure/TestMe/Browser/436619[‫ ص‬06:47:17 11/12/1437]


MyPastest

Difficulty: Average

Peer Responses

Session Progress

Responses Correct: 0

Responses Incorrect: 36

Responses Total: 36

Responses - % Correct: 0%

Blog
About Pastest
Contact Us
Help

© Pastest 2016

https://mypastest.pastest.com/Secure/TestMe/Browser/436619[‫ ص‬06:47:17 11/12/1437]


MyPastest

Prefer to use the old MyPastest? Access it here »

Back to Filters

Question 33 of 90

Previous Question
Which one of the following agents accounts for the majority of cases of ophthalmia neonatorium?

A Herpes simplex

B Herpes zoster

C Chlamydia trachomatis

D Neisseria gonorrhoeae

E Human immunodeficiency virus (HIV)

Explanation
Ophthalmia neonatorium is a conjunctivitis occurring within the first 28 days of life.

Sexually
transmitted infectious agents such as Chlamydia trachomatis and Neisseria gonorrhoeae are usually
acquired from the maternal genital tract during childbirth.

The commonest cause is Chlamydia trachomatis which accounts for up to 40% of cases.

Non sexually transmitted infective agents include S. Aureus. Viruses may also be accountable agents.

It was a notifiable condition until 2010.


44934

Next Question Tag Question

Feedback

https://mypastest.pastest.com/Secure/TestMe/Browser/436619[‫ ص‬06:47:38 11/12/1437]


MyPastest

End Session
Difficulty: Average

Peer Responses

Session Progress

Responses Correct: 0

Responses Incorrect: 37

Responses Total: 37

Responses - % Correct: 0%

Blog
About Pastest
Contact Us
Help

© Pastest 2016

https://mypastest.pastest.com/Secure/TestMe/Browser/436619[‫ ص‬06:47:38 11/12/1437]


MyPastest

Prefer to use the old MyPastest? Access it here »

Back to Filters

Question 34 of 90

Which of the following statements is true regarding the treatment of group B streptococcal carriage in expectant
mothers in order to reduce neonatal infection?

A Treatment is not required but neonates should be closely monitored after birth

B Oral antibiotics should be offered to expectant mothers once maternal group B streptococcal carriage
is confirmed

C Intravenous antibiotics should be offered during labour to high risk women for group B streptococcal
disease

D Intravenous
antibiotics should be offered immediately after childbirth to babies born to high risk
women for group B streptococcal disease

E Oral antibiotics should be offered during labour to high risk women for group B streptococcal disease

Explanation
Group B streptococcal (GBS) disease is the number
one infective cause of death in neonates which often causes
meningitis,
pneumonia and septicaemia.

At risk women should be offered intravenous antibiotics during labour as oral antibiotics are not effective.

Unlike many developed countries where GBS disease has fallen, there is currently no screening programme for
GBS in the UK.

Many
doctors and midwives advise testing at around 35-37 weeks gestation using enriched culture medium which
is more sensitive that standard swab
media but not available on the NHS.

http://gbss.org.uk/who-we-are/about-gbs/gbs-prevention/
44936

https://mypastest.pastest.com/Secure/TestMe/Browser/436619[‫ ص‬06:48:00 11/12/1437]


MyPastest

End Session
Tag Question

Feedback

Difficulty: Average

Peer Responses

Previous Question

Session Progress

Responses Correct: 0

Responses Incorrect: 38

Responses Total: 38

Responses - % Correct: 0%

Blog
About Pastest
Contact Us
Help

© Pastest 2016

Next Question

https://mypastest.pastest.com/Secure/TestMe/Browser/436619[‫ ص‬06:48:00 11/12/1437]


MyPastest

Prefer to use the old MyPastest? Access it here »

Back to Filters

Question 35 of 90

Previous Question
Which one of the following statements is true regarding maternal-foetal transmission of HIV?

A Effective therapy and precautions can reduce the risk of neonatal HIV infection from around 25% to
1%.

B Effective therapy and precautions can reduce the risk of neonatal HIV infection from around 40% to
1%.

C Effective therapy and precautions can reduce the risk of neonatal HIV infection from around 15% to
5%.

D Effective therapy and precautions can reduce the risk of neonatal HIV infection from around 50% to
5%.

E Effective therapy and precautions can reduce the risk of neonatal HIV infection from 80% to 5%.

Explanation
All pregnant women should be offered an HIV test as part of antenatal screening.

HIV may be transmitted through breast milk, during pregnancy or through childbirth.

Treatment
of HIV during pregnancy with anti-retroviral agents, delivery via elective c-section and avoiding
breastfeeding reduces the risk of neonatal HIV infection from around 25% to 1%.
44937

Next Question
Tag Question

https://mypastest.pastest.com/Secure/TestMe/Browser/436619[‫ ص‬06:48:20 11/12/1437]


MyPastest

Feedback

End Session
Difficulty: Average

Peer Responses

Session Progress

Responses Correct: 0

Responses Incorrect: 39

Responses Total: 39

Responses - % Correct: 0%

Blog
About Pastest
Contact Us
Help

© Pastest 2016

https://mypastest.pastest.com/Secure/TestMe/Browser/436619[‫ ص‬06:48:20 11/12/1437]


MyPastest

Prefer to use the old MyPastest? Access it here »

Back to Filters

Question 36 of 90

Previous Question
The risk of acquiring antenatal cytomegalovirus infection may be reduced by which one of the following?

A Immunisation of teenage girls

B Screening all pregnant women antenatally

C Anti-viral agents in CMV positive women during childbirth

D Avoidance of exposure to toddler urine

E Avoidance of handling cat litter

Explanation
Cytomegalovirus (CMV) forms part of the TORCH complex. Around 50% of UK pregnant women have
antibodies to CMV. A small number of women become infected with CMV during pregnancy although many are
asymptomatic. Women may be exposed to CMV through a number of sources including toddler urine.

Features
of congenital CMV infection include IUGR, microcephaly, hydrocephalus, cerebral calcification,
learning difficulties, hearing loss and chorioretinitis.

There is no routine CMV vaccine or antenatal screening programme. Toxoplasmosis may be acquired through
handling cat litter.
44938

Next Question Tag Question

Feedback

https://mypastest.pastest.com/Secure/TestMe/Browser/436619[‫ ص‬06:49:54 11/12/1437]


MyPastest

End Session
Difficulty: Average

Peer Responses

Session Progress

Responses Correct: 0

Responses Incorrect: 40

Responses Total: 40

Responses - % Correct: 0%

Blog
About Pastest
Contact Us
Help

© Pastest 2016

https://mypastest.pastest.com/Secure/TestMe/Browser/436619[‫ ص‬06:49:54 11/12/1437]


MyPastest

Prefer to use the old MyPastest? Access it here »

Back to Filters

Question 37 of 90

Regarding the risk of acquiring foetal varicella syndrome, which one of the following statements are true?

A The risk of foetal varicella infection is minimised through routine vaccination at age 13

B The risk of foetal varicella infection is high in women who have been ante-natally vaccinated against
varicella zoster virus

C The risk of foetal varicella infection is high in women who have been ante-natally exposed varicella
zoster virus

D The risk of foetal varicella infection is highest during the first 20 weeks of pregnancy

E Maternal shingles is a major risk factor for foetal varicella infection

Explanation
Foetal varicella syndrome may manifest with microcephaly, learning difficulties, seizures and eye, bladder and
limb defects.

The risks of complications after maternal zoster exposure, however, is low.

Chicken pox infection during the first half of pregnancy poses the highest risk, particularly between 13 and 20
weeks.

There is no routine vaccination programme although a live vaccine is available.

Infection after 36 weeks may cause neonatal chicken pox.


44939

Next Question

https://mypastest.pastest.com/Secure/TestMe/Browser/436619[‫ ص‬06:50:25 11/12/1437]


MyPastest

Tag Question

Feedback End Session

Difficulty: Average

Peer Responses

Previous Question

Session Progress

Responses Correct: 0

Responses Incorrect: 41

Responses Total: 41

Responses - % Correct: 0%

Blog
About Pastest
Contact Us
Help

© Pastest 2016

https://mypastest.pastest.com/Secure/TestMe/Browser/436619[‫ ص‬06:50:25 11/12/1437]


MyPastest

Prefer to use the old MyPastest? Access it here »

Back to Filters

Question 38 of 90

Previous Question
After a fall in incidence in tuberculosis (TB) world-wide over the last two or three decades following the
development of successful anti-TB chemotherapy, there is now an increase in cases in both adults and children.

Which of the following is least likely to constitute a risk factor for acquiring Tuberculosis?

A Brief contact with a patient with open TB

B Children of parents who originate from a country with a high prevalence of TB

C Homelessness

D Immunosuppression

E Malnutrition

Explanation
TB is increasing in the UK especially in immigrant patients and those with human immunodeficiency virus (HIV).

Risk factors include malnutrition and immunosuppression, prolonged exposure


to people with open TB,
overcrowding , homelessness, living in a country or having patients from a country of high prevalence.

Brief exposure to an affected patient is unlikely to cause infection.

TB is a notifiable disease
44940

Next Question
Tag Question

https://mypastest.pastest.com/Secure/TestMe/Browser/436619[‫ ص‬06:50:49 11/12/1437]


MyPastest

Feedback

End Session
Difficulty: Average

Peer Responses

Session Progress

Responses Correct: 0

Responses Incorrect: 42

Responses Total: 42

Responses - % Correct: 0%

Blog
About Pastest
Contact Us
Help

© Pastest 2016

https://mypastest.pastest.com/Secure/TestMe/Browser/436619[‫ ص‬06:50:49 11/12/1437]


MyPastest

Prefer to use the old MyPastest? Access it here »

Back to Filters

Question 39 of 90

Which one of the following statements is true regarding polio?

A It
is a vaccine preventable disease that mainly affects children aged 5-10
years and can take a matter of
hours before paralysis develops

B It is a bacterial infection that typically affects children under the age of three and takes years before
paralysis develops

C It
is a viral infection that typically affects children under the age of three and can take a matter of
hours before paralysis develops

D It
is a vaccine preventable bacterial infection that mainly affects children below three years of age and
can take a matter of hours before paralysis develops

E It
is a vaccine preventable viral infection that mainly affects children over three years of age and can
take a matter of days before paralysis develops

Explanation
Polio is a viral infection that is preventable through vaccination.

It typically affects children under the age of three and it may only take a few hours before paralysis develops.

Most
infected people are not affected but may transmit the disease to thousands of others through faecal matter or
respiratory droplets.

Symptomatic children may present with fatigue and malaise, headache, limb pain, headache and vomiting.

0.5% of affected children will develop flaccid paralysis.

https://www.gov.uk/government/collections/polio-guidance-data-and-analysis
44943

https://mypastest.pastest.com/Secure/TestMe/Browser/436619[‫ ص‬06:51:11 11/12/1437]


MyPastest

End Session
Tag Question

Feedback

Difficulty: Average

Peer Responses

Previous Question

Session Progress

Responses Correct: 0

Responses Incorrect: 43

Responses Total: 43

Responses - % Correct: 0%

Blog
About Pastest
Contact Us
Help

© Pastest 2016

Next Question

https://mypastest.pastest.com/Secure/TestMe/Browser/436619[‫ ص‬06:51:11 11/12/1437]


MyPastest

Prefer to use the old MyPastest? Access it here »

Back to Filters

Question 40 of 90

A 3-year-old child is brought to the emergency department by his family with fever, sweats, headache, chills and
vomiting a week after returning to the UK from Uganda.

Which one of the following tests should be arranged next to establish a rapid diagnosis of his condition?

A Blood cultures

B Blood film

C C-reactive protein

D Full blood count

E Rapid diagnostic test

Explanation
Malaria is endemic in the tropical world,
especially sub-Saharan Africa and parts of south-east Asia. P. falciparum
malaria is the most severe and potentially fatal disease and is the predominant species in Africa. This child may
have P. falciparum malaria.

In
the UK it is assumed that all cases of P. falciparum malaria are chloroquine resistant and so treatment is with
quinine, orally if possible, or intravenously if severely unwell for 3-7 days.

Rapid diagnostic dipstick testing for parasite antigen is required given the child is unwell and early treatment
essential.

Nevertheless, other infective agents should be considered including dengue fever, haemorrhagic fevers, hepatitis
and HIV.
Next Question 44944

https://mypastest.pastest.com/Secure/TestMe/Browser/436619[‫ ص‬06:51:35 11/12/1437]


MyPastest

Tag Question
End Session
Feedback

Difficulty: Average

Peer Responses

Previous Question

Session Progress

Responses Correct: 0

Responses Incorrect: 44

Responses Total: 44

Responses - % Correct: 0%

Blog
About Pastest
Contact Us
Help

© Pastest 2016

https://mypastest.pastest.com/Secure/TestMe/Browser/436619[‫ ص‬06:51:35 11/12/1437]


MyPastest

Prefer to use the old MyPastest? Access it here »

Back to Filters

Question 41 of 90

You see parents in clinic who express concern about their daughter whom they believe may have been exposed to
the Hepatitis A virus having recently spent time with a grandparent who has subsequently
been diagnosed with the
infection.

What is the typical incubation period of hepatitis A?

A The incubation period is 1-7 days

B The incubation period is 7-14 days

C The incubation period is 7-21 days

D The incubation period is 14-21 days

E The incubation period is 14-28 days

Explanation
Hepatitis A is a viral infection preventable by vaccination and good hygiene.

Transmission is by faeco-oral route, usually by ingesting water contaminated with infected faeces.

The incubation period is typically 14-28 days and symptoms include malaise, diarrhoea, abdominal pain and
sometimes jaundice.

Younger children tend to be less severely affected.

http://www.who.int/mediacentre/factsheets/fs328/en/
Next Question 44945

https://mypastest.pastest.com/Secure/TestMe/Browser/436619[‫ ص‬06:51:57 11/12/1437]


MyPastest

Tag Question
End Session
Feedback

Difficulty: Average

Peer Responses

Previous Question

Session Progress

Responses Correct: 0

Responses Incorrect: 45

Responses Total: 45

Responses - % Correct: 0%

Blog
About Pastest
Contact Us
Help

© Pastest 2016

https://mypastest.pastest.com/Secure/TestMe/Browser/436619[‫ ص‬06:51:57 11/12/1437]


MyPastest

Prefer to use the old MyPastest? Access it here »

Back to Filters

Question 42 of 90

Which one of the following infections is notifiable disease in England?

A Chicken pox

B Glandular fever

C Impetigo

D Leptospirosis

E Scarlet fever

Explanation
Scarlet fever is the only notifiable disease. See

http://www.gov.uk/guidance/notifiable-diseases-and-causative-organisms-how-to-report#list-of-notifiable-diseases

Typically
the incubation period for scarlet fever is around 2-5 days although the
range is anything from a day to a
week. It is caused by group A streptococcus. Those affected are infectious for up to 3 weeks.

Clinical features:

Fever (>38.3οc)
Headache
Lymphadenopathy
Malaise
Nausea
Rash (days 2-5 of illness, starts on neck and face, then spreads)
Red cheeks

https://mypastest.pastest.com/Secure/TestMe/Browser/436619[‫ ص‬06:52:19 11/12/1437]


MyPastest

Red/purulent tonsils
Sore throat
Strawberry tongue End Session
Tachycardia

Complications:

Cerebral abscess
Glomerulonephritis
Meningitis
Myocarditis
Previous
Osteomylitis Question
Otitis media
Pneumonia
Quinsy
Sinusitis
Rheumatic fever
Sepsis
Skin desquamation
Venous sinus thrombosis

45056

Tag Question

Feedback

Difficulty: Average

Peer Responses

Next Question

https://mypastest.pastest.com/Secure/TestMe/Browser/436619[‫ ص‬06:52:19 11/12/1437]


MyPastest

Session Progress

Responses Correct: 0

Responses Incorrect: 46

Responses Total: 46

Responses - % Correct: 0%

Blog
About Pastest
Contact Us
Help

© Pastest 2016

https://mypastest.pastest.com/Secure/TestMe/Browser/436619[‫ ص‬06:52:19 11/12/1437]


MyPastest

Prefer to use the old MyPastest? Access it here »

Back to Filters

Question 43 of 90

In order to reduce the spread of norovirus, how would you advise infected patients and their families?

A Recovery usually occurs within 1-2 days after which return to school is permitted

B Recovery
usually occurs within 1-2 days but children should not return to school
for at least 2 days
after the last episode of vomiting and diarrhoea

C Recovery
usually occurs within 1-2 days but children should not return to school
for at least 3 days
after the last episode of vomiting and diarrhoea

D Recovery
usually occurs within 1-2 days but children should not return to school
for at least 5 days
after the last episode of vomiting and diarrhoea

E Recovery
usually occurs within 1-2 days but children should not return to school
for at least seven
days after the last episode of vomiting and diarrhoea

Explanation
Norovirus is highly infectious and typically strikes in the winter hence its other name "winter vomiting bug".

It
classically causes vomiting and diarrhoea. Alcohol gels may not kill viral particals hence washing with warm
soapy water and meticulous hygiene measures are required including keeping away from institutions such as
school and nursery for at least 48 hours after the last episode of vomiting or diarrhoea.

45057

Next Question

Tag Question

https://mypastest.pastest.com/Secure/TestMe/Browser/436619[‫ ص‬06:52:49 11/12/1437]


MyPastest

Feedback End Session

Difficulty: Average

Peer Responses

Previous Question

Session Progress

Responses Correct: 0

Responses Incorrect: 47

Responses Total: 47

Responses - % Correct: 0%

Blog
About Pastest
Contact Us
Help

© Pastest 2016

https://mypastest.pastest.com/Secure/TestMe/Browser/436619[‫ ص‬06:52:49 11/12/1437]


MyPastest

Prefer to use the old MyPastest? Access it here »

Back to Filters

Question 44 of 90

A 3-month-old child is brought to see you with bronchiolitis.

What oxygen saturation (whilst breathing in air) is an indication for admission to hospital?

A Less than 91%

B Less than 92%

C Less than 93%

D Less than 94%

E Less than 95%

Explanation
Bronchiolitis typically occurs in winter/spring and may be caused by respiratory syncytial virus (RSV),
parainfluenza virus, influenza or adenovirus. It mainly affects infants less than one
year of age. Premature and
those with congenital diseases are particularly at risk of admission. Breastfeeding is considered to be protective.

Admission criteria (NICE Guidance 9 see link below):

When assessing a child in a secondary care setting, admit them to hospital if they have any of the following:

apnoea (observed or reported)


persistent oxygen saturation of less than 92% when breathing air
inadequate
oral fluid intake (50–75% of usual volume, taking account of risk
factors [see recommendation
1.3.3] and using clinical judgement)
persisting
severe respiratory distress, for example grunting, marked chest recession, or a respiratory rate of
over 70 breaths/minute.

https://mypastest.pastest.com/Secure/TestMe/Browser/436619[‫ ص‬06:53:20 11/12/1437]


MyPastest

https://www.nice.org.uk/guidance/ng9

End Session
45058

Tag Question

Feedback

Previous Question
Difficulty: Average

Peer Responses

Session Progress

Responses Correct: 0

Responses Incorrect: 48

Responses Total: 48

Responses - % Correct: 0%

Blog
About Pastest
Contact Us
Help

© Pastest 2016

Next Question

https://mypastest.pastest.com/Secure/TestMe/Browser/436619[‫ ص‬06:53:20 11/12/1437]


MyPastest

Prefer to use the old MyPastest? Access it here »

Back to Filters

Question 45 of 90

Which one of the following statements is true regarding scabies?

A Mites can survive away from skin for up to 12 hours

B Mites can survive away from skin for up to 24 hours

C Mites can survive away from skin for up to 36 hours

D Mites can survive away from skin for up to 48 hours

E Mites can survive away from skin for up to 72 hours

Explanation
Scabies is a common condition which is often associated with social deprivation and crowded conditions such as
nursing homes. Skin contact for a significant time is required for person-person infection. It may be contracted
from bed linen, towels and furniture.

Clinical features:
Patients may be asymptomatic for 4 weeks unless previously infected. Pruritic skin lesions, papules and vesicles
may be noted. The burrows may be ≥15mm and are found in flexures and fingerwebs, the mite may be seen at one
end. Skin may look excoriated and eczematous. Super-infestation (Norwegian/crusted scabies) confers a mortality
risk.

Management
is with aqueous-based preparations such as malathon (Derbac M) and permethrin 5%. Secondary
infections should be treated with systemic antibiotics and residual dry skin with emollients.

Parents should be advised to wash all clothing, bedlinen and towels a the end of treatment.
45059
Next Question

https://mypastest.pastest.com/Secure/TestMe/Browser/436619[‫ ص‬06:53:40 11/12/1437]


MyPastest

Tag Question
End Session
Feedback

Difficulty: Average

Peer Responses

Previous Question

Session Progress

Responses Correct: 0

Responses Incorrect: 49

Responses Total: 49

Responses - % Correct: 0%

Blog
About Pastest
Contact Us
Help

© Pastest 2016

https://mypastest.pastest.com/Secure/TestMe/Browser/436619[‫ ص‬06:53:40 11/12/1437]


MyPastest

Prefer to use the old MyPastest? Access it here »

Back to Filters

Question 46 of 90

Which one of the following infections is typically associated with orchitis?

A Chicken pox

B Measles

C Mumps

D Rubella

E Scarlet fever

Explanation
Mumps is a viral infection and notifiable illness.

https://www.gov.uk/guidance/notifiable-diseases-and-causative-organisms-how-to-report#list-of-notifiable-
diseases

Epidemiology:

Clinical features:

Incubation 18-21 days.

Inflammatory phase:

Parotitis
Orchitis
Oophoritis

https://mypastest.pastest.com/Secure/TestMe/Browser/436619[‫ ص‬06:54:02 11/12/1437]


MyPastest

Complications:

Infertility (male) End Session


Meningitis
Encephalitis
Pancreatitis
Spontaneous
miscarriage (slight risk if contracted 12-16 weeks' gestation.) No malformation occurs if
contacted during pregnancy.

45060

Previous Question

Tag Question

Feedback

Difficulty: Average

Peer Responses

Session Progress

Responses Correct: 0

Responses Incorrect: 50

Responses Total: 50

Responses - % Correct: 0%

Next Question
Blog
About Pastest
Contact Us
Help

© Pastest 2016

https://mypastest.pastest.com/Secure/TestMe/Browser/436619[‫ ص‬06:54:02 11/12/1437]


MyPastest

https://mypastest.pastest.com/Secure/TestMe/Browser/436619[‫ ص‬06:54:02 11/12/1437]


MyPastest

Prefer to use the old MyPastest? Access it here »

End Session

Back to Filters

Question 47 of 90

Previous Question
A 14-year-old boy presents with a florid morbilliform type rash after being given amoxicillin for an upper
respiratory chest infection (URTI).

What is the most likely cause of the URTI?

A Glandular fever

B Measles

C Parvovirus

D Rhinovirus

E Streptococcus A

Explanation
It is likely this patient had glandular fever and was inadvertently given amoxicillin causing an immune reaction.
Measles would be unlikely given the MMR vaccination programme. Parvovirus causes slapped cheek syndrome
and can also account for foetal hydrops and aplastic anaemia. Complications of this delayed hypersensitivity
reaction includes Steven-Johnson syndrome.
45066

Tag Question
Next Question
Feedback

https://mypastest.pastest.com/Secure/TestMe/Browser/436619[‫ ص‬06:54:22 11/12/1437]


MyPastest

Difficulty: Average

Peer Responses

Session Progress

Responses Correct: 0

Responses Incorrect: 51

Responses Total: 51

Responses - % Correct: 0%

Blog
About Pastest
Contact Us
Help

© Pastest 2016

https://mypastest.pastest.com/Secure/TestMe/Browser/436619[‫ ص‬06:54:22 11/12/1437]


MyPastest

Prefer to use the old MyPastest? Access it here »

Back to Filters

Question 48 of 90

A child presents with slapped cheek syndrome. She has florid red cheeks with perioral sparing. Her mother asks
for how long she is infectious.

Which one of the following statements is true?

A She is infectious for 3-5 days after rash appears

B She is infectious for 5-7 days after rash appears

C She is infectious for 7-14 days after rash appears

D She is infectious for 7-21 days after rash appears

E She is no longer infectious

Explanation
Slapped cheek syndrome is caused by parvovirus B19. It is otherwise known as fifth disease or erythema
infectiosum. Children aged 3 to 15 years are particularly affected especially in winter/spring. The incubation
period is between 4 and 14 days - most usually 1 week.

Clinical features:

Infectious before rash presents, infectivity wanes once rash appears


Coryza
Low grade fever
mild headache
Sore throat
Red cheeks with perioral sparing
Lacy morbilliform rash on limbs

https://mypastest.pastest.com/Secure/TestMe/Browser/436619[‫ ص‬06:54:42 11/12/1437]


MyPastest

Complications
of parvovirus B19 include aplastic anaemia especially in patients with sickle cell disease and foetal
hydrops if infection occurs during pregnancy (especially under 20 weeks). End Session
45067

Tag Question

Feedback

Previous Question Difficulty: Average

Peer Responses

Session Progress

Responses Correct: 0

Responses Incorrect: 52

Responses Total: 52

Responses - % Correct: 0%

Blog
About Pastest
Contact Us
Help

© Pastest 2016

Next Question

https://mypastest.pastest.com/Secure/TestMe/Browser/436619[‫ ص‬06:54:42 11/12/1437]


MyPastest

Prefer to use the old MyPastest? Access it here »

Back to Filters

Question 49 of 90

You are treating a child with uncomplicated eczema.The father asks which topical steroid cream is the most
potent.

Which cream from the following list is the most potent?

A Betamethasone valerate

B Clobetasol

C Clobetasone

D Hydrocortisone

E Mometasone

Explanation
Understanding the potency of steroids is essential for the careful management of eczema.

From most to least potent:

Clobestol
Mometasone
Betamethasone
Clobetasone
Hydrocortisone

Note; ointments tend to be more potent and greasier than creams.

http://www.mims.co.uk/topical-steroids-comparison-potencies-formulations/dermatology/article/882440
45068

https://mypastest.pastest.com/Secure/TestMe/Browser/436619[‫ ص‬06:55:02 11/12/1437]


MyPastest

End Session

Tag Question

Feedback

Difficulty: Average

Peer Responses

Previous Question

Session Progress

Responses Correct: 0

Responses Incorrect: 53

Responses Total: 53

Responses - % Correct: 0%

Blog
About Pastest
Contact Us
Help

© Pastest 2016

Next Question

https://mypastest.pastest.com/Secure/TestMe/Browser/436619[‫ ص‬06:55:02 11/12/1437]


MyPastest

Prefer to use the old MyPastest? Access it here »

Back to Filters

Question 50 of 90

A child presents with joint and abdominal pain. A non-blanching rash is noted on the buttocks and legs.

What is the most likely diagnosis?

A Acute lymphoblastic leukaemia

B Aplastic anaemia

C Henoch-Schönlein purpura

D Immune thrombocytopaenic purpura

E Scurvy

Explanation
Clinical features of HSP include:

Abdominal pain
Headache
Joint swelling and pain
Nausea
Rash (purpuric, particularly on buttocks and legs)
Small vessel vasculitis
Vomiting

Complications include:

Bowel infarction

https://mypastest.pastest.com/Secure/TestMe/Browser/436619[‫ ص‬06:55:23 11/12/1437]


MyPastest

GI haemorrhage
Hypertension
Intussusception End Session
Myocardial infarction (MI)
Nephritis
Nephrotic syndrome
Renal failure
Neuropathy
Seizure
Death

Previous Question 45069

Tag Question

Feedback

Difficulty: Average

Peer Responses

Session Progress

Responses Correct: 0

Responses Incorrect: 54

Responses Total: 54

Responses - % Correct: 0%

Next Question

https://mypastest.pastest.com/Secure/TestMe/Browser/436619[‫ ص‬06:55:23 11/12/1437]


MyPastest

Blog
About Pastest
Contact Us
Help

© Pastest 2016

https://mypastest.pastest.com/Secure/TestMe/Browser/436619[‫ ص‬06:55:23 11/12/1437]


MyPastest

Prefer to use the old MyPastest? Access it here »

Back to Filters

Question 51 of 90

A 5-year-old girl has recently attended a birthday party where one of the other guests has subsequently been
diagnosed with scarlet fever. Her grandmother is concerned about the risk of her grandchild catching scarlet fever,
having been brought up on tales from her parents of isolation hospitals and the burning of toys and bed linen
of
scarlet fever victims. You reassure the grandmother that scarlet fever is neither as common nor as virulent as it
once was, but counsel that it is very infectious and that there continue to be periodic outbreaks of a much milder
infection.

What is the incubation period of scarlet fever?

A 1-7 days

B 5-10 days

C 7-14 days

D 7-21 days

E Up to 28 days

Explanation
Typically the incubation period for scarlet fever is around 2-5 days although the range is anything from a day to a
week. It is caused by group A streptococcus. Those affected are infectious for up to 3 weeks.

Clinical features:

Fever (>38.3°C)
Headache,
Lymphadenopathy
Malaise

https://mypastest.pastest.com/Secure/TestMe/Browser/436619[‫ ص‬06:55:45 11/12/1437]


MyPastest

Nausea
Rash (day two to five of illness, starts on neck and face, then spreads)
Red cheeks End Session
Red/purulent tonsils
Sore throat
Strawberry tongue
Tachycardia

Complications:

Cerebral abscess
Previous
Glomerulonephritis Question
Meningitis
Myocarditis
Osteomyelitis
Otitis media
Pneumonia
Quinsy
Sinusitis
Rheumatic fever
Sepsis
Skin desquamation
Venous sinus thrombosis

45070

Tag Question

Feedback

Difficulty: Average

Peer Responses

Next Question

https://mypastest.pastest.com/Secure/TestMe/Browser/436619[‫ ص‬06:55:45 11/12/1437]


MyPastest

Session Progress

Responses Correct: 0

Responses Incorrect: 55

Responses Total: 55

Responses - % Correct: 0%

Blog
About Pastest
Contact Us
Help

© Pastest 2016

https://mypastest.pastest.com/Secure/TestMe/Browser/436619[‫ ص‬06:55:45 11/12/1437]


MyPastest

Prefer to use the old MyPastest? Access it here »

Back to Filters

Question 52 of 90

Previous Question
According to national public health guidelines, a child should be kept away from school, nursery or childminders
following a bout of diarrhoea.

For what length of time should the child be excluded after their last episode of diarrhoea?

A 24 hours

B 48 hours

C 72 hours

D 96 hours

E 120 hours

Explanation
Public health guidelines state that children should be 48 hours free from vomiting or diarrhoea before returning to
school, nursery or under the care of a childminder. Infective particles can continue to be shed once symptoms have
ceased hence this guideline helps to reduce the risk of spread of disease to other children and adults.

Note; bacterial gastroenteritis e.g. E. coli may require longer exclusion periods especially if age 5 or less, or
evidence of bacterial clearance.

www.gov.uk
45071

Next Question
Tag Question

https://mypastest.pastest.com/Secure/TestMe/Browser/436619[‫ ص‬06:56:06 11/12/1437]


MyPastest

Feedback

End Session
Difficulty: Average

Peer Responses

Session Progress

Responses Correct: 0

Responses Incorrect: 56

Responses Total: 56

Responses - % Correct: 0%

Blog
About Pastest
Contact Us
Help

© Pastest 2016

https://mypastest.pastest.com/Secure/TestMe/Browser/436619[‫ ص‬06:56:06 11/12/1437]


MyPastest

Prefer to use the old MyPastest? Access it here »

End Session

Back to Filters

Question 53 of 90

Previous Question
Which one of the following communicable diseases requires temporary exclusion from school or nursery?

A Chicken pox

B Conjuntivitus

C Glandular fever

D Head lice

E Viral meningitis

Explanation
Children with chicken pox should be excluded from
school, nursery and should not be under the care of a child
minder until all lesions have crusted over.

https://www.gov.uk
45072

Tag Question

Feedback

Next Question
Difficulty: Average

Peer Responses

https://mypastest.pastest.com/Secure/TestMe/Browser/436619[‫ ص‬06:56:25 11/12/1437]


MyPastest

Session Progress

Responses Correct: 0

Responses Incorrect: 57

Responses Total: 57

Responses - % Correct: 0%

Blog
About Pastest
Contact Us
Help

© Pastest 2016

https://mypastest.pastest.com/Secure/TestMe/Browser/436619[‫ ص‬06:56:25 11/12/1437]


MyPastest

Prefer to use the old MyPastest? Access it here »

End Session

Back to Filters

Question 54 of 90

Previous Question
Which one of the following is considered to be a risk factor for chorioamnionitis?

A Multiple vaginal examinations

B Prolonged rupture of membranes

C Prolonged second stage of labour

D Untreated group B streptococcus bacteriuria

E All of the above

Explanation
Chorioamnionitis is a serious condition that may account for premature labour, still birth, cerebral palsy, neonatal
and maternal sepsis, neonatal meningitis and pneumonia. There are multiple risk factors for chorioamnionitis.

Education of expectant mothers is critical to reduce risk and ensure early treatment e.g. screening and treatment
for group B streptococcus,
lifestyle measures and seeking early advice in the presence of maternal
fever,
abdominal pain or foul smelling vaginal discharge.
45087

Tag Question

Feedback Next Question

Difficulty: Average

https://mypastest.pastest.com/Secure/TestMe/Browser/436619[‫ ص‬06:56:45 11/12/1437]


MyPastest

Peer Responses

Session Progress

Responses Correct: 0

Responses Incorrect: 58

Responses Total: 58

Responses - % Correct: 0%

Blog
About Pastest
Contact Us
Help

© Pastest 2016

https://mypastest.pastest.com/Secure/TestMe/Browser/436619[‫ ص‬06:56:45 11/12/1437]


MyPastest

Prefer to use the old MyPastest? Access it here »

Back to Filters

Question 55 of 90

Previous Question
You're asked to see a week-old baby with irritability, a high
pitched cry and bulging fontanelle. A clinical
diagnosis of meningitis is made.

Which one of the following microbes is the most likely aetiological agent in this age group?

A Group B Streptococcus

B Haemophilus influenzae type B

C Mycobacterium tuberculosis

D Neisseria meningitidis

E Straphylococcus aureus

Explanation
Causes of meningitis in infants over three months of age include: Neisseria meningitidis (meningococcus),
Streptococcus pneumoniae (pneumococcus) and Haemophilus influenzae type b (Hib). The most common causes
of meningitis in neonates include Streptococcus agalactiae (Group B streptococcus), Escherichia coli, S
pneumoniae and Listeria monocytogenes.

https://www.nice.org.uk/guidance/cg102
45089

Next Question Tag Question

Feedback

https://mypastest.pastest.com/Secure/TestMe/Browser/436619[‫ ص‬06:57:05 11/12/1437]


MyPastest

End Session
Difficulty: Average

Peer Responses

Session Progress

Responses Correct: 0

Responses Incorrect: 59

Responses Total: 59

Responses - % Correct: 0%

Blog
About Pastest
Contact Us
Help

© Pastest 2016

https://mypastest.pastest.com/Secure/TestMe/Browser/436619[‫ ص‬06:57:05 11/12/1437]


MyPastest

Prefer to use the old MyPastest? Access it here »

End Session

Back to Filters

Question 56 of 90

Previous Question
Which one of the following signs is NOT a feature of serious febrile illness in infants (< 12 months of age)?

A Capillary refill time of 4 seconds

B Concerned parents

C Pulse of 156 beats per minute

D Respiratory rate of 72 breaths per minute

E Significant reduction in urine output

Explanation
Understanding the signs and symptoms of serious illness are essential for exams and clinical practice. Some
parameters change with age e.g. pulse and respiratory rate. Parental concerns should never be ignored. A pulse
rate of 120-160 is normal in infants.

https://www.nice.org.uk/guidance/cg102
45090

Tag Question

Feedback
Next Question
Difficulty: Average

https://mypastest.pastest.com/Secure/TestMe/Browser/436619[‫ ص‬06:57:25 11/12/1437]


MyPastest

Peer Responses

Session Progress

Responses Correct: 0

Responses Incorrect: 60

Responses Total: 60

Responses - % Correct: 0%

Blog
About Pastest
Contact Us
Help

© Pastest 2016

https://mypastest.pastest.com/Secure/TestMe/Browser/436619[‫ ص‬06:57:25 11/12/1437]


MyPastest

Prefer to use the old MyPastest? Access it here »

Back to Filters

Question 57 of 90

Previous Question
You are investigating a child for suspected UTI.

Which one of the following acids should urine be preserved in if it can not be cultured or refrigerated within 4
hours?

A Acetic

B Boric

C Hydrochloric

D Lactic

E Sulphuric

Explanation
According to NICE CG54:

"If urine is to be cultured but cannot be cultured within 4 hours of collection, the sample
should be refrigerated or
preserved with boric acid immediately."

If boric acid is used, the correct volume of urine is required to avoid bacterial toxicity and a false negative culture.

https://www.nice.org.uk/guidance/cg54
45092

Next Question
Tag Question

https://mypastest.pastest.com/Secure/TestMe/Browser/436619[‫ ص‬06:57:44 11/12/1437]


MyPastest

Feedback

End Session
Difficulty: Average

Peer Responses

Session Progress

Responses Correct: 0

Responses Incorrect: 61

Responses Total: 61

Responses - % Correct: 0%

Blog
About Pastest
Contact Us
Help

© Pastest 2016

https://mypastest.pastest.com/Secure/TestMe/Browser/436619[‫ ص‬06:57:44 11/12/1437]


MyPastest

Prefer to use the old MyPastest? Access it here »

End Session

Back to Filters

Question 58 of 90

Previous Question
Infection with group B streptococcus (GBS) accounts for 40% of early onset neonatal infections.

What is the percentage carriage of GBS in women of reproductive age?

A 25%

B 30%

C 35%

D 40%

E 45%

Explanation
Early onset neonatal infections occur within 72 hours of birth. Implicated bacteria are predominantly group B
streptococci and E. Coli.
Sepsis, Pneumonia and meningitis may result. Symptoms include irritability, floppiness,
poor feeding, low or high temperature, respiratory distress and skin changes.

https://www.nice.org.uk/guidance/cg149
45094

Tag Question

Feedback
Next Question

https://mypastest.pastest.com/Secure/TestMe/Browser/436619[‫ ص‬06:58:03 11/12/1437]


MyPastest

Difficulty: Average

Peer Responses

Session Progress

Responses Correct: 0

Responses Incorrect: 62

Responses Total: 62

Responses - % Correct: 0%

Blog
About Pastest
Contact Us
Help

© Pastest 2016

https://mypastest.pastest.com/Secure/TestMe/Browser/436619[‫ ص‬06:58:03 11/12/1437]


MyPastest

Prefer to use the old MyPastest? Access it here »

End Session

Back to Filters

Question 59 of 90

Previous Question
Which one of the following oral antibiotics should be used first line in the treatment of impetigo?

A Amoxicillin

B Clarithromycin

C Co-amoxiclav

D Doxycycline

E Flucloxacillin

Explanation
Flucloxacillin is first line therapy when oral antibiotics are required. Whilst coamoxiclav may be used to cover
both staphylococcus and streptococcus,
it shouldn't be considered first line. Clarithromycin has good cover and is
used second line, although, many doctors use it first line due to
convenience. Doxycycline is contraindicated in
children.

http://cks.nice.org.uk/impetigo#!scenario
45096

Tag Question

Feedback Next Question

Difficulty: Average

https://mypastest.pastest.com/Secure/TestMe/Browser/436619[‫ ص‬06:58:23 11/12/1437]


MyPastest

Peer Responses

Session Progress

Responses Correct: 0

Responses Incorrect: 63

Responses Total: 63

Responses - % Correct: 0%

Blog
About Pastest
Contact Us
Help

© Pastest 2016

https://mypastest.pastest.com/Secure/TestMe/Browser/436619[‫ ص‬06:58:23 11/12/1437]


MyPastest

Prefer to use the old MyPastest? Access it here »

End Session

Back to Filters

Question 60 of 90

Previous Question
You are advising parents of a 5-year-old with acute infective conjunctivitis.

Which one of the following statements is correct?

A Most children will get better without treatment within 24 hours

B Most children will get better without treatment within 3 days

C Most children will get better without treatment within 5-7 days

D Most children will get better without treatment within 7-10 days

E Most children will get better without treatment within 7-14 days

Explanation
Most children with simple acute conjunctivitis will get better without antibiotic treatment within 7-14 days.
Topical treatments are often overused. There is no requirement for
children to remain away from school. Care
should be taken in neonates and complicated disease such as periorbital cellulitis. 45097

Tag Question

Feedback

Next Question
Difficulty: Average

https://mypastest.pastest.com/Secure/TestMe/Browser/436619[‫ ص‬06:58:43 11/12/1437]


MyPastest

Peer Responses

Session Progress

Responses Correct: 0

Responses Incorrect: 64

Responses Total: 64

Responses - % Correct: 0%

Blog
About Pastest
Contact Us
Help

© Pastest 2016

https://mypastest.pastest.com/Secure/TestMe/Browser/436619[‫ ص‬06:58:43 11/12/1437]


MyPastest

Prefer to use the old MyPastest? Access it here »

End Session

Back to Filters

Question 61 of 90

Previous Question
Which one of the following conditions are complications of impetigo?

A Cellulitus

B Guttate psoriasis

C Sepsis

D Staphylococcal scalded skin syndrome

E All of the above

Explanation
Whilst most cases of impetigo are easily managed in primary care, children will need referral for complications
and appropriate safety setting for parents is essential. In addition to the complications listed above, scarlet fever,
scarring and post streptococcal glomerulonephritis may also be seen. Patients with resistant, aggressive,
complicated or recurrent impetigo may require investigation for underlying immune paresis e.g. leukaemia.
45098

Tag Question

Feedback

Next Question
Difficulty: Average

https://mypastest.pastest.com/Secure/TestMe/Browser/436619[‫ ص‬06:59:02 11/12/1437]


MyPastest

Peer Responses

Session Progress

Responses Correct: 0

Responses Incorrect: 65

Responses Total: 65

Responses - % Correct: 0%

Blog
About Pastest
Contact Us
Help

© Pastest 2016

https://mypastest.pastest.com/Secure/TestMe/Browser/436619[‫ ص‬06:59:02 11/12/1437]


MyPastest

Prefer to use the old MyPastest? Access it here »

Back to Filters

Question 62 of 90

What is the usual stat treatment dose of oral dexamethasone in the treatment of childhood croup?

A 0.05mg/kg

B 0.10mg/kg

C 0.15mg/kg

D 0.25mg/kg

E 0.5mg/kg

Explanation
Croup is a viral acute laryngotracheobronchitis and is unusual above 6 years of age. Causes include parainfluenza,
adenovirus, RSV, rhinovirus, influenza and even measles.

Clinical features are a barking, seal-like cough, sore throat, runny nose, mild/moderate fever, inspiratory stridor,
respiratory distress. Symptoms tend to resolve after three days.

Red flags:

Agitation or reduced consciousness


Central cyanosis
Reduced air entry
Severe intercostal recession
Stridor

45101

https://mypastest.pastest.com/Secure/TestMe/Browser/436619[‫ ص‬06:59:22 11/12/1437]


MyPastest

End Session
Tag Question

Feedback

Difficulty: Average

Peer Responses

Previous Question

Session Progress

Responses Correct: 0

Responses Incorrect: 66

Responses Total: 66

Responses - % Correct: 0%

Blog
About Pastest
Contact Us
Help

© Pastest 2016

Next Question

https://mypastest.pastest.com/Secure/TestMe/Browser/436619[‫ ص‬06:59:22 11/12/1437]


MyPastest

Prefer to use the old MyPastest? Access it here »

End Session

Back to Filters

Question 63 of 90

Previous Question
According to NICE, what is the appropriate duration of treatment with oral antibiotics for a child of over three
months of age with cystitis?

A 3 days

B 5 days

C 7-10 days

D 10 days

E None of the above

Explanation
According to NICE, children of over three months of age with a lower urinary tract infection should be treated
with a three day course of oral antibiotics. Pyelonephritis may be treated with
oral antibiotics such as co-
amoxiclav or cephalosporins for 7-10 days.

https://www.nice.org.uk/guidance/cg54
45102

Tag Question

Feedback Next Question

Difficulty: Average

https://mypastest.pastest.com/Secure/TestMe/Browser/436619[‫ ص‬07:00:08 11/12/1437]


MyPastest

Peer Responses

Session Progress

Responses Correct: 0

Responses Incorrect: 67

Responses Total: 67

Responses - % Correct: 0%

Blog
About Pastest
Contact Us
Help

© Pastest 2016

https://mypastest.pastest.com/Secure/TestMe/Browser/436619[‫ ص‬07:00:08 11/12/1437]


MyPastest

Prefer to use the old MyPastest? Access it here »

End Session

Back to Filters

Question 64 of 90

Previous Question
According to NICE, what antibiotics are NOT appropriate in the initial treatment of lower urinary tract
infection/cystitis in children over three months of age?

A Amoxicillin

B Cephradine

C Co-amoxiclav

D Nitrofurantion

E Trimethoprim

Explanation
All except co-amoxiclav are appropriate treatments for lower urinary tract infections. Prescribing should be in line
with local resistance advice and guidelines. Co-amoxiclav is indicated for upper urinary tract infections.
Asymptomatic bacteriuria is not an indication for antibiotics.

https://www.nice.org.uk/guidance/cg54
45104

Tag Question

Feedback Next Question

Difficulty: Average

https://mypastest.pastest.com/Secure/TestMe/Browser/436619[‫ ص‬07:00:37 11/12/1437]


MyPastest

Peer Responses

Session Progress

Responses Correct: 0

Responses Incorrect: 68

Responses Total: 68

Responses - % Correct: 0%

Blog
About Pastest
Contact Us
Help

© Pastest 2016

https://mypastest.pastest.com/Secure/TestMe/Browser/436619[‫ ص‬07:00:37 11/12/1437]


MyPastest

Prefer to use the old MyPastest? Access it here »

End Session

Back to Filters

Question 65 of 90

Previous Question
When managing children with otitis media, which of the following scenarios should be considered for immediate
management with antibiotics?

A Bilateral otitis media in children under one year of age

B Bilateral otitis media in children under two years of age

C Bilateral otitis media in children under three years of age

D Bilateral otitis media in children under four years of age

E Bilateral otitis media in children under five years of age

Explanation
According to NICE guidelines on respiratory tract
infections, children under the age of 2 with bilateral otitis media
should be considered for immediate treatment with antibiotics. A no- or delayed antibiotic approach may
otherwise be considered for acute otitis
media, acute sore throat/acute pharyngitis/acute tonsillitis, common cold,
acute rhinosinusitis, acute cough/bronchitis.

https://www.nice.org.uk/guidance/cg69
45105

Tag Question
Next Question
Feedback

https://mypastest.pastest.com/Secure/TestMe/Browser/436619[‫ ص‬07:01:06 11/12/1437]


MyPastest

Difficulty: Average

Peer Responses

Session Progress

Responses Correct: 0

Responses Incorrect: 69

Responses Total: 69

Responses - % Correct: 0%

Blog
About Pastest
Contact Us
Help

© Pastest 2016

https://mypastest.pastest.com/Secure/TestMe/Browser/436619[‫ ص‬07:01:06 11/12/1437]


MyPastest

Prefer to use the old MyPastest? Access it here »

Back to Filters

Question 66 of 90

When advising parents on the natural history of the common cold, what is the average total length of illness?

A 4 days

B 1 week

C 1.5 weeks

D 2.5 weeks

E 3 weeks

Explanation
According to NICE, the average total length of illness is:

Acute otitis: 4 days


Acute sore throat/acute pharyngitis/acute tonsillitis: 1 week
Common cold: 1½ weeks
Acute rhinosinusitis: 2½ weeks
Acute cough/acute bronchitis: 3 weeks

https://www.nice.org.uk/guidance/cg69 45107

Next Question

Tag Question

https://mypastest.pastest.com/Secure/TestMe/Browser/436619[‫ ص‬07:01:26 11/12/1437]


MyPastest

Feedback End Session

Difficulty: Average

Peer Responses

Previous Question

Session Progress

Responses Correct: 0

Responses Incorrect: 70

Responses Total: 70

Responses - % Correct: 0%

Blog
About Pastest
Contact Us
Help

© Pastest 2016

https://mypastest.pastest.com/Secure/TestMe/Browser/436619[‫ ص‬07:01:26 11/12/1437]


MyPastest

Prefer to use the old MyPastest? Access it here »

Back to Filters

Question 67 of 90

When counselling a teenage, what is the incubation period for glandular fever?

A <1 week

B 1-2 weeks

C 2-4 weeks

D 1-2 months

E 2-4 months

Explanation
Glandular fever has a long incubation period of 1-2 months. Clinical features include sore throat, tonsillitis, white
tonsillar exudate, fatigue, fever, nausea, cervical lymphadenopathy, and mild hepatosplenomegaly.

Complications include:

Arthralgia
Cranial nerve palsy
Encephalitis
Guillain-Barré syndrome
Haemolysis
Hepatitis
Jaundice
Meningitis
Myocarditis
Nephritis

https://mypastest.pastest.com/Secure/TestMe/Browser/436619[‫ ص‬07:01:47 11/12/1437]


MyPastest

Optic neuritis
Postviral fatigue/depression
Rash with amoxicillin/ampicillin
End Session
Splenic rupture
Transverse myelitis

45108

Previous Question Tag Question

Feedback

Difficulty: Average

Peer Responses

Session Progress

Responses Correct: 0

Responses Incorrect: 71

Responses Total: 71

Responses - % Correct: 0%

Blog
About Pastest
Contact Us
Help

© Pastest 2016
Next Question

https://mypastest.pastest.com/Secure/TestMe/Browser/436619[‫ ص‬07:01:47 11/12/1437]


MyPastest

Prefer to use the old MyPastest? Access it here »

End Session

Back to Filters

Question 68 of 90

Previous Question
You are counselling a family about chicken pox.

For how many days prior to vesicles appearing is it infectious?

A 0

B 1

C 2

D 3

E 4

Explanation
Chicken pox is infectious for 2 days prior to vesicles appearing to when they all crust over. The incubation period
is 9-23 days. Complications are several fold and include secondary infection, ocular, neurological, liver and
respiratory disorders.
45110

Tag Question

Feedback
Next Question
Difficulty: Average

https://mypastest.pastest.com/Secure/TestMe/Browser/436619[‫ ص‬07:02:07 11/12/1437]


MyPastest

Peer Responses

Session Progress

Responses Correct: 0

Responses Incorrect: 72

Responses Total: 72

Responses - % Correct: 0%

Blog
About Pastest
Contact Us
Help

© Pastest 2016

https://mypastest.pastest.com/Secure/TestMe/Browser/436619[‫ ص‬07:02:07 11/12/1437]


MyPastest

Prefer to use the old MyPastest? Access it here »

Back to Filters

Question 69 of 90

According to NICE guidelines, which one of the following is a red flag when managing a child with bronchiolitis?

A Mild chest wall recession

B No wet nappies for 6 hours

C Peripheral cyanosis

D Persistent oxygen saturations of less than 94%

E Respiratory rate >70 breaths/minute

Explanation
According to NICE guidelines on bronchiolitis, referral to an acute facility (usually) by ambulance is required
under any one of the following circumstances:

apnoea (observed or reported)


child looks seriously unwell to a healthcare professional
severe respiratory distress, for example grunting, marked chest recession, or a respiratory rate of over 70
breaths/minute
central cyanosis
persistent oxygen saturation of less than 92% when breathing air

http://www.nice.org.uk/guidance/NG9/chapter/1-recommendations
Next Question 45111

https://mypastest.pastest.com/Secure/TestMe/Browser/436619[‫ ص‬07:02:26 11/12/1437]


MyPastest

Tag Question
End Session
Feedback

Difficulty: Average

Peer Responses

Previous Question

Session Progress

Responses Correct: 0

Responses Incorrect: 73

Responses Total: 73

Responses - % Correct: 0%

Blog
About Pastest
Contact Us
Help

© Pastest 2016

https://mypastest.pastest.com/Secure/TestMe/Browser/436619[‫ ص‬07:02:26 11/12/1437]


MyPastest

Prefer to use the old MyPastest? Access it here »

Back to Filters

Question 70 of 90

Previous Question
When assessing risk for early onset neonatal infection, which one of the following risk factors does NICE consider
a red flag?

A Intrapartum fever higher than 38°C

B Invasive group B streptococcal infection in a previous baby

C Pre-labour rupture of the membranes

D Pre-term birth following spontaneous labour (before 37 weeks' gestation)

E Suspected or confirmed infection in another baby in the case of multiple pregnancy

Explanation
All factors in the list above are risk factors. Suspected or confirmed infection in another baby in the case of a
multiple pregnancy is considered by NICE to be a red flag. The other red
flag, however, is "Parenteral antibiotic
treatment given to the woman for confirmed or suspected invasive bacterial infection (such as septicaemia) at any
time during labour, or in the 24-hour periods before
and after the birth. (This does not refer to intrapartum
antibiotic prophylaxis)" see:

http://www.nice.org.uk/guidance/cg149/chapter/1-recommendations#risk-factors-for-infection-and-clinical-
indicators-of-possible-infection-2
45115

Next Question
Tag Question

https://mypastest.pastest.com/Secure/TestMe/Browser/436619[‫ ص‬07:02:48 11/12/1437]


MyPastest

Feedback

End Session
Difficulty: Average

Peer Responses

Session Progress

Responses Correct: 0

Responses Incorrect: 74

Responses Total: 74

Responses - % Correct: 0%

Blog
About Pastest
Contact Us
Help

© Pastest 2016

https://mypastest.pastest.com/Secure/TestMe/Browser/436619[‫ ص‬07:02:48 11/12/1437]


MyPastest

Prefer to use the old MyPastest? Access it here »

End Session

Back to Filters

Question 71 of 90

Previous Question
When assessing risk for early onset neonatal infection, which one of the following signs does NICE consider NOT
to be
a red flag?

A Need for mechanical ventilation in a term baby

B Respiratory distress starting more than 4 hours after birth

C Seizures

D Signs of shock

E Tachycardia

Explanation
All the features listed above are red flags except for tachycardia. Nevertheless, tachycardia or bradycardia may be
a
sign of early onset neonatal infection and should be taken seriously.

http://www.nice.org.uk/guidance/cg149/chapter/1-recommendations#risk-factors-for-infection-and-clinical-
indicators-of-possible-infection-2
45116

Tag Question

Feedback Next Question

Difficulty: Average

https://mypastest.pastest.com/Secure/TestMe/Browser/436619[‫ ص‬07:03:07 11/12/1437]


MyPastest

Peer Responses

Session Progress

Responses Correct: 0

Responses Incorrect: 75

Responses Total: 75

Responses - % Correct: 0%

Blog
About Pastest
Contact Us
Help

© Pastest 2016

https://mypastest.pastest.com/Secure/TestMe/Browser/436619[‫ ص‬07:03:07 11/12/1437]


MyPastest

Prefer to use the old MyPastest? Access it here »

End Session

Back to Filters

Question 72 of 90

Previous Question
When discussing constipation in children, which one of the following is considered to be a red flag symptoms?

A Abdominal distension with vomiting

B Constipation from birth

C Failure to pass meconium for greater than 48 hours after childbirth

D Previously undiagnosed leg weakness

E All of the above

Explanation
A clear understanding of red flags is essential for this exam. Whilst most children will suffer from idiopathic
constipation, recognition of any underlying pathology masquerading as constipation is a required skill for doctors
who manage children. Underlying causes may include neurological diseases and gastrointestinal
disorders such as
Hirschsprung's disease and intestinal obstruction.

http://www.nice.org.uk/guidance/cg99/chapter/1-recommendations
45118

Tag Question

Feedback Next Question

Difficulty: Average

https://mypastest.pastest.com/Secure/TestMe/Browser/436619[‫ ص‬07:03:26 11/12/1437]


MyPastest

Peer Responses

Session Progress

Responses Correct: 0

Responses Incorrect: 76

Responses Total: 76

Responses - % Correct: 0%

Blog
About Pastest
Contact Us
Help

© Pastest 2016

https://mypastest.pastest.com/Secure/TestMe/Browser/436619[‫ ص‬07:03:26 11/12/1437]


MyPastest

Prefer to use the old MyPastest? Access it here »

Back to Filters

Question 73 of 90

A child is at a birthday party when she suddenly complains of


an itchy tongue, starts scratching her throat,
sneezes, develops lip swelling, difficulty swallowing and urticaria.

On arrival of the paramedics, what is the next BEST step?

A Give intramuscular adrenaline

B Give intramuscular antihistamine

C Give intravenous adrenaline

D Give oral piriton

E Give subcutaneous adrenaline

Explanation
This child has severe respiratory complications in her upper airway and has generalised urticaria indicating
anaphylaxis. She needs urgent intramuscular adrenaline. The likely allergen is a food – possibly nuts in a cake or
chocolate eaten at
the party.

Anaphylaxis is an extreme and severe allergic reaction that is potentially fatal. It is caused by degranulation of
mast cells mediated by IgE. Anaphylaxis can be triggered by any of a very broad range of allergens, the most
common being food (children), drugs (adults) and venom.

Features of anaphylaxis

Acute and rapid onset of symptoms:

life-threatening airway, and/or breathing, and/or circulation problems


usually skin changes

https://mypastest.pastest.com/Secure/TestMe/Browser/436619[‫ ص‬07:03:47 11/12/1437]


MyPastest

Life-threatening problems requiring intramuscular adrenaline


Life-threatening airway: swelling, hoarseness, stridor
End Sessioncirculation: pale,
Life-threatening
breathing: tachypnoea, wheeze, cyanosis, confusion • Life-threatening
clammy, drowsy/coma.

A useful website is http://www.anaphylaxis.org.uk/

Antihistamines
and hydrocortisone are recommended in the treatment of anaphylaxis. Parents should be advised to
dial 999 after administration of adrenaline
because there may be a return of symptoms requiring further treatment.

Children
with anaphylaxis should wear a MedicAlert bracelet/card and carry their
adrenaline with them at all
times. The most commonly prescribed autoinjector is the EpiPen (there is also an Anapen autoinjector):

Previous
EpiPen junior: 0.15 mgQuestion
adrenaline (<6 years)
EpiPen: 0.3 mg adrenaline (6–12 years)
Intramuscular adrenaline 0.5 mg (>12 years).

Blood test for tryptase may be helpful in diagnosing anaphylaxis.


45172

Tag Question

Feedback

Difficulty: Average

Peer Responses

Session Progress

Responses Correct: Next Question 0

Responses Incorrect: 77

https://mypastest.pastest.com/Secure/TestMe/Browser/436619[‫ ص‬07:03:47 11/12/1437]


MyPastest

Responses Total: 77

Responses - % Correct: 0%

Blog
About Pastest
Contact Us
Help

© Pastest 2016

https://mypastest.pastest.com/Secure/TestMe/Browser/436619[‫ ص‬07:03:47 11/12/1437]


MyPastest

Prefer to use the old MyPastest? Access it here »

Back to Filters

Question 74 of 90

According to NICE guidelines on gastro-oesophageal disease in children and young people, which of the
following symptoms is
considered a red flag?

A Abdominal distension

B Acute diarrhoea

C Haematemesis in a breastfeeding child whose mother has cracked nipples

D Onset of reflux symptoms at 3 months of age

E Vomiting gastric contents

Explanation
It is essential to recognise red flags for examinations and clinical practice. In children with gastrointestinal
suspected disease, the following are considered red flags:

Frequent, forceful (projectile) vomiting especially in infants less than 2 months of age. This may suggest
pyloric stenosis
Bile-stained (green or yellow-green) vomit which can be a sign of intestinal obstruction
Haematemesis (but not swallowed blood)
Onset of regurgitation and/or vomiting after 6 months old or persisting after 1 year old
Blood in stool
Abdominal distension, tenderness or palpable mass
Chronic diarrhoea

http://www.nice.org.uk/guidance/NG1/chapter/1-recommendations

https://mypastest.pastest.com/Secure/TestMe/Browser/436619[‫ ص‬07:04:11 11/12/1437]


MyPastest

45992

End Session

Tag Question

Feedback

Difficulty: Average

Peer Responses
Previous Question

Session Progress

Responses Correct: 0

Responses Incorrect: 78

Responses Total: 78

Responses - % Correct: 0%

Blog
About Pastest
Contact Us
Help

© Pastest 2016

Next Question

https://mypastest.pastest.com/Secure/TestMe/Browser/436619[‫ ص‬07:04:11 11/12/1437]


MyPastest

Prefer to use the old MyPastest? Access it here »

Back to Filters

Question 75 of 90

An 11-month-old infant presented with a 2-week history of fever. He is immunised up to age. Clinical
examination revealed high grade fever 39°C, blood pressure 85/50mmHg, HR 70/min, RR 24/min. Clinical
examination was unremarkable. Complete Blood Count revealed Hb:
12g/dL, HCT: 35%, WBC: 2.3x109/L,
Platelets: 98x109/L.

What is the MOST likely cause of this infant’s illness?

A Adenovirus

B Dengue virus

C Leptospira sp

D Plasmodium knowlesi

E Salmonella typhi

Explanation
In this infant, the most likely cause is Salmonella typhi.
Given the prolonged fever with lack of significant clinical
examination
finding, relative bradycardia, neutropaenia and thrombocytopaenia, typhoid fever is the leading
diagnosis.

Adenovirus may cause a similar blood picture with similar duration of fever, however, clinically the infant may
have rather striking features of an upper respiratory tract infection and conjunctivitis.

Dengue fever could


give similar features too however the prolonged fever is unlikely. In addition, a high
haematocrit level is associated with the thrombocytopaenia. Relative bradycardia is not a feature of dengue fever.

Leptospirosis do present early on with leucopoenia


and thrombocytopenia however it is rare in this age group. In
addition,
jaundice is a key feature, especially when the disease has entered the second week.

Plasmodium knowlesi is an emerging cause of malaria in South-East Asia. It can present with similar features to

https://mypastest.pastest.com/Secure/TestMe/Browser/436619[‫ ص‬07:04:32 11/12/1437]


MyPastest

this case however, the infant will most likely also display features of hepatosplenomegaly. In addition, a drop in
haemoglobin and jaundice will
be a key feature too.
46087
End Session

Tag Question

Feedback

Difficulty: Average
Previous Question
Peer Responses

Session Progress

Responses Correct: 0

Responses Incorrect: 79

Responses Total: 79

Responses - % Correct: 0%

Blog
About Pastest
Contact Us
Help

© Pastest 2016

Next Question

https://mypastest.pastest.com/Secure/TestMe/Browser/436619[‫ ص‬07:04:32 11/12/1437]


MyPastest

Prefer to use the old MyPastest? Access it here »

Back to Filters

Question 76 of 90

A 3-month-old unimmunised infant from Borneo was brought by a


distraught mother with complaints of an
inconsolable cry and less activity than normal for 3 days. The infant developed a fit in the emergency department.
He was resuscitated and admitted to the HDU. A CT
brain showed acute hydrocephalus and he was planned for an
urgent extra-ventricular drainage.

CSF analysis:
CSF Leucocytes 90cellsx106/L (90% Lymphocytes, 10% Neutrophils)

CSF Erythrocytes NIL

CSF Protein 1.8g/L

CSF Glucose 0.5mmol/L

What is the MOST likely causative organism?

A Escherichia coli

B Group B streptococcus

C Listeria monocytogenes

D Mycobacterium tuberculosis

E Neisseria meningitides

Explanation
At 3 months of age, this infant is at risk of meningitis from all the organisms stated. Tuberculosis is the most
likely cause as this is an unimmunised infant. Acute hydrocephalus is a
prominent feature of TB meningitis, and
TB meningitis is a significant cause of morbidity and mortality in children with tuberculosis. The high
total white

https://mypastest.pastest.com/Secure/TestMe/Browser/436619[‫ ص‬07:04:53 11/12/1437]


MyPastest

count in the CSF analysis with lymphocyte predominance, an extremely high protein and low CSF glucose are
highly suggestive of TB meningitis.
End Session
E.coli may be a cause of meningitis in this age group however, acute hydrocephalus and CSF lymphocytosis are
not feature.

Group B streptococcus
is a common cause of meningitis is infants < 3 months of age, however, acute
hydrocephalus is not a primary feature and CSF analysis would show a high neutrophil count rather than
lymphocytosis.

Listeria monocytogenes
is a rare cause of meningitis but should be actively looked for in infants < 3 months of
age. It is fastidious and not easy to culture. The CSF lymphocytosis IS a feature in Listeria meningitis, however
acute
hydrocephalus is not a primary feature. Given the history of the lack of immunity in a Bornean child, this
diagnosis is unlikely.

Previous
Neisseria meningitides Question
is a known cause of meningitis however CSF neutrophilia is expected.
46088

Tag Question

Feedback

Difficulty: Average

Peer Responses

Session Progress

Responses Correct: 0

Responses Incorrect: 80

Responses Total: 80

Responses - % Correct: 0%
Next Question

https://mypastest.pastest.com/Secure/TestMe/Browser/436619[‫ ص‬07:04:53 11/12/1437]


MyPastest

Blog
About Pastest
Contact Us
Help

© Pastest 2016

https://mypastest.pastest.com/Secure/TestMe/Browser/436619[‫ ص‬07:04:53 11/12/1437]


MyPastest

Prefer to use the old MyPastest? Access it here »

Back to Filters

Question 77 of 90

A 5-year-old boy presented with history of fever and being unwell for the past week. His family has just returned
to the UK from a family trip to an island in Malaysia. He was pale on examination with hepatosplenomegaly on
abdominal palpation. There were no lymphadenopathy. His blood works are as follows:
Hb: 6.5g/dL
WBC: 3.0x109/L
Platelets: 76x109/L
Bilirubin: 25mg/dL
ALT: 15U/L AST: 35U/L

What is the MOST useful investigation to aid in diagnosis?

A Blood culture

B Blood film for malarial parasites

C Blood for Malaria PCR

D Peripheral Blood Film

E Typhoid serology

Explanation
Malaria is the tropical disease most commonly imported into the UK with 1500-2000 cases reported each year,
and 10-20 deaths. Approximately three-quarters of reported malaria cases in the UK
are caused by Plasmodium
falciparum, which is capable of invading a high proportion of red blood cells and rapidly leading to severe or life-
threatening multi-organ disease. Given the history of a holiday in South-East Asia, ruling out malaria is of utmost
importance. Performing a thick and thin blood smear to be examined microscopically by an expert can reveal
malarial parasites. It is sensitive and specific
in expert hands. This test is can be down within 30 minutes.
Commencement of treatment is crucial as Falciparum malaria is life threatening.

https://mypastest.pastest.com/Secure/TestMe/Browser/436619[‫ ص‬07:05:15 11/12/1437]


MyPastest

http://www.who.int/mediacentre/factsheets/fs094/en/

A blood culture will definitely be done for an ill patient, however, it is not the most urgent test in aiding the
diagnosis. End Session
Blood for malaria PCR / rapid diagnostic tests is vital in identifying the exact species especially with the
emergence of Plasmodium knowlesi that can be easily mistaken for Plasmodium malaria by the untrained
microscopist.

With a history of fever, hepatosplenomegaly, and pancytopaenia, acute lymphoblastic leukaemia (ALL) is a
differential. Given the short history, lack of lymphadenopathy, and the raised serum bilirubin, malaria is still the
most likely diagnosis.

Typhoid fever gives a


more prolonged and protracted course of fever. Although it may cause pancytopaenia,
splenomegaly is most likely seen rather than a hepatosplenomegaly.
46089
Previous Question

Tag Question

Feedback

Difficulty: Average

Peer Responses

Session Progress

Responses Correct: 0

Responses Incorrect: 81

Responses Total: 81

Responses - % Correct: 0%

Next Question

https://mypastest.pastest.com/Secure/TestMe/Browser/436619[‫ ص‬07:05:15 11/12/1437]


MyPastest

Blog
About Pastest
Contact Us
Help

© Pastest 2016

https://mypastest.pastest.com/Secure/TestMe/Browser/436619[‫ ص‬07:05:15 11/12/1437]


MyPastest

Prefer to use the old MyPastest? Access it here »

Back to Filters

Question 78 of 90

A 14-year-old boy was brought to the emergency department with fever, abdominal pain and a generalised feeling
of being unwell over the past 3 days. Clinical examination revealed a pale and jaundiced
child. He was
tachypnoeic. Abdominal examination revealed hepatosplenomegaly with a soft systolic murmur over the left
sternal edge. A dextrostix read 2.9mmol/L. He subsequently developed a generalised tonic clonic seizure with was
aborted with IV Diazepam and a
bolus of Dextrose 10%. He had a drop in GCS, was intubated and admitted
to the
PICU. Parents have been giving over the counter aspirin for the past 3 days. Further history revealed that he was in
a student exchange programme to the Philippines 3 months back.

What is the MOST likely diagnosis?

A Cerebral malaria

B Dengue encephalitis

C Leptospirosis

D Reye’s syndrome

E Streptococcal meningitis

Explanation
Cerebral malaria is the most probable diagnosis in this case. Pallor, jaundice, hepatosplenomegaly and
hypoglycaemia with recent foreign travel to a malaria endemic area makes cerebral malaria most likely. Due to the
severe haemolysis, patients develop severe anaemia that is symptomatic (soft systolic murmur) with jaundice and
hepatosplenomegaly. Metabolic acidosis (with tachypnoea being the feature) is common in view of circulatory
collapse, and cerebral manifestation is common in later presentations. Patients can present months after exposure.

http://britishinfection.org/files/3514/1617/4227/malariatreatmentBIS07.pdf

Dengue
is endemic in the Philippines, Thailand and Malaysia. In dengue, anaemia, jaundice and hepatomegaly
may be a feature. Hypoglycaemia is rarely a clinical finding. Splenomegaly is present in only 10% of cases of

https://mypastest.pastest.com/Secure/TestMe/Browser/436619[‫ ص‬07:05:36 11/12/1437]


MyPastest

dengue fever. In addition, the incubation period for dengue is 5-10 days, and it is unlikely to present a month later
after an exposure. Dengue fever is severe during subsequent infections after a primary infection that is usually
mild.
End Session
Although jaundice, anaemia and hepatomegaly is a common finding in leptospirosis, splenomegaly is not.
Leptospira encephalitis is very rare. Severe forms of leptospirosis present with pulmonary haemorrhage with
ARDS. Incubation period 7-12 days.

In view of the history of aspirin ingestion, hepatomegaly, tachypnoea (indicating acidosis), hypoglycaemia and a
low GCS may be indicative of Reye’s. However, this usually affects a younger age group and splenomegaly is
rarely a feature.

Streptococcal meningitis is a possibility and indeed with such an ill presentation, it


will be prudent to start the
patient on IV antibiotics. However splenomegaly does not fit and although streptococcal infection may cause
a
haemolytic disease, this itself is rare.
Previous Question 46090

Tag Question

Feedback

Difficulty: Average

Peer Responses

Session Progress

Responses Correct: 0

Responses Incorrect: 82

Responses Total: 82

Responses - % Correct: 0%

Next Question

https://mypastest.pastest.com/Secure/TestMe/Browser/436619[‫ ص‬07:05:36 11/12/1437]


MyPastest

Blog
About Pastest
Contact Us
Help

© Pastest 2016

https://mypastest.pastest.com/Secure/TestMe/Browser/436619[‫ ص‬07:05:36 11/12/1437]


MyPastest

Prefer to use the old MyPastest? Access it here »

Back to Filters

Question 79 of 90

A newborn delivered at term with a birth weight of 3.2kgs was


admitted to the NICU. Her Burmese mother was
diagnosed as having sputum-positive tuberculosis 1 week ago after a bout of haemoptysis and prolonged cough.
The newborn is active on handling. She is not in respiratory distress. Her neonatal examination was normal.

What is the MOST important next step in her management?

A Administer BCG

B Get a Mantoux test done

C Perform a gastric lavage for AFB x 3

D Order a chest radiograph

E Start Isoniazide

Explanation
As the mother was only recently diagnosed as sputum positive pulmonary TB, and (presumably) will have had
only 1 week
of anti-TB treatment, she is still highly contagious. The newborn has been clinically assessed for TB
and the clinical findings are not suggestive of congenital tuberculosis.

BCG is deferred, until a Mantoux is performed at 3 months of age. If the Mantoux is <5mm, a BCG vaccination is
then given and Isoniazide prophylaxis is stopped.

Mantoux test - Utility of Mantoux test in neonates is poor due to low reactogenicity and poor helper T cell
responses.

As the newborn does not show signs of active TB, sending a gastric lavage for AFB unnecessary.

A chest radiograph in an asymptomatic newborn would be of no yield.

The newborn should be started on isoniazid (and pyridoxine) until a reassessment at the age of 3 months; or until

https://mypastest.pastest.com/Secure/TestMe/Browser/436619[‫ ص‬07:05:59 11/12/1437]


MyPastest

there is evidence of congenital TB.

https://www.nice.org.uk/guidance/ng33/chapter/Recommendations
End Session 46091

Tag Question

Feedback

Difficulty: Average
Previous Question
Peer Responses

Session Progress

Responses Correct: 0

Responses Incorrect: 83

Responses Total: 83

Responses - % Correct: 0%

Blog
About Pastest
Contact Us
Help

© Pastest 2016

Next Question

https://mypastest.pastest.com/Secure/TestMe/Browser/436619[‫ ص‬07:05:59 11/12/1437]


MyPastest

Prefer to use the old MyPastest? Access it here »

Back to Filters

Question 80 of 90

A 3-month-old infant presented to the emergency department with 1 week history of fever, cough, and rapid
breathing. She was in severe respiratory distress and was intubated by the paediatric team and
transferred to the
PICU. She weighed 2.9kgs. Examination revealed hepatosplenomegaly with shotty cervical lymphadenopathy.
She was ventilated on high ventilator settings. A homogenous, diffuse interstitial infiltrates was seen on chest
radiograph. Her complete blood count showed:
Hb: 9g/dl
WBC: 4.5x109/L (Neutrophil 2x109/L, Lymphocyte 0.8x109/L)
Platelets: 140x109/L

What is the MOST likely diagnosis?

A Aspergillosis

B CMV pneumonitis

C Pneumocystis jirovecii pneumonia

D Pulmonary tuberculosis

E RSV pneumonia

Explanation
In an infant with poor weight gain, failure to thrive, hepatosplenomegaly and lymphadenopathy who is severely
ill, it is important to rule out immunodeficiency. The low total white cell count and lymphopaenia in this case is
significant. HIV I, one of the primary causes of immunodeficiency. PCP is a common complication of HIV
infection and the prognosis for infants < 6 months old is poor.

The CXR finding in this case is not suggestive of Aspergillosis, that usually is cavitatory.

CMV pneumonitis is a possibility in AIDS however the CXR findings are not specific and given the higher
probability of PCP, this is not the answer of choice.

https://mypastest.pastest.com/Secure/TestMe/Browser/436619[‫ ص‬07:06:19 11/12/1437]


MyPastest

Pulmonary tuberculosis is an opportunistic infection as well in the immunocompromised. Tuberculosis in an


immunocompromised patient will reveal consolidatory changes, cavities, as well as pneumatocoeles in the CXR.
End Session
RSV pneumonia is unlikely in such a clinical picture in view of the clinical presentation and counts.
46092

Tag Question

Feedback

Previous Question
Difficulty: Average

Peer Responses

Session Progress

Responses Correct: 0

Responses Incorrect: 84

Responses Total: 84

Responses - % Correct: 0%

Blog
About Pastest
Contact Us
Help

© Pastest 2016

Next Question

https://mypastest.pastest.com/Secure/TestMe/Browser/436619[‫ ص‬07:06:19 11/12/1437]


MyPastest

Prefer to use the old MyPastest? Access it here »

Back to Filters

Question 81 of 90

A 3-year-old boy from Borneo, Malaysia was presented with a history of fever for 1 week with mild abdominal
pain. He was worked up for malaria and the blood film showed Plasmodium vivax. He was started on chloroquine.

What is the next best step in his management?

A Blood culture to be sent to the lab

B Check his G6PD status

C Notify Public Health

D Start Primaquine

E Repeat the blood film for malarial parasites

Explanation
All the above answers will be steps in his management. The most important step in his management however will
be to
determine his G6PD status as primaquine will need to be started in Plasmodium vivax
infection to eradicate
the hypnozoites. Starting primaquine in a child who is G6PD deficient will cause severe haemolytic disease.

A blood culture may be necessary depending on his clinical condition ( gram negative sepsis is possible as a
superimposed infection in malaria).

Malaria is a notifiable disease but at this juncture, it is not the most important step in management.

Primaquine is to be started, but not before testing G6PD status.

Blood film for malarial parasites are to be repeated AFTER commencement of treatment.
46093
Next Question

https://mypastest.pastest.com/Secure/TestMe/Browser/436619[‫ ص‬07:06:39 11/12/1437]


MyPastest

Tag Question
End Session
Feedback

Difficulty: Average

Peer Responses

Previous Question

Session Progress

Responses Correct: 0

Responses Incorrect: 85

Responses Total: 85

Responses - % Correct: 0%

Blog
About Pastest
Contact Us
Help

© Pastest 2016

https://mypastest.pastest.com/Secure/TestMe/Browser/436619[‫ ص‬07:06:39 11/12/1437]


MyPastest

Prefer to use the old MyPastest? Access it here »

Back to Filters

Question 82 of 90

A 4-year-old boy presents with a right sided submandibular swelling. Examination revealed a violaceous, non-
tender 3 x 4cm swelling
at the right submandibular region. He is otherwise well. FNAC was done and the culture
results showed ‘non tuberculous mycobacterium seen’.

What is the BEST option of management?

A Amoxycillin 90mg/kg/day for 10 days

B Azithromycin 10mg/kg daily for 5 days

C Isoniazid 10mg/kg daily prophylaxis

D Surgical excision

E Conservative management

Explanation
Lymphadenopathy is common in this paediatric age group, usually following upper respiratory tract infections.

Atypical
mycobacterium is a common cause as well. Mycobacterium avium-intracellulare is the most common in
the western world. In a well child, surgical excision is the best treatment option.

During initial presentation, Amoxycillin is recommended for non-malignant lymphadenopathy. If


lymphadenopathy persists, a FNAC/ excision biopsy is
then planned.

Azithromycin can be used in cases where excision is contraindicated.

Isoniazid prophylaxis is not indicated as this is not Latent TB.

Conservative management is not the treatment of choice in this case.


46094

https://mypastest.pastest.com/Secure/TestMe/Browser/436619[‫ ص‬07:06:59 11/12/1437]


MyPastest

End Session
Tag Question

Feedback

Difficulty: Average

Peer Responses

Previous Question

Session Progress

Responses Correct: 0

Responses Incorrect: 86

Responses Total: 86

Responses - % Correct: 0%

Blog
About Pastest
Contact Us
Help

© Pastest 2016

Next Question

https://mypastest.pastest.com/Secure/TestMe/Browser/436619[‫ ص‬07:06:59 11/12/1437]


MyPastest

Prefer to use the old MyPastest? Access it here »

Back to Filters

Question 83 of 90

A 2-year-old Southern Filipino boy who recently arrived in the UK, is brought to the emergency department with
complaints of fever and rapid breathing. He is unimmunised. Examination reveals a wasted, malnourished child
who is fretful. He is febrile and tachypnoeic. RR: 50/min. Lung examination revealed crepitation bilaterally. His
chest radiograph revealed a collapse consolidation of the right middle lobe with a perihilar calcification and
lymphadenopathy.

What is the MOST likely diagnosis?

A Community Acquired Pneumonia

B CMV Pneumonitis

C Lymphocytic alveolitis

D Pneumocystis jirovecii pneumonia

E Pulmonary tuberculosis

Explanation
Coming from an endemic area of tuberculosis, pulmonary TB is the most likely diagnosis for this child. Perihilar
lymphadenopathy is a prime feature in TB. Obstruction of the bronchus/bronchial leads to the collapse
consolidation which is a common
finding in paediatric pulmonary tuberculosis. Malnutrition and HIV predisposes
children to opportunistic infections such as TB.

Community acquired pneumonia may be a possibility however, in immunocompromised children, there should be
a high index of suspicion for opportunistic infection.

CMV pneumonitis gives a homogenous picture of pulmonary infiltrates/ ground glass appearance rather than a
frank consolidation.

Lymphocytic alveolitis is usually seen as ground glass opacities and very rarely as a consolidation.

https://mypastest.pastest.com/Secure/TestMe/Browser/436619[‫ ص‬07:07:18 11/12/1437]


MyPastest

PCP gives a fine reticular interstitial pulmonary pattern. Although they may be perihilar, lymph node enlargement
is not a feature, and only
seen in 10% of the cases.
46095
End Session

Tag Question

Feedback

Difficulty: Average
Previous Question
Peer Responses

Session Progress

Responses Correct: 0

Responses Incorrect: 87

Responses Total: 87

Responses - % Correct: 0%

Blog
About Pastest
Contact Us
Help

© Pastest 2016

Next Question

https://mypastest.pastest.com/Secure/TestMe/Browser/436619[‫ ص‬07:07:18 11/12/1437]


MyPastest

Prefer to use the old MyPastest? Access it here »

Back to Filters

Question 84 of 90

A newborn delivered at term with a birth weight of 2.4kgs was


admitted to the NICU with an impression of
congenital TB. Her mother is HIV positive and is on HAART. She was diagnosed as having sputum-positive
tuberculosis 1 week ago after a bout of haemoptysis and prolonged cough.

The following are features of congenital tuberculosis EXCEPT?

A Diarrhoea

B Low birth weight

C Hepatomegaly

D Irritability

E Splenomegaly

Explanation
Diarrhoea is not a feature of congenital tuberculosis.

Clinical manifestations of congenital TB include poor feeding (100%), fever (100%), irritability (100%), failure to
thrive (100%), cough (88.9%), and respiratory distress (66.7%).

Examination reveals hepatosplenomegaly (100%), splenomegaly (77.8%), and abdominal distension (77.8%).

Apnoea, vomiting, cyanosis, jaundice, seizures and petechiae have been reported in less than 10 per cent of cases.

Ref: Congenital tuberculosis: critical reappraisal of clinical findings and diagnostic procedures.
AUHageman J, Shulman S, Schreiber M, Luck S, Yogev R
SOPediatrics. 1980;66(6):980.
46096

https://mypastest.pastest.com/Secure/TestMe/Browser/436619[‫ ص‬07:07:39 11/12/1437]


MyPastest

End Session
Tag Question

Feedback

Difficulty: Average

Peer Responses

Previous Question

Session Progress

Responses Correct: 0

Responses Incorrect: 88

Responses Total: 88

Responses - % Correct: 0%

Blog
About Pastest
Contact Us
Help

© Pastest 2016

Next Question

https://mypastest.pastest.com/Secure/TestMe/Browser/436619[‫ ص‬07:07:39 11/12/1437]


MyPastest

Prefer to use the old MyPastest? Access it here »

Back to Filters

Question 85 of 90

An unwell child presents with fever, focal seizures, altered consciousness and focal neurology.

What is the most likely diagnosis?

A Bacterial meningitis

B Head injury

C Herpes simplex encephalitis

D Septic shock

E Urinary tract infection

Explanation
It's important to be aware of how febrile conditions present as this may guide empirical treatment. NICE
guidelines state that children who present with fever and focal symptoms
such as the case above may have herpes
simplex encephalitis.

Bacterial Meningitis – fever plus any of the following:

Neck stillness
Bulging fontanelle
Decreased level of consciousness
Convulsive status epilepticus

Urinary
tract infection (consider in any child aged younger than 3 months with fever) or with fever plus one or
more of the following in older children:

https://mypastest.pastest.com/Secure/TestMe/Browser/436619[‫ ص‬07:08:00 11/12/1437]


MyPastest

Vomiting
Poor feeding
Lethargy End Session
Irritiability
Abdominal pain or tenderness
Urinary frequency or dysuria

Sepsis:

Abnormalities in temperature, heart rate, respiratory rate and white blood count in the context of bacterial
infection
Previous Question
https://www.nice.org.uk/guidance/cg160/chapter/1-recommendations
46112

Tag Question

Feedback

Difficulty: Average

Peer Responses

Session Progress

Responses Correct: 0

Responses Incorrect: 89

Responses Total: 89
Next Question
Responses - % Correct: 0%

https://mypastest.pastest.com/Secure/TestMe/Browser/436619[‫ ص‬07:08:00 11/12/1437]


MyPastest

Blog
About Pastest
Contact Us
Help

© Pastest 2016

https://mypastest.pastest.com/Secure/TestMe/Browser/436619[‫ ص‬07:08:00 11/12/1437]


MyPastest

Prefer to use the old MyPastest? Access it here »

Back to Filters

Question 86 of 90

Previous Question
Which one of the bacterial species below is incorrectly categorised?

A Clostridium tetani – gram positive

B Haemophilus influenzae – gram negative

C Listeria monocytogenes – gram positive

D Pseudomonasaeruginosa – gram negative

E Salmonella typhi – gram positive

Explanation
Salmonella typhiis a gram negative organism. The other bacterial species are correctly categorised.

Gram staining is a common technique used to differentiate two large groups of bacteria based on their different
cell wall constituents. Gram
staining distinguishes between Gram positive and Gram negative groups by colouring
cells red or violet. Gram positive bacteria stain violet due to the presence of a thick layer of peptidoglycan in their
cell walls. This retains the crystal violet these cells are stained with. Gram negative bacteria stain red, because of a
thinner peptidoglycan wall, which does not retain the crystal violet during the process.

Gram staining is one of the first steps in identifying which infectious agent is present.
46179

Next Question Tag Question

Feedback

https://mypastest.pastest.com/Secure/TestMe/Browser/436619[‫ ص‬07:08:21 11/12/1437]


MyPastest

End Session
Difficulty: Average

Peer Responses

Session Progress

Responses Correct: 0

Responses Incorrect: 90

Responses Total: 90

Responses - % Correct: 0%

Blog
About Pastest
Contact Us
Help

© Pastest 2016

https://mypastest.pastest.com/Secure/TestMe/Browser/436619[‫ ص‬07:08:21 11/12/1437]


MyPastest

Prefer to use the old MyPastest? Access it here »

Back to Filters

Question 87 of 90

Previous Question
Which respiratory virus is responsible for the majority of cases ofbronchiolitis in the UK?

A Adenovirus

B Human metapneumovirus

C Influenza

D Parainfluenza

E Respiratory syncytial virus

Explanation
Respiratory syncytial virus (RSV) causes the majority of cases of bronchiolitis in the UK. All the other viruses –
Adenovirus, human metapneumovirus, influenza and parainfluenza are all known causes of bronchiolitis to a
lesser extent.

Bronchiolitis is the most common disease of the lower respiratory tract during the first year of life. It usually
presents with cough with increased work of breathing and it often affects a child's ability to feed. Symptoms are
usually mild and might only last for a few days, but in some cases the disease can cause severe illness.

Reference website: https://www.nice.org.uk/guidance/NG9/documents/bronchiolitis-in-children-final-scope2


46180

Next Question Tag Question

Feedback

https://mypastest.pastest.com/Secure/TestMe/Browser/436619[‫ ص‬07:08:41 11/12/1437]


MyPastest

End Session
Difficulty: Average

Peer Responses

Session Progress

Responses Correct: 0

Responses Incorrect: 91

Responses Total: 91

Responses - % Correct: 0%

Blog
About Pastest
Contact Us
Help

© Pastest 2016

https://mypastest.pastest.com/Secure/TestMe/Browser/436619[‫ ص‬07:08:41 11/12/1437]


MyPastest

Prefer to use the old MyPastest? Access it here »

Back to Filters

Question 88 of 90

Previous Question
Which one of the bacterial species below is incorrectly categorised?

A Enterococcusfaecalis – gram negative

B Klebsiella pneumoniae – gram negative

C Staphylococcus aureus – gram positive

D Streptococcus pneumoniae – gram positive

E Vibrio cholerae – gram negative

Explanation
Enterococcus faecalisis a gram positive cocci. The other bacterial species are correctly categorised.

Gram staining is a common technique used to differentiate two large groups of bacteria based on their different
cell wall constituents. Gram
staining distinguishes between Gram positive and Gram negative groups by colouring
cells red or violet. Gram positive bacteria stain violet due to the presence of a thick layer of peptidoglycan in their
cell walls. This retains the crystal violet these cells are stained with. Gram negative bacteria stain red, because of a
thinner peptidoglycan wall, which does not retain the crystal violet during the process.

Gram staining is one of the first steps in identifying which infectious agent is present.
46181

Next Question Tag Question

Feedback

https://mypastest.pastest.com/Secure/TestMe/Browser/436619[‫ ص‬07:09:02 11/12/1437]


MyPastest

End Session
Difficulty: Average

Peer Responses

Session Progress

Responses Correct: 0

Responses Incorrect: 92

Responses Total: 92

Responses - % Correct: 0%

Blog
About Pastest
Contact Us
Help

© Pastest 2016

https://mypastest.pastest.com/Secure/TestMe/Browser/436619[‫ ص‬07:09:02 11/12/1437]


MyPastest

Prefer to use the old MyPastest? Access it here »

Back to Filters

Question 89 of 90

Which immunoglobulin is deficient in premature babies putting them at increased risk of infection?

A Ig A

B Ig D

C Ig E

D Ig G

E Ig M

Explanation
The human body defends itself against infection in three ways – physical, cellular and humoral mechanisms.
Neonates, especially premature babies are deficient across all three defences placing them at increased risk of
infection.

The correct answer, IgG is deficient in the premature infant. IgG is actively and passively transported across the
placenta from approximately week 20 of pregnancy. By full term baby’s IgG is higher than his mother’s.
Following delivery IgG falls (half life
~ 3 weeks) until he produces his own. This transient
hypogammaglobulinaemia (low IgG, IgA, IgM) is rarely clinically important for a term baby but for a preterm
infant for whom insufficient
IgG crossed the placenta this is a time of considerable infection risk.

In the normal neonate of any gestation IgA, IgD, IgE and IgM should NOT be circulating. If they are found in
cord blood or the very early neonatal period it suggests fetal infection.
46206

Next Question

https://mypastest.pastest.com/Secure/TestMe/Browser/436619[‫ ص‬07:09:21 11/12/1437]


MyPastest

Tag Question

Feedback End Session

Difficulty: Average

Peer Responses

Previous Question

Session Progress

Responses Correct: 0

Responses Incorrect: 93

Responses Total: 93

Responses - % Correct: 0%

Blog
About Pastest
Contact Us
Help

© Pastest 2016

https://mypastest.pastest.com/Secure/TestMe/Browser/436619[‫ ص‬07:09:21 11/12/1437]


MyPastest

Prefer to use the old MyPastest? Access it here »

Back to Filters

Question 90 of 90

An 8 month old baby girl attends clinic. Her parents are worried she is not gaining weight, has watery, greasy
stools, is pale, lethargic and is not yet sitting by herself. She has been treated by the GP twice for otitis media and
once for pneumonia. The GP is also prescribing creams for eczema. Blood tests show a mild normochromic –
normocytic anaemia, mild thrombocytopenia and marked neutropenia.Newborn blood spot and sweat test are
negative.

What is the most likely diagnosis for this child?

A Coeliac Disease

B Cystic Fibrosis

C Diamond – Blackfan Anaemia

D Fanconi Anaemia

E Schwachman – Diamond Syndrome

Explanation
This baby should have the diagnosis (E) Schwachman-Diamond Syndrome (SDS) carefully considered. SDS is the
most
common cause of pancreatic exocrine insufficiency after Cystic Fibrosis
(steatorrhoea, FTT). Pearson's
syndrome, Johanssen–Blizzard syndrome, and severe malnutrition are other extremely rare possibilities.In contrast
to CF, pancreatic function often improves with
age. In addition, children with SDS have evidence of neutropenia
(most common), anaemia, thrombocytopenia or pancytopenia as evidence of bone marrow failure. The risk of
AML is increased. Older children show skeletal dysplasia and short stature. Severe eczema is often seen.

SDS is a genetic disorder caused by a gene mutation on Chromosome 7. It is usually inherited in an autosomal
recessive manner. 10% of cases are de novo.

SDS is the third most common congenital bone marrow


failure syndrome after Fanconi and Diamond–Blackfan
anaemia. Neither of these syndromes also present with pancreatic insufficiency.

https://mypastest.pastest.com/Secure/TestMe/Browser/436619[‫ ص‬07:09:42 11/12/1437]


MyPastest

Cystic Fibrosis should certainly be tested for but in this case both the newborn blood spot and sweat test are
negative for CF.

Coeliac disease can present with sequelae of malnutrition but it again does not present with pancreatic
insufficiency (although this can sometimes be co-existant) or bone marrow failure.

Webpage: http://www.gosh.nhs.uk/medical-information-0/search-medical-conditions/shwachman-diamond-
syndrome
46207

End Session

Previous Question Tag Question

Feedback

Difficulty: Average

Peer Responses

Session Progress

Responses Correct: 0

Responses Incorrect: 94

Responses Total: 94

Responses - % Correct: 0%

Blog
About Pastest
Contact Us
Help

© Pastest 2016

https://mypastest.pastest.com/Secure/TestMe/Browser/436619[‫ ص‬07:09:42 11/12/1437]


MyPastest

Prefer to use the old MyPastest? Access it here »

Back to Filters

Question 1 of 35
Next Question
Previous Question

Which of the following statements about prostaglandin synthesis is correct?

A It is activated by glucocorticoids End Session

B It is produced by lipoxygenase

C It is activated by aspirin

D It is produced by cyclooxygenase

E It causes vasoconstriction

Explanation
Prostaglandins (PGs) are biosynthesised from straight-chain C20 fatty acids, arachidonic acid being the precursor.
The responsible enzyme is cyclooxygenase, leukotrienes are formed by lipoxygenase. Glucocorticoids and aspirin
inhibit the prostaglandin synthesis. Prostaglandins lead to vasodilatation. The amounts of PGs in the tissues are
very low, indicating that they are biosynthesised immediately before hormone-stimulated release. PGs seem to be
metabolised by all body tissues. 12045

Tag Question

Feedback

Difficulty: Average

https://mypastest.pastest.com/Secure/TestMe/Browser/436619[‫ ص‬11:59:44 10/12/1437]


MyPastest

Peer Responses

Session Progress

Responses Correct: 0

Responses Incorrect: 1

Responses Total: 1

Responses - % Correct: 0%

Blog
About Pastest
Contact Us
Help

© Pastest 2016

https://mypastest.pastest.com/Secure/TestMe/Browser/436619[‫ ص‬11:59:44 10/12/1437]


MyPastest

Prefer to use the old MyPastest? Access it here »

Back to Filters

Question 2 of 35
Next Question

Which of the following statements is most accurate with regards to the chemoreceptor control of breathing?

A The aortic body is located in the root of the aorta

B When the CO2 concentration of plasma falls, the central chemoreceptors stimulate the inspiratory area
of the brain

C Central chemoreceptors are sensitive to the H+ content of the CSF

D The carotid bodies are located on the external carotid artery

E Central chemoreceptors are sensitive to the O2 content of the CSF

Explanation
The chemoreceptors that regulate respiration are located both centrally and peripherally. Normally control is
exercised by the central receptors located in the medulla, which respond to the CSF hydrogen ion concentration, in
turn determined by CO2, which diffuses freely across the blood-brain barrier from the arterial blood. The response
is both quick and sensitive to small changes in arterial CO2 (PaCO2). Whilst central chemoreceptors are therefore
(indirectly) sensitive to levels of carbon dioxide, peripheral chemoreceptors are also highly sensitive to oxygen.
There are three clusters of peripheral chemoreceptors: the aortic body located in the wall of the arch of the aorta,
and the two nodular carotid bodies in the left and right common carotid arteries. When the concentration of carbon
dioxide in the blood rises (hypercapnia), the central and peripheral chemoreceptors stimulate the inspiratory areas
to
stimulate the rate and depth of breathing. This hyperventilation causes
more carbon dioxide to be exhaled until
levels return to normal. 12075

https://mypastest.pastest.com/Secure/TestMe/Browser/436619[‫ م‬12:00:20 10/12/1437]


MyPastest

Tag Question

Feedback

Difficulty: Average

Peer Responses

Previous Question

End Session

Session Progress

Responses Correct: 0

Responses Incorrect: 2

Responses Total: 2

Responses - % Correct: 0%

Blog
About Pastest
Contact Us
Help

© Pastest 2016

https://mypastest.pastest.com/Secure/TestMe/Browser/436619[‫ م‬12:00:20 10/12/1437]


MyPastest

Prefer to use the old MyPastest? Access it here »

Back to Filters

Question 3 of 35
Next Question
Previous Question

Pulmonary gas exchange occurs under which of the following physiological principles?

A End Session
Gas exchange can occur in the final seven branches of the bronchoalveolar tree

B The first 12 branches of the bronchial tree are collectively known as the conducting zone

C The equilibration of gases takes about 2.5 s in the resting lung

D Only about 0.15% of oxygen is carried in solution in the plasma

E Carbon dioxide is less water-soluble than oxygen

Explanation
Gas exchange can occur in the final seven branches of the bronchoalveolar tree (the respiratory zone). The first 16
branches of the bronchial tree are collectively known as the conducting zone. The equilibration of gases takes
about 0.25 s in the resting lung.
About 1.5% of oxygen is carried in solution in the plasma. Carbon dioxide is more
water-soluble than oxygen. 12077

Tag Question

Feedback

Difficulty: Difficult

Peer Responses

https://mypastest.pastest.com/Secure/TestMe/Browser/436619[‫ م‬12:03:52 10/12/1437]


MyPastest

Session Progress

Responses Correct: 0

Responses Incorrect: 3

Responses Total: 3

Responses - % Correct: 0%

Blog
About Pastest
Contact Us
Help

© Pastest 2016

https://mypastest.pastest.com/Secure/TestMe/Browser/436619[‫ م‬12:03:52 10/12/1437]


MyPastest

Prefer to use the old MyPastest? Access it here »

Back to Filters

Question 4 of 35
Next Question
Previous Question

Which one of the following is MOST likely to increase during exercise?

A Peripheral vascular resistance End Session

B Pulmonary vascular resistance

C Stroke volume

D Diastolic pressure

E Venous compliance

Explanation
During exercise, increased oxygen consumption and increased venous return to the heart result in an increase in
cardiac output and an increase in blood flow to both skeletal muscle and coronary circulation, when oxygen
utilisation is greatest. The increase in cardiac output is due to an increase in both heart rate and stroke volume.
Systemic arterial pressure also increases in response to the increase in cardiac output. However, the fall in total
peripheral resistance, which is caused by dilatation of the blood vessels within the exercising muscles, results in a
decrease in diastolic blood pressure. The pulmonary vessels undergo passive dilatation as more blood
flows into
the pulmonary circulation. As a result, pulmonary vascular resistance decreases. The decrease in venous
compliance, caused by sympathetic stimulation, helps to maintain ventricular filling during diastole. 12078

Tag Question

Feedback

https://mypastest.pastest.com/Secure/TestMe/Browser/436619[‫ م‬12:04:15 10/12/1437]


MyPastest

Difficulty: Easy

Peer Responses

Session Progress

Responses Correct: 0

Responses Incorrect: 4

Responses Total: 4

Responses - % Correct: 0%

Blog
About Pastest
Contact Us
Help

© Pastest 2016

https://mypastest.pastest.com/Secure/TestMe/Browser/436619[‫ م‬12:04:15 10/12/1437]


MyPastest

Prefer to use the old MyPastest? Access it here »

Back to Filters

Question 5 of 35
Next Question
Previous Question

Which one of the following is higher at the apex of the lung than at the base when a person is standing?

A V/Q ratio End Session

B Ventilation

C PaCO2

D Compliance

E Blood flow

Explanation
The alveoli at the apex of the lung are larger than those at the base so their compliance is less. Because of the
reduced compliance, less inspired gas goes to the apex than to the base. Also, because the apex is above the heart
level, less blood flows through the apex than through the base. However, the reduction in air flow is less than the
reduction in blood flow, so that the V/Q ratio at the top of the lung is greater than it is at the bottom. The increased
V/Q ratio at
the apex makes PaCO2 lower and PaO2 higher at the apex than they are at the base. 12079

Tag Question

Feedback

Difficulty: Average

https://mypastest.pastest.com/Secure/TestMe/Browser/436619[‫ م‬12:04:34 10/12/1437]


MyPastest

Peer Responses

Session Progress

Responses Correct: 0

Responses Incorrect: 5

Responses Total: 5

Responses - % Correct: 0%

Blog
About Pastest
Contact Us
Help

© Pastest 2016

https://mypastest.pastest.com/Secure/TestMe/Browser/436619[‫ م‬12:04:34 10/12/1437]


MyPastest

Prefer to use the old MyPastest? Access it here »

Back to Filters

Question 6 of 35
Next Question
Previous Question

The secretion of growth hormone is increased by?

A Hyperglycaemia End Session

B Exercise

C Somatostatin

D Growth hormone

E Free fatty acids

Explanation
Growth hormone (GH) is synthesised, stored, and secreted by the endocrine cells of the anterior pituitary. Its
release is stimulated by growth hormone-releasing hormone and inhibited by somatostatin. Numerous factors
serve as a stimulus for GH release, including hypoglycaemia (e.g. insulin administration), moderate to severe
exercise, stress due to emotional disturbances, illness, and fever, and dopamine agonists such as bromocriptine.
Growth hormone itself will inhibit further release of growth hormone due to negative feedback on the pituitary
gland and hypothalamus.
12080

Tag Question

Feedback

https://mypastest.pastest.com/Secure/TestMe/Browser/436619[‫ م‬12:04:53 10/12/1437]


MyPastest

Difficulty: Average

Peer Responses

Session Progress

Responses Correct: 0

Responses Incorrect: 6

Responses Total: 6

Responses - % Correct: 0%

Blog
About Pastest
Contact Us
Help

© Pastest 2016

https://mypastest.pastest.com/Secure/TestMe/Browser/436619[‫ م‬12:04:53 10/12/1437]


MyPastest

Prefer to use the old MyPastest? Access it here »

Back to Filters

Question 7 of 35
Next Question

A 9-year-old boy presents with recurrent episodes of early morning dizziness, which resolve rapidly when he has
his breakfast.

Which of the following metabolic pathways is most likely to be functioning abnormally?

A Gluconeogenesis

B Glycolysis

C Glycogen synthesis

D Ketogenesis

E The tricarboxylic acid cycle (citric acid cycle, Krebs’ cycle)

Explanation
The history is typical of hypoglycaemia (early morning dizziness resolving on eating). Blood glucose
concentration is maintained in the fasting state through glucose release from glycogen (glycogenolysis) and
gluconeogenesis (glucose synthesis from lactate and
other precursors, the problem in this case, as glucose is not
made from
precursors overnight).

Glycolysis consumes glucose: the activity of the glycolytic pathway is reduced during starvation, to conserve
glucose, with complementary increases in the production and oxidation of
ketones and fatty acids. Abnormalities
of the tricarboxylic acid cycle are very rare and do not cause hypoglycaemia.
13740

Tag Question

https://mypastest.pastest.com/Secure/TestMe/Browser/436619[‫ م‬12:05:12 10/12/1437]


MyPastest

Feedback

Difficulty: Average

Peer Responses

Previous Question

End Session

Session Progress

Responses Correct: 0

Responses Incorrect: 7

Responses Total: 7

Responses - % Correct: 0%

Blog
About Pastest
Contact Us
Help

© Pastest 2016

https://mypastest.pastest.com/Secure/TestMe/Browser/436619[‫ م‬12:05:12 10/12/1437]


MyPastest

Prefer to use the old MyPastest? Access it here »

Back to Filters

Question 8 of 35
Next Question
Previous Question

A 15-year-old boy, previously rather sedentary in his habits but alarmed by his increasing weight, is persuaded by
his new girlfriend to join a gym, and begins an exercise programme. Although there are several similar
people
enrolled in the programme, he finds that he is unable to keep up
with them because of muscle pain, particularly in
his legs. Within a few minutes of resting, however, the pains resolve. Investigation revealed a slight fall in blood
lactate concentration during exercise.

Which of the following is most likely to be the cause of his complaint?

A Decreased muscle glucose uptake

B Decreased hepatic glycogenolysis

C Decreased muscle glycogenolysis

D Impaired hepatic gluconeogenesis

E Premature atherosclerosis

Explanation
An ischaemic cause for his muscle pain is excluded by the failure of a rise in lactate concentration during exercise.
Although muscle can take up glucose from the blood, during intense activity it is partly dependent on glucose
supplied by the breakdown of muscle glycogen. The enzyme responsible is muscle phosphorylase, and the history
is typical of McArdle’s disease, i.e. an inherited deficiency of this enzyme.
13742

Tag Question

https://mypastest.pastest.com/Secure/TestMe/Browser/436619[‫ م‬12:05:52 10/12/1437]


MyPastest

Feedback

Difficulty: Average

Peer Responses

Session Progress
End Session
Responses Correct: 0

Responses Incorrect: 8

Responses Total: 8

Responses - % Correct: 0%

Blog
About Pastest
Contact Us
Help

© Pastest 2016

https://mypastest.pastest.com/Secure/TestMe/Browser/436619[‫ م‬12:05:52 10/12/1437]


MyPastest

Prefer to use the old MyPastest? Access it here »

Back to Filters

Question 9 of 35

A 7-year-old child presents to the emergency department with vomiting, drowsiness and a tachycardia of 145bpm.
The capillary blood gas shows: pH 7.20, pCO2 3, pO2 4.2, HCO3 11, BE -10, Blood glucose 30.

Which ONE of the following BEST describes the blood gas result?

A Compensated metabolic acidosis

B Compensated respiratory acidosis

C Decompensated metabolic acidosis

D Decompensated respiratory acidosis

E Mixed metabolic and respiratory acidosis

Explanation
Normal blood gas values:

pH 7.35-7.45
pCO2 4.5-6
HCO3 22-26
BE -2 - +2

The above is a decompensated metabolic acidosis as there is low bicarbonate in the presence of a negative base
excess and an acidic pH.

A compensated metabolic acidosis would have to have a normal pH in the presence of a low bicarbonate and a
negative base excess so stem A is incorrect. You would expect respiratory compensation from a lower than
expected pCO2.

https://mypastest.pastest.com/Secure/TestMe/Browser/436619[‫ م‬12:06:13 10/12/1437]


MyPastest

A compensated respiratory acidosis would have a normal pH in the presence of a higher than normal pCO2
and a
higher bicarbonate and positive base excess. You would expect this in chronic respiratory disorders and therefore
stem B is incorrect.

A decompensated respiratory acidosis would have an abnormal acidic pH in the presence of a raised pCO2. This is
most commonly the case with acute respiratory failure.

A mixed metabolic and respiratory acidosis would have an abnormally low pH in the presence of a high pCO2 and
a low bicarbonate and a negative base excess.

The first step in blood gas analysis is to look at the pH – acidic or alkaline or normal.

The second step is to establish the pCO2 – high, low or normal to see if there is respiratory acidosis or evidence of
compensation (full or complete). Next Question
Previous Question
The third step is to analyse the HCO3 or base excess level. In general, a low bicarbonate or a negative base excess
reflects a metabolic acidosis.
47536

End Session

Tag Question

Feedback

Difficulty: Average

Peer Responses

Session Progress

Responses Correct: 0

Responses Incorrect: 9

Responses Total: 9

Responses - % Correct: 0%

https://mypastest.pastest.com/Secure/TestMe/Browser/436619[‫ م‬12:06:13 10/12/1437]


MyPastest

Blog
About Pastest
Contact Us
Help

© Pastest 2016

https://mypastest.pastest.com/Secure/TestMe/Browser/436619[‫ م‬12:06:13 10/12/1437]


MyPastest

Prefer to use the old MyPastest? Access it here »

Back to Filters

Question 10 of 35
Next Question

A 7-year-old child presents to the emergency department with vomiting, drowsiness and a tachycardia of 145bpm.
The capillary blood gas shows: pH 7.20, pCO2 3, pO2 4.2, HCO3 11, BE -10, Blood glucose 30.

What is the most LIKELY diagnosis?

A Addison’s disease

B Cardiogenic shock

C Diabetic ketoacidosis

D Septicaemia

E Severe gastroenteritis

Explanation
The picture of vomiting, tachycardia and metabolic acidosis with an incredibly raised BM is suggestive of DKA.

Addison’s
disease is unlikely because the history would have other suggestive features such as long term steroid
use and hypoglycaemia would be an expected rather than hyperglycaemia.

Cardiogenic shock is unlikely because the history would include a background of cardiac problems or if the cause
is secondary to a tachyarrhythmia than the heart rate would be much higher.

Septicaemia is unlikely because other features such as rash, fevers, respiratory symptoms/signs or meningism
would be evident.

Severe gastroenteritis is unlikely because this is more likely to result in a hypoglycaemia.


47537

https://mypastest.pastest.com/Secure/TestMe/Browser/436619[‫ م‬12:06:33 10/12/1437]


MyPastest

Tag Question

Feedback

Difficulty: Average

Peer Responses

Previous Question

End Session

Session Progress

Responses Correct: 0

Responses Incorrect: 10

Responses Total: 10

Responses - % Correct: 0%

Blog
About Pastest
Contact Us
Help

© Pastest 2016

https://mypastest.pastest.com/Secure/TestMe/Browser/436619[‫ م‬12:06:33 10/12/1437]


MyPastest

Prefer to use the old MyPastest? Access it here »

Back to Filters

Question 11 of 35

A 7-year-old child presents to the emergency department with vomiting, drowsiness and a tachycardia of 145bpm.
The capillary blood gas shows: pH 7.30, pCO2 2.5, pO2 4.2, HCO3 9, BE -14, Blood glucose 2.2. This child is
normally on regular corticosteroids.

Which ONE of the following BEST describes the blood gas result?

A A fully compensated metabolic acidosis

B A fully compensated respiratory acidosis

C A partially compensated metabolic acidosis

D A partially compensated respiratory acidosis

E Mixed metabolic and respiratory acidosis

Explanation
Normal blood gas values:

pH 7.35-7.45
pCO2 4.5-6
HCO3 22-26
BE -2 - +2

Stem C, a partially compensated metabolic acidosis, is correct because the results show a near normal pH with a
low pCO2 and a low bicarbonate and negative base excess which is indicative of this finding.

A fully compensated metabolic acidosis would have to have a normal pH in the presence of a low bicarbonate and
a low pCO2 so A is incorrect.

https://mypastest.pastest.com/Secure/TestMe/Browser/436619[‫ م‬12:06:54 10/12/1437]


MyPastest

A fully compensated respiratory acidosis would have a normal pH in the presence of a high pCO2 and a high
bicarbonate with a positive base excess, therefore stem B is incorrect.

A blood gas result of low pH and high pCO2 with a high bicarbonate and positive base excess would indicate a
partially compensated respiratory acidosis so D is incorrect.

A blood gas result of an abnormally low pH in the presence of a high pCO2 and a low bicarbonate and a negative
base excess would indicate a mixed metabolic and respiratory acidosis so E is incorrect.

The first step in blood gas analysis is to look at the pH – acidic or alkaline or normal.

The second step is to establish the pCO2 – high, low or normal to see if there is respiratory acidosis or evidence of
compensation (full or complete).
Next Question
The third step isPrevious Question
to analyse the HCO3 or base excess level. In general, a low bicarbonate or a negative base excess
reflects a metabolic acidosis.
47538

End Session

Tag Question

Feedback

Difficulty: Average

Peer Responses

Session Progress

Responses Correct: 0

Responses Incorrect: 11

Responses Total: 11

Responses - % Correct: 0%

https://mypastest.pastest.com/Secure/TestMe/Browser/436619[‫ م‬12:06:54 10/12/1437]


MyPastest

Blog
About Pastest
Contact Us
Help

© Pastest 2016

https://mypastest.pastest.com/Secure/TestMe/Browser/436619[‫ م‬12:06:54 10/12/1437]


MyPastest

Prefer to use the old MyPastest? Access it here »

Back to Filters

Question 12 of 35

A 7-year-old child presents to the emergency department with vomiting, drowsiness and a tachycardia of 145bpm.
The capillary blood gas shows: pH 7.30, pCO2 2.5, pO2 4.2, HCO3 9, BE -14, Blood glucose 2.2. This child is
normally on regular corticosteroids.

What is the most LIKELY diagnosis?

A Addison’s Disease

B Cardiogenic shock

C Diabetic ketoacidosis

D Septicaemia

E Severe gastroenteritis

Explanation
This child is normally on regular corticosteroids
for Addison’s disease. Addison’s disease is the inability to
produce adequate endogenous steroids from the adrenals. This child is
in an addisonian crisis secondary to a
severe lack of cortisol and aldosterone. An Addisonian crisis can occur in the presence of intercurrent illness,
stress or surgery where the quantity of cortisol available is insufficient to meet the additional need.

The picture of vomiting, tachycardia and metabolic acidosis with a raised BM is suggestive of DKA.

Cardiogenic shock is unlikely because the history would include a background of cardiac problems or if the cause
is secondary to a tachyarrhythmia than the heart rate would be much higher.

Septicaemia is unlikely because other features such as rash, fever, respiratory symptoms/signs or meningism
would be evident.

Severe
gastroenteritis is unlikely as a sole cause of this presentation as there is background information of long
term steroid use. An episode of severe gastroenteritis can induce an Addisonian crisis, especially if regular oral

https://mypastest.pastest.com/Secure/TestMe/Browser/436619[‫ م‬12:07:14 10/12/1437]


MyPastest

steroids cannot be tolerated due to vomiting.


47539

Tag Question

Feedback

Next Question
Difficulty: Average
Previous Question
Peer Responses

End Session

Session Progress

Responses Correct: 0

Responses Incorrect: 12

Responses Total: 12

Responses - % Correct: 0%

Blog
About Pastest
Contact Us
Help

© Pastest 2016

https://mypastest.pastest.com/Secure/TestMe/Browser/436619[‫ م‬12:07:14 10/12/1437]


MyPastest

Prefer to use the old MyPastest? Access it here »

Back to Filters

Question 13 of 35

A 7-year-old child presents to the emergency department with vomiting, drowsiness and a tachycardia of 145bpm.
The capillary blood gas shows: pH 7.30, pCO2 2.5, pO2 4.2, HCO3 9, BE -14, Blood glucose 2.2. This child is
normally on regular corticosteroids.

With the above diagnosis what other biochemical features would you expect?

A Hypernatraemic hyperkalaemic dehydration

B Hypernatraemic hypokalaemic dehydration

C Hyponatraemic hyperkalaemic dehydration

D Hyponatraemic hypokalaemic dehydration

E None of the above

Explanation
Severe adrenal insufficiency is characteristed by
low plasma sodium and high plasma potassium. A low blood
sugar during an addisonian crisis is a late feature. The electrolyte disturbances seen in adrenal insufficiency is
secondary to a lack of cortisol and aldosterone. Aldosterone functions by increasing urinary potassium secretion
and therefore a lack of this hormone will result in hyperkalaemia. Aldosterone also enhances renal sodium
reabsorption and therefore lack of this hormone will result in hyponatraemia and hypotension.

Stems A and B are incorrect as hypernatraemia would


not be expected with a lack of aldosterone as this normally
leads to ‘salt-wasting.’ Hypernatraemic dehydration occurs from free water loss as intracellular water is drawn out
of cells into the extracellular fluid and therefore preserving the extracellular fluid volume.

Stem D is incorrect as hypokalaemia would not be expected. The aldosterone deficiency would result in reduced
urinary potassium secretion and therefore higher levels of blood potassium.

Stem E is incorrect as the correct answer from stem C is explained as above.


47540

https://mypastest.pastest.com/Secure/TestMe/Browser/436619[‫ م‬12:07:34 10/12/1437]


MyPastest

Tag Question

Feedback

Difficulty: Average

Next Question
Previous Question Peer Responses

End Session

Session Progress

Responses Correct: 0

Responses Incorrect: 13

Responses Total: 13

Responses - % Correct: 0%

Blog
About Pastest
Contact Us
Help

© Pastest 2016

https://mypastest.pastest.com/Secure/TestMe/Browser/436619[‫ م‬12:07:34 10/12/1437]


MyPastest

Prefer to use the old MyPastest? Access it here »

Back to Filters

Question 14 of 35

A 7-year-old child presents to the emergency department with vomiting, drowsiness and a tachycardia of 145bpm.
The capillary blood gas shows: pH 7.35, pCO2 2.5, pO2 4.2, HCO3 10, BE -11, Blood glucose 3.5. This child is
normally fit and well and is currently apyrexial.

Which ONE of the following BEST describes the blood gas result?

A A fully compensated metabolic acidosis

B A fully compensated respiratory acidosis

C A partially compensated metabolic acidosis

D A partially compensated respiratory acidosis

E Mixed metabolic and respiratory acidosis

Explanation
Normal blood gas values:

pH 7.35-7.45
pCO2 4.5-6
HCO3 22-26
BE -2 - +2

Stem A, a fully compensated metabolic acidosis, is correct because the results show a normal pH with a low pCO2
and a low bicarbonate and negative base excess which is indicative of this finding.

A fully compensated respiratory acidosis acidosis would have a normal pH in the presence of a high pCO2 and a
high bicarbonate with a positive base excess, therefore stem B is incorrect.

https://mypastest.pastest.com/Secure/TestMe/Browser/436619[‫ م‬12:07:57 10/12/1437]


MyPastest

A partially compensated metabolic acidosis would show a near normal or low pH with a low pCO2 and a low
bicarbonate and a negative base excess and therefore stem C is incorrect.

A blood gas result of low pH and high pCO2 with a high bicarbonate and positive base excess would indicate a
partially compensated respiratory acidosis so D is incorrect.

A blood gas result of an abnormally low pH in the presence of a high pCO2 and a low bicarbonate and a negative
base excess would indicate a mixed metabolic and respiratory acidosis so E is incorrect.

The first step in blood gas analysis is to look at the pH – acidic or alkaline or normal.

The second step is to establish the pCO2 – high, low or normal to see if there is respiratory acidosis or evidence of
compensation (full or complete).
Next Question
The third step isPrevious Question
to analyse the HCO3 or base excess level. In general, a low bicarbonate or a negative base excess
reflects a metabolic acidosis.
47541

End Session

Tag Question

Feedback

Difficulty: Average

Peer Responses

Session Progress

Responses Correct: 0

Responses Incorrect: 14

Responses Total: 14

Responses - % Correct: 0%

https://mypastest.pastest.com/Secure/TestMe/Browser/436619[‫ م‬12:07:57 10/12/1437]


MyPastest

Blog
About Pastest
Contact Us
Help

© Pastest 2016

https://mypastest.pastest.com/Secure/TestMe/Browser/436619[‫ م‬12:07:57 10/12/1437]


MyPastest

Prefer to use the old MyPastest? Access it here »

Back to Filters

Question 15 of 35

A 7-year-old child presents to the emergency department with a


2 day history of vomiting, diarrhoea, drowsiness
and a tachycardia of 145bpm. The capillary blood gas shows: pH 7.35, pCO2 2.5, pO2 4.2, HCO3 10, BE -11,
Blood glucose 3.5. This child is normally fit and well and is currently apyrexial.

What is the most LIKELY diagnosis?

A Addison’s Disease

B Cardiogenic shock

C Diabetic ketoacidosis

D Septicaemia

E Severe gastroenteritis

Explanation
This child is normally fit and well and presents acutely unwell over the last 2 days with vomiting and diarrhoea.
There is usually a low grade or no pyrexia noted in cases of gastroenteritis. This most likely is an episode of
gastroenteritis. The blood gas reflects dehydration secondary to the diarrhoea and vomiting.

Addison’s disease is the inability to produce adequate endogenous steroids from the adrenals. This child is in an
addisonian crisis secondary to a severe lack of cortisol and aldosterone. An Addisonian crisis can occur in the
presence of intercurrent illness, stress or surgery where the quantity of cortisol available is insufficient to meet
the
additional need. There are no features to suggest this as an underlying cause in the above scenario.

Cardiogenic shock is unlikely because the history would include a background of cardiac problems or if the cause
is secondary to a tachyarrhythmia than the heart rate would be much higher.

The picture of vomiting, tachycardia and metabolic acidosis with a raised BM is suggestive of DKA.

Septicaemia is unlikely because other features such as rash, fever, respiratory symptoms/signs or meningism

https://mypastest.pastest.com/Secure/TestMe/Browser/436619[‫ م‬12:08:17 10/12/1437]


MyPastest

would be evident.
47542

Tag Question

Feedback

Next Question
Difficulty: Average
Previous Question
Peer Responses

End Session

Session Progress

Responses Correct: 0

Responses Incorrect: 15

Responses Total: 15

Responses - % Correct: 0%

Blog
About Pastest
Contact Us
Help

© Pastest 2016

https://mypastest.pastest.com/Secure/TestMe/Browser/436619[‫ م‬12:08:17 10/12/1437]


MyPastest

Prefer to use the old MyPastest? Access it here »

Back to Filters

Question 16 of 35

A 7-year-old child presents to the emergency department with vomiting, drowsiness and a tachycardia of 145bpm.
The capillary blood gas shows: pH 7.15, pCO2 8.0, pO2 2.0, HCO3 10, BE -12, Blood glucose 5. This child has
been pyrexial.

Which ONE of the following BEST describes the blood gas result?

A A fully compensated metabolic acidosis

B A fully compensated respiratory acidosis

C A partially compensated metabolic acidosis

D A partially compensated respiratory acidosis

E Mixed metabolic and respiratory

Explanation
Normal blood gas values:

pH 7.35-7.45
pCO2 4.5-6
HCO3 22-26
BE -2 - +2

The correct answer is stem E because a blood gas result of an abnormally low pH in the presence of a high pCO2
and a low bicarbonate and a negative base excess would indicate a mixed metabolic and respiratory acidosis.

Stem A, a fully compensated metabolic acidosis, is incorrect because the results show a normal pH with a low
pCO2 and a low bicarbonate and negative base excess which is indicative of this finding.

https://mypastest.pastest.com/Secure/TestMe/Browser/436619[‫ م‬12:08:47 10/12/1437]


MyPastest

Stem B is incorrect because a fully compensated respiratory acidosis would have a normal pH in the presence of a
high pCO2 and a high bicarbonate with a positive base excess, therefore stem B is incorrect.

A partially compensated metabolic acidosis would show a near normal or low pH with a low pCO2 and a low
bicarbonate and a negative base excess and therefore stem C is incorrect.

A blood gas result of low pH and high pCO2 with a high bicarbonate and positive base excess would indicate a
partially compensated respiratory acidosis so D is incorrect.

The first step in blood gas analysis is to look at the pH – acidic or alkaline or normal.

The second step is to establish the pCO2 – high, low or normal to see if there is respiratory acidosis or evidence of
compensation (full or complete).
Next Question
The third step isPrevious Question
to analyse the HCO3 or base excess level. In general, a low bicarbonate or a negative base excess
reflects a metabolic acidosis.
47543

End Session

Tag Question

Feedback

Difficulty: Average

Peer Responses

Session Progress

Responses Correct: 0

Responses Incorrect: 16

Responses Total: 16

Responses - % Correct: 0%

https://mypastest.pastest.com/Secure/TestMe/Browser/436619[‫ م‬12:08:47 10/12/1437]


MyPastest

Blog
About Pastest
Contact Us
Help

© Pastest 2016

https://mypastest.pastest.com/Secure/TestMe/Browser/436619[‫ م‬12:08:47 10/12/1437]


MyPastest

Prefer to use the old MyPastest? Access it here »

Back to Filters

Question 17 of 35

A 7-year-old child presents to the emergency department with vomiting, drowsiness and a tachycardia of 145bpm.
The capillary blood gas shows: pH 7.15, pCO2 8.0, pO2 2.0, HCO3 10, BE -12, Blood glucose 5. This child has a
fever of 41C and a rapidly spreading non-blanching rash.

What is the most LIKELY diagnosis?

A Addison’s Disease

B Cardiogenic shock

C Diabetic ketoacidosis

D Septicaemia

E Severe gastroenteritis

Explanation
This child has severe septicaemia and it is reflected with the presence of a mixed respiratory and metabolic
acidosis. This is evidence of severe sepsis as there is end organ involvement and it is likely that this child needs
further ventilator and circulatory support. A history of fever in conjunction with a non-blanching rash is
suggestive of this diagnosis. The most likely cause is a meningococcal septicaemia and this needs to be treated as
per
APLS guidance followed by prompt administration of ceftriaxone.

Addison’s disease is the inability to produce adequate endogenous


steroids from the adrenals. This child is in an
addisonian crisis secondary to a severe lack of cortisol and aldosterone. An Addisonian crisis can occur in the
presence of intercurrent illness, stress or surgery where the quantity of cortisol available is insufficient to meet
the
additional need. There are no features to suggest this as an underlying cause in the above scenario.

Cardiogenic shock is unlikely because the history would include a background of cardiac problems or if the cause
is secondary to a tachyarrhythmia than the heart rate would be much higher.

https://mypastest.pastest.com/Secure/TestMe/Browser/436619[‫ م‬12:09:08 10/12/1437]


MyPastest

The picture of vomiting, tachycardia and metabolic acidosis with a raised BM is suggestive of DKA.

There is usually a low grade or no pyrexia noted in cases of gastroenteritis and is often accompanied by diarrhoea
and therefore stem
E is unlikely as an answer.
47544

Tag Question

Feedback Next Question


Previous Question
Difficulty: Average

Peer Responses
End Session

Session Progress

Responses Correct: 0

Responses Incorrect: 17

Responses Total: 17

Responses - % Correct: 0%

Blog
About Pastest
Contact Us
Help

© Pastest 2016

https://mypastest.pastest.com/Secure/TestMe/Browser/436619[‫ م‬12:09:08 10/12/1437]


MyPastest

Prefer to use the old MyPastest? Access it here »

Back to Filters

Question 18 of 35

A 12-year-old Asian girl complains of a 2-month history of tiredness, spasm of her hands and tingling of her
hands. She is vegetarian and has always been a fussy eater. On examination she is on the 9th centile for weight
and 25th centile for height. Baseline bloods show a calcium of 1.9mmol/L. Her full blood count and urea &
electrolytes are normal.

What is the most LIKELY diagnosis?

A Eating disorder

B Hypoparathyroidism

C Psychogenic cause

D Renal disease

E Vitamin D deficiency

Explanation
This child most likely has symptoms of hypocalcaemia because hypocalcaemia can result in tetany, carpopedal
spasms and tingling of the hands. Vitamin D deficiency is the most likely underlying cause in view of her ethnicity
and dietary requirements. She requires vitamin D and calcium replacement.

An
eating disorder is unlikely because there are no other features in the history to suggest this such as vomiting,
self-induced weight loss and no body mass index is provided in the information above.

The diagnosis of hypoprathyroidism would need to be based on a low PTH level, which is not provided in the
scenario above. In view of the ethnicity and dietary habits, vitamin D deficiency in the above scenario
is much
more likely than hypoparathyroidism. Hypoparathyroidism is secondary to an underproduction of parathyroid
hormone from the parathyroid glands. This results in hypocalcaemia. The management is limited as this hormone
cannot be administered in an artificial form and
therefore calcium replacement is the mainstay.

https://mypastest.pastest.com/Secure/TestMe/Browser/436619[‫ م‬12:10:02 10/12/1437]


MyPastest

Stem C is incorrect as her blood calcium is clearly abnormal and with symptoms that fit hypocalcaemia a
psychogenic cause is unlikely.

Stem D is highly unlikely as there are no features in the history to suggest the background for acute or chronic
renal impairment. Also the serum urea and electrolytes are normal which would not be the case if there was renal
impairment.
47545

Next Question Tag Question


Previous Question
Feedback

Difficulty: Average
End Session
Peer Responses

Session Progress

Responses Correct: 0

Responses Incorrect: 18

Responses Total: 18

Responses - % Correct: 0%

Blog
About Pastest
Contact Us
Help

© Pastest 2016

https://mypastest.pastest.com/Secure/TestMe/Browser/436619[‫ م‬12:10:02 10/12/1437]


MyPastest

Prefer to use the old MyPastest? Access it here »

Back to Filters

Question 19 of 35
Next Question

A 12-year-old Asian girl has recently been given a diagnosis of vitamin D deficiency incidentally on some routine
blood tests. Her parents are concerns about the effects of this deficiency.

Which ONE of the following best describes the function of vitamin D?

A Decreasing bone formation

B Decreasing bone mineralisation

C Decreasing the intestinal absorption of phosphate

D Increasing calcium mobilisation from the bone

E Increasing the intestinal absorption of calcium

Explanation
Vitamin D is obtained from the diet or the action
of sunlight on the skin. Vitamin D from the circulation is
metabolised in the liver to 25-hydroxyvitamin D and then this is hydroxylated in the
kidney to 1,25-
dihydroxyvitamin D. Vitamin D acts on the intestine to increase calcium and phosphate absorption, increases bone
mineralisation
and increases bone formation and therefore stem A is correct.

Stem B is incorrect as vitamin D acts to increase the intestinal absorption of phosphate.

Stems C and D are incorrect as vitamin D improves bone mineralisation and promotes bone formation.

Stem E is incorrect as vitamin D does not increase calcium mobilisation


from the bone. This is a function of
parathyroid hormone.
47546

https://mypastest.pastest.com/Secure/TestMe/Browser/436619[‫ م‬12:10:34 10/12/1437]


MyPastest

Tag Question

Feedback

Difficulty: Average

Peer Responses

Previous Question

End Session

Session Progress

Responses Correct: 0

Responses Incorrect: 19

Responses Total: 19

Responses - % Correct: 0%

Blog
About Pastest
Contact Us
Help

© Pastest 2016

https://mypastest.pastest.com/Secure/TestMe/Browser/436619[‫ م‬12:10:34 10/12/1437]


MyPastest

Prefer to use the old MyPastest? Access it here »

Back to Filters

Question 20 of 35
Next Question

A 10-year-old girl has recently been given a diagnosis of vitamin D deficiency after a history of feeling generally
lethargic. Her parents are anxious and are wondering what can cause vitamin D deficiency.

Which of the following statements best describes the cause of vitamin D deficiency?

A Dietary insufficiency

B Hepatic or renal disease

C Inadequate sunshine exposure

D Malabsorption

E All of the above

Explanation
All of the above are causes of vitamin D deficiency.

Malabsorptive conditions such as coeliac disease and pancreatic insufficiency cause vitamin D deficiency.

As Vitamin D is converted into active forms in the liver and kidney, hepatic and renal disease can contribute to
vitamin D deficiency.

Dermal synthesis of vitamin D from cholesterol is highly influenced by the amount of sun exposure. Synthesis of
vitamin D in the skin is the major natural source of this vitamin especially from UVB radiation.

Very few foods contain vitamin D and therefore dietary habits are also important. Foods such as oily fish, eggs
and fortified cereals are the main sources of dietary vitamin D.
47547

https://mypastest.pastest.com/Secure/TestMe/Browser/436619[‫ م‬12:10:55 10/12/1437]


MyPastest

Tag Question

Feedback

Difficulty: Average

Peer Responses

Previous Question

End Session

Session Progress

Responses Correct: 0

Responses Incorrect: 20

Responses Total: 20

Responses - % Correct: 0%

Blog
About Pastest
Contact Us
Help

© Pastest 2016

https://mypastest.pastest.com/Secure/TestMe/Browser/436619[‫ م‬12:10:55 10/12/1437]


MyPastest

Prefer to use the old MyPastest? Access it here »

Back to Filters

Question 21 of 35
Next Question

A 9-year-old girl has recently had bloods for lethargy and excessive muscle pain. Her bloods showed a low
vitamin D and calcium level. Her parents are not sure as to what the management options are.

Which ONE of the following best describes the management of vitamin D deficiency in the above case?

A Oral calcium supplementation alone

B Oral vitamin D and calcium supplementation

C Oral vitamin D plus calcium if there is hypocalcaemia

D Oral vitamin D supplementation alone

E None of the above

Explanation
Stem C is the correct answer as additional calcium should be added to oral vitamin D if there is evidence of
hypocalcaemia.

Stem A is incorrect as treating the hypocalcaemia alone will not rectify the underlying cause of low vitamin D.

Stem
B is incorrect as if the calcium level is normal then additional administration of calcium may increase the
risk of hypercalcaemia.

Stem
D is incorrect as oral vitamin D alone may help the symptoms in the long term but the hypocalcaemia will
remain uncorrected for a longer period of time.
47548

https://mypastest.pastest.com/Secure/TestMe/Browser/436619[‫ م‬12:11:15 10/12/1437]


MyPastest

Tag Question

Feedback

Difficulty: Average

Peer Responses

Previous Question

End Session

Session Progress

Responses Correct: 0

Responses Incorrect: 21

Responses Total: 21

Responses - % Correct: 0%

Blog
About Pastest
Contact Us
Help

© Pastest 2016

https://mypastest.pastest.com/Secure/TestMe/Browser/436619[‫ م‬12:11:15 10/12/1437]


MyPastest

Prefer to use the old MyPastest? Access it here »

Back to Filters

Question 22 of 35

A 2-year-old child presents with bowing of her legs and delayed closure of her fontanelle. She is a fussy eater. She
is otherwise fit and well and is on the 25th centile for height and weight.

What other investigations would you undertake to confirm most likely diagnosis?

A Serum calcium, alkaline phosphatase, vitamin D and phosphate

B Serum calcium, alkaline phosphatase, vitamin D and X-rays

C Serum calcium, alkaline phosphatase, vitamin D, phosphate and parathyroid hormone

D Serum calcium, alkaline phosphatase, vitamin D, phosphate and parathyroid hormone and X-rays

E None of the above

Explanation
The investigations in stem D would help diagnose rickets. Diagnosis would be confirmed:

ALP: high (125 - 320 U/L)


PTH: high (1.2 – 5.8pmol/L)
Calcium: low or normal (2.25-2.50mmol/L)
Phosphate: low or normal (0.81 – 1.45 mmol/L)
Vitamin D: low (24 – 65pg/mL)
X-ray: widened metaphyses with cupping and splaying

Stem A would not be the most helpful as without radiological evidence of rickets is helpful in establishing the
extent of rickets. Also parathyroid hormone would help to the biochemical diagnosis as it would be elevated.

Stem B is incorrect as the phosphate level would be helpful in establishing the presence of hypophosphataemic
rickets.

Stem C is incorrect as a radiology may help to guide the diagnosis and/or the severity of rickets.

https://mypastest.pastest.com/Secure/TestMe/Browser/436619[‫ م‬12:11:38 10/12/1437]


MyPastest

Stem E is incorrect as the presence of delayed fontanelle closure and bowed legs requires investigations for
rickets. These are subtle features of rickets but must be taken seriously.
47549

Tag Question

Feedback
Next Question
Previous Question
Difficulty: Average

Peer Responses

End Session

Session Progress

Responses Correct: 0

Responses Incorrect: 22

Responses Total: 22

Responses - % Correct: 0%

Blog
About Pastest
Contact Us
Help

© Pastest 2016

https://mypastest.pastest.com/Secure/TestMe/Browser/436619[‫ م‬12:11:38 10/12/1437]


MyPastest

Prefer to use the old MyPastest? Access it here »

Back to Filters

Question 23 of 35

A 5-year-old child presents with bowed legs, tiredness and muscle spasms. She has previously had a greenstick
fracture of her radius from a minor fall 6 months ago. Her weight is on the 25th centile
and her height is on the 9th
centile.

Based on the above diagnosis, what other clinical feature would you expect?

A Constipation

B Delayed dentition

C Delayed development

D Weight gain

E Widened fontanelle

Explanation
A feature of rickets is delayed dentition. Vitamin D ensures that serum calcium and phosphate levels are sufficient
to allow effective mineralisation of the teeth and bone.

Stem B
is incorrect as constipation is not a feature. Constipation can be seen
with hypercalcaemia and this is not
the case with vitamin D deficiency as a low or a normal level of calcium is seen.

Delayed development is a common feature of vitamin D deficiency. However, growth restriction may be noted.

Stem D is incorrect as vitamin D deficiency often results in normal or faltered growth pattern.

Although a widened and delayed fontanelle closure is noted with rickets, this would not be a feature that is
expected in a 4-year-old child.
47550

https://mypastest.pastest.com/Secure/TestMe/Browser/436619[‫ م‬12:11:57 10/12/1437]


MyPastest

Tag Question

Feedback

Difficulty: Average

Next
PeerQuestion
Responses
Previous Question

End Session

Session Progress

Responses Correct: 0

Responses Incorrect: 23

Responses Total: 23

Responses - % Correct: 0%

Blog
About Pastest
Contact Us
Help

© Pastest 2016

https://mypastest.pastest.com/Secure/TestMe/Browser/436619[‫ م‬12:11:57 10/12/1437]


MyPastest

Prefer to use the old MyPastest? Access it here »

Back to Filters

Question 24 of 35

A 3-year-old presented to the paediatric outpatient clinic with a family history of vitamin D deficiency and the GP
was concerned about the possibility of rickets n this child.

Which statement best describes the clinical features associated with rickets in this age group?

A Bowed legs, craniotabes and proximal myopathy

B Bowed legs, craniotabes and rachitic rosary

C Bowed legs, proximal myopathy and rachitic rosary

D Bowed legs, rachitic rosary and tetany

E Bowed legs, tetany and proximal myopathy

Explanation
The clinical features of rickets include and therefore best matches stem B as the correct answer:

Craniotabes: softening of the skull with delayed closure of fontanelles


Rachitic rosary: enlargement of the costochondral junctions
Bowed legs: in children that are weight bearing
Delayed dentition
Symptoms of hypocalcaemia: tetany, carpopedal spasm and tingling of the hands usually occurs in the
adolescent age band.

Stems A and C are incorrect as proximal myopathy is not a feature of rickets.

Stem D is incorrect as tetany would be expected in an older child who has rickets.

https://mypastest.pastest.com/Secure/TestMe/Browser/436619[‫ م‬12:12:17 10/12/1437]


MyPastest

Stem E is incorrect as proximal myopathy is not a predominant feature and also tetany would present in an older
child.
47551

Tag Question

Feedback

Next Question
Previous Question Difficulty: Average

Peer Responses

End Session

Session Progress

Responses Correct: 0

Responses Incorrect: 24

Responses Total: 24

Responses - % Correct: 0%

Blog
About Pastest
Contact Us
Help

© Pastest 2016

https://mypastest.pastest.com/Secure/TestMe/Browser/436619[‫ م‬12:12:17 10/12/1437]


MyPastest

Prefer to use the old MyPastest? Access it here »

Back to Filters

Question 25 of 35

You are teaching a group of 4th year medical students and are
asked the following what is the role of parathyroid
hormone in bone metabolism?

Of the following what would be the best response?

A Decreases renal calcium absorption and mobilises calcium and phosphate from the bone

B Decreases renal phosphate absorption and increases skeletal mineralisation

C Increases gut calcium absorption and increases skeletal mineralisation

D Increases renal calcium absorption and mobilises calcium and phosphate from the bone

E Increases renal phosphate absorption and increases skeletal mineralisation

Explanation
Stem C is correct. There are 4 parathyroid glands which are located behind the thyroid gland in the neck. They are
responsible for secreting parathyroid hormone as a response to low levels of ionised calcium in the blood. The role
of PTH is to increase renal calcium absorption and mobilises calcium and phosphate from the bone.

Stem A is incorrect because PTH increases renal calcium absorption and mobilises calcium and phosphate from
the bone.

Stem B is incorrect because PTH is to increase renal calcium absorption and reduces skeletal mineralisation.

Stem D is incorrect because PTH decreases skeletal mineralisation.

Stem E is incorrect because this is a function of vitamin D to increase gut calcium absorption and increase bone
mineralisation
47552

https://mypastest.pastest.com/Secure/TestMe/Browser/436619[‫ م‬12:12:38 10/12/1437]


MyPastest

Tag Question

Feedback

Difficulty: Average

Next
PeerQuestion
Responses
Previous Question

End Session

Session Progress

Responses Correct: 0

Responses Incorrect: 25

Responses Total: 25

Responses - % Correct: 0%

Blog
About Pastest
Contact Us
Help

© Pastest 2016

https://mypastest.pastest.com/Secure/TestMe/Browser/436619[‫ م‬12:12:38 10/12/1437]


MyPastest

Prefer to use the old MyPastest? Access it here »

Back to Filters

Question 26 of 35

A 3-week-old baby presents with vomiting, poor feeding and respiratory distress.

What is the LEAST likely diagnosis?

A Congenital cardiac disease

B Inborn error of metabolism

C Intussusception

D Meningitis

E Septicaemia

Explanation
In an unwell neonate the main causes to consider are sepsis with or without the presence of meningitis, an
underlying cardiac condition or an inborn error of metabolism.

Inborn errors of metabolism can present with a range of symptoms depending on the age of the child. In the
neonatal period the commonest presentation will include poor feeding, poor suck, vomiting, hypotonia, respiratory
distress, apnoeas, multi-organ failure, seizures and encephalopathy. This is often mistaken for sepsis.

Septicaemia with or without meningitis can present with a wide range of symptoms including vomiting,
poor
feeding, lethargy, respiratory distress, fever and dehydration.

Congenital cardiac conditions can also present in the neonatal period in a similar manner with respiratory distress,
poor feeding and circulatory collapse.

Babies of this age group may rarely have bowel obstruction but Intussusception often presents in older babies.
47553

https://mypastest.pastest.com/Secure/TestMe/Browser/436619[‫ م‬12:12:57 10/12/1437]


MyPastest

Tag Question

Feedback

Difficulty: Average

Next
PeerQuestion
Responses
Previous Question

End Session

Session Progress

Responses Correct: 0

Responses Incorrect: 26

Responses Total: 26

Responses - % Correct: 0%

Blog
About Pastest
Contact Us
Help

© Pastest 2016

https://mypastest.pastest.com/Secure/TestMe/Browser/436619[‫ م‬12:12:57 10/12/1437]


MyPastest

Prefer to use the old MyPastest? Access it here »

Back to Filters

Question 27 of 35

A 5-year-old child with global developmental delay and challenging behaviour has recently been diagnosed with
an inborn error of metabolism following extensive investigations.

Which ONE of the following symptoms is NOT characteristic of an inborn error of metabolism in this age group?

A Recurrent episodes of encephalopathy

B Recurrent hypocalcaemia

C Recurrent hypoglycaemia

D Stroke like episodes

E Unexplained seizures

Explanation
Hypocalcaemia is more characteristic of metabolic
bone disorders such as vitamin D deficiency or
hypoparathyroidism. Other causes of hypocalcaemia include severe sepsis, hypoalbuminaemia and
hypomagnesemia.

The presentation of metabolic disease in the


older child is different to that of a neonate. Older children tend to
present with episodes of recurrent vomiting associated with dehydration,
unexplained seizures, episodes of
encephalopathy, stroke-like episodes,
hepatic or renal failure and cardiomyopathy. Metabolic conditions which
present with hypoglycaemia include fatty acid oxidation defects (e.g. MCADD) and glycogen storage disorders.

Encephalpathy is a feature of metabolic disorders such as urea cycle defects, maple syrup urine disease, fatty acid
oxidation disorders and organic acidaemias.

Stroke-like episodes and unexplained seizures are predominantly features of mitochondrial disorders such as
MELAS and MERRF.

Other features of metabolic disease include unusual odours (e.g. Maple syrup urine disorder, musty in

https://mypastest.pastest.com/Secure/TestMe/Browser/436619[‫ م‬12:13:17 10/12/1437]


MyPastest

phenylketonuria), developmental delay or regression, hepatosplenomegaly (e.g. mucopolysaccharidosis, certain


glycogen storage disorders & sphingolipidoses), opthalomological findings (e.g. cherry red spot in lysosomal
disorders, cataracts in galactosaemia and cloudy cornea in Hurler’s syndrome).

Certain
phenotypes may also aid clinical diagnosis of metabolic disorders for example a Marfanoid habitus in
homocysteinuria and course facies in mucopolysaccharidosis.
47554

Next Question Tag Question


Previous Question
Feedback

Difficulty: Average
End Session
Peer Responses

Session Progress

Responses Correct: 0

Responses Incorrect: 27

Responses Total: 27

Responses - % Correct: 0%

Blog
About Pastest
Contact Us
Help

© Pastest 2016

https://mypastest.pastest.com/Secure/TestMe/Browser/436619[‫ م‬12:13:17 10/12/1437]


MyPastest

Prefer to use the old MyPastest? Access it here »

Back to Filters

Question 28 of 35

A 6-day-old baby presents with poor feeding, vomiting, encephalopathy and acidosis. He was born at 38 weeks
gestation with no risk factors for sepsis and was well up until this presentation. The parents are consanguineous
and there is a family history of sudden infant deaths.

Which of the following investigations will be of LEAST specific in obtaining a diagnosis?

A Acylcarnitine profile

B Serum amino acids

C Serum ammonia

D Serum lactate

E Urine organic acids

Explanation
There are many baseline investigations that are involved in a metabolic screen. It is important to know which
investigations will be the most useful in helping to provide a more specific cause.

Serum ammonia:

Urea cycle defects (high ammonia)

Serum amino acids:

Maple syrup urine disorder (high leucine/isoleucine/valine)


Urea cycle defect (high glutamine)
Phenylketonuria (High phenylalanine)
Tyrosinaemia (high tyrosine)

https://mypastest.pastest.com/Secure/TestMe/Browser/436619[‫ م‬12:13:39 10/12/1437]


MyPastest

Non-ketotic hyperglycinaemia (high glycine)

Urine organic acids:

Propronic acidaemia (high propronate)


Isovaleric acidaemia (high isovalerate)
Urea cycle defects (high orotic acid in OTC)
Tyrosinaemia (high succinylacetone)
Fatty acid oxidation disorders (high dicarboxylic acid)

Acylcarnitine profile:
Next Question
Previous
Fatty Question
acid oxidation defects (low carnitine)

Plasma lactate:

Elevated
in mitochondrial disorders, glycogen storage disease 1, pyruvate dehydrogenase
End Session
deficiency and fatty acid oxidation defects but raised with any problem that reduces tissue
perfusion e.g. sepsis, dehydration therefore non-specific

Other:

Mucopolysaccharidoses (urinary glycosaminoglycans)


Hypoketotic hypoglycaemia (fatty acid oxidation defect e.g. MCAD)
Urine reducing substances (present in galactossaemia)

47555

Tag Question

Feedback

Difficulty: Average

Peer Responses

https://mypastest.pastest.com/Secure/TestMe/Browser/436619[‫ م‬12:13:39 10/12/1437]


MyPastest

Session Progress

Responses Correct: 0

Responses Incorrect: 28

Responses Total: 28

Responses - % Correct: 0%

Blog
About Pastest
Contact Us
Help

© Pastest 2016

https://mypastest.pastest.com/Secure/TestMe/Browser/436619[‫ م‬12:13:39 10/12/1437]


MyPastest

Prefer to use the old MyPastest? Access it here »

Back to Filters

Question 29 of 35

A 4-week-old baby presents with prolonged jaundice. He has been exclusively breastfeeding since birth with
normal stools. He has had faltering growth ever since he was treated for E. Coli sepsis at 1 week of age. His last
haemoglobin was 110.

What is the MOST likely diagnosis?

A Biliary atresia

B Breast milk jaundice

C Galactosaemia

D Haemolytic jaundice

E Physiological jaundice

Explanation
Galactosaemia is a rare autosomal recessive disorder. It is secondary to the deficiency of the enzyme galactose-1-
uridyl transferase, which is an enzyme that converts galactose into glucose. Galactose is a product of lactose.
Babies often present with vomiting, diarrhoea, faltering growth and jaundice. Other features include cataracts and
E.Coli urinary tract infection. The presence of urine reducing substances helps guide the diagnosis. These babies
are managed with a lactose-free diet.

Faltering growth is a red flag and therefore physiological and breast milk jaundice should not be high in the list of
differentials. The history is
suggestive of a pathological cause.

Haemolytic jaundice would have suggestive FBC and blood film findings.

The history of normal stools excludes biliary atresia as this would present with pale stools.
47556

https://mypastest.pastest.com/Secure/TestMe/Browser/436619[‫ م‬12:14:00 10/12/1437]


MyPastest

Tag Question

Feedback

Difficulty: Average

Next
PeerQuestion
Responses
Previous Question

End Session

Session Progress

Responses Correct: 0

Responses Incorrect: 29

Responses Total: 29

Responses - % Correct: 0%

Blog
About Pastest
Contact Us
Help

© Pastest 2016

https://mypastest.pastest.com/Secure/TestMe/Browser/436619[‫ م‬12:14:00 10/12/1437]


MyPastest

Prefer to use the old MyPastest? Access it here »

Back to Filters

Question 30 of 35

A 4-year-old child presents with diarrhoea, vomiting and drowsiness. Her blood glucose is 1.9 mmol/L and urine
dip is negative. The liver function test is mildly abnormal. There is a family history of
sudden death.

What is the MOST likely diagnosis?

A Acute hepatic failure

B Fatty acid oxidation defect

C Galactosaemia

D Gastroenteritis

E Hyperinsulinism

Explanation
This child most likely has MCADD (Medium chain acyl-CoA dehydrogenase deficiency). MCADD may present
with an intercurrent illness or following a prolonged period of fasting. The presentation is that of hypoketotic
hypoglycaemia. Other presentations include lethargy, seizures, coma and acute hepatic failure. Some individuals
remain asymptomatic but with the advent of newborn screening
this is much less likely now.

The enzyme MCAD results in dehydrogenation of fatty acids as they undergo beta oxidation within the
mitochondria. Fatty acid beta oxidation provides energy after the body has depleted stores of glycogen and
glucose, such as during illness or prolonged fasting. The pathways that result in ketone bodies being produced as
an alternative energy source is therefore interrupted.

Management includes avoidance of prolonged fasting. During intercurrent illness, additional supplementation of
glucose is essential. This may be in the form of enteral glucose polymer or an intravenous infusion of glucose.

Acute heptatic failure is unlikely because there are no other features in the history suggestive of this such as recent
fever, jaundice and new medication induced. Moreover, you would not expect mildly abnormal liver function tests

https://mypastest.pastest.com/Secure/TestMe/Browser/436619[‫ م‬12:14:22 10/12/1437]


MyPastest

with
acute hepatic failure which is significant enough to be symptomatic

Galactosaemia is unlikely because this is most commonly diagnosed in the neonatal period either from newborn
bloodspot screening or symptoms such as faltering growth, jaundice, cataracts or history of E. coli sepsis. It would
be unlikely for a child in this age group to present with new onset Galactosaemia having had a normal diet up until
this presentation.

Simple gastroenteritis can present with symptoms such as vomiting, lethargy and hypoglycaemia are common,
however you would expect the urine dip to be positive for ketones. Those children who are hypoglycaemic
secondary to an underlying illness should mount a ketotic response. Hypoketotic hypoglycaemia is characteristic
of MCADD.

Next Question
Hyperinsulinism is a very rare cause of hypoglycaemia. Examples of this cause would include persisting
hyperinsulinismPrevious Question
of infancy, Beckwith-Wiedemann syndrome and insulinoma.
47557

End Session
Tag Question

Feedback

Difficulty: Average

Peer Responses

Session Progress

Responses Correct: 0

Responses Incorrect: 30

Responses Total: 30

Responses - % Correct: 0%

https://mypastest.pastest.com/Secure/TestMe/Browser/436619[‫ م‬12:14:22 10/12/1437]


MyPastest

Blog
About Pastest
Contact Us
Help

© Pastest 2016

https://mypastest.pastest.com/Secure/TestMe/Browser/436619[‫ م‬12:14:22 10/12/1437]


MyPastest

Prefer to use the old MyPastest? Access it here »

Back to Filters

Question 31 of 35

A 7-year-old children presents with acute onset vomiting, encephalopathy and hypoglycaemia.

Which ONE of the following is the LEAST likely cause of hypoglycaemia?

A Fatty acid oxidation defect

B Glycogen storage disorders

C Ketotic hypoglycaemia

D Multi-organ failure

E Peroxismal disorder

Explanation
Peroxismal disorders do not present with hypoglycaemia. Disorders within this category of metabolic medicine
often present with dysmorphism, pshycomotor retardation, seizures, leucodystrophy (on MRI head), hepatic
dysfunction and visual/hearing impairment.

Fatty acid oxidation defects such as MCADD is a cause of hypoglycaemia which is often seen with prolonged
periods of fasting or an intercurrent illness.

Glycogen storage disorders occur due to the inability to breakdown glycogen adequately to supply glucose for
energy. This can present at various ages. There are 6 subtypes. Hepatomegaly is universal to all subtypes. Some
subtypes present with severe hypoglycaemia and high lactate. Some subtypes present with more muscle weakness
such as hypotonia, rhabdomyolysis and cardiac involvement.

Ketotic hypoglycaemia is the most frequently encountered form of hypoglycaemia in children. This is poorly
understood and refers to a subgroup of children who tend to have a reduced fasting tolerance. This is also
commonly seen in babies who have
intrauterine growth restriction or small for dates. This is a diagnosis
of
exclusion and other causes must be excluded prior to this. Management includes an emergency feeding plan

https://mypastest.pastest.com/Secure/TestMe/Browser/436619[‫ م‬12:14:42 10/12/1437]


MyPastest

during episodes of intercurrent illness and to prevent periods of prolonged fasting.

Multi-organ failure causes hypoglycaemia. This picture can be seen with tyrosinaemia, galactosaemia and
mitochondrial disorders.
47558

Tag Question

Feedback Next Question


Previous Question
Difficulty: Average

Peer Responses
End Session

Session Progress

Responses Correct: 0

Responses Incorrect: 31

Responses Total: 31

Responses - % Correct: 0%

Blog
About Pastest
Contact Us
Help

© Pastest 2016

https://mypastest.pastest.com/Secure/TestMe/Browser/436619[‫ م‬12:14:42 10/12/1437]


MyPastest

Prefer to use the old MyPastest? Access it here »

Back to Filters

Question 32 of 35

A 5-year-old boy has a background of behavioural difficulties, global developmental delay and dysmorphic
features. He is the second child of consanguineous parents who lost their first child to
sudden unexplained death in
infancy. He had normal development until 15 months of age following which he had some developmental
regression. He also has a background of recurrent headaches and he has recently been
diagnosed with
hypothyroidism. On examination, he has soft dysmorphic features and a short stature.

Which feature from the history and examination is the MOST indicative of a metabolic disorder?

A Abnormal thyroid function

B Behavioural difficulties

C Dysmorphic features

D Family history of sudden infant death

E Recurrent headaches

Explanation
Family history of sudden infant death is an indicator for metabolic disease. Metabolic disorders can present with a
range of serious symptoms such as seizures, encephalopathy and hypoglycaemia which can contribute to sudden
death. As most inborn errors of metabolism are inherited in an autosomal recessive fashion and
there may be a
history of consanguinity, an important marker is family history.

Abnormal thyroid function is an endocrine disorder and it is very rare for metabolic conditions to be associated
specifically with a metabolic condition.

Recurrent headaches are not a common


feature seen with metabolic diseases. The differential diagnoses for
recurrent headaches is vast such as migraines to space occupying lesions.

Although dysmorphic features can be seen in metabolic conditions such as mucopolysaccharidoses and peroxismal

https://mypastest.pastest.com/Secure/TestMe/Browser/436619[‫ م‬12:15:06 10/12/1437]


MyPastest

disorders, this is very non-specific. Dysmorphic features are also seen with a vast range of other genetic
chromosomal defects too.

Behavioural difficulties can be a non specific marker of metabolic disease, but it can also be a part of a range of
other problems including autistic spectrum disorders to attention deficit hyperactivity disorder and therefore is not
the most indicative marker of metabolic disease.

There
are various factors in the history and examination that are indicative of a possible inborn error of
metabolism. These features include:

Parental consanguinity
Family history of sudden infant deaths
Next Question
Previous
Previous recurrent Question
miscarriages
Neurological deterioration followed by a preceding period of being well
Encephalopathy
Seizures
Dysmorphic features End Session
Maternal HELLP syndrome in pregnancy (Haemolysis; Elevated Liver enzymes; Low Platelet count)

47559

Tag Question

Feedback

Difficulty: Average

Peer Responses

Session Progress

Responses Correct: 0

Responses Incorrect: 32

https://mypastest.pastest.com/Secure/TestMe/Browser/436619[‫ م‬12:15:06 10/12/1437]


MyPastest

Responses Total: 32

Responses - % Correct: 0%

Blog
About Pastest
Contact Us
Help

© Pastest 2016

https://mypastest.pastest.com/Secure/TestMe/Browser/436619[‫ م‬12:15:06 10/12/1437]


MyPastest

Prefer to use the old MyPastest? Access it here »

Back to Filters

Question 33 of 35
Next Question

A 5-year-old boy has a background of behavioural difficulties, global developmental delay and dysmorphic
features. He is
the second child of consanguineous parents who lost their first child to sudden unexplained death in
infancy. He had normal development until 15 months of age following which he had some developmental
regression. He also has a background of recurrent headaches and he has recently been
diagnosed with
hypothyroidism. On examination, he is noted to have course facies, corneal clouding and hepatosplenomegaly.

Which ONE of the following is the MOST likely diagnosis?

A Hurler syndrome (type 1 MPS)

B Hunter syndrome (type 2 MPS)

C Sanfillipo syndrome (type 3 MPS)

D Morquio syndrome (type 4 MPS)

E Maroteaux syndrome (type 5 MPS)

Explanation
Mucopolysaccharidoses are caused by deficiency of lysosomal enzymes that are involved in breaking down
glycosaminoglycans.
They are inherited in an autosomal recessive manner except Hunter syndrome which is X-
linked. The clinical features depend on the presence
of enzymatic activity within each tissue and therefore present
with a variable degree of progressive mental and physical deterioration. There are characteristic dysmorphic
features such as course facies, macroglossia and micrognathia. Hurler syndrome is associated with corneal
clouding. Hunter syndrome is similar to Hurler but there is no corneal clouding. Sanfillipo syndrome is associated
with major developmental and intellectual delay. Maroteaux and Morquio syndromes mainly have skeletal
dysplasias but less of an effect on the central nervous system. 47560

https://mypastest.pastest.com/Secure/TestMe/Browser/436619[‫ م‬12:15:27 10/12/1437]


MyPastest

Tag Question

Feedback

Difficulty: Average

Peer Responses

Previous Question

End Session

Session Progress

Responses Correct: 0

Responses Incorrect: 33

Responses Total: 33

Responses - % Correct: 0%

Blog
About Pastest
Contact Us
Help

© Pastest 2016

https://mypastest.pastest.com/Secure/TestMe/Browser/436619[‫ م‬12:15:27 10/12/1437]


MyPastest

Prefer to use the old MyPastest? Access it here »

Back to Filters

Question 34 of 35

The community midwife completes a routine day 5 heel prick blood test on a term baby.

Which ONE of the following conditions is NOT tested for?

A Glutaric aciduria type 1

B Isovaleric acidaemia

C Jaundice

D Medium chain acyl-CoA dehydrogenase deficiency

E Phenylketonuria

Explanation
Jaundice is often detected in the newborn period by simple physical examination of the skin and sclera. Serum
bilirubin levels are measured is there is suspicion of clinical jaundice in order to determine whether treatment is
required e.g. monitoring or phototherapy. Although heel prick tests are carried out in the newborn period to
monitor serum bilirubin levels, this is different to blood that is taken for Guthrie. The metabolic condition that can
present with
jaundice in the newborn period include galactosaemia.

The following conditions are tested for as part of a routine newborn screen using a Guthrie card:

Metabolic conditions:

Phenylketonuria
MCADD
Maple syrup urine disorder
Isovaleric acidaemia
Glutaric aciduria type 1

https://mypastest.pastest.com/Secure/TestMe/Browser/436619[‫ م‬12:15:47 10/12/1437]


MyPastest

Homocystinuria

Other:

Sickle cell disease


Cystic fibrosis
Congential hypothyroidism

47561

Next Question
Previous Question

Tag Question

Feedback End Session

Difficulty: Average

Peer Responses

Session Progress

Responses Correct: 0

Responses Incorrect: 34

Responses Total: 34

Responses - % Correct: 0%

Blog
About Pastest
Contact Us
Help

© Pastest 2016

https://mypastest.pastest.com/Secure/TestMe/Browser/436619[‫ م‬12:15:47 10/12/1437]


MyPastest

https://mypastest.pastest.com/Secure/TestMe/Browser/436619[‫ م‬12:15:47 10/12/1437]


MyPastest

Prefer to use the old MyPastest? Access it here »

Back to Filters

Question 35 of 35

A 2-week-old baby has recently been diagnosed with an inborn error of metabolism based on routine newborn
screening results. This baby is currently asymptomatic and the parents were informed that had this condition
remained undetected or if the strict restriction diet is not followed the symptoms would include developmental
delay, microcephaly and eczema. The Parents have also been informed that management will include monitoring
of blood levels and supplementation with tyrosine.

What is the most LIKELY diagnosis?

A Congenital hypothyroidism

B Homocystinuria

C Maple syrup urine disease

D Phenylketonuria

E Tyrosinaemia

Explanation
Phenylketonuria is the commonest inborn error of metabolism whereby there is a deficiency of the enzyme
phenylalanine hydroxylase. This results in an overall deficiency of tyrosine and an increase in phenylalanine.
Persistently raised levels of phenylalanine are associated with mental retardation, microcephaly and eczema. The
diagnosis is based on an elevated level of phenylalanine and decreased tyrosine. The management includes
phenylalanine-restricted diet with supplementation of tyrosine. Bloods levels are monitored. No special
precautions are required during illness or surgery.

Congenital hypothyroidism is unlikely because the management would not include a restriction diet and hormone
replacement with thyroxine would be required.

Homocystinuria is unlikely because the symptoms would include a Marfanoid like habitus, joint stiffness and
developmental delay. Moreover, the management would include pyridoxine and folate supplementation.

https://mypastest.pastest.com/Secure/TestMe/Browser/436619[‫ م‬12:16:06 10/12/1437]


MyPastest

MSUD is unlikely because the clinical features include encephalopathy within the newborn period, a sweet odour
and possible seizures. The management would include controlling leucine levels with a low protein diet and
replacing with the enzyme cofactor thiamine.

Tyrosinaemia is unlikely as this would present with early onset severe liver disease, faltering growth and proximal
renal tubulopathy. The management is a low phenylalanine and low tyrosine diet.
47562

End Session

Previous Question Tag Question

Feedback

Difficulty: Average

Peer Responses

Session Progress

Responses Correct: 0

Responses Incorrect: 35

Responses Total: 35

Responses - % Correct: 0%

Blog
About Pastest
Contact Us
Help

© Pastest 2016

https://mypastest.pastest.com/Secure/TestMe/Browser/436619[‫ م‬12:16:06 10/12/1437]


MyPastest

Prefer to use the old MyPastest? Access it here »

Back to Filters

Question 1 of 17

A 14-year-old boy attends the clinic with a 1-day history of pain and swelling in his left knee. He is known to have
factor VIII deficiency. On examination there is restriction of joint movement. The joint is hot, swollen and
extremely painful. ESR and CRP levels are normal. Test for rheumatoid factor is negative.

What is the most likely diagnosis?

A Pyogenic arthritis

B Juvenile rheumatoid arthritis

C Haemophilic arthritis

D Juvenile idiopathic arthritis

E Rickets

Explanation
This boy has haemophilic arthritis. The pain and swelling is due to bleeding within the joint. The erythrocyte
sedimentation rate (ESR) and C-reactive protein (CRP) levels would be raised in pyogenic/septic arthritis.
‘Juvenile idiopathic arthritis’ is the term given to a spectrum of arthritides occurring in childhood. Other
subgroups are juvenile rheumatoid arthritis (rheumatoid factor is positive), juvenile ankylosing spondylitis,
psoriatic arthritis and enteropathic arthritis. In rickets there is a normal amount of bony tissue but a reduced
quantity of mineral content due to vitamin D and calcium deficiency. Knock-knees/bow-legs and hypocalcaemia
are features of this condition. Next Question
12666

https://mypastest.pastest.com/Secure/TestMe/Browser/436619[‫ ص‬09:52:06 10/12/1437]


MyPastest

Previous Question
Tag Question
End Session
Feedback

Difficulty: Easy

Peer Responses

Session Progress

Responses Correct: 0

Responses Incorrect: 1

Responses Total: 1

Responses - % Correct: 0%

Blog
About Pastest
Contact Us
Help

© Pastest 2016

https://mypastest.pastest.com/Secure/TestMe/Browser/436619[‫ ص‬09:52:06 10/12/1437]


MyPastest

Prefer to use the old MyPastest? Access it here »

Back to Filters

Question 2 of 17

An 8-year-old Japanese boy complains of pain in his elbows and knees with swelling of his hands and feet. On
examination, his temperature is 39 °C, pulse 120/m in and blood pressure 100/60 mmHg.
His tongue is red in
colour. Conjunctival congestion and cervical lymphadenopathy are noted.

What is the most likely diagnosis?

A Kawasaki disease

B Sjögren syndrome

C Diffuse cutaneous systemic sclerosis

D Behçet syndrome

E Felty syndrome

Explanation
Kawasaki disease is an acute systemic disorder of
childhood that predominantly occurs in Japan (800 cases per
million in children under the age of 5 years). The causative factor is not known, but mycoplasma and HIV
infection may be associated in some cases. The principal clinical features are fever persisting for more than 5 days,
bilateral non-purulent conjunctival congestion, cervical lymphadenopathy, polymorphous rash, arthralgia, palmar
erythema and strawberry tongue. Diffuse cutaneous systemic sclerosis is associated with skin, renal and gut
involvement. Arthralgia, morning stiffness and flexor tenosynovitis are common. Behçet syndrome is a vasculitis
of unknown aetiology that characteristically targets venules. Felty syndrome is the association of splenomegaly
and neutropenia with rheumatoid arthritis. Lymphadenopathy is common and there is a predisposition to recurrent
infections.
12667

https://mypastest.pastest.com/Secure/TestMe/Browser/436619[‫ ص‬09:52:58 10/12/1437]


MyPastest

Previous Question
End Session
Tag Question

Feedback

Difficulty: Easy

Peer Responses

Session Progress

Responses Correct: 0

Responses Incorrect: 2

Responses Total: 2

Responses - % Correct: 0%

Blog
About Pastest
Contact Us
Help

© Pastest 2016

Next Question

https://mypastest.pastest.com/Secure/TestMe/Browser/436619[‫ ص‬09:52:58 10/12/1437]


MyPastest

Previous Question
Prefer to use the old MyPastest? Access it here »

Back to Filters

Question 3 of 17

A 12-year-old girl complains of pain in her hip and knee joints, as well as fever, bloody diarrhoea and abdominal
pain. A barium enema shows rose-thorn ulcers.

What is the most characteristic feature seen on colonoscopy in this condition?

A Red-raw mucosa

B Pseudopolyps

C Discrete ulcers

D Colonic dilatation

E Diverticula

Explanation
Both Crohn’s disease and ulcerative colitis may be associated with enteropathic arthritis. Barium studies in
Crohn’s disease may show strictures, rose-thorn ulcers and cobblestone mucosal surfaces. Colonoscopy may show
discrete ulcers. Red-raw mucosa, pseudopolyps and evidence of colonic dilatation are features of ulcerative colitis.
Diverticular disease does not present with arthralgia and is unusual in children.
12668

Next Question
Tag Question

Feedback

https://mypastest.pastest.com/Secure/TestMe/Browser/436619[‫ ص‬09:53:43 10/12/1437]


MyPastest

End Session
Difficulty: Average

Peer Responses

Session Progress

Responses Correct: 0

Responses Incorrect: 3

Responses Total: 3

Responses - % Correct: 0%

Blog
About Pastest
Contact Us
Help

© Pastest 2016

https://mypastest.pastest.com/Secure/TestMe/Browser/436619[‫ ص‬09:53:43 10/12/1437]


MyPastest

Prefer to use the old MyPastest? Access it here »

Back to Filters

Question 4 of 17

A 17-year-old girl complains of a 2-week history of fever, chest pain, stiffness, swelling in the wrists and fingers
and oedema in both legs. She also has a rash on her palms and over her cheeks and complains of excessive loss of
hair while combing.

Given the likely diagnosis, which of the following results is most likely to be found in the blood test?

A Autoimmune haemolytic anaemia

B Rheumatoid factor

C Low serum complement levels

D Anti double-stranded DNA antibodies

E Anticardiolipin antibodies

Explanation
This patient most probably has systemic lupus erythematosus, given the clinical features. Systemic lupus
erythematosus is nine times commoner in women and the age of onset is usually between 15 and 25 years. Due to
the formation of immune complexes, serum complement levels are invariably reduced during active disease.
Rheumatoid factor is positive in 25% and anticardiolipin antibodies are present in 35–45% of cases. Anti-dsDNA
occurs in around half the patients, but may be positive in around 80% of patients, with severe systemic
involvement. Autoimmune haemolytic anaemia may occasionally occur in this condition.
12669
Next Question

https://mypastest.pastest.com/Secure/TestMe/Browser/436619[‫ ص‬09:54:54 10/12/1437]


MyPastest

Previous Question Tag Question

Feedback End Session

Difficulty: Difficult

Peer Responses

Session Progress

Responses Correct: 0

Responses Incorrect: 4

Responses Total: 4

Responses - % Correct: 0%

Blog
About Pastest
Contact Us
Help

© Pastest 2016

https://mypastest.pastest.com/Secure/TestMe/Browser/436619[‫ ص‬09:54:54 10/12/1437]


MyPastest

Prefer to use the old MyPastest? Access it here »

Back to Filters

Question 5 of 17

You see in A&E a 17-year-old who presents with a raised red and scaly lesion on his glans penis, red
discolouration and pain in both eyes and pain and swelling of his right knee. Over the past
few days he has noticed
painless red plaques on his hands and feet. A diagnosis of Reiter’s syndrome is suspected.

Which clinical feature would best support this diagnosis?

A H istory of a flu-l ike illness 4–6 weeks pr ior to symptoms

B Presence of keratoderma blenorrhagica

C Family history of ulcerative colitis

D Positive gonococcal culture of urethral discharge

E Arthritis affecting the upper limb joints

Explanation
Reiter’s syndrome is a triad of urethritis, arthritis and conjunctivitis, which follows 4–6 weeks after a
genitourinary (chlamydial) infection. ‘Reactive arthritis’ is the term given to arthritis following gastrointestinal
(shigella, yersinia) infection. Keratoderma blenorrhagica (brown aseptic abscesses on the soles and palms) may
occur
and are highly diagnostic. Ulcerative colitis may give rise to enteropathic arthritis. A urethral discharge in
Reiter’s syndrome is usually sterile. The arthritis commonly affects the knees, ankles or feet.
12670

Next Question

https://mypastest.pastest.com/Secure/TestMe/Browser/436619[‫ ص‬09:55:17 10/12/1437]


MyPastest

Previous Question Tag Question

Feedback End Session

Difficulty: Difficult

Peer Responses

Session Progress

Responses Correct: 0

Responses Incorrect: 5

Responses Total: 5

Responses - % Correct: 0%

Blog
About Pastest
Contact Us
Help

© Pastest 2016

https://mypastest.pastest.com/Secure/TestMe/Browser/436619[‫ ص‬09:55:17 10/12/1437]


MyPastest

Prefer to use the old MyPastest? Access it here »

Back to Filters

Question 6 of 17

A 4-year-old girl with a 1-day history of increasing hip pain is unable to stand. Her WCC is 20 × 109 /l, ESR 90
mm/1st h and CRP 275 mg/l. A radiograph of the hip shows a widened joint space.

What is the most likely diagnosis?

A Perthes disease

B Slipped upper femoral epiphysis

C Septic arthritis

D Congenital dislocation of hip

E Osteomyelitis

Explanation
The features are suggestive of septic arthritis. A painful, hot, swollen joint with increased synovial fluid, indicated
by the widened joint space on X-ray, is characteristic of this
condition.

Perthes disease is osteochondritis of the femoral head, affecting children aged between 3 and 11 years. It presents
with pain in the hip or knee and causes a limp.Slipped upper femoral epiphysis affects children aged 10-16 years.
Some 20% are bilateral. About half the patients are obese and hypogonadal. Congenital dislocation of hips is seen
in neonates. Girls are more commonly affected than boys. If not detected in infancy, older children may present
with delay in walking, an abnormal waddling gait and an inability to fully abduct the affected hip. The distal
femur, or upper tibia, is the most common site for osteomyelitis. Although X-ray changes
are not apparent for a
few days, they then show haziness and loss of density of the affected bone followed by subperiosteal reaction and,
later, sequestrum and involucrum.
12671

https://mypastest.pastest.com/Secure/TestMe/Browser/436619[‫ ص‬09:56:00 10/12/1437]


MyPastest

Previous Question
End Session
Tag Question

Feedback

Difficulty: Easy

Peer Responses

Session Progress

Responses Correct: 0

Responses Incorrect: 6

Responses Total: 6

Responses - % Correct: 0%

Blog
About Pastest
Contact Us
Help

© Pastest 2016

Next Question

https://mypastest.pastest.com/Secure/TestMe/Browser/436619[‫ ص‬09:56:00 10/12/1437]


MyPastest

Previous Question
Prefer to use the old MyPastest? Access it here »

Back to Filters

Question 7 of 17

You review a 17-year-old who has suffered from Still's disease (systemic juvenile idiopathic arthritis) since
childhood.

Which of the following are common features of Still's disease?

A Negative rheumatoid factor

B Normal erythrocyte sedimentation rate (ESR)

C Normal C-reactive protein (CRP)

D Thrombocytopenia

E Splenic atrophy

Explanation
Still’s disease (which accounts for 10% of cases of juvenile idiopathic arthritis (JIA) affects boys and girls equally
up to the age of five years of age, after this point, girls are more commonly affected. Adult-onset Still’s is rare.
Clinical features include fever, maculopapular rash, myalgia and generalised lymphadenopathy.
Hepatosplenomegaly, pericarditis and pleurisy also occur. Laboratory tests reveal raised ESR and CRP,
neutrophilia and thrombocytosis, though autoantibodies are negative.
12672

Next Question

Tag Question

https://mypastest.pastest.com/Secure/TestMe/Browser/436619[‫ ص‬09:56:21 10/12/1437]


MyPastest

Feedback

End Session
Difficulty: Average

Peer Responses

Session Progress

Responses Correct: 0

Responses Incorrect: 7

Responses Total: 7

Responses - % Correct: 0%

Blog
About Pastest
Contact Us
Help

© Pastest 2016

https://mypastest.pastest.com/Secure/TestMe/Browser/436619[‫ ص‬09:56:21 10/12/1437]


MyPastest

Prefer to use the old MyPastest? Access it here »

Back to Filters

Question 8 of 17

A 14-year-old girl, who is otherwise fit, complains of backache and fatigue. Her parents have also noticed that she
is increasingly becoming round-shouldered. On examination movements are normal. A smooth lump is seen in the
thoracic region.

What would be the most characteristic feature of an X-ray of the thoracic spine?

A Anterior displacement of one thoracic vertebra upon another

B Absent neural arch

C Osteophyte formation on posterior facet joints

D Deep notches on the anterior corners of the vertebrae

E Scoliosis

Explanation
This girl has Scheuermann’s disease. It usually affects those between 13 and 16 years of age. The normal
ossification of ring epiphyses of several thoracic vertebrae is affected. Deforming forces are greatest at the anterior
border of the vertebrae so vertebrae are narrower anteriorly, resulting in kyphosis. Anterior displacement occurs in
spondylolisthesis (defect in the pars interarticularis of the neural arch). Osteophyte formation is seen in
osteoarthritis. Kyphosis, and not scoliosis, is the presenting feature of Scheuermann’s disease.
12673

Next Question

https://mypastest.pastest.com/Secure/TestMe/Browser/436619[‫ ص‬09:56:43 10/12/1437]


MyPastest

Previous Question Tag Question

Feedback End Session

Difficulty: Difficult

Peer Responses

Session Progress

Responses Correct: 0

Responses Incorrect: 8

Responses Total: 8

Responses - % Correct: 0%

Blog
About Pastest
Contact Us
Help

© Pastest 2016

https://mypastest.pastest.com/Secure/TestMe/Browser/436619[‫ ص‬09:56:43 10/12/1437]


MyPastest

Prefer to use the old MyPastest? Access it here »

Back to Filters

Question 9 of 17

A 7-year-old boy presents with arthritis affecting his wrists, knees and ankles following an upper respiratory tract
infection.
He then develops testicular swelling, bloody stools, renal impairment with haematuria and proteinuria
and a rash affecting the lower limbs and
buttocks.

What is the most likely diagnosis?

A Henoch–Schönlein purpura (HSP)

B Polyarteritis nodosa

C Crohn's disease

D Reiter syndrome

E Systemic lupus erythematosus

Explanation
The clinical scenario has enough information to diagnose HSP. Colicky abdominal pain is classical but bloody
stools can occur as part of intussusception. About 3% of patients
develop significant renal disease particularly
displaying signs of haematuria and proteinuria although end stage renal failure is extremely
rare. Approximately
50% of children have a history of preceding upper respiratory tract infection (URTI). Commonest age at
presentation for HSP is 4–15 years and there is a 2 : 1 male to female ratio. Prednisolone treatment may be
considered where there is significant renal disease; otherwise only supportive care is often required. Prognosis is
excellent in general with most patients achieving spontaneous recovery within 4 weeks.
12674
Next Question

https://mypastest.pastest.com/Secure/TestMe/Browser/436619[‫ ص‬09:57:01 10/12/1437]


MyPastest

Previous Question Tag Question

Feedback End Session

Difficulty: Easy

Peer Responses

Session Progress

Responses Correct: 0

Responses Incorrect: 9

Responses Total: 9

Responses - % Correct: 0%

Blog
About Pastest
Contact Us
Help

© Pastest 2016

https://mypastest.pastest.com/Secure/TestMe/Browser/436619[‫ ص‬09:57:01 10/12/1437]


MyPastest

Previous Question
Prefer to use the old MyPastest? Access it here »

End Session

Back to Filters

Question 10 of 17

A 3-year-old girl is taken to her GP with a 10-day history of


fever. She has now developed a truncal, macular
papular rash. On examination she is irritable and is found to have cervical lymphadenopathy and reddened
conjunctiva and non-purulent discharge. Initial blood test shows raised white cell count, platelets, CRP and ESR.

What is the single most likely diagnosis?

A Kawasaki disease

B Measles

C Rheumatic fever

D Scarlet fever

E Systemic lupus erythmatosus

Explanation
In the absence of description of a focus of infection, Kawasaki disease must be considered. Ideally, if presenting
within 5-days, immunoglobulin should be prescribed and aspirin commenced. CRP is raised and white cell count
mildly raised. Scarlet fever is not associated with eye problems. Measles rash commences behind
the ears and on
the face, and eye problems are suppurative. The other conditions could be considered in the differential diagnosis.
14588

Next Question
Tag Question

Feedback

https://mypastest.pastest.com/Secure/TestMe/Browser/436619[‫ ص‬09:57:20 10/12/1437]


MyPastest

Difficulty: Easy

Peer Responses

Session Progress

Responses Correct: 0

Responses Incorrect: 10

Responses Total: 10

Responses - % Correct: 0%

Blog
About Pastest
Contact Us
Help

© Pastest 2016

https://mypastest.pastest.com/Secure/TestMe/Browser/436619[‫ ص‬09:57:20 10/12/1437]


MyPastest

Previous Question
Prefer to use the old MyPastest? Access it here »

End Session

Back to Filters

Question 11 of 17

Which of the following investigations is the one that


has the greatest relevance to the ophthalmic monitoring of a
child with
juvenile idiopathic arthritis?

A Anti-neutrophil cytoplasmic antibodies (ANCAs)

B Erythrocyte sedimentation rate (ESR)

C C-reactive protein

D Anti-nuclear antibodies (ANA)

E Extractable nuclear antigens

Explanation
All these investigations are useful when making the diagnosis. Once the diagnosis has been made, the
investigation that has the biggest impact on management is ANAs. There is a close correlation between being
ANA positive and development of iridocyclitis (which can lead to blindness). ANA-positive children must have
close ophthalmological follow-up.
22236

Tag Question

Feedback
Next Question

Difficulty: Average

https://mypastest.pastest.com/Secure/TestMe/Browser/436619[‫ ص‬09:57:38 10/12/1437]


MyPastest

Peer Responses

Session Progress

Responses Correct: 0

Responses Incorrect: 11

Responses Total: 11

Responses - % Correct: 0%

Blog
About Pastest
Contact Us
Help

© Pastest 2016

https://mypastest.pastest.com/Secure/TestMe/Browser/436619[‫ ص‬09:57:38 10/12/1437]


MyPastest

Prefer to use the old MyPastest? Access it here »

Back to Filters

Question 12 of 17

A previously well 8-year-old boy presents to the Emergency Department with a mild fever and painful swelling of
his right
wrist and left ankle, his right ankle having been affected the previous
week. Twenty days ago he had
tonsillitis. On examination his temperature is 38°C, respiration 30/min, pulse 120/min, blood pressure (BP) 80
mmHg systolic, capillary refill 2 s. He has normal 1st and 2nd
heart sounds, with an additional significant cardiac
murmur and a pericardial rub, but no palpable liver. His CRP is 150 mg/L and his ECG shows a slightly prolonged
P-R interval. Of the following, the most appropriate treatment is:

A Low-dose aspirin

B Intravenous flucloxacillin

C Diuretics

D Bed rest

E ACTH (adrenocorticotrophic hormone)

Explanation
Acute rheumatic fever is a very likely diagnosis with two to three minor features (fever, raised CRP and prolonged
P-R interval) and two major features (flitting polyarthropathy and pericarditis) after an URTI. Acute
myocarditis/pericarditis is one of the few indications for bed rest. High-dose aspirin is indicated; penicillin is often
given, although of equivocal value in this situation. Diuretics are indicated only for cardiac failure and
corticosteroids rather than ACTH are often given for prolonged fever and/or inflammation.
22390

Next Question

Tag Question

https://mypastest.pastest.com/Secure/TestMe/Browser/436619[‫ ص‬09:57:58 10/12/1437]


MyPastest

Previous Question
Feedback End Session

Difficulty: Difficult

Peer Responses

Session Progress

Responses Correct: 0

Responses Incorrect: 12

Responses Total: 12

Responses - % Correct: 0%

Blog
About Pastest
Contact Us
Help

© Pastest 2016

https://mypastest.pastest.com/Secure/TestMe/Browser/436619[‫ ص‬09:57:58 10/12/1437]


MyPastest

Prefer to use the old MyPastest? Access it here »

Back to Filters

Question 13 of 17

Theme: Systemic diseases

A Behçet disease
B Dermatomyositis
C Kawasaki disease
D Lyme disease
E Pauciarticular juvenile idiopathic arthritis
F Polyarticular juvenile idiopathic arthritis
G Rheumatic fever
H Septic arthritis
I Still disease
J SLE

For
each of the following case scenarios, choose the most likely diagnosis from the list above. Each item may be
used once, more than once or not at all.

Scenario 1

A 3-year-old boy has had a fever for 6 days, swollen hands, conjunctivitis and cracked lips. Investigation reveals:
C-reactive protein (CRP) 67 mg/L, erythrocyte sedimentation rate (ESR) 40 mm, Hb 12.1 g/dL, WCC 16 × 109/L
(neutrophils 7 × 109/L, lymphocytes 8 × 109/L), plts 480 × 109/L.

Your answer was incorrect

Select one...

C - Kawasaki disease

The major criteria of Kawasaki disease


are: cervical lymphadenopathy (> 2 cm), non-purulent conjunctivitis,
mucositis, high spiking fever for > 5 days, polymorphous rash/extremity change.

Scenario 2

A 12-year-old girl presents with feeling unwell, lethargy and pain in her knees and legs. On examination she is
febrile (37.7ºC), and has an erythematous rash on the face, particularly the eyelids. Investigations show: ESR 50
mm, normal FBC, creatine kinase (CK) 700 IU/L.

https://mypastest.pastest.com/Secure/TestMe/Browser/436619[‫ ص‬09:58:19 10/12/1437]


MyPastest

Previous
Your answer was incorrect Question
Select one... End Session

B - Dermatomyositis

The age of the girl makes some diagnoses more likely than others (eg dermatomyositis, SLE). However, the rash
affecting the eyelids is characteristic of dermatomyositis, along with the raised CK. The pain in the knees and legs
is due to the myositis and not an arthritis as such.

Scenario 3

A 7-year-old girl presents with pain in both knees, left ankle and right wrist. On examination, the joints involved
are swollen, tender and have limitation of range of movement. Investigations show CRP 35 g/dL, anti-nuclear
antibody (ANA) positive, rheumatoid factor negative, dsDNA antibodies negative, anti-streptolysin
O test (ASOT)
negative.

Your answer was incorrect

Select one...

E - Pauciarticular juvenile idiopathic arthritis

Double-stranded DNA (dsDNA) negative makes SLE unlikely, ASOT negative renders rheumatic fever unlikely.
The clinical picture fits with that of juvenile idiopathic arthritis (previously called juvenile chronic arthritis). This
is pauciarticular as it involves four or less joints. The ANA positive means that there is
a high risk of iridocyclitis
and the patient must be screened for this.
22411

Tag Question

Feedback

Difficulty: Average

Session Progress

Responses Correct: 0

Responses Incorrect: Next Question 15

Responses Total: 15

Responses - % Correct: 0%

https://mypastest.pastest.com/Secure/TestMe/Browser/436619[‫ ص‬09:58:19 10/12/1437]


MyPastest

Blog
About Pastest
Contact Us
Help

© Pastest 2016

https://mypastest.pastest.com/Secure/TestMe/Browser/436619[‫ ص‬09:58:19 10/12/1437]


MyPastest

Previous Question
Prefer to use the old MyPastest? Access it here »

Back to Filters

Question 14 of 17

A healthy looking 3-year-old Caucasian girl has had a swollen


knee for 6 months and has had an intermittent limp.
Her FBC, ESR and coagulation are normal.

What is the most likely diagnosis?

A Leukaemia

B Oligoarticular JIA

C Septic arthritis

D Sickle cell

E Systemic onset JIA

Explanation
The most likely diagnosis is oligoarticular JIA. It generally appears in children under 6 years of age and is
predominantly a female condition. This is an arthritis affecting four or fewer joints - usually a knee, ankle, elbow
or even a single finger. Persistent oligoarticular is defined as an arthritis affecting four or fewer joints after 6
months.

Sickle cell is unlikely in a Caucasian girl, leukaemia is unlikely in a normal girl with a normal FBC, 6 months is
too long for a septic arthritis, and she has no systemic features.
46304

Next Question

Tag Question

https://mypastest.pastest.com/Secure/TestMe/Browser/436619[‫ ص‬09:59:39 10/12/1437]


MyPastest

Feedback

End Session
Difficulty: Average

Peer Responses

Session Progress

Responses Correct: 0

Responses Incorrect: 16

Responses Total: 16

Responses - % Correct: 0%

Blog
About Pastest
Contact Us
Help

© Pastest 2016

https://mypastest.pastest.com/Secure/TestMe/Browser/436619[‫ ص‬09:59:39 10/12/1437]


MyPastest

Previous Question
Prefer to use the old MyPastest? Access it here »

End Session

Back to Filters

Question 15 of 17

A healthy looking 5-year old Caucasian girl has had a swollen


knee for 6 months and has had an intermittent limp.
Her FBC, ESR and coagulation are normal. She is subsequently diagnosed with oligoarticular JIA.

Given this diagnosis, what is the most useful next step?

A Bone marrow examination

B Blood film

C Haemoglobin electrophoresis

D Septic screen

E Slit-lamp examination of the eyes

Explanation
A slit-lamp examination of the eyes is important to see whether there is any uveitis (associated with oligo JIA).

Iridocyclitis
is bilateral in two-thirds of cases and is independent of the joints affected. Prognosis depends on early
detection and good management . Frequent ophthalmological assessments (3 - 6 months) are recommended.
46309

Tag Question
Next Question
Feedback

https://mypastest.pastest.com/Secure/TestMe/Browser/436619[‫ ص‬09:59:58 10/12/1437]


MyPastest

Difficulty: Average

Peer Responses

Session Progress

Responses Correct: 0

Responses Incorrect: 17

Responses Total: 17

Responses - % Correct: 0%

Blog
About Pastest
Contact Us
Help

© Pastest 2016

https://mypastest.pastest.com/Secure/TestMe/Browser/436619[‫ ص‬09:59:58 10/12/1437]


MyPastest

Prefer to use the old MyPastest? Access it here »

Back to Filters

Question 16 of 17

A 5-year-old boy is woken at night by pain in his legs. His mother rubs them and gives him paracetamol. In the
morning, he is fine with no pain. His height and weight are on the 50th centile and he is doing well at school. His
physical examination is normal.

What is the most likely diagnosis?

A Benign nocturnal limb pains

B Factor VIII deficiency

C Oligoarticular JIA

D Sickle cell anaemia

E Systemic onset JIA

Explanation
Benign nocturnal limb pains are often also referred to as 'growing pains' however there is no substantive evidence
that the limbs are 'growing' and causing the pain.

This boy is exhibiting no evidence of disease and this is a common story, 'growing pains' are thought to be due to
muscle pain secondary to hypermobility and exercise. There are no long term consequence to experiencing
'growing pains' and paracetamol or ibuprofen are generally sufficient to
help the child sleep. The pain will
disappear in the morning.
46314

Next Question

Tag Question

https://mypastest.pastest.com/Secure/TestMe/Browser/436619[‫ ص‬10:00:16 10/12/1437]


MyPastest

Previous Question
Feedback End Session

Difficulty: Average

Peer Responses

Session Progress

Responses Correct: 0

Responses Incorrect: 18

Responses Total: 18

Responses - % Correct: 0%

Blog
About Pastest
Contact Us
Help

© Pastest 2016

https://mypastest.pastest.com/Secure/TestMe/Browser/436619[‫ ص‬10:00:16 10/12/1437]


MyPastest

Prefer to use the old MyPastest? Access it here »

Back to Filters

Question 17 of 17

A 6-year-old girl reviewed in general paediatric clinic has a


6 month history of lower limb pains, worse in the
evenings. She had no morning stiffness. On examination she had no joint swelling but hypermobility of both
knees. You note she has flat feet.

What is the most important advice to give now ?

A Slight swelling of the knees lasting for less than 24 hours may suggest a secondary inflammatory
arthritis

B She needs to avoid strenuous sports at school

C Treatment of associated flat feet with insoles will reduce knee pain

D Use of analgesics will lead to a dependence syndrome and should be avoided

E Using knee splints which support the joint will reduce long term symptoms

Explanation
Joint hypermobility is very common in childhood and can be associated with mechanical joint pains. It is one of
the most
common musculoskeletal complaints in childhood related to muscular weakness in association wit
hypermobile joints.

Maintaining normal
exercise and function is the mainstay of treatment as this builds up strength. The condition
will improve with age. Parents often report mild joint swelling but this is unlikey to be JIA. Hypermobility is short
lived and never associated with large joint effusions.
46316

End Session

https://mypastest.pastest.com/Secure/TestMe/Browser/436619[‫ ص‬10:00:36 10/12/1437]


MyPastest

Previous Question Tag Question

Feedback

Difficulty: Average

Peer Responses

Session Progress

Responses Correct: 0

Responses Incorrect: 19

Responses Total: 19

Responses - % Correct: 0%

Blog
About Pastest
Contact Us
Help

© Pastest 2016

https://mypastest.pastest.com/Secure/TestMe/Browser/436619[‫ ص‬10:00:36 10/12/1437]


MyPastest

Prefer to use the old MyPastest? Access it here »

Back to Filters

Question 1 of 71

A newborn baby boy has facial deformities and a small-misproportioned head. He is irritable, hypotonic and has
severe tremors.

What diagnosis, related to antenatal care, would best account for these symptoms?

A Fetal alcohol syndrome (FAS)

B AIDS

C Down syndrome

D Microcephalus

E Hydrocephalus

Explanation
FAS was first reported as a syndrome in 1973 and is now thought to be one of the major causes of mental
retardation, having an incidence of 0.2–3 per 1000 live births. It has been estimated that between 10 and 20% of
mild mental retardation cases are caused by maternal alcohol use. Severity and timing of alcohol consumption,
bingeing, polydrug use (including smoking) during pregnancy, genetic variation and low socioeconomic status are
all aetiological factors. Alcohol inhibits N-methyl-d-aspartate (NMDA) receptors, which mediate the postsynaptic
excitatory effects of glutamate, and this is thought to have an effect on cell proliferation.

Affected
newborns are often irritable, hypotonic, experience severe tremors and show other signs of alcohol
withdrawal. The cardinal signs are facial features, growth deficit and central nervous system impairment. Facial
features include epicanthic folds, microcephaly, short ­
palpebral fissure, underdeveloped philtrum and a thin
upper lip. There are often associated behavioural difficulties including hyperactivity and sleep disturbance. Optic
nerve hypoplasia with poor visual acuity, hearing loss and receptive and expressive language deficits can also be
seen. Cardiac and renal abnormalities include atrial and ventricular septal defects, renal hypoplasia and bladder
diverticula.

https://mypastest.pastest.com/Secure/TestMe/Browser/436619[‫ ص‬06:04:09 11/12/1437]


MyPastest

12645
End Session

Tag Question

Feedback

Difficulty: Easy

Peer Responses

Session Progress

Responses Correct: 0

Responses Incorrect: 1

Responses Total: 1

Responses - % Correct: 0%

Blog
About Pastest
Contact Us
Help

© Pastest 2016

Previous Question
Next Question

https://mypastest.pastest.com/Secure/TestMe/Browser/436619[‫ ص‬06:04:09 11/12/1437]


MyPastest

Prefer to use the old MyPastest? Access it here »


End Session

Back to Filters

Question 2 of 71

A newborn baby is born with a midline lumbrosacral cystic lesion. Occipitofrontal circumference was above the
90th percentile.

The next best step is?

A Cranial ultrasound

B Renal tract ultrasound

C Muscle charting by the physiotherapists

D Surgical closure of the back

E Ventricular tap

Explanation
All of the above need to happen but, as the
baby may have a hydrocephalus, you don't surgically intervene and try
to rectify the midline defect until this has been clarified since urgent
need for shunting may then be precipitated.
Ventricular tap in newborns
is not undertaken without cranial ultrasound and demonstrated clinical need.
14590

Previous Question
Next Question
Tag Question

Feedback

Difficulty: Easy

https://mypastest.pastest.com/Secure/TestMe/Browser/436619[‫ ص‬06:05:19 11/12/1437]


MyPastest

Peer Responses

Session Progress

Responses Correct: 0

Responses Incorrect: 2

Responses Total: 2

Responses - % Correct: 0%

Blog
About Pastest
Contact Us
Help

© Pastest 2016

https://mypastest.pastest.com/Secure/TestMe/Browser/436619[‫ ص‬06:05:19 11/12/1437]


MyPastest

Prefer to use the old MyPastest? Access it here »


End Session

Back to Filters

Question 3 of 71

A newborn infant is noted to be profoundly hypotonic at birth; he has a good heart rate but is in respiratory
distress. Which of
the following syndromes is most likely to be the cause?

A Down syndrome

B Prader–Willi

C Noonan syndrome

D Werdnig–Hoffman disease (spinomuscular atrophy type 1)

E Beckwith–Wiedemann syndrome

Explanation
Although Down, Prader–Willi, and Beckwith–Wiedemann syndromes can all present with neonatal hypotonia,
they are not severe enough to cause respiratory insufficiency. The combination of hypotonia and respiratory
distress would be more fitting with spinomuscular atrophy type 1.
21509

Previous Question
Next Question Tag Question

Feedback

Difficulty: Average

https://mypastest.pastest.com/Secure/TestMe/Browser/436619[‫ ص‬06:05:49 11/12/1437]


MyPastest

Peer Responses

Session Progress

Responses Correct: 0

Responses Incorrect: 3

Responses Total: 3

Responses - % Correct: 0%

Blog
About Pastest
Contact Us
Help

© Pastest 2016

https://mypastest.pastest.com/Secure/TestMe/Browser/436619[‫ ص‬06:05:49 11/12/1437]


MyPastest

Prefer to use the old MyPastest? Access it here »

Back to Filters

Question 4 of 71

You are called to counsel a woman in premature labour


at 23/40. Which of the following is the most appropriate
information to
give initially?

A Unfortunately at this gestation there is 10–20% survival rate, and of the survivors 50% will have some
degree of handicap

B Unfortunately
at this gestation there are the following survival and handicap statistics (tell her the
figures for your own unit)

C Survival
at this gestation is generally poor, and survivors may have long-term problems – (then discuss
with her the option of not resuscitating if the baby is in a poor condition)

D Babies
at this gestation are very immature and will have a long and difficult time in neonatal intensive
care (then discuss the likely problems and interventions that may be necessary)

E Babies at this gestation are extremely early and often do not survive but we will do everything we can

Explanation
Discussion of potential morbidity and mortality with parents in threatened premature labour is difficult but very
important. If there is a threatened delivery at 23 weeks’ gestation it is important to discuss with the parents the
possibility of
not resuscitating if the condition of the baby is poor or the gestation
looks to be earlier, because the
outcome is likely to be very poor. It is useful to give parents morbidity and mortality figures – either
national
figures from the EPICURE study (Pediatrics 2000, 106:659–71) or, preferably, the figures for your own unit.
Next Question 22350

Tag Question

https://mypastest.pastest.com/Secure/TestMe/Browser/436619[‫ ص‬06:06:10 11/12/1437]


MyPastest

End Session
Feedback

Difficulty: Average

Peer Responses

Session Progress

Responses Correct: 0

Responses Incorrect: 4

Responses Total: 4

Responses - % Correct: 0%

Blog
About Pastest
Contact Us
Help

© Pastest 2016

Previous Question

https://mypastest.pastest.com/Secure/TestMe/Browser/436619[‫ ص‬06:06:10 11/12/1437]


MyPastest

Prefer to use the old MyPastest? Access it here »

Back to Filters

Question 5 of 71

A 2-day-old neonate of 28 weeks’ gestation has a


right-side intraventricular haemorrhage with no ventricular
dilatation while on the ventilator. Which of the following is the best advice to give to the parents?

A There are likely to be no significant long-term effects

B There should be no significant long-term effects provided that the ventricle doesn’t dilate

C There may be some mild impairment of the left arm/leg

D There may be some very mild concentration difficulties in childhood

E It is probable that there will be no significant long-term effects but his development will be closely
followed just in case

Explanation
Grading of intraventricular haemorrhage is as follows:

Grade 1: ependymal (germinal matrix)


Grade 2: intraventricular without ventricular dilatation
Grade 3: intraventricular with ventricular dilatation
Grade 4: parenchymal.

Prognosis
depends on grade of bleed: grades 1 and 2 have a good prognosis with no
long-term effects. In grade 3
there may be possible impairment on the contralateral side, depending on the degree of progression of dilatation,
and in grade 4 there is likely to be motor impairment on contralateral side.
22352

https://mypastest.pastest.com/Secure/TestMe/Browser/436619[‫ ص‬06:06:37 11/12/1437]


MyPastest

End Session

Tag Question

Feedback

Difficulty: Easy

Peer Responses

Session Progress

Responses Correct: 0

Responses Incorrect: 5

Responses Total: 5

Responses - % Correct: 0%

Blog
About Pastest
Contact Us
Help

© Pastest 2016

Previous Question
Next Question

https://mypastest.pastest.com/Secure/TestMe/Browser/436619[‫ ص‬06:06:37 11/12/1437]


MyPastest

Prefer to use the old MyPastest? Access it here »


End Session

Back to Filters

Question 6 of 71

Which of the following is the most likely pathology on a cranial ultrasonography performed on day 1 of the life of
a pre-term baby born at 25 weeks’ gestation?

A Hydrocephalus

B Gyral pattern

C Periventricular leukomalacia

D Intraventricular haemorrhage

E Cerebral atrophy

Explanation
The initial cranial ultrasound scan in a 25/40 gestation infant is primarily to look for intraventricular haemorrhage,
because the period in which it is most likely to happen is the initial 48 h. The other features are important but may
well not be seen initially.
22371

Previous Question
Next Question Tag Question

Feedback

Difficulty: Average

https://mypastest.pastest.com/Secure/TestMe/Browser/436619[‫ ص‬06:07:22 11/12/1437]


MyPastest

Peer Responses

Session Progress

Responses Correct: 0

Responses Incorrect: 6

Responses Total: 6

Responses - % Correct: 0%

Blog
About Pastest
Contact Us
Help

© Pastest 2016

https://mypastest.pastest.com/Secure/TestMe/Browser/436619[‫ ص‬06:07:22 11/12/1437]


MyPastest

Prefer to use the old MyPastest? Access it here »


End Session

Back to Filters

Question 7 of 71

What is the most important investigation to perform in a 3-week-old newborn baby boy who is feeding well and
thriving, but is referred with jaundice?

A Direct and indirect bilirubin

B Thyroid function tests (TFTs)

C LFTs

D Urine organic acids

E Coombs’ test

Explanation
All of these are important elements of a prolonged jaundice screen. Biliary atresia is the important cause of
conjugated hyperbilirubinaemia. It is surgically corrected by the Kasai procedure and this should be done before
60 days (8 weeks) of life otherwise the outcome is markedly worse. TFTs are theoretically not so important to
perform because the Guthrie test should screen for hypothyroidism. The other tests are not as urgent as the split
bilirubin.
22373

Previous Question
Next Question
Tag Question

Feedback

https://mypastest.pastest.com/Secure/TestMe/Browser/436619[‫ ص‬06:08:02 11/12/1437]


MyPastest

Difficulty: Easy

Peer Responses

Session Progress

Responses Correct: 0

Responses Incorrect: 7

Responses Total: 7

Responses - % Correct: 0%

Blog
About Pastest
Contact Us
Help

© Pastest 2016

https://mypastest.pastest.com/Secure/TestMe/Browser/436619[‫ ص‬06:08:02 11/12/1437]


MyPastest

Prefer to use the old MyPastest? Access it here »

Back to Filters

Question 8 of 71

Theme: Respiratory distress in the newborn

A Congenital cystic adenomatous malformation

B Congenital diaphragmatic hernia

C Congenital pneumonia

D Meconium aspiration syndrome

E Persistent pulmonary hypertension of the newborn

F Pneumothorax

G Pulmonary haemorrhage

H Pulmonary hypoplasia

I Surfactant deficient lung disease (hyaline membrane disease)

J Transient tachypnoea of newborn

For
each of the following scenarios, choose the most likely diagnosis from the list above. Each item may be used
once, more than once or not at all.

Scenario 1

An infant born at 24 weeks’ gestation who has respiratory distress at birth.

Your answer was incorrect

Select one...

I - Surfactant deficient lung disease (hyaline membrane disease)

Although all of the options are possible, I is the most likely one from the information given. Pulmonary
hypoplasia
and persistent pulmonary hypertension of the newborn are also possible but the most common cause of respiratory
distress syndrome
(RDS) at birth at 24 weeks’ gestation is hyaline membrane disease
(HMD).

Scenario 2

https://mypastest.pastest.com/Secure/TestMe/Browser/436619[‫ ص‬06:08:26 11/12/1437]


MyPastest

An infant born at 33 weeks’ gestation with profound respiratory distress. Antenatal history reveals that there was
rupture of membranes at 16 weeks, and the mother has been on erythromycin.
End Session

Your answer was incorrect

Select one...

H - Pulmonary hypoplasia

As the fetal lungs require amniotic fluid to develop properly, rupture of membranes at 16 weeks will lead to
oligohydramnios and subsequent pulmonary hypoplasia. Erythromycin reduces incidence of congenital
pneumonia. HMD is less likely at 33 weeks’ gestation.

Scenario 3

A baby born by elective caesarean section at term (for breech presentation) after an uncomplicated pregnancy. At
15 min of age the baby is noted to be grunting.

Your answer was incorrect

Select one...

J - Transient tachypnoea of newborn

Transient tachypnoea of the newborn is


the most likely diagnosis because caesarean sections don’t provide
stimulation for absorption of lung fluid. This would be the most
likely diagnosis if there were an uncomplicated
antenatal course.
22403

Tag Question

Feedback

Difficulty: Average

Previous Question Session Progress

Responses Correct: Next Question 0

Responses Incorrect: 10

Responses Total: 10

Responses - % Correct: 0%

https://mypastest.pastest.com/Secure/TestMe/Browser/436619[‫ ص‬06:08:26 11/12/1437]


MyPastest

Blog
About Pastest
Contact Us
Help

© Pastest 2016

https://mypastest.pastest.com/Secure/TestMe/Browser/436619[‫ ص‬06:08:26 11/12/1437]


MyPastest

Prefer to use the old MyPastest? Access it here »

Back to Filters

Question 9 of 71

Which of the following statements concerning necrotising enterocolitis (NEC) is the most factually correct?

A Breast
milk and to a lesser extent donor breast milk decrease the probability of developing NEC in
high-risk neonates when compared with formula-fed control infants

B Infective organisms are identified in 80% of cases of NEC

C NEC is a diagnosis exclusive to preterm babies

D Probiotics have no role in the prevention of NEC in very-low-birthweight neonates

E Standardising feeding regimens in neonatal units had no effect in decreasing the incidence of NEC
within those units

Explanation
Necrotising enterocolitis (NEC) is a condition of unknown aetiology that occurs most commonly in infants
<1500g and is characterised by transmural intestinal necrosis. Presentation is with abdominal distension and
tenderness, with increasing bile-stained aspirates, blood in stools, apnoea and acidosis.

Predisposing factors are:

Prematurity
Intrauterine growth retardation (IUGR), abnormal Doppler scans
Hypoxia
Hypotension
Perinatal compromise
Sepsis • Hypothermia
Exchange transfusion/umbilical catheterisation

https://mypastest.pastest.com/Secure/TestMe/Browser/436619[‫ ص‬06:08:55 11/12/1437]


MyPastest

Anaemia
End Session
Polycythaemia
Formula feeding/hyperosmolar feeds
Cardiac
causes with impaired gut perfusion including PDA, coarctation, left ventricular outflow tract
obstruction, transposition of the great arteries.

Preventive strategies include:

Standardised feeding regimens within units


Maternal breast milk • Donor breast milk
In
growth-restricted infants with abnormal Doppler scans parenteral nutrition with minimal enteral feeding
for the first week was commonly practised; however, recent data (ADEPT 2010) shows no significant
difference in NEC between early and late fed infants.
A possible role for pre- and probiotics (emerging evidence, ongoing trials).

NEC can occur in outbreaks but in <50% of cases an organism is found. Organisms isolated include Escherichia
coli, Staphylococcus epidermidis, rotavirus and Clostridium perfringens.

Term
neonates who develop NEC usually have a predisposing factor such as perinatal hypoxia–ischaemia or
congenital heart disease.
44033

Tag Question

Feedback

Difficulty: Average

Peer Responses

Previous Question
Next Question

Session Progress

https://mypastest.pastest.com/Secure/TestMe/Browser/436619[‫ ص‬06:08:55 11/12/1437]


MyPastest

Responses Correct: 0

Responses Incorrect: 11

Responses Total: 11

Responses - % Correct: 0%

Blog
About Pastest
Contact Us
Help

© Pastest 2016

https://mypastest.pastest.com/Secure/TestMe/Browser/436619[‫ ص‬06:08:55 11/12/1437]


MyPastest

Prefer to use the old MyPastest? Access it here »

Back to Filters

Question 10 of 71

Which of the following statements concerning birth injuries is the most factually correct?

A Caput succedaneum is limited by suture lines

B Fracture of the cervical spine sustained at delivery will be evident on radiograph

C Phrenic
nerve palsy with diaphragmatic weakness associated with Erb palsy generally needs surgical
intervention as an emergency to improve respiratory function

D The clavicle is the most common bone to fracture during labour and delivery

E Waiter’s tip positioning of the arm is seen with Klumpke paralysis

Explanation
One per cent of infants sustain fractures
at the time of their birth, clavicular fractures being the most common
(usually obvious on radiograph). Risk factors include shoulder dystocia,
macrosomia and breech delivery.

Injury to the cervical spine is rare but can occur (with or without radiological changes). Excessive rotation during
cephalic deliveries results in injury at the C1–2 level whereas breech deliveries with excessive traction result in
damage at C6–T1. Hypotonia, hyporeflexia and hypoventilation ensue. Magnetic resonance imaging (MRI) is
useful for diagnosis. Permanent quadriplegia is likely after this injury.

Brachial
plexus injuries usually result from traction associated with shoulder dystocia; 97% are from cephalic
deliveries, 1–2% from breech deliveries and 1% from caesarean sections.

Upper brachial plexus, Erb–Duchenne paralysis (C5–6): affected arm


is held in the waiter’s tip position.
Biceps and supinator reflexes are absent. Hand grasp is preserved. Associated diaphragmatic (phrenic
nerve) palsy represents a severe injury and may require surgical treatment.
Lower brachial plexus injury, Klumpke paralysis (C8–T1), claw hand deformity, absent triceps jerk and

https://mypastest.pastest.com/Secure/TestMe/Browser/436619[‫ ص‬06:09:18 11/12/1437]


MyPastest

palmar grasp with occasional ipsilateral Horner syndrome (eyelid ptosis and papillary miosis).
End Session
Total brachial plexus injury, Erb–Duchenne–Klumpke palsy: combines features of both. Depending on the
severity of the injury recovery may take up to 3 months, but is incomplete in about a third of cases.
Surgery may be indicated if recovery is not evident by this time.

Regarding the scalp, cephalohaematoma rather than caput succedaneum is limited by suture lines.
44034

Tag Question

Feedback

Difficulty: Average

Peer Responses

Session Progress

Responses Correct: 0

Responses Incorrect: 12

Responses Total: 12

Responses - % Correct: 0%

Previous Question

Blog
About Pastest
Contact Us
Help
Next Question
© Pastest 2016

https://mypastest.pastest.com/Secure/TestMe/Browser/436619[‫ ص‬06:09:18 11/12/1437]


MyPastest

Prefer to use the old MyPastest? Access it here »

Back to Filters

Question 11 of 71

Which of the following statements about intestinal obstruction/bile-stained vomit on the first day of life is most
factually correct?

A A history of meconium-stained amniotic fluid rules out intestinal obstruction as a cause for vomiting in
a neonate

B Bile-stained vomiting in a neonate requires a barium enema to rule out malrotation

C Distal intestinal obstruction in the neonate most commonly presents with vomiting within the first 24
hours of life

D Most cases of duodenal atresia occur as isolated abnormalities

E Non-bile-stained vomiting on the first day of life suggests a diagnosis of hypertrophic pyloric stenosis

Explanation
Duodenal atresia

This
accounts for 25–40% of all intestinal atresias, but is rare, occurring in approximately 1 child per 10 000
births. Most cases of duodenal atresia will be isolated abnormalities but associated abnormalities do occur; 20–
30% of cases are associated with Down syndrome. Other associations include malrotation, oesophageal atresia,
anorectal malformations and congenital heart disease. Fifty per cent will be diagnosed in utero during antenatal
scanning. Early onset of bilious vomiting within hours of birth is the norm for postnatal presentation. Fifteen per
cent of babies with duodenal atresia will not have bile-stained vomiting due to the atresia being proximal to the
ampulla of Vater. Classical radiological appearance is of the ‘double bubble’. Treatment is surgical.

Pyloric stenosis

This
has multifactorial inheritance. There is projectile non-bilious vomiting, with onset most commonly between
age 3 weeks and 5 months. The
baby presents hungry and with weight loss. Firstborn boys are more commonly
affected. Palpable pyloric tumour and visible gastric peristalsis are present. Mild unconjugated

https://mypastest.pastest.com/Secure/TestMe/Browser/436619[‫ ص‬06:09:41 11/12/1437]


MyPastest

hyperbilirubinaemia and hypochloraemic/hypokalaemic metabolic alkalosis also occur. Ultrasonography confirms


End Session
the diagnosis. Treatment is by pyloromyotomy (Ramstedt procedure) after correction of electrolyte and acid–base
disturbance.

Malrotation

This
refers to incomplete rotation of the midgut during fetal development, resulting in the small bowel occupying
the right side of the abdomen and
the large bowel the left. The caecum is often subhepatic. The mesentery
along
with the superior mesenteric artery is attached by a narrow stalk, which can twist around itself producing a midgut
volvulus. Infants can present with bile-stained vomiting with or without abdominal
distension in the first few
weeks of life, or later with episodes of abdominal colic and vomiting. Diagnosis is by upper GI contrast; barium
enema may show a malpositioned caecum but is less reliable.

Meconium can still be passed with obstruction of the upper small bowel.
Bilious vomiting in a neonate is a
surgical emergency until proven otherwise.

Other causes of intestinal obstruction in the neonatal period include Hirschsprung disease, meconium ileus and
imperforate anus. Distal obstructions may present with abdominal distension, delayed passage of meconium and
absence of transitional stools, rather than bilious vomiting.
44053

Tag Question

Feedback

Difficulty: Average

Peer Responses

Previous Question
Session Progress
Next Question
Responses Correct: 0

Responses Incorrect: 13

Responses Total: 13

Responses - % Correct: 0%

https://mypastest.pastest.com/Secure/TestMe/Browser/436619[‫ ص‬06:09:41 11/12/1437]


MyPastest

Blog
About Pastest
Contact Us
Help

© Pastest 2016

https://mypastest.pastest.com/Secure/TestMe/Browser/436619[‫ ص‬06:09:41 11/12/1437]


MyPastest

Prefer to use the old MyPastest? Access it here »

Back to Filters

Question 12 of 71

Which of the following statements about seizures in neonates is most factually correct?

A The overall prognosis of neonatal seizures in term babies is poor, with approximately 30% achieving a
good long-term outcome

B Neonatal seizures are twice as common in the preterm neonate as in the term neonate

C Infection is the second most common cause of seizures in the neonatal period

D Anticonvulsants in the neonatal period achieve good seizure control in 80% of patients

E There
is evidence that anticonvulsive medication commonly used to control neonatal seizures may
have detrimental effects on developmental outcome when used in neonates; however, the balance of
risks is still in favour of treating seizures to try to get the best outcome possible for the patient

Explanation
Convulsions are the most common neurological sign seen in the newborn; they are four times more common in the
preterm neonate (although more commonly unrecognised) than the term neonate, and overall rates are around
2.6/1000 live births.

Seizure types can be diverse, subtle and difficult to detect, and can vary within the same patient over a short period
of time.

Subtle
manifestations of neonatal seizures can include lip smacking, tongue thrusting, mouthing, cycling
movements of the lower limbs and swimming movements of the upper limbs.

Apnoea can be a common manifestation of subtle seizures, particularly in the preterm population.

Causes of neonatal seizures:

Hypoxic ischaemic encephalopathy (38-40%)

https://mypastest.pastest.com/Secure/TestMe/Browser/436619[‫ ص‬06:10:04 11/12/1437]


MyPastest

Cerebral infarction (20%)


End Session
MCA (middle cerebral artery) infarction
sagittal sinus thrombosis

Intraventricular haemorrhage (12–20%)


Congenital malformation (5–10%)
Infection (3–20%)
Hypoglycaemia/hypocalcaemia (3–19%)
Hyponatraemia
Hypernatraemia
Inborn error of metabolism (1%)
Kernicterus
Benign familial neonatal convulsions
Benign sleep myoclonus
Fifth day fits
Drug withdrawal
Idiopathic

Not
all clinical manifestations of seizure activity in the neonate correspond to abnormal EEG findings; conversely,
not all electrical seizure activity on EEG corresponds to overt abnormal movements (electroconvulsive
dissociation).

Anticonvulsants commonly used to control neonatal seizures have relatively low rates of EEG seizure control 43–
62%.

Data
suggest that the antiepileptic drugs (AEDs) commonly used to control seizures in the neonatal period may in
themselves lead to neuronal death
as a result of apoptosis, with lasting effects on development.

These
effects appear to be additive with the use of more than one AED. However, there is also evidence that
frequent seizures cause additional damage to the developing brain and so, in refractory seizures, a balance
needs to
be struck between treatment side effects and the risks of ongoing or recurrent seizure activity.

Prognosis
of neonatal seizures reported in one study of term babies was relatively good, with a reported favourable
outcome in 72%; however, this decreases to 50% if seizures secondary to HIE are considered separately.
44906

Previous Question
Next Question Tag Question

Feedback

Difficulty: Average

https://mypastest.pastest.com/Secure/TestMe/Browser/436619[‫ ص‬06:10:04 11/12/1437]


MyPastest

Peer Responses

Session Progress

Responses Correct: 0

Responses Incorrect: 14

Responses Total: 14

Responses - % Correct: 0%

Blog
About Pastest
Contact Us
Help

© Pastest 2016

https://mypastest.pastest.com/Secure/TestMe/Browser/436619[‫ ص‬06:10:04 11/12/1437]


MyPastest

Prefer to use the old MyPastest? Access it here »

Back to Filters

Question 13 of 71

Which of the following statements about congenital dislocation of the hip is the most factually correct?

A Congenital dislocation of the hip is more common in females and more commonly affects the right hip

B Outcome is improved with early detection because early surgical intervention is possible

C If a Pavlik harness can be fitted before 6 weeks of age, the treatment will be successful in
approximately 90% of patients

D Avascular
necrosis of the femoral head is a recognised complication of corrective
surgery but not
Pavlik harness treatment of developmental dysplasia of the hip

E The Pavlik harness is most commonly worn for 6 months, followed by a weaning period of a further 6
months

Explanation
Congenital dislocation or developmental dysplasia of the hip is a common disorder present in 15–20/1000 live
births, with frank dislocation in 1–2/1000 births.

It
is either present at birth or develops over the first 3 months of life as a consequence of the femoral head being
partially or fully displaced from the acetabulum.

Risk factors for congenital dislocation of the hip:

Family history (positive in 20%)


Female sex (female:male 6:1)
Breech presentation (factor in 30%)
Spina bifida
Being firstborn

https://mypastest.pastest.com/Secure/TestMe/Browser/436619[‫ ص‬06:10:26 11/12/1437]


MyPastest

Oligohydramnios
End Session
The left hip is more likely to be dislocated than the right. Ultrasonography is the best test for diagnosis. The
clinical tests of hip instability are the Ortolani (relocation) and Barlow (dislocation) tests.

The management of this condition varies according to the time of diagnosis. The earlier the diagnosis, the better
the prognosis. Currently all neonates are screened at birth and at 6–8 weeks by clinical examination. Prognosis is
better for those picked up on screening (<3 months) than those children presenting later (>3 months). Delayed
diagnosis beyond 3 months almost always requires surgical intervention but the timing of surgery is debatable.
Surgery is sometimes delayed to try to decrease the complication of avascular necrosis of the femoral head.
Treatment using the Pavlik harness has good prognosis if started early, <6 weeks, with 90% successful outcome. It
should be fitted as early as possible and kept in situ for 6 weeks with weaning over the next 6 weeks. The
hips
should be monitored closely and treatment discontinued if there is
no response due to the increased risk of
avascular necrosis of the cartilaginous femoral head.
44908

Tag Question

Feedback

Difficulty: Average

Peer Responses

Session Progress

Responses Correct: Previous Question 0

Responses Incorrect: 15
Next Question
Responses Total: 15

Responses - % Correct: 0%

https://mypastest.pastest.com/Secure/TestMe/Browser/436619[‫ ص‬06:10:26 11/12/1437]


MyPastest

Blog
About Pastest
Contact Us
Help

© Pastest 2016

https://mypastest.pastest.com/Secure/TestMe/Browser/436619[‫ ص‬06:10:26 11/12/1437]


MyPastest

Prefer to use the old MyPastest? Access it here »

Back to Filters

Question 14 of 71

You are seeing a well, thriving, breastfed, 14-day-old term baby with jaundice.

What is the most appropriate investigation?

A α1-Anti-trypsin phenotype

B Urine amino acids

C Thyroid function tests

D Blood culture

E Conjugated and unconjugated bilirubin

Explanation
Prolonged jaundice is defined as jaundice lasting more than 14 days in term babies and 21 days in preterm babies.

It is common and affects 15–40% of term breastfed infants.

It
is imperative to have full assessment of all children with prolonged jaundice to exclude the rare but potentially
catastrophic missed cause of this common condition, biliary atresia (jaundice is not a symptom, it
is a clinical
sign).

THE NICE guidelines (2010) for management of prolonged jaundice state the following:

Visual inspection for pale chalky stools and/or dark urine is obligatory.
Measure conjugated bilirubin, FBC (full blood count), blood group and direct antiglobulin test (DAT), and
culture urine.
Ensure that routine metabolic screening had been performed (Guthrie/neonatal blood spot test).
Conjugated
bilirubin levels >25 υmol/l should be referred for expert advice because they may indicate

https://mypastest.pastest.com/Secure/TestMe/Browser/436619[‫ ص‬06:10:49 11/12/1437]


MyPastest

underlying serious liver disease https://www.nice.org.uk/guidance/cg98


End Session
This baby is most likely to have physiological breast milk jaundice. Breastfeeding should not be stopped.

The
Guthrie/neonatal blood spot card currently screens for congenital hypothyroidism, phenylketonuria, sickle cell
disease, cystic fibrosis and medium-chain acyl-CoA dehydrogenase deficiency (MCADD).
44912

Tag Question

Feedback

Difficulty: Average

Peer Responses

Session Progress

Responses Correct: 0

Responses Incorrect: 16

Responses Total: 16

Responses - % Correct: 0%

Previous Question
Blog
About Pastest
Contact Us
Help
Next Question
© Pastest 2016

https://mypastest.pastest.com/Secure/TestMe/Browser/436619[‫ ص‬06:10:49 11/12/1437]


MyPastest

Prefer to use the old MyPastest? Access it here »

Back to Filters

Question 15 of 71

You are asked to review a 2-day-old baby with poor feeding. On examination the baby is floppy with absent
reflexes although appearing otherwise well and alert. She is admitted and undergoes a full
septic screen and
treated with antibiotics. There is no change in her condition and the septic screen is negative.

Which of the following investigations is most likely to yield a definitive diagnosis for this infant's presenting
features?

A Creatine kinase

B Muscle biopsy

C Nerve conduction studies

D Lumbar puncture

E Genetic testing for deletion on chromosome 5q

Explanation
This is the picture of congenital/severe spinal muscular atrophy (SMA) type 1, also called (eponymously)
Werdnig–Hoffman disease. It is an autosomal recessive disorder affecting 1 in 10 000 live births. Cases
present
any time from birth to 6 months of age, with 95% having symptoms before age 3 months and 60% being floppy at
birth, and 30% reporting decreased movement in utero.

Affected infants show progressive muscle weakness and hypotonia with absent reflexes, leading to problems with
sucking, swallowing and respiratory function. Patients have intact extraocular muscles and facial weakness is
minimal or absent. Babies often appear alert and there is no evidence of cerebral involvement.

Examination reveals diffuse muscle weakness and hypotonia greater in the proximal than the distal muscles. Deep
tendon reflexes are absent; tongue fasciculations are usually present but may be confused with the normal random
movements unless atrophy is present. Arthrogryphosis can be present at birth as a consequence of in utero
hypotonia. Creatine kinase (CK) is typically normal in SMA type 1 and only mildly raised/normal in the other

https://mypastest.pastest.com/Secure/TestMe/Browser/436619[‫ ص‬06:11:10 11/12/1437]


MyPastest

types. Cerebrospinal fluid (CSF) findings are normal.


End Session
Prenatal
and postnatal genetic testing are now available to identify this condition; 95% of cases of SMA involve a
homozygous deletion of the SMN1
gene on the long arm of chromosome 5 (5q).

Muscle biopsy may be needed if genetic testing is not definitive.

Neonatal nerve conduction studies are difficult to interpret and may be delayed until 6 months of age.

There
is no curative treatment and, even with supportive measures, the median
survival is 7 months with a 95%
mortality rate by 18 months, with respiratory infection leading to death in most cases.
44914

Tag Question

Feedback

Difficulty: Average

Peer Responses

Session Progress

Responses Correct: 0

Responses Incorrect: 17

Responses Total: 17

Responses - % Correct: 0%
Previous Question
Next Question
Blog
About Pastest
Contact Us
Help

© Pastest 2016

https://mypastest.pastest.com/Secure/TestMe/Browser/436619[‫ ص‬06:11:10 11/12/1437]


MyPastest

https://mypastest.pastest.com/Secure/TestMe/Browser/436619[‫ ص‬06:11:10 11/12/1437]


MyPastest

Prefer to use the old MyPastest? Access it here »

Back to Filters

Question 16 of 71

A systolic murmur is heard in an asymptomatic, pink, term baby with normal pulses and otherwise normal
examination, with no dysmorphic features on the routine first day neonatal check.

What is the MOST appropriate action to take immediately?

A Pre-and post-ductal saturations

B Four limb blood pressure

C Echo same day

D Chest radiograph

E Discharge with written advice about possible symptoms of heart disease and clinical review in one
week

Explanation
The prevalence of murmurs in the neonatal period varies widely. Many types of congenital heart disease can be
associated with an asymptomatic murmur. Non-pathological
murmurs in the neonate are generally related to blood
flow through the ductus arteriosus or turbulent flow through the pulmonary arteries. In
one study of 50 infants
with murmurs, 60% had PDA – all closed by
6 weeks; 50% had pulmonary branch stenosis – all resolved by 6
months

Management:

Careful
clinical examination of the whole cardiovascular system is essential and this should be repeated
daily if the infant remains in hospital.
Four limb blood pressure is variable and not reliable in identifying duct-dependent heart disease.
Chest radiograph is unlikely to be helpful.

https://mypastest.pastest.com/Secure/TestMe/Browser/436619[‫ ص‬06:11:36 11/12/1437]


MyPastest

ECG can be helpful, particularly if abnormal axis or consistent with LVH – indicating cardiac disease.
End Session
Pulse
oximetry should be performed in all neonates with clinical murmurs. It may detect subtle or evolving
hypoxaemia and difference between pre- and
post-ductal saturation may reveal a duct-dependent lesion,
saturations <95% being regarded as abnormal.
Refer any child with significant symptoms or signs (other than the murmur), those who have dysmorphic
features or other congenital abnormality or a harsh/loud murmur, or who show hypoxaemia on pulse
oximetry.
Departments
vary but in one large audit all children who were well with low intensity murmurs (10% of
neonates) and normal pulse oximetry were discharged with written advice about significant symptoms and
reviewed weekly and, if the murmur was still present at 2 weeks, a referral to cardiology was made. A
third of referrals were found to have a structural heart defect. Only
6% of infants fitting these criteria
discharged in the latest 4 years of audit had a structural heart defect, none of which was life threatening.

44916

Tag Question

Feedback

Difficulty: Average

Peer Responses

Previous Question Session Progress

Responses Correct:
Next Question 0

Responses Incorrect: 18

Responses Total: 18

Responses - % Correct: 0%

https://mypastest.pastest.com/Secure/TestMe/Browser/436619[‫ ص‬06:11:36 11/12/1437]


MyPastest

Blog
About Pastest
Contact Us
Help

© Pastest 2016

https://mypastest.pastest.com/Secure/TestMe/Browser/436619[‫ ص‬06:11:36 11/12/1437]


MyPastest

Prefer to use the old MyPastest? Access it here »


End Session

Back to Filters

Question 17 of 71

A child is born having not had any scans during pregnancy. On


examination it is noted she is lymphoedematous,
has widely spaced nipples, a systolic murmur and absent femoral pulses.

What is the most likely diagnosis?

A Angelman syndrome

B DiGeorge syndrome

C Edward syndrome

D Turner syndrome

E Williams syndrome

Explanation
Turner syndrome affects around 1 in 3000 live births. Features include wide-spaced nipples, short fourth
metacarpals, low hairline, high-arched palate, lymphoedema, horseshoe kidney, and cardiac anomalies -
commonly coarcation of the aorta.
45986

Previous Question
Next Question
Tag Question

Feedback

Difficulty: Average

https://mypastest.pastest.com/Secure/TestMe/Browser/436619[‫ ص‬06:11:56 11/12/1437]


MyPastest

Peer Responses

Session Progress

Responses Correct: 0

Responses Incorrect: 19

Responses Total: 19

Responses - % Correct: 0%

Blog
About Pastest
Contact Us
Help

© Pastest 2016

https://mypastest.pastest.com/Secure/TestMe/Browser/436619[‫ ص‬06:11:56 11/12/1437]


MyPastest

Prefer to use the old MyPastest? Access it here »


End Session

Back to Filters

Question 18 of 71

Which of the following are found in Angelman syndrome?

A Ataxia

B Microcephaly

C Seizures

D Unprovoked clapping

E All of the above

Explanation
Angelman syndrome is rare. in addition to the above, quasi-purposeful mouth and tongue movements, and
unprovoked laughing also occur.
45988

Tag Question
Previous Question
Feedback
Next Question
Difficulty: Average

Peer Responses

https://mypastest.pastest.com/Secure/TestMe/Browser/436619[‫ ص‬06:12:16 11/12/1437]


MyPastest

Session Progress

Responses Correct: 0

Responses Incorrect: 20

Responses Total: 20

Responses - % Correct: 0%

Blog
About Pastest
Contact Us
Help

© Pastest 2016

https://mypastest.pastest.com/Secure/TestMe/Browser/436619[‫ ص‬06:12:16 11/12/1437]


MyPastest

Prefer to use the old MyPastest? Access it here »

Back to Filters

Question 19 of 71

Which one of the following is a feature of Williams syndrome?

A Hypocalcaema

B Hypotension

C Pulmonary artery stenosis

D Pulmonary valve regurgitation

E Subvalvular aortic stenosis

Explanation
Williams syndrome is a rare condition with the following features:

Clinodactyly
Dental anomalies
Epicanthic folds
Flat nasal bridge
Learning difficulties
Pectus excavatum
Attention deficit
Hypercalcaemia
Hypertension
Pulmonary artery stenosis
Pulmonary stenosis
Supravalvular aortic valve stenosis

https://mypastest.pastest.com/Secure/TestMe/Browser/436619[‫ ص‬06:12:39 11/12/1437]


MyPastest

End Session 45990

Tag Question

Feedback

Difficulty: Average

Peer Responses

Session Progress

Responses Correct: 0

Responses Incorrect: 21

Responses Total: 21

Responses - % Correct: 0%

Blog
About Pastest
Contact Us
Help

© Pastest 2016

Previous Question
Next Question

https://mypastest.pastest.com/Secure/TestMe/Browser/436619[‫ ص‬06:12:39 11/12/1437]


MyPastest

Prefer to use the old MyPastest? Access it here »

Back to Filters

Question 20 of 71

What is the most common cardiac abnormality found in Noonan syndrome?

A Coarctation of the aorta

B Double inlet left ventricle

C Hypoplastic left heart

D Patent ductus arteriosus

E Pulmonary stenosis

Explanation
Noonan syndrome is an autosomal dominant condition with the following features:

Mental retardation
Orbital hypertelorism
Blue/green eyes
Nystagmus
Thrombocytopenia
Clotting factor deficiencies
Lymphoedema
Delayed puberty
Low-set ears
Short webbed neck
Low hairline
Deep groove in upper lip

https://mypastest.pastest.com/Secure/TestMe/Browser/436619[‫ ص‬06:13:00 11/12/1437]


MyPastest

Poor facial expression End Session


Pectus excavatum/carinatum
Wide-set nipples

Cardiac
defects (50% cases) especially right-sided, commonly pulmonary stenosis. Also pulmonary artery
stenosis, septal defects, HCM. May have normal or short stature. Normal female fertility.
45991

Tag Question

Feedback

Difficulty: Average

Peer Responses

Session Progress

Responses Correct: 0

Responses Incorrect: 22

Responses Total: 22

Responses - % Correct: 0%

Previous Question
Next Question
Blog
About Pastest
Contact Us
Help

© Pastest 2016

https://mypastest.pastest.com/Secure/TestMe/Browser/436619[‫ ص‬06:13:00 11/12/1437]


MyPastest

Prefer to use the old MyPastest? Access it here »

Back to Filters

Question 21 of 71

You are taking a history from a woman in clinic who is trying


to conceive. She mentions enjoying a glass of wine
at the week-end and
asks whether this can continue during pregnancy.

What maximum safe level of alcohol can women be advised to consume during pregnancy in order to eliminate
the risk of damage to the fetus?

A 1 unit per week

B 2 units per week

C 3 units per week

D No more than one unit per day

E None

Explanation
Women should always be advised to avoid alcohol in pregnancy. There is no defined safe limit and avoidance is
the best policy. Consumption during early pregnancy can increase the risk of miscarriage.

Fetal alcohol syndrome is a dose dependent condition. Features are variable and include:

Acute neonatal alcohol withdrawal


ASD, Ataxia
Epicanthic folds
IUGR
Learning disabilities
Microcephaly
Miscarriage

https://mypastest.pastest.com/Secure/TestMe/Browser/436619[‫ ص‬06:13:21 11/12/1437]


MyPastest

Narrow palpebral fissures


End Session
Neonatal hypotonia
PDA
Ptosis
Seizures
Skeletal deformities
Smooth philtrum
Thin superior lip
Urinary tract deformities
VSD.

46010

Tag Question

Feedback

Difficulty: Easy

Peer Responses

Session Progress

Responses Correct: 0
Previous Question
Responses Incorrect: 23

Responses Total: Next Question 23

Responses - % Correct: 0%

https://mypastest.pastest.com/Secure/TestMe/Browser/436619[‫ ص‬06:13:21 11/12/1437]


MyPastest

Blog
About Pastest
Contact Us
Help

© Pastest 2016

https://mypastest.pastest.com/Secure/TestMe/Browser/436619[‫ ص‬06:13:21 11/12/1437]


MyPastest

Prefer to use the old MyPastest? Access it here »

Back to Filters

Question 22 of 71

A floppy newborn baby is noticed to have epicanthic folds, a thin upper lip and smooth philtrum. He has been
monitored closely throughout pregnancy for intra-uterine growth retardation (IUGR) and a ventricular septal
defect.

What is the most likely diagnosis?

A Down syndrome

B Edward syndrome

C Fetal alcohol syndrome

D Holt-Oram syndrome

E Turner syndrome

Explanation
this is a description of an infant with fetal alcohol syndrome. It is likely that the effects on any one fetus are
determined by the degree, timing and duration of exposure as well as the
susceptibility of the fetus which is likely
to be genetically determined.

Women should always be advised to avoid alcohol in pregnancy. There is no defined safe limit and avoidance is
the best policy. Consumption during early pregnancy can increase the risk of miscarriage.

As mentioned above, fetal alcohol syndrome is a dose


dependent condition. Features are variable and include:
Acute neonatal alcohol withdrawal, ASD, Ataxia, Epicanthic folds, IUGR, Learning disabilities, Microcephaly,
Miscarriage, Narrow palpebral fissures, Neonatal hypotonia, PDA, Ptosis, Seizures, Skeletal deformities, Smooth
philtrum, Thin superior lip, Urinary tract deformities, VSD.
46013

https://mypastest.pastest.com/Secure/TestMe/Browser/436619[‫ ص‬06:13:45 11/12/1437]


MyPastest

End Session

Tag Question

Feedback

Difficulty: Average

Peer Responses

Session Progress

Responses Correct: 0

Responses Incorrect: 24

Responses Total: 24

Responses - % Correct: 0%

Blog
About Pastest
Contact Us
Help

© Pastest 2016

Previous Question
Next Question

https://mypastest.pastest.com/Secure/TestMe/Browser/436619[‫ ص‬06:13:45 11/12/1437]


MyPastest

Prefer to use the old MyPastest? Access it here »

Back to Filters

Question 23 of 71

A newborn child is noted not to be moving his left arm properly. Of relevance he had shoulder dystocia during
childbirth. His arm is noted to be hanging down with the shoulder internally rotated, elbow extended, forearm
pronated and wrist flexed.

What is the most likely diagnosis?

A Cerebral palsy

B Erb palsy

C Pseudo bulbar palsy

D Radial nerve palsy

E Ulnar nerve palsy

Explanation
Epidemiology:

Approximately 1 in 2,000 births

Aetiology/risk factors:

Brachial plexus injury (C5-C8, T1) commonly due to birth trauma (shoulder dystocia, clavicular fracture)
C5 most common nerve root to be affected in birth injuries
Nerve may be avulsed from spine, torn or stretched

90% heal without treatment

https://mypastest.pastest.com/Secure/TestMe/Browser/436619[‫ ص‬06:14:06 11/12/1437]


MyPastest

46053
End Session

Tag Question

Feedback

Difficulty: Average

Peer Responses

Session Progress

Responses Correct: 0

Responses Incorrect: 25

Responses Total: 25

Responses - % Correct: 0%

Blog
About Pastest
Contact Us
Help

© Pastest 2016

Previous Question
Next Question

https://mypastest.pastest.com/Secure/TestMe/Browser/436619[‫ ص‬06:14:06 11/12/1437]


MyPastest

Prefer to use the old MyPastest? Access it here »


End Session

Back to Filters

Question 24 of 71

How many newborn babies who have spent at least 48 hours in special care have hearing loss in one or both ears?

A 1 in 1000

B 1 in 500

C 1 in 100

D 1 in 50

E 1 in 10

Explanation
Whilst between 1-2 of newborns per 1000 births have hearing difficulties in one or both ears, this increases to 1 in
100 in those admitted to special care for at least 2 days.

The newborn hearing test is designed to identify and support these children at the earliest opportunity. It initially
consists of an otoacoustic emissions test and if this does not produce a satisfactory clear result then the auditory
brain-stem response test will be used.

If these tests are not 'passed' then the baby should be referred for audiology assessment.
46054

Previous Question
Next Question

Tag Question

Feedback

https://mypastest.pastest.com/Secure/TestMe/Browser/436619[‫ ص‬06:14:27 11/12/1437]


MyPastest

Difficulty: Average

Peer Responses

Session Progress

Responses Correct: 0

Responses Incorrect: 26

Responses Total: 26

Responses - % Correct: 0%

Blog
About Pastest
Contact Us
Help

© Pastest 2016

https://mypastest.pastest.com/Secure/TestMe/Browser/436619[‫ ص‬06:14:27 11/12/1437]


MyPastest

Prefer to use the old MyPastest? Access it here »

Back to Filters

Question 25 of 71

A new mother who gave birth at home and subsequently moved in


with her parents some 100 miles distant for the
first 6 weeks, contacts
her GP as on returning home she realises she has missed her baby's appointment for routine
hearing screening assessment.

Whilst
the newborn hearing test is best carried out within the first four or five weeks of life, until what age can it
be performed?

A 8 weeks

B 10 weeks

C 12 weeks

D 15 weeks

E 24 weeks

Explanation
The new born hearing test is designed to identify
hearing impairment at the earliest opportunity in order to
maximise the
child's language and educational potential. Whilst it is ideally performed within the first 4 or 5
weeks of life, it can be done up to three months of age.

For a 'well' baby, the otoacoustic emissions test is generally sufficient to make an accurate assessment however if
the result of this is unclear or you are assessing a 'special care' baby, then the auditory brain-stem response test
will be employed also. If neither of these tests is conclusive or 'passed' then referral for audiology assessment is
indicated.

For further information see:

http://www.nhs.uk/Conditions/pregnancy-and-baby/Pages/newborn-hearing-test.aspx
46055

https://mypastest.pastest.com/Secure/TestMe/Browser/436619[‫ ص‬06:14:48 11/12/1437]


MyPastest

End Session

Tag Question

Feedback

Difficulty: Average

Peer Responses

Session Progress

Responses Correct: 0

Responses Incorrect: 27

Responses Total: 27

Responses - % Correct: 0%

Blog
About Pastest
Contact Us
Help

© Pastest 2016

Previous Question
Next Question

https://mypastest.pastest.com/Secure/TestMe/Browser/436619[‫ ص‬06:14:48 11/12/1437]


MyPastest

Prefer to use the old MyPastest? Access it here »

Back to Filters

Question 26 of 71

A 5-day-old baby is brought in for her newborn blood spot test (heel prick test).

How many conditions are tested for in the neonatal heel prick test?

A 5

B 6

C 7

D 8

E 9

Explanation
The following 9 conditions are tested for in the heel prick test:

Sickle cell anaemia


Cystic fibrosis
Congenital hypothyroidism
Phenylketonuria (PKU)
Medium-chain acyl-CoA dehydrogenase deficiency (MCADD)
Maple syrup urine disease (MSUD)
Isovaleric acidaemia (IVA)
Glutaric aciduria type 1 (GA1)
Homocystinuria (pyridoxine unresponsive) (HCU)

46056

https://mypastest.pastest.com/Secure/TestMe/Browser/436619[‫ ص‬06:15:10 11/12/1437]


MyPastest

End Session

Tag Question

Feedback

Difficulty: Average

Peer Responses

Session Progress

Responses Correct: 0

Responses Incorrect: 28

Responses Total: 28

Responses - % Correct: 0%

Blog
About Pastest
Contact Us
Help

© Pastest 2016

Previous Question
Next Question

https://mypastest.pastest.com/Secure/TestMe/Browser/436619[‫ ص‬06:15:10 11/12/1437]


MyPastest

Prefer to use the old MyPastest? Access it here »

Back to Filters

Question 27 of 71

You are looking at the results of a neonatal heel prick test.

Which one of the following conditions is NOT screened for in this test?

A Galactosaemia

B Homocystinuria (pyridoxine unresponsive)

C Maple syrup urine disease

D Medium-chain acyl-CoA dehydrogenase deficiency

E Phenylketonuria

Explanation
Galactosaemia is an autosomally recessive inherited condition (chr 9) and is characterised by the inability to break
down galactose once milk-feeding starts.

Features include:

Vomiting
Diarrhoea
Lethargy
Failure to thrive/develop
Neonatal jaundice
Hepatosplenomegaly
Bulging fontanelle

https://mypastest.pastest.com/Secure/TestMe/Browser/436619[‫ ص‬06:15:35 11/12/1437]


MyPastest

The following 9 conditions are tested for in the heel prick test:
End Session
Sickle cell anaemia
Cystic fibrosis
Congenital hypothyroidism
Phenylketonuria (PKU)
Medium-chain acyl-CoA dehydrogenase deficiency (MCADD)
Maple syrup urine disease (MSUD)
Iisovaleric acidaemia (IVA)
Glutaric aciduria type 1 (GA1)
Homocystinuria (pyridoxine unresponsive) (HCU)

46057

Tag Question

Feedback

Difficulty: Average

Peer Responses

Session Progress

Responses Correct: 0
Previous Question
Responses Incorrect: 29
Next Question
Responses Total: 29

Responses - % Correct: 0%

https://mypastest.pastest.com/Secure/TestMe/Browser/436619[‫ ص‬06:15:35 11/12/1437]


MyPastest

Blog
About Pastest
Contact Us
Help

© Pastest 2016

https://mypastest.pastest.com/Secure/TestMe/Browser/436619[‫ ص‬06:15:35 11/12/1437]


MyPastest

Prefer to use the old MyPastest? Access it here »

Back to Filters

Question 28 of 71

A newborn child is found to have phenylketonuria on heel prick testing.

What is the incidence of PKU?

A 5 in 10000 live births

B 4 in 10000 live births

C 3 in 10000 live births

D 2 in 10000 live births

E 1 in 10000 live births

Explanation
PKU is an autosomal recessive condition and is the most common inborn error of metabolism in the UK. It has an
incidence of 1:10,000 and a carrier rate of 1:50.

Clinical features:

Eczema
Failure to thrive
If untreated: Mental disability, Microcephaly, poor pigmentation (blue eyes, light skin), Seizures
If treated: Asymptomatic, IQ approximately 10 points below unaffected sibling

Management:

Genetic counselling
Lifelong dietary phenylalanine restriction

https://mypastest.pastest.com/Secure/TestMe/Browser/436619[‫ ص‬06:15:56 11/12/1437]


MyPastest

End Session
As
phenylalanine is an essential amino acid it cannot be totally excluded from the diet. Phenylalanine is
teratogenic and so affected girls need strict dietary control pre-conception and during pregnancy to achieve best
outcomes.
46059

Tag Question

Feedback

Difficulty: Average

Peer Responses

Session Progress

Responses Correct: 0

Responses Incorrect: 30

Responses Total: 30

Responses - % Correct: 0%

Blog
About Pastest
Contact Us
Help
Previous Question
© Pastest 2016
Next Question

https://mypastest.pastest.com/Secure/TestMe/Browser/436619[‫ ص‬06:15:56 11/12/1437]


MyPastest

Prefer to use the old MyPastest? Access it here »

Back to Filters

Question 29 of 71

A baby is born spontaneously at 32 weeks. Soon after birth the midwife is concerned he is floppy, drowsy and
irritable. His blood sugars and CRP are normal. He goes on to have a seizure after which urgent neuroimaging is
requested.

What is the most likely cause?

A Intracranial haemorrhage

B Muscular dystrophy

C Myaesthenia gravis

D Spinal muscular atrophy

E Syringomyelia

Explanation
It is likely there is a central cause for the hypotonia in this baby given the irritability and seizures. All the other
causes are peripheral and should not affect central function.

A useful list of differential diagnoses can be found here:

http://www.adhb.govt.nz/newborn/Guidelines/Neurology/Hypotonia.htm
Previous Question 46060

Next Question

Tag Question

https://mypastest.pastest.com/Secure/TestMe/Browser/436619[‫ ص‬06:16:17 11/12/1437]


MyPastest

Feedback End Session

Difficulty: Average

Peer Responses

Session Progress

Responses Correct: 0

Responses Incorrect: 31

Responses Total: 31

Responses - % Correct: 0%

Blog
About Pastest
Contact Us
Help

© Pastest 2016

https://mypastest.pastest.com/Secure/TestMe/Browser/436619[‫ ص‬06:16:17 11/12/1437]


MyPastest

Prefer to use the old MyPastest? Access it here »

Back to Filters

Question 30 of 71

You are asked to see a 6-month-old hypotonic baby and note he


has undescended testicles, failure to thrive,
almond shaped eyes and a thin upper lip. His mother mentions feeding is a challenge. He has had all his screening
tests and immunisations to date.

What is the most likely diagnosis?

A Homocystinuria (pyridoxine unresponsive)

B Noonan syndrome

C Phenylketonuria

D Prader-Willi syndrome

E Refsum's disease

Explanation
Prader-Willi syndrome is caused by an abnormality in chromosome 15. Features include:

Cryptorchidism
Excessive hunger (but feeding difficulties in the first year of life)
Failure to thrive (and excessive eating and weight thereafter)
Hypotonia
Mental handicap

Answer stems A and C are tested for in the neonatal heel prick test.

Noonan's causes:

https://mypastest.pastest.com/Secure/TestMe/Browser/436619[‫ ص‬06:16:42 11/12/1437]


MyPastest

Mental retardation
End Session
Orbital hypertelorism
Blue/green eyes
Nystagmus
Thrombocytopenia
Clotting factors 8, 11, 12 deficiencies
Lymphoedema
Delayed puberty
Low-set ears
Short webbed neck
Low hairline
Deep groove in upper lip
Poor facial expression
Pectus excavatum/carinatum
Wide-set nipples
Cardiac defects

The features of Refsum's disease include:

Ataxia
Bone defects
Cataract, Deafness
Ichthyosis
Nerve thickening
Neuropathy
Retinitis pigmentosa
Cardiomyopathy

46061

Previous Question Tag Question

Feedback Next Question

Difficulty: Average

Peer Responses

https://mypastest.pastest.com/Secure/TestMe/Browser/436619[‫ ص‬06:16:42 11/12/1437]


MyPastest

Session Progress

Responses Correct: 0

Responses Incorrect: 32

Responses Total: 32

Responses - % Correct: 0%

Blog
About Pastest
Contact Us
Help

© Pastest 2016

https://mypastest.pastest.com/Secure/TestMe/Browser/436619[‫ ص‬06:16:42 11/12/1437]


MyPastest

Prefer to use the old MyPastest? Access it here »


End Session

Back to Filters

Question 31 of 71

A neonate has been observed, by his parents, twitching and jerking in his sleep causing them considerable alarm.

Which one of the following is not a cause of neonatal seizure?

A Hypoxic ischaemic encephalopathy

B Intracranial haemorrhage

C Intracranial infection

D Metabolic disorder

E Neonatal myoclonus

Explanation
Neonatal myoclonus is not a true seizure but the involuntary jerking of muscles caused by involuntary contraction
or relaxation of a muscle or muscle group. It is a benign condition, common in pre-term infants and seen during
periods of quiet sleep but can closely mimic true seizures and frequently causes concern.

Common causes of true neonatal seizures include all of the other listings and also congenital cerebral
malformations and focal ischaemic stroke.
46062

Previous Question
Next Question

Tag Question

Feedback

https://mypastest.pastest.com/Secure/TestMe/Browser/436619[‫ ص‬06:17:04 11/12/1437]


MyPastest

Difficulty: Average

Peer Responses

Session Progress

Responses Correct: 0

Responses Incorrect: 33

Responses Total: 33

Responses - % Correct: 0%

Blog
About Pastest
Contact Us
Help

© Pastest 2016

https://mypastest.pastest.com/Secure/TestMe/Browser/436619[‫ ص‬06:17:04 11/12/1437]


MyPastest

Prefer to use the old MyPastest? Access it here »

Back to Filters

Question 32 of 71

What is the most likely time of life to have a seizure?

A 0 - 7 days old

B 7 - 14 days old

C 14 - 21 days old

D 21 - 28 days old

E 28 - 35 days old

Explanation
Seizures are not uncommon in the first week of life. Causes are many fold and include:

Hypoxic ischaemic encephalopathy


Intracranial haemorrhage
Intracranial infections
Congenital cerebral malformations
Metabolic disorders
Focal ischaemic stroke

Next Question 46063

https://mypastest.pastest.com/Secure/TestMe/Browser/436619[‫ ص‬06:17:24 11/12/1437]


MyPastest

End
Tag Session
Question

Feedback

Difficulty: Average

Peer Responses

Session Progress

Responses Correct: 0

Responses Incorrect: 34

Responses Total: 34

Responses - % Correct: 0%

Blog
About Pastest
Contact Us
Help

© Pastest 2016

Previous Question

https://mypastest.pastest.com/Secure/TestMe/Browser/436619[‫ ص‬06:17:24 11/12/1437]


MyPastest

Prefer to use the old MyPastest? Access it here »


End Session

Back to Filters

Question 33 of 71

You are called to a crash c-section. After the birth you are concerned about the baby’s condition and assess her
heart rate.

According to resuscitation guidelines, at which point should you commence chest compressions?

A Less than 40 beats per minute

B Less than 60 beats per minute

C Less than 100 beats per minute

D Less than 120 beats per minute

E Less than 160 beats per minute

Explanation
Neonatal resuscitation guidelines are easy question fodder for exams. Definitely required reading. Chest
compressions should be commenced if the heart rate is less than 60.

Full information can be found on the following website:


https://www.resus.org.uk/_resources/assets/attachment/full/0/811.pdf
46104

Previous Question
Next Question

Tag Question

Feedback

https://mypastest.pastest.com/Secure/TestMe/Browser/436619[‫ ص‬06:17:43 11/12/1437]


MyPastest

Difficulty: Average

Peer Responses

Session Progress

Responses Correct: 0

Responses Incorrect: 35

Responses Total: 35

Responses - % Correct: 0%

Blog
About Pastest
Contact Us
Help

© Pastest 2016

https://mypastest.pastest.com/Secure/TestMe/Browser/436619[‫ ص‬06:17:43 11/12/1437]


MyPastest

Prefer to use the old MyPastest? Access it here »


End Session

Back to Filters

Question 34 of 71

You are performing resuscitation on a neonate.

How many chest compressions should be administered for each breath?

A 1

B 2

C 3

D 4

E 5

Explanation
Chest compressions should only take place after the baby has received 5 inflation breaths such that they are
aerated. The ratio of compressions to inflations in newborn resuscitation is 3:1

For further information see the following website:


https://www.resus.org.uk/_resources/assets/attachment/full/0/811.pdf
46105

Previous Question
Next Question
Tag Question

Feedback

https://mypastest.pastest.com/Secure/TestMe/Browser/436619[‫ ص‬06:18:03 11/12/1437]


MyPastest

Difficulty: Average

Peer Responses

Session Progress

Responses Correct: 0

Responses Incorrect: 36

Responses Total: 36

Responses - % Correct: 0%

Blog
About Pastest
Contact Us
Help

© Pastest 2016

https://mypastest.pastest.com/Secure/TestMe/Browser/436619[‫ ص‬06:18:03 11/12/1437]


MyPastest

Prefer to use the old MyPastest? Access it here »

Back to Filters

Question 35 of 71

During resuscitation of the newborn who is not breathing spontaneously, the lungs may need to be aerated given
they are fluid filled in utero.

What is the appropriate pressure to be used when giving inflation breaths to a newborn child born at term?

A 15 cmH2O

B 20 cmH2O

C 25 cmH2O

D 30 cmH2O

E 35 cmH2O

Explanation
According to UK national guidelines, 5 inflation breaths should be given with a gas pressure of 30cmH2O for
term babies. Each breath should be given for 2-3 seconds. Pre-term babies should be aerated with a lower pressure
of 20-25cmH2O.

For further information see the following website:


https://www.resus.org.uk/_resources/assets/attachment/full/0/811.pdf
46106
Previous Question
Next Question

Tag Question

Feedback

https://mypastest.pastest.com/Secure/TestMe/Browser/436619[‫ ص‬06:18:31 11/12/1437]


MyPastest

End Session

Difficulty: Average

Peer Responses

Session Progress

Responses Correct: 0

Responses Incorrect: 37

Responses Total: 37

Responses - % Correct: 0%

Blog
About Pastest
Contact Us
Help

© Pastest 2016

https://mypastest.pastest.com/Secure/TestMe/Browser/436619[‫ ص‬06:18:31 11/12/1437]


MyPastest

Prefer to use the old MyPastest? Access it here »

Back to Filters

Question 36 of 71

Following an emergency c-section, following which the baby required lung inflation and chest compression there
is still inadequate circulation.

Approximately how many babies require more intervention than inflation breaths and chest compressions to
achieve effective sustained circulation?

A 1 in 10

B 1 in 100

C 1 in 1000

D 1 in 10000

E 1 in 100000

Explanation
Most babies will not require more than inflation breaths and chest compressions. Only around 1 in 1000 babies
will fall into this category so it is unusual to require drugs in neonatal resuscitation.

When required, drugs used include adrenaline (1:10,000), sodium bicarbonate, and dextrose and are usually
delivered by umbilical venous catheter.
Previous
For further information Question
please see:
46107
Next Question

Tag Question

https://mypastest.pastest.com/Secure/TestMe/Browser/436619[‫ ص‬06:18:50 11/12/1437]


MyPastest

Feedback End Session

Difficulty: Average

Peer Responses

Session Progress

Responses Correct: 0

Responses Incorrect: 38

Responses Total: 38

Responses - % Correct: 0%

Blog
About Pastest
Contact Us
Help

© Pastest 2016

https://mypastest.pastest.com/Secure/TestMe/Browser/436619[‫ ص‬06:18:50 11/12/1437]


MyPastest

Prefer to use the old MyPastest? Access it here »


End Session

Back to Filters

Question 37 of 71

You are asked to assess a feverish baby in the emergency department. The child looks poorly, has a weak,
continuous cry, displays decreased activity.

Which one of the following clinical features would not be considered as high risk according to NICE?

A Continuous cry

B Decreased activity

C Grunting

D Looks ill

E Weak cry

Explanation
According to NICE, all the above features are RED
except decreased activity which is amber. Eye balling a patient
as to whether they look ill or well is an essential skill.

A full list of high risk symptoms and signs can be found in the following NICE guidance document.
https://www.nice.org.uk/guidance/cg160/chapter/1-recommendations
46108

Previous Question
Next Question

Tag Question

Feedback

https://mypastest.pastest.com/Secure/TestMe/Browser/436619[‫ ص‬06:19:10 11/12/1437]


MyPastest

Difficulty: Average

Peer Responses

Session Progress

Responses Correct: 0

Responses Incorrect: 39

Responses Total: 39

Responses - % Correct: 0%

Blog
About Pastest
Contact Us
Help

© Pastest 2016

https://mypastest.pastest.com/Secure/TestMe/Browser/436619[‫ ص‬06:19:10 11/12/1437]


MyPastest

Prefer to use the old MyPastest? Access it here »

Back to Filters

Question 38 of 71

Following a premature birth where the mother’s membranes ruptured more than 24 hours before delivery you have
a 6-hour-old baby exhibiting signs of respiratory distress. You have concerns about the risk of sepsis.

Which of the following statements best describes sepsis?

A Disseminated meningococcal disease

B Dysregulated inflammatory response to infection

C Liver and renal failure

D Shock

E Tachycardia

Explanation
There is a huge movement towards recognising sepsis early as early intervention with antibiotics and fluids can
save lives. Sepsis is caused by an over-zealous immune reaction to infection
which can lead to shock, multi-organ
failure and eventually death.

Risk factors include parenteral antibiotic treatment given to the mother for a confirmed or suspected bacterial
infection during labour or
24 hours either side of the birth, or a suspected or confirmed case in another baby in the
case of multiple pregnancy.

Red flag indications would include:

Requirement for mechanical ventilation in a term baby


Seizures
Shock
Respiratory distress appearing over 4 hours after the birth

https://mypastest.pastest.com/Secure/TestMe/Browser/436619[‫ ص‬06:19:32 11/12/1437]


MyPastest

End Session
http://www.bmj.com/content/350/bmj.h3017
46109

Tag Question

Feedback

Difficulty: Average

Peer Responses

Session Progress

Responses Correct: 0

Responses Incorrect: 40

Responses Total: 40

Responses - % Correct: 0%

Blog
About Pastest
Contact Us
Help

© Pastest 2016
Previous Question
Next Question

https://mypastest.pastest.com/Secure/TestMe/Browser/436619[‫ ص‬06:19:32 11/12/1437]


MyPastest

Prefer to use the old MyPastest? Access it here »

Back to Filters

Question 39 of 71

“Sepsis is a life threatening condition that arises


when the body’s response to an infection injures its own tissues
and organs. Sepsis leads to shock, multiple organ failure and death especially if not recognized early and treated
promptly.” Sepsis Trust.org

Which of the following would not be considered to be consistent with sepsis?

A Hyperglycaemia

B Low bilirubin

C Low core temperature

D Low white count

E Oedema

Explanation
It is essential to be able to recognise sepsis. These are easy exam questions and essential knowledge for anybody
dealing with patients.

Information about the ‘Surviving Sepsis Campaign’ can be found via the following link:
http://www.sccm.org/Documents/SSC-Guidelines.pdf
46110
Previous Question
Next Question

Tag Question

Feedback

https://mypastest.pastest.com/Secure/TestMe/Browser/436619[‫ ص‬06:19:54 11/12/1437]


MyPastest

End Session

Difficulty: Average

Peer Responses

Session Progress

Responses Correct: 0

Responses Incorrect: 41

Responses Total: 41

Responses - % Correct: 0%

Blog
About Pastest
Contact Us
Help

© Pastest 2016

https://mypastest.pastest.com/Secure/TestMe/Browser/436619[‫ ص‬06:19:54 11/12/1437]


MyPastest

Prefer to use the old MyPastest? Access it here »


End Session

Back to Filters

Question 40 of 71

You are called to see a child in the emergency department. She is in septic shock. The child weighs approximately
10kg.

How much fluid should she receive as an initial fluid bolus?

A 20ml

B 200ml

C 2L

D 20L

E 200L

Explanation
This child, with hypovolaemic shock needs a fluid bolus of 20ml/kg body weight.

Thus: 20ml x 10kg = 200ml.


46111

Previous Question
Next Question Tag Question

Feedback

Difficulty: Average

https://mypastest.pastest.com/Secure/TestMe/Browser/436619[‫ ص‬06:20:16 11/12/1437]


MyPastest

Peer Responses

Session Progress

Responses Correct: 0

Responses Incorrect: 42

Responses Total: 42

Responses - % Correct: 0%

Blog
About Pastest
Contact Us
Help

© Pastest 2016

https://mypastest.pastest.com/Secure/TestMe/Browser/436619[‫ ص‬06:20:16 11/12/1437]


MyPastest

Prefer to use the old MyPastest? Access it here »

Back to Filters

Question 41 of 71

You are asked about the complications of sepsis by the parents of a sick child.

Which one of the following is a metabolic complication of meningococcal sepsis?

A Hypocalcaemia

B Hypoglycaemia

C Hypokalaemia

D Hypomagnesaemia

E All of the above

Explanation
Sepsis is critical to diagnose. Equally a knowledge of the potential underlying causes, how to investigate,
treatment and management of complications is required.

Early symptoms may include:

Fever or low body temperature


Chills and shivering
Fast heartbeat
Fast breathing

Complications may also include post sepsis syndrome which includes a variety of physical, psychological and
emotional factors. For example:

Lethargy

https://mypastest.pastest.com/Secure/TestMe/Browser/436619[‫ ص‬06:20:40 11/12/1437]


MyPastest

Muscle weakness
End Session
Swollen limbs/joint pain
Chest pain or breathlessness
Anxiety
Nightmares

https://www.nice.org.uk/guidance/cg102/resources/meningitis-bacterial-and-menigococcal-septicaemia-in-under-
16s-recognition-diagnosis-and-management-35109325611205
46113

Tag Question

Feedback

Difficulty: Average

Peer Responses

Session Progress

Responses Correct: 0

Responses Incorrect: 43

Responses Total: 43

Responses - % Correct: 0%
Previous Question
Next Question

Blog
About Pastest
Contact Us
Help

© Pastest 2016

https://mypastest.pastest.com/Secure/TestMe/Browser/436619[‫ ص‬06:20:40 11/12/1437]


MyPastest

https://mypastest.pastest.com/Secure/TestMe/Browser/436619[‫ ص‬06:20:40 11/12/1437]


MyPastest

Prefer to use the old MyPastest? Access it here »


End Session

Back to Filters

Question 42 of 71

Which one of the following conditions are more likely to persist in premature babies compared to term babies?

A Asthma

B Hearing difficulties

C Learning difficulties

D Visual disorders

E All of the above

Explanation
Prematurity is fraught with both long and short term
complications. Longer term complications also include dental
problems, cerebral palsy and feeding difficulties. The impact on the family is related to the degree of disability,
particularly as the baby grows bigger and becomes more difficult to manage. 46149

Previous Question Tag Question

Feedback Next Question

Difficulty: Average

Peer Responses

https://mypastest.pastest.com/Secure/TestMe/Browser/436619[‫ ص‬06:21:03 11/12/1437]


MyPastest

Session Progress

Responses Correct: 0

Responses Incorrect: 44

Responses Total: 44

Responses - % Correct: 0%

Blog
About Pastest
Contact Us
Help

© Pastest 2016

https://mypastest.pastest.com/Secure/TestMe/Browser/436619[‫ ص‬06:21:03 11/12/1437]


MyPastest

Prefer to use the old MyPastest? Access it here »


End Session

Back to Filters

Question 43 of 71

You’re discussing retinopathy of prematurity to some parents. You explain that blood vessels grow out of the optic
disc but don’t reach the peripheral retina until after birth.

At what age do vessels reach the peripheral retina?

A 1 month

B 2 months

C 3 months

D 4 months

E 5 months

Explanation
Theunvascularised retina is highly sensitive to oxygen damage – particularly in premature babies. It is rare in the
UK and unless the baby is very premature. In addition to prematurity
and low birthweight, oxygen therapy is also
a risk factor given vessels
require relative hypoxia to develop. 46150

Previous Question
Next Question
Tag Question

Feedback

Difficulty: Average

https://mypastest.pastest.com/Secure/TestMe/Browser/436619[‫ ص‬06:21:26 11/12/1437]


MyPastest

Peer Responses

Session Progress

Responses Correct: 0

Responses Incorrect: 45

Responses Total: 45

Responses - % Correct: 0%

Blog
About Pastest
Contact Us
Help

© Pastest 2016

https://mypastest.pastest.com/Secure/TestMe/Browser/436619[‫ ص‬06:21:26 11/12/1437]


MyPastest

Prefer to use the old MyPastest? Access it here »


End Session

Back to Filters

Question 44 of 71

Which one of the following conditions is not considered to be a risk factor for developing bronchopulmonary
dysplasia (BPD)?

A Female gender

B Intra-uterine growth retardation

C Maternal smoking

D Prematurity

E Prolonged oxygenation

Explanation
Prematurity, particularly children born before 32 weeks are susceptible to alveolar damage and BPD. Males are
more commonly affected than females by virtue of more male premature births than female. BPD is poorly
understood - it is thought that prolonged artificial oxygenation at high concentration plays a part it's the
pathophysiology. 46151

Previous Question
Next Question Tag Question

Feedback

Difficulty: Average

https://mypastest.pastest.com/Secure/TestMe/Browser/436619[‫ ص‬06:21:49 11/12/1437]


MyPastest

Peer Responses

Session Progress

Responses Correct: 0

Responses Incorrect: 46

Responses Total: 46

Responses - % Correct: 0%

Blog
About Pastest
Contact Us
Help

© Pastest 2016

https://mypastest.pastest.com/Secure/TestMe/Browser/436619[‫ ص‬06:21:49 11/12/1437]


MyPastest

Prefer to use the old MyPastest? Access it here »


End Session

Back to Filters

Question 45 of 71

Which one of the following are potential complications of a strawberry naevus?

A Amblyopia

B Bleeding

C Cardiac failure

D Consumptive coagulopathy

E All of the above

Explanation
Strawberry naevi are common childhood lesions with a
number of potential complications including bleeding,
high output heart
failure, consumptive coagulopathy and obstruction eg airway, eyesight. 46152

Tag Question
Previous Question
Feedback
Next Question
Difficulty: Average

Peer Responses

https://mypastest.pastest.com/Secure/TestMe/Browser/436619[‫ ص‬06:22:11 11/12/1437]


MyPastest

Session Progress

Responses Correct: 0

Responses Incorrect: 47

Responses Total: 47

Responses - % Correct: 0%

Blog
About Pastest
Contact Us
Help

© Pastest 2016

https://mypastest.pastest.com/Secure/TestMe/Browser/436619[‫ ص‬06:22:11 11/12/1437]


MyPastest

Prefer to use the old MyPastest? Access it here »


End Session

Back to Filters

Question 46 of 71

A newborn is noted to have several strawberry naevi. You're concerned about internal haemangiomas.

How many naevi should warrant further investigation?

A More than 1

B More than 2

C More than 3

D More than 4

E More than 5

Explanation
Strawberry naevi are common in childhood. Around a third are noted at birth. More than five lesions should raise
suspicion of internal lesions e.g liver 46153

Previous Question Tag Question

Feedback
Next Question

Difficulty: Average

Peer Responses

https://mypastest.pastest.com/Secure/TestMe/Browser/436619[‫ ص‬06:22:35 11/12/1437]


MyPastest

Session Progress

Responses Correct: 0

Responses Incorrect: 48

Responses Total: 48

Responses - % Correct: 0%

Blog
About Pastest
Contact Us
Help

© Pastest 2016

https://mypastest.pastest.com/Secure/TestMe/Browser/436619[‫ ص‬06:22:35 11/12/1437]


MyPastest

Prefer to use the old MyPastest? Access it here »


End Session

Back to Filters

Question 47 of 71

Mongolian blue spots are least likely to be found in which ethnic background?

A African

B Asian

C Mediterranean

D Middle Eastern

E White European

Explanation
Mongolion blue spots are usually found on the back and are uniform in colour. Most tend to disappear by puberty.
46154

Tag Question

Feedback
Previous Question
Next Question
Difficulty: Average

Peer Responses

https://mypastest.pastest.com/Secure/TestMe/Browser/436619[‫ ص‬06:22:56 11/12/1437]


MyPastest

Session Progress

Responses Correct: 0

Responses Incorrect: 49

Responses Total: 49

Responses - % Correct: 0%

Blog
About Pastest
Contact Us
Help

© Pastest 2016

https://mypastest.pastest.com/Secure/TestMe/Browser/436619[‫ ص‬06:22:56 11/12/1437]


MyPastest

Prefer to use the old MyPastest? Access it here »


End Session

Back to Filters

Question 48 of 71

Which one of the following statements is true regarding the epidemiology of developmental dysplasia of the hip
(DDH)?

A DDH is equally as common in females and males and can affect either hip in the same proportion

B DDH is more common in females and predominantly affects the left hip

C DDH is more common in females and predominantly affects the right hip

D DDH is more common in males and predominantly affects the left hip

E DDH is more common in males and predominantly affects the right hip

Explanation
DDH has an incidence of 2:1000 live births and is
more common in females by a ratio of 6:1 F:M. The left hip is
more likely to be affected than the right. Both hips are affected in a third of cases.

Ref: Oxford handbook of clinical specialities


46155

Previous Question
Next Question Tag Question

Feedback

Difficulty: Average

https://mypastest.pastest.com/Secure/TestMe/Browser/436619[‫ ص‬06:23:17 11/12/1437]


MyPastest

Peer Responses

Session Progress

Responses Correct: 0

Responses Incorrect: 50

Responses Total: 50

Responses - % Correct: 0%

Blog
About Pastest
Contact Us
Help

© Pastest 2016

https://mypastest.pastest.com/Secure/TestMe/Browser/436619[‫ ص‬06:23:17 11/12/1437]


MyPastest

Prefer to use the old MyPastest? Access it here »


End Session

Back to Filters

Question 49 of 71

Risk factors for developmental dysplasia of the hip (DDH) includes which one of the following?

A Breech presentation

B Multiparity

C Polyhydramnios

D Prematurity

E Small for dates baby

Explanation
Risks for DDH include female gender, breech presentation, oligohydramnios,high birth weight, primiparity and
term plus babies. 46156

Tag Question
Previous Question
Feedback
Next Question
Difficulty: Average

Peer Responses

https://mypastest.pastest.com/Secure/TestMe/Browser/436619[‫ ص‬06:23:38 11/12/1437]


MyPastest

Session Progress

Responses Correct: 0

Responses Incorrect: 51

Responses Total: 51

Responses - % Correct: 0%

Blog
About Pastest
Contact Us
Help

© Pastest 2016

https://mypastest.pastest.com/Secure/TestMe/Browser/436619[‫ ص‬06:23:38 11/12/1437]


MyPastest

Prefer to use the old MyPastest? Access it here »


End Session

Back to Filters

Question 50 of 71

What is the imaging modality of choice to assess for the presence of developmental dysplasia of the hip (DDH) in
a baby born breech?

A Bone scan

B CT

C MRI

D USS

E X-Ray

Explanation
Ultrasound is the imaging modality of choice to look for DDH. Units screen high risk children e.g. Breech 46157

Tag Question
Previous Question
Feedback
Next Question
Difficulty: Average

Peer Responses

https://mypastest.pastest.com/Secure/TestMe/Browser/436619[‫ ص‬06:24:00 11/12/1437]


MyPastest

Session Progress

Responses Correct: 0

Responses Incorrect: 52

Responses Total: 52

Responses - % Correct: 0%

Blog
About Pastest
Contact Us
Help

© Pastest 2016

https://mypastest.pastest.com/Secure/TestMe/Browser/436619[‫ ص‬06:24:00 11/12/1437]


MyPastest

Prefer to use the old MyPastest? Access it here »


End Session

Back to Filters

Question 51 of 71

You're speaking to some parents about their 9 month old son who had a late diagnosis of developmental dysplasia
of the hip (DDH) after apparently normal screening tests.

When discussing the condition you mention that DDH has what incidence?

A 1 in 1000 live births

B 2 in 1000 live births

C 3 in 1000 live births

D 4 in 1000 live births

E 5 in 1000 live births

Explanation
DDH has an incidence of 2 in 1000 live births. It may be missed by routine newborn and 6 week screening
examinations.
46158

Previous Question
Next Question Tag Question

Feedback

Difficulty: Average

https://mypastest.pastest.com/Secure/TestMe/Browser/436619[‫ ص‬06:24:22 11/12/1437]


MyPastest

Peer Responses

Session Progress

Responses Correct: 0

Responses Incorrect: 53

Responses Total: 53

Responses - % Correct: 0%

Blog
About Pastest
Contact Us
Help

© Pastest 2016

https://mypastest.pastest.com/Secure/TestMe/Browser/436619[‫ ص‬06:24:22 11/12/1437]


MyPastest

Prefer to use the old MyPastest? Access it here »

Back to Filters

Question 52 of 71

A baby boy weighing 4.9kg is admitted to the neonatal unit after a difficult delivery and an extended resuscitation.
You are called
as he is increasingly unstable with petechial bleeding under the skin on his legs and oozing from
around the umbilicus. His clotting results show prolonged PT, APPT and thrombin time.

What is the most likely diagnosis?

A Beckwith-Wiedemann Syndrome

B Disseminated Intravascular Coagulation

C Haemorrhagic Disease of the Newborn (HDN)

D Meningococcal septicaemia

E Severe neonatal thrombocytopaenia

Explanation
This baby is most likely to have DIC triggered by
severe birth asphyxia secondary to his large size and difficult
delivery. DIC usually presents with petechiae and oozing from puncture
sites but bleeding can be elsewhere.
Thrombin formation triggered by microbial endotoxins or thromboplastin release from damaged tissues and
endothelial cells causes intravascular coagulation. This consumes platelets, factors II, VIII, XIII and fibrinogen.
The fibrinolytic system is also activated and the fibrin degradation products further exacerbate the bleeding. The
PT, APTT and thrombin time are all increased because of the consumption of intrinsic coagulation factors and
fibrinogen. Platelets are also usually reduced and red blood cells distorted.

Beckwith-Wiedemann Syndrome (BWS) should be considered in macrosomic babies with macroglossia,


visceromegaly, hypoglycaemia, ear crease pits and a number of other potential syndromic features. Children with
BWS may have clotting abnormalities if congenital heart disease co-exists. Polycythaemia is a neonatal feature.
This child may
need consideration for BWS but it is not the most likely cause of this acute problem.

HDN or Vitamin K deficiency bleeding can be severe due to deficiency of vitamin K dependent clotting factors II,

https://mypastest.pastest.com/Secure/TestMe/Browser/436619[‫ ص‬06:24:46 11/12/1437]


MyPastest

VII, IX and X. There is a chance vitamin K can be forgotten in a busy resuscitation environment. The classical
End
form presents on day 2-4 of life with umbilical stump haemorrhage and bleeding Session
elsewhere. PT and APTT are
very prolonged but thrombin time and fibrinogen levels are normal - hence HDN is not the right answer.

Meningococcal septicaemia and any severe sepsis may lead to DIC and the presentation of this baby. The history
is not suggestive that an infective process is the most likely underlying cause of his acute problem. He should of
course be screened for infection and treated accordingly.

Severe neonatal thrombocytopaeniamay present with purpura, cephalhaematoma and bruising. Internal organ and
puncture site bleeding is rare in otherwise well term neonates (likely to have neonatal
alloimmunethrombocytopaenia or maternal ITP). There are a range of causes in the unwell child including
congenital infection, giant haemangiomas (Kasabach-Merritt syndrome), marrow abnormalities, drugs, organic
acidaemias etc. In this baby the overall picture is best represented by DIC rather than thrombocytopaenia alone.
46197

Tag Question

Feedback

Difficulty: Average

Peer Responses

Session Progress

Responses Correct: 0

Responses Incorrect: 54
Previous Question
Responses Total: 54

Responses - % Correct:
Next Question 0%

Blog
About Pastest
Contact Us
Help

https://mypastest.pastest.com/Secure/TestMe/Browser/436619[‫ ص‬06:24:46 11/12/1437]


MyPastest

© Pastest 2016

https://mypastest.pastest.com/Secure/TestMe/Browser/436619[‫ ص‬06:24:46 11/12/1437]


MyPastest

Prefer to use the old MyPastest? Access it here »

Back to Filters

Question 53 of 71

What post-birth event encourages closure of the ductus venosus in a newborn baby?

A First breath and fall in pulmonary vascular resistance

B First breast feed and receipt of colostrum

C Increased pulmonary blood flow

D Sharp rise in the partial pressure of arterial oxygen (PaO2)

E Umbilical cord clamping and separation from mother

Explanation
The ductus venosus channels some of the returning
oxygenated blood into the umbilical vein from the placenta
through the liver, into the inferior vena cava (IVC) and then the right atrium. Once the umbilical cord is clamped
and the connection to the placenta is
severed the closure of the ductus venosus is triggered. Closure is complete in
76% newborns by day 7 of life). The flow in the IVC then falls and systemic vascular resistance rises.

Increased pulmonary blood flow supports closure of the foramen ovale by raising the left atrial pressure.

The ductus arteriosus closes secondary to the rise in PaO2 and falling prostaglandin levels in circulation.

The other events are all important and physiologically normal but they do not directly trigger closure of the ductus
venosus or otherPrevious
ducts. Question
46198
Next Question

Tag Question

https://mypastest.pastest.com/Secure/TestMe/Browser/436619[‫ ص‬06:25:07 11/12/1437]


MyPastest

Feedback End Session

Difficulty: Average

Peer Responses

Session Progress

Responses Correct: 0

Responses Incorrect: 55

Responses Total: 55

Responses - % Correct: 0%

Blog
About Pastest
Contact Us
Help

© Pastest 2016

https://mypastest.pastest.com/Secure/TestMe/Browser/436619[‫ ص‬06:25:07 11/12/1437]


MyPastest

Prefer to use the old MyPastest? Access it here »


End Session

Back to Filters

Question 54 of 71

Which one of the following conditions is breast feeding considered to confer a reduced risk for the baby?

A Diabetes

B Gastroenteritis

C Lower respiratory tract infections

D Sudden infant death syndrome

E All of the above

Explanation
Breastfeeding has significant health benefits for both the mother and baby. Younger mothers are less likely to
breast feed which may put them and their babies at risk. Education is therefore
an essential component of care and
should include the advantages of breast feeding. 47133

Previous Question Tag Question

Feedback Next Question

Difficulty: Average

Peer Responses

https://mypastest.pastest.com/Secure/TestMe/Browser/436619[‫ ص‬06:25:28 11/12/1437]


MyPastest

Session Progress

Responses Correct: 0

Responses Incorrect: 56

Responses Total: 56

Responses - % Correct: 0%

Blog
About Pastest
Contact Us
Help

© Pastest 2016

https://mypastest.pastest.com/Secure/TestMe/Browser/436619[‫ ص‬06:25:28 11/12/1437]


MyPastest

Prefer to use the old MyPastest? Access it here »


End Session

Back to Filters

Question 55 of 71

Which of the following conditions does breast feeding reduce the risk of in the mother?

A Breast cancer

B Cardiovascular disease

C Diabetes

D Ovarian cancer

E All of the above

Explanation
Breast feeding offers health advantages to both the mother and child. The risk of premenopausal breast cancer is
reduced in breast feeding mothers. Type 2
diabetes mellitus as well as metabolic syndrome are also reduced. It's
essential health care professionals educate mothers accordingly but equally don't make mothers feel guilty if they
are unable to breast feed. 47134

Previous Question
Tag Question
Next Question
Feedback

Difficulty: Average

Peer Responses

https://mypastest.pastest.com/Secure/TestMe/Browser/436619[‫ ص‬06:25:50 11/12/1437]


MyPastest

Session Progress

Responses Correct: 0

Responses Incorrect: 57

Responses Total: 57

Responses - % Correct: 0%

Blog
About Pastest
Contact Us
Help

© Pastest 2016

https://mypastest.pastest.com/Secure/TestMe/Browser/436619[‫ ص‬06:25:50 11/12/1437]


MyPastest

Prefer to use the old MyPastest? Access it here »


End Session

Back to Filters

Question 56 of 71

Which one of the following infections is NOT transmitted through breast milk?

A Borrelia burgdorferi

B Haemophilus influenzae

C Hepatitis B

D Human immunodeficiency virus

E Mycobacterium tuberculosis

Explanation
HIV may be transmitted via breast milk and antiretroviral drugs should be offered. Guidelines differ and the UK
suggests avoid breast feeding.

Borrelia burgdorferi causes Lyme disease in case you've forgotten!

Other infectious diseases where breast feeding should be avoided include group B strep and Neisseria
gonorrhoeae.
47135

Previous Question
Next Question
Tag Question

Feedback

https://mypastest.pastest.com/Secure/TestMe/Browser/436619[‫ ص‬06:26:17 11/12/1437]


MyPastest

Difficulty: Average

Peer Responses

Session Progress

Responses Correct: 0

Responses Incorrect: 58

Responses Total: 58

Responses - % Correct: 0%

Blog
About Pastest
Contact Us
Help

© Pastest 2016

https://mypastest.pastest.com/Secure/TestMe/Browser/436619[‫ ص‬06:26:17 11/12/1437]


MyPastest

Prefer to use the old MyPastest? Access it here »


End Session

Back to Filters

Question 57 of 71

What is the most likely infectious agent implicated in mastitis?

A Group B streptococcus

B Haemophilus influenzae

C Mycobacterium tuberculosis

D Neisseria gonorrhoeae

E Staphylococcus aureus

Explanation
Staphylococcus aureus is the most likely infectious agent associated with mastitis. Treatment is antibiotics. Breast
feeding
(or pumping) should be continued. Abscesses are a complication and are more likely in smokers. 47136

Tag Question
Previous Question
Feedback
Next Question
Difficulty: Average

Peer Responses

https://mypastest.pastest.com/Secure/TestMe/Browser/436619[‫ ص‬06:26:37 11/12/1437]


MyPastest

Session Progress

Responses Correct: 0

Responses Incorrect: 59

Responses Total: 59

Responses - % Correct: 0%

Blog
About Pastest
Contact Us
Help

© Pastest 2016

https://mypastest.pastest.com/Secure/TestMe/Browser/436619[‫ ص‬06:26:37 11/12/1437]


MyPastest

Prefer to use the old MyPastest? Access it here »


End Session

Back to Filters

Question 58 of 71

By how much is successful breast feeding reduced in women who have breast augmentation?

A 10%

B 15%

C 20%

D 25%

E 30%

Explanation
Breast feeding may be significantly impaired by breast augmentation and women need to made aware of this.
Equally, breast engorgement which occurs due to vascular congestion reduces nipple protrusion and subsequent
ability to breast feed successfully. In
the latter, regular feeding or expressing is required 47137

Previous Question Tag Question

Feedback Next Question

Difficulty: Average

Peer Responses

https://mypastest.pastest.com/Secure/TestMe/Browser/436619[‫ ص‬06:26:57 11/12/1437]


MyPastest

Session Progress

Responses Correct: 0

Responses Incorrect: 60

Responses Total: 60

Responses - % Correct: 0%

Blog
About Pastest
Contact Us
Help

© Pastest 2016

https://mypastest.pastest.com/Secure/TestMe/Browser/436619[‫ ص‬06:26:57 11/12/1437]


MyPastest

Prefer to use the old MyPastest? Access it here »


End Session

Back to Filters

Question 59 of 71

From what age is it acceptable to offer a child cow's milk?

A 4 months

B 6 months

C 8 months

D 10 months

E 12 months

Explanation
Cow's milk should not be offered to infants below one year of age. This is because the sodium content is too high
and the iron concentration is too low. Protein levels and cow's milk also not appropriate for children below one
year of age. 47138

Previous Question Tag Question

Feedback Next Question

Difficulty: Average

Peer Responses

https://mypastest.pastest.com/Secure/TestMe/Browser/436619[‫ ص‬06:27:16 11/12/1437]


MyPastest

Session Progress

Responses Correct: 0

Responses Incorrect: 61

Responses Total: 61

Responses - % Correct: 0%

Blog
About Pastest
Contact Us
Help

© Pastest 2016

https://mypastest.pastest.com/Secure/TestMe/Browser/436619[‫ ص‬06:27:16 11/12/1437]


MyPastest

Prefer to use the old MyPastest? Access it here »


End Session

Back to Filters

Question 60 of 71

Which one of the following statements is correct regarding the definition of neonatal deaths?

A Neonatal deaths occur only up to the first 24 hours of life

B Neonatal deaths occur only up to the first 72 hours of life

C Neonatal deaths occur only up to the first 7 days of life

D Neonatal deaths occur only up to the first 21 days of life

E Neonatal deaths occur only up to the first 28 days of life

Explanation
Neonatal deaths occur within the first 28 days of life whereas perinatal deaths are stillbirths plus deaths under 7
days.

Stillbirths are classified as in utero death after 24 weeks’ gestation.


47139

Previous Question
Tag Question
Next Question
Feedback

Difficulty: Average

Peer Responses

https://mypastest.pastest.com/Secure/TestMe/Browser/436619[‫ ص‬06:27:36 11/12/1437]


MyPastest

Session Progress

Responses Correct: 0

Responses Incorrect: 62

Responses Total: 62

Responses - % Correct: 0%

Blog
About Pastest
Contact Us
Help

© Pastest 2016

https://mypastest.pastest.com/Secure/TestMe/Browser/436619[‫ ص‬06:27:36 11/12/1437]


MyPastest

Prefer to use the old MyPastest? Access it here »


End Session

Back to Filters

Question 61 of 71

Which one of the following is the biggest risk factor for stillbirth?

A Fetal growth restriction

B Maternal age

C Maternal obesity

D Pre-eclampsia

E Prematurity

Explanation
Growth restriction is critically important to diagnose antenatally in order for the cause to be found and managed,
and
for close monitoring. Where scans are missed, restriction continues undiagnosed increasing the risk of death.
This is a significant public health issue and active management is required to educate women. 47140

Previous Question Tag Question

Feedback Next Question

Difficulty: Average

Peer Responses

https://mypastest.pastest.com/Secure/TestMe/Browser/436619[‫ ص‬06:27:57 11/12/1437]


MyPastest

Session Progress

Responses Correct: 0

Responses Incorrect: 63

Responses Total: 63

Responses - % Correct: 0%

Blog
About Pastest
Contact Us
Help

© Pastest 2016

https://mypastest.pastest.com/Secure/TestMe/Browser/436619[‫ ص‬06:27:57 11/12/1437]


MyPastest

Prefer to use the old MyPastest? Access it here »

Back to Filters

Question 62 of 71

A caucasian mother gives birth to her second child; a baby girl. Her partner is unemployed and their income is
wholly derived from
state benefits.

Which of the following is a risk factor for neonatal death and still birth?

A Baby of female gender

B Caucasian ethnicity

C Multiparity

D Parental unemployment

E Singleton pregnancy

Explanation
Perinatal and neonatal death is multifactorial covering social and medical factors, and maternal and fetal factors.
In this scenario, low socio-economic status is a risk factor.

Other risk factors include:

African origin
Male gender infant
Nulliparity
Non-singleton pregnancies
Multi-parity (third and subsequent pregnancies)

A good article can be found at http://patient.info/doctor/stillbirth-and-neonatal-death


47141

https://mypastest.pastest.com/Secure/TestMe/Browser/436619[‫ ص‬06:28:17 11/12/1437]


MyPastest

End Session

Tag Question

Feedback

Difficulty: Average

Peer Responses

Session Progress

Responses Correct: 0

Responses Incorrect: 64

Responses Total: 64

Responses - % Correct: 0%

Blog
About Pastest
Contact Us
Help

© Pastest 2016

Previous Question
Next Question

https://mypastest.pastest.com/Secure/TestMe/Browser/436619[‫ ص‬06:28:17 11/12/1437]


MyPastest

Prefer to use the old MyPastest? Access it here »


End Session

Back to Filters

Question 63 of 71

What is the hazard ratio for neonatal death in mothers aged 40 or over compared to those aged 25-29?

A 1.3

B 1.9

C 2.3

D 2.9

E 3.3

Explanation
Maternal age is a risk factor neonatal death. The
hazard ratios for stillbirth with respect to maternal age are even
more
dramatic. Several other risk factors are at play including socio-economic factors, parity, maternal health and
lifestyle, and fetal
abnormalities.

http://patient.info/doctor/stillbirth-and-neonatal-death
47142

Previous Question
Next Question Tag Question

Feedback

Difficulty: Average

https://mypastest.pastest.com/Secure/TestMe/Browser/436619[‫ ص‬06:28:37 11/12/1437]


MyPastest

Peer Responses

Session Progress

Responses Correct: 0

Responses Incorrect: 65

Responses Total: 65

Responses - % Correct: 0%

Blog
About Pastest
Contact Us
Help

© Pastest 2016

https://mypastest.pastest.com/Secure/TestMe/Browser/436619[‫ ص‬06:28:37 11/12/1437]


MyPastest

Prefer to use the old MyPastest? Access it here »


End Session

Back to Filters

Question 64 of 71

A refugee newly arrived in the UK is booked into clinic for her first antenatal appointment. She believes she is
approximately 12 weeks pregnant.

An initial antenatal booking appointment beyond how many weeks, confers an increased risk of stillbirth?

A 10

B 11

C 12

D 13

E 14

Explanation
Education regarding pre-pregnancy health and appropriate antenatal care is essential to minimising risk of still
birth and neonatal death. Late booking increases still birth risk. Maternal age, socio-economic factors, parity,
maternal health and lifestyle and fetal abnormalities all play a part in risk of death, many
of which are preventable.

http://patient.info/doctor/stillbirth-and-neonatal-death
47143

Previous Question
Next Question

Tag Question

Feedback

https://mypastest.pastest.com/Secure/TestMe/Browser/436619[‫ ص‬06:29:04 11/12/1437]


MyPastest

Difficulty: Average

Peer Responses

Session Progress

Responses Correct: 0

Responses Incorrect: 66

Responses Total: 66

Responses - % Correct: 0%

Blog
About Pastest
Contact Us
Help

© Pastest 2016

https://mypastest.pastest.com/Secure/TestMe/Browser/436619[‫ ص‬06:29:04 11/12/1437]


MyPastest

Prefer to use the old MyPastest? Access it here »

Back to Filters

Question 65 of 71

A 43-year-old first time mother gives birth to a still born child at 28 weeks’ gestation. A congenital abnormality
had been detected however the mother, after consideration of the situation, decided to continue with the
pregnancy.

What proportion of stillbirths are attributable to congenital abnormalities?

A Less than 2%

B Less than 5%

C Less than 10%

D Less than 15%

E Less than 20%

Explanation
A myriad of risk factors contribute to stillbirth, particularly restricted growth. Congenital factors account for less
than 10% of cases. Education regarding pre-pregnancy health and
appropriate antenatal care is essential to
minimising risk of still birth and neonatal death. Late antenatal booking after 13 weeks gestation increases still
birth risk. Maternal age, socio-economic factors, parity, and maternal health and lifestyle all play a part in risk of
death, many of which are preventable.

http://patient.info/doctor/stillbirth-and-neonatal-death
Next Question 47144

Tag Question

https://mypastest.pastest.com/Secure/TestMe/Browser/436619[‫ ص‬06:29:34 11/12/1437]


MyPastest

End Session
Feedback

Difficulty: Average

Peer Responses

Session Progress

Responses Correct: 0

Responses Incorrect: 67

Responses Total: 67

Responses - % Correct: 0%

Blog
About Pastest
Contact Us
Help

© Pastest 2016

Previous Question

https://mypastest.pastest.com/Secure/TestMe/Browser/436619[‫ ص‬06:29:34 11/12/1437]


MyPastest

Prefer to use the old MyPastest? Access it here »

Back to Filters

Question 66 of 71

According to NICE guidelines, which of the following factors pose an increased risk of severe
hyperbilirubinaemia?

A Bottle feeding

B Gestation < 34 weeks

C Gestational chicken pox infection at 30 weeks

D History of a previous sibling requiring phototherapy for jaundice

E Jaundice within 48 hours

Explanation
The history of a previous sibling requiring phototherapy for jaundice should alert you to the possibility of severe
hyperbilirubinaemia. This is also more likely to be seen in babies who are exclusively breastfed and in those born
before 38 weeks. Jaundice seen within the first 24 hours of birth is also considered to be a risk factor.

Babies of mothers who develop chickenpox within 5 days before, to 2 days after, delivery will require zoster
immunoglobulin (ZIG), If the baby develops chickenpox then treatment is with intravenous aciclovir. This is not a
factor suggestive of an increased risk of severe hyperbilirubinaemia.

https://www.nice.org.uk/guidance/cg98/chapter/1-Guidance
Previous Question
47145

Next Question

Tag Question

https://mypastest.pastest.com/Secure/TestMe/Browser/436619[‫ ص‬06:30:02 11/12/1437]


MyPastest

Feedback End Session

Difficulty: Average

Peer Responses

Session Progress

Responses Correct: 0

Responses Incorrect: 68

Responses Total: 68

Responses - % Correct: 0%

Blog
About Pastest
Contact Us
Help

© Pastest 2016

https://mypastest.pastest.com/Secure/TestMe/Browser/436619[‫ ص‬06:30:02 11/12/1437]


MyPastest

Prefer to use the old MyPastest? Access it here »


End Session

Back to Filters

Question 67 of 71

You have a baby less than 24 hours old on the neonatal ward with suspected jaundice.

According
to NICE guidelines, the serum bilirubin level should be measured within
how many hours in all babies
with suspected or obvious jaundice within the first 24 hours of life?

A 2

B 4

C 6

D 8

E 10

Explanation
Children in this category should have a bilirubin
checked within 2 hours and then every 6 hours until the level is
below treatment threshold AND the levels are either stable or reducing.

For further information see: https://www.nice.org.uk/guidance/cg98/chapter/1-Guidance


47146

Previous Question
Next Question
Tag Question

Feedback

https://mypastest.pastest.com/Secure/TestMe/Browser/436619[‫ ص‬06:30:25 11/12/1437]


MyPastest

Difficulty: Average

Peer Responses

Session Progress

Responses Correct: 0

Responses Incorrect: 69

Responses Total: 69

Responses - % Correct: 0%

Blog
About Pastest
Contact Us
Help

© Pastest 2016

https://mypastest.pastest.com/Secure/TestMe/Browser/436619[‫ ص‬06:30:25 11/12/1437]


MyPastest

Prefer to use the old MyPastest? Access it here »

Back to Filters

Question 68 of 71

You are asked to measure the bilirubin level of an infant over the postnatal age of 24 hours and intend using a
transcutaneous bilirubinometer.

According to NICE, a transcutaneous bilirubinometer should only be used in babies born at which of the following
gestational ages?

A 31 weeks or more

B 32 weeks or more

C 33 weeks or more

D 34 weeks or more

E 35 weeks or more

Explanation
It is essential that doctors understand the limitations of the tests we perform. That includes when we should and
shouldn't use a test as well as an understanding of the sensitivity, specificity, positive and negative predictive
values. A transcutaneous
bilirubinometer is only recommended for use in babies with a gestational age of 35
weeks or more and a postnatal age of over 24 hours. It is a device that uses reflected light to measure the yellow
colour (bilirubin level) in the skin.

https://www.nice.org.uk/guidance/cg98/chapter/1-Guidance
Next Question 47147

Tag Question

https://mypastest.pastest.com/Secure/TestMe/Browser/436619[‫ ص‬06:30:45 11/12/1437]


MyPastest

End Session
Feedback

Difficulty: Average

Peer Responses

Session Progress

Responses Correct: 0

Responses Incorrect: 70

Responses Total: 70

Responses - % Correct: 0%

Blog
About Pastest
Contact Us
Help

© Pastest 2016

Previous Question

https://mypastest.pastest.com/Secure/TestMe/Browser/436619[‫ ص‬06:30:45 11/12/1437]


MyPastest

Prefer to use the old MyPastest? Access it here »

Back to Filters

Question 69 of 71

You identify a baby on the ward with hyperbilirubinaemia as being at increased risk of developing kernicterus.

According to NICE, what rate of increase in bilirubin concentration is a risk factor for developing kernicterus?

A Above 5.5 micromol/litre per hour

B Above 6.5 micromol/litre per hour

C Above 7.5 micromol/litre per hour

D Above 8.5 micromol/litre per hour

E Above 9.5 micromol/litre per hour

Explanation
Kernicterus is a rare condition whereby dangerous
levels of bilirubin damage the central nervous system.
Symptoms include
reduced consciousness, floppiness, back arching, seizures and death. Long term sequelae
include cerebral palsy and hearing loss.

Risk factors for kernicterus include any of the following:

A serum bilirubin level >340 umol/l in babies with a gestational age ≥37 weeks
A rapidly rising bilirubin level >8.5 umol/l per hour
Clinical features of acute bilirubin encephalopathy
Next Question
Further information can be found via this link: https://www.nice.org.uk/guidance/cg98/chapter/1-Guidance
47148

https://mypastest.pastest.com/Secure/TestMe/Browser/436619[‫ ص‬06:31:07 11/12/1437]


MyPastest

End Session
Tag Question

Feedback

Difficulty: Average

Peer Responses

Session Progress

Responses Correct: 0

Responses Incorrect: 71

Responses Total: 71

Responses - % Correct: 0%

Blog
About Pastest
Contact Us
Help

© Pastest 2016

Previous Question

https://mypastest.pastest.com/Secure/TestMe/Browser/436619[‫ ص‬06:31:07 11/12/1437]


MyPastest

Prefer to use the old MyPastest? Access it here »

Back to Filters

Question 70 of 71

A neonate of Asian parents presents with jaundice appearing less than 24 hours after birth and you are concerned
about glucose-6-phosphate dehydrogenase deficiency.

What is the mode of inheritance of this condition?

A Autosomal dominant

B Autosomal recessive

C Mitochondrial

D X-linked

E None of the above

Explanation
Glucose-6-phosphate dehydrogenase deficiency causes reduced red cell glutathione levels which in turn, reduces
protection against oxidative stress and haemolysis. It may present with neonatal jaundice. If suspected then in
order to assess the baby for underlying disease a blood glucose-6-phosphate dehydrogenase level test may be
clinically indicated.

https://www.nice.org.uk/guidance/cg98/chapter/1-Guidance
47149
Previous Question
Next Question

Tag Question

Feedback

https://mypastest.pastest.com/Secure/TestMe/Browser/436619[‫ ص‬06:31:29 11/12/1437]


MyPastest

End Session

Difficulty: Average

Peer Responses

Session Progress

Responses Correct: 0

Responses Incorrect: 72

Responses Total: 72

Responses - % Correct: 0%

Blog
About Pastest
Contact Us
Help

© Pastest 2016

https://mypastest.pastest.com/Secure/TestMe/Browser/436619[‫ ص‬06:31:29 11/12/1437]


MyPastest

Prefer to use the old MyPastest? Access it here »

Back to Filters

Question 71 of 71

Biliary atresia should be considered in term babies persistently jaundiced for over what period of time?

A 7 days

B 10 days

C 14 days

D 21 days

E 28 days

Explanation
Biliary atresia is a rare but serious condition. Around 50 cases occur every year in the UK. Symptoms include
obstructive
jaundice (dark urine and pale stool). Surgery within 8 weeks is recommended. Survival is around 90%
using current treatment regimes. Without treatment children will survive to around 18 months. 47150

End Session

Previous Question Tag Question

Feedback

Difficulty: Average

Peer Responses

https://mypastest.pastest.com/Secure/TestMe/Browser/436619[‫ ص‬06:31:51 11/12/1437]


MyPastest

Session Progress

Responses Correct: 0

Responses Incorrect: 73

Responses Total: 73

Responses - % Correct: 0%

Blog
About Pastest
Contact Us
Help

© Pastest 2016

https://mypastest.pastest.com/Secure/TestMe/Browser/436619[‫ ص‬06:31:51 11/12/1437]


MyPastest

Prefer to use the old MyPastest? Access it here »

Next Question

Back to Filters

Question 1 of 61

A 7-year-old boy presents with generalised oedema. Urinalysis shows marked albuminuria. Blood tests reveal
hypoalbuminaemia
and hyperlipidaemia. A renal biopsy appears normal on light microscopy.

What would be the most likely finding on electron microscopy?

A Deposition of electron-dense material on the capillary basement membrane

B Splitting of the capillary basement membrane

C Fusion of foot processes of the glomerular epithelial cells

D Thinning of the capillary basement membrane

E Fibrils of amyloid protein in the mesangium

Explanation
The most common cause of nephrotic syndrome in children is minimal-change nephropathy. Loss of foot
processes may be seen in other proteinuric states such as membranous glomerulonephritis and diabetic
nephropathy, but light microscopic changes would also be evident in these conditions. Splitting of the capillary
basement membrane is seen in mesangiocapillary glomerulonephritis, while thinning is noted in thin glomerular
basement membrane disease.
12819

Tag Question

https://mypastest.pastest.com/Secure/TestMe/Browser/436619[‫ ص‬07:51:49 10/12/1437]


MyPastest

Feedback

Previous Question Difficulty: Average

Peer Responses

End Session

Session Progress

Responses Correct: 0

Responses Incorrect: 1

Responses Total: 1

Responses - % Correct: 0%

Blog
About Pastest
Contact Us
Help

© Pastest 2016

https://mypastest.pastest.com/Secure/TestMe/Browser/436619[‫ ص‬07:51:49 10/12/1437]


MyPastest

Prefer to use the old MyPastest? Access it here »

Back to Filters

Question 2 of 61

A 2-year-old boy has vitamin D-resistant rickets. Investigations show: serum calcium 2.6 mmol/l, phosphate 0.5
mmol/l and alkaline phosphatase 1040 U/l. Parathyroid hormone and bicarbonate levels are normal.

What is the most probable diagnosis?

A Distal renal tubular acidosis

B Hypophosphataemic rickets

C Vitamin D-dependent rickets

D Proximal renal tubular acidosis

E Hyperparathyroidism

Explanation

Alkaline Parathyroid
Disorder Serum calcium Serum phosphate
phosphatase hormone

Hypophosphataemic Normal or
Decreased Increased Normal
rickets decreased

Vitamin D-dependent Normal or


Decreased Increased Increased
rickets decreased

Hyperparathyroidism Increased Decreased Increased Increased

https://mypastest.pastest.com/Secure/TestMe/Browser/436619[‫ ص‬07:52:35 10/12/1437]


MyPastest

Nutritional rickets Decreased Decreased Increased Increased

Next Question
Proximal tubular acidosis also causes hypophosphataemic rickets but the bicarbonate level will be reduced. Distal
tubular acidosis causes vitamin D-dependent rickets.
Previous Question 12820

End Session

Tag Question

Feedback

Difficulty: Easy

Peer Responses

Session Progress

Responses Correct: 0

Responses Incorrect: 2

Responses Total: 2

Responses - % Correct: 0%

Blog
About Pastest
Contact Us
Help

© Pastest 2016

https://mypastest.pastest.com/Secure/TestMe/Browser/436619[‫ ص‬07:52:35 10/12/1437]


MyPastest

Prefer to use the old MyPastest? Access it here »

Next Question
Previous Question
Back to Filters

Question 3 of 61

A 6-week-old baby is vomiting excessively due to pyloric stenosis. What significant finding would you expect in a
blood test?

A Increased serum chloride levels

B Hypernatraemia

C Hypokalaemia

D Decreased serum bicarbonate level

E Plasma pH > 8.0

Explanation
The loss of gastric fluid rich in hydrochloric acid results in a metabolic alkalosis because bicarbonate generated
during the production of gastric acid returns to the circulation. Although sodium and potassium loss in the gastric
juice is variable, the obligate urinary loss of these cations is intensified by bicarbonaturia, which occurs during
disequilibrium. A plasma pH of 8.0 is incompatible with life. The mortality rate is 80% when the pH is greater
than 7.65.
12821

Tag Question

Feedback

https://mypastest.pastest.com/Secure/TestMe/Browser/436619[‫ ص‬07:52:56 10/12/1437]


MyPastest

Difficulty: Easy

Peer Responses

End Session

Session Progress

Responses Correct: 0

Responses Incorrect: 3

Responses Total: 3

Responses - % Correct: 0%

Blog
About Pastest
Contact Us
Help

© Pastest 2016

https://mypastest.pastest.com/Secure/TestMe/Browser/436619[‫ ص‬07:52:56 10/12/1437]


MyPastest

Prefer to use the old MyPastest? Access it here »

Next Question
Previous Question
Back to Filters

Question 4 of 61 End Session

A 1-year-old boy presents with a poor urinary stream since birth.

Which of the following investigations would be diagnostic in this case?

A Micturating cystourethrography

B Ultrasound of the bladder

C Excretion urography

D Uroflowmetry

E Computed tomography

Explanation
A poor urinary stream suggests a urinary tract obstruction (usually infravesical). The most common cause in a
male child is posterior urethral valves. The best diagnostic method is a
micturating cystourethrography. The other
option is endoscopy. Both these investigations would clearly show the site of obstruction.
12823

Tag Question

Feedback

Difficulty: Easy

https://mypastest.pastest.com/Secure/TestMe/Browser/436619[‫ ص‬07:53:15 10/12/1437]


MyPastest

Peer Responses

Session Progress

Responses Correct: 0

Responses Incorrect: 4

Responses Total: 4

Responses - % Correct: 0%

Blog
About Pastest
Contact Us
Help

© Pastest 2016

https://mypastest.pastest.com/Secure/TestMe/Browser/436619[‫ ص‬07:53:15 10/12/1437]


MyPastest

Prefer to use the old MyPastest? Access it here »

Next Question

Back to Filters

Question 5 of 61

A 4-year-old boy complains of abdominal pain and inability to pass urine in a good stream. An ultrasound scan of
the abdomen shows both kidneys are enlarged, irregular, and cystic. Further investigation shows that they are
dysplastic.

What is the most likely cause of this condition?

A Posterior urethral valve

B Bladder extrophy

C Pelvi-ureteric junction obstruction

D Recurrent urinary tract infection

E Vesicoureteric reflux

Explanation
Renal dysplasia occurs due to an abnormality in metanephric differentiation. It is characterised histologically by
the persistence in the kidney of abnormal structures such as cartilage, undifferentiated mesenchyme and immature
collecting ducts and by abnormal lobar organisation. Renal dysplasia is usually associated with posterior
urethral
valves. The latter should always be considered in a boy with poor urinary stream or a thick-walled trabeculated
bladder on ultrasound.
12824

Tag Question

https://mypastest.pastest.com/Secure/TestMe/Browser/436619[‫ ص‬07:53:52 10/12/1437]


MyPastest

Feedback

Previous Question Difficulty: Average

Peer Responses

End Session

Session Progress

Responses Correct: 0

Responses Incorrect: 5

Responses Total: 5

Responses - % Correct: 0%

Blog
About Pastest
Contact Us
Help

© Pastest 2016

https://mypastest.pastest.com/Secure/TestMe/Browser/436619[‫ ص‬07:53:52 10/12/1437]


MyPastest

Prefer to use the old MyPastest? Access it here »

Back to Filters

Question 6 of 61

A 3-year-old child is brought to the clinic with a history of lethargy, failure to thrive, excessive thirst and constant
bed-wetting. A blood test shows hypokalaemia and metabolic alkalosis. There is hyperplasia of the
juxtaglomerular apparatus seen on renal biopsy.

What is the most probable diagnosis?

A Cystinosis

B Bartter’s syndrome

C Minimal-change nephropathy

D Nephroblastoma

E Medullary cystic disease

Explanation
Bartter’s syndrome consists of metabolic hypokalaemia, alkalosis, hypercalciuria, normal blood pressure and
elevated plasma renin and aldosterone. It is an autosomal-recessive condition leading to tubular defects in sodium
chloride transport and increased intrarenal production of PGE2. A renal biopsy is diagnostic.

Cystinosis,
another autosomal-recessive disorder, causes hyperchloraemic acidosis and rickets. Medullary cystic
disease (juvenile nephronophthisis) is also inherited in an autosomal-recessive manner. Diagnosis is based on the
family history and renal biopsy, which shows interstitial inflammation, tubular atrophy and medullary cysts.

Minimal-change
nephropathy usually occurs in older children and presents with proteinuria. Hypokalaemia and
alkalosis do not occur. Glomeruli appear normal on light microscopy. Fusion of the foot processes of epithelial
cells is seen on electron microscopy. Nephroblastoma (Wilms' tumour) is seen mainly within the first 3 years of
life and may be bilateral. It presents as an abdominal mass. Metabolic changes are not seen in this disease.
12825

https://mypastest.pastest.com/Secure/TestMe/Browser/436619[‫ ص‬07:54:13 10/12/1437]


MyPastest

Next Question
Previous Question Tag Question

Feedback

End Session
Difficulty: Easy

Peer Responses

Session Progress

Responses Correct: 0

Responses Incorrect: 6

Responses Total: 6

Responses - % Correct: 0%

Blog
About Pastest
Contact Us
Help

© Pastest 2016

https://mypastest.pastest.com/Secure/TestMe/Browser/436619[‫ ص‬07:54:13 10/12/1437]


MyPastest

Prefer to use the old MyPastest? Access it here »

Next Question
Previous Question
Back to Filters

Question 7 of 61 End Session

A 15-year-old boy presents with dark discolouration of urine. There is a history of upper respiratory tract infection
two weeks earlier. If untreated, he may go on to develop:

A Nephrotic syndrome

B Immune deficiency with normal complement (C3) levels

C Acute renal failure

D Fits or a stroke

E Nephroblastoma

Explanation
Bacterial infections, usually subacute, and
typically Group A Streptococcal, may cause a variety of histological
patterns of glomerulonephritis, but usually with plentiful immunoglobulin deposition and often with evidence of
complement consumption (low C3). A low C3 level is indicative of poor prognosis in nephrotic syndrome, but is
common in glomerulonephritis. Renal failure occurs in less than 1% of cases of post-streptococcal nephritis.
Hypertensive encephalopathy may occur and result in fits, coma and neurological events like stroke.
12826

Tag Question

Feedback

https://mypastest.pastest.com/Secure/TestMe/Browser/436619[‫ ص‬07:54:35 10/12/1437]


MyPastest

Difficulty: Difficult

Peer Responses

Session Progress

Responses Correct: 0

Responses Incorrect: 7

Responses Total: 7

Responses - % Correct: 0%

Blog
About Pastest
Contact Us
Help

© Pastest 2016

https://mypastest.pastest.com/Secure/TestMe/Browser/436619[‫ ص‬07:54:35 10/12/1437]


MyPastest

Prefer to use the old MyPastest? Access it here »

Back to Filters

Question 8 of 61

A 15-year-old girl presents with red spots on her buttocks and legs, joint pains, oedema, hypertension and
proteinuria. She had an upper respiratory tract infection 2-3 weeks ago.

What is the most likely diagnosis?

A Proliferative glomerulonephritis

B Goodpasture’s syndrome

C Wegener’s granulomatosis

D Henoch–Schönlein purpura

E Systemic lupus erythematosus

Explanation
This girl most probably has Henoch–Schönlein purpura. It presents with a purpuric rash over the buttocks and
extensor surfaces. Some one-third of patients have
nephritis. Joint pains and acute abdominal pain are common.
The fault lies in the vasculature; the platelets are normal. It often follows an acute respiratory tract infection and
usually follows a benign course over weeks or months. Complications include massive gastrointestinal
haemorrhage, ileus and, rarely, renal failure.

Wegener’s granulomatosis would have additional features such as nasal ulceration, rhinitis, otitis media and
pulmonary symptoms, such as haemoptysis. Variable shadows may be seen on chest X-ray, particularly multiple
nodules. Lung involvement (pulmonary haemorrhage) is seen in Goodpasture’s syndrome: the chest X-ray may
show diffuse infiltrates in the lower zones. Renal involvement in systemic lupus erythematosus occurs in 50% of
patients but is seldom so rapidly progressive as to cause oedema, hypertension and proteinuria in 2–3 weeks time.
Proliferative glomerulonephritis would present with macroscopic haematuria, but purpura is not a feature of this
condition.
12827

https://mypastest.pastest.com/Secure/TestMe/Browser/436619[‫ ص‬07:54:56 10/12/1437]


MyPastest

Next Question
Previous Question
Tag Question

Feedback

End Session
Difficulty: Easy

Peer Responses

Session Progress

Responses Correct: 0

Responses Incorrect: 8

Responses Total: 8

Responses - % Correct: 0%

Blog
About Pastest
Contact Us
Help

© Pastest 2016

https://mypastest.pastest.com/Secure/TestMe/Browser/436619[‫ ص‬07:54:56 10/12/1437]


MyPastest

Prefer to use the old MyPastest? Access it here »

Next Question
Previous Question
Back to Filters

Question 9 of 61 End Session

Primary vesicoureteric reflux is most commonly found in which patient population?

A Girls aged 3–10 years

B Prepubertal boys

C Newborn girls

D Teenage girls

E Boys over 15 years of age

Explanation
Vesicoureteric reflux refers to the retrograde flow of urine from the bladder to the upper urinary tract. This may
occur due to incompetence of the valve at the vesicoureteric junction. It is most commonly detected the earliest in
newborn girls.
12828

Tag Question

Feedback

Difficulty: Difficult

https://mypastest.pastest.com/Secure/TestMe/Browser/436619[‫ ص‬07:55:16 10/12/1437]


MyPastest

Peer Responses

Session Progress

Responses Correct: 0

Responses Incorrect: 9

Responses Total: 9

Responses - % Correct: 0%

Blog
About Pastest
Contact Us
Help

© Pastest 2016

https://mypastest.pastest.com/Secure/TestMe/Browser/436619[‫ ص‬07:55:16 10/12/1437]


MyPastest

Prefer to use the old MyPastest? Access it here »

Back to Filters

Question 10 of 61

Theme: Renal diseases

A Bartter syndrome

B Factitious illness

C Haemolytic uraemic syndrome (HUS)

D Henoch–Schönlein purpura (HSP) nephritis

E Mesangial IgA nephropathy

F Nephrotic syndrome

G Post-streptococcal glomerulonephritis

H Reflux nephropathy

I Systemic lupus erythematosus (SLE)

J Urinary tract infection (UTI)

For
each of the following clinical scenarios, choose the most likely diagnosis from the above list. Each item may
be used once, more than once or not at all.

Scenario 1

A 4-year-old child is brought to see you as his mother noticed that his urine is pink. He has been complaining of
headaches. His blood pressure is 130/88 mmHg. Blood tests reveal: Na+ 130 mmol/L, K+ 4.9 mmol/L, urea (Ur)
12 mmol/L, creatinine (Cr) 96 µmol/L. Complement C3 is reduced but C4 is normal. Albumin is 29 g/L.

Your answer was incorrect

Select one...

G - Post-streptococcal glomerulonephritis

Red urine probably indicates haematuria. He is hypertensive and is having symptomatic headaches; these two
features fit with a nephritis. A low C3 and normal C4 are typical for post-streptococcal glomerular nephritis.

https://mypastest.pastest.com/Secure/TestMe/Browser/436619[‫ ص‬07:55:37 10/12/1437]


MyPastest

Scenario 2

A 13-year-old child presents with blood in


the urine. He reports having an upper respiratory tract infection (URTI)
3 days ago. His blood pressure is 120/60 mmHg. Urine analysis reveals only blood 3+, protein 2+. Renal function
is normal. Complement levels are normal. Next Question
Previous Question
Your answer was incorrect

Select one...
End Session
E - Mesangial IgA nephropathy

Though he has had a recent URTI, the onset of the symptoms is too soon after the URTI for post-streptococcal
glomerular nephritis (which is normally more than a week after). In addition his blood pressure is normal (which
doesn’t fit with a nephritis), and the complement levels are normal therefore mesangial IgA
nephropathy is the
most likely diagnosis. IgA nephropathy most commonly
presents with recurrent macroscopic haematuria
following an URTI; mild associated proteinuria is common.

Scenario 3

A 6-year-old child who has had diarrhoea for the past week is brought to the Emergency Department. Her baseline
bloods reveal: Na+ 133 mmol/L, K+ 5.4 mmol/L, Cr 90 µmol/L, Ur 42 mmol/L, Hb 6.3 g/dL, white cell count
(WCC) 2.3 × 109/L, platelets (plts) 95 × 109/L.

Your answer was incorrect

Select one...

C - Haemolytic uraemic syndrome (HUS)

Any renal impairment following a diarrhoeal episode should raise the suspicion of HUS. This is confirmed by a
very high urea, low Hb and low platelets. Examination of a blood film would show a microangiopathic haemolytic
anaemia. HUS is the most common cause of acute renal failure in childhood. It is caused by verotoxins produced
by Escherichia coli 0157 (a cause of bloody diarrhoea). Over half of the cases require dialysis.
22024

Tag Question

Feedback

Difficulty: Average

Session Progress

https://mypastest.pastest.com/Secure/TestMe/Browser/436619[‫ ص‬07:55:37 10/12/1437]


MyPastest

Responses Correct: 0

Responses Incorrect: 12

Responses Total: 12

Responses - % Correct: 0%

Blog
About Pastest
Contact Us
Help

© Pastest 2016

https://mypastest.pastest.com/Secure/TestMe/Browser/436619[‫ ص‬07:55:37 10/12/1437]


MyPastest

Prefer to use the old MyPastest? Access it here »

Next Question
Previous Question
Back to Filters

Question 11 of 61 End Session

An 8-month-old infant had a confirmed UTI at age 5 months and


is currently on prophylactic antibiotics. Renal
ultrasonographyat the time of the UTI showed left renal pelvicalyceal dilatation.

What is the most appropriate investigation to perform next?

A Repeat renal tract ultrasonography

B Micturating cystourethrogram

C DMSA (99mTc-dimercaptosuccinic acid) scan

D MAG-3 (Mercapto Acetyl Tri Glycine) scan

E Repeat urine for MC&S (microscopy, culture and sensitivity)

Explanation
In an infant who has had a confirmed UTI and pevicalyceal dilatation on ultrasonography, a MCUG is important
to look for VUR. If present, VUR greatly increases the risk for renal scarring when associated with UTIs and so
prophylactic antibiotics are important to try to prevent repeat UTIs.
22360

Tag Question

Feedback

Difficulty: Average

https://mypastest.pastest.com/Secure/TestMe/Browser/436619[‫ ص‬07:55:57 10/12/1437]


MyPastest

Peer Responses

Session Progress

Responses Correct: 0

Responses Incorrect: 13

Responses Total: 13

Responses - % Correct: 0%

Blog
About Pastest
Contact Us
Help

© Pastest 2016

https://mypastest.pastest.com/Secure/TestMe/Browser/436619[‫ ص‬07:55:57 10/12/1437]


MyPastest

Prefer to use the old MyPastest? Access it here »

Next Question
Previous Question
Back to Filters

Question 12 of 61 End Session

A 2-year-old white boy is diagnosed with nephrotic syndrome.

Which of the following is the most likely underlying pathology?

A Minimal change disease

B Mesangial proliferative disease

C Focal segmental glomerular sclerosis

D ‘Finnish’-type microcystic disease

E Cystinosis

Explanation
Minimal change disease is by far the most
common cause of nephrotic syndrome in childhood. The next most
common is focal segmental glomerular sclerosis. Finnish microcystic disease is a
rare cause of nephrotic
syndrome seen only in infancy.
22383

Tag Question

Feedback

Difficulty: Easy

https://mypastest.pastest.com/Secure/TestMe/Browser/436619[‫ ص‬07:56:18 10/12/1437]


MyPastest

Peer Responses

Session Progress

Responses Correct: 0

Responses Incorrect: 14

Responses Total: 14

Responses - % Correct: 0%

Blog
About Pastest
Contact Us
Help

© Pastest 2016

https://mypastest.pastest.com/Secure/TestMe/Browser/436619[‫ ص‬07:56:18 10/12/1437]


MyPastest

Prefer to use the old MyPastest? Access it here »

Next Question
Previous Question
Back to Filters

Question 13 of 61 End Session

A diagnosis of Henoch–Schönlein purpura (HSP) is made in a 7-year-old child. There is a tace of blood on a urine
dipstick. His blood pressure is normal. There is mild arthralgia controlled with ibuprofen and paracetamol. What is
the most appropriate course of action?

A Referral for renal biopsy

B Weekly blood pressure and urine dipsticks until clear

C Discharge from follow-up

D Weekly serum urea and electrolytes (U&Es)

E Monthly outpatient clinic appointments for 6 months

Explanation
In the context of a normal blood pressure and only a trace of urine, there is little evidence of a severe nephritis
needing renal biopsy. It would be unwise to discharge a child at this stage. The most appropriate monitoring is
blood pressure and urine dipsticks until clear – this is non-invasive and can accurately monitor the development of
nephritis.
22389

Tag Question

Feedback

https://mypastest.pastest.com/Secure/TestMe/Browser/436619[‫ ص‬07:56:37 10/12/1437]


MyPastest

Difficulty: Average

Peer Responses

Session Progress

Responses Correct: 0

Responses Incorrect: 15

Responses Total: 15

Responses - % Correct: 0%

Blog
About Pastest
Contact Us
Help

© Pastest 2016

https://mypastest.pastest.com/Secure/TestMe/Browser/436619[‫ ص‬07:56:37 10/12/1437]


MyPastest

Prefer to use the old MyPastest? Access it here »

Back to Filters

Question 14 of 61

Which of the following statements about scrotal pain is most factually correct?

A If
torsion of the testis cannot be excluded clinically exploration of the scrotum is the only safe
management of an acutely inflamed scrotum

B Postoperatively a previously torted viable testis will go on to display completely normal


sermatogenesis in adult life

C Testicular torsion accounts for just over 50% of the cases of acute scrotum presenting in childhood

D Most torted testes remain viable up to 18 hours after the onset of symptoms

E It is impossible to differentiate between testicular torsion and torsion of the hydatid of Morgagni

Explanation
The acute scrotum is defined as the sudden onset of testicular pain; there are many causes.

Testicular torsion (25%)

Torsion can occur at any time during childhood but peaks in the neonatal and peripubertal periods.
Testis is acutely very tender.
Testis lies high in the scrotum accompanied by scrotal redness and oedema.
It is indefensible if in doubt not to explore the testis if torsion is suspected.
Infarction occurs within 10 h from onset of symptoms.
Spermatogenesis is decreased post-torsion in a viable testis.
Loss of testis may be managed cosmetically with a prosthesis.

Torted hydatid of Morgagni

https://mypastest.pastest.com/Secure/TestMe/Browser/436619[‫ ص‬07:56:59 10/12/1437]


MyPastest

Pedunculated small cystic remnant of the müllerian ducts usually attached to the upper pole of the testis.
Torsion of the appendage presents with acute scrotal swelling, localised tenderness ? hydrocele.
Next Question
Transillumination reveals a blue/black spot lying at the superior pole of the testis.
Previous
If diagnosis Question
is clear there is no indication for surgery.
Natural history of spontaneous resolution of symptoms within 48 hours.

Orchitis, epididymo-orchitis End Session


Orchitis is now rare in childhood post-MMR.
Epididymo-orchitis: most cases are viral, if urine culture positive then 7 days of oral antibiotics are
indicated.
Due to differential scrotal exploration is needed to exclude torsion of the testis if the diagnosis is not
certain.
Prognosis is good but recurrence should raise suspicion of abnormal urinary anatomy.

Idiopathic scrotal oedema

Inflamed scrotum/hemi scrotum in young boys that may spread up the shaft of the penis or onto the
perineum
Unknown aetiology
Self-limiting, resolves within 1–4 days
Treatment – analgesia
If superinfection occurs (rare) antibiotic cover with flucloxacillin is indicated.

Further differential diagnoses of an acutely painful testis include an incarcerated inguinal hernia or a tense
hydrocele.

Painless
swelling of the scrotum can occur due to a number of different causes including HSP, hydrocele and
generalised causes of oedema such as nephrotic syndrome.
44039

Tag Question

Feedback

Difficulty: Average

Peer Responses

https://mypastest.pastest.com/Secure/TestMe/Browser/436619[‫ ص‬07:56:59 10/12/1437]


MyPastest

Session Progress

Responses Correct: 0

Responses Incorrect: 16

Responses Total: 16

Responses - % Correct: 0%

Blog
About Pastest
Contact Us
Help

© Pastest 2016

https://mypastest.pastest.com/Secure/TestMe/Browser/436619[‫ ص‬07:56:59 10/12/1437]


MyPastest

Prefer to use the old MyPastest? Access it here »

Back to Filters

Question 15 of 61

Which of the following statements about nocturnal enuresis is most factually correct?

A Behavioural
strategies such as star charts are not beneficial in the management of nocturnal enuresis
because the symptoms are beyond cognitive control.

B Increasing
day-time fluid intake can be an effective intervention in helping to decrease the symptom of
nocturnal enuresis in some children.

C Longstanding
day-time symptoms of urinary urgency, incontinence, frequent (over seven) or
infrequent (fewer than four) episodes of urination can be associated with enuresis in patients with
diabetes mellitus.

D Nocturnal
enuresis in children under the age of 10 years has equal sex distribution with increasing
proportions of males affected after 13 years of age.

E Nocturnal enuresis is a primary condition (the child never having achieved dryness for 6 consecutive
months) in 90% of cases.

Explanation
Definition: wetting the bed three to more nights per month or one night per week:

8–10% of 5–6 year olds


5–7% of 7–10 year olds
1–3% by adulthood
Enuresis
can be monosymptomatic (two-thirds) or non-monosymptomatic (a third), In combination with
lower urinary tract symptoms/bladder dysfunction including daytime urgency, frequency and small
daytime voiding volumes
The male:female ratio is 2:1

https://mypastest.pastest.com/Secure/TestMe/Browser/436619[‫ ص‬07:57:23 10/12/1437]


MyPastest

The incidence doubles with a positive parental history


70% incidence if both parents suffered
About 20–30% are secondary (after >6 months of night-time dryness).
Next Question
Previous Question
Enuresis mechanisms

An inability to wake during sleep in response to need to void


Nocturnal polyuria (two-thirds respond to DDAVP)
Reduced bladder capacity (small size ? overactive bladder)

Management

General

Simple advice on fluid intake (may need to increase during the day, nil <1 hr before bed)
Regular toileting during the day (more than four but less than seven times)
Avoid anger/blame from parents
Advice
on reward system to use alone or together with other therapies (star charts for compliance with
treatment – not dry nights)
Suggest a trial without nappies for all dry by day for >6 months
Avoid lifting
Treat co-morbidities (constipation, obstructive sleep apnoea, recurrent UTIs)
Consider alternative diagnoses: diabetes mellitus, maltreatment, learning difficulties/attention deficit
hyperactivity disorder)
Under age 5 years – reassurance.

Specific

Enuresis alarm: 35% learn to wake, 65% sleep through (65% success, 42% relapse)
Desmopressin: >50% ? in wet nights in 60–70%); break for 1 week every 3 months to assess progress
Anticholinergics, eg oxybutynin: ? detrusor overactivity; can be used in combination with desmopressin.

An
adequate (not excessive) fluid intake must be maintained when on DDAVP and the medication should be
stopped if vomiting occurs in order to prevent severe hyponatraemia.
44158

Tag Question

https://mypastest.pastest.com/Secure/TestMe/Browser/436619[‫ ص‬07:57:23 10/12/1437]


MyPastest

Feedback

Difficulty: Average

Peer Responses

End Session

Session Progress

Responses Correct: 0

Responses Incorrect: 17

Responses Total: 17

Responses - % Correct: 0%

Blog
About Pastest
Contact Us
Help

© Pastest 2016

https://mypastest.pastest.com/Secure/TestMe/Browser/436619[‫ ص‬07:57:23 10/12/1437]


MyPastest

Prefer to use the old MyPastest? Access it here »

Back to Filters

Question 16 of 61

Which of the following statements about urinary tract infection (UTI) in children is most factually correct?

A All upper renal tract infections in children <3 years old should be treated initially with intravenous
antibiotics.

B Prophylaxis for recurrent UTIs is no longer indicated according to NICE guidelines for the
management of UTIs in children.

C Pseudomonas UTIs are associated with renal stones.

D The
degree of scarring found on a DMSA scan is proportional to the number of previous episodes of
upper urinary tract infections.

E Treatment
with a 3-day course of oral antibiotics is appropriate in a 2-year-old child with proven UTI
and symptoms of a lower UTI.

Explanation

Ten per cent of girls and 3% of boys have had a UTI by 16 years.
Vesicoureteric reflux (VUR) is present in approximately a third of children with a UTI.
Children aged <3 years should have microscopy not dipstick.
E. coli accounts for 70–90% of community-acquired UTIs.
Non-E. coli UTIs are associated with increased underlying obstructive structural abnormalities

The
National Institute for Health and Clinical Excellence (NICE) guidance (2007) recommends assessing for signs
of severe underlying pathology, and defines atypical UTIs as:

Poor urine flow

https://mypastest.pastest.com/Secure/TestMe/Browser/436619[‫ ص‬08:01:56 10/12/1437]


MyPastest

Previous UTIs
Recurrent pyrexia of unknown origin
Antenatal diagnosis of renal abnormality
Next Question
Family history of VUR or renal disease
Previous Question
Constipation
Dysfunctional voiding
Enlarged bladder
Abdominal mass End Session
Evidence of spinal lesion
Poor growth
Hypertension
Seriously ill child/septicaemia
Raised serum creatinine
Failure to respond to treatment in 48 hours
Infection with non-E. coli organisms.

Although
UTIs do predispose to scarring, the degree of damage is unpredictable and evidence has swung against
intensive investigation. NICE guidelines detail imaging needed for follow-up stratified by age group at diagnosis,
response to treatment by 48 hours, and recurrent and atypical
presentations.

Treatment depends on age and severity of symptoms and ability to tolerate oral antibiotics.

>3 months with upper tract signs:


– oral antibiotics for 7–10 days,
– unable to tolerate oral antibiotics: 48 h of intravenous antibiotics followed by 8 days of oral
Lower tract signs – 3 days of oral antibiotics
Prophylaxis should be considered in recurrent UTIs
Treatment should be with a different antibiotic if infection occurs on prophylaxis.

Follow-up
long term is indicated for all with bilateral renal abnormalities, impaired renal function, hypertension
and/or proteinuria.

https://www.nice.org.uk/guidance/cg54
44160

Tag Question

Feedback

https://mypastest.pastest.com/Secure/TestMe/Browser/436619[‫ ص‬08:01:56 10/12/1437]


MyPastest

Difficulty: Average

Peer Responses

Session Progress

Responses Correct: 0

Responses Incorrect: 18

Responses Total: 18

Responses - % Correct: 0%

Blog
About Pastest
Contact Us
Help

© Pastest 2016

https://mypastest.pastest.com/Secure/TestMe/Browser/436619[‫ ص‬08:01:56 10/12/1437]


MyPastest

Prefer to use the old MyPastest? Access it here »

Next Question

Back to Filters

Question 17 of 61

Which of the following statements about hypertension in childhood is most factually correct?

A Ambulatory BP measurements are not acceptable in the vast majority of children due to discomfort.

B Hypertension in childhood is defined as a systolic blood pressure (BP) >120 mmHg.

C The use of a small blood pressure cuff will produce a falsely low reading of blood pressure.

D The vast majority of cases of hypertension in children are secondary to endocrine disease.

E The
vast majority of children are asymptomatic but hypertensive encephalopathy presenting as severe
hypertension with headache, vomiting, hyperreflexia and seizures is a medical emergency requiring
admission to hospital for a controlled gradual decrease in BP.

Explanation
Hypertension in children is rare and usually secondary to renal (80%) or endocrine pathology however, primary
hypertension in childhood is an increasingly recognised problem.

Hypertension
in childhood is defined as a BP over the 95th centile for age, height and gender. All children with
BPs >25 mmHg above the 95th centile should be admitted for immediate investigation and treatment. Those with
less severe hypertension require home ambulatory BP measurements to
clarify the diagnosis.

To measure the blood pressure manually, the bladder of the cuff itself should cover at least two-thirds of the
circumference of the arm. A small cuff will result in a high reading.
44161

Tag Question

https://mypastest.pastest.com/Secure/TestMe/Browser/436619[‫ ص‬08:02:17 10/12/1437]


MyPastest

Feedback

Previous Question Difficulty: Average

Peer Responses

End Session

Session Progress

Responses Correct: 0

Responses Incorrect: 19

Responses Total: 19

Responses - % Correct: 0%

Blog
About Pastest
Contact Us
Help

© Pastest 2016

https://mypastest.pastest.com/Secure/TestMe/Browser/436619[‫ ص‬08:02:17 10/12/1437]


MyPastest

Prefer to use the old MyPastest? Access it here »

Back to Filters

Question 18 of 61

Which of the following statements concerning undescended testis is most factually correct?

A By 12 months of age 8% of previously undescended testes remain outside the scrotum.

B Cases of retractile testes should undergo orchidopexy to decrease the associated risk of testicular
torsion.

C Early orchidopexy has been shown to reduce infertility in later life.

D It
is essential to perform a karyotype to exclude mixed gonadal dysgenesis
and disorders of sex
differentiation in all cases of undescended testes
and retractile testes.

E Undescended testes are less common in the low-birthweight and preterm infants.

Explanation

Of babies boys born at term 4–5% have either unilateral or bilateral UDT.
Some 1.5% have UDT by the age of 6 months.
Spontaneous descent is rare after 6 months.
Impalpable testes are not necessarily absent and may be intra-abdominal.
UDT rates ? for preterm and low-birthweight babies and an increased rate of:
– 6–10% with a previously affected sibling
– 70% – undescended testis is on the right
– 10–25% of cases are bilateral.
Of
cases 10–20% have a completely impalpable testis and in bilateral
cases a disorder of sex
differentiation must be considered and investigated.
A small proportion of testes ascend to become permanently impalpable, having been in the scrotum at

https://mypastest.pastest.com/Secure/TestMe/Browser/436619[‫ ص‬08:02:38 10/12/1437]


MyPastest

birth (1.2% at 6 years, 2.2% at 9 years and 1.1% at 13 years).

Important differential diagnoses to consider include:


Next Question
RetractilePrevious
testis (can beQuestion
‘milked’ down into the scrotum)
Maldescended testis (abnormal route of descent and so will never spontaneously descend)
Absent testis
Mixed gonadal dysgenesis (often 45XO/46XY mosaic) associated with End
hypospadias
Sessionand UDT
Disorder of sex differentiation (DSD).

There is an increased risk of infertility and of malignancy in children with undescended testis.

In
order to minimise these, orchidopexy should be carried out before the end of the second year, increasingly
between 6 and 12 months.

Early surgical intervention can improve future fertility and may decrease the risk of malignancy.

The incidence of testicular tumours in adults is 3/10 000.

The risk in males with a history of undescended testis is 4–10 times greater.

Sixty per cent of the tumours are seminomas, most of the remainder being teratomas.

Important associations of undescended testis include:

Spinal muscular atrophy


Myotonic dystrophy
X-linked ichthyosis
Kallmann syndrome
Prune-belly syndrome.

44166

Tag Question

Feedback

Difficulty: Average

Peer Responses

https://mypastest.pastest.com/Secure/TestMe/Browser/436619[‫ ص‬08:02:38 10/12/1437]


MyPastest

Session Progress

Responses Correct: 0

Responses Incorrect: 20

Responses Total: 20

Responses - % Correct: 0%

Blog
About Pastest
Contact Us
Help

© Pastest 2016

https://mypastest.pastest.com/Secure/TestMe/Browser/436619[‫ ص‬08:02:38 10/12/1437]


MyPastest

Prefer to use the old MyPastest? Access it here »

Back to Filters

Question 19 of 61

Which of the following statements about Wilms tumours is most factually correct?

A Forty per cent of cases of Wilms tumours are bilateral.

B Hemihypertrophy, genitourinary abnormalities and aniridia are all associated with an increased risk of
developing Wilms tumours.

C The most common presentation of Wilms tumour is with painful haematuria.

D There
has been no genetic basis for Wilms tumour identified to date but in some families there is a
very high incidence of Wilms tumour.

E Wilms tumours account for approximately 50% of abdominal masses in childhood.

Explanation
Wilms tumours are characterised by a malignant proliferation of the metanephric blastema (primitive embryonic
renal) cells.

The vast majority of Wilms tumours arise sporadically with no other predisposing malformation syndromes or
family history.

However,
in a small number of cases there is a significant family history of Wilms tumour or one of the following
recognised predisposing conditions listed below.

In these circumstances regular screening for the development of Wilms tumour with yearly or twice yearly renal
ultrasound examinations is advisable:

Isolated hemihypertrophy
Beckwith–Wiedemann syndrome
Neurofibromatosis

https://mypastest.pastest.com/Secure/TestMe/Browser/436619[‫ ص‬08:08:49 10/12/1437]


MyPastest

Drash syndrome (Wilms tumour + pseudohermaphroditism + glomerulopathy)


WAGR syndrome (Wilms tumour + aniridia + genitourinary malformations + learning disability)
Ambiguous genitalia
Next Question
Nephropathy
Previous Question
Aniridia
Genitourinary malformations
Learning disability
Trisomy 18. End Session

Wilms tumour accounts for 10% of all paediatric tumours; 10% are bilateral.

The mean age of diagnosis is 3.5 years.

The most common presenting feature is an abdominal mass.

Abdominal
pain occurs in 30–40% of cases and hypertension, haematuria, anaemia and fever secondary to tumour
necrosis may be associated.

Ultrasonography is the best investigation to make the diagnosis.

Chromosomal analysis and gene mapping are currently available.

Treatment is with surgery and chemotherapy, with the addition of radiotherapy for more advanced disease.

The overall prognosis is good, with 80–90% 5-year survival rate.


44185

Tag Question

Feedback

Difficulty: Average

Peer Responses

https://mypastest.pastest.com/Secure/TestMe/Browser/436619[‫ ص‬08:08:49 10/12/1437]


MyPastest

Session Progress

Responses Correct: 0

Responses Incorrect: 21

Responses Total: 21

Responses - % Correct: 0%

Blog
About Pastest
Contact Us
Help

© Pastest 2016

https://mypastest.pastest.com/Secure/TestMe/Browser/436619[‫ ص‬08:08:49 10/12/1437]


MyPastest

Prefer to use the old MyPastest? Access it here »

Back to Filters

Question 20 of 61

Which of the following statements concerning vesicoureteric reflux is most factually correct?

A Spontaneous remission of vesicoureteric reflux is rare.

B The
aim of prophylactic antibiotics in children with vesicoureteric reflux is to minimise the harmful
consequences of renal scarring and progression towards renal impairment and subsequent failure.

C The damage caused by vesicoureteric reflux can be reversed with the use of corrective surgery.

D Vesicoureteric reflux can most accurately be diagnosed using a DMSA scan.

E Vesicoureteric reflux occurs in less than 5% of children who present with a confirmed urinary tract
infection.

Explanation
Of children who present to hospital with a confirmed urinary tract infection, 30–40% have VUR (boys > girls).

Of those with UTIs, up to 30% develop renal scars.

Over
30% of patients who are found to have hydronephrosis on antenatal ultrasound scanning are shown to have
VUR on investigations performed postnatally.

There is a strong genetic predisposition to the development of VUR, with an occurrence rate of 25–30% in
siblings and 66% in children of affected individuals.

The diagnosis of VUR is by micturating cystography.

Severity is graded from I to IV, with grade IV implying presence of intrarenal reflux.

A
MAG-3 indirect cystogram can be used once continence has been achieved;
this is both less sensitive and less
specific although better tolerated.

The object of management is to prevent UTIs and in doing so prevent renal scarring (reflux nephropathy).

https://mypastest.pastest.com/Secure/TestMe/Browser/436619[‫ ص‬08:09:12 10/12/1437]


MyPastest

This
is best achieved using low-dose prophylactic antibiotics to prevent infections and appropriate antibiotics if
breakthrough infection occurs based on urine culture and sensitivity.

Next Question
Most reflux resolves by the age of 5 years. If not, and recurrent infections occur with progressive renal scarring,
ureteric reimplantation is indicated.
Previous Question
DMSA is the best investigation to look for renal scars.

Complications of reflux nephropathy include:

Hypertension End Session


Impaired renal function (? urine concentrating ability and ? GFR)
Renal failure.

44189

Tag Question

Feedback

Difficulty: Average

Peer Responses

Session Progress

Responses Correct: 0

Responses Incorrect: 22

Responses Total: 22

Responses - % Correct: 0%

https://mypastest.pastest.com/Secure/TestMe/Browser/436619[‫ ص‬08:09:12 10/12/1437]


MyPastest

Blog
About Pastest
Contact Us
Help

© Pastest 2016

https://mypastest.pastest.com/Secure/TestMe/Browser/436619[‫ ص‬08:09:12 10/12/1437]


MyPastest

Prefer to use the old MyPastest? Access it here »

Back to Filters

Question 21 of 61

A 2-week-old baby boy whose mother had no antenatal care is referred for assessment of failure to thrive.

History
gives few clues but he has a rather poor urine stream. On examination he is scrawny with a preauricular
tag and has palpable loin masses.

The NEXT step should be:

A Catheterisation

B Intravenous urography

C Renal biopsy

D Start antibiotics and arrange review

E Urgent renal ultrasonography

Explanation
This baby has renal failure secondary to posterior urethral valves (PUVs) until proved otherwise and the story is
typical.

The diagnosis of PUVs is limited to baby boys and is the most common urological cause of renal failure in
childhood.

PUVs
cause bladder obstruction that can vary in severity ranging from disease so severe that it is incompatible
with postnatal life through to
disease so minimal that it manifests only in later life.

In
70% of cases the diagnosis is suspected on antenatal ultrasonography with findings of a thick-walled, poorly
emptying bladder, dilated posterior urethra, bilateral hydronephrosis, echogenic or cystic change in the kidneys ?
oligohydramnios.

Those cases not diagnosed antenatally can present in infancy with UTI, poor urinary stream, failure to thrive and

https://mypastest.pastest.com/Secure/TestMe/Browser/436619[‫ ص‬08:09:53 10/12/1437]


MyPastest

abdominal mass (palpable bladder or kidneys) ,or later in childhood with persistent day- and night-time wetting or
intermittent retention.

The morbidity of PUVs is not just limited to transient urethral obstruction.


Next Question
Obstruction
of the urinary tract antenatally at a time of critical organogenesis may result in lifelong dysfunction in
Previous Question
both kidney and bladder function.

Ultrasonography
at presentation, or in the postnatal period if suspected antenatally, confirms obstruction at the
urethral level with ureteric and pelvicalyceal dilatation.

The gold standard for diagnosis remains the micturating cystourethrogram.


End Session
Management includes catheterisation followed by endoscopic resection of valves once the patient is stable.

It
is important to treat any pulmonary insufficiency secondary to oligohydramnios, exclude UTI, start prophylactic
antibiotics, and carefully manage fluid and electrolyte balances to maximise outcome.

Approximately
a third of patients born with PUVs progress to end-stage renal disease;
this represents 10–15% of
all children undergoing renal transplantation.

Preauricular tags are common and, though there is an occasional association with renal abnormalities,
they do not,
as such, warrant further investigation other than a careful history and clinical examination.
44191

Tag Question

Feedback

Difficulty: Average

Peer Responses

Session Progress

Responses Correct: 0

Responses Incorrect: 23

Responses Total: 23

https://mypastest.pastest.com/Secure/TestMe/Browser/436619[‫ ص‬08:09:53 10/12/1437]


MyPastest

Responses - % Correct: 0%

Blog
About Pastest
Contact Us
Help

© Pastest 2016

https://mypastest.pastest.com/Secure/TestMe/Browser/436619[‫ ص‬08:09:53 10/12/1437]


MyPastest

Prefer to use the old MyPastest? Access it here »

Back to Filters

Question 22 of 61

A 5-year-old boy is referred with a 2-week history of general malaise and facial swelling.

There is a history of recent upper respiratory tract infection. On examination, he has generalised oedema with
shifting dullness on abdominal percussion. He has 4+ proteinuria and BP 140/100 mmHg. His initial
investigations show an albumin of 18 g/l normal renal function and a normal C3 and C4.

The BEST initial management is:

A Dietary review and start high protein diet

B High-dose oral furosemide

C Immediate transfer to paediatric nephrology service

D Infusion of 20% human albumin solution

E Start high-dose prednisolone

Explanation
Nephrotic syndrome usually presents with generalised oedema. Ascites and pleural effusions may develop in more
severe cases.

There
is often a history of URTI that precedes the onset of oedema and children may be lethargic and irritable with
poor appetite, and may have
diarrhoea and abdominal pain.

Blood pressure is usually low or normal. Hypovolaemia is a common finding although difficult to assess.

Symptoms and signs associated with hypovolaemia include:

Cap refill time >2 s


Toe core temperature gap of >2°C

https://mypastest.pastest.com/Secure/TestMe/Browser/436619[‫ ص‬08:10:14 10/12/1437]


MyPastest

Hypotension/hypertension
Persistent tachycardia
Abdominal pain
Next Question
Urinary sodium <10 mmol/l.
Previous Question
Hypovolaemia
should be corrected with fluid resuscitation and cautious use of 4.5% albumin or, more rarely, very
cautious 20% albumin.
End Session
Treatment is with a course of prednisolone starting at 60 mg/m2 daily of prednisolone until remission of the
2
proteinuria (95% in <28 days), followed by 40 mg/m on alternate days for a further 4 weeks before reduction to
stop.

Most
children have a relapsing–remitting course with 60% having five or more relapses over several years;
however, in the long term most show
a decline in the rate of relapses and most patients achieve long-term
remission by adulthood.

Steroid-sparing agents used in cases found to be steroid dependent or for cases showing frequent relapses include
levamisole, cyclophosphamide, ciclosporin and rituximab (CD20 monoclonal antibody).

These agents should all be used under the guidance of a paediatric nephrologist.

Referral to a paediatric nephrologist should be considered for the following:

Age <12 months


Age >10 years
Macroscopic or persistent microscopic haematuria
Impaired renal function not attributable to hypovolaemia
Persistent hypertension.
Decreased C3 complement
Rash or arthropathy
Failure to induce remission with 28 days of prednisolone at 60 mg/m2.

44192

Tag Question

Feedback

Difficulty: Average

Peer Responses

https://mypastest.pastest.com/Secure/TestMe/Browser/436619[‫ ص‬08:10:14 10/12/1437]


MyPastest

Session Progress

Responses Correct: 0

Responses Incorrect: 24

Responses Total: 24

Responses - % Correct: 0%

Blog
About Pastest
Contact Us
Help

© Pastest 2016

https://mypastest.pastest.com/Secure/TestMe/Browser/436619[‫ ص‬08:10:14 10/12/1437]


MyPastest

Prefer to use the old MyPastest? Access it here »

Next Question

Back to Filters

Question 23 of 61

A 4-year-old girl is referred with periumbilical pain and urinary frequency for some months.

Multiple MSUs have been negative. She has regressed behaviourally with frequent temper tantrums at home, but
is well behaved at nursery school.
Other members of her family have been well. She has a new baby sister who is
6 months old. Examination including BP is normal.

The MOST appropriate approach would be:

A Arrange a screen of blood tests and abdominal ultrasonography

B Arrange to be seen jointly with social services on suspicion of abuse

C Reassure that this is a normal reaction to new siblings and that no investigation is needed

D Start antibiotic prophylaxis for possible missed urinary tract infections while arranging a renal isotope
scan and cystogram

E Surgical opinion

Explanation
The symptoms are typically non-organic.

However,
a thorough history should be taken and comprehensive examination, including urinalysis and blood
pressure and measuring and plotting of height and weight, should be performed.

If there is nothing to note in the history, examination or urinalysis, it is important not to escalate anxiety in this
situation; an appropriate next step would be reassurance for the child and the family that there is nothing serious
amiss and that she will improve with time as the family dynamics settle down into a new rhythm.
44197

https://mypastest.pastest.com/Secure/TestMe/Browser/436619[‫ ص‬08:10:34 10/12/1437]


MyPastest

Tag Question

Feedback
Previous Question
Difficulty: Average

Peer Responses End Session

Session Progress

Responses Correct: 0

Responses Incorrect: 25

Responses Total: 25

Responses - % Correct: 0%

Blog
About Pastest
Contact Us
Help

© Pastest 2016

https://mypastest.pastest.com/Secure/TestMe/Browser/436619[‫ ص‬08:10:34 10/12/1437]


MyPastest

Prefer to use the old MyPastest? Access it here »

Back to Filters

Question 24 of 61

An-8-year-old boy attends the emergency department with acute


testicular pain. On examination he has a tender
testicle. On trans-illumination a faint blue dot can be seen.

What is the most likely diagnosis?

A Acute testicular Torsion

B Epidiymo-orcitis

C Torsion of the hydatid cyst of the testis

D Idiopathic Scrotal Oedema

E Inguinal Hernia

Explanation
The differential diagnoses in acute testicular pain/swelling are listed in the possible answers. Testicular torsion
typically affects adolescent males and presents with acute pain swelling and a horizontally lying testicle.
Epididymo-orchitis is diagnosed at surgery with a thickened erythematous epididymis. Ectopic ureteric insertion
into vas must be excluded by ultrasound scan on an out patient basis. The hydatid cyst or cyst of Morgagni
represents the remnants of the mullarian structure. In the pre-pubertal child a surge in hormones can stimulate
growth and chance of torsion. It is diagnosed by surgical exploration or by a blue dot sign. Idiopathic scrotal
oedema
tends to be within the scrotum itself and the testis can be examined in
the groin to exclude pain in the
testis. All acute scrotal pain must be
explored to exclude testicular torsion, unless a surgical registrar is convinced
of a tortid hydatid cyst.

Acute testicular torsion is false, although the reason for exploration at surgery is to rule out testicular torsion this
typically occurs in pubertal children. An 8-year-old is more likely to represent tortid hydatid cyst. The blue dot
sign is diagnostic and can negate the need for surgical exploration. The age of the child also makes epidiymo-
orcitis less likely.

https://mypastest.pastest.com/Secure/TestMe/Browser/436619[‫ ص‬08:11:01 10/12/1437]


MyPastest

Idiopathic scrotal
oedema presents with an erythematous and thickened scrotum. This can cross the midline and
involve the whole scrotum however again, an 8-year-old is more likely to represent tortid hydatid cyst.

Finally,
an inguinal hernia would present with a mass in the groin extending to the scrotum that you cannot get
Next Question
above plus the age of the child makes this less likely.
Previous Question 45040

End Session
Tag Question

Feedback

Difficulty: Average

Peer Responses

Session Progress

Responses Correct: 0

Responses Incorrect: 26

Responses Total: 26

Responses - % Correct: 0%

Blog
About Pastest
Contact Us
Help

© Pastest 2016

https://mypastest.pastest.com/Secure/TestMe/Browser/436619[‫ ص‬08:11:01 10/12/1437]


MyPastest

Prefer to use the old MyPastest? Access it here »

Back to Filters

Question 25 of 61

Parents bring their 6-week-old baby for a cultural circumcision. On examination the urethra opens ventrally just
before the
glans. The penis has a slight tilt towards the feet and the foreskin doesn’t meet. You also notice both
testis are retractile in nature.

Which of the following statements concerning circumcision in this case is correct?

A Continue with the circumcision and refer to paediatric urology outpatients

B Cancel circumcision and referral to endocrinology to rule of disorder of sexual differentiation

C Cancel circumcision and perform epispadias repair at 6 to 12 months of age

D Perform a primary repair of defect as part of the circumcision

E Cancel circumcision and perform a hypospadias repair at 6 to 12 months

Explanation
Hypospadias affects 1 in 300 males and is
characterised by a triad of a foreshortened ventral opening, a chordee
(tilt ventrally) and hooded foreskin. In the absence of true undescended
testis this can be referred to surgeons for
correction around the age of 1 year. If seen at birth with bilateral undescended testis, disorder of sexual
development must be considered. The foreskin is essential for most repairs.

Answer stem A is false, as this is a hypospadias. The foreskin may be required during surgical reconstruction

Answer
stem B is false as retractile testis is a normal feature in pre-pubertal boys. Simple hypospadias does not
require a referral to endocrinology

Answer stem C is false because epispadias represents an opening on the dorsal side not the ventral side
of the
penis and is much less common than hypospadias.

Answer stem D is false as primary repair in hypospadias occurs from 6-months of age.

https://mypastest.pastest.com/Secure/TestMe/Browser/436619[‫ ص‬08:11:26 10/12/1437]


MyPastest

Answer
stem E is true; this is hypospadias with chordee (the penile tilt) and a
hooded foreskin. Most paediatric
urologists start primary surgical repair after the age of 6-18 months. It is important to tell the parents
that
circumcision in these patients is contra-indicated as the foreskin
will be used in the repair.
Next Question 45041

Previous Question

End
Tag Session
Question

Feedback

Difficulty: Average

Peer Responses

Session Progress

Responses Correct: 0

Responses Incorrect: 27

Responses Total: 27

Responses - % Correct: 0%

Blog
About Pastest
Contact Us
Help

© Pastest 2016

https://mypastest.pastest.com/Secure/TestMe/Browser/436619[‫ ص‬08:11:26 10/12/1437]


MyPastest

Prefer to use the old MyPastest? Access it here »

Back to Filters

Question 26 of 61

On routine Inguinal Hernia repair of a 4-year-old girl a testis is seen in the inguinal region.

What is the most appropriate course of action?

A Finish
the operation leaving gonad in situ and inform the parents more tests are required to determine
whether their child is actually a boy

B Obtain
consent for blood test to perform chromosomal analysis and arrange referral to multi-regional
centre for Disorder of sexual development (DSD)

C Obtain consent to take a biopsy of the testis as part of the procedure to send for DSD work up

D Obtain consent to perform a diagnostic laparoscopy to look for a normal ovary on the other side

E Give a dose of intravenous steroids intra-operatively as an adrenal crisis is possible

Explanation
The most likely diagnosis is complete androgen insensitivity syndrome (CAIS). This is present in 1% of girls with
bilateral inguinal hernias. A blood test for chromosome analysis or to determine the presence of the SRY gene is
indicated. This can usually be performed with permission while the child is asleep. An ultrasound scan to look for
presence of a uterus and biopsy of the gonad
if mixed gonadal dysgenesis is suspected is usually carried out at a
specialised centre. Children with CAIS usually present with either an inguinal hernia or more commonly with
primary amenorrhea with secondary sexual development.

Answer stem A is false, although it is correct to finish the hernia repair it would be useful to
obtain a blood test in
order to assess for presence of the SRY gene to rule out CAIS.

Answer stem B is true, this is as


much as would be expected outside of a centre of excellence for DSD. CASI is
present in 1% of bilateral female hernias.

Answer stem C is false. CAIS is most likely therefore testicular biopsy is not required as they tend to remain to

https://mypastest.pastest.com/Secure/TestMe/Browser/436619[‫ ص‬08:11:56 10/12/1437]


MyPastest

allow for normal pubertal development.

Answer stem D is false. Unless performing the hernia by laparoscopic repair, this is best done at the multiregional
centre for DSD.
Next Question
Answer stem E is false, the most common DSD is congenital adrenal hyperplasia. This can lead to a salt and
Previous Question
steroid crisis at birth. Blood is typically sent for 17-OH Progesterone and 11-deoxycortisol. 24-hour urinary
steroids are also collected.
45042

End Session

Tag Question

Feedback

Difficulty: Average

Peer Responses

Session Progress

Responses Correct: 0

Responses Incorrect: 28

Responses Total: 28

Responses - % Correct: 0%

Blog
About Pastest
Contact Us
Help

© Pastest 2016

https://mypastest.pastest.com/Secure/TestMe/Browser/436619[‫ ص‬08:11:56 10/12/1437]


MyPastest

Prefer to use the old MyPastest? Access it here »

Back to Filters

Question 27 of 61

A 3.5kg newborn baby boy has not passed urine at 24 hours of age. A 6f feeding tube is able to be passed into the
bladder and drains 30mls of urine. An Ultrasound scan reveals gross right-sided hydroureteronephrosis with very
little renal tissue and a thick walled bladder.

What is the most likely diagnosis?

A Posterior Urethral Valves with poor prognostic sign

B Unilateral hydroureteronephrosis secondary to pelvico-ureteric junction obstruction

C Multicystic dysplastic Kidney

D Posterior Urethral valves with a good prognostic sign

E Horseshoe Kidney

Explanation
Posterior urethral valves is the most likely explanation. Any male born baby with bilateral hydronephrosis or
unilateral hydroureteronephrosis must have valves excluded. Good prognostic indicators include pop off
phenomenon where one kidney takes all the pressure and the other is protector or an urinoma forms. A 6 French
feeding tube is slightly smaller than a 6 French catheter and is small for a 3.5 kg child.

Answer stem A is false, the gross dilatation of the right kidney has been protective of the left and is a good
prognostic sign.

Answer stem B is false, there is hydroureteronephosis meaning the ureter is also dilated below the level of the
renal pelvis making unilateral hydroureteronephrosis secondary to pelvico-ureteric junction obstruction
unlikely.

Answer stem C is false, multicystic dysplastic kidney would typically not demonstrate a dilated ureter or thick
walled bladder.

Answer stem D is true, the pop-off phenomenon where one kidney receives all the damage is a good prognostic

https://mypastest.pastest.com/Secure/TestMe/Browser/436619[‫ ص‬08:12:48 10/12/1437]


MyPastest

sign.

Answer stem E is false; multicystic dysplastic kidney would typically show a pelvic single kidney.
45043
Next Question
Previous Question

Tag Question
End Session
Feedback

Difficulty: Average

Peer Responses

Session Progress

Responses Correct: 0

Responses Incorrect: 29

Responses Total: 29

Responses - % Correct: 0%

Blog
About Pastest
Contact Us
Help

© Pastest 2016

https://mypastest.pastest.com/Secure/TestMe/Browser/436619[‫ ص‬08:12:48 10/12/1437]


MyPastest

Prefer to use the old MyPastest? Access it here »

Back to Filters

Question 28 of 61

A 3-month-old child comes to clinic with an antenatal history


of an abnormal right kidney. An ultrasound
demonstrates a large cyst assumed to be renal pelvis and very dilated calyces with little renal tissue. The kidney
length is 6cm. A MAG3 scan reveals no tracer uptake or excretion on one side.

Which of the following statements concerning this child is most correct?

A Diagnosis of Multicystic Dysplastic Kidney (MCDK), unilateral nephrectomy next available list

B Pelvico-ureteric Obstruction, urgent pyeloplasty

C Child is too young for a MAG3 to be of use, a repeat scan at 6 months is indicated

D Posterior urethral valves with protective ‘pop-off’ phenomenon

E Vesico-ureteric obstruction. Ureteric stent required

Explanation
Multicystic dysplastic kidney has no function of nuclear imaging. A hydronephrosis secondary to PUJ obstruction
would be expected to have some secretion on a MAG-3 study. Often grossly dilated cysts can be misinterpreted as
PUJ obstruction. A MAG-3 allows differential diagnosis of PUJ/ MCDK to be performed. Tecnitium99 is taken up
by cells in the distal convoluted tubules from about 3-months of age, but some information can be obtained as
early as 6
weeks. MCDK > 5cm requires surgical excision as is associated with higher risk of infection,
developing carcinoma and hypertension.

Answer stem A is true, multiple cystic structures with no renal function are diagnostic of MCDK.

Answer
stem B is false, some tracer uptake and excretion would expected to be seen if pelvic-uteteric obstruction
was the diagnosis.

Answer
stem C is false, Technetium99 is taken up by the cells of the distal convoluted tubule from 3-months of
age, but from 6 weeks and assessment of whether or not there is functioning renal tissue can be made.

https://mypastest.pastest.com/Secure/TestMe/Browser/436619[‫ ص‬08:13:34 10/12/1437]


MyPastest

Answer stem D is false, as the scan gives no indication of a dilated ureter or thickened bladder wall.

Answer stem E is false; a dilated ureter is not seen on ultrasound scan.


45044
Next Question
Previous Question

Tag Question
End Session
Feedback

Difficulty: Average

Peer Responses

Session Progress

Responses Correct: 0

Responses Incorrect: 30

Responses Total: 30

Responses - % Correct: 0%

Blog
About Pastest
Contact Us
Help

© Pastest 2016

https://mypastest.pastest.com/Secure/TestMe/Browser/436619[‫ ص‬08:13:34 10/12/1437]


MyPastest

Prefer to use the old MyPastest? Access it here »

Back to Filters

Question 29 of 61

An 8-year-old boy is seen in clinic. The parents are worried because they are unable to see his testes. They state
they were present at birth. On examination both testes are clearly in the groin. On manipulation they can be bought
into the scrotum.

Which of the following statements best represents the diagnosis and the course of action?

A Bilateral Ascending testis, routine bilateral orchidopexy

B Bilateral Retractile testis, routine bilateral orchidopexy

C Bilateral Undescended testis, routine bilateral orchidopexy

D Bilateral Ascending testis, discharge with reassurance

E Bilateral retractile testis, discharge with reassurance

Explanation
Testicular descent can continue until about 3-months of age. The pre-pubertal testis is small enough that with
activation of the cremasteric reflex they can enter the inguinal canal.
This in itself is a normal finding providing
the testis can be brought into the scrotum without tension and does not require surgical correction. With time the
testis will lie within the scrotum. Orchidopexy for true undescended testis can be performed from 6-months of age
as there is evidence of sertoli cell up regulation in animal models from this stage and testis that remain
undescended or intra-abdominal for years have reduced fertility.

Answer
stem A is false, an ascending testis is one, typically, following hernia or orchidopexy surgery that was in
the testis but with time is within the inguinal canal and cannot be brought into the scrotum. Orchidopexy is
required for this.

Answer stem B
is false, this is the diagnosis, but no surgery is required as when the
testis enlarges with puberty it
will remain within the scrotum.

https://mypastest.pastest.com/Secure/TestMe/Browser/436619[‫ ص‬08:14:34 10/12/1437]


MyPastest

Answer stem C is false; the testes can be brought into the scrotum, therefore they are not undescended.

Answer
stem D is false, the testes are retractile not ascending; ascending testis cannot be brought into the scrotum
and would require orchidopexy.
Next Question
Answer stem E is true; this history is a normal finding in pre-pubertal children and resolves with puberty.
Previous Question 45045

End Session
Tag Question

Feedback

Difficulty: Average

Peer Responses

Session Progress

Responses Correct: 0

Responses Incorrect: 31

Responses Total: 31

Responses - % Correct: 0%

Blog
About Pastest
Contact Us
Help

© Pastest 2016

https://mypastest.pastest.com/Secure/TestMe/Browser/436619[‫ ص‬08:14:34 10/12/1437]


MyPastest

Prefer to use the old MyPastest? Access it here »

Back to Filters

Question 30 of 61

A 6-year-old boy is seen in clinic. The foreskin everts but is non-retractile, white material is seen coming from
inside the foreskin. The parents report that the foreskin balloons on micturition.

Which of the following is diagnosis and management plan?

A Balantitis xerotica obliterans, circumcision required

B Phimosis, circumcision required

C Phimosis, discharge with reassurance

D Balanitis secondary to candida, treat with antifungals

E Phimosis, prescribe 1% hydrocortisone and encourage retraction of foreskin

Explanation
At birth the foreskin is retractile in 1%
of babies. This increases with age and 1% of 16 years have a non-retractile
foreskin. The foreskin is attached to the glans of the penis by preputial adhesions and as they break down they can
cause mild erythema, ballooning of the foreskin on micturition and white debris. Physiological phimosis is a
normal non-retractile foreskin and with time
will become retractile. BXO is a thickened non-retractile foreskin
which is scarred and with time leads to a pin hole opening and is the only absolute medical indication for a
circumcision.

Answer stem A is false; BXO typically has a thickened white foreskin which does not evert and requires a
circumcision.

Answer
stem B is false the numbers needed to treat to prevent a UTI in a normal male child is 111 circumcisions,
this increases to 1 in 4 in gross bilateral vesico-ureteric reflux and 1 in 3 for posterior urethral
valves.

Answer stem C is true, this is a normal non-retractile foreskin, no treatment is required.

Answer stem D is false, the white discharge is the skin debris from the preputial adhesions breaking down.

https://mypastest.pastest.com/Secure/TestMe/Browser/436619[‫ ص‬08:15:08 10/12/1437]


MyPastest

Answer stem E is false, this can sometimes be used when there is thickening of the foreskin but mostly is not
required.
45046
Next Question
Previous Question

Tag Question
End Session
Feedback

Difficulty: Average

Peer Responses

Session Progress

Responses Correct: 0

Responses Incorrect: 32

Responses Total: 32

Responses - % Correct: 0%

Blog
About Pastest
Contact Us
Help

© Pastest 2016

https://mypastest.pastest.com/Secure/TestMe/Browser/436619[‫ ص‬08:15:08 10/12/1437]


MyPastest

Prefer to use the old MyPastest? Access it here »

Back to Filters

Question 31 of 61

A 6-month-old baby boy is seen in clinic with a non-palpable left testicle.

What is the most likely, next course of action?

A Perform an ultrasound scan

B Perform a CT scan

C Review in clinic in 1-years time as further descent is possible

D Diagnostic laparoscopy

E Groin exploration

Explanation
Testicular descent is possible up until 3-months of age. Current BAPU guidelines state that orchidopexy can be
performed after 6-months of age. If a testis is non-palpable it may be absent intra-abdominal, non-palpable but in
the groin due to fat distribution or an abnormal testis. No form of imagining is reliable enough to demonstrate
whether a testis is present or not. The correct course of action is to examine the child under anaesthesia and if the
testis is non palpable to perform a diagnostic laparoscopy to locate the
testis or see if the sperm duct and vessels
proceed through the internal inguinal ring in which case conversion to groin exploration is performed.

Answer stem A is false, ultrasound scan is not reliable to consistently demonstrate whether a testis is present in the
groin and may miss an intra-abdominal testis.

Answer stem B is false. A CT scan may not reliably locate a testis in the inguinal canal or intra-abdominally.

Answer
stem C is false, there is evidence in animal models that the testis already starts to undergo cell changes
from 3-months of age. Current BAPU guidelines are to perform orchidopexy from 6-months of age.

Answer stem D is true; this is the only reliable way to locate the testis.

https://mypastest.pastest.com/Secure/TestMe/Browser/436619[‫ ص‬08:15:47 10/12/1437]


MyPastest

Answer stem E is false, since the advent of laparoscopy which allows for more accurate diagnosis of an intra-
abdominal testis.
45048

Next Question
Previous Question

Tag Question
End Session
Feedback

Difficulty: Average

Peer Responses

Session Progress

Responses Correct: 0

Responses Incorrect: 33

Responses Total: 33

Responses - % Correct: 0%

Blog
About Pastest
Contact Us
Help

© Pastest 2016

https://mypastest.pastest.com/Secure/TestMe/Browser/436619[‫ ص‬08:15:47 10/12/1437]


MyPastest

Prefer to use the old MyPastest? Access it here »

Next Question
Previous Question
Back to Filters

Question 32 of 61 End Session

A 6-week old baby has a testis palpable in the groin but which cannot be manipulated into the scrotum.

What should be the next course of action?

A Routine Orchidopexy

B Review in clinic in 3-months and perform orchidopexy if still undescended

C Review in clinic in 6-months and perform orchidopexy if still undescended

D Review in clinic in 1-year and perform orchidopexy if still undescended

E Review in clinic in 18-months’ time and perform orchidopexy if still undescended.

Explanation
The correct time to perform routine orchidopexy is from 6-months of age. After this time testicular cells already
start to show signs of sertoli cell up regulation and reduced fertility.

Answer stems A and B are false, this baby is too young, the correct age is 6-months and testis may continue to
descend for 6-months.

Answer stems D and E are false - after the age of 6-months, testicular cells already start to undergo change.
45049

Tag Question

Feedback

https://mypastest.pastest.com/Secure/TestMe/Browser/436619[‫ ص‬08:17:37 10/12/1437]


MyPastest

Difficulty: Average

Peer Responses

Session Progress

Responses Correct: 0

Responses Incorrect: 34

Responses Total: 34

Responses - % Correct: 0%

Blog
About Pastest
Contact Us
Help

© Pastest 2016

https://mypastest.pastest.com/Secure/TestMe/Browser/436619[‫ ص‬08:17:37 10/12/1437]


MyPastest

Prefer to use the old MyPastest? Access it here »

Next Question

Back to Filters

Question 33 of 61

An 8-year-old girl presents with constant dripping of urine despite being able to void a full volume bladder. An
ultrasound scan shows both kidneys enlarged.

What is the most likely diagnosis?

A Horseshoe kidney

B Duplex kidney with ureterocele

C Duplex kidney with ectopic ureter

D Polycystic kidney disease

E Overactive bladder

Explanation
This is a typical history of a duplex system with an ectopic ureter inserting below the bladder neck. The Weigert-
Myer law states that the upper moiety ureter inserts inferior and medial to the lower moiety ureter.

Answer stem A is false as a horseshoe kidney would display as a single pelvic kidney on ultrasound scan.

Answer stem B is false, an ureterocele would be a cystic lesion within the bladder or may prolapse at birth.

Answer stem C is true, The upper moiety ureter inserts below the bladder neck and bladder control.

Answer stem D is false, continence should not be affected by polycystic kidney disease.

Answer stem E is false; an overactive bladder would cause frequency and urgency neither of which is mentioned
in the history.
45050

https://mypastest.pastest.com/Secure/TestMe/Browser/436619[‫ ص‬08:19:18 10/12/1437]


MyPastest

Tag Question

Feedback
Previous Question
Difficulty: Average

Peer Responses End Session

Session Progress

Responses Correct: 0

Responses Incorrect: 35

Responses Total: 35

Responses - % Correct: 0%

Blog
About Pastest
Contact Us
Help

© Pastest 2016

https://mypastest.pastest.com/Secure/TestMe/Browser/436619[‫ ص‬08:19:18 10/12/1437]


MyPastest

Prefer to use the old MyPastest? Access it here »

Next Question
Previous Question
Back to Filters

Question 34 of 61

A 7-year-old is admitted to hospital for severe gastroenteritis and dehydration requiring IV fluids. As part of his
monitoring he is having blood tests to detect for signs of acute kidney injury.

Which one of the following is NOT used to detect acute renal injury?

A 25% or more reduction in eGFR over 7 days

B 50% or more elevation in creatinine over 7 days

C Potassium >6mmol/l

D Rise in creatinine of >25micromol/l in 48 hours

E Urine output <0.5m/kg/hr for >8 hours

Explanation
Hyperkalaemia is the result of acute renal injury
and is not used to diagnose the condition. NICE guidelines on the
detection of acute renal injury, provides advice for the detection of acute renal injury in patients that are at risk for
this. If any one of the above factors are present (not including potassium level) then renal
injury should be
suspected and investigated.

Answer stems A, B, D and E are all used in the detection of acute renal injury.

Ref: http://pathways.nice.org.uk/pathways/acute-kidney-injury
47019

Tag Question

https://mypastest.pastest.com/Secure/TestMe/Browser/436619[‫ ص‬08:19:46 10/12/1437]


MyPastest

Feedback

Difficulty: Average

Peer Responses

End Session

Session Progress

Responses Correct: 0

Responses Incorrect: 36

Responses Total: 36

Responses - % Correct: 0%

Blog
About Pastest
Contact Us
Help

© Pastest 2016

https://mypastest.pastest.com/Secure/TestMe/Browser/436619[‫ ص‬08:19:46 10/12/1437]


MyPastest

Prefer to use the old MyPastest? Access it here »

Next Question

Back to Filters

Question 35 of 61

A 5-year-old is admitted to your ward for acute renal injury secondary to glomerulonephritis. He is receiving
medications to manage the complications of his condition. You are considering renal replacement therapy.

Which one of the following is NOT an indication for renal replacement therapy?

A Fluid overload unresponsive to diuretics

B Hyperkalaemia despite medications

C Hypertension on the 95th centile

D Metabolic acidosis despite medical treatment

E Pulmonary oedema

Explanation
NICE has produced criteria on when to refer acute
renal injury for renal replacement therapy such as dialysis.
Hypertension does not feature on this list.

Answer stems A, B and D are indications for referral for renal replacement therapy if they
are not responding to
medical therapy. The presence of pulmonary oedema may warrant referral regardless of medical management.
Additional
indications would be complications from uraemia such as encephalopathy.

Ref: http://pathways.nice.org.uk/pathways/acute-kidney-injury

http://www.emeesykidney.nhs.uk/professional-area/individual-guidelines
47020

https://mypastest.pastest.com/Secure/TestMe/Browser/436619[‫ ص‬08:22:15 10/12/1437]


MyPastest

Tag Question

Feedback

Previous Question Difficulty: Average

Peer Responses

End Session

Session Progress

Responses Correct: 0

Responses Incorrect: 37

Responses Total: 37

Responses - % Correct: 0%

Blog
About Pastest
Contact Us
Help

© Pastest 2016

https://mypastest.pastest.com/Secure/TestMe/Browser/436619[‫ ص‬08:22:15 10/12/1437]


MyPastest

Prefer to use the old MyPastest? Access it here »

Back to Filters

Question 36 of 61

A 6-year-old is admitted to the ward for investigation into glomerulonephritis, as he has peripheral oedema and
blood in his urine. A medical student asks whether glomerulonephritis causes acute renal injury that is pre-renal,
renal or obstructive to the kidney.

Which one of the list below results in pre-renal acute renal injury?

A Gastroenteritis

B Glomerulonephritis

C Haemolytic uraemic syndrome

D Henoch-SchÖnlein purpura

E Posterior urethral valves

Explanation
Gastroenteritis can result in intravascular volume depletion and reduced perfusion to the kidneys resulting in pre-
renal failure. Restoration of volume and perfusion will reverse the renal damage unless the insult has been
longstanding in which case acute
tubular necrosis can ensue from the hypoxic damage and resulting ischaemia.

B - This causes intrinsic renal damage.

C - This causes intrinsic renal damage and is reported to be the most common cause of acute renal injury in
children.

D - This causes intrinsic renal damage.

E - This causes obstructive renal damage in males.

Ref: http://emedicine.medscape.com/article/243492-overview#a5
47021

https://mypastest.pastest.com/Secure/TestMe/Browser/436619[‫ ص‬08:22:41 10/12/1437]


MyPastest

Next Question
Previous Question Tag Question

Feedback

Difficulty: Average End Session

Peer Responses

Session Progress

Responses Correct: 0

Responses Incorrect: 38

Responses Total: 38

Responses - % Correct: 0%

Blog
About Pastest
Contact Us
Help

© Pastest 2016

https://mypastest.pastest.com/Secure/TestMe/Browser/436619[‫ ص‬08:22:41 10/12/1437]


MyPastest

Prefer to use the old MyPastest? Access it here »

Back to Filters

Question 37 of 61

A 4-year-old presents with diarrhoea and vomiting and has not


passed urine for 24 hours. You suspect renal failure
and send bloods to
the lab. Whilst you are awaiting the results you consider that the child may have pre-renal
failure from gastroenteritis and fluid depletion, but consider intrinsic renal failure from Haemolytic uraemic
syndrome.

Which urine results below suggest pre-renal failure?

A Urine sodium <20mmol/l and urine osmolarity<350mosm/l

B Urine sodium <20mmol/l and urine osmolarity>500mosm/l

C Urine sodium >40mmol/l and urine osmolarity<350mosm/l

D Urine sodium >40mmol/l and urine osmolarity>350mosm/l

E Urine sodium >40mmol/l and urine osmolarity>500mosm/l

Explanation
The correct answer is B Urine sodium < 20mmol/l and urine osmolarity >500 mosm/l.

Gastroenteritis
is a common cause of hypovolaemia in children and can result in under perfusion of the kidneys
and pre-renal failure.In pre-renal failure the kidneys are still able to reabsorb sodium and concentrate urine as the
tubular cells are working, resulting in the above values [1].

A -
This suggests intrinsic renal failure as the kidney is unable to conserve sodium or concentrate the urine, this
form is seen with haemolytic uraemic syndrome.

C – This suggests you’re unable to reabsorb sodium but able to concentrate your urine and does not reflect either
form of renal injury.

D – This suggests you can reabsorb sodium but are unable to concentrate your urine and does not reflect either
form of renal injury.

https://mypastest.pastest.com/Secure/TestMe/Browser/436619[‫ ص‬08:29:58 10/12/1437]


MyPastest

E– This suggests you are unable to reabsorb sodium and partially able to concentrate your urine and does not
reflect either form of renal injury.

Next Question
Ref: [1] M Longmore et al. Oxford handbook of clinical medicine 8thed
47022
Previous Question

End Session
Tag Question

Feedback

Difficulty: Average

Peer Responses

Session Progress

Responses Correct: 0

Responses Incorrect: 39

Responses Total: 39

Responses - % Correct: 0%

Blog
About Pastest
Contact Us
Help

© Pastest 2016

https://mypastest.pastest.com/Secure/TestMe/Browser/436619[‫ ص‬08:29:58 10/12/1437]


MyPastest

Prefer to use the old MyPastest? Access it here »

Back to Filters

Question 38 of 61

A 10-year-old girl moves into your catchment area and presents to your clinic saying she has been monitored in
the past for her chronic renal failure from reflux nephropathy. You advise her of symptoms that can be the result
of chronic renal disease and arrange appropriate monitoring of her.

Which of the following is not seen in chronic renal disease?

A Anaemia

B Hypertension

C Hypocalcaemia

D Hypokalaemia

E Short stature

Explanation
In chronic kidney disease there is reduced renal excretion of potassium leading to hyperkalaemia which if left
untreated can be fatal. Hypoaldosteronismand metabolic acidosis, which both occur in chronic renal
failure,exacerbate this.

A – This occurs, in part, from reduced synthesis of erythropoietin.

B – This reflects renal osteodystrophy. The kidneys convert vitamin D into its active form, when they are unable
to do this calcium levels fall resulting in secondary hyperparathyroidism.

C – This is in part from an increase in vascular resistance and volume expansion.

E – This affects some children with chronic renal disease and appears to be multi-factorial in its cause.

Ref:

http://emedicine.medscape.com/article/984358-clinical

https://mypastest.pastest.com/Secure/TestMe/Browser/436619[‫ ص‬08:31:24 10/12/1437]


MyPastest

https://www.aakp.org/education/resourcelibrary/ckd-resources/item/growth-failure-in-children-with-chronic-
kidney-disease.html
47023
Next Question
Previous Question

Tag Question
End Session
Feedback

Difficulty: Average

Peer Responses

Session Progress

Responses Correct: 0

Responses Incorrect: 40

Responses Total: 40

Responses - % Correct: 0%

Blog
About Pastest
Contact Us
Help

© Pastest 2016

https://mypastest.pastest.com/Secure/TestMe/Browser/436619[‫ ص‬08:31:24 10/12/1437]


MyPastest

Prefer to use the old MyPastest? Access it here »

Back to Filters

Question 39 of 61

A 7-year-old comes to see you in clinic with her mother. She is dry during the day but has always had nocturnal
enuresis. Her mother is not keen for treatment, but would like lifestyle advice on the management of her condition.

Which of the following is most likely to be helpful and is in line with NICE guidance?

A Avoiding caffeine based drinks can help

B Fluid intake should be restricted to 75% of normal maintenance

C Lifting and waking promotes long term dryness

D Punishing wet nights can be helpful

E Rewarding dry nights can be helpful

Explanation
NICE advice is that caffeine based drinks be avoided. Adequate fluid intake should be maintained alongside a
healthy diet. Voiding urine throughout the day should be encouraged, as well as ensuring the toilet is used prior to
bed.

B - Normal maintenance fluids should be given, which may need to be adjusted for in hot weather or with
exercise.

C - Carrying or waking the child overnight to take them to the bathroom does not seem to result in long term
dryness.

D - Punishing is not helpful in the management of bed wetting.

E - Rewarding behaviours rather than dry nights is advised, for example


using the toilet well during the day, or
reaching their fluid requirements.

Ref:

https://mypastest.pastest.com/Secure/TestMe/Browser/436619[‫ ص‬08:41:54 10/12/1437]


MyPastest

https://www.nice.org.uk/guidance/cg111
47024

Next Question
Previous Question

Tag Question

Feedback End Session

Difficulty: Average

Peer Responses

Session Progress

Responses Correct: 0

Responses Incorrect: 41

Responses Total: 41

Responses - % Correct: 0%

Blog
About Pastest
Contact Us
Help

© Pastest 2016

https://mypastest.pastest.com/Secure/TestMe/Browser/436619[‫ ص‬08:41:54 10/12/1437]


MyPastest

Prefer to use the old MyPastest? Access it here »

Back to Filters

Question 40 of 61

You see a child in clinic who is complaining of wetting herself. It is causing her and her family much distress and
they would like something to stop it. You ask about lifestyle measures and are happy that she has appropriate
toileting use and fluid intake. You consider desmopressin.

Which one of the following would STOP you from prescribing desmopressin?

A Child aged 6

B Day and night wetting

C Daytime wetting only

D Learning difficulties

E Sickle cell disease

Explanation
Desmopressin is a synthetic form of vasopressin/anti-diuretic hormone and helps to prevent urinary voiding. NICE
guidelines on management of enuresis states that desmopressin should not be used for daytime wetting only,
where there are no problems
with nocturnal enuresis.

A - Children over 7 can be offered desmopressin. However it may be considered for children aged 5-7 if felt to be
needed.

B - It can be considered in children who have both day and night wetting.

D - If lifestyle measures are being adhered to then desmopressin can be


considered in children with learning or
developmental problems.

E – Damage to the kidneys from sickle cells blocking the vascular


supply can result in nocturnal enuresis.
Desmopressin can be used but should be stopped in times of a sickle crisis.

https://mypastest.pastest.com/Secure/TestMe/Browser/436619[‫ ص‬08:43:28 10/12/1437]


MyPastest

Ref: https://www.nice.org.uk/guidance/cg111
47025

Next Question
Previous Question

Tag Question

Feedback End Session

Difficulty: Average

Peer Responses

Session Progress

Responses Correct: 0

Responses Incorrect: 42

Responses Total: 42

Responses - % Correct: 0%

Blog
About Pastest
Contact Us
Help

© Pastest 2016

https://mypastest.pastest.com/Secure/TestMe/Browser/436619[‫ ص‬08:43:28 10/12/1437]


MyPastest

Prefer to use the old MyPastest? Access it here »

Back to Filters

Question 41 of 61

You are the registrar for the specialist nocturnal enuresis clinic. The first patient you see is an 8-year-old girl who
has always had nocturnal enuresis and daytime wetting. She has tried desmopressin and had a slight benefit but is
still wetting at night, with occasional episodes during the day.

Which treatment could you consider adding in to her care, after discussion with your consultant?

A Anticholinergic

B Furosemide

C Imipramine

D Spironolactone

E Thiazides

Explanation
If desmopressin fails to work or only partially works, then a specialist with expertise in prescribing combination
therapy can prescribe an anticholinergic as an adjunct. This should not be used for nocturnal enuresis without
daytime wetting and should not be
combined with tricyclic therapy.

B – This is a diuretic and not indicated.

C - Imipramine is a Tricyclic medication and can be used when other measures fail. It should be prescribed by a
specialist.

D - This is a diuretic and not indicated.

E - This is a diuretic and not indicated.

Ref: https://www.nice.org.uk/guidance/cg111
47026

https://mypastest.pastest.com/Secure/TestMe/Browser/436619[‫ ص‬08:45:57 10/12/1437]


MyPastest

Next Question
Previous Question Tag Question

Feedback

Difficulty: Average End Session

Peer Responses

Session Progress

Responses Correct: 0

Responses Incorrect: 43

Responses Total: 43

Responses - % Correct: 0%

Blog
About Pastest
Contact Us
Help

© Pastest 2016

https://mypastest.pastest.com/Secure/TestMe/Browser/436619[‫ ص‬08:45:57 10/12/1437]


MyPastest

Prefer to use the old MyPastest? Access it here »

Back to Filters

Question 42 of 61

You see an 8-year-old in clinic who has primary nocturnal enuresis each night. She is taking good amounts of
uncaffeinated fluids and is using the toilet appropriately during the day. She would like to try treatment for her
bedwetting and the family would prefer long term results.

Which of the following is the most suitable first line therapy in this case?

A Alarm treatment

B Anticholinergic therapy

C Desmopressin

D Reward system

E Tricyclics

Explanation
Alarms offer more long term solutions than desmopressin. Desmopressin is better for immediate relief of
symptoms for example, if a child is going to a sleep over with friends. Alarms should be tried for 4 weeks then the
patient is reassessed to see if things are beginning to improve. There are different versions such as buzzers for the
hearing impaired.

B – Anticholinergic therapy is second line treatment, started by specialists in nocturnal enuresis

C - Desmopressin tends to give more immediate relief of symptoms and is less likely to result in long term relief
than alarms

D – Had there been some dry nights then a reward system could have been used

E - Tricyclics are a second line treatment, started by specialists in nocturnal enuresis

Ref: https://www.nice.org.uk/guidance/cg111
47027

https://mypastest.pastest.com/Secure/TestMe/Browser/436619[‫ ص‬08:46:19 10/12/1437]


MyPastest

Next Question
Previous Question Tag Question

Feedback

End Session
Difficulty: Average

Peer Responses

Session Progress

Responses Correct: 0

Responses Incorrect: 44

Responses Total: 44

Responses - % Correct: 0%

Blog
About Pastest
Contact Us
Help

© Pastest 2016

https://mypastest.pastest.com/Secure/TestMe/Browser/436619[‫ ص‬08:46:19 10/12/1437]


MyPastest

Prefer to use the old MyPastest? Access it here »

Back to Filters

Question 43 of 61

A 7-year-old boy presents to your clinic with secondary nocturnal enuresis. He is otherwise fit and well and has no
family history of illness. He has a good fluid intake but is a fussy eater. On examination he has a mass in the left
iliac fossa. Mum thinks he is doing well at school, although he has recently changed schools.

What is the likely cause of this boy’s enuresis?

A Constipation

B Emotional upset

C Sickle cell disease

D Type 1 diabetes

E Urinary tract infection

Explanation
Constipation is not an uncommon cause of enuresis. The hard stool presses on the bladder resulting in the leakage
of urine. The mass in the left iliac fossa is the impacted stool. Faecal disimpaction is necessary, followed by
maintenance therapy, and should result in the resolution of his symptoms.

B - This can cause secondary enuresis. It may be from a change in schools, being bullied, parental separation.
Child abuse can also result
in this and needs to be considered. Here the child is happy and has a mass in the
abdomen.

C - Sickle cell disease can result in enuresis, however is unlikely in this case.

D - Polyuria from diabetes can be a cause of secondary nocturnal enuresis. Urine dip for glucose will help exclude
this, along with the history. This child has no supporting features of this diagnosis.

E - Can result in leakage of urine and voiding difficulties. Urine dip


will be able to confirm this. This child has a
mass in the abdomen which a UTI does not explain.

https://mypastest.pastest.com/Secure/TestMe/Browser/436619[‫ ص‬08:46:39 10/12/1437]


MyPastest

47028

Next Question
Previous Question
Tag Question

Feedback
End Session
Difficulty: Average

Peer Responses

Session Progress

Responses Correct: 0

Responses Incorrect: 45

Responses Total: 45

Responses - % Correct: 0%

Blog
About Pastest
Contact Us
Help

© Pastest 2016

https://mypastest.pastest.com/Secure/TestMe/Browser/436619[‫ ص‬08:46:39 10/12/1437]


MyPastest

Prefer to use the old MyPastest? Access it here »

Back to Filters

Question 44 of 61

A 7-year-old presents to your assessment unit with periorbital oedema. A urine dipstick is 3+ positive for blood
and 1+ positive for protein. From the history you discover that he had a pharyngitis 10 days ago. There is no past
medical history or relevant family history.

What is the most likely diagnosis?

A Alports syndrome

B Glomerulonephritis

C IgA nephropathy

D Nephrotic syndrome

E Wilms tumour

Explanation
This child has post streptococcus glomerulonephritis (GMN). This is the cause of 80% of paediatric cases of
GMN. Signs of renal injury occur 1-2 weeks following the throat infection, unlike IgA nephropathy which occurs
at the time of infection.
There may be some protein as well as blood in the urine, however nephrotic syndrome
requires at least 3+ protein on dipstick to be considered.

Alports syndrome is an X-linked dominant condition associated with deafness, a family history would also be
expected.

IgA nephropathy, also called Berger’s disease, presents with haematuria at the time of an upper respiratory tract
infection.

Nephrotic syndrome requires at least 3+ protein on urine dip.

Wilms tumour - a nephroblastoma, is more likely to be found with an abdominal mass. There may be evidence of
associated syndromes or associations such as hemi-hypertrophy.

https://mypastest.pastest.com/Secure/TestMe/Browser/436619[‫ ص‬08:47:00 10/12/1437]


MyPastest

Ref: http://www.emeesykidney.nhs.uk/professional-area/individual-guidelines
47029

Next Question
Previous Question

Tag Question

Feedback End Session

Difficulty: Average

Peer Responses

Session Progress

Responses Correct: 0

Responses Incorrect: 46

Responses Total: 46

Responses - % Correct: 0%

Blog
About Pastest
Contact Us
Help

© Pastest 2016

https://mypastest.pastest.com/Secure/TestMe/Browser/436619[‫ ص‬08:47:00 10/12/1437]


MyPastest

Prefer to use the old MyPastest? Access it here »

Back to Filters

Question 45 of 61

A 2-day-old baby girl is referred to you for assessment as her mother has noticed blood when changing her
daughter’s nappy. On examination there is an orange-red discolouration in the nappy. The baby had normal
antenatal scans and was given IM Vitamin K at birth, she
is currently breast fed.

What is the most likely diagnosis?

A Haemorrhagic disease of the newborn

B Polycystic kidney disease

C Urate crystals

D Vaginal bleed

E Wilms tumour

Explanation
If urine becomes concentrated in the neonate then
urate crystals can become deposited within the nappy. These
may look red on initial examination, however on closer inspection usually have a slight orange colouring to them.
They can be a sign of concentrated urine however in the absence of dehydration, only reassurance is needed.

Haemorrhagic disease of the newborn occurs due to vitamin K deficiency and there have been reported cases of
haematuria resulting from this. It is rare following routine administration of Vitamin K at birth, which we know
from the history, this neonate has had.

B -
The autosomal recessive form of polycystic kidney disease can occur in neonates and result in haematuria,
oliguria or polyuria. It may be associated with abdominal masses and Potter syndrome, which are not seen
in this
case.

Vaginal bleed occurs due to maternal oestrogen withdrawal and should resolve within a week from birth, however
this would look red rather than orange.

https://mypastest.pastest.com/Secure/TestMe/Browser/436619[‫ ص‬08:48:33 10/12/1437]


MyPastest

Wilms tumour may present in neonates and be associated with an abdominal mass or syndrome, neither of which
is present here.
47030

Next Question
Previous Question

Tag Question
End Session
Feedback

Difficulty: Average

Peer Responses

Session Progress

Responses Correct: 0

Responses Incorrect: 47

Responses Total: 47

Responses - % Correct: 0%

Blog
About Pastest
Contact Us
Help

© Pastest 2016

https://mypastest.pastest.com/Secure/TestMe/Browser/436619[‫ ص‬08:48:33 10/12/1437]


MyPastest

Prefer to use the old MyPastest? Access it here »

Back to Filters

Question 46 of 61

A 4-year-old boy is referred for assessment as he has had diarrhoea for 2 days which has now developed blood
staining to it. The child looks mildly dehydrated, but well perfused with a capillary refill
time of <2 seconds
centrally. He is able to produce a concentrated urine sample which dips positive for blood. You admit him for oral
rehydration, but see some petechiae on his legs so perform bloods which you send to the lab.

What is the most likely reason for his haematuria?

A Contamination from faecal blood

B Haemolytic Uraemic Syndrome

C Henoch-SchÖnlein purpura

D Salmonella food poisoning

E Sepsis

Explanation
The history of bloody diarrhoea and haematuria suggests haemolytic uraemic syndrome (HUS). The petechiae are
from the associated thrombocytopenia, and his bloods may also show anaemia and renal impairment. In children
this condition usually results from E.coli O157
infection and there may be a history of farm contact or recent BBQ
eating under-cooked burgers. It is the toxin which causes the symptoms and as such antibiotics should not be
given, unless sepsis is evident, as these can cause bacterial wall lysis and release of more toxin.

Henoch-SchÖnlein
purpura presents following a recent illness with haematuria and a purpuric rash, usually on the
legs, there may also be joint swelling and
abdominal pain.

Contamination from faecal blood is unlikely as it does not explain the petechial rash.

Salmonella food poisoning is unlikely as there is no history of food ingestion given.

Diarrhoea and petechiae could be a sign of sepsis however he is well perfused making this option less likely.

https://mypastest.pastest.com/Secure/TestMe/Browser/436619[‫ ص‬08:48:57 10/12/1437]


MyPastest

47031

Next Question
Previous Question
Tag Question

Feedback
End Session
Difficulty: Average

Peer Responses

Session Progress

Responses Correct: 0

Responses Incorrect: 48

Responses Total: 48

Responses - % Correct: 0%

Blog
About Pastest
Contact Us
Help

© Pastest 2016

https://mypastest.pastest.com/Secure/TestMe/Browser/436619[‫ ص‬08:48:57 10/12/1437]


MyPastest

Prefer to use the old MyPastest? Access it here »

Back to Filters

Question 47 of 61

A 5-year-old boy presents to your assessment unit with a palpable purpuric rash to his legs and buttocks, with
associated swelling and pain in his right knee, resulting in him limping. He has had a recent viral upper respiratory
tract infection but is otherwise fit and well. A urine dip reveals haematuria only.

What is the most likely diagnosis?

A Haemophilia

B Henoch-SchÖnlein purpura

C Septic arthritis

D Transient synovitis

E Urinary tract infection

Explanation
Henoch-SchÖnlein purpura (HSP) is an IgA vasculitis that affects young children, with a slight predominance for
males. It may be preceded by an upper respiratory tract infection. It is associated with a palpable purpura that
tends to affect the legs and buttocks but can be seen in other locations such as the arms. There may
be joint
swelling, scrotal oedema and abdominal pain which may progress
to intussusception in a minority. Renal injury
can result in nephrotic
syndrome but only rarely causes renal failure.

Bleeding in haemophilia could cause haematuria and haemarthrosis, however the case above is more classic of
HSP.

Septic arthritis would explain the limp but not the rash.

Transient synovitis occurs after a viral illness and can cause a limp, but does not explain the rash.

Haematuria can be a sign of a UTI, but doesn’t explain the other features present.

https://mypastest.pastest.com/Secure/TestMe/Browser/436619[‫ ص‬08:49:18 10/12/1437]


MyPastest

Ref:

http://www.niddk.nih.gov/health-information/health-topics/kidney-disease/henoch-sch%C3%B6nlein-purpura-
hsp/Pages/facts.aspx
Next Question 47032

Previous Question

End Session
Tag Question

Feedback

Difficulty: Average

Peer Responses

Session Progress

Responses Correct: 0

Responses Incorrect: 49

Responses Total: 49

Responses - % Correct: 0%

Blog
About Pastest
Contact Us
Help

© Pastest 2016

https://mypastest.pastest.com/Secure/TestMe/Browser/436619[‫ ص‬08:49:18 10/12/1437]


MyPastest

Prefer to use the old MyPastest? Access it here »

Back to Filters

Question 48 of 61

A child presents with periorbital and peripheral oedema. You


perform a urine dip which shows 3+ of protein. You
suspect nephrotic syndrome and perform relevant blood tests.

Which of the following suggests this is atypical nephrotic syndrome?

A Age more than 11 years

B Microscopic haematuria

C No family history

D Normal creatinine

E Normotensive

Explanation
The majority of children with nephrotic syndrome respond to steroid therapy and have minimal change disease;
those who have atypical presentations are less likely to follow this pattern. Being older than 11 years or younger
than 1 year is an atypical presentation and warrants discussion with the paediatric nephrologist for possible biopsy
prior to therapy.

Macroscopic not microscopic haematuria is an atypical feature.

Elevated creatinine is an atypical feature.

Hypertension is an atypical feature.

Family history of nephrotic syndrome is atypical.

Ref: http://www.emeesykidney.nhs.uk
47033

https://mypastest.pastest.com/Secure/TestMe/Browser/436619[‫ ص‬08:49:45 10/12/1437]


MyPastest

Next Question
Previous Question Tag Question

Feedback

Difficulty: Average End Session

Peer Responses

Session Progress

Responses Correct: 0

Responses Incorrect: 50

Responses Total: 50

Responses - % Correct: 0%

Blog
About Pastest
Contact Us
Help

© Pastest 2016

https://mypastest.pastest.com/Secure/TestMe/Browser/436619[‫ ص‬08:49:45 10/12/1437]


MyPastest

Prefer to use the old MyPastest? Access it here »

Back to Filters

Question 49 of 61

A 13-year-old boy is referred to you as his GP has incidentally noticed protein in the urine on two occasions when
the child came to see him after school. The boy is healthy and has no signs
of oedema or recent infection. He is the
first child you see in your morning clinic and the urine dip you obtain is negative for protein.

What was the most likely cause of this child’s proteinuria?

A Alports syndrome

B Cystitis

C Nephritic syndrome

D Nephrotic syndrome

E Orthostatic proteinuria

Explanation
Orthostatic proteinuria occurs when the kidneys are able to conserve urine when the patient is recumbent, such as
sleeping at night, but leak protein with standing or in exercise. This results in early morning urine being negative
for protein but late in the day urine being positive. It is mostly seen in tall thin adolescents and is benign.

Alport’s syndrome is a hereditary condition associated with haematuria and deafness.

Urinary tract infections can cause proteinuria, but leucocyte esterase and nitrites would also be expected in a child
of this age.

Nephritic syndrome can be associated with proteinuria, but haematuria would also be present.

In nephrotic syndrome, proteinuria would be present on all occasions and associated with oedema.
47034

https://mypastest.pastest.com/Secure/TestMe/Browser/436619[‫ ص‬08:50:17 10/12/1437]


MyPastest

Next Question
Previous Question Tag Question

Feedback

Difficulty: Average End Session

Peer Responses

Session Progress

Responses Correct: 0

Responses Incorrect: 51

Responses Total: 51

Responses - % Correct: 0%

Blog
About Pastest
Contact Us
Help

© Pastest 2016

https://mypastest.pastest.com/Secure/TestMe/Browser/436619[‫ ص‬08:50:17 10/12/1437]


MyPastest

Prefer to use the old MyPastest? Access it here »

Back to Filters

Question 50 of 61

An 8-year-old with previous nephrotic syndrome presents to your assessment unit as they have proteinuria.

Which of the following is required for diagnosis of a relapse?

A Any proteinuria detected

B Protein 2+ or more for three consecutive days

C Protein 2+ or more on one occasion

D Protein 3+ or more for three consecutive days

E Protein 3+ or more on one occasion

Explanation
More than 80% of children will have a relapse and
parents should be warned about this and advised how to
monitor for this
at initial presentation. Relapse is said to occur when the child has Protein 3+ or more for three
consecutive days; treatment is with steroids.

Stem A is incorrect as less than 3+ does not reflect a relapse.

Stem B is incorrect as more proteinuria is required and for a longer duration.

Stem C is the correct number of days required, however the level of protein detected is too low to suggest relapse.

Stem E is the correct amount of protein, but a longer duration is required.

Ref:
http://www.emeesykidney.nhs.uk/professional-area/individual-guidelines
http://www.rch.org.au/clinicalguide/guideline_index/Nephrotic_Syndrome/
47035

https://mypastest.pastest.com/Secure/TestMe/Browser/436619[‫ ص‬08:51:22 10/12/1437]


MyPastest

Next Question
Previous Question Tag Question

Feedback

Difficulty: Average End Session

Peer Responses

Session Progress

Responses Correct: 0

Responses Incorrect: 52

Responses Total: 52

Responses - % Correct: 0%

Blog
About Pastest
Contact Us
Help

© Pastest 2016

https://mypastest.pastest.com/Secure/TestMe/Browser/436619[‫ ص‬08:51:22 10/12/1437]


MyPastest

Prefer to use the old MyPastest? Access it here »

Back to Filters

Question 51 of 61

A 5-year-old presents to your assessment unit with headaches.


You perform a blood pressure measurement using
an appropriate size cuff and find that his systolic blood pressure is just above the 95th centile. A urine dip is
positive for both protein and blood. He is not
oedematous and has no recent history of infection. Mum remembers
that his grandmother took BP medication all her life.

What is the most likely cause of his BP?

A Brain tumour

B Coarctation of the aorta

C Glomerulonephritis

D Nephrotic syndrome

E Polycystic kidney disease

Explanation
Polycystic kidney disease (PCKD) is the most likely answer in view of the blood and protein in the urine as well
as the family history, since childhood PCKD is inherited in an autosomal recessive manner. Blood and protein in
urine of hypertensive patients suggests a renal cause, however hypertension can cause end organ failure
such as
renal disease. End organ failure tends to be due to longstanding severe hypertension, although it can be difficult to
determine cause and effect.

A brain tumour can cause hypertension, however blood and protein in the urine suggests renal disease.

In coarctation of the aorta, BP would be higher in the arms than the legs and femoral pulses may feel weak or
difficult to detect. Here blood and protein in the urine suggests renal disease.

The lack of a recent illness in the history makes glomerulonephritis less likely.

The lack of oedema in the history makes nephrotic syndrome less likely.

https://mypastest.pastest.com/Secure/TestMe/Browser/436619[‫ ص‬08:52:58 10/12/1437]


MyPastest

Ref:

http://www.emeesykidney.nhs.uk/professional-area/individual-guidelines
Next Question 47036

Previous Question

End
Tag Session
Question

Feedback

Difficulty: Average

Peer Responses

Session Progress

Responses Correct: 0

Responses Incorrect: 53

Responses Total: 53

Responses - % Correct: 0%

Blog
About Pastest
Contact Us
Help

© Pastest 2016

https://mypastest.pastest.com/Secure/TestMe/Browser/436619[‫ ص‬08:52:58 10/12/1437]


MyPastest

Prefer to use the old MyPastest? Access it here »

Back to Filters

Question 52 of 61

A 4-year-old sees their GP with a 2 week history of polydipsia and polyuria. Mum is unsure if there has been any
weight loss but thinks he may have had the flu recently as he appears “run down”. Urine dip is positive for glucose
and ketones. The child is well perfused with no abdominal pain or vomiting. The GP calls you to ask about
management of this child.

Which of the following is the most appropriate answer?

A Ambulance to the emergency department for diabetic ketoacidosis (DKA) protocol

B Arrange serum blood glucose test

C Refer to rapid access clinic

D Send to children’s assessment unit today

E Urgent referral to diabetes clinic

Explanation
This child potentially has type 1 diabetes and needs to be seen on the same day that the problem is identified. The
child does not have any features of DKA and can most likely be started on subcutaneous insulin once initial
bloods for a newly diagnosed diabetic are performed, including a gas. Advice in these cases should always be to
send the child in immediately to the assessment unit to prevent them entering into DKA.

As this child does not have abdominal pain or vomiting, he is unlikely to have DKA. Blood gas should be obtained
to confirm this, then subcutaneous insulin started so
stem A is false.

Arranging for a serum blood glucose test risks


delaying the diagnosis and resulting in the child entering DKA. If
the
diagnosis is suspected the child needs to be seen by a paediatrician immediately and they will perform same
day bloods so stem B is false.

Again, referring to a rapid access clinic risks delaying the diagnosis and resulting in the child entering DKA so D

https://mypastest.pastest.com/Secure/TestMe/Browser/436619[‫ ص‬08:53:36 10/12/1437]


MyPastest

is an incorrect response.

Even urgent referral to a diabetes clinic risks delaying the diagnosis and resulting in the child entering DKA so is
an inappropriate response.
Next Question
Ref:
Previous Question
http://pathways.nice.org.uk/pathways/diabetes-in-children-and-young-people#
47037

End Session

Tag Question

Feedback

Difficulty: Average

Peer Responses

Session Progress

Responses Correct: 0

Responses Incorrect: 54

Responses Total: 54

Responses - % Correct: 0%

Blog
About Pastest
Contact Us
Help

© Pastest 2016

https://mypastest.pastest.com/Secure/TestMe/Browser/436619[‫ ص‬08:53:36 10/12/1437]


MyPastest

Prefer to use the old MyPastest? Access it here »

Back to Filters

Question 53 of 61

A 6-year-old presents to your assessment unit as mum is concerned that he may have cancer. He has a 2 week
history of weight loss and lethargy. On further questioning you discover that he drinks excessively and there is a
strong family history of autoimmune conditions.

What is the most likely cause of his symptoms?

A Diabetes insipidus

B Gastroenteritis

C Habitual polydipsia

D Leukaemia

E Type 1 diabetes mellitus

Explanation
Lethargy, weight loss and polydipsia with a family history of autoimmune conditions is strongly suggestive of
type 1
diabetes. The polydipsia is due to polyuria from glucose being present
in the urine. A blood glucose and
urine dip will help confirm your diagnosis. This child requires insulin to halt the catabolic state that
his body is in.

The family history of autoimmune conditions makes diabetes insipidus less likely as a diagnosis here.

Excessive losses from gastroenteritis could cause these symptoms however this child is not experiencing diarrhoea
or vomiting.

Habitual polydipsia is less likely given the weight loss and lethargy.

Leukemia
can present with the above features described in the history, including
polydipsia and polyuria, but the
family history of autoimmune conditions makes diabetes more likely in this case.
47038

https://mypastest.pastest.com/Secure/TestMe/Browser/436619[‫ ص‬08:54:03 10/12/1437]


MyPastest

Next Question
Previous Question Tag Question

Feedback

Difficulty: Average End Session

Peer Responses

Session Progress

Responses Correct: 0

Responses Incorrect: 55

Responses Total: 55

Responses - % Correct: 0%

Blog
About Pastest
Contact Us
Help

© Pastest 2016

https://mypastest.pastest.com/Secure/TestMe/Browser/436619[‫ ص‬08:54:03 10/12/1437]


MyPastest

Prefer to use the old MyPastest? Access it here »

Back to Filters

Question 54 of 61

A 3-year-old with polyuria and polydipsia presents for a water deprivation test. Previous urine dip has been
negative for glucose, protein and blood. The test is started and once he loses 3% of
his body weight, blood and
urine osmolarity are performed which show levels of >300 and <300 respectively. Desmopressin is given which
results in his urine osmolarity increasing to >750.

What is the most likely cause of this child’s polydipsia and polyuria?

A Acute renal failure

B Cranial Diabetes Insipidus

C Diabetes Mellitus

D Habitual polydipsia

E Nephrogenic Diabetes Insipidus

Explanation
During the water deprivation test, this child had
a concentrated serum osmolarity and yet he was not concentrating
his urine to try and correct for this, indicating diabetes insipidus. When Desmopressin was administered it resulted
in him being able to concentrate his urine. Therefore his kidneys are responding to Desmopressin but he must not
be producing his own endogenous form of antidiuretic hormone, resulting in the diagnosis of cranial DI. The next
step would be to obtain an MRI to investigate the reason for this.

Acute renal failure usually results in oliguria so stem A is incorrect.

Diabetes Mellitus should have been investigated for prior to this stage
of setting up a water deprivation test. The
absence of glucose in the urine dip excludes this as the cause.

In habitual polydipsia the


child should be able to concentrate his urine appropriately to water deprivation so stem
D is false.

https://mypastest.pastest.com/Secure/TestMe/Browser/436619[‫ ص‬08:54:24 10/12/1437]


MyPastest

In nephrogenic diabetes insipidus the kidneys are not responding to endogenous antidiuretic hormone and so
would not be able to concentrate urine to Desmopressin either. Stem E is therefore incorrect.

Ref: http://emedicine.medscape.com/article/919886-workup#c9
Next Question 47039

Previous Question

End Session
Tag Question

Feedback

Difficulty: Average

Peer Responses

Session Progress

Responses Correct: 0

Responses Incorrect: 56

Responses Total: 56

Responses - % Correct: 0%

Blog
About Pastest
Contact Us
Help

© Pastest 2016

https://mypastest.pastest.com/Secure/TestMe/Browser/436619[‫ ص‬08:54:24 10/12/1437]


MyPastest

Prefer to use the old MyPastest? Access it here »

Back to Filters

Question 55 of 61

A 6-year-old with polyuria and polydipsia presents for a water deprivation test. Previous urine dip has been
negative for glucose, protein and blood. The test is started and once he loses 3% of his body weight, his blood
osmolarityis >300 and his urine osmolarity is <300. Desmopressin is given but no change in his osmolarity is
seen.

What is the likely cause of this child’s polydipsia and polyuria?

A Acute renal failure

B Cranial Diabetes Insipidus

C Diabetes Mellitus

D Habitual polydipsia

E Nephrogenic Diabetes Insipidus

Explanation
In this scenario, despite depriving the child of fluids, he is still producing large volumes of dilute urine suggesting
diabetes insipidus. When Desmopressin (an exogenous version of antidiuretic hormone) is given there is no
change in his ability to concentrate urine, therefore his kidneys are not responding to the hormone and he has renal
diabetes insipidus. This condition can be congenital or acquired.

Acute renal failure is associated with oliguria so stem A is incorrect.

In cranial DI there is a problem with the hypothalamus’ production of antidiuretic hormone. Therefore when
exogenous Desmopressin is given the kidneys are able to respond to this and concentrate urine unlike in this
scenario, so stem B is incorrect.

The absence of glycosuria goes against a diagnosis of diabetes mellitus.

The inability to concentrate urine to water deprivation makes habitual polydipsia unlikely.

https://mypastest.pastest.com/Secure/TestMe/Browser/436619[‫ ص‬08:54:47 10/12/1437]


MyPastest

Ref:

http://emedicine.medscape.com/article/919886-workup#c9
Next Question 47040

Previous Question

End
Tag Session
Question

Feedback

Difficulty: Average

Peer Responses

Session Progress

Responses Correct: 0

Responses Incorrect: 57

Responses Total: 57

Responses - % Correct: 0%

Blog
About Pastest
Contact Us
Help

© Pastest 2016

https://mypastest.pastest.com/Secure/TestMe/Browser/436619[‫ ص‬08:54:47 10/12/1437]


MyPastest

Prefer to use the old MyPastest? Access it here »

Back to Filters

Question 56 of 61

A 2-year-old undergoes a water deprivation test for polydipsia and polyuria. Urine dip is negative for glucose,
blood and protein. During the water deprivation test his plasma osmolarity is 285
and his urine osmolarity
increases to >750, he is very distressed by
the test.

What is the most likely cause of his polydipsia and polyuria?

A Acute renal failure

B Cranial Diabetes Insipidus

C Diabetes Mellitus

D Habitual polydipsia

E Nephrogenic Diabetes Insipidus

Explanation
Habitual polydipsia, also known as primary polydipsia, is seen in toddlers and preschool children. Serum
osmolarity is usually in the lower end of normal although occasionally it may fall to below normal and be
associated with hyponatraemia. In most cases children are able to concentrate their urine when water is restricted,
however if their polydipsia has been prolonged there may be a
“washout effect” where the renal medulla has
reduced osmolarity and so cannot concentrate the urine effectively. This situation can cause a delayed
concentrating response to water deprivation.

A – This usually results in oliguria.

B – This would be associated with an inappropriately dilute urine for plasma osmolarity.

C – The absence of glycosuria makes this unlikely.

E– This would be associated with an inappropriately dilute urine for plasma osmolarity.

https://mypastest.pastest.com/Secure/TestMe/Browser/436619[‫ ص‬08:55:08 10/12/1437]


MyPastest

Ref:

http://emedicine.medscape.com/article/919886-workup#c9
47041
Next Question
Previous Question

Tag Question
End Session
Feedback

Difficulty: Average

Peer Responses

Session Progress

Responses Correct: 0

Responses Incorrect: 58

Responses Total: 58

Responses - % Correct: 0%

Blog
About Pastest
Contact Us
Help

© Pastest 2016

https://mypastest.pastest.com/Secure/TestMe/Browser/436619[‫ ص‬08:55:08 10/12/1437]


MyPastest

Prefer to use the old MyPastest? Access it here »

Back to Filters

Question 57 of 61

A 4-year-old with high fevers, has a clean catch urine dip performed. It is positive for leucocyte esterase but
negative for nitrites.

What is the most suitable next step?

A Discharge home and await culture

B Look for an alternative source of infection

C Obtain repeat sample in case of contamination

D Start IV antibiotics

E Start oral antibiotics

Explanation
This child is capable of holding urine within the
bladder and therefore could produce nitrites to common
pathogens. The absence of these makes a UTI less likely, however, if there were urinary
symptoms then a UTI
could still be your diagnosis. As this child is only having fevers, the presence of leucocytes may reflect illness
elsewhere in the body, such as a viral illness or appendicitis. Microscopy should be obtained and an alternative
source of infection looked for. As this was a clean catch sample, contamination is less likely.

A – These results in the absence of urinary symptoms may reflect an infection somewhere else in the body and this
needs to be looked for and treated.

C – As the sample was a clean catch (the gold standard) contamination is unlikely.

D – In the absence of urinary symptoms this dip does not suggest a


UTI and antibiotics should not be prescribed
without a focus. Additionally oral antibiotics in a well child of this age usually suffice.

E - In the absence of urinary symptoms this dip does not suggest a UTI and antibiotics should not be prescribed
without a focus.

https://mypastest.pastest.com/Secure/TestMe/Browser/436619[‫ ص‬08:55:29 10/12/1437]


MyPastest

REF: https://www.nice.org.uk/guidance/cg54
47042

Next Question
Previous Question

Tag Question

Feedback End Session

Difficulty: Average

Peer Responses

Session Progress

Responses Correct: 0

Responses Incorrect: 59

Responses Total: 59

Responses - % Correct: 0%

Blog
About Pastest
Contact Us
Help

© Pastest 2016

https://mypastest.pastest.com/Secure/TestMe/Browser/436619[‫ ص‬08:55:29 10/12/1437]


MyPastest

Prefer to use the old MyPastest? Access it here »

Back to Filters

Question 58 of 61

A 4-year-old has a positive urine dip for nitrites and leucocytes. You are going to start antibiotics and know that
for an upper UTI/pyelonephritis he requires a course of 7-10 days and for a lower UTI/cystitis he will require 3
days.

Which of the following suggests pyelonephritis may be present?

A Dysuria

B Enuresis

C Frequency

D Mild abdominal pain

E Temp >38 degrees

Explanation
A fever of more than 38 degrees suggests pyelonephritis. Other features that may make you consider this would be
loin tenderness or pain, vomiting and other systemic features. Oral antibiotics can still be given if the child is well
in himself.

A
- Pain on passing urine may be elicited from the history. Even in younger children the parents may notice
discomfort on voiding urine.

B - A continent child who starts to wet themselves may have a UTI, other causes for this can be constipation as
well as psychological disturbance and child abuse.

C - Passing urine more often suggests a lower UTI.

D - Mild abdominal pain can be seen in lower UTI’s, loin pain or tenderness is more suggestive of pyelonephritis.

REF:

https://mypastest.pastest.com/Secure/TestMe/Browser/436619[‫ ص‬08:55:51 10/12/1437]


MyPastest

https://www.nice.org.uk/guidance/cg54
47043

Next Question
Previous Question

Tag Question

Feedback End Session

Difficulty: Average

Peer Responses

Session Progress

Responses Correct: 0

Responses Incorrect: 60

Responses Total: 60

Responses - % Correct: 0%

Blog
About Pastest
Contact Us
Help

© Pastest 2016

https://mypastest.pastest.com/Secure/TestMe/Browser/436619[‫ ص‬08:55:51 10/12/1437]


MyPastest

Prefer to use the old MyPastest? Access it here »

Back to Filters

Question 59 of 61

An 8-month-old has been diagnosed as having a UTI by their GP. The GP calls you to ask whether imaging is
required.

Which of the following is NOT a requirement for imaging?

A E.coli UTI

B Pseudomonas UTI

C Recurrent UTI

D Septicaemia

E Unwell 48hrs post antibiotics

Explanation
E.Coli accounts for the majority of UTI’s in children and is thus a common organism. If this child responds well to
treatment and has no recurrence of his symptoms, then no form of imaging is required, as per NICE advice for
children aged 6 months to 3 years. Had the child been less than 6 months of age, then an USS at 6 weeks would be
necessary.

B - Pseudomonas is an atypical UTI as it is a non-E.coli organism. This requires USS within the acute illness
phase as it may reflect an underlying pathology, and DMSA at 4-6 months.

C – Recurrent UTI consists of: 2 or more episodes of pyelonephritis OR 1 pyelonephritis and 1 cystitis OR 3 or
more cystitis.
It requires USS at 6 weeks and DMSA at 4-6 months.

D - Septicaemia is a sign of an atypical UTI. This requires USS within the acute illness as it may reflect an
underlying pathology and DMSA at 4-6 months.

E - This is a sign of an atypical UTI. This requires USS


within the acute illness as may reflect underlying
pathology and DMSA at 4-6 months.

https://mypastest.pastest.com/Secure/TestMe/Browser/436619[‫ ص‬08:56:12 10/12/1437]


MyPastest

Ref: https://www.nice.org.uk/guidance/cg54
47044

Next Question
Previous Question

Tag Question

Feedback End Session

Difficulty: Average

Peer Responses

Session Progress

Responses Correct: 0

Responses Incorrect: 61

Responses Total: 61

Responses - % Correct: 0%

Blog
About Pastest
Contact Us
Help

© Pastest 2016

https://mypastest.pastest.com/Secure/TestMe/Browser/436619[‫ ص‬08:56:12 10/12/1437]


MyPastest

Prefer to use the old MyPastest? Access it here »

Back to Filters

Question 60 of 61

A 2-month-old presents to your assessment unit. They have a fever of 38.5 degrees and mum reports that the urine
has smelt offensive. You suspect a UTI and obtain a urine sample, which is positive for leucocyte esterase and you
send for urgent microscopy. The
infant looks well in herself and her temperature has responded to paracetamol.

What is the next step in your management of this infant?

A Obtain a chest X-ray

B Perform bloods

C Perform bloods and a lumbar puncture

D Start IV antibiotics

E Start oral antibiotics

Explanation
As this child is <3 months old and has a fever
>38 degrees he needs to be managed in combination with NICE
guidance
on fever. He requires a FBC, CRP and blood culture. As he is >1month of age and appears well, it is
reasonable to await FBC and CRP results prior to deciding whether an LP is required. He will require IV
antibiotics in the interim whilst awaiting results.

A - As there are no respiratory signs present, a chest X-ray is not required.

C – Bloods and an LP can be debated and may vary with local practice. If the infant was <1 month or appeared
unwell then LP should be performed after obtaining bloods. As this infant does not fulfil this criteria, NICE
recommends awaiting blood results prior to LP.

D and E – As the infant is <3 months, but >1month, bloods are required to aid this decision.

Ref:

https://mypastest.pastest.com/Secure/TestMe/Browser/436619[‫ ص‬08:56:33 10/12/1437]


MyPastest

https://www.nice.org.uk/guidance/cg160
47045

Next Question
Previous Question

Tag Question

Feedback End Session

Difficulty: Average

Peer Responses

Session Progress

Responses Correct: 0

Responses Incorrect: 62

Responses Total: 62

Responses - % Correct: 0%

Blog
About Pastest
Contact Us
Help

© Pastest 2016

https://mypastest.pastest.com/Secure/TestMe/Browser/436619[‫ ص‬08:56:33 10/12/1437]


MyPastest

Prefer to use the old MyPastest? Access it here »

Back to Filters

Question 61 of 61

A 4-year-old has a pseudomonas UTI confirmed by MCS.


It is sensitive to ciprofloxacin and she has just started
her treatment. She has not had any other UTI’s.

Which imaging strategy is most suitable for this patient?

A Acute USS

B DMSA at 4-6 months

C MCUG

D None required

E USS within 6 weeks

Explanation
NICE recommends that if a child >3 years of age has an atypical UTI i.e. non-E.coli
UTI, then an USS should be
obtained within the acute illness. If however, she responds to treatment and has no other features of an atypical
UTI, then this USS could be obtained within 6 weeks.

A DMSA at 4-6 months is not required in atypical UTI’s over 3 years so stems B and C are incorrect.

NICE recommends USS so stem D is incorrect.

As
we do not know yet if she will respond well to treatment, as treatment has only just started, an acute USS is
more appropriate in this case.

Ref:

https://www.nice.org.uk/guidance/cg54
47046

https://mypastest.pastest.com/Secure/TestMe/Browser/436619[‫ ص‬08:56:55 10/12/1437]


MyPastest

End Session

Previous Question Tag Question

Feedback

Difficulty: Average

Peer Responses

Session Progress

Responses Correct: 0

Responses Incorrect: 63

Responses Total: 63

Responses - % Correct: 0%

Blog
About Pastest
Contact Us
Help

© Pastest 2016

https://mypastest.pastest.com/Secure/TestMe/Browser/436619[‫ ص‬08:56:55 10/12/1437]


MyPastest

Prefer to use the old MyPastest? Access it here »

Back to Filters

Question 1 of 35

Bell’s palsy in childhood:

A Is an upper motor neurone lesion

B Is commonly bilateral

C Responds well to steroids


Next Question

D Commonly preceded by a viral infection

E Prognosis is often poor

Explanation
Bell’s palsy (idiopathic facial palsy) is an acute unilateral lower motor neurone palsy. It typically occurs
approximately two weeks after infection with viruses such as Epstein–Barr, herpes simplex, herpes zoster or
mumps. A careful assessment may suggest an alternative aetiology for the acute onset of weakness e.g. acute otitis
media, Lyme disease, hypertension.

In childhood, Bell’s palsy is probably a post-infectious (i.e. immune


mediated) phenomenon, while in adults there
is increasing evidence that
the majority of cases follow reactivation of previous HSV infection.

The
prognosis is generally good in childhood with full recovery in the majority but permanent weakness is
observed in around 5%. Steroids and aciclovir may have some benefit in adults with recent onset of weakness,
but
the evidence of the treatment benefit is much less convincing in childhood. Exposure keratitis is an important
complication and children should be managed with eye drops and taping of the eyelid at night until
recovery is
complete.
10427

https://mypastest.pastest.com/Secure/TestMe/Browser/436619[‫ ص‬07:04:45 10/12/1437]


MyPastest

Tag Question

Feedback

Difficulty: Easy

Peer Responses

Previous Question
Session Progress

Responses Correct: 0

Responses Incorrect: 1

Responses Total: 1

Responses - % Correct: 0%

End Session

Blog
About Pastest
Contact Us
Help

© Pastest 2016

https://mypastest.pastest.com/Secure/TestMe/Browser/436619[‫ ص‬07:04:45 10/12/1437]


MyPastest

Prefer to use the old MyPastest? Access it here »

Back to Filters

Question 2 of 35

The following are features of Asperger’s Syndrome, except:

A Impaired social interaction


Previous Question

B Repetitive behaviour

C
Next Question
Restricted, obsessional idiosyncratic interests

D Language impairment

E Near normal intelligence End Session

Explanation
Asperger’s syndrome is a condition at
the mild end of the autistic spectrum. There is normal early language
development and intellectual functioning. In autism, there is severe language and intellectual impairment. Other
features, like impairment in
social interaction, stereotypical behaviour and difficulty in communication, are
common in both. However, in a few cases Asperger’s Syndrome may go undiagnosed.

Group social skills training is the hallmark of the intervention.

As in the case of autism, it is picked up and diagnosed usually by 3 years of age.


11624

Tag Question

Feedback

https://mypastest.pastest.com/Secure/TestMe/Browser/436619[‫ ص‬07:05:13 10/12/1437]


MyPastest

Difficulty: Average

Peer Responses

Session Progress

Responses Correct: 0

Responses Incorrect: 2

Responses Total: 2

Responses - % Correct: 0%

Blog
About Pastest
Contact Us
Help

© Pastest 2016

https://mypastest.pastest.com/Secure/TestMe/Browser/436619[‫ ص‬07:05:13 10/12/1437]


MyPastest

Prefer to use the old MyPastest? Access it here »

Back to Filters

Question 3 of 35

The following are true regarding attention deficit hyperactivity disorder (ADHD), except:

A There are problems with hyperactivity, impulsiveness and inattention

B Girls are more often affected

C
Next Question
They develop emotional, social problems and poor school performance.

D Treatment
involves a comprehensive treatment programme involving psychological/behavioural
interventions, educational support, drug therapy and social services.

E Methylphenidate is used for treatment as a part of comprehensive treatment plan.

Explanation
ADHD is seen in 1% of school-aged children.
It is more common in boys. It is characterised by inattention,
hyperactivity and impulsiveness. The behaviour should be present for more than 6 months in more than one
setting. It should be inconsistent with the child’s developmental age and there should be no other valid cause (e.g.
other psychiatric illness) explaining this behaviour. They have poor school performance and also face problems at
home. They are at risk of substance abuse, committing crime and unemployment as adults.

A multidisciplinary approach is used for treatment. Methylphenidate is used as a drug treatment. It is a CNS
stimulant, given twice daily, in the morning and at lunch time. An evening dose is avoided as it interferes with
sleep. It is not recommended in children below 6 years of age. Side effects include retardation in growth and
hypertension.
11625

https://mypastest.pastest.com/Secure/TestMe/Browser/436619[‫ ص‬07:05:34 10/12/1437]


MyPastest

Tag Question

Feedback

Difficulty: Easy

Peer Responses

Previous Question
Session Progress

Responses Correct: 0

Responses Incorrect: 3

Responses Total: 3

Responses - % Correct: 0%

End Session

Blog
About Pastest
Contact Us
Help

© Pastest 2016

https://mypastest.pastest.com/Secure/TestMe/Browser/436619[‫ ص‬07:05:34 10/12/1437]


MyPastest

Prefer to use the old MyPastest? Access it here »

Back to Filters

Question 4 of 35

Neuroregression is seen in:

A Hypoxic ischaemic encephalopathy

B Neonatal meningitis

C Intrauterine infection
Next Question

D Subacute Sclerosing Panencephalitis (SSPE)

E Foetal alcohol syndrome

Explanation
SSPE is a chronic encephalitis following chronic persisting measles infecton in the brain tissue. It is
seen post
measles in children and young adults. It begins as subtle behavioural changes and deterioration of school work
which is of insidious onset but is followed by bizarre behaviour and frank dementia.
There are no features of acute
encephalitis except for headache.
Diffuse neurological disease becomes more severe, mycoclonus is seen,
cerebellar ataxia, retinopathy, optic atrophy may appear. Dementia may progress to stupor and coma.

There is no effective treatment to stop the progression of the disease. Supportive treatment in the form of
anticonvulsants can be given.

Neuroregression is the
loss of milestones which were already attained. This is seen in SSPE, Huntington’s disease,
Wilson’s Disease, Rett’s syndrome and metabolic diseases like Tay Sachs disease and Niemann Pick disease.
Developmental delay is seen in conditions like Foetal Alcohol syndrome, intrauterine infections, HIE and neonatal
meningitis.
11626

https://mypastest.pastest.com/Secure/TestMe/Browser/436619[‫ ص‬07:05:57 10/12/1437]


MyPastest

Tag Question

Feedback

Difficulty: Easy

Peer Responses

Previous Question
Session Progress

Responses Correct: 0

Responses Incorrect: 4

Responses Total: 4

Responses - % Correct: 0%

End Session

Blog
About Pastest
Contact Us
Help

© Pastest 2016

https://mypastest.pastest.com/Secure/TestMe/Browser/436619[‫ ص‬07:05:57 10/12/1437]


MyPastest

Prefer to use the old MyPastest? Access it here »

Back to Filters

Question 5 of 35

Which of the following is true regarding Medulloblastoma:

A Forms < 10 % of the embryonal tumors


Previous Question

B Seen mostly in infancy

C Seen in medulla oblongata


Next Question

D It rarely metastasises

E May obstruct the 4th ventr i cle and cause hydrocephalus End Session

Explanation
Medulloblastoma is a common tumour in childhood. It forms > 90% of embryonal tumours. It is seen
predominantly in males. Median age is 5-7 years. It occurs in the cerebellar vermis predominantly, and sometimes
seen in cerebellar hemispheres. CT scan shows contrast enhancing mass in the posterior fossa causing obstruction
to the 4th ventricle and hydrocephalus. It is the most highly malignant brain tumour in childhood, rapidly growing,
and may metastasise along the CSF pathways.

Treatment includes surgery and radiotherapy.


11627

Tag Question

Feedback

https://mypastest.pastest.com/Secure/TestMe/Browser/436619[‫ ص‬07:06:29 10/12/1437]


MyPastest

Difficulty: Average

Peer Responses

Session Progress

Responses Correct: 0

Responses Incorrect: 5

Responses Total: 5

Responses - % Correct: 0%

Blog
About Pastest
Contact Us
Help

© Pastest 2016

https://mypastest.pastest.com/Secure/TestMe/Browser/436619[‫ ص‬07:06:29 10/12/1437]


MyPastest

Prefer to use the old MyPastest? Access it here »

Back to Filters

Question 6 of 35

A 5-month-old is brought in by her stepdad with


impaired consciousness. She is having irregular breathing with
intermittent apnoeic episodes, she also has old bruises on her limbs and
has retinal hemorrhages. You suspect:

A
Previous Question
Status epilepticus with postictal state

B Meningococcal sepsis
Next Question
C Metabolic disorder

D Non-accidental injury
End Session
E Poisoning

Explanation
The features are consistent with NAI, particularly Shaken Baby Syndrome. A CT scan may show acute and
chronic bleeds in the brain. The skeletal bone survey may indicate present or healed fractures.
11628

Tag Question

Feedback

Difficulty: Easy

Peer Responses

https://mypastest.pastest.com/Secure/TestMe/Browser/436619[‫ ص‬07:06:48 10/12/1437]


MyPastest

Session Progress

Responses Correct: 0

Responses Incorrect: 6

Responses Total: 6

Responses - % Correct: 0%

Blog
About Pastest
Contact Us
Help

© Pastest 2016

https://mypastest.pastest.com/Secure/TestMe/Browser/436619[‫ ص‬07:06:48 10/12/1437]


MyPastest

Prefer to use the old MyPastest? Access it here »

Back to Filters

Question 7 of 35

The following are signs of raised intracranial tension, except:

A Cushing’s triad of bradycardia, hypertension and irregular breathing


Previous Question

B Papilloedema

C Sunken fontanelles
Next Question

D Unequal pupils

E Decorticate posturing End Session

Explanation
Initial signs of raised intracranial tension include headache, confusion and vomiting. Bulging fontanelles, not
sunken, can be seen in infants. Papillodema indicates raised ICP. Unequal pupils, Cushing's triad and decorticate
posturing are late signs
indicating impending herniation of the brain.

Raised ICP is a contraindication for lumbar puncture except in a bulging fontanelle without focal neurological
deficits.

Causes of raised ICP include: space occupying lesions, meningitis, encephalitis, trauma and intracranial bleeds.
11629

Tag Question

Feedback

https://mypastest.pastest.com/Secure/TestMe/Browser/436619[‫ ص‬07:07:08 10/12/1437]


MyPastest

Difficulty: Easy

Peer Responses

Session Progress

Responses Correct: 0

Responses Incorrect: 7

Responses Total: 7

Responses - % Correct: 0%

Blog
About Pastest
Contact Us
Help

© Pastest 2016

https://mypastest.pastest.com/Secure/TestMe/Browser/436619[‫ ص‬07:07:08 10/12/1437]


MyPastest

Prefer to use the old MyPastest? Access it here »

Back to Filters

Question 8 of 35

An 8-year-old boy develops ataxia, absent deep tendon reflexes, dysarthtic speech, nystagmus. His intelligence is
well preserved. Weakness of distal musculature of hands and feet noted. He is
also diagnosed to have hypertrophic
cardiomyopathy.

The probable diagnosis is?

A Ataxia telangiectasia

B Friedreich’s ataxia

C Abetalipoprotienemia

D Acute cerebellar ataxia

E Acute labrynthitis

Explanation
Friedreich’s ataxia is also called spino-cerebellar degeneration. It is an autosomal recessive condition. Onset of
ataxia is around 10 years of age. Intelligence is preserved. Cerebellar impairment, distal muscle weakness, pes
cavus, hammer toes and progressive kyphoscoliosis are present. Deep tendon reflexes are absent, particularly the
ankle jerk. Marked loss of vibration and position sense occurs because of degeneration of the posterior columns.
Hypertrophic cardiomyopathy can occur with progresson to congestive cardiac failure.

Ataxia Telangiectasia: ataxia develops earlier, around 2 years of age, with loss of ambulation by adolescence.
Nystagmus
is present. Telangiectasia becomes evident by mid-childhood and is found on bulbar conjunctiva, over
the bridge of the nose and on the ears. Abnormalities of immunologic function leads to frequent sinopulmonary
infections, there is decreased serum, secretory IgA. These
children have increased risk of developing
lymphoreticular malignancies.

Abetalipoproteinemia: begins in childhood with steatorrhea and failure to thrive. Neurological symptoms appear in
late childhood. These include ataxia, retinitis pigmentosa, peripheral neuritis, abnormalities in postion and

https://mypastest.pastest.com/Secure/TestMe/Browser/436619[‫ ص‬07:07:40 10/12/1437]


MyPastest

vibration sense, muscle weakness and mental retardation. Vitamin E is undetectable in serum in these patients.

Acute
cerebellar ataxia: occurs in children of 1-3 years of age and is a diagnosis of exclusion. It often follows
acute viral infections like varicella, coxsackie virus or echovirus by 2-3 weeks. It is an autoimmune response to
viral agent affecting cerebellum. Prognosis is excellent with complete recovery present. Very small numbers have
long term sequelae like ataxia, incoordination, speech disorder and behavioural problems.

Acute labrynthitis: difficult to differentiate from acute cerebellar ataxia in a toddler. It is associated with middle
ear infections, vertigo and vomiting.
11631

Tag Question

Feedback

Previous Question
Difficulty: Easy

Peer Responses
Next Question

End Session

Session Progress

Responses Correct: 0

Responses Incorrect: 8

Responses Total: 8

Responses - % Correct: 0%

Blog
About Pastest
Contact Us
Help

© Pastest 2016

https://mypastest.pastest.com/Secure/TestMe/Browser/436619[‫ ص‬07:07:40 10/12/1437]


MyPastest

Prefer to use the old MyPastest? Access it here »

Back to Filters

Question 9 of 35

A 4-year-old child presents with a history of headache, vomiting of 6 hours duration and altered sensorium of 1
hour duration. He has a seizure in A&E. On examination he is toxic looking, febrile, has neck stiffness. He is
noted to have a 4th cranial nerve palsy. A provisional diagnosis of meningitis is made.

The most appropriate course of action is?

A Do a lumbar puncture and start IV antibiotics


Next Question
B Start IV antibiotics followed by a lumbar puncture

C Start IV antibiotics promptly after blood culture is taken. Do a CT scan and plan lumbar puncture
accordingly.

D Fundoscopy and lumbar puncture

E MRI followed by lumbar puncture. Meantime start antibiotics

Explanation
The clinical diagnosis of meningitis can be made from this presentation. To prove the causative organism causing
meningitis, a lumbar puncture is diagnostic. However raised intracranial
tension is a contraindication for a lumbar
puncture, except when the anterior fontanelle is open.

In the above case, there is focal neurological deficit in the form of 4th
nerve palsy. In such a case, it is safe to do a
CT scan and rule out raised intracranial tension and any mass effect causing it. There is a risk of coning when a
lumbar puncture is done in presence of raised ICP.
11632

https://mypastest.pastest.com/Secure/TestMe/Browser/436619[‫ ص‬07:08:03 10/12/1437]


MyPastest

Tag Question

Feedback

Difficulty: Easy

Peer Responses

Previous Question
Session Progress

Responses Correct: 0

Responses Incorrect: 9

Responses Total: 9

Responses - % Correct: 0%

End Session

Blog
About Pastest
Contact Us
Help

© Pastest 2016

https://mypastest.pastest.com/Secure/TestMe/Browser/436619[‫ ص‬07:08:03 10/12/1437]


MyPastest

Prefer to use the old MyPastest? Access it here »

Back to Filters

Question 10 of 35

Which of the following organisms is the most common causative agent for meningitis in the first 2 months of life?

A Group B Streptococci

B Staphylococcus

C Viral

D Listeria

E H. influenza

Explanation
In the first two months of life, the bacteria that cause meningitis reflects the maternal flora. They include
Group B
Streptococcus and E coli being more common, and Staphylococcus and Listeria being less common.

In the 2 months to 1 year age group the most common organisms are:

1. Streptococcus pnuemoniae
2. Neisseria meningitidis
3. Haemophilus influenza

In older children common organisms include:

1. Neisseria meningitidis
2. Streptococcus pnuemoniae
3. Haemophilus influenza

Prior to the introduction of H. influenza B vaccine, 70% of bacterial meningitis were caused by this organism.

Antibiotics for the treatment of meningitis depends on the age and the causative organism.
11633

https://mypastest.pastest.com/Secure/TestMe/Browser/436619[‫ ص‬07:08:24 10/12/1437]


MyPastest

Tag Question

Feedback

Difficulty: Easy

Peer Responses

Previous Question

Next Question
Session Progress

Responses Correct: 0

Responses Incorrect: 10

Responses Total: End Session 10

Responses - % Correct: 0%

Blog
About Pastest
Contact Us
Help

© Pastest 2016

https://mypastest.pastest.com/Secure/TestMe/Browser/436619[‫ ص‬07:08:24 10/12/1437]


MyPastest

Prefer to use the old MyPastest? Access it here »

Back to Filters

Question 11 of 35

A 6-month-old boy, Tom, is brought to A&E by paramedics. He is irritable and has been feeding poorly for the
last 2 days. He is febrile, has a high pitched cry and pustular discharge from both ears. His fontanelle is full. There
were no other neurological signs. IV access was obtained, bloods were sent for septic workup and a lumbar
puncture was performed. In the meantime IV antibiotics were started.

The expected CSF picture for bacterial meningitis would be:

A CSF glucose: blood glucose 0.5, protein 0.1g/l. Cells 200, all lymphocytes

B CSF glucose: blood glucose 0.4, protein 0.2g/l. Cells 400, 390 polymorphs, 10 lymphocytes

C CSF glucose: blood glucose 0.5, protein 2g/l. Cells 100, all lymphocytes

D CSF glucose: blood glucose 0.6, protein 0.025g/l. Cells 3, all lymphocytes

E CSF glucose: blood glucose 0.5, protein 0.05g/l. Cells 200, mononuclear cells

Explanation
The CSF characteristically in bacterial meningitis will show:

Turbid appearance
Multiple polymorphs
Few lymptocytes
Ogansisms on microscopy
Protein up to 5g/l
Glucose less than two thirds of the blood level

As both polymorphs and lymphocytes are seen, the only answer that is correct is option B.

https://mypastest.pastest.com/Secure/TestMe/Browser/436619[‫ ص‬07:08:46 10/12/1437]


MyPastest

Option A is a CSF picture in viral meningitis

Option
B is a classical bacterial meningitis picture. CSF would be turbid. Organisms may be seen on gram staining
and grown on culture. Partially treated bacterial meningitis may have cells ranging from 5 to 10,000 and
cells may
be polymorphs or if treated for extended period may be mononuclear cells. Protein will be same as in bacterial
meningitis, glucose may be normal or marginally low. The organism may or may not be seen on gram stain and
cultures may be sterile owing to pre-treatment.

Option
C shows very high protein, lymphocytosis in CSF and low or marginally low glucose in CSF and this
indicates TB meningitis. The CSF may have cobweb appearance on standing. Acid Fast Bacilli may be isolated on
special smears.

Option D shows the normal CSF picture. Up to 5 cells in a child is normal, all cells being lymphocytes In a
neonate, up
to 15 cells are normal. Any polymorph in a child is abnormal but 1-2 in
a neonate is normal.

Option E shows fungal meningitis. Cells may


be polymorphs in early stages, but later it is mononuclear cells.
Budding yeast may be seen. Antigens are used for detecting the type of organisms in all the meningitis. Bacterial
antigens, cryptococcal antigens and viral antigens can be detected. It is more useful I partially treated pyogenic
meningitis where the cultures may be negative. Tubercular meningitis has CSF positive for tubercular Ag detected
by PCR. Previous Question
Exam Question theme June 2015
11634

Next Question

Tag Question

Feedback End Session

Difficulty: Easy

Peer Responses

Session Progress

Responses Correct: 0

Responses Incorrect: 11

https://mypastest.pastest.com/Secure/TestMe/Browser/436619[‫ ص‬07:08:46 10/12/1437]


MyPastest

Responses Total: 11

Responses - % Correct: 0%

Blog
About Pastest
Contact Us
Help

© Pastest 2016

https://mypastest.pastest.com/Secure/TestMe/Browser/436619[‫ ص‬07:08:46 10/12/1437]


MyPastest

Prefer to use the old MyPastest? Access it here »

Back to Filters

Question 12 of 35

Which of the following statements regarding status epilepticus is true?

A Defined as convulsion lasting for more than 1 hour

B Always seen in a known epileptic

C
Next Question
The drugs can be administered intramuscularly if IV access is difficult

D The
mode of treatment: ABC management. Maintain blood glucose. Up to 2 doses of IV
lorazapam/diazapam. Rectal paraldehyde. IV phenytoin. Consider general anaesthesia if still continues
to fit.

E Pre-hospital management includes rectal diazepam and buccal midazolam given both together.

Explanation
Status epilepticus is defined as a continuous convulsion lasting longer than 30 minutes or the occurrence of serial
convulsions between which there is no return of consciousness.
These are common in poorly controlled epileptics
but are also seen in febrile convulsions, electrolyte abnormalities, encephalitis, inborn errors of metabolism and
HIE in newborns, tumours and Reye syndrome.

Pre-hospital
treatment includes either buccal midazolam or per-rectal diazepam. In the hospital always use the
intravenous route. The intramuscular route is not preferred as drugs are sequestered in the muscle. If IV access not
available, try the intra-osseous route. Not more than 2 doses of benzodiazapines are recommended. Either per-
rectal diazepam or buccal midazolam can be used but not both together at the same time. Overdose of
benzodiazapines cause respiratory depression.
11636

https://mypastest.pastest.com/Secure/TestMe/Browser/436619[‫ ص‬07:09:08 10/12/1437]


MyPastest

Tag Question

Feedback

Difficulty: Easy

Peer Responses

Previous Question
Session Progress

Responses Correct: 0

Responses Incorrect: 12

Responses Total: 12

Responses - % Correct: 0%

End Session

Blog
About Pastest
Contact Us
Help

© Pastest 2016

https://mypastest.pastest.com/Secure/TestMe/Browser/436619[‫ ص‬07:09:08 10/12/1437]


MyPastest

Prefer to use the old MyPastest? Access it here »

Back to Filters

Question 13 of 35

Theme: Neurocutaneous Syndromes

A Sturge Weber Syndrome


B Neurofibromatosis 2
C Neurofibromatosis 1
D Tuberous sclerosis
E Von Hippel Lindau
F Hypomelanosis of Ito

For
each of the following scenarios, choose the best diagnosis from the list above. Each option may be used once,
more than once or not at all.

Scenario 1

A child has facial naevus at birth on R side involving upper face and eyelid. At 8 months she develops L sided
focal tonic clonic convulsions, resistant to anticonvulsants. Following this she is found to develop L sided
hemiparesis and developmental regression in 2nd year of life. CT Scan head was done and there was evidence of
intracranial calcification showing railtrack appearance.

Your answer was incorrect

Select one...

A - Sturge Weber Syndrome

In
the first case, the features are highly characteristic of Sturge Weber syndrome. These children have a port wine
stain from birth over the forehead and upper eyelid of one side of the face which later may be accompanied by
seizures. There are increased capillaries around the trigeminal nerve and underlying malformation of the blood
vessels in the
brain. This causes the intracranial calcification typically called traintracks. They may develop
hemiplegia on the contralateral side to port wine stain with developmental delay more often than regression.

Scenario 2

4-year-old Harry has numerous café-au-lait spots on his body. On slit lamp examination of his eyes,
hamartomatous lesions consistent with Lisch nodules are found. His
father was diagnosed to have a similar
condition.

https://mypastest.pastest.com/Secure/TestMe/Browser/436619[‫ ص‬07:09:57 10/12/1437]


MyPastest

Your answer was incorrect

Select one...

C - Neurofibromatosis 1

Neurofibromatosis Type 1 has criteria set out as below. Two of the following are requried for the diagnosis:

6
or more café-au-lait macules over 5 mm in greatest diameter in pre-pubertal individuals and over 15 mm
in greatest diameter in post-pubertal individuals
2 or more neurofibromas of any type or 1 plexiform neurofibroma
Freckling in the axillary or inguinal regions
Optic glioma
2 or more Lisch nodules (iris hamartomas)
A distinctive osseous lesion such as sphenoid dysplasia or thinning of the long bone cortex with or without
pseudarthrosis
A first degree relative (parent, sibling or offspring) with NF-1 by the above criteria

Scenario 3

14-year-old Jack has bilateral 8th nerve masses consistent with acoustic neuroma as seen on MRI.

Your answer was incorrect

Select one...

B - Neurofibromatosis 2

Neurofibromatosis Type 2 is diagnosed on a wide range of presenting symptoms. They consist of bilateral acoustic
neuromas, tumours in other cranial nerves or meningiomas, or spinal astrocytomas or ependymomas.

Scenario 4

8-month-old Charlotte has infantile spasms with hypsarrythmic EEG. On careful examination she is found to have
ash leaf like hypopigmented lesions on the skin. A CT scan of the head shows tubers in the subependymal region.

Your answer was incorrect

Select one...

D - Tuberous sclerosis

Ash leaf patches are seen in patients with tuberous sclerosis. These patients are also prone to infantile spasms.

Explanation

https://mypastest.pastest.com/Secure/TestMe/Browser/436619[‫ ص‬07:09:57 10/12/1437]


MyPastest

 
11639

Tag Question

Feedback

Difficulty: Average

Session Progress

Responses Correct: 0

Responses Incorrect: 16
Previous Question
Responses Total: 16

Responses - % Correct: 0%

Next Question

Blog
About Pastest
Contact Us
Help

© Pastest 2016
End Session

https://mypastest.pastest.com/Secure/TestMe/Browser/436619[‫ ص‬07:09:57 10/12/1437]


MyPastest

Prefer to use the old MyPastest? Access it here »

Back to Filters

Question 14 of 35

Theme: Anticonvulsants

A Carbamazepine
B No anticonvulsants indicated
C Phenobarbitone
D ACTH
E Ethosuximide

Match the appropriate treatment with the scenarios described below. Each option may be used once, more than
once or not at all.

Exam question theme from June 2015

Scenario 1

Benign Partial Epilepsy of Childhood / Rolandic Seizures with frequent episodes

Your answer was incorrect

Select one...

A - Carbamazepine

BPEC / Rolandic Seizures: Anticonvulsants are only necessary for patients who have frequent seizures, not to be
started after the 1st convulsion. Carbamazepine is the preferred drug, continued for 2 years seizure free interval or
up to 14-16 years when spontaneous remission of
BPEC occurs.

Scenario 2

Benign myoclonus of infancy

Your answer was incorrect

Select one...

B - No anticonvulsants indicated

https://mypastest.pastest.com/Secure/TestMe/Browser/436619[‫ ص‬07:10:19 10/12/1437]


MyPastest

Benign myoclonus of infancy is not an indication for starting anticonvulsants and it is self-limiting by age of 2
years.

Scenario 3

Febrile seizures

Your answer was incorrect

Select one...

B - No anticonvulsants indicated

Anticonvulsants are not indicated in Febrile seizures. Fever control indicated and per-rectal diazepam or buccal
midazolam is given if there is a convulsion.

Scenario 4

Neonatal Seizures in a child with Hypoxic Ischaemic Encephalopathy

Your answer was incorrect

Select one...

C - Phenobarbitone

Phenobarbitone is preferred in neonatal seizures with HIE.

Scenario 5

Infantile Spasms

Your answer was incorrect

Select one...

D - ACTH

In infantile spasms ACTH is used. Vigabatrin is used in infantile spasms seen with Tuberous Sclerosis.

Scenario 6

Absence Seizures

Your answer was incorrect

Select one...

E - Ethosuximide

https://mypastest.pastest.com/Secure/TestMe/Browser/436619[‫ ص‬07:10:19 10/12/1437]


MyPastest

Ethosuximide, Lamotrigine or Valproate is used for treatment of absence seizures.

The following table gives guidance on the treatment for epilepsy syndromes.

Seizure Partial Partial-Benign Infantile


Generalised
Type Symptomatic Rolandic spasms

1st line Sodium Valproate Carbamazepine Nothing Vigabatrin

Lamotrogine /
2nd Carbamazepine (not
Lamotrogine Carbamazepine ACTH
line used for absence
seizures)

Gabapentin /
3rd
Topiramate / Topiramate
line
Clonazepam

4th Previous Question


Phenytoin
line

Explanation
Next Question
Note: The side effects of commonly used anticonvulsants can also be a probable question.
11642

End Session

Tag Question

Feedback

Difficulty: Average

Session Progress

Responses Correct: 0

Responses Incorrect: 22

Responses Total: 22

Responses - % Correct: 0%

Blog
About Pastest
Contact Us
Help

https://mypastest.pastest.com/Secure/TestMe/Browser/436619[‫ ص‬07:10:19 10/12/1437]


MyPastest

© Pastest 2016

https://mypastest.pastest.com/Secure/TestMe/Browser/436619[‫ ص‬07:10:19 10/12/1437]


MyPastest

Prefer to use the old MyPastest? Access it here »

Back to Filters

Question 15 of 35

A Duchenne Muscular Dystrophy


B Becker Muscular Dystrophy
C Werdnig Hoffman Disease
D Transverse myelitis
E Guillian Barre Syndrome

Diagnose the cause of weakness using the most appropriate option. Each option may be used once, more than
once, or not at all.

Scenario 1

6 year old Tim was first brought to your clinic as a 3 year old by his mother when she noticed that his gait
had
become unstable. His developmental milestones were normal. On examining him, you notice lordosis, he had
Trendelenberg gait and a positive Gower’s sign. Over the past 3 years his weakness has progressed and he has
been wheel chair bound for the past 3 months. His maternal uncle had similar complaints and died in childhood
due to some associated cardiac problem which the mother is not sure of.

Your answer was incorrect

Select one...

A - Duchenne Muscular Dystrophy

Duchenne Muscular Dystrophy is very commonly seen in Western Europe. It is X linked recessive. Female
carriers are asymptomatic. Mutation in dystrophin gene. DMD patients produce no dystrophin. Normal
at birth and
infancy, symptomatic in toddler age group. If it presents in infancy, poor head control may be manifested.
Progressive proximal muscle weakness with psuedohypertrophy of these muscles. Wheelchair bound by
adolosence. Resiratory failure can supervene. Dilated Cardiomyopathy and learning difficulty can be associated.
High serum creatine kinase and absent dystrophin in muscle biopsy confirms diagnosis.

Becker Muscular Dystrophy is a milder form of DMD. Age of presentation is in adolescent years.

Scenario 2

3 month old Tia is having generalised weakness with severe hypotonia, lies flaccid with very few movements and
feeding difficulty. She has fasciculation of tongue and very poor head control for her age. Deep tendon reflexes

https://mypastest.pastest.com/Secure/TestMe/Browser/436619[‫ ص‬07:10:45 10/12/1437]


MyPastest

are absent. Antenatal and birth history reveals that decreased foetal movements were
appreciated. Otherwise it was
uneventful.

Your answer was incorrect

Select one...

C - Werdnig Hoffman Disease

Spinal muscular atrophy: Anterior horn cells in spinal cord degenerates.

Type
1 - Werdnig Hoffman Disease - Onset very early in life, decreased foetal movements, severe weakness and
hypotonia in infants, feeding difficulty, respiratory difficulty. Death occurs in first 18 months of life from
respiratory failure.

Type 2 - Onset after first 3 months, able to sit, not able to stand or walk. Prognosis depends on extent of
respiratory muscle involvement.

Type 3 - Kugelberg-Walender Disease. Able to walk but proximal muscle weakness present. Prognosis good.
Previous Question
Scenario 3

4 year old Katie presented with a refusal to walk since last night. There was marked weakness of lower limbs,
Next Question
power 2/5 and hypotonic. Absent deep tendon reflexes. After admission over the next few hours the weakness has
ascended to involve the trunk and she is unable to sit. There is a history of viral upper respiratory tract infection 10
days ago.

Your answer was incorrect

Select one...

E - Guillian Barre Syndrome

Guillian Barre Syndrome: Acute ascending symmetrical weakness following a viral infection – urti or
gastroenteritis. It is a demyelinating neuropathy. Deep tendon reflexes are absent. Bulbar involvement seen in 50%
of the cases involving dysphagia, facial weakness and impending respiratory failure. Autonomic disturbances are
seen in few cases. Involvement of autonomic system and bulbar involvement are the poor prognostic factors.

Explanation
 
11645

Tag Question

Feedback

https://mypastest.pastest.com/Secure/TestMe/Browser/436619[‫ ص‬07:10:45 10/12/1437]


MyPastest

Difficulty: Average

Session Progress

Responses Correct: 0

Responses Incorrect: 25

Responses Total: 25

Responses - % Correct: 0%

Blog
About Pastest
Contact Us
Help

© Pastest 2016

End Session

https://mypastest.pastest.com/Secure/TestMe/Browser/436619[‫ ص‬07:10:45 10/12/1437]


MyPastest

Prefer to use the old MyPastest? Access it here »

Back to Filters

Question 16 of 35

Theme: Estimated developmental age

A 2 years
B 3 years
C 4 years
D 9 months
E 8 months
F 7 months

Choose
the appropriate developmental age for each of the descriptions below. Each option can be used once,
more than once or not at all.

Scenario 1

Rides tricycle, dresses and undresses self


when helped with buttons. Gives full name and knows own sex. Sings
some
nursery rhymes. Identifies few colours, copies circle and plays with other children in a group.

Your answer was incorrect

Select one...

B - 3 years

At age 3 years a child should be able to ride a tricycle (a way to remember is '3 wheels at 3').

Scenario 2

Pulls self to standing position and stands


steadily holding on to furniture. Sits on his own and steadily without risk
of falling. Has developed pincer grasp. Can release objects deliberately and let them go. Plays peek-a-boo.

Your answer was incorrect

Select one...

D - 9 months

https://mypastest.pastest.com/Secure/TestMe/Browser/436619[‫ ص‬07:11:10 10/12/1437]


MyPastest

At age 9 months a child can start to cruise around furniture and sit unsupported (8 months). Pincer grasp is
developed at 9 months to 1 year.

Scenario 3

Rolls from supine to prone and vice versa.


Sits with hands forward for support. Transfers objects from one hand to
other. Takes all objects to mouth. Respond to name. Says “da” “ba ” (monosyllable babble).

Your answer was incorrect

Select one...

F - 7 months

At age 7 months a child can roll in both directions and can sit supporting themselves. Transferring objects starts at
6 months.
2 years:
Previous Question
Goes up and down the stairs alone, two feet at a time.
Turns pages of book singly.
Puts on shoes, socks and pants.
Turns door knobs, unscrews lid.
Next Question
Washes and dries hands.
Parallel play - watches others play and plays near them without playing with them.
Uses words like I, me, you.

4 years:

Up and down the stairs, one foot at a time.


Skips.
Copies a square.
Talks almost like an adult.

Explanation

11646

Tag Question

Feedback

https://mypastest.pastest.com/Secure/TestMe/Browser/436619[‫ ص‬07:11:10 10/12/1437]


MyPastest

Difficulty: Average

Session Progress

Responses Correct: 0

Responses Incorrect: 28

Responses Total: 28

Responses - % Correct: 0%

Blog
About Pastest
Contact Us
Help

© Pastest 2016

End Session

https://mypastest.pastest.com/Secure/TestMe/Browser/436619[‫ ص‬07:11:10 10/12/1437]


MyPastest

Prefer to use the old MyPastest? Access it here »

Back to Filters

Question 17 of 35

A 16-year-old female presents with a 3-day history of progressive weakness and numbness of her legs, urinary
retention and back pain some 2 weeks following an upper respiratory infection. On examination there is spastic
paraparesis, sensory level up to T5, extensor plantars. Examination of her cranial nerves and upper limbs is
normal. MRI of the spine is normal. CSF analysis reveals 50 cells/mm3, over 90% lymphocytes with normal
protein and glucose levels and negative oligoclonal bands.

What is the most likely diagnosis?

Next Question
A Anterior spinal artery occlusion

B Guillain–Barré syndrome

C Multiple sclerosis

D Postinfectious transverse myelitis

E Thoracic disc prolapse

Explanation
Transverse myelitis usually follows an upper respiratory tract infection. It causes a complete spinal cord
syndrome. MRI of the spine may either show an intrinsic inflammatory lesion or can be normal. Multiple sclerosis
usually causes a partial spinal cord syndrome (asymmetrical paraparesis). Anterior spinal artery occlusion causes
an acute-onset spinal cord syndrome with spinal shock (flaccid paraplegia). Guillain–Barré syndrome causes lower
motor neurone signs.
12659

https://mypastest.pastest.com/Secure/TestMe/Browser/436619[‫ ص‬07:11:37 10/12/1437]


MyPastest

Tag Question

Feedback

Difficulty: Average

Peer Responses

Previous Question
Session Progress

Responses Correct: 0

Responses Incorrect: 29

Responses Total: 29

Responses - % Correct: 0%

End Session

Blog
About Pastest
Contact Us
Help

© Pastest 2016

https://mypastest.pastest.com/Secure/TestMe/Browser/436619[‫ ص‬07:11:37 10/12/1437]


MyPastest

Prefer to use the old MyPastest? Access it here »

Back to Filters

Question 18 of 35

A 16-year-old boy presents with difficulty in walking and foot drop. There is weakness of dorsiflexion and
eversion of the right foot, with a small area of sensory loss over the dorsum of that foot.

What is the most likely diagnosis?


Previous Question
A Posterior tibial nerve lesion

B Sciatic nerve lesion


Next Question

C L5 root lesion

D Common peroneal nerve lesion

E Deep peroneal nerve lesion

Explanation
The common peroneal (L5, S1) nerve arises from the division of the sciatic nerve in the popliteal fossa. It passes
close to the head of the fibula and can be damaged by pressure in this area. It divides into a superficial and a deep
branch. The deep peroneal
nerve supplies the tibialis anterior, extensor hallucis longus and extensor digitorum
longus muscles, which dorsiflex the foot and toes. The superficial nerve supplies the peroneus longus and brevis
muscles, which evert the foot. Damage to the posterior tibial nerve produces weakness of planter flexion and
inversion of the foot.
12660

Tag Question

https://mypastest.pastest.com/Secure/TestMe/Browser/436619[‫ ص‬07:12:22 10/12/1437]


MyPastest

Feedback

Difficulty: Average

Peer Responses

Session Progress

Responses Correct: 0

Responses Incorrect: 30

Responses Total: 30

Responses - % Correct: 0%

End Session
Blog
About Pastest
Contact Us
Help

© Pastest 2016

https://mypastest.pastest.com/Secure/TestMe/Browser/436619[‫ ص‬07:12:22 10/12/1437]


MyPastest

Prefer to use the old MyPastest? Access it here »

Back to Filters

Question 19 of 35

A 16-year-old girl presents with a resting tremor of both arms. She is dysarthric and ataxic with some grimacing of
facial musculature.

What is the most likely diagnosis?


Previous Question
A Alzheimer’s disease

B Functional illness
Next Question

C Huntington’s chorea

D Neuroacanthocytosis

E Wilson’s disease

Explanation
The combination of cerebellar signs and tremor in a young patient makes Wilson’s disease the most likely
diagnosis. Wilson’s disease (hepatolenticular degeneration) is a rare inherited autosomal recessive disorder of
copper metabolism. Ophthalmic examination may reveal Kayser–Fleischer rings (a rusty-coloured ring around the
iris), and eye movements may be restricted. There may be signs of cirrhosis. Central nervous system signs may
include ataxia, tremor, and cognitive decline. Serum ceruloplasmin is low, and serum copper is low in spite of the
copper deposits in tissues. Urinary copper is high. A full blood count may reveal haemolytic anaemia or decreased
white blood cells. Serum uric acid levels are low.
12661

Tag Question

https://mypastest.pastest.com/Secure/TestMe/Browser/436619[‫ ص‬07:12:43 10/12/1437]


MyPastest

Feedback

Difficulty: Average

Peer Responses

Session Progress

Responses Correct: 0

Responses Incorrect: 31

Responses Total: 31

Responses - % Correct: 0%

End Session
Blog
About Pastest
Contact Us
Help

© Pastest 2016

https://mypastest.pastest.com/Secure/TestMe/Browser/436619[‫ ص‬07:12:43 10/12/1437]


MyPastest

Prefer to use the old MyPastest? Access it here »

Back to Filters

Question 20 of 35

Which one of the following statements is MOST accurate about chronic subdural haematoma?

A The trauma to the head is usually minor and often forgotten by the patient
Previous Question

B Neck stiffness is an early feature

C Headache is often absent


Next Question

D Lumbar puncture and CSF analysis should be done immediately on patient arrival

End Session
E Injury to the middle meningeal artery is the usual cause of the haematoma

Explanation
The clinical syndrome develops weeks and even months after the original trauma. The minor head injury might
not be remembered by the patient. Headache is the earliest feature and may be present almost from the time of the
injury, subsequently, subtle mental changes may occur such as lethargy, loss of initiative, somnolence and even
confusion. If untreated, patients may progress to signs and symptoms of tentorial herniation. The bleeding is from
cerebral veins and neck stiffness is a feature of arachnoid matter irritation due to subarachnoid haemorrhage.
Lumbar puncture should be avoided because of the potential risk of tentorial herniation.
12662

Tag Question

Feedback

https://mypastest.pastest.com/Secure/TestMe/Browser/436619[‫ ص‬07:13:03 10/12/1437]


MyPastest

Difficulty: Average

Peer Responses

Session Progress

Responses Correct: 0

Responses Incorrect: 32

Responses Total: 32

Responses - % Correct: 0%

Blog
About Pastest
Contact Us
Help

© Pastest 2016

https://mypastest.pastest.com/Secure/TestMe/Browser/436619[‫ ص‬07:13:03 10/12/1437]


MyPastest

Prefer to use the old MyPastest? Access it here »

Back to Filters

Question 21 of 35

A boy presented with recurrent convulsions. The convulsions usually occur at night and are confirmed by an
eyewitness. Clinically examination revealed a café-au-lait spot on the right buttock.

Which of the following investigations would be most helpful in suggesting a diagnosis of neurofibromatosis?

A Computed tomography (CT) brain

B Magnetic resonance imaging (MRI) brain

C Slit lamp examination of the eye

D EEG

E Cutaneous biopsy of buttock lesion

Explanation
The implication is that this child may have neurofibromatosis (NF). Early cutaneous signs of NF include cafe-au-
lait
spots and axillary freckling. Diagnosis of NF type 1 is confirmed on slit lamp examination of the eye, which
reveals Lisch nodules (hyperpigmented iris harmatomas).

Peripheral NF (NF1) tends to be


associated with skin neurofibromas and multiple cafe-au-lait spots (6 or more >
5mm in prepubertal children, > 15mm in postpubertal children).

Central NF (NF2) is associated with number of neural tumours including 8th-nerve sheath neurofibromas (acoustic
neuromas), meningioma, glioma and plexiform neuroma. Cutaneous neurofibromas occur with less frequency than
in peripheral NF.

Associated abnormalities include scoliosis, orbital haemangioma, local gigantism of


a limb, phaeochromocytoma,
renal artery stenosis, pulmonary fibrosis, obstructive cardiomyopathy and fibrous dysplasia of bone.
12663

https://mypastest.pastest.com/Secure/TestMe/Browser/436619[‫ ص‬07:13:25 10/12/1437]


MyPastest

Tag Question

Feedback

Difficulty: Average

Peer Responses

Previous Question

Next Question
Session Progress

Responses Correct: 0

Responses Incorrect: 33

Responses Total: End Session 33

Responses - % Correct: 0%

Blog
About Pastest
Contact Us
Help

© Pastest 2016

https://mypastest.pastest.com/Secure/TestMe/Browser/436619[‫ ص‬07:13:25 10/12/1437]


MyPastest

Prefer to use the old MyPastest? Access it here »

Back to Filters

Question 22 of 35

A 6-year-old girl presented following her first seizure


which was a generalised tonic/clonic episode which lasted
for about 5 minutes. There was no associated fever. There was a 9-month history of headaches that occurred once
in about 3-4 weeks that were quite severe and needed analgesics. These were usually diurnal and not accompanied
by
nausea or vomiting. In between she had been normal with no change in her moods or behaviour. There is a
strong family history of migraine with the mother and the maternal grandmother being affected. Clinical
examination was normal.

The most relevant investigation in the evaluation of the patient is?


Next Question
A Fasting blood sugar

B EEG

C CT scan of the brain

D MRI scan of the brain

E Trial with Pizotifen

Explanation
The most relevant investigation here is an MRI scan. The first seizure in a child with significant headache needs
urgent evaluation, preferably by MRI, as some space occupying lesions could be missed on a CT scan. The
absence of vomiting, early morning headaches, neurological changes and normal optic discs do not preclude an
early diagnosis of a cerebral tumour. The EEG may indicate a focus but its negativity should not be relied upon
not to scan the brain. The diagnosis of migraine in the above circumstances should be by elimination of the above.
The child here had a left parietal lobe glioma.
14598

https://mypastest.pastest.com/Secure/TestMe/Browser/436619[‫ ص‬07:13:46 10/12/1437]


MyPastest

Tag Question

Feedback

Difficulty: Difficult

Peer Responses

Previous Question

Session Progress

Responses Correct: 0

Responses Incorrect: 34

Responses Total: 34

Responses - % Correct: 0%
End Session

Blog
About Pastest
Contact Us
Help

© Pastest 2016

https://mypastest.pastest.com/Secure/TestMe/Browser/436619[‫ ص‬07:13:46 10/12/1437]


MyPastest

Prefer to use the old MyPastest? Access it here »

Back to Filters

Question 23 of 35

Theme: Seizures

A Meningitis
B Brain tumour
C Subdural haematoma
D Sturge-Weber syndrome
E Tuberous sclerosis
F Neurofibromatosis
G Hypoglycaemia
H Hypocalcaemia
I Migraine
J Complex partial seizure

For
each of the following scenarios, select the one most likely diagnosis. Each option may be used once, more
than once, or not at all.

Scenario 1

A 15-year-old girl has been unwell for 2 days. She is brought to the Emergency Department complaining of a
severe headache and nausea. Her temperature is 38.7°C. She has been unable to tolerate fluids or food for 24
hours. She has a history of febrile convulsions and complex partial seizures. On examination she has
a capillary
refill of 3 seconds and nuchal rigidity. She has a prolonged generalised tonic clonic seizure while in the
Department.

Your answer was incorrect

Select one...

A - Meningitis

The background history is irrelevant. Pyrexia, headache, meningism and signs of illness indicate meningitis which
must be considered first. Resuscitation measures should
put in place.

Scenario 2

A 10-year-old boy has a history of infantile spasms, generalised tonic clonic seizures, learning difficulties and

https://mypastest.pastest.com/Secure/TestMe/Browser/436619[‫ ص‬07:14:08 10/12/1437]


MyPastest

autistic features. He has three sisters who also have seizures. On examination he has cafe-au-lait patches and an
acne-like rash in his naso-labial grooves. His mother is being investigated for hypertension and renal tumours.

Your answer was incorrect

Select one...

E - Tuberous sclerosis

Tuberous sclerosis is one of a group of conditions called neuro-cutaneous disorders. They are characterised by
genetic and development anomalies in structures of ectodermal origin:

Skin
CNS
PNS

These conditions are usually autosomal dominant, have a high new mutation rate and variable penetrance.
Skin
lesions in TS include: Cafe-au-lait patches, shagreen patches, hypo-pigmented ash-leaf patches, adenoma
sebaceum (acne-like rash), subungual or periungual fibromata.

Next Question
This boy's rash is
adenoma sebaceum, he has a history of infantile spasms and his mother has high blood pressure
due to renal benign angiomyolipomas.

Scenario 3

A 5-day-old baby boy has three short


focal fits at home. Pregnancy was normal. Delivery was by ventouse. Apgars
were normal. He received IM vitamin K and he has been breast-feeding well. Clinical examination is
unremarkable. You notice his mother walks with a limp and has recently had a pathological fracture of her radius.
She tells you she has been advised that she needs more sunlight.

Your answer was incorrect

Select one...

H - Hypocalcaemia

This case demonstrates a mother who has Vitamin D deficiency and as a consequence, her son has low levels also
plus seizures due to hypocalcaemia.

Explanation
 
14891

https://mypastest.pastest.com/Secure/TestMe/Browser/436619[‫ ص‬07:14:08 10/12/1437]


MyPastest

Tag Question

Feedback

Difficulty: Average

Session Progress

Responses Correct: 0

Responses Incorrect: 37

Responses Total: 37

Responses - % Correct: 0%

Previous Question
Blog
About Pastest
Contact Us
Help

© Pastest 2016

End Session

https://mypastest.pastest.com/Secure/TestMe/Browser/436619[‫ ص‬07:14:08 10/12/1437]


MyPastest

Prefer to use the old MyPastest? Access it here »

Back to Filters

Question 24 of 35

Theme: Epilepsy

A Absence epilepsy
B Complex partial seizures (of the temporal lobe)
C Febrile convulsion
D Idiopathic generalised tonic–clonic epilepsy
E Infantile spasms
F Night terror
G Partial seizure
H Pseudoseizure
I Rigors
J Rolandic epilepsy (BECTS)

For
each of the following cases, choose the most likely diagnosis from the above list. Each item may be used once,
more than once or not at all.

Scenario 1

An 8-year-old boy develops episodes of twitching of the left side of his mouth and cheek. They occur any time
during the day and but frequently at night. He has centrotemporal spikes
on EEG.

Your answer was incorrect

Select one...

J - Rolandic epilepsy (BECTS)

These features best fit with ‘benign’ rolandic epilepsy or benign epilepsy with centrotemporal spikes (BECTS).
The age of onset and the type of seizure are typical. The centrotemporal spikes are diagnostic. The term ‘benign’
rolandic epilepsy is slightly misleading in that, although the outcome is usually good with the seizures
disappearing by adulthood, the nocturnal seizures are increasingly linked to SUDEP (sudden unexplained death in
epilepsy). The risk of SUDEP lowers the threshold of many clinicians to start anti-epileptics.

Scenario 2

The EEG of 5-year-old child with poor concentration at school shows 3/s spike-and-wave discharge provoked by

https://mypastest.pastest.com/Secure/TestMe/Browser/436619[‫ ص‬07:14:30 10/12/1437]


MyPastest

hyperventilation.

Your answer was incorrect

Select one...

A - Absence epilepsy

Absence seizures are often first noticed as ‘poor concentration’ at school. The child is concentrating but is having
absences that may be mistaken for daydreaming. Classically, 3/s spikes on EEG are seen. In an outpatient setting
hyperventilation is a useful stimulus to bring on a seizure, thus making the diagnosis.

Scenario 3

A 6-year-old child has episodes of nausea and abdominal pain, followed by repetitive chewing and jerking of the
left arm for 1–2 minutes. He is confused afterwards.

Your answer was incorrect


Previous Question
Select one...

B - Complex partial seizures (of the temporal lobe)


Next Question
The prodromal features described, and the type of seizure described, are common in temporal lobe epilepsy.
22020

End Session

Tag Question

Feedback

Difficulty: Average

Session Progress

Responses Correct: 0

Responses Incorrect: 40

Responses Total: 40

Responses - % Correct: 0%

https://mypastest.pastest.com/Secure/TestMe/Browser/436619[‫ ص‬07:14:30 10/12/1437]


MyPastest

Blog
About Pastest
Contact Us
Help

© Pastest 2016

https://mypastest.pastest.com/Secure/TestMe/Browser/436619[‫ ص‬07:14:30 10/12/1437]


MyPastest

Prefer to use the old MyPastest? Access it here »

Back to Filters

Question 25 of 35

A 7-year-old boy with severe cerebral palsy and developmental


delay was referred for investigation because of
uncontrolled seizures and respiratory problems, initially diagnosed and treated as asthma. He attends a special
school. He is a ‘difficult feeder’ and had
clinical evidence of a chest infection on admission. He was noted to have
recurrent spasmodic movements with arching of the back, throwing the arms and head and lurching forward.
These were associated with gurgling noises in the throat but no loss of awareness. These were more frequent
during respiratory illnesses and especially during or immediately after meals. Two EEGs were reported as normal.
Antiepileptic
drugs have had no significant effect.

The clinical picture is most compatible with:

A Dystonic cerebral palsy

B Uncontrolled asthma

C Partial complex seizures

D Severe gastro-oesophageal reflux

E Immunodeficiency

Explanation
This 7-year-old with cerebral palsy has ‘dystonic’ movements associated with feeding, which also appear to
exacerbate with chest infections. These have not been noted at
other times and specifically no intention tremor or
incoordination has been noted. The constellation of symptoms is compatible with severe gastro-oesophageal reflux
and oesophagitis, which can lead to dystonic movements as a result of the extreme discomfort, especially at and
after
meals. The reflux can be the cause of recurrent chestiness and wheeze, which may present as or exacerbate
pre-existing asthma. The diagnosis of
epilepsy needs to be revised in the absence of a compatible clinical picture,
absence of response to antiepileptic drugs and presence of normal EEGs.
22366

https://mypastest.pastest.com/Secure/TestMe/Browser/436619[‫ ص‬07:14:50 10/12/1437]


MyPastest

Tag Question

Feedback

Difficulty: Easy

Peer Responses

Previous Question

Next Question
Session Progress

Responses Correct: 0

Responses Incorrect: 41

Responses Total: End Session 41

Responses - % Correct: 0%

Blog
About Pastest
Contact Us
Help

© Pastest 2016

https://mypastest.pastest.com/Secure/TestMe/Browser/436619[‫ ص‬07:14:50 10/12/1437]


MyPastest

Prefer to use the old MyPastest? Access it here »

Back to Filters

Question 26 of 35

A 22-month-old boy is seen because there are concerns


about his motor development. He sat unsupported at 10
months, crawled at 1 year and started to walk unsupported at 20 months. His social, language and fine motor
development appear age appropriate. Which of the
following investigations would be the most useful?
Previous Question
A Detailed physiotherapy assessment

B TFTS Next Question

C Creatine kinase (CK) level

D Cranial MRI (magnetic resonance imaging) End Session

E Electromyogram (EMG)

Explanation
Isolated gross motor delay in a child with otherwise normal development must raise the question of muscular
dystrophy. The most useful test in diagnosing this is a CK level. A detailed physiotherapy assessment would in all
likelihood raise the question of muscular dystrophy but a CK would be needed to confirm it. Thyroid function
testing is important in generalised developmental delay, but less do in this case. An MRI and EMG are unlikely to
be helpful initially.
22388

Tag Question

Feedback

https://mypastest.pastest.com/Secure/TestMe/Browser/436619[‫ ص‬07:15:14 10/12/1437]


MyPastest

Difficulty: Average

Peer Responses

Session Progress

Responses Correct: 0

Responses Incorrect: 42

Responses Total: 42

Responses - % Correct: 0%

Blog
About Pastest
Contact Us
Help

© Pastest 2016

https://mypastest.pastest.com/Secure/TestMe/Browser/436619[‫ ص‬07:15:14 10/12/1437]


MyPastest

Prefer to use the old MyPastest? Access it here »

Back to Filters

Question 27 of 35

Regarding self-injury in children with neurodevelopmental disorders - which of the following conditions is NOT
associated with high risk of self-injurious behaviour?

A Cornelia de Lange syndrome

B Fragile X syndome

C Noonan syndrome

D PraderWilli syndrome

E Tuberous syndome

Explanation
The physiological mechanisms underlying self-injurious behaviour in neurodevelopmental disorders are complex
and
relate to abnormalities in sensory processing and pain perception. Self-injury may occur as a response to
chronic pain, but can also cause chronic pain secondary to tissue damage; early intervention is therefore
required
to break self-perpetuating cycle.

Noonan syndrome is a relatively common multiple aomaly syndrome which shares many features with Turner
syndrome, including webbing of the neck, cardiovascular abnormalities and reduced growth. 10% have learning
difficulties but self-injury is not known to be associated.

A is incorrect; there
is a high prevalence of self-injury in Cornelia de Lange syndrome, especially lip biting and
skin picking at the chin, often in response to
pain associated with gastro-oesophageal reflux.

B is incorrect;
children with fragile X syndrome may injure themselves by biting the hand (classically at the
‘anatomical snuff box’ at the base of the thumb) in response to anxiety or excitement.

D is incorrect; Prader-Willi syndrome is associated with skin picking and scratching, which can be quite severe.

E
is incorrect; self-injury is reported in up to 40% of children with TS,
particular those with lower levels of

https://mypastest.pastest.com/Secure/TestMe/Browser/436619[‫ ص‬07:15:33 10/12/1437]


MyPastest

adaptive functioning.

References:
Symons, 2011.Self-Injurious Behavior in Neurodevelopmental Disorders: Relevance of Nociceptive
and Immune Mechanisms. NeurosciBiobehav Rev. 2011 Apr; 35(5): 1266–1274.
http://www.ncbi.nlm.nih.gov/pmc/articles/PMC3086601/

Eden et al., 2014.Self-injury and aggression in tuberous sclerosis complex: cross syndrome comparison and
associated risk markers. J NeurodevDisord. 2014; 6(1): 10.
http://www.ncbi.nlm.nih.gov/pmc/articles/PMC4017702/
47173

Tag Question

Feedback

Previous Question
Difficulty: Average

Peer Responses
Next Question

End Session

Session Progress

Responses Correct: 0

Responses Incorrect: 43

Responses Total: 43

Responses - % Correct: 0%

Blog
About Pastest
Contact Us
Help

© Pastest 2016

https://mypastest.pastest.com/Secure/TestMe/Browser/436619[‫ ص‬07:15:33 10/12/1437]


MyPastest

Prefer to use the old MyPastest? Access it here »

Back to Filters

Question 28 of 35

Regarding developmental problems in Fragile X syndrome (FXS), which of the following statements is most
accurate?

A Around one third of children with FXS meet diagnostic criteria for autistic spectrum disorder

B Children with FXS tend to close the developmental gap with their peers in adolescence

C Intellectual disability is usually in the severe range

D Nonverbal communication skills are usually superior to spoken language

E Premutation carriers do not suffer developmental problems

Explanation
FXS is the second commonest genetic cause of developmental problems (after Down syndrome). There is
significant phenotypic overlap with idiopathic neurodevelopmental problems including
autistic spectrum disorder
and ADHD.

B is incorrect; the impact


of FXS usually becomes more apparent in adolescence, with slowing of skill acquisition
due to increased demands on sequential information processing.

C is incorrect; intellectual disability is usually in the mild to moderate range.

D is incorrect; although spoken language tends to be delayed in acquisition and repetitive in style, nonverbal
blocks to communication including gaze aversion are even more significant.

E is incorrect; most premutation carriers (those with 55-200 triplet repeats at the FXMR1 locus) have normal IQ
but some do suffer developmental problems.

References: Turk, 2011.Fragile X syndrome: lifespan developmental implications for those without as well
as with
intellectual disability.CurrOpin Psychiatry. 2011 Sep;24(5):387-97
47174

https://mypastest.pastest.com/Secure/TestMe/Browser/436619[‫ ص‬07:15:53 10/12/1437]


MyPastest

Tag Question

Feedback

Difficulty: Average

Peer Responses

Previous Question

Next Question
Session Progress

Responses Correct: 0

Responses Incorrect: 44

Responses Total: End Session 44

Responses - % Correct: 0%

Blog
About Pastest
Contact Us
Help

© Pastest 2016

https://mypastest.pastest.com/Secure/TestMe/Browser/436619[‫ ص‬07:15:53 10/12/1437]


MyPastest

Prefer to use the old MyPastest? Access it here »

Back to Filters

Question 29 of 35

A child is seen by a psychologist who undertakes psychometric tests and estimates their full scale IQ to be 55.

Which of the following categories most accurately describes the severity of this child’s intellectual disability?

A Borderline intellectual disability

B Mild intellectual disability

C Moderate intellectual disability

D Profound intellectual disability

E Severe intellectual disability

Explanation
Measurement and categorisation of learning disability or intellectual disability is helpful insofar as it helps the
family and involved professionals to understand the child’s needs
and the kind of support they may require. A
holistic approach is required in which IQ testing forms only one part of the assessment of the child’s strengths and
needs.

The division between categories is not clear cut, but by convention the following IQ ranges apply:

>70 Normal
50-70 Mild intellectual disability
35-49 Moderate intellectual disability
20-34 Severe intellectual disability
<20 Profound intellectual disability

Reference: http://www.bild.org.uk/information/faqs/
47175

https://mypastest.pastest.com/Secure/TestMe/Browser/436619[‫ ص‬07:16:17 10/12/1437]


MyPastest

Tag Question

Feedback

Difficulty: Average

Peer Responses

Previous Question

Next Question
Session Progress

Responses Correct: 0

Responses Incorrect: 45

Responses Total: End Session 45

Responses - % Correct: 0%

Blog
About Pastest
Contact Us
Help

© Pastest 2016

https://mypastest.pastest.com/Secure/TestMe/Browser/436619[‫ ص‬07:16:17 10/12/1437]


MyPastest

Prefer to use the old MyPastest? Access it here »

Back to Filters

Question 30 of 35

Regarding intellectual function in children, which of the following statements about intellectual quotient (IQ) is
most accurate?

A A small number of genes (10-20) account for 22-46% of phenotypic variation in childhood IQ

B IQ at age 17-18 years correlates poorly with IQ at age 5-7

C IQ = chronological age ÷mental age

D The standard deviation of IQ in the population is 15

E 5% of the population are expected to have IQ < 70

Explanation
Various IQ tests are used by psychologists in the
measurement of intellectual function. Most modern tests use
deviation scoring in which each standard deviation up or down from the norming sample median is 15 points from
the median score of 100 so D is correct.

A is incorrect; studies indicate intelligence is heritable but highly polygenic, with only 22-46% of the variance
accounted for by the top 100 single nucleotide polymorphisms (sequence variations).

B is incorrect; there is a strong correlation between IQ at age 17-18 years and IQ at 5-7 (r=0.86).

C is incorrect; the original Stanford-Binet IQ score was calculated by the (now obsolete) method of dividing the
subject’s mental age by their chronological age.

E is incorrect; according to the normal distribution, 2.2% of the population are expected to have IQ < 70.

References:
Benyamin et al., 2014.Childhood intelligence is heritable, highly polygenic and associated with
FNBP1L. Mol Psychiatry. 2014 Feb; 19(2): 253–258. http://www.ncbi.nlm.nih.gov/pmc/articles/PMC3935975/
47176

https://mypastest.pastest.com/Secure/TestMe/Browser/436619[‫ ص‬07:16:38 10/12/1437]


MyPastest

Tag Question

Feedback

Difficulty: Average

Peer Responses

Previous Question

Next Question
Session Progress

Responses Correct: 0

Responses Incorrect: 46

Responses Total: End Session 46

Responses - % Correct: 0%

Blog
About Pastest
Contact Us
Help

© Pastest 2016

https://mypastest.pastest.com/Secure/TestMe/Browser/436619[‫ ص‬07:16:38 10/12/1437]


MyPastest

Prefer to use the old MyPastest? Access it here »

Back to Filters

Question 31 of 35

A 14-year-old girl presents to the outpatient clinic with learning difficulties and behavioural problems. She has
recently moved to the area and the previous medical history is not known, but the family say the problems are
longstanding. She is keen to socialise and able to hold a conversation but asks repetitive questions which annoy
those around her. She is highly insistent on routine and has severe temper outbursts if prevented from accessing
food; however, if access is
not restricted she may eat for up to 2 hours without stopping, and as a
result she has
become severely obese.

Which investigation is most likely to identify the underlying cause for this girl’s problems?

A CSF neurotransmitter levels

B DNA methylation analysis

C Lipid profile

D MRI brain

E Pituitary function tests

Explanation
The combination of severe, compulsive hyperphagia, mild intellectual disability and a behavioural profile
characterised by rigidity, stubbornness and tantrums is highly suggestive of Prader-Willi syndrome (PWS). Failure
of the normal satiety
cascade leads to severe overeating and obesity. The syndrome is caused by microdeletion of
paternal chromosome 15q11.2-q13 (75% of cases), maternal uniparentaldisomy for chromosome 15 (20%) or
imprinting defect (5%). Diagnosis is best achieved by methylation studies of 15q11.2-q13; routine chromosomal
microarray will usually fail to detect theabnormality. Food security (locked cupboards) is essential to manage
overeating and pharmacological treatments are also available.

A is incorrect; CSF neurotransmitter levels are not diagnostic of PWS.

C is incorrect; lipid profile may be abnormal but is not diagnostic of PWS.

https://mypastest.pastest.com/Secure/TestMe/Browser/436619[‫ ص‬07:17:01 10/12/1437]


MyPastest

D is incorrect; MRI brain is normal in PWS.

E is incorrect; some children with PWS suffer hypothalamic deficiency (with associated hypothyroidism and
abnormal pubertal development), but this is not diagnostic of the underlying cause, and not all children are
affected.

References:

https://www.fpwr.org/gb/about-prader-willi-syndrome/

http://bmcmedgenet.biomedcentral.com/articles/10.1186/1471-2350-11-70

Martinez Michel et al., 2016.A review of chemosensory perceptions, food


preferences and food-related behaviours
in subjects with Prader-Willi Syndrome.Appetite. Apr 1;99:17-24
47177

Previous Question Tag Question

Feedback

Next Question
Difficulty: Average

Peer Responses

End Session

Session Progress

Responses Correct: 0

Responses Incorrect: 47

Responses Total: 47

Responses - % Correct: 0%

Blog
About Pastest
Contact Us
Help

https://mypastest.pastest.com/Secure/TestMe/Browser/436619[‫ ص‬07:17:01 10/12/1437]


MyPastest

© Pastest 2016

https://mypastest.pastest.com/Secure/TestMe/Browser/436619[‫ ص‬07:17:01 10/12/1437]


MyPastest

Prefer to use the old MyPastest? Access it here »

Back to Filters

Question 32 of 35

Regarding Prader-Willi syndrome (PWS), which of the following statements is most accurate?

A A small proportion grow out of hyperphagia in later adolescence

B Behavioural profile is dominated by drive to seek novel experiences

C Infants tend to be hypertonic and gain weight excessively

D Pain tolerance is usually poor

E There is increased incidence of psychotic disorders

Explanation
In addition to hyperphagia and food seeking, children with PWS have a characteristic behavioural phenotype
including insistence on sameness and repetition, skin picking, tantrums and oppositionality. They suffer increased
risk of psychotic disorder, affective disorder with psychotic component, and obsessive-compulsive disorder.

A is incorrect; although there are behavioural and pharmacological approaches to aid in the management,
compulsive hyperphagia due to decreased satiety is universal in PWS.

B is incorrect; children with PWS tend to be highly insistent upon routine, and there is clinical overlap with
autistic spectrum disorders.

C
is incorrect; infants with PWS are hypotonic, and dietetic/OT input is often needed to maximise weight gain in
infancy; excessive weight gain does not usually occur until the second or third year of life.

References:

https://www.fpwr.org/gb/about-prader-willi-syndrome/

Dykens& Shah, 2003.Psychiatric disorders in Prader-Willi syndrome: epidemiology and management. CNS Drugs
17(3):167-78.
47178

https://mypastest.pastest.com/Secure/TestMe/Browser/436619[‫ ص‬07:17:22 10/12/1437]


MyPastest

Tag Question

Feedback

Difficulty: Average

Peer Responses

Previous Question

Next Question
Session Progress

Responses Correct: 0

Responses Incorrect: 48
End Session
Responses Total: 48

Responses - % Correct: 0%

Blog
About Pastest
Contact Us
Help

© Pastest 2016

https://mypastest.pastest.com/Secure/TestMe/Browser/436619[‫ ص‬07:17:22 10/12/1437]


MyPastest

Prefer to use the old MyPastest? Access it here »

Back to Filters

Question 33 of 35

A 5-year-old boy presents to the outpatient clinic with social communication concerns. His language is severely
delayed and he usually indicates his needs by grunting or becoming distressed. He rarely engages in shared
attention with parents. In play he prefers to bang toys together or spin them around rather than explore their
function, and he tends to ignore other children who try to interact with
him.

In the previous medical history there was a small abnormality seen in the heart on the antenatal scan, but the
mother cannot recall what this was, and it was never followed up.

On physical examination you note a number of flat red spots on the face. His mother reports that they sometimes
bleed. There are also two firm, non-tender nodules on the oral mucosa.

Which of the following is the least useful investigation to arrange at this stage?

A CT/MRI brain

B CT/MRI chest

C Echocardiogram

D MRI/USS abdomen

E Ophthalmological examination

Explanation
Tuberous sclerosis complex (TSC) is an autosomal dominant multisystem disease characterised by the
development of hamartomas (focal benign overgrowths) in the brain, kidneys, heart, lungs and skin. Up to half of
children with TS meet diagnostic criteria
for autistic spectrum disorders. Definite diagnosis can be made on the
basis of two major features or one major feature plus two minor features. Many of the cutaneous features do not
appear until later childhood. In this case facial angiofibromas (which may not be elevated in early childhood) and
possible cardiac rhabdomyoma are major features,
and intraoral fibromas are a minor feature.

https://mypastest.pastest.com/Secure/TestMe/Browser/436619[‫ ص‬07:17:43 10/12/1437]


MyPastest

It is recommended to screen children at diagnosis for CNS, renal, cardiac and ocular involvement. CT imaging of
the lungs is recommended for female patients in their late teens to screen for lymphangioleiomyomatosis, but is
not required at this age.

A is incorrect; neuroimaging is mandatory for identification of tubers, calcified sub-ependymal nodules, sub-
ependymal giant cell astrocytoma, hydrocephalus, cortical dysplasia and/or migration lines.

C is incorrect; echocardiogram is advisable in this case where there is a history of possible antenatal cardiac
rhabdomyoma. These are present in up to half of infants with TSC
but tend to regress over time.

D is incorrect; renal imaging is mandatory to screen for renal angiomyolipomas or cysts.

E is incorrect; examination by an ophthalmologist is recommended to screen for visual problems and/or


abnormalities of the retina.

References: http://www.tuberous-sclerosis.org

http://www.tsalliance.org/pages.aspx?content=731
47497

Previous Question

Tag Question
Next Question
Feedback

Difficulty: Average

Peer Responses End Session

Session Progress

Responses Correct: 0

Responses Incorrect: 49

Responses Total: 49

Responses - % Correct: 0%

https://mypastest.pastest.com/Secure/TestMe/Browser/436619[‫ ص‬07:17:43 10/12/1437]


MyPastest

Blog
About Pastest
Contact Us
Help

© Pastest 2016

https://mypastest.pastest.com/Secure/TestMe/Browser/436619[‫ ص‬07:17:43 10/12/1437]


MyPastest

Prefer to use the old MyPastest? Access it here »

Back to Filters

Question 34 of 35

A 2-year-old girl with developmental delay is referred to the


paediatric outpatient clinic by the audiologist
following identification of moderate severity sensorineural hearing loss in the left ear and severe mixed
conductive/sensorineural hearing loss in the right ear. There is a history of recurrent ear infections. Her
development in early infancy was normal but has slowed markedly in her second year. She has no words and
limited babble. She is able to crawl but cannot stand or walk independently. There has been little or no
developmental progress in the past 4 months.

On examination her height and weight are below the 0.4th centile (previously 25-50th centile at 3 months of age).
There is facial dysmorphism with a protruding tonguethat is rather large relative to the teeth. The liver edge is
palpable 3cm below the costal margin on the right. There is also
a firm, mobile mass palpable below the costal
margin on the left.

MRI brain under sedation has failed and the child is awaiting MRI under GA at the tertiary centre.

Which further investigation is most likely to lead to a definitive diagnosis for this child?

A Acylcarnitine profile

B DNA microarray

C Plasma amino acids

D Urine glycosaminoglycans

E Urine organic acids

Explanation
The history of normal development followed by delay and plateau, reduced growth, mixed hearing impairment,
recurrent upper respiratory tract infections, micrognathia/macroglossia and hepatosplenomegaly is suggestive of
mucopolysaccharidosis type I (MPS I). This is a rare (1 in 100,000) lysosomal storage disorder caused by
mutations in the IDUA gene. Failure to breakdown glycosaminoglycans (GAGs) leads to accumulation and

https://mypastest.pastest.com/Secure/TestMe/Browser/436619[‫ ص‬07:18:04 10/12/1437]


MyPastest

multiorgan disease. Elevated levels of


GAGs can be detected in the urine.

Other features include clouding of the cornea, cardiomyopathy and coarse facial features. Subtypes include Hurler
syndrome and Scheie syndrome but there is significant overlap of clinical features between types. The prognosis
in
Hurler syndrome is poor; death usually occurs before the second decade due to cardiorespiratory complications.

A is incorrect; acylcarnitine profile is abnormal in fatty acid oxidation defects and organic acid disorders.

B is incorrect; microarray is used for detection of large scale chromosomal deletions or rearrangements, but cannot
detect single gene mutations.

C is incorrect; plasma amino acids are abnormal in urea cycle disorders, homocystinuria and non-
ketotichyperglycinaemia.

E is incorrect; urine organic acids are abnormal in organic acid disorders.

References:

http://rarediseases.org/rare-diseases/lysosomal-storage-disorders/

https://ghr.nlm.nih.gov/condition/mucopolysaccharidosis-type-i#resources
Previous
Cleary and Green, Questiondelay: when to suspect and how to investigate for an inborn error of
2005.Developmental
metabolism. Arch Dis Child 90:1128-1132. http://adc.bmj.com/content/90/11/1128.full
47498

Next Question

Tag Question

Feedback End Session

Difficulty: Average

Peer Responses

Session Progress

Responses Correct: 0

Responses Incorrect: 50

https://mypastest.pastest.com/Secure/TestMe/Browser/436619[‫ ص‬07:18:04 10/12/1437]


MyPastest

Responses Total: 50

Responses - % Correct: 0%

Blog
About Pastest
Contact Us
Help

© Pastest 2016

https://mypastest.pastest.com/Secure/TestMe/Browser/436619[‫ ص‬07:18:04 10/12/1437]


MyPastest

Prefer to use the old MyPastest? Access it here »

Back to Filters

Question 35 of 35

A girl aged 2 years and nine months presents to the outpatient clinic with delayed acquisition of speech and
developmental delay. She babbles but speaks no identifiable words whatsoever. Her receptive language is
relatively preserved but she is severely delayed in all developmental domains and is not yet walking.

On examination she is an excitable girl who laughs frequently and is keen to crawl around the room and engage in
social interaction. Her eye contact is good. She makes flapping movements of the hands when she is excited. Her
purposeful upper limb movements are rather ataxic.

What is the most likely explanation for this child’s speech and language delay?

A Angelman syndrome

B Autistic spectrum disorder

C Rett syndrome

D Specific language impairment

E Williams syndrome

Explanation
Angelman syndrome (AS) is a neurodevelopmental disorder caused by loss of expression of the maternally
inherited UBE3A gene in the brain. Incidence is as high as 1 in 10,000. Several key features are described in this
scenario including severe developmental delay with little or no expressive language (but preserved motivation for
social contact), ataxia, hypermotor behaviours and a happy demeanour.

B is incorrect; children with ASD and the severe degree of developmental impairment described tend to have
reduced motivation for social contact, poor eye contact and a withdrawn demeanour. Ataxia is unusual in isolated
ASD.

C is incorrect; the Rett’s phenotype can be similar to AS, but children with typical Rett’s characteristically

https://mypastest.pastest.com/Secure/TestMe/Browser/436619[‫ ص‬07:18:26 10/12/1437]


MyPastest

manifest developmental regression, which is not described in this case.

D is incorrect;
children with specific language impairment do not have developmental delay in other domains and
would not be expected to demonstrate ataxia.

E is correct; Williams syndrome shares with AS a strong motivation for social contact, but developmental delay is
usually mild to moderate rather than severe.

References:

Robb et al., 1989. The ‘happy puppet’ syndrome of Angelman: review of the clinical features. Arch Dis Child
64(1): 83–86.

http://www.ncbi.nlm.nih.gov/pmc/articles/PMC1791803/
47499

End Session

Previous Question Tag Question

Feedback

Difficulty: Average

Peer Responses

Session Progress

Responses Correct: 0

Responses Incorrect: 51

Responses Total: 51

Responses - % Correct: 0%

Blog
About Pastest
Contact Us
Help

https://mypastest.pastest.com/Secure/TestMe/Browser/436619[‫ ص‬07:18:26 10/12/1437]


MyPastest

© Pastest 2016

https://mypastest.pastest.com/Secure/TestMe/Browser/436619[‫ ص‬07:18:26 10/12/1437]


MyPastest

Previous Question
Prefer to use the old MyPastest? Access it here »

Back to Filters

Question 1 of 29

Abnormal movements in infancy are caused by a wide range of aetiologies with variable prognosis.

Which of the following is most likely to be associated with a normal developmental outcome?

A Alternating hemiplegia of childhood

B Infantile spasms

C Reflex anoxic seizures

D Startle disease

E Vitamin B12 deficiency

46121

Next Question Tag Question

Feedback

Difficulty: Average
End Session
Peer Responses

https://mypastest.pastest.com/Secure/TestMe/Browser/436619[‫ ص‬06:51:53 10/12/1437]


MyPastest

Session Progress

Responses Correct: 0

Responses Incorrect: 1

Responses Total: 1

Responses - % Correct: 0%

Blog
About Pastest
Contact Us
Help

© Pastest 2016

https://mypastest.pastest.com/Secure/TestMe/Browser/436619[‫ ص‬06:51:53 10/12/1437]


MyPastest

Previous Question
Prefer to use the old MyPastest? Access it here »

Back to Filters

Question 2 of 29

A 14-month-old girl with quadriplegic cerebral palsy, failure


to thrive and profound developmental delay presents
with a two month history of episodic back arching and dystonic posturing of the upper limbs. Episodes occur
several times daily, most often in the evening when she lies on the floor to watch television after her nasogastric
tube feed. She has a previous history of neonatal seizures which the family reports were caused by hypoxic
ischaemic injury at birth, and were thought to have resolved.

Which investigation is most likely to lead to a definitive explanation of this child’s movement disorder?

A EEG

B Genetic testing

C MRI brain

D Nerve conduction study

E Upper GI endoscopy

46122

Next Question

Tag Question
End Session
Feedback

Difficulty: Average

Peer Responses

https://mypastest.pastest.com/Secure/TestMe/Browser/436619[‫ ص‬06:52:30 10/12/1437]


MyPastest

Session Progress

Responses Correct: 0

Responses Incorrect: 2

Responses Total: 2

Responses - % Correct: 0%

Blog
About Pastest
Contact Us
Help

© Pastest 2016

https://mypastest.pastest.com/Secure/TestMe/Browser/436619[‫ ص‬06:52:30 10/12/1437]


MyPastest

Prefer to use the old MyPastest? Access it here »

Back to Filters

Question 3 of 29

A 15-year-old girl presents to the outpatient clinic with a three month history of daily headache. Her parents report
she is sleeping more than previously and is quicker to become agitated, but they had attributed this to “becoming a
teenager”. In the background history she is a previously well child who started puberty when she was 9 years old.
She has felt light-headed on a few occasions but there is no shortness of breath or pallor and she has not yet started
her periods. Her appetite has been increased recently. Her routine observations and neurological examination are
normal.

What is the most likely diagnosis?

A Anaemia

B Brain tumour

C Carbon monoxide poisoning

D Hypothyroidism

E Illicit drug use

Explanation
Children and teenagers presenting with headache should be evaluated for a wide range of secondary causes. This
teenager has a pituitary fossa tumour which has led to hormonally-mediated changes in her personality, diurnal
rhythm and appetite. The crucial clinical feature to elicit from this history is that of arrested puberty; time from
onset of puberty to menarche is usually four years or
less.

80-90% of pituitary fossa tumours in children are craniopharyngiomas, which although almost never malignant,
usually require surgical resection +/- adjunctive radiotherapy due to compression of local structures. As the
tumour grows larger it may compress the optic chiasm leading to bitemporal hemianopia, but the neurological
examination in the early stages may be normal.

A is incorrect; anaemia severe enough to cause headache and agitation would also cause tachycardia and pallor.

https://mypastest.pastest.com/Secure/TestMe/Browser/436619[‫ ص‬06:52:53 10/12/1437]


MyPastest

C
is incorrect; chronic carbonQuestion
Previous monoxide poisoning is an under-recognised cause of daily headache, but does not
cause pubertal arrest.

D is
incorrect; hypothyroidism may present with headache, increased sleeping
and arrested puberty, but appetite is
usually reduced, and change in mood typically manifests as depression rather than agitation.

E is incorrect; drug abuse may lead to headache and personality change, but is not known to cause pubertal arrest.

References:

http://www.ncbi.nlm.nih.gov/pmc/articles/PMC2743125/

http://www.headsmart.org.uk/
46123

Tag Question

Feedback

Difficulty: Average

Peer Responses

Session Progress

Responses Correct: Next Question 0

Responses Incorrect: 3

Responses Total: 3

Responses - % Correct: 0%
End Session

Blog
About Pastest
Contact Us
Help

© Pastest 2016

https://mypastest.pastest.com/Secure/TestMe/Browser/436619[‫ ص‬06:52:53 10/12/1437]


MyPastest

https://mypastest.pastest.com/Secure/TestMe/Browser/436619[‫ ص‬06:52:53 10/12/1437]


MyPastest

Prefer to use the old MyPastest? Access it here »

Back to Filters

Question 4 of 29

A previously well 11-year-old boy presents to the emergency department with a two week history of headache
occurring most days. Pain
is in the suboccipital area bilaterally and radiates down into the neck. He describes the
pain as a continuous pressure. Severity is 5/10; he is able to mobilise around the house during episodes, which last
up to 2-3 hours, but is distracted from his usual activities. Episodes usually occur in the evening but do not wake
him from sleep. There is no
history of nausea, vomiting, fever or rash.

On examination the
patient is fully alert and oriented, and complaining of headache. Blood
pressure is 102/62, HR
114, RR 22, T37.4°C. Fundoscopy is normal although he complains of pain when the light is shone into the eye.
There is no cranial bruit. Neck extension and rotation are painless but neck flexion causes pain. The posterior
aspect of the neck and shoulders
are tender to palpation. The functional neurological examination is normal.

What is the most appropriate initial investigation for this child?

A CT Head

B Inflammatory markers

C Lumbar puncture

D MRI head

E None

Explanation
This child meets the ICHD-3 criteria for episodic tension type headache with pericranial muscle tenderness.

At least two of the following:

Bilateral location
Pressing or tightening (non-pulsating) quality

https://mypastest.pastest.com/Secure/TestMe/Browser/436619[‫ ص‬06:53:16 10/12/1437]


MyPastest

Mild or moderate intensity


Previous Question
Not aggravated by routine physical activity such as walking or climbing stairs

Both of the following:

No nausea or vomiting
No more than one of photophobia or phonophobia

Diagnosis is on the basis of clinical assessment; investigations are not required in the absence of features
suggestive of raised intracranial pressure, space occupying lesion or infection.

A and D are incorrect. Neuroimaging may be required in children with headache and one or more of the following:

Not meeting diagnostic criteria for one of the primary headache disorders
Abnormal neurological examination
Rapidly progressing subacute headache
New headache in an immunosuppressed child
First or worst headache
Systemic symptoms
History of seizures
Headaches awakening the child from sleep
Intractable vomiting

B is incorrect. There is nothing in the history or examination to suggest infection or inflammation as a likely
aetiology in this case.

C is incorrect. Clinical presentations in which infection, neuroinflammation or subarachnoid haemorrhage are


suspected may merit LP. Of note there is no history of fever, nausea or vomiting in this case. Localised muscle
tenderness in the neck is not unusual in tension-type headache.

References:

Kacperski et al.The optimal management of headaches in children and adolescents.TherAdvNeurolDisord. 2016


Jan; 9(1): 53–68.
Next Question
Headache Classification Committee of the International Headache Society (IHS). The International Classification
of Headache Disorders, 3rd edition. Cephalalgia 2013; 33:629.
46124

End Session

Tag Question

Feedback

https://mypastest.pastest.com/Secure/TestMe/Browser/436619[‫ ص‬06:53:16 10/12/1437]


MyPastest

Difficulty: Average

Peer Responses

Session Progress

Responses Correct: 0

Responses Incorrect: 4

Responses Total: 4

Responses - % Correct: 0%

Blog
About Pastest
Contact Us
Help

© Pastest 2016

https://mypastest.pastest.com/Secure/TestMe/Browser/436619[‫ ص‬06:53:16 10/12/1437]


MyPastest

Prefer to use the old MyPastest? Access it here »

Back to Filters

Question 5 of 29

A 15-year-old girl presents to the outpatient clinic with a 2-year history of headache episodes which have been
getting worse for the past six months. For the last two months she has been suffering daily headaches for which
she takes regular 500mg paracetamol 3-4 times daily and 200mg ibuprofen 2-3 times daily. Six weeks ago she was
admitted to hospital overnight and given oramorph, which fully relieved her headache for a period of 12 hours.
MRI head during that admission was normal. There is a family history of cyclical vomiting syndrome and
migraine with aura. On examination she weighs 67kg, has a BP of 115/68, and the systemic and neurological
examination is unremarkable.

What is the most appropriate next step in the management?

A Stop ibuprofen and optimise dose of daily paracetamol

B Stop ibuprofen and paracetamol

C Stop ibuprofen, optimise dose of daily paracetamol and start sublingualtriptan

D Stop paracetamol and optimise dose of daily ibuprofen

E Stop paracetamol, optimise dose of daily ibuprofen and start codeine phosphate

Explanation
This child most likely has a primary headache disorder of migraine which has been transformed into a medication
overuse headache.

Excessive use of simple analgesia can cause medication overuse headache in children with primary headache
disorders,
and may contribute to headache in 20-30% of children with chronic daily
headache. Complete and
abrupt withdrawal of analgesia is accepted as the management of choice. After the withdrawal period they may be
reintroduced, but medications to abort headache should be used no more than three times weekly, regardless of
indication for use. When a headache episode occurs, medication should be used as quickly as possible, or at the
onset of aura. Paracetamol is less effective than ibuprofen or triptans for management of acute migraine.

https://mypastest.pastest.com/Secure/TestMe/Browser/436619[‫ ص‬06:53:37 10/12/1437]


MyPastest

All other answers are incorrect as medications to abort headache should be used no more than three times weekly.
Previous Question
References:

Kacperski et al.The optimal management of headaches in children and adolescents.TherAdvNeurolDisord. 2016


Jan; 9(1): 53–68.

Munksgaard and Jensen. Medication overuse headache. Headache. 2014 Jul-Aug;54(7):1251-7.


46125

Tag Question

Feedback

Difficulty: Average

Peer Responses

Session Progress

Responses Correct: 0

Responses Incorrect: 5

Responses Total: 5

Responses - % Correct: Next Question 0%

Blog
About Pastest
Contact Us
Help
End Session
© Pastest 2016

https://mypastest.pastest.com/Secure/TestMe/Browser/436619[‫ ص‬06:53:37 10/12/1437]


MyPastest

Previous Question
Prefer to use the old MyPastest? Access it here »

Back to Filters

Question 6 of 29

An 8-year-old boy presented to the emergency department with a


sudden onset of generalised tonic-clonic seizure
while he was in class.
The seizure lasted for 5 minutes, and was aborted spontaneously. There was no preceding
aura. He has had a fever for the past 3 days that resolved after taking paracetamol. He recorded a temperature of
38.5°C, and throat examination revealed bilateral exudative tonsillitis. He has previously been treated for febrile
seizure, once at
the age of 18 months, and subsequently at 2 years of age, and again at 5
years. His mother
revealed that she too suffered from repeated febrile seizures when she was young. His growth and development
are up to age and he is an above average student at school.

What is the MOST likely diagnosis?

A Epilepsy with myoclonic absences

B Febrile seizure plus

C Juvenile absence epilepsy

D Juvenile myoclonic epilepsy

E Temporal lobe epilepsy


Next Question
Explanation
46637

End Session

Tag Question

Feedback

https://mypastest.pastest.com/Secure/TestMe/Browser/436619[‫ ص‬06:54:13 10/12/1437]


MyPastest

Difficulty: Average

Peer Responses

Session Progress

Responses Correct: 0

Responses Incorrect: 6

Responses Total: 6

Responses - % Correct: 0%

Blog
About Pastest
Contact Us
Help

© Pastest 2016

https://mypastest.pastest.com/Secure/TestMe/Browser/436619[‫ ص‬06:54:13 10/12/1437]


MyPastest

Previous Question
Prefer to use the old MyPastest? Access it here »

Back to Filters

Question 7 of 29

A previously well 8-month-old girl was filmed by her mother having abnormal movements. She brought her child
to the clinic today as these abnormal movements are becoming more frequent. The recording shows
a conscious
and well 8-month-old who holds her hips in flexion and adduction, in a rhythmic motion. Her face appeared
flushed. The event lasted for 1 minute, until her mother called out her name. The infant appeared uneasy for a
brief moment before continuing as before. Her mother adds that this activity usually occurs when she is in the
back of
the car on quiet rides or when she is alone. Her development is up to age and she has a normal head
circumference.

What is the next BEST step in managing this infant?

A Arrange for an EEG

B Arrange for an MRI Brain

C Referral to the Paediatric Neurologist

D Reassurance and follow up

E Start the child on Sodium Valproate


Next Question
46638

End Session
Tag Question

Feedback

Difficulty: Average

https://mypastest.pastest.com/Secure/TestMe/Browser/436619[‫ ص‬06:54:33 10/12/1437]


MyPastest

Peer Responses

Session Progress

Responses Correct: 0

Responses Incorrect: 7

Responses Total: 7

Responses - % Correct: 0%

Blog
About Pastest
Contact Us
Help

© Pastest 2016

https://mypastest.pastest.com/Secure/TestMe/Browser/436619[‫ ص‬06:54:33 10/12/1437]


MyPastest

Previous Question
Prefer to use the old MyPastest? Access it here »

Back to Filters

Question 8 of 29

A 6-week-old infant is brought in to the emergency department


by worried parents with a complaint of ‘seizures’.
Both parents are paramedics in the hospital. They explained that their baby has jerky movements involving both
her upper and lower limbs that occurs
2-3 times every time she sleeps. These episodes never wake her and she
sleeps through them, stopping when parents wake her up. She feeds well. She has no inconsolable cry. She was
born at term with no perinatal complications. Neurological examination performed was normal. This is their first
child, and it worries them as they have a cousin whose 3-year-old child was recently diagnosed with epilepsy.

What is the MOST likely diagnosis?

A Benign neonatal sleep myoclonus

B Myoclonic epilepsy of infancy

C Self-gratification disorder

D Shuddering spells

E West syndrome

Next Question 46639

End
Tag Session
Question

Feedback

Difficulty: Average

https://mypastest.pastest.com/Secure/TestMe/Browser/436619[‫ ص‬06:54:53 10/12/1437]


MyPastest

Peer Responses

Session Progress

Responses Correct: 0

Responses Incorrect: 8

Responses Total: 8

Responses - % Correct: 0%

Blog
About Pastest
Contact Us
Help

© Pastest 2016

https://mypastest.pastest.com/Secure/TestMe/Browser/436619[‫ ص‬06:54:53 10/12/1437]


MyPastest

Previous Question
Prefer to use the old MyPastest? Access it here »

Back to Filters

Question 9 of 29

A 4-month-old infant presents with stiffening episodes of his


body, arms and legs. This occurs frequently
throughout the day and he can have 10-15 episodes per day. He was born term with a Ventouse assisted delivery.
During examination, his head lags on being lifted.

Which investigation will MOST likely identify the aetiology of this diagnosis?

A Chromosomal study

B MRI Brain

C Serum and urine amino acid

D Serum biotinidase

E TSC – 1 gene study

46640

Next Question

Tag Question

Feedback
End Session
Difficulty: Average

Peer Responses

https://mypastest.pastest.com/Secure/TestMe/Browser/436619[‫ ص‬06:55:12 10/12/1437]


MyPastest

Session Progress

Responses Correct: 0

Responses Incorrect: 9

Responses Total: 9

Responses - % Correct: 0%

Blog
About Pastest
Contact Us
Help

© Pastest 2016

https://mypastest.pastest.com/Secure/TestMe/Browser/436619[‫ ص‬06:55:12 10/12/1437]


MyPastest

Previous Question
Prefer to use the old MyPastest? Access it here »

Back to Filters

Question 10 of 29

A 5-year-old boy developed a right-sided focal seizure and was subsequently admitted to the general ward for
management. He was afebrile and previously well. His mother adds that over the past 2 months, he has been
struggling to speak, only being able to speak in one
or two words. He was previously able to speak in full 5-8
word sentences appropriate for his age.

Which of the following is the MOST likely diagnosis?

A Absence seizure

B Autism spectrum disorder

C Benign Epilepsy of Childhood with Centro-Temporal Spikes (BECTS)

D Landau Kleffner Syndrome

E Lennox Gastaut Syndrome

46641

Next Question

Tag Question

Feedback End Session

Difficulty: Average

Peer Responses

https://mypastest.pastest.com/Secure/TestMe/Browser/436619[‫ ص‬06:55:41 10/12/1437]


MyPastest

Session Progress

Responses Correct: 0

Responses Incorrect: 10

Responses Total: 10

Responses - % Correct: 0%

Blog
About Pastest
Contact Us
Help

© Pastest 2016

https://mypastest.pastest.com/Secure/TestMe/Browser/436619[‫ ص‬06:55:41 10/12/1437]


MyPastest

Previous Question
Prefer to use the old MyPastest? Access it here »

Back to Filters

Question 11 of 29

A 3-year-old boy presented in status epilepticus. He had an episode of pallor with retching and vomiting before
the onset of seizures. He has been previously well with normal developmental milestones. Occipital spikes were
seen on EEG.

What is the MOST likely diagnosis?

A Benign Epilepsy of Childhood with Centro-Temporal Spikes (BECTS)

B Childhood Occipital Epilepsy (Gastaut type)

C Mitochondrial Encephalopathy, Lactic Acidosis and Stroke episodes (MELAS)

D Panayiotopoulos Syndrome

E Temporal lobe epilepsy

46642

Next Question

Tag Question

Feedback
End Session
Difficulty: Average

Peer Responses

https://mypastest.pastest.com/Secure/TestMe/Browser/436619[‫ ص‬06:56:01 10/12/1437]


MyPastest

Session Progress

Responses Correct: 0

Responses Incorrect: 11

Responses Total: 11

Responses - % Correct: 0%

Blog
About Pastest
Contact Us
Help

© Pastest 2016

https://mypastest.pastest.com/Secure/TestMe/Browser/436619[‫ ص‬06:56:01 10/12/1437]


MyPastest

Previous Question
Prefer to use the old MyPastest? Access it here »

Back to Filters

Question 12 of 29

An 18-month-old boy was sent to nursery for the first time today. He cried incessantly, turned blue and limp, and
developed bilateral jerky movements of his upper and lower limbs. This lasted for less than a minute. He has had
previous episodes before, usually during play at home with his elder sister who is 3 years old. He usually sleeps
after these episodes.

What is the MOST helpful step in determining the diagnosis?

A Complete Blood Count

B CT Brain

C Electroencephalogram (EEG)

D MRI Brain

E Video recording

46643

Next Question

Tag Question

Feedback End Session

Difficulty: Average

Peer Responses

https://mypastest.pastest.com/Secure/TestMe/Browser/436619[‫ ص‬06:56:21 10/12/1437]


MyPastest

Session Progress

Responses Correct: 0

Responses Incorrect: 12

Responses Total: 12

Responses - % Correct: 0%

Blog
About Pastest
Contact Us
Help

© Pastest 2016

https://mypastest.pastest.com/Secure/TestMe/Browser/436619[‫ ص‬06:56:21 10/12/1437]


MyPastest

Prefer to use the old MyPastest? Access it here »

Back to Filters

Question 13 of 29

A 15-year-old boy was out with his friends in a dance club. He collapsed and developed jerky movements of his
upper limbs, according
to his friend who brought him to the casualty. Further history revealed
that he has been
clumsy recently, frequently dropping his toothbrush while brushing his teeth in the morning.

Which of the following drug should be avoided in treating him?

A Carbamazepine

B Lamotrigine

C Levetiracetam

D Topiramate

E Valproate

Explanation
The history is suggestive of Juvenile Myoclonic Epilepsy. Sleep deprivation and alcohol consumption precipitates
seizure
attacks. Management will include performing an EEG which will show periodic 3Hz spikes.

Carbamazepine, Phenytoin and Oxcarbazepine aggravate myoclonic seizures and should be avoided.

The combination of valproate and lamotrigine may have synergistic effects, but caution must be taken given the
increased risk of serious skin reactions with rapid titration of lamotrigine, particularly in children younger than 12
years.

Levetiracetam is an effective add on therapy in JME.

Topiramate is an adjunctive therapy, and not contraindicated in JME.

https://www.nice.org.uk/guidance/cg137
46644

https://mypastest.pastest.com/Secure/TestMe/Browser/436619[‫ ص‬06:56:57 10/12/1437]


MyPastest

Previous Question

Tag Question

Feedback

Difficulty: Average

Peer Responses

Session Progress

Responses Correct: 0

Responses Incorrect: 13

Responses Total: 13

Responses - % Correct: 0%

Blog
About Pastest
Contact Us
Help

© Pastest 2016
Next Question

End Session

https://mypastest.pastest.com/Secure/TestMe/Browser/436619[‫ ص‬06:56:57 10/12/1437]


MyPastest

Previous Question
Prefer to use the old MyPastest? Access it here »

Back to Filters

Question 14 of 29

A 3-month-old infant presents with arching of his back, usually after a feed. It occurs 4-5 times per day. He was
born at 29 weeks gestation with a birth weight of 950g and had a stormy neonatal period. He developed chronic
lung disease and is now on home oxygen therapy.

What is the MOST likely diagnosis?

A Benign neonatal myoclonus

B Benign paroxysmal torticollis in infancy

C Cerebral palsy

D Hyperekplexia

E Sandifer syndrome

46645

Next Question

Tag Question

Feedback
End Session
Difficulty: Average

Peer Responses

https://mypastest.pastest.com/Secure/TestMe/Browser/436619[‫ ص‬06:57:16 10/12/1437]


MyPastest

Session Progress

Responses Correct: 0

Responses Incorrect: 14

Responses Total: 14

Responses - % Correct: 0%

Blog
About Pastest
Contact Us
Help

© Pastest 2016

https://mypastest.pastest.com/Secure/TestMe/Browser/436619[‫ ص‬06:57:16 10/12/1437]


MyPastest

Previous Question
Prefer to use the old MyPastest? Access it here »

Back to Filters

Question 15 of 29

An 18-month-old infant slipped by the poolside and hit the back of his head. Immediately afterwards he turned
pale and limp, and passed out. He regained consciousness after his parents carried him away
from the pool. He
sustained a small swelling over the occipital part of
his head. He has been well otherwise with normal
developmental milestones to date.

What is the MOST likely diagnosis?

A Absence seizure

B Breath holding spell

C Posterior fossa bleed

D Prolonged QT syndrome

E Reflex anoxic seizure

46646

Next Question

Tag Question

Feedback End Session

Difficulty: Average

Peer Responses

https://mypastest.pastest.com/Secure/TestMe/Browser/436619[‫ ص‬06:57:36 10/12/1437]


MyPastest

Session Progress

Responses Correct: 0

Responses Incorrect: 15

Responses Total: 15

Responses - % Correct: 0%

Blog
About Pastest
Contact Us
Help

© Pastest 2016

https://mypastest.pastest.com/Secure/TestMe/Browser/436619[‫ ص‬06:57:36 10/12/1437]


MyPastest

Prefer to use the old MyPastest? Access it here »

Back to Filters

Question 16 of 29

A 6-year-old, previously well girl presents to the emergency department with a two week history of headache,
lethargy and vomiting. On examination her BP is 134/72, HR 88, RR 24, T37.4°C, sats 99% O/A. The child is
sitting on her mother’s lap, speaking in short sentences, alert and oriented but irritable. Reflexes are brisk in the
right lower limb with upgoing plantars bilaterally and 5-6 beats of clonus at the right ankle.

Urgent MRI brain and spine is performed and reveals a large space-occupying lesion with surrounding oedema in
the left cerebral hemisphere. There is effacement of the left lateral ventricle and deviation of the midline away
from the lesion. Diffusion weighted imaging shows low signal (no restriction of diffusion).

Analgesia has been given, neurosurgeons have been contacted, and the child is being nursed with the head
elevated. Routine
bloods are pending.

What is the most important medication or medications to commence at this point?

A Ceftriaxone, metronidazole and azithromycin

B Dexamethasone

C Dexamethasone and omeprazole

D Labetalol

E Labetalol and propranolol

Explanation
This child has a space-occupying lesion which is causing mass effect and possibly raised intracranial pressure,
leading to vomiting, lethargy and irritability. Diffusion weighted imaging supports the diagnosis of brain tumour.

In this situation high dose dexamethasone (with gastroprotection) would reduce surrounding oedema and alleviate
symptoms while the patient is transferred to a neurosurgical centre for surgical resection of the tumour.

A is incorrect; the absence of fever, immunosuppression or antecedent infection in the history makes brain abscess

https://mypastest.pastest.com/Secure/TestMe/Browser/436619[‫ ص‬06:58:01 10/12/1437]


MyPastest

an unlikely differential in
this case. Additonally, brain abscess would show high signal on diffusion weighted
Previous Question
imaging (diffusion is restricted due to the viscosity
of the pus).

B is incorrect; high dose steroid without gastroprotection would expose the child to higher risk of peptic
ulceration.

D
and E are incorrect; the blood pressure is abnormally high but it would
be dangerous to treat this aggressively in
the context of possibly raised intracranial pressure: higher than normal mean arterial pressure may be required to
maintain cerebral perfusion pressure if intracranial pressure is elevated. Advice should be sought from the renal
and neurosurgical teams.
46713

Tag Question

Feedback

Difficulty: Average

Peer Responses

Session Progress

Responses Correct: 0

Responses Incorrect:
Next Question 16

Responses Total: 16

Responses - % Correct: 0%

End Session

Blog
About Pastest
Contact Us
Help

© Pastest 2016

https://mypastest.pastest.com/Secure/TestMe/Browser/436619[‫ ص‬06:58:01 10/12/1437]


MyPastest

https://mypastest.pastest.com/Secure/TestMe/Browser/436619[‫ ص‬06:58:01 10/12/1437]


MyPastest

Previous Question
Prefer to use the old MyPastest? Access it here »

Back to Filters

Question 17 of 29

An 8-year-old boy is brought to the emergency department from


school with a one hour history of headache,
localised to the front of the head bilaterally, and described as a “pressing” pain of moderate severity. He feels
nauseous and has vomited once. There is no fever, neck stiffness, rash or encephalopathy. There is no aura, visual
disturbance, phonophobia or photophobia. The neurological examination is
normal. He reports a history of similar
episodes occurring approximately once monthly for the past six months.

What is the most likely diagnosis?

A Cluster headache

B Intracranial space-occupying lesion

C Migraine without aura

D Subarachnoid haemorrhage

E Tension-type headache

46714

Next Question

Tag Question
End Session
Feedback

Difficulty: Average

Peer Responses

https://mypastest.pastest.com/Secure/TestMe/Browser/436619[‫ ص‬06:58:22 10/12/1437]


MyPastest

Session Progress

Responses Correct: 0

Responses Incorrect: 17

Responses Total: 17

Responses - % Correct: 0%

Blog
About Pastest
Contact Us
Help

© Pastest 2016

https://mypastest.pastest.com/Secure/TestMe/Browser/436619[‫ ص‬06:58:22 10/12/1437]


MyPastest

Previous Question
Prefer to use the old MyPastest? Access it here »

Back to Filters

Question 18 of 29

A 9-year-old boy with known epilepsy and no known allergies is brought to the emergency department resus by
ambulance having had a generalised tonic-clonic seizure at home which started at 10:55. Buccal midazolam was
given by paramedics at 11:04 leading to cessation of tonic-clonic movements at 11:06. They applied O2 via face
mask and sited an intravenous cannula. Blood glucose was 5.2mmol/L. He remained unrousable for the remainder
of the journey. On arrival at the emergency department resus at 11:16 his airway is patent, there is good air entry,
breathing is regular with RR 12 and saturations 98% on 10L O2,
HR is 122, CRT is 2 seconds. He is completely
unresponsive to pain. During examination you observe onset of subtle but persistent jerking movements of the jaw
and left upper limb at 11:19. The pupils are bilaterally constricted and deviated to the right side.

At 11:20, what is the best next step in the management?

A Administer buccal midazolam now

B Administer IV lorazepam now

C Intubate and prepare IV phenytoin

D Observe for further 4 minutes while preparing IV lorazepam

E Observe for further 4 minutes while preparing IV phenytoin


Next Question
46715

End Session

Tag Question

Feedback

https://mypastest.pastest.com/Secure/TestMe/Browser/436619[‫ ص‬06:58:42 10/12/1437]


MyPastest

Difficulty: Average

Peer Responses

Session Progress

Responses Correct: 0

Responses Incorrect: 18

Responses Total: 18

Responses - % Correct: 0%

Blog
About Pastest
Contact Us
Help

© Pastest 2016

https://mypastest.pastest.com/Secure/TestMe/Browser/436619[‫ ص‬06:58:42 10/12/1437]


MyPastest

Previous Question
Prefer to use the old MyPastest? Access it here »

Back to Filters

Question 19 of 29

A GP phones for advice regarding a 12-year-old girl with a first suspected seizure. There are no previous medical
problems. The episode occurred at home yesterday. She was playing computer games at home with her older
brother who describes sudden onset of tonic clonic jerking affecting all four limbs with complete loss of
consciousness. There was no colour change. The episode self-resolved at 2 minutes and was followed by 30
minutes of post-ictal drowsiness. She is now back to her normal self and completely asymptomatic.

The GP is happy that the patient is clinically well and has given the relevant advice regarding risk avoidance and
what to do in the event of future episodes.

What is the best advice regarding paediatric follow up?

A Admission to hospital for further assessment and decision regarding follow up

B Referral to the paediatric outpatient clinic (4-6 weeks appointment)

C Referral to the paediatric outpatient clinicin event of further seizures

D Referral to the tertiary paediatric neurologist

E Urgent referral to see a paediatrician within 2 weeks


Next Question
46716

End Session
Tag Question

Feedback

https://mypastest.pastest.com/Secure/TestMe/Browser/436619[‫ ص‬06:59:06 10/12/1437]


MyPastest

Difficulty: Average

Peer Responses

Session Progress

Responses Correct: 0

Responses Incorrect: 19

Responses Total: 19

Responses - % Correct: 0%

Blog
About Pastest
Contact Us
Help

© Pastest 2016

https://mypastest.pastest.com/Secure/TestMe/Browser/436619[‫ ص‬06:59:06 10/12/1437]


MyPastest

Previous Question
Prefer to use the old MyPastest? Access it here »

Back to Filters

Question 20 of 29

In children and young people, which of the following is associated with the highest risk of sudden unexpected
death in epilepsy (SUDEP)?

A Idiopathic epilepsy

B Late onset epilepsy

C Poor antiepileptic compliance

D Supine position in sleep

E Symptomatic epilepsy

46717

Next Question Tag Question

Feedback

Difficulty: Average
End Session
Peer Responses

https://mypastest.pastest.com/Secure/TestMe/Browser/436619[‫ ص‬06:59:26 10/12/1437]


MyPastest

Session Progress

Responses Correct: 0

Responses Incorrect: 20

Responses Total: 20

Responses - % Correct: 0%

Blog
About Pastest
Contact Us
Help

© Pastest 2016

https://mypastest.pastest.com/Secure/TestMe/Browser/436619[‫ ص‬06:59:26 10/12/1437]


MyPastest

Prefer to use the old MyPastest? Access it here »

Back to Filters

Question 21 of 29

Stacey is a 15-year-old girl brought to the emergency department following two episodes of collapse and abnormal
movements.

The
first episode occurred at home yesterday. Stacey was in the kitchen speaking on the telephone with her father.
The episode was witnessed by the mother. She dropped the phone, became unresponsive and began up-rolling
both eyes, but initially remained standing. After around 30 seconds she gradually collapsed to the floor and began
arching her back and making erratic movements of the upper limbs bilaterally. The eyes were tightly closed. The
movements ended after 2-3 minutes and she began
responding to questions, but remained on the floor with her
eyes closed. She was brought to the emergency department and discharged following normal examination, routine
bloods and ECG.

The second episode occurred today at school. She was entering the classroom with her friends when she became
scared and complained of being unable to see
clearly. She was helped to a chair where she sat for 10 seconds
speaking incoherently before collapsing to the floor. Irregular jerking movements were observed in the right lower
limb and left upper limb. On this occasion there was no back arching. Abnormal movements continued for 10
minutes, after which she was able to sit unsupported but did not speak for 30 minutes. There was some bleeding
from the tongue which she bit during the episode.

Stacey suffers from recurrent abdominal pain and is under outpatient paediatric follow up for this. She is otherwise
well and lives with her mother and younger brother, who are also well. There is a history of epilepsy in the
maternal grandmother. Stacey is described as a high achiever who is expected to perform well in her upcoming
GCSE mock examinations.

What is the most likely diagnosis?

A Cataplexy

B Complex partial epilepsy

C Idiopathic generalised epilepsy

D Non-epileptic attack disorder

https://mypastest.pastest.com/Secure/TestMe/Browser/436619[‫ ص‬06:59:50 10/12/1437]


MyPastest

E Vasovagal syncope Question


Previous with secondary anoxic seizures

46718

Tag Question

Feedback

Difficulty: Average

Peer Responses

Session Progress

Responses Correct: 0

Responses Incorrect: 21

Responses Total: 21

Responses - % Correct: 0%

Next Question
Blog
About Pastest
Contact Us
Help

© Pastest 2016

End Session

https://mypastest.pastest.com/Secure/TestMe/Browser/436619[‫ ص‬06:59:50 10/12/1437]


MyPastest

Previous Question
Prefer to use the old MyPastest? Access it here »

Back to Filters

Question 22 of 29

A 5-year-old boy weighing 18kg is taking 180mg BD of sodium valproate for generalised tonic clonic seizures
which began 7 months ago. His parents do not feel there has been any change in seizure frequency (1-2/month)
since commencement of valproate as first line treatment 6 months ago. He is otherwise well without
developmental problems. There have been no prolonged seizures, no adverse drug effects, and no deterioration in
school performance, although he does miss some school time when seizures occur. The parents are keen to try a
different anti-epileptic drug (AED).

What is the best treatment plan for this child?

A Gradually wean off valproate after alternative AED has been introduced

B Gradually wean off valproate then start alternative AED

C Optimise dose of valproate before considering alternative AED

D Start second AED in combination with valproate

E Stop valproate and start alternative AED today

Explanation Next Question


46719

End Session
Tag Question

Feedback

https://mypastest.pastest.com/Secure/TestMe/Browser/436619[‫ ص‬07:00:10 10/12/1437]


MyPastest

Difficulty: Average

Peer Responses

Session Progress

Responses Correct: 0

Responses Incorrect: 22

Responses Total: 22

Responses - % Correct: 0%

Blog
About Pastest
Contact Us
Help

© Pastest 2016

https://mypastest.pastest.com/Secure/TestMe/Browser/436619[‫ ص‬07:00:10 10/12/1437]


MyPastest

Previous Question
Prefer to use the old MyPastest? Access it here »

Back to Filters

Question 23 of 29

A 15-year-old girl and her mother attend the paediatric outpatient clinic with recent onset of generalised tonic
clonic (GTC) seizures. There have been four episodes over two weeks. She is otherwise
well and developmentally
normal, although the school report that her attention in the classroom has deteriorated recently, and she often stops
in the middle of tasks and forgets what she is doing.

The EEG is non-diagnostic. You agree with the patient and mother to start treatment. The patient asks to speak to
you alone and reveals that she is sexually active with her boyfriend. She is taking the progestogen-only
contraceptive pill but sometimes forgets to take it.

What is the most appropriate first line anti-epileptic drug (AED) to commence for this patient?

A Carbamazepine

B Lamotrigine

C Oxcarbazepine

D Sodium valproate

E Topiramate
Next Question
46720

End Session
Tag Question

Feedback

https://mypastest.pastest.com/Secure/TestMe/Browser/436619[‫ ص‬07:00:31 10/12/1437]


MyPastest

Difficulty: Average

Peer Responses

Session Progress

Responses Correct: 0

Responses Incorrect: 23

Responses Total: 23

Responses - % Correct: 0%

Blog
About Pastest
Contact Us
Help

© Pastest 2016

https://mypastest.pastest.com/Secure/TestMe/Browser/436619[‫ ص‬07:00:31 10/12/1437]


MyPastest

Previous Question
Prefer to use the old MyPastest? Access it here »

Back to Filters

Question 24 of 29

A 13-year-old previously well girl presents to the rapid access outpatient clinic with a three week history of
clumsiness on waking in the mornings. Her parents had originally attributed this to “becoming a teenager”, but
recently they have also noticed sudden, involuntary and frequent jerking movements at the breakfast table. The
jerks occur in isolation with preserved awareness and look like “electric shocks”.

EEG demonstrates 3-6 Hz generalised polyspike and wave activity.

What is the most likely syndromic diagnosis?

A Benign Rolandic epilepsy of childhood

B Childhood absence epilepsy

C Dravet syndrome

D Juvenile myoclonic epilepsy

E Myoclonic-astatic epilepsy

46721
Next Question

Tag Question
End Session
Feedback

Difficulty: Average

https://mypastest.pastest.com/Secure/TestMe/Browser/436619[‫ ص‬07:00:52 10/12/1437]


MyPastest

Peer Responses

Session Progress

Responses Correct: 0

Responses Incorrect: 24

Responses Total: 24

Responses - % Correct: 0%

Blog
About Pastest
Contact Us
Help

© Pastest 2016

https://mypastest.pastest.com/Secure/TestMe/Browser/436619[‫ ص‬07:00:52 10/12/1437]


MyPastest

Prefer to use the old MyPastest? Access it here »

Back to Filters

Question 25 of 29

A 5-year-old boy presents to the emergency department following a second suspected seizure. The first episode
occurred 9 months previously. He was in bed asleep when he suddenly awoke, complained of feeling sick and
vomited. This was followed by 15 minutes of unresponsiveness with staring and head and eye deviation to the left
side. There was no tonic-clonic activity in the limbs. After the event the child gradually returned to normal and
was discharged from the emergency department with advice that the event may or may not have been
a seizure.
Since then he has been well until today and remained developmentally normal.

On this occasion the abnormal episode again began in sleep, during a daytime nap. He awoke, vomited and
became
unresponsive. Eyes were deviated to the left and the pupils were very large. The parents also noticed he
became pale and clammy. This continued until 20 minutes post onset when he started having jerking movements
of all 4 limbs, prompting paramedics to administer buccal midazolam, terminating the seizure. He has now woken
up and is feeling groggy but is speaking normally and almost back to his normal self.

What is the most likely syndromic diagnosis?

A Benign epilepsy with centrotemporal spikes

B Idiopathic generalised epilepsy

C Landau-Kleffner syndrome

D Lennox-Gastaut syndrome

E Panayiotopoulos syndrome

Explanation
A multi-axial approach to epilepsy classification
is recommended in which diagnosis is organised by seizure type,
syndrome and aetiology. Correct syndromic diagnosis is a key part of planning treatment and providing prognostic
information.

The seizure type in this case is focal, autonomic seizures, with secondary generalisation in the second episode. The

https://mypastest.pastest.com/Secure/TestMe/Browser/436619[‫ ص‬07:01:14 10/12/1437]


MyPastest

syndromic diagnosis of Panayiotopoulos syndrome (also known as early-onset benign partial epilepsy with
Previous
occipital paroxysms) is madeQuestion
on the basis of prominent autonomic symptoms, very low frequency of episodes,
sleep onset, prolonged duration and preserved neurodevelopment. It is one of the commonest childhood epilepsy
syndromes, comprising 10% or more of all epilepsy in childhood. Onset is usually at 4-7 years of age but may be
earlier. Autonomic features including vomiting are followed by tonic eye
deviation, impairment of consciousness
and sometimes evolution into secondary GTC. The EEG may be normal or may demonstrate bilateral occipital
spikes, especially in sleep. Prognosis is excellent; most children go into spontaneous remission after a few seizures
and treatment is often unnecessary.

A is incorrect; benign epilepsy with centrotemporal spikes (BECTS; also known as benign Rolandic epilepsy of
childhood) is characterised by focal seizures in sleep, but autonomic features and vomiting are not usually present.

B is incorrect; the episodes in this case are focal rather than generalised (although secondary generalisation is
described in the second episode).

C
is incorrect; Landau–Kleffner syndrome is characterised by EEG abnormalities which are seen in sleep, but the
clinical presentation is one of regression in language skills.

D is incorrect; Lennox–Gastaut syndrome is characterised by cognitive impairment and multiple seizure types
including atonic, tonic, tonic–clonic and atypical absence seizures.

References:

https://www.nice.org.uk/guidance/cg137

https://www.epilepsy.org.uk/info/syndromes/
46722

Tag Question

Feedback

Difficulty: Average

Peer Responses

Next Question

End Session

Session Progress

https://mypastest.pastest.com/Secure/TestMe/Browser/436619[‫ ص‬07:01:14 10/12/1437]


MyPastest

Responses Correct: 0

Responses Incorrect: 25

Responses Total: 25

Responses - % Correct: 0%

Blog
About Pastest
Contact Us
Help

© Pastest 2016

https://mypastest.pastest.com/Secure/TestMe/Browser/436619[‫ ص‬07:01:14 10/12/1437]


MyPastest

Previous Question
Prefer to use the old MyPastest? Access it here »

Back to Filters

Question 26 of 29

A 3-month-old girl is referred urgently from the GP with a history of increasing head size since birth. The OFC,
initially on the 25th centile, is now above the 91st. The mother speaks little English but on taking further history it
becomes apparent the infant is generally unwell with poor feeding and recurrent nosebleeds. On examination the
child is irritable and unable to fix and follow. There is significant head lag. On closer examination of the head
you
notice bilateral proptosis and a cranial bruit.

What is the most likely diagnosis?

A Aqueductal stenosis

B Congenital leukaemia

C Neurofibromatosis type 1

D Sotos syndrome

E Vein of Galen malformation

47171

Next Question

Tag Question
End Session
Feedback

Difficulty: Average

Peer Responses

https://mypastest.pastest.com/Secure/TestMe/Browser/436619[‫ ص‬07:01:35 10/12/1437]


MyPastest

Session Progress

Responses Correct: 0

Responses Incorrect: 26

Responses Total: 26

Responses - % Correct: 0%

Blog
About Pastest
Contact Us
Help

© Pastest 2016

https://mypastest.pastest.com/Secure/TestMe/Browser/436619[‫ ص‬07:01:35 10/12/1437]


MyPastest

Previous Question
Prefer to use the old MyPastest? Access it here »

Back to Filters

Question 27 of 29

An 8-year-old girl is referred to the outpatient clinic with learning difficulties, seizures and microcephaly. The
family are refugees who recently moved to the UK; an underlying diagnosis was never
identified in the country of
origin.

On examination she is an active and mobile child who is happy to be examined and laughs frequently during the
consultation. Cognitive impairment is quite significant, however her receptive language is relatively preserved and
she has good non-verbal communication skills. She is constantly on the go with a jerky gait. She holds her arms
high and flexed at the elbow, and frequently flaps her hands. Her OFC is 47cm (0.4th-2nd centile).

What is the most likely genetic cause of this child’s underlying diagnosis?

A DHCR7 mutation

B MECP2 mutation

C Microdeletion of maternal 15q11.2-q13

D UBE3A mutation

E Uniparental disomy of chromosome 14


Next Question
47172

End Session
Tag Question

Feedback

https://mypastest.pastest.com/Secure/TestMe/Browser/436619[‫ ص‬07:01:56 10/12/1437]


MyPastest

Difficulty: Average

Peer Responses

Session Progress

Responses Correct: 0

Responses Incorrect: 27

Responses Total: 27

Responses - % Correct: 0%

Blog
About Pastest
Contact Us
Help

© Pastest 2016

https://mypastest.pastest.com/Secure/TestMe/Browser/436619[‫ ص‬07:01:56 10/12/1437]


MyPastest

Previous Question
Prefer to use the old MyPastest? Access it here »

Back to Filters

Question 28 of 29

A 4-year-old child presents to the emergency department with headache, vomiting and macrocephaly.
Neuroimaging reveals obstructive hydrocephalus secondary to sub-ependymal giant cell astrocytoma (SEGA).
Evaluation with Wood’s lamp reveals multiple hypopigmented macules
and diagnosis is made of tuberous
sclerosis complex (TSC).

Which is the most accurate statement regarding TSC in children?

A ADHD is the commonest neuropsychiatric manifestation

B CT is superior to MRI for identification of cortical hamartomas

C Mutations in TSC1 are the commonest cause

D Neuroimaging is recommended every 1-3 years in asymptomatic patients

E SEGAs are the commonest CNS lesion

47495

Next Question

Tag Question

Feedback End Session

Difficulty: Average

Peer Responses

https://mypastest.pastest.com/Secure/TestMe/Browser/436619[‫ ص‬07:02:17 10/12/1437]


MyPastest

Session Progress

Responses Correct: 0

Responses Incorrect: 28

Responses Total: 28

Responses - % Correct: 0%

Blog
About Pastest
Contact Us
Help

© Pastest 2016

https://mypastest.pastest.com/Secure/TestMe/Browser/436619[‫ ص‬07:02:17 10/12/1437]


MyPastest

Prefer to use the old MyPastest? Access it here »

Back to Filters

Question 29 of 29

An 11-month-old girl presents to the emergency department with a one-week history of abnormal movements
occurring several times daily. She is observed to cry out then flex the trunk and limbs. The left side flexes more
than the right and the head and eyes deviate to the right side. She remains stiff for several seconds then relaxes.
The movement is repeated several times over a minute so.

There is no family history of epilepsy. The father has a history of renal cysts but is otherwise well. The parents are
not consanguineous.

On examination the child is quite irritable but the neurological examination is normal. She sits with support and
rolls over but cannot crawl. There is no evident dysmorphism or cutaneous abnormalities.

When
shaking the father’s hand at the end of the consultation you notice flesh-coloured lesions distorting the nail
bed of several fingers.

What is the most likely underlying cause for this child’s seizure disorder?

A Neurofibromatosis type I

B Neurofibromatosis type II

C Neurosarcoidosis

D Tuberous sclerosis complex

E West syndrome

47496

End Session

Tag Question

https://mypastest.pastest.com/Secure/TestMe/Browser/436619[‫ ص‬07:02:38 10/12/1437]


MyPastest

Previous Question
Feedback

Difficulty: Average

Peer Responses

Session Progress

Responses Correct: 0

Responses Incorrect: 29

Responses Total: 29

Responses - % Correct: 0%

Blog
About Pastest
Contact Us
Help

© Pastest 2016

https://mypastest.pastest.com/Secure/TestMe/Browser/436619[‫ ص‬07:02:38 10/12/1437]


MyPastest

Prefer to use the old MyPastest? Access it here »

Back to Filters

Question 1 of 31

A 6-month-old boy was seen by his GP and referred for admission to hospital for investigation of pallor. He was
otherwise well
and examination was normal. His mother said that his appetite was good.

His blood investigations showed:

CBC : Hb: 9.0 g/dl (11 – 16 g/dl)

MCV: 66fl (76 – 96 fl)

WBC : 5x109/L (4 – 11x109/L)

Ferritin : 9ug/L (12 – 200ug/L)

Serum iron : 7umol/l (14 – 31umol/l)

What is the most likely diagnosis?

A Iron deficiency anaemia

B Leukaemia

C Thalassaemia minor

D Vitamin B12 deficiency

E Sickle cell disease

Explanation
The blood investigations indicate a microcytic hypochromic anaemia. The white blood cell count is within the
normal range and does not show a pancytopaenic picture,making leukaemia
unlikely. The decreased levels of
serum iron and ferritin point to the iron deficiency. In thalassaemia minor and sickle cell disease the ferritin and
iron levels are usually normal. Vitamin B 12 deficiency is a
cause of megaloblastic anaemia (increased MCV). At
this age, iron deficiency is due to inadequate iron in the diet, e.g. in delayed weaning.T

https://mypastest.pastest.com/Secure/TestMe/Browser/436619[‫ ص‬06:38:07 10/12/1437]


MyPastest

11688

Next Question
Tag Question

Feedback
Previous Question
End Session
Difficulty: Easy

Peer Responses

Session Progress

Responses Correct: 0

Responses Incorrect: 1

Responses Total: 1

Responses - % Correct: 0%

Blog
About Pastest
Contact Us
Help

© Pastest 2016

https://mypastest.pastest.com/Secure/TestMe/Browser/436619[‫ ص‬06:38:07 10/12/1437]


MyPastest

Prefer to use the old MyPastest? Access it here »

Back to Filters

Question 2 of 31

A 3-month-old baby boy is brought to the Paediatric emergency due to vomiting, pallor and weight loss. On
examination, he was afebrile, tachypnoeic and restless. He produced no sound on crying. His mother admitted to
boiling his milk regularly. A chest X- ray taken was normal.

The baby is likely to have:

A Thiamine deficiency

B Gastroenteritis

C Bronchiolitis

D Aspiration syndrome

E Pyloric stenosis

Explanation
Thiamine (vitamin B1) deficiency, results in Beri-beri. Clinical features include lethargy, restlessness, vomiting,
tachypnoea, cyanosis and a characteristic aphonic cry. Peripheral neuritis, paraesthesia of the toes and feet, calf
tenderness and decreased reflexes may occur at a later stage. Vitamin B1 is neutralised by boiling. Thiamine
occurs in unrefined cereals, fresh foods, wholegrain bread, fresh meat, legumes, green vegetables, fruit and milk.
If a baby is mainly fed breast milk from a mother who has thiamine deficiency, then their child may develop it
also.

Gastroenteritis
and pyloric stenosis would explain the vomiting, pallor and weight loss, but not the aphonic cry.
Pyloric stenosis occurs due to hypertrophy of the pylorus of the stomach. It presents around 3-6 weeks of age with
recurrent non-bilous vomiting leading to a hypochloraemic hypokalaemic metabolic alkalosis. The baby becomes
hungry soon after vomiting.

Bronchiolitis presents in infants up to 2 years of age with a peak between 3-6 months. Symptoms are productive
cough, fever, tachypnoea and decreased feeding. Chest X-ray shows hyperinflation, and crackles +/- wheezing can

https://mypastest.pastest.com/Secure/TestMe/Browser/436619[‫ ص‬06:38:58 10/12/1437]


MyPastest

be heard on auscultation of the chest. There is nothing in the history to suggest aspiration.
11689

Next Question

Tag Question

Previous Question
Feedback
End Session
Difficulty: Average

Peer Responses

Session Progress

Responses Correct: 0

Responses Incorrect: 2

Responses Total: 2

Responses - % Correct: 0%

Blog
About Pastest
Contact Us
Help

© Pastest 2016

https://mypastest.pastest.com/Secure/TestMe/Browser/436619[‫ ص‬06:38:58 10/12/1437]


MyPastest

Prefer to use the old MyPastest? Access it here »

Back to Filters

Question 3 of 31

A 6-year-old boy was seen at the Paediatric emergency department following trauma to his right arm. An X-ray
taken showed a greenstick fracture of his radius and decreased mineralisation of his bones. He was on regular
Phenytoin due to grand mal seizures.

His blood investigations showed:

Calcium: 1.6 mmol/l (2.12 – 2.65 mmol/l)

Phosphate : 0.6 mmol/l (0.8 – 1.45 mmol/l)

Alkaline phosphate : 450iu/L (30 – 300iu/L)

PTH : 11.0pmol/L (0.8 – 8.5 pmol/L)

The most likely diagnosis is:

A Phenytoin toxicity

B Osteogenesis imperfecta

C Hypophosphatasia

D Vitamin D dependent rickets

E Familial hypophosphataemic rickets

Explanation
Calcium and phosphate levels are regulated namely by PTH (parathyroid hormone) and Vitamin D. Vitamin D,
obtained from the diet and from sun exposure is converted into 25 hydroxyvitamin D in the liver. This is converted
by 1a hydroxylase into the metabolically active form 1,25 dihydroxyvitmain D in the kidney. As a result there is:

Increased calcium and phosphate absorption from the GIT

https://mypastest.pastest.com/Secure/TestMe/Browser/436619[‫ ص‬06:39:22 10/12/1437]


MyPastest

Increased calcium absorption from the distal tubule in the kidney


Increased phosphate absorption from the proximal tubule in the kidney

Next Question
In vitamin D deficiency (nutritional or, secondary to drug effects):

Decreased Vitamin D
Decreased calcium
DecreasedPrevious
phosphate Question
Increased PTH End Session

In hypophosphataemic rickets, there is abnormal reabsorption of phosphate by the kidney, leading to:

Decreased phosphate
Normal calcium
Normal PTH
High urinary phosphate

Vitamin D dependent rickets is an autosomal recessive (AR) disorder leading to deficiency of 1a hydroxylase in
the kidneys. Therefore there is:

Decreased vitamin D
Decreased calcium
Decreased phosphate
Increased PTH
Decreased 1,25 (OH)D

Hypophosphatasia
is an AR recessive disorder characterised by decreased alkaline phosphate activity. Clinical
features resemble those of rickets with bowing and shortening of the legs and fractures. It may present in a lethal
neonatal form, a severe infantile form or a milder form in childhood or adolescence. It can be excluded here as
alkaline phosphate levels are increased.
11690

Tag Question

Feedback

Difficulty: Average

Peer Responses

https://mypastest.pastest.com/Secure/TestMe/Browser/436619[‫ ص‬06:39:22 10/12/1437]


MyPastest

Session Progress

Responses Correct: 0

Responses Incorrect: 3

Responses Total: 3

Responses - % Correct: 0%

Blog
About Pastest
Contact Us
Help

© Pastest 2016

https://mypastest.pastest.com/Secure/TestMe/Browser/436619[‫ ص‬06:39:22 10/12/1437]


MyPastest

Prefer to use the old MyPastest? Access it here »

Next Question
Back to Filters

Question 4 of 31

A previously healthy 4-year-old girl was brought to hospital due to the sudden onset of recurrent vomiting,
headache and photophobia. She had been left with her grandmother in the afternoon. On examination, the child
appeared lethargic, was afebrile and had a purple tongue. There was no neck stiffness.

The most likely diagnosis is:

A Vitamin A toxicity

B Meningitis

C Brain tumour

D Vitamin D toxicity

E Head injury

Explanation
Acute Vitamin A toxicity presents with signs of raised intracranial pressure including vomiting, nausea,
drowsiness, papilloedema and diplopia. The child may also complain of headaches, dizziness, blurred vision and
poor muscle coordination. The fact that the child was left alone with her grandmother and the purple staining to
the tongue may indicate that the child could have ingested something. A brain tumour would not present with such
an acute picture. Head injury with a subsequent intracranial bleed is possible with this scenario. However, it would
not explain the purple tongue. A child with meningitis presents with vomiting, headache and photophobia. At the
age of 4 years neck stiffness should easily be elicited if the child has meningitis. The lack of pyrexia also make
meningitis less likely (although does not necessarily exclude it). A petechial rash may also be
present. Vitamin D
toxicity presents with vomiting and headache, polyuria, muscle weakness and hypercalcaemia.
11691

https://mypastest.pastest.com/Secure/TestMe/Browser/436619[‫ ص‬06:39:56 10/12/1437]


MyPastest

Tag Question

Feedback

Difficulty: Average

Previous Question Peer Responses

End Session

Session Progress

Responses Correct: 0

Responses Incorrect: 4

Responses Total: 4

Responses - % Correct: 0%

Blog
About Pastest
Contact Us
Help

© Pastest 2016

https://mypastest.pastest.com/Secure/TestMe/Browser/436619[‫ ص‬06:39:56 10/12/1437]


MyPastest

Prefer to use the old MyPastest? Access it here »

Back to Filters

Question 5 of 31

Kwash iorkor (PEM):

A Is due to a poor calorie intake only

B Body weight is less than 60% of normal

C Children affected have generalised oedema

D There is atrophy of muscle and adipose tissue

E Its effects are always reversed after nutritional rehabilitation

Explanation
Kwashiorkor, protein energy malnutrition (PEM), is due to protein deficiency, whilst calorie intake is adequate.
Weight for age is 60-80% of expected. Clinical manifestations include

Preserved subcutaneous tissue


Muscular atrophy
Localised/generalised oedema
Hepatomegaly due to fatty infiltration of the liver
Sparse, easily pluckable hair
Hyperpigmented hyperkeratosis and macular rashes
Atrophied tongue and lymphatics

Marasmus,
on the other hand, is due to severe calorie depletion and body weight is less than 60%. There is excess
loss of subcutaneous tissue and muscle.

If growth and development have been extensively impaired, physical and mental retardation may be permanent in

https://mypastest.pastest.com/Secure/TestMe/Browser/436619[‫ ص‬06:40:16 10/12/1437]


MyPastest

both cases.
11693

Next Question

Tag Question

Previous Question
Feedback
End Session
Difficulty: Average

Peer Responses

Session Progress

Responses Correct: 0

Responses Incorrect: 5

Responses Total: 5

Responses - % Correct: 0%

Blog
About Pastest
Contact Us
Help

© Pastest 2016

https://mypastest.pastest.com/Secure/TestMe/Browser/436619[‫ ص‬06:40:16 10/12/1437]


MyPastest

Prefer to use the old MyPastest? Access it here »

Back to Filters

Question 6 of 31

A Tetracycline

B Isoniazid

C Warfarin

D Sodium valproate

E Carbimazole

F Phenytoin

G Amiodarone

H Salbutamol via inhaler

I Haloperidol

J Chloramphenicol

Which of the above medications, if ingested via breast milk, will produce the following adverse effects?
Each option can be used once, more than once or not at all.

Scenario 1

A 1-year-old baby boy with discoloration of the teeth.

Your answer was incorrect

Select one...

A - Tetracycline

Tetracycline causes yellowish discoloration of the teeth and should be avoided while breast feeding. It is contra-
indicated in children under 12 years also, since it causes this staining and occasionally dental hypoplasia.

Scenario 2

A 4-day-old girl admitted for investigation following a convulsion.

Your answer was incorrect

https://mypastest.pastest.com/Secure/TestMe/Browser/436619[‫ ص‬06:40:38 10/12/1437]


MyPastest

Select one...

B - Isoniazid
Next Question
Isoniazid, an antituberculous drug, is associated with an increased risk of convulsions and peripheral
neuropathies
in the newborn. Prophylactic Pyridoxine is advisable in the mother and infant.

Scenario 3
Previous Question
End Session
A 6-week-old baby boy is brought to the emergency. He is hypotensive, tachypnoeic and cyanosed. His abdomen
is distended.

Your answer was incorrect

Select one...

J - Chloramphenicol

Chloramphenicol may cause bone marrow toxicity in an infant. Grey baby syndrome, i.e. abdominal distension,
cyanosis and circulatory collapse is rare but can occur.
Carbimazole
appears in breast milk, but breastfeeding is not contraindicated as long as low doses are used and
neonatal development is monitored.

Warfarin, Sodium valproate, Salbutamol, and Haloperidol can be used safely by breastfeeding mothers.

Explanation
Tetracycline causes yellowish discoloration of the teeth. Isoniazid, an antituberculous drug, is
associated with an increased risk of convulsions and neuropathies in the newborn. Chloramphenicol
may cause bone marrow toxicity in an infant. Grey baby syndrome, i.e. abdominal distension,
cyanosis and circulatory collapse is rare but can occur. Carbimazole appears in breast milk, but
breastfeeding is not contraindicated as long as low doses are used and neonatal development is
monitored. Warfarin, Sodium valproate, Salbutamol, and Haloperidol can be used safely by
breastfeeding mothers.
11694

Tag Question

Feedback

Difficulty: Average

https://mypastest.pastest.com/Secure/TestMe/Browser/436619[‫ ص‬06:40:38 10/12/1437]


MyPastest

Session Progress

Responses Correct: 0

Responses Incorrect: 8

Responses Total: 8

Responses - % Correct: 0%

Blog
About Pastest
Contact Us
Help

© Pastest 2016

https://mypastest.pastest.com/Secure/TestMe/Browser/436619[‫ ص‬06:40:38 10/12/1437]


MyPastest

Prefer to use the old MyPastest? Access it here »

Back to Filters

Question 7 of 31

A Vitamin B12
B Vitamin K
C Zinc
D Vitamin B1
E Folate
F Vitamin A
G Vitamin B6
H Biotin
I Copper
J Vitamin B3

Which of the above deficiencies explains each of the following clinical scenarios?
Each option may be used once, more than once or not at all.

Scenario 1

A 3-day-old baby boy was brought to hospital as his mother noted that blood was coming out of his umbilicus.

Your answer was incorrect

Select one...

B - Vitamin K

Vitamin K deficiency is rare in neonates but causes haemorrhagic disease of the newborn. Bleeding can occur
from the umbilicus and cause intracranial bleeds. This can be prevented by the administration of IM vitamin K just
after birth.

Scenario 2

A 6-month-old girl admitted with chronic diarrhoea and recurrent infections. On examination, she had a perioral
rash and a white tongue.

Your answer was incorrect

Select one...

https://mypastest.pastest.com/Secure/TestMe/Browser/436619[‫ ص‬06:41:00 10/12/1437]


MyPastest

C - Zinc

Next Question
This describes acrodermatitis enteropathica (Zinc deficiency). Presentation includes a history of :

chronic diarrhoea,
perianal and perioral rashes or lesions,
reddish hair and alopecia
Previous
ocular Question
changes e.g. blepharitis, conjunctivitis, photophobia and corneal dystrophy

The white tongue is due to associated Candida infections. End Session


Scenario 3

A 10-year-old girl known to suffer from Crohn’s disease presented with pallor and peripheral neuropathy.

Your answer was incorrect

Select one...

A - Vitamin B12

Vitamin B12 is usually absorbed in the terminal ileum, which in Crohn’s disease patients can be inflammed and
ulcerated. Vitamin B12 deficiency will cause a macrocytic anaemia.

Explanation
 
11695

Tag Question

Feedback

Difficulty: Average

Session Progress

Responses Correct: 0

Responses Incorrect: 11

Responses Total: 11

Responses - % Correct: 0%

https://mypastest.pastest.com/Secure/TestMe/Browser/436619[‫ ص‬06:41:00 10/12/1437]


MyPastest

Blog
About Pastest
Contact Us
Help

© Pastest 2016

https://mypastest.pastest.com/Secure/TestMe/Browser/436619[‫ ص‬06:41:00 10/12/1437]


MyPastest

Prefer to use the old MyPastest? Access it here »

Back to Filters

Question 8 of 31

You see a mum who has brought in her 6-month-old son who has been fed exclusively on breastmilk. She is
worried because he looks pale and her attempt to wean him was unsuccessful.

Blood results were Hb 6.6, Wcc 7.0, and low serum iron.

Blood film shows hypochromic, microcytic anaemia.

The diagnosis is?

A Sickle cell anaemia

B Beta thalasaemia

C Haemolytic anaemia

D Iron deficiency anaemia

E Sideroblastic anaemia

Explanation
The abundant iron stores in the newborn infant mean that even though breast-milk and cow’s milk do not make a
significant contribution to dietary iron, depletion of iron stores does not occur before around 6 months of age in a
breast-fed infant.

From the age of about 6 months, iron must be added to the diet of full-term infants either in the form of iron-
supplements, or in food naturally containing iron. In iron deficiency anaemia, the blood film is hypochromic,
showing microcytic anaemia with low serum iron, low
transferrin saturation. Plasma ferritin is also low.

Sickle cell anaemia occurs in patients with sickle cell disease, which is due to substitution of valine for glutamic
acid at position 6 on the beta globin chain. When the Haemoglobulin S molecules are deoxygenated or acidotic,
the molecules polymerise. This causes the red cell to become sickle shaped resulting in reduced life span of the red
cell leading to haemolysis. This occurs at around 6 months of age. The iron level is usually normal in patients with

https://mypastest.pastest.com/Secure/TestMe/Browser/436619[‫ ص‬06:41:20 10/12/1437]


MyPastest

sickle cell disease.

Beta thalassaemia - serum iron is normal or increased, ferritin normal.

Next Question
Sideroblastic anaemia - serum iron and ferritin are increased.
11709

Previous Question
End Session
Tag Question

Feedback

Difficulty: Easy

Peer Responses

Session Progress

Responses Correct: 0

Responses Incorrect: 12

Responses Total: 12

Responses - % Correct: 0%

Blog
About Pastest
Contact Us
Help

© Pastest 2016

https://mypastest.pastest.com/Secure/TestMe/Browser/436619[‫ ص‬06:41:20 10/12/1437]


MyPastest

Prefer to use the old MyPastest? Access it here »

Next Question
Back to Filters

Previous Question
Question 9 of 31

Milk constituents:

A Carbohydrate
main component is sucrose that is easily digestible and fermented by the lactobacilli in
the gut to produce acid stools

B Main protein in breast-milk is whey, which is more digestible than cow’s milk protein casein.

C Breast-milk has high level of vitamin K

D Breast-milk contains low level of vitamin A, C, and, E compared to cow’s milk

E The average calorific requirements for a one-month infant are 200kcal/kg/day

Explanation
The main carbohydrate component in all the types of milk is lactose. Whey is the main protein in breast-milk but
also other important components are the immunoglobulins - IgA, IgM, Lysozyme,
Lactoferrin and Lactoalbumin.

Vitamin K level is lower in breast-milk hence haemorrhagic disease of the newborn is more likely in breast-fed
infants but there are higher levels of Vitamin A, C, and E than in cow’s milk.

The average calorific requirements for a


one-month infant are 100kcal/kg/day. Standard full fat cow’s milk
contains about 67kcal/100ml.
11710

Tag Question

Feedback

https://mypastest.pastest.com/Secure/TestMe/Browser/436619[‫ ص‬06:41:41 10/12/1437]


MyPastest

Difficulty: Average

Peer Responses

End Session

Session Progress

Responses Correct: 0

Responses Incorrect: 13

Responses Total: 13

Responses - % Correct: 0%

Blog
About Pastest
Contact Us
Help

© Pastest 2016

https://mypastest.pastest.com/Secure/TestMe/Browser/436619[‫ ص‬06:41:41 10/12/1437]


MyPastest

Prefer to use the old MyPastest? Access it here »

Back to Filters

Question 10 of 31

Theme: Infant nutrition

A 2 months
B 6 months
C 7 months
D 9 months
E 1 year
F 18 months
G 2 years
H 3 years
I 4 years
J 5 years

For
each of the following foodstuffs, choose the most appropriate age for their introduction (in a normal infant)
from the list above. Each item may be used once, more than once or not at all.

Scenario 1

Honey

Your answer was incorrect

Select one...

E - 1 year

Honey must not be given to children under the age of one year due to the risk of botulism.

Scenario 2

Baby rice

Your answer was incorrect

Select one...

https://mypastest.pastest.com/Secure/TestMe/Browser/436619[‫ ص‬06:42:04 10/12/1437]


MyPastest

B - 6 months

Baby rice mixed with milk is a good food to start weaning. Weaning should start at around 6 months (the WHO
recommends exclusive breast-feeding until 6 months of age).
Next Question
Other foods suitable to begin with are mashed cooked vegetables and fruit.

Scenario 3

Puréed meat and vegetables


Previous Question
Your answer was incorrect End Session

Select one...

C - 7 months

Once an infant is used to eating baby rice, mashed vegetables and fruit (at 6 months) then other foods can be
added, such as: puréed or mashed-up meat, fish and chicken; lentils (dhal) or pulses; full-fat dairy products (low
sugar varieties).
22410

Tag Question

Feedback

Difficulty: Average

Session Progress

Responses Correct: 0

Responses Incorrect: 16

Responses Total: 16

Responses - % Correct: 0%

Blog
About Pastest
Contact Us
Help

© Pastest 2016

https://mypastest.pastest.com/Secure/TestMe/Browser/436619[‫ ص‬06:42:04 10/12/1437]


MyPastest

Prefer to use the old MyPastest? Access it here »

Next Question
Back to Filters

Previous Question
Question 11 of 31
End Session

What is the ideal growth rate of a newborn baby when receiving appropriate nutritional input?

A 2g/kg/day

B 15g/kg/day

C 25g/kg/day

D 30g/kg/day

E 40g/kg/day

Explanation
To grow and develop normally babies must have the
energy and correct balance of carbohydrate, fat, proteins,
vitamins and
minerals.

The aim of nutritional support is to achieve a growth rate of approximately 15g/kg/day whilst maintaining a
normal body composition. Providing this nutrition enterally is the best method and is much easier and safer than
other routes.
46199

Tag Question

Feedback

Difficulty: Average

https://mypastest.pastest.com/Secure/TestMe/Browser/436619[‫ ص‬06:42:23 10/12/1437]


MyPastest

Peer Responses

Session Progress

Responses Correct: 0

Responses Incorrect: 17

Responses Total: 17

Responses - % Correct: 0%

Blog
About Pastest
Contact Us
Help

© Pastest 2016

https://mypastest.pastest.com/Secure/TestMe/Browser/436619[‫ ص‬06:42:23 10/12/1437]


MyPastest

Prefer to use the old MyPastest? Access it here »

Next Question
Back to Filters

Question 12 of 31

How much energy does a well preterm baby in a thermo-neutral environment require to maintain essential body
functions?

A 25kcal/kg/day

B 50kcal/kg/day

C 80kcal/kg/day

D 100kcal/kg/day

E 120kcal/kg/day

Explanation
A well pre-term baby in a thermo-neutral environment needs 50/kcal/kg per day just to maintain essential body
functions. If he is to grow, he needs an additional 5 kcal for every gram of weight gain. Sick babies need more
calories. Some babies grow well on 90-120kcal/kg/day, particularly if baby can absorb fat well but many need
140-160kcal/kg/day.

The protein: calorie ratio within the nutrition supplied is also very important. Enough calories need to be supplied
to ensure baby uses all the dietary protein for cell-building and he does not need to break down protein for energy.

References: Rennie JM, Robertson NRC, A Manual of Neonatal Intensive Care 4th Edition, Infant Nutritional
Requirements. Arnold Publishers ISBN0-340-72010-7.
46200

Tag Question

https://mypastest.pastest.com/Secure/TestMe/Browser/436619[‫ ص‬06:42:42 10/12/1437]


MyPastest

Feedback

Difficulty: Average

Peer Responses
Previous Question
End Session

Session Progress

Responses Correct: 0

Responses Incorrect: 18

Responses Total: 18

Responses - % Correct: 0%

Blog
About Pastest
Contact Us
Help

© Pastest 2016

https://mypastest.pastest.com/Secure/TestMe/Browser/436619[‫ ص‬06:42:42 10/12/1437]


MyPastest

Prefer to use the old MyPastest? Access it here »

Next Question
Back to Filters

Question 13 of 31

How many calories are found in mature human breast milk?

A 25kcal/100ml

B 40kcal/100ml

C 60kcal/100ml

D 70kcal/100ml

E 100kcal/100ml

Explanation
The correct answer is 70kcal/100ml. Breast milk is the most ideal nutritional source for babies. At Term, mothers
own milk (Day 7 onwards) contains approximately 73kcal/100ml. Breastmilk produced for a preterm baby (Day
7+) contains 74kcal/100ml and drip breast milk (often collected for storage whilst feeding from the alternate
breast) is more dilute and contains only 54kcal/100ml.

Preterm baby specialist formula milk contains approximately 80kcal/100ml. Cow’s milk contains approximately
67kcal/100ml.

Fat is the main energy source for babies and should constitute at least
30% of their total calorie intake per day (5-
9g/kg/day). For the newborn baby, the move from mainly glucose metabolism in utero to fat metabolism based
energy supply marks a major physiological adjustment.

References: Rennie JM, Robertson NRC, A Manual of Neonatal Intensive Care 4th Edition, Infant Nutritional
Requirements. Arnold Publishers ISBN0-340-72010-7.
46201

https://mypastest.pastest.com/Secure/TestMe/Browser/436619[‫ ص‬06:43:02 10/12/1437]


MyPastest

Tag Question

Feedback

Difficulty: Average

Previous Question Peer Responses

End Session

Session Progress

Responses Correct: 0

Responses Incorrect: 19

Responses Total: 19

Responses - % Correct: 0%

Blog
About Pastest
Contact Us
Help

© Pastest 2016

https://mypastest.pastest.com/Secure/TestMe/Browser/436619[‫ ص‬06:43:02 10/12/1437]


MyPastest

Prefer to use the old MyPastest? Access it here »

Back to Filters

Question 14 of 31

Which daily calorie requirement range is matched correctly with its paediatric age group?

A 90-120kcal/kg/day for children aged 0 – 1 year

B 75-90kcal/kg/day for children aged 1 - 3 years

C 60-75kcal/kg/day for children aged 5 – 10 years

D 30-60kcal/kg/day for children aged 11 - 18 years

E 20-30kcal/kg/day for children aged 15 – 18 years

Explanation
The correct answer is: A 90-120kcal/kg/day for children aged 0 – 1 year.

Recommended daily calorie requirements for children and adults are as follows:

Age 0 – 1 year require 90-120kcal/kg/day.


Age 1 – 7 years require 75-90kcal/kg/day.
Age 7 – 12 years require 60-75kcal/kg/day.
Age 12 – 18 years require 30-60kcal/kg/day.
Adults over 18 years require between 20 – 30kcal/kg/day.

The above figures are for healthy children maintaining normal range weights with average levels of energy
expenditure. Sick children or those competing in high level sports for example may well require specialist diets
and significant additional calorie intake.

References: Rennie JM, Robertson NRC, A Manual of Neonatal Intensive Care 4th Edition, Infant Nutritional
Requirements. Arnold Publishers ISBN0-340-72010-7.
46202

https://mypastest.pastest.com/Secure/TestMe/Browser/436619[‫ ص‬06:43:22 10/12/1437]


MyPastest

Next Question Tag Question

Feedback

Previous Question
Difficulty: Average
End Session
Peer Responses

Session Progress

Responses Correct: 0

Responses Incorrect: 20

Responses Total: 20

Responses - % Correct: 0%

Blog
About Pastest
Contact Us
Help

© Pastest 2016

https://mypastest.pastest.com/Secure/TestMe/Browser/436619[‫ ص‬06:43:22 10/12/1437]


MyPastest

Prefer to use the old MyPastest? Access it here »

Next Question
Back to Filters

Previous Question
Question 15 of 31
End Session

How many calories are found in 50ml of 5% dextrose saline solution?

A 0.5kcal in 50mls

B 2.5kcal in 50mls

C 5kcal in 50mls

D 7kcal in 50mls

E 8.5kcal in 50mls

Explanation
The correct answer is 8.5kcal/50mls. A 5% glucose solution contains 170kcal/L, 0.17kcal per ml.

Normal saline and water (with which dextrose may be diluted) contain zero calories.

References: Rennie JM, Robertson NRC, A Manual of Neonatal Intensive Care 4th Edition, Infant Nutritional
Requirements. Arnold Publishers ISBN0-340-72010-7.
46203

Tag Question

Feedback

Difficulty: Average

https://mypastest.pastest.com/Secure/TestMe/Browser/436619[‫ ص‬06:43:44 10/12/1437]


MyPastest

Peer Responses

Session Progress

Responses Correct: 0

Responses Incorrect: 21

Responses Total: 21

Responses - % Correct: 0%

Blog
About Pastest
Contact Us
Help

© Pastest 2016

https://mypastest.pastest.com/Secure/TestMe/Browser/436619[‫ ص‬06:43:44 10/12/1437]


MyPastest

Prefer to use the old MyPastest? Access it here »

Back to Filters

Question 16 of 31

A 4 month old baby is brought to A & E by his mother with


vomiting, lethargy and abnormal movements. His Red
Book indicates failure to thrive. He has no abnormal temperature measurements and is not dysmorphic. During the
history you note his parents follow a strict
diet without meat, dairy, eggs or other animal products.

What could be the cause of this presentation?

A Calcium Deficiency

B Iron deficiency

C Vitamin A deficiency

D Vitamin B12 deficiency

E Vitamin D deficiency

Explanation
The infant of a mother following a vegan diet is at most risk of being Vitamin B12 deficient. Vitamin B12 is
primarily available from animal products and without adequate intake or supplementation the mother will become
deficient during pregnancy and in
the provision of B12 via breastmilk. Vitamin B12 deficiency usually presents
between 2 – 12 months of age with vomiting, lethargy, failure to thrive, hypotonia, developmental arrest or
regression and abnormal movements. The symptoms are serious and although treatable with vitamin
supplementation there may be long term sequelae. In the vegan diet vitamin B12 sources may include fermented
soybean and yeast products.

This baby may also be at risk of deficiencies ofiron, vitamin A and vitamin D in particular.

Iron deficiency may present with lethargy, developmental delay, microcytic hypochromic anaemia, angular
stomatitis, koilonychia.

Vitamin A deficiency may present with increased morbidity (and mortality) from infections, follicular

https://mypastest.pastest.com/Secure/TestMe/Browser/436619[‫ ص‬06:44:04 10/12/1437]


MyPastest

hyperkeratosis, xerophthalmia (dry eyes), night blindness.

Vitamin D deficiency may present with rickets.

Calcium deficiency in the infant is less likely with the vegan diet as calcium is present in many green, leafy
vegetables, tofu, brazil nuts and almonds. Next Question
All mothers following vegan or other free-from diets should be supported appropriately to ensure they meet the
nutritional needs of their own body and of their infant via diet inclusions alone or with supplementation.
46204
Previous Question
End Session

Tag Question

Feedback

Difficulty: Average

Peer Responses

Session Progress

Responses Correct: 0

Responses Incorrect: 22

Responses Total: 22

Responses - % Correct: 0%

Blog
About Pastest
Contact Us
Help

© Pastest 2016

https://mypastest.pastest.com/Secure/TestMe/Browser/436619[‫ ص‬06:44:04 10/12/1437]


MyPastest

Prefer to use the old MyPastest? Access it here »

Back to Filters

Question 17 of 31

A 3-year-old girl comes to see you in clinic, accompanied by her mother. She is severely obese, and when you plot
her weight on a centile chart, it is well above the 99.6th centile, and almost extends into the height centile chart
above. You see that she had
been born by spontaneous vaginal delivery (SVD) on the 25th centile for weight, but
quickly gained weight within the first 6 months of life, and continued to do so.

What is the MOST appropriate next course of action?

A Diet and exercise, and refer to social services

B Diet and lifestyle measures, and send bloods for genetics

C Diet and lifestyle measures alone

D Psychology referral

E Refer for gastric banding surgery

Explanation
The correct answer is B. Adherence to a healthy, balanced diet and increased activity are crucial measures that will
help
to reduce this girl’s weight. However, the severe early onset obesity suggests that there may be a genetic
cause for her obesity. Leptin deficiency, though rare, can cause severe hyperphagia and is the only treatable form
of monogenic obesity.

A is incorrect. Although in cases of severe obesity in children, social services involvement should be considered if
parents are not engaging with professionals to address the issue, this is not the most appropriate next course of
action here.

C is also incorrect. Whilst diet and lifestyle measures are crucial, the early onset obesity makes genetic
investigations appropriate.

D is incorrect. Psychologists play an important role in the management of obesity, and both individual
and family

https://mypastest.pastest.com/Secure/TestMe/Browser/436619[‫ ص‬06:44:47 10/12/1437]


MyPastest

therapy can help to tackle reasons for using food as a coping strategy. This may be more appropriate in an older
child, rather than in a 3-year-old.

E is not an appropriate course of action. Gastric banding may be considered in an older child with extreme obesity
where all other avenues have been exhausted.
Next Question 47151

Previous Question
End Session
Tag Question

Feedback

Difficulty: Average

Peer Responses

Session Progress

Responses Correct: 0

Responses Incorrect: 23

Responses Total: 23

Responses - % Correct: 0%

Blog
About Pastest
Contact Us
Help

© Pastest 2016

https://mypastest.pastest.com/Secure/TestMe/Browser/436619[‫ ص‬06:44:47 10/12/1437]


MyPastest

Prefer to use the old MyPastest? Access it here »

Next Question
Back to Filters

Question 18 of 31

A 10-year-old boy with Klinefelter syndrome attends your Paediatric Endocrinology clinic. He had been finding
weight management particularly difficult, and at the last clinic appointment, was prescribed a medication which
has helped him greatly and has resulted in
weight loss. His parents are very pleased with this, although his stools
are very offensive smelling.

What is the medication that he was MOST likely prescribed?

A Lactulose

B Metformin

C Orlistat

D Polyethylene glycol

E Senna

Explanation
The correct answer is C. Orlistat prevents the absorption of lipids, and can be an effective medicine, in
combination with a calorie controlled diet. Steatorrhea, as well as bowel urgency and flatulence with discharge are
commonly reported side effects.

Lactulose, polyethylene glycol (Movicol) and senna are all laxatives that are not suitable drugs to prescribe for
weight management. A, D and
E are therefore incorrect.

B is also wrong. Metformin can be a helpful drug to consider, but is unlikely to result in offensive stools.
47152

https://mypastest.pastest.com/Secure/TestMe/Browser/436619[‫ ص‬06:45:09 10/12/1437]


MyPastest

Tag Question

Feedback

Difficulty: Average

Peer Responses
Previous Question
End Session

Session Progress

Responses Correct: 0

Responses Incorrect: 24

Responses Total: 24

Responses - % Correct: 0%

Blog
About Pastest
Contact Us
Help

© Pastest 2016

https://mypastest.pastest.com/Secure/TestMe/Browser/436619[‫ ص‬06:45:09 10/12/1437]


MyPastest

Prefer to use the old MyPastest? Access it here »

Next Question
Back to Filters

Question 19 of 31

A 6-year-old girl is seen on the children’s assessment unit. She has a longstanding history of constipation,
hirsutism and tends to “feels the cold” a lot. On examination, you note that she is overweight.

Which one of the following actions will MOST likely reveal the cause of her symptoms?

A OGTT

B Plot BMI on a BMI centile chart

C Plot weight and height on a centile chart

D Take a detailed family history

E TFTs

Explanation
The correct answer is E, TFTs (Thyroid Function Tests). This girl most likely has hypothyroidism, and treatment
with levothyroxine, alongside weight management strategies is most likely to result in weight loss.

OGTT (Oral Glucose Tolerance Test) may be


appropriate if a diagnosis of diabetes mellitus is being considered.
However, this will not reveal the cause of all her symptoms. A is therefore incorrect.

B and C are also wrong. Although these are important actions to take, they will not reveal the underlying cause.

D may be helpful, particularly if other members of her family are known to have thyroid disorders, and a family
history should form part of the assessment. However, TFTs are more likely to confirm her diagnosis.
47153

https://mypastest.pastest.com/Secure/TestMe/Browser/436619[‫ ص‬06:45:32 10/12/1437]


MyPastest

Tag Question

Feedback

Difficulty: Average

Peer Responses
Previous Question
End Session

Session Progress

Responses Correct: 0

Responses Incorrect: 25

Responses Total: 25

Responses - % Correct: 0%

Blog
About Pastest
Contact Us
Help

© Pastest 2016

https://mypastest.pastest.com/Secure/TestMe/Browser/436619[‫ ص‬06:45:32 10/12/1437]


MyPastest

Prefer to use the old MyPastest? Access it here »

Back to Filters

Question 20 of 31

A 1-year-old boy present to the emergency department with a barking cough and stridor. He is treated with
dexamethasone and his respiratory symptoms improve. He has had multiple previous similar emergency
department attendances requiring steroids. His weight is 21kg.

Which is MOST likely to be an effective strategy in managing his weight?

A Ban steroid use

B Inform public health

C Refer to general outpatient clinic for detailed assessment

D Refer to respiratory

E Take a dietary history and give advice before discharge from the emergency department

Explanation
The correct answer in these circumstances is E. This question has been taken from a case study where it was found
that the infant was drinking 4 x 5 oz bottles of formula milk each night, as well as eating four meals during the
day. Mother was advised to wean him
off the overnight formula milk, and he lost 5kg over 4 months.

A is incorrect. This would not be appropriate, and steroids may be required for further acute episodes.

B is incorrect. Public health play an important role in measures to reduce obesity in children, including healthy
eating, increasing physical activity and improving mental well-being. The focus is more on overseeing health
prevention and promotion, rather than dealing with individual cases such as this one.

C and D are incorrect. This patient, although presenting for a different reason, should have a detailed history taken
without delay so that measures to improve his health can be instigated straight away.
47154

https://mypastest.pastest.com/Secure/TestMe/Browser/436619[‫ ص‬06:45:52 10/12/1437]


MyPastest

Next Question Tag Question

Feedback

Previous Question
Difficulty: Average
End Session
Peer Responses

Session Progress

Responses Correct: 0

Responses Incorrect: 26

Responses Total: 26

Responses - % Correct: 0%

Blog
About Pastest
Contact Us
Help

© Pastest 2016

https://mypastest.pastest.com/Secure/TestMe/Browser/436619[‫ ص‬06:45:52 10/12/1437]


MyPastest

Prefer to use the old MyPastest? Access it here »

Back to Filters

Question 21 of 31

A 12-year-old girl is seen in clinic for weight management. Her weight is 137kg. She is upset and angry about her
size, and the psychologist is trying to work with her. On examination, she has darkened, thickened patches of skin
in the axillae and neck. There are multiple linear burns across her lower legs, which she tells you are from her
mother’s hair straighteners. She denies self-harm and you
notice that her mother looks uncomfortable.

What is the MOST appropriate next course of action?

A Call the police

B Discuss with your consultant

C Discuss with your consultant and social services

D Discuss with your consultant and social services, get medical photography

E Refer back to the psychologist

Explanation
D is the correct answer. It is likely that this girl is suffering abuse which is affecting her confidence and self-
esteem. Given appropriate support and encouragement away from the abuser, she is likely to re-establish her self-
confidence and lose the excess weight. The first step is therefore to seek senior advice and alert social services and
use the opportunity of the child being in the hospital to obtain documentary evidence of the injuries.

A is incorrect. This case should first be discussed with the consultant, and a
plan made which will involve a social
services referral.

B and C, although correct, are not the best answers. Medical photography, alongside clear, thorough
documentation (using a safeguarding medical proforma) will provide clear documented evidence of the injuries.

E
is not appropriate. Whilst ongoing psychology involvement is important,
the priority here is dealing with this
girl’s safety and the concerns about non accidental injury.

https://mypastest.pastest.com/Secure/TestMe/Browser/436619[‫ ص‬06:46:13 10/12/1437]


MyPastest

Note-the dark thickened patches are areas of acanthosis nigricans, which are associated with obesity.
47155

Next Question

Tag Question

Previous Question
Feedback
End Session
Difficulty: Average

Peer Responses

Session Progress

Responses Correct: 0

Responses Incorrect: 27

Responses Total: 27

Responses - % Correct: 0%

Blog
About Pastest
Contact Us
Help

© Pastest 2016

https://mypastest.pastest.com/Secure/TestMe/Browser/436619[‫ ص‬06:46:13 10/12/1437]


MyPastest

Prefer to use the old MyPastest? Access it here »

Next Question
Back to Filters

Question 22 of 31

A 1-year-old girl is brought to see you in clinic. Her mother


is concerned because the child has been eating chalk.
The mother also mentions that yesterday she was in the garden and was eating soil.

What is the MOST appropriate next course of action?

A Assess for autism

B Bloods to check vitamin 25(OH)D3 level

C Bloods to check vitamin E level

D FBC, ferritin and iron studies

E Refer to CAMHS

Explanation
The correct answer is D. This girl has pica which has been linked with iron deficiency, and blood tests will reveal
this.

A is incorrect. Pica may be noted in individuals with autism, but assessment for autistic traits at this age would be
very difficult, and there are no such features suggested in the history.

Whilst it is possible that this girl is deficient in multiple vitamins due to inadequate nutrition, vitamin D or E
deficiencies do not account for her
symptoms. B and C are therefore incorrect.

E is not an appropriate course of action in a one year old with no suggestion of mental disorder in the history. In
an older child, it is important to note that schizophrenia has been linked to pica.
47156

https://mypastest.pastest.com/Secure/TestMe/Browser/436619[‫ ص‬06:46:33 10/12/1437]


MyPastest

Tag Question

Feedback

Difficulty: Average

Peer Responses
Previous Question
End Session

Session Progress

Responses Correct: 0

Responses Incorrect: 28

Responses Total: 28

Responses - % Correct: 0%

Blog
About Pastest
Contact Us
Help

© Pastest 2016

https://mypastest.pastest.com/Secure/TestMe/Browser/436619[‫ ص‬06:46:33 10/12/1437]


MyPastest

Prefer to use the old MyPastest? Access it here »

Next Question
Back to Filters

Question 23 of 31

A 15-year-old girl of Pakistani origin complains of general tiredness and vague aches and pains. She has a
25(OH)D3 level checked which is 18 ng/ml (45nmol/l).

What is the SINGLE best treatment?

A Alfacalcidiol

B Calcitriol

C Cholecalciferol

D No treatment required

E Paricalcitol

Explanation
The correct answer is C. Vitamin D deficiency can
occur in people whose exposure to sunlight is limited. At
18ng/ml, this
girl is severely deficient. Cholecalciferol is the suitable treatment for this condition.

Alfacalcidol and calcitriol should be prescribed if patients with severe renal impairment need vitamin D therapy.
A and B are therefore incorrect.

D is incorrect. This girl has severe deficiency and does require treatment.

E is wrong. Paricalcitol is used in chronic renal failure, for the prevention and treatment of secondary
hyperparathyroidism.

https://www.evidence.nhs.uk/formulary/bnf/current/9-nutrition-and-blood/96-vitamins/964-vitamin-d
47157

https://mypastest.pastest.com/Secure/TestMe/Browser/436619[‫ ص‬06:46:53 10/12/1437]


MyPastest

Tag Question

Feedback

Difficulty: Average

Previous Question Peer Responses

End Session

Session Progress

Responses Correct: 0

Responses Incorrect: 29

Responses Total: 29

Responses - % Correct: 0%

Blog
About Pastest
Contact Us
Help

© Pastest 2016

https://mypastest.pastest.com/Secure/TestMe/Browser/436619[‫ ص‬06:46:53 10/12/1437]


MyPastest

Prefer to use the old MyPastest? Access it here »

Back to Filters

Question 24 of 31

A 19-month-old female was referred for delay in walking. She had been born by emergency C section with a
birthweight of 3.98kg. Her length had fallen from the 50th centile at birth to the 0.4th
centile at 19 months. Other
family members had also been “late walkers.” On examination, she appeared short with widened wrist and knee
joints. She had bow legs. She crawled with her wrists flexed and walked on her knees. Hand X-rays revealed
metaphyseal abnormalities affecting the distal aspects of the 2nd and 5th metacarpals.
X-ray of the right arm
showed metaphyseal abnormalities of the proximal humerus and distal radius and ulna, in addition to flaring and
cupping, with widening of the growth plates. FBC, U&Es and LFTs were
normal. Her corrected Ca2+ was
2.11mmol/l, phosphate 0.72 mmol/l, alkaline phosphatase 1745 IU/l.

What is the MOST likely diagnosis?

A Autosomal dominant hypophosphataemic rickets

B Chronic hepatic failure

C Chronic renal failure

D Nutritional rickets

E X-linked hypophosphataemic rickets

Explanation
The correct answer is E. This girl has features of rickets. With other family members affected, the most likely
cause is
X-linked hypophosphataemic rickets, which accounts for over 80% of all familial hypophosphatemia. The
remaining 20% of familial hypophosphatemia have autosomal dominant hypophosphataemia (ADHR).

A is incorrect because ADHR is less common, as explained above.

B and C are wrong, because there are multiple family members affected, suggesting a familial hypophosphatemia.

D is incorrect. Nutritional rickets secondary to vitamin D deficiency does not explain the blood results, nor other

https://mypastest.pastest.com/Secure/TestMe/Browser/436619[‫ ص‬06:47:13 10/12/1437]


MyPastest

family members being affected.


47158

Next Question

Tag Question

Previous Question
Feedback
End Session
Difficulty: Average

Peer Responses

Session Progress

Responses Correct: 0

Responses Incorrect: 30

Responses Total: 30

Responses - % Correct: 0%

Blog
About Pastest
Contact Us
Help

© Pastest 2016

https://mypastest.pastest.com/Secure/TestMe/Browser/436619[‫ ص‬06:47:13 10/12/1437]


MyPastest

Prefer to use the old MyPastest? Access it here »

Back to Filters

Question 25 of 31

A 1-year-old boy is seen by the out-of-hours GP at 6pm with cough and coryza. He looks pale on examination.
Mother says that he loves cows’ milk and drinks plenty. He is referred to the children’s assessment unit and an
FBC reveals an Hb of 6.4g/dL. All observations are within normal range.

What is the single MOST appropriate course of action?

A Admit and arrange a blood transfusion urgently

B Admit and start sodium feredetate

C Discharge

D Discharge and ask GP to start sodium feredetate

E Take a detailed dietary history, admit and start sodium feredetate

Explanation
The correct answer is E. This is most likely iron
deficiency anaemia secondary to inadequate intake. A detailed
dietary history should be taken to confirm this. It takes several months to build up iron stores, but many patients
can have chronic anaemia without
symptoms or haemodynamic instability. Although parental opinions on dietary
intake may be helpful, they are not necessarily correct.

A
is incorrect. A blood transfusion is not indicated in chronic iron deficiency anaemia. The benefits and risks of a
blood transfusion out of
hours also need to be considered.

B is not the most complete answer because ensuring a detailed dietary history is taken immediately is the more
appropriate approach.

C and D are wrong. This patient needs to be monitored and observed to ensure haemodynamic stability, and to
ensure that the parents are educated about diet.
47159

https://mypastest.pastest.com/Secure/TestMe/Browser/436619[‫ ص‬06:47:34 10/12/1437]


MyPastest

Next Question Tag Question

Feedback

Previous Question
Difficulty: Average
End Session
Peer Responses

Session Progress

Responses Correct: 0

Responses Incorrect: 31

Responses Total: 31

Responses - % Correct: 0%

Blog
About Pastest
Contact Us
Help

© Pastest 2016

https://mypastest.pastest.com/Secure/TestMe/Browser/436619[‫ ص‬06:47:34 10/12/1437]


MyPastest

Prefer to use the old MyPastest? Access it here »

Back to Filters

Question 26 of 31

A 7-year-old boy with end stage renal disease complains of malaise, lethargy and bleeding gums. His teeth are
loose.

What is the MOST appropriate treatment?

A Ascorbic acid

B Cholecalciferol

C Vitamin A supplements

D Vitamin E supplements

E Vitamin K injection IM

Explanation
The correct answer is A, ascorbic acid. This child has scurvy. Although rare in the UK, patients with chronic renal
disease remain at risk of this disease due to dietary restriction, increased urinary loss of vitamin C with the use of
diuretics, and through diosalytes (for patients on dialysis). The treatment is ascorbic
acid.

B, C, D and E are incorrect, as they will not treat a vitamin C deficiency.

Cholecalciferol is also known as vitamin D3 and is used for the prevention and treatment of vitamin D deficiency.
In infants and young children, vitamin D deficiency may result in slow growth and rickets.

Vitamin A deficiency causes night blindness, poor growth, xerophthalmia, follicular hyperplasia and impaired
resistance to infection. Dietary sources are milk, fat, fruit and vegetables, eggs and liver.

Vitamin E is an antioxidant found in green vegetables


and vegetable oils. Deficiency causes ataxia, peripheral
neuropathy and retinitis pigmentosa.

Vitamin K injection IM is given to neonates at birth as they are relatively deficient in vitamin K and are

https://mypastest.pastest.com/Secure/TestMe/Browser/436619[‫ ص‬06:47:55 10/12/1437]


MyPastest

potentially at risk of serious bleeding. Levels fall further in breastfed infants in the first few days of life. This
condition was previously termed ‘haemorrhagic disease of newborn’ and is now known a, vitamin K deficiency
bleeding (VKDB) of early infancy.

https://www.evidence.nhs.uk/formulary/bnfc/current/9-nutrition-and-blood/96-vitamins/966-vitamin-k
Next Question 47160

Previous Question
End Session
Tag Question

Feedback

Difficulty: Average

Peer Responses

Session Progress

Responses Correct: 0

Responses Incorrect: 32

Responses Total: 32

Responses - % Correct: 0%

Blog
About Pastest
Contact Us
Help

© Pastest 2016

https://mypastest.pastest.com/Secure/TestMe/Browser/436619[‫ ص‬06:47:55 10/12/1437]


MyPastest

Prefer to use the old MyPastest? Access it here »

Back to Filters

Question 27 of 31

The mother of a 6 month old baby asks you for advice on how to wean. The baby was exclusively breastfed until 2
months of age, and she now has a mixture of bottle and breastfeeds.

What is the MOST appropriate advice to give to the mother?

A Cow’s milk should be introduced at 6 months

B Infant led weaning can be tried first if the mother is happy with this

C She should wait for the baby to grow teeth before introducing solids

D Soft pureed foods must be given initially

E Sweet foods such as pureed pear and apple should be introduced first

Explanation
The correct answer is B. Many parents are now choosing this method of weaning instead of the conventional
spoon feeding method. Benefits are that it may be a healthier, less expensive way of introducing complementary
foods, which babies are able to choose and enjoy more. 6 months is the recommended age to start weaning.

A is incorrect. Cow’s milk should not be introduced until a baby reaches 12 months.

C is incorrect. Many babies are able to enjoy soft finger foods before they have any teeth.

D is incorrect. Infant led weaning can be tried if the parents wish to do so.

E is incorrect. Many health visitors will advise starting savoury foods


first, to get infants used to these “first
tastes”, before introducing sweet tastes such as apple and pear.

Cameron S et al. How feasible is baby led weaning as an approach to infant feeding? Nutrients 2012 Nov;
4(11):1575-1609

https://www.nice.org.uk/guidance/PH11/chapter/4-Recommendations#infant-formula

https://mypastest.pastest.com/Secure/TestMe/Browser/436619[‫ ص‬06:48:17 10/12/1437]


MyPastest

47478

Next Question
Tag Question

Feedback
Previous Question
End Session
Difficulty: Average

Peer Responses

Session Progress

Responses Correct: 0

Responses Incorrect: 33

Responses Total: 33

Responses - % Correct: 0%

Blog
About Pastest
Contact Us
Help

© Pastest 2016

https://mypastest.pastest.com/Secure/TestMe/Browser/436619[‫ ص‬06:48:17 10/12/1437]


MyPastest

Prefer to use the old MyPastest? Access it here »

Back to Filters

Question 28 of 31

A fifteen week old female infant is brought to clinic by her parents. This is their first child and, although she had
previously been
troubled by eczema, it is now well controlled with the use of regular emollients. The parents
would like some advice about weaning.

What is the MOST appropriate advice to give?

A Breastfed babies can be weaned earlier than formula fed

B Sugary foods can be introduced before 6 months, but not savoury

C Weaning must not commence before 17 weeks

D Weaning must not commence before 6 months

E We encourage weaning at 6 months of age, and this is linked to a reduction in childhood obesity

Explanation
C is correct. The Department of Health and the World Health Organisation recommend that weaning should start
at 6 months of age i.e. once they are able to stay sat up, hold their head steady and co-ordinate their eyes, hands
and mouth. Some parents choose to wean earlier, but should be advised that weaning must not start before 17
weeks at the very earliest.

A is wrong. There is no guideline that recommends a difference in weaning between breast and bottle fed babies.

B is incorrect. Savoury foods should be introduced first.

D is incorrect. Although this should be encouraged, many parents will still choose to wean earlier than 6 months.

C is the better answer of the two.

E, whilst true, is also not the best answer because C is advice that parents must be given.
47479

https://mypastest.pastest.com/Secure/TestMe/Browser/436619[‫ ص‬06:48:37 10/12/1437]


MyPastest

Next Question Tag Question

Feedback

Previous Question
Difficulty: Average
End Session
Peer Responses

Session Progress

Responses Correct: 0

Responses Incorrect: 34

Responses Total: 34

Responses - % Correct: 0%

Blog
About Pastest
Contact Us
Help

© Pastest 2016

https://mypastest.pastest.com/Secure/TestMe/Browser/436619[‫ ص‬06:48:37 10/12/1437]


MyPastest

Prefer to use the old MyPastest? Access it here »

Back to Filters

Question 29 of 31

A one-year-old boy is referred by the GP. There is a lot of parental anxiety about the child’s nutrition. Mother had
iron deficiency anaemia in pregnancy and was therefore on iron supplements throughout pregnancy, and has
continued on these with good compliance. She also takes multivitamins containing vitamin D. The child was
exclusively breastfed until 6 months of age, and then weaned onto solid foods, but he still has 2 good breastfeed
per day lasting approximately 20 minutes. He has a varied diet, including red meat, poultry, cereals and
vegetables. The extended family have been putting pressure on the mother to stop breastfeeding, because they feel
that this will limit the
child’s brain development. The parents want some advice.

That is the MOST appropriate NEXT course of action?

A Gradually wean breastfeeds and replace with cow’s milk

B Gradually wean breastfeeds and replace with follow on milk

C Reassure the parents

D Refer to family health visitor and follow up in outpatient clinic in 6 months time

E Take bloods for iron, iron studies and vitamin D

Explanation
C is the correct answer. The World Health Organisation recommends exclusive breastfeeding for the first six
months
of an infant's life. Thereafter, local, nutritious foods should be introduced, while breastfeeding continues
for up to two years or beyond.

A and B are therefore incorrect. Follow up formula is unnecessary.

D is not correct. The family health visitor may still be involved and offer further reassurance, but a further
outpatient appointment is not necessary in this otherwise healthy child.

E is not correct. This child is well and is eating a healthy, varied diet. He is therefore
unlikely to be deficient in

https://mypastest.pastest.com/Secure/TestMe/Browser/436619[‫ ص‬06:48:57 10/12/1437]


MyPastest

iron or vitamin D.
47480

Next Question

Tag Question

Previous Question
Feedback
End Session
Difficulty: Average

Peer Responses

Session Progress

Responses Correct: 0

Responses Incorrect: 35

Responses Total: 35

Responses - % Correct: 0%

Blog
About Pastest
Contact Us
Help

© Pastest 2016

https://mypastest.pastest.com/Secure/TestMe/Browser/436619[‫ ص‬06:48:57 10/12/1437]


MyPastest

Prefer to use the old MyPastest? Access it here »

Back to Filters

Question 30 of 31

A six day old infant is admitted to the Paediatric ward. She was born at full term by normal vaginal delivery
weighing 3.46kg and has
been breastfed since birth. She has now lost 11% of her birthweight and
bilirubin is
above the phototherapy line. On examination, her fontanelle is soft, CRT<2 seconds, she is active and alert. U&Es
have been sent, and the sodium on a blood gas is 144. You have started phototherapy, but still need to make a plan
about fluids and feeding. Mother wants to continue breastfeeding, but is also happy to introduce some formula
milk.

What is the MOST appropriate course of action?

A Bottle feed formula milk at 150mls/kg/day

B Continue to exclusively breastfeed and monitor sodium

C IV fluids at 150mls/kg/day

D IV fluids at 180mls/kg/day

E Offer NG or cup feeds alongside breastfeeding

Explanation
E is the correct answer. This is not an uncommon presentation, and mothers often feel that low production of
breastmilk has caused these problems. If the mother wishes to continue breastfeeding, then she should be
supported in doing so. Involvement of
a breastfeeding advisor can help. Introducing a bottle may make latching
difficult and cause further problems with establishing breastfeeding. Cup and NG are therefore reasonable
suggestions.

A is not the best answer. See above.

B is not the best answer. With a loss in weight of >11%, the child is at risk of dehydration and so top ups of
expressed breastmilk and/or formula should be given.

https://mypastest.pastest.com/Secure/TestMe/Browser/436619[‫ ص‬06:49:18 10/12/1437]


MyPastest

C and D are wrong. IV fluids are not necessary in this situation. The baby is not clinically dehydrated at this stage.
The total fluid intake does not need to exceed 150mls/kg/day
in a term baby of adequate weight.
47481

Next Question

Tag Question
Previous Question
Feedback End Session

Difficulty: Average

Peer Responses

Session Progress

Responses Correct: 0

Responses Incorrect: 36

Responses Total: 36

Responses - % Correct: 0%

Blog
About Pastest
Contact Us
Help

© Pastest 2016

https://mypastest.pastest.com/Secure/TestMe/Browser/436619[‫ ص‬06:49:18 10/12/1437]


MyPastest

Prefer to use the old MyPastest? Access it here »

Back to Filters

Question 31 of 31

A midwife calls you from the postnatal ward. A baby was born last night to a mother who had not booked at the
hospital. Her notes are
not available and she does not speak English. However, a lady who is present with her has
managed to communicate to the midwife that Zika virus had been confirmed in pregnancy. You are busy on the
neonatal unit, but will be able to go and review her in an hour or two. The midwife wants to know what to do in
the meantime.

What is the MOST appropriate course of action?

A Advise the mother not to breastfeed

B Ask them to wait for you to review the baby

C Encourage breastfeeding

D Obtain donor breastmilk and feed the baby

E Test for HIV, and wait for the result before starting breastfeeding

Explanation
The correct answer is C. Current WHO breastfeeding recommendations remain valid in mothers known to have, or
with suspected Zika virus. Zika virus RNA has been detected in the breastmilk of two mothers with Zika virus
infection, but virus kinetics and viral load is unknown. Furthermore, no adverse neurological outcomes
have been
reported in infants who have postnatally acquired Zika virus.
Therefore, based on current evidence, the benefits of
breastfeeding still outweigh any potential risk.

A is also incorrect. See explanation above.

B is incorrect. The infant should be given a feed within the first hour of birth, as per WHO recommendations.

D is incorrect. The baby can be breastfed directly by her own mother.

E
is incorrect. This would be inappropriate. The infant should be given a
feed within the first hour of birth. There

https://mypastest.pastest.com/Secure/TestMe/Browser/436619[‫ ص‬06:49:39 10/12/1437]


MyPastest

is no suggestion that the mother has HIV, and waiting for a result would take too long.

http://apps.who.int/iris/bitstream/10665/204473/1/WHO_ZIKV_MOC_16.5_eng.pdf?ua=1&ua=1
47482

End Session

Previous Question Tag Question

Feedback

Difficulty: Average

Peer Responses

Session Progress

Responses Correct: 0

Responses Incorrect: 37

Responses Total: 37

Responses - % Correct: 0%

Blog
About Pastest
Contact Us
Help

© Pastest 2016

https://mypastest.pastest.com/Secure/TestMe/Browser/436619[‫ ص‬06:49:39 10/12/1437]


MyPastest

Prefer to use the old MyPastest? Access it here »

Back to Filters

Question 1 of 19

A 10-year-old boy was taken to his GP as his mother noted that occasionally he tilted his head towards his right
shoulder. She also remarked that his left eye seemed to move up suddenly when he looked towards his nose.

Which of the following extra-ocular muscles is most likely to be affected?

A Superior rectus

B Superior oblique

C Lateral rectus

D Inferior oblique

E Medial rectus

Explanation
Nerve supply of extra-ocular muscles is as follows:

Lateral rectus 6th


Superior oblique 4th
Inferior oblique 3rd
Superior rectus 3rd
Medial rectus 3rd
Inferior rectus 3rd

The
superior oblique muscle causes the eye to move downwards and medially. The unopposed inferior oblique, in
this case, causes the eye to deviate upwards and medially. Children tend to tilt their head (torticollis) to the side
opposite to the affected eye with their chin down toward the shoulder and their face turned away from the affected

https://mypastest.pastest.com/Secure/TestMe/Browser/436619[‫ ص‬06:24:22 10/12/1437]


MyPastest

side so as to decrease the diplopia.


11698
Previous Question

Next Question
Tag Question

Feedback

Difficulty: Average
End Session

Peer Responses

Session Progress

Responses Correct: 0

Responses Incorrect: 1

Responses Total: 1

Responses - % Correct: 0%

Blog
About Pastest
Contact Us
Help

© Pastest 2016

https://mypastest.pastest.com/Secure/TestMe/Browser/436619[‫ ص‬06:24:22 10/12/1437]


MyPastest

Prefer to use the old MyPastest? Access it here »

Back to Filters

Question 2 of 19

During an opthalmic examination, a 6-year-old boy’s right pupil is noted to remain dilated when tested with a
torch. Eye movements were normal. Further examination revealed multiple hyperpigmented areas in his groin,
each about 3mm in diameter. Larger, but paler pigmented areas were scattered over his trunk and on his right
upper limb.

What is the most likely diagnosis?

A Tuberous sclerosis

B Neurofibromatosis type 1

C Ataxia telangiectasia

D Neurofibromatosis type 2

E Incontinentia pigmenti

Explanation
Neurofibromatosis type 1 is characterised by:

6
or more café-au-lait spots more than 5mm in diameter in prepubertal individuals or more than 15mm in
postpubertal individuals
Axillary or inguinal freckling (2-3 mm hyperpigmented areas)
Lisch nodules (iris hamartomas)
2
or more neurofibromas or 1 plexiform neuroma. Neurofibromas are cutaneous rubbery lesions with
purpilish discoloration of the overlying skin. Plexiform neuromas are due to diffuse thickening of nerve
trunks (usually in the orbital or temporal regions)
Osseous lesions, e.g. sphenoid dysplasia

https://mypastest.pastest.com/Secure/TestMe/Browser/436619[‫ ص‬06:24:57 10/12/1437]


MyPastest

Optic
gliomas. These are benign tumours of the optic nerve. They are mostly asymptomatic, but can
present asPrevious
unilateral visual loss. The affected pupil does not constrict on light stimulation.
Question

Neurofibromatosis type 2 is associated with the presence of acoustic neuromas. No eye changes are noted.
Next Question
Tuberous sclerosis presents with hamartomas of the retina and optic nerve, optic atrophy and papilloedema.

In
ataxia telangiectasia, the pupil is not affected. There is disorder of conjugate eye movements. Telangiectasiae of
the eyes and skin are usually present.

Incontinentia pigmenti is associated with corneal opacities, cataracts, micropthalmos and optic atrophy.
11699
End Session

Tag Question

Feedback

Difficulty: Average

Peer Responses

Session Progress

Responses Correct: 0

Responses Incorrect: 2

Responses Total: 2

Responses - % Correct: 0%

Blog
About Pastest
Contact Us
Help

© Pastest 2016

https://mypastest.pastest.com/Secure/TestMe/Browser/436619[‫ ص‬06:24:57 10/12/1437]


MyPastest

https://mypastest.pastest.com/Secure/TestMe/Browser/436619[‫ ص‬06:24:57 10/12/1437]


MyPastest

Prefer to use the old MyPastest? Access it here »

Back to Filters

Question 3 of 19

An 8-year-old boy was taken to his ophthalmologist as he complained that he was unable to see the blackboard on
looking towards his far left side. His visual fields were checked.

What part of the visual pathway is most likely to be affected?

A Optic chiasma

B Edinger Westphal nucleus

C Optic tract

D Optic nerve

E Lateral geniculate body

Explanation
The boy is complaining of left homonymous hemianopia which results due to a lesion affecting the optic tract.

Optic
nerve - blindness in one eye; amaurosis fugax is a transient loss of vision in one eye due to
temporary lack of blood supply to the retina
Optic chiasma - bitemporal hemianopia
Optic tract - homonymous hemianopia
Optic radiation: Upper fibres - inferior homonymous quadrantanopia; Lower fibres - superior
homonymous quadrantanopia
Visual cortex - homonymous hemianopia with macular sparing.

11700

https://mypastest.pastest.com/Secure/TestMe/Browser/436619[‫ ص‬06:25:19 10/12/1437]


MyPastest

Previous Question

Tag Question
Next Question
Feedback

Difficulty: Average

Peer Responses End Session

Session Progress

Responses Correct: 0

Responses Incorrect: 3

Responses Total: 3

Responses - % Correct: 0%

Blog
About Pastest
Contact Us
Help

© Pastest 2016

https://mypastest.pastest.com/Secure/TestMe/Browser/436619[‫ ص‬06:25:19 10/12/1437]


MyPastest

Prefer to use the old MyPastest? Access it here »

Next Question
Back to Filters

Question 4 of 19

A 5-year-old girl presents to the emergency department with pain, redness, and increased lacrimation of her right
eye. She was also complaining of photophobia. During the past weeks she had been investigated for a swelling in
her right knee and left wrist. On examination, she complained of pain in her right eye when the torch was passed
in front of her left eye.

A likely diagnosis for her eye disease would be?

A Migraine

B Viral conjunctivitis

C Anterior uveitis

D Keratitis

E Posterior uveitis

Explanation
Anterior uveitis is the most likely answer.
This involves inflammation of the uveal tract, i.e. the iris, ciliary body
and choroids. The history of joint pain and swelling in only 2 joints points to oligoarticular juvenile arthritis. Signs
of anterior uveitis include tenderness over the upper eyelid and circumcorneal erythema. Slit lamp examination
may show congestion of the iris and white precipitates on the anterior cornea. Treatment includes steroid drops
and analgesia.

Viral conjunctivitis is characterised by conjunctival redness and increased lacrimation of the eye, but doesn’t
usually cause pain or photophobia. Keratitis causes pain with photophobia. However these would not explain the
associated joint findings.
11701

https://mypastest.pastest.com/Secure/TestMe/Browser/436619[‫ ص‬06:25:41 10/12/1437]


MyPastest

Previous Question
Tag Question

Feedback

Difficulty: Easy

Peer Responses

End Session

Session Progress

Responses Correct: 0

Responses Incorrect: 4

Responses Total: 4

Responses - % Correct: 0%

Blog
About Pastest
Contact Us
Help

© Pastest 2016

https://mypastest.pastest.com/Secure/TestMe/Browser/436619[‫ ص‬06:25:41 10/12/1437]


MyPastest

Prefer to use the old MyPastest? Access it here »

Back to Filters

Question 5 of 19

Which of the following statements regarding nystagmus is true?

A Vertical nystagmus is due to a disorder of the vestibular system

B Carbamazepine is a known causative agent

C In vestibular nystagmus, the slow phase is directed away from the side of the lesion

D Is always acquired in nature

E Cerebellar nystagmus improves on looking to the side of the lesion

Explanation
Nystagmus describes the involuntary rhythmic oscillations of 1 or both eyes. It can be congenital or occurs
secondary to tumours, intracranial disease, visual defects or drugs, as in this case. Nystagmus can have a slow and
a fast component, i.e. jerk nystagmus, or a fast component only, i.e. pendular nystagmus. Different types of
nystagmus include:

Vertical nystagmus: occurs due to lesions of the brainstem at the pontomedullary junction, e.g Arnold
Chiari malformation
Vestibular nystagmus: is due to disorders of the labyrinth system. The slow phase is directed to the side of
the lesion
Cerebellar nystagmus : fast component is directed towards the side of the lesion

The
triad of head nodding, torticollis, and pendular nystagmus is called spasmus nutans. It presents in the first 2
years of life and can resolve
spontaneously.
11702

https://mypastest.pastest.com/Secure/TestMe/Browser/436619[‫ ص‬06:26:14 10/12/1437]


MyPastest

Previous Question

Tag Question
Next Question
Feedback

Difficulty: Difficult

Peer Responses End Session

Session Progress

Responses Correct: 0

Responses Incorrect: 5

Responses Total: 5

Responses - % Correct: 0%

Blog
About Pastest
Contact Us
Help

© Pastest 2016

https://mypastest.pastest.com/Secure/TestMe/Browser/436619[‫ ص‬06:26:14 10/12/1437]


MyPastest

Prefer to use the old MyPastest? Access it here »

Next Question
Back to Filters

Question 6 of 19

Which of the following best describes bacterial conjunctivitis?

A Neonatal conjunctivitis occurs in infants 6 weeks or older

B Can be treated with saline drops only

C Approximately 20% of infants with chlamydial conjunctivitis can develop pneumonia

D Pseudomonas aeruginosa is the commonest causative agent

E Presentation is always with a mucopurulent discharge from the affected eye

Explanation
Conjunctivitis is due to bacterial, viral, toxic or allergic agents or in association with systemic disease. Neonatal
(by definition a neonate is a baby up to the age of 28 days) conjunctivitis occurs in the first month of life and is
usually acquired
during vaginal delivery. Causative organisms include:

Neisseria
gonorrhoea: incubation period of 2-5 days. It presents with serosanguious discharge which later
becomes pustular. It is treated with
IV ceftriaxone or cefotaxime and saline irrigation.
Chlamydia
trachomatis: incubation period 5-14 days. It presents with mild to severe inflammation of the
eyelids and a purulent discharge. Treated with topical tetracycline and oral erythromycin.

Other
uncommon organisms include Pseudomonas aeruginosa, Strep. and Staph. Species. Complications include
corneal ulceration and perforation, iridocyclitis and rarely panophthalmitis.
11703

https://mypastest.pastest.com/Secure/TestMe/Browser/436619[‫ ص‬06:26:36 10/12/1437]


MyPastest

Previous Question Tag Question

Feedback

Difficulty: Difficult

Peer Responses

End Session

Session Progress

Responses Correct: 0

Responses Incorrect: 6

Responses Total: 6

Responses - % Correct: 0%

Blog
About Pastest
Contact Us
Help

© Pastest 2016

https://mypastest.pastest.com/Secure/TestMe/Browser/436619[‫ ص‬06:26:36 10/12/1437]


MyPastest

Prefer to use the old MyPastest? Access it here »

Back to Filters

Question 7 of 19

A Parinaud sydrome
B Myasthenia Gravis
C Glaucoma
D Duane syndrome
E Oculomotor nerve palsy
F Sjogren’s syndrome
G Mobius’ syndrome
H Brain tumour
I Trochlear nerve palsy
J Brown’s syndrome

Which
of the above clinical conditions best describes the following settings?
Each option can be used once, more
than once or not at all.

Scenario 1

A 3-month-old girl with inability to abduct the right eye. Eye retraction was noted on adduction when she was
distracted with a rattle.

Your answer was incorrect

Select one...

D - Duane syndrome

Duane syndrome is a congenital eye movement disorder characterised by the inability of the eye to turn out
(abduction). The eyeball retracts into socket on adduction with narrowing of the palpebral fissure.It is caused by
incorrect connections of the eye muscles, causing some eye muscles to contract when they shouldn’t and other eye
muscles not to contract when they should. The majority of patients remain symptom free and able to maintain
binocular vision with only a slight face turn, if this is not possible then surgery may be needed.

Scenario 2

An 8-week-old baby boy is brought by his mother due to a persistent esotropia. She also noted that her son did not
close his eyes properly when he was asleep.

https://mypastest.pastest.com/Secure/TestMe/Browser/436619[‫ ص‬06:27:13 10/12/1437]


MyPastest

Your answer was incorrect

Previous Question
Select one...

G - Mobius’ syndrome
Next Question
Mobius' syndrome is very rare and characterised by facial paralysis and the inability to move the eyes
from side to
side. It occurs as cranial nerves VI and VII are underdeveloped. Limb (clubbed feet, missing digits) and chest wall
abnormalities sometimes occur with the syndrome. Esotropia (squint) occurs and one or both eyes turn inwards so
the baby appears cross-eyed.
There is no cure so treatment is supportive. Surgery can correct crossed eyes, protect
the cornea via tarsorraphy, and improve limb and jaw deformities. End Session
Scenario 3

An 8-year-old boy presents with inability to look up on adduction following an upper respiratory tract infection.

Your answer was incorrect

Select one...

J - Brown’s syndrome

Brown's syndrome is also a rare disorder characterised by defects in eye movement. There is a congential
version
or it can be acquired as a consequence post infection or inflammation. It is caused by a defect in the superior
oblique tendon sheath, so the eye has difficulty looking upwards particulary during adduction. Treatment involves
mainly surgical correction, but anti-inflammatory medication has been used or injections of steroids into the
trochlea.

Explanation
 
11704

Tag Question

Feedback

Difficulty: Average

Session Progress

Responses Correct: 0

Responses Incorrect: 9

https://mypastest.pastest.com/Secure/TestMe/Browser/436619[‫ ص‬06:27:13 10/12/1437]


MyPastest

Responses Total: 9

Responses - % Correct: 0%

Blog
About Pastest
Contact Us
Help

© Pastest 2016

https://mypastest.pastest.com/Secure/TestMe/Browser/436619[‫ ص‬06:27:13 10/12/1437]


MyPastest

Prefer to use the old MyPastest? Access it here »

Back to Filters

Question 8 of 19

A Vincristine

B Toxoplasmosis

C Rhabdomyoma

D Horner’s syndrome

E Neurofibromatosis

F Marcus Gunn syndrome

G Occulomotor nerve palsy

H Botulism

I Sturge-Weber syndrome

J Myasthenia Gravis

Which
of the causes of ptosis above is likely to result in the following clinical settings? Each option can be used
once, more than once or not at all.

Scenario 1

A 1-week-old baby was taken to the GP as his mother noted that his right eyelid was droopy. On examination, he
had miosis of the right pupil.

Your answer was incorrect

Select one...

D - Horner’s syndrome

Horner’s
syndrome is usually acquired but can also be congenital. Due to the defect in the sympathetic nervous
system the infant can present with ptosis (dropping of the upper eyelid), miosis (constricted pupil), decreased
sweating on the affected side and a slow response of the pupil
to light.

Scenario 2

https://mypastest.pastest.com/Secure/TestMe/Browser/436619[‫ ص‬06:27:36 10/12/1437]


MyPastest

On returning from school, an 8-year-old girl told her mother that she couldn’t keep her eyes open when she
was
looking up toward the volleyball net at school.
Previous Question
Your answer was incorrect

Select one... Next Question


J - Myasthenia Gravis

Myasthenia gravis is characterised by fatiguability and muscles become progressively weaker during activity.
Muscles controlling eyelid movement can be affected and in some cases may End Session
be limited to these muscles. The
child may have ptosis of one or both the eyes or diplopia.

Scenario 3

A 6-year-old girl was entertaining her school friends by showing them how her right eyelid would go up as she
opened her mouth.

Your answer was incorrect

Select one...

F - Marcus Gunn syndrome

Marcus Gunn syndrome is also known as 'jaw-winking syndrome' and can be inherited in an autosomal dominant
manner. The stimulation of the trigeminal nerve by contraction of the pterygoid muscles of jaw results in the
excitation of the branch of the oculomotor nerve that innervates the levator palpebrae superioris
ipsilaterally.
Treatment not usually needed, but in severe cases surgery with excision of the levator palperae muscles and then
frontalis
brow suspension may be an option.

Explanation
 
11705

Tag Question

Feedback

Difficulty: Average

Session Progress

Responses Correct: 0

https://mypastest.pastest.com/Secure/TestMe/Browser/436619[‫ ص‬06:27:36 10/12/1437]


MyPastest

Responses Incorrect: 12

Responses Total: 12

Responses - % Correct: 0%

Blog
About Pastest
Contact Us
Help

© Pastest 2016

https://mypastest.pastest.com/Secure/TestMe/Browser/436619[‫ ص‬06:27:36 10/12/1437]


MyPastest

Prefer to use the old MyPastest? Access it here »


Previous Question

Next Question
Back to Filters

Question 9 of 19
End Session

Which of the following statements is true in the epidemiology of toxoplasmosis?

A Adult cats are the major host

B Sheep act as an intermediate host

C Humans cannot pass the disease to one another

D Ophthalmological involvement is rare in humans

E Primary infection is common in HIV-related disease

Explanation
Kittens are the primary host; sheep and cattle eat food contaminated with soil contaminated with kitten faeces, and
humans ingest the organisms in poorly cooked meat. Humans pass the disease between one another vertically (in
utero) but not from one another. Eye involvement is usual. HIV is associated with reactivation of the disease.
12664

Tag Question

Feedback

Difficulty: Difficult

Peer Responses

https://mypastest.pastest.com/Secure/TestMe/Browser/436619[‫ ص‬06:27:57 10/12/1437]


MyPastest

Session Progress

Responses Correct: 0

Responses Incorrect: 13

Responses Total: 13

Responses - % Correct: 0%

Blog
About Pastest
Contact Us
Help

© Pastest 2016

https://mypastest.pastest.com/Secure/TestMe/Browser/436619[‫ ص‬06:27:57 10/12/1437]


MyPastest

Prefer to use the old MyPastest? Access it here »

Back to Filters

Question 10 of 19

Which of the following conditions is most likely to permanently impair visual development in a 2-year-old child?

A Blepharitis

B Congenital ptosis

C Heterochromia

D Nasolacrimal duct obstruction

E Recurrent chalazion

Explanation
Visual development in a child can be permanently impaired when there is deprivation of visual stimulation or
when the oculomotor function is impaired. Congenital ptosis can represent both these components, where impaired
levator function of the eyelid causes the eye to essentially be closed (if the ptosis crosses over the visual axis).
Most congenital ptosis will improve over time with conservative treatment such as glasses, and do not require
surgical intervention.

Stem A, Blepharitis is incorrect. This is a predominantly inflammatory


condition which affects the eyelids and
rarely presents any issue with visual impairment.

Heterochromia is incorrect. Some individuals are simply born with different iris colours and therefore this is not
automatically an indicator of disease. However, heterochromia in the presence of other abnormalities such as
anisocoria,
ptosis and anhyrosis should raise suspicion for Horner syndrome.

Nasolacrimal duct obstruction is seen in over 5% of infants, and of these, approximately 90% will resolve
spontaneously within a year. It can be a unilateral or bilateral finding, and is not normally associated
with
concerns regarding visual impairment.

Recurrent chalazion (meibomian cysts) is incorrect. These can be a consequence of blepharitis, and often look

https://mypastest.pastest.com/Secure/TestMe/Browser/436619[‫ ص‬06:28:19 10/12/1437]


MyPastest

dramatic (a painless, hard, round nodule caused by inflammation of the meisbomian gland which results from
obstruction of the gland duct), which leads to its referral to secondary
care. Most small chalazia resolve
Previous Question
spontaneously but incision and drainage may be necessary. Their temporary nature makes the condition unlikely to
be a threat to visual development although recurrent chalazion may be a sign of chronic meibomian gland
dysfunction and need referral to a paediatric ophthalmologist.
Next Question 47161

End
Tag Session
Question

Feedback

Difficulty: Average

Peer Responses

Session Progress

Responses Correct: 0

Responses Incorrect: 14

Responses Total: 14

Responses - % Correct: 0%

Blog
About Pastest
Contact Us
Help

© Pastest 2016

https://mypastest.pastest.com/Secure/TestMe/Browser/436619[‫ ص‬06:28:19 10/12/1437]


MyPastest

Prefer to use the old MyPastest? Access it here »

Back to Filters

Question 11 of 19

You are arranging necessary referrals for a child following a diagnosis.

Which of the following diagnoses is LEAST likely to require immediate formal ocular assessment.

A Albinism

B Chickenpox

C Incontinentiapigmenti

D Juvenile idiopathic arthritis

E Marfan syndrome

Explanation
With chicken pox, a child may develop a localised
red lesion on the conjunctivae, but usually these self-resolve
and no intervention is required so B is the correct answer.

Options A, C, D and E will all require formal immediate ophthalmological assessments.

Children with albinism are prone to develop visual problems, and often require help to support visual
development. It affects each individual differently, from minimal to being legally blind. These include nystagmus,
squints, foveal hypoplasia, refractive errors and optic nerve
misrouting.

Incontinentiapigmenti is a rare X-linked inherited condition which had been named for its microscopic
appearance. Affected
children can develop retinal vascular occlusions and its sequelae (haemorrhage,
neovascularisation and tractional retinal detachment).

Juvenile idiopathic arthritis is a condition that may be associated with uveitis, cataracts and glaucoma. It is often
bilateral and patients frequently have mild symptoms.

Marfan syndrome is often


associated with high myopia, ectopialentis and carries a higher risk of
developing

https://mypastest.pastest.com/Secure/TestMe/Browser/436619[‫ ص‬06:29:01 10/12/1437]


MyPastest

retinal detachments. Always consider a referral to ophthalmology if Marfan syndrome is suspected.


47162
Previous Question

Next Question
Tag Question

Feedback

Difficulty: Average
End Session

Peer Responses

Session Progress

Responses Correct: 0

Responses Incorrect: 15

Responses Total: 15

Responses - % Correct: 0%

Blog
About Pastest
Contact Us
Help

© Pastest 2016

https://mypastest.pastest.com/Secure/TestMe/Browser/436619[‫ ص‬06:29:01 10/12/1437]


MyPastest

Prefer to use the old MyPastest? Access it here »

Back to Filters

Question 12 of 19

You see an 18-month-old girl in clinic who has been referred with an abnormal red reflex. On examination of the
eyes you found the right cornea to be opaque.

Which of the following is the most likely diagnosis?

A Coats’ disease

B Congenital cataract

C Congenial glaucoma

D Osteogenesisimperfecta

E Retinoblastoma

Explanation
Red reflex can be observed when the ocular media is clear (from the cornea to the retina) and devoid of any
opacities. An abnormal red reflex occurs when there is interference to the clear media or the posterior pole of the
eye (which the light reflects from). An abnormal red reflex can display as a white, yellow or black reflection.

Congenital glaucoma begins within the first 3 years of life and can present with corneal clouding from oedema
which impairs the appearance of the red reflex. Glaucoma is damage of the optic nerve
with visual field loss
caused by, or related to, elevated pressure within the eye. Normal intraocular pressure in infants and young
children is <20mmHg.

Coats’ disease, also known as exudative retinitis, is a very rare congenital condition occurring more frequently in
boys than girls. Peripheral retinal telangiectasias and aneurismal dilation lead to extensive areas of exudates giving
the retina a yellow-white appearance which may produce leukocoria. Peak age
is 8 – 10 years and most commonly
under 18 years. The condition is usually bilateral.

Congenital cataract gives rise to abnormal red reflex from opacity within the lens. Congenital cataract is the most

https://mypastest.pastest.com/Secure/TestMe/Browser/436619[‫ ص‬06:29:26 10/12/1437]


MyPastest

common remedial cause of blindness in the developed world.

Previous
Ocular manifestation Question
of osteogenesisimperfecta includes development of a blueish hue on the sclera, from thinned
sclera.

Retinoblastoma
cause abnormal red reflex from pale lesions within the retina. It is caused by a mutation in a
Next Question
growth-suppressor gene. Both alleles have to be affected to develop the tumour. The most common age of
diagnosis is between 1 year and 18 months.
47163

End Session
Tag Question

Feedback

Difficulty: Average

Peer Responses

Session Progress

Responses Correct: 0

Responses Incorrect: 16

Responses Total: 16

Responses - % Correct: 0%

Blog
About Pastest
Contact Us
Help

© Pastest 2016

https://mypastest.pastest.com/Secure/TestMe/Browser/436619[‫ ص‬06:29:26 10/12/1437]


MyPastest

Prefer to use the old MyPastest? Access it here »

Back to Filters

Question 13 of 19

You are assessing the sight of a newborn before discharge.

Which of the following are you LEAST likely to be able to assess at this stage?

A Extraocular movements

B Gross visual acuity

C Pupils

D Red reflex

E Visual fields

Explanation
Assessment of a newborn’s visual fields is unnecessary as well as nearly impossible, although gross information
could be captured from their reaction to a peripheral stimulus. Formal visual fields can be assessed, if required, by
the age of 9 months but is often dependent on the child’s ability to concentrate on a task. The infant will be sat on
a carer’s lap and the examiner faces the child, attracting their attention centrally using a toy or a light. A child with
normal fields will readily move his or her head or eyes in the direction of the object or light source.

Even though
a newborn is unable to cooperate fully with examination, much vital information regarding the
child’s ocular status could be determined and is based on their reaction and response to stimulation. This includes
visual acuity, extraocular movements, pupils and red reflex.

Extraocular movements can be assessed simply by observing the child in its cot, with a stimulus that moves in the
6 main
directions of gaze.

A newborn’s visual acuity is estimated to be about 6/240, and they usually display attentiveness to bright or large
objects. Either eye must be occluded to assess individual vision, through their reaction.

Assessment of a newborn’s pupils is usually limited to gross measures such as colour size and reaction to light. A

https://mypastest.pastest.com/Secure/TestMe/Browser/436619[‫ ص‬06:30:22 10/12/1437]


MyPastest

swinging light test provides important information about the relative integrity of the optic nerve.

Previous
Red reflex can be performed Question
with the direct ophthalmoscope. The light source must be shined into both pupils
together in order for the clinician to make a direct comparison between the two eyes.
47164

Next Question

Tag Question

Feedback
End Session

Difficulty: Average

Peer Responses

Session Progress

Responses Correct: 0

Responses Incorrect: 17

Responses Total: 17

Responses - % Correct: 0%

Blog
About Pastest
Contact Us
Help

© Pastest 2016

https://mypastest.pastest.com/Secure/TestMe/Browser/436619[‫ ص‬06:30:22 10/12/1437]


MyPastest

Prefer to use the old MyPastest? Access it here »

Back to Filters

Question 14 of 19

A 5-year-old boy was seen in the emergency department after a


mild fall, which resulted in a large bruise on both
his thighs. When you examine him, you note blue discolouration to both his sclera.

Which of the following conditions is most likely to be the underlying cause of his presentation?

A Juvenile idiopathic arthritis

B Nephrotic syndrome

C Osteogenesis imperfecta

D Pseudoxanthoma elasticum

E Wilson’s disease

Explanation
Sufferers of osteogenesis imperfecta are born with deficiency in collagen production (normally type 1). Diagnosis
is usually made through the combination of clinical features, together with
collagen or genetic testing.
Abnormalities in collagen production leads to easy bruising from minor bumps, due to thinned vessels and skin.
This can also lead to abnormalities in the appearance and function of the eye (bluish thinned sclera) and patients
are recommended
to have an eye test every 2-3 years.

www.oif.org

Juvenile idiopathic arthritis is incorrect as easy bruising is not a feature of this condition. Neither do sufferers
present with thinned sclera, but rather red eyes from uveitis.

Nephrotic syndrome is incorrect as the child would usually be in a hypercoaguable state, and often have issues
with oedema (anasarca) which includes facial and eyelid oedema. The sclera is not involved.

Pseudoxanthoma elasticum is incorrect. Accumulation of minerals and fragmentation of elastic lamina can lead to
vascular occlusions but not easy bruising per
se. Sufferers are also likely to develop posterior segment ocular

https://mypastest.pastest.com/Secure/TestMe/Browser/436619[‫ ص‬06:33:36 10/12/1437]


MyPastest

problems such as angioid streaks.

Previous
Children with Wilson’s Question
disease are prone to easy bruising from thrombocytopenia; the ocular appearance is
described as Kayser-Fleischer rings which appear as a dark
hue on the cornea that seems to encircle the iris so this
is an incorrect answer.
47165
Next Question

Tag Question
End Session
Feedback

Difficulty: Average

Peer Responses

Session Progress

Responses Correct: 0

Responses Incorrect: 18

Responses Total: 18

Responses - % Correct: 0%

Blog
About Pastest
Contact Us
Help

© Pastest 2016

https://mypastest.pastest.com/Secure/TestMe/Browser/436619[‫ ص‬06:33:36 10/12/1437]


MyPastest

Prefer to use the old MyPastest? Access it here »

Back to Filters

Question 15 of 19

An 8-month-old male child was brought to the emergency department with mild swelling and yellow sticky
discharge around his left eye with no eye redness. According to his parents, he has always had sticky matter
accumulating around his eyes since birth, although his
right eye seemed to have resolved of its own accord about
month ago. The child is otherwise well and undistressed.

Which of the following is the most likely diagnosis?

A Congenital glaucoma

B Eczema herpeticum

C Nasolacrimal ducts obstruction

D Neonatorum ophthalmia

E Periorbital cellulitis

Explanation
Nasolacrimal duct obstruction is commonly seen, where infants develop either unilateral or bilateral ocular
discharge with occasional lump that can be palpated in the nasolacrimal region. Most of these resolve before the
age of 1, or occasionally later, by age
2.

Congenital glaucoma is incorrect as ocular discharge is not a


feature of this condition. Instead, watery eye in a
child can point to
this diagnosis.

Eczema herpeticum is incorrect, as this is usually an acute short lasting episode that resolves following treatment
and supportive care. There would also be multiple cutaneous vesicular lesions typical of a herpetic cause.

Neonatorum ophthalmia is incorrect as infants with this condition present with greenish discharge
and
conjunctivitis, with onset from birth. This is most commonly picked up early and treated in the community.

Periorbital cellulitis is incorrect as the child would be likely to display distress, have associated ptosis and present

https://mypastest.pastest.com/Secure/TestMe/Browser/436619[‫ ص‬06:33:57 10/12/1437]


MyPastest

acutely.
47166
Previous Question

Next Question
Tag Question

Feedback

Difficulty: Average
End Session

Peer Responses

Session Progress

Responses Correct: 0

Responses Incorrect: 19

Responses Total: 19

Responses - % Correct: 0%

Blog
About Pastest
Contact Us
Help

© Pastest 2016

https://mypastest.pastest.com/Secure/TestMe/Browser/436619[‫ ص‬06:33:57 10/12/1437]


MyPastest

Prefer to use the old MyPastest? Access it here »

Back to Filters

Question 16 of 19

You examine a 4-year-old boy in the emergency department, who


complained that he was not able to see at all
when he rubs his right eye. On examination you observe leucocoria in his left eye. You take a
brief history, no
neurological symptoms are mentioned.

Which of the following is LEAST likely to be part of your immediate management plan for this child?

A Arrange ocular ultrasound

B Contact the on call ophthalmologist immediately

C Intracranial imaging

D Obtain a detailed family history

E Perform fundoscopy

Explanation
Intracranial imaging is usually unnecessary and certainly the least urgent of the above. It may be indicated if the
child or his parents report neurological symptoms – which should be asked about specifically during detailed
history taking.

Ocular ultrasound can be arranged or performed within the department, if


the facility is readily available. This
would convey much useful information regarding the posterior part of the eyeball.

In any case of leucocoria, the child requires specialist assessment as soon as possible to ascertain the nature of an
abnormal red reflex.

The first clinician who assesses the child should perform fundoscopy. This would allow as much information as
possible to be conveyed to the ophthalmology team.

A detailed family history is vital, as the likes of retinoblastoma and congenital cataract are hereditary.
47167

https://mypastest.pastest.com/Secure/TestMe/Browser/436619[‫ ص‬06:34:23 10/12/1437]


MyPastest

Previous Question

Tag Question
Next Question
Feedback

Difficulty: Average

Peer Responses
End Session

Session Progress

Responses Correct: 0

Responses Incorrect: 20

Responses Total: 20

Responses - % Correct: 0%

Blog
About Pastest
Contact Us
Help

© Pastest 2016

https://mypastest.pastest.com/Secure/TestMe/Browser/436619[‫ ص‬06:34:23 10/12/1437]


MyPastest

Prefer to use the old MyPastest? Access it here »

Back to Filters

Question 17 of 19

You examine an 11-year-old girl who complained of predominantly left red eye, with mild photophobia. You
examine her alongside the ophthalmology doctor, who then prescribes topical steroids
and cycloplegic.

Which of the following is the action of topical cycloplegia (such as cyclopentolate or atropine)?

A Can cause permanent mydriasis

B Has antimicrobial properties

C Helps improve vision immediately on application

D Prevents iris-lens adhesions

E Works through pure antimuscarinic pathway

Explanation
Cycloplegics cause mydriasis (pupillary dilatation), and forms an essential part of treatment for uveitis. Its role is
mainly to prevent posterior synechiae (iris-lens adhesions), as well as to reduce symptoms of photophobia so D is
the correct answer.

A is incorrect as the effect of cycloplegia wears off over time, variation depending on the type. The duration of
action for cyclopentolate is about 24 hours while atropine may take around 1-2 weeks before wearing off
completely.

B is incorrect, as topical cycloplegia is not known to make any contribution in combating infectious processes.

C is incorrect as the action of cycloplegia induces temporary paralysis


of the iris muscles and ciliary body,
preventing optical focus.

E is incorrect as cycloplegic medications are both antimuscarinic and anticholinergic in nature.


47168

https://mypastest.pastest.com/Secure/TestMe/Browser/436619[‫ ص‬06:34:48 10/12/1437]


MyPastest

Previous Question

Tag Question
Next Question
Feedback

Difficulty: Average

Peer Responses End Session

Session Progress

Responses Correct: 0

Responses Incorrect: 21

Responses Total: 21

Responses - % Correct: 0%

Blog
About Pastest
Contact Us
Help

© Pastest 2016

https://mypastest.pastest.com/Secure/TestMe/Browser/436619[‫ ص‬06:34:48 10/12/1437]


MyPastest

Prefer to use the old MyPastest? Access it here »

Back to Filters

Question 18 of 19

A 3-year-old boy attended the emergency department with his parents. They noticed that his left eye seems to turn
in when he tries to complete his colouring book. The father mentions that he had a similar squint as a young child
which was treated with an eye patch.

The ophthalmology doctor, who assessed the child with you, says that it is likely to be an accommodative squint.

Which of the following is true about childhood squints?

A Correcting refractive error is vital

B Eye patching as a treatment is rarely helpful

C Squints are rarely hereditary

D Squints normally resolve without treatment

E Surgical treatment is inevitable

Explanation
The first step in the management of childhood squint is detection and correction of refractive error. The use of
prescription glasses for a young child must be encouraged to allow the child the best chance to see, which is
essential in the development of visual pathways.

Eye patching is an essential part of treatment for childhood squints. Theoretically, patching the good eye
encourages the better eye to pick up images to see and helps develop the visual pathway for this concerned eye.

Childhood squints often run in families, and it is not uncommon for the parent or grandparent of the child to give a
history of wearing eye patches, prescribed glasses, or squint surgery.

A child with a squint often requires intervention, which is mostly conservative such as eye patching or correction
of any refractive errors. Without treatment, a child could end up with amblyopia.

https://mypastest.pastest.com/Secure/TestMe/Browser/436619[‫ ص‬06:35:08 10/12/1437]


MyPastest

Surgical correction of squint is essential if conservative measures such as eye patching or correcting refractive
error is unsatisfactory in helping the child develop his/her vision. Children often require a visit to an orthoptist at
Previous Question
least, to ensure their level of vision is stable.
47169

Next Question

Tag Question

Feedback End Session

Difficulty: Average

Peer Responses

Session Progress

Responses Correct: 0

Responses Incorrect: 22

Responses Total: 22

Responses - % Correct: 0%

Blog
About Pastest
Contact Us
Help

© Pastest 2016

https://mypastest.pastest.com/Secure/TestMe/Browser/436619[‫ ص‬06:35:08 10/12/1437]


MyPastest

Prefer to use the old MyPastest? Access it here »

Back to Filters

Question 19 of 19

A 10-year-old girl with behavioural difficulties was referred


by her GP with right knee pain. Her GP noted that her
eyes are also red, although the girl denied any ocular discomfort.

Which of the following should be suspected?

A Anterior uveitis

B Congenital cataract

C Corneal foreign body

D Periorbital cellulitis

E Uncorrected refractive error

Explanation
When a child with joint problems present with a red eye, the likelihood of an anterior uveitis must be excluded by
involving an ophthalmologist in the assessment.

Cataract does not give rise to a red eye, but to leucocoria. Often this would have been picked up at an earlier age.

A
patient with a corneal foreign body almost would almost invariably give
a suspicious history (onset while
playing outdoors etc), as well as have marked symptoms of ocular discomfort (red, watery and painful eyes).

Periorbital cellulitis is a condition that often presents with a child who is acutely unwell and distressed.

A child with an uncorrected refractive error often complains of being


unable to see in class, and does not normally
present with ocular redness.
47170

End Session

https://mypastest.pastest.com/Secure/TestMe/Browser/436619[‫ ص‬06:35:33 10/12/1437]


MyPastest

Previous Question Tag Question

Feedback

Difficulty: Average

Peer Responses

Session Progress

Responses Correct: 0

Responses Incorrect: 23

Responses Total: 23

Responses - % Correct: 0%

Blog
About Pastest
Contact Us
Help

© Pastest 2016

https://mypastest.pastest.com/Secure/TestMe/Browser/436619[‫ ص‬06:35:33 10/12/1437]


MyPastest

Prefer to use the old MyPastest? Access it here »

Back to Filters

Question 1 of 1

A 7-year-old boy presents with pain in his right leg.


He is afebrile. On examination you note that the range of
movement in his right hip is limited by pain. The most likely diagnosis is:

A Perthes’ disease

B Irritable hip

C Septic arthritis

D Slipped upper femoral epiphysis (SUFE)

E Osgood–Schlatter disease

Explanation
As this child is afebrile and has no previous history of a viral illness, septic arthritis and irritable hip are unlikely.
Irritable hip is uncommon in this age group anyway (usually affecting those aged 2–4 years). Osgood–Schlatter
disease is
pain caused by inflammation of the patella tendon at the tibial insertion. Slipped upper femoral
epiphysis occurs classically in adolescence (M:F = 3:2).
22219

End Session

Previous Question Tag Question

Feedback

Difficulty: Average

https://mypastest.pastest.com/Secure/TestMe/Browser/436619[‫ ص‬10:36:40 08/12/1437]


MyPastest

Peer Responses

Session Progress

Responses Correct: 0

Responses Incorrect: 1

Responses Total: 1

Responses - % Correct: 0%

Blog
About Pastest
Contact Us
Help

© Pastest 2016

https://mypastest.pastest.com/Secure/TestMe/Browser/436619[‫ ص‬10:36:40 08/12/1437]


MyPastest

Next Question
Prefer to use the old MyPastest? Access it here »

Previous Question
Back to Filters
End Session
Question 1 of 47

A 7-year-old boy presents to the Emergency Department


with an acute onset of scrotal pain. On examination there
is scrotal tenderness, swelling, associated hydrocoele and redness of the skin.

Which is the most likely cause?

A Torsion of the testicle

B Epididymo-orchitis

C Mumps

D Torsion of the hydatid of Morgagni

E An incarcerated inguinal hernia

Explanation
All of the causes listed are possible causes of an acute red scrotum, but in view of his age, the most likely cause is
torsion of the hydatid of Morgagni. Testicular torsion may occur at any age but is most common at adolescence
(age 12-16 years). Torsion of the hydatid usually occurs in prepubertal males.

Exam theme question from June 2015


14888

Tag Question

Feedback

https://mypastest.pastest.com/Secure/TestMe/Browser/436619[‫ ص‬05:52:46 10/12/1437]


MyPastest

Difficulty: Difficult

Peer Responses

Session Progress

Responses Correct: 0

Responses Incorrect: 1

Responses Total: 1

Responses - % Correct: 0%

Blog
About Pastest
Contact Us
Help

© Pastest 2016

https://mypastest.pastest.com/Secure/TestMe/Browser/436619[‫ ص‬05:52:46 10/12/1437]


MyPastest

Next Question
Prefer to use the old MyPastest? Access it here »

Back to Filters

Question 2 of 47

An 18-month-old boy presents with vomiting, abdominal pain and rectal bleeding. He is dehydrated and there is
an abdominal mass. A Meckel’s scan is negative.

What is the MOST likely diagnosis?

A Enteric duplication

B Intussusception

C Meckel’s diverticulum

D Midgut volvulus

E Appendicitis

Explanation
In this age group intussusception is the leading diagnosis.

A
duplication cyst could give all these features, although it may contain
sufficient gastric epithelium to produce a
positive Meckel’s scan. However, it would be a comparatively rare diagnosis.

A
Meckel’s diverticulum is still possible (scintigraphy has poor sensitivity) and it could have bled, but there are
also features of gut obstruction. Although Meckel’s diverticulum can be the cause of gut obstruction, it is less
likely to be bleeding and obstructing at the
same time.

Midgut volvulus is plausible and is important to consider in these circumstances. However, in this age group, it is
less likely than intussusception. Acute appendicitis can certainly cause gut obstruction and a palpable mass, but
rarely presents
with rectal bleeding.
21875

https://mypastest.pastest.com/Secure/TestMe/Browser/436619[‫ ص‬05:53:20 10/12/1437]


MyPastest

Tag Question

Feedback
Previous Question
Difficulty: Easy
End Session
Peer Responses

Session Progress

Responses Correct: 0

Responses Incorrect: 2

Responses Total: 2

Responses - % Correct: 0%

Blog
About Pastest
Contact Us
Help

© Pastest 2016

https://mypastest.pastest.com/Secure/TestMe/Browser/436619[‫ ص‬05:53:20 10/12/1437]


MyPastest

Prefer to use the old MyPastest? Access it here »

Back to Filters

Question 3 of 47

Inguinal hernias in premature boy infants are MOST likely to:

A Resolve spontaneously

B Result in testicular atrophy

C Recur after surgery

D Transform into hydrocoeles

E Be direct hernias

Explanation
Inguinal hernias in premature boy neonates have a substantial chance of recurring (in about 5% of cases). Inguinal
hernias
in children are nearly always indirect, the gut passing through the peritoneal channel made by the
processus vaginalis. The direct form of hernia (a bulge through the posterior wall of the inguinal canal) is very rare
in the absence of an underlying connective tissue disorder. Spontaneous resolution has been recorded, and would
be expected to involve a transient transformation into a hydrocoele. The patent processus would have to close to
the point where the entry of gut through the deep ring into the inguinal region was prevented. This would
convert
the hernia into a hydrocoele, where fluid could still flow into
and out of the sac via the narrowed, but still patent,
processus. The endpoint of the natural history would see the processus closing completely, leaving evidence of
neither a hernia nor a hydrocoele. However, although both these situations are encountered anecdotally, they are
relatively rare. If an inguinal hernia becomes incarcerated, it
may damage adjacent tissues. The effects of local
pressure cause such damage. Pressure on the testicular vessels can cause ischaemia and atrophy of the testis.
However, another cause of testicular vessel damage is the herniotomy operation needed to repair the hernia, which
may prove to be a very difficult procedure. It may be impossible to identify which mechanism was responsible if
testicular atrophy follows hernia incarceration. 21878

https://mypastest.pastest.com/Secure/TestMe/Browser/436619[‫ ص‬05:53:48 10/12/1437]


MyPastest

Next Question

Previous Question Tag Question

Feedback
End Session
Difficulty: Average

Peer Responses

Session Progress

Responses Correct: 0

Responses Incorrect: 3

Responses Total: 3

Responses - % Correct: 0%

Blog
About Pastest
Contact Us
Help

© Pastest 2016

https://mypastest.pastest.com/Secure/TestMe/Browser/436619[‫ ص‬05:53:48 10/12/1437]


MyPastest

Next Question
Prefer to use the old MyPastest? Access it here »

Previous Question
Back to Filters
End Session
Question 4 of 47

A 2-year-old presents with a very painful scrotum. On


examination the scrotum is swollen and inflamed on the
right side. The testis on the right is not swollen although it is tender at the upper pole. What is the most likely
diagnosis?

A Testicular torsion

B Idiopathic scrotal oedema

C Torted hydatid of Morgagni

D Epididymo-orchitis

E Mumps orchitis

Explanation
The most likely diagnosis is a torted cyst of Morgagni (an embryonic remnant on the poles of the testes).
However, the
most important diagnosis to exclude is testicular torsion and so most will undergo surgical
exploration. Mumps orchitis is rare now because of
MMR. Infertility secondary to mumps orchitis is rare (< 1%).
22356

Tag Question

Feedback

Difficulty: Average

https://mypastest.pastest.com/Secure/TestMe/Browser/436619[‫ ص‬05:54:33 10/12/1437]


MyPastest

Peer Responses

Session Progress

Responses Correct: 0

Responses Incorrect: 4

Responses Total: 4

Responses - % Correct: 0%

Blog
About Pastest
Contact Us
Help

© Pastest 2016

https://mypastest.pastest.com/Secure/TestMe/Browser/436619[‫ ص‬05:54:33 10/12/1437]


MyPastest

Prefer to use the old MyPastest? Access it here »

Back to Filters

Question 5 of 47

A 15-week-old baby presents with vomiting and weight loss to the emergency department. Further examination
demonstrates a palpable mass centrally. A routine ultrasound scan demonstrated a pylorus with a
length of 8mm
and a target sign centrally.

What is the most likely diagnosis?

A Intussusception

B Pyloric Stenosis

C Meckels Diverticulum

D Gastro-Oesophageal reflux

E Duplication Cyst

Explanation
The most likely diagnosis here is intussusception. This occurs in children aged 3-12 months of age.

Pyloric
stenosis typically affects babies aged 4-12 weeks, a pyloric muscle thickness of greater than 3mm and/ or a
length of greater than 12mm is abnormal.

A target sign is a diagnostic feature for intussusception which is where one loop of bowel sits within another.

Answer
stem A is correct - a target sign on ultrasound scan is diagnostic of intussusception therefore stems D & E
are false. A pyloric length of > 12mm is pathological, a pyloric mass is typically right upper quadrant, so B is
false. Meckels are almost never palpable therefore stem C is false.

44953

https://mypastest.pastest.com/Secure/TestMe/Browser/436619[‫ ص‬05:54:52 10/12/1437]


MyPastest

Next Question

Previous Question Tag Question

Feedback
End Session
Difficulty: Average

Peer Responses

Session Progress

Responses Correct: 0

Responses Incorrect: 5

Responses Total: 5

Responses - % Correct: 0%

Blog
About Pastest
Contact Us
Help

© Pastest 2016

https://mypastest.pastest.com/Secure/TestMe/Browser/436619[‫ ص‬05:54:52 10/12/1437]


MyPastest

Prefer to use the old MyPastest? Access it here »

Back to Filters

Question 6 of 47

A 3-month-old baby presents with persistent discharging umbilicus. The Health Visitor has tried silver nitrate with
no success.
The parents say there is a yellow tinged discharge that sometimes stains the clothes and clear fluid
coming from the umbilicus.

This is most likely to represent?

A Patent Vitelo-intestinal Remnant

B Patent Urachal Sinus

C Umbilical Granuloma

D Patent umbilical vein

E Normal secretions from an umbilical hernia

Explanation
There are five contents that pass through the foetal umbilicus. Two
umbilical arteries coming from the internal
iliac arteries which form the medial ligaments, an umbilical vein which enters the left branch of the portal vein and
becomes the ductus venosus. Both a patent urachus (bladder connection) and vitelo-intestinal duct (attachment to
small bowel) can cause discharge.

The V.I remnant is more likely to represent bowel contents and connects a mid small bowel. It can be seen in 2%
of the population as a Meckel’s diverticulum.

Vitelo-intestinal
remnant although a possibility is more likely to be staining of intestinal contents or pus therefore
A is false.

Clear
fluid discharging from the umbilicus is suspicious of urachal cyst/ remnant. The next step is either to arrange
an ultrasound scan or perform a surgical exploration. Stem B is correct.

Umbilical granuloma should normally resolve with cauterisation eg silver nitrate therefore stem C is false.

https://mypastest.pastest.com/Secure/TestMe/Browser/436619[‫ ص‬05:55:12 10/12/1437]


MyPastest

Umbilical hernias tend not to have secretions therefore stem E is false.


Next Question 44954

Previous Question

End
Tag Session
Question

Feedback

Difficulty: Average

Peer Responses

Session Progress

Responses Correct: 0

Responses Incorrect: 6

Responses Total: 6

Responses - % Correct: 0%

Blog
About Pastest
Contact Us
Help

© Pastest 2016

https://mypastest.pastest.com/Secure/TestMe/Browser/436619[‫ ص‬05:55:12 10/12/1437]


MyPastest

Prefer to use the old MyPastest? Access it here »

Back to Filters

Question 7 of 47

A 13-year-old Afro-Caribbean boy presents with acute abdominal Pain. On examination he has a soft abdomen he
is tachycardia. Investigations demonstrate an Hb of 6 g/dl and he has had fresh red rectal bleeding.

What is the most likely diagnosis?

A Acute Appendicitis

B Sickle Cell Crisis

C Inflammatory Bowel Disease

D Meckels Diverticulum

E Intussusception

Explanation
Fresh red bleeding can be cause by haemorrhoids, polyps or a massive GI bleed. Typically small bowel and
stomach bleeding cause melaena. In Meckels diverticulum, ectopic gastric
mucosa may be present and lead to
erosion of the surrounding enteric mucosa causing massive bleeding enough to reduce haemoglobin levels.

A. False, Appendicitis would show signs of guarding and peritonism clinically.

B. False, there would be no rectal bleeding with a sickle cell crisis.

C.
False, Inflammatory bowel disease may present with bloody stool, but typically there is also mucous, weight
loose and chronicity to the history.

D. True. Fresh red bleeding enough to cause a drop in Hb is most likely to come from a Meckels diverticulum
with ectopic gastric mucosa

E. False, intussusception can cause rectal bleeding if severe but typically this is a ‘red currant Jelly stool’.
Intussusception is most likely to affect children aged 4-12 months
44955

https://mypastest.pastest.com/Secure/TestMe/Browser/436619[‫ ص‬05:55:54 10/12/1437]


MyPastest

Next Question

Previous Question Tag Question

Feedback End Session

Difficulty: Average

Peer Responses

Session Progress

Responses Correct: 0

Responses Incorrect: 7

Responses Total: 7

Responses - % Correct: 0%

Blog
About Pastest
Contact Us
Help

© Pastest 2016

https://mypastest.pastest.com/Secure/TestMe/Browser/436619[‫ ص‬05:55:54 10/12/1437]


MyPastest

Next Question
Prefer to use the old MyPastest? Access it here »

Back to Filters

Question 8 of 47

A 13-year Afro-Caribbean old boy presents with temperature of 37.2oC and acute abdominal pain. He has
previously had a splenectomy for Sickle cell disease. Clinically he is jaundiced.

An ultrasound scan demonstrates a common bile duct diameter of 10mm.

What is the most likely diagnosis?

A Acute Appendicitis

B Cholecystitis

C Impacted Gall Stone

D Sickle Cell Crisis

E Pancreatitis

Explanation
Gall stones in children can be caused by haematological diseases such as sickle cell anaemia and thalassemia.
Cholesterol stones are also becoming more prevalent. A dilated common bile duct (> 10mm in adults) suggests
gall stone impaction. The presence of pyrexia indicates cholecystitis.

Acute appendicitis would not cause jaundice.

44956

https://mypastest.pastest.com/Secure/TestMe/Browser/436619[‫ ص‬05:57:01 10/12/1437]


MyPastest

Tag Question

Feedback

Previous Question Difficulty: Average

Peer Responses End Session

Session Progress

Responses Correct: 0

Responses Incorrect: 8

Responses Total: 8

Responses - % Correct: 0%

Blog
About Pastest
Contact Us
Help

© Pastest 2016

https://mypastest.pastest.com/Secure/TestMe/Browser/436619[‫ ص‬05:57:01 10/12/1437]


MyPastest

Prefer to use the old MyPastest? Access it here »

Back to Filters

Question 9 of 47

A 13-year-old Afro-Caribbean boy presents with acute abdominal pain. He has previously had a splenectomy for
Sickle cell disease. He has a temperature of 38.5oC and is tachycardic.

On
examination he is guarding and has had non-bilious vomiting. An ultrasound scan demonstrates gall stones in
the gall bladder, free fluid
in the pelvis and an appendix with a diameter of 8mm.

What is the most likely diagnosis?

A Acute Appendicitis

B Cholecystitis

C Impacted Gall Stone

D Sickle Cell Crisis

E Gastroenteritis

Explanation
Acute appendicitis presents typically with central abdominal pain migrating to the right iliac fossa anorexia,
nausea and fever. The temperature typically does not go beyond 40oC.
Although appendicitis is a clinical
diagnosis, an ultrasound scan showing an appendix width of greater than 6mm is diagnostic.

Pyrexia, guarding and pain are all in keeping with appendicitis so A is the best answer.

Cholecystitis
is very rare in children and although gall stones are present there is no jaundice or biliary tree
dilatation suggested by the scan so B is false.

There is no mention of common bile duct dilatation so C is false and a sickle cell crisis should not cause guarding,
or an ultrasound scan finding of a dilated appendix base so D is false.

gastroenteritis would not produce the ultrasound scan findings described above with a thickened appendix so E is

https://mypastest.pastest.com/Secure/TestMe/Browser/436619[‫ ص‬05:57:24 10/12/1437]


MyPastest

false.
44957
Next Question

Previous Question

End
Tag Session
Question

Feedback

Difficulty: Average

Peer Responses

Session Progress

Responses Correct: 0

Responses Incorrect: 9

Responses Total: 9

Responses - % Correct: 0%

Blog
About Pastest
Contact Us
Help

© Pastest 2016

https://mypastest.pastest.com/Secure/TestMe/Browser/436619[‫ ص‬05:57:24 10/12/1437]


MyPastest

Prefer to use the old MyPastest? Access it here »

Back to Filters

Question 10 of 47

A septic 3-year-old child has a chest X-ray showing a complete white-out on the left side with midline shift of the
heart to the right side. She is in intensive care and is ventilated.

An ultrasound scan of the chest shows a septated collection measuring 40mm in depth.

What is the most appropriate management of this patient?

A Insertation of chest drain with an underwater seal

B Insertation of pigtail seldinger technique chest drain with connection to collection chamber and
underwater seal.

C Insertation of wide bore chest drain connected to a collection chamber, underwater seal and suction

D Insertion of wide bore chest drain with a collection chamber and underwater seal.

E Repeated aspiration of collection to allow for controlled drainage

Explanation
The correct management of a symptomatic empyema is insertion of a fine chest drain (around 12 french). It is
important not to put the patient on immediate suction as there may be significant fluid shift. There is evidence that
patients treated with small sized chest drains for empyema go home 1-2 days sooner, this is because of the use of
urokinase, the drains tend not to block. This is not true of pneumothorax or chest drains used in trauma.

Stem A is false. There will be a significant amount of fluid from the empyema, if this drains into the chamber with
the underwater seal it will increase the pressure if any air leak is present.

Stem
B is correct because smaller drains are shown to lead to earlier discharge and if suction is commenced
immediately this may lead to a fluid shift.

Stem C is false. Suction may well be needed if there is a bronchopulmonary fistula, however there is no mention
of whether a pneumothorax is present in the question. Controlled
drainage to avoid fluid shift initially is indicated.

https://mypastest.pastest.com/Secure/TestMe/Browser/436619[‫ ص‬05:58:10 10/12/1437]


MyPastest

Smaller drains are used in empyema.


Next Question
D is false because smaller drains lead to earlier discharge in empyema and E is false as this does not allow for
treatment with urokinase or continued drainage.
44959

Previous Question

End Session

Tag Question

Feedback

Difficulty: Average

Peer Responses

Session Progress

Responses Correct: 0

Responses Incorrect: 10

Responses Total: 10

Responses - % Correct: 0%

Blog
About Pastest
Contact Us
Help

© Pastest 2016

https://mypastest.pastest.com/Secure/TestMe/Browser/436619[‫ ص‬05:58:10 10/12/1437]


MyPastest

Prefer to use the old MyPastest? Access it here »

Back to Filters

Question 11 of 47

A 3-day-old 3.5kg term baby presents having not stooled since


birth. He has bilious aspirates on naso-gastric
decompression and a distended abdomen with visible loops. A contrast shows contracted rectum
going into dilated
upper sigmoid colon.

Which of the following is the most likely diagnosis?

A Meconium Ileus

B Anorectal Malformation

C Malrotaion and midgut volvulus

D Hirschsprung Disease

E Meconium Plug

Explanation
The presentation of a distended abdomen is more typical of distal obstruction in a neonate. The main differentials
here are Hirschsprung disease, meconium ileus, meconium plug, small left colon (associated with maternal
diabetes) and anorectal
malformation. Nearly all patients with Hirschsprung disease have delayed passage of
meconium beyond 24 hours, however this is only true of around 1% of normal term babies.

A contrast enema is the logical next step to aid diagnosis. If a transition zone of
collapsed to dilated bowel is seen
this is classic for Hirschsprungs and is present in 25% of Hirschsprungs patients.

Diagnosis is by rectal suction biopsy to look for the absence of ganglion cells.

Meconium
ileus is false because the transition for meconium ileus is in the terminal ileum from collapsed to
dilated bowel. Anorectal malformation is false as there would be either an anterior stenotic anus or absent anus or
rectal atresia. Malrotation and midgut volvulus is false as the picture is typical of distal bowel obstruction and
meconium plug is false because filling defects would be seen throughout the colon on contrast enema, and it may

https://mypastest.pastest.com/Secure/TestMe/Browser/436619[‫ ص‬05:58:32 10/12/1437]


MyPastest

well be therapeutic in nature.


Next Question
44960

Previous Question

End Session
Tag Question

Feedback

Difficulty: Average

Peer Responses

Session Progress

Responses Correct: 0

Responses Incorrect: 11

Responses Total: 11

Responses - % Correct: 0%

Blog
About Pastest
Contact Us
Help

© Pastest 2016

https://mypastest.pastest.com/Secure/TestMe/Browser/436619[‫ ص‬05:58:32 10/12/1437]


MyPastest

Prefer to use the old MyPastest? Access it here »

Back to Filters

Question 12 of 47

A 2-day-old baby boy returns to hospital having not opened his bowels. On examination there is no visible anus
and meconium is seen
coming from the urethra.

Which of the following statements is most likely to be correct?

A This is an anorectal malformation with a recto-urethral fistula. Primary anorectoplasty as first


procedure is indicated

B This is a low anorectal malformation which is decompressible. Perineal cutback indicated for urgent
decompression

C This is rectal atresia, a defunctioning colostomy is needed

D This is an anorectal malformation with recto-urethral fistula and urgent micturating cystourethrogram
needed

E This is an anorectal malformation with a recto urethral fistula, urgent defunctioning colostomy needed

Explanation
This is an anorectal malformation with a recto-urethral or vesical fistula. It is not possible to do a primary
procedure without first defunctioning colostomies, followed by a micturating cystourethrogram to accurately
locate the site of the fistula to allow surgical approach to be planned.

Primary
anoplasty is possible in babies with beads of meconium at the anus, radiological or ultrasonography
evidence of anus within 2cm of perineum,
or there is a perineal fistula.

Answer stem A is false because primary anorectoplasty should be done in 3-6 months to allow the child to grow
and reduce the risk of damage to existing continence anatomy.

Answer stem B. is false because there is meconium coming from the urethra so by definition this is a high lesion.

Answer stem C is false as there is no anus visible, and you would expect to see the anus in rectal atresia.

https://mypastest.pastest.com/Secure/TestMe/Browser/436619[‫ ص‬05:58:54 10/12/1437]


MyPastest

Although
a micturating cystourethrogram will be required, this can be done on an
outpatient basis. A colostomy is
what is required urgently. Next Question
An urgent colostomy followed by staged planned surgery in the next 3-6 months is what is required.
44961
Previous Question

End Session

Tag Question

Feedback

Difficulty: Average

Peer Responses

Session Progress

Responses Correct: 0

Responses Incorrect: 12

Responses Total: 12

Responses - % Correct: 0%

Blog
About Pastest
Contact Us
Help

© Pastest 2016

https://mypastest.pastest.com/Secure/TestMe/Browser/436619[‫ ص‬05:58:54 10/12/1437]


MyPastest

Prefer to use the old MyPastest? Access it here »

Back to Filters

Question 13 of 47

A 4-day-old baby of Caucasian parents presents to the emergency department with green vomit following a bottle
feed. A plain abdominal X-ray shows a large stomach bubble but a paucity of gas distally.

Mother had group B streptococcus positive vaginal swabs. There is a history of meconium stained liquor.

Which of the following statements best describes your management plan?

A Treat the child with first line antibiotics, place a nasogastric tube and admit for observation

B Observe the child on the ward. If feeding well and no inflammatory markers, discharge with
reassurance

C Arrange for an urgent upper GI contrast study to rule out malrotation, cover sepsis with antibiotics

D Arrange for a lower GI contrast study to rule out meconium ileus and treat cover sepsis with
antibiotics.

E Cover sepsis and arrange for a suction rectal biopsy to rule out Hirschsprungs disease.

Explanation
The most important time critical diagnosis to rule out in this patient is a small bowel malrotation volvulus. An
urgent upper GI contrast study must be performed to assess for normal position of the DJ flexure to the left of the
transverse process of L1 at the height of the pylorus. It is important to cover with antibiotics due to the history of
group be streptococcus.

Answer stems A and B are false because this child has potentially life threatening malrotation volvulus, a time
critical diagnosis to rule out.

Answer
stems D and E are false because this is a picture (or description) of an upper, not lower, GI obstruction
and malrotation MUST be ruled out.

https://mypastest.pastest.com/Secure/TestMe/Browser/436619[‫ ص‬05:59:33 10/12/1437]


MyPastest

44962

Next Question

Previous Question
Tag Question
End Session
Feedback

Difficulty: Average

Peer Responses

Session Progress

Responses Correct: 0

Responses Incorrect: 13

Responses Total: 13

Responses - % Correct: 0%

Blog
About Pastest
Contact Us
Help

© Pastest 2016

https://mypastest.pastest.com/Secure/TestMe/Browser/436619[‫ ص‬05:59:33 10/12/1437]


MyPastest

Next Question
Prefer to use the old MyPastest? Access it here »

Back to Filters

Question 14 of 47

A 3-day-old Down syndrome baby has a bilious vomit on the ward. On examination the baby has a grossly
distended abdomen with visible bowel loops.

Which diagnosis, associated with Down syndrome, is most likely?

A Duodenal web/atresia

B Hirschsprungs disease

C Meconium plug syndrome

D Meconium ileus

E Malrotation volvulus

Explanation
Hirschsprung disease is most likely. This
is a picture of distal obstruction with visible bowel loops and a grossly
distended abdomen. Malrotation or duodenal pathologies are a high obstruction and would not be distended.

Duodenal
web/atresia is false because although this is associated with Down syndrome with 1/3 of duodenal
atresia present in trisomy 21, this would not present with a distended abdomen, which suggests a distal
obstruction.

Meconium plug syndrome is not associated with Down syndrome children.

Meconium ileus is associated with cystic fibrosis not Down syndrome and malrotation volvulus is a picture of
distal not proximal obstruction.
44963

https://mypastest.pastest.com/Secure/TestMe/Browser/436619[‫ ص‬05:59:54 10/12/1437]


MyPastest

Tag Question

Feedback

Previous Question
Difficulty: Average

End Session
Peer Responses

Session Progress

Responses Correct: 0

Responses Incorrect: 14

Responses Total: 14

Responses - % Correct: 0%

Blog
About Pastest
Contact Us
Help

© Pastest 2016

https://mypastest.pastest.com/Secure/TestMe/Browser/436619[‫ ص‬05:59:54 10/12/1437]


MyPastest

Prefer to use the old MyPastest? Access it here »

Back to Filters

Question 15 of 47

A 3-day-old 4.2kg baby boy of Caucasian parents presents with


a distended abdomen, not opening bowels since
birth, and bilious aspirates on nasogastric tube insertion.

A contrast enema demonstrates microcolon with filling defects throughout. The contrast refluxes into dilated
ileum.

What is the most likely diagnosis?

A Meconium ileus

B Hirschsprungs Disease

C Meconium plug

D Duplication cyst

E Malrotation volvulus

Explanation
Meconium ileus is the correct diagnosis. The contrast refluxes into dilated ileum suggesting this is the site of
obstruction. The filling defects are also typical. Contrast enema maybe therapeutic as well as diagnostic.
Meconium Ileus is diagnostic for Cystic Fibrosis in 95% of cases.

This is a typical clinical picture of meconium ileus. The contrast enema maybe therapeutic but if not, a
laparotomy, enterotomy and distal washout is required.

In HIrschsprung disease the contrast enema would show a transition zone typically in the sigmoid colon.

A meconium plug tends not to have a microcolon and a duplication cyst would show an isolated filling defect.

In Malrotation volvulus, contrast enema may be normal or show an abnormally sited caecum.
44964

https://mypastest.pastest.com/Secure/TestMe/Browser/436619[‫ ص‬06:00:33 10/12/1437]


MyPastest

Next Question

Previous Question Tag Question

Feedback
End Session
Difficulty: Average

Peer Responses

Session Progress

Responses Correct: 0

Responses Incorrect: 15

Responses Total: 15

Responses - % Correct: 0%

Blog
About Pastest
Contact Us
Help

© Pastest 2016

https://mypastest.pastest.com/Secure/TestMe/Browser/436619[‫ ص‬06:00:33 10/12/1437]


MyPastest

Prefer to use the old MyPastest? Access it here »

Back to Filters

Question 16 of 47

A 2-week old Down syndrome baby presents with bilious vomiting and failure to thrive. An upper GI contrast
enema shows a dilated duodenum and a duodenal-jejunal flexure to the right of the transverse process of L1
vertebrae.

What is the most likely diagnosis?

A Duodenal web associated with Down Syndrome

B Pancreas divisum

C Hirschsprungs disease

D Duodenal duplication cyst

E Intestinal malrotation

Explanation
Although the history would lead you to suspect duodenal pathology, the upper GI contrast is diagnostic of
malrotation. The DJ flexure should be to the LEFT of the transverse process of L1 vertebrae at the height of the
pylorus. Any position other
than this depicts malrotation.

Although the history is suggestive of duodenal web associated with Down syndrome or Hirschprungs, the contrast
findings are diagnostic of malrotation.

Pancreas divisum would show a hold up or obstruction at the second part of the duodenum.

Duodenal duplication cyst would be seen well by cross-sectional imaging if the duodenum showed evidence of
external compression.

44965

https://mypastest.pastest.com/Secure/TestMe/Browser/436619[‫ ص‬06:00:54 10/12/1437]


MyPastest

Next Question

Previous Question Tag Question

Feedback
End Session
Difficulty: Average

Peer Responses

Session Progress

Responses Correct: 0

Responses Incorrect: 16

Responses Total: 16

Responses - % Correct: 0%

Blog
About Pastest
Contact Us
Help

© Pastest 2016

https://mypastest.pastest.com/Secure/TestMe/Browser/436619[‫ ص‬06:00:54 10/12/1437]


MyPastest

Next Question
Prefer to use the old MyPastest? Access it here »

Previous Question
Back to Filters

Question 17 of 47

A new-born baby with confirmed trisomy 21 of prenatal Harmony


blood test presents with bilious vomiting and an
abdominal X-ray at 24-hours of age showing a ‘double bubble’ appearance.

What is the most likely diagnosis?

A Hirschsprungs Disease

B Malrotation volvulus

C Ileal atresia

D Duodenal Atresia

E Meconium Ileus & Cystic fibrosis

Explanation
A double bubble appearance on abdominal X-ray is a classic diagnostic for duodenal atresia. Gas should reach the
stomach almost immediately after birth, the ileo-caecal valve by 6 hours and rectum by 24 hours.

Answer stem A is false as this is a picture of proximal not distal obstruction. B is false as there would be some gas
distally as well as a double bubble and C is false as there would be more visible loops.

Answer stem E is false as this would show a picture of multiple dilated loops.
44966

Tag Question

https://mypastest.pastest.com/Secure/TestMe/Browser/436619[‫ ص‬06:01:24 10/12/1437]


MyPastest

Feedback

Difficulty: Average

Peer Responses

End Session

Session Progress

Responses Correct: 0

Responses Incorrect: 17

Responses Total: 17

Responses - % Correct: 0%

Blog
About Pastest
Contact Us
Help

© Pastest 2016

https://mypastest.pastest.com/Secure/TestMe/Browser/436619[‫ ص‬06:01:24 10/12/1437]


MyPastest

Next Question
Prefer to use the old MyPastest? Access it here »

Previous Question
Back to Filters
End Session
Question 18 of 47

What percentage of infants born with meconium Ileus have cystic fibrosis?

A 0-20%

B 20-40%

C 40-60%

D 60-90%

E >90%

Explanation
Meconium Ileus is associated with cystic fibrosis in over 95% of cases. It does not predict the clinical course of
cystic fibrosis though. About 15% of new-born cystic fibrosis patients present with meconium ileus.

44967

Tag Question

Feedback

Difficulty: Average

Peer Responses

https://mypastest.pastest.com/Secure/TestMe/Browser/436619[‫ ص‬06:01:46 10/12/1437]


MyPastest

Session Progress

Responses Correct: 0

Responses Incorrect: 18

Responses Total: 18

Responses - % Correct: 0%

Blog
About Pastest
Contact Us
Help

© Pastest 2016

https://mypastest.pastest.com/Secure/TestMe/Browser/436619[‫ ص‬06:01:46 10/12/1437]


MyPastest

Prefer to use the old MyPastest? Access it here »

Back to Filters

Question 19 of 47

A 15-year old girl presents with Right Iliac fossae discomfort and a palpable 6cm mass. She is apyrexic and
inflammatory markers are normal. An abdominal X-ray shows what appears to be a tooth in the right lower
quadrant.

What is the most likely diagnosis?

A Appendix mass with a faecolith

B Ectopic pregnancy

C Ovarian teratoma

D Tortid ovarian cyst

E Faecal loading

Explanation
The most likely diagnosis is an ovarian teratoma with a tooth within the tumour. The girl needs routine tumour
marker aFP, bHCG and oophorectomy. The majority of these lesions are benign.

Answer stem A is False because although this may show calcification, obstructive or constitutional symptoms
would be expected.

An
ectopic pregnancy would present with pain, and shock with or without vaginal bleeding. It is important to
perform a pregnancy test in all girls older than 12 years being assessed for abdominal pain.

A tortid ovarian cyst would present with pain and peritonims and faecal loading, although very common, does not
explain the presence of the tooth.

Ovarian teratoma is the most likely diagnosis; arising from all three cell layers any tissue may be present.
44968

https://mypastest.pastest.com/Secure/TestMe/Browser/436619[‫ ص‬06:02:18 10/12/1437]


MyPastest

Next Question

Previous Question Tag Question

Feedback
End Session
Difficulty: Average

Peer Responses

Session Progress

Responses Correct: 0

Responses Incorrect: 19

Responses Total: 19

Responses - % Correct: 0%

Blog
About Pastest
Contact Us
Help

© Pastest 2016

https://mypastest.pastest.com/Secure/TestMe/Browser/436619[‫ ص‬06:02:18 10/12/1437]


MyPastest

Prefer to use the old MyPastest? Access it here »

Back to Filters

Question 20 of 47

A 14-year-old girl presents with mild abdominal pain in the left lower quadrant for the last 3 months. An
ultrasound scan demonstrates a 7cm simple ovarian cyst.

What is the most appropriate course of action?

A Referral to an oncologist for urgent cross-sectional imaging and staging of likely malignancy

B Functional ovarian cyst. Ovary sparing cystectomy

C Open Oophorectomy

D Cyst aspiration and oophorectomy

E Conservative management

Explanation
The most prudent course of action is an ovary sparing cystectomy. The history is 3-months and is already chronic,
furthermore the cyst is greater than 5cm in size and at risk of
torsion. This will relieve the cause of pain, reduce
the risk of torsion and save ovarian function.

Answer stem A is false as this is a simple cyst and not suggestive of malignancy so imaging and referral are not
indicated.

Open oophorectomy was done in the past howeer this is very aggressive and the modern approach is ovary
sparing.

As the cyst is 6cm and at risk of torsion conservative management is not appropriate.

44969

https://mypastest.pastest.com/Secure/TestMe/Browser/436619[‫ ص‬06:02:43 10/12/1437]


MyPastest

Next Question

Previous Question Tag Question

Feedback
End Session
Difficulty: Average

Peer Responses

Session Progress

Responses Correct: 0

Responses Incorrect: 20

Responses Total: 20

Responses - % Correct: 0%

Blog
About Pastest
Contact Us
Help

© Pastest 2016

https://mypastest.pastest.com/Secure/TestMe/Browser/436619[‫ ص‬06:02:43 10/12/1437]


MyPastest

Next Question
Prefer to use the old MyPastest? Access it here »

Back to Filters

Question 21 of 47

A 3-month-old baby boy presents with a sudden history of a lump in the right groin extending to the scrotum
which transilluminates on examination.

What is the most likely diagnosis?

A Indirect inguinal hernia

B Direct inguinal hernia

C Encysted hydrocele of the cord

D Communicating hydrocele

E Undescended testis

Explanation
This is an inguinal hernia. Hernias are reducible, you cannot get above them, the rule that hydroceles
transilluminate is true in adults, but in neonates hernias do too and it
is not a reassuring sign. If in doubt an
ultrasound scan may be useful.

Answer stem B is false because inguinal hernias in children are nearly always indirect.

Encysted hydrocele of the cord would present with another discrete lump palpable above the testis.

Answer stem D is false, as the history would be longer in a communicating hydrocele and E is false as the lump
would not extend to the groin but be able to be 'milked' into the scrotum.
44970

https://mypastest.pastest.com/Secure/TestMe/Browser/436619[‫ ص‬06:03:14 10/12/1437]


MyPastest

Tag Question

Feedback

Previous Question Difficulty: Average

Peer Responses End Session

Session Progress

Responses Correct: 0

Responses Incorrect: 21

Responses Total: 21

Responses - % Correct: 0%

Blog
About Pastest
Contact Us
Help

© Pastest 2016

https://mypastest.pastest.com/Secure/TestMe/Browser/436619[‫ ص‬06:03:14 10/12/1437]


MyPastest

Next Question
Prefer to use the old MyPastest? Access it here »

Back to Filters

Question 22 of 47

A 4-month-old baby presents with a swollen right scrotum. On examination it is non-tender and transilluminates.
You are able to get above the swelling.

What would be the next step in the management of this condition?

A Arrange for repair of the inguinal hernia on the next elective operating list

B Urgent scrotal exploration for acutely swollen testis

C Review in clinic in 1 year and proceed to ligation of patent processus vaginals if still present

D Arrange for an ultrasound scan for definitive diagnosis

E Discharge with reassurance

Explanation
The diagnosis is a hydrocele. These are common in new-born babies and 95% resolve by the age of 2 years. If still
present at this stage a hydrocele repair by ligating the processus
vaginalis is performed. The parents should be
advised to seek further medical attention if the condition is still present at the child's 2nd birthday.

Answer stem A is false as the diagnosis is of a hydrocele as you are able to get above the swelling.

Answer stem B is false - you have been told that the testis is non-tender.

Answer
stem C is false - 95% of hydroceles resolve spontaneously. It is not necessary to see them all back in
clinic and D is false as the diagnosis is obviously a hydrocele - no ultrasound is required.

44971

https://mypastest.pastest.com/Secure/TestMe/Browser/436619[‫ ص‬06:03:45 10/12/1437]


MyPastest

Tag Question

Feedback
Previous Question
Difficulty: Average
End Session
Peer Responses

Session Progress

Responses Correct: 0

Responses Incorrect: 22

Responses Total: 22

Responses - % Correct: 0%

Blog
About Pastest
Contact Us
Help

© Pastest 2016

https://mypastest.pastest.com/Secure/TestMe/Browser/436619[‫ ص‬06:03:45 10/12/1437]


MyPastest

Prefer to use the old MyPastest? Access it here »

Back to Filters

Question 23 of 47

A 9-year-old boy is seen who is unable to retract his previously retractile foreskin. On examination the foreskin is
white and
thickened. He has been treated for 3 months with 1% hydrocortisone.

What would be the next step in the management of this condition?

A Treatment for further 3-months with 1% hydrocortisone cream

B Dorsal slit under general anaesthesia

C Treatment with systemic broad spectrum antibiotics

D Circumcision with urethral calibration

E Circumcision

Explanation
The diagnosis here is balanitis xerotica obliterans (BXO) and is the only true medical indication for circumcision.
These boys can get meatal stenosis in 10% of cases and therefore it is important to perform urethral calibration.

BXO
will not usually respond to steroids, although for some clinicians this is first line treatment. If there is no
response to steroids after
3 months then a circumcision is indicated so A is false.

The scarring will continue even with a dorsal slit. The whole foreskin needs to be removed so B is false.

Although
a dorsal slit might settle an acute flare up of balanitis, this case is
chronically scarred and a circumcision
is required.

E is false because although a circumcision is required, in 10% of cases the urethra can be narrow and calibration is
important.
44972

https://mypastest.pastest.com/Secure/TestMe/Browser/436619[‫ ص‬06:04:06 10/12/1437]


MyPastest

Next Question

Previous Question Tag Question

Feedback
End Session
Difficulty: Average

Peer Responses

Session Progress

Responses Correct: 0

Responses Incorrect: 23

Responses Total: 23

Responses - % Correct: 0%

Blog
About Pastest
Contact Us
Help

© Pastest 2016

https://mypastest.pastest.com/Secure/TestMe/Browser/436619[‫ ص‬06:04:06 10/12/1437]


MyPastest

Next Question
Prefer to use the old MyPastest? Access it here »

Previous Question
Back to Filters

Question 24 of 47

An 18-month-old girl is seen with a 0.5cm firm/cystic lump on


the lateral aspect of her eyebrow. It has been
present since birth but is slowly enlarging.

What is the most likely diagnosis?

A Branchial remnant

B Sebaceous cyst

C External angular dermoid

D Vascular malformation

E Lymphatic malformation

Explanation
This is an external angular dermoid, these typically form where there are overlapping tissue planes such as the
midline. They contain a caseous material. The cysts are excised for cosmetic purposes or when they encroach on
vision but are benign in nature. The case described is in the classic position.

Branchial remnant typically presents as a supraclavicular pit and sebaceous cysts tend to be present on the scalp.

There
is nothing in the history to suggest vascular involvement such as pigment change or ‘bag of worms’ feel so
D is false and lymphatic malformations are typically found in the neck or under the axilla so E is false.
44973

Tag Question

https://mypastest.pastest.com/Secure/TestMe/Browser/436619[‫ ص‬06:04:29 10/12/1437]


MyPastest

Feedback

Difficulty: Average

Peer Responses

End Session

Session Progress

Responses Correct: 0

Responses Incorrect: 24

Responses Total: 24

Responses - % Correct: 0%

Blog
About Pastest
Contact Us
Help

© Pastest 2016

https://mypastest.pastest.com/Secure/TestMe/Browser/436619[‫ ص‬06:04:29 10/12/1437]


MyPastest

Prefer to use the old MyPastest? Access it here »

Back to Filters

Question 25 of 47

A 6-year-old girl has a midline neck cyst which moves on tongue protrusion.

What is the most likely next step in the management of this condition?

A Needs surgical excision on next routine elective list

B Needs an ultrasound scan to look for normal thyroid gland

C Needs an ultrasound scan to look for suprahyoid extension for the tract

D Needs treatment with prophylactic antibiotics for a week prior to surgery

E Review in clinic in 1 year to monitor progression, likely to resolve spontaneously

Explanation
This is a thyroglossal cyst. The most important information to gather prior to surgery is to determine whether
there
is a normal thyroid gland and that this does not represent the soul functioning thyroid tissue of this patient. The
operation involves taking the central portion of the hyoid bone to reduces the chances of recurrence.

Answer stem A is false, this is dangerous without ensuring there is a normal thyroid gland on ultrasound
scan as
there is a potential to render the patient thyroid.

Answer stem B is t rue, this is essential.

Answer
stem C is false, an ultrasound scan is needed to look for normal thyroid gland, but the extent of the tract
can be determined at surgical
exploration.

Answer stem D is false, although operating during infection increases the chances of recurrence.

Answer stem E is false - this will not resolve without surgery.


44974

https://mypastest.pastest.com/Secure/TestMe/Browser/436619[‫ ص‬06:04:49 10/12/1437]


MyPastest

Next Question

Previous Question Tag Question

Feedback
End Session
Difficulty: Average

Peer Responses

Session Progress

Responses Correct: 0

Responses Incorrect: 25

Responses Total: 25

Responses - % Correct: 0%

Blog
About Pastest
Contact Us
Help

© Pastest 2016

https://mypastest.pastest.com/Secure/TestMe/Browser/436619[‫ ص‬06:04:49 10/12/1437]


MyPastest

Next Question
Prefer to use the old MyPastest? Access it here »

Back to Filters

Question 26 of 47

On a 6-week baby check an umbilical hernia is noted.

What advice would you give the parents?

A A routine referral to paediatric surgeons is required

B At risk incarceration, urgent referral to on-call paediatric surgeons

C Tape a 50 pence coin over the defect to prevent incarceration

D This is likely to resolve in 90% of cases by the age of 4 years. Repair is performed is still present

E Umbilical hernias in girls should not be repaired as the defect can give way, should they come
pregnant

Explanation
Umbilical hernias are common and over 90%
resolve by the age of 4 years. If they have not resolved than this is
an acceptable age to operate as the chance of recurrence is much lower. Some Afro-Caribbean children have large
defects, but again, if possible a
repair after 3 years of age results in a lower chance of recurrence.

Answer stem A is false, the majority of these resolve, they should be discharged with reassurance.

Answer stem B is false, inguinal hernias are at risk of incarceration not umbilical hernias.

Answer stem C is false, although this is done is some rural parts of Africa it doesn’t aid resolution.

Answer stem D is true, repair at an older age is also associated with a lower recurrence.

Answer stem E false, this is an indication to perform in girls to prevent an unsightly hernia during pregnancy.
44975

https://mypastest.pastest.com/Secure/TestMe/Browser/436619[‫ ص‬06:05:30 10/12/1437]


MyPastest

Tag Question

Feedback
Previous Question
Difficulty: Average
End Session
Peer Responses

Session Progress

Responses Correct: 0

Responses Incorrect: 26

Responses Total: 26

Responses - % Correct: 0%

Blog
About Pastest
Contact Us
Help

© Pastest 2016

https://mypastest.pastest.com/Secure/TestMe/Browser/436619[‫ ص‬06:05:30 10/12/1437]


MyPastest

Prefer to use the old MyPastest? Access it here »

Back to Filters

Question 27 of 47

An 8-year-old girl was taken to her GP 5 days ago with lower abdominal pain and leukocytes 3+ on urine dipstick.
She was treated with
trimethoprim. She now has low-grade pyrexia a CRP of 254 and a palpable
mass in the right
iliac fossa. She is able to eat and drink.

What is the most likely diagnosis and management (where appropriate)?

A This is acute appendicitis, commence broad spectrum antibiotics and arrange appendicectomy

B This is an appendix abscess and drainage via interventional radiology is advised

C This is mass palpable kidney secondary to obstructive pyelonephritis

D This is an appendix mass and intravenous antibiotics where observation is the initial treatment

E Suspected Crohn's stricture, arrange for laparotomy and ileo-colic resection

Explanation
This is an appendix mass. The correct management is broad-spectrum intravenous antibiotics such as co-
amoxyclav and amikacin plus observation. The child should be allowed to eat and drink. If there are on-going
temperature spikes, signs of obstruction or severe colicky abdominal pain, than surgery is required. The majority
of patients respond to conservative management and then a discussion about the pros and cons of interval
appendicetomy can take place.

Answer stem A false, this is a partially treated appendicitis which has formed an appendix mass.

Answer stem B is alse, she would have a swinging pyrexia with a collection.

Answer stem C is false, she would have high-grade pyrexia and be constitutionally unwell if this were the case.

Answer stem D ist rue, conservative management is the preferred treatment as surgical exploration at this stage
can result in increased morbidity.

https://mypastest.pastest.com/Secure/TestMe/Browser/436619[‫ ص‬06:05:52 10/12/1437]


MyPastest

Answer stem E is false, the history would be longer with symptoms of weight loss and altered bowel habit.
44976
Next Question

Previous Question

End
Tag Session
Question

Feedback

Difficulty: Average

Peer Responses

Session Progress

Responses Correct: 0

Responses Incorrect: 27

Responses Total: 27

Responses - % Correct: 0%

Blog
About Pastest
Contact Us
Help

© Pastest 2016

https://mypastest.pastest.com/Secure/TestMe/Browser/436619[‫ ص‬06:05:52 10/12/1437]


MyPastest

Next Question
Prefer to use the old MyPastest? Access it here »

Previous Question
Back to Filters
End Session
Question 28 of 47

An 11-year-old boy has been conservatively managed for an appendix mass. The parents want to know whether to
have an interval appendicectomy done to prevent the chances of having appendicitis again.

What percentage of patients with conservatively managed appendix mass are likely to recurrent appendicitis?

A 0-20%

B 20-40%

C 40-60%

D 60-80%

E 80-100%

Explanation
The chance of having appendicitis again after appendix mass is around 17% in children. There remains
controversy
whether to perform an interval appendicectomy or not.

44977

Tag Question

Feedback

https://mypastest.pastest.com/Secure/TestMe/Browser/436619[‫ ص‬06:06:12 10/12/1437]


MyPastest

Difficulty: Average

Peer Responses

Session Progress

Responses Correct: 0

Responses Incorrect: 28

Responses Total: 28

Responses - % Correct: 0%

Blog
About Pastest
Contact Us
Help

© Pastest 2016

https://mypastest.pastest.com/Secure/TestMe/Browser/436619[‫ ص‬06:06:12 10/12/1437]


MyPastest

Prefer to use the old MyPastest? Access it here »

Back to Filters

Question 29 of 47

Four days after a laparoscopic appendicectomy for a perforated appendix, you are asked to see a 5-year-old boy
who has had repeated temperature spikes of 38.5 oC throughout the day. He has opened his bowels with diarrhoea.
His chest is clear on examination.

What is the most likely cause of his condition?

A Urinary Tract Infection

B Intra-abdominal collection

C Bacterial resistance to current antibiotic therapy

D Lower lobe pneumonia

E Wound infection

Explanation
Perforated appendicitis is more common in
the paediatric population and can be present in up to 50% of cases.
This is due to poor localisation of abdominal pain, inability to express
their discomfort and the difficulty
surrounding surgical assessment of an unwell child. About a third of perforated appendicitis can go on to develop
a complication. The most common are wound infections, intra-abdominal collections and intra-abdominal
adhesions. Laparoscopic appendicectomy has a higher rate of intra-abdominal collection than an open
appendicetomy, which has increased rate of intra-abdominal adhesions and wound infections.

Answer stem A is false, this is unlikely as the antibiotics used to treat appendicitis should cover organism causing
UTIs.

Answer stem C is false,


although this may be the case MC&S from the initial operation would
have been back at
48 hours and antibiotics changed accordingly. Most units are aware of local sensitivities and organisms and have a
protocol
for antibiotics to take this into account.
44978

https://mypastest.pastest.com/Secure/TestMe/Browser/436619[‫ ص‬06:07:02 10/12/1437]


MyPastest

Next Question

Previous Question Tag Question

Feedback
End Session

Difficulty: Average

Peer Responses

Session Progress

Responses Correct: 0

Responses Incorrect: 29

Responses Total: 29

Responses - % Correct: 0%

Blog
About Pastest
Contact Us
Help

© Pastest 2016

https://mypastest.pastest.com/Secure/TestMe/Browser/436619[‫ ص‬06:07:02 10/12/1437]


MyPastest

Prefer to use the old MyPastest? Access it here »

Back to Filters

Question 30 of 47

A new-born baby has an abdominal wall defect diagnosed antenatally. When admitted to the neonatal unit, there is
a sac covering
a 3cm defect with what appears to be intestine; no liver is visible. The baby has no dysmorphic
features other than a large tongue.

What is the immediate danger with this baby?

A Beckwith-Wiedermann syndrome, risk of hypoglycaemia

B Trisomy 21, risk of cardiac defect

C Trisomy 18, risk of cardiac defect

D Pierre Robin syndrome, risk to airway

E Tracheomalacia, risk to airway

Explanation
This is exomphalos minor. Exomphalos major is defined as a defect > 5cm or with liver within the sac.
Exomphalos minor is associated with Beckwith-Wiedermann syndrome. This commonly presents with a triad of
exomphalos minor, large protruding tongue and hypoglycaemia. It is this, which is the immediate concern in this
child.

Answer stem A is true this is associated with exomphalos minor and there is a risk of hypoglycaemia.

Answer stem B is false, although associated with exomphalos, there are no dysmorphic features on this baby
making this unlikely.

Answer stem C is false, although associated with exomphalos, these infants rarely make it to term and this baby
has no dysmorphic features.

Answer stem D is false, this is associated with a small jaw and a tongue which slips back and occludes the airway.

https://mypastest.pastest.com/Secure/TestMe/Browser/436619[‫ ص‬06:07:24 10/12/1437]


MyPastest

Answer stem E is false, tracheomalacia is associated with oesophageal atresia.


44979
Next Question

Previous Question

End
Tag Session
Question

Feedback

Difficulty: Average

Peer Responses

Session Progress

Responses Correct: 0

Responses Incorrect: 30

Responses Total: 30

Responses - % Correct: 0%

Blog
About Pastest
Contact Us
Help

© Pastest 2016

https://mypastest.pastest.com/Secure/TestMe/Browser/436619[‫ ص‬06:07:24 10/12/1437]


MyPastest

Next Question
Prefer to use the old MyPastest? Access it here »

Previous Question
Back to Filters
End Session
Question 31 of 47

You are called to a delivery of a new-born baby. There is a defect to the right of the umbilicus with bowel
protruding through the defect. On first visit to the neonatal unit the parents ask when they will be able to fully
breastfeed their baby?

What advice would you give them?

A 0-7 days of age

B 7-14 days of age

C 14-21 days of age

D 21-28 days of age

E 28-35 days of age

Explanation
This is gastroschisis, this tends to be an isolated defect associated with young mothers. The median time to full
feeds is between 24 to 28 days in most large series.

44980

Tag Question

Feedback

https://mypastest.pastest.com/Secure/TestMe/Browser/436619[‫ ص‬06:07:43 10/12/1437]


MyPastest

Difficulty: Average

Peer Responses

Session Progress

Responses Correct: 0

Responses Incorrect: 31

Responses Total: 31

Responses - % Correct: 0%

Blog
About Pastest
Contact Us
Help

© Pastest 2016

https://mypastest.pastest.com/Secure/TestMe/Browser/436619[‫ ص‬06:07:43 10/12/1437]


MyPastest

Prefer to use the old MyPastest? Access it here »

Back to Filters

Question 32 of 47

An 8-year-old girl presents with 1 day history of bilious vomiting and abdominal distension. Her inflammatory
markers are normal. Mother tells you she had an operation at 3 days of age for malrotation. An abdominal X-ray
shows several severely dilated loops of small bowel with no gas in the rectum.

What is the likely diagnosis and initial management?

A Recurrent volvulus, urgent laparotomy required

B Intra-abdominal adhesions, surgery for division of intra-abdominal adhesions

C Intra-abdominal adhesions, naso-gastric decompression and intravenous fluids

D Gastroenteritis with incompetent pylorus, intravenous fluids

E Likely sepis with bilious vomiting, intravenous fluids and antibiotics

Explanation
Malrotation typically presents in the first month of life with bilious vomiting. There is a lifetime risk of intra-
abdominal adhesions. This presents with bilious vomiting, dilated bowel loops on plain abdominal film. 2/3 of
adhesional obstruction resolves by conservative management with nasogastric decompression and intravenous
fluids. If this fails to resolve after 24-48 hours or if there are signs of peritonism a laparotomy is indicated.

Answer stem A is false, the child is now 8-years-old and recurrent volvulus is unlikely as bowel should be fixed
by intra-abdominal adhesions.

Answer stem B isf alse, although surgery may be required the majority of adhesional obstructive cases respond to
conservative management

Answer stem C is true, the majority of adhesional obstructions respond with conservative management.

Answer stem D is false,


it is true that patients with malrotation can have bilious vomiting in gastroenteritis, the
abdominal X-ray is suggestive of obstruction.

https://mypastest.pastest.com/Secure/TestMe/Browser/436619[‫ ص‬06:08:04 10/12/1437]


MyPastest

Answer stem E is false, whilst a septic ileus can give bilious vomiting, this patient has no inflammatory markers
suggestive of sepsis. Next Question
44981

Previous Question

End Session
Tag Question

Feedback

Difficulty: Average

Peer Responses

Session Progress

Responses Correct: 0

Responses Incorrect: 32

Responses Total: 32

Responses - % Correct: 0%

Blog
About Pastest
Contact Us
Help

© Pastest 2016

https://mypastest.pastest.com/Secure/TestMe/Browser/436619[‫ ص‬06:08:04 10/12/1437]


MyPastest

Prefer to use the old MyPastest? Access it here »

Back to Filters

Question 33 of 47

A horse kicks a 12-year-old girl in the abdomen, whilst she is at riding camp. A CT scan demonstrates a grade IV
splenic injury. The
child has a pulse of 110 bpm with a blood pressure of 110/70.

What is the most likely management of this child?

A Transfer the interventional radiology for embolisation of splenic artery

B Transfer 1 adult unit of blood reassess

C Admit to the ward for a week of bed-rest

D Admit for 3 days and discharge home

E Transfer to theatre for emergency splenectomy

Explanation
Splenic trauma is managed conservatively in the majority of cases. A double contrast CT scan is performed during
the trauma call and a contrast blush suggesting ingoing bleeding can be managed by embolization. Splenic injury
is graded I-V, listed below. The
current recommendations for blunt splenic trauma is grade plus 2 days of bed rest.
Major injury >III requires repeat imaging to assess for pseudo aneurysm at 5-10 days post injury.

Haematoma
I Subcapsular, <10% surface area. Capsular tear, <1cm parenchymal depth
Laceration

Haematoma Subcapsular, 10-50% surface area. Intraparenchymal, <5cm diameter 1-3cm parenchymal
II
Laceration depth not involving a oarenchymal vessel

Subcapsular, >50% surface area or expanding. Ruptured subcapsular or


parenchymak
Haematoma haematoma. Intraparenchymal haematoma >5cm, >3cm parenchymal depth or involving
III

https://mypastest.pastest.com/Secure/TestMe/Browser/436619[‫ ص‬06:08:27 10/12/1437]


MyPastest

Laceration trabecular vessels


Next Question
IV Laceration Laceration of segmental or hilar vessels producing major devascularisation

LacerationPrevious Question
V Completely shattered spleen. Hilar vascular injury which devascularised spleen
Vascular

End Session
Answer stem A is false. The child is haemodynamically stable and there is nothing to suggest on-going bleeding
needing embolisation.

Answer stem B is false, the child is haemodynamically stable and does not need transfusion as this may precipitate
further bleeding

Answer stem C is true, this is the current recommended treatment for children with high-grade splenic injury.

Answer stem D is false, this is too soon to discharge and there is a risk of a pseudo aneurysm around 5-10 days.

Answer stem E is false,


it is very rare to need to perform a splenectomy for blunt abdominal trauma and the child
is currently haemodynamically stable.
44982

Tag Question

Feedback

Difficulty: Average

Peer Responses

Session Progress

Responses Correct: 0

Responses Incorrect: 33

Responses Total: 33

https://mypastest.pastest.com/Secure/TestMe/Browser/436619[‫ ص‬06:08:27 10/12/1437]


MyPastest

Responses - % Correct: 0%

Blog
About Pastest
Contact Us
Help

© Pastest 2016

https://mypastest.pastest.com/Secure/TestMe/Browser/436619[‫ ص‬06:08:27 10/12/1437]


MyPastest

Prefer to use the old MyPastest? Access it here »

Back to Filters

Question 34 of 47

A 2-year-old girl comes to clinic with two small pits just above the clavicle. The parents say it stains the white
babygro yellow. It has never been infected or swollen.

What is the most likely diagnosis?

A Branchial arch remnants

B Thyroglossal cysts

C 1st Pharyngeal arch anomaly

D Chronically infected lymph node abscess

E Pilomatrixoma

Explanation
This represents a second branchial arch remnant. 1st pharyngeal arches form the bones musculature and nerves of
the face and produce typical facial anomalies. Most branchial sinus remnants undergo routine surgical excision
and may require a stepladder of incisions.

Answer stem A is true, this is the typical site for branchial remnant.

Answer stem B is false, this is a midline neck cyst

Answer stem C is false, this produces typical facial appearances.

Answer stem D is false, tuberculosis infected lymph nodes can lead to chronic sinus but this would rarely be
symmetrical.

Answer stem E is false, this is typically a calcifying skin lesion of a hair cell follicle.
44983

https://mypastest.pastest.com/Secure/TestMe/Browser/436619[‫ ص‬06:09:21 10/12/1437]


MyPastest

Next Question

Previous Question Tag Question

Feedback
End Session
Difficulty: Average

Peer Responses

Session Progress

Responses Correct: 0

Responses Incorrect: 34

Responses Total: 34

Responses - % Correct: 0%

Blog
About Pastest
Contact Us
Help

© Pastest 2016

https://mypastest.pastest.com/Secure/TestMe/Browser/436619[‫ ص‬06:09:21 10/12/1437]


MyPastest

Prefer to use the old MyPastest? Access it here »

Back to Filters

Question 35 of 47

A 12-year-old boy presents with a low grade temperature of 38.3oC,


a CRP of 256 and a palpable mass in the right
iliac fossa. He is eating
and drinking. An ultrasound scan demonstrates and appendix mass.

What is the appropriate management?

A Give broad spectrum intravenous antibiotics, arrange for CT scan to confirm a diagnosis

B Give broad spectrum intravenous antibiotics and arrange for urgent laparotomy

C Give broad spectrum oral antibiotics and discharge home for clinic review in 1 week

D Give
broad spectrum intravenous antibiotics, admit to the ward, perform operation only if signs of
obstruction or on-going sepsis

E Give oral antibiotics and arrange for interval appendicectomy in 6-12 weeks

Explanation
An appendix mass is on the whole managed medically with intravenous antibiotics and monitoring for signs of
obstruction or on-going sepsis. If the child is not responding to medical management, then surgery is performed.
The morbidity associated with operating on an appendix mass in greatly increased and consent for a
limited right
hemi-colectomy must be taken. The decision whether or not
to perform an interval appendicectomy is
controversial and currently subject to a multicentre national trial. The likelihood of another episode of appendicitis
is 1 in 5.

Answer stem A is false, ultrasound and clinical examination is sufficient to confirm the diagnosis, especially in a
boy.

Answer stem B is false, the majority of appendix masses respond to conservative management.

Answer stem C is false, the raised CRP indicates significant inflammatory response and intravenous antibiotics are
indicated.

https://mypastest.pastest.com/Secure/TestMe/Browser/436619[‫ ص‬06:09:43 10/12/1437]


MyPastest

Answer stem D is true. This is the correct management; bilious vomiting, pain and on-going fever indicate failure
of medical management. Next Question
Answer stem E is false, intravenous antibiotics are indicated due to sepsis, interval appendicectomy is
controversial and is not routinely practiced by all centres.
Previous Question 45027

End Session

Tag Question

Feedback

Difficulty: Average

Peer Responses

Session Progress

Responses Correct: 0

Responses Incorrect: 35

Responses Total: 35

Responses - % Correct: 0%

Blog
About Pastest
Contact Us
Help

© Pastest 2016

https://mypastest.pastest.com/Secure/TestMe/Browser/436619[‫ ص‬06:09:43 10/12/1437]


MyPastest

Prefer to use the old MyPastest? Access it here »

Back to Filters

Question 35 of 47

A 12-year-old boy presents with a low grade temperature of 38.3oC,


a CRP of 256 and a palpable mass in the right
iliac fossa. He is eating
and drinking. An ultrasound scan demonstrates and appendix mass.

What is the appropriate management?

A Give broad spectrum intravenous antibiotics, arrange for CT scan to confirm a diagnosis

B Give broad spectrum intravenous antibiotics and arrange for urgent laparotomy

C Give broad spectrum oral antibiotics and discharge home for clinic review in 1 week

D Give
broad spectrum intravenous antibiotics, admit to the ward, perform operation only if signs of
obstruction or on-going sepsis

E Give oral antibiotics and arrange for interval appendicectomy in 6-12 weeks

Explanation
An appendix mass is on the whole managed medically with intravenous antibiotics and monitoring for signs of
obstruction or on-going sepsis. If the child is not responding to medical management, then surgery is performed.
The morbidity associated with operating on an appendix mass in greatly increased and consent for a
limited right
hemi-colectomy must be taken. The decision whether or not
to perform an interval appendicectomy is
controversial and currently subject to a multicentre national trial. The likelihood of another episode of appendicitis
is 1 in 5.

Answer stem A is false, ultrasound and clinical examination is sufficient to confirm the diagnosis, especially in a
boy.

Answer stem B is false, the majority of appendix masses respond to conservative management.

Answer stem C is false, the raised CRP indicates significant inflammatory response and intravenous antibiotics are
indicated.

https://mypastest.pastest.com/Secure/TestMe/Browser/436619[‫ ص‬06:14:26 10/12/1437]


MyPastest

Answer stem D is true. This is the correct management; bilious vomiting, pain and on-going fever indicate failure
of medical management. Next Question
Answer stem E is false, intravenous antibiotics are indicated due to sepsis, interval appendicectomy is
controversial and is not routinely practiced by all centres.
Previous Question 45027

End Session

Tag Question

Feedback

Difficulty: Average

Peer Responses

Session Progress

Responses Correct: 0

Responses Incorrect: 35

Responses Total: 35

Responses - % Correct: 0%

Blog
About Pastest
Contact Us
Help

© Pastest 2016

https://mypastest.pastest.com/Secure/TestMe/Browser/436619[‫ ص‬06:14:26 10/12/1437]


MyPastest

Prefer to use the old MyPastest? Access it here »

Back to Filters

Question 36 of 47

Following a laparoscopic appendicectomy for a perforated appendix an 8-year-old girl develops a swinging
temperature of 38.6oC.

What is the appropriate course of management?

A Arrange for an ultrasound scan to look for intra-abdominal collection

B Change antibiotics to second line and involve microbiology

C Arrange for a chest x-ray and physiotherapy to manage likely chest infection

D Examine wounds for infection and allow them to open if infected

E Place an NG tube and allow decompression of likely post-operative adhesions

Explanation
Post-operative complications in appendicitis in children are more common than in adults. This is because
a greater
proportion of children present with perforated appendicitis and up to 1/3 of these can have a complication.
Surgical specific complications include: intra-abdominal collection, more common following
perforated
appendicitis and laparoscopic surgery; wound infection, more
common in open surgery and perforated
appendicitis; lower lobe pneumonia due to actelectasis, this is more common following open surgery; and poor
post-operative analgesia leading to shallow breathing and adhesions.

Answer stem A is true, the fact that this operation was perforated and laparoscopic make this the most likely
problem at this time post-operatively.

Answer stem B is false, although this may be necessary, swabs taken from theatre should have been available at
48 hours.

Answer stem C is false, this would be more likely if this had been an open operation or one using a midline
incision.

https://mypastest.pastest.com/Secure/TestMe/Browser/436619[‫ ص‬06:16:55 10/12/1437]


MyPastest

Answer
stem D is false, intra-abdominal collection is more likely to cause such a swinging pyrexia and more
likely with laparopscopic surgery. Open
surgery has a higher wound infection rate.
Next Question
Answer
stem E is false, adhesional obstruction would present with colicky abdominal pain, distended abdomen
and bilious vomiting, not with signs of sepsis.
Previous Question 45028

End Session

Tag Question

Feedback

Difficulty: Average

Peer Responses

Session Progress

Responses Correct: 0

Responses Incorrect: 36

Responses Total: 36

Responses - % Correct: 0%

Blog
About Pastest
Contact Us
Help

© Pastest 2016

https://mypastest.pastest.com/Secure/TestMe/Browser/436619[‫ ص‬06:16:55 10/12/1437]


MyPastest

Prefer to use the old MyPastest? Access it here »

Back to Filters

Question 37 of 47

You are called to the post natal ward to see a baby with an abdominal wall defect. There is intestine present, and a
sac covering the contents. The baby has tongue protrusion.

What is the next immediate course of action?

A Take blood for urgent chromosome analysis

B Wrap the bowel in cling film and arrange for transfer to surgical unit

C Auscultate the heart to check for duct dependent cardiac defects

D Reduce the bowel contents and apply the cord clamp flush with the skin

E Check the blood sugar

Explanation
A sac-covered abdominal wall defect is exomphalos; gastroschisis does not have a sac. Paradoxically the smaller
defects (not containing liver or < 5cm) are more likely to be associated with trisomies and syndromes. The critical
syndrome to rule out in exomphalos minor is Beckwith-Wiedemann Syndrome which is an over growth syndrome
characterised by: hypoglycaemia; macroglossia; macrosomia; ear creases; exomphalos; and hepatoblastoma/
increased risk of childhood cancer.

Answer stem A is false, although this is necessary, urgent blood sugar to assess for possibility
of neonatal
hypoglycaemia is most important next step.

Answer stem B is false, this is not necessary as the bowel already has a
sac, as opposed to gastroschisis where this
is done. Urgent blood sugar to assess for possibility of neonatal hypoglycaemia is most important next step.

Answer stem C is false, although cardiac defects are associated with exomphalos they tend to be VSD/ ASD and
not duct dependent. Urgent blood sugar to assess for possibility of neonatal hypoglycaemia is most important next
step.

https://mypastest.pastest.com/Secure/TestMe/Browser/436619[‫ ص‬06:17:19 10/12/1437]


MyPastest

Answer stem D is false, although this has been reported in some centres
there is a risk of placing the clamp across
enteric contents. Urgent blood sugar to assess for possibility of neonatal hypoglycaemia is most important next
Next Question
step.

Answer stem E is true, the likely diagnosis is Beckwith-Wiedemann syndrome and hypoglycaemia is a possibility.
Previous Question 45029

End Session

Tag Question

Feedback

Difficulty: Average

Peer Responses

Session Progress

Responses Correct: 0

Responses Incorrect: 37

Responses Total: 37

Responses - % Correct: 0%

Blog
About Pastest
Contact Us
Help

© Pastest 2016

https://mypastest.pastest.com/Secure/TestMe/Browser/436619[‫ ص‬06:17:19 10/12/1437]


MyPastest

Next Question
Prefer to use the old MyPastest? Access it here »

Previous Question
Back to Filters

Question 38 of 47

Which one of the following is not associated with exomphalos?

A Pentalogy of Cantrell

B Cardiac defects

C VACTERL

D Bladder exstrophy

E Trisomy 13

Explanation
Exomphalos is present in 1 in 4000 live births. It has a mortality of 25%, cardiac defects 50% and is associated
with trisomy 13, 18 and 21. It is also associated with neural tube defects and midline syndromes such as pentalogy
of Cantrell and bladder exstrophy.

Pentalogy of Cantrell consists of exomphalos, anterior diaphragmatic hernia, sternal cleft, ectopic cordis
and
cardiac defect (VSD).

VACTERL is an association of defects consisting of; (V)Vertebral, (A)Ano-rectal anomalies, (C)Cardiac ,


(TE)trachea-oEsophagealfistural/ atresia, (R)Renal and (L)Limb anomalies.
45030

Tag Question

Feedback

https://mypastest.pastest.com/Secure/TestMe/Browser/436619[‫ ص‬06:17:39 10/12/1437]


MyPastest

Difficulty: Average

Peer Responses

End Session

Session Progress

Responses Correct: 0

Responses Incorrect: 38

Responses Total: 38

Responses - % Correct: 0%

Blog
About Pastest
Contact Us
Help

© Pastest 2016

https://mypastest.pastest.com/Secure/TestMe/Browser/436619[‫ ص‬06:17:39 10/12/1437]


MyPastest

Prefer to use the old MyPastest? Access it here »

Back to Filters

Question 39 of 47

A 1–year-old girl is brought to the emergency department with bilious vomiting and abdominal distension.
Abdominal radiograph shows distended bowel loops and no gas in the rectum. Mother tells you she had an
operation at 2 days old for twisted intestine. A capillary blood gas shows a pH of 7.34 and lactate of 2.

What is the most appropriate management?

A Nasogastric decompression, broad spectrum antibiotics and immediate surgery for suspected recurrent
volvulus

B Nasogastric decompression, broad spectrum antibiotics and immediate surgery for suspected intra-
abdominal adhesions

C Manage as per gastroenteritis, allow oral rehydration therapy, as bilious vomiting can be normal in
previous malrotation

D Arrange for an urgent upper GI contrast to assess for position of DJ flexure to assess for recurrence

E Naso-gastric
decompression, intra venous fluids and admit. The majority of adhesional obstruction
resolves without need for surgery

Explanation
Malrotation is present in up to 1 in 500 births. The correct management is an operation called a ‘Ladds Procedure’,
to widen the mesenteric base to reduce the risk of recurrence. The operation creates adhesions which help fix the
bowel reducing the chance of volvulus. About 1/3 of patients can have problems
with vague abdominal symptoms
post operatively and there is a lifetime risk of intra-abdominal adhesions. 2/3 of adhesional obstructions resolve
with conservative management (nasogastric decompression & intra-venous fluids) and without the need for
surgery.

Answer
stem A is false, recurrent volvulus is highest in the immediate post-operative period. Intra-abdominal
adhesions are far more common, especially with a normal blood gas.

https://mypastest.pastest.com/Secure/TestMe/Browser/436619[‫ ص‬06:18:22 10/12/1437]


MyPastest

Answer stem B
is false, the majority of adhesive small bowel obstruction responds to nasogastric decompression
and gut rest for 24-48 hours.
Next Question
Answer
stem C is false, although bilious vomiting can be normal in this population, the abdominal radiograph
clearly dictates a picture of obstruction.
Previous Question
Answer stem D is false, even after surgical correction the DJ flexure remains in an abnormal position and does not
reassure against recurrent volvulus.
End Session
Answer stem E is true for the reasons given in the stem.
45031

Tag Question

Feedback

Difficulty: Average

Peer Responses

Session Progress

Responses Correct: 0

Responses Incorrect: 39

Responses Total: 39

Responses - % Correct: 0%

Blog
About Pastest
Contact Us
Help

© Pastest 2016

https://mypastest.pastest.com/Secure/TestMe/Browser/436619[‫ ص‬06:18:22 10/12/1437]


MyPastest

https://mypastest.pastest.com/Secure/TestMe/Browser/436619[‫ ص‬06:18:22 10/12/1437]


MyPastest

Prefer to use the old MyPastest? Access it here »

Back to Filters

Question 40 of 47

A 6-month-old baby with bilious vomiting, a palpable mass and


drawing up of legs is treated by an air reduction
enema for intussusception. During the procedure the infant becomes suddenly distended and begins to drop their
saturations.

What would be your immediate course of action?

A Stop the procedure and arrange for surgical ‘open’ reduction of intussusception

B Call the cardiac arrest team, the child has hypovolemic shock

C Immediate needle decompression of pneumoperitoneum

D Stop
the procedure, the air has reduced the colon and is refluxing into normal small bowel. Removal of
rectal catheter allows decompression.

E All babies that have air reduction enema should have nasal cannula as this is a common response to air
reduction enema

Explanation
Air reduction enema is the main stay of treatment for intussusception and is successful in around 90% of cases. A
serious potential risk of this procedure is perforation of the colon and a pneumoperitoneum, leading to rapid
distension of the abdomen and splinting of the diaphragm. Immediate needle decompression of the abdomen is a
lifesaving procedure that temporizes the situation while emergency definitive surgery is organised.

Answer stem A is false, whilst this is the case in 10% of ‘failed’ air reduction enema and pneumoperitoneum from
colonic perforation needs urgent needle decompression of the abdomen.

Answer
stem B is false, whilst this may happen simultaneously life saving measure to deal with the
pneumoperitoneum from colonic perforation needs
urgent needle decompression of the abdomen.

Answer stem C is true, this is rapid and effective lifesaving procedure.

https://mypastest.pastest.com/Secure/TestMe/Browser/436619[‫ ص‬06:18:47 10/12/1437]


MyPastest

Answer
stem D is false, this can be seen on image intensifier but does not lead to significant abdominal distension
causing splinting of the diaphragm. Next Question
Answer stem E is false, this is not needed and is the result of pneumoperitoneum from colonic perforation needs
urgent needle decompression of the abdomen.
Previous Question 45032

End Session

Tag Question

Feedback

Difficulty: Average

Peer Responses

Session Progress

Responses Correct: 0

Responses Incorrect: 40

Responses Total: 40

Responses - % Correct: 0%

Blog
About Pastest
Contact Us
Help

© Pastest 2016

https://mypastest.pastest.com/Secure/TestMe/Browser/436619[‫ ص‬06:18:47 10/12/1437]


MyPastest

Next Question
Prefer to use the old MyPastest? Access it here »

Back to Filters

Question 41 of 47

A 6-month-old baby boy presents to the emergency department with an acutely swollen mass extending from the
groin to the scrotum. He
has had one episode of green vomiting.

What is the most likely diagnosis and course of action?

A Incarcerated direct inguinal hernia, analgesia, sedation and attempt to reduce

B Incarcerated indirect inguinal hernia, analgesia, sedation and attempt to reduce

C Incarcerated femoral hernia, emergency surgery

D Incarcerated direct inguinal hernia, emergency surgery

E Incarcerated indirect inguinal hernia, emergency surgery

Explanation
All paediatric inguinal hernias are indirect and are a consequence of a patent processusvaginalis. The initial
management is attempted reduction under analgesia. If this is not possible than surgery is performed. Most UK
centres following reduction would than offer surgery within the next 2-3 days once the swelling has reduced.

Answer stem A is false, all paediatric hernias are indirect in nature.

Answer stem B is true, this is the correct diagnosis and management.

Answer stem C is false, this is rare in children and swelling tends to be slightly more lateral.

Answer stem D is false, this is an indirect hernia, and attempted reduction should be performed first.

Answer stem E is false, attempted reduction should be performed and only surgery if this is not possible.
45033

https://mypastest.pastest.com/Secure/TestMe/Browser/436619[‫ ص‬06:19:08 10/12/1437]


MyPastest

Tag Question

Feedback
Previous Question
Difficulty: Average
End Session
Peer Responses

Session Progress

Responses Correct: 0

Responses Incorrect: 41

Responses Total: 41

Responses - % Correct: 0%

Blog
About Pastest
Contact Us
Help

© Pastest 2016

https://mypastest.pastest.com/Secure/TestMe/Browser/436619[‫ ص‬06:19:08 10/12/1437]


MyPastest

Prefer to use the old MyPastest? Access it here »

Back to Filters

Question 42 of 47

A newborn baby boy presents with choking on feeding. The midwife attempts to pass an NG tube but is unable to
do so. A Chest /Abdominal X-ray is performed which shows an NG tube coiled at T3/4 and a
gasless abdomen.

Which of the following operations is likely to be required?

A Right sided thoracotomy, ligation of trachea-oesophageal fistula and repair of oesophageal atresia

B Oesophagostomy

C Laparotomy and open gastrostomy

D Right sided thoracotomy, attempted primary anastomosis and gastric pull-up if this is unsuccessful

E Bronchoscopy to look for upper pouch fistula

Explanation
This is type A or ‘pure’ oesophageal atresia as there is a gasless abdomen.These represent long-gap oesophageal
atresia. There is only a 1 in 5 chance of a trachea-oesophageal fistula with the upper pouch and no connection with
the lower oesophageal pouch. Normal enteral feeding needs to be established via a gastrostomy before regular gap
assessments and attempted primary anastomosis at 3-6 months. If this is not possible either a gastric pull-up,
jejunal or colonic interposition is performed.
Bronchoscopy may be undertaken at this time.

Answer
stem A is false, this would be the case if there was gas within the abdomen and a distal trachea-
oesohageal (type C) oesophageal atresia.

Answer
stem B is false, this is reserved when primary anastomosis is not possible and is very rarely used in the
western world due to adequate suctioning with Replogal tubes.

Answer stem C is
true, this is a pure oesophageal atresia and enteral feeding needs to be established before
delayed primary repair in 3-6 months.

Answer stem D is false, this is the second operation in 3-6 months; gastrostomy is needed as a first procedure.

https://mypastest.pastest.com/Secure/TestMe/Browser/436619[‫ ص‬06:19:29 10/12/1437]


MyPastest

Answer
stem E is false, this may be undertaken at the time of delayed definitive closure by a gastrostomy is
Next Question
needed as a first procedure to establish enteral feeding.
45034

Previous Question

End Session
Tag Question

Feedback

Difficulty: Average

Peer Responses

Session Progress

Responses Correct: 0

Responses Incorrect: 42

Responses Total: 42

Responses - % Correct: 0%

Blog
About Pastest
Contact Us
Help

© Pastest 2016

https://mypastest.pastest.com/Secure/TestMe/Browser/436619[‫ ص‬06:19:29 10/12/1437]


MyPastest

Next Question
Prefer to use the old MyPastest? Access it here »

Previous Question
Back to Filters
End Session
Question 43 of 47

Which of the following is not an indication for laparotomy for Necrotizing enterocolitis?

A Pneumoperitoneum

B Pneumotosis coli

C Failure of medical management

D Discolouration of the abdomen with signs of peritonism

E Fixed abdominal loop on serial radiographs

Explanation
The indications for operative intervention during the acute phase necrotising enterocolitis are divided into absolute
and relative. Absolute indications for laparotomy are pneumoperitoneum and failure of medical management with
worsening of
clinical situation 48 hours into the onset of disease. Relative indications include peritonism,
palapable mass, discoloration of the abdomen, fixed abdominal loop on serial radiographs.

Pneumotosis coli is a radiological feature of Bell stage 2 NEC.


45035

Tag Question

Feedback

Difficulty: Average

https://mypastest.pastest.com/Secure/TestMe/Browser/436619[‫ ص‬06:19:53 10/12/1437]


MyPastest

Peer Responses

Session Progress

Responses Correct: 0

Responses Incorrect: 43

Responses Total: 43

Responses - % Correct: 0%

Blog
About Pastest
Contact Us
Help

© Pastest 2016

https://mypastest.pastest.com/Secure/TestMe/Browser/436619[‫ ص‬06:19:53 10/12/1437]


MyPastest

Next Question
Prefer to use the old MyPastest? Access it here »

Back to Filters

Question 44 of 47

A 3-day-old baby is brought to see you because the parents complain of an abnormal umbilicus. On examination
there appears to be 2 loops of bowel on a stalk, with intestinal contents coming from it.

What is the most likely diagnosis?

A Vitello-intestinal remnant

B Missed gastroschisis

C Incorrectly applied bowel clamp causing bowel injury

D Cloacalexstrophy

E Exomphalos with ruptured sac

Explanation
This is a typical finding of a vitello-intestinal duct. Two loops of bowel can intussuscept through the
duct giving a
characteristic ‘ram’s horn’ appearance.

Answer stem A is true, this is a complete duct with intussusceptions.

Answer stem B is false, gastroschisis should have intact intestine and be obvious at birth.

Answer stem C is false, if this had occurred the child would be septic from ischaemic bowel and faecal peritonitis.

Answer stem D is false, this is typically infra-umbilical and has a large bladder plate with hind gut either side.

Answer stem E is false, there should not be any enteric contents if this were the case.
45036

https://mypastest.pastest.com/Secure/TestMe/Browser/436619[‫ ص‬06:20:15 10/12/1437]


MyPastest

Tag Question

Feedback

Previous Question
Difficulty: Average

End Session
Peer Responses

Session Progress

Responses Correct: 0

Responses Incorrect: 44

Responses Total: 44

Responses - % Correct: 0%

Blog
About Pastest
Contact Us
Help

© Pastest 2016

https://mypastest.pastest.com/Secure/TestMe/Browser/436619[‫ ص‬06:20:15 10/12/1437]


MyPastest

Prefer to use the old MyPastest? Access it here »

Back to Filters

Question 45 of 47

A 3-year-old girl of Caucasian parents presents with a 3-month history of rectal prolapse following potty training.

What would be the correct initial management?

A Commence on laxatives and arrange a rectal biopsy

B Examination under anaesthesia and Sclerotherapy

C Dietary and potty training advice, arrange a sweat test

D Discharge with reassurance

E Arrange for proctogram to rule out intussusception

Explanation
Rectal prolapse following potty training is common in children. General reassurance with simple potty training
advice how to reduce the prolapse along with about how to sit correctly and avoid straining is usually sufficient. In
recurrent prolapse in Caucasian children cystic fibrosis is possible and should be ruled out. If this is negative
simple laxatives are the next line and if bleeding or ulceration is present management is sclerotherapy although
this is rare.

Answer stem A is false, late diagnosis Hirschsprung’s tends to present with failure to thrive, abdominal distension
and explosive stool.

Answer stem B is
false, sclerotherapy is reserved for refractory cases not responding to
dietary life style advice or
simple laxatives.

Answer stem C is true, cystic fibrosis can be associated with rectal prolapsed.

Answer stem D is false, a sweat test is required to rule out cystic fibrosis.

Answer stem E is false, this is usually in older children.


45037

https://mypastest.pastest.com/Secure/TestMe/Browser/436619[‫ ص‬06:20:37 10/12/1437]


MyPastest

Next Question

Previous Question Tag Question

Feedback End Session

Difficulty: Average

Peer Responses

Session Progress

Responses Correct: 0

Responses Incorrect: 45

Responses Total: 45

Responses - % Correct: 0%

Blog
About Pastest
Contact Us
Help

© Pastest 2016

https://mypastest.pastest.com/Secure/TestMe/Browser/436619[‫ ص‬06:20:37 10/12/1437]


MyPastest

Prefer to use the old MyPastest? Access it here »

Back to Filters

Question 46 of 47

An 8-year-old boy with faecal soiling secondary to constipation and over flow incontinence has not responded to
over a year
of medical management. A biopsy has ruled out Hirschsprungs disease. Parents ask about surgical
management.

Which of the following procedures is most likely to be appropriate?

A Defunctioning Ileostomy

B Appendicostomy for anterior continence enemas

C Laparotomy and resection megarectum

D Left hemi-colectomy to increase transit time

E Bishop-Koop stoma

Explanation
Idiopathic constipation leading to faecal
incontinence is managed in a stepwise progression, first with laxatives
such as movicol, enemas and stronger laxatives, in younger children inter-sphincteric injection of botox may be
performed, following this either anal irrigation system or antegradecontinence enemas are performed. If this is not
an operation than a defunctioning ileostomy may be performed to allow the megarectum to shrink.

Answer stem A is false, although an option in extreme cases, an ACE stoma would be more appropriate in this
child.

Answer stem B is true, this allows for colonic washouts and quite rapid continance is achieved.

Answer stem C is false, this is performed if ACE stoma fails due to mega-rectum.

Answer stem D is false, this is reserved for slow-transit colons.

Answer stem E is false, this is a historic way of washing out and managing meconium ileus.
45038

https://mypastest.pastest.com/Secure/TestMe/Browser/436619[‫ ص‬06:20:59 10/12/1437]


MyPastest

Next Question

Previous Question Tag Question

Feedback End Session

Difficulty: Average

Peer Responses

Session Progress

Responses Correct: 0

Responses Incorrect: 46

Responses Total: 46

Responses - % Correct: 0%

Blog
About Pastest
Contact Us
Help

© Pastest 2016

https://mypastest.pastest.com/Secure/TestMe/Browser/436619[‫ ص‬06:20:59 10/12/1437]


MyPastest

Prefer to use the old MyPastest? Access it here »

Back to Filters

Question 47 of 47

A 6-year-old child with global developmental delay presents with failure to thrive and diarrhoea following change
of gastrostomy to a
low profile device.

Which of the following investigations is most likely to help with the diagnosis and management of this child?

A Plot weight on centiles chart and change feed accordingly

B Chest X-ray for post-operative actelactesis and aspiration

C Lower GI contrast

D AXR to look signs of subacute obstruction

E Contrast feed to look for colonic site of gastrostomy

Explanation
A complication of gastrostomy placement is colonic injury or entrapment when the original gastrostomy is
inserted. This may remain undiagnosed until gastrostomy device is changed and as the stomach falls away the low
profile device is sited in
the colon. A contrast feed demonstrates this most accurately.

Answer stem A is false, this is a technical complication from the surgery and not related to feed intolerance.

Answer stem B is false, there is no reason this should suddenly be an issue.

Answer
stem C is false, although this may demonstrate the problem it is not as
accurate as putting contrast down
the gastrostomy itself.

Answer stem D is false, there would be abdominal distension and bilious vomiting if this was the case.

Answer
stem E is true, the colon was trapped at first gastrostomy and at change the button device is within the
colon and not the stomach.
45039

https://mypastest.pastest.com/Secure/TestMe/Browser/436619[‫ ص‬06:21:21 10/12/1437]


MyPastest

End Session

Previous Question Tag Question

Feedback

Difficulty: Average

Peer Responses

Session Progress

Responses Correct: 0

Responses Incorrect: 47

Responses Total: 47

Responses - % Correct: 0%

Blog
About Pastest
Contact Us
Help

© Pastest 2016

https://mypastest.pastest.com/Secure/TestMe/Browser/436619[‫ ص‬06:21:21 10/12/1437]


MyPastest

Prefer to use the old MyPastest? Access it here »

Back to Filters

Question 1 of 9

Which of the following principles is essential when palliating children?

A Effective communication

B Family centered care

C Multi-disciplinary approach

D Sensitive professional approach

E All of the above

Explanation
Palliative care in children requires a professional, sensitive team that communicates effectively and responsively
to the needs of the child and family concerned. A shared understanding of the mutually agreed plan is needed such
that everyone involved is clear of what to do when events change.

http://www.rainbows.co.uk/wp-content/uploads/2011/06/Rainbows-Hospice-Basic-Symptom-Control-In-
Paediatric-Palliative-Care-8th-Ed-2011-protected.pdf
Next Question 47238

Previous Question
End
Tag Session
Question

Feedback

Difficulty: Average

https://mypastest.pastest.com/Secure/TestMe/Browser/436619[‫ ص‬05:41:00 10/12/1437]


MyPastest

Peer Responses

Session Progress

Responses Correct: 0

Responses Incorrect: 1

Responses Total: 1

Responses - % Correct: 0%

Blog
About Pastest
Contact Us
Help

© Pastest 2016

https://mypastest.pastest.com/Secure/TestMe/Browser/436619[‫ ص‬05:41:00 10/12/1437]


MyPastest

Prefer to use the old MyPastest? Access it here »

Back to Filters

Question 2 of 9

Which of the following may be a cause of anorexia in children with malignancy?

A Depression

B Oral thrush

C Pain

D Radiotherapy

E All of the above

Explanation
Anorexia may have a number of causes which, in addition to the conditions noted above, also include nausea,
medication,
cancer cachexia, taste disturbance and constipation. If no clear cause is found or if it isn't treatable,
parents may require support and education. Practical management e.g. Small meals, child's favourite food
and
even consideration of steroids may be offered.
47239

Next Question

Previous Question
Tag Question
End Session
Feedback

Difficulty: Average

https://mypastest.pastest.com/Secure/TestMe/Browser/436619[‫ ص‬05:41:52 10/12/1437]


MyPastest

Peer Responses

Session Progress

Responses Correct: 0

Responses Incorrect: 2

Responses Total: 2

Responses - % Correct: 0%

Blog
About Pastest
Contact Us
Help

© Pastest 2016

https://mypastest.pastest.com/Secure/TestMe/Browser/436619[‫ ص‬05:41:52 10/12/1437]


MyPastest

Prefer to use the old MyPastest? Access it here »

  Logged in as Ahmed fouad

Back to Filters
Question 3 of 9

When palliating children, which one of the following would NOT be Difficulty: Average
considered to be a cause of dyspnoea?
Peer Responses

A Anaemia

B Pain

C Raised intracranial pressure

D Renal failure

E None of the above Session Progress

Responses Correct: 0

Explanation Responses Incorrect: 3

All four conditions A-D would be considered to be


causes of breathlessness. Responses Total: 3

The list if not exhaustive and there may be multiple factors occurring at once. Responses - % Correct: 0%
It is essential all systems are considered when assessing patients.
http://www.togetherforshortlives.org.uk/assets/0000/5325/TfSL_Basic_Symptom_Control_In_Paediatric_Palliative_Care_-
_Ninth_Edition_-_PDF.pdf 47240

Next Question

Tag Question Feedback


Previous Question
End Session

Blog
About Pastest
Contact Us
Help

© Pastest 2016

https://mypastest.pastest.com/Secure/TestMe/Browser/436619[‫ ص‬05:43:13 10/12/1437]


MyPastest

https://mypastest.pastest.com/Secure/TestMe/Browser/436619[‫ ص‬05:43:13 10/12/1437]


MyPastest

Prefer to use the old MyPastest? Access it here »

Back to Filters

Question 4 of 9

A child with a known mediastinal tumour presents with headache and facial swelling.

What is the most likely diagnosis?

A Acute disseminated encephalomyelitis

B Anaphylaxis

C Cerebral metastasis

D Nephrotic syndrome

E Superior vena cava obstruction

Explanation
Symptoms of SVCO include headache, facial plethora, dizziness and breathlessness. It's considered an oncological
emergency. Treatments include steroids, radiotherapy, chemotherapy and stenting.
47241

Next Question

Previous Question Tag Question

Feedback
End Session

Difficulty: Average

Peer Responses

https://mypastest.pastest.com/Secure/TestMe/Browser/436619[‫ ص‬05:44:59 10/12/1437]


MyPastest

Session Progress

Responses Correct: 0

Responses Incorrect: 4

Responses Total: 4

Responses - % Correct: 0%

Blog
About Pastest
Contact Us
Help

© Pastest 2016

https://mypastest.pastest.com/Secure/TestMe/Browser/436619[‫ ص‬05:44:59 10/12/1437]


MyPastest

Prefer to use the old MyPastest? Access it here »

Back to Filters

Question 5 of 9

You are choosing an anti-emetic for a terminally ill child.

Which one of the following acts on the medulla oblongata?

A Chlorpromazine

B Cyclizine

C Domperidone

D Haloperidol

E Metoclopramide

Explanation
Cyclizine works on the medulla oblongata. The other anti-emetics work by different mechanisms. Cyclizine has
anti-muscarinic properties which may cause urinary retention. It also has sedating anti-histaminic effects. It starts
to work in around 30 mins of administration and peaks at around 2 hours.
47242
Next Question

Previous Question
Tag Question
End Session
Feedback

Difficulty: Average

https://mypastest.pastest.com/Secure/TestMe/Browser/436619[‫ ص‬05:45:23 10/12/1437]


MyPastest

Peer Responses

Session Progress

Responses Correct: 0

Responses Incorrect: 5

Responses Total: 5

Responses - % Correct: 0%

Blog
About Pastest
Contact Us
Help

© Pastest 2016

https://mypastest.pastest.com/Secure/TestMe/Browser/436619[‫ ص‬05:45:23 10/12/1437]


MyPastest

Prefer to use the old MyPastest? Access it here »

Back to Filters

Question 6 of 9

What is the most common cause of hiccup in a terminally ill child?

A Cerebral metastasis

B Gastric distension

C Pleural effusion

D Renal failure

E Superior vena caval obstruction

Explanation
Hiccups may cause a great deal of distress in a terminally ill child, affecting their ability to communicate, to eat, to
sleep which in turn may lead to weight loss, anxiety even depression. In any event their quality of life is badly
affected. The commonest cause of hiccups in terminally ill patients is gastric distension.

Gastric reflux, diaphragmatic irritation and renal failure may also be contributory.
47243
Next Question

Previous Question
Tag Question
End Session
Feedback

Difficulty: Average

https://mypastest.pastest.com/Secure/TestMe/Browser/436619[‫ ص‬05:46:27 10/12/1437]


MyPastest

Peer Responses

Session Progress

Responses Correct: 0

Responses Incorrect: 6

Responses Total: 6

Responses - % Correct: 0%

Blog
About Pastest
Contact Us
Help

© Pastest 2016

https://mypastest.pastest.com/Secure/TestMe/Browser/436619[‫ ص‬05:46:27 10/12/1437]


MyPastest

Prefer to use the old MyPastest? Access it here »

Back to Filters

Question 7 of 9

In the context of rapidly escalating pain in a child nearing the end of their life, intranasal morphine should be
given at what dose in a opiate naïve patient?

A 0.5mg/kg

B 0.1mg/kg

C 0.05mg/kg

D 0.01mg/kg

E 0.005mg/kg

Explanation
IV morphine may be given intra-nasally when rapid
analgesia is required. Opiate naïve children will need a lower
dose than children already tolerant.
47398

Next Question

Previous Question Tag Question

Feedback End Session

Difficulty: Average

Peer Responses

https://mypastest.pastest.com/Secure/TestMe/Browser/436619[‫ ص‬05:46:46 10/12/1437]


MyPastest

Session Progress

Responses Correct: 0

Responses Incorrect: 7

Responses Total: 7

Responses - % Correct: 0%

Blog
About Pastest
Contact Us
Help

© Pastest 2016

https://mypastest.pastest.com/Secure/TestMe/Browser/436619[‫ ص‬05:46:46 10/12/1437]


MyPastest

Prefer to use the old MyPastest? Access it here »

Back to Filters

Question 8 of 9

You are asked to see a young terminally ill child in urinary retention.

Which of the following actions would be an appropriate immediate next step?

A Enema

B Intravenous naloxone

C Placing child in a warm bath

D Radiotherapy to spinal cord

E Urinary catheterisation

47400

Next Question

Tag Question

Feedback End Session


Previous Question Difficulty: Average

Peer Responses

https://mypastest.pastest.com/Secure/TestMe/Browser/436619#Top[‫ ص‬05:48:12 10/12/1437]


MyPastest

Session Progress

Responses Correct: 0

Responses Incorrect: 9

Responses Total: 9

Responses - % Correct: 0%

Blog
About Pastest
Contact Us
Help

© Pastest 2016

https://mypastest.pastest.com/Secure/TestMe/Browser/436619#Top[‫ ص‬05:48:12 10/12/1437]


MyPastest

Prefer to use the old MyPastest? Access it here »

Back to Filters

Question 9 of 9

You are asked to convert 10mg oral morphine to subcutaneous morphine.

What is the equivalent subcutaneous dose?

A 20mg

B 10mg

C 5mg

D 2mg

E 1mg

Explanation
Opiate conversions are a common challenge when managing terminally ill patients. Oral to subcutaneous
morphine conversion is in a ratio of 2:1.
47401

End Session

Previous Question Tag Question

Feedback

Difficulty: Average

Peer Responses

https://mypastest.pastest.com/Secure/TestMe/Browser/436619#Top[‫ ص‬05:48:30 10/12/1437]


MyPastest

Session Progress

Responses Correct: 0

Responses Incorrect: 9

Responses Total: 9

Responses - % Correct: 0%

Blog
About Pastest
Contact Us
Help

© Pastest 2016

https://mypastest.pastest.com/Secure/TestMe/Browser/436619#Top[‫ ص‬05:48:30 10/12/1437]


MyPastest

Prefer to use the old MyPastest? Access it here »


Previous Question

Back to Filters

Question 1 of 86

A 5-year-old girl, known to have asthma has had a cold for 2 days. She wakes up suddenly short of breath and has
difficulty speaking in full sentences. On examination she has a respiratory rate of
50/min, and sounds wheezy.

Which treatment is of unknown effectiveness:

A Inhaled ipratropium bromide added to salbutamol (after initial stabilisation)

B Oxygen

C Nebulised ipratropium bromide

D Metered dose inhaler via spacer device

E Oral prednisolone

Explanation
Inhaled ipratropium bromide added to salbutamol (after initial stabilistation) is of unknown effectiveness (Clinical
Evidence, BMJ). Management of acute asthma attack is very important and if possible bronchodilators should be
given via a spacer device.
11663

Next Question
Tag Question

Feedback

End Session
Difficulty: Difficult

https://mypastest.pastest.com/Secure/TestMe/Browser/436619[‫ ص‬04:59:00 10/12/1437]


MyPastest

Peer Responses

Session Progress

Responses Correct: 0

Responses Incorrect: 1

Responses Total: 1

Responses - % Correct: 0%

Blog
About Pastest
Contact Us
Help

© Pastest 2016

https://mypastest.pastest.com/Secure/TestMe/Browser/436619[‫ ص‬04:59:00 10/12/1437]


MyPastest

Prefer to use the old MyPastest? Access it here »


Previous Question

Back to Filters

Question 2 of 86

A 2-year-old child has been seen recently by an optometrist working in a hospital eye department. The child’s
parents say that eye drops were instilled but they do not know what they
were called. The child now presents with
dry mouth, flushing and dryness of the skin, dilated pupils and increased heart rate.

What is this likely to be an adverse effect from?

A Atropine sulphate

B Phenylephrine

C Timolol maleate

D Carbachol

E Prednisolone

Explanation
The child has symptoms of atropine poisoning. Atropine is a cycloplegic sometimes used in the eye hospital to
paralyse a young child’s accommodation to reveal their full refractive error as the accommodation will no longer
effect the results.
The fatal dose for a child is only 10mg. 100mg will kill an adult. CNS effects occur only in
advanced stages of atropine poisoning, milder effects occur at an earlier stage, so the patient will be thirsty, hot
and dry, have a dry mouth, and they will have an increased heart rate.
11666

Next Question

Tag Question

https://mypastest.pastest.com/Secure/TestMe/Browser/436619[‫ ص‬04:59:28 10/12/1437]


MyPastest

Feedback

Difficulty: Easy

Peer Responses

Session Progress

Responses Correct: 0

Responses Incorrect: 2

Responses Total: 2

Responses - % Correct: 0%

Blog
About Pastest
Contact Us
Help

© Pastest 2016

End Session

https://mypastest.pastest.com/Secure/TestMe/Browser/436619[‫ ص‬04:59:28 10/12/1437]


MyPastest

Prefer to use the old MyPastest? Access it here »


Previous Question

Back to Filters

Question 3 of 86

A child presents with sudden onset pain, photophobia, lacrimation, blurred vision, circumcorneal redness and a
small pupil.

What is often used in combination with atropine to prevent worsening of this condition?

A Chloramphenicol

B Phenylephrine

C Sno tears

D Sodium cromoglycate eye drops

E Aciclovir

Explanation
The child has symptoms of anterior uveitis and needs to be treated with phenylephrine to prevent posterior
synechiae.
11668

Next Question Tag Question

Feedback

Difficulty: Average
End Session

https://mypastest.pastest.com/Secure/TestMe/Browser/436619[‫ ص‬04:59:48 10/12/1437]


MyPastest

Peer Responses

Session Progress

Responses Correct: 0

Responses Incorrect: 3

Responses Total: 3

Responses - % Correct: 0%

Blog
About Pastest
Contact Us
Help

© Pastest 2016

https://mypastest.pastest.com/Secure/TestMe/Browser/436619[‫ ص‬04:59:48 10/12/1437]


MyPastest

Prefer to use the old MyPastest? Access it here »


Previous Question

Back to Filters

Question 4 of 86

A child presents with a left eye proptosis, swollen erythematous eyelid, immobility of the left eye and fever.

Which is the most appropriate treatment?

A Cefuroxime IV

B Chloramphenicol IV

C Steroids IV

D Oral cephalosphorin

E Hydrocortisone IV

Explanation
The child has symptoms associated with orbital cellulitis, therefore systemic antibiotics need to be given. Topical
chloramphenicol eye drops may be used.
11669

Tag Question
Next Question
Feedback

Difficulty: Easy

End Session
Peer Responses

https://mypastest.pastest.com/Secure/TestMe/Browser/436619[‫ ص‬05:00:08 10/12/1437]


MyPastest

Session Progress

Responses Correct: 0

Responses Incorrect: 4

Responses Total: 4

Responses - % Correct: 0%

Blog
About Pastest
Contact Us
Help

© Pastest 2016

https://mypastest.pastest.com/Secure/TestMe/Browser/436619[‫ ص‬05:00:08 10/12/1437]


MyPastest

Prefer to use the old MyPastest? Access it here »


Previous Question

Back to Filters

Question 5 of 86

A mum suspects her child has one eye that turns in by itself sometimes and thinks her child has a squint.

What is the most suitable preparation to use to assess the child’s refractive state for a 4-year-old Caucasian child?

A No drops

B 1% Cyclopentolate

C 0.5% Amethocaine

D 1% Tropicamide

E 1% Atropine sulphate

Explanation
Topical cycloplegic agents are used to dilate the pupil and to relax the ciliary muscle for ocular examination.
Drugs of choice vary with opinions but 1% cyclopentolate would be a suitable choice of cycloplegic in this
situation. Cycloplegics paralyse accommodation and therefore reveal an individual's full refractive error
without
the effects of accommodation altering the prescription. If a prescription is found, it is prescribed if it can control or
relieve the
squint. Tropicamide is a weak mydriatic but will not paralyse ciliary musculature sufficiently to
prevent accommodation. Atropine has a longer
duration of action than cyclopentolate. Amethocaine is a local
anaesthetic.
11670

Next Question

Tag Question

https://mypastest.pastest.com/Secure/TestMe/Browser/436619[‫ ص‬05:00:27 10/12/1437]


MyPastest

Feedback

Difficulty: Difficult

Peer Responses

Session Progress

Responses Correct: 0

Responses Incorrect: 5

Responses Total: 5

Responses - % Correct: 0%

Blog
About Pastest
Contact Us
Help

© Pastest 2016

End Session

https://mypastest.pastest.com/Secure/TestMe/Browser/436619[‫ ص‬05:00:27 10/12/1437]


MyPastest

Prefer to use the old MyPastest? Access it here »

Back to Filters

Question 6 of 86

A mum suspects her child has one eye that turns in by itself sometimes and suspects her child has a squint.

What
is the most suitable preparation to use to assess the child’s refractive state for a pre-school child with dark
irises?

A No drops

B 1% Phenylephrine

C 0.5% Cyclopentolate

D 1% Tropicamide

E 1% Atropine sulphate

Explanation
Topical cycloplegic agents are used to dilate the pupil and to relax the ciliary muscle for ocular examination.
Drugs of choice vary with opinion but 1% Atropine sulphate would be the
preferred agent for an individual with
darkly pigmented irises. Cycloplegics paralyse accommodation and therefore reveal an individual's
full refractive
error without the effects of accommodation altering the
prescription. If a prescription is found, it is prescribed if it
can control or relieve the squint. Tropicamide may be used where pupil dilatation alone is required (e.g. to
facilitate fundoscopy). In many ophthalmology clinics, 1% cyclopentolate is the preferred agent for cycloplegia.
Atropine performs better than cyclopentolate in children with dark irises, which tend to be more resistant to
Next Question
mydriasis. Drops may need to be instilled some hours before examination in some of these individuals.
11671

https://mypastest.pastest.com/Secure/TestMe/Browser/436619[‫ ص‬05:00:52 10/12/1437]


MyPastest

Tag Question

Previous Question
Feedback

Difficulty: Difficult

Peer Responses

Session Progress

Responses Correct: 0

Responses Incorrect: 6

Responses Total: 6

Responses - % Correct: 0%

Blog
About Pastest
Contact Us
Help

© Pastest 2016

End Session

https://mypastest.pastest.com/Secure/TestMe/Browser/436619[‫ ص‬05:00:52 10/12/1437]


MyPastest

Prefer to use the old MyPastest? Access it here »

Back to Filters

Question 7 of 86

Theme: Treatment options

A Whey hydrolysate milk

B Sucrose solution

C Soya based infant feeds

D Herbal tea

E Desmopressin

F Carbamazepine

G Lamotrigine

H Paracetamol

I Ibuprofen

J Clear fluids

For
each of the following scenarios select the most likely beneficial treatment option from the list above. Each
option may be used once, more
than once, or not at all.

Scenario 1

A 9-year-old boy develops sudden episodes,


only lasting a few seconds, of unconsciousness. The EEG shows
regular symmetrical three cycles a second generalised spike and wave complexes.

Your answer was incorrect

Select one...

G - Lamotrigine

The first case is describing petit mal (absence seizures). The most appropriate treatment from the list is therefore
Lamotrigine.

Scenario 2

https://mypastest.pastest.com/Secure/TestMe/Browser/436619[‫ ص‬05:01:14 10/12/1437]


MyPastest

An 8-year-old boy is still wetting the bed at night and wants to go to a sleep-over party.

Your answer was incorrect


Previous Question
Select one...

E - Desmopressin

An 8 year old with nocturnal enuresis could be managed at home with an alarm, however as a temporary measure,
desmopressin has been shown to be of benefit.

Scenario 3

A 3-month-old boy is described as crying whilst pulling up his legs. These bouts last for more than three hours
each time and have been occuring more than three days a week. All investigations have been normal.

Your answer was incorrect

Select one...

A - Whey hydrolysate milk

This
baby is experiencing infantile colic. The BMJ clinical evidence site (http://clinicalevidence.bmj.com) has
found evidence that, compared with
cows’ milk formula, whey hydrolysate milk may be more effective at
reducing the duration of crying.

Explanation
 
11672

Tag Question

Feedback

Difficulty: Average

Next Question
Session Progress

Responses Correct: 0

Responses Incorrect: 9

Responses Total: 9

Responses - % Correct:
End Session 0%

https://mypastest.pastest.com/Secure/TestMe/Browser/436619[‫ ص‬05:01:14 10/12/1437]


MyPastest

Blog
About Pastest
Contact Us
Help

© Pastest 2016

https://mypastest.pastest.com/Secure/TestMe/Browser/436619[‫ ص‬05:01:14 10/12/1437]


MyPastest

Prefer to use the old MyPastest? Access it here »

Back to Filters

Question 8 of 86

Theme: Medications

A Tetracycline

B Cefotaxime

C Chloramphenicol

D Amoxicillin

E Gentamicin

F Trimethoprim

G Teicoplanin

H Cefaclor

I Flucloxacillin

J Benzylpenicillin

Choose
the drug most responsible or used for the clinical situation shown below. Each option may be used once,
more than once, or not at all.

Scenario 1

A 5-year-old given this drug has now developed staining and dental hypoplasia.

Your answer was incorrect

Select one...

A - Tetracycline

Tetracycline is contraindicated in growing children as it causes yellow staining of the teeth with poor development
of them also. This staining can cause permanent discolouration.

Scenario 2

A 4-year-old boy who has failed his hearing test and was treated for neonatal sepsis with this antibiotic.

https://mypastest.pastest.com/Secure/TestMe/Browser/436619[‫ ص‬05:01:41 10/12/1437]


MyPastest

Your answer was incorrect

Previous Question
Select one...

E - Gentamicin

Gentamicin causes nephrotoxicity, by causing necrosis to the proximal tubule, leading to acute renal failure. It also
causes otoxicity as the drug is toxic to the sensory cells of the ear.

Scenario 3

A 6 month old girl who has antentally diagnosised hydronephrosis, had two confirmed urinary tract infections
(UTIs) and is now on a long-term prophylactic antibiotic.

Your answer was incorrect

Select one...

F - Trimethoprim

Trimethoprim is most commonly used as prophylaxis against urinary tract infections in children, particularly those
with underlying abnormalities from birth.

Explanation
 
11673

Tag Question

Feedback

Difficulty: Average

Session Progress

Responses Correct: 0
Next Question
Responses Incorrect: 12

Responses Total: 12

Responses - % Correct: 0%

End Session

https://mypastest.pastest.com/Secure/TestMe/Browser/436619[‫ ص‬05:01:41 10/12/1437]


MyPastest

Blog
About Pastest
Contact Us
Help

© Pastest 2016

https://mypastest.pastest.com/Secure/TestMe/Browser/436619[‫ ص‬05:01:41 10/12/1437]


MyPastest

Prefer to use the old MyPastest? Access it here »


Previous Question

Back to Filters

Question 9 of 86

Which of the following is a characteristic clinical finding of opioid poisoning?

A Pupillary dilatation

B Hypothermia

C Deep respiration

D Bradycardia

E Sweating and lacrimation

Explanation
Opioid poisoning is classically associated with pinpoint pupils, reduced respiratory rate, bradycardia, drowsiness
and coma. Hypothermia is a feature of barbiturate poisoning, while sweating and lacrimation are seen in cases of
opiate withdrawal.
12731

Next Question
Tag Question

Feedback

Difficulty: Average
End Session

https://mypastest.pastest.com/Secure/TestMe/Browser/436619[‫ ص‬05:02:03 10/12/1437]


MyPastest

Peer Responses

Session Progress

Responses Correct: 0

Responses Incorrect: 13

Responses Total: 13

Responses - % Correct: 0%

Blog
About Pastest
Contact Us
Help

© Pastest 2016

https://mypastest.pastest.com/Secure/TestMe/Browser/436619[‫ ص‬05:02:03 10/12/1437]


MyPastest

Prefer to use the old MyPastest? Access it here »

Back to Filters

Question 10 of 86

A 17-year-old presents to A&E following the ingestion of an unknown quantity of aspirin.

Which of the following statements is true concerning salicylate poisoning?

A Tinnitus occurs at plasma salicylate concentrations of 200 mg/l

B Metabolic acidosis is the first acid–base disturbance seen

C Hypoglycaemia is common in adults

D Acidosis reduces the rate of salicylate transfer across the blood–brain barrier

E Multi-dose activated charcoal is indicated in the management

Explanation
Symptoms following salicylate ingestion include nausea, vomiting, flushing, hyperventilation and sweating.
Tinnitus typically occurs at plasma salicylate concentrations above 400–500 mg/l. Hypoglycaemia following
salicylate ingestion is commonly seen in children but not in adults. Severe toxic effects are CNS effects, including
confusion, coma and fits. Initially there is a respiratory alkalosis due to the direct stimulation of the respiratory
centre and hyperventilation; then a metabolic acidosis occurs due to uncoupling of oxidative phosphorylation and
the build up of lactic acid and fatty acid metabolites. The metabolic acidosis can increase the transfer of salicylates
Next Question
across the blood–brain barrier, thereby increasing CNS toxicity.
12732

https://mypastest.pastest.com/Secure/TestMe/Browser/436619[‫ ص‬05:02:25 10/12/1437]


MyPastest

Tag Question

Previous Question
Feedback

Difficulty: Average

Peer Responses

Session Progress

Responses Correct: 0

Responses Incorrect: 14

Responses Total: 14

Responses - % Correct: 0%

Blog
About Pastest
Contact Us
Help

© Pastest 2016

End Session

https://mypastest.pastest.com/Secure/TestMe/Browser/436619[‫ ص‬05:02:25 10/12/1437]


MyPastest

Prefer to use the old MyPastest? Access it here »


Previous Question

Back to Filters

Question 11 of 86

A 16-year-old student suffers from hay fever. She is about to take her GCSE examinations and is worried that the
medications may cause drowsiness. She has recently been prescribed erythromycin for an ear infection.

Which antihistamine would be most suitable for treating her hayfever?

A Cetirizine

B Loratadine

C Fexofenadine

D Chlorphenamine maleate

E Terfenadine

Explanation
Cetirizine, loratadine and fexofenadine are prescribed for allergic rhinitis (hay fever) and all three are equally
effective. However, loratadine and fexofenadine interact with erythromycin and other macrolides which inhibit
their metabolism. Chlorphenamine maleate and terfenadine cause drowsiness and also interact with erythromycin.

12733

Next Question

Tag Question

Feedback
End Session

https://mypastest.pastest.com/Secure/TestMe/Browser/436619[‫ ص‬05:02:46 10/12/1437]


MyPastest

Difficulty: Difficult

Peer Responses

Session Progress

Responses Correct: 0

Responses Incorrect: 15

Responses Total: 15

Responses - % Correct: 0%

Blog
About Pastest
Contact Us
Help

© Pastest 2016

https://mypastest.pastest.com/Secure/TestMe/Browser/436619[‫ ص‬05:02:46 10/12/1437]


MyPastest

Prefer to use the old MyPastest? Access it here »

Back to Filters

Question 12 of 86

You are called by a local GP who asks for advice about the best antibiotic to use to treat an acute infection in a
woman who is
breast-feeding her 2-week-old baby.

Which of the following medications is safe in breast-feeding?

A Flucloxacillin

B Ciprofloxacin

C Ofloxacin

D Clarithromycin

E Fluconazole

Explanation
The penicillins are relatively safe in breast-feeding. In contrast, ciprofloxacin and ofloxacin are quite lipophilic
and are therefore secreted in significant quantities in breast milk. Clarithromycin is not recommended by the
manufacturer unless the benefits of treatment outweigh the risk. Fluconazole is also not recommended for use by
the manufacturer in patients who are breast-feeding. It is important to note that significant changes in cytochrome
P450 metabolism take place during the first few months of life, so that safety in a parent or adult population is no
guarantee of safety in the neonate. Next Question
12734

https://mypastest.pastest.com/Secure/TestMe/Browser/436619[‫ ص‬05:03:10 10/12/1437]


MyPastest

Tag Question

Previous Question
Feedback

Difficulty: Average

Peer Responses

Session Progress

Responses Correct: 0

Responses Incorrect: 16

Responses Total: 16

Responses - % Correct: 0%

Blog
About Pastest
Contact Us
Help

© Pastest 2016

End Session

https://mypastest.pastest.com/Secure/TestMe/Browser/436619[‫ ص‬05:03:10 10/12/1437]


MyPastest

Prefer to use the old MyPastest? Access it here »


Previous Question

Back to Filters

Question 13 of 86

A young woman, who is on the oral contraceptive pill, is treated by her GP for contact with a proven case of
meningitis. A week later she develops irregular bleeding. A pregnancy test done a month later is positive.

Which medication is most likely to have been prescribed by the GP?

A Rifampicin

B Tetracycline

C Amoxicillin

D Cefalexin

E Ciprofloxacin

Explanation
Of the choices given, only ciprofloxacin and rifampicin are used as prophylaxis for meningitis contacts. Current
evidence suggests that the effectiveness of oral contraceptives is reduced by the interaction with drugs that induce
hepatic enzyme activity such as carbamazepine, phenytoin, phenobarbital, topiramate and
rifampicin. These
medications induce the hepatic enzymes to the extent of reducing the activity of a number of drugs that are
metabolised by the liver, including oral contraceptive pills.
Next Question 12735

Tag Question

https://mypastest.pastest.com/Secure/TestMe/Browser/436619[‫ ص‬05:03:50 10/12/1437]


MyPastest

Feedback

Difficulty: Average

Peer Responses

Session Progress

Responses Correct: 0

Responses Incorrect: 17

Responses Total: 17

Responses - % Correct: 0%

Blog
About Pastest
Contact Us
Help

© Pastest 2016

End Session

https://mypastest.pastest.com/Secure/TestMe/Browser/436619[‫ ص‬05:03:50 10/12/1437]


MyPastest

Prefer to use the old MyPastest? Access it here »


Previous Question

Back to Filters

Question 14 of 86

A 15-year-old girl is referred after a deliberate paracetamol overdose. She reports having taken five pills 45
minutes ago. What is the most appropriate course of action?

A Administer activated charcoal and take paracetamol levels at 4 h

B Administer activated charcoal and start intravenous N-acetylcysteine, then take paracetamol levels at 4
h

C Start intravenous N-acetylcysteine, and take levels at 4 h

D Wait and take blood at 4 h for paracetamol levels

E Discharge her as it is not a harmful dose

Explanation
Five pills is not a toxic dose. Activated charcoal is indicated if the amount ingested is >150mg/kg and <1 hour
prior to presentation. Levels for paracetamol and salicylate should
be taken at 4 h to assess whether treatment is
needed (because she may have concealed what and how much she actually took). All overdoses should, ideally, be
discussed with the National Poisons Information Service (NPIS), especially if staggered overdose or more than
one substance was taken. Most emergency departments now have online access to ‘Toxbase’, the NPIS website
that allows quick referencing
for overdose management.
22386

Next Question

Tag Question

Feedback

https://mypastest.pastest.com/Secure/TestMe/Browser/436619[‫ ص‬05:04:09 10/12/1437]


MyPastest

Difficulty: Average

Peer Responses

Session Progress

Responses Correct: 0

Responses Incorrect: 18

Responses Total: 18

Responses - % Correct: 0%

Blog
About Pastest
Contact Us
Help

© Pastest 2016

End Session

https://mypastest.pastest.com/Secure/TestMe/Browser/436619[‫ ص‬05:04:09 10/12/1437]


MyPastest

Prefer to use the old MyPastest? Access it here »


Previous Question

Back to Filters

Question 15 of 86

A 3-year-old is brought to the Emergency Department having been found with an empty pill bottle of his
grandmother’s. There is no information as to what the tablets were. On examination he is drowsy, and pale. His
respiratory rate (RR) is 60/min, his BP 70/30 mmHg and his pulse 170/min and very thready. You also note that
he has dilated pupils. Which of the following drugs were the tablets most likely to have been?

A ß-blockers

B Aspirin

C Paracetamol

D Iron

E Tricyclic antidepressant (TCA)

Explanation
The most serious side effect of TCAs is the development of ventricular tachycardia. This child has a reduced
conscious level, and is profoundly tachycardic, tachypnoeic and hypotensive; these would all be seen with VT and
also the CNS effects of
TCAs. b Blockers would more likely cause a bradycardia. Aspirin can cause an initial
hyperventilation and then a metabolic acidosis.
22391

Next Question

Tag Question

Feedback
End Session

https://mypastest.pastest.com/Secure/TestMe/Browser/436619[‫ ص‬05:04:29 10/12/1437]


MyPastest

Difficulty: Average

Peer Responses

Session Progress

Responses Correct: 0

Responses Incorrect: 19

Responses Total: 19

Responses - % Correct: 0%

Blog
About Pastest
Contact Us
Help

© Pastest 2016

https://mypastest.pastest.com/Secure/TestMe/Browser/436619[‫ ص‬05:04:29 10/12/1437]


MyPastest

Prefer to use the old MyPastest? Access it here »

Back to Filters

Question 16 of 86

Theme: Drugs

A Intravenous acyclovir

B Per os fluconaxole

C Intravenous ribavirin

D Intravenous ganciclovir

E Per os azithromycin

F Intravenous vancomycin

G Intravenous flucloxacillin

H Intravenous co-amoxiclav

I Intravenous cefotaxime

J Intravenous emphotericin

For
each of the clinical scenarios below select, from the list above, the most appropriate theraputic agent. Each
item may be used once, more than
once or not at all.

Scenario 1

A 12-year old presents with cough, fever and feeling unwell. Her saturations are 98% in air and her respiratory
rate is 24/min without recession. There are diffuse crackles throughout her lung fields on auscultation.

Your answer was incorrect

Select one...

E - Per os azithromycin

This 12-year-old has presented with a clinical chest infection but has no signs of acute respiratory distress.
A
macrolide is a very reasonable choice in this situation because it will cover common pathogens such as
pneumococci but also mycoplasmas, which must be considered in a child of this age. Azithromycin is a newer
macrolide that is given once a day for 3 days; this low dosing regimen is argued to improve compliance.

https://mypastest.pastest.com/Secure/TestMe/Browser/436619[‫ ص‬05:04:53 10/12/1437]


MyPastest

Scenario 2

Previous
A 2-year old child Question
presents with a 2-day history of headache and mood change. She is febrile. She has a focal
seizure with secondary generalization in the Emergency Department. Computed tomography (CT) shows changes
in the temporal lobe.

Your answer was incorrect

Select one...

A - Intravenous acyclovir

In any child with headache and fever, meningitis/encephalitis should be considered. If this child has a seizure, the
index of suspicion is much higher. Temporal lobe changes are classically seen on CT in herpes encephalitis.
Intravenous acyclovir
is the therapy of choice for herpes encephalitis. In a real-life scenario, this child would also
have received a broad-spectrum antibiotic (eg intravenous cefotaxime) to cover bacterial causes.

Scenario 3

An 18-month-old child is admitted with infected eczema affecting both elbows, the chest and behind the left knee.
He is febrile (38°C).

Your answer was incorrect

Select one...

H - Intravenous co-amoxiclav

The two most likely pathogens in this situation are S. Aureus and Streptococcus spp.
As the child is displaying
systemic symptoms and the infection is in more than two areas, it would be reasonable to treat with a broad-
spectrum antibiotic that covers both pathogens. Augmentin would be
appropriate in this case. Intravenous
flucloxacillin alone is not sufficient unless combined with benzylpenicillin.
22416

Tag Question

Feedback
Next Question
Difficulty: Average

Session Progress

Responses Correct: End Session 0

https://mypastest.pastest.com/Secure/TestMe/Browser/436619[‫ ص‬05:04:53 10/12/1437]


MyPastest

Responses Incorrect: 22

Responses Total: 22

Responses - % Correct: 0%

Blog
About Pastest
Contact Us
Help

© Pastest 2016

https://mypastest.pastest.com/Secure/TestMe/Browser/436619[‫ ص‬05:04:53 10/12/1437]


MyPastest

Prefer to use the old MyPastest? Access it here »


Previous Question

Back to Filters

Question 17 of 86

Regarding the treatment of tuberculosis, which one of the following drugs is likely to cause red discoloured urine?

A Cycloserine

B Ethambutol

C Isoniazid

D Pyrazinamide

E Rifampacin

Explanation
The side effects of key drugs is a popular exam question.

Rifampacin causes red/orange discolouration of body secretions.

Cycloserine is used for drug resistant TB. Side effects may include confusion, dizziness, drowsiness and tremor.

Side
effects of Isoniazid and ethambutol may include dark urine and patients
given pyrazinamide may experience
nausea, vomiting, joint pain and fatigue.
44942

Next Question

Tag Question

Feedback
End Session

https://mypastest.pastest.com/Secure/TestMe/Browser/436619[‫ ص‬05:05:16 10/12/1437]


MyPastest

Difficulty: Average

Peer Responses

Session Progress

Responses Correct: 0

Responses Incorrect: 23

Responses Total: 23

Responses - % Correct: 0%

Blog
About Pastest
Contact Us
Help

© Pastest 2016

https://mypastest.pastest.com/Secure/TestMe/Browser/436619[‫ ص‬05:05:16 10/12/1437]


MyPastest

Prefer to use the old MyPastest? Access it here »

Back to Filters

Question 18 of 86

An 8-year-old girl is referred for neurophysiological testing


for frequent episodes of sudden, brief apparent loss of
consciousness, accompanied by lip smacking. Electroencephalography demonstrated a spike
and wave pattern.
She has no other medical problems or current medications.

What is the most appropriate therapy to commence?

A Clonazepam

B Ethosuximide

C Gabapentin

D Lamotrigine

E Vigabatrin

Explanation
This is a classical description of absence seizures with automatic movements associated with brief periods of loss
of consciousness that can leave the afflicted person appearing vacant. The electroencephalogram demonstrates a
pathognomonic spike and wave pattern.

Ethosuximide and valproate sodium are the drugs of choice for absence seizures. Lamotrigine can be used if
these
first line agents are unsuitable or not effective.

Next Question
Second
line therapies could include clobazam, clonazepam, leviteracetam amongst others. In general,
carbamazepine, gabapentin, phenytoin, and vigabatrin are not recommended for absence seizures.

[Ref BNF for children]


45263

https://mypastest.pastest.com/Secure/TestMe/Browser/436619[‫ ص‬05:05:36 10/12/1437]


MyPastest

Previous Question Tag Question

Feedback

Difficulty: Average

Peer Responses

Session Progress

Responses Correct: 0

Responses Incorrect: 24

Responses Total: 24

Responses - % Correct: 0%

Blog
About Pastest
Contact Us
Help

© Pastest 2016

End Session

https://mypastest.pastest.com/Secure/TestMe/Browser/436619[‫ ص‬05:05:36 10/12/1437]


MyPastest

Prefer to use the old MyPastest? Access it here »

Back to Filters

Question 19 of 86

An 12-year-old girl develops brief episodes of involuntary muscle twitching occurring early in the morning and
before falling asleep, leading to her frequently knocking over objects. Referral to a paediatrician was suggested
after she suffered a single grand mal seizure. There is a family history of several members of the family with
epilepsy syndromes.

What is the most appropriate therapy to commence?

A Carbamazepine

B Leviteracetam

C Nothing

D Topiramate

E Valproate sodium

Explanation
Juvenile myoclonic epilepsy is most common in females in their adolescent years. Myoclonic jerks precede
generalized tonic-clonic seizures in most patients by several months, though not all patients with juvenile
myoclonic epilepsy suffer generalised tonic-clonic seizures. There is a family history of epilepsy
syndromes in
many patients.

Valproate sodium is the drug of choice; alternatives include topiramate and leviteracetam.

Juvenile
myoclonic epilepsy requires treatment if it is interfering with lifestyle and carbamazepine is not
recommended for the treatment of myoclonic jerks.

[Ref BNF for children]


45264

https://mypastest.pastest.com/Secure/TestMe/Browser/436619[‫ ص‬05:05:57 10/12/1437]


MyPastest

Previous Question

Tag Question

Feedback

Difficulty: Average

Peer Responses

Session Progress

Responses Correct: 0

Responses Incorrect: 25

Responses Total: 25

Responses - % Correct: 0%

Blog
About Pastest
Contact Us
Help

© Pastest 2016

Next Question

End Session

https://mypastest.pastest.com/Secure/TestMe/Browser/436619[‫ ص‬05:05:57 10/12/1437]


MyPastest

Prefer to use the old MyPastest? Access it here »

Back to Filters

Question 20 of 86

An 8-year-old girl with epilepsy is found to have loss of consciousness, accompanied by lip smacking.
Electroencephalography demonstrated a spike and wave pattern. She has no other medical problems
or current
medications.

What is the most appropriate therapy to commence?

A Clonazepam

B Ethosuximide

C Gabapentin

D Lamotrigine

E Vigabatrin

Explanation
This is a classical description of absence seizures with automatic movements associated with brief periods of loss
of consciousness that can leave the afflicted person appearing vacant. The electroencephalogram demonstrates a
pathognomonic spike and wave pattern.

Ethosuximide and valproate sodium are the drugs of choice for absence seizures. Lamotrigine can be used if
these
first line agents are unsuitable or not effective.

Next Question
Second
line therapies could include clobazam, clonazepam, leviteracetam amongst others. In general,
carbamazepine, gabapentin, phenytoin, and vigabatrin are not recommended for absence seizures.

[Ref BNF for children]


45265

https://mypastest.pastest.com/Secure/TestMe/Browser/436619[‫ ص‬05:06:20 10/12/1437]


MyPastest

Previous Question Tag Question

Feedback

Difficulty: Average

Peer Responses

Session Progress

Responses Correct: 0

Responses Incorrect: 26

Responses Total: 26

Responses - % Correct: 0%

Blog
About Pastest
Contact Us
Help

© Pastest 2016

End Session

https://mypastest.pastest.com/Secure/TestMe/Browser/436619[‫ ص‬05:06:20 10/12/1437]


MyPastest

Prefer to use the old MyPastest? Access it here »

Back to Filters

Question 21 of 86

An 8-year-old girl is diagnosed with attention deficit hyperactivity disorder (ADHD).

What is the most appropriate therapy to commence?

A Atomexetine

B Citalopram

C Gabapentin

D Imipramine

E Methamphetamine

Explanation
Recommended management options include atomoxetine, dexamfetamine and methylphenidate.

Citalopram is used primarily for depression.

Gabapentin is used as an anti-epileptic and neuropathic analgesic.

Imipramine is a tri-cyclic antidepressant that can be used off-label in ADHD.

Methamphetamine is an illicit stimulant drug.

https://www.nice.org.uk/guidance/ta98
Next Question 45266

Tag Question

https://mypastest.pastest.com/Secure/TestMe/Browser/436619[‫ ص‬05:06:45 10/12/1437]


MyPastest

Previous Question
Feedback

Difficulty: Average

Peer Responses

Session Progress

Responses Correct: 0

Responses Incorrect: 27

Responses Total: 27

Responses - % Correct: 0%

Blog
About Pastest
Contact Us
Help

© Pastest 2016

End Session

https://mypastest.pastest.com/Secure/TestMe/Browser/436619[‫ ص‬05:06:45 10/12/1437]


MyPastest

Prefer to use the old MyPastest? Access it here »

Back to Filters

Question 22 of 86

An 8-year-old girl is diagnosed with attention deficit hyperactivity disorder (ADHD). She is commenced on
methylphenidate as per NICE guidance.

What are the most important parameters to be recorded at least 6 monthly or after each dose change?

A Glycated haemoglobin and fasting glucose

B Pulse and blood pressure

C Pupillary responses to light and accomodation

D QTc and PR interval

E Respiratoty rate and transcutaneous arterial oxygen saturation

Explanation
Recommended monitoring parameters include
6-monthly (or after each dose change) pulse and blood pressure as
the stimulant effect of methylphenidate can increase HR and BP as it is a sympathomimetic.

It has no effect on glycaemic indices.

It has no effect on pupillary responses that can used to assess therapeutic efficacy or toxicity.

It has no known effect on QT or PR segments of the ECG.

It is not known to affect respiration adversely. Next Question


[Ref BNF for children]
45267

https://mypastest.pastest.com/Secure/TestMe/Browser/436619[‫ ص‬05:07:07 10/12/1437]


MyPastest

Tag Question
Previous Question
Feedback

Difficulty: Average

Peer Responses

Session Progress

Responses Correct: 0

Responses Incorrect: 28

Responses Total: 28

Responses - % Correct: 0%

Blog
About Pastest
Contact Us
Help

© Pastest 2016

End Session

https://mypastest.pastest.com/Secure/TestMe/Browser/436619[‫ ص‬05:07:07 10/12/1437]


MyPastest

Prefer to use the old MyPastest? Access it here »

Back to Filters

Question 23 of 86

An 8-year-old girl is diagnosed with attention deficit hyperactivity disorder (ADHD). She is commenced on
methylphenidate as per NICE guidance.

What are the most important parameters to be recorded at least 6 monthly or after each dose change?

A Capillary refill time and jugular venous pressure

B Height and weight

C Liver enzymes and serum bilirubin

D Serum potassium and sodium

E Urinary frequency and flow rate

Explanation
Recommended monitoring parameters include
6-monthly (or after each dose change) height and weight as the
stimulant effect of methylphenidate can cause anorexia and increase metabolic rate leading to weight loss or
growth retardation.

It has no effect on fluid retention or cardiac output.

It has no effect on liver function tests or function.

It has no effect on serum electrolytes.


Next Question
It is not known to affect bladder function.
45268

https://mypastest.pastest.com/Secure/TestMe/Browser/436619[‫ ص‬05:07:28 10/12/1437]


MyPastest

Tag Question

Previous Question
Feedback

Difficulty: Average

Peer Responses

Session Progress

Responses Correct: 0

Responses Incorrect: 29

Responses Total: 29

Responses - % Correct: 0%

Blog
About Pastest
Contact Us
Help

© Pastest 2016

End Session

https://mypastest.pastest.com/Secure/TestMe/Browser/436619[‫ ص‬05:07:28 10/12/1437]


MyPastest

Prefer to use the old MyPastest? Access it here »

Back to Filters

Question 24 of 86

A 3-year-old boy presents to his primary care physician with cough and hoarseness which has been worse
particularly at night for the past 4 days with intermittent fevers.

Examination reveals a barking cough with mildly elevated respiratory rate and normal oxygen saturations and no
stridor or drooling.

What is the most appropriate therapy for this condition?

A Amoxicillin orally

B Dexamethasone orally

C Epinephrine by nebulisation

D Magnesium intravenously

E Salbutamol inhaled via spacer device

Explanation
This is a classical description of viral croup, which should be treated with oral, iv or nebulised steroid, with
nebulised epinephrine reserved for those who do not respond.

Nebulised epinephrine is reserved for those who do not respond to steroid treatment of croup.

Magnesium iv can be used in acute asthma.

Salbutamol inhaled can be used in acute asthma.

Amoxicillin is used to treat many suspected respiratory bacterial infections.

[Ref BNF for children]


45269

https://mypastest.pastest.com/Secure/TestMe/Browser/436619[‫ ص‬05:07:50 10/12/1437]


MyPastest

Previous Question

Tag Question

Feedback

Difficulty: Average

Peer Responses

Session Progress

Responses Correct: 0

Responses Incorrect: 30

Responses Total: 30

Responses - % Correct: 0%

Blog
About Pastest
Contact Us
Help

© Pastest 2016

Next Question

End Session

https://mypastest.pastest.com/Secure/TestMe/Browser/436619[‫ ص‬05:07:50 10/12/1437]


MyPastest

Prefer to use the old MyPastest? Access it here »

Back to Filters

Question 25 of 86

An 8-year-old boy presents to his primary care physician with


nocturnal cough and wheeze despite PRN (pro re
nata - as necessary) use
of salbutamol inhaler via a spacer. He is using his inhaler 4 times weekly on average.

What is the most appropriate next step in therapy for this condition?

A Amoxicillin orally

B Standard dose beclomethasone inhaler

C High dose budesonide inhaler

D Prednisolone orally

E Salmeterol inhaled

Explanation
He is currently at step 1 on the BTS ladder for chronic asthma. He should move to step 2, of which first line
is
standard dose inhaled steroid.

There are no signs of infection to warrant oral antibiotics.

High dose inhaled steroid is part of step 3.

Prednisolone orally would be indicated in acute asthma.

Next Question
Addition of long acting beta agonist is part of step 3

[Ref BTS guidelines for asthma]


45270

https://mypastest.pastest.com/Secure/TestMe/Browser/436619[‫ ص‬05:08:11 10/12/1437]


MyPastest

Tag Question
Previous Question
Feedback

Difficulty: Average

Peer Responses

Session Progress

Responses Correct: 0

Responses Incorrect: 31

Responses Total: 31

Responses - % Correct: 0%

Blog
About Pastest
Contact Us
Help

© Pastest 2016

End Session

https://mypastest.pastest.com/Secure/TestMe/Browser/436619[‫ ص‬05:08:11 10/12/1437]


MyPastest

Prefer to use the old MyPastest? Access it here »

Back to Filters

Question 26 of 86

A 2-year-old boy with asthma is taken to his local emergency department, by local taxi, with acute symptoms of
shortness of breath and wheeze. On assessment, he has oxygen saturations of 85% on room air and is agitated. He
has been treated with nebulized beta agonist and ipratropium and high flow oxygen without any improvement in
clinical state. He is now drowsy. He has received appropriate intravenous corticosteroid therapy.

What is the most appropriate next step in therapy for his condition?

A Vancomycin iv

B Salbutamol iv

C Aminophylline orally

D Epinephrine nebuliser

E Magnesium iv

Explanation
He has life threatening asthma for which is acute treatment always includes high flow oxygen, beta agonists and
ipratropium nebulized and corticosteroid orally or iv. Further treatments if there is not a rapid improvement can
include any of the following: nebulized magnesium; beta agonist iv or aminophylline iv.

There is no indication for routine antibiotics as most exacerbations are non infection related or viral in origin.

Aminophylline can be used by iv route.

Epinephrine nebulisers can be used in Croup.

Magnesium can used nebulized in young children in preference to iv.

[Ref BTS guidelines for asthma]


45271

https://mypastest.pastest.com/Secure/TestMe/Browser/436619[‫ ص‬05:08:32 10/12/1437]


MyPastest

Previous Question

Tag Question

Feedback

Difficulty: Average

Peer Responses

Session Progress

Responses Correct: 0

Responses Incorrect: 32

Responses Total: 32

Responses - % Correct: 0%

Blog
About Pastest
Contact Us
Help

© Pastest 2016

Next Question

End Session

https://mypastest.pastest.com/Secure/TestMe/Browser/436619[‫ ص‬05:08:32 10/12/1437]


MyPastest

Prefer to use the old MyPastest? Access it here »

Back to Filters

Question 27 of 86

A 5-year-old boy with a history of penicillin allergy is mistakenly given co-amoxiclav orally for a middle ear
infection. Shortly
after his first oral dose, he developes wheeze, rash, and feels unwell.
He is rushed to the local
emergency department where his BP is unrecordable on admission. The diagnosis of anaphylaxis is made. He
requires an immediate 150 micrograms of epinephrine 1:1000.

Which of the following volumes/routes is most appropriate?

A 0.015 mL iv

B 0.15 mL im

C 1.5 mL nebulised

D 15mL via endo-tracheal tube

E 150mL orally

Explanation
1:1000 dilution means 1mg in 1mL. Immediate management of anaphylaxis is intra-muscular as first line
[0.15mL].

iv route can be used by experienced practitioners with appropriate monitoring but this amount will only give
15mcg.

Nebulised route is not appropriate.

ET route can used in cardiac arrest but this dose is excessive [normally 10x iv route].

Oral route is inappropriate.

[Ref BNF for children]

https://mypastest.pastest.com/Secure/TestMe/Browser/436619[‫ ص‬05:09:28 10/12/1437]


MyPastest

45272

Previous Question

Tag Question

Feedback

Difficulty: Average

Peer Responses

Session Progress

Responses Correct: 0

Responses Incorrect: 33

Responses Total: 33

Responses - % Correct: 0%

Blog
About Pastest
Contact Us
Help

© Pastest 2016

Next Question

End Session

https://mypastest.pastest.com/Secure/TestMe/Browser/436619[‫ ص‬05:09:28 10/12/1437]


MyPastest

Prefer to use the old MyPastest? Access it here »

Back to Filters

Question 28 of 86

A 5-year-old boy with a history of anaphylaxis to penicillin requires intravenous antibiotics for a gram-positive
skin infection.

Which of the medications would be considered safe and appropriate?

A Gentamicin

B Co-amoxiclav

C Cefuroxime

D Tazocin®

E Clarithromycin

Explanation
In true penicillin anaphylaxis, both penicillins and cephalosporins should be avoided. Clarithromycin is a
macrolide and has good gram positive cover.

Coamoxiclav contains amoxicillin and is therefore contraindicated here.

Gentamicin is a aminoglycoside and has no gram positive cover on its own.

Cefuroxime is a beta lactam containing cephalosporin and is therefore contraindicated.

Next Question
Tazocin® contains piperacillin and is therefore contra-indicated and should be prescribed as
piperacillin/tazobactam to avoid confusion

[Ref BNF for children]


45273

https://mypastest.pastest.com/Secure/TestMe/Browser/436619[‫ ص‬05:09:49 10/12/1437]


MyPastest

Previous Question Tag Question

Feedback

Difficulty: Average

Peer Responses

Session Progress

Responses Correct: 0

Responses Incorrect: 34

Responses Total: 34

Responses - % Correct: 0%

Blog
About Pastest
Contact Us
Help

© Pastest 2016

End Session

https://mypastest.pastest.com/Secure/TestMe/Browser/436619[‫ ص‬05:09:49 10/12/1437]


MyPastest

Prefer to use the old MyPastest? Access it here »

Back to Filters

Question 29 of 86

A 4-month-old boy is up-to-date with the current UK immunisation schedule. He has no significant medical
history.

Which combination of vaccines are currently recommended at 4 months of age?

A Diphtheria, tetanus, pertussis, polio, haemophilus influenzae type b, meningococcal group B,


Pneumococcal (13 serotypes)

B Diphtheria, pertussis, polio, haemophilus influenzae type b, meningococcal group B, rotavirus, BCG

C Diphtheria, tetanus, pertussis, polio, haemophilus influenzae type b, rotavirus, influenza

D Diphtheria, tetanus, pertussis, polio, haemophilus influenzae type b, rotavirus, measles, mumps,
rubella [MMR]

E Diphtheria, tetanus, pertussis, polio, haemophilus influenzae type b, meningococcal groups ACYW,
rotavirus

Explanation
The current immunisation schedule for the
UK at 4 months is: diphtheria, tetanus, pertussis, polio, haemophilus
influenzae type b, meningococcal group B, pneumococcal (13 serotypes).

BCG
is not routine; it is recommended at birth for children where one parent/grandparent is from a high incidence
country (>4/10000 of the population have TB) or were born in a high incidence area.

Influenza is recommended in ‘flu season to pregnant women and persons with specific medical conditions and to
all >65 year olds and
children between 2-6 MMR is first given at 12 months.

Meningococcal groups ACYW is given at 14 years of age.

[Ref www.gov.uk]
45274

https://mypastest.pastest.com/Secure/TestMe/Browser/436619[‫ ص‬05:10:09 10/12/1437]


MyPastest

Previous Question

Tag Question

Feedback

Difficulty: Average

Peer Responses

Session Progress

Responses Correct: 0

Responses Incorrect: 35

Responses Total: 35

Responses - % Correct: 0%

Blog
About Pastest
Contact Us
Help

© Pastest 2016

Next Question

End Session

https://mypastest.pastest.com/Secure/TestMe/Browser/436619[‫ ص‬05:10:09 10/12/1437]


MyPastest

Prefer to use the old MyPastest? Access it here »

Back to Filters

Question 30 of 86

A 13-year-old boy is about to start year 9 at school. He is up-to-date with the current UK immunisation schedule.
He has no significant medical history.

Which combination of vaccines are currently recommended to be given at this age?

A Diphtheria, tetanus, polio, meningococcal groups ACYW

B Diphtheria, pertussis, polio, BCG

C Pneumococcal, influenza, shingles

D Human papilloma virus 6,11,16 and 18, meningococcal groups B and C

E Influenza, measles, mumps, rubella [MMR]

Explanation
The current immunisation schedule for the UK in year 9 is diphtheria, tetanus, polio, meningococcal groups
ACYW.

Pertussis is not recommended in adolescence. BCG is not routine; it is recommended at birth for children where
one parent/grandparent is from a
high incidence country (>4/10000 of the population have TB) or were born in a
high incidence area.

Influenza is recommended in ‘flu season to pregnant women and persons with specific medical conditions and to
all >65 year olds and children between 2-6. Shingles vaccine is recommended to 70y old. Pneumococcal vaccine
is recommended at 2, 4 and 12 months and 65 years old.

HPV is only given to girls currently at 12-13 years old. Meningococcal B and C are only given together at 12
months of age.

MMR
is first given at 12 months and again at 40 months. Influenza is recommended in ‘flu season to pregnant
women and persons with specific medical conditions and to all >65 year olds and children between 2-6.

https://mypastest.pastest.com/Secure/TestMe/Browser/436619[‫ ص‬05:10:30 10/12/1437]


MyPastest

www.gov.uk
45275
Previous Question

Tag Question

Feedback

Difficulty: Average

Peer Responses

Session Progress

Responses Correct: 0

Responses Incorrect: 36

Responses Total: 36

Responses - % Correct: 0%

Blog
About Pastest
Contact Us
Help

© Pastest 2016

Next Question

End Session

https://mypastest.pastest.com/Secure/TestMe/Browser/436619[‫ ص‬05:10:30 10/12/1437]


MyPastest

Prefer to use the old MyPastest? Access it here »

Back to Filters

Question 31 of 86

A 13-year-old boy presents to his local emergency department with headache, neck stiffness and fever.

Which is the most appropriate empirical therapy?

A Cefotaxime iv

B Cefuroxime iv

C Ceftazidime iv

D Chloramphenicol iv

E Ampicillin iv

Explanation
The clinical description suggests meningitis. Management from 3months to 18 years empirically until organism
and sensitivities are known are either cefotaxime or ceftriaxone iv.

Cefuroxime does not cover the likely causative agents sufficiently in acute bacterial meningitis.

Ceftazidime does not cover the likely causative agents sufficiently in acute bacterial meningitis.

Chloramphenicol is a useful alternative if beta-lactam allergic Ampicillin iv is used as an adjunct in neonates to


cover for Listeria spp.

[Ref BNF for children] Next Question


45276

https://mypastest.pastest.com/Secure/TestMe/Browser/436619[‫ ص‬05:10:51 10/12/1437]


MyPastest

Tag Question

Previous Question
Feedback

Difficulty: Average

Peer Responses

Session Progress

Responses Correct: 0

Responses Incorrect: 37

Responses Total: 37

Responses - % Correct: 0%

Blog
About Pastest
Contact Us
Help

© Pastest 2016

End Session

https://mypastest.pastest.com/Secure/TestMe/Browser/436619[‫ ص‬05:10:51 10/12/1437]


MyPastest

Prefer to use the old MyPastest? Access it here »

Back to Filters

Question 32 of 86

A 2-month-old boy presents to his local emergency department floppy with fever and a spreading, non-blanching
petechial rash.

Which is the most appropriate empirical therapy?

A Ceftriaxone iv + ampiciiin iv

B Cefotaxime iv + vancomycin iv

C Cefotaxime iv + amoxicillin po

D Chloramphenicol iv

E Co-trimoxazole iv

Explanation
The clinical description suggests meningitis. Management from <3 months needs to cover for Listeria spp.
Recommended iv antibiotic therapy is either ceftriaxone or cefotaxime + ampicillin or amoxicllin

Vancomycin
can be added to dual therapy if recent prolonged use of antibiotics or recent travel to high beta-
lactam resistance area.

Oral amoxicillin therapy is not appropriate for meningitis.

Chloramphenicol is a useful alternative if beta-lactam allergic but does not cover Listeria spp.
Next Question
Co-trimoxazole is useful in beta-lactam allergy in neonates to cover for confirmed Listeria spp. meningitis

[Ref BNF for children]


45277

https://mypastest.pastest.com/Secure/TestMe/Browser/436619[‫ ص‬05:11:11 10/12/1437]


MyPastest

Previous Question Tag Question

Feedback

Difficulty: Average

Peer Responses

Session Progress

Responses Correct: 0

Responses Incorrect: 38

Responses Total: 38

Responses - % Correct: 0%

Blog
About Pastest
Contact Us
Help

© Pastest 2016

End Session

https://mypastest.pastest.com/Secure/TestMe/Browser/436619[‫ ص‬05:11:11 10/12/1437]


MyPastest

Prefer to use the old MyPastest? Access it here »

Back to Filters

Question 33 of 86

A 4-year-old girl presents to her primary care physician with


low grade fever and itchy patches over her cheeks,
chin and neck, that have areas of erythema and widespread golden-honey coloured crusting. Her father informs
you that several other children in her nursery are similarly affected currently.

Which is the most appropriate empirical therapy?

A Flucloxacillin orally

B Mupirocin topically

C Clindamycin orally

D Azithromycin orally

E Phenoxymethylpenicillin orally

Explanation
The clinical description suggests impetigo which is typically a staphylococcal aureus infection that responds to
oral flucloxacillin.

Topical antibiotics are only indicated when there are isolated small patches of affected skin without systemic
involvement.

Clindamycin is indicated for erysipelas in beta lactam allergic children.

Azithromycin is an alternative in beta lactam allergic children with impetigo.

Phenoxymethylpenicillin is used for likely streptococcal infections, such as erysipelas.

[Ref BNF for children]


45278

https://mypastest.pastest.com/Secure/TestMe/Browser/436619[‫ ص‬05:11:56 10/12/1437]


MyPastest

Previous Question

Tag Question

Feedback

Difficulty: Average

Peer Responses

Session Progress

Responses Correct: 0

Responses Incorrect: 39

Responses Total: 39

Responses - % Correct: 0%

Blog
About Pastest
Contact Us
Help

© Pastest 2016

Next Question

End Session

https://mypastest.pastest.com/Secure/TestMe/Browser/436619[‫ ص‬05:11:56 10/12/1437]


MyPastest

Prefer to use the old MyPastest? Access it here »

Back to Filters

Question 34 of 86

A 9-year-old girl presents to her primary care physician with


itchy, red, watering eyes. On examination, there is
yellow mucous discharge of the eyelid margin with injected and oedematous conjunctiva.

Which is the most appropriate empirical therapy?

A Chloramphenicol topically

B Erythromycin orally

C Cefotaxime iv

D Hypromellose topically

E Flucloxacillin orally

Explanation
The clinical description suggests bacterial conjunctivitis which is typically a staphylococcal aureus infection that
responds to topical chloramphenicol.

Oral erythromycin is reserved for congenital chlamydial conjunctivitis.

Iv antibiotics such as cefotaxime are reserved for congenital gonococcal conjunctivitis.

Clindamycin is indicated for erysipelas in beta lactam allergic children.

Hypromellose is useful for dry eyes.

Flucloxacillin orally is not necessary for bacterial conjunctivitis.

[Ref BNF for children]


45279

https://mypastest.pastest.com/Secure/TestMe/Browser/436619[‫ ص‬05:12:19 10/12/1437]


MyPastest

Previous Question

Tag Question

Feedback

Difficulty: Average

Peer Responses

Session Progress

Responses Correct: 0

Responses Incorrect: 40

Responses Total: 40

Responses - % Correct: 0%

Blog
About Pastest
Contact Us
Help

© Pastest 2016

Next Question

End Session

https://mypastest.pastest.com/Secure/TestMe/Browser/436619[‫ ص‬05:12:19 10/12/1437]


MyPastest

Prefer to use the old MyPastest? Access it here »

Back to Filters

Question 35 of 86

A 10-year-old girl is treated with broad spectrum antibiotics


for a post-operative anastomotic leak following
surgery for Hirschsprung’s disease. One week later, she develops fever, severe
abdominal pain and bloody
diarrhoea. Initial stool testing reveals positivity for glutamate dehydrogenase [GDH].

Which is the most appropriate treatment?

A Metronidazole orally

B Erythromycin orally

C Ciprofloxacin orally

D Loperamide orally

E Racecadotril orally

Explanation
GDH is a screening test that is performed
to rapidly identify faecal samples that are likely to represent Clostridium
difficile-associated diarrhoea. The clinical description suggests Clostridium difficile-associated diarrhoea
following broad spectrum antibiotic use and first line therapy is oral metronidazole.

Oral erythromycin is pro-motility and has no role in Clostridium difficile-associated diarrhoea.

Ciprofloxacin is occasionally required for bacterial enteritis caused by non- Clostridium difficile organisms such
as Campylobacter or Salmonella.

Loperamide is an anti-motility agent


that is not recommended for routine use in infective diarrhoea syndromes or
in children < 12-y old.

Racecadotril orally is licensed, as an adjunct to rehydration, for the symptomatic treatment of uncomplicated acute
diarrhoea in children >3m-old.

https://mypastest.pastest.com/Secure/TestMe/Browser/436619[‫ ص‬05:13:33 10/12/1437]


MyPastest

[Ref BNF for children]


45280

Previous Question

Tag Question

Feedback

Difficulty: Average

Peer Responses

Session Progress

Responses Correct: 0

Responses Incorrect: 41

Responses Total: 41

Responses - % Correct: 0%

Blog
About Pastest
Contact Us
Help

© Pastest 2016

Next Question

End Session

https://mypastest.pastest.com/Secure/TestMe/Browser/436619[‫ ص‬05:13:33 10/12/1437]


MyPastest

Prefer to use the old MyPastest? Access it here »

Back to Filters

Question 36 of 86

A 15-year-old girl with type-1 diabetes mellitus is diagnosed


with Clostridium difficile-associated diarrhoea
following a prolonged antibacterial treatment schedule for cellulitis. She has failed to respond to first line
treatment with oral metronidazole. She is allergic
to penicillins and aminoglycosides.

Which is the most next step in treatment?

A Vancomycin orally

B Pooled faecal transplant

C Metronidazole iv

D Vancomycin iv

E Chlorhexidine via NG tube

Explanation
Second line treatment for Clostridium difficile-associated diarrhoea is oral vancomycin.

Pooled
faecal transplant is effective for the management for resistant Clostridium difficile-associated diarrhoea but
has yet to be tested in children and so should only be used under specialist direction.

Metronidazole iv is third line as it is secreted into the bowel as part


of elimination Iv vancomycin will not reach
the GI tract.
Next Question
There is no indication to attempt sterilization of the GI tract with antiseptic agents.

[Ref BNF for children]


45281

https://mypastest.pastest.com/Secure/TestMe/Browser/436619[‫ ص‬05:13:58 10/12/1437]


MyPastest

Tag Question
Previous Question
Feedback

Difficulty: Average

Peer Responses

Session Progress

Responses Correct: 0

Responses Incorrect: 42

Responses Total: 42

Responses - % Correct: 0%

Blog
About Pastest
Contact Us
Help

© Pastest 2016

End Session

https://mypastest.pastest.com/Secure/TestMe/Browser/436619[‫ ص‬05:13:58 10/12/1437]


MyPastest

Prefer to use the old MyPastest? Access it here »

Back to Filters

Question 37 of 86

A 15-year-old girl with type-1 diabetes mellitus presents with vomiting and tachypnoea. Her capillary blood
glucose is 17.1 mmol/L; arterial pH is 7.2, [K+] is 4.5 mmol/L and she has 3+ ketones in urine. She is not shocked.
She weighs 60kg.

Which is the most appropriate initial fluid prescription?

A 500mL 0.9% saline + 20mmol KCl iv; rate 100mL/h

B 1L 0.9% saline + 20mmol KCl iv; rate 125 mL/h

C 500mL water orally over 2h

D 500mL 1.26% sodium bicarbonate iv; rate 50mL/h

E 600mL hydroxyethylstarch iv; rate STAT

Explanation
She has mild DKA (diabetic ketoacidosis) as the pH is >7.1 – she is estimated to have 5% volume loss. Initial
fluid therapy is based of the BSPED (British Society for Paediatric Endocrinology and Diabetes) formula:

Estimated loss/48 + restricted maintenance fluid requirements per hour.


Here, 5% of 60kg = 3000mL
3000/48 = 62.5 mL
Restricted maintenance fluids for a child in DKA >40kg is 40mL/h
Therefore total required is 62.5+40 = 102.5 [approx. 100mL/h]
The recommended fluid is 0.9% saline with at least 40mmol KCl per litre unless the patient is totally anuric.

Answer b has not enough potassium and is too high a rate.

Answer c is wrong as oral fluid rehydration is unreliable.

Sodium bicarbonate should not be used in paediatric DKA.

https://mypastest.pastest.com/Secure/TestMe/Browser/436619[‫ ص‬05:14:20 10/12/1437]


MyPastest

Colloid is not better than crystalloid and is not recommended. Bolus fluid should only be used in initial
management if the patient is shocked [10ml/kg 0.9% saline iv]
Previous Question
http://www.bsped.org.uk/about/about_committees.aspx
45282

Tag Question

Feedback

Difficulty: Average

Peer Responses

Session Progress

Responses Correct: 0

Responses Incorrect: 43

Responses Total: 43

Responses - % Correct: 0%

Blog
About Pastest
Contact Us
Help

© Pastest 2016
Next Question

End Session

https://mypastest.pastest.com/Secure/TestMe/Browser/436619[‫ ص‬05:14:20 10/12/1437]


MyPastest

Prefer to use the old MyPastest? Access it here »

Back to Filters

Question 38 of 86

A 15-year-old girl with type-1 diabetes mellitus presents with vomiting and tachypnoea. Her capillary blood
glucose is 17.1 mmol/L; arterial pH is 7.2, [K+] is 4.5 mmol/L and she has 3+
ketones in urine. She is not shocked.
She weighs 60kg. She is normally on glargine insulin 10u nocte and mealtime soluble insulin adjusted for
carbohydrate load of her meals.

Which is the most appropriate initial insulin prescription?

A 6u/h soluble insulin iv, started 1h after fluids initiated

B Immediate bolus 10u soluble insulin iv

C Immediate 5u glargine insulin sc

D No insulin required

E Continue basal/bolus regime to coincide with mealtimes

Explanation
Fixed-rate insulin [0.05 or 0.1 u/h] is the preferred insulin regime, to be started 1- 2h post iv fluid initiation.

Boluses are not recommended any more.

If a patient is on a long acting insulin, this should be continued as normal, as well as the fixed-rate insulin.

Insulin is required to stop ketogenesis.


Next Question
It is not appropriate simply to continue the pre-existing regime in DKA.

[Ref bsped.org.uk]
45283

https://mypastest.pastest.com/Secure/TestMe/Browser/436619[‫ ص‬05:14:42 10/12/1437]


MyPastest

Previous Question Tag Question

Feedback

Difficulty: Average

Peer Responses

Session Progress

Responses Correct: 0

Responses Incorrect: 44

Responses Total: 44

Responses - % Correct: 0%

Blog
About Pastest
Contact Us
Help

© Pastest 2016

End Session

https://mypastest.pastest.com/Secure/TestMe/Browser/436619[‫ ص‬05:14:42 10/12/1437]


MyPastest

Prefer to use the old MyPastest? Access it here »

Back to Filters

Question 39 of 86

A 15-year-old girl with type-1 diabetes mellitus is an in-patient following an appendicectomy. She complains of
feeling anxious
and sweaty. She is alert and not confused. Her capillary blood glucose is measured at 3.8 mmol/L.
She has no iv access. She weighs 50kg and is currently on a basal/bolus regime and has been eating well today.

Which is the most appropriate management?

A Oral glucose gel, 75g orally

B Diet Lucozade®, 50mL orally

C Reduce current lnsulin prescription by 25%

D Glucagon 1mg im

E Cannulate and give 10mL 50% glucose iv

Explanation
She has symptomatic hypoglycaemia without
neurological stupor and therefore oral treatment with at least 10g
glucose (this contains 30g) is appropriate.

Diet drinks have no glucose and so will not raise blood glucose.

Reducing future insulin will not treat current hypoglycaemia.

Glucagon im is reserved for patients not responding to oral treatment or those unable to take oral glucose or
receive iv glucose quickly.

50%
glucose is not recommended as it is hypertonic and is very irritant if extravasation occurs. 10% is the most
appropriate glucose strength for iv replacement.

[Ref BNF for children]


45285

https://mypastest.pastest.com/Secure/TestMe/Browser/436619[‫ ص‬05:15:07 10/12/1437]


MyPastest

Previous Question

Tag Question

Feedback

Difficulty: Average

Peer Responses

Session Progress

Responses Correct: 0

Responses Incorrect: 45

Responses Total: 45

Responses - % Correct: 0%

Blog
About Pastest
Contact Us
Help

© Pastest 2016

Next Question

End Session

https://mypastest.pastest.com/Secure/TestMe/Browser/436619[‫ ص‬05:15:07 10/12/1437]


MyPastest

Prefer to use the old MyPastest? Access it here »

Back to Filters

Question 40 of 86

A 15-year-old girl with type-1 diabetes mellitus is an in-patient following an appendicectomy. She complains of
feeling anxious
and sweaty. She is alert and not confused. Her capillary blood glucose is measured at 3.8 mmol/L.
She has no iv access. She weighs 50kg and is currently on a basal/bolus regime and has been eating well today.

Which of the following prescriptions will ensure she receives at least 10g glucose to raise her blood glucose?

A 100mL 10% glucose infusion fluid

B 1 teaspoon sugar orally

C 1 lump sugar orally

D 10mL neat Ribena Blackcurrant®, diluted into 100mL water

E 10mL 50% glucose infusion fluid

Explanation
Recommended treatments for hypoglycaemia should contain 10-20g glucose. 10% glucose has 100 mg glucose
per 1 ml fluid, so 100mL is 10g.

2 teaspoons of sugar is estimated to contain 10g glucose.

3 lumps of sugar is estimated to contain 10g glucose.

19mL Ribena Blackcurrant® is estimated to contain 10g glucose.


Next Question
50% glucose has only 500 mg glucose per 1 ml fluid, so 10mL is only 5g.

[Ref BNF for children]


45286

https://mypastest.pastest.com/Secure/TestMe/Browser/436619[‫ ص‬05:15:27 10/12/1437]


MyPastest

Previous Question Tag Question

Feedback

Difficulty: Average

Peer Responses

Session Progress

Responses Correct: 0

Responses Incorrect: 46

Responses Total: 46

Responses - % Correct: 0%

Blog
About Pastest
Contact Us
Help

© Pastest 2016

End Session

https://mypastest.pastest.com/Secure/TestMe/Browser/436619[‫ ص‬05:15:27 10/12/1437]


MyPastest

Prefer to use the old MyPastest? Access it here »

Back to Filters

Question 41 of 86

A neonate has severe respiratory distress syndrome and has a significant left-to-right shunt due to a patent ductus
arteriosus (PDA).

Which of the following medications is the most appropriate treatment?

A Ibuprofen iv

B Indometacin orally

C Hyperbaric oxygen therapy

D Paracetamol per rectal

E Aspirin orally

Explanation
Ibuprofen has less nephrotoxic and gastrotoxic effects at the doses used for PDA, than equivalent indomethacin
doses.

Indometacin is the most commonly used treatment for PDA but is iv.

Hyperbaric oxygen therapy is most commonly used for carbon monoxide poisoning.

Paracetamol has no role in PDA management.

Next Question
Oral aspirin is indicated in Kawasaki disease and other thrombotic cardiovascular diseases

[Ref BNF for children]


45287

https://mypastest.pastest.com/Secure/TestMe/Browser/436619[‫ ص‬05:15:47 10/12/1437]


MyPastest

Tag Question
Previous Question
Feedback

Difficulty: Average

Peer Responses

Session Progress

Responses Correct: 0

Responses Incorrect: 47

Responses Total: 47

Responses - % Correct: 0%

Blog
About Pastest
Contact Us
Help

© Pastest 2016

End Session

https://mypastest.pastest.com/Secure/TestMe/Browser/436619[‫ ص‬05:15:47 10/12/1437]


MyPastest

Prefer to use the old MyPastest? Access it here »

Back to Filters

Question 42 of 86

A 12-year-old boy is being trained with an enuresis alarm but needs to go on a school trip away from home for 2
nights.

What is the most appropriate management option?

A Desmopressin orally at night

B Imipramine orally

C Fluid restriction

D Bendroflumethiazide orally

E Tolvaptan orally

Explanation
Desmopressin is indicated (orally or SL(sublingual)) for short term relief or for those in whom an alarm is not
suitable or not working.

Tricyclic antidepressants (TCAs) can be used under specialist supervision for enuresis as second line
pharmacotherapy.

Fluid restriction is not advised.

Bendroflumethiazide is a diuretic and will increase urine production.

Tolvaptan
increases free water excretion and so is the opposite of the desired effect wanted here by increasing
urine production.

[Ref BNF for children]


45289

https://mypastest.pastest.com/Secure/TestMe/Browser/436619[‫ ص‬05:16:07 10/12/1437]


MyPastest

Previous Question

Tag Question

Feedback

Difficulty: Average

Peer Responses

Session Progress

Responses Correct: 0

Responses Incorrect: 48

Responses Total: 48

Responses - % Correct: 0%

Blog
About Pastest
Contact Us
Help

© Pastest 2016

Next Question

End Session

https://mypastest.pastest.com/Secure/TestMe/Browser/436619[‫ ص‬05:16:07 10/12/1437]


MyPastest

Prefer to use the old MyPastest? Access it here »

Back to Filters

Question 43 of 86

When prescribing medications in children with significant renal impairment which of the most following
statements is most accurate when compared to prescribing in children with normal renal
function?

A Loading doses remain unchanged

B Renally excreted drugs must be totally avoided

C It will take an increased number of half-lives to achieve steady state

D Dose adjustment is required for all medications

E Dose adjustment is best done related to serum creatinine

Explanation
Loading doses are related to volume of distribution which largely remains unchanged in renal disease.

Not all drugs that are in part renally excreted need to be avoided as sometimes there is no option i.e
aminoglycosides in endocarditis.

The half life will be increased but the steady state pharmacokinetic principle of first order metabolised drugs
remains that steady state is achieved by 4-5 half lives even in renal impairment.

eGFR is a better guide to real GFR than serum creatinine, which is inaccurate at extremes of size for age.

[Ref BNF for children]


Next Question 45291

Tag Question

https://mypastest.pastest.com/Secure/TestMe/Browser/436619[‫ ص‬05:17:00 10/12/1437]


MyPastest

Previous Question
Feedback

Difficulty: Average

Peer Responses

Session Progress

Responses Correct: 0

Responses Incorrect: 49

Responses Total: 49

Responses - % Correct: 0%

Blog
About Pastest
Contact Us
Help

© Pastest 2016

End Session

https://mypastest.pastest.com/Secure/TestMe/Browser/436619[‫ ص‬05:17:00 10/12/1437]


MyPastest

Prefer to use the old MyPastest? Access it here »

Back to Filters

Question 44 of 86

A 12-year-old boy with metastatic bone cancer is being treated with morphine for intractable bone pain. He is
requiring 30mg total daily of immediate release morphine to control his symptoms adequately. He is struggling to
take medications by mouth due to pharyngeal metastases. It is decided to convert him to diamorphine via syringe
driver sub-cutaneously over 24h.

What is the most appropriate daily dose of diamorphine to control his symptoms?

A 10mg

B 15mg

C 20mg

D 30mg

E 60mg

Explanation
10mg morphine is approx. 3mg sub-cutaneous diamorphine. Therefore he needs approx. 10mg diamorphine daily
over 24h via driver.

15mg is too high

20mg is too high

30mg is too high

60mg is too high

[Ref BNF for children]


45293

https://mypastest.pastest.com/Secure/TestMe/Browser/436619[‫ ص‬05:19:28 10/12/1437]


MyPastest

Previous Question

Tag Question

Feedback

Difficulty: Average

Peer Responses

Session Progress

Responses Correct: 0

Responses Incorrect: 50

Responses Total: 50

Responses - % Correct: 0%

Blog
About Pastest
Contact Us
Help

© Pastest 2016

Next Question

End Session

https://mypastest.pastest.com/Secure/TestMe/Browser/436619[‫ ص‬05:19:28 10/12/1437]


MyPastest

Prefer to use the old MyPastest? Access it here »


Previous Question

Back to Filters

Question 45 of 86

What is the leading type of road traffic accident (as a victim) leading to death for a child under the age of 15?

A Pedestrian

B Car passenger

C Pedal cyclist

D Car driver

E On a motorbike

Explanation
Child pedestrians account for the majority of RTA deaths.

Pedal cyclists are the second most common victimology for RTAs.

Due to seat belt laws and improvement in car safety, car deaths are not that common.

Very few children die whilst driving car as they are under age.

Very few children die whilst on motorbikes as they are underage and rarely ride pillion.

[Ref Office National Statistics]


45294

Next Question

Tag Question

Feedback

https://mypastest.pastest.com/Secure/TestMe/Browser/436619[‫ ص‬05:19:51 10/12/1437]


MyPastest

Difficulty: Average

Peer Responses

Session Progress

Responses Correct: 0

Responses Incorrect: 51

Responses Total: 51

Responses - % Correct: 0%

Blog
About Pastest
Contact Us
Help

© Pastest 2016

End Session

https://mypastest.pastest.com/Secure/TestMe/Browser/436619[‫ ص‬05:19:51 10/12/1437]


MyPastest

Prefer to use the old MyPastest? Access it here »

Back to Filters

Question 46 of 86

A 12-year-old boy is being treated for Attention Deficit Hyperactivity disorder (ADHD) with imipramine.
Imipramine is licensed for use in children for nocturnal enuresis only.

What is the best description of the use of imipramine in this child?

A Off-label

B Inappropriate

C Illegal

D Use of an unlicensed medication

E Investigational medicinal product

Explanation
Imipramine is licensed but is used for a different indication – this if off label use.

It is recommended for use in ADHD as an alternative and is not inappropriate.

It is recommended for use in ADHD as has a licence for use in the UK and is not subject to legal restrictions and is
therefore legal.

It is licensed IMPs are medications under evaluation in clinical trials prior to licensing.

[Ref BNF for children] Next Question


45298

https://mypastest.pastest.com/Secure/TestMe/Browser/436619[‫ ص‬05:20:11 10/12/1437]


MyPastest

Tag Question

Previous Question
Feedback

Difficulty: Average

Peer Responses

Session Progress

Responses Correct: 0

Responses Incorrect: 52

Responses Total: 52

Responses - % Correct: 0%

Blog
About Pastest
Contact Us
Help

© Pastest 2016

End Session

https://mypastest.pastest.com/Secure/TestMe/Browser/436619[‫ ص‬05:20:11 10/12/1437]


MyPastest

Prefer to use the old MyPastest? Access it here »

Back to Filters

Question 47 of 86

A 12-year-old boy with auto-immune hypoadrenalism is diagnosed with epilepsy and started on carbamazepine.
He is on regular corticosteroid replacement, 3 times daily.

What is the the most appropriate dose adjustment required for his corticosteroid replacement?

A Increase each dose, keep three times daily

B Same dose, reduce to twice daily

C No change required

D Reduced dose, twice daily

E Add an extra dose at night

Explanation
Carbamazepine is an enzyme inducer and leads enhanced metabolism, therefore higher doses are required. There
is
no need to change dosing schedule.

Corticosteroid replacement is designed to replicate the natural circadian rhythm of cortisol and is most commonly
given 3 times daily.

His steroid requirement will increase as he is now prescribed an enzyme inducer.

Corticosteroid replacement is designed to replicate the natural circadian rhythm of cortisol and is most commonly
given 3 times daily. His steroid requirement will increase as he is now prescribed an enzyme inducer.

Extra steroid at night can disturb sleep and will not cover him over a 24h period.

[Ref BNF for children]


45299

https://mypastest.pastest.com/Secure/TestMe/Browser/436619[‫ ص‬05:21:13 10/12/1437]


MyPastest

Previous Question

Tag Question

Feedback

Difficulty: Average

Peer Responses

Session Progress

Responses Correct: 0

Responses Incorrect: 53

Responses Total: 53

Responses - % Correct: 0%

Blog
About Pastest
Contact Us
Help

© Pastest 2016

Next Question

End Session

https://mypastest.pastest.com/Secure/TestMe/Browser/436619[‫ ص‬05:21:13 10/12/1437]


MyPastest

Prefer to use the old MyPastest? Access it here »

Back to Filters

Question 48 of 86

A 9-year-old boy with a renal transplant develops TB, presumably as a reactivation due to immunospuppression
with ciclosporin.
He is started on standard quadruple therapy. His ciclosporin level is found to be substantially
sub-therapeutic at next check 3 weeks later.

What is most likely explanation for the dose increase in ciclosporin?

A Drug-induced hepatotoxicity by isoniazid

B Poor adherence

C Reduced absorption due to incompatibility of multiple oral agents

D Induction of liver enzymes by rifampicin

E Inhibition of liver enzymes by ethambutol

Explanation
Rifampicin is an enzyme inducer and leads
enhanced metabolism and therefore lower levels of ciclosporin that
peaks 2-3 weeks after drug initation.

Although poor adherence should always be interrogated, it is less likely than the
expected pharmacokinetic action
of rifampicin.

If these medications were incompatible, they would be contra-indicated for concomitant use.

INH-hepatotoxicity would if anything increase ciclosporin levels.

Ethambutol does not affect liver function and enzyme inhibition would lead to higher ciclosporin levels.

[Ref BNF for children]


45300

https://mypastest.pastest.com/Secure/TestMe/Browser/436619[‫ ص‬05:21:35 10/12/1437]


MyPastest

Previous Question

Tag Question

Feedback

Difficulty: Average

Peer Responses

Session Progress

Responses Correct: 0

Responses Incorrect: 54

Responses Total: 54

Responses - % Correct: 0%

Blog
About Pastest
Contact Us
Help

© Pastest 2016

Next Question

End Session

https://mypastest.pastest.com/Secure/TestMe/Browser/436619[‫ ص‬05:21:35 10/12/1437]


MyPastest

Prefer to use the old MyPastest? Access it here »

Back to Filters

Question 49 of 86

A 15-year-old boy presents with pulmonary oedema following a protracted febrile illness associated with weight
loss. On examination he is clubbed and has a loud pan-systolic murmur and basal crepitations.
He is diagnosed
with infective endocarditis following presumptive blood
culture results and transoesophageal echocardiography.

He is started on iv antibiotics including iv gentamicin. 1 week later he complains of progressive hearing loss.

Which of his other medications has a pharmacodynamics interaction with iv gentamicin to predispose to
ototoxicity?

A Iv flucloxacillin

B Iv furosemide

C Sc low molecular weight heparin

D Oral paracetamol

E Oral ibuprofen

Explanation
Furosemide and gentamicin can both cause ototoxicity and the risk is enhanced when both are coprescribed.

Flucloxacillin does not cause ototoxicity; it can cause cholestatic jaundice.

LMWH do not cause ototoxicity; they can cause low platelets.

Paracetamol does not cause ototoxicity.

Ibuprofen does not cause ototoxicity; it can cause gastric ulceration and fluid retention.

[Ref BNF for children]


45301

https://mypastest.pastest.com/Secure/TestMe/Browser/436619[‫ ص‬05:21:54 10/12/1437]


MyPastest

Previous Question

Tag Question

Feedback

Difficulty: Average

Peer Responses

Session Progress

Responses Correct: 0

Responses Incorrect: 55

Responses Total: 55

Responses - % Correct: 0%

Blog
About Pastest
Contact Us
Help

© Pastest 2016

Next Question

End Session

https://mypastest.pastest.com/Secure/TestMe/Browser/436619[‫ ص‬05:21:54 10/12/1437]


MyPastest

Prefer to use the old MyPastest? Access it here »

Back to Filters

Question 50 of 86

A 12-year-old girl is diagnosed with juvenile idiopathic arthritis and started on weekly methotrexate.

Which of the following medications should be avoided?

A Allopurinol orally

B Diclofenac orally

C Folinic acid orally

D Morphine orally

E Paracetamol orally

Explanation
NSAIDs reduce methotrexate excretion and therefore increase toxicity and should be avoided if possible.

Paracetamol has no interaction with MTX.

Opiates have no interaction with MTX.

Allopurinol has no interaction with MTX.

Methotrexate is an anti-folate drug and therefore the patient should be folic acid replete or supplemented with
folinic acid.

[Ref BNF for children] Next Question


45302

https://mypastest.pastest.com/Secure/TestMe/Browser/436619[‫ ص‬05:22:14 10/12/1437]


MyPastest

Tag Question

Previous Question
Feedback

Difficulty: Average

Peer Responses

Session Progress

Responses Correct: 0

Responses Incorrect: 56

Responses Total: 56

Responses - % Correct: 0%

Blog
About Pastest
Contact Us
Help

© Pastest 2016

End Session

https://mypastest.pastest.com/Secure/TestMe/Browser/436619[‫ ص‬05:22:14 10/12/1437]


MyPastest

Prefer to use the old MyPastest? Access it here »

Back to Filters

Question 51 of 86

A 12-year-old girl with heterozygous familial hypercholesterolaemia on high dose atorvastatin is admitted with
wheeze,
fever and productive cough. She is found to have a community acquired pneumonia and is started on
treatment.

Which of the following medications would necessitate a temporary cessation of statin therapy?

A Amoxicillin orally

B Clarithromycin orally

C Enoxaparin sc

D Paracetamol iv

E Salbutamol by nebuliser

Explanation
Clarithromycin is a potent CYP 450 3A4 inhibitor and is contraindicated for concomitant use with statins as it
increases the risk of rhabdomyolysis.

Amoxicillin has no ineraction with atorvastatin.

Salbutamol has no ineraction with atorvastatin.

Enoxaparin has no ineraction with atorvastatin.


Next Question
Paracetamol has no ineraction with atorvastatin.

[Ref BNF for children]


45303

https://mypastest.pastest.com/Secure/TestMe/Browser/436619[‫ ص‬05:22:35 10/12/1437]


MyPastest

Previous Question Tag Question

Feedback

Difficulty: Average

Peer Responses

Session Progress

Responses Correct: 0

Responses Incorrect: 57

Responses Total: 57

Responses - % Correct: 0%

Blog
About Pastest
Contact Us
Help

© Pastest 2016

End Session

https://mypastest.pastest.com/Secure/TestMe/Browser/436619[‫ ص‬05:22:35 10/12/1437]


MyPastest

Prefer to use the old MyPastest? Access it here »

Back to Filters

Question 52 of 86

A 14-year-old girl with cystic fibrosis is admitted with an infective exacerbation. She is profoundly wheezy and is
started on iv aminophylline as she has had little response to nebulised salbutamol. Her sputum culture is positive
for Pseudomonal spp and her empirical anti-bacterial therapy is due to be altered.

Which of the following anti-bacterial therapies should not be considered?

A Azithromycin orally

B Ceftazidime iv

C Ciprofloxacin iv + nebulised colistin

D Imipenem iv

E Piperacillin/tazobactam iv + tobramycin iv

Explanation
Ciprofloxacin is a potent enzyme inhibitor and enhances the serum levels of theophyllines and predisposes
to
cardiac toxicities.

Azithromycin orally has anti-pseudomonal activity and no interaction with theophyllines.

Ceftazidime iv has anti-pseudomonal activity and no interaction with theophyllines.

Piperacillin/tazobactam iv + tobramycin iv have anti-pseudomonal activity and no interaction with theophyllines.


Next Question
Imipenem iv has anti-pseudomonal activity and no interaction with theophyllines.

[Ref BNF for children]


45304

https://mypastest.pastest.com/Secure/TestMe/Browser/436619[‫ ص‬05:23:13 10/12/1437]


MyPastest

Previous Question Tag Question

Feedback

Difficulty: Average

Peer Responses

Session Progress

Responses Correct: 0

Responses Incorrect: 58

Responses Total: 58

Responses - % Correct: 0%

Blog
About Pastest
Contact Us
Help

© Pastest 2016

End Session

https://mypastest.pastest.com/Secure/TestMe/Browser/436619[‫ ص‬05:23:13 10/12/1437]


MyPastest

Prefer to use the old MyPastest? Access it here »

Back to Filters

Question 53 of 86

A 15-year-old boy is currently well treated with azathioprine for Crohn’s disease. He has a history of gout.

For
which of the following new medications he could be started on would he need to dose-reduce azathioprine to a
quarter of his usual dose?

A Allopurinol

B Indometacin

C Paracetamol

D Febuxostat

E Codeine

Explanation
Allopurinol is an anti-gout medication that enhances effects and increases toxicity of azathioprine so it is
mandated to reduce the dose of azathioprine to one quarter of usual dose. Allopurinol inhibits xanthine oxidase,
which usually inactivates the active metabolite 6-MCP.

Manufacturer of azathioprine advises possible increased risk of myelosuppression when azathioprine is given with
indomethacin but no dose reduction is mandated.

Paracetamol has no interaction with azathioprine.

Febuxostat is contraindicated with azathioprine.

Codeine has no interaction with azathioprine.

[Ref BNF for children]


45306

https://mypastest.pastest.com/Secure/TestMe/Browser/436619[‫ ص‬05:24:15 10/12/1437]


MyPastest

Previous Question

Tag Question

Feedback

Difficulty: Average

Peer Responses

Session Progress

Responses Correct: 0

Responses Incorrect: 59

Responses Total: 59

Responses - % Correct: 0%

Blog
About Pastest
Contact Us
Help

© Pastest 2016

Next Question

End Session

https://mypastest.pastest.com/Secure/TestMe/Browser/436619[‫ ص‬05:24:15 10/12/1437]


MyPastest

Prefer to use the old MyPastest? Access it here »

Back to Filters

Question 54 of 86

A 14-year-old girl is admitted to the emergency department with regular fast palpitations and dizziness. On
assessment, she has a regular, narrow-complex tachycardia at a rate of 200 bpm. She has no adverse features or
significant medical history.

Which of the following management options is the most appropriate and effective first line management?

A Facial immersion with ice cold water

B Adenosine 3mg iv

C Valsalva maneouvre

D Carotid sinus massage

E Synchronised DC cardioversion

Explanation
Ice-water to the face is considered to be a safe,
quick, effective and non-invasive treatment for paediatric
supraventricular tachycardia (SVT).

Adenosine also appears safe and effective but is more invasive and therefore is second line.

Valsalva and carotid sinus massage are less effective and DC cardioversion is reserved those who fail medication
treatment.

http://www.bestbets.org/bets/bet.php?id=2798
Next Question
45307

https://mypastest.pastest.com/Secure/TestMe/Browser/436619[‫ ص‬05:24:36 10/12/1437]


MyPastest

Tag Question

Previous Question
Feedback

Difficulty: Average

Peer Responses

Session Progress

Responses Correct: 0

Responses Incorrect: 60

Responses Total: 60

Responses - % Correct: 0%

Blog
About Pastest
Contact Us
Help

© Pastest 2016

End Session

https://mypastest.pastest.com/Secure/TestMe/Browser/436619[‫ ص‬05:24:36 10/12/1437]


MyPastest

Prefer to use the old MyPastest? Access it here »

Back to Filters

Question 55 of 86

A 14-year-old girl is admitted to the emergency department with regular fast palpitations and dizziness. On
assessment, she has a regular, narrow-complex tachycardia at a rate of 200 bpm. She spontaneously cardioverts
with ice-cold facial immersion. This is her third episode in one year requiring emergency department attendance.
She
has never had adverse features and has a past medical history of asthma. She is not any oral medications
currently.

What is the most appropriate prophylactic management?

A Verapamil 40mg three times daily orally

B Amiodarone 200mg daily orally

C Sotalol 40mg twice daily orally

D Digoxin 62.5 mcg daily orally

E Adenosine 6mg PRN orally

Explanation
Verapamil is indicated for prophylaxis for supraventricular tachycardia (SVT).

Amiodarone
has many adverse effects and has a very long half-life. Oral once daily
dosing will not achieve steady
state for potentially 1 year and so is ineffective prescribed like this and is best avoided as maintenance therapy in
children if possible.

Sotalol would be first line but as the girl is asthmatic so it is contra-indicated.

Digoxin is not recommended for SVT.

Adenosine can not be used PRN (pro re nata - (when necessary)) and PRN use in any case is not prophylactic but
reactive.

https://mypastest.pastest.com/Secure/TestMe/Browser/436619[‫ ص‬05:24:57 10/12/1437]


MyPastest

Ref BNF for children


45309

Previous Question

Tag Question

Feedback

Difficulty: Average

Peer Responses

Session Progress

Responses Correct: 0

Responses Incorrect: 61

Responses Total: 61

Responses - % Correct: 0%

Blog
About Pastest
Contact Us
Help

© Pastest 2016

Next Question

End Session

https://mypastest.pastest.com/Secure/TestMe/Browser/436619[‫ ص‬05:24:57 10/12/1437]


MyPastest

Prefer to use the old MyPastest? Access it here »

Back to Filters

Question 56 of 86

A 12-year-old girl on warfarin for deep vein thrombosis (DVT)


has a routine INR check. She has no alarming
symptoms of bleeding. Her INR is 9.0.

What is the most appropriate management?

A Stop warfarin; give phytomenadione orally; restart warfarin sodium when INR <5.0

B Withhold 1 or 2 doses of warfarin and reduce subsequent maintenance dose

C Stop warfarin; give phytomenadione iv; restart warfarin when INR <5.0

D Stop warfarin; give phytomenadione and prothrombin complex iv

E Switch to phenindione

Explanation
She has no evidence of minor or major bleeding. Current recommendations would be to stop warfarin and give
oral vitamin K, only restarting the medication when INR <5.0.

Withholding 1 or 2 doses of warfarin and reducing subsequent maintenance dose is the management when INR 5-
8 and no bleeding.

Stopping warfarin and giving vitamin K iv and restarting warfarin when INR <5.0 is applicable when INR >5 and
minor bleeding.

Vitamin K and prothrombin complex iv are indicated in any form of major bleeding on warfarin irrespective of the
INR.

Changing to another coumarin is inappropriate.

Ref BNF for children


45310

https://mypastest.pastest.com/Secure/TestMe/Browser/436619[‫ ص‬05:25:34 10/12/1437]


MyPastest

Previous Question

Tag Question

Feedback

Difficulty: Average

Peer Responses

Session Progress

Responses Correct: 0

Responses Incorrect: 62

Responses Total: 62

Responses - % Correct: 0%

Blog
About Pastest
Contact Us
Help

© Pastest 2016

Next Question

End Session

https://mypastest.pastest.com/Secure/TestMe/Browser/436619[‫ ص‬05:25:34 10/12/1437]


MyPastest

Prefer to use the old MyPastest? Access it here »

Back to Filters

Question 57 of 86

A 16-year-old girl on warfarin for deep vein thrombosis (DVT)


presents with large volume haematuria with clots.
Her urine dip shows blood only. She has no symptoms of UTI. Her INR is 3.5.

What is the most appropriate management?

A Stop warfarin; give phytomenadione orally; restart warfarin sodium when INR <5.0

B Withhold 1 or 2 doses of warfarin and reduce subsequent maintenance dose

C Stop warfarin; give phytomenadione iv; restart warfarin when INR <5.0

D Stop warfarin; give phytomenadione and prothrombin complex iv

E Oral tranexamic acid

Explanation
Vitamin K and prothrombin complex iv are indicated in any form of major bleeding on warfarin irrespective of the
INR, such as large volume extra-glomerular bleeding in this case.

If INR >8 with no bleeding, current recommendations would be oral vitamin K and restart when INR <5.0.

Withholding 1 or 2 doses of warfarin and reducing subsequent maintenance dose is the management when INR 5-
8 and no bleeding.

Stopping warfarin and giving vitamin K iv and restarting warfarin when INR <5.0 is applicable when INR >5 and
minor bleeding.

Although tranexamic acid can be used as a haemostatic, without reversing the INR it will not suffice.

Ref BNF for children


45311

https://mypastest.pastest.com/Secure/TestMe/Browser/436619[‫ ص‬05:25:54 10/12/1437]


MyPastest

Previous Question

Tag Question

Feedback

Difficulty: Average

Peer Responses

Session Progress

Responses Correct: 0

Responses Incorrect: 63

Responses Total: 63

Responses - % Correct: 0%

Blog
About Pastest
Contact Us
Help

© Pastest 2016

Next Question

End Session

https://mypastest.pastest.com/Secure/TestMe/Browser/436619[‫ ص‬05:25:54 10/12/1437]


MyPastest

Prefer to use the old MyPastest? Access it here »

Back to Filters

Question 58 of 86

A 13-year-old girl is given treatment-dose enoxaparin for high risk orthopaedic surgery. Despite this, she develops
a post-operative deep vein thrombosis (DVT).

What blood test is most useful to check the anti-coagulant effect of enoxaparin?

A Prothrombin time

B Bleeding time

C Activated partial thromoboplastin time

D Anti factor Xa level

E Factor VIII levels

Explanation
Anti factor Xa level is the only blood test that can confirm the anticoagulant effect of LMWHs, which usually do
not require routine monitoring.

Prothrombin time is measure of the extrinsic coagulation pathway, most commonly used to assess warfarin effect.

Activated partial thromoboplastin time is a measure of the intrinsic coagulation pathway, most commonly used to
assess unfractionated heparin
effect.

Bleeding time is an obsolete test of platelet function.


Next Question
Factor VIII levels is used to diagnosis Haemophilia A.

(Ref BNF for children)


45312

https://mypastest.pastest.com/Secure/TestMe/Browser/436619[‫ ص‬05:26:27 10/12/1437]


MyPastest

Previous Question Tag Question

Feedback

Difficulty: Average

Peer Responses

Session Progress

Responses Correct: 0

Responses Incorrect: 64

Responses Total: 64

Responses - % Correct: 0%

Blog
About Pastest
Contact Us
Help

© Pastest 2016

End Session

https://mypastest.pastest.com/Secure/TestMe/Browser/436619[‫ ص‬05:26:27 10/12/1437]


MyPastest

Prefer to use the old MyPastest? Access it here »

Back to Filters

Question 59 of 86

A 15-year-old girl is given metoclopramide iv in the emergency department for nausea and vomiting. Shortly after
receiving the drug, she develops severe torsion of her neck and fixed upwards gaze. She is conscious and
distressed.

What is the most appropriate management?

A High flow oxygen

B Procyclidine iv

C Activated oral charcoal

D Diazepam per rectal

E Re-breathing in a paper bag

Explanation
This describes a severe anti-dopaminergic dystonic reaction to metoclopramide. Management is with an anti-
muscarinic agent.

Oxygen has no role here, other than it can abort cluster headache.

Oral charcoal is useful in most oral overdoses that present within 1 hour of ingestion.

Diazepam pr is recommended for community management of status epilepticus.


Next Question
Re-breathing in a paper bag is a mechanism to abort psychological hyperventilation.

(Ref BNF for children)


45313

https://mypastest.pastest.com/Secure/TestMe/Browser/436619[‫ ص‬05:26:47 10/12/1437]


MyPastest

Previous Question Tag Question

Feedback

Difficulty: Average

Peer Responses

Session Progress

Responses Correct: 0

Responses Incorrect: 65

Responses Total: 65

Responses - % Correct: 0%

Blog
About Pastest
Contact Us
Help

© Pastest 2016

End Session

https://mypastest.pastest.com/Secure/TestMe/Browser/436619[‫ ص‬05:26:47 10/12/1437]


MyPastest

Prefer to use the old MyPastest? Access it here »

Back to Filters

Question 60 of 86

A 14-year-old girl with a metallic mitral valve replacement is being prepared for major spinal surgery. She is
started on iv unfractionated heparin and is due to have a APTTr check in 6 hours post initiation. 15min after
starting the infusion, she develops a severe epistaxis that does not terminate with manual pressure.

What is the most appropriate reversal agent for unfractionated heparin anticoagulation?

A Protamine iv

B Vitamin K iv

C Prothrombin complex iv

D Tranexamic acid iv

E Pooled platelet transfusion

Explanation
Protamine is the specific reversal agent for unfractioned heparin (UFH).

Vitamin K is a moderately quick method to reverse the effect of warfarin.

Prothrombin complex is an extremely quick method to reverse the effect of warfarin.

Tranexamic acid is a haemostatic agent and will not reverse the APTTr.

Next Question
Platelet transfusion is rarely needed in any situation if the platelet count is >20 [150-400] and is not a reversal
agent.

(Ref BNF for children)


45314

https://mypastest.pastest.com/Secure/TestMe/Browser/436619[‫ ص‬05:27:08 10/12/1437]


MyPastest

Previous Question Tag Question

Feedback

Difficulty: Average

Peer Responses

Session Progress

Responses Correct: 0

Responses Incorrect: 66

Responses Total: 66

Responses - % Correct: 0%

Blog
About Pastest
Contact Us
Help

© Pastest 2016

End Session

https://mypastest.pastest.com/Secure/TestMe/Browser/436619[‫ ص‬05:27:08 10/12/1437]


MyPastest

Prefer to use the old MyPastest? Access it here »

Back to Filters

Question 61 of 86

A 5-year-old boy with epilepsy secondary to hydrocephalus with an abdominal shunt in situ presents with a
seizure that has lasted 15 minutes by the time he arrives in the emergency department. He has already had
treatment with rectal diazepam in the community.

What is the most appropriate next step in management?

A Lorazepam iv

B Midazolam orally

C Phenytoin iv

D Propofol iv

E Paracetamol orally

Explanation
Benzodiazepine treatment is the first line for status eplipeticus and should at least be repeated once before moving
on to the next treatment choice.

Midazolam buccally is an option for status; oral treatment risks aspiration.

Phenytoin iv is the next step once benzodiazepine therapy has been exhausted.

Propofol has no role in status eplipeticus.


Next Question
Paracetamol is used to prevent ongoing febrile convulsions in young children.

(Ref BNF for children)


45315

https://mypastest.pastest.com/Secure/TestMe/Browser/436619[‫ ص‬05:27:43 10/12/1437]


MyPastest

Previous Question Tag Question

Feedback

Difficulty: Average

Peer Responses

Session Progress

Responses Correct: 0

Responses Incorrect: 67

Responses Total: 67

Responses - % Correct: 0%

Blog
About Pastest
Contact Us
Help

© Pastest 2016

End Session

https://mypastest.pastest.com/Secure/TestMe/Browser/436619[‫ ص‬05:27:43 10/12/1437]


MyPastest

Prefer to use the old MyPastest? Access it here »

Back to Filters

Question 62 of 86

A 15-year-old boy is diagnosed with a bleeding duodenal ulcer after presenting with haematemesis. Biopsy
confirms H. Pylori infection. He has no significant medical history.

What is the most appropriate treatment?

A Amoxicillin, clarithromycin for 1 week; omeprazole for 4 weeks

B Amoxicilin, metronidazole and omeprazole for 2 weeks

C Amoxicillin, clarithromycin for 2 weeks; lifelong omeprazole

D Amoxicillin, omeprazole for 1 week

E Amoxicillin, metronidazole, omeprazole for 1 week

Explanation
H. pylori eradication should contain only 2 of amoxicillin, clarithromycin or metronidazole for 1 week preferably.
Proton pump inhibitors should be a time-limited course: ideally 1 week unless presents with haematemesis, in
which case 4
weeks is recommended.

A 2 week course of triple


therapy is no longer recommended as first line PPIs should be time-limited and 2 week
courses of antibiotics are not recommended anymore.

Dual therapy is not recommended at all.

Amoxicillin, metronidazole, omeprazole for 1 week would be appropriate If he did not present with haematemesis;
PPI should be for 4 weeks then stopped.

(Ref BNF for children)


45316

https://mypastest.pastest.com/Secure/TestMe/Browser/436619[‫ ص‬05:28:06 10/12/1437]


MyPastest

Previous Question

Tag Question

Feedback

Difficulty: Average

Peer Responses

Session Progress

Responses Correct: 0

Responses Incorrect: 68

Responses Total: 68

Responses - % Correct: 0%

Blog
About Pastest
Contact Us
Help

© Pastest 2016

Next Question

End Session

https://mypastest.pastest.com/Secure/TestMe/Browser/436619[‫ ص‬05:28:06 10/12/1437]


MyPastest

Prefer to use the old MyPastest? Access it here »

Back to Filters

Question 63 of 86

A 15-year-old boy presents with bradycardia and hypotension following a deliberate overdose of his mother’s
atenolol.

What is the most appropriate first line treatment?

A Atropine iv

B Epinephrine im

C Salbutamol nebulised

D Glucagon iv

E Insulin/dextrose infusion

Explanation
Although it is unlikely to reverse a significant beta blocker overdose on its own, atropine is first line treatment for
all bradycardias associated with haemodynamic compromise.

Epinephrine im is used exclusively for anaphylaxis.

Salbutamol will have no effect on heart rate in beta blocker overdose and all beta receptors are blocked in excess.

Glucagon iv is second line treatment.

Next Question
High dose insulin/dextrose infusion is recommended third line treatment
by NPIS though should only be used
under specialist supervision.

(Ref BNF for children; Toxbase.org)


45317

https://mypastest.pastest.com/Secure/TestMe/Browser/436619[‫ ص‬05:28:27 10/12/1437]


MyPastest

Previous Question Tag Question

Feedback

Difficulty: Average

Peer Responses

Session Progress

Responses Correct: 0

Responses Incorrect: 69

Responses Total: 69

Responses - % Correct: 0%

Blog
About Pastest
Contact Us
Help

© Pastest 2016

End Session

https://mypastest.pastest.com/Secure/TestMe/Browser/436619[‫ ص‬05:28:27 10/12/1437]


MyPastest

Prefer to use the old MyPastest? Access it here »

Back to Filters

Question 64 of 86

A 15-year-old boy presents with bradycardia and hypotension following a deliberate overdose of his mother’s
atenolol. His is given atropine with little effect.

What is the most appropriate next step in treatment?

A Insulin bolus iv

B Isoprenaline iv

C Salbutamol iv

D Glucagon bolus iv

E External pacing

Explanation
Glucagon iv is second line treatment, as it bypasses the beta receptor and increases cAMP via other mechanisms in
cardiac myocytes to increase force and rate of contraction.

Isoprenaline is an option in beta blocker overdose due to its beta-1 and beta-2 agonist activity, it is rarely effective.

Salbutamol will have no effect on heart rate in beta blocker overdose and all beta- 2 receptors are blocked in
excess.

Glucagon iv is second line treatment.

Beta blocker overdose renders the myocardium relatively insensitive to electrical stimulation and very high
thresholds are required that can usually only be achieved by invasive pacing.

(Ref BNF for children; toxbase.org)


45318

https://mypastest.pastest.com/Secure/TestMe/Browser/436619[‫ ص‬05:28:51 10/12/1437]


MyPastest

Previous Question

Tag Question

Feedback

Difficulty: Average

Peer Responses

Session Progress

Responses Correct: 0

Responses Incorrect: 70

Responses Total: 70

Responses - % Correct: 0%

Blog
About Pastest
Contact Us
Help

© Pastest 2016

Next Question

End Session

https://mypastest.pastest.com/Secure/TestMe/Browser/436619[‫ ص‬05:28:51 10/12/1437]


MyPastest

Prefer to use the old MyPastest? Access it here »

Back to Filters

Question 65 of 86

A 5-year-old boy presents with diffuse abdominal pain following accidental ingestion of iron sulfate tablets from
his father’s supply.

What is the a specific treatment to enhance elimination of iron tablets?

A Desferrioxamine iv

B Whole bowel irrigation

C Multi-dose activated charcoal orally

D Dimercaprol iv

E Sodium bicarbonate iv

Explanation
Desferrioxamine is the specific chelator for significant iron dose cases.

Whole bowel irrigation is rarely indicated; it is occasionally used with body packers and would reduce absorption
rather than increase elimination in any case.

MDAC is used with drugs that recirculate via the entero-hepatic circulation; charcoal cannot bind iron.

Dimercaprol is an obsolete treatment for heavy metal poisoning.

Next Question
Sodium bicarbonate is used in TCA overdose to manage cardiac problems and in salicylate overdose to enhance
urinary elimination.

(Ref BNF for children; toxbase.org)


45319

https://mypastest.pastest.com/Secure/TestMe/Browser/436619[‫ ص‬05:29:36 10/12/1437]


MyPastest

Previous Question Tag Question

Feedback

Difficulty: Average

Peer Responses

Session Progress

Responses Correct: 0

Responses Incorrect: 71

Responses Total: 71

Responses - % Correct: 0%

Blog
About Pastest
Contact Us
Help

© Pastest 2016

End Session

https://mypastest.pastest.com/Secure/TestMe/Browser/436619[‫ ص‬05:29:36 10/12/1437]


MyPastest

Prefer to use the old MyPastest? Access it here »

Back to Filters

Question 66 of 86

A 5-year-old boy presents with diffuse abdominal pain, microcytic anaemia and increasing disruptive behaviour
over the past 3 months since moving into a new family home. He is a small, yet otherwise
healthy child with no
abnormalities on physical examination.

What is the most likely explanation?

A Chronic lead exposure

B Depression

C Lyme disease

D Petit mal epilepsy

E Zollinger-Ellison syndrome

Explanation
Lead poisoning:

Abdominal pain with no physical findings


Neuropsychaitric features
Microcytic anaemia
± blue gum line (v rare)

Organic causes should always be sought prior to psychiatric ones; depression could not explain the anaemia.

Lyme disease can cause a range of different abnormalities depending on the stage at diagnosis; there should be a
history of tick bite or spreading rash and microcytosis requires an alternative diagnosis.
ZE syndrome presents with recurrent, widespread gastric ulcers but does not typically affect behaviour.

https://mypastest.pastest.com/Secure/TestMe/Browser/436619[‫ ص‬05:29:58 10/12/1437]


MyPastest

Petit mal is described as absence episodes where the child will appear vacant and then pick up exactly where they
left off.
Previous Question
(Ref BNF for children; toxbase.org)
45320

Tag Question

Feedback

Difficulty: Average

Peer Responses

Session Progress

Responses Correct: 0

Responses Incorrect: 72

Responses Total: 72

Responses - % Correct: 0%

Blog
About Pastest
Contact Us
Help

© Pastest 2016
Next Question

End Session

https://mypastest.pastest.com/Secure/TestMe/Browser/436619[‫ ص‬05:29:58 10/12/1437]


MyPastest

Prefer to use the old MyPastest? Access it here »

Back to Filters

Question 67 of 86

A 10-year-old boy with bone cancer mistakenly takes his newly


prescribed MST continus, several times within the
first morning he has it, on a PRN basis. He becomes increasingly drowsy and is brought to the local emergency
department. On arrival, he is on high flow oxygen and his cardiac observations are normal. Oxygen saturations are
100% on oxygen, RR 4/min and GCS is calculated to be 5/15.

What is the most appropriate management?

A Flumazenil iv

B Intubate and ventilate until wears off

C Let sleep off

D Naxoxone im

E Naloxone infusion iv

Explanation
MST continus is a long acting version of morphine; naloxone infusion titrated to neurological and respiratory
response will be required as one off treatments will wear off and the opiate toxidrome will recur.

He is unable to reliably maintain a safe airway with such a low conscious level; the long half life of this version of
morphine means it will be many hours before the effects wear off so he should not be left to sleep it off.

Intubation is an option if there is no appropriate response to naloxone.

Flumazenil can be used in confirmed benzodiazepine overdose

(Ref BNF for children; toxbase.org)


45321

https://mypastest.pastest.com/Secure/TestMe/Browser/436619[‫ ص‬05:30:20 10/12/1437]


MyPastest

Previous Question

Tag Question

Feedback

Difficulty: Average

Peer Responses

Session Progress

Responses Correct: 0

Responses Incorrect: 73

Responses Total: 73

Responses - % Correct: 0%

Blog
About Pastest
Contact Us
Help

© Pastest 2016

Next Question

End Session

https://mypastest.pastest.com/Secure/TestMe/Browser/436619[‫ ص‬05:30:20 10/12/1437]


MyPastest

Prefer to use the old MyPastest? Access it here »

Back to Filters

Question 68 of 86

A 14-year-old boy takes a mixed overdose of unknown medications from his parents' and grandparents' prescribed
medications.

For which medication is multi-dose activated charcoal administration not recommended to enhance elimination
management?

A Carbamazepine

B Phenobarbital

C Quinine

D Theophylline

E Tramadol

Explanation
Tramadol in overdose gives both the opiate toxidrome, managed by naloxone and can give serotoninergic
symptoms that may conversely require small doses of benzodiazepines. The
predominant toxidrome should be
managed first.

Carbamazepine undergoes entero-hepatic recirculation and therefore MDAC is recommended.

Phenobarbital undergoes entero-hepatic recirculation and therefore MDAC is recommended.

Quinine undergoes entero-hepatic recirculation and therefore MDAC is recommended.

Theophylline undergoes entero-hepatic recirculation and therefore MDAC is recommended.

(Ref BNF for children; Toxbase.org)


45322

https://mypastest.pastest.com/Secure/TestMe/Browser/436619[‫ ص‬05:31:07 10/12/1437]


MyPastest

Previous Question

Tag Question

Feedback

Difficulty: Average

Peer Responses

Session Progress

Responses Correct: 0

Responses Incorrect: 74

Responses Total: 74

Responses - % Correct: 0%

Blog
About Pastest
Contact Us
Help

© Pastest 2016

Next Question

End Session

https://mypastest.pastest.com/Secure/TestMe/Browser/436619[‫ ص‬05:31:07 10/12/1437]


MyPastest

Prefer to use the old MyPastest? Access it here »

Back to Filters

Question 69 of 86

An 11-year-old girl takes an unwitnessed overdose of aspirin;


she denies taking anything else, though there were
many other medications belonging to her parents at home.

Which clinical feature is not consistent with the salicylate toxidrome and should make you think of other co-
ingestions?

A Clonus

B Hyperventilation

C Metabolic acidosis

D Tinnitus

E Pulmonary oedema

Explanation
The main features of salicylate poisoning
are hyperventilation, tinnitus, deafness, vasodilatation, and sweating.
Coma and pulmonary oedema are uncommon but indicate very severe poisoning. The associated acid-base
disturbances are complex. Clonus is a
classic feature of a serotoninergic overdose, such as amphetamines.

Initial respiratory alkalosis due to direct stimulation of the respiratory centres is followed by metabolic acidosis
due to uncoupling of oxidative phosphorylation and ketoacid formation due to fatty acid metabolism.

(Ref BNF for children; toxbase.org) Next Question


45323

https://mypastest.pastest.com/Secure/TestMe/Browser/436619[‫ ص‬05:31:41 10/12/1437]


MyPastest

Tag Question

Previous Question
Feedback

Difficulty: Average

Peer Responses

Session Progress

Responses Correct: 0

Responses Incorrect: 75

Responses Total: 75

Responses - % Correct: 0%

Blog
About Pastest
Contact Us
Help

© Pastest 2016

End Session

https://mypastest.pastest.com/Secure/TestMe/Browser/436619[‫ ص‬05:31:41 10/12/1437]


MyPastest

Prefer to use the old MyPastest? Access it here »

Back to Filters

Question 70 of 86

An 11-year-old girl takes an unwitnessed overdose of aspirin;


she denies taking anything else, though there were
many other medications belonging to her parents at home.

Which of the following management options is never indicated in aspirin overdose?

A Intralipid iv

B Oral activated charcoal

C Iv sodium bicarbonate

D Haemodialysis

E Supportive management and repeat levels

Explanation
Intralipid is useful in fat-soluble overdoses (such as local anaesthetic overdose) that are resistant to other
treatment. Aspirin is water soluble, hence its suitability for haemodialytic removal.

Oral activated charcoal is indicated within 1 hour of overdose to prevent absorption. Iv sodium bicarbonate
alkalinises the urine, which enhances the solubility of aspirin in water/urine to aid elimination.

Haemodialysis is useful in severe overdose (convulsions, pulmonary oedema, persistent acidosis, levels >700
mg/L) as aspirin is water soluble and so should be almost entirely removed within one circuit of dialysis.

Absorption of aspirin may be slow and the plasma-salicylate concentration may continue to rise for several hours,
requiring repeated measurement.

(Ref BNF for children; Toxbase.org)


45324

https://mypastest.pastest.com/Secure/TestMe/Browser/436619[‫ ص‬05:32:02 10/12/1437]


MyPastest

Previous Question

Tag Question

Feedback

Difficulty: Average

Peer Responses

Session Progress

Responses Correct: 0

Responses Incorrect: 76

Responses Total: 76

Responses - % Correct: 0%

Blog
About Pastest
Contact Us
Help

© Pastest 2016

Next Question

End Session

https://mypastest.pastest.com/Secure/TestMe/Browser/436619[‫ ص‬05:32:02 10/12/1437]


MyPastest

Prefer to use the old MyPastest? Access it here »

Back to Filters

Question 71 of 86

An 11-year-old girl takes a deliberate, unknown number of paracetamol tablets but immediately regrets it and calls
999. She arrives in the hospital 2 hours after consuming all the tablets in one go. She has a history of HIV and is
on antiretroviral therapy though does not know the name.

Which of the following management options is most appropriate?

A Arrange immediate haemodialysis

B Arrange transfer for acute liver transplant

C Check paracetamol level in further 2 hours' time

D Check paracetamol level now

E Give full treatment dose N-acetylcysteine iv now

Explanation
Between 4-15 hours the normogram is very accurate and levels should not be checked prior to 4 hours post
ingestion.

2 hours post ingestion is too early to check the level; 4 hours post is
the earliest checks can be made. Treatment
with NAC can be delayed until the 4 hour level is returned and compared to the nomogram.

Haemodialysis is only indicated if she develops hepato-renal syndrome that is likely to occur 72-96 hours post
ingestion.

Indications for acute liver transplantation include persistant acidosis, hepatorenal syndrome and worsening
coagulopathy.

(Ref BNF for children; Toxbase.org)


45325

https://mypastest.pastest.com/Secure/TestMe/Browser/436619[‫ ص‬05:32:25 10/12/1437]


MyPastest

Previous Question

Tag Question

Feedback

Difficulty: Average

Peer Responses

Session Progress

Responses Correct: 0

Responses Incorrect: 77

Responses Total: 77

Responses - % Correct: 0%

Blog
About Pastest
Contact Us
Help

© Pastest 2016

Next Question

End Session

https://mypastest.pastest.com/Secure/TestMe/Browser/436619[‫ ص‬05:32:25 10/12/1437]


MyPastest

Prefer to use the old MyPastest? Access it here »

Back to Filters

Question 72 of 86

An 11-year-old girl takes a deliberate, unknown number of paracetamol tablets but regrets it and calls 999. She
arrives in the hospital at 5am - 2 hours after consuming the last of her paracetamol tablets. She started taking the
first of the tablets at midnight. She has no significant medical history.

What of the following management options is most appropriate?

A Check paracetamol level in further 2h time

B Check paracetamol level now

C Give full treatment dose N-acetylcysteine iv now

D Discharge if LFTs and INR are normal

E Vitamin K iv

Explanation
In staggered overdose (tablets taken over
a longer time frame than 1 hour), paracetamol levels are not useful to
guide whether to treat or not; unless you doubt the history in which case a 'zero’ level would mean there was no
paracetamol ingestion.
In staggered overdose estimated to be >75 mg/kg or of unknown amount, full treatment
dose NAC should be given immediately.

Liver damage peaks at 72-96 hours so it is too early to check these markers.

Next Question
Although liver damage will be demonstrated by a rising INR; it is important to track this and correction with
vitamin K will obscure worsening liver damage.

(Ref BNF for children; Toxbase.org)


45326

https://mypastest.pastest.com/Secure/TestMe/Browser/436619[‫ ص‬05:32:47 10/12/1437]


MyPastest

Previous Question Tag Question

Feedback

Difficulty: Average

Peer Responses

Session Progress

Responses Correct: 0

Responses Incorrect: 78

Responses Total: 78

Responses - % Correct: 0%

Blog
About Pastest
Contact Us
Help

© Pastest 2016

End Session

https://mypastest.pastest.com/Secure/TestMe/Browser/436619[‫ ص‬05:32:47 10/12/1437]


MyPastest

Prefer to use the old MyPastest? Access it here »

Back to Filters

Question 73 of 86

A 15-year-old girl is admitted after a staggered paracetamol overdose. She is commenced on intravenous N-
acetylcyteine therapy over a
21 hour period.

What is the most appropriate monitoring option that should be checked to decide on continuing antidote therapy
past 21 hours?

A Core temperature

B Glasgow coma scale

C INR

D Serum ALP

E Serum potassium

Explanation
INR (and pH and serum creatinine) are the best markers of severity of paracetamol overdose and organ damage.

In uncomplicated paracetamol overdose, conscious level should only be affected by liver failure, which will be
assessed best by INR.

Liver
function is important to monitor after paracetamol overdose and ALT is a
more sensitive marker of
inflammatory liver damage.

Renal function is important to monitor and is best assessed by serum creatinine.

Temperature is not normally an important feature of paracetamol toxicity.

(Ref BNF for children; Toxbase.org)


45327

https://mypastest.pastest.com/Secure/TestMe/Browser/436619[‫ ص‬05:33:08 10/12/1437]


MyPastest

Previous Question

Tag Question

Feedback

Difficulty: Average

Peer Responses

Session Progress

Responses Correct: 0

Responses Incorrect: 79

Responses Total: 79

Responses - % Correct: 0%

Blog
About Pastest
Contact Us
Help

© Pastest 2016

Next Question

End Session

https://mypastest.pastest.com/Secure/TestMe/Browser/436619[‫ ص‬05:33:08 10/12/1437]


MyPastest

Prefer to use the old MyPastest? Access it here »

Back to Filters

Question 74 of 86

A 15-year-old boy with reactive depression took an overdose of his mother’s amitriptyline. On presentation, he
was tachycardic
and drowsy. His ECG showed a sinus tachycardia with QRS duration of 130ms.

What is the most important treatment option at this point?

A Amiodarone iv

B Atenolol 50mg orally

C Calcium gluconate iv

D Hydrocortisone iv

E Sodium bicarbonate iv

Explanation
TCA overdose leads to sodium channel blocking in the myocardium that predisposes to tachyarhythmias. The first
sign of impending cardiovascular complications is prolongation of the QRS complex. Sodium bicarbonate
increases TCA protein binding, dislodges TCAs from the sodium channel and increases TCA elimination.

TCA
overdose renders the myocardium relatively insensitive to traditional antiarrhythmics. In any case,
amiodarone is rarely used in children as a
prophylactic medication unless there has been a ventricular rhythm
disturbance.

Calcium gluconate protects cardiac myocytes from the effects of hyperkalaemia.

Hydrocortisone has a role in the emergency treatment of anaphylactic and Addisonian reactions.

There is no indication for beta blockade which will increase cardiovascular instability.
45328

https://mypastest.pastest.com/Secure/TestMe/Browser/436619[‫ ص‬05:33:30 10/12/1437]


MyPastest

Previous Question

Tag Question

Feedback

Difficulty: Average

Peer Responses

Session Progress

Responses Correct: 0

Responses Incorrect: 80

Responses Total: 80

Responses - % Correct: 0%

Blog
About Pastest
Contact Us
Help

© Pastest 2016

Next Question

End Session

https://mypastest.pastest.com/Secure/TestMe/Browser/436619[‫ ص‬05:33:30 10/12/1437]


MyPastest

Prefer to use the old MyPastest? Access it here »

Back to Filters

Question 75 of 86

Following a house fire, twin 10-year-old boys present to the local emergency department with fatigue, lethargy,
headache and dizziness. Their parents have similar but milder symptoms. They are diagnosed with carbon
monoxide poisoning.

What additional information in either child would be most pressing to necessitate consideration of hyperbaric
oxygen therapy.

A Carboxyhaemoglobin 10%

B Cherry red lips

C Elevated respiratory rate

D Soot around nostrils

E Syncope

Explanation
Carbon Monoxide poisoning is most commonly related to faulty flues related to house fires, heaters, or boilers.
Exposure leads to confusion, neuropsychiatric symptoms, cardiac
dysfunction, and tissue hypoxia.

The most common symptoms in children are:

headache (37.5%)
dizziness (18%)
nausea (17.3%)
loss of consciousness (7.7%)
shortness of breath (6.7%)
and loss of muscle control (3.5%)

https://mypastest.pastest.com/Secure/TestMe/Browser/436619[‫ ص‬05:33:54 10/12/1437]


MyPastest

We
can measure COHb levels (<5% in normal, <10% smokers) and first line treatment is with high flow oxygen.
Previous
Referral for hyperbaric Question
oxygen treatment should be discussed with the National Poisons Information Service if the
patient is pregnant or in cases of severe poisoning such as if the patient is, or has been unconscious, or has
psychiatric or neurological features other than a headache, or has myocardial ischaemia
or an arrhythmia, or has a
blood COHb concentration > 20%.
COHb levels of 10% can be found in smokers and is not indicative of severe CO poisoning.

Elevated RR is a cardinal feature of CO poisoning and is not an indication for hyperbaric therapy in of itself.

Soot or singed hair suggest significant exposure to heat/fire but is not of itself and indication for hyperbaric
therapy.

Cherry red lips are a classical feature of CO poisoning but is clinically very rare ante-mortem and does not
correlate to clinical outcomes or therapy.
45329

Tag Question

Feedback

Difficulty: Average

Peer Responses

Session Progress

Responses Correct: 0

Responses Incorrect: 81
Next Question
Responses Total: 81

Responses - % Correct: 0%

End Session

https://mypastest.pastest.com/Secure/TestMe/Browser/436619[‫ ص‬05:33:54 10/12/1437]


MyPastest

Blog
About Pastest
Contact Us
Help

© Pastest 2016

https://mypastest.pastest.com/Secure/TestMe/Browser/436619[‫ ص‬05:33:54 10/12/1437]


MyPastest

Prefer to use the old MyPastest? Access it here »

Back to Filters

Question 76 of 86

A 15-year-old girl is admitted with agitation after taking MDMA (ecstacy). She has a medical history of asthma
and is on the oral contraceptive pill and salbutamol inhaler PRN.

On examination:
Temperature 38.5°C, HR 110/min, RR 35/min, O2 sat 99% on air. BP 176/112 mmHg. GCS 15/15. Agitated and
bruxism.

What is the most appropriate management at this stage?

A Diazepam 10mg orally

B Paracetamol iv

C Observe in ED for 4h

D Safe to discharge

E 1L chilled 0.9% saline iv

Explanation
She is exhibiting all the classical features of the serotonin toxidrome. Diazepam is first line management for the
tachycardia and hypertension, hyperthermia, agitation and bruxism.

Hyperthermia is caused by muscular overactivity; paracetamol does not treat the primary issue.

She requires monitoring until she no longer exhibits any features of the serotonin toxidrome.

She is not safe to discharge, as she is significantly hyperthermic and has abnormal CV observations.

Chilled iv fluids can be used in hyperthermia does not respond to diazepam; other options include dantrolene and
cyproheptadine under NPIS
advice.

(Ref BNF for children; Toxbase.org)

https://mypastest.pastest.com/Secure/TestMe/Browser/436619[‫ ص‬05:34:17 10/12/1437]


MyPastest

45330

Previous Question

Tag Question

Feedback

Difficulty: Average

Peer Responses

Session Progress

Responses Correct: 0

Responses Incorrect: 82

Responses Total: 82

Responses - % Correct: 0%

Blog
About Pastest
Contact Us
Help

© Pastest 2016

Next Question

End Session

https://mypastest.pastest.com/Secure/TestMe/Browser/436619[‫ ص‬05:34:17 10/12/1437]


MyPastest

Prefer to use the old MyPastest? Access it here »

Back to Filters

Question 77 of 86

A 15-year-old girl is commenced on the combined oral contraceptive pill (COCP) for contraceptive reasons.

Select the most appropriate monitoring option that should be routinely arranged for this patient.

A Blood Pressure

B Breast examination

C Liver function tests

D Oxygen saturations

E Urine microscopy

Explanation
COCPs can increase BP and significant BP elevation to significant hypertensive levels is an indication to cease.

There is no indication for routine breast examination because of COCP use.

There is no indication for routine liver function tests (LFTs) because of COCP use.

Although COCP use is associated with small increases in risk of venous thromboembolism (VTE) and therefore
pulmonary embolism (PE), there is no indication for routine oxygen sats because of COCP use as PE can not be
effectively screened for.

Next Question
There is no indication for routine urine microscopy because of COCP use.
45331

https://mypastest.pastest.com/Secure/TestMe/Browser/436619[‫ ص‬05:34:38 10/12/1437]


MyPastest

Tag Question

Previous Question
Feedback

Difficulty: Average

Peer Responses

Session Progress

Responses Correct: 0

Responses Incorrect: 83

Responses Total: 83

Responses - % Correct: 0%

Blog
About Pastest
Contact Us
Help

© Pastest 2016

End Session

https://mypastest.pastest.com/Secure/TestMe/Browser/436619[‫ ص‬05:34:38 10/12/1437]


MyPastest

Prefer to use the old MyPastest? Access it here »


Previous Question

Back to Filters

Question 78 of 86

Which of these topical corticosteroids for skin disorders is the most potent?

A Betnovate

B Dermovate

C Eumovate

D Hydrocortisone

E Trimovate

Explanation
The BNF (British National Formulary) classifies steroid strengths as mild, moderate, potent and very potent.

MILD: Hydrocortisone
MODERATE: Eumovate, Trimovate
POTENT: Betnovate
VERY POTENT: Dermovate

A full list of topical steroid potencies can be found on the BNF website. Website:
http://www.evidence.nhs.uk/formulary/bnf/current/13-skin/134-topical-corticosteroids/topical-corticosteroid-
preparation-potencies
46182
Next Question

Tag Question

https://mypastest.pastest.com/Secure/TestMe/Browser/436619[‫ ص‬05:34:58 10/12/1437]


MyPastest

Feedback

Difficulty: Average

Peer Responses

Session Progress

Responses Correct: 0

Responses Incorrect: 84

Responses Total: 84

Responses - % Correct: 0%

Blog
About Pastest
Contact Us
Help

© Pastest 2016

End Session

https://mypastest.pastest.com/Secure/TestMe/Browser/436619[‫ ص‬05:34:58 10/12/1437]


MyPastest

Prefer to use the old MyPastest? Access it here »


Previous Question

Back to Filters

Question 79 of 86

Which of the medications below is the correct treatment for Pertussis (whooping cough)?

A Amoxicillin

B Ampicillin

C Cephalexin

D Clarithromycin

E Rifampicin

Explanation
Pertussis (whooping cough) is an acute infection caused by the gram-negative bacterium Bordetella pertussis. It is
treated with macrolide antibiotics such as clarithromycin, azithromycin and erythromycin.

Clarithromycin is also used as prophylaxis for contacts of pertussis-infected patients.

The other antibiotics listed have a wide range of uses but are not commonly used for the treatment of pertussis.

Website: http://www.evidence.nhs.uk/formulary/bnf/current/5-infections/51-antibacterial-drugs
46183

Next Question

Tag Question

Feedback
End Session

https://mypastest.pastest.com/Secure/TestMe/Browser/436619[‫ ص‬05:35:19 10/12/1437]


MyPastest

Difficulty: Average

Peer Responses

Session Progress

Responses Correct: 0

Responses Incorrect: 85

Responses Total: 85

Responses - % Correct: 0%

Blog
About Pastest
Contact Us
Help

© Pastest 2016

https://mypastest.pastest.com/Secure/TestMe/Browser/436619[‫ ص‬05:35:19 10/12/1437]


MyPastest

Prefer to use the old MyPastest? Access it here »

Back to Filters

Question 80 of 86

You are treating a 4 year old girlin intensive care for tuberculosis meningitis. She is receiving Izoniazid,
Rifampicin, Pyrazinamide and Ethambutol.

What other drug should you add in this situation?

A Acyclovir

B Cefotaxime

C Dexomethasone

D Immunoglobulin

E Meropenem

Explanation
TB meningitis is comparatively rare in the UK. In
areas where TB prevalence is high, TB meningitis is most
common in children aged 0 - 4 years, and in areas where TB prevalence is low, most
cases of TB meningitis are in
adults.

TB infection usually begins elsewhere in the body (usually lungs), and then travels through the bloodstream to the
meninges where small abscesses (microtubercles) are formed. When these abscesses burst, TB meningitis results.

Corticosteroids are added to the treatment regime for TB meningitis to help to reduce the severe CNS
inflammatory process and secondary damage.

The other answers above are not specifically used for TB meningitis. Acyclovir is an anti-viral medication which
may be used in herpes simplex encephalitis. IV immunoglobulin may be used in Kawasaki Disease. Cefotaxime is
used in other forms of bacterial meningitis with
meropenem used as per pharmacy advice only in cases
unresponsive to standard antibiotics (such as Pseudomonas aeruginosa meningitis in which other treatments have
failed).
46184

https://mypastest.pastest.com/Secure/TestMe/Browser/436619[‫ ص‬05:35:41 10/12/1437]


MyPastest

Previous Question

Tag Question

Feedback

Difficulty: Average

Peer Responses

Session Progress

Responses Correct: 0

Responses Incorrect: 86

Responses Total: 86

Responses - % Correct: 0%

Blog
About Pastest
Contact Us
Help

© Pastest 2016

Next Question

End Session

https://mypastest.pastest.com/Secure/TestMe/Browser/436619[‫ ص‬05:35:41 10/12/1437]


MyPastest

Prefer to use the old MyPastest? Access it here »

Back to Filters

Question 81 of 86

Which of the following antiepileptic drugs is most associated with optic neuritis?

A Lamotrigine

B Phenytoin

C Sodium Valproate

D Topiramate

E Vigabatrin

Explanation
The correct answer is vigabatrin which is associated with visual field defects in up to a third of patients with
additional links with optic neuritis and prominent behavioural side effects.

Key side effects of the other medications:

Lamotrigine – Bone marrow failure.


Phenytoin - Coarsening of the facial appearance, acne, hirsutism and gingival hyperplasia.
Sodium Valproate – widespread metabolic effects, liver toxicity, congenital malformations.
Topiramate
- acute myopia with secondary angle-closure glaucoma, typically occurring within 1 month of starting
treatment. Increased risk of cleft
palate in fetus.

All anti-epilepic medications have a heady list of general side effects including nausea, vomiting, rashes,
hypersensitivity etc and all patients and families should be asked to report side effects promptly for assessment.
Many are teratogenic and this must be considered in female patients.

Further information: https://www.evidence.nhs.uk/formulary/bnf/current/4-central-nervous-system/48-


antiepileptic-drugs/481-control-of-the-epilepsies
46185

https://mypastest.pastest.com/Secure/TestMe/Browser/436619[‫ ص‬05:36:03 10/12/1437]


MyPastest

Previous Question

Tag Question

Feedback

Difficulty: Average

Peer Responses

Session Progress

Responses Correct: 0

Responses Incorrect: 87

Responses Total: 87

Responses - % Correct: 0%

Blog
About Pastest
Contact Us
Help

© Pastest 2016

Next Question

End Session

https://mypastest.pastest.com/Secure/TestMe/Browser/436619[‫ ص‬05:36:03 10/12/1437]


MyPastest

Prefer to use the old MyPastest? Access it here »

Back to Filters

Question 82 of 86

Which drug should NOT be prescribed concomitantly with CNS stimulants to treat ADHD?

A Aspirin

B Colecalciferol

C Dexamphetamine

D Imipramine

E Lorazepam

Explanation
Imipramine (a tricyclic antidepressant) should not be used in conjunction with CNS stimulants such as
methylphenidate due to the risk of severe adverse affects. Reported problems of children on both of these
medications included severe cognitive and mood
deteriorations. It is likely that methylphenidate interrupts the
normal metabolism of imipramine.

There are no reported contra-indications on taking aspirin with methylphenidate. However, in children under 16
years of age aspirin use is contra-indicated because of the risk of Reye’s Syndrome (a rapidly progressive
encephalopathy linked with aspirin use).

Colecalciferol is used in children with low Vitamin D. There are no known contra-indications with
methylphenidate.

Dexamphetamine and methylphenidate may be prescribed to treat Narcolepsy and ADHD.

Lorazepam
is occasionally used to reduce anxiety symptoms in patients taking CNS stimulants to control their
ADHD. It is more likely to be used in older
teenagers or adults.
46186

https://mypastest.pastest.com/Secure/TestMe/Browser/436619[‫ ص‬05:36:26 10/12/1437]


MyPastest

Previous Question

Tag Question

Feedback

Difficulty: Average

Peer Responses

Session Progress

Responses Correct: 0

Responses Incorrect: 88

Responses Total: 88

Responses - % Correct: 0%

Blog
About Pastest
Contact Us
Help

© Pastest 2016

Next Question

End Session

https://mypastest.pastest.com/Secure/TestMe/Browser/436619[‫ ص‬05:36:26 10/12/1437]


MyPastest

Prefer to use the old MyPastest? Access it here »

Back to Filters

Question 83 of 86

At what age is Aspirin (Acetylsalicylic Acid) considered safe to prescribe to children and young people for
analgesia?

A Over 8 years of age

B Over 10 years of age

C Over 12 years of age

D Over 16 years of age

E Over 18 years of age

Explanation
For children under 16 years of age aspirin use is
contra-indicated because of the risk of Reye’s Syndrome. This is
a rare but very serious rapidly progressive encephalopathy and liver damage that may lead to death. Reye’s
Syndrome has mainly been described as affecting individuals shortly after a viral illness (ie chicken pox) or
influenza-like symptom set, most of whom used aspirin for symptom control. It is thought that mitochondrial
damage occurs leading to the encephalopathic progression.

The exceptions to this age restriction are the use of aspirin’s anti-platelet properties to treat Kawasaki Disease or
to prevent thrombus formation after cardiac surgery.

Website: https://www.evidence.nhs.uk/formulary/bnfc/current/2-cardiovascular-system/29-antiplatelet-
drugs/aspirin-antiplatelet
Next Question
46187

https://mypastest.pastest.com/Secure/TestMe/Browser/436619[‫ ص‬05:36:48 10/12/1437]


MyPastest

Tag Question

Previous Question
Feedback

Difficulty: Average

Peer Responses

Session Progress

Responses Correct: 0

Responses Incorrect: 89

Responses Total: 89

Responses - % Correct: 0%

Blog
About Pastest
Contact Us
Help

© Pastest 2016

End Session

https://mypastest.pastest.com/Secure/TestMe/Browser/436619[‫ ص‬05:36:48 10/12/1437]


MyPastest

Prefer to use the old MyPastest? Access it here »

Back to Filters

Question 84 of 86

Pharmacological treatment for obesity may be started in children over the age of 12 years under specialist
paediatric management. Lifestyle, behavioural, dietary and physical exercise interventions must have been
commenced.

What additional criteria does NICE specify before drug treatment may be considered?

A Child must have a Body Mass Index above the 99th centile for age.

B Child must have a fully supportive family setting.

C Child must have at least one first degree relative with morbid obesity.

D Child must have orthopaedic problems, sleep apnoea or severe psychological co-morbidities.

E Child must have significant dietary intolerances or allergies restricting standard healthy diet intake.

Explanation
Childhood obesity is considered by the World Health Organisation to be one of the most serious public health
challenges in the 21st century. There are significant and ongoing health risks for obese children and teenagers who
are more likely to become obese adults. UK statistics (2014/15), show that 19.1% of children in Year 6 (aged 10-
11) were obese and a further 14.2% were overweight. Of children in Reception (aged 4-5), 9.1% were obese and
another 12.8% were overweight. This means a third of 10-11 year olds and
over a fifth of 4-5 year olds were
overweight or obese (http://www.noo.org.uk/NOO_about_obesity/child_obesity).

NICE
guidelines recommend that pharmacological management (with Orlistat) is
only considered in children over
12 years old (apart from in exceptional circumstances) for whom all other measures (lifestyle, behaviour, dietary,
physical activity) are underway. In addition, the child must have some major physical problem such as orthopaedic
issues, sleep apnoea or severe psychological co-morbidities.

Orlistat must only be used in a specific multi-disciplinary paediatric team setting who can offer drug monitoring,
psychological support, behavioural interventions, interventions to increase physical activity, interventions to

https://mypastest.pastest.com/Secure/TestMe/Browser/436619[‫ ص‬05:37:11 10/12/1437]


MyPastest

improve diet.

With regards to the other answer options:


Previous Question
Obesity
is classified as a Body Mass Index (BMI) at or above the 95th centile for age and gender. A child is
overweight if their BMI is between the 85th – 95th centiles.

Many obese children have at least one


parent or close relative who is significantly overweight or obese. A fully
supportive family is very important for short and long term weight
management success. Often the whole family
benefits from healthy lifestyle and diet education.

Most children with severe dietary intolerances or allergies struggle to maintain weight. This is not a criteria to
commence Orlistat.
46188

Tag Question

Feedback

Difficulty: Average

Peer Responses

Session Progress

Responses Correct: 0

Responses Incorrect: 90

Responses Total: 90

Responses - % Correct:
Next Question 0%

Blog
About Pastest
Contact Us
Help
End Session
© Pastest 2016

https://mypastest.pastest.com/Secure/TestMe/Browser/436619[‫ ص‬05:37:11 10/12/1437]


MyPastest

https://mypastest.pastest.com/Secure/TestMe/Browser/436619[‫ ص‬05:37:11 10/12/1437]


MyPastest

Prefer to use the old MyPastest? Access it here »


Previous Question

Back to Filters

Question 85 of 86

Which psychiatric disorder is most commonly associated with Type 2 diabetes and insulin resistance as a
consequence of drug treatment?

A Anxiety Disorder

B Attention Deficit Hyperactivity Disorder

C Bi-polar Disorder

D Depression

E Schizophrenia

Explanation
Schizophrenia is associated with the development of Type 2 Diabetes and insulin resistance. First-generation
antipsychotic drugs (such as haloperidol) are less likely to cause diabetes than second-generation (‘atypical’)
antipsychotic drugs (such as olanzapine, respiridone). Anti-psychotics can have a wide range of unpleasant side
effects which need to be monitored, including the BMI of the child and diabetes emergence.

The other psychiatric disorders all have pharmacological treatments with their own
potential side effects, however
Schizophrenia is the key disorder linked to diabetes type 2.
46189

Next Question

Tag Question

Feedback End Session

https://mypastest.pastest.com/Secure/TestMe/Browser/436619[‫ ص‬05:37:34 10/12/1437]


MyPastest

Difficulty: Average

Peer Responses

Session Progress

Responses Correct: 0

Responses Incorrect: 91

Responses Total: 91

Responses - % Correct: 0%

Blog
About Pastest
Contact Us
Help

© Pastest 2016

https://mypastest.pastest.com/Secure/TestMe/Browser/436619[‫ ص‬05:37:34 10/12/1437]


MyPastest

Prefer to use the old MyPastest? Access it here »

Back to Filters

Question 86 of 86

A number of paediatric drug doses are calculated using Body Surface Area. Examples include ondansetron and
certain cytotoxic drugs.

What is the approximate body surface area of a one year old, 10kg child?

A 0.10 m2

B 0.49 m2

C 0.65 m2

D 1.00 m2

E 1.20 m2

Explanation
A 10kg child will have a body surface area (BSA) of approximately 0.49m2. A full list of BSA calculations for
children over and under 40kg can be found in the Children’s BNF and via the
following link:

https://www.evidence.nhs.uk/formulary/bnfc/current/body-surface-area-in-children

A 0.10 m2 - approximate BSA for a 1kg baby.


C 0.65 m2 - approximate BSA for a 15kg child.
D 1.00 m2 - approximate BSA for a 28-29kg child.
E 1.20 m2 - approximate BSA for a 35-38kg child.

It is not necessary to remember exact BSA calculations however an awareness of approximate sensible values is
helpful. All drug calculations should be done with full precision using tools provided.
46190

End Session

https://mypastest.pastest.com/Secure/TestMe/Browser/436619[‫ ص‬05:37:55 10/12/1437]


MyPastest

Previous Question Tag Question

Feedback

Difficulty: Average

Peer Responses

Session Progress

Responses Correct: 0

Responses Incorrect: 92

Responses Total: 92

Responses - % Correct: 0%

Blog
About Pastest
Contact Us
Help

© Pastest 2016

https://mypastest.pastest.com/Secure/TestMe/Browser/436619[‫ ص‬05:37:55 10/12/1437]


MyPastest

Prefer to use the old MyPastest? Access it here »

Back to Filters

Question 1 of 53

A 5-year-old boy had a number of previous admissions for ‘virally induced wheeze’. He has been using a ß-
agonist inhaler at home on required basis. He presented to surgery and his mum tells you that he becomes wheezy
with changes in temperature, laughter, exercise and with viral illnesses. He has to use his inhaler three times per
week and coughs on most days, usually at night.

The best next step in his management would be?

A Increase the number of puffs of his ß-agonist that he is having

B Add a low-dose steroid inhaler

C Add a high-dose steroid inhaler

D Add a long-acting ß-agonist

E Add ipratropium inhaler

Explanation
There are multiple, typical points in the history
pointing to poorly controlled asthma. You should be familiar with
the British Thoracic Society Guidelines for the management of asthma as this
is a common problem and it is
virtually guaranteed that there will be questions on asthma and its management in the exam. In brief, management
of chronic episodic asthma is to start with a short-acting ß-agonist as required. Add low-dose inhaled steroids if
symptoms are frequent and use of ß-agonist is also frequent. If symptoms persist, add a long-acting ß-agonist (e.g
salmeterol) or try a leukotriene receptor antagonist (e.g. montelukast). The next step will be to increase the dose of
inhaled steroids. If symptoms still persist, consider continuous or frequent use of oral steroids.

The British Thoracic Society Guidelines can be downloaded from the following website:

http://www.brit-thoracic.org.uk/guidelines-and-quality-standards/asthma-guideline/
10425

https://mypastest.pastest.com/Secure/TestMe/Browser/436619[‫ ص‬12:04:11 09/12/1437]


MyPastest

Tag Question

Feedback

Next Question
Difficulty: Easy

Peer Responses

End Session

Previous Question

Session Progress

Responses Correct: 0

Responses Incorrect: 1

Responses Total: 1

Responses - % Correct: 0%

Blog
About Pastest
Contact Us
Help

© Pastest 2016

https://mypastest.pastest.com/Secure/TestMe/Browser/436619[‫ ص‬12:04:11 09/12/1437]


MyPastest

Prefer to use the old MyPastest? Access it here »

Back to Filters

Question
Next 2 of 53
Question

You are seeing a 4-week-old female infant in clinic. She has been making persistent noises whilst breathing,
especially when upset. She is otherwise well and feeding and growing nicely. Examination
today reveals a
healthy-looking infant. She is not in distress but then
she becomes upset when you handle her and develops an
inspiratory wheeze. There is obvious subcostal, intercostal and suprasternal recession but no cyanosis. She soon
settles down and the symptoms subside. Examination is otherwise unremarkable.

The most likely diagnosis is?

A Neonatal tetany

B Subglottic stenosis

C Vascular ring

D Laryngomalacia

E Laryngeal nerve palsy

Explanation
Inspiratory wheeze signifies stridor. She is only 4 weeks of age so, for this symptom to be described as persistent,
it must more or less have been there since birth. Around 65%
of persistent stridor in infancy is due to
laryngomalacia. It usually presents at birth, but can present at any stage up to 4 weeks. Most children with
laryngomalacia thrive and feed normally. It usually resolves spontaneously by 18 months of age. Other conditions
to consider
include neonatal tetany, subglottic stenosis, subglottic haemangioma, laryngeal nerve palsy, laryngeal
web, vascular ring, congenital floppy larynx and goitre - but these are less common.
10426

https://mypastest.pastest.com/Secure/TestMe/Browser/436619[‫ ص‬12:04:37 09/12/1437]


MyPastest

Tag Question

Feedback

Difficulty: Easy

Peer Responses

End Session

Session Progress

Responses Correct:
Previous Question 0

Responses Incorrect: 2

Responses Total: 2

Responses - % Correct: 0%

Blog
About Pastest
Contact Us
Help

© Pastest 2016

https://mypastest.pastest.com/Secure/TestMe/Browser/436619[‫ ص‬12:04:37 09/12/1437]


MyPastest

Prefer to use the old MyPastest? Access it here »

Back to Filters

Question 3 of 53

Theme: asthma management

A Salbutamol inhaler regularly for the next 12 hours and three times in the first hour
B Start salbutamol via appropriate inhaler as required
C Salbutamol nebulisers regularly for the next 12 hours and three times in the first hour
D Start ipratropium via appropriate inhaler regularly
E Start ipratropium via appropriate inhaler as required
F Give a 5 day course of oral steroids
G Give steroid via appropriate inhaler regularly
H Give steroid via appropriate inhaler as required
I No treatment required

For
each of the scenarios choose the first 'short term' plan of action. Each option may be used once, more than
once, or not at all.

Scenario 1

A 3-year-old boy presents with episodic wheeze worse when playing or running around.

Your answer was incorrect

Select one...

B - Start salbutamol via appropriate inhaler as required

This child is experiencing symptoms of acute asthma triggered by exercise. In this instance a short
acting ß-
agonist via spacer should be the first line treatment. Therapy should be reviewed at a later date to assess the
frequency of use and adequacy of symptom control.

Scenario 2

A 7-year-old known asthmatic boy presents with a 2-hour history of acute shortness of breath. He has widespread
wheeze but is talking and his O2 saturations are 94% in room air.

Your answer was incorrect

https://mypastest.pastest.com/Secure/TestMe/Browser/436619[‫ ص‬12:05:00 09/12/1437]


MyPastest

Select one...

A - Salbutamol inhaler regularly for the next 12 hours and 3 times in the 1st hour

The 7-year-old boy is having a moderate acute exacerbation of asthma. As he is not requiring oxygen, inhalers are
the most appropriate treatment. He would benefit from having 10 puffs every 20 minutes for the first hour and will
then need to use the inhaler regularly for the next 12 hours and possibly even longer.

Therapy should be reviewed according to clinical response and a short course of steroids considered if symptoms
have not resolved quickly. Next Question
Scenario 3

A 10-year-old known asthmatic attends surgery with a painful right toe. He has had several hospital admissions
for
asthma in the past, but has not had an asthma attack for one year. He has no night-time symptoms and describes no
exercise limitation. He was prescribed daily inhaled steroids via large volumeEnd
spacerSession
4 years ago, but his mother
concedes that he has not been using this much over the last 12 months.

Your answer was incorrect

Select one...
Previous Question
I - No treatment required

The 10-year-old boy has no interval symptoms despite not using his inhaled steroids. It would be appropriate
to
stop all his treatment if he has not required inhaled steroids and has been symptom free for one year. Asthma
therapy should be reviewed regularly.
11551

Tag Question

Feedback

Difficulty: Average

Session Progress

Responses Correct: 0

Responses Incorrect: 5

Responses Total: 5

Responses - % Correct: 0%

https://mypastest.pastest.com/Secure/TestMe/Browser/436619[‫ ص‬12:05:00 09/12/1437]


MyPastest

Blog
About Pastest
Contact Us
Help

© Pastest 2016

https://mypastest.pastest.com/Secure/TestMe/Browser/436619[‫ ص‬12:05:00 09/12/1437]


MyPastest

Prefer to use the old MyPastest? Access it here »

Back to Filters

Question
Next 4 of 53
Question

Which one of the following features is MOST characteristic of cystic fibrosis?

A Inherited as autosomal dominant

B Previous
Pancreatic Question
insufficiency is almost always identified in adult patients

C In patients with recurrent chest infections Burkholderia cepacia (B. cepacia) is the most frequent
organ ism isolated from sputum

D Family members who carry the gene are at risk of developing mild recurrent bronchitis

E Patients typically have reduced levels of sodium and chloride in the sweat

Explanation
Cystic fibrosis (CF) is an autosomal recessive disease affecting both eccrine and exocrine gland function,
characterised by elevated levels of sodium and chloride in the sweat. It is caused by abnormal viscid secretions
from mucous glands leading to chronic pulmonary disease and pancreatic insufficiency, which will be evident in
more than 90% of adult cases. Recurrent chest infections are usually caused by Pseudomonas aeruginosa and
Staphylococcus aureus. B. cepacia occurs in
only 5–10% of cases. The carrier frequency is 1:25 in the Caucasian
population, but these heterozygotes are clinically normal.
12190

Tag Question

https://mypastest.pastest.com/Secure/TestMe/Browser/436619[‫ ص‬12:05:22 09/12/1437]


MyPastest

Feedback

Difficulty: Average

Peer Responses

End Session

Session Progress

Responses Correct: 0

Responses Incorrect: 6

Responses Total: 6

Responses - % Correct: 0%

Blog
About Pastest
Contact Us
Help

© Pastest 2016

https://mypastest.pastest.com/Secure/TestMe/Browser/436619[‫ ص‬12:05:22 09/12/1437]


MyPastest

Prefer to use the old MyPastest? Access it here »

Back to Filters

Question
Next 5 of 53
Question

Most cases of community acquired pneumonia in the under 5s are caused by which one of the following?

A Streptococcus pneumoniae

B Previous
Mycoplasma Question
pneumoniae

C Staphylococcus aureus

D Haemophilus influenzae

E Viral pneumonia

Explanation
Community acquired pneumonia is, by definition, contracted in the community rather than in hospital. In the
northern hemisphere, community acquired pneumonia affects approximately 12/1000 people per year, particularly
during winter and at the extremes of age (incidence: < 1 year old 30–50/1000 per year; 71–85 years 50/1000 per
year). Over 100 microorganisms have been implicated, but most cases are caused by viruses whereas most
bacterial cases are caused by Streptococcus pneumoniae particularly in the under 5s whereas Mycoplasma is more
common in the over 5s.
12191

Tag Question

Feedback

https://mypastest.pastest.com/Secure/TestMe/Browser/436619[‫ ص‬12:05:43 09/12/1437]


MyPastest

Difficulty: Average

Peer Responses

End Session

Session Progress

Responses Correct: 0

Responses Incorrect: 7

Responses Total: 7

Responses - % Correct: 0%

Blog
About Pastest
Contact Us
Help

© Pastest 2016

https://mypastest.pastest.com/Secure/TestMe/Browser/436619[‫ ص‬12:05:43 09/12/1437]


MyPastest

Prefer to use the old MyPastest? Access it here »

Back to Filters

Question 6 of 53

A 16-year-old trainee nursery nurse presents to A&E acutely unwell. She has a cough and has become
progressively worse, now with severe shortness of breath. Her flatmate reports that she has become unsteady on
her feet over the past few days. There is no important past history of note, she smokes 20 cigarettes per day. There
is a striking rash over her scalp, face and trunk with a mixed crop of vesicles and pustules suggestive of
chickenpox. On examination she has bilateral cerebellar ataxia.

Which diagnosis fits best with this clincal picture?

A Acute varicella zoster virus infection with CNS and lung involvement

B Bacterial pneumonia

C Herpes simplex encephalitis

D Generalised viral infection of unknown origin

E Likely immunocompromised patient

Explanation
Although uncommon, lung involvement with acute chickenpox infection is seen. The pneumonitis generally
begins some 1–6 days after the appearance of the skin eruption and is more common in young adults, with
cigarette smokers being particularly at risk. This woman was not exposed to chickenpox as a child, and exposure
has now occurred due to her work as a nursery nurse. The chest radiograph normally shows diffuse lung changes
throughout both lung fields, but symptoms are commonly worse than the X-ray picture. CNS involvement is rarer,
occurring in around 1 in 1000 cases, and presenting with acute truncal ataxia. Treatment is with intravenous
aciclovir, with support from the high-dependency/intensive care unit as needed.
12192

https://mypastest.pastest.com/Secure/TestMe/Browser/436619[‫ ص‬12:06:43 09/12/1437]


MyPastest

Tag Question

Feedback

Next Question
Difficulty: Easy

Peer Responses

End Session

Previous Question

Session Progress

Responses Correct: 0

Responses Incorrect: 8

Responses Total: 8

Responses - % Correct: 0%

Blog
About Pastest
Contact Us
Help

© Pastest 2016

https://mypastest.pastest.com/Secure/TestMe/Browser/436619[‫ ص‬12:06:43 09/12/1437]


MyPastest

Prefer to use the old MyPastest? Access it here »

Back to Filters

Question 7 of 53

A 16-year-old girl presents to A&E with a severe asthma attack. What is the most important therapy to relieve her
bronchoconstriction?

A Propranolol

B Salbutamol

C Oxygen

D Glucocorticosteroids

E Cromoglicate

Explanation
The actions of ß-agonists in asthma are the
result of stimulation of ß-adrenoreceptors that are located in the
airways, on airway epithelium, and in submucosal glands, airway and vascular smooth muscle. ß-adrenergic
receptors in the airways are entirely ß2, with the exception of some ß1-receptors on submucosal glands. ß2-agonists
can influence airway function through several mechanisms:

relaxation of bronchial smooth muscle by direct effect on ß2-receptors


inhibition of mast-cell mediator release, and
enhanced mucociliary clearance.

Inhalation of a ß2-agonist by a patient with asthma increases airway calibre and reduces airway hyper
responsiveness. ß2-agonists
also cause tachycardia and increased cardiac output, systemic vasodilatation and
increased muscle blood flow. The tachycardia and increased cardiac output are the results of both stimulation of
cardiac ß-adrenoreceptors and a reflex response to peripheral vasodilatation. In addition, ß2-agonists cause tremor
and have metabolic actions, of which hypokalaemia is probably the only potentially important clinical effect.

https://mypastest.pastest.com/Secure/TestMe/Browser/436619[‫ ص‬12:07:06 09/12/1437]


MyPastest

12193

Tag Question

Feedback

Next Question
Difficulty: Easy

Peer Responses

End Session

Previous Question

Session Progress

Responses Correct: 0

Responses Incorrect: 9

Responses Total: 9

Responses - % Correct: 0%

Blog
About Pastest
Contact Us
Help

© Pastest 2016

https://mypastest.pastest.com/Secure/TestMe/Browser/436619[‫ ص‬12:07:06 09/12/1437]


MyPastest

Prefer to use the old MyPastest? Access it here »

Back to Filters

Question 8 of 53

A 16-year-old boy has been complaining of shortness of breath for the last 2 days. On examination bronchial
breathing is heard over the right lower lobe.

What is the most likely diagnosis for this clinical finding?

A Pneumothorax

B Pneumonia

C Asthma

D Emphysema

E Chronic obstructive pulmonary disease

Explanation
Bronchial breathing is heard over an airless lung, such as in consolidation, atelectasis or dense fibrosis. There is
some resemblance to the sounds heard over the normal trachea, but, by comparison with normal breath sounds,
bronchial breathing is higher in pitch and more blowing in quality. It does not have to be loud. Bronchial breath
sounds are classically heard throughout both inspiration and expiration. Very quiet breath sounds are heard over
hyperinflated lungs, as in emphysema, or when breath sounds are prevented from reaching the chest wall by a
layer of air, fluid or fibrosis.

The classic presentation of pneumonia is of a cough and fever with the variable presence of sputum production,
dyspnoea and pleurisy. Most patients have constitutional symptoms such as malaise, fatigue and asthenia, and
many also have gastrointestinal symptoms. Although patients with pneumonia usually possess these characteristic
clinical features, there can be major differences in presentation based on the host and the aetiological agent.
12194

https://mypastest.pastest.com/Secure/TestMe/Browser/436619[‫ ص‬12:07:29 09/12/1437]


MyPastest

Tag Question

Feedback

Next Question
Difficulty: Easy

Peer Responses

End Session

Previous Question

Session Progress

Responses Correct: 0

Responses Incorrect: 10

Responses Total: 10

Responses - % Correct: 0%

Blog
About Pastest
Contact Us
Help

© Pastest 2016

https://mypastest.pastest.com/Secure/TestMe/Browser/436619[‫ ص‬12:07:29 09/12/1437]


MyPastest

Prefer to use the old MyPastest? Access it here »

Back to Filters

Question
Next 9 of 53
Question

A 12-year-old girl is admitted to the emergency department. She had been found by her mother and younger sister
when they returned back from shopping. When they got back to the flat they tell you she was unsteady on her feet
and having problems with her memory. They tell you they have been using old fashioned gas fires as it
has been
so cold. You are wondering whether she could have carbon monoxide poisoning.

Which test will be most helpful in determining this?

A Clinical examination

B Pulse oximetry

C Arterial blood oxygen level

D Exhaled breath test

E Chest radiograph

Explanation
Breath carbon monoxide levels can be measured using devices intended for smoking cessation work. They can
also be measured on a co-oximeter, measuring carboxyhaemoglobin levels. Clinical examination is unhelpful, as
patients are not cyanosed, but a cherry-red colour. Pulse oximetry appears normal, due to carboxyhaemoglobin
having similar absorption spectra to oxyhaemoglobin. Arterial oxygen levels may be normal, as may a chest
radiograph. Treatment for carbon monoxide poisoning is to give high levels of inspired oxygen, by ventilation (or
hyperbaric means if necessary), to displace the carbon monoxide and increase the amount of dissolved oxygen.
12538

https://mypastest.pastest.com/Secure/TestMe/Browser/436619[‫ ص‬12:07:51 09/12/1437]


MyPastest

Tag Question

Feedback

Difficulty: Average

Peer Responses

End Session

Previous Question Session Progress

Responses Correct: 0

Responses Incorrect: 11

Responses Total: 11

Responses - % Correct: 0%

Blog
About Pastest
Contact Us
Help

© Pastest 2016

https://mypastest.pastest.com/Secure/TestMe/Browser/436619[‫ ص‬12:07:51 09/12/1437]


MyPastest

Prefer to use the old MyPastest? Access it here »

Back to Filters

Question 10 of 53
Next Question

A 2-year-old girl presents to your Emergency Department


with a history of cough and a 6-hour history of noisy
End Session
breathing. She has a barking cough, mild recession and mild stridor.

What is the best course of action?

A Take her to the resuscitation area and ask anaesthetists to attend urgently
Previous Question
B Give an oral dose of dexamethosone (600 micrograms/kg)

C Request a lateral X-ray

D Give an oral dose of dexamethasone (150 micrograms/kg) and observe for a further 4 hours

E Prescribe oral budesonide 1mg/kg

Explanation
The child has croup and she is showing signs of mild illness. In an A&E department, the most appropriate thing to
do would be to give an oral dose of dexamethasone (150 micrograms/kg) and observe for a further 4 hours. There
is little evidence for the higher dose strategy giving more benefits than the lower one. An X-ray would not help
distinguish the diagnosis and as she is maintaining her own airway there is no need for an anaesthetist.
14889

Tag Question

Feedback

https://mypastest.pastest.com/Secure/TestMe/Browser/436619[‫ ص‬12:08:12 09/12/1437]


MyPastest

Difficulty: Average

Peer Responses

Session Progress

Responses Correct: 0

Responses Incorrect: 12

Responses Total: 12

Responses - % Correct: 0%

Blog
About Pastest
Contact Us
Help

© Pastest 2016

https://mypastest.pastest.com/Secure/TestMe/Browser/436619[‫ ص‬12:08:12 09/12/1437]


MyPastest

Prefer to use the old MyPastest? Access it here »

Back to Filters

Question 11 of 53
Next Question

A 7-year-old girl was admitted with high fever and a painful cough productive of purulent sputum. The clinical
picture was that of right lower lobe pneumonia which was confirmed radiologically. She had a similar episode 9
months previously while away on holiday and was hospitalised for 3 days. The chest X-ray report indicated
identical changes at that time. There was no contact history of TB and she has not
had BCG. She was commenced
on a course of antibiotics.

The next most logical step in her management is?

A Bronchoscopy

B Mantoux test

C CT scan of the thorax

D Clinical and radiological review at follow-up

E Ventilation perfusion scan

Explanation
This child has had two acute lung infections affecting the same area of the lung in a period of nine months. As far
as the history goes she recovered fully after the first attack and had been well in the intervening period. However,
the original X-ray was not available for perusal though a copy of the report
was seen. The history does not suggest
a chronic respiratory problem and the most logical step is to review her response to antibiotics both clinically and
radiologially before further investigations are instigated. In the presence of recurrent clinical and/or radiological
abnormality, elimination of a foreign body or chronic infection (e.g. TB, pulmonary sequestration and immune
deficiency, etc) may need to be considered.
14890

https://mypastest.pastest.com/Secure/TestMe/Browser/436619[‫ ص‬12:08:33 09/12/1437]


MyPastest

Tag Question

Feedback

Difficulty: Average

Peer Responses

End Session

Previous Question Session Progress

Responses Correct: 0

Responses Incorrect: 13

Responses Total: 13

Responses - % Correct: 0%

Blog
About Pastest
Contact Us
Help

© Pastest 2016

https://mypastest.pastest.com/Secure/TestMe/Browser/436619[‫ ص‬12:08:33 09/12/1437]


MyPastest

Prefer to use the old MyPastest? Access it here »

Back to Filters

Question 13 of 53

Theme: Respiratory conditions

A a-1 anti-trypsin
B Atopic asthma
C Cystic fibrosis
D Foreign body inhalation
E Kartagener syndrome
F Mycoplasma infection
G Pertussis
H Recurrent aspiration
I Vascular ring
J Tuberculosis

For
each of the following cases please choose the most likely diagnosis from the above list. Each item may be used
once, more than once or not at all.

Scenario 1

A 2-year-old is referred with difficulty in breathing. She is noted to be wheezy – more on the right than left. A
chest X-ray reveals right-sided hyperexpansion.

Your answer was incorrect

Select one...

D - Foreign body inhalation

In a toddler who presents with difficulty in breathing, foreign body aspiration must always be considered.
Toddlers are inquisitive and will put most small objects in their mouths. Any asymmetry in expansion or wheeze
should increase the index of suspicion.

Scenario 2

An 18-month-old infant of a travelling family presents with an acute respiratory illness. Investigations at the time
show a marked lymphocytosis. He has a persistent cough 2 months later.

https://mypastest.pastest.com/Secure/TestMe/Browser/436619[‫ ص‬12:09:44 09/12/1437]


MyPastest

Your answer was incorrect

Select one...

G - Pertussis

An immunisation history must always be


asked for (and the personal child health record (‘red book’) should be
used). A cough that occurs in paroxysms, subconjunctival haemorrhages from repeated coughing and a marked
Next Question
lymphocytosis are all classic features of whooping cough. Treatment is supportive but erythromycin can be given
to limit infectivity to other children. The cough can last up to 3 months (the 100 days cough).

Scenario 3

A 3-year-old child has a recurrent nocturnal cough. He had mild eczema as an infant. He is otherwise well.

Your answer was incorrect End Session

Select one...

B - Atopic asthma
Previous Question
A previous history of an atopic condition or a family history of atopy increases the risk of asthma (and
other atopic
conditions). A nocturnal cough is a recognised presentation of asthma in pre-school children. A trial of an inhaled
bronchodilator via a spacer is worthwhile.
22186

Tag Question

Feedback

Difficulty: Average

Session Progress

Responses Correct: 0

Responses Incorrect: 19

Responses Total: 19

Responses - % Correct: 0%

https://mypastest.pastest.com/Secure/TestMe/Browser/436619[‫ ص‬12:09:44 09/12/1437]


MyPastest

Blog
About Pastest
Contact Us
Help

© Pastest 2016

https://mypastest.pastest.com/Secure/TestMe/Browser/436619[‫ ص‬12:09:44 09/12/1437]


MyPastest

Prefer to use the old MyPastest? Access it here »

Back to Filters

Question 12 of 53

Theme: Respiratory medicine

A Intensive chest physiotherapy


B 24-hour oesophageal pH monitoring
C Prescribe high dose inhaled steroids via a spacer +/- mask
D Measure sweat electrolytes
E Request a diary of twice daily peak flow measurements
F Refer to the dietician for a detailed nutritional assessment
G Prescribe 2 weeks of oral ciprofloxacin
H Oral amoxicillin for a 7 day course
I Refer for a routine flexible bronchoscopy

For
each of the scenarios below, select the most appropriate way to proceed
from the options above. Each option
may be used once, more than once, or not at all.

Scenario 1

A 2-year-old child with poor growth has seen the GP with recurrent chestiness and delayed speech. He is on
inhaled sodium cromoglycate via a spacer with equivocal benefit. He was admitted at 9 months of age with
bronchiolitis. In a recent referral to the ENT department they detected nasal polyps and prescribe intranasal
steroids.

Your answer was incorrect

Select one...

D - Measure sweat electrolytes

This child is displaying


features suggestive of CF, in which case a sweat test is the most appropriate thing to do
next from the options given.

Scenario 2

A 4-year-old girl has had troublesome nocturnal cough on about 4 nights per week for over a year. Another GP
thought she might have asthma and prescribed budesonide via a
turbohaler which has made no difference to her
symptoms.

https://mypastest.pastest.com/Secure/TestMe/Browser/436619#Top[‫ ص‬12:10:46 09/12/1437]


MyPastest

Your answer was incorrect

Select one...

C - Prescribe high dose inhaled steroids via a spacer +/- mask

This child appears to have asthma and needs to be treated with inhaled steroids via a spacer.

Scenario 3

A 23-month-old boy presents to you with mild breathlessness, fever and cough for just under 24 hours. His parents
report that he is a very active boy who is always running around
and is 'into everything'. On examination he has
decreased air entry on the right with occasional wheeze.

Your answer was incorrect Next Question


Select one...

Previous Question
H - Oral amoxicillin for a 7 day course
End Session
This child has a right sided pneumonia and needs oral antibiotics for 7 days.

Explanation
 
14892

Tag Question

Feedback

Difficulty: Average

Session Progress

Responses Correct: 0

Responses Incorrect: 19

Responses Total: 19

Responses - % Correct: 0%

https://mypastest.pastest.com/Secure/TestMe/Browser/436619#Top[‫ ص‬12:10:46 09/12/1437]


MyPastest

Blog
About Pastest
Contact Us
Help

© Pastest 2016

https://mypastest.pastest.com/Secure/TestMe/Browser/436619#Top[‫ ص‬12:10:46 09/12/1437]


MyPastest

Prefer to use the old MyPastest? Access it here »

Back to Filters

Question 13 of 53

Theme: Respiratory conditions

A a-1 anti-trypsin
B Atopic asthma
C Cystic fibrosis
D Foreign body inhalation
E Kartagener syndrome
F Mycoplasma infection
G Pertussis
H Recurrent aspiration
I Vascular ring
J Tuberculosis

For
each of the following cases please choose the most likely diagnosis from the above list. Each item may be used
once, more than once or not at all.

Scenario 1

A 2-year-old is referred with difficulty in breathing. She is noted to be wheezy – more on the right than left. A
chest X-ray reveals right-sided hyperexpansion.

Your answer was incorrect

Select one...

D - Foreign body inhalation

In a toddler who presents with difficulty in breathing, foreign body aspiration must always be considered.
Toddlers are inquisitive and will put most small objects in their mouths. Any asymmetry in expansion or wheeze
should increase the index of suspicion.

Scenario 2

An 18-month-old infant of a travelling family presents with an acute respiratory illness. Investigations at the time
show a marked lymphocytosis. He has a persistent cough 2 months later.

https://mypastest.pastest.com/Secure/TestMe/Browser/436619#Top[‫ ص‬12:11:05 09/12/1437]


MyPastest

Your answer was incorrect

Select one...

G - Pertussis

An immunisation history must always be


asked for (and the personal child health record (‘red book’) should be
used). A cough that occurs in paroxysms, subconjunctival haemorrhages from repeated coughing and a marked
lymphocytosis are all classic features of whooping cough. Treatment is supportive but erythromycin can be given
to limit infectivity to other children. The cough can last up to 3 months (the 100 days cough).

Scenario 3

A 3-year-old child has a recurrent nocturnal cough. He had mild eczema as an infant. He is otherwise well.

Your answer was incorrect


Next Question

Select one...
Previous Question
B - Atopic asthma
End Session
A previous history of an atopic condition or a family history of atopy increases the risk of asthma (and
other atopic
conditions). A nocturnal cough is a recognised presentation of asthma in pre-school children. A trial of an inhaled
bronchodilator via a spacer is worthwhile.
22186

Tag Question

Feedback

Difficulty: Average

Session Progress

Responses Correct: 0

Responses Incorrect: 19

Responses Total: 19

Responses - % Correct: 0%

https://mypastest.pastest.com/Secure/TestMe/Browser/436619#Top[‫ ص‬12:11:05 09/12/1437]


MyPastest

Blog
About Pastest
Contact Us
Help

© Pastest 2016

https://mypastest.pastest.com/Secure/TestMe/Browser/436619#Top[‫ ص‬12:11:05 09/12/1437]


MyPastest

Prefer to use the old MyPastest? Access it here »

Back to Filters

Question 14 of 53

Next Question
In a child whose asthma is not controlled on a regular inhaled steroid and occasional ß2-agonist, the single next
best action would be to:

A Previous
Add long-acting Question
ß2-agonist

End Session
B Increase the dose of inhaled steroid

C Check the inhaler technique

D Add a leukotriene inhibitor

E Add a short course of oral steroids

Explanation
The most important step to take is to check inhaler technique before going to the next stage of the British Thoracic
Society (BTS) guidelines. If the current medication is not being effectively given, there is no point in increasing
the medication.
22191

Tag Question

Feedback

Difficulty: Average

https://mypastest.pastest.com/Secure/TestMe/Browser/436619#Top[‫ ص‬12:11:26 09/12/1437]


MyPastest

Peer Responses

Session Progress

Responses Correct: 0

Responses Incorrect: 20

Responses Total: 20

Responses - % Correct: 0%

Blog
About Pastest
Contact Us
Help

© Pastest 2016

https://mypastest.pastest.com/Secure/TestMe/Browser/436619#Top[‫ ص‬12:11:26 09/12/1437]


MyPastest

Prefer to use the old MyPastest? Access it here »

Back to Filters

Question 15 of 53

Next Question
A 4-month-old infant presents with a fever, cough and
reduced feeds. Her respiratory rate is 60/min with mild
recession, wheeze and crackles throughout. The most likely diagnosis is:

A Croup Previous Question


End Session
B Virally induced atopic wheeze

C Cystic fibrosis

D Bacterial chest infection

E Bronchiolitis

Explanation
These are the clinical features of bronchiolitis – most frequently caused by RSV, but also adenovirus and influenza
virus. These features do not really fit with croup. Virally induced atopic wheeze is possible.
22194

Tag Question

Feedback

Difficulty: Easy

Peer Responses

https://mypastest.pastest.com/Secure/TestMe/Browser/436619#Top[‫ ص‬12:11:47 09/12/1437]


MyPastest

Session Progress

Responses Correct: 0

Responses Incorrect: 21

Responses Total: 21

Responses - % Correct: 0%

Blog
About Pastest
Contact Us
Help

© Pastest 2016

https://mypastest.pastest.com/Secure/TestMe/Browser/436619#Top[‫ ص‬12:11:47 09/12/1437]


MyPastest

Prefer to use the old MyPastest? Access it here »

Back to Filters

Question 16 of 53

Next Question
You diagnose a 7-year-old girl as having a generalised chest infection. Her respiratory rate is 22/min, SaO2 98% in
air; there is no recession. What would be the most appropriate course of action?

A AdmitPrevious Question
for intravenous antibiotics

End Session
B Allow home but arrange for intravenous antibiotics to be given by the home care nurses

C A 7-day course of oral amoxicillin

D A 7-day course of oral erythromycin

E Admit for oral antibiotics

Explanation
In a child with no respiratory distress there is
no need to admit unless oral antibiotics cannot be tolerated. The BTS
guidelines for community-acquired pneumonia in children suggest the use of a macrolide antibiotic if Mycoplasma
sp. is a possible causative organism. If, however, there are features that would suggest pneumococcal chest
infection (focal consolidation/signs), then amoxicillin is the suggested first-line treatment.
22346

Tag Question

Feedback

Difficulty: Difficult

https://mypastest.pastest.com/Secure/TestMe/Browser/436619#Top[‫ ص‬12:12:20 09/12/1437]


MyPastest

Peer Responses

Session Progress

Responses Correct: 0

Responses Incorrect: 22

Responses Total: 22

Responses - % Correct: 0%

Blog
About Pastest
Contact Us
Help

© Pastest 2016

https://mypastest.pastest.com/Secure/TestMe/Browser/436619#Top[‫ ص‬12:12:20 09/12/1437]


MyPastest

Prefer to use the old MyPastest? Access it here »

Back to Filters

Question 17 of 53

Next Question
A 5-year-old boy has a history of recurrent nocturnal
cough and frequent difficulty in breathing on exercise. He
has recently
been admitted with moderately severe wheeze that responded to nebulised
bronchodilator therapy and
a dose of oral prednisolone. What do you prescribe on discharge?

Previous Question
A A home nebulliser with salbutamol as needed
End Session
B Becotide 200 µg twice daily via a metered dose inhaler (MDI)

C Flixotide 100 µg twice daily via a low-volume spacer

D Budesonide 200 µg twice daily via a dry powder device

E Montelukast 5 mg at night

Explanation
The most practical and reliable device to deliver
inhaled medication to a 5-year-old is a spacing device, which can
also be used for preventative measures and is particularly effective as a relief medication. Nebulisers are relatively
impractical, less portable,
take longer to deliver each dose and expensive. Most 5-year-olds are usually unable to
use a dry powder device very reliably and even less so
MDIs. Oral leukotriene antagonists are not first-line
therapy in asthma
prevention.
22365

Tag Question

Feedback

https://mypastest.pastest.com/Secure/TestMe/Browser/436619#Top[‫ ص‬12:12:41 09/12/1437]


MyPastest

Difficulty: Difficult

Peer Responses

Session Progress

Responses Correct: 0

Responses Incorrect: 23

Responses Total: 23

Responses - % Correct: 0%

Blog
About Pastest
Contact Us
Help

© Pastest 2016

https://mypastest.pastest.com/Secure/TestMe/Browser/436619#Top[‫ ص‬12:12:41 09/12/1437]


MyPastest

Prefer to use the old MyPastest? Access it here »

Back to Filters

Question 18 of 53

Next Question
A child who has a 2-day history of fever has a bright-red, infected, left tympanic membrane. The child is eating
and drinking well. What would be the most appropriate course of action?

A Previous
Referral Question
to ENT (ear, nose, throat) surgeons as an emergency

End Session
B Referral to ENT surgeon as an outpatient

C 5-day course of oral phenoxymethylpenicillin

D Advice to parents that this is a self-limiting condition and is best left alone

E Regular antipyretics and analgesics with review in 48h

Explanation
Antibiotics are of limited benefit in the treatment of otitis media. They can shorten the duration of the illness but
the number needed to treat is 17. Of the cases of otitis media, 80% resolve without treatment. Supportive measures
such as antipyretics and analgesics are important measures to recommend to parents. (Sharon Sanders, Paul P.
Glasziou, Chris Del Mar, and Maroeska Rovers. (2009) "Antibiotics for acute otitis media in children (Review)"
Cochrane database of systematic reviews, (2), 1-43)
22369

Tag Question

Feedback

https://mypastest.pastest.com/Secure/TestMe/Browser/436619#Top[‫ ص‬12:13:02 09/12/1437]


MyPastest

Difficulty: Average

Peer Responses

Session Progress

Responses Correct: 0

Responses Incorrect: 24

Responses Total: 24

Responses - % Correct: 0%

Blog
About Pastest
Contact Us
Help

© Pastest 2016

https://mypastest.pastest.com/Secure/TestMe/Browser/436619#Top[‫ ص‬12:13:02 09/12/1437]


MyPastest

Prefer to use the old MyPastest? Access it here »

Back to Filters

Question 19 of 53

Next Question
A parent comes to you for advice after her child’s grommet insertion. What is the most important thing to tell her?

A Swimming in a chlorinated pool is permitted


Previous Question
B Swimming in the sea should be avoided

C There may be short-term hearing loss

D Aeroplane flights should be avoided

E Plugging the ears for baths and showers is advisable

Explanation
Soaps and shampoos are irritant to the middle ear, and the soap in the water theoretically allows the liquid to pass
more easily through the hole in the grommet itself. As daily washing is the most common of these activites, it
follows that this is the most important advice. Swimming in swimming pools is permitted although diving and
underwater swimming should be discouraged. Sea water is more likely to cause infection than chlorinated pool
water, but swimming may be allowed if the ears are appropriately plugged. There should not be any hearing loss
with grommets, and grommets may ease any pressure equalisation problems on aeroplane flights. There is little
evidence base to this and much variation in advice from ENT surgeons re management, but the above is based on
the Prodigy clinical knowledge summaries.
22372

Tag Question

Feedback

https://mypastest.pastest.com/Secure/TestMe/Browser/436619#Top[‫ ص‬12:13:24 09/12/1437]


MyPastest

Difficulty: Average

Peer Responses

Session Progress

Responses Correct: 0

Responses Incorrect: End Session 25

Responses Total: 25

Responses - % Correct: 0%

Blog
About Pastest
Contact Us
Help

© Pastest 2016

https://mypastest.pastest.com/Secure/TestMe/Browser/436619#Top[‫ ص‬12:13:24 09/12/1437]


MyPastest

Prefer to use the old MyPastest? Access it here »

Back to Filters

Question 20 of 53

Next Question
A 4-year-old child presents with a coryzal symptoms, cough, fever and abdominal pain. There is no dysuria and
urine dipstick is unremarkable. On examination the pain is in the left upper quadrant but is not severe. Which is
the most likely cause?

Previous Question
A UTI
End Session
B Appendicitis

C Pyelonephritis

D Left basal chest infection

E Mesenteric adenitis

Explanation
The history and site of the pain are unlikely for
appendicitis, mesenteric adenitis and pyelonephritis. A UTI is
unlikely
with no symptoms and a negative dipstick. A basal pneumonia is an important differential diagnosis in a
febrile child who presents with abdominal pain.
22384

Tag Question

Feedback

Difficulty: Easy

https://mypastest.pastest.com/Secure/TestMe/Browser/436619#Top[‫ ص‬12:13:43 09/12/1437]


MyPastest

Peer Responses

Session Progress

Responses Correct: 0

Responses Incorrect: 26

Responses Total: 26

Responses - % Correct: 0%

Blog
About Pastest
Contact Us
Help

© Pastest 2016

https://mypastest.pastest.com/Secure/TestMe/Browser/436619#Top[‫ ص‬12:13:43 09/12/1437]


MyPastest

Prefer to use the old MyPastest? Access it here »

Back to Filters

Question 21 of 53

A 7-year-old child presents complaining of itchy eyes and nasal congestion. Her mother reports the symptoms
tend to come and go. They are generally worse during the summer months. Examination reveals
mild conjunctival
injection but nil else of note.

What is the BEST option for management?

A Cetirizine

B Chlorpeniramine

C Intranasal antihistamine

D Intranasal corticosteroid

E Intranasal decongestant

Explanation
Cetirizine - Cetirizine is a non-sedating antihistamine and should be considered first line management of allergic
rhinitis in this case. NICE guidelines state an oral antihistamine should be prescribed first line for the management
of allergic rhinitis in the following situations:

Conjunctivitis is also present


Children aged 2 to 5 years of age
Patient preference for oral treatment

Chlorpeniramine - Oral antihistamines are indeed first line treatment


in this case however Chlorpheniramine
(Piriton) is a sedating antihistamine. This is likely to impair educational performance. A non-sedating
antihistamine should be used.

https://mypastest.pastest.com/Secure/TestMe/Browser/436619#Top[‫ ص‬12:16:23 09/12/1437]


MyPastest

Intranasal antihistamine – Intransal antihistamines are recommended first line in most cases with the exception of
the following situations; the presence of symptomatic conjunctivitis, children aged 2 to 5 years of age, patient
preference for oral treatment. In this case the child is complaining of itchy eyes suggesting allergic conjunctivitis.
In most other cases intranasal antihistamines (azelastine) should be prescribed first line.

Intranasal corticosteroid – Intranasal corticosteroids are only recommended first line in cases of persistent
symptoms of allergic rhinitis where nasal blockage is the primary complaint, or nasal polyps are present.

Intranasal decongestant – Although nasal decongestants are useful in the management of allergic rhinitis oral
antihistamines would be first line in this case (see NICE guidelines below).

http://cks.nice.org.uk/allergic-rhinitis
46710

Next Question
Tag Question

Previous Question
Feedback

End Session
Difficulty: Average

Peer Responses

Session Progress

Responses Correct: 0

Responses Incorrect: 27

Responses Total: 27

Responses - % Correct: 0%

Blog
About Pastest
Contact Us
Help

© Pastest 2016

https://mypastest.pastest.com/Secure/TestMe/Browser/436619#Top[‫ ص‬12:16:23 09/12/1437]


MyPastest

https://mypastest.pastest.com/Secure/TestMe/Browser/436619#Top[‫ ص‬12:16:23 09/12/1437]


MyPastest

Prefer to use the old MyPastest? Access it here »

Back to Filters

Question 22 of 53

A 7-year-old child presents complaining of nasal congestion and discharge. Her mother reports the symptoms tend
to be worse during the summer months. The child denies any discomfort or itchiness of her eyes and on
examination there is no evidence of conjunctival injection.

What is the BEST option for management?

A Cetirizine

B Chlorpeniramine

C Intranasal antihistamine

D Intranasal corticosteroid

E Intranasal decongestant

Explanation
Intranasal antihistamine – Intransal antihistamines are recommended first line in most cases with the exception of
the following situations; the presence of symptomatic conjunctivitis, children aged 2 to 5 years of age, patient
preference for oral treatment.

Cetirizine - Oral antihistamines are first line treatment in children 2-5 years of age or if conjunctivitis is also
present (which is not the case in this question), otherwise for intermittent allergic rhinitis intranasal antihistamines
are first line.
When oral antihistamines are used a non-sedating antihistamine, such as
cetirizine, is a better choice
than a sedating antihistamine such as chlorpeniramine (Piriton).

Chlorpeniramine - Oral antihistamines are first line treatment in children 2-5 years of age or if conjunctivitis is
also present (which is not the case in this question),
otherwise for intermittent allergic rhinitis intranasal
antihistamines are first line. Chlorpheniramine (Piriton) is a sedating antihistamine, likely to impact on educational
performance, when oral antihistamines are used a non-sedating antihistamine is a better choice.

https://mypastest.pastest.com/Secure/TestMe/Browser/436619#Top[‫ ص‬12:16:43 09/12/1437]


MyPastest

Intranasal
corticosteroid – Intranasal corticosteroids are only recommended first line in cases of persistent
symptoms of allergic rhinitis where nasal blockage is the primary complaint, or nasal polyps are present.

Intranasal
decongestant – Although nasal decongestants are sometime in the management of allergic rhinitis
intranasal antihistamines would be first
line in this case (see NICE guidelines below).

http://cks.nice.org.uk/allergic-rhinitis
46711

Tag Question

Feedback
Next Question
Difficulty: Average

Previous Question Peer Responses

End Session

Session Progress

Responses Correct: 0

Responses Incorrect: 28

Responses Total: 28

Responses - % Correct: 0%

Blog
About Pastest
Contact Us
Help

© Pastest 2016

https://mypastest.pastest.com/Secure/TestMe/Browser/436619#Top[‫ ص‬12:16:43 09/12/1437]


MyPastest

Prefer to use the old MyPastest? Access it here »

Back to Filters

Question 23 of 53

A 9-year-old child with a history of infantile atomic dermatitis presents complaining of periods of severely itchy
eyes, sneezing and a blocked nose. He has noticed this typically occurs on weekends which he spends at his
father’s house. On direct questioning he informs you that his father’s house is not as clean
as his mother’s and that
his father has a number of pets including two dogs and a cat.

What is the BEST first line management plan?

A Advise the use of house dust mite impermeable covers for pillows and mattresses at his father’s home

B Arrange skin prick testing

C Request blood tests for Total and specific IgE levels

D Start inhaled corticosteroids

E Start oral antihistamine

Explanation
Arrange skin prick testing – Allergy testing is indicated when the aetiology is unclear and particularly before
arduous allergen avoidance strategies, such as house dust mite avoidance measures, or removal of a pet from the
family home are advised. Skin prick testing is the allergy testing of choice. Avoidance of confirmed animal
allergens is considered important to control symptoms of allergic rhinitis and reduce the risk of asthma
developing. Stem B is correct.

Advise the use of house dust mite impermeable covers for pillows and mattresses at his father’s home - House
dust mite avoidance measures are arduous, expensive and lack a solid evidence base. House dust mite allergy
should be confirmed before avoidance measures are recommended so stem A is incorrect.

Request
blood tests for Total IgE levels - Allergy testing is indicated when the aetiology is unclear and particularly
before arduous allergen avoidance strategies, such as house dust mite avoidance measures, or removal of a pet
from the family home are advised. Skin prick testing is
the allergy testing of choice. If skin prick testing is not

https://mypastest.pastest.com/Secure/TestMe/Browser/436619#Top[‫ ص‬12:17:03 09/12/1437]


MyPastest

available total and specific IgE levels, RAST (radioallergosorbent test) or ELISA (enzyme-linked immunosorbent
assay) may be useful. So answer stem C is not the best answer.

Start inhaled corticosteroids – Inhaled


corticosteroids are useful however allergen avoidance, if possible, is
fundamental in the management of allergic rhinitis so testing for allegens is the better answer.

Start oral antihistamine – Oral antihistamines are useful however allergen avoidance is fundamental
in the
management of allergic rhinitis. so again, stem B is the best answer.
46712

Tag Question

Feedback Next Question

Difficulty: Average
Previous Question
Peer Responses
End Session

Session Progress

Responses Correct: 0

Responses Incorrect: 29

Responses Total: 29

Responses - % Correct: 0%

Blog
About Pastest
Contact Us
Help

© Pastest 2016

https://mypastest.pastest.com/Secure/TestMe/Browser/436619#Top[‫ ص‬12:17:03 09/12/1437]


MyPastest

Prefer to use the old MyPastest? Access it here »

Back to Filters

Question 24 of 53

A 3-year-old child is admitted to the paediatric ward with a lower respiratory tract infection. Reviewing the history
she has been unwell for 48 hours with cough and fever on the background of longstanding nasal congestion, itchy
eyes and recurrent cough. Auscultation of her chest reveals bibasal crepitations. ENT examination
reveals mild
conjunctival injection and a nasal polyp. You plot her weight which is below the 0.4th centile.

What is the most appropriate investigation?

A CXR and Sweat test

B CXR and Total IgE

C Specific IgE and Skin prick testing

D Total IgE and Skin prick testing

E Total IgE and Specific IgE

Explanation
The most appropriate investigation is CXR and Sweat test - Nasal congestion, cough and conjunctivitis may point
to a
diagnosis of allergic rhinitis however the examination finding of nasal
polyps is rare and careful consideration
should therefore be given to other causes. On identification of nasal polyps in the context of a history of recurrent
cough, respiratory tract infection and faltering growth, Cystic Fibrosis should be excluded. Chest X-ray is useful
to look for evidence of acute and chronic lower respiratory tract infection
and also rules out dextrocardia and situs
inversus which would support a
diagnosis of Primary Ciliary Dyskinesia. Sweat test is the gold standard
investigation for the diagnosis of Cystic Fibrosis.

Remaining answer stems are incorrect in the context of the scenario but would help in the following ways:

Total IgE & Specific IgE


– Elevated total IgE levels support a diagnosis of allergic disease but do not identify a
particular allergen. Specific IgE testing
(such as RAST ® or ImmunoCAP®) can identify specific allergens which
may be useful particularly when symptoms are perennial and therefore are often as a result of indoor allergens.

https://mypastest.pastest.com/Secure/TestMe/Browser/436619#Top[‫ ص‬12:17:27 09/12/1437]


MyPastest

Total IgE & Skin prick testing


– Elevated total IgE levels support a diagnosis of allergic disease as before but
further tests are required to confirm what the specific allergen is (such as RAST ®, ImmunoCAP® or skin prick
testing.). Skin prick testing is performed by pricking the skin on the volar aspect of forearm with potential
allergens along with a negative (saline) and positive (histamine) control. The result is read at twenty
minutes after
administration of the allergens. The size of wheal which
develops relates to likelihood of clinical allergy.

Specific IgE & Skin prick testing


- Specific IgE testing (such as RAST ®or ImmunoCAP®) can identify specific
allergens which may be useful particularly when symptoms are perennial and therefore are often as a result of
indoor allergens. Skin prick testing will also identify specific allergens all
as above.

CXR & Total IgE - Chest x-ray is useful to look for evidence of acute and chronic lower respiratory tract infection
and also rules out dextrocardia and situs inversus which would
support a diagnosis of Primary Ciliary Dyskinesia.
Elevated total IgE levels support a diagnosis of allergic disease but further tests are required to confirm what the
specific allergen is (such as allergen specific IgE, skin prick testing etc.) all as above.
47183
Next Question

Previous Question
Tag Question
End Session
Feedback

Difficulty: Average

Peer Responses

Session Progress

Responses Correct: 0

Responses Incorrect: 30

Responses Total: 30

Responses - % Correct: 0%

https://mypastest.pastest.com/Secure/TestMe/Browser/436619#Top[‫ ص‬12:17:27 09/12/1437]


MyPastest

Blog
About Pastest
Contact Us
Help

© Pastest 2016

https://mypastest.pastest.com/Secure/TestMe/Browser/436619#Top[‫ ص‬12:17:27 09/12/1437]


MyPastest

Prefer to use the old MyPastest? Access it here »

Back to Filters

Question 25 of 53

A 14-year-old girl with cystic fibrosis is noted to have lost


4 Kg since her last clinic appointment 6 weeks ago. She
has not had any respiratory tract infections in this time. She denies any changes to her diet or bowel habit but does
admit to feeling more lethargic over
the past few months. On examination she appears underweight, her chest
is
clear and abdomen soft with no palpable masses.

What is most likely to explain this?

A Anorexia nervosa

B Diabetes

C Inflammatory bowel disease

D Intentional weight loss

E Liver cirrhosis

Explanation
The most likely diagnosis is diabetes. This 14-year-old has developed CF related diabetes which is a distinct entity
form type 1 and type 2 diabetes mellitus. Inspissated secretions lead to damage and scarring of the pancreas which
primarily results in deficient insulin production although insulin resistance also has a role
in the pathogenesis. A
history of acute weight loss should raise the possibility of diabetes in any child but a high degree of suspicion is
required for children with CF. Other symptoms may include polydipsia, polyuria, lethargy and unexplained
deterioration of lung function. All patients with CF are routinely screened for diabetes when followed up in
clinic.
An oral glucose tolerance test (OGTT ) is the gold standard investigation and should be performed on an annual
basis.

Weight
loss may be intentional or occur as a result of medical or psychiatric conditions. Eating disorders should
be considered in all underweight adolescents, particularly females, however in a child known to have cystic
fibrosis medical complications of the condition (such as CF related diabetes) should be ruled out first. A diagnosis

https://mypastest.pastest.com/Secure/TestMe/Browser/436619#Top[‫ ص‬12:17:52 09/12/1437]


MyPastest

of anorexia nervosa requires the following; BMI ≤17.5, self-induced weight loss, body image distortion (dread of
being fat, excessively low target weight) and endocrine disorder (e.g. amenorrhoea).

Inflammatory bowel disease commonly presents in adolescents and may result in significant weight loss however
in this case there is no history of change in bowel habit such as diarrhoea and blood/mucous per rectum, so
C is
less likely.

Stem D is unlikely. The majority of patients


who have lost weight intentionally will be forthcoming with the fact
that they have purposefully done so. A careful history of diet and exercise, particularly in adolescent girls, along
with a collateral history is necessary to explore the possibility of an eating disorder.

Liver
cirrhosis may occur in cystic fibrosis as inspissated bile leads to obstruction of the intrahepatic biliary ducts
resulting in fibrosis. The condition most commonly presents with hepatomegaly and there is no suggestion of such
a finding in this scenario. Ursodeoxycholic acid is the mainstay of medical treatment.
47184

Next Question

Previous Question Tag Question

Feedback
End Session

Difficulty: Average

Peer Responses

Session Progress

Responses Correct: 0

Responses Incorrect: 31

Responses Total: 31

Responses - % Correct: 0%

https://mypastest.pastest.com/Secure/TestMe/Browser/436619#Top[‫ ص‬12:17:52 09/12/1437]


MyPastest

Blog
About Pastest
Contact Us
Help

© Pastest 2016

https://mypastest.pastest.com/Secure/TestMe/Browser/436619#Top[‫ ص‬12:17:52 09/12/1437]


MyPastest

Prefer to use the old MyPastest? Access it here »

Back to Filters

Question 26 of 53

A 13-month-old is referred with concerns of poor weight gain.


She has fallen from the 25th centile to below the
second. Her parents
report she is often ‘chesty’ and has had two previous admissions for chest infections. She has
6-8 bowel motions per day. On
examination she is clearly small for her age, auscultation reveals bilateral coarse
crepitations. Cardiovascular examination is unremarkable. Abdominal examination reveals a palpable liver edge
but nil else of note.

Which of the following is the most likely diagnosis?

A Cystic Fibrosis

B Glycogen storage disease

C Immunodeficiency

D Inflammatory bowel disease

E Primary Ciliary Dyskinesia

Explanation
This child is most likely to have cystic fibrosis
which is an autosomal dominantly inherited disorder that affects
multiple systems. Classical presentation is with recurrent respiratory tract infections and gastrointestinal
disturbance associated with pancreatic insufficiency. Mutations of the Cystic Fibrosis Transmembrane Regulator
(CFTR) gene lead to impaired chloride secretion which results in movement of sodium and water into cells
leaving viscous
secretions behind. In the lungs this predisposes to recurrent infection and subsequent
bronchiectasis. Pancreatic insufficiency leads
to poor fat absorption and therefore frequent fatty stools. CF should
be ruled out early in children presenting with respiratory, gastrointestinal or growth concerns.

Glycogen storage disease


is caused by an autosomal recessive inherited enzyme defect which results in an
inability to break down glycogen into glucose. The classical presentations are with hypoglycaemia at birth and
hepatomegaly
on examination. Although this child has a palpable liver edge the diagnosis of a glycogen storage

https://mypastest.pastest.com/Secure/TestMe/Browser/436619#Top[‫ ص‬12:18:16 09/12/1437]


MyPastest

disorder would not explain the respiratory and gastrointestinal signs and symptoms.

Primary immunodeficiency should be considered in cases of recurrent infection and faltering growth. The
umbrella term covers a wide range of conditions arising due to defects of lymphocytes, neutrophils and
complement. Cystic Fibrosis is by far more common however and should be
ruled out early in children presenting
with respiratory, gastrointestinal or growth concerns.

Inflammatory bowel disease classically presents with diarrhoea, blood/mucous per rectum, abdominal pain and
faltering growth. It is rare in a child as young as this and does not explain the respiratory symptoms and signs.

Primary Ciliary Dyskinesia is an autosomal recessive disorder resulting in abnormal ciliary function. Impaired
clearance of mucous then results in
recurrent respiratory infections progressing to bronchiectasis. The diagnosis
should be amongst the differential for recurrent respiratory tract infections and faltering growth but does not fit the
gastrointestinal symptoms.
47185

Next Question

Previous Question Tag Question

Feedback
End Session

Difficulty: Average

Peer Responses

Session Progress

Responses Correct: 0

Responses Incorrect: 32

Responses Total: 32

Responses - % Correct: 0%

https://mypastest.pastest.com/Secure/TestMe/Browser/436619#Top[‫ ص‬12:18:16 09/12/1437]


MyPastest

Blog
About Pastest
Contact Us
Help

© Pastest 2016

https://mypastest.pastest.com/Secure/TestMe/Browser/436619#Top[‫ ص‬12:18:16 09/12/1437]


MyPastest

Prefer to use the old MyPastest? Access it here »

Back to Filters

Question 27 of 53

A 15-month-old child with faltering growth and recurrent admissions with lower respiratory tract infections is
suspected of having cystic fibrosis. A sweat test is performed.

Which of the following results would support the diagnosis of CF?

A Chloride 18 mmol/L

B Chloride 22 mmol/L

C Chloride 70 mmol/L

D Na 22 mmol/L

E Na 70 mmol/L

Explanation
The sweat test is the gold standard diagnostic test for cystic fibrosis. The CF phenotype occurs due to mutations of
the Cystic Fibrosis Transmembrane Regulator (CFTR) gene which leads to impaired function of a chloride
channel. In the epithelium of the respiratory tract, gastrointestinal tract, biliary tree and pancreas the
functioning
chloride channel acts to move chloride out of the cell and into the lumen. This function is impaired in CF, chloride
remains intracellular which results in movement of sodium and water into cells leaving viscous secretions within
the lumen. The function of the chloride channel in the skin is the opposite, it acts to move chloride in to the
epithelium as opposed to out. In CF abnormal function means that high levels of chloride are found on the skin
forming the basis of the sweat test. Sweat is collected on filter paper following the administration of pilocarpine
and the help of a small electric current. It is not usually performed in the first 2 weeks of life. A positive sweat
test, and therefore a likely diagnosis of CF, is defined by a chloride level of ≥ 60 mmol/L.

Stems A and B Chloride 18/22 mmol/L – As mentioned above, a positive sweat test, and therefore a
likely
diagnosis of CF, is defined by a chloride level of ≥ 60 mmol/L. CF is very unlikely in children more than six
months of age with a chloride level <39 mmol/L.

https://mypastest.pastest.com/Secure/TestMe/Browser/436619#Top[‫ ص‬12:19:04 09/12/1437]


MyPastest

Stems D and E are incorrect because measurement of chloride forms the basis of the sweat test, not sodium.
47186

Tag Question

Feedback

Difficulty: Average

Peer Responses
Next Question

Previous Question
End Session

Session Progress

Responses Correct: 0

Responses Incorrect: 33

Responses Total: 33

Responses - % Correct: 0%

Blog
About Pastest
Contact Us
Help

© Pastest 2016

https://mypastest.pastest.com/Secure/TestMe/Browser/436619#Top[‫ ص‬12:19:04 09/12/1437]


MyPastest

Prefer to use the old MyPastest? Access it here »

Back to Filters

Question 28 of 53

A 14-year-old boy attends the paediatric outpatient clinic for review. He reports he has been having frequent
bowel motions but is
otherwise well. His routine medications include flucloxacillin, colomycin, vitamins and
creon. On examination he appears small for his age but is comfortable at rest with no signs of respiratory distress.
Respiratory examination reveals Harrison’s Sulcus and bilateral crepitations which clear on coughing. He is not
cyanosed and has no evidence of finger clubbing. His abdomen is soft and non-tender.

Which of the following organisms is UNLIKELY to be associated with his underlying condition?

A Burkholderia Cepacia

B Haemophilus Influenzae

C Pseudomonas Aeruginosa

D Salmonella typhi

E Staphylococcus Aureus

Explanation
This 14-year-old has cystic fibrosis. Salmonella
typhi is responsible for typhoid fever. Studies have shown that
proteins coded by the CFTR gene, which is abnormal in cystic fibrosis, are used by salmonella to enter the
epithelium from the gut. This suggests those with CFTR mutations, including heterozygotes (carriers of
CF), may
have resistance to infection with typhoid which provides a theory to explain why mutations of CFTR remain so
prevalent when in the recent past, patients with cystic fibrosis typically died before reproductive age.

Burkholderia Cepacia - The Burkholderia Cepacia family of bacteria are particularly problematic for
children with
cystic fibrosis. Infections can lead to a rapid decline in pulmonary function and treatment can be difficult.

Haemophilus Influenzae frequently colonises patients with cystic fibrosis in infancy or early childhood.

Pseudomonas
aeruginosa is an extremely common organism responsible for chronic infection. Multiple drug

https://mypastest.pastest.com/Secure/TestMe/Browser/436619#Top[‫ ص‬12:19:27 09/12/1437]


MyPastest

resistance is becoming problematic.

Staphylococcus Aureus infection is common. Most children with CF will be on oral flucloxacillin prophylactically.
Methicillin Resistant Staphylococcus Aureus (MRSA) is becoming an increasing problem.
47187

Tag Question

Feedback

Difficulty: Average
Next Question
Peer Responses

Previous Question
End Session

Session Progress

Responses Correct: 0

Responses Incorrect: 34

Responses Total: 34

Responses - % Correct: 0%

Blog
About Pastest
Contact Us
Help

© Pastest 2016

https://mypastest.pastest.com/Secure/TestMe/Browser/436619#Top[‫ ص‬12:19:27 09/12/1437]


MyPastest

Prefer to use the old MyPastest? Access it here »

Back to Filters

Question 29 of 53

A mother attends the emergency department with both her 3 –year-old and 18-month-old children. She has noticed
a discharge from the 18-month-olds left ear. The child has no significant past medical history and has been well
today. Earlier in the week the children were playing together, briefly unsupervised, the mother recalls
how the 18-
month-old did cry out loudly but by the time she had left the kitchen and reached the children they were
continuing to play. On examination the child is systemically well. Examination of the left ear
reveals a foul
smelling copious discharge. The child is reluctant to let you near the ear.

What is the most likely diagnosis?

A Cholesteotoma

B Foreign body

C Mastoiditis

D Skull fracture

E Viral upper respiratory tract infection

Explanation
It is relatively common for toddlers, or their younger siblings, to present to the paediatric emergency department
with
a small object lodged in the external ear canal or nostril. Offensive discharge may develop if the incident is
unwitnessed and the object remains within the canal. Removal of the object should be done with caution to avoid
pushing it further into the ear canal. Refer to ENT.

Cholesteotoma is an abnormal collection of squamous epithelial cells within the middle ear and mastoid process.
The expanding growth is erosive to the local structures. The classical presenting symptoms are discharge and
hearing impairment. Prompt identification and treatment of cholesteotoma is essential as left untreated
complications include hearing loss, facial nerve injury and potential for intracranial infection. A ‘polyp’ on
otoscopy is a cholesteotoma until proven otherwise. Refer to an ENT surgeon for assessment. The history of ear

https://mypastest.pastest.com/Secure/TestMe/Browser/436619#Top[‫ ص‬12:20:00 09/12/1437]


MyPastest

discharge in this case is short and on the background of an acute


episode of pain in an unsupervised toddler,
foreign body is the most likely diagnosis.

Mastoiditis is infection of the mastoid air cells. This classically presents with an inflamed tender mastoid process
which results in a protruding ear. Intravenous antibiotics are required.

A skull fracture may be obtained from accidental or non-accidental injury. When head injury is suspected always
examine the
ears carefully to exclude haemotympanum and CSF ottorhoea that would suggest a base of skull
fracture. In this case the child is otherwise well and the discharge has an offensive odour pointing towards a
foreign
body.

Viral upper respiratory tract infection would be associated with symptoms such as coryza, cough, sore throat, poor
oral intake and fever. The throat and tympanic membranes may be erythematous
on inspection but ear discharge
would not be expected without a secondary bacterial ear infection.
47188

Next Question

Previous Question Tag Question

Feedback
End Session

Difficulty: Average

Peer Responses

Session Progress

Responses Correct: 0

Responses Incorrect: 35

Responses Total: 35

Responses - % Correct: 0%

https://mypastest.pastest.com/Secure/TestMe/Browser/436619#Top[‫ ص‬12:20:00 09/12/1437]


MyPastest

Blog
About Pastest
Contact Us
Help

© Pastest 2016

https://mypastest.pastest.com/Secure/TestMe/Browser/436619#Top[‫ ص‬12:20:00 09/12/1437]


MyPastest

Prefer to use the old MyPastest? Access it here »

Back to Filters

Question 30 of 53

A mother attends the emergency department with both her 3-year-old and 18-month-old children. The 18-month-
old has been vomiting and lethargic over the past 2-3 hours. The child has no significant past medical history and
was well in the morning. Earlier in the afternoon the children were playing together, briefly unsupervised as mum
was in the kitchen. The mother recalls how the 18-month-old did cry out suddenly and required comforting for ten
minutes following which he continued to play. On examination the 18-month-old is alert but miserable. ENT
examination reveals a lightly blood stained serous discharge from the left ear. Otoscopy is difficult, as the child is
combative, but the tympanic membrane appears dark.

What is the most appropriate investigation?

A CT Head

B MRI Head

C Meningococcal/Pneumococcal PCR, Blood culture, Lumbar puncture

D Nasopharyngeal swab

E Send discharge for microscopy, culture and sensitivities

Explanation
A significant head injury should be suspected in this scenario. This child has signs of a base of skull fracture with
haemotympanum on ototscopy and CSF otorrhoea. A CT scan is indicated as
per NICE guidelines. This should be
within one hour if ANY of the following features are present; Suspicion of NAI, post-traumatic seizure, GCS < 14
(<15 in infants) on initial assessment, GCS <
15 two hours post injury, suspected open or depressed skull fracture,
suspected basal skull fracture, focal neurological deficit, bruising or swelling > 5cm in an infant.

As explained above, CT is the first line investigation for significant head injury in both children and adults. MRI
can provide additional information but in view of safety, logistical reasons and limited resources CT is
recommended as first line.

https://mypastest.pastest.com/Secure/TestMe/Browser/436619#Top[‫ ص‬12:21:06 09/12/1437]


MyPastest

Meningococcal/Pneumococcal PCR, Blood culture, Lumbar puncture - These investigations are indicated for
suspected meningitis / sepsis. There is not a history of fever in this case nor description of meningism or non-
blanching rash. The clinical findings suggest a base of skull fracture.

Nasopharyngeal swab is
useful to identify infectious organisms within the respiratory tract, particularly viruses.
One of the most common uses is to identify the causative organism in bronchiolitis. The clinical findings in this
case
however suggest a base of skull fracture, there is no history of fever to suggest infection.

Sending the fluid to the laboratory would allow confirmation that it is in fact CSF but this is unnecessary as a
suspicion of significant head injury with haemotympanum is indication for immediate imaging which will identify
a skull fracture. MC&S would be useful if a local ear infection was suspected but there is no history of fever to
suggest infection and there are signs of significant
head injury.

NICE Guidelines:

https://www.nice.org.uk/guidance/conditions-and-diseases/injuries--accidents-and-wounds/head-injuries
Next Question 47189

Previous Question
End
Tag Session
Question

Feedback

Difficulty: Average

Peer Responses

Session Progress

Responses Correct: 0

Responses Incorrect: 36

Responses Total: 36

Responses - % Correct: 0%

https://mypastest.pastest.com/Secure/TestMe/Browser/436619#Top[‫ ص‬12:21:06 09/12/1437]


MyPastest

Blog
About Pastest
Contact Us
Help

© Pastest 2016

https://mypastest.pastest.com/Secure/TestMe/Browser/436619#Top[‫ ص‬12:21:06 09/12/1437]


MyPastest

Prefer to use the old MyPastest? Access it here »

Back to Filters

Question 31 of 53

A mother attends clinic with both her 3-year-old and 18-month-old children. She has noticed a discharge from the
3-year-old’s left ear over the past two weeks. He has a history of frequent episodes of ‘ear infection’ but has been
otherwise well. There is no reported fever. On examination the child is systemically well. The child is co-operative
with otoscopy which shows an intact pink tympanic membrane on the right but a possible perforation on the left.

What is the next best step in management?

A Arrange CT Head

B Oral Amoxicillin

C Reassurance

D Refer for ENT opinion

E Topical aminoglycosides

Explanation
This child has Chronic Suppurative Otitis Media which is chronic inflammation of the middle ear. It presents with
recurrent discharge (> 2 weeks) which escapes the middle ear through a
perforated tympanic membrane. There is
absence of acute infection, i.e. no fever or pain, however a previous history of acute otitis media supports the
diagnosis. Examination is not uncomfortable for the child and identifies a perforated tympanic membrane. There
may be associated hearing impairment which typically resolves when the tympanic membrane heals.
Complications include mastoiditis, facial nerve palsy and intracranial infection. If Chronic Supporative Otitis
Media is suspected ENT referral is indicated, the ears should not be swabbed and no treatment should be given.

As this child has Chronic Supporative Otitis Media the child should be referred to an ENT surgeon.
CT head may
be indicated for complications such as mastoiditis but this should be under the care of an ENT specialist.

Amoxicillin
is used for the treatment of acute otitis media however if Chronic Supporative Otitis Media is
suspected, as in this case, the child should
be referred to an ENT surgeon. The ears should not be swabbed and no

https://mypastest.pastest.com/Secure/TestMe/Browser/436619#Top[‫ ص‬12:23:48 09/12/1437]


MyPastest

treatment should be given.

Parents/Carers may be reassured that


any hearing impairment is likely to resolve completely as the tympanic
membrane heals however referral to an ENT surgeon is indicated as the condition can be associated with
complications such as mastoiditis, facial nerve palsy and intracranial infection.

Topical aminoglycosides are contraindicated in the presence of tympanic membrane perforation so stem E is
incorrect.

NICE guidelines: http://cks.nice.org.uk/otitis-media-chronic-suppurative


47190

Next Question Tag Question

Feedback

Previous Question
Difficulty: Average
End Session
Peer Responses

Session Progress

Responses Correct: 0

Responses Incorrect: 37

Responses Total: 37

Responses - % Correct: 0%

Blog
About Pastest
Contact Us
Help

© Pastest 2016

https://mypastest.pastest.com/Secure/TestMe/Browser/436619#Top[‫ ص‬12:23:48 09/12/1437]


MyPastest

https://mypastest.pastest.com/Secure/TestMe/Browser/436619#Top[‫ ص‬12:23:48 09/12/1437]


MyPastest

Prefer to use the old MyPastest? Access it here »

Back to Filters

Question 31 of 53

A mother attends clinic with both her 3-year-old and 18-month-old children. She has noticed a discharge from the
3-year-old’s left ear over the past two weeks. He has a history of frequent episodes of ‘ear infection’ but has been
otherwise well. There is no reported fever. On examination the child is systemically well. The child is co-operative
with otoscopy which shows an intact pink tympanic membrane on the right but a possible perforation on the left.

What is the next best step in management?

A Arrange CT Head

B Oral Amoxicillin

C Reassurance

D Refer for ENT opinion

E Topical aminoglycosides

Explanation
This child has Chronic Suppurative Otitis Media which is chronic inflammation of the middle ear. It presents with
recurrent discharge (> 2 weeks) which escapes the middle ear through a
perforated tympanic membrane. There is
absence of acute infection, i.e. no fever or pain, however a previous history of acute otitis media supports the
diagnosis. Examination is not uncomfortable for the child and identifies a perforated tympanic membrane. There
may be associated hearing impairment which typically resolves when the tympanic membrane heals.
Complications include mastoiditis, facial nerve palsy and intracranial infection. If Chronic Supporative Otitis
Media is suspected ENT referral is indicated, the ears should not be swabbed and no treatment should be given.

As this child has Chronic Supporative Otitis Media the child should be referred to an ENT surgeon.
CT head may
be indicated for complications such as mastoiditis but this should be under the care of an ENT specialist.

Amoxicillin
is used for the treatment of acute otitis media however if Chronic Supporative Otitis Media is
suspected, as in this case, the child should
be referred to an ENT surgeon. The ears should not be swabbed and no

https://mypastest.pastest.com/Secure/TestMe/Browser/436619#Top[‫ ص‬12:24:55 09/12/1437]


MyPastest

treatment should be given.

Parents/Carers may be reassured that


any hearing impairment is likely to resolve completely as the tympanic
membrane heals however referral to an ENT surgeon is indicated as the condition can be associated with
complications such as mastoiditis, facial nerve palsy and intracranial infection.

Topical aminoglycosides are contraindicated in the presence of tympanic membrane perforation so stem E is
incorrect.

NICE guidelines: http://cks.nice.org.uk/otitis-media-chronic-suppurative


47190

Next Question Tag Question

Feedback

Previous Question
Difficulty: Average
End Session
Peer Responses

Session Progress

Responses Correct: 0

Responses Incorrect: 37

Responses Total: 37

Responses - % Correct: 0%

Blog
About Pastest
Contact Us
Help

© Pastest 2016

https://mypastest.pastest.com/Secure/TestMe/Browser/436619#Top[‫ ص‬12:24:55 09/12/1437]


MyPastest

https://mypastest.pastest.com/Secure/TestMe/Browser/436619#Top[‫ ص‬12:24:55 09/12/1437]


MyPastest

Prefer to use the old MyPastest? Access it here »

Back to Filters

Question 32 of 53

A 2-year-child presents to the emergency department with a three day history of coryzal symptoms and gradual
onset of increased work of breathing. On examination the child appears very comfortable at
rest and fairly playful.
There is mild intercostals and subcostal recession along with tracheal tug. The temperature is 38.1oC and the
oxygen saturations are 98% in room air. There is an audible inspiratory
stridor.

What is the best next step in management?

A Dexamethasone 0.15mg/Kg PO

B IM Adrenaline 150 micrograms

C Perform a full ENT examination

D Request ENT and Anaesthetic support

E Send Nasopharyngeal swab

Explanation
Dexamethasone 0.15mg/Kg PO is the best answer. This child has croup which results from an acute viral illness
causing inflammation of the upper respiratory tract. It classically presents with stridor, hoarse voice, fever and
‘barking cough’ on the
background of a preceding coryzal illness. Steroids are the first line
treatment, oral
dexamethasone is currently the most commonly used. The
condition is usually mild with only five percent of
children requiring admission to hospital however intubation and ventilation is necessary for the most severely
affected.

Intramuscular adrenaline is used in the management of anaphylaxis so B is an incorrect answer. In a


child with
severe croup nebulised adrenaline may be used to buy time for steroid treatment to take effect. A 1 in 1000
solution is used at a
dose of 400 micrograms/Kg (maximum 5mg) and may be repeated after thirty minutes. The
effect of the nebulised adrenaline lasts approximately two to three hours, close observation is required for
recurrence of symptoms.

https://mypastest.pastest.com/Secure/TestMe/Browser/436619#Top[‫ ص‬12:25:17 09/12/1437]


MyPastest

Although this child is not in extremis, examination of the throat should be avoided in any children with stridor as
this could provoke worsening of the airway obstruction so C is incorrect.

Expert anaesthetic and ENT support should be requested for severe airway obstruction which is relatively rare for
simple viral croup. In a systemically unwell / toxic child, or when drooling is significant, bacterial tracheitis and
epiglottitis should be
considered and anaesthetic / ENT assistance requested urgently. In this case the child is
comfortable with mild respiratory distress, treatment with steroids is warranted along with close observation.

Nasopharyngeal
swab is useful to identify infectious organisms within the respiratory tract, particularly viruses.
One of the most common uses is to identify
the causative organism in bronchiolitis to allow cohorting of admitted
infants. The clinical findings in this case however suggest croup (viral laryngotracheobronchitis). The child has
stridor and therefore swabbing the throat is contraindicated, as is examination of the throat,
as this may provoke
worsening of the airway obstruction so E is incorrect.
47191

Next Question

Previous Question Tag Question

Feedback End Session

Difficulty: Average

Peer Responses

Session Progress

Responses Correct: 0

Responses Incorrect: 38

Responses Total: 38

Responses - % Correct: 0%

Blog
About Pastest
Contact Us
Help

https://mypastest.pastest.com/Secure/TestMe/Browser/436619#Top[‫ ص‬12:25:17 09/12/1437]


MyPastest

© Pastest 2016

https://mypastest.pastest.com/Secure/TestMe/Browser/436619#Top[‫ ص‬12:25:17 09/12/1437]


MyPastest

Prefer to use the old MyPastest? Access it here »

Back to Filters

Question 1 of 53

A 5-year-old boy had a number of previous admissions for ‘virally induced wheeze’. He has been using a ß-
agonist inhaler at home on required basis. He presented to surgery and his mum tells you that he becomes wheezy
with changes in temperature, laughter, exercise and with viral illnesses. He has to use his inhaler three times per
week and coughs on most days, usually at night.

The best next step in his management would be?

A Increase the number of puffs of his ß-agonist that he is having

B Add a low-dose steroid inhaler

C Add a high-dose steroid inhaler

D Add a long-acting ß-agonist

E Add ipratropium inhaler

Explanation
There are multiple, typical points in the history
pointing to poorly controlled asthma. You should be familiar with
the British Thoracic Society Guidelines for the management of asthma as this
is a common problem and it is
virtually guaranteed that there will be questions on asthma and its management in the exam. In brief, management
of chronic episodic asthma is to start with a short-acting ß-agonist as required. Add low-dose inhaled steroids if
symptoms are frequent and use of ß-agonist is also frequent. If symptoms persist, add a long-acting ß-agonist (e.g
salmeterol) or try a leukotriene receptor antagonist (e.g. montelukast). The next step will be to increase the dose of
inhaled steroids. If symptoms still persist, consider continuous or frequent use of oral steroids.

The British Thoracic Society Guidelines can be downloaded from the following website:

http://www.brit-thoracic.org.uk/guidelines-and-quality-standards/asthma-guideline/
10425

https://mypastest.pastest.com/Secure/TestMe/Browser/436619[‫ ص‬04:32:39 10/12/1437]


MyPastest

Tag Question

Feedback

Next Question
Difficulty: Easy

Peer Responses

End Session

Previous Question

Session Progress

Responses Correct: 0

Responses Incorrect: 1

Responses Total: 1

Responses - % Correct: 0%

Blog
About Pastest
Contact Us
Help

© Pastest 2016

https://mypastest.pastest.com/Secure/TestMe/Browser/436619[‫ ص‬04:32:39 10/12/1437]


MyPastest

Prefer to use the old MyPastest? Access it here »

Back to Filters

Question
Next 2 of 53
Question

You are seeing a 4-week-old female infant in clinic. She has been making persistent noises whilst breathing,
especially when upset. She is otherwise well and feeding and growing nicely. Examination
today reveals a
healthy-looking infant. She is not in distress but then
she becomes upset when you handle her and develops an
inspiratory wheeze. There is obvious subcostal, intercostal and suprasternal recession but no cyanosis. She soon
settles down and the symptoms subside. Examination is otherwise unremarkable.

The most likely diagnosis is?

A Neonatal tetany

B Subglottic stenosis

C Vascular ring

D Laryngomalacia

E Laryngeal nerve palsy

Explanation
Inspiratory wheeze signifies stridor. She is only 4 weeks of age so, for this symptom to be described as persistent,
it must more or less have been there since birth. Around 65%
of persistent stridor in infancy is due to
laryngomalacia. It usually presents at birth, but can present at any stage up to 4 weeks. Most children with
laryngomalacia thrive and feed normally. It usually resolves spontaneously by 18 months of age. Other conditions
to consider
include neonatal tetany, subglottic stenosis, subglottic haemangioma, laryngeal nerve palsy, laryngeal
web, vascular ring, congenital floppy larynx and goitre - but these are less common.
10426

https://mypastest.pastest.com/Secure/TestMe/Browser/436619[‫ ص‬04:32:59 10/12/1437]


MyPastest

Tag Question

Feedback

Difficulty: Easy

Peer Responses

End Session

Session Progress

Responses Correct:
Previous Question 0

Responses Incorrect: 2

Responses Total: 2

Responses - % Correct: 0%

Blog
About Pastest
Contact Us
Help

© Pastest 2016

https://mypastest.pastest.com/Secure/TestMe/Browser/436619[‫ ص‬04:32:59 10/12/1437]


MyPastest

Prefer to use the old MyPastest? Access it here »

Back to Filters

Question 3 of 53

Theme: asthma management

A Salbutamol inhaler regularly for the next 12 hours and three times in the first hour
B Start salbutamol via appropriate inhaler as required
C Salbutamol nebulisers regularly for the next 12 hours and three times in the first hour
D Start ipratropium via appropriate inhaler regularly
E Start ipratropium via appropriate inhaler as required
F Give a 5 day course of oral steroids
G Give steroid via appropriate inhaler regularly
H Give steroid via appropriate inhaler as required
I No treatment required

For
each of the scenarios choose the first 'short term' plan of action. Each option may be used once, more than
once, or not at all.

Scenario 1

A 3-year-old boy presents with episodic wheeze worse when playing or running around.

Your answer was incorrect

Select one...

B - Start salbutamol via appropriate inhaler as required

This child is experiencing symptoms of acute asthma triggered by exercise. In this instance a short
acting ß-
agonist via spacer should be the first line treatment. Therapy should be reviewed at a later date to assess the
frequency of use and adequacy of symptom control.

Scenario 2

A 7-year-old known asthmatic boy presents with a 2-hour history of acute shortness of breath. He has widespread
wheeze but is talking and his O2 saturations are 94% in room air.

Your answer was incorrect

https://mypastest.pastest.com/Secure/TestMe/Browser/436619[‫ ص‬04:33:21 10/12/1437]


MyPastest

Select one...

A - Salbutamol inhaler regularly for the next 12 hours and 3 times in the 1st hour

The 7-year-old boy is having a moderate acute exacerbation of asthma. As he is not requiring oxygen, inhalers are
the most appropriate treatment. He would benefit from having 10 puffs every 20 minutes for the first hour and will
then need to use the inhaler regularly for the next 12 hours and possibly even longer.

Therapy should be reviewed according to clinical response and a short course of steroids considered if symptoms
have not resolved quickly. Next Question
Scenario 3

A 10-year-old known asthmatic attends surgery with a painful right toe. He has had several hospital admissions
for
asthma in the past, but has not had an asthma attack for one year. He has no night-time symptoms and describes no
exercise limitation. He was prescribed daily inhaled steroids via large volumeEnd
spacerSession
4 years ago, but his mother
concedes that he has not been using this much over the last 12 months.

Your answer was incorrect

Select one...
Previous Question
I - No treatment required

The 10-year-old boy has no interval symptoms despite not using his inhaled steroids. It would be appropriate
to
stop all his treatment if he has not required inhaled steroids and has been symptom free for one year. Asthma
therapy should be reviewed regularly.
11551

Tag Question

Feedback

Difficulty: Average

Session Progress

Responses Correct: 0

Responses Incorrect: 5

Responses Total: 5

Responses - % Correct: 0%

https://mypastest.pastest.com/Secure/TestMe/Browser/436619[‫ ص‬04:33:21 10/12/1437]


MyPastest

Blog
About Pastest
Contact Us
Help

© Pastest 2016

https://mypastest.pastest.com/Secure/TestMe/Browser/436619[‫ ص‬04:33:21 10/12/1437]


MyPastest

Prefer to use the old MyPastest? Access it here »

Back to Filters

Question
Next 4 of 53
Question

Which one of the following features is MOST characteristic of cystic fibrosis?

A Inherited as autosomal dominant

B Previous
Pancreatic Question
insufficiency is almost always identified in adult patients

C In patients with recurrent chest infections Burkholderia cepacia (B. cepacia) is the most frequent
organ ism isolated from sputum

D Family members who carry the gene are at risk of developing mild recurrent bronchitis

E Patients typically have reduced levels of sodium and chloride in the sweat

Explanation
Cystic fibrosis (CF) is an autosomal recessive disease affecting both eccrine and exocrine gland function,
characterised by elevated levels of sodium and chloride in the sweat. It is caused by abnormal viscid secretions
from mucous glands leading to chronic pulmonary disease and pancreatic insufficiency, which will be evident in
more than 90% of adult cases. Recurrent chest infections are usually caused by Pseudomonas aeruginosa and
Staphylococcus aureus. B. cepacia occurs in
only 5–10% of cases. The carrier frequency is 1:25 in the Caucasian
population, but these heterozygotes are clinically normal.
12190

Tag Question

https://mypastest.pastest.com/Secure/TestMe/Browser/436619[‫ ص‬04:33:41 10/12/1437]


MyPastest

Feedback

Difficulty: Average

Peer Responses

End Session

Session Progress

Responses Correct: 0

Responses Incorrect: 6

Responses Total: 6

Responses - % Correct: 0%

Blog
About Pastest
Contact Us
Help

© Pastest 2016

https://mypastest.pastest.com/Secure/TestMe/Browser/436619[‫ ص‬04:33:41 10/12/1437]


MyPastest

Prefer to use the old MyPastest? Access it here »

Back to Filters

Question
Next 5 of 53
Question

Most cases of community acquired pneumonia in the under 5s are caused by which one of the following?

A Streptococcus pneumoniae

B Previous
Mycoplasma Question
pneumoniae

C Staphylococcus aureus

D Haemophilus influenzae

E Viral pneumonia

Explanation
Community acquired pneumonia is, by definition, contracted in the community rather than in hospital. In the
northern hemisphere, community acquired pneumonia affects approximately 12/1000 people per year, particularly
during winter and at the extremes of age (incidence: < 1 year old 30–50/1000 per year; 71–85 years 50/1000 per
year). Over 100 microorganisms have been implicated, but most cases are caused by viruses whereas most
bacterial cases are caused by Streptococcus pneumoniae particularly in the under 5s whereas Mycoplasma is more
common in the over 5s.
12191

Tag Question

Feedback

https://mypastest.pastest.com/Secure/TestMe/Browser/436619[‫ ص‬04:34:26 10/12/1437]


MyPastest

Difficulty: Average

Peer Responses

End Session

Session Progress

Responses Correct: 0

Responses Incorrect: 7

Responses Total: 7

Responses - % Correct: 0%

Blog
About Pastest
Contact Us
Help

© Pastest 2016

https://mypastest.pastest.com/Secure/TestMe/Browser/436619[‫ ص‬04:34:26 10/12/1437]


MyPastest

Prefer to use the old MyPastest? Access it here »

Back to Filters

Question 6 of 53

A 16-year-old trainee nursery nurse presents to A&E acutely unwell. She has a cough and has become
progressively worse, now with severe shortness of breath. Her flatmate reports that she has become unsteady on
her feet over the past few days. There is no important past history of note, she smokes 20 cigarettes per day. There
is a striking rash over her scalp, face and trunk with a mixed crop of vesicles and pustules suggestive of
chickenpox. On examination she has bilateral cerebellar ataxia.

Which diagnosis fits best with this clincal picture?

A Acute varicella zoster virus infection with CNS and lung involvement

B Bacterial pneumonia

C Herpes simplex encephalitis

D Generalised viral infection of unknown origin

E Likely immunocompromised patient

Explanation
Although uncommon, lung involvement with acute chickenpox infection is seen. The pneumonitis generally
begins some 1–6 days after the appearance of the skin eruption and is more common in young adults, with
cigarette smokers being particularly at risk. This woman was not exposed to chickenpox as a child, and exposure
has now occurred due to her work as a nursery nurse. The chest radiograph normally shows diffuse lung changes
throughout both lung fields, but symptoms are commonly worse than the X-ray picture. CNS involvement is rarer,
occurring in around 1 in 1000 cases, and presenting with acute truncal ataxia. Treatment is with intravenous
aciclovir, with support from the high-dependency/intensive care unit as needed.
12192

https://mypastest.pastest.com/Secure/TestMe/Browser/436619[‫ ص‬04:35:06 10/12/1437]


MyPastest

Tag Question

Feedback

Next Question
Difficulty: Easy

Peer Responses

End Session

Previous Question

Session Progress

Responses Correct: 0

Responses Incorrect: 8

Responses Total: 8

Responses - % Correct: 0%

Blog
About Pastest
Contact Us
Help

© Pastest 2016

https://mypastest.pastest.com/Secure/TestMe/Browser/436619[‫ ص‬04:35:06 10/12/1437]


MyPastest

Prefer to use the old MyPastest? Access it here »

Back to Filters

Question 7 of 53

A 16-year-old girl presents to A&E with a severe asthma attack. What is the most important therapy to relieve her
bronchoconstriction?

A Propranolol

B Salbutamol

C Oxygen

D Glucocorticosteroids

E Cromoglicate

Explanation
The actions of ß-agonists in asthma are the
result of stimulation of ß-adrenoreceptors that are located in the
airways, on airway epithelium, and in submucosal glands, airway and vascular smooth muscle. ß-adrenergic
receptors in the airways are entirely ß2, with the exception of some ß1-receptors on submucosal glands. ß2-agonists
can influence airway function through several mechanisms:

relaxation of bronchial smooth muscle by direct effect on ß2-receptors


inhibition of mast-cell mediator release, and
enhanced mucociliary clearance.

Inhalation of a ß2-agonist by a patient with asthma increases airway calibre and reduces airway hyper
responsiveness. ß2-agonists
also cause tachycardia and increased cardiac output, systemic vasodilatation and
increased muscle blood flow. The tachycardia and increased cardiac output are the results of both stimulation of
cardiac ß-adrenoreceptors and a reflex response to peripheral vasodilatation. In addition, ß2-agonists cause tremor
and have metabolic actions, of which hypokalaemia is probably the only potentially important clinical effect.

https://mypastest.pastest.com/Secure/TestMe/Browser/436619[‫ ص‬04:35:28 10/12/1437]


MyPastest

12193

Tag Question

Feedback

Next Question
Difficulty: Easy

Peer Responses

End Session

Previous Question

Session Progress

Responses Correct: 0

Responses Incorrect: 9

Responses Total: 9

Responses - % Correct: 0%

Blog
About Pastest
Contact Us
Help

© Pastest 2016

https://mypastest.pastest.com/Secure/TestMe/Browser/436619[‫ ص‬04:35:28 10/12/1437]


MyPastest

Prefer to use the old MyPastest? Access it here »

Back to Filters

Question 8 of 53

A 16-year-old boy has been complaining of shortness of breath for the last 2 days. On examination bronchial
breathing is heard over the right lower lobe.

What is the most likely diagnosis for this clinical finding?

A Pneumothorax

B Pneumonia

C Asthma

D Emphysema

E Chronic obstructive pulmonary disease

Explanation
Bronchial breathing is heard over an airless lung, such as in consolidation, atelectasis or dense fibrosis. There is
some resemblance to the sounds heard over the normal trachea, but, by comparison with normal breath sounds,
bronchial breathing is higher in pitch and more blowing in quality. It does not have to be loud. Bronchial breath
sounds are classically heard throughout both inspiration and expiration. Very quiet breath sounds are heard over
hyperinflated lungs, as in emphysema, or when breath sounds are prevented from reaching the chest wall by a
layer of air, fluid or fibrosis.

The classic presentation of pneumonia is of a cough and fever with the variable presence of sputum production,
dyspnoea and pleurisy. Most patients have constitutional symptoms such as malaise, fatigue and asthenia, and
many also have gastrointestinal symptoms. Although patients with pneumonia usually possess these characteristic
clinical features, there can be major differences in presentation based on the host and the aetiological agent.
12194

https://mypastest.pastest.com/Secure/TestMe/Browser/436619[‫ ص‬04:35:49 10/12/1437]


MyPastest

Tag Question

Feedback

Next Question
Difficulty: Easy

Peer Responses

End Session

Previous Question

Session Progress

Responses Correct: 0

Responses Incorrect: 10

Responses Total: 10

Responses - % Correct: 0%

Blog
About Pastest
Contact Us
Help

© Pastest 2016

https://mypastest.pastest.com/Secure/TestMe/Browser/436619[‫ ص‬04:35:49 10/12/1437]


MyPastest

Prefer to use the old MyPastest? Access it here »

Back to Filters

Question
Next 9 of 53
Question

A 12-year-old girl is admitted to the emergency department. She had been found by her mother and younger sister
when they returned back from shopping. When they got back to the flat they tell you she was unsteady on her feet
and having problems with her memory. They tell you they have been using old fashioned gas fires as it
has been
so cold. You are wondering whether she could have carbon monoxide poisoning.

Which test will be most helpful in determining this?

A Clinical examination

B Pulse oximetry

C Arterial blood oxygen level

D Exhaled breath test

E Chest radiograph

Explanation
Breath carbon monoxide levels can be measured using devices intended for smoking cessation work. They can
also be measured on a co-oximeter, measuring carboxyhaemoglobin levels. Clinical examination is unhelpful, as
patients are not cyanosed, but a cherry-red colour. Pulse oximetry appears normal, due to carboxyhaemoglobin
having similar absorption spectra to oxyhaemoglobin. Arterial oxygen levels may be normal, as may a chest
radiograph. Treatment for carbon monoxide poisoning is to give high levels of inspired oxygen, by ventilation (or
hyperbaric means if necessary), to displace the carbon monoxide and increase the amount of dissolved oxygen.
12538

https://mypastest.pastest.com/Secure/TestMe/Browser/436619[‫ ص‬04:36:11 10/12/1437]


MyPastest

Tag Question

Feedback

Difficulty: Average

Peer Responses

End Session

Previous Question Session Progress

Responses Correct: 0

Responses Incorrect: 11

Responses Total: 11

Responses - % Correct: 0%

Blog
About Pastest
Contact Us
Help

© Pastest 2016

https://mypastest.pastest.com/Secure/TestMe/Browser/436619[‫ ص‬04:36:11 10/12/1437]


MyPastest

Prefer to use the old MyPastest? Access it here »

Back to Filters

Question 10 of 53
Next Question

A 2-year-old girl presents to your Emergency Department


with a history of cough and a 6-hour history of noisy
End Session
breathing. She has a barking cough, mild recession and mild stridor.

What is the best course of action?

A Take her to the resuscitation area and ask anaesthetists to attend urgently
Previous Question
B Give an oral dose of dexamethosone (600 micrograms/kg)

C Request a lateral X-ray

D Give an oral dose of dexamethasone (150 micrograms/kg) and observe for a further 4 hours

E Prescribe oral budesonide 1mg/kg

Explanation
The child has croup and she is showing signs of mild illness. In an A&E department, the most appropriate thing to
do would be to give an oral dose of dexamethasone (150 micrograms/kg) and observe for a further 4 hours. There
is little evidence for the higher dose strategy giving more benefits than the lower one. An X-ray would not help
distinguish the diagnosis and as she is maintaining her own airway there is no need for an anaesthetist.
14889

Tag Question

Feedback

https://mypastest.pastest.com/Secure/TestMe/Browser/436619[‫ ص‬04:36:36 10/12/1437]


MyPastest

Difficulty: Average

Peer Responses

Session Progress

Responses Correct: 0

Responses Incorrect: 12

Responses Total: 12

Responses - % Correct: 0%

Blog
About Pastest
Contact Us
Help

© Pastest 2016

https://mypastest.pastest.com/Secure/TestMe/Browser/436619[‫ ص‬04:36:36 10/12/1437]


MyPastest

Prefer to use the old MyPastest? Access it here »

Back to Filters

Question 11 of 53
Next Question

A 7-year-old girl was admitted with high fever and a painful cough productive of purulent sputum. The clinical
picture was that of right lower lobe pneumonia which was confirmed radiologically. She had a similar episode 9
months previously while away on holiday and was hospitalised for 3 days. The chest X-ray report indicated
identical changes at that time. There was no contact history of TB and she has not
had BCG. She was commenced
on a course of antibiotics.

The next most logical step in her management is?

A Bronchoscopy

B Mantoux test

C CT scan of the thorax

D Clinical and radiological review at follow-up

E Ventilation perfusion scan

Explanation
This child has had two acute lung infections affecting the same area of the lung in a period of nine months. As far
as the history goes she recovered fully after the first attack and had been well in the intervening period. However,
the original X-ray was not available for perusal though a copy of the report
was seen. The history does not suggest
a chronic respiratory problem and the most logical step is to review her response to antibiotics both clinically and
radiologially before further investigations are instigated. In the presence of recurrent clinical and/or radiological
abnormality, elimination of a foreign body or chronic infection (e.g. TB, pulmonary sequestration and immune
deficiency, etc) may need to be considered.
14890

https://mypastest.pastest.com/Secure/TestMe/Browser/436619[‫ ص‬04:36:57 10/12/1437]


MyPastest

Tag Question

Feedback

Difficulty: Average

Peer Responses

End Session

Previous Question Session Progress

Responses Correct: 0

Responses Incorrect: 13

Responses Total: 13

Responses - % Correct: 0%

Blog
About Pastest
Contact Us
Help

© Pastest 2016

https://mypastest.pastest.com/Secure/TestMe/Browser/436619[‫ ص‬04:36:57 10/12/1437]


MyPastest

Prefer to use the old MyPastest? Access it here »

Back to Filters

Question 12 of 53

Theme: Respiratory medicine

A Intensive chest physiotherapy


B 24-hour oesophageal pH monitoring
C Prescribe high dose inhaled steroids via a spacer +/- mask
D Measure sweat electrolytes
E Request a diary of twice daily peak flow measurements
F Refer to the dietician for a detailed nutritional assessment
G Prescribe 2 weeks of oral ciprofloxacin
H Oral amoxicillin for a 7 day course
I Refer for a routine flexible bronchoscopy

For
each of the scenarios below, select the most appropriate way to proceed
from the options above. Each option
may be used once, more than once, or not at all.

Scenario 1

A 2-year-old child with poor growth has seen the GP with recurrent chestiness and delayed speech. He is on
inhaled sodium cromoglycate via a spacer with equivocal benefit. He was admitted at 9 months of age with
bronchiolitis. In a recent referral to the ENT department they detected nasal polyps and prescribe intranasal
steroids.

Your answer was incorrect

Select one...

D - Measure sweat electrolytes

This child is displaying


features suggestive of CF, in which case a sweat test is the most appropriate thing to do
next from the options given.

Scenario 2

A 4-year-old girl has had troublesome nocturnal cough on about 4 nights per week for over a year. Another GP
thought she might have asthma and prescribed budesonide via a
turbohaler which has made no difference to her
symptoms.

https://mypastest.pastest.com/Secure/TestMe/Browser/436619[‫ ص‬04:37:19 10/12/1437]


MyPastest

Your answer was incorrect

Select one...

C - Prescribe high dose inhaled steroids via a spacer +/- mask

This child appears to have asthma and needs to be treated with inhaled steroids via a spacer.

Scenario 3 Next Question


A 23-month-old boy presents to you with mild breathlessness, fever and cough for just under 24 hours. His parents
report that he is a very active boy who is always running around
and is 'into everything'. On examination he has
decreased air entry on the right with occasional wheeze.

Your answer was incorrect


End Session
Select one...

H - Oral amoxicillin for a 7 day course


Previous Question
This child has a right sided pneumonia and needs oral antibiotics for 7 days.

Explanation
 
14892

Tag Question

Feedback

Difficulty: Average

Session Progress

Responses Correct: 0

Responses Incorrect: 16

Responses Total: 16

Responses - % Correct: 0%

https://mypastest.pastest.com/Secure/TestMe/Browser/436619[‫ ص‬04:37:19 10/12/1437]


MyPastest

Blog
About Pastest
Contact Us
Help

© Pastest 2016

https://mypastest.pastest.com/Secure/TestMe/Browser/436619[‫ ص‬04:37:19 10/12/1437]


MyPastest

Prefer to use the old MyPastest? Access it here »

Back to Filters

Question 13 of 53

Theme: Respiratory conditions

A a-1 anti-trypsin
B Atopic asthma
C Cystic fibrosis
D Foreign body inhalation
E Kartagener syndrome
F Mycoplasma infection
G Pertussis
H Recurrent aspiration
I Vascular ring
J Tuberculosis

For
each of the following cases please choose the most likely diagnosis from the above list. Each item may be used
once, more than once or not at all.

Scenario 1

A 2-year-old is referred with difficulty in breathing. She is noted to be wheezy – more on the right than left. A
chest X-ray reveals right-sided hyperexpansion.

Your answer was incorrect

Select one...

D - Foreign body inhalation

In a toddler who presents with difficulty in breathing, foreign body aspiration must always be considered.
Toddlers are inquisitive and will put most small objects in their mouths. Any asymmetry in expansion or wheeze
should increase the index of suspicion.

Scenario 2

An 18-month-old infant of a travelling family presents with an acute respiratory illness. Investigations at the time
show a marked lymphocytosis. He has a persistent cough 2 months later.

https://mypastest.pastest.com/Secure/TestMe/Browser/436619[‫ ص‬04:37:41 10/12/1437]


MyPastest

Your answer was incorrect

Select one...

G - Pertussis

An immunisation history must always be


asked for (and the personal child health record (‘red book’) should be
used). A cough that occurs in paroxysms, subconjunctival haemorrhages from repeated coughing and a marked
Next Question
lymphocytosis are all classic features of whooping cough. Treatment is supportive but erythromycin can be given
to limit infectivity to other children. The cough can last up to 3 months (the 100 days cough).

Scenario 3

A 3-year-old child has a recurrent nocturnal cough. He had mild eczema as an infant. He is otherwise well.

Your answer was incorrect End Session

Select one...

B - Atopic asthma
Previous Question
A previous history of an atopic condition or a family history of atopy increases the risk of asthma (and
other atopic
conditions). A nocturnal cough is a recognised presentation of asthma in pre-school children. A trial of an inhaled
bronchodilator via a spacer is worthwhile.
22186

Tag Question

Feedback

Difficulty: Average

Session Progress

Responses Correct: 0

Responses Incorrect: 19

Responses Total: 19

Responses - % Correct: 0%

https://mypastest.pastest.com/Secure/TestMe/Browser/436619[‫ ص‬04:37:41 10/12/1437]


MyPastest

Blog
About Pastest
Contact Us
Help

© Pastest 2016

https://mypastest.pastest.com/Secure/TestMe/Browser/436619[‫ ص‬04:37:41 10/12/1437]


MyPastest

Prefer to use the old MyPastest? Access it here »

Back to Filters

Question 14 of 53
Next Question

In a child whose asthma is not controlled on a regular inhaled steroid and occasional ß2-agonist, the single next
End Session
best action would be to:

A Add long-acting ß2-agonist

B
Previous Question
Increase the dose of inhaled steroid

C Check the inhaler technique

D Add a leukotriene inhibitor

E Add a short course of oral steroids

Explanation
The most important step to take is to check inhaler technique before going to the next stage of the British Thoracic
Society (BTS) guidelines. If the current medication is not being effectively given, there is no point in increasing
the medication.
22191

Tag Question

Feedback

Difficulty: Average

https://mypastest.pastest.com/Secure/TestMe/Browser/436619[‫ ص‬04:38:02 10/12/1437]


MyPastest

Peer Responses

Session Progress

Responses Correct: 0

Responses Incorrect: 20

Responses Total: 20

Responses - % Correct: 0%

Blog
About Pastest
Contact Us
Help

© Pastest 2016

https://mypastest.pastest.com/Secure/TestMe/Browser/436619[‫ ص‬04:38:02 10/12/1437]


MyPastest

Prefer to use the old MyPastest? Access it here »

Back to Filters

Question 15 of 53
Next Question

A 4-month-old infant presents with a fever, cough and


reduced feeds. Her respiratory rate is 60/min with mild
End Session
recession, wheeze and crackles throughout. The most likely diagnosis is:

A Croup

B
Previous Question
Virally induced atopic wheeze

C Cystic fibrosis

D Bacterial chest infection

E Bronchiolitis

Explanation
These are the clinical features of bronchiolitis – most frequently caused by RSV, but also adenovirus and influenza
virus. These features do not really fit with croup. Virally induced atopic wheeze is possible.
22194

Tag Question

Feedback

Difficulty: Easy

Peer Responses

https://mypastest.pastest.com/Secure/TestMe/Browser/436619[‫ ص‬04:38:22 10/12/1437]


MyPastest

Session Progress

Responses Correct: 0

Responses Incorrect: 21

Responses Total: 21

Responses - % Correct: 0%

Blog
About Pastest
Contact Us
Help

© Pastest 2016

https://mypastest.pastest.com/Secure/TestMe/Browser/436619[‫ ص‬04:38:22 10/12/1437]


MyPastest

Prefer to use the old MyPastest? Access it here »

Back to Filters

Question 16 of 53
Next Question

You diagnose a 7-year-old girl as having a generalised chest infection. Her respiratory rate is 22/min, SaO2 98% in
End Session
air; there is no recession. What would be the most appropriate course of action?

A Admit for intravenous antibiotics

B
Previous Question
Allow home but arrange for intravenous antibiotics to be given by the home care nurses

C A 7-day course of oral amoxicillin

D A 7-day course of oral erythromycin

E Admit for oral antibiotics

Explanation
In a child with no respiratory distress there is
no need to admit unless oral antibiotics cannot be tolerated. The BTS
guidelines for community-acquired pneumonia in children suggest the use of a macrolide antibiotic if Mycoplasma
sp. is a possible causative organism. If, however, there are features that would suggest pneumococcal chest
infection (focal consolidation/signs), then amoxicillin is the suggested first-line treatment.
22346

Tag Question

Feedback

Difficulty: Difficult

https://mypastest.pastest.com/Secure/TestMe/Browser/436619[‫ ص‬04:38:41 10/12/1437]


MyPastest

Peer Responses

Session Progress

Responses Correct: 0

Responses Incorrect: 22

Responses Total: 22

Responses - % Correct: 0%

Blog
About Pastest
Contact Us
Help

© Pastest 2016

https://mypastest.pastest.com/Secure/TestMe/Browser/436619[‫ ص‬04:38:41 10/12/1437]


MyPastest

Prefer to use the old MyPastest? Access it here »

Back to Filters

Question 17 of 53
Next Question

A 5-year-old boy has a history of recurrent nocturnal


cough and frequent difficulty in breathing on exercise. He
End Session
has recently
been admitted with moderately severe wheeze that responded to nebulised
bronchodilator therapy and
a dose of oral prednisolone. What do you prescribe on discharge?

A A home nebulliser with salbutamol as needed


Previous Question
B Becotide 200 µg twice daily via a metered dose inhaler (MDI)

C Flixotide 100 µg twice daily via a low-volume spacer

D Budesonide 200 µg twice daily via a dry powder device

E Montelukast 5 mg at night

Explanation
The most practical and reliable device to deliver
inhaled medication to a 5-year-old is a spacing device, which can
also be used for preventative measures and is particularly effective as a relief medication. Nebulisers are relatively
impractical, less portable,
take longer to deliver each dose and expensive. Most 5-year-olds are usually unable to
use a dry powder device very reliably and even less so
MDIs. Oral leukotriene antagonists are not first-line
therapy in asthma
prevention.
22365

Tag Question

Feedback

https://mypastest.pastest.com/Secure/TestMe/Browser/436619[‫ ص‬04:39:01 10/12/1437]


MyPastest

Difficulty: Difficult

Peer Responses

Session Progress

Responses Correct: 0

Responses Incorrect: 23

Responses Total: 23

Responses - % Correct: 0%

Blog
About Pastest
Contact Us
Help

© Pastest 2016

https://mypastest.pastest.com/Secure/TestMe/Browser/436619[‫ ص‬04:39:01 10/12/1437]


MyPastest

Prefer to use the old MyPastest? Access it here »

Back to Filters

Question 18 of 53
Next Question

A child who has a 2-day history of fever has a bright-red, infected, left tympanic membrane. The child is eating
End Session
and drinking well. What would be the most appropriate course of action?

A Referral to ENT (ear, nose, throat) surgeons as an emergency

B
Previous Question
Referral to ENT surgeon as an outpatient

C 5-day course of oral phenoxymethylpenicillin

D Advice to parents that this is a self-limiting condition and is best left alone

E Regular antipyretics and analgesics with review in 48h

Explanation
Antibiotics are of limited benefit in the treatment of otitis media. They can shorten the duration of the illness but
the number needed to treat is 17. Of the cases of otitis media, 80% resolve without treatment. Supportive measures
such as antipyretics and analgesics are important measures to recommend to parents. (Sharon Sanders, Paul P.
Glasziou, Chris Del Mar, and Maroeska Rovers. (2009) "Antibiotics for acute otitis media in children (Review)"
Cochrane database of systematic reviews, (2), 1-43)
22369

Tag Question

Feedback

https://mypastest.pastest.com/Secure/TestMe/Browser/436619[‫ ص‬04:39:25 10/12/1437]


MyPastest

Difficulty: Average

Peer Responses

Session Progress

Responses Correct: 0

Responses Incorrect: 24

Responses Total: 24

Responses - % Correct: 0%

Blog
About Pastest
Contact Us
Help

© Pastest 2016

https://mypastest.pastest.com/Secure/TestMe/Browser/436619[‫ ص‬04:39:25 10/12/1437]


MyPastest

Prefer to use the old MyPastest? Access it here »

Back to Filters

Question 19 of 53
Next Question

A parent comes to you for advice after her child’s grommet insertion. What is the most important thing to tell her?

A Swimming in a chlorinated pool is permitted

B Swimming in the sea should be avoided


Previous Question

C There may be short-term hearing loss

D Aeroplane flights should be avoided

E Plugging the ears for baths and showers is advisable

Explanation
Soaps and shampoos are irritant to the middle ear, and the soap in the water theoretically allows the liquid to pass
more easily through the hole in the grommet itself. As daily washing is the most common of these activites, it
follows that this is the most important advice. Swimming in swimming pools is permitted although diving and
underwater swimming should be discouraged. Sea water is more likely to cause infection than chlorinated pool
water, but swimming may be allowed if the ears are appropriately plugged. There should not be any hearing loss
with grommets, and grommets may ease any pressure equalisation problems on aeroplane flights. There is little
evidence base to this and much variation in advice from ENT surgeons re management, but the above is based on
the Prodigy clinical knowledge summaries.
22372

Tag Question

Feedback

https://mypastest.pastest.com/Secure/TestMe/Browser/436619[‫ ص‬04:39:55 10/12/1437]


MyPastest

Difficulty: Average

Peer Responses

End Session

Session Progress

Responses Correct: 0

Responses Incorrect: 25

Responses Total: 25

Responses - % Correct: 0%

Blog
About Pastest
Contact Us
Help

© Pastest 2016

https://mypastest.pastest.com/Secure/TestMe/Browser/436619[‫ ص‬04:39:55 10/12/1437]


MyPastest

Prefer to use the old MyPastest? Access it here »

Back to Filters

Question 20 of 53
Next Question

A 4-year-old child presents with a coryzal symptoms, cough, fever and abdominal pain. There is no dysuria and
End Session
urine dipstick is unremarkable. On examination the pain is in the left upper quadrant but is not severe. Which is
the most likely cause?

A UTI
Previous Question
B Appendicitis

C Pyelonephritis

D Left basal chest infection

E Mesenteric adenitis

Explanation
The history and site of the pain are unlikely for
appendicitis, mesenteric adenitis and pyelonephritis. A UTI is
unlikely
with no symptoms and a negative dipstick. A basal pneumonia is an important differential diagnosis in a
febrile child who presents with abdominal pain.
22384

Tag Question

Feedback

Difficulty: Easy

https://mypastest.pastest.com/Secure/TestMe/Browser/436619[‫ ص‬04:41:10 10/12/1437]


MyPastest

Peer Responses

Session Progress

Responses Correct: 0

Responses Incorrect: 26

Responses Total: 26

Responses - % Correct: 0%

Blog
About Pastest
Contact Us
Help

© Pastest 2016

https://mypastest.pastest.com/Secure/TestMe/Browser/436619[‫ ص‬04:41:10 10/12/1437]


MyPastest

Prefer to use the old MyPastest? Access it here »

Back to Filters

Question 21 of 53

A 7-year-old child presents complaining of itchy eyes and nasal congestion. Her mother reports the symptoms
tend to come and go. They are generally worse during the summer months. Examination reveals
mild conjunctival
injection but nil else of note.

What is the BEST option for management?

A Cetirizine

B Chlorpeniramine

C Intranasal antihistamine

D Intranasal corticosteroid

E Intranasal decongestant

Explanation
Cetirizine - Cetirizine is a non-sedating antihistamine and should be considered first line management of allergic
rhinitis in this case. NICE guidelines state an oral antihistamine should be prescribed first line for the management
of allergic rhinitis in the following situations:

Conjunctivitis is also present


Children aged 2 to 5 years of age
Patient preference for oral treatment

Chlorpeniramine - Oral antihistamines are indeed first line treatment


in this case however Chlorpheniramine
(Piriton) is a sedating antihistamine. This is likely to impair educational performance. A non-sedating
antihistamine should be used.

https://mypastest.pastest.com/Secure/TestMe/Browser/436619[‫ ص‬04:41:40 10/12/1437]


MyPastest

Intranasal antihistamine – Intransal antihistamines are recommended first line in most cases with the exception of
the following situations; the presence of symptomatic conjunctivitis, children aged 2 to 5 years of age, patient
preference for oral treatment. In this case the child is complaining of itchy eyes suggesting allergic conjunctivitis.
In most other cases intranasal antihistamines (azelastine) should be prescribed first line.

Intranasal corticosteroid – Intranasal corticosteroids are only recommended first line in cases of persistent
symptoms of allergic rhinitis where nasal blockage is the primary complaint, or nasal polyps are present.

Intranasal decongestant – Although nasal decongestants are useful in the management of allergic rhinitis oral
antihistamines would be first line in this case (see NICE guidelines below).

http://cks.nice.org.uk/allergic-rhinitis
Next Question
46710

End Session
Tag Question

Feedback

Previous Question
Difficulty: Average

Peer Responses

Session Progress

Responses Correct: 0

Responses Incorrect: 27

Responses Total: 27

Responses - % Correct: 0%

Blog
About Pastest
Contact Us
Help

© Pastest 2016

https://mypastest.pastest.com/Secure/TestMe/Browser/436619[‫ ص‬04:41:40 10/12/1437]


MyPastest

https://mypastest.pastest.com/Secure/TestMe/Browser/436619[‫ ص‬04:41:40 10/12/1437]


MyPastest

Prefer to use the old MyPastest? Access it here »

Back to Filters

Question 22 of 53

A 7-year-old child presents complaining of nasal congestion and discharge. Her mother reports the symptoms tend
to be worse during the summer months. The child denies any discomfort or itchiness of her eyes and on
examination there is no evidence of conjunctival injection.

What is the BEST option for management?

A Cetirizine

B Chlorpeniramine

C Intranasal antihistamine

D Intranasal corticosteroid

E Intranasal decongestant

Explanation
Intranasal antihistamine – Intransal antihistamines are recommended first line in most cases with the exception of
the following situations; the presence of symptomatic conjunctivitis, children aged 2 to 5 years of age, patient
preference for oral treatment.

Cetirizine - Oral antihistamines are first line treatment in children 2-5 years of age or if conjunctivitis is also
present (which is not the case in this question), otherwise for intermittent allergic rhinitis intranasal antihistamines
are first line.
When oral antihistamines are used a non-sedating antihistamine, such as
cetirizine, is a better choice
than a sedating antihistamine such as chlorpeniramine (Piriton).

Chlorpeniramine - Oral antihistamines are first line treatment in children 2-5 years of age or if conjunctivitis is
also present (which is not the case in this question),
otherwise for intermittent allergic rhinitis intranasal
antihistamines are first line. Chlorpheniramine (Piriton) is a sedating antihistamine, likely to impact on educational
performance, when oral antihistamines are used a non-sedating antihistamine is a better choice.

https://mypastest.pastest.com/Secure/TestMe/Browser/436619[‫ ص‬04:42:29 10/12/1437]


MyPastest

Intranasal
corticosteroid – Intranasal corticosteroids are only recommended first line in cases of persistent
symptoms of allergic rhinitis where nasal blockage is the primary complaint, or nasal polyps are present.

Intranasal
decongestant – Although nasal decongestants are sometime in the management of allergic rhinitis
intranasal antihistamines would be first
line in this case (see NICE guidelines below).

http://cks.nice.org.uk/allergic-rhinitis
46711

Next Question

Tag Question

Feedback
End Session
Difficulty: Average

Peer Responses

Previous Question

Session Progress

Responses Correct: 0

Responses Incorrect: 28

Responses Total: 28

Responses - % Correct: 0%

Blog
About Pastest
Contact Us
Help

© Pastest 2016

https://mypastest.pastest.com/Secure/TestMe/Browser/436619[‫ ص‬04:42:29 10/12/1437]


MyPastest

Prefer to use the old MyPastest? Access it here »

Back to Filters

Question 23 of 53

A 9-year-old child with a history of infantile atomic dermatitis presents complaining of periods of severely itchy
eyes, sneezing and a blocked nose. He has noticed this typically occurs on weekends which he spends at his
father’s house. On direct questioning he informs you that his father’s house is not as clean
as his mother’s and that
his father has a number of pets including two dogs and a cat.

What is the BEST first line management plan?

A Advise the use of house dust mite impermeable covers for pillows and mattresses at his father’s home

B Arrange skin prick testing

C Request blood tests for Total and specific IgE levels

D Start inhaled corticosteroids

E Start oral antihistamine

Explanation
Arrange skin prick testing – Allergy testing is indicated when the aetiology is unclear and particularly before
arduous allergen avoidance strategies, such as house dust mite avoidance measures, or removal of a pet from the
family home are advised. Skin prick testing is the allergy testing of choice. Avoidance of confirmed animal
allergens is considered important to control symptoms of allergic rhinitis and reduce the risk of asthma
developing. Stem B is correct.

Advise the use of house dust mite impermeable covers for pillows and mattresses at his father’s home - House
dust mite avoidance measures are arduous, expensive and lack a solid evidence base. House dust mite allergy
should be confirmed before avoidance measures are recommended so stem A is incorrect.

Request
blood tests for Total IgE levels - Allergy testing is indicated when the aetiology is unclear and particularly
before arduous allergen avoidance strategies, such as house dust mite avoidance measures, or removal of a pet
from the family home are advised. Skin prick testing is
the allergy testing of choice. If skin prick testing is not

https://mypastest.pastest.com/Secure/TestMe/Browser/436619[‫ ص‬04:42:53 10/12/1437]


MyPastest

available total and specific IgE levels, RAST (radioallergosorbent test) or ELISA (enzyme-linked immunosorbent
assay) may be useful. So answer stem C is not the best answer.

Start inhaled corticosteroids – Inhaled


corticosteroids are useful however allergen avoidance, if possible, is
fundamental in the management of allergic rhinitis so testing for allegens is the better answer.

Start oral antihistamine – Oral antihistamines are useful however allergen avoidance is fundamental
in the
management of allergic rhinitis. so again, stem B is the best answer.
46712

Next Question

Tag Question

Feedback End Session

Difficulty: Average

Peer Responses
Previous Question

Session Progress

Responses Correct: 0

Responses Incorrect: 29

Responses Total: 29

Responses - % Correct: 0%

Blog
About Pastest
Contact Us
Help

© Pastest 2016

https://mypastest.pastest.com/Secure/TestMe/Browser/436619[‫ ص‬04:42:53 10/12/1437]


MyPastest

Prefer to use the old MyPastest? Access it here »

Back to Filters

Question 24 of 53

A 3-year-old child is admitted to the paediatric ward with a lower respiratory tract infection. Reviewing the history
she has been unwell for 48 hours with cough and fever on the background of longstanding nasal congestion, itchy
eyes and recurrent cough. Auscultation of her chest reveals bibasal crepitations. ENT examination
reveals mild
conjunctival injection and a nasal polyp. You plot her weight which is below the 0.4th centile.

What is the most appropriate investigation?

A CXR and Sweat test

B CXR and Total IgE

C Specific IgE and Skin prick testing

D Total IgE and Skin prick testing

E Total IgE and Specific IgE

Explanation
The most appropriate investigation is CXR and Sweat test - Nasal congestion, cough and conjunctivitis may point
to a
diagnosis of allergic rhinitis however the examination finding of nasal
polyps is rare and careful consideration
should therefore be given to other causes. On identification of nasal polyps in the context of a history of recurrent
cough, respiratory tract infection and faltering growth, Cystic Fibrosis should be excluded. Chest X-ray is useful
to look for evidence of acute and chronic lower respiratory tract infection
and also rules out dextrocardia and situs
inversus which would support a
diagnosis of Primary Ciliary Dyskinesia. Sweat test is the gold standard
investigation for the diagnosis of Cystic Fibrosis.

Remaining answer stems are incorrect in the context of the scenario but would help in the following ways:

Total IgE & Specific IgE


– Elevated total IgE levels support a diagnosis of allergic disease but do not identify a
particular allergen. Specific IgE testing
(such as RAST ® or ImmunoCAP®) can identify specific allergens which
may be useful particularly when symptoms are perennial and therefore are often as a result of indoor allergens.

https://mypastest.pastest.com/Secure/TestMe/Browser/436619[‫ ص‬04:43:16 10/12/1437]


MyPastest

Total IgE & Skin prick testing


– Elevated total IgE levels support a diagnosis of allergic disease as before but
further tests are required to confirm what the specific allergen is (such as RAST ®, ImmunoCAP® or skin prick
testing.). Skin prick testing is performed by pricking the skin on the volar aspect of forearm with potential
allergens along with a negative (saline) and positive (histamine) control. The result is read at twenty
minutes after
administration of the allergens. The size of wheal which
develops relates to likelihood of clinical allergy.

Specific IgE & Skin prick testing


- Specific IgE testing (such as RAST ®or ImmunoCAP®) can identify specific
allergens which may be useful particularly when symptoms are perennial and therefore are often as a result of
indoor allergens. Skin prick testing will also identify specific allergens all
as above.
Next Question
CXR & Total IgE - Chest x-ray is useful to look for evidence of acute and chronic lower respiratory tract infection
and also rules out dextrocardia and situs inversus which would
support a diagnosis of Primary Ciliary Dyskinesia.
Elevated total IgE levels support a diagnosis of allergic disease but further tests are required to confirm what the
specific allergen is (such as allergen specific IgE, skin prick testing etc.) all as above.
47183

End Session

Tag Question
Previous Question
Feedback

Difficulty: Average

Peer Responses

Session Progress

Responses Correct: 0

Responses Incorrect: 30

Responses Total: 30

Responses - % Correct: 0%

https://mypastest.pastest.com/Secure/TestMe/Browser/436619[‫ ص‬04:43:16 10/12/1437]


MyPastest

Blog
About Pastest
Contact Us
Help

© Pastest 2016

https://mypastest.pastest.com/Secure/TestMe/Browser/436619[‫ ص‬04:43:16 10/12/1437]


MyPastest

Prefer to use the old MyPastest? Access it here »

Back to Filters

Question 25 of 53

A 14-year-old girl with cystic fibrosis is noted to have lost


4 Kg since her last clinic appointment 6 weeks ago. She
has not had any respiratory tract infections in this time. She denies any changes to her diet or bowel habit but does
admit to feeling more lethargic over
the past few months. On examination she appears underweight, her chest
is
clear and abdomen soft with no palpable masses.

What is most likely to explain this?

A Anorexia nervosa

B Diabetes

C Inflammatory bowel disease

D Intentional weight loss

E Liver cirrhosis

Explanation
The most likely diagnosis is diabetes. This 14-year-old has developed CF related diabetes which is a distinct entity
form type 1 and type 2 diabetes mellitus. Inspissated secretions lead to damage and scarring of the pancreas which
primarily results in deficient insulin production although insulin resistance also has a role
in the pathogenesis. A
history of acute weight loss should raise the possibility of diabetes in any child but a high degree of suspicion is
required for children with CF. Other symptoms may include polydipsia, polyuria, lethargy and unexplained
deterioration of lung function. All patients with CF are routinely screened for diabetes when followed up in
clinic.
An oral glucose tolerance test (OGTT ) is the gold standard investigation and should be performed on an annual
basis.

Weight
loss may be intentional or occur as a result of medical or psychiatric conditions. Eating disorders should
be considered in all underweight adolescents, particularly females, however in a child known to have cystic
fibrosis medical complications of the condition (such as CF related diabetes) should be ruled out first. A diagnosis

https://mypastest.pastest.com/Secure/TestMe/Browser/436619[‫ ص‬04:43:59 10/12/1437]


MyPastest

of anorexia nervosa requires the following; BMI ≤17.5, self-induced weight loss, body image distortion (dread of
being fat, excessively low target weight) and endocrine disorder (e.g. amenorrhoea).

Inflammatory bowel disease commonly presents in adolescents and may result in significant weight loss however
in this case there is no history of change in bowel habit such as diarrhoea and blood/mucous per rectum, so
C is
less likely.

Stem D is unlikely. The majority of patients


who have lost weight intentionally will be forthcoming with the fact
that they have purposefully done so. A careful history of diet and exercise, particularly in adolescent girls, along
with a collateral history is necessary to explore the possibility of an eating disorder.
Next Question
Liver
cirrhosis may occur in cystic fibrosis as inspissated bile leads to obstruction of the intrahepatic biliary ducts
resulting in fibrosis. The condition most commonly presents with hepatomegaly and there is no suggestion of such
a finding in this scenario. Ursodeoxycholic acid is the mainstay of medical treatment.
47184

End Session

Tag Question

Previous Question
Feedback

Difficulty: Average

Peer Responses

Session Progress

Responses Correct: 0

Responses Incorrect: 31

Responses Total: 31

Responses - % Correct: 0%

https://mypastest.pastest.com/Secure/TestMe/Browser/436619[‫ ص‬04:43:59 10/12/1437]


MyPastest

Blog
About Pastest
Contact Us
Help

© Pastest 2016

https://mypastest.pastest.com/Secure/TestMe/Browser/436619[‫ ص‬04:43:59 10/12/1437]


MyPastest

Prefer to use the old MyPastest? Access it here »

Back to Filters

Question 26 of 53

A 13-month-old is referred with concerns of poor weight gain.


She has fallen from the 25th centile to below the
second. Her parents
report she is often ‘chesty’ and has had two previous admissions for chest infections. She has
6-8 bowel motions per day. On
examination she is clearly small for her age, auscultation reveals bilateral coarse
crepitations. Cardiovascular examination is unremarkable. Abdominal examination reveals a palpable liver edge
but nil else of note.

Which of the following is the most likely diagnosis?

A Cystic Fibrosis

B Glycogen storage disease

C Immunodeficiency

D Inflammatory bowel disease

E Primary Ciliary Dyskinesia

Explanation
This child is most likely to have cystic fibrosis
which is an autosomal dominantly inherited disorder that affects
multiple systems. Classical presentation is with recurrent respiratory tract infections and gastrointestinal
disturbance associated with pancreatic insufficiency. Mutations of the Cystic Fibrosis Transmembrane Regulator
(CFTR) gene lead to impaired chloride secretion which results in movement of sodium and water into cells
leaving viscous
secretions behind. In the lungs this predisposes to recurrent infection and subsequent
bronchiectasis. Pancreatic insufficiency leads
to poor fat absorption and therefore frequent fatty stools. CF should
be ruled out early in children presenting with respiratory, gastrointestinal or growth concerns.

Glycogen storage disease


is caused by an autosomal recessive inherited enzyme defect which results in an
inability to break down glycogen into glucose. The classical presentations are with hypoglycaemia at birth and
hepatomegaly
on examination. Although this child has a palpable liver edge the diagnosis of a glycogen storage

https://mypastest.pastest.com/Secure/TestMe/Browser/436619[‫ ص‬04:44:26 10/12/1437]


MyPastest

disorder would not explain the respiratory and gastrointestinal signs and symptoms.

Primary immunodeficiency should be considered in cases of recurrent infection and faltering growth. The
umbrella term covers a wide range of conditions arising due to defects of lymphocytes, neutrophils and
complement. Cystic Fibrosis is by far more common however and should be
ruled out early in children presenting
with respiratory, gastrointestinal or growth concerns.

Inflammatory bowel disease classically presents with diarrhoea, blood/mucous per rectum, abdominal pain and
faltering growth. It is rare in a child as young as this and does not explain the respiratory symptoms and signs.

Primary Ciliary Dyskinesia is an autosomal recessive disorder resulting in abnormal ciliary function. Impaired
Next Question
clearance of mucous then results in
recurrent respiratory infections progressing to bronchiectasis. The diagnosis
should be amongst the differential for recurrent respiratory tract infections and faltering growth but does not fit the
gastrointestinal symptoms.
47185

End Session

Tag Question

Previous Question
Feedback

Difficulty: Average

Peer Responses

Session Progress

Responses Correct: 0

Responses Incorrect: 32

Responses Total: 32

Responses - % Correct: 0%

https://mypastest.pastest.com/Secure/TestMe/Browser/436619[‫ ص‬04:44:26 10/12/1437]


MyPastest

Blog
About Pastest
Contact Us
Help

© Pastest 2016

https://mypastest.pastest.com/Secure/TestMe/Browser/436619[‫ ص‬04:44:26 10/12/1437]


MyPastest

Prefer to use the old MyPastest? Access it here »

Back to Filters

Question 27 of 53

A 15-month-old child with faltering growth and recurrent admissions with lower respiratory tract infections is
suspected of having cystic fibrosis. A sweat test is performed.

Which of the following results would support the diagnosis of CF?

A Chloride 18 mmol/L

B Chloride 22 mmol/L

C Chloride 70 mmol/L

D Na 22 mmol/L

E Na 70 mmol/L

Explanation
The sweat test is the gold standard diagnostic test for cystic fibrosis. The CF phenotype occurs due to mutations of
the Cystic Fibrosis Transmembrane Regulator (CFTR) gene which leads to impaired function of a chloride
channel. In the epithelium of the respiratory tract, gastrointestinal tract, biliary tree and pancreas the
functioning
chloride channel acts to move chloride out of the cell and into the lumen. This function is impaired in CF, chloride
remains intracellular which results in movement of sodium and water into cells leaving viscous secretions within
the lumen. The function of the chloride channel in the skin is the opposite, it acts to move chloride in to the
epithelium as opposed to out. In CF abnormal function means that high levels of chloride are found on the skin
forming the basis of the sweat test. Sweat is collected on filter paper following the administration of pilocarpine
and the help of a small electric current. It is not usually performed in the first 2 weeks of life. A positive sweat
test, and therefore a likely diagnosis of CF, is defined by a chloride level of ≥ 60 mmol/L.

Stems A and B Chloride 18/22 mmol/L – As mentioned above, a positive sweat test, and therefore a
likely
diagnosis of CF, is defined by a chloride level of ≥ 60 mmol/L. CF is very unlikely in children more than six
months of age with a chloride level <39 mmol/L.

https://mypastest.pastest.com/Secure/TestMe/Browser/436619[‫ ص‬04:45:10 10/12/1437]


MyPastest

Stems D and E are incorrect because measurement of chloride forms the basis of the sweat test, not sodium.
47186

Tag Question

Feedback
Next Question
Difficulty: Average

Peer Responses

End Session

Previous Question

Session Progress

Responses Correct: 0

Responses Incorrect: 33

Responses Total: 33

Responses - % Correct: 0%

Blog
About Pastest
Contact Us
Help

© Pastest 2016

https://mypastest.pastest.com/Secure/TestMe/Browser/436619[‫ ص‬04:45:10 10/12/1437]


MyPastest

Prefer to use the old MyPastest? Access it here »

Back to Filters

Question 28 of 53

A 14-year-old boy attends the paediatric outpatient clinic for review. He reports he has been having frequent
bowel motions but is
otherwise well. His routine medications include flucloxacillin, colomycin, vitamins and
creon. On examination he appears small for his age but is comfortable at rest with no signs of respiratory distress.
Respiratory examination reveals Harrison’s Sulcus and bilateral crepitations which clear on coughing. He is not
cyanosed and has no evidence of finger clubbing. His abdomen is soft and non-tender.

Which of the following organisms is UNLIKELY to be associated with his underlying condition?

A Burkholderia Cepacia

B Haemophilus Influenzae

C Pseudomonas Aeruginosa

D Salmonella typhi

E Staphylococcus Aureus

Explanation
This 14-year-old has cystic fibrosis. Salmonella
typhi is responsible for typhoid fever. Studies have shown that
proteins coded by the CFTR gene, which is abnormal in cystic fibrosis, are used by salmonella to enter the
epithelium from the gut. This suggests those with CFTR mutations, including heterozygotes (carriers of
CF), may
have resistance to infection with typhoid which provides a theory to explain why mutations of CFTR remain so
prevalent when in the recent past, patients with cystic fibrosis typically died before reproductive age.

Burkholderia Cepacia - The Burkholderia Cepacia family of bacteria are particularly problematic for
children with
cystic fibrosis. Infections can lead to a rapid decline in pulmonary function and treatment can be difficult.

Haemophilus Influenzae frequently colonises patients with cystic fibrosis in infancy or early childhood.

Pseudomonas
aeruginosa is an extremely common organism responsible for chronic infection. Multiple drug

https://mypastest.pastest.com/Secure/TestMe/Browser/436619[‫ ص‬04:45:38 10/12/1437]


MyPastest

resistance is becoming problematic.

Staphylococcus Aureus infection is common. Most children with CF will be on oral flucloxacillin prophylactically.
Methicillin Resistant Staphylococcus Aureus (MRSA) is becoming an increasing problem.
47187

Next Question Tag Question

Feedback

Difficulty: Average
End Session
Peer Responses

Previous Question

Session Progress

Responses Correct: 0

Responses Incorrect: 34

Responses Total: 34

Responses - % Correct: 0%

Blog
About Pastest
Contact Us
Help

© Pastest 2016

https://mypastest.pastest.com/Secure/TestMe/Browser/436619[‫ ص‬04:45:38 10/12/1437]


MyPastest

Prefer to use the old MyPastest? Access it here »

Back to Filters

Question 29 of 53

A mother attends the emergency department with both her 3 –year-old and 18-month-old children. She has noticed
a discharge from the 18-month-olds left ear. The child has no significant past medical history and has been well
today. Earlier in the week the children were playing together, briefly unsupervised, the mother recalls
how the 18-
month-old did cry out loudly but by the time she had left the kitchen and reached the children they were
continuing to play. On examination the child is systemically well. Examination of the left ear
reveals a foul
smelling copious discharge. The child is reluctant to let you near the ear.

What is the most likely diagnosis?

A Cholesteotoma

B Foreign body

C Mastoiditis

D Skull fracture

E Viral upper respiratory tract infection

Explanation
It is relatively common for toddlers, or their younger siblings, to present to the paediatric emergency department
with
a small object lodged in the external ear canal or nostril. Offensive discharge may develop if the incident is
unwitnessed and the object remains within the canal. Removal of the object should be done with caution to avoid
pushing it further into the ear canal. Refer to ENT.

Cholesteotoma is an abnormal collection of squamous epithelial cells within the middle ear and mastoid process.
The expanding growth is erosive to the local structures. The classical presenting symptoms are discharge and
hearing impairment. Prompt identification and treatment of cholesteotoma is essential as left untreated
complications include hearing loss, facial nerve injury and potential for intracranial infection. A ‘polyp’ on
otoscopy is a cholesteotoma until proven otherwise. Refer to an ENT surgeon for assessment. The history of ear

https://mypastest.pastest.com/Secure/TestMe/Browser/436619[‫ ص‬04:46:08 10/12/1437]


MyPastest

discharge in this case is short and on the background of an acute


episode of pain in an unsupervised toddler,
foreign body is the most likely diagnosis.

Mastoiditis is infection of the mastoid air cells. This classically presents with an inflamed tender mastoid process
which results in a protruding ear. Intravenous antibiotics are required.

A skull fracture may be obtained from accidental or non-accidental injury. When head injury is suspected always
examine the
ears carefully to exclude haemotympanum and CSF ottorhoea that would suggest a base of skull
fracture. In this case the child is otherwise well and the discharge has an offensive odour pointing towards a
foreign
body.
Next Question
Viral upper respiratory tract infection would be associated with symptoms such as coryza, cough, sore throat, poor
oral intake and fever. The throat and tympanic membranes may be erythematous
on inspection but ear discharge
would not be expected without a secondary bacterial ear infection.
47188

End Session

Tag Question

Previous Question
Feedback

Difficulty: Average

Peer Responses

Session Progress

Responses Correct: 0

Responses Incorrect: 35

Responses Total: 35

Responses - % Correct: 0%

https://mypastest.pastest.com/Secure/TestMe/Browser/436619[‫ ص‬04:46:08 10/12/1437]


MyPastest

Blog
About Pastest
Contact Us
Help

© Pastest 2016

https://mypastest.pastest.com/Secure/TestMe/Browser/436619[‫ ص‬04:46:08 10/12/1437]


MyPastest

Prefer to use the old MyPastest? Access it here »

Back to Filters

Question 30 of 53

A mother attends the emergency department with both her 3-year-old and 18-month-old children. The 18-month-
old has been vomiting and lethargic over the past 2-3 hours. The child has no significant past medical history and
was well in the morning. Earlier in the afternoon the children were playing together, briefly unsupervised as mum
was in the kitchen. The mother recalls how the 18-month-old did cry out suddenly and required comforting for ten
minutes following which he continued to play. On examination the 18-month-old is alert but miserable. ENT
examination reveals a lightly blood stained serous discharge from the left ear. Otoscopy is difficult, as the child is
combative, but the tympanic membrane appears dark.

What is the most appropriate investigation?

A CT Head

B MRI Head

C Meningococcal/Pneumococcal PCR, Blood culture, Lumbar puncture

D Nasopharyngeal swab

E Send discharge for microscopy, culture and sensitivities

Explanation
A significant head injury should be suspected in this scenario. This child has signs of a base of skull fracture with
haemotympanum on ototscopy and CSF otorrhoea. A CT scan is indicated as
per NICE guidelines. This should be
within one hour if ANY of the following features are present; Suspicion of NAI, post-traumatic seizure, GCS < 14
(<15 in infants) on initial assessment, GCS <
15 two hours post injury, suspected open or depressed skull fracture,
suspected basal skull fracture, focal neurological deficit, bruising or swelling > 5cm in an infant.

As explained above, CT is the first line investigation for significant head injury in both children and adults. MRI
can provide additional information but in view of safety, logistical reasons and limited resources CT is
recommended as first line.

https://mypastest.pastest.com/Secure/TestMe/Browser/436619[‫ ص‬04:46:32 10/12/1437]


MyPastest

Meningococcal/Pneumococcal PCR, Blood culture, Lumbar puncture - These investigations are indicated for
suspected meningitis / sepsis. There is not a history of fever in this case nor description of meningism or non-
blanching rash. The clinical findings suggest a base of skull fracture.

Nasopharyngeal swab is
useful to identify infectious organisms within the respiratory tract, particularly viruses.
One of the most common uses is to identify the causative organism in bronchiolitis. The clinical findings in this
case
however suggest a base of skull fracture, there is no history of fever to suggest infection.

Sending the fluid to the laboratory would allow confirmation that it is in fact CSF but this is unnecessary as a
suspicion of significant head injury with haemotympanum is indication for immediate imaging which will identify
Next Question
a skull fracture. MC&S would be useful if a local ear infection was suspected but there is no history of fever to
suggest infection and there are signs of significant
head injury.

NICE Guidelines:

https://www.nice.org.uk/guidance/conditions-and-diseases/injuries--accidents-and-wounds/head-injuries
47189
End Session

Previous Question Tag Question

Feedback

Difficulty: Average

Peer Responses

Session Progress

Responses Correct: 0

Responses Incorrect: 36

Responses Total: 36

Responses - % Correct: 0%

https://mypastest.pastest.com/Secure/TestMe/Browser/436619[‫ ص‬04:46:32 10/12/1437]


MyPastest

Blog
About Pastest
Contact Us
Help

© Pastest 2016

https://mypastest.pastest.com/Secure/TestMe/Browser/436619[‫ ص‬04:46:32 10/12/1437]


MyPastest

Prefer to use the old MyPastest? Access it here »

Back to Filters

Question 31 of 53

A mother attends clinic with both her 3-year-old and 18-month-old children. She has noticed a discharge from the
3-year-old’s left ear over the past two weeks. He has a history of frequent episodes of ‘ear infection’ but has been
otherwise well. There is no reported fever. On examination the child is systemically well. The child is co-operative
with otoscopy which shows an intact pink tympanic membrane on the right but a possible perforation on the left.

What is the next best step in management?

A Arrange CT Head

B Oral Amoxicillin

C Reassurance

D Refer for ENT opinion

E Topical aminoglycosides

Explanation
This child has Chronic Suppurative Otitis Media which is chronic inflammation of the middle ear. It presents with
recurrent discharge (> 2 weeks) which escapes the middle ear through a
perforated tympanic membrane. There is
absence of acute infection, i.e. no fever or pain, however a previous history of acute otitis media supports the
diagnosis. Examination is not uncomfortable for the child and identifies a perforated tympanic membrane. There
may be associated hearing impairment which typically resolves when the tympanic membrane heals.
Complications include mastoiditis, facial nerve palsy and intracranial infection. If Chronic Supporative Otitis
Media is suspected ENT referral is indicated, the ears should not be swabbed and no treatment should be given.

As this child has Chronic Supporative Otitis Media the child should be referred to an ENT surgeon.
CT head may
be indicated for complications such as mastoiditis but this should be under the care of an ENT specialist.

Amoxicillin
is used for the treatment of acute otitis media however if Chronic Supporative Otitis Media is
suspected, as in this case, the child should
be referred to an ENT surgeon. The ears should not be swabbed and no

https://mypastest.pastest.com/Secure/TestMe/Browser/436619[‫ ص‬04:47:01 10/12/1437]


MyPastest

treatment should be given.

Parents/Carers may be reassured that


any hearing impairment is likely to resolve completely as the tympanic
membrane heals however referral to an ENT surgeon is indicated as the condition can be associated with
complications such as mastoiditis, facial nerve palsy and intracranial infection.

Topical aminoglycosides are contraindicated in the presence of tympanic membrane perforation so stem E is
incorrect.

NICE guidelines: http://cks.nice.org.uk/otitis-media-chronic-suppurative


47190
Next Question

Tag Question
End Session
Feedback

Difficulty: Average

Previous Question
Peer Responses

Session Progress

Responses Correct: 0

Responses Incorrect: 37

Responses Total: 37

Responses - % Correct: 0%

Blog
About Pastest
Contact Us
Help

© Pastest 2016

https://mypastest.pastest.com/Secure/TestMe/Browser/436619[‫ ص‬04:47:01 10/12/1437]


MyPastest

https://mypastest.pastest.com/Secure/TestMe/Browser/436619[‫ ص‬04:47:01 10/12/1437]


MyPastest

Prefer to use the old MyPastest? Access it here »

Back to Filters

Question 32 of 53

A 2-year-child presents to the emergency department with a three day history of coryzal symptoms and gradual
onset of increased work of breathing. On examination the child appears very comfortable at
rest and fairly playful.
There is mild intercostals and subcostal recession along with tracheal tug. The temperature is 38.1oC and the
oxygen saturations are 98% in room air. There is an audible inspiratory
stridor.

What is the best next step in management?

A Dexamethasone 0.15mg/Kg PO

B IM Adrenaline 150 micrograms

C Perform a full ENT examination

D Request ENT and Anaesthetic support

E Send Nasopharyngeal swab

Explanation
Dexamethasone 0.15mg/Kg PO is the best answer. This child has croup which results from an acute viral illness
causing inflammation of the upper respiratory tract. It classically presents with stridor, hoarse voice, fever and
‘barking cough’ on the
background of a preceding coryzal illness. Steroids are the first line
treatment, oral
dexamethasone is currently the most commonly used. The
condition is usually mild with only five percent of
children requiring admission to hospital however intubation and ventilation is necessary for the most severely
affected.

Intramuscular adrenaline is used in the management of anaphylaxis so B is an incorrect answer. In a


child with
severe croup nebulised adrenaline may be used to buy time for steroid treatment to take effect. A 1 in 1000
solution is used at a
dose of 400 micrograms/Kg (maximum 5mg) and may be repeated after thirty minutes. The
effect of the nebulised adrenaline lasts approximately two to three hours, close observation is required for
recurrence of symptoms.

https://mypastest.pastest.com/Secure/TestMe/Browser/436619[‫ ص‬04:47:22 10/12/1437]


MyPastest

Although this child is not in extremis, examination of the throat should be avoided in any children with stridor as
this could provoke worsening of the airway obstruction so C is incorrect.

Expert anaesthetic and ENT support should be requested for severe airway obstruction which is relatively rare for
simple viral croup. In a systemically unwell / toxic child, or when drooling is significant, bacterial tracheitis and
epiglottitis should be
considered and anaesthetic / ENT assistance requested urgently. In this case the child is
comfortable with mild respiratory distress, treatment with steroids is warranted along with close observation.

Nasopharyngeal
swab is useful to identify infectious organisms within the respiratory tract, particularly viruses.
One of the most common uses is to identify
the causative organism in bronchiolitis to allow cohorting of admitted
Next Question
infants. The clinical findings in this case however suggest croup (viral laryngotracheobronchitis). The child has
stridor and therefore swabbing the throat is contraindicated, as is examination of the throat,
as this may provoke
worsening of the airway obstruction so E is incorrect.
47191

End Session

Tag Question

Feedback
Previous Question

Difficulty: Average

Peer Responses

Session Progress

Responses Correct: 0

Responses Incorrect: 38

Responses Total: 38

Responses - % Correct: 0%

Blog
About Pastest
Contact Us
Help

https://mypastest.pastest.com/Secure/TestMe/Browser/436619[‫ ص‬04:47:22 10/12/1437]


MyPastest

© Pastest 2016

https://mypastest.pastest.com/Secure/TestMe/Browser/436619[‫ ص‬04:47:22 10/12/1437]


MyPastest

Prefer to use the old MyPastest? Access it here »

Back to Filters

Question 33 of 53

You are called to urgently review a 5-year-old in the emergency department. He has been brought in by his mother
with increased work of breathing. On examination he looks unwell, and is sat
leaning forward on his mother’s lap.
The nurse informs you his temperature is 40.1°C. He has signs of respiratory distress with recessions and audible
stridor.

What is the most likely causative pathogen?

A Haemophilus Influenzae B

B Moraxella catarrhalis

C Parainfluenza

D Respiratory Syncitial Virus

E Staphylococcus Aureus

Explanation
This child has bacterial tracheitis which is most commonly caused by Staphylococcus Aureus
infection.
Presentation can be similar to croup with the presence of a
barking cough but bacterial tracheitis is associated with
a systemically unwell or ‘toxic looking’ child with high fever. Epiglottitis is associated with Haemophilus
Influenzae B
infection and is rare since widespread HIB immunisation. Epiglottitis may present similarly to
bacterial tracheitis however with epiglottitis onset is typically very acute, drooling is a prominent feature, barking
cough is absent and stridor is soft. If bacterial tracheitis or epiglottitis is suspected minimise distress to the child,
provide supplementary oxygen if possible and request expert assistance from the most senior anaesthetic and ENT
doctors available. Definitive management requires intubation and intravenous antibiotics.

Haemophilus Influenzae B as explained above is associated with epiglottitis but is unlikely to the causative
pathogen in this scenario for the reasons already explained.

Moraxella catarrhalis
is a gram negative diplococcus associated with upper respiratory tract infection such as

https://mypastest.pastest.com/Secure/TestMe/Browser/436619[‫ ص‬04:47:50 10/12/1437]


MyPastest

otitis media, sinusitis and pharyngitis. It is a cause of bacterial tracheitis however Staphylococcus Aureus is the
most commonly identified organism in most studies.

Respiratory Syncitial Virus is the commonest cause of bronchiolitis which is a lower respiratory tract infection
affecting infants. It presents with cough, coryza, poor feeding and respiratory distress. Management is supportive.

Parainfluenza is the commonest cause of viral croup which classically presents with stridor, hoarse voice, fever
and ‘barking cough’ on the background of a preceding coryzal illness. The child in this case has severe symptoms
and is systemically unwell / toxic with high fever suggesting bacterial tracheitis or epiglottitis.
47192

Next Question

Tag Question
End Session
Feedback

Difficulty: Average

Previous Question Peer Responses

Session Progress

Responses Correct: 0

Responses Incorrect: 39

Responses Total: 39

Responses - % Correct: 0%

Blog
About Pastest
Contact Us
Help

© Pastest 2016

https://mypastest.pastest.com/Secure/TestMe/Browser/436619[‫ ص‬04:47:50 10/12/1437]


MyPastest

Prefer to use the old MyPastest? Access it here »

Back to Filters

Question 34 of 53

A 19 month old baby presents to the paediatric assessment unit with a 24 hour history of fever and being generally
miserable. His mother informs you she has noticed him pulling on his right eat frequently. On examination he is
systemically well in himself with observations within normal limits. He is currently afebrile. On otoscopy you
note a bulging and erythematous tympanic membrane. There is nil else of note.

What is the next best step in management?

A Amoxicillin, 5 day course

B Erythromycin

C Penicillin V, 5 day course

D Reassure

E Refer to ENT for IV antibiotics and drainage

Explanation
Amoxicillin, 5 day course – This young child has acute otitis media (AOM). Reassurance is all that is usually
required as the condition tends to be self limiting with symptom resolution within a few days. When antibiotics are
prescribed first line treatment is with a five day course of amoxicillin. A macrolide such as erythromycin should
be used for those with penicillin allergy.

Erythromycin
– This young child has acute otitis media (AOM). Reassurance is
all that is usually required as the
condition tends to be self limiting
with symptom resolution within a few days. When antibiotics are prescribed
first line treatment is with a five day course of amoxicillin. A macrolide such as erythromycin should be used for
those with penicillin allergy.

Penicillin V, 5 day course – This young child has acute otitis media (AOM). Reassurance is all that is usually
required as the condition tends to be self limiting with symptom resolution within a few days. When antibiotics are
prescribed first line treatment is with a five day course of amoxicillin. A macrolide such as erythromycin should

https://mypastest.pastest.com/Secure/TestMe/Browser/436619[‫ ص‬04:48:13 10/12/1437]


MyPastest

be used for those with penicillin allergy.

Reassure – This young child has acute otitis media (AOM) which is characterised by the presence of inflammation
of the middle ear with an associated effusion occurring as a result of an acute infection. It is common, particularly
in children younger than ten years of age, and may be due to viral or bacterial organisms. Reassurance is all that is
usually required as the condition tends to be
self limiting with symptom resolution within a few days. Antibiotics
may be considered for those with systemic upset, symptoms which have been ongoing for four or more days,
children younger than two years of age with infection in both ears and children with tympanic membrane
perforation or discharge within the ear canal. When antibiotics are prescribed first line treatment is with a five day
course of amoxicillin.
Next Question
Refer to ENT for IV antibiotics and drainage – This young child has acute otitis media (AOM). Reassurance is all
that is usually required as the condition tends to be self limiting with symptom resolution within a few days.
Antibiotics may be used in certain circumstances. Referral to an ENT specialist would be indicated
for
complications e.g. mastoiditis.

Nice Guidelines: http://cks.nice.org.uk/otitis-media-acute End Session


47513

Previous Question
Tag Question

Feedback

Difficulty: Average

Peer Responses

Session Progress

Responses Correct: 0

Responses Incorrect: 40

Responses Total: 40

Responses - % Correct: 0%

https://mypastest.pastest.com/Secure/TestMe/Browser/436619[‫ ص‬04:48:13 10/12/1437]


MyPastest

Blog
About Pastest
Contact Us
Help

© Pastest 2016

https://mypastest.pastest.com/Secure/TestMe/Browser/436619[‫ ص‬04:48:13 10/12/1437]


MyPastest

Prefer to use the old MyPastest? Access it here »

Back to Filters

Question 35 of 53

A 7 year old boy is referred via the emergency department with a history of pyrexia and worsening headache. He
was seen by his GP
3 days ago complaining of a sore throat and ear pain and diagnosed with
acute otitis media.
On examination you note asymmetry of the prominence of the pinna. There is focal tenderness posterior to the
pinna on the right.

What is the next best step in management?

A Amoxicillin, 5 day course

B Erythromycin

C IV antibiotics& refer to ENT

D Penicillin V, 5 day course

E Reassure

Explanation
Amoxicillin, 5 day course – A five day course of amoxicillin is the first line antibiotic for acute otitis media
however this child has developed mastoiditis which required intravenous antibiotics.

Erythromycin – A macrolide, such


as erythromycin or clarithromycin, is first line treatment for acute otitis media
in those allergic to penicillin. This child has developed mastoiditis however and therefore requires intravenous
antibiotics and assessment by an ENT surgeon.

IV antibiotics& refer to ENT – This seven year old child has developed mastoiditis as a complication of acute
otitis media. Mastoiditis occurs when infection spreads to the mastoid cells (small air containing spaces) within
the mastoid bone. The diagnosis is usually easily made as the localised swelling results in protuberance of the
pinna on the affected side, the mastoid area itself becomes erythematous and tender. Investigations consist of
blood tests to check inflammatory markers, blood culture and in some circumstances a CT scan is required.
Intravenous antibiotics are the mainstay of treatment. Surgery is required in some cases. Other complications of

https://mypastest.pastest.com/Secure/TestMe/Browser/436619[‫ ص‬04:48:36 10/12/1437]


MyPastest

acute otitis media include hearing impairment, perforation of the tympanic membrane, facial nerve palsy,
intracranial abscess and venous sinus thrombosis.

Penicillin V, 5 day course – This child has developed mastoiditis as a complication of acute
otitis media.
Admission for ENT opinion and intravenous antibiotics is
warranted to prevent spread of the infection
intracranially.

Reassure
– This seven year old child has developed mastoiditis as a complication of acute otitis media. Admission
for ENT opinion and intravenous antibiotics is warranted to prevent spread of the infection intracranially.

Nice Guidelines: http://cks.nice.org.uk/otitis-media-acute


Next Question 47514

End Session
Tag Question

Feedback

Previous Question Difficulty: Average

Peer Responses

Session Progress

Responses Correct: 0

Responses Incorrect: 41

Responses Total: 41

Responses - % Correct: 0%

Blog
About Pastest
Contact Us
Help

© Pastest 2016

https://mypastest.pastest.com/Secure/TestMe/Browser/436619[‫ ص‬04:48:36 10/12/1437]


MyPastest

https://mypastest.pastest.com/Secure/TestMe/Browser/436619[‫ ص‬04:48:36 10/12/1437]


MyPastest

Prefer to use the old MyPastest? Access it here »

Back to Filters

Question 36 of 53

An 18 month old baby presents to the paediatric assessment unit with a 24 hour history of fever and being
generally miserable. His mother informs you she has noticed him pulling on his ears frequently. On examination
he is coryzal but systemically well in himself with observations within normal limits. He is currently afebrile. His
chest
is clear with no signs of respiratory distress, heart sounds are normal
with no added sounds. His abdomen is
soft non-tender. His throat is erythematous and tympanic membranes pink bilaterally.

What is the most likely diagnosis?

A Cholesteotoma

B Chronic suppurative otitis media

C Otitis externa

D Sinusitis

E Viral upper respiratory tract infection

Explanation
Cholesteotoma – Cholesteotoma is an abnormal collection of squamous epithelial cells within the middle ear and
mastoid process. The expanding growth is erosive to the local structures. The classical presenting symptoms are
discharge and hearing
impairment. Prompt identification and treatment of cholesteotoma is essential as left
untreated complications include hearing loss, facial nerve injury and potential for intracranial infection. A ‘polyp’
on otoscopy is a cholesteotoma until proven otherwise. Refer to an ENT surgeon for assessment.

Chronic suppurative otitis media – Chronic Suppurative Otitis Media which is chronic inflammation of the middle
ear. It presents with recurrent discharge (> 2 weeks) which escapes the middle ear through a perforated tympanic
membrane. There is absence of acute infection, i.e.
no fever or pain, however a previous history of acute otitis
media supports the diagnosis. Examination is not uncomfortable for the child and identifies a perforated tympanic
membrane. There may be associated hearing impairment which typically resolves when the tympanic membrane

https://mypastest.pastest.com/Secure/TestMe/Browser/436619[‫ ص‬04:48:57 10/12/1437]


MyPastest

heals.

Otitis externa – Otitis Externa is inflammation of the external auditory canal. Examination findings consist of an
inflamed, oedematous ear canal usually associated with significant pain and discharge.

Sinusitis – Sinusitis refers to inflammation of the lining of the sinuses (frontal , maxillary or ethmoidal). This may
occur as a result of viral or bacterial infection.
In older children complaints of a ‘ blocked nose’ or ‘fullness’
predominate however in younger children symptoms and signs are frequently non-specific. Parents may report that
the child is breathing through their mouth or feeding poorly. Examination may reveal focal tenderness over the
forehead or cheeks.
Next Question
Viral upper respiratory tract infection – Viral upper respiratory tract infections are extremely common in
childhood. Symptoms include coryza, cough, sore throat, poor oral intake and fever. The throat and tympanic
membranes may be erythematous on inspection. Management in most cases is supportive with analgesia /
antipyretics and reassurance.
47515
End Session

Tag Question
Previous Question
Feedback

Difficulty: Average

Peer Responses

Session Progress

Responses Correct: 0

Responses Incorrect: 42

Responses Total: 42

Responses - % Correct: 0%

https://mypastest.pastest.com/Secure/TestMe/Browser/436619[‫ ص‬04:48:57 10/12/1437]


MyPastest

Blog
About Pastest
Contact Us
Help

© Pastest 2016

https://mypastest.pastest.com/Secure/TestMe/Browser/436619[‫ ص‬04:48:57 10/12/1437]


MyPastest

Prefer to use the old MyPastest? Access it here »

Back to Filters

Question 37 of 53

A 6 year old girl is brought to the emergency department complaining of right sided earache. She describes the
pain as an ‘ache that comes and goes’ but has worsened over the past 24
hours. On examination she appears
uncomfortable at rest but otherwise systemically well. Her observations are within normal limits. On otoscopy the
ear canal appears healthy, the tympanic membrane is pink and non-bulging with the light reflex present. Inspection
of the oral cavity identifies significant dental caries with soft tissue swelling adjacent to the right upper first molar.

What is the best next step in management?

A Analgesia and admit for IV antibiotics

B Analgesia and advise to attend dentist

C Analgesia, oral antibiotics and discharge

D Analgesia, oral antibiotics plus incision & drainage of swelling

E Analgesia plus incision & drainage of swelling

Explanation
Analgesia and admit for IV antibiotics – This six year old has developed a dental abscess. This occurs when an
insult to the pulp of the tooth leads to necrosis and infection which may subsequently spread to the adjacent soft
tissue. Analgesia is of course an important part of management. Antibiotics are not routinely prescribed but should
be commenced if the child is systemically unwell. Intravenous antibiotics would be indicated if the child displayed
signs of sepsis. The child should be referred to a dentist for definitive management in the way of root canal
surgery of dental extraction.

Analgesia
and advise to attend dentist – This six year old has developed a
dental abscess. This occurs when an
insult to the pulp of the tooth leads to necrosis and infection which may subsequently spread to the adjacent soft
tissue. Dental abscesses cause significant pain in adults
but in children pain is commonly less of a feature in the
presentation.
Management in the paediatric emergency department or assessment unit is
to provide analgesia and

https://mypastest.pastest.com/Secure/TestMe/Browser/436619[‫ ص‬04:49:20 10/12/1437]


MyPastest

advise an urgent dental appointment. Commence


antibiotics if systemically unwell.

Analgesia, oral antibiotics and discharge – This six year old has developed a dental abscess. This occurs when an
insult to the pulp of the tooth leads to necrosis and infection which may subsequently spread to the adjacent soft
tissue. Analgesia is of course an important part of management. Antibiotics are not routinely prescribed but should
be commenced if the child is systemically unwell. The child should be referred to a dentist for definitive
management.

Analgesia, oral
antibiotics plus incision & drainage of swelling – This six year old has developed a dental abscess.
This occurs when an insult to the pulp of the tooth leads to necrosis and infection which may subsequently spread
Next Question
to the adjacent soft tissue. Analgesia is an important part of management for the paediatrician and antibiotics may
be used especially if there are systemic symptoms. Definitive management requires dental expertise. Incision and
drainage of the gum may be a temporary measure used by the dental practitioner however root canal surgery or
dental extraction is usually required.

Analgesia
plus incision & drainage of swelling – This six year old has developed a dental abscess. This occurs
End
when an insult to the pulp of the tooth leads to necrosis and infection which may Session spread to the
subsequently
adjacent soft tissue. Analgesia is an important part of management for the paediatrician but definitive management
requires dental expertise. Incision and drainage of the gum may be a temporary measure used by the dental
practitioner however root canal surgery or dental extraction is usually required.
47516

Previous Question

Tag Question

Feedback

Difficulty: Average

Peer Responses

Session Progress

Responses Correct: 0

Responses Incorrect: 43

Responses Total: 43

https://mypastest.pastest.com/Secure/TestMe/Browser/436619[‫ ص‬04:49:20 10/12/1437]


MyPastest

Responses - % Correct: 0%

Blog
About Pastest
Contact Us
Help

© Pastest 2016

https://mypastest.pastest.com/Secure/TestMe/Browser/436619[‫ ص‬04:49:20 10/12/1437]


MyPastest

Prefer to use the old MyPastest? Access it here »

Back to Filters

Question 38 of 53

A 13 year old girl presents out of hours complaining of right


sided earache. She is in the area for a surfing
competition and states
the ear has been itchy over the past few days but has become excruciatingly painful today.
On examination she appears uncomfortable at rest but otherwise systemically well. Her observations are within
normal limits. Otoscopy is difficult as the girl flinches in pain, the ear canal appears oedematous. The tympanic
membrane is difficult to see past the oedema and discharge present in the external canal. The oropharynx appears
normal with no erythema or tonsilar exudate.

What is the best next step in management?

A Admit for IV antibiotics

B Oral antibiotics

C Reassurance

D Refer to ENT

E Topical antibiotics

Explanation
Admit for IV antibiotics – This thirteen year old girl has developed acute otitis externa (AOE) which is
inflammation of the external ear canal. First line management is with analgesia and topical acidic, antibiotic or
combined antibiotic and corticosteroid preparations.

Oral antibiotics – This thirteen year old girl has developed acute otitis externa (AOE) which is
inflammation of the
external ear canal. First line management is with analgesia and topical preparations. Only consider the use of an
oral antibiotic (flucloxacillin or erythromycin) for severe infections such as the spread of cellulitis beyond the ear
canal. Advice from an ENT specialist may be required if considering starting oral antibiotics.

Reassurance
– This thirteen year old girl has developed acute otitis externa (AOE) which is inflammation of the
external ear canal. First line management is with analgesia and topical acidic, antibiotic or combined antibiotic and

https://mypastest.pastest.com/Secure/TestMe/Browser/436619[‫ ص‬04:50:20 10/12/1437]


MyPastest

corticosteroid preparations.

Refer to ENT –
This thirteen year old girl has developed acute otitis externa (AOE) which is inflammation of the
external ear canal. First line management is with analgesia and topical preparations. Seek ENT advice in the case
of treatment failure, if there is cellulitis extending past the margin of the external ear canal, extreme pain or
extensive swelling and discharge likely to require suction or the insertion of an ear wick. Admit urgently under
ENT if suspecting malignant otitis externa (with extension of the condition into the adjacent bone and spreading
osteomyelitis).

Topical antibiotics – This thirteen year


old girl has developed acute otitis externa (AOE) which is inflammation
of
Next Question
the external ear canal. Otitis externa may be classified as acute (< 3 weeks) or chronic (> 3 months). Frequent
exposure to water, e.g. swimming, is a risk factor for the condition. Itching, pain, hearing loss and discharge are
common complaints. Examination demonstrates oedema of the external auditory canal with discharge. Pain
may
be elicited on movement of the tragus or pinna. First line treatment is with topical drops/sprays for one week.
Acidic preparations such as 2% acetic acid, antibiotic or combined antibiotic and corticosteroid preparations may
be used (1% hydrocortisone and 0.3% gentamicin). Acetic acid may be used as the first line treatment for mild
End Session
cases without discharge or hearing impairment. On selecting an appropriate topical preparation remember that
topical aminoglycosides are contraindicated if the tympanic membrane is perforated and that chloramphenicol ear
drops cause contact dermatitis in approximately 10% of people. Clinoquinol (Locorten–Vioform®), a combination
antibacterial and antifungal, may be preferred over aminoglycoside containing preparations due to the theoretical
reduced risk of ototoxicity and dermatitis however there is no clear evidence to support
the use of one topical
preparartion overPrevious
another. Question

Nice Guidelines: http://cks.nice.org.uk/otitis-externa


47517

Tag Question

Feedback

Difficulty: Average

Peer Responses

Session Progress

https://mypastest.pastest.com/Secure/TestMe/Browser/436619[‫ ص‬04:50:20 10/12/1437]


MyPastest

Responses Correct: 0

Responses Incorrect: 44

Responses Total: 44

Responses - % Correct: 0%

Blog
About Pastest
Contact Us
Help

© Pastest 2016

https://mypastest.pastest.com/Secure/TestMe/Browser/436619[‫ ص‬04:50:20 10/12/1437]


MyPastest

Prefer to use the old MyPastest? Access it here »

Back to Filters

Question 39 of 53

You are called by the ward nursing staff to see a 9 year old boy who is having a nosebleed. He is currently on IV
antibiotics for a lower respiratory tract infection and has been recovering well. The nurse informs you that the
nosebleed began approximately 3 minutes ago. She has checked his observations whilst awaiting your arrival;
heart rate 100, respiratory rate 24, SpO2 98%, systolic blood pressure 110 mmHg, temperature 37.7°C. On
examination the child is sat up in bed
crying with ongoing bleeding from the nose.

What is the best next step in management?

A Apply pressure to the soft part of the nose for 15 minutes

B Call ENT team to review

C Give paracetamol

D IV labetalol

E Oral nifedipine

Explanation
Apply pressure to the soft part of the nose for 15 minutes – Epistaxis is common in children and adults (although it
is rare in young children, < 2 years of age). Approximately 90% are anterior bleeds arising from little’s area
(where the venous plexus forms on the septum). Basic first aid is all that is required in the management of the vast
majority. This involves pinching together the soft part of the nose, below the cartilage, for ten to fifteen minutes.
Once the bleeding stops with simple first aid consider prescribing a topical antiseptic cream such as Naseptin
(Chlorhexidine and Neomycin) to be applied to the nostrils for ten days.
If bleeding continues following simple
first aid measures nasal cautery or nasal packing may be required. When assessing the child with
epistaxis look for
haemodynamic compromise, this is rare with epistaxis
but can be life threatening.

Call ENT team to review – Epistaxis is common in children and adults (although it is rare in young
children, < 2
years of age). Basic first aid is all that is required in the management of the vast majority. Refer to ENT in cases of

https://mypastest.pastest.com/Secure/TestMe/Browser/436619[‫ ص‬04:50:45 10/12/1437]


MyPastest

suspected posterior bleed, treatment failure or recurrent epistaxis.

Give
paracetamol – This child is febrile with a lower respiratory tract infection. The antipyretic effect of
paracetamol may reduce his fever and as a consequence reduce his cardiac output however this is not
the priority
in the management of epistaxis, simple compression is.

IV
labetalol – IV labetalol may be used in the management of hypertensive crisis. Epistaxis may occur as a result
of hypertension however this is rarely the cause in children and in this case you are supplied with a blood pressure
reading.

Oral nifedipine – Oral nifedipine may be used in the management of hypertensive crisis. Epistaxis may occur as a
Next Question
result of hypertension however this is rarely the cause in children and in this case you are supplied with a blood
pressure reading.

Nice Guidelines: http://cks.nice.org.uk/epistaxis-nosebleeds


47518

End Session

Tag Question
Previous Question
Feedback

Difficulty: Average

Peer Responses

Session Progress

Responses Correct: 0

Responses Incorrect: 45

Responses Total: 45

Responses - % Correct: 0%

https://mypastest.pastest.com/Secure/TestMe/Browser/436619[‫ ص‬04:50:45 10/12/1437]


MyPastest

Blog
About Pastest
Contact Us
Help

© Pastest 2016

https://mypastest.pastest.com/Secure/TestMe/Browser/436619[‫ ص‬04:50:45 10/12/1437]


MyPastest

Prefer to use the old MyPastest? Access it here »

Back to Filters

Question 40 of 53

A 14 year old presents to the emergency department following a


blow to the face playing rugby. He is holding a
blood stained towel under his nose. His father informs you that he had a nosebleed which took 10-15 minutes to
settle but has now stopped completely. The boy is
now complaining that of difficulty breathing through his nose.
On inspection there is visible bruising to the nose and upper lip, inspection inside the nares reveals bilateral red
swellings arising from
the septum.

What is the best next step in management?

A Apply pressure to the soft part of the nose for 15 minutes

B CT head

C Intranasal corticosteroid

D Reassure and discharge with head injury advice

E Refer to ENT team

Explanation
Apply pressure to the soft part of the nose for 15 minutes – The nosebleed has now stopped therefore compression
is no longer required. The boy has developed a septal haematoma after traumatic injury. Referral to an ENT
surgeon is indicated as untreated there is high risk of cartilage breakdown and complications related to infection

CT head – This fourteen year old has developed a


septal haematoma after traumatic injury. Referral to an ENT
surgeon is
indicated as untreated there is high risk of cartilage breakdown and complications related to infection.
The nasal septum is visible under direct vision or rhinoscopy, CT head is not indicated.

Intranasal corticosteroid – This fourteen year old has developed a


septal haematoma after traumatic injury.
Referral to an ENT surgeon is
indicated as untreated there is high risk of cartilage breakdown and complications
related to infection. Management consists of drainage and
antibiotics. There is no role for intranasal
corticosteroids.

https://mypastest.pastest.com/Secure/TestMe/Browser/436619[‫ ص‬04:51:10 10/12/1437]


MyPastest

Reassure
and discharge with head injury advice – This fourteen year old has developed a septal haematoma after
traumatic injury. Referral to an
ENT surgeon is indicated as untreated there is high risk of cartilage breakdown
and complications related to infection.

Refer to ENT team – This fourteen year old has developed a septal haematoma after traumatic injury. Blood
collects between the septal cartilage and
the perichondrium. Classical presentation is with symptoms of nasal
obstruction and pain following a nasal injury. Take care not to misdiagnose a septal haematoma as a blood clot on
the septum, in the case of a septal haematoma the swelling will typically be visible on both sides of the septum
although this is not always the case. Referral
to an ENT surgeon is indicated as untreated there is high risk of
Next Question
cartilage breakdown and complications related to infection.
47519

End Session
Tag Question

Feedback

Previous Question Difficulty: Average

Peer Responses

Session Progress

Responses Correct: 0

Responses Incorrect: 46

Responses Total: 46

Responses - % Correct: 0%

Blog
About Pastest
Contact Us
Help

© Pastest 2016

https://mypastest.pastest.com/Secure/TestMe/Browser/436619[‫ ص‬04:51:10 10/12/1437]


MyPastest

https://mypastest.pastest.com/Secure/TestMe/Browser/436619[‫ ص‬04:51:10 10/12/1437]


MyPastest

Prefer to use the old MyPastest? Access it here »

Back to Filters

Question 41 of 53

A 15 year old presents to the emergency department following a


blow to the face playing rugby. He is holding a
towel on his nose which is saturated with blood. The bleeding ceases with direct compression. On examination a
pink / blue mass is visible filling the right nostril. There is nil else to find on ENT examination. On direct
questioning the boy reports worsening problems with a ‘blocked nose’.

What is the most likely diagnosis?

A Hereditary haemorrhagic telangiectasia

B Idiopathic epistaxis

C Juvenile Angiofibroma

D Nasopharyngeal teratoma

E Septal haematoma

Explanation
Hereditary haemorrhagic telangiectasia - Hereditary haemorrhagic telangiectasia (Osler-Weber-Rendu Syndrome)
is an inherited vasculitis that classically presents with epistaxis and the
presence of telangiectasia on the fingers
and mucous membranes.

Idiopathic
epistaxis – Epistaxis is common in children and adults (although it is rare in young children, < 2 years
of age). Approximately 90% are anterior bleeds arising from little’s area (where the venous plexus forms on the
septum). Basic first aid is all that is required in
the management of the vast majority. This involves pinching
together the soft part of the nose, below the cartilage, for ten to fifteen minutes. In this case the presence of a ‘pink
/ blue mass... filling the right nostril’ rules out idiopathic epistaxis.

Juvenile
Angiofibroma – Juvenile angiofibromas are rare tumours which most
commonly occur in adolescent
males. They typically present with nasal obstruction, recurrent ear infection and epistaxis which may be life
threatening due to the high vascularity of the tumour. Examination reveals a mass which is red – blue in colour.

https://mypastest.pastest.com/Secure/TestMe/Browser/436619[‫ ص‬04:51:33 10/12/1437]


MyPastest

They are benign but


may be locally destructive. Treatment is with surgical excision.

Nasopharyngeal
carcinoma – Nasopharyngeal carcinoma is rare accounting for somewhere in the region of 1% of
childhood malignancies. There is an association with EBV infection. Males are more commonly affected than
females and in the paediatric population the condition usually presents in adolescence. Presentation is typically
with obstruction, recurrent ear infection and epistaxis. On examination significant cervical lymphadenopathy is
palpable (which in this question is not the case). Treatment is mainly with radiotherapy.

Septal haematoma – A
septal hamatoma occurs when blood collects between the septal cartilage
and the
perichondrium. Classical presentation is with symptoms of nasal obstruction and pain following a nasal injury.
Next Question
Take care not to misdiagnose a septal haematoma as a blood clot on the septum, in the case of a septal haematoma
the swelling will typically be visible on both sides of the septum although this is not always the case. Referral to
an ENT surgeon is indicated as untreated there is high risk of cartilage breakdown and complications related to
infection. In this case the chronic history of nasal obstruction prior to the traumatic injury along with the presence
of a blue coloured unilateral swelling points to a diagnosis of Juvenile Angifibroma.
47520
End Session

Previous Question Tag Question

Feedback

Difficulty: Average

Peer Responses

Session Progress

Responses Correct: 0

Responses Incorrect: 47

Responses Total: 47

Responses - % Correct: 0%

https://mypastest.pastest.com/Secure/TestMe/Browser/436619[‫ ص‬04:51:33 10/12/1437]


MyPastest

Blog
About Pastest
Contact Us
Help

© Pastest 2016

https://mypastest.pastest.com/Secure/TestMe/Browser/436619[‫ ص‬04:51:33 10/12/1437]


MyPastest

Prefer to use the old MyPastest? Access it here »

Back to Filters

Question 42 of 53

You are teaching a group of medical students about ENT conditions in children. One of the students tells you she
has a 2 year old sister who has been diagnosed with ‘glue ear’. She asks
you what the risk factors are for this
condition.

Which of the following is NOT a risk factor for otitis media with effusion?

A Breast feeding

B Cleft palate

C Cystic fibrosis

D Down's syndrome

E Parental smoking

Explanation
Breast feeding – ‘Glue ear’ refers to otitis media with effusion. The condition is characterised by
fluid collecting
behind the tympanic membrane within the middle ear in the absence of acute inflammation. A number of risk
factors are well documented. It is more common in children who are formula fed, breastmilk is protective.

Cleft palate – Otitis media with


effusion is more common in children with cleft palate due to impaired function of
the Eustachian tube.

Cystic fibrosis – Otitis media with effusion is more common in children with cystic fibrosis.

Down's
syndrome – Otitis media with effusion is more common in children with trisomy 21. This is thought to be
as a result of impaired immunity.

Parental smoking – Otitis Media with Effusion is more common in children living in a household with a smoker.

NICE Guidelines: http://cks.nice.org.uk/otitis-media-with-effusion


47521

https://mypastest.pastest.com/Secure/TestMe/Browser/436619[‫ ص‬04:51:55 10/12/1437]


MyPastest

Tag Question

Feedback

Next Question
Difficulty: Average

Peer Responses

End Session

Previous Question

Session Progress

Responses Correct: 0

Responses Incorrect: 48

Responses Total: 48

Responses - % Correct: 0%

Blog
About Pastest
Contact Us
Help

© Pastest 2016

https://mypastest.pastest.com/Secure/TestMe/Browser/436619[‫ ص‬04:51:55 10/12/1437]


MyPastest

Prefer to use the old MyPastest? Access it here »

Back to Filters

Question 43 of 53

A 5 year old presents with his mother who is concerned that over the past week he has been listening to the TV on
excessive volumes.
He has mentioned his ears have been ‘popping’ but has been
otherwise well. There have been
no reported concerns form school. He has had a number of diagnoses of acute otitis media in the past, most
recently 3 weeks ago. On examination otoscopy reveals a right sided retracted pale tympanic membrane with a
diffuse light reflex.

What is the next best step in management?

A Prescribe Amoxicillin

B Prescribe decongestants

C Prescribe Erythromycin

D Reassure and arrange follow up

E Refer to ENT

Explanation
Prescribe Amoxicillin – This child has otitis media with effusion (OME) which is commonly known as ‘glue ear’.
In the vast majority of cases of OME spontaneous resolution
occurs and a period of watchful waiting for six to
twelve weeks is appropriate. Drugs such as antibiotics and decongestants are not indicated for the management of
OME.

Prescribe decongestants – This child has otitis media with effusion (OME) which is commonly known as ‘glue
ear’. In the vast majority of cases
of OME spontaneous resolution occurs and a period of watchful waiting for six
to twelve weeks is appropriate. Drugs such as antibiotics and decongestants are not indicated for the management
of OME.

Prescribe
Erythromycin – This child has otitis media with effusion (OME) which is commonly known as ‘glue
ear’. In the vast majority
of cases of OME spontaneous resolution occurs and a period of watchful waiting for six

https://mypastest.pastest.com/Secure/TestMe/Browser/436619[‫ ص‬04:52:17 10/12/1437]


MyPastest

to twelve weeks is appropriate. Drugs such as antibiotics and decongestants are not indicated for the management
of OME.

Reassure and arrange follow up – This child has otitis media with effusion (OME) which is commonly known as
‘glue ear’. The condition is characterised by fluid collecting behind the tympanic membrane within the middle ear
in the absence of acute inflammation. The most common presentation is with hearing loss and for
this reason
presentation can be late in infants and young children. Other features may include a sensation of fullness within
the ear and pain. There is often a preceding history of recurrent upper respiratory
tract infections. In the vast
majority of cases of OME spontaneous resolution occurs and a period of watchful waiting for six to twelve weeks
is appropriate. Drugs such as antibiotics and decongestants are not indicated for the management of OME.
Next Question
Refer to ENT – This child has otitis media with effusion (OME) which is commonly known as ‘glue ear’. In the
vast majority of cases of OME spontaneous resolution occurs and a period of watchful waiting for six to twelve
weeks is appropriate. If the symptoms persist past this time period referral should be made for an audiology
assessment or to an ENT specialist. Earlier referral is appropriate if severe hearing difficulty is suspected or the
hearing impairment is disturbing the child’s developmental/educational progression. Refer children with trisomy
End Session
21 or cleft palate for immediate ENT assessment.

NICE Guidelines: http://cks.nice.org.uk/otitis-media-with-effusion


47522

Previous Question

Tag Question

Feedback

Difficulty: Average

Peer Responses

Session Progress

Responses Correct: 0

Responses Incorrect: 49

Responses Total: 49

https://mypastest.pastest.com/Secure/TestMe/Browser/436619[‫ ص‬04:52:17 10/12/1437]


MyPastest

Responses - % Correct: 0%

Blog
About Pastest
Contact Us
Help

© Pastest 2016

https://mypastest.pastest.com/Secure/TestMe/Browser/436619[‫ ص‬04:52:17 10/12/1437]


MyPastest

Prefer to use the old MyPastest? Access it here »

Back to Filters

Question 44 of 53

A 5 year old with Down’s syndrome presents with his mother who is concerned that over the past week he has had
difficulty hearing. He has been complaining of a painful right ear but has been otherwise well. There have been no
reported concerns form school. He has had a number of diagnoses of acute otitis media in the past, most recently 3
weeks ago. On examination otoscopy reveals a right sided retracted pale tympanic membrane with a diffuse light
reflex.

What is the next best step in management?

A Prescribe Amoxicillin

B Prescribe decongestants

C Prescribe Erythromycin

D Reassure and arrange follow up

E Refer to ENT

Explanation
Prescribe Amoxicillin – This child has otitis media with effusion (OME) which is commonly known as ‘glue ear’.
Drugs such as antibiotics and decongestants are not indicated for the management of OME. In the vast majority of
cases of OME spontaneous resolution occurs and a period of watchful waiting for six to twelve weeks is
appropriate however this child has trisomy 21 and
therefore warrants immediate referral.

Prescribe decongestants – This child has otitis media with effusion (OME) which is commonly known as ‘glue
ear’. Drugs such as antibiotics and
decongestants are not indicated for the management of OME. In the vast
majority of cases of OME spontaneous resolution occurs and a period of watchful waiting for six to twelve weeks
is appropriate however this child has trisomy 21 and therefore warrants immediate referral.

Prescribe
Erythromycin – This child has otitis media with effusion (OME) which is commonly known as ‘glue
ear’. Drugs such as antibiotics and decongestants are not indicated for the management of OME. In the vast

https://mypastest.pastest.com/Secure/TestMe/Browser/436619[‫ ص‬04:52:43 10/12/1437]


MyPastest

majority of cases of OME spontaneous resolution occurs


and a period of watchful waiting for six to twelve weeks
is appropriate
however this child has trisomy 21 and therefore warrants immediate referral.

Reassure and arrange follow up – This child has otitis media with effusion (OME) which is commonly known as
‘glue ear’. In the vast majority of cases of OME spontaneous resolution occurs and a period of watchful waiting
for six to twelve weeks is appropriate. This child has trisomy 21 however and this is an indication for immediate
referral.

Refer to ENT – This child has otitis media with effusion (OME) which is commonly known as ‘glue ear’. The
condition is characterised by fluid collecting behind the tympanic membrane within the middle ear in the absence
Next Question
of acute inflammation. The most common presentation is with hearing loss and for this reason presentation can be
late in infants and
young children. Other features may include a sensation of fullness within the ear and pain.
There is often a preceding history of recurrent upper respiratory tract infections. In the vast majority of cases of
OME spontaneous resolution occurs and a period of watchful waiting for six to twelve weeks is appropriate
however this child has trisomy 21 and therefore warrants immediate referral for ENT assessment.
Other
indications for early referral include cleft palate and suspicion of severe hearing difficulty or that hearing
End Session
impairment is disturbing the child’s developmental/educational progression.

NICE Guidelines: http://cks.nice.org.uk/otitis-media-with-effusion


47523

Previous Question

Tag Question

Feedback

Difficulty: Average

Peer Responses

Session Progress

Responses Correct: 0

Responses Incorrect: 50

Responses Total: 50

https://mypastest.pastest.com/Secure/TestMe/Browser/436619[‫ ص‬04:52:43 10/12/1437]


MyPastest

Responses - % Correct: 0%

Blog
About Pastest
Contact Us
Help

© Pastest 2016

https://mypastest.pastest.com/Secure/TestMe/Browser/436619[‫ ص‬04:52:43 10/12/1437]


MyPastest

Prefer to use the old MyPastest? Access it here »

Back to Filters

Question 45 of 53

A 5 year returns for review six weeks after being diagnosed with otitis media with effusion. His mother is anxious
that things have
not improved, he has difficulty hearing when there is any background noise. There have been no
concerns from school. You wish to confirm the diagnosis of otitis media with effusion.

What is the most appropriate investigation to confirm the diagnosis?

A CT Head

B Distraction testing

C Otoscopy

D Tuning fork testing (Weber and Rinne test)

E Tympanometry

Explanation
CT Head – There is no indication for the use of CT imaging in the investigation or management of otitis media
with effusion.

Distraction testing – Referral to audiology


services for formal audiometric testing is indicated for persisting
symptoms of otitis media with effusion. Distraction is typically used for children aged between the age of 7 and 30
months. The examiner makes sounds behind the infant/child ensuring the object producing the sound is out of
view. The response to the sound is observed. The child in this case is five years of age, play or pure tone
audiometry may be used in a child of this age.

Otoscopy – Otoscopy is an essential part of any ENT examination. It may identify features of OME, such as an
effusion, as well as alternative diagnoses however a tympanic membrane with normal appearance does not rule out
OME.

Tuning
fork testing (Weber and Rinne test) – Tuning fork tests are not recommended routinely for aiding

https://mypastest.pastest.com/Secure/TestMe/Browser/436619[‫ ص‬04:53:27 10/12/1437]


MyPastest

diagnosis of otitis media with effusion.

Tympanometry – Otitis media with effusion (OME), commonly known as ‘glue ear’, is characterised by fluid
collecting behind the tympanic membrane within the middle ear in the absence of acute inflammation. The most
common presentation is with
hearing loss and for this reason presentation can be late in infants and young
children. In the vast majority of cases of OME spontaneous resolution occurs and a period of watchful waiting for
six to twelve weeks is appropriate. If symptoms persist referral for a hearing test or to an ENT specialist is
indicated. Tympanometry assesses the response of the tympanic membrane to sound and can be used to help
diagnoses OME. Hand-held tympanometers are readily available but require specialist training.
Next Question
NICE Guidelines: http://cks.nice.org.uk/otitis-media-with-effusion
47524

End Session
Tag Question

Feedback

Previous Question Difficulty: Average

Peer Responses

Session Progress

Responses Correct: 0

Responses Incorrect: 51

Responses Total: 51

Responses - % Correct: 0%

Blog
About Pastest
Contact Us
Help

© Pastest 2016

https://mypastest.pastest.com/Secure/TestMe/Browser/436619[‫ ص‬04:53:27 10/12/1437]


MyPastest

https://mypastest.pastest.com/Secure/TestMe/Browser/436619[‫ ص‬04:53:27 10/12/1437]


MyPastest

Prefer to use the old MyPastest? Access it here »

Back to Filters

Question 46 of 53

The mother of a 3 year old attends the surgery to address concerns she has prior to travelling abroad on holiday.
The child had Grommets inserted six weeks ago for otitis media with effusion.

Which of the following should be discouraged for children with grommets?

A Bathing in soapy water

B Flying

C Snorkeling

D Swimming

E Using in ear headphones

Explanation
Bathing in soapy water – Grommets are very small tubes inserted into the tympanic membrane that allows fluid to
drain form the middle ear and equalises the pressure. They are usually unproblematic and gradually work their
way out of the ear drum over time. There are few ‘do’s and ‘don’ts’ when it comes to grommets but immersion of
the child’s head in soapy water should be avoided as the soap reduces the surface tension of
the water increasing
the likelihood of it entering the middle ear.

Flying – Flying should not cause any problems to children with grommets in-situ.

Snorkeling
– Swimming and snorkelling are not associated with any increased infection risk and swimming caps
or ear plugs are not necessary. Diving, especially to significant depth, should be avoided.

Swimming
– Swimming is not associated with any increased infection risk and swimming caps or ear plugs are
not necessary. Advise against diving.

Using in ear headphones – There is no contraindication to using in ear headphones. As with all children the advice
should be to keep volumes to a minimum to prevent damage to hearing.

https://mypastest.pastest.com/Secure/TestMe/Browser/436619[‫ ص‬04:53:50 10/12/1437]


MyPastest

NICE Guidelines: http://cks.nice.org.uk/otitis-media-with-effusion


47525

Tag Question

Feedback Next Question


Difficulty: Average

Peer Responses
End Session

Previous Question

Session Progress

Responses Correct: 0

Responses Incorrect: 52

Responses Total: 52

Responses - % Correct: 0%

Blog
About Pastest
Contact Us
Help

© Pastest 2016

https://mypastest.pastest.com/Secure/TestMe/Browser/436619[‫ ص‬04:53:50 10/12/1437]


MyPastest

Prefer to use the old MyPastest? Access it here »

Back to Filters

Question 47 of 53

An overweight 6 year old boy presents with parental concerns that his breathing is ‘strange’ at night, and that this
has been the case for some months now. This is associated with frequent and
loud snoring. They show you a
video, filmed on a mobile phone, where significant recession is clearly visible. Clinical examination reveals
bilaterally tonsillar hypertrophy.

What is the best next step in management?

A Capillary blood gas

B CXR

C Refer for ENT opinion

D Trial of oral phenoxymethilpenicillin and arrange review

E Weight loss advice

Explanation
Capillary blood gas – This child has clinical signs of adenotonsillar hypertrophy, symptoms of persistent snoring
and features of obstructive sleep apnoea (OSA). Suspected OSA warrants referral to an ENT specialist.
Investigation consists of overnight oxygen saturation monitoring or polysomnography.

CXR – This child has clinical signs of adenotonsillar hypertrophy, symptoms of persistent snoring and features of
obstructive sleep apnoea (OSA). Suspected OSA warrants referral to an ENT specialist. Chest x-ray is not
indicated.

Refer for ENT opinion – This child


has clinical signs of adenotonsillar hypertrophy, symptoms of persistent
snoring and features of obstructive sleep apnoea (OSA). Suspected OSA warrants referral to an ENT specialist.
Suspect OSA in children with snoring, apnoea, increased work of breathing or unusual posturing (e.g. head held in
neck extension) when sleeping. Disturbed sleep can also result in daytime symptoms such as impaired
concentration
and school performance.

https://mypastest.pastest.com/Secure/TestMe/Browser/436619[‫ ص‬04:54:20 10/12/1437]


MyPastest

Trial of oral phenoxymethilpenicillin and arrange review – Phenoxymehylpenicillin (Penicillin V) is the


first line
treatment for tonsillitis however with a history of several months and no suggestion of fever or sore throat acute
tonsillitis is unlikely. This child has clinical signs of adenotonsillar hypertrophy, symptoms of persistent snoring
and features of obstructive sleep apnoea (OSA). Suspected OSA warrants referral to an ENT specialist.

Weight
loss advice – Obesity is a risk factor for obstructive sleep apnoea (OSA) amongst other health problems.
Although the child’s weight should be highlighted and advice and support for weight loss provided referral to an
ENT specialist is indicated when features of OSA
are present.

NICE Guidelines: http://cks.nice.org.uk/obstructive-sleep-apnoea-syndrome


Next Question 47526

End Session
Tag Question

Feedback

Previous Question Difficulty: Average

Peer Responses

Session Progress

Responses Correct: 0

Responses Incorrect: 53

Responses Total: 53

Responses - % Correct: 0%

Blog
About Pastest
Contact Us
Help

© Pastest 2016

https://mypastest.pastest.com/Secure/TestMe/Browser/436619[‫ ص‬04:54:20 10/12/1437]


MyPastest

https://mypastest.pastest.com/Secure/TestMe/Browser/436619[‫ ص‬04:54:20 10/12/1437]


MyPastest

Prefer to use the old MyPastest? Access it here »

Back to Filters

Question 48 of 53

An 8 year old boy presents with parental concerns that he has


always snored loudly at night but has recently had
episodes where he has stopped breathing all together whilst asleep. You aim to focus your
history on possible
complications that may affect him.

Which of the following is NOT a complication associated with his underlying diagnosis?

A Behavioural deterioration

B Daytime somnolence

C Nocturnal enuresis

D Pulmonary hypertension

E Weight gain

Explanation
Behavioural deterioration – Daytime sleepiness, poor concentration, behavioural deterioration and impaired
educational performance are significant problems for many children with obstructive sleep apnoea.

Daytime somnolence – Daytime sleepiness, poor concentration, behavioural deterioration and impaired
educational performance are significant problems for many children with obstructive sleep apnoea.

Nocturnal enuresis – Nocturnal enuresis has a recognised association with obstructive sleep apnoea. This is
thought to be due to difficulty wakening from sleep.

Pulmonary
hypertension – Long-term hypoxia as a result of obstructive sleep
apnoea can lead to the development
of pulmonary hypertension.

Weight gain – This child has obstructive sleep apnoea (OSA). Suspect OSA in children who have the following
when asleep; snoring, apnoea, increased work of breathing or unusual posturing, restless sleep
or bedwetting.
Daytime symptoms include excessive sleepiness, irritability and behavioural deterioration. Obesity is a risk factor

https://mypastest.pastest.com/Secure/TestMe/Browser/436619[‫ ص‬04:55:03 10/12/1437]


MyPastest

for
obstructive sleep apnoea however faltering growth is a recognised complication in severe cases.

NICE Guidelines: http://cks.nice.org.uk/obstructive-sleep-apnoea-syndrome


47527

Tag Question
Next Question
Feedback

Difficulty: Average

Peer Responses End Session

Previous Question

Session Progress

Responses Correct: 0

Responses Incorrect: 54

Responses Total: 54

Responses - % Correct: 0%

Blog
About Pastest
Contact Us
Help

© Pastest 2016

https://mypastest.pastest.com/Secure/TestMe/Browser/436619[‫ ص‬04:55:03 10/12/1437]


MyPastest

Prefer to use the old MyPastest? Access it here »

Back to Filters

Question 49 of 53

A 5 year old boy presented after the school reported a deterioration in his concentration and performance. His
parents had pre-existing concerns that his breathing was abnormal when sleeping. They reported loud snoring and
respiratory pauses. His parents feel this is likely to be as a result of frequent episodes of tonsillitis. On examination
the child has extensive bilateral tonsillar hypertrophy. You have referred the boy for ENT review with a working
diagnosis of obstructive sleep apnoea (OSA). His parents would like to know what is likely to happen next and if
he can have his tonsils removed.

What is the best advice to give his parents?

A Adenotonsillectomy is indicated if there is a history of peritonsillar abscess

B Adenotonsillectomy is no longer the treatment of choice in the management of OSA

C Adenotonsillectomy may be required following polysomnography

D BiPAP is first line treatment

E CPAP is first line treatment

Explanation
Adenotonsillectomy is indicated if there is a history of peritonsillar abscess – A peritonsillar abscess (quinsy) is a
complication of tonsillitis. Management requires incision
and drainage along with antibiotics and tonsillectomy is
often recommended following recovery. In the case of obstructive sleep apnoea
most children with evidence of
adenotonsillar hypertrophy will be offered adenotonsillectomy irrespective of a history of peritonsillar abscess.

Adenotonsillectomy is no longer the treatment of choice


in the management of OSA – Adenotonsillectomy is
frequently offered to children with obstructive sleep apnoea and clinical evidence of adenotonsillar hypertrophy.
Adenotonsillectomy has become less frequently performed for recurrent uncomplicated tonsillitis.

Adenotonsillectomy
may be required following polysomnography – Sleep studies (polysomnography) are the gold
standard diagnostic investigation for obstructive sleep apnoea in both adults and children. Once the diagnosis is

https://mypastest.pastest.com/Secure/TestMe/Browser/436619[‫ ص‬04:55:27 10/12/1437]


MyPastest

confirmed most children with evidence of adenotonsillar hypertrophy will be offered adenotonsillectomy.

BiPAP is first line treatment – Bilevel positive airway pressure (BiPAP) has been shown to be no more effective
than continuous positive airway pressure (CPAP) in the treatment of obstructive sleep apnoea. CPAP is first line in
the management of moderate to severe OSA in adults. In the paediatric population CPAP should be considered if
adenotonsillectomy is deemed unlikely to be of benefit or is contraindicated.

CPAP is first line treatment – Continuous Positive Airway Pressure (CPAP) is first line in the management of
moderate to severe OSA in adults. In the paediatric population CPAP should be considered if adenotonsillectomy
is deemed unlikely to be of benefit or is contraindicated.
Next Question
NICE Guidelines: http://cks.nice.org.uk/obstructive-sleep-apnoea-syndrome
47528

End Session
Tag Question

Feedback

Previous Question
Difficulty: Average

Peer Responses

Session Progress

Responses Correct: 0

Responses Incorrect: 55

Responses Total: 55

Responses - % Correct: 0%

Blog
About Pastest
Contact Us
Help

© Pastest 2016

https://mypastest.pastest.com/Secure/TestMe/Browser/436619[‫ ص‬04:55:27 10/12/1437]


MyPastest

https://mypastest.pastest.com/Secure/TestMe/Browser/436619[‫ ص‬04:55:27 10/12/1437]


MyPastest

Prefer to use the old MyPastest? Access it here »

Back to Filters

Question 50 of 53

Which of the following is NOT a risk factor for Sudden Infant Death Syndrome (SIDS)?

A A dummy / pacifier

B Being placed to sleep in a prone position

C Co-sleeping

D Male sex

E Prematurity

Explanation
A dummy / pacifier – Sudden Infant Death Syndrome (SIDS) is the term given to the sudden and unexplained
death of
an infant for which no cause is identified after detailed post mortem. Although the condition is not fully
understood a variety of associations have been identified. A number of studies have shown that infants who
regularly use a dummy have a lower risk of SIDS.

Being
placed to sleep in a prone position – All infants should be placed in a supine position when put down to
sleep. Infants put to sleep on their fronts have been found to be up to six times more at risk
of SIDS.

Co-sleeping – New parents should be advised to avoid co-sleeping particularly if they have used alcohol or are
feeling excessively tired. Co-sleeping on a sofa may increase the risk of SIDS fiftyfold.

Male sex – Sudden Infant Death Syndrome (SIDS)


is more common in males. In 2012 64% of all sudden and
unexplained infant deaths occurred in males. In 2013 55% of cases were in males.

Prematurity
– Prematurity and low birth weight is associated with an increased risk of SIDS. Being born with a
low birth weight increases the risk approximately threefold.

For more information on SIDS see: http://www.lullabytrust.org.uk/


47529

https://mypastest.pastest.com/Secure/TestMe/Browser/436619[‫ ص‬04:55:50 10/12/1437]


MyPastest

Tag Question

Feedback

Next Question
Difficulty: Average

Peer Responses

End Session

Previous Question

Session Progress

Responses Correct: 0

Responses Incorrect: 56

Responses Total: 56

Responses - % Correct: 0%

Blog
About Pastest
Contact Us
Help

© Pastest 2016

https://mypastest.pastest.com/Secure/TestMe/Browser/436619[‫ ص‬04:55:50 10/12/1437]


MyPastest

Prefer to use the old MyPastest? Access it here »

Back to Filters

Question 51 of 53

To reduce the risk of SIDS parents should be advised that their baby should sleep in a moses basket or cot in the
same room as them for how many months after birth?

A 3 months

B 6 months

C 9 months

D 12 months

E 15 months

Explanation
3 months – In order to reduce the risk of SIDS parents should be advised that their baby should sleep in the same
room as them, in a moses basket or cot, for the first six months of its life.

6 months – Sudden Infant Death Syndrome (SIDS) is the term given to the sudden and unexplained death of an
infant for which no cause is identified after detailed post mortem. Although the condition is not fully understood a
variety of associations have been identified In order to reduce the risk of SIDS parents should be advised
that their
baby should sleep in the same room as them, in a moses basket or cot, for the first six months of its life.

9 months – In order to reduce the risk of SIDS parents should be advised that their baby should sleep in the same
room as them, in a moses basket
or cot, for the first six months of its life.

12 months –
In order to reduce the risk of SIDS parents should be advised that their baby should sleep in the same
room as them, in a moses basket or cot, for the first six months of its life.

15 months – In order to reduce the risk of SIDS parents should be advised that their baby should sleep in the same
room as them, in a moses basket or cot, for the first six months of its life.

For information on SIDS see: http://www.lullabytrust.org.uk/

https://mypastest.pastest.com/Secure/TestMe/Browser/436619[‫ ص‬04:56:12 10/12/1437]


MyPastest

47530

Tag Question

Feedback

Next Question
Difficulty: Average

Peer Responses

End Session

Previous Question

Session Progress

Responses Correct: 0

Responses Incorrect: 57

Responses Total: 57

Responses - % Correct: 0%

Blog
About Pastest
Contact Us
Help

© Pastest 2016

https://mypastest.pastest.com/Secure/TestMe/Browser/436619[‫ ص‬04:56:12 10/12/1437]


MyPastest

Prefer to use the old MyPastest? Access it here »

Back to Filters

Question 52 of 53

A 13 year old girl is referred to the paediatric assessment unit with a widespread macular rash. She has recently
seen her GP with a
flu like illness and sore throat for which she was prescribed Amoxicllin with a diagnosis of
tonsillitis. On examination she has widespread cervical lymphadenopathy and petechiae on her palate.

What is the most likely underlying pathogen?

A Coronavirus

B Epstein Barr Virus

C Group A Streptococcus

D Parainfluenza

E Rhinovirus

Explanation
Coronavirus – This is one of the causes of the common cold and is frequently responsible for sore throats however
in this case there are other factors suggesting EBV infection.

Epstein
Barr Virus – Epstein Barr virus (EBV) infection most commonly results in upper respiratory tract
symptoms with fever and sore throat. Lymphadenopathy is commonly palpable on examination. Splenomegaly
may also occur and for this reason patients diagnosed with glandular fever should be advised to avoid contact
sports for one month following diagnosis. The presence of petechiae on the soft palate is also associated with EBV
infection. The vast majority of patients with active EBV infection will develop a widespread rash if given
amoxicillin
therefore phenoxymethylpenicillin (Penicillin V) is recommended for the
treatment of a suspected
bacterial throat infection to prevent this complication occurring for individuals who in fact have EBV infection.

Group
A Streptococcus – Streptococci are classified into two main groups; alpha-haemolytic and beta-haemolytic.
Group A Streptococcus belongs to the beta-haemolytic group. Group A streptococcal infection is a common cause
of bacterial tonsillitis. It is possible that the child in this case had a streptococcal tonsillitis and has had a drug

https://mypastest.pastest.com/Secure/TestMe/Browser/436619[‫ ص‬04:56:36 10/12/1437]


MyPastest

reaction to the amoxicillin however her age, flu-like symptoms and petechiae on her palate all point to the classic
association of EBV infection and amoxicillin resulting in a rash.

Parainfluenza – Parainfluenza viruses are one of many causes of the common cold and can cause pharyngitis.
They are the main cause of viral croup. In
this case there are other factors suggesting EBV infection.

Rhinovirus
– Rhinoviruses are one of many causes of the common cold and are
frequently responsible for sore
throats however in this case there are other factors suggesting EBV infection.
47531

Next Question

Tag Question

Feedback End Session

Difficulty: Average

Peer Responses
Previous Question

Session Progress

Responses Correct: 0

Responses Incorrect: 58

Responses Total: 58

Responses - % Correct: 0%

Blog
About Pastest
Contact Us
Help

© Pastest 2016

https://mypastest.pastest.com/Secure/TestMe/Browser/436619[‫ ص‬04:56:36 10/12/1437]


MyPastest

Prefer to use the old MyPastest? Access it here »

Back to Filters

Question 53 of 53

An 11 year old girl presents to the paediatric assessment unit with a 4 day history of worsening sore throat and one
day of fever.
She denies any history of cough and is able to tolerate small volumes of oral fluids. On examination
she is febrile with a temperature of 38°5 C, ENT examination reveals bilateral tonsillar enlargement with
exudate.
There are shotty cervical lymph nodes palpable.

What is the best next step in management?

A Amoxicillin, 5 days

B Amoxicillin, 10 days

C Penicillin V, 5 days

D Penicillin V, 10 days

E Reassurance

Explanation
Amoxicillin, 5 days – This child has tonsillitis. Although this may be viral or bacterial using the Centor scoring
system many clinicians would treat this child as a bacterial tonsillitis with a ten day course of Penicillin V.

Amoxicillin, 10 days – This child has tonsillitis. Although this may be viral or bacterial using the Centor scoring
system many clinicians would treat
this child as a bacterial tonsillitis with a ten day course of Penicillin V.

Penicillin V, 5 days – This child has tonsillitis. Although this may be viral or bacterial using the Centor scoring
system many clinicians would treat this child as a bacterial tonsillitis with a ten day course of Penicillin V.

Penicillin V, 10 days – Although the majority of throat infections are viral approximately 10-30% of children with
tonsillitis have group A streptococcal infection. Identifying which children to treat with antibiotics poses a
challenge. Scoring systems are available as a clinical aid, the most well known of which is the Centor score. One
point is given for each of the four following criteria; presence of tonsillar exudates, tender anterior cervical

https://mypastest.pastest.com/Secure/TestMe/Browser/436619[‫ ص‬04:56:59 10/12/1437]


MyPastest

lymphadenopathy, history of
fever and absence of cough. The likelihood of Group A streptococcal infection for a
score of 1 is between 2-23%, with a score of 4 it increases to between 25 – 86%. When considering the use of
throat
swabs to investigate pharungitis / tonsillitis and guide management it is important to remember that the
carrier rate for group A streptococcus
may be as high as 40% therefore a positive swab does not confirm the
diagnosis of bacterial tonsillitis. Antibiotics have been shown to provide a modest benefit in the treatment of sore
throats reducing symptom duration approximately 16 hours within the first week. Once the
clinical decision to
treat with antibiotics has been made this should be with a ten day course of phenoxymethylpenicillin (Penicillin
V). Amoxicillin should be avoided as if EBV infection is the true cause of the sore throat 90% of individuals given
amoxicillin will develop a rash. A ten day course of penicillin V should be used as opposed to a seven day course
as studies have demonstrated a greater treatment failure rate with the shorter seven day course.

Reassurance – This child has tonsillitis. Although this may be viral or bacterial using the Centor scoring system
many clinicians would treat this child as a bacterial tonsillitis with a ten day course of Penicillin V.

NICE Guidelines: http://cks.nice.org.uk/sore-throat-acute


47532

End Session

Previous Question Tag Question

Feedback

Difficulty: Average

Peer Responses

Session Progress

Responses Correct: 0

Responses Incorrect: 59

Responses Total: 59

Responses - % Correct: 0%

https://mypastest.pastest.com/Secure/TestMe/Browser/436619[‫ ص‬04:56:59 10/12/1437]


MyPastest

Blog
About Pastest
Contact Us
Help

© Pastest 2016

https://mypastest.pastest.com/Secure/TestMe/Browser/436619[‫ ص‬04:56:59 10/12/1437]


MyPastest

Next Question
Prefer to use the old MyPastest? Access it here »

Back to Filters

Question 1 of 16

You see a child in clinic with newly diagnosed type 1 diabetes.

What is considered to be the target Hba1c in children and young people with type 1 diabetes in the latest NICE
guidelines?

A HbA1C - 48mmol/mol

B HbA1C - 54mmol/mol

C HbA1C - 58mmol/mol

D HbA1C - 60mmol/mol

E HbA1C - 64mmol/mol

Explanation
NICE have encouraged much stricter glucose control than previous guidance - with the caveat that children and
young
people, and their families are given education, emotional support and obesity management, and do not feel
pressured, hence a personal management plan is required. Tighter control is considered to offer a much better
outlook by reducing the risk of complications such as nephropathy and retinopathy, hence a maximum HbA1c of
48mmol/mol is recommended.
Previous Question
https://www.nice.org.uk/guidance/ng18
47198

Tag Question

https://mypastest.pastest.com/Secure/TestMe/Browser/436619[‫ م‬11:56:30 08/12/1437]


MyPastest

Feedback

Difficulty: Average

Peer Responses
End Session

Session Progress

Responses Correct: 0

Responses Incorrect: 1

Responses Total: 1

Responses - % Correct: 0%

Blog
About Pastest
Contact Us
Help

© Pastest 2016

https://mypastest.pastest.com/Secure/TestMe/Browser/436619[‫ م‬11:56:30 08/12/1437]


MyPastest

Next Question
Prefer to use the old MyPastest? Access it here »

Back to Filters

Question 2 of 16

You have a young child in clinic whose parents are reluctant to attend clinic for routine HbA1c checks.

How often should HbA1c be checked in children and young people with type 2 diabetes according to the latest
NICE guidelines?

A Every month

B Every 2 months

C Every 3 months

D Every 4 months

E Every 6 months

Explanation
NICE updated their guidelines on diabetes in children and young people in 2015 (NG 18). There is a focus on the
patient and their family and a clear steer towards reducing complication
risk. Education, emotional support and
obesity management is discussed delivered through personal management plans to engage and create a sense
of
ownership by the patient and their family.

HbA1c should be checked every 3 months to ensure tight control is maintained, to enable early intervention in the
context of poor control and to provide a topic
of discussion during reviews.

https://www.nice.org.uk/guidance/ng18
47199

https://mypastest.pastest.com/Secure/TestMe/Browser/436619[‫ م‬11:56:54 08/12/1437]


MyPastest

Tag Question

Feedback

Difficulty: Average

End Session
Peer Responses

Session Progress

Responses Correct: 0

Responses Incorrect: 2

Responses Total: 2

Responses - % Correct: 0%

Blog
About Pastest
Contact Us
Help

© Pastest 2016

Previous Question

https://mypastest.pastest.com/Secure/TestMe/Browser/436619[‫ م‬11:56:54 08/12/1437]


MyPastest

Next Question
Prefer to use the old MyPastest? Access it here »

Back to Filters

Question 3 of 16

With reference to the appropriate Nice guidance, how often should children and young people with type 1 diabetes
attend secondary care clinics?

A Every 2 months

B Every 3 months

C Every 4 months

D Every 6 months

E Annually

Explanation
NICE updated their guidelines on diabetes in children and young people in 2015 (NG18).

A diagnosis of Type 1 diabetes is being made much more regularly than in previous decades in the UK. NICE
quotes the 2013-14 National Diabetes Audit as having identified ‘26,500 children and young people with type 1
diabetes’. Accordingly there is increased need
to focus on the patient and their family and for NICE to give a clear
steer towards reducing complication risk. Education, emotional support and obesity management is discussed and
delivered through personal management plans to engage and create a sense of ownership by the patient and their
family.

Those patients with type 1 disease should be reviewed in a specialist clinic four times a year to ensure patients and
their families continue to be well educated, to monitor HbA1c and to ensure complications are identified early.

https://www.nice.org.uk/guidance/ng18
47200

https://mypastest.pastest.com/Secure/TestMe/Browser/436619[‫ م‬11:57:15 08/12/1437]


MyPastest

Tag Question

Feedback

Difficulty: Average
End Session
Peer Responses

Session Progress

Responses Correct: 0

Responses Incorrect: 3

Responses Total: 3

Responses - % Correct: 0%

Blog
About Pastest
Contact Us
Help

© Pastest 2016

Previous Question

https://mypastest.pastest.com/Secure/TestMe/Browser/436619[‫ م‬11:57:15 08/12/1437]


MyPastest

Prefer to use the old MyPastest? Access it here »

Back to Filters

Question 4 of 16

You have a patient who was diagnosed with diabetes at the age
of 5. She is approaching the age at which
retinopathy screening should
be commenced in line with Nice guidelines.

From what age should children and young people with Type 1 diabetes have retinopathy screening?

A 6 years

B 8 years

C 10 years

D 12 years

E 14 years

Explanation
NICE updated their guidelines on diabetes in children and young people in 2015 (NG18).

There is a focus on the patient and their family and a clear steer towards reducing complication risk. Education,
emotional support and obesity management is discussed delivered through personal management plans to engage
and create a sense of ownership by the patient and their
family.

Those with type 1 disease should have thyroid function checked at diagnosis and annually from then on.
Retinopathy screening should occur from age 12 in both type 1 and type 2 diabetes with a view to early
intervention. Type 1 diabetics in particular are at greater risk of cataract.

Additionally from age 12 children should be monitored annually for moderately increased albuminuria
(albumin:creatinine ration (ACR) 3-30 mg/mmol; ‘microalbuminuria’) to detect diabetic kidney disease and for
signs of hypertension.

https://www.nice.org.uk/guidance/ng18
47201

https://mypastest.pastest.com/Secure/TestMe/Browser/436619[‫ م‬11:57:37 08/12/1437]


MyPastest

Next Question

Tag Question

Feedback End Session

Difficulty: Average

Peer Responses

Session Progress

Responses Correct: 0

Responses Incorrect: 4

Responses Total: 4

Responses - % Correct: 0%

Blog
About Pastest
Contact Us
Help

© Pastest 2016

Previous Question

https://mypastest.pastest.com/Secure/TestMe/Browser/436619[‫ م‬11:57:37 08/12/1437]


MyPastest

Prefer to use the old MyPastest? Access it here »

Back to Filters

Question 5 of 16

With reference to relevant NICE guidelines:

From
what age should children and young people with type 1 diabetes have nephropathy screening with an annual
early morning urine sample for albumin:creatinine ratio?

A From diagnosis

B 8 years

C 10 years

D 12 years

E 14 years

Explanation
NICE updated their guidelines on diabetes in children and young people in 2015 (NG18). There is a focus on the
patient and their family and a clear steer towards reducing complication
risk. Education, emotional support and
obesity management is discussed delivered through personal management plans to engage and create a sense
of
ownership by the patient and their family.

Those with type 1


disease should have thyroid function checked at diagnosis and annually from then on.
Retinopathy and nephropathy screening should be offered annually from age 12 in type 1 disease whereas children
and young people
with type 2 diabetics should have urine ACR from diagnosis.

In
either case the first sample of the day should be used to measure the albumin:creatinine ratio however if this is
not available then a ‘random’ sample may be substituted however this is associated with an increased risk of false
positive results.

Where the initial albumin:creatinine ratio is above 3 mg/mmol but below 30 mg/mmol, then the test should be
repeated on 2 further occasions using ‘early morning urine’ before proceeding to investigation and therapy. If the

https://mypastest.pastest.com/Secure/TestMe/Browser/436619[‫ م‬11:57:59 08/12/1437]


MyPastest

initial test is 30 mg/mmol or more (proteinuria) then investigate further without additional testing.
Next Question
https://www.nice.org.uk/guidance/ng18
47202

End Session
Tag Question

Feedback

Difficulty: Average

Peer Responses

Session Progress

Responses Correct: 0

Responses Incorrect: 5

Responses Total: 5

Responses - % Correct: 0%

Blog
About Pastest
Contact Us
Help

© Pastest 2016 Previous Question

https://mypastest.pastest.com/Secure/TestMe/Browser/436619[‫ م‬11:57:59 08/12/1437]


MyPastest

Prefer to use the old MyPastest? Access it here »

Back to Filters

Question 6 of 16

A 4-month-old baby is brought into the emergency department having been referred by her GP. On examination
the baby has a lower respiratory tract infection, cough and increased work of breathing. Her
mother, who is a
single mum aged 17, reports that she thought the child
had a cold which began about 5 days ago but is increasingly
concerned as the child is finding it difficult to breastfeed and she feels hot.

With regard to appropriate NICE guidelines, what pathogens may be associated with the most likely diagnosis?

A Adenovirus

B Influenza virus

C Metapneumovirus

D Respiratory syncytial virus

E All of the above

Explanation
The most likely diagnosis is bronchiolitis. This
is covered by NICE Guidelines 9 which was last updated in 2015.
It is certainly one of the guidelines worth reading and absorbing for exams as
well as clinical practice. One in
three children will suffer from bronchiolitis in the first year and 2% – 3% of these will be admitted to hospital .
The peak incidence occurs between 3-6 months although younger children may be affected. Apnoea may be the
only presenting feature in infants. Whilst RSV is the commonest cause, all the other agents listed may be
implicated.

Most cases of bronchiolitis can be nursed at home however NICE recommends that GPs take into account factors
which might make such a course less ideal such
as social circumstances, skill and confidence of the carer in
looking after the ill child at home and their ability to recognise red flag symptoms or the geographical distance
from help should there be deterioration in the child’s condition.

Red flag symptoms are listed as:

https://mypastest.pastest.com/Secure/TestMe/Browser/436619[‫ م‬11:58:20 08/12/1437]


MyPastest

Worsening work of breathing Next Question


50 – 75% of normal fluid intake (evidenced by no wet nappy for 12 hours)
Apnoea or cyanosis
Exhaustion (failing to respond to social cues/excessively sleepy and difficult to rouse)

End Session
https://www.nice.org.uk/guidance/ng9
47203

Tag Question

Feedback

Difficulty: Average

Peer Responses

Session Progress

Responses Correct: 0

Responses Incorrect: 6

Responses Total: 6

Responses - % Correct: 0%
Previous Question

Blog
About Pastest
Contact Us
Help

© Pastest 2016

https://mypastest.pastest.com/Secure/TestMe/Browser/436619[‫ م‬11:58:20 08/12/1437]


MyPastest

https://mypastest.pastest.com/Secure/TestMe/Browser/436619[‫ م‬11:58:20 08/12/1437]


MyPastest

Prefer to use the old MyPastest? Access it here »

Back to Filters

Question 7 of 16

A 3-week-old child has been referred by red light ambulance to the emergency department with suspected
bronchiolitis.

Which
of the following red flag symptoms does NICE advise may be the only presenting clinical sign of
bronchiolitis, in those under 6-weeks of age, and requires immediate referral to hospital usually by ambulance?

A Apnoea (observed or reported)

B Child looks seriously unwell to a healthcare professional

C Marked chest wall recession

D Poorly responsive

E Reduced fluid intake such that nappies are dry for 12 hours

Explanation
NICE updated their guidelines on bronchiolitis in
2015 (NG9). It is certainly one of the guidelines worth reading
for exams as well clinical practice. NICE suggests, one in three children will suffer from bronchiolitis within their
first year of life. The peak
incidence occurs between 3-6 months although younger children may be affected.

Apnoea may be the only presenting feature in infants.


This is a red flag symptom. All the above stems are
indicative of bronchiolitis. Children should be referred for immediate emergency hospital care, often by 999
ambulance in the following circumstances:

Apnoea (observed or reported)


Child looks seriously unwell to a healthcare professional (stem B)
Severe respiratory distress (eg grunting, marked chest wall recession, respiratory rate over 70 breaths per
minute (stem C)

https://mypastest.pastest.com/Secure/TestMe/Browser/436619[‫ م‬11:58:41 08/12/1437]


MyPastest

Central cyanosis
Next Question
Persistent oxygen saturation of less than 92% when breathing air.

Consideration should be given to referring children to hospital if:

Respiratory rate of over 60 breaths per minute


End (stem
Difficulty breastfeeding or inadequate oral fluid (50 – 75% of usual volume) Session
E)
Clinical dehydration

The majority of patients will be discharged home with strict advice to ensure parents will seek advice if the
situation deteriorates. Clear documentation is advised. The decision whether to direct a parent to hospital should
take into account the carer’s ability to look after their sick child at home. Nice includes the following as relevant
factors:

Social circumstances
Skill and confidence of carer
Ability to spot red flag symptoms (for example as described in stem D)
Distance from professional help if child worsens

https://www.nice.org.uk/guidance/ng9
47204

Tag Question

Feedback

Difficulty: Average

Peer Responses

Previous Question

Session Progress

https://mypastest.pastest.com/Secure/TestMe/Browser/436619[‫ م‬11:58:41 08/12/1437]


MyPastest

Responses Correct: 0

Responses Incorrect: 7

Responses Total: 7

Responses - % Correct: 0%

Blog
About Pastest
Contact Us
Help

© Pastest 2016

https://mypastest.pastest.com/Secure/TestMe/Browser/436619[‫ م‬11:58:41 08/12/1437]


MyPastest

Next Question
Prefer to use the old MyPastest? Access it here »

Back to Filters

Question 8 of 16

According to the 2015 NICE guidelines on bronchiolitis, at what peripheral oxygen saturation is oxygen
supplementation recommended?

A Persistently less than 88%

B Persistently less than 89%

C Persistently less than 90%

D Persistently less than 91%

E Persistently less than 92%

Explanation
NICE updated their guidelines on bronchiolitis in
2015. It is certainly one of the guidelines worth reading for
exams as well clinical practice. One in three children will suffer from bronchiolitis within their first year. The peak
incidence occurs between
3-6 months although younger children may be affected.

Apnoea may be the only presenting feature in infants. The majority of patients will be discharged home with strict
advice to ensure parents will seek further help if the situation deteriorates. Nevertheless there are criteria whereby
admission and indeed oxygen therapy is advised. Persistent oxygen saturations less than 92% is an indication for
oxygen supplementation.

https://www.nice.org.uk/guidance/ng9
47205

Tag Question

https://mypastest.pastest.com/Secure/TestMe/Browser/436619[‫ م‬11:59:03 08/12/1437]


MyPastest

Feedback

Difficulty: Average

End Session
Peer Responses

Session Progress

Responses Correct: 0

Responses Incorrect: 8

Responses Total: 8

Responses - % Correct: 0%

Blog
About Pastest
Contact Us
Help

© Pastest 2016

Previous Question

https://mypastest.pastest.com/Secure/TestMe/Browser/436619[‫ م‬11:59:03 08/12/1437]


MyPastest

Prefer to use the old MyPastest? Access it here »

Back to Filters

Question 9 of 16

A child has been brought into the emergency department with a diagnosis of bronchiolitis.

According to the new (2015) NICE guidelines, which of the following therapies may be used in the treatment of
bronchiolitis?

A Antibiotics

B Ipratropium bromide

C Hypertonic saline

D Oxygen

E Salbutamol

Explanation
NICE updated their guidelines on bronchiolitis in
2015. It is certainly one of the guidelines worth reading for
exams as well clinical practice. One in three children are likely to suffer from bronchiolitis in their first year. The
peak incidence occurs between 3-6
months although younger children may be affected. Apnoea may be the only
presenting feature in infants. The majority of patients will be discharged home with strict advice to ensure parents
will seek advice if
the situation deteriorates. Nevertheless there are criteria whereby admission and indeed oxygen
therapy is advised. Persistent oxygen saturations less than 92% is an indication for oxygen supplementation. The
other treatments are specifically listed as not to be used to treat bronchiolitis by NICE given a lack of evidence of
effectiveness. The full list is as follows:

Antibiotics
Hypertonic saline
Adrenaline (nebulised)
Salbutamol

https://mypastest.pastest.com/Secure/TestMe/Browser/436619[‫ م‬11:59:29 08/12/1437]


MyPastest

Montelukast
Ipratropium bromide
Next Question
Systemic or inhaled corticosteroids
A combination of systemic corticosteroids and nebulised adrenaline

https://www.nice.org.uk/guidance/ng9 End Session


47206

Tag Question

Feedback

Difficulty: Average

Peer Responses

Session Progress

Responses Correct: 0

Responses Incorrect: 9

Responses Total: 9

Responses - % Correct: 0%

Previous Question

Blog
About Pastest
Contact Us
Help

© Pastest 2016

https://mypastest.pastest.com/Secure/TestMe/Browser/436619[‫ م‬11:59:29 08/12/1437]


MyPastest

Next Question
Prefer to use the old MyPastest? Access it here »

Back to Filters

Question 10 of 16

You see a 3-month-old child in clinic with a short history of


vomiting and apparent pain. Given the absence of
diarrhoea, her parents
are concerned about the presence of gastro-oesophageal reflux disease.

With
reference to the relevant NICE guidelines, which one of the following conditions would NOT be considered
as a viable differential diagnosis in
this scenario?

A Cows’ milk intolerance

B Gastro-oesophageal disease

C Pyloric stenosis

D Raised intracranial pressure

E Urinary tract infection

Explanation
NICE updated their guidance on gastro-oesophageal
disease in 2015 (NG1). It is essential to consider red flag
features in
any case scenario. Vomiting with distress in the absence of diarrhoea widens the differential and also
includes intestinal obstruction.

Cows’ milk intolerance would generally cause diarrhoea with some


blood in older children so would not be
considered a differential diagnosis in this scenario.

Non-gastric causes such as UTI and raised intracranial pressure should always be considered in the differential
diagnosis.

https://www.nice.org.uk/guidance/ng1
47207

https://mypastest.pastest.com/Secure/TestMe/Browser/436619[‫ م‬11:59:49 08/12/1437]


MyPastest

Tag Question

Feedback

Difficulty: Average End Session

Peer Responses

Session Progress

Responses Correct: 0

Responses Incorrect: 10

Responses Total: 10

Responses - % Correct: 0%

Blog
About Pastest
Contact Us
Help

© Pastest 2016

Previous Question

https://mypastest.pastest.com/Secure/TestMe/Browser/436619[‫ م‬11:59:49 08/12/1437]


MyPastest

Next Question
Prefer to use the old MyPastest? Access it here »

Back to Filters

Question 11 of 16

You see a child with symptoms of gastro-intestinal reflux without distress or additional concerning features. The
parents ask if
the symptoms will ever improve.

What percentage of children will be symptom free by one year of age according to NICE ?

A 60%

B 70%

C 80%

D 90%

E 100%

Explanation
NICE updated their guidance on gastro-oesophageal
disease in 2015 (NG1). Generally questions with ‘all/100%’
or ‘nothing’ responses are incorrect answers; there are usually caveats in medicine.

NICE recommends when giving advice


to parents about gastro-oesophageal reflux (GOR) that you remember that
a well infant will commonly regurgitate milk after feeding (around 40% of infants), and that this may be frequent
(5% of infants will posset as
many as 6 times each day). This usually begins before the child is 8 weeks old but
has generally ceased (in approximately 90% of cases) by the age of 1 year.

https://www.nice.org.uk/guidance/ng1
47208

https://mypastest.pastest.com/Secure/TestMe/Browser/436619[‫ ص‬12:00:12 09/12/1437]


MyPastest

Tag Question

Feedback

Difficulty: Average

End Session
Peer Responses

Session Progress

Responses Correct: 0

Responses Incorrect: 11

Responses Total: 11

Responses - % Correct: 0%

Blog
About Pastest
Contact Us
Help

© Pastest 2016

Previous Question

https://mypastest.pastest.com/Secure/TestMe/Browser/436619[‫ ص‬12:00:12 09/12/1437]


MyPastest

Prefer to use the old MyPastest? Access it here »

Back to Filters

Question 14 of 16

You are talking to the parents of a child who has been displaying concerning behaviours for the past month.
Psychosis is being considered.

According to NICE quality standards on bipolar disorder, psychosis and schizophrenia published in 2015, within
what time frame should a child with a first episode of psychosis be assessed?

A 1 day

B 7 days

C 2 weeks

D 4 weeks

E 6 weeks

Explanation
NICE updated their quality standards on bipolar disorder, psychosis and schizophrenia in 2015 (QS102). They
recommend specialist mental health assessment occurs within 2 weeks of first psychotic presentation. Clearly the
standard needs tailoring to the individual’s risk and symptomatology and immediate admission may be required
where there is a danger to the patient or others. Equally, florid psychosis would usually require immediate
admission.

By starting the assessment within 2 weeks it is intended that there will be


improved outcomes for both the child
and their family as long term untreated psychosis is associated with worse outcomes. It is also intended that fewer
young people will require admittance to hospital.

In practice children and young people who have been suffering delusions or hallucinations for 4 weeks or more
should start their assessment within 2 weeks of referral to a specialist mental health service such as CAMHS
(Child and Adolescent Mental Health Service).

https://mypastest.pastest.com/Secure/TestMe/Browser/436619#Top[‫ ص‬12:01:51 09/12/1437]


MyPastest

https://www.nice.org.uk/guidance/qs102
Next Question 47211

Previous Question
End Session
Tag Question

Feedback

Difficulty: Average

Peer Responses

Session Progress

Responses Correct: 0

Responses Incorrect: 14

Responses Total: 14

Responses - % Correct: 0%

Blog
About Pastest
Contact Us
Help

© Pastest 2016

https://mypastest.pastest.com/Secure/TestMe/Browser/436619#Top[‫ ص‬12:01:51 09/12/1437]


MyPastest

Prefer to use the old MyPastest? Access it here »

Back to Filters

Question 15 of 16

A 14-year-old girl has been suffering migraine type headaches


for the past 6 months. You suspect she may be
suffering from a menstrual-related migraine and ask her to keep a headache diary.

How
many consecutive menstrual cycles should headache be associated with, before a diagnosis of menstrual-
related migraine can be made?

A 1

B 2

C 3

D 4

E 5

Explanation
This question references NICE Guidance 150 – Headaches in over 12s: diagnosis and management.

Definitions are easy marks for examiners. Headache in teenagers are common and it is important to know the
definitions to help support the patient and family, and indeed to ensure sinister diagnoses are not missed.

Migraines need to be associated with 2 out 3 consecutive periods before a diagnosis of menstruation associated
migraine can be made or else the association may be more by chance.

Frovatriptan or zolmitriptan may be offered if preventative measures such as avoiding triggers and standard
therapies such as topiramate or propanolol fail to work. Aspirin should not be offered in teenagers under 16. It is
important to note the oral contraceptive pill is contraindicated in migraine with aura.

https://www.nice.org.uk/guidance/CG150
47212

https://mypastest.pastest.com/Secure/TestMe/Browser/436619#Top[‫ ص‬12:02:12 09/12/1437]


MyPastest

Next Question

Previous Question Tag Question

Feedback End Session

Difficulty: Average

Peer Responses

Session Progress

Responses Correct: 0

Responses Incorrect: 15

Responses Total: 15

Responses - % Correct: 0%

Blog
About Pastest
Contact Us
Help

© Pastest 2016

https://mypastest.pastest.com/Secure/TestMe/Browser/436619#Top[‫ ص‬12:02:12 09/12/1437]


MyPastest

Next Question
Prefer to use the old MyPastest? Access it here »

Back to Filters

Question 1 of 16

You see a child in clinic with newly diagnosed type 1 diabetes.

What is considered to be the target Hba1c in children and young people with type 1 diabetes in the latest NICE
guidelines?

A HbA1C - 48mmol/mol

B HbA1C - 54mmol/mol

C HbA1C - 58mmol/mol

D HbA1C - 60mmol/mol

E HbA1C - 64mmol/mol

Explanation
NICE have encouraged much stricter glucose control than previous guidance - with the caveat that children and
young
people, and their families are given education, emotional support and obesity management, and do not feel
pressured, hence a personal management plan is required. Tighter control is considered to offer a much better
outlook by reducing the risk of complications such as nephropathy and retinopathy, hence a maximum HbA1c of
48mmol/mol is recommended.
Previous Question
https://www.nice.org.uk/guidance/ng18
47198

Tag Question

https://mypastest.pastest.com/Secure/TestMe/Browser/436619[‫ ص‬04:25:41 10/12/1437]


MyPastest

Feedback

Difficulty: Average

Peer Responses
End Session

Session Progress

Responses Correct: 0

Responses Incorrect: 1

Responses Total: 1

Responses - % Correct: 0%

Blog
About Pastest
Contact Us
Help

© Pastest 2016

https://mypastest.pastest.com/Secure/TestMe/Browser/436619[‫ ص‬04:25:41 10/12/1437]


MyPastest

Next Question
Prefer to use the old MyPastest? Access it here »

Back to Filters

Question 2 of 16

You have a young child in clinic whose parents are reluctant to attend clinic for routine HbA1c checks.

How often should HbA1c be checked in children and young people with type 2 diabetes according to the latest
NICE guidelines?

A Every month

B Every 2 months

C Every 3 months

D Every 4 months

E Every 6 months

Explanation
NICE updated their guidelines on diabetes in children and young people in 2015 (NG 18). There is a focus on the
patient and their family and a clear steer towards reducing complication
risk. Education, emotional support and
obesity management is discussed delivered through personal management plans to engage and create a sense
of
ownership by the patient and their family.

HbA1c should be checked every 3 months to ensure tight control is maintained, to enable early intervention in the
context of poor control and to provide a topic
of discussion during reviews.

https://www.nice.org.uk/guidance/ng18
47199

https://mypastest.pastest.com/Secure/TestMe/Browser/436619[‫ ص‬04:26:04 10/12/1437]


MyPastest

Tag Question

Feedback

Difficulty: Average

End Session
Peer Responses

Session Progress

Responses Correct: 0

Responses Incorrect: 2

Responses Total: 2

Responses - % Correct: 0%

Blog
About Pastest
Contact Us
Help

© Pastest 2016

Previous Question

https://mypastest.pastest.com/Secure/TestMe/Browser/436619[‫ ص‬04:26:04 10/12/1437]


MyPastest

Next Question
Prefer to use the old MyPastest? Access it here »

Back to Filters

Question 3 of 16

With reference to the appropriate Nice guidance, how often should children and young people with type 1 diabetes
attend secondary care clinics?

A Every 2 months

B Every 3 months

C Every 4 months

D Every 6 months

E Annually

Explanation
NICE updated their guidelines on diabetes in children and young people in 2015 (NG18).

A diagnosis of Type 1 diabetes is being made much more regularly than in previous decades in the UK. NICE
quotes the 2013-14 National Diabetes Audit as having identified ‘26,500 children and young people with type 1
diabetes’. Accordingly there is increased need
to focus on the patient and their family and for NICE to give a clear
steer towards reducing complication risk. Education, emotional support and obesity management is discussed and
delivered through personal management plans to engage and create a sense of ownership by the patient and their
family.

Those patients with type 1 disease should be reviewed in a specialist clinic four times a year to ensure patients and
their families continue to be well educated, to monitor HbA1c and to ensure complications are identified early.

https://www.nice.org.uk/guidance/ng18
47200

https://mypastest.pastest.com/Secure/TestMe/Browser/436619[‫ ص‬04:26:24 10/12/1437]


MyPastest

Tag Question

Feedback

Difficulty: Average
End Session
Peer Responses

Session Progress

Responses Correct: 0

Responses Incorrect: 3

Responses Total: 3

Responses - % Correct: 0%

Blog
About Pastest
Contact Us
Help

© Pastest 2016

Previous Question

https://mypastest.pastest.com/Secure/TestMe/Browser/436619[‫ ص‬04:26:24 10/12/1437]


MyPastest

Prefer to use the old MyPastest? Access it here »

Back to Filters

Question 4 of 16

You have a patient who was diagnosed with diabetes at the age
of 5. She is approaching the age at which
retinopathy screening should
be commenced in line with Nice guidelines.

From what age should children and young people with Type 1 diabetes have retinopathy screening?

A 6 years

B 8 years

C 10 years

D 12 years

E 14 years

Explanation
NICE updated their guidelines on diabetes in children and young people in 2015 (NG18).

There is a focus on the patient and their family and a clear steer towards reducing complication risk. Education,
emotional support and obesity management is discussed delivered through personal management plans to engage
and create a sense of ownership by the patient and their
family.

Those with type 1 disease should have thyroid function checked at diagnosis and annually from then on.
Retinopathy screening should occur from age 12 in both type 1 and type 2 diabetes with a view to early
intervention. Type 1 diabetics in particular are at greater risk of cataract.

Additionally from age 12 children should be monitored annually for moderately increased albuminuria
(albumin:creatinine ration (ACR) 3-30 mg/mmol; ‘microalbuminuria’) to detect diabetic kidney disease and for
signs of hypertension.

https://www.nice.org.uk/guidance/ng18
47201

https://mypastest.pastest.com/Secure/TestMe/Browser/436619[‫ ص‬04:26:50 10/12/1437]


MyPastest

Next Question

Tag Question

Feedback End Session

Difficulty: Average

Peer Responses

Session Progress

Responses Correct: 0

Responses Incorrect: 4

Responses Total: 4

Responses - % Correct: 0%

Blog
About Pastest
Contact Us
Help

© Pastest 2016

Previous Question

https://mypastest.pastest.com/Secure/TestMe/Browser/436619[‫ ص‬04:26:50 10/12/1437]


MyPastest

Prefer to use the old MyPastest? Access it here »

Back to Filters

Question 5 of 16

With reference to relevant NICE guidelines:

From
what age should children and young people with type 1 diabetes have nephropathy screening with an annual
early morning urine sample for albumin:creatinine ratio?

A From diagnosis

B 8 years

C 10 years

D 12 years

E 14 years

Explanation
NICE updated their guidelines on diabetes in children and young people in 2015 (NG18). There is a focus on the
patient and their family and a clear steer towards reducing complication
risk. Education, emotional support and
obesity management is discussed delivered through personal management plans to engage and create a sense
of
ownership by the patient and their family.

Those with type 1


disease should have thyroid function checked at diagnosis and annually from then on.
Retinopathy and nephropathy screening should be offered annually from age 12 in type 1 disease whereas children
and young people
with type 2 diabetics should have urine ACR from diagnosis.

In
either case the first sample of the day should be used to measure the albumin:creatinine ratio however if this is
not available then a ‘random’ sample may be substituted however this is associated with an increased risk of false
positive results.

Where the initial albumin:creatinine ratio is above 3 mg/mmol but below 30 mg/mmol, then the test should be
repeated on 2 further occasions using ‘early morning urine’ before proceeding to investigation and therapy. If the

https://mypastest.pastest.com/Secure/TestMe/Browser/436619[‫ ص‬04:27:11 10/12/1437]


MyPastest

initial test is 30 mg/mmol or more (proteinuria) then investigate further without additional testing.
Next Question
https://www.nice.org.uk/guidance/ng18
47202

End Session
Tag Question

Feedback

Difficulty: Average

Peer Responses

Session Progress

Responses Correct: 0

Responses Incorrect: 5

Responses Total: 5

Responses - % Correct: 0%

Blog
About Pastest
Contact Us
Help

© Pastest 2016 Previous Question

https://mypastest.pastest.com/Secure/TestMe/Browser/436619[‫ ص‬04:27:11 10/12/1437]


MyPastest

Prefer to use the old MyPastest? Access it here »

Back to Filters

Question 6 of 16

A 4-month-old baby is brought into the emergency department having been referred by her GP. On examination
the baby has a lower respiratory tract infection, cough and increased work of breathing. Her
mother, who is a
single mum aged 17, reports that she thought the child
had a cold which began about 5 days ago but is increasingly
concerned as the child is finding it difficult to breastfeed and she feels hot.

With regard to appropriate NICE guidelines, what pathogens may be associated with the most likely diagnosis?

A Adenovirus

B Influenza virus

C Metapneumovirus

D Respiratory syncytial virus

E All of the above

Explanation
The most likely diagnosis is bronchiolitis. This
is covered by NICE Guidelines 9 which was last updated in 2015.
It is certainly one of the guidelines worth reading and absorbing for exams as
well as clinical practice. One in
three children will suffer from bronchiolitis in the first year and 2% – 3% of these will be admitted to hospital .
The peak incidence occurs between 3-6 months although younger children may be affected. Apnoea may be the
only presenting feature in infants. Whilst RSV is the commonest cause, all the other agents listed may be
implicated.

Most cases of bronchiolitis can be nursed at home however NICE recommends that GPs take into account factors
which might make such a course less ideal such
as social circumstances, skill and confidence of the carer in
looking after the ill child at home and their ability to recognise red flag symptoms or the geographical distance
from help should there be deterioration in the child’s condition.

Red flag symptoms are listed as:

https://mypastest.pastest.com/Secure/TestMe/Browser/436619[‫ ص‬04:27:33 10/12/1437]


MyPastest

Worsening work of breathing Next Question


50 – 75% of normal fluid intake (evidenced by no wet nappy for 12 hours)
Apnoea or cyanosis
Exhaustion (failing to respond to social cues/excessively sleepy and difficult to rouse)

End Session
https://www.nice.org.uk/guidance/ng9
47203

Tag Question

Feedback

Difficulty: Average

Peer Responses

Session Progress

Responses Correct: 0

Responses Incorrect: 6

Responses Total: 6

Responses - % Correct: 0%
Previous Question

Blog
About Pastest
Contact Us
Help

© Pastest 2016

https://mypastest.pastest.com/Secure/TestMe/Browser/436619[‫ ص‬04:27:33 10/12/1437]


MyPastest

https://mypastest.pastest.com/Secure/TestMe/Browser/436619[‫ ص‬04:27:33 10/12/1437]


MyPastest

Prefer to use the old MyPastest? Access it here »

Back to Filters

Question 7 of 16

A 3-week-old child has been referred by red light ambulance to the emergency department with suspected
bronchiolitis.

Which
of the following red flag symptoms does NICE advise may be the only presenting clinical sign of
bronchiolitis, in those under 6-weeks of age, and requires immediate referral to hospital usually by ambulance?

A Apnoea (observed or reported)

B Child looks seriously unwell to a healthcare professional

C Marked chest wall recession

D Poorly responsive

E Reduced fluid intake such that nappies are dry for 12 hours

Explanation
NICE updated their guidelines on bronchiolitis in
2015 (NG9). It is certainly one of the guidelines worth reading
for exams as well clinical practice. NICE suggests, one in three children will suffer from bronchiolitis within their
first year of life. The peak
incidence occurs between 3-6 months although younger children may be affected.

Apnoea may be the only presenting feature in infants.


This is a red flag symptom. All the above stems are
indicative of bronchiolitis. Children should be referred for immediate emergency hospital care, often by 999
ambulance in the following circumstances:

Apnoea (observed or reported)


Child looks seriously unwell to a healthcare professional (stem B)
Severe respiratory distress (eg grunting, marked chest wall recession, respiratory rate over 70 breaths per
minute (stem C)

https://mypastest.pastest.com/Secure/TestMe/Browser/436619[‫ ص‬04:27:56 10/12/1437]


MyPastest

Central cyanosis
Next Question
Persistent oxygen saturation of less than 92% when breathing air.

Consideration should be given to referring children to hospital if:

Respiratory rate of over 60 breaths per minute


End (stem
Difficulty breastfeeding or inadequate oral fluid (50 – 75% of usual volume) Session
E)
Clinical dehydration

The majority of patients will be discharged home with strict advice to ensure parents will seek advice if the
situation deteriorates. Clear documentation is advised. The decision whether to direct a parent to hospital should
take into account the carer’s ability to look after their sick child at home. Nice includes the following as relevant
factors:

Social circumstances
Skill and confidence of carer
Ability to spot red flag symptoms (for example as described in stem D)
Distance from professional help if child worsens

https://www.nice.org.uk/guidance/ng9
47204

Tag Question

Feedback

Difficulty: Average

Peer Responses

Previous Question

Session Progress

https://mypastest.pastest.com/Secure/TestMe/Browser/436619[‫ ص‬04:27:56 10/12/1437]


MyPastest

Responses Correct: 0

Responses Incorrect: 7

Responses Total: 7

Responses - % Correct: 0%

Blog
About Pastest
Contact Us
Help

© Pastest 2016

https://mypastest.pastest.com/Secure/TestMe/Browser/436619[‫ ص‬04:27:56 10/12/1437]


MyPastest

Next Question
Prefer to use the old MyPastest? Access it here »

Back to Filters

Question 8 of 16

According to the 2015 NICE guidelines on bronchiolitis, at what peripheral oxygen saturation is oxygen
supplementation recommended?

A Persistently less than 88%

B Persistently less than 89%

C Persistently less than 90%

D Persistently less than 91%

E Persistently less than 92%

Explanation
NICE updated their guidelines on bronchiolitis in
2015. It is certainly one of the guidelines worth reading for
exams as well clinical practice. One in three children will suffer from bronchiolitis within their first year. The peak
incidence occurs between
3-6 months although younger children may be affected.

Apnoea may be the only presenting feature in infants. The majority of patients will be discharged home with strict
advice to ensure parents will seek further help if the situation deteriorates. Nevertheless there are criteria whereby
admission and indeed oxygen therapy is advised. Persistent oxygen saturations less than 92% is an indication for
oxygen supplementation.

https://www.nice.org.uk/guidance/ng9
47205

Tag Question

https://mypastest.pastest.com/Secure/TestMe/Browser/436619[‫ ص‬04:28:18 10/12/1437]


MyPastest

Feedback

Difficulty: Average

End Session
Peer Responses

Session Progress

Responses Correct: 0

Responses Incorrect: 8

Responses Total: 8

Responses - % Correct: 0%

Blog
About Pastest
Contact Us
Help

© Pastest 2016

Previous Question

https://mypastest.pastest.com/Secure/TestMe/Browser/436619[‫ ص‬04:28:18 10/12/1437]


MyPastest

Prefer to use the old MyPastest? Access it here »

Back to Filters

Question 9 of 16

A child has been brought into the emergency department with a diagnosis of bronchiolitis.

According to the new (2015) NICE guidelines, which of the following therapies may be used in the treatment of
bronchiolitis?

A Antibiotics

B Ipratropium bromide

C Hypertonic saline

D Oxygen

E Salbutamol

Explanation
NICE updated their guidelines on bronchiolitis in
2015. It is certainly one of the guidelines worth reading for
exams as well clinical practice. One in three children are likely to suffer from bronchiolitis in their first year. The
peak incidence occurs between 3-6
months although younger children may be affected. Apnoea may be the only
presenting feature in infants. The majority of patients will be discharged home with strict advice to ensure parents
will seek advice if
the situation deteriorates. Nevertheless there are criteria whereby admission and indeed oxygen
therapy is advised. Persistent oxygen saturations less than 92% is an indication for oxygen supplementation. The
other treatments are specifically listed as not to be used to treat bronchiolitis by NICE given a lack of evidence of
effectiveness. The full list is as follows:

Antibiotics
Hypertonic saline
Adrenaline (nebulised)
Salbutamol

https://mypastest.pastest.com/Secure/TestMe/Browser/436619[‫ ص‬04:29:02 10/12/1437]


MyPastest

Montelukast
Ipratropium bromide
Next Question
Systemic or inhaled corticosteroids
A combination of systemic corticosteroids and nebulised adrenaline

https://www.nice.org.uk/guidance/ng9 End Session


47206

Tag Question

Feedback

Difficulty: Average

Peer Responses

Session Progress

Responses Correct: 0

Responses Incorrect: 9

Responses Total: 9

Responses - % Correct: 0%

Previous Question

Blog
About Pastest
Contact Us
Help

© Pastest 2016

https://mypastest.pastest.com/Secure/TestMe/Browser/436619[‫ ص‬04:29:02 10/12/1437]


MyPastest

Next Question
Prefer to use the old MyPastest? Access it here »

Back to Filters

Question 10 of 16

You see a 3-month-old child in clinic with a short history of


vomiting and apparent pain. Given the absence of
diarrhoea, her parents
are concerned about the presence of gastro-oesophageal reflux disease.

With
reference to the relevant NICE guidelines, which one of the following conditions would NOT be considered
as a viable differential diagnosis in
this scenario?

A Cows’ milk intolerance

B Gastro-oesophageal disease

C Pyloric stenosis

D Raised intracranial pressure

E Urinary tract infection

Explanation
NICE updated their guidance on gastro-oesophageal
disease in 2015 (NG1). It is essential to consider red flag
features in
any case scenario. Vomiting with distress in the absence of diarrhoea widens the differential and also
includes intestinal obstruction.

Cows’ milk intolerance would generally cause diarrhoea with some


blood in older children so would not be
considered a differential diagnosis in this scenario.

Non-gastric causes such as UTI and raised intracranial pressure should always be considered in the differential
diagnosis.

https://www.nice.org.uk/guidance/ng1
47207

https://mypastest.pastest.com/Secure/TestMe/Browser/436619[‫ ص‬04:29:23 10/12/1437]


MyPastest

Tag Question

Feedback

Difficulty: Average End Session

Peer Responses

Session Progress

Responses Correct: 0

Responses Incorrect: 10

Responses Total: 10

Responses - % Correct: 0%

Blog
About Pastest
Contact Us
Help

© Pastest 2016

Previous Question

https://mypastest.pastest.com/Secure/TestMe/Browser/436619[‫ ص‬04:29:23 10/12/1437]


MyPastest

Next Question
Prefer to use the old MyPastest? Access it here »

Back to Filters

Question 11 of 16

You see a child with symptoms of gastro-intestinal reflux without distress or additional concerning features. The
parents ask if
the symptoms will ever improve.

What percentage of children will be symptom free by one year of age according to NICE ?

A 60%

B 70%

C 80%

D 90%

E 100%

Explanation
NICE updated their guidance on gastro-oesophageal
disease in 2015 (NG1). Generally questions with ‘all/100%’
or ‘nothing’ responses are incorrect answers; there are usually caveats in medicine.

NICE recommends when giving advice


to parents about gastro-oesophageal reflux (GOR) that you remember that
a well infant will commonly regurgitate milk after feeding (around 40% of infants), and that this may be frequent
(5% of infants will posset as
many as 6 times each day). This usually begins before the child is 8 weeks old but
has generally ceased (in approximately 90% of cases) by the age of 1 year.

https://www.nice.org.uk/guidance/ng1
47208

https://mypastest.pastest.com/Secure/TestMe/Browser/436619[‫ ص‬04:29:44 10/12/1437]


MyPastest

Tag Question

Feedback

Difficulty: Average

End Session
Peer Responses

Session Progress

Responses Correct: 0

Responses Incorrect: 11

Responses Total: 11

Responses - % Correct: 0%

Blog
About Pastest
Contact Us
Help

© Pastest 2016

Previous Question

https://mypastest.pastest.com/Secure/TestMe/Browser/436619[‫ ص‬04:29:44 10/12/1437]


MyPastest

Next Question
Prefer to use the old MyPastest? Access it here »

Back to Filters End Session

Question 12 of 16

You are considering the treatment options for a breastfed baby who continues to suffer frequent regurgitation
associated with distress. The regurgitation sometimes occurs at night. A breastfeeding assessment has been carried
out by a competent person.

According
to NICE guidelines on gastro-intestinal reflux disease (2015), which of
the following would be the next
treatment step?

A Further review of feeding history with advice

B Proton pump inhibitors

C Positioning management for sleeping infant

D Smaller volume and higher frequency feeds as part of stepped approach

E Trial alginate therapy for 1 – 2 weeks

Explanation
47209

Previous Question

Tag Question

Feedback

Difficulty: Average

https://mypastest.pastest.com/Secure/TestMe/Browser/436619[‫ ص‬04:30:18 10/12/1437]


MyPastest

Peer Responses

Session Progress

Responses Correct: 0

Responses Incorrect: 12

Responses Total: 12

Responses - % Correct: 0%

Blog
About Pastest
Contact Us
Help

© Pastest 2016

https://mypastest.pastest.com/Secure/TestMe/Browser/436619[‫ ص‬04:30:18 10/12/1437]


MyPastest

Next Question
Prefer to use the old MyPastest? Access it here »

Back to Filters End Session

Question 13 of 16

You are on the ward discussing gastrointestinal reflux (GOR) and its complications with a couple.

With
reference to appropriate NICE guidelines, which one of the following is
a possible complication of GOR
rather than a ‘red flag symptom’ suggestive of a disorder other than GOR?

A Apnoea

B Abdominal distension

C Bulging fontanelle

D Dysuria

E Frequent, projectile vomiting

47210

Tag Question
Previous Question
Feedback

Difficulty: Average

Peer Responses

https://mypastest.pastest.com/Secure/TestMe/Browser/436619[‫ ص‬04:30:39 10/12/1437]


MyPastest

Session Progress

Responses Correct: 0

Responses Incorrect: 13

Responses Total: 13

Responses - % Correct: 0%

Blog
About Pastest
Contact Us
Help

© Pastest 2016

https://mypastest.pastest.com/Secure/TestMe/Browser/436619[‫ ص‬04:30:39 10/12/1437]


MyPastest

Prefer to use the old MyPastest? Access it here »

Back to Filters

Question 14 of 16

You are talking to the parents of a child who has been displaying concerning behaviours for the past month.
Psychosis is being considered.

According to NICE quality standards on bipolar disorder, psychosis and schizophrenia published in 2015, within
what time frame should a child with a first episode of psychosis be assessed?

A 1 day

B 7 days

C 2 weeks

D 4 weeks

E 6 weeks

Explanation
NICE updated their quality standards on bipolar disorder, psychosis and schizophrenia in 2015 (QS102). They
recommend specialist mental health assessment occurs within 2 weeks of first psychotic presentation. Clearly the
standard needs tailoring to the individual’s risk and symptomatology and immediate admission may be required
where there is a danger to the patient or others. Equally, florid psychosis would usually require immediate
admission.

By starting the assessment within 2 weeks it is intended that there will be


improved outcomes for both the child
and their family as long term untreated psychosis is associated with worse outcomes. It is also intended that fewer
young people will require admittance to hospital.

In practice children and young people who have been suffering delusions or hallucinations for 4 weeks or more
should start their assessment within 2 weeks of referral to a specialist mental health service such as CAMHS
(Child and Adolescent Mental Health Service).

https://mypastest.pastest.com/Secure/TestMe/Browser/436619[‫ ص‬04:31:02 10/12/1437]


MyPastest

https://www.nice.org.uk/guidance/qs102
Next Question 47211

End Session
Tag Question

Feedback

Difficulty: Average

Peer Responses

Session Progress

Responses Correct: 0

Responses Incorrect: 14

Responses Total: 14

Responses - % Correct: 0%

Blog
About Pastest
Contact Us
Help

© Pastest 2016

Previous Question

https://mypastest.pastest.com/Secure/TestMe/Browser/436619[‫ ص‬04:31:02 10/12/1437]


MyPastest

Prefer to use the old MyPastest? Access it here »

Back to Filters

Question 15 of 16

A 14-year-old girl has been suffering migraine type headaches


for the past 6 months. You suspect she may be
suffering from a menstrual-related migraine and ask her to keep a headache diary.

How
many consecutive menstrual cycles should headache be associated with, before a diagnosis of menstrual-
related migraine can be made?

A 1

B 2

C 3

D 4

E 5

Explanation
This question references NICE Guidance 150 – Headaches in over 12s: diagnosis and management.

Definitions are easy marks for examiners. Headache in teenagers are common and it is important to know the
definitions to help support the patient and family, and indeed to ensure sinister diagnoses are not missed.

Migraines need to be associated with 2 out 3 consecutive periods before a diagnosis of menstruation associated
migraine can be made or else the association may be more by chance.

Frovatriptan or zolmitriptan may be offered if preventative measures such as avoiding triggers and standard
therapies such as topiramate or propanolol fail to work. Aspirin should not be offered in teenagers under 16. It is
important to note the oral contraceptive pill is contraindicated in migraine with aura.

https://www.nice.org.uk/guidance/CG150
47212

https://mypastest.pastest.com/Secure/TestMe/Browser/436619[‫ ص‬04:31:24 10/12/1437]


MyPastest

Next Question

Tag Question

Feedback End Session

Difficulty: Average

Peer Responses

Session Progress

Responses Correct: 0

Responses Incorrect: 15

Responses Total: 15

Responses - % Correct: 0%

Blog
About Pastest
Contact Us
Help

© Pastest 2016

Previous Question

https://mypastest.pastest.com/Secure/TestMe/Browser/436619[‫ ص‬04:31:24 10/12/1437]


MyPastest

Prefer to use the old MyPastest? Access it here »

Back to Filters

Question 16 of 16

You see a 15-year-old teenage girl in clinic for migraine without aura. She is sexually active and does not take her
contraceptive
pill reliably.

According to the NICE guideline on headache (CG150), what would you consider to be the most clinically
effective and appropriate first line drug in the prophylaxis of her migraine?

A Amitryptyline

B Gabapentin

C Pizotifen

D Propanolol

E Topiramate

47213

End Session

Previous Question Tag Question

Feedback

Difficulty: Average

Peer Responses

https://mypastest.pastest.com/Secure/TestMe/Browser/436619[‫ ص‬04:31:45 10/12/1437]


MyPastest

Session Progress

Responses Correct: 0

Responses Incorrect: 16

Responses Total: 16

Responses - % Correct: 0%

Blog
About Pastest
Contact Us
Help

© Pastest 2016

https://mypastest.pastest.com/Secure/TestMe/Browser/436619[‫ ص‬04:31:45 10/12/1437]

You might also like